You are on page 1of 1694

SLR-BB 1

*SLRBB1*
Seat
No.

Set

F.E. (Part I) Examination, 2014


ENGG. MATHEMATICS I (New)
Day and Date : Monday, 8-12-2014
Time : 10.00 a.m. to 1.00 p.m.

Max. Marks : 70

N. B. : 1) Q. No. 1 is compulsory. It should be solved in first 30 minutes


in Answer Book Page No. 3. Each question carries one mark.
2) Attempt any three questions from each Section.
3) Figures to the right indicate full marks.
4) Use of calculator is allowed.
5) Answer MCQ/Objective type questions on Page No. 3 only.
Dont forget to mention, Q.P. Set (A/B/C/D) on Top of Page.
MCQ/Objective Type Questions
Duration : 30 Minutes

Marks : 14

1. Choose the correct option :


1) If y =

14

1
, then yn = ___________
(x + 2)3

( 1)n (n + 2) !
a)
( x + 2)n + 3

( 1)n (n + 2) !
b)
2( x + 2)n + 3

( 1)n (n + 1) !
c)
( x + 2)n + 3

d) None of these

2) If y = log(1 + x), then yn =


( 1)n 1(n 1) !
a)
(1 + x)n

(1)n n !
b)
(1 + x )n + 1

( 1)n 1 (n 1) !
( 1)n n !
c)
d)
(1 + x)n + 1
(1 + x )n

3) In the Taylors series expansion of y = 1 about x = 1, the coefficient of


x
(x 1)2 is
a) 1
b) 1
c) 21
d) 21
4) lim x log x =
x0

a) 1

b) 2

5) The value of ii is equal to


a) e 2

b) e 2

c)

d) 0

c) e

d) 0
P.T.O.

SLR-BB 1

*SLRBB1*

-24

6) sin + i cos =
4
4
a) i
b) i
7) cosh1z =
a) log z + z 2 + 1

c) log z + z 2 1

c) 1

d) 1

b) log z z 2 + 1

d) None of these

8) If the eigen values of a matrix [ A ]3 3 are 2, 3, 4 then the eigen values of


A1 are

a) 2, 3, 4

b) 1 , 1 , 1
2 3 4

c) 1 , 1 , 1
2 3 4

2 1
9) The product of the eigen values of
is
3 6
a) 1
b) 2
c) 6
u
y
= _______
10) If u = x 2 y y then x
+y
y
x
x
a) 0
b) u
c) 2u

d) Not exist

d) 9

d) 3u

11) If A is a square matrix of order n and if |A| = 0 then the rank of Matrix A is
________
a) n
b) n + 1
c) greater than n d) less than n
u u
12) If u = x2 y2 then x + y = ________ at point (1, 1) is
a) 4
b) 0
c) 2
d) 2
u, v
13) If u = x2 and y2 = v then J x, y = __________

a) 4xy
b) 4xy
c) 2xy

d) 2xy

z
1 y
14) If z = tan x then y = ________

x
a) x 2 + y 2

b)

x
x y2
2

y
x + y2
______________

c)

x
d) x 2 + y 2

Set A

*SLRBB1*

SLR-BB 1

-3-

Seat
No.

F.E. (Part I) Examination, 2014


ENGG. MATHEMATICS I (New)
Day and Date : Monday, 8-12-2014
Time : 10.00 a.m. to 1.00 p.m.

Marks : 56

N. B. : 1) Attempt any three questions from each Section.


2) Figures to the right indicate full marks.
3) Use of calculator is allowed.
SECTION I
2. a) Find the nth derivative of y =

2x + 3
.
(3x + 2) (2x 1)

b) Find the nth derivative of y = eax cos2x sinx.

3
3

c) If y = cos(msin1x), prove that (1 x 2 )y n + 2 (2n + 1)xyn + 1 + (m2 n2 )yn = 0 . 4


3. Attempt any three :

a) Find the values of a and b such that lim

x0

a sin 2x b tan x
= 1.
x3

b) Expand log(1 + sinx) up to x3.


1 + x 2 1
up to x7.
c) Expand tan1

d) Expand log(cosx) about 3 .


4. a) If and are the roots of the equation x2 2x + 4 = 0, find the value of
15 + 15 .

b) Find the continued product of all the values of (i)2/3.

c) Express sin 6 in terms of powers of sin and cos .

3
Set A

SLR-BB 1

*SLRBB1*

-4-

5. a) If sin( + i) = cos + i sin prove that cos 4 = sin2 .

b) Prove that sec h1(sin ) = log(cot 2) .

x i
1
c) Prove that i log
= 2 tan x .
x
i
+

SECTION II
6. Attempt the following :
a) If u = logr and r2 = (x a)2 + (y b)2 + (z c)2 then show that,
2
2u + 2u + y = 1 .
x 2 y 2 z2 r 2

+ y2
u
u 1
b) If u = tan
then prove that, x
+y
= sin 2u .

x
y 2
xy

3u
c) If u = exyz then find x y z .

1 x

7. Attempt the following :


a) For what values of the set of equations 2x 3y + 6z 5t = 3, y 4z + t = 1,
4x 5y + 8z 9t = has
i) a unique solution
ii) an infinitely many solutions.

b) Solve by matrix method, 3x + y 5z = 0, 5x + 3y 6z = 0, x + y 2z = 0,


x 5y + z = 0.
1

c) Find the rank of matrix by normal form A = 1


3

3
1

1
1 1 .
1
1

Set A

*SLRBB1*

SLR-BB 1

-5-

8. Attempt the following :

a) Find the approximate value of (0.98)2 + (2.01)2 + (1.94)2


approximation.

1
2

by theory of
3

b) If x = eu cos v and y = eu sin v then prove that JJ1 = 1.

c) Find the maximum and minimum value of f(x, y) = sinx + siny + sin(x + y).

9. Attempt the following :


a) Are the vectors X1 = [1, 1, 1, 1], X2 = [3, 1, 0, 1], X3 = [1, 1, 2, 1] are
linearly dependent ? If so express X3 as a linear combination of remaining
two.

b) Find eigen values and eigen vector corresponding to greatest eigen value of
matrix A given below.
4
A = 5
2

2
3
4

2
2
1

c) Find the characteristic equation of matrix A given below and show that it is
satisfied by A, where
2
A = 1
1

1
2
1

1
1
2

Hence find A1.

4
_____________________

Set A

Set A

SLR-BB 100

*SLRBB100*
Seat
No.

Set

T.E. (Mech.) (Part I) Examination, 2014


MACHINE TOOL DESIGN (Professional Elective I) (New)
Day and Date : Wednesday, 10-12-2014
Time : 3.00 p.m. to 6.00 p.m.

Total Marks : 100

Instructions : 1) Figures to the right indicate full marks.


2) Assume suitable data if necessary and mention it clearly.
3) Q. No. 1 is compulsory. It should be solved in first 30 minutes in
Answer Book Page No. 3. Each question carries one mark.
4) Answer MCQ/Objective type questions on Page No. 3 only.
Dont forget to mention, Q.P. Set (A/B/C/D) on Top of Page.
MCQ/Objective Type Questions
Duration : 30 Minutes

Marks : 20

1. Solve the following objective type questions :

20

1) The external appearance is important in


a) consumer durable like refrigerators and audiovisual equipment
b) industrial products like cranes and hoists
c) machine elements like gearbox, coupling or pressure vessel
d) none of the above
2) Steels used for welded assemblies are
a) medium carbon steel
b) mild steel
c) high carbon steel
d) alloy steel
3) A machine tool is a power driven machine, capable of
a) Holding a tool
b) Holding the job
c) Directing and guiding the tool and job
d) All of above
4) General purpose machine tools are used
a) For high production rate
b) In normal workshops and repair shops
c) For large production volumes
d) For automated production
5) Gearbox is produced by
a) design by drawing
c) design synthesis

b) design by craft evolution


d) simultaneous design

6) A V-slideway is preferred in lathe bed because it


a) it easy to manufacture
b) adjusts the clearance under the weight of moving member
c) is a simple design
d) no specific advantage
P.T.O.

SLR-BB 100

-2-

*SLRBB100*

7) In DNC main computer is the one used for part programming and for other data
processing operations.
a) true
b) false
8) Value of coefficient of efficiency of belt drive with flat belt is
a) 0.98
b) 0.90
c) 0.50
d) 0.75
9) Breaking in electrical control can be obtained
a) by increasing magnetic flux of motor
b) by decreasing potential of generator
c) a) or b)
d) none of above
10) Vibration in a machine tool can occur under the influence of ________
a) unbalanced rotating force b) transmission of vibratory forces to job and tool
c) both a) and b)
d) none of above
11) Plastic guides are used because of
a) less wear
b) less friction
c) easy fabrication
d) all of above
12) Active isolators are used to prevent __________
a) outside transmission of the vibratory forces
b) vibration of the neighbouring sources being transmitted to the machine
c) both a) and b)
d) none of these
13) Passive isolators are used to prevent ___________
a) outside transmission of the vibratory forces
b) vibration of the neighbouring sources being transmitted to the machine
c) both a) and b)
d) none of these
14) Shock mounts are also known as __________
a) dampers
b) isolators
c) springs
d) insulations
15) One of the consideration for designing the controlling systems is all handles or
shifting mechanism must be concentrated in as narrow an area of normal working
as possible.
a) true
b) false
16) The choice of material as well as process is made at the early design stage because
type of material affects the detailed aspects of
a) design
b) cost
c) design as well as cost
d) none of the above
17) Centralized control is suitable for heavy duty machine tools.
a) true
b) false
18) Machine structures includes ____________
a) Beds
b) Columns
c) Box-type housings
d) All of above
19) Production machine tools are
a) used for small batch manufacture
b) normal machine tools fitted with jigs or fixtures
c) used for large production volumes
d) for one-off production of complex geometries
20) The lathe spindles at the nose end have
a) internal screw threads
b) external screw threads
c) no threads
d) tapered threads
______________
Set A

*SLRBB100*

-3-

SLR-BB 100

Seat
No.

T.E. (Mech.) (Part I) Examination, 2014


MACHINE TOOL DESIGN (Professional Elective I) (New)
Day and Date : Wednesday, 10-12-2014
Time : 3.00 p.m. to 6.00 p.m.

Marks : 80

Instructions : 1) Answer any three questions from each Section.


2) Figure to right indicate full marks.
3) Assume suitable data if necessary and mention it clearly.

SECTION I
2. a) Explain with neat block diagram important stages of machine tool design process. 7

b) Define machine tool and classify machine tool on the basis of different criteria. 7
3. a) Explain basic design procedure for machine tool structure.

b) What are different functions and types of guide ways ?

4. a) Explain meter in circuit in detail with neat sketch.

b) Classify stepless regulation. Explain any one in detail.


5. Write short note on (Any 3) :

7
(43=12)

i) Meter out circuit


ii) Antifriction bearing
iii) Spindle material
iv) Stiffness improvement method.
Set A

SLR-BB 100

-4-

*SLRBB100*

SECTION II
6. a) Explain with neat sketch spindle end.

b) Explain ergonomically consideration in design of control member with


examples.

7. a) Explain ergonomic aspects levers.

b) Explain preloading of antifriction bearing with neat sketch.


8. a) Explain electrical automatic control system.

b) Explain manual automatic control system with neat sketch.


9. Write short note on (Any 3) :

7
7

7
(43=12)

i) Aerostatic sideways
ii) Stability analysis
iii) DNC
iv) ATC.
_____________________

Set A

SLR-BB 101

*SLR-BB-101*
Seat
No.

Set

T.E. (Mechanical) (Part I) (New) Examination, 2014


MACHINE DESIGN I
Day and Date : Friday, 12-12-2014
Time : 3.00 p.m. to 6.00 p.m.

Max. Marks : 100

Instructions : i) Q. No. 1 is compulsory. It should be solved in first 30 minutes in


Answer Book Page No. 3.
ii) Answer MCQ/Objective type questions on Page No. 3 only. Dont
forget to mention, Q.P. Set (A/B/C/D) on Top of Page.
iii) Figures to the right indicate full marks.
iv) Assume suitable data if necessary and mention it clearly.
Objective Question Paper
Duration : 30 Minutes

Marks : 20

1. Objective type Questions :


A) Match the following :
a) Knuckle Pin Design
b) Helical Spring Design
c) Fulcrum Pin Design
d) Muff Coupling Design
e) Key Design

5
p)
q)
r)
s)
t)

Shear and Crushing load


Pure Shear load
Shear and Bending load
Combined torsion and Shear load
Crushing load

B) Single correct answer type objective question :

1) Hardness is the resistance for materials to


a) Temporary deformation

b) Permanent deformation

c) Fracture

d) Both a) and b)

2) Roughness value (microns) for honing-lapping is


a) 0.5 to 0.05

b) 2.5 to 0.25

c) 3.5 to 1.5

d) 6.5 to 0.5

3) Knuckle joint is used to transmit _________ from one rod to another rod.
a) Power

b) Load

c) Frictional Force

d) Both a) and b)

4) The maximum principal stress theory is the proper choice for


a) Brittle material

b) Ductile material

c) Nonmetals

d) Both a) and b)

5) The endurance limit (Sse) of component subjected to fluctuating torsional shear stresses
is obtained from endurance limit in reversed bending using distortion energy theory is
a) Sse=0.5se

b) Sse=0.557se

c) Sse=0.577se

d) Sse=0.555se
P.T.O.

SLR-BB 101

*SLR-BB-101*

-2-

C) Multiple correct answers objective questions :

10

1) The causes of stress concentration is


a) Variation is material property

b) Variation of load

c) Variation of factor of safety

d) Variation in cross sectional area

2) Nondestructive testing is classified into


a) Crack detection by ultra-sonic detection
b) Radio graphical examination by using X-rays
c) Visual examination
d) Sonographic examination
3) Terms related to shaft design are
a) Bending stress
b) Torsional shear stress
c) Combined bending and torsional shear stress
d) Bending pressure
4) The function of spring are
a) To absorb shock

b) To store energy

c) To measure force

d) None of the above

5) Fatigue design can be obtained for


a) Finite life

b) Intermediate life

c) Infinite life

d) Average life
______________

Set A

*SLR-BB-101*

-3-

SLR-BB 101

Seat
No.

T.E. (Mechanical) (Part I) (New) Examination, 2014


MACHINE DESIGN I
Day and Date : Friday, 12-12-2014
Time : 3.00 p.m. to 6.00 p.m.
Instructions :

Marks : 80

i) Answer any two questions from each Section.


ii) Figures to the right indicate full marks.
iii) Assume suitable data if necessary and mention it clearly.
SECTION I

2. a) What is the factor of safety ? Explain the guidelines for selecting the magnitude
of factor of safety in designing various machine elements.

b) Explain IS coding for steels with suitable examples.

c) Conveyor running at 700 RPM is driven by electric motor of rating 10 KW at 1440 RPM
using V belt. Centre distance between pulleys is 1 m. For 24 hr. continuous operation,
select suitable belt using data sheet supplied with question paper.
3. a) Discuss with neat sketch the design of various parts of turn buckle with
relevant design expressions and standard proportions.

10

10

b) Design a right angled bell crank lever as shown in Fig. Load W = 5500N. The lever is
made of forged steel, assume
Perm. tensile bending stress = 75 MPa
Perm. shear stress = 60 MPa
Perm. bearing stress = 10 MPa.

assume b = 3t and length of pin at F = 1.25 diam. of pin; design pin at F and
find width and thickness of lever.

10

Set A

SLR-BB 101

-4-

*SLR-BB-101*

4. a) A machine component is subjected to two-dimensional stresses. The tensile stress


in the X direction varies from 40 to 100 N/mm2 while the tensile stress in the Y
direction varies from 10 to 80 N/mm2. The frequency of variation of these stresses
is equal . The corrected endurance limit of the component is 270 N/mm2. The ultimate
tensile strength of the material of component is 620 N/mm2. Determine the factor of
safety used by the designer.

10

b) A forged steel bar, 40 mm in diameter is subjected to a reversed bending stress of


300 N/mm2. The bar is made of steel 40C8 (Sut = 650 N/mm2). Calculate the life of
the bar for a reliability of 90%.
Take : Surface Finish factor = Ka = 0.43, Size factor, Kb = 0.85,
Reliability factor, Kc = 0.897.

10
SECTION II

5. a) What are design considerations for forging ?

b) Discuss the merits and demerits of solid shaft and hollow shaft.

c) A rotating shaft, 45 mm in diameter is made of plain carbon steel 40C8


(Syt = 380 N/mm2). It is subjected to a steady torsional moment of 225 N-m and
Maximum bending moment of 850 N-m. Calculate the factor of safety based on
i) Maximum principal stress theory.
ii) Maximum shear stress theory.

10

6. a) Draw the labeled sketch for semi elliptical leaf spring used in automobiles
and explain nipping of-leaf spring.

b) Explain design with non-metals.

c) A closed coil helical compression spring of 16 active coils having spring


stiffness of 120 N/mm is cut in to two springs of 06 and 10 turns. Determine the
spring stiffness of resulting spring. If it is cut into two equal springs,
what will be the spring stiffness of new springs ?

10

7. a) Compare welded and riveted joint.


b) Explain the design of muff coupling with the help of suitable sketch.
c) The bracket as shown in fig. is to carry a load of 45 kN. Determine the size of the
rivet if the shear stress is not to exceed 40 MPa. Assume all rivets of
the same size.

Fig

5
5
10

Set A

*SLR-BB-101*

SLR-BB 101

-5-

Table 1 : Dimensions of standard cross-sections

Belt Section

Width Thickness Minimum pitch diameter


W (mm)
T(mm)
of pulley (mm)

13

125

17

11

200

22

14

300

32

19

500

38

23

630

Table 2 : Conversion of inside length to pitch length of the belt


A

Difference between pitch length


and inside length (mm)

36

43

56

79

92

SPEED OF FASTER SHAFT (r.p.m.)

Belt section

Set A

SLR-BB 101

*SLR-BB-101*

-6-

Table 4 : Power rating of V-belts


( 5 = 180 ; speed of the faster pulley = 1440 r.p.m.)
(D = pulley diameter (mm) ; PR = power rating in kW)
Section
A
Section
B
Section
C
Section
D

75

80

85

90

100

106

112

118

125

1.12

1.38

1.50

1.63

1.81

2.00

140

150

160

170

180

190

200

PR 2.24 2.46 2.77

3.30

3.60

4.00

4.39

4.77

5.23

224

236

250

265

280

300

315

PR 6.14 6.81 7.68

8.28

9.40

PR 0.73 0.86 0.99


D

125

200

350

132

212

375

400

10.10 11.10 12.10

12.50

425

PR 15.7 17.5 19.3 20.60

5 (degrees)
Fig. 2 Correction factor for arc of contact (V-grooves on both pulleys)
Set A

*SLR-BB-101*

-7-

SLR-BB 101

Table 6 Correction factor Fi for belt length


(Li = nominal inside length of the belt in mm)

_____________________

Set A

Set A

SLR-BB 103

*SLRBB103*
Seat
No.

Set

T.E. (Mech.) (Part II) Examination, 2014


MACHINE DESIGN II
Day and Date : Tuesday, 25-11-2014
Time : 10.00 a.m. to 1.00 p.m.
Instructions :

Max. Marks : 100

1) Q. No. 1 is compulsory. It should be solved in first 30 minutes in


Answer Book Page No. 3.
2) Answer MCQ/Objective type questions on Page No. 3 only. Dont
forget to mention, Q.P. Set (A/B/C/D) on Top of Page.
3) Figures to the right indicate full marks.
MCQ/Objective Type Questions

Duration : 30 Minutes
I.

Marks : 20

Match the pair :

List (i)

List (ii)

A) Form factor

P) Inaccuracies in tooth profile

B) Minimum running friction

Q) Hydrostatic bearing

C) Minimum starting friction

R) Pneumatic cylinder

D) Dynamic load on gear tooth

S) Boiler shell

E) Thick cylinder

T) Hydrodynamic bearing
U) Number of teeth

II. Multiple Choice questions (Carry two marks each) :

A) The Sommerfeld number depends upon


a)

ZN
P

b)

c
d

c)

d
c

d)

PN
z

B) For maximum power transmission condition for gears


a) Wear strength must be high
b) Beam strength must be high
c) Dynamic load must be equal to static load
d) Factor of safety must be one

P.T.O.

SLR-BB 103

*SLRBB103*

-2-

C) Optimum design can be carried out for the following cases by using Johnsons method
a) Normal specification

b) Abnormal specifications

c) Redundant specification

d) All of these

D) In hydrostatic bearing, there is


a) metal to metal contact
b) No metal to metal contact
c) Partial metal to metal contact
d) Starting friction is very less
III. Single correct answer objective questions (Carry one mark each) :

A) Bevel gears used for velocity ratio as one and for perpendicular shafts, are called as
a) Spiral bevel gears

b) Crown gears

c) Mitre gears

d) Hypoid gears

B) The rolling contact bearings are known as


a) Hydrodynamic bearing
b) Thick film lubricated bearing
c) Thin film lubricated bearing
d) Anti friction bearing
C) For velocity ratio of 52, the number of starts on the worm preferably will be
a) Two

b) Four

c) One

d) Six

D) The type of end closure for which the thickness required will be minimum is
a) Hemispherical end closure
b) Torrispherical end closure
c) Flat end closure
d) Elliptical end closure
E) The bearing number 6204 specifies the bore diameter in mm as
a) 40

b) 20

c) 15

d) 17

F) In which gear teeth slight variation in centre distance does not affect the velocity ratio ?
a) Involute

b) Cycloidal

c) Spiral

d) None of these

G) The objective of preloading of bearing is to increase


a) Static load capacity

b) Dynamic load capacity

c) Reliability

d) Fatigue life
______________
Set A

*SLRBB103*

SLR-BB 103

-3-

Seat
No.

T.E. (Mech.) (Part II) Examination, 2014


MACHINE DESIGN II
Day and Date : Tuesday, 25-11-2014
Time : 10.00 a.m. to 1.00 p.m.
Instructions :

1)
2)
3)
4)

Marks : 80

Solve any two questions from each Section I and II.


Make necessary Assumption, if required and mention it clearly.
Figures to the right indicate full marks.
Make suitable assumptions, if required.
SECTION I

1. a) Write note on gear lubrication.

b) A 20 full depth involutes spur gear pair is used to transmit 10 KW power from and electric
motor running at 1440 rpm to machine running at 500 rpm. The number of teeth on pinion
are 25. The pinion and gear are made of alloy steel (Sut = 800 N/mm2). The service factor
and factor of safety are 1.25 and 1.5 resp. The face width of gear tooth is 10 times of
module. The gears are to be case Hardened to 500 BHN. Design gear pair by using
velocity factor to account for Dynamic load. Use Cv = 6/(6 + v), Y for 20 teeth = 0.3393. 10
c) Derive the expression for maximum power transmission condition for spur gears.

2. a) The following data is given for a pair of helical gears made up of steel
Speed of pinion = 720 r.p.m.

Number of teeth on pinion = 35

Gear ratio = 2 :1

Normal module = 5 mm

Face width = 50 mm

Helix angle = 23

Normal pressure angle = 20

Ultimate tensile strength = 450 N/mm2

Surface hardness required = 300 BHN

Factor of safety = 2

Service factor = 1.25

Grade of machining = 8

For grade 8
Tooth error, e = 16 + 1.25 [mn +0.25] microns
Assume Lewis form factor Y for formative number of teeth on pinion as Y = 0.3986
Calculate :
i) The beam strength
ii) The dynamic load by Spotts equation

ii) The wear strength


iv) Power transmitting capacity.

14

b) Explain the virtual number of teeth and its significance in the design of Helical
gears.

3. a) Give the classification of pressure vessel as according to the IS 2825 : 1969 code.

6
Set A

SLR-BB 103

*SLRBB103*

-4-

b) A high pressure cylinder consists of an inner cylinder of inner and outer diameters
200 mm and 300 mm respectively. It is jacketed by an outer cylinder of outside diameter
400 mm. The difference between the outer diameter of the inner cylinder and the inner
diameter of the jacket before assembly is 0.25 mm. Calculate the shrinkage pressure and
the maximum tensile stress induced in any of the cylinders.
Assume E = 207 kN/mm2.

c) Write a note on openings and nozzles in pressure vessels.

SECTION II
4. a) Describe with neat figures, the various types of mounting arrangements for
bevel gearing.

b) Explain thermal considerations in case of worm gears.

c) A pair of worm and worm wheel is designated as 2/52/10/4

10

10 kW power at 720 rpm is supplied to the worm shaft. The coefficient of friction is 0.04
and the pressure angle is 20. Calculate the tangential, axial and radial components of
the resultant gear tooth force acting on the worm wheel.
5. a) Derive the Reynolds equation for the pressure distribution in a journal bearing
with thick film lubrication in axial and along the circumference.

10

b) A pair of straight bevel gears has a velocity ratio of 2 : 1. The pitch circle diameter of the
pinion is 80 mm at the large end of the tooth. 5 kW power is supplied to the pinion, that
rotates at 800 rpm. The face width is 40 mm and the pressure angle is 20. Calculate the
tangential, radial and axial components of the resultant tooth force acting on the pinion.
10
6.

a) A ball bearing is operating on a work cycle consisting of three parts; a radial load of
3000 N at 1440 r.p.m. for one quarter cycle, a radial load of 5000 N at 720 r.p.m. for one
half cycle, and a radial load of 2500 N at 1440 r.p.m. for the remaining cycle. The expected
life of bearing is 10,000 Hrs. Calculate the
i) Average speed of rotation

ii) Equivalent load

iii) Life in million revolutions and iv) Dynamic load carrying capacity of the bearing.
b) Following data is given for a 360 hydrodynamic bearing :
radial load = 3.2 kN

journal speed = 1490 r.p.m.

journal diameter = 50 mm

bearing length = 50 mm

radial clearance = 0.05 mm

viscosity of lubricant = 25 cP

10

Assuming that the total heat generated in the bearing is carried by the total oil flow in the
bearing, calculate :
i) Coefficient of friction
ii) Power lost in friction
ii) Minimum oil film thickness
iv) Flow requirement in lit/min and
v) Temperature rise
r f = 3.22 and S = 0.121 h0 = 0.4
For d1 = 1 , use c
c
Q
rcnsI = 4.33

()

10

( )

_____________________

Set A

SLR-BB 104

*SLRBB104*
Seat
No.

Set

T.E. (Mech.) (Part II) Examination, 2014


TOOL ENGG.
Day and Date : Wednesday, 26-11-2014
Time : 10.00 a.m. to 2.00 p.m.
N.B. :

Max. Marks : 100

1) Q. No. 1 is compulsory. It should be solved in first 30 minutes


in Answer Book Page No. 3.
2) Answer MCQ/Objective type questions on Page No. 3 only.
Dont forget to mention, Q.P. Set (A/B/C/D) on Top of Page.
MCQ/Objective Type Questions

Duration : 30 Minutes

Marks : 20

1. A) Single correct answer type questions :

(52=10)

1) Material having lowest cutting speed is


a) C.I.

b) Bronze

c) Brass

d) High C steel

2) Tool signature consists of


a) 4 elements

b) 5 elements

c) 6 elements

d) 7 elements

3) Helix angle of drill tool for steel material is


a) 125

b) 104

c) 114

d) 118

4) In punching operation clearance is provided in


a) Punch

b) Die

c) Punch and Die

d) No clearance is provided

5) In tool specification 0-8-6-6-8-90-1 nose radius is


a) 0 mm

b) 8 mm

c) 6 mm

d) 1 mm

P.T.O.

SLR-BB 104

*SLRBB104*

-2-

B) MCQ with multiple correct answers :

(52=10)

6) Jig is employed for


a) Facing

b) Reaming

c) Centre drilling

d) Spot facing

7) High rake angle of tool means


a) Greater tool life

b) Greater strength

c) Lower lip angle

d) High lip angle

8) Fixed cost include


a) Labour cost

b) Power cost

c) Setup cost

d) Cost of machine

9) For manufacturing washer die used is


a) Compound die

b) Combination die

c) Gay and follow die

d) Progressive die

10) Machinability Index is determined w.r.to following


a) Tool life

b) Surface finish

c) Machining conditions

d) Operator

______________

Set A

*SLRBB104*

SLR-BB 104

-3-

Seat
No.

T.E. (Mech.) (Part II) Examination, 2014


TOOL ENGG.
Day and Date : Wednesday, 26-11-2014
Time : 10.00 a.m. to 2.00 p.m.

Marks : 80

Instructions : 1) Q. 1 and Q. 5 are compulsory. Attempt any two questions from


remaining.
2) Assume suitable data wherever necessary.
3) Figures to the right indicates full marks.
4) All the dimensions are in mm.
SECTION I
1. Design press tool for the component shown in Fig. I and make the necessary
calculations. Draw two view of the same.
20

Material Aluminium
Punched hole 5
t = 5 mm

s = 130 N/mm2

Figure I
OR

Design a draw die which is having a diameter of 50 mm and height of 80 mm, made of mild
steel. The thickness is 0.8 mm and inner radius is 1.8 mm. Make the necessary calculations
and draw two views. vTS = 150 N/mm2.

20

2. a) Following data refers to the orthogonal cutting.


5
r = 0.4 (chip thickness ratio)
= 10 (rake angle)
Calculate : 1) Shear angle 2) Shear strain
b) In turning operation it was observed that, tool life was 100 mins and 50 mins at
cutting speed of 25 m/min and 100 m/min respectively. Find out tool life at
200 m/min, under same cutting conditions.
5
3. a) Differentiate between orthogonal and oblique cutting.
5
b) Discuss different types of chip in detail.
5
4. Write short notes on any two :
10
a) What is cutting force ? What are the methods of reducing cutting force ?
b) C.O.P.
c) Cutting tool materials.

Set A

SLR-BB 104

-4-

*SLRBB104*

SECTION II

5.

2 holes 8
slot 30 6 3
4 holes 3

Design a drill jig for a component shown in figure II for 2 holes of 8 .


OR
Design a slotting fixture for a component shown in figure II for 3063 slot.

20

6. a) A tixture costing Rs.2,000/-, the old fixture which originally cost Rs. 1,400/- has
scrap value of Rs. 500/. The new fixture will save 20 paise/piece and % of overhead
charged to this fixture is 30%. Taking M = 3%, T = 24%, I = 16 % and amortization
is 1.5 years. Calculate no. of pieces which must be produced to break even.
So that, the fixture may pay for insert in one year.
b) Explain 3 2 1 principle with a neat sketch.
7. a) Draw tool geometry of a single point cutting tool and explain the same.
b) Describe the nomenclature of drill tool.
8. Write short notes on any two :
1) Return On Investment (ROI)
2) Types of jig bush
3) Draw a neat sketch of button clamp.

5
5
5
5
10

_____________________

Set A

SLR-BB 105

*SLRBB105*
Seat
No.

Set

T.E. (Mechanical) (Part II) Examination, 2014


HEAT AND MASS TRANSFER
Day and Date : Thursday, 27-11-2014
Time : 10.00 a.m. to 1.00 p.m.
Instructions :

Max. Marks : 100

1) Assume suitable data if required.


2) Use of scientific calculator is allowed.
3) Q. No. 1 is compulsory. It should be solved in first 30 minutes in
Answer Book Page No. 3. Each question carries one mark.
4) Answer MCQ/Objective type questions on Page No. 3 only. Dont
forget to mention, Q.P. Set (A/B/C/D) on Top of Page.

MCQ/Objective Type Questions


Duration : 30 Minutes

Marks : 20

1. 1) Thermal conductivity is expressed as


a) W/mK
b) W/m2K

c) W/hmK

d) W/m

2) The radial heat transfer rate through hollow cylinder increases as the ratio of outer radius
to inner radius
a) Decreases
b) Increases
c) Constant
d) None of the above
3) The quantity of heat radiation depends on
a) Area of body only
c) Temperature of body only

b) Shape of body only


d) All of the above

4) The overall heat transfer co-efficient is used in case of


a) Conduction
b) Radiation
c) Convection
d) Conduction and convection
2

5) The relation T = 0 is refered to as _________ equation.


a) Poissions
b) Laplace
c) Fourier Heat Conduction
d) None of the above
6) Fins are provided on surface to __________
a) Increase temperature gradient
b) Increase heat transfer co-efficient
c) Increase heat transfer area
d) All of the above
7) Thermal diffusivity is
a) Dimensionless parameter
c) Physical property of material

b) Mathematical formula only


d) Function of temperature

P.T.O.

SLR-BB 105

*SLRBB105*

-2-

8) Planck law is applicable to _________ radiations.


a) Monochromatic
b) Thermal
c) Temperature
d) None of the above
9) According to Weins Law, the wavelength corresponding to maximum energy is proportional
to
c) T 3
d) T 4
a) T
b) T 2
10) At thermal equilibrium absorptivity is _________ emissivity.
a) Greater than
b) Lesser than
c) Equal to
d) Unpredictable
11) In air preheater for boiler heat is least transferred by
a) Radiation
b) Conduction
c) Convection
d) Conduction and convection
12) In which of following cases heat is transferred by all three modes viz., Conduction,
Convection and Radiation ?
a) Steam Condenser
b) Boiler
c) Electric heater
d) All of the above
13) Nusselt number for forced convection is a function of _________ number and
________ number.
a) Re, Pr
b) Re, Gr
c) Pr, Gr
d) None of the above
14) Why are multipass heat exchangers used ?
a) To obtain high heat transfer co-efficient
b) To reduce the pressure drop
c) To get the compact unit
d) All of the above
15) The steam condenser in a thermal power plant is __________ type heat exchanger.
a) Recuperator
b) Direct contact
c) Regenerator
d) None of the above
16) The velocity profile for fully developed flow in a tube is
a) Hyperbolic
b) Linear
c) Parabolic

d) Exponential

17) For gases Prandtl number is


a) Near unity
b) 5 50

c) 60 100

d) 110 200

18) The unit of mass transfer co-efficient is


b) W/m2 k
a) m2/s

c) m/s2

d) None of these

19) ________ number correlate the relative thickness of the hydrodynamic and thermal
boundary layer
a) Mach number
b) Nusselt number
c) Grashoff number
d) Prandtl number
20) The convective heat transfer co-efficients for boiling and condensation usually lie in the
range of
b) 200 2500 W/m2K
a) 50 500W/m2K
c) 300 5000 W/m2K
d) 2500 10000 W/m2K

______________

Set A

*SLRBB105*

-3-

SLR-BB 105

Seat
No.

T.E. (Mechanical) (Part II) Examination, 2014


HEAT AND MASS TRANSFER
Day and Date : Thursday, 27-11-2014
Time : 10.00 a.m. to 1.00 p.m.
Instructions :

Max. Marks : 80

1) Attempt any two questions from each Section.


2) Assume suitable data if required.
3) Use of scientific calculator is allowed.
SECTION I

2. a) Derive an expression for steady state one dimensional heat flow through a hollow cylinder
without heat generation.
7
b) Derive general heat conduction equation in Cartesian co-ordinate for isotropic materials.

c) A wire of 6.5 mm diameter at a temperature of 60C is to be insulated by material having


k = 0.174w/mK. Convective heat transfer co-efficient h = 8.722W/m2K. The ambient
temperature is 20C. Find maximum percentage increase in heat dissipation when wire is
insulated.

3. a) Derive an expression for critical radius of insulation for cylinder and state its applications.

b) Define fin effictiveness and fin efficiency. Explain the effect of Biot number on effectiveness
of fins.

c) An aluminium rod 2.5 cm in diameter and 10 cm long protrudes from a wall surface which
is maintained at 250C. The rod is exposed to an environment at 15C. convect heat
transfer co-efficient is 15 W/m2K. Calculate heat lost by rod. Assume rod tip is insulated.
Take K for aluminium = 200 W/mK. Also find fin efficiency and temperature at end of fin.

4. a) Write a short note on Laws of Radiation.

b) Explain following terms :


i) Solid angle
ii) Kirchoffs law.

c) Find the percentage reduction in heat transfer with a polished aluminium radiation shield
with emissivity 0.05 when placed between two very large parallel plates with emissivities
0.5 and 0.3.

Set A

SLR-BB 105

-4-

*SLRBB105*

SECTION II
5. a) Define Reynolds number, Prandtl number, Nusselt number, Grashoff number and explain
their physical significance.

b) Explain with neat sketch boiling curve and its different regions.

c) Air at 20C is flowing over a vertical plate of dimensions 1m0.5m. The plate is maintained
at 180C. Find the heat loss from the plate. Assume 0.5m side to be vertical.
Given : properties of air at (180 + 20)/=100C are v = 23.18106m2/s k = 0.0321W/mK
Pr = 0.688. Use correlation Nu = 0.13(Gr.Pr)1/3.

6. a) Derive an expression for determining the effectiveness of parallel flow heat exchanger.

b) A counter double pipe heat exchanger using superheated steam to heat the water. The
water flow rate is 10500 kg/h. The steam enters heat exchanger at 180C and leave at
110C. The inlet an exit temperatures of water are 30C and 70C respectively. If overall
heat transfer co-efficient is 814W/m2K, calculate the heat transfer area. What would be
the increase in area if fluid flows were parallel ?

c) Air at 30C is flowing over a horizontal tube with a velocity of 25m/s. The tube diameter is
5cm. The tube surface temperature is 124C. Calculate the heat loss per unit length from
tube surface.
Take : properties of air at (124 + 30)/2=77C are v = 20.92106m2/s k = 0.03W/mK Pr = 0.7
Use correlation Nu = 0.027(Re)0.805.(Pr)0.33.
7. a) Explain in detail about Fouling factor in heat exchangers.
b) State analogy between momentum, heat and mass transfer.

6
6
7

c) With Buckinghams theorem establish a model correlation in terms of dimensionless


groups for heat transfer co-efficient of forced convection.

_____________________

Set A

SLR-BB 106

*SLRBB106*
Seat
No.

Set

T.E. (Mech.) (Part II) Examination, 2014


AUTOMATIC CONTROL ENGINEERING
Day and Date : Friday, 28-11-2014
Time : 10.00 a.m. to 1.00 p.m.

Max. Marks : 100

Instructions : 1) Q. No. 1 is compulsory. It should be solved in first 30 minutes

in Answer book Page No. 3. Each question carries one mark.


2) Answer MCQ/Objective type questions on Page No. 3 only. Dont
forget to mention, Q.P. Set (A/B/C/D) on Top of Page.
MCQ/Objective Type Questions
Duration : 30 Minutes

Marks : 20

Type (1) :
1.

4
Column I

A)
B)
C)
D)

M
C
M
V
V
C
M
C

Column II
KH

P)

Q) K a1 . K H

R) K
G2

S) A K G1

Type (2) :
2. A Refrigerator is an example of closed loop control system.
A) True
B) False

3. If X1, X2 and X3 are the state variables and if X2 = D X1 , X3 = D X2 , then the technique used is
series programming.
A) True
B) False

4. In Force-Voltage analog, damping coefficient B is analogus to resistance R.


A) True
B) False

5. If Gain Margin is ve, the system is stable


A) True
B) False

P.T.O.

SLR-BB 106

*SLRBB106*

-2-

Type (3) :
6. In inverse-analog for a compressible fluid flow system, the mass flood rate is analogous to
A) Current
B) Voltage
C) Resistance
D) Capacitance

7. Traffic signal system is an example of


A) Open loop control
C) Manual control

1
B) Closed loop control
D) P-control

8. In a feed back control system, if G1 is the feedforward function H1 is the feedback function to
give negative feedback, then the closed loop T.F C(s)|R(s) is given by
H1
G1
H1
G1
A)
B)
C)
D)
1 + G1H1
1 G1H1
1 G1H1
1 + G1H1
9. A force-voltage analog is also called
A) Direct analog
B) Indirect analog

1
C) Inverse analog

D) Analogus analog

10. Effect/Effects of addition of zeros is/are


A) System becomes unstable
B) Gain margin decreases
C) The root of locus changes its shape and shifts to the left half of s-plane
D) All of above

11. When the root locus does not cross jw-axis then gain(K) is
A) Positive
B) 0
C)

D) 1 to 100

12. State space approach is mostly suitable for


A) Simple linear systems
B) Complex and nonlinear system
C) Simple and derivative system
D) None
13. If a row in Rouths array has all zeros, the system
A) Is unstable
B) Is stable
C) Has two roots in the right plane
D) Has two or more roots symmetrically places about the origin

14. The best control can be achieved with


A) P-control
B) I-control

1
C) P + D control

D) P + I + D control

15. In Magnitude plot of Bode-plots the slope changes at


A) The point of zero magnitude
B) Every corner frequency
C) Gain crossover frequency
D) Phase crossover frequency

16. Off set-error is the characteristic of


A) D-control
B) I-control

1
C) ON/OFF control

17. In Bode-Plots a simple pole will contribute a slope of


A) 20 db/decade
B) 40 db/decade
C) 40 db/decade
______________

D) P-control
1
D) 20 2b/decade
Set A

*SLRBB106*

-3-

SLR-BB 106

Seat
No.

T.E. (Mech.) (Part II) Examination, 2014


AUTOMATIC CONTROL ENGINEERING
Day and Date : Friday, 28-11-2014
Marks : 80
Time : 10.00 a.m. to 1.00 p.m.
Instructions :
1) Answer any three questions from each Section.
2) Figures to the right indicate full marks.
3) Use of non-programmable calculators is allowed.
4) Assume additional suitable data if necessary and state it clearly.
5) Use university graph-papers and semi-log papers if required.
SECTION I
1. a) For the lever system as shown in Fig. (1-a), determine the relations between,

i) f and xii)T and (where, T = Torque applied, = angle of deflection in rad.).


b) Determine overall block diagram representation for a field controlled D.C. motor.
2. a) Obtain the linear approximation to calculate the volume V of a sphere having the
diameter D. If reference value of the diameter is Di = 60 mm, determine the percentage
error in the volume due to this approximation when D = 59 mm.
b) Draw the grounded chair representation for the system as shown in Fig. (2-b) and determine
the relations between,

i) f and x

ii) x and y.

3. a) Reduce the block diagram shown in Fig. (3-a) and obtain C(s)/R(s).

8
6

7
8

b) Explain the need of control and the meaning of automatic control by a suitable example.
5
Set A

SLR-BB 106

-4-

*SLRBB106*

4. a) The block diagram of a feedback control system is as shown in Fig. (4-a). At the reference
point, Vi = 80, Ci = 100 and Mi = 50 and Ui = 40. Determine the new values of C when,

i) V changes to 90 while U is held fixed at 40


ii) U changes to 50 while V is held fixed at 80.

b) Explain the concept of equilibrium in a system under steady state with the help of controller
O/P M against system O/P C plot.

SECTION II
5. a) What do you mean by integral and derivative controller ? State necessary equations and
give the plots.
b) Sketch the root loci diagram for unity feedback system with it transfer function
G(s ) =

10

k (s + 3 )

s s + 2s + 2 (s + 5 ) (s + 9 )
2

6. a) For a flow control system with P-controller, the flow rate is 10m1/hr per% of controller
output initial set point is at 50% and gain Kp = 15%. If flow rate changes from 500 m3/hr.
to 600 m3/hr. Determine the new controller output and the offset error.

b) By using Rouths stability criterion, determine 1 the range of k for stability for the given
characteristics equation.
s4 + 2s3 + (4 + k) s2 + 9s + 25 = 0

b) Plot bode plot and determine the gain margin and phase margin for the given unity feedback
control system

s(s + 0.1s ) (1 + 0.5 s )

8. a) Determine the state space representation using series programming and draw the
computer diagram for a system with
y (t ) =

7
6

7. a) Write short note on angle condition and magnitude condition of root locus.

G(s ) =

2D + 6

(D + 4 ) (2D + 3 ) (3D + 2 )

b) What are various important factors in the system equation considered to draw Bode plot ?
Explain bode-plots for the poles at the origin.

_____________________

Set A

SLR-BB 107

*SLRBB107*
Seat
No.

Set

T.E. (Mechanical Engineering) (Part II) Examination, 2014


CAD/CAM
Day and Date : Saturday, 29-11-2014
Time : 10.00 a.m. to 1.00 p.m.

Max. Marks : 100

Note : 1) Attempt any three questions from Section I.


2) Q. No. 5 is compulsory from Section II and solve
any two from remaining questions.
3) Draw meaningful sketches wherever necessary with pencil
only.
4) Figures to right indicate full marks.
5) Make suitable assumptions, if required and state them clearly.
6) Q. No. 1 is compulsory. It should be solved in first 30
minutes in Answer book Page No. 3. Each question carries
one mark.
7) Answer MCQ/Objective type questions on Page No. 3 only.
Dont forget to mention, Q.P. Set (A/B/C/D) on Top of Page.
MCQ/Objective Type Questions
Duration : 30 Minutes

Marks : 20

1. 1) G 90 preparatory code is used for


a) Absolute presetting
b) Absolute co-ordinate setting
c) Metric unit setting
d) None of the above
2) Miscellaneous function used for Coolant off is
a) M 07
b) M 09
c) M 08
d) M 30

20

3) Listing of relative cutter and work piece positions in manual part programming is
a) Manuscript
b) Programming c) Automation
d) None of the above
4) Creation, analysis, modification and optimization is
a) CIM
b) CAD
c) CAM
5) Modelling means
a) 2D to 3D
b) 3D to 2D
c) 2I/ 2D

d) CAD/CAM
d) All of the above

6) Commercial CAD software package available for kinematics analysis and


Virtual Prototyping.
a) CATIA V5
b) Hypermesh
c) ADAMS
d) Master Cam
7) Concatenations can be done using
a) Viewing coordinate
c) Cylindrical coordinate

b) Polar coordinate
d) Homogeneous coordinate

P.T.O.

SLR-BB 107
8) Reflection about X axis is
a) Y flips
b) X flips

*SLRBB107*

-2-

c) X-Y flips

d) None of the above

9) The coordinate system in which all points are defined with respect to same
origin is
a) Absolute coordinate system
b) Incremental coordinate system
c) Both a) and b)
d) None
10) Lathes, shapers, planers, boring machines are using
a) Multipoint tools
b) Single point tools
c) Both a) and b)
d) None
11) AutoDesk Inventor is ________ software.
a) CAM
b) CAE
c) CAD

d) AutoCad

12) Is not a CAD data Exchange translator


a) IGES
b) STEP
c) DXE

d) GKS

13) The machine zero on lathe is generally set on the machine at


a) Top right side
b) Top left side
c) At top mid position
d) None of the above
14) Machining centre used for machining heavier work pieces with large metal
removal rate is
a) VMC
b) HMC
c) Both a) and b) d) None
15) ________ is a computer assisted part programming language.
a) MCU
b) APT
c) ATC
d) MDI
16) Pro-MACHINICA is
a) Application software
c) Networking software

b) Operating software
d) None of the above

17) Geometric modeling corresponds to


a) Synthesis phase of CAD
b) Analysis phase of CAD
c) Synthesis and Analysis phase of CAD
d) None of the above
18) NC machines used for drilling, boring, reaming, tapping is
a) Contouring type b) Straight cut
c) Point-to-point d) None of the above
19) DC Servo motors are used as drive motors in
a) Open loop control
b) Close loop control
c) Both a) and b)
d) None of the above
20) FANUC,CNUMERIC,SIEMENS are CNC
a) Programming language
b) Controllers
c) Machines
d) None
______________

Set A

*SLRBB107*

-3-

SLR-BB 107

Seat
No.

T.E. (Mechanical Engineering) (Part II) Examination, 2014


CAD/CAM
Day and Date : Saturday, 29-11-2014
Time : 10.00 a.m. to 1.00 p.m.

Marks : 80

Note : 1) Attempt any three questions from Section I.


2) Q. No. 5 is compulsory from Section II and solve any
two from remaining questions.
3) Draw meaningful sketches wherever necessary with pencil
only.
4) Figures to right indicate full marks.
5) Make suitable assumptions, if required and state them clearly.
SECTION I
1. a) Explain CAD/CAM data exchange.

b) Explain GKS, PHIGS and IGES.

2. a) What is the role of CAD/CAM in CIM ? Explain.


b) Explain the role of CAPP in manufacturing industry.

7
6

3. a) Explain the flexibilities associated with FMS.

b) Explain the concept of Group Technology.

4. Write notes on any three :

14

a) B-spline curve
b) Hardware in CAD
c) Input devices in CAD
d) CAD/CAM product cycle.

Set A

SLR-BB 107

-4-

*SLRBB107*

SECTION II
5. a) For following component make a part program on CNC machine.
Work material : Mild steel
Tool materials : HSS
Work size : 120*100*20

10

b) Explain coordinate system group in part programming.

6. a) What are the types of machining centres ? Write detail about horizontal
machining centre.
b) Explain DNC with necessary sketch.

7
6

7. a) Explain machine zero and work zero of machining centre and Turning centre
(Lathe).
b) Differentiate between NC and CNC.

7
6

8. Write short note on (any 4) :


a) Classification of NC machine system
b) Tool magazine
c) Active plane selection group in part programming
d) Advantages and limitations of CNC systems
e) Steps in NC programming.
_____________________

14

Set A

SLR-BB 108

*SLRBB108*
Seat
No.

Set

B.E. (Mech.) (Part I) Examination, 2014


INTERNAL COMBUSTION ENGINE
Day and Date : Tuesday, 2-12-2014
Time : 3.00 p.m. to 6.00 p.m.

Max. Marks : 100

Instructions : 1) Q. No. 1 is compulsory. It should be solved in first 30


minutes in Answer Book Page No. 3. Each question carries
one mark.
2) Use of scientific calculator is allowed.
3) Assume suitable data, if necessary and state it clearly.
4) Answer MCQ/Objective type questions on Page No. 3 only.
Dont forget to mention, Q.P. Set (A/B/C/D) on Top of Page.
MCQ/Objective Type Questions
Duration : 30 Minutes

Marks : 20

1. Choose the correct option :

20

1) Engines of different cylinder dimensions, power and speed are compared on the
basis of
a) Maximum pressure
b) Fuel consumption
c) Mean effective pressure
d) Unit power
2) In an actual SI engine the pumping loss with respect to speed
a) Decreases
b) Increases
c) Remains constant
d) None of above
3) The choke is closed when the engine is
a) Accelerating
b) Hot
c) Cold
4) When an engine is idling it requires
a) No fuel in air
c) Rich fuel air mixture

b) Lean fuel air mixture


d) Stoichiometric mixture

5) Fuel injection pressure in solid injection system is around


a) < 10 bar
b) 10 20 bar
c) 30 50 bar
6) Main advantage of pintaux nozzle is
a) Better cold starting performance
c) Good penetration

d) Idling

d) 200 250 bar

b) Ability to distribute the fuel


d) Good atomization

7) The centrifugal type supercharger is preferable only for


a) Low speed
b) High speed
c) High pressure
d) None of the above
P.T.O.

SLR-BB 108

*SLRBB108*

-2-

8) The advantage of Roots supercharger is


a) High pressure
b) Minimum maintenance
c) Consumes less power
d) Occupies less space
9) Petrol engines are adjusted to give minimum brake specific fuel consumption at
a) No load
b) 20 30% full load
c) About 70% of full load
d) Near full load
10) A diesel engine as compared to petrol engine both running at full load is
a) Less efficient
b) More efficient
c) Equally efficient
d) None of the above
11) The best fuel for CI engines are
a) Straight chain paraffins
c) Branched chain paraffins

b) Aromatics
d) Naphthanes

12) Hydrocarbon emission is maximum in


a) 4 stroke SI engine
c) 2 stroke SI engine

b) 4 stroke CI engine
d) 2 stroke CI engine

13) Addition of normal Heptane


a) Resists auto ignition
c) Does not affect auto ignition

b) Accelerate auto ignition


d) None of the above

14) The self ignition temperature of diesel compared to petrol


a) Is higher
b) Is lower
c) Is same
d) Depends on quality of fuel
15) Reference fuels for knock rating of SI engine fuels would include
a) Iso-octane and alpha methyl naphthalene
b) Normal octane and aniline
c) Iso-octane and n-heptane
d) Iso-octane and n-hexane
16) Compression ratio of SI engine is kept less than CI engine because
a) To reduce the weight of engine
b) For better performance
c) To operate it at high speed
d) To control the pre-ignition of charge
17) In a CI engine the squish is created
a) Towards the end of compression stroke
b) At the end of suction stroke
c) At the beginning of suction stroke
d) During combustion
18) Overhead engine is also known as
a) T head
b) L head

c) F head

19) The octane number of CNG is approximately


a) 97
b) 120
c) 87

d) I head
d) 77

20) The most popular device of measuring engine output is


a) Electrical dynamometer
b) Hydraulic dynamometer
c) Hand brake dynamometer
d) Water brake dynamometer
______________

Set A

*SLRBB108*

-3-

SLR-BB 108

Seat
No.

B.E. (Mech.) (Part I) Examination, 2014


INTERNAL COMBUSTION ENGINE
Day and Date : Tuesday, 2-12-2014

Marks : 80

Time : 3.00 p.m. to 6.00 p.m.


Instructions : 1) Attempt any two questions from each Section.
2) Use of scientific calculator is allowed.
3) Assume suitable data, if necessary and state it clearly.
SECTION I
2. a) Give detail classification of an I.C. Engine with their applications.

b) Explain deviation in theoretical and actual Otto cycle.

c) Explain limitations of simple carburetor and suggest the methods to overcome


them.

3. a) A sample jet carburetor is required to supply 5 kg of air and 0.5 kg of fuel per
minute. The fuel specific gravity is 0.75. The air is initially at 1 bar and 300 k.
Calculate the throat diameter of the choke for a flow velocity of 100 m/s.
Velocity coefficient is 0.8. If the pressure drops across the fuel metering
orifice is 0.80 of that of the choke. Calculate orifice diameter assuming
10
Cdf = 0.60 and = 1.4.
b) Briefly explain the various supercharging methods for an I.C. engine.

c) Explain significance of Pintle and Pintaux nozzle.

4. a) List three requirements of an injection system. Explain with sketch distributed


type injection system.

b) Explain important four factors considered for engine selection for marine
applications.
c) Explain the need of C.I. engine governing. List the different types of governors.

7
5
Set A

SLR-BB 108

-4-

*SLRBB108*

SECTION II
5. a) Discuss the abnormal combustion in S.I. engine.

b) Explain the combustion phenomenon with distinct phases in C.I. engine.

c) Explain fuel rating of S.I. engine.

6. a) Explain the procedure of Morse Test for S.I. engine.

b) The following details were noted in a test on four cylinders, four stroke engine,
bore 100 mm, stroke 120 mm, speed of engine 1600 rpm, fuel consumption
= 0.2 kg/min; calorific value of fuel is 44000/KJ/Kg; difference in tension on either
side of the brake pulley = 40 Kg; brake circumference is 300 cm. If the mechanical
efficiency is 80%, calculate :
i) Brake thermal efficiency
ii) Indicated thermal efficiency
iii) Indicated mean effective pressure
iv) Brake specific fuel consumption.
10
c) Explain suitability of two present alternative fuels for S.I. engine.
7. a) Write a note on catalytic converter.

5
7

b) Discuss delay period in C.I. engine and its effect on diesel knock.

c) Write a note on an electronic engine management system.

_____________________

Set A

SLR-BB 109

*SLRBB109*
Seat
No.

Set

B.E. (Mech.) (Part I) Examination, 2014


POWER PLANT AND ENERGY ENGINEERING
Day and Date : Thursday, 4-12-2014
Time : 3.00 p.m. to 6.00 p.m.

Max. Marks : 100

Instructions : 1) Q. No. 1 is compulsory. It should be solved in first 30 minutes in


Answer Book Page No. 3. Each question carries one mark.
2) Answer MCQ/Objective type questions on Page No. 3 only.
Dont forget to mention, Q.P. Set (A/B/C/D) on Top of Page.
3) Figures to right indicate full marks.
4) Make suitable assumption if necessary and state it clearly.
MCQ/Objective Type Questions
Duration : 30 Minutes

Marks : 20

1. Choose the correct answer :


1) The layout of electrical equipments in the power plant consist of the
a) Arrangement of bus bar
b) Circuit breakers
c) Transformer
d) All of the above
2) The functions performed by the switch gear are
a) Faulty plant is automatically disconnected
b) To break short circuit
c) To facilitate redistribution of load
d) All of these
3) The function of solar collector is to convert
a) Solar energy in to electricity
b) Solar energy in to radiation
c) Solar energy in to thermal energy
d) All of these
4) Maximum power density available in the wind
a) ( D2 V 3 ) 8

b) ( AVi3 ) 2

5) Objective of energy audit is


a) Identify the quality
c) Either a) nor b)

c) ( D2 Vi2 ) 9

d) ( AVi3 ) 27

b) Identify the cost of various energy inputs


d) Both a) and b)

6) Non conventional energy sources are available in the form of


a) Solar energy, biomass energy, ocean energy, wind energy
b) Bio energy, ocean energy, wind energy, wood energy
c) Ocean energy, wind energy, geothermal energy, wood energy
d) Wood energy, geothermal energy, wind energy, ocean energy
P.T.O.

SLR-BB 109

*SLRBB109*

-2-

7) The turbine which is used in a tidal power plant for getting continuous power is
a) Simple impulse turbine
b) Reversible type
c) Propeller type
d) Any one
8) Conservation of energy means using ________ energy for the same level of activity.
a) More
b) Less
c) Partial
d) Zero
9) Approximately ___________ of all energy is accounted for during the detailed
energy audit.
a) 95%
b) 85%
c) 75%
d) 65%
10) Geothermal power plants as compared to fossil fuel plants load factor have
a) equal
b) lower
c) higher
d) none of these
11) The energy demand increases day by day because of
a) Population increases
b) Industrialization increases
c) Transportation increases
d) All of these
12) Load factor of power station is generally
a) Equal to unity
b) Less than unity
c) More than unity
d) None of the above
13) Demand factor is defined as
a) Average load / maximum demand
b) Maximum demand / connected load
c) Connected load / maximum demand
d) Maximum demand connected load
14) Pumped storage hydroelectric power plant is more suitable use as
a) Peak load plant
b) Base load plant
c) Intermediate load plant
d) All of the above
15) The general type of tariff is
a) bc + y
b) by + c

c) ax2

d) ax + by + c

16) ___________ tariff method is used for industrial customers.


a) Two part tariff
b) Three part tariff
c) Block meter rate
d) Straight line meter
17) Flat plat collector absorbs
a) Direct radiation only
c) Direct and diffuse both

b) Diffuse radiation only


d) Either a) or b)

18) The main purpose of installed generator is to drive the


a) Alternator
b) Rotor
c) Exciter

d) None of these

19) The core of the transformer is made up of


a) Laminations
b) Single block
c) Hollow casting
d) Decomposable material
20) Pyreheliometer is used to measure
a) Beam radiation
c) Global radiation

b) Diffuse radiation
d) All of the above

______________
Set A

*SLRBB109*

-3-

SLR-BB 109

Seat
No.

B.E. (Mech.) (Part I) Examination, 2014


POWER PLANT AND ENERGY ENGINEERING
Marks : 80

Day and Date : Thursday, 4-12-2014


Time : 3.00 p.m. to 6.00 p.m.

Instructions : 1) Answer any two questions from each Section.


2) Figures to right indicate full marks.
3) Make suitable assumption if necessary and state it clearly.
SECTION I
1. a) Discuss the future energy demands in India.

b) Describe with neat sketches various types of loads on power plant and their
variation.

c) The peak power station is 35 mw. The load having maximum demands of
20 mw, 10 nw, 5 mw and 7 mw are connected to the power station. The capacity of
power station is 40 mw and annual load factor is 55%. Find :
i) Average load on the power station
ii) Demand factor
iii) Diversity factor.

2. a) Explain clearly what do you mean by a load curve ? Explain these two with the
help of sketches.
i) Street lighting load curve
ii) Industrial load curve of one shift.

b) Give a classification of circuit breaker in brief and mention its principle.

c) Draw a sketch of power transformer showing all its details. Also explain
major parts of it.

3. Write short note on any four :

(54=20)

a) Pumped storage plant


b) Different load curve
c) Tariff method
d) Selection of generation equipment
e) Power factor improving methods.

Set A

SLR-BB 109

-4-

*SLRBB109*

SECTION II
4. a) With neat sketch represent the various types of sun earth angles.

b) Explain with neat sketch pyranometer.

c) Determine LAT and declination at Ahmedabad (longitude 7240E, latitude


2000N) corresponding to 1430h IST on December 15, (given E-513) std.
time longitude = 82.5.

5. a) Discuss the classification of wind machine.

b) Classify the geothermal resources and explain liquid dominated type


geothermal system.

c) State advantages and disadvantages of wave energy.

6. a) Explain various method of tidal power.

b) What is an objective of Energy Audit ? Explain any one energy flow diagram.

c) Discuss energy conservation in following industry :


i) Sugar Industry
ii) HVAC
iii) Illumination.

_____________________

Set A

SLR-BB 110

*SLR-BB-110*
Seat
No.

Set

B.E. (Mechanical) (Part I) Examination, 2014


OPERATIONS RESEARCH
Day and Date : Saturday, 6-12-2014
Time : 3.00 p.m. to 6.00 p.m.

Max. Marks : 100

Instructions : 1) Type I Questions carry one mark each.


2) Type II Questions carry two marks each.
3) Answer MCQ/Objective type questions on Page No. 3 only.
Dont forget to mention, Q.P. Set (A/B/C/D) on Top of Page.
MCQ/Objective Type Questions
Duration : 30 Minutes

Marks : 20

Type I Questions :
1. In Simplex technique, the conversion of solution is
a) from bad to worse
b) from worse to bad
c) from good to bad

d) none of the above

2. In OR models, physical model is an example of


a) Analogue model
b) Iconic model
c) Symbolic model

d) Descriptive model

3. In a balanced assignment problem always demand and supply units are


a) Zero and one

b) One and zero

c) One and one

d) Zero and zero

4. The degeneracy in transportation problems occurs if number of independent


allocations made is
a) equal to (m + n 1)
b) equal to (m + n)
c) greater than (m + n 1)

d) less than (m + n 1)

5. The dynamic programming problem can be divided into a sequence of smaller


sub-problems called as
a) branches

b) steps

c) stages

d) all of the above

6. In a two person game if there exists no saddle point, then both the players play with
a) pure strategy
b) infinite strategies
c) mixed strategies

d) none of the above


P.T.O.

SLR-BB 110

*SLR-BB-110*

-2-

7. Network of a residential area for LPG gas supply through pipes, is an example of
a) shortest route method

b) minimum spanning tree problem

c) maximal flow problem

d) all of the above

8. Group replacement policy is adapted for


a) large number of identical items
b) low cost items
c) items that fail suddenly

d) all of the above

9. Which of the following criterion is also called as weighted average criterion ?


a) Laplace criterion
c) Minimax criterion

b) Hurwicz criterion
d) Maximin criterion

10. In a project network, the critical path


a) may be shortest path
c) may be longest path

b) is the longest path


d) none of the above

Type II questions :
1. In unbalance transportation problem when supply is more than demand, then
a) dummy destination is added
c) both a) and b)

b) dummy source is added


d) none of the above
2 4
2 5
c) 5/18

2. The value of the game whose payoff matrix is


a) 2/5

b) 18/5

d) 13/18

3. In PERT, if the optimistic time and pessimistic time of an activity are 2 days and
14 days. Then, the variance for the activity will be
a) one day

b) four days

c) two days

d) twelve days

4. When money value changing with time is 14%, the discount factor for the third
year is
a) 1
b) 0.877
c) 0.769
d) 0.674
5. In a LPP if all the constraints are having greater or equal sign, then for maximization
objective the solution is
a) feasible solution

b) unbound solution

c) no solution

d) all of the above


______________
Set A

*SLR-BB-110*

SLR-BB 110

-3-

Seat
No.

B.E. (Mechanical) (Part I) Examination, 2014


OPERATIONS RESEARCH
Day and Date : Saturday, 6-12-2014
Time : 3.00 p.m. to 6.00 p.m.

Marks : 80

Instructions : 1) Answer any three questions from Section I and Section II.
2) Figures to the right indicate full marks.
3) Use of non programmable calculator is allowed.
SECTION I
1. a) Solve the following LPP by graphical method.
Max Z = 5x1 + 8x2,
subject to
2x1 + 4x2 12
2x1 + x2 = 10
5x1 + 2x2 10
x1, x2 = 0.

b) Explain the importance of OR in executive decision making process.

2. a) A marketing company has four territories open and four salesmen available
for an assignment. The following matrix gives monthly sales revenue in
thousands of rupees.
Territories
I

II

III

IV

88

73

58

44

52

63

42

32

Salesmen C

52

63

42

32

33

42

50

25

Find the optimum assignment to maximize the total sales.

8
Set A

SLR-BB 110

*SLR-BB-110*

-4-

b) Construct the dual of the following LPP


Min Z = 5x1 + 3x2
Subject to,
3x1 + x2 10
x1 + x2 6
3x1 + 2x2 6
x1, x2 0.

3. a) The unit transportation costs in rupees are given in the following table :
i) Find IFBS by VAM

ii) Find optimum solution


Destinations

Sources

D Supply unit

300

400

500

250

350

400

Demand units

200

b) Explain slack, surplus and artificial variables used in simplex technique.


4. Write a note on any two of the following :

(72=14)

i) Dynamic Programming
ii) Sensitivity Analysis
iii) Phases in OR study.
SECTION II
5. a) Define the following terms used in Games theory :

i) Payoff matrix
ii) Pure and mixed strategies
iii) Two person zero sum game.

Set A

*SLR-BB-110*

SLR-BB 110

-5-

b) Solve the following competitive game by graphical method and determine the
optimal strategies for both the players.

Player B
a1
Player A a2

b1

b2

b3

b4

b6

6. a) The details for the small project are given below with three time estimates in
days
Activity

Optimistic time

Most likely time

Pessimistic time

12

14

13

12

15

24

14

17

34

35

12

15

45

12

46

14

56

i) Draw the network and calculate the expected project duration.


ii) What is the probability that the project will be completed in 33 days ?
iii) What due date has 90% chance of being met ?
Refer the following standard table :
Z

0.63

0.90

1.28

0.51

0.735

0.815

0.90

0.695

b) Compare PERT and CPM.

10
3

Set A

SLR-BB 110

*SLR-BB-110*

-6-

7. a) The data on operating cost per year and the resale price of certain equipment
whose purchase price is Rs. 10,000 are given below :
Year

Operating cost 1,500 1,900 2,300 2,900 3,600 4,500 5,500


Resale price

5,000 2,500 1,250

600

400

400

400

Determine the optimum replacement age of the equipment.

b) Find the minimum cable length required to connect six bungalows (shown in
the network) given with distance in meters.

8. a) Elaborate the concept of decision making under condition of risk with suitable
example.

b) Explain in brief :
i) Group replacement policy

ii) Types of floats.

_____________________

Set A

SLR-BB 111

*SLRBB111*
Seat
No.

Set

B.E. (Mech.) (Part I) Examination, 2014


FINITE ELEMENT METHOD
Day and Date : Tuesday, 9-12-2014
Time : 3.00 p.m. to 6.00 p.m.

Max. Marks : 100

Instructions : 1)
2)
3)
4)

Answer any two questions from each Section.


Figures to the right indicate full marks.
Assume suitable data if necessary and mention it clearly.
Q. No. 1 is compulsory. It should be solved in first 30 minutes in
Answer book Page No. 3. Each question carries one mark.
5) Answer MCQ/Objective type questions on Page No. 3 only. Dont
forget to mention, Q.P. Set (A/B/C/D) on Top of Page.
MCQ/Objective Type Questions

Duration : 30 Minutes

Marks : 20

1. Choose the correct answer :


1) In finite element method the term finite stands for
a) Finite number of element
b) Finite number of nodes
c) Finite number of equations
d) Finite number of degrees of freedom
2) Finite element commercial software package hypermesh is basically a
a) Preprocessor
b) Post processor
c) Solver
d) None of the above
3) In three dimensional Simplex element the number of nodes located at corners are
a) 6
b) 3
c) 4
d) Infinite
4) The element with quadratic interpolation function is called as
a) Quadratic element
b) Cubic element
c) Linear

d) None of the above

5) If the physical formulation of the problem is described as a differential equation, then the
most popular solution method is the
a) Method of weighted Residuals
b) Variational Formulation
c) Direct stiffness matrix
d) None of the above
6) If the physical problem can be formulated as the minimization of a functional, then the
_____________ is usually used.
a) Method of Weighted Residuals
b) Variational Formulation
c) Direct stiffness matrix
d) None of the above
7) Consider a differential equation Dy( x) + Q = 0 . Suppose that y = h(x) is an approximate
solution to it. Substitution then gives Dh(x ) + Q = R , where R is a
a) Non-zero residual b) Zero residual
c) Resultant
d) None of the above
8) The MWR (Method of Weighted Residual) requires that
a) The weighting functions
c) Error term

W (x) R(x) = 0 where Wi(x) are


i

b) Weighted residual
d) None of the above

P.T.O.

SLR-BB 111

*SLRBB111*

-2-

9) Prandtls stress function is used in analysis of


a) Torsion problem
b) Fluid problem
c) Heat conduction problem
d) Electric conduction problem
10) The coordinate system that are local to each element but are dimensionless and have a
maximum absolute magnitude of unity.
a) Global coordinate system
b) Cartesian coordinate system
c) Natural coordinate system
d) Polar coordinate system
11) The element having constant value of field variable (eg. displacement, temperature) in the
circumferential direction is
a) Planar symmetric b) Repetitive symmetric c) Cyclic symmetric d) Axisymmetric
12) Sum of all the natural coordinate of the node is
a) One
b) Zero
c) Can not predicted
13) In structural analysis problems the primary field variable is
a) Temperature
b) Heat flux
c) Displacement

d) None of the above


d) Stress

14) Bodies which are long and whose geometry and loading do not vary significantly in the
longitudinal direction can be modeled using
a) Plane stress representation
b) Plane strain representation
c) Symmetric representation
d) None of the above
15) Geometrical modeling of Dams with geometrical approximation is
a) Plane stress representation
b) Plane strain representation
c) Symmetric representation
d) None of the above
16) Natural co-ordinate is
a) Local coordinate b) Global coordinate c) Area coordinate
17) Each node of a plate element is having
a) One Dof
b) Three Dof

c) Two Dof

d) Volume coordinate
d) Six Dof

18)

For two dimensional element 2 if node shown by node number 4 is I node the local dof
is shown by
a) U4 U5
b) U3 U4
c) U7 U8
d) U8 U9
19) If the function is of first order then number of sampling point required to solve the integration
by Gauss Quadrature is
a) 1
b) 2
c) 3
d) 4
20) Each node of beam element is having following degree of freedom
a) 3
b) 4
c) 1
d) 2
______________

Set A

*SLRBB111*

-3-

SLR-BB 111

Seat
No.

B.E. (Mech.) (Part I) Examination, 2014


FINITE ELEMENT METHOD
Day and Date : Tuesday, 9-12-2014
Time : 3.00 p.m. to 6.00 p.m.

Marks : 80

Instructions : 1) Answer any two questions from each Section.


2) Figures to the right indicate full marks.
3) Assume suitable data if necessary and mention it clearly.
SECTION I
2. a) Explain finite element procedure.
b) Explain simplification through symmetry.
c) Use finite element method to predict the temperature distribution and heat flow through the
composite skin as shown in fig.

7
6
7

Assuming a unit cross sectional area for each element.


3. a) Derive shape functions in terms of natural co-ordinates (L1, L2, L3) for one dimensional
element with three nodes and a quadratic interpolation function. Plot the same.
b) Explain simplex, complex and multiplex element.
c) What is FEM ? Write its advantages and disadvantages.
4. Write short notes on (any five) :

7
6
7

(54=20)

a) Size and number of elements


b) Principle of minimum potential energy
c) An axisymmetric element
d) Classify and explain weighted residual method
e) Interpolation functions
f)

Applications of FEM.
Set A

SLR-BB 111

-4-

*SLRBB111*

SECTION II
5. a) Solve by Galerkin method and compare u(0.5) with exact solution :
d2u
dx 2

+ u x2 = 0

0 x 1

Boundary conditions u(0)=0,

du
dx

(1) = 1 .

OR
Derive Jacobian matrix of one dimensional higher order quadratic element.
b) Explain dynamic problems.
c) Explain mesh design and refinement.

6
7

6. a) What do you mean by bandwidth ? Show with suitable example that the bandwidth depends
directly on the order that the nodes are numbered.

b) Write one dimensional, two dimensional and three dimensional element and their interpolation
function. Sketch the same.
c) Explain model validity in FEM.

6
7

7. Write short notes on (any five) :

(54=20)

a) Model check
b) Elimination and penalty approach
c) Element distortions
d) Two dimensional elasticity
e) Properties of global stiffness matrix
f)

Geometric approximations.
________________

Set A

SLR-BB 112

*SLRBB112*
S

B.E. (Mechanical) (Part I) Examination, 2014


EXPERIMENT STRESS ANALYSIS (Elective I)
Day and Date : Thursday, 11-12-2014
Time : 3.00 p.m. to 6.00 p.m.

Max. Marks : 100

Instructions : 1) Make necessary assumptions, if required and mention it clearly.


2) Figures to the right indicate full marks.
3) Q. No. 1 is compulsory. It should be solved in first 30
minutes in Answer Book Page No. 3.
4) Answer MCQ/Objective type questions on Page No. 3
only. Dont forget to mention, Q.P. Set (A/B/C/D) on Top
of Page.
Duration : 30 Minutes

MCQ/Objective Type Questions

Marks : 20

1. A. Match the pairs :


Column (1)

(41=4)
Column (2)

a) Foil strain gauges

p) Unknown state of strain

b) Torque of rotating shaft


c) Rosette

q) Strain gauges at 45
r) Sensitive to cross-axis sensitivity

d) Polarizer

s) Unique axis for passing light


t) Better heat dissipation

B. Match the pairs :

(41=4)

Column (1)
a) N-Type Strain Gauge

Column (2)
p) Tilted method

b) P-Type Strain Gauge

q) Generally resistance increases with increase


in tensile strain

c) Oblique incidence

r) Generally resistance decreases with


increase in tensile strain
s) Permanent birefringence

d) Quarter wave plate

t) Insensitive to cross-axis sensitivity


P.T.O.

SLR-BB 112

*SLRBB112*

-2-

C. Solve multiple correct answers :

(42=8)

i) Stress difference is directly proportional to


a) Fringe order
b) Material fringe value
c) Model thickness
d) Light used
ii) Following are the types of strain gauges
a) Semi-conductor
b)
c) Bridge
d)
iii) Brittle coatings can be used for
a) Locating strain directions
b)
c) Increasing model thickness
d)

Electrical resistance
Elliptical
Measuring strains
Interference

iv) Strain gauge type transducers can be used for measuring


a) Pressure
b) Force
c) Angle
d) Length
D. Solve classical objectives :

(41=4)

i) Opposite nature strain gauges are used in


a) Adjacent arms
b) Opposite arms
c) a) and b)
d) All arms
ii) Same nature strain gauges are used in
a) Adjacent arms
b) Opposite arms
c) a) and b)
d) All arms
iii) Output voltage in case of four arm sensitive bridge as compared with one
arm sensitive is
a) Same
b) Double
c) Tripple
d) Four times (quadruplicated)
iv) Babinet Soleil Method is known as
a) Compensation b) Theoretical

c) Stressed

d) None

______________

Set A

*SLRBB112*
S

-3-

SLR-BB 112

B.E. (Mechanical) (Part I) Examination, 2014


EXPERIMENTAL STRESS ANALYSIS (Elective I)
Day and Date : Thursday, 11-12-2014
Time : 3.00 p.m. to 6.00 p.m.

Marks : 80

Instructions : 1) Solve any two questions from each Section I and II.
2) Make necessary assumptions, if required and mention it
clearly.
3) Figures to the right indicate full marks.
SECTION I
2. a) Explain in brief :

i) Isoclinics
ii) Isochromatics.
b) Derive the expression for the light intensity observed through analyzer when
the stressed model is kept in plane polariscope.
10
c) How to distinguish Isoclinics from Isochromatic fringes in plane polariscope ? 4
3. a) Enlist the materials used for sheet casting and explain the procedure for
sheet casting.

b) A circular disc made of photo elastic material is used for calibration. The disc
diameter is 60 mm and its thickness is 6 mm. It is subjected to equal and
opposite compressive loads of 300 N along the vertical diameter. The observed
fringe order at the disc centre and two other points 6 mm and 12 mm along
the horizontal diameter is 0.4, 0.35 and 0.3 respectively. Calculate the average
value of the material fringe constant.

c) Explain Calibration method of Beam Model.

4. a) Explain oblique incidence method for separation of principal stress.

b) Explain the scaling of model results to prototype.

c) What is the purpose of electrical analogy method ? Explain the method.

Set A

SLR-BB 112

*SLRBB112*

-4-

SECTION II
5. a) Discuss the mechanism of Moire fringe formation.

b) Discuss temperature compensation related Wheatstone bridge and derive


expression gauge factor.

12

6. a) State working principle for electrical resistance type strain gauge. Discuss
different types of electrical resistance type of strain gauges with the help of
neat sketches.

b) The strain readings as measured by a T-delta rosette mounted on a steel


component shows the following strain indications :
= 255 m/m,
= 305 m/m,
= 294 m/m and
= 65 m/m.
Calculate the maximum principal strain direction, the principal stresses,
principal strains and the maximum shear stress. Take E = 200 GPa and
= 0.3.


>

12

7. a) Derive the expression for the output voltage of a two arm and four arm sensitive
Wheatstones bridge subjected to compressive and tensile linear strains.
Comment on the output.
10
b) Discuss the methods used for balancing Wheatstone bridge.

10

_____________________

Set A

SLR-BB 113

*SLRBB113*
Seat
No.

Set

B.E. (Mech.) (Part I) Examination, 2014


Elective I : PROCESS ENGINEERING
Day and Date : Thursday, 11-12-2014

Max. Marks : 100

Time : 3.00 p.m. to 6.00 p.m.


Instructions : 1) Assume suitable data wherever necessary and state it clearly.
2) Q. No. 1 is compulsory. It should be solved in first 30 minutes
in Answer book Page No. 3.
3) Answer MCQ/Objective type questions on Page No. 3 only.
Dont forget to mention, Q.P. Set (A/B/C/D) on Top of Page.
MCQ/Objective Type Questions
Duration : 30 Minutes

Marks : 20

1. I) MCQ with more than one answer correct :

10

1) Match the pairs :

(22=4)

a) Special purpose m/c

p) Less investment cost

b) General purpose m/c

q) Fewer m/cs reqd.


r) Single set up
s) Batch production

2) Match the pairs :

(23=6)

a) Operating cost

p) Unpredictable cost

b) Intangible cost

q) Direct labour cost

c) Investment cost

r) Depreciation and taxes


s) Material costs
t) Downtime costs
u) Installation cost
P.T.O.

SLR-BB 113

*SLRBB113*

-2-

II) MCQ with only one answer correct (1 mark each) :


1) Match the pairs :

10
(12=2)

a) Design tolerance stack

p) Tool engineer

b) Process tolerance stack

q) Product engineer
r) Process engineer

2) Match the pairs :

(15=5)

Sequence of operation

Classification of operation

i) Forging of a blank

p) Principle process operation

ii) Machining of spur gear on milling m/c q) Major operation


iii) Key way cutting on slotting m/c

r) Auxiliary operation

iv) Heat treatment of gear

s) Supporting operation

v) Inspection, packaging and dispatch

t) Basic process operation

3) Match the pairs :

(13=3)

a) Job production

p) Capstan lathe

b) Batch production

q) Automatic lathe

c) Mass production

r) Centre lathe
______________

Set A

*SLRBB113*

-3-

SLR-BB 113

Seat
No.

B.E. (Mech.) (Part I) Examination, 2014


Elective I : PROCESS ENGINEERING
Day and Date : Thursday, 11-12-2014
Time : 3.00 p.m. to 6.00 p.m.

Marks : 80

Instructions : 1) Q. No. 2 and Q. No. 6 are compulsory.


2) Attempt any two questions from Q. No. 3, 4, 5 and any two
questions from Q. No. 7, 8, 9.
3) Assume suitable data wherever necessary and state it clearly.
SECTION I
2. Provide following information for the part shown in Fig. 1. The no. of pieces to be
produced is two :
a) Sequence of operation indicating machine selected, holding method, machining
data for each set up.
13
b) Specification of gages and inspection equipments.
4
c) ISO specification or any other commercial specification for min. five tools.
3
3. a) Explain duties and responsibilities of process engineer in an organisation.
b) What do you understand by part print analysis ?

5
5

4. a) What is selective assembly ? What are its merits and demerits ?


b) Explain procedure to study feasibility.

5
5

5. Write notes on any two :


a) Tolerance stack
b) Causes of work piece variation
c) Steps leading to proper sequence of operation
d) Documents released by process engineering dept.

(52=10)

SECTION II
6. Draw process picture sheets for any four operations to be performed on the part
shown in Fig 1. Process picture sheet shall include location, clamping and all
other relevant details.
20
7. a) Discuss cost factors in m/c selection.
b) What are the disadvantages of combining the operation ?

5
5

8. a) Explain advantages of general purpose m/cs. What are their limitations ?


b) Explain various sources of information for the process engineer during
selection of equipment.

5
5
Set A

SLR-BB 113

-4-

*SLRBB113*

9. Write notes on any two of the following :


(25=10)
a) Steps leading to proper sequence of operation b) Retrieval type CAPP
c) Advantages of commercial tooling
d) Low cost processing.

Fig. 1
______________

Set A

SLR-BB 114

*SLRBB114*
Seat
No.

Set

B.E. (Mechanical) (Part II) Examination, 2014


REFRIGERATION AND AIR CONDITIONING
Day and Date : Tuesday, 25-11-2014
Time : 3.00 p.m. to 6.00 p.m.

Max. Marks : 100

Instructions : 1) Attempt any two questions from each Section.


2) Use of non-programmable calculator, steam table, psychometry
chart and tables are allowed.
3) Assume suitable data wherever necessary.
4) Q. No. 1 is compulsory. It should be solved in first 30 minutes in
Answer Book Page No. 3. Each question carries one mark.
5) Answer MCQ/Objective type questions on Page No. 3 only. Dont
forget to mention, Q.P. Set (A/B/C/D) on Top of Page.
MCQ/Objective Type Questions
Duration : 30 Minutes

Marks : 20

1. 1) A reversible refrigerator working between two fixed temperatures.


A) Gives higher COP when working substance has latent heat
B) Gives higher COP when working substance has specific heats
C) Gives the same COP irrespective of working substance
D) None of the above

2) The common air-cooling system used in air-craft is


A) Simple air-cooling
B) Boot-strap
C) Simple air cooling with evaporator
D) None of the above

3) In a VCC the effect of super heating the vapour after the exit of evaporator.
A) Increases the work done
B) Increases the heat rejection to the condenser
C) Increases the refrigeration effect
D) All of the above

4) Use of liquid vapour regenerative heat exchanger is justified because


A) COP of the system improves
B) KW/TOR decreases
C) Suction volume of compressor/TOR decreases
D) Superheating of vapour in heat-exchanger is preferable to superheating in evaporator

5) For the given condenser and evaporator pressures in VCC-cycle, the multi-staging
as compared to single staging will increase the refrigeration capacity
A) More for R-717 than R-12
B) More for R-12 than R-717
C) Same for both
D) May be more or less for R-12 and R-717 depending upon actual condenser pressure
6) For the given evaporator and condenser temperatures, single stage compressor compared
to two stage compressor for the same clearance volume percentage gives.
B) Lower volumetric
A) Higher volumetric
C) Same volumetric

D) None of the above


P.T.O.

SLR-BB 114

*SLRBB114*

-2-

7) A electrolux refrigerator is called as


A) Single fluid absorption system
C) Three fluids absorption system

1
B) Two fluids absorption system
D) None of the above

8) The clearance factor is the ratio of


Vs

A) V
c

Vc

1
Vc

B) V
s

C) V + V
s
c

D)

Vs + Vc
Vc

Where Vc and Vs are clearance and swept volume of the compressor ?


9) The centrifugal compressor is not suitable for a refrigerant plant using
A) R-11
B) R-12
C) R-22
D) R-717

10) For one-ton refrigeration plant, if the heat rejection ratio of condenser is 1.2, the COP of
the plant is
A) 6
B) 5
C) 4
D) 3
11) In a unsaturated air, the condition of vapour is
A) Saturated
B) Superheated
C) Wet

D) None of the above

12) During sensible heating, the DBT of moist air


A) Increases
B) Decreases
C) Remains same

D) None of the above

13) Air washer works as


A) Filter only

D) Cooler only

1
1
1
1

B) Humidifier only

C) Both A) and B)

14) The minimum temperature to which the moist air can be cooled is
A) WBT
B) DPT
C) Any one of A) or B)
D) None of the above

15) The WBT depression is zero when the RH of the air is


A) 100%
B) 80%
C) 60%

16) The WBT of air whose R.H is 100% is


A) >DBT
B) <DBT

D) Zero
1

C) =DBT

17) The air DBT = WBT = 20C, then ET (Effective Temperature) is


A) <20C
B) >20C
C) =20

D) None of the above


1
D) Unpredictable

18) The condition of supply air passing over cooling coil of BF to an air-conditional
space lies at
A) Intersection of RSHF and GSHF lines
B) Intersection of RSHF and saturation curve
C) Intersection of GSHF and saturation curve
D) None of the above

19) The conditioned air supplied to a room must have a capacity to take
A) Sensible heat load
B) Latent heat load
C) Sensible and latent heat load
D) Any of the above

20) In a rectangular duct (a b), (a > b) the aspect ratio is given by

A) ba

B) (a + b)

C) (a b)

D) a b

______________

Set A

*SLRBB114*

SLR-BB 114

-3-

Seat
No.

B.E. (Mechanical) (Part II) Examination, 2014


REFRIGERATION AND AIR CONDITIONING
Day and Date : Tuesday, 25-11-2014
Time : 3.00 p.m. to 6.00 p.m.
Instructions :

Marks : 80

1) Attempt any two questions from each Section.


2) Use of non-programmable calculator, steam table, psychometry
chart and tables are allowed.
3) Assume suitable data wherever necessary.
SECTION I

2. a) A simple air cooled system is used for an airplane having a load of 10 tonnes. The
atmospheric pressure and temperature are 0.9 bar and 10C respectively. The pressure
increases to 1.01325 bar due to ramming. The air is then further compressed in a
compressor to 3-5 bar. The temperature of the air is reduced by 50C in the heat exchanger.
The pressure in the Cabin is 1.01 bar and temperature of air leaving the cabin is 25C.
Determine :
i) Power required
ii) COP of the system. Assume that compression and expansion process as isentropic.

10

b) What is secondary refrigerants ? Explain its use or application by example.

c) Draw a neat diagram of LiBr absorption system and explain its working.

3. a) R-12 vapor compression system operating at a condenser temperature of 40C and on


evaporator temperature of 5C develops 15TR. Determine :
i) Mass flow rate of refrigerant
ii) Volume of refrigerant handled by compressor
iii) HP per TR
iv) COP.

12

Use following data :


T set
C

P (bar)

hf
kJ/Kg

hg
kJ/Kg

sf
kJ/Kg-K

sg
kJ/KgK

Vf m3/Kg

Vg
M6/Kg

2.6096

195

349

0.9831

1.5571

0.7078 103

0.06496

40

9.6065

238

367

1.1298

1.54051

0.7980103

0.01817

Cp = 0.65 KJ/Kg-K
b) Why multi-evaporator system is needed ? Explain any one on p-h diagram.

8
Set A

SLR-BB 114

*SLRBB114*

-4-

4. Attempt any four :

(45=20)

a) What do you mean by TR and show 1TR = 3.516 kW ?


b) Show for Bell Coleman cycle COP =

(rp ) r 11
r

rp = pressure ratio

c) Explain flooded evaporators.


d) Explain VCC with the help of T-S 4 P-h diagram.
e) Short note on nomenclature of refrigerants.
SECTION II
5. a) For a DBT of 38C and RH of 70%. Calculate the following for air if barometric pressure is
1 bar.
i) Specific humidity
ii) Enthalpy of air
iii) DPT
iv) Partial pressure of dry air.

b) What is effective temperature ? Explain factors affecting it.

c) Define :
i) RSHF

6
ii) ESHF

iii) By pass factor.

6. a) A building has the following loads :

12

RSH = 58.15 kW RLH = 14.54 kV


The symmer outside and inside design conditions are :
Outside 40C DB and 27C WB
Inside 25C DB and 50% RH.
70 cmm of ventilation air is used. Determine :
i) Ventilation load
ii) ESHF
iii) ADP
iv) Dehumidified air quantity
Assume by pass factor of 0.15.
b) What are the different factors considered in load estimation sheet for comfort application ? 8
7. a) What do you understand by static and velocity pressure in duct ?

b) Describe the different methods of air conditioning duct design. Why are doupers required
in some systems ?

14

_____________________

Set A

SLR-BB 115

*SLRBB115*
Seat
No.

Set

B.E. (Mech.) (Part II) Examination, 2014


INDUSTRIAL AND QUALITY MANAGEMENT
Day and Date : Thursday, 27-11-2014
Time : 3.00 p.m. to 6.00 p.m.
Instructions :

Max. Marks : 100

1) Figures to the right indicate full marks.


2) Solve objectives on answer sheet separately provided with obj.que.
paper only.
3) Q. No. 1 is compulsory. It should be solved in first 30 minutes in
Answer book Page No. 3. Each question carries one mark.
4) Answer MCQ/Objective type questions on Page No. 3 only. Dont
forget to mention, Q.P. Set (A/B/C/D) on Top of Page.
MCQ/Objective Type Questions

Duration : 30 Minutes

Marks : 20

1. Conventional MCQ :

(201=20)

1) At the ___________ , the conceptual and design abilities and human skills are especially
valuable
A) Middle management level
B) Tope Management level
C) Lower management level
D) All levels
2) Entrepreneurial role, disturbance-handler role are _____________ managerial roles.
A) Interpersonal
B) Informational
C) Decision
D) Leading
3) Territorial departmentation is especially attractive to ____________ firms.
A) Medium Scale
B) Small Scale
C) All Scale
D) Large Scale
4) Self-esteem, status, affiliation with others, affection these comes under
_____________ needs.
A) Basic
B) Primary
C) Third level of
D) Secondary
5) Vroom theory may be stated as
A) Valance = Force Expectancy
C) Result = Balance Expectancy

B) Force = Valance Expectancy


D) Expectancy = Valance Force

6) ______________ is any paid form of non personnel presentation and promotion of goods
and services or ideas by an identified sponsored.
A) Marketing
B) Selling
C) Advertising
D) None of above
7) To earn more profit, the cost of capital should be _____________
A) Maximized
B) Minimized
C) Same
D) Cant Say
8) ______________ funds are required to meet the working capital needs like investment in
the inventories, debtors, bank balance, cash on hand, marketable security etc.
A) Long term
B) Short term
C) Middle term
D) Fixed
9) _______________ management called country club management.
A) 9.1
B) 9.9
C) 5.5
D) 1.9
P.T.O.

SLR-BB 115

*SLRBB115*

-2-

10) _____________ leader uses his or her power very little, if at all, giving subordinates a
high degree of independence in their operations.
A) Autocratic
B) The free rein
C) Democratic
D) Ideal
11) The quality of design is concerned with the ______________ for the
manufacture of the product
A) Exactness of specification
B) Fitness of specification
C) Correctness of specification
D) Tightness of specifications
12) The costs associated with measuring, evaluating or auditing the product come under
_____________
A) Cost of appraisal
B) Cost of prevention
C) Cost of internal failure
D) Cost of external failure
13) In a double sampling plan if the 1st sample is neither good enough nor bad enough then
the decision on acceptance or rejection is based on the evidence of
________________
A) Last two combined
B) First and last combined
C) First and second sample combined
D) First three combined
14) The larger the sample size and acceptance number _____________ is the slope of OC
curve.
A) Straight
B) Curve
C) Steeper
D) Horizontal
15) AOQ stands for _____________
A) Average Outgoing Quality
C) Average Outward Quality

B) Additional Outgoing Quality


D) Advanced Outgoing Quality

16) The number of items in a sample is called as _____________


A) Lot size
B) Batch size
C) Test quantity

D) Sample size

17) The items are subjected to _____________ test must be inspected by sampling inspection
only.
A) Quality
B) Destructive
C) Production
D) Single
18) The ISO : 9000 certificate awarded is normally valid for ____________ years
A) 1 year
B) 2 years
C) 3 years
D) 5 years
19) In any sampling plan if c is the acceptance number then the rejection number is
______________
A) c
B) c 1
C) c + 2
D) c + 1
20) The producer can decrease his risk by submitting the lots of _____________
A) Same as AQL
B) Better than AQL
C) Less than AQL
D) Only AQL
______________

Set A

*SLRBB115*

-3-

SLR-BB 115

Seat
No.
B.E. (Mech.) (Part II) Examination, 2014
INDUSTRIAL AND QUALITY MANAGEMENT
Day and Date : Thursday, 27-11-2014
Time : 3.00 p.m. to 6.00 p.m.
Instructions :

Marks : 80

1) Solve any three questions from Section I.


2) Solve any three questions from Section II.
3) Figures to the right indicate full marks.
SECTION I

2.

3.

4.

5.

a) What are quidelines to overcome weak delegation of authority ?

b) What are causes of accident ? Explain technical causes and human causes.

a) Explain importance, objective and function of maintenance.

b) Distinguish between narrow span and wide span.

a) Explain candidate selection process of company in detail.

b) Explain the term capital, its importance and its classification.

a) Explain Maslows theory and McGregors theory.

b) What are managerial skills and managerial roles ?

SECTION II
6.

7.

a) Name any three quality gurus and their contribution in quality.

b) What is acceptance sampling ? Explain sampling plans and its types.

a) Describe the various steps necessary for obtaining ISO : 9000 standard registration.

b) Explain actual and ideal OC curve for a sampling plan. Also explain all concepts such as
producers risk, consumers risk, AQL, LTPD, AOQL etc.
8.

a) Explain the following as applied to quality control :


1) Appraisal costs

2) Prevention costs 3) Failure costs

b) Explain scatter diagrams and Pareto analysis.

7
4) Optimum costs
6
Set A

SLR-BB 115
9.

*SLRBB115*

-4-

a) The following table gives the number of missing rivets noted at aircraft final inspection :

Air plane
No

No. of missing
rivets

Air plane
No

No. of
Air plane
No. of
missing rivets
No
missing rivets

10

12

19

11

16

11

23

20

14

12

16

21

10

19

13

22

22

11

14

25

23

15

15

15

24

28

16

25

11

17

21

18

14

Find C. compute trial control limits, and plot control chart for C. What values of C would
you suggest for the subsequent period ?
b) Write a note on X R chart.

8
6

_____________________

Set A

SLR-BB 116

*SLRBB116*
Seat
No.

Set

B.E. (Mech.) (Part II) Examination, 2014


MECHATRONICS
Day and Date : Saturday, 29-11-2014
Time : 3.00 p.m. to 6.00 p.m.

Max. Marks : 100

Instructions : 1) Q. No. 1 is compulsory. It should be solved in first 30 minutes


in Answer Book Page No. 3.
2) Answer MCQ/Objective type questions on Page No. 3 only.
Dont forget to mention, Q.P. Set (A/B/C/D) on Top of Page.
3) Use of non-programmable calculator is allowed.
4) Figures to the right indicate full marks.
5) Assume suitable data if necessary and state it clearly.
MCQ/Objective Type Questions
Duration : 30 Minutes

Marks : 20

1. Choose the correct answer :


1) Pyrometer is used to measure
a) Strain
b) Pressure

20
1
c) Displacement

d) Temperature

2) Filter that transmit all frequencies below a defined cut-off frequency are known as 1
a) low-pass filters
b) high pass filters
c) band-pass filters
d) any of these
3) MOSFET can be employed as a control switch for
a) DC motor
b) AC motor
c) Stepper motor d) All the above

4) Which of the basic logic gates are considered as universal gate ?


a) AND gate and OR gate
b) NAND gate and NOR gate
c) AND gate and NAND gate
d) NOR gate and NAND gate

5) The Program Storage Enable (PSEN) pin for 8051 microcontroller is


a) Write signal pin for external program memory
b) Read signal pin for external program memory
c) Address latch enable pin
d) All the above

P.T.O.

SLR-BB 116

*SLRBB116*

-2-

6) Application of latching contact in PLC


a) Performs timing operation
b) ON/OFF control using two separate switches
c) Perform counting operation
d) Serial to parallel data transfer

7) Condition monitoring of machine means


a) Cleaning of machinery
b) Measurement of vibrations
c) Determining the machine status d) Use of sensors in machine

8) Protocol governs
a) Data format
c) Timing

1
b) Error control
d) Memory of data

9) In OSI model the function of session layer is


a) Provide actual user information
b) Deals with communication paths, addressing and routing
c) Sending and receiving message
d) All the above

10) Which of the following sensors is non contact type robotic sensor ?
a) Force sensors
b) Touch sensors
c) Position sensor
d) None of the above

11) The 8051 has _____________ bytes of program memory and ___________
bytes of data memory.
a) 128, 4096
c) 4096, 2048

2
b) 4096, 128
d) 2048, 4096

12) Fibre optic sensor used for


a) Measurement of load
c) Measurement of light intensity

2
b) Measurement of displacement
d) Measurement of torque

13) The octal equivalent of the decimal number (247)10 is


a) 368
b) 367
c) 371
14) Interfacing is required for
a) Electrical buffering/isolation
c) Code conversion

2
d) 374
2

b) Sequencing of data
d) Data movement

15) Micro optical sensor is used for measurement of


a) Displacement b) Torque
c) Rotation

2
d) Temperature

______________
Set A

*SLRBB116*

-3-

SLR-BB 116

Seat
No.

B.E. (Mech.) (Part II) Examination, 2014


MECHATRONICS
Day and Date : Saturday, 29-11-2014
Time : 3.00 p.m. to 6.00 p.m.

Marks : 80

Instructions : 1) Answer any two questions from each Section.


2) Figures to the right indicate full marks.
3) Assume suitable data if necessary and state it clearly.
SECTION I
2. a) Define Mechatronics. Explain microprocessor based engine management
system with a neat sketch.

b) Why signal conditioning is required ? Explain different types of filters.

c) What is an Op-amp ? Explain differential operational amplifier.

3. a) Discuss the construction and working of stepper motors. List the advantages,
limitations and applications.
8
b) Describe the architecture of 8085 microprocessor with neat sketch.

c) Explain the working of relays with neat sketch. Also briefly write about the
gear train types.

4. Write short notes on (any four) :

20

a) ADCs and DACs


b) Data acquisition system
c) Karnaugh maps
d) Hydraulic actuators
e) Registers.

Set A

SLR-BB 116

-4-

*SLRBB116*

SECTION II
5. a) Explain the correct schematic 8051 based washing machine controller.

b) What is delay on, delay off timer ? Draw timing circuit that will switch on an
output for 12 s then switch it off.

c) Explain the forms of bus, star and ring networks.

6. a) Explain the sensors used for machine condition monitoring.

b) Device a system, using a PLC, which can be used to control the movement
of a piston in a Cylinder using 4/2 solenoid operated pilot valve. The piston
is to move in one direction when Proximity sensor at one end of the stroke
closes contact and in the other direction when proximity sensor at other end
of the stroke indicates its arrival there.

c) What do you understand by sourcing and sinking input and output modules ?
List the special features of PLCs.

7. Write short notes on (any four) :

20

a) Input/output processing in PLC


b) OSI networking models
c) Communication protocols
d) Advantages and disadvantages of PLCs
e) Serial and parallel data transmission.
_____________________

Set A

SLR-BB 117

*SLRBB117*
Seat
No.

Set

B.E. (Mech.) (Part II) Examination, 2014


AUTOMOBILE ENGINEERING (Elective II)
Day and Date : Monday, 1-12-2014
Time : 3.00 p.m. to 6.00 p.m.

Max. Marks : 100

Instructions : 1) Figures to right indicate full marks.


2) Assume suitable data if necessary and mention it clearly.
3) Q. No. 1 is compulsory. It should be solved in first 30 minutes
in Answer book Page No. 3. Each question carries one mark.
4) Answer MCQ/Objective type questions on Page No. 3 only.
Dont forget to mention, Q.P. Set (A/B/C/D) on Top of Page.
MCQ/Objective Type Questions
Duration : 30 Minutes

Marks : 20

1. Solve the following objective questions :

(120=20)

1) The clutch is located between the transmission and


a) Engine
b) Rear axle
c) Propeller shaft
d) Differential
2) Cushioning spring in clutch plate are meant to reduce
a) Torsional vibration
b) Vehicle speed
c) Jerky start
d) None of these
3) The purpose of transmission in automobile is
a) To vary speed of automobile
b) To vary torque at road wheels
c) To vary power of automobile
d) None of the above
4) By using synchronising device the two involved adjacent gear have their speed
a) Increased
b) Reduced
c) Equalized
d) Unequalised
5) The smallest gears inside the differential casing are
a) Pinion gears
b) Sun gears
c) Side gears
d) Ring gears
6) Function of slip joint is
a) Bend side way
c) Change inclination

b) Transfer torque at angle


d) Change length

P.T.O.

SLR-BB 117

-2-

*SLRBB117*

7) In automatic transmission system


a) Clutch lever, brake lever, accelerator pedal, gear shifting lever are present
b) Only accelerator pedal is absent
c) Clutch and gear shifting lever is absent
d) Only brake pedal is absent
8) Speed of Engine compared to wheel is
a) Less
b) More
c) Equal
d) None of above
9) Draw bar pull is
a) Extra road resistance which can overcome
b) Extra air resistance which can overcome
c) Extra load which vehicle can sustain
d) None of above
10) Generally load on rear axle compared to front axle is
a) More
b) Less
c) Equal
d) None of above
11) Cornering force divided by slip angle is called __________
a) Self righting torque
b) Cornering power
c) Pneumatic trail
d) Caster trail
12) The turning circle radius for a car is approximately
a) 1 meter
b) 2 meters
c) 10 meters
d) 30 meters
13) The brake efficiency of a new vehicle is about __________
a) 30%
b) 50%
c) 80%
d) 100%
14) Fading of brakes occurs _______
a) At high speeds
b) At low speeds
c) During continuous application
d) When brake lining is worn
15) Over charging of battery _________
a) Will bring about chemical changes in active material
b) Will increase the capacity of battery
c) Will raise the specific gravity of the electrolyte
d) None of above will occurs
16) The frequency of light per minute flashing in the direction indicator is
a) 50
b) 80
c) 800
d) None of above
17) Weight of vehicle produces in the side members of frame _______
a) Vertical bending
b) Horizontal bending
c) Torsion
d) All of these
18) For aiming the head lights the distance between the head light and the screen
should be _________
a) 1.6 meter
b) 4.6 meters
c) 7.6 meters
d) 10.6 meters
19) In starter motor the field winding are wound around __________
a) Armature
b) Commutator
c) Brush
d) Pole shoes
20) The ratio of the distributor shaft and the crank shaft speed is __________
a) One
b) One half
c) One fourth
d) Two

______________

Set A

*SLRBB117*

-3-

SLR-BB 117

Seat
No.

B.E. (Mech.) (Part II) Examination, 2014


AUTOMOBILE ENGINEERING (Elective II)
Day and Date : Monday, 1-12-2014
Time : 3.00 p.m. to 6.00 p.m.
Instructions : 1)
2)
3)
4)

Marks : 80

Answer any two questions from each Section.


Use of non-programmable calculator is allowed.
Figures to right indicate full marks.
Assume suitable data if necessary and mention it clearly.

SECTION I
2. a) Explain front wheel drive layout in detail.

b) What are requirement of good passenger car ? Discuss suitability considerations


for passenger car.

c) Describe with figure fluid clutch and compare it with friction clutch.

3. a) Explain single plate clutch with neat diagram.

b) Explain with figure working of synchromesh gear box.

c) What is torque converter ? Explain its construction and working with figure.

4. a) What is function of propeller shaft ? Draw neat sketch of propeller shaft


and explain it.

b) A truck weighs 100111 N and the engine develops 97 KW at 2400 rpm. The
transmission efficiency is 90% in top gear of 3.4 : 1 and 85% in third gear of 8.4 : 1.
The performance of the vehicle is such that it will just reach a speed 86.8 km/hr at
2400 rpm at wide open throttle when running on the level in still air, and at the
same engine speed in third gear it will just climb a gradient 1 in 14 if the total
resistance in N is given by the formula-R = K.W + KaAV2 + WSin where A is in m2,
V in km/hr and W in N. Calculate K and Ka and hence engine power required for
climbing a grade of 1 in 40 at 48 km/hr in top gear. How much more weight can be
added to the vehicle to use the engine power fully under the above condition ?
Frontal area of truck = 5.575 m2.
12

Set A

SLR-BB 117

-4-

*SLRBB117*

SECTION II
5. a) Explain with fig. different steering geometry used in front wheel steering system.

b) Explain any two types of steering gear boxes with the help of neat sketches.

c) Show that the breaking all wheel is more effective than breaking at gear wheel.

6. a) The wheel base of vehicle travelling on a wet road slopping upward at an angle
1
= Sin 0.1 is 4.5 m. Its centre of gravity is 1.5 m ahead of rear axle and 0.8 m
above the road. The coefficient of adhesion between vehicle tyres and the road is 0.25.
The vehicle employes brakes on all the four wheels. Determine
i) Ratio of breaking force on front and rear wheels if skidding is avoided.
ii) Stopping distance for the vehicle travelling at speed of 40 km./hr. when engine
is stopped and brakes are applied.
8
b) Explain with neat sketch double wish-bone type suspension used in vehicle.

c) What is voltage and current regulator ? Explain with fig.

7. Write short notes (Any four) :

(45=20)

a) Electric horn
b) Charging system of automobile
c) Two wheeler layout
d) Mach Pharson strut assembly
e) Air suspension system
_____________________

Set A

SLR-BB 118

*SLRBB118*
Seat
No.

Set

B.E. (Mechanical) (Part II) Examination, 2014


PRODUCTION MANAGEMENT (Elective II)
Day and Date : Monday, 1-12-2014
Time : 3.00 p.m. to 6.00 p.m.

Max. Marks : 100

Instructions : 1) Solve any two questions from each Section.


2) Q. No. 1 is compulsory. It should be solved in first 30 minutes in
Answer Book Page No. 3. Each question carries one mark.
3) Answer MCQ/Objective type questions on Page No. 3 only.
Dont forget to mention, Q.P. Set (A/B/C/D) on Top of Page.
MCQ/Objective Type Questions
Duration : 30 Minutes
1.

Marks : 20

i) Demand for an item that can be linked to the demand for another item is called as
_________
a) Independent demand
b) Dependent demand
c) Isolated demand
d) None of above
ii) Delphi technique of forecasting is
a) Qualitative technique
c) Both of above

b) Quantitative technique
d) None of above

iii) In service system, the decision making is difficult due to


a) Tangible nature of product
b) Intangible nature of product
c) Both above
d) None of above
iv) In manufacturing management, the term dispatching is used to describe
a) Dispatch of sales order
b) Dispatch of finished product
c) Dispatch of mails
d) Dispatch of work orders through shop floor
v) Gantt chart is a
a) Scheduling tool
c) Maintenance tool

b) Forecasting tool
d) MRP tool

vi) The value of (smoothening constant) in exponential smoothening method of


forecasting is
a) Between 1 to 1
b) Between 0 to 1
c) Between 2 to 2
d) Cant be defined
vii) MRP means
a) Material Reorder Point
c) Material Requirement Planning

b) Material Reorder Planning


d) Material Release Point

viii) The prime input for aggregate planning is


a) Active forecasting
b) Passive forecasting
c) Demand forecasting
d) None of above
P.T.O.

SLR-BB 118

-2-

*SLRBB118*

ix) In ABC analysis, A items are those which represent


a) Small percentage of the total annual consumption value
b) High percentage of the total annual consumption value
c) Small percentage of closing inventory value
d) High percentage of closing inventory value
x) Kanban is a Japanese term indicating
a) Method of line balancing
b) Information for production and withdrawn of items
c) Priority dispatching
d) Allocation of manpower to machines
xi) TPM means
a) Total Product Management
b) Total Process Management
c) Total Production Management
d) Total Productive Maintenance
xii) Annual carrying cost for a given annual demand
a) Will increase with increase in number of orders placed per annum
b) Will decrease with the increase in the number of orders placed per annum
c) Is independent of number of orders placed
d) Will increase or decrease depending upon nature of item
xiii) Release date is equal to
a) Due date + lead time
b) Due date lead time
c) Lead time due date
d) None of above
xiv) When the ordering cost is increased to 4 times, the EOQ will be increased to
a) 2 times
b) 3 times
c) 8 times
d) 4 times
xv) The directional (positive/negative) tendency is indicated in following type of error
in forecasting
a) Mean absolute deviation
b) Bias
c) Mean square error
d) None of above
xvi) Forward scheduling is commonly used in
a) Job shop
b) Mass production
c) Mixed production
d) Continuous production
xvii) Interest cost on the material bought is considered in
a) Ordering cost
b) Inventory carrying cost
c) Shortage cost
d) None of above
xviii) Line balancing is commonly used in
a) Job production
b) Batch production
c) Mass production
d) None of above
xix) Quick and inexpensive set up is a feature of
a) JIT
b) Kanban
c) MRP
d) None of above
xx) One of the major output of MRP is
a) Actual planned order release
b) Inventory status file
c) Master production schedule
d) Bill of material
______________

Set A

*SLRBB118*

SLR-BB 118

-3-

Seat
No.

B.E. (Mechanical) (Part II) Examination, 2014


PRODUCTION MANAGEMENT (Elective II)
Day and Date : Monday, 1-12-2014
Time : 3.00 p.m. to 6.00 p.m.

Marks : 80

Instruction : Solve any two questions from each Section.


SECTION I
2. a) Define production/operation management. Explain the objectives and scope of
the same.

10

b) Compare make to stock (MTS) with make to order (MTO) production.

c) Explain exponential smoothening method of forecasting.

3. a) A company has the following sales pattern. Calculate sales forecast for the period
08 and 09 using linear regression method.
10
Year

Sales in Thousands

14

20

20

28

30

32

38

b) Define capacity. Explain various types of capacities.

c) Explain types of errors in forecasting.

4. a) Five jobs are to be processed on two machines A and B, in the order A and B.
Processing time in hours are given below.
Jobs

Machine A

10

Machine B

Determine the sequence that minimises total elapsed time. Find out total elapsed
time and idle time (if any) on machine B.
10
b) Explain steps in production planning.

c) Write a note on line balancing.

Set A

SLR-BB 118

*SLRBB118*

-4-

SECTION II
5. a) Annual requirement of an item is 3000 units. Each item costs the company Rs. 10/-.
The manufacturer offers a discount of 5% if 500 units or more are purchased. The
cost of ordering is Rs. 30/- per order. Inventory carrying cost is 15%. Whether it is
advisable to accept the discount ? Comment.
10
b) Explain the various costs associated with inventory control.

c) Explain MRP.

6. a) The cost of a machine is Rs. 6,100/-. Its scrap value is Rs. 100/-. The annual
maintenance cost found from experience is as follows (Rs).
Year
Maintenance cost Rs.

100 250 400

600 900 1200 1600 2000

When should the machine be replaced ?

10

b) Explain preventive maintenance.

c) Explain Kanban.

7. a) Discuss principles of material handling.

10

b) Discuss supply chain management.

c) Explain Reliability.

5
_____________________

Set A

SLR-BB 119

*SLRBB119*
Seat
No.

Set

S.E. (Electronics) (Part I) Examination, 2014


ENGINEERING MATHEMATICS III
Day and Date : Tuesday, 9-12-2014
Time : 10.00 a.m. to 1.00 p.m.

Max. Marks : 100

N.B. : 1)
2)
3)
4)

Attempt any three questions from each Section.


Figures to right indicate full marks.
Use of calculator is allowed
Q. No. 1 is compulsory. It should be solved in first 30 minutes in Answer
Book Page No. 3. Each question carries one mark.
5) Answer MCQ/Objective type questions on Page No. 3 only. Dont forget to
mention, Q.P. Set (A/B/C/D) on Top of Page.
MCQ/Objective Type Questions

Duration : 30 Minutes

Marks : 20

1. Choose the correct answer :


1) The particular integral of (D3 3D2 + 4) y = e2x is _______________
a)

x 2x
e
6

b)

x 2 2x
e
6

(201=20)

c)

x 3 2x
e
6

d) 0

c)

e 35
s

d) None of these

c)

t4
24

d)

t3
6

d)

2) L{( t 3)} = ___________


a) e35

b) e35

3) L1 1 = _____________
4
s
5
t
t3
b)
a)
5
3

4) L1
= ______________
s

a)

b)

c)

5) The general solution of partial differential equation pq = 3 is


a) z = ax + 3y + c
b) z = 3x + by + c
3
x + by + c
b

c) z =

6) If L{ J0 (t )} =

a)

1
s +1
2

d) None of these

, then L{J0(2t)} = ______________


b)

c)

s +4
s +1
1
7)
{ X} is equals to ___________
D+a

a)

X e

ax

dx

b)

X e

ax

dx

2
s +4
2

c) e a x eax X dx

d)

1
2

s +4
2

d) e a x X e ax dx
P.T.O.

SLR-BB 119

*SLRBB119*

-2-

8) Among the following, which equation is not a non linear partial differential equation ?
p + q =1

a)

b) y 2p + x 2 q = x 2 y 2 z

c) p 3 + q3 = 27z

d) pq = xy

9) L1 {1} = _____________
a) (t )

b) H( t )

c)

1
s

d) None of these

d2 y
10) If L{y(t)}= y (s), then L 2 = ____________
dt

b) s2 y (s) + sy(0) + y (0)

a) s y (s) y(0)

c) s2 y (s) s y (0) y (0)


d) s2 y (s) sy(0) y (0)
11) If f(x) = x is represented by Fourier series in ( , ) then ____________ is the constant term.

b)
2
12) The period of sin7x is ___________

a)

d) 2

c) 0

2
7
13) The speed of a particle moving along the curve x = 2 sin 3t, y = 2 cos 3t, z = 8t at time t = 0 is
___________
a) 8
b) 10
c) 14
d) 20

a) 7

b) 2

c)

d)

14) If r = xi + yj + zk and a is constant vector then (a r ) = _________


b) r
c) a r
d) 0
a) a
2
2
15) The unit tangent vector to the curve x = t + 1, y = 4t 3, z = 3t 6t at t = 1 is
1

a)

(i + j + k)

b)

(i + 2 j + k)

c)

1
5

(i + 2 j)

d)

1
5

(i + 2k)

1 , 0 x < 1
16) The Fourier cosine transform of f(x) =
is _____________
0 , x > 1
2
s

a)

17) From the result

2
sin s

b)

c)

2 sin s
s

cos sx sin sk
, | x | < k
ds =
we can deduce that
s
0 , | x | > k

a) 0

2
s. sin s

sin ks
ds = ____
s

c) 2

c)

b) 1

d)

18) Inverse Fourier sine transform of Fs(s) is given by ___________


a)

1
Fs (s) cos sx ds
0

b)

1
2
Fs (s) cos sx ds c)
Fs (s) sin sx ds

0
0

19) If z {f(k)} = F (z) then z{f(k n)} = ______________


a) z k F(z)
b) zk F(z)
c) zn F(z)

d)

2
Fs (s) sin sx ds
0

d) zn F(z)

1 , k 0
20) If u(k) =
then z{u(k)} = _____________
0 , k < 0

a)

z
z 1

b)

1
z 1

c)

1
z 1

d)

z
z 1

______________
Set A

*SLRBB119*

-3-

SLR-BB-119

Seat
No.

S.E. (Electronics) (Part I) Examination, 2014


ENGINEERING MATHEMATICS III
Day and Date : Tuesday, 9-12-2014
Time : 10.00 a.m. to 1.00 p.m.

Marks : 80

N.B. : 1) Attempt any three questions from each Section.


2) Figures to right indicate full marks.
3) Use of calculator is allowed
SECTION I
2. A) Attempt any two :
i) Solve : (D2 + 2aD + a2) y = 8 cos (ax)
ii) Solve :(D2 + 6D + 9) y = 5x log 2 + e3x
iii) Solve : (5z 7y) p + (7x 3z) q = 3y 5x
B)

An electric circuit consists of an inductance L, capacitance C and an e.m.f. E0cos( t), so that the charge q
satisfies the differential equation.

E
d2q
1
+
q = 0 cos t . If 2 = 1 and q = q0, i = i0 when t = 0, then find charge q interms of t.
2
LC
L
dt
LC

3. a) Solve : z(p2 q2) = x y


b)
c)

Solve : p2 pq = 1 z2
Solve :

(D2

4) y =

e3xx2.

2s + 1
1
.
4. a) Find : L
2
(
s
3
)(
s
4
)

b)

Find : L e 2 t 1 + sin 4 t .

c)

Find Laplace transform of periodic function given by


f(t) = t
= t

0<t<

<t<2

where f(t) = f(t + 2 )

5. a) Find : L{ te 2 tH(t 3)} .

s2 a2
2
2
s b

b)

Find : L log

c)

The differential equation of a circuit containing resistance R, capacitance C is given by

dq 1
+ q = 40e 3 t + 20e 6t . If R = 20 , C = 0.01 farad and q(0) = 0, then using Laplace transform find the
dt C

charge q interms of time t.

Set A

SLR-BB 119

*SLRBB119*

-4-

SECTION II

6.

2 x 2
in ( , )

12 4

a)

Find the Fourier series for f(x ) =

b)

Show that the vector field F = (x 2 + xy 2 ) i + (y 2 + x 2 y )j is irrotational and find its scalar potential.

c)

Find the velocity and acceleration of a particle moving along the curve x = 2 sin 3t, y = 2 cos 3t, z = 8t. Also find
their magnitudes.

OR

7.

8.

9.

c)

Find the rate of change of = xyz in the direction normal to the surface x2y + y2 x + yz2 = 3 at the point (1, 1, 1).

a)

If a is a constant vector and r, r have usual meaning, prove that a =

b)

Obtain the Fourier expansion of x sin x (0 < x < 2 ).

a)

Find the inverse z-transform of F(z ) =

b)

Find z {K2 eak}.

c)

Find Fourier sine transform of f(x ) =

a)

Find the inverse z-transform of F(z ) =

b)

2
Express ex.cos x as Fourier cosine integral and show that ex.cos x =

1
r

a 3( a r ) r
+
.
r3
r5

6
7

3z 2 18z + 26
,3 < z < 4 .
(z 2)(z 3)(z 4)

5
5

sin x , 0 < x < 1


.
0 , x >1

z2
1
, | z |< .
1
1
5

z 4 z 5

hence evaluate

2 + 2
d .
(4 + 4)

2 + 2
cos x d and
( 4 + 4)
8

_____________________

Set A

SLR-BB 120

*SLRBB120*
Seat
No.

Set

S.E. (Electronics) (Part I) Examination, 2014


ELECTRONIC CIRCUIT ANALYSIS AND DESIGN I
Day and Date : Thursday, 11-12-2014
Time : 10.00 a.m. to 1.00 p.m.

Total Marks : 100

Instructions : 1)
2)
3)
4)

All questions are compulsory.


Assume suitable data if required.
Use of data sheet is allowed.
Q. No. 1 is compulsory. It should be solved in first 30 minutes in
Answer book Page No. 3. Each question carries one mark.
5) Answer MCQ/Objective type questions on Page No. 3 only. Dont
forget to mention, Q.P. Set (A/B/C/D) on Top of Page.
MCQ/Objective Type Questions

Duration : 30 Minutes
1. Choose the correct answer :

Marks : 20
(201=20)

1) Ripple frequency of output waveform of a voltage doubler circuit when fed with a 50 Hz
sine wave is
a) 100 Hz
b) 25 Hz
c) 50 Hz
d) 200 Hz
2) If the load resistance of a inductor filtered full wave rectifier is reduced, ripple voltage
a) Increases
b) Decreases
c) Is not affected
d) Has different frequency
3) Zener diode is used as a
a) Fixed reference voltage
b) Peak clipper in wave shaping circuit
c) Meter protection against damage from application of excessive voltage
d) All of the above
4) The main function of clipping circuit is to
a) Remove certain portion of input signal above or below certain level
b) Protection from high voltage
c) Suppress amplitude variation in input signal
d) All of the above
5) The capacitance which exists in a forward biased junction is __________ capacitance.
a) Diffusion
b) Storage
c) Transition
d) Both a) and b)
6) Silicon material is widely used as a semiconductor over Germanium because ______
a) It is cheaper
b) Available in abundant
c) Has got low reverse saturation current d) All of the above
7) A zener diode
a) Has a high forward voltage rating
b) Is useful as an amplifier
c) Has a sharp breakdown at low reverse voltage
d) Can not be operated in forward biasing mode
8) No load voltage across a full wave rectifier is 30 V. After connecting a capacitor across
its output terminals it becomes ___________
a) 47.12 V
b) 30 V
c) 0.7 V
d) 42.42 V

P.T.O.

SLR-BB 120

*SLRBB120*

-2-

9) The output of a voltage doubler is ____________


a) DC
b) AC
c) Zero
d) None of the above
10) The transformer utilisation factor of Bridge rectifier is ____________
a) 0.693
b) 0.81
c) 1.21
d) 0.48
11) The base emitter voltage ________ at the rate of 2.5mV/C with ________ in temperature.
a) Increases, increases
b) Decreases, increases
c) Decreases, decreases
d) Increases, decreases
12) The Q point of NPN CE amplifier should not be selected to cutoff region because
a) It leads to thermal runway
b) It may cause output current to be clipped off in negative half cycle
c) It may cause output voltage to be clipped off in positive half cycle
d) Both b) and c)
13) The self bias arrangement gives an improved Q point stability when
a) RE is low
b) is small but RE is large
c) Both and RE are large
d) is large but RE is small
14) In case of BJT bias stability is achieved by
a) Keeping base current constant
b) Keeping temperature constant
c) Changing base current in order to keep IC and VCE constant
d) Both a) and b)
15) In an astable multivibrator has C1 = C2 = 1000pF and R1 = R2 = 20 K then frequency
of oscillation is
a) 36.23 KHz
b) 3.625 KHz
c) 0.3625 KHz
d) 362.5 KHz
16) Pinch-off voltage Vp for an FET is the drain voltage at which :
a) Channel width is reduced to minimum
b) All free charges get removed from the channel
c) ID reaches to its constant saturation level
d) All above
17) The common-drain JFET amplifier has
a) A very high input impedance
b) Provides 0 phase shift
c) Used at front end of measuring instruments like CRO
d) All above
18) n-channel MOSFETs are superior to p-channel MOSFETs because :
a) Mobility of electrons is greater than that of holes
b) Resistance of n-channel MOSFETs is less than p-channel MOSFETs
c) Higher packing density for n-channel MOSFETs
d) All above
19) FETs are used in mixers of TV and FM receivers for the reason ___________
a) They have got no drift
b) They have low noise margins
c) They have square law distortion
d) They have high voltage gains
20) Assume that N channel EMOSFET shown above is ideal and its threshold voltage is 2V.
The voltage V0 is

a) 12 V

b) 0 V

c) 11 V

______________

d) 0.5 V

Set A

*SLRBB120*

-3-

SLR-BB 120

Seat
No.

S.E. (Electronics) (Part I) Examination, 2014


ELECTRONIC CIRCUIT ANALYSIS AND DESIGN I
Day and Date : Thursday, 11-12-2014
Time : 10.00 a.m. to 1.00 p.m.

Marks : 80

Instructions : 1) All questions are compulsory.


2) Assume suitable data if required.
3) Use of data sheet is allowed.
SECTION I
2. Attempt any four :

(45=20)

1) Give the relation between diode current and applied voltage. Also plot this relation. A Si
semiconductor p-n junction diode is exposed to 27C. If current flowing through it is 10mA,
find how much voltage is applied to it. Assume minority carrier current to be 2.5nA.
2) A center tapped rectifier is subjected to an input AC voltage of 15 V-0-15 V. Find DC o/p
voltage, RMS load voltage, DC load current and PIV rating of diode across a load of 10K.
Assume practical diode.
3) Draw and explain a circuit to clip a sinusoidal waveforms above 4 V and below 2 V if
applied input signal peak voltage is 5 V, with i/p and o/p waveforms.
4) Find the o/p voltage across RL. Refer the following circuit diagram.
a) for RL = 3.3 K
b) for RL = 1 K .

5) Explain the operation of a bridge rectifier when a capacitor of high value is connected
across it.

Set A

SLR-BB 120

-4-

*SLRBB120*

3. Attempt any two :

(210=20)

1) Design power supply to provide an unregulated DC voltage of 50 V at 400 mA. Use only
two filtering components to provide ripple of 0.1%.
2) Explain the important ratings of diode and internal capacitances observed in semiconductor
p-n junction diode.
3) A filter with bridge rectifier has C1 = C2 = 500 f, L = 100 mH with 0.5 winding
resistance. Instead of load resistor there is a constant 90mA current. The AC input voltage
to FWR is 15 Vrms and diodes each have 0.7 V drop when forward biased. Calculate DC
o/p voltage and peak o/p ripple voltage. Assume input signal frequency to be 60 Hz.
SECTION II
4. Attempt any four :

(45=20)

a) Why bias compensation is required ? Explain bias compensation techniques using


thermistor.
b) Q-point at saturation and at cut off results to distortion. Justify answer with suitable
waveforms.
c) Design an Astable multivibrator for output amplitude of 14 V and square wave of 2 KHz.
Use Si transistor hfe(min) = 60, Ic(max) = 8 mA VCE(sat) = 0.2 V.
d) Explain working of self biased binary circuit. Sketch associated waveforms.
e) Explain effect of bypass capacitor, coupling capacitor and internal junction capacitors on
the frequency response of amplifier.
5. Attempt any two :

(210=20)

a) Explain construction, working and characteristics of N channel JFET.


b) Derive an expression for stability factor S of voltage divider bias circuit. Explain how it
stabilizes the operating point.
Design voltage divider bias circuit for the following specification.
VCC = 12 V, VCEQ = 6 V, ICEQ = 4mA, min = 120.
c) Derive the expression for Ai, Ri, Ais, Av, Yo in terms of hybrid parameter for CE
configuration.
_____________________

Set A

SLR-BB 121

*SLRBB121*
Seat
No.

Set

S.E. (Electronics Engineering) (Part I) Examination, 2014


NETWORK THEORY AND ANALYSIS
Day and Date : Saturday, 13-12-2014
Time : 10.00 a.m. to 1.00 p.m.

Max. Marks : 100

Instructions : 1) Q. No. 1 is compulsory. It should be solved in first 30 minutes in Answer


Book Page No. 3. Each question carries one mark.
2) Figures to the right indicate full marks.
3) Assume suitable data, if necessary.
4) Answer MCQ/Objective type questions on Page No. 3 only. Dont forget to
mention, Q.P. Set (A/B/C/D) on Top of Page.
MCQ/Objective Type Questions
Duration : 30 Minutes

Marks : 20

1. Choose the correct answer :

(201=20)

1) To calculate Thevenins equivalent impedance value in a circuit


a) All independent voltage sources are opened and all independent current sources are short
circuited
b) Both voltage and current sources are open circuited
c) All voltage and current sources are shorted
d) All voltage sources are shorted while current sources are opened
2) The characteristic impedance of a low pass filter in attenuation band is
a) Purely imaginary
b) Zero
c) Complex quantity

d) Real value

3) In Parallel Resonance of R L C circuit having a R L as series branch and C forming parallel


branch
a) Max impedance and current is at the frequency that of resonance
b) Value of max impedance = L / (CR)
c) Branch currents are 180 degree phase shifted with each other
d) fr =

1
2

1
R2
2

LC L

4) For a coil with inductance L and resistance R in series with a capacitor C has
a) Resonance impedance as zero
b) Resonance impedance R
c) Resonance impedance L/CR
d) Resonance impedance as infinity
5) An attenuator is a
a) Rs network

b) RL network

c) RC network

d) LC network

6) A series resonant circuit is inductive at f = 1000 Hz. The circuit will be capacitive some where at
a) f > 1000 Hz
b) f < 1000 Hz
c) f equal to 1000 Hz and by adding a resistance in series
d) f = 1000 + fo (resonance frequency)
7) A network function is said to have simple pole or simple zero if
a) The poles and zeroes are on the real axis
b) The poles and zeroes are repetitive
c) The poles and zeroes are complex conjugate to each other
d) The poles and zeroes are not repeated
P.T.O.

SLR-BB 121

*SLRBB121*

-2-

8) The tie-set schedule gives the relation between


a) Branch currents and link currents
c) Branch currents and link voltages

b) Branch voltages and link currents


d) None of the above

9) A 20 V source has an internal resistance of 90 ohm. If load resistance of 60 ohm is connected to the
voltage source, the load power PL is
a) 1.06 mW
b) 1.06 W
c) 106 mW
d) 10.6 W
10) The z-parameters of the shown T-network at Fig. 1 are given by

Fig. 1
a) 5, 8, 12, 0

b) 14, 8, 8, 20

c) 8, 20, 13, 12

d) 6, 8, 8, 12

11) To a highly inductive circuit, a small capacitance is added in series. The angle between voltage and
current will
a) Decrease
b) Increase
c) Remain nearly the same
d) Become indeterminant
12) An RLC series circuit is said to be inductive if
a) L > 1/ C

b) L = 1/ C

c) L < 1/ C

d) L = C

13) In the circuit shown, maximum power will be transferred when ?

a) ZL = (5.6 + j 5.4)

b) ZL = (5.6 j 5.4)

c) ZL = (5.4 j 5.6)

d) ZL = (5.4 + j 5.6)

14) For a two port reciprocal network, the three transmission parameters are given by A = 4, B = 7 and
C = 5. The value of D is equal to
a) 8.5
b) 9
c) 9.5
d) 8
15) An ideal filter should have
a) Zero attenuation in the pass band
c) Infinite attenuation in the pass band

b) Zero attenuation in the attenuation band


d) Infinite attenuation in the attenuation band

16) For an m-derived high pass filter, the cut off frequency is 4 KHz and the filter has an infinite
attenuation at 3.6 KHz, the value of m is
a) 0.436
b) 4.36
c) 0.34
d) 0.6
17) If Zoc = 120 and Zsc = 30 , the characteristic impedance is
a) 30
b) 60
c) 120

d) 150

18) The dynamic impedance of a parallel RLC circuit at resonance is


a) C / LR
b) R / LC
c) L / CR

d) LC / R

19) For a linear passive bilateral network


a) h12 = h21
b) h12 = h21

d) h12 = g12

20) A function is given by f(s) =


a) Real axis of s-plane
c) At infinity

c) h12 = g12

1
. It would have a zero at
s+3
b) Imaginary axis of s-plane
d) At the origin
______________

Set A

*SLRBB121*

-3-

SLR-BB 121

Seat
No.

S.E. (Electronics Engineering) (Part I) Examination, 2014


NETWORK THEORY AND ANALYSIS
Day and Date : Saturday, 13-12-2014
Time : 10.00 a.m. to 1.00 p.m.

Marks : 80

Instructions : 1) All questions are compulsory.


2) Figures to the right indicate full marks.
3) Assume suitable data, if necessary.
2. Attempt any four :

(45=20)

a) State and verify Millmans Theorem.


b) In the circuit shown in fig. a use mesh analysis to find out the power delivered to the 4 resistor.
To what voltage should the 100 V battery be changed so that no power delivered to the 4
resistor ?

Fig. a
c) A series RLC circuits consist of a 50 resistance, 0.2 H inductance and 10 F capacitor with
an applied voltage of 20 V. Determine the resonant frequency. Find the Q factor of the circuit.
Compute the lower and upper frequency limits and also find the bandwidth of the circuit.
d) For the two port bilateral network, prove that the relation between the transmission parameter is
AD BC = 1.
e) The port currents of a two-port network are given by
l1 = 2.5 V1 V2
l2 = V1 + 5 V2
Find the equivalent -network.
3. Attempt any two :

(210=20)

a) For the network shown below, determine using Thevenins theorem voltage across the capacitor.

Set A

SLR-BB 121

-4-

*SLRBB121*

b) Impedance Z2 and Z3 in parallel are in series with impedance Z1 across a 100 V, 50 Hz a.c. supply.
Z1 = (6.25 + j 1.25) , Z2 = (5 + j 0) and Z3 = (5 j Xc) . Determine the value of capacitance
of Xc such that the total current of the circuit will be in phase with the total voltage. What is the circuit
current and power ?
c) Determine the z and y parameters of the network shown below :

4. Attempt any four :

(45=20)

a) For the circuit shown below, find the current equation when the switch is opened at t = 0.

b) Design a low pass filter (both and T-sections) having a cut-off frequency of 2 KHz to operate with
a terminated load resistance of 500 .
c) Design an m-derived T-section filter (high pass) with a cut-off frequency 10 KHz, design impedance
of 200 and m = 0.4.
d) Design a symmetrical lattice attenuator to have a characteristics impedance of 800 and
attenuation of 20 dB.
e) State the restrictions on poles and zeroes for driving point functions.
5. Attempt any two :

(210=20)

a) Explain the sinusoidal response of series RL circuit.


b) Explain L-type asymmetrical attenuator and derive the design equations. Design a L-type
attenuator to operate into a load resistance of 600 with an attenuation of 20 dB.
c) For the given network function, draw the pole zero diagram and hence obtain the time domain
response i(t) l (s) =

5s

(s + 1) (s 2 + 4s + 8 )

_____________________

Set A

SLR-BB 122

*SLRBB122*
Seat
No.

Set

S.E. (Part I) (Electronics Engg.) Examination, 2014


DIGITAL LOGIC DESIGN
Day and Date : Tuesday, 16-12-2014
Time : 10.00 a.m. to 1.00 p.m.

Max. Marks : 100

Instructions : 1) Figures to right indicate full marks.


2) Assume suitable data whenever necessary.
3) Q. No. 1 is compulsory. It should be solved in first 30 minutes
in Answer Book Page No. 3. Each question carries one mark.
4) Answer MCQ/Objective type questions on Page No. 3 only.
Dont forget to mention, Q.P. Set (A/B/C/D) on Top of Page.
MCQ/Objective Type Questions
Duration : 30 Minutes

Marks : 20

1. Choose the correct alternative :


1) A binary number with n bits all of which are 1s has the value
b) 2n
c) 2n 1
d) 2n 1
a) n2 1

(120=20)

2) A combinational circuit can be designed using only


a) AND gates
b) OR gates
c) OR and X-NOR gates
d) All the above
3) A + B = B + A; AB = BA represent which laws ?
a) Commutative
b) Associative c) Distributive

d) Idempotence

4) The code used for labelling cells of the K-map is


a) Natural BCD
b) Hexadecimal c) Gray

d) Octal

5) The NOR-NOR realization is equivalent to


a) AND-NOR realization
b) NOT-AND realization
c) OR-NOT realization
d) OR-AND realization
6) Parallel adders are
a) Combinational logic circuit
b) Sequential logic circuit
c) Both of the above
d) None of the above
7) A combinational logic circuit which is used to send data coming from a single
source to two or more separate destinations is called
a) A decoder
b) An encoder
c) A multiplexer d) A demultiplexer
8) The number of subfamilies TTL has is
a) 4
b) 8
c) 6

d) 10
P.T.O.

SLR-BB 122

*SLRBB122*

-2-

9) The number of 1 : 16 demultiplexers required for designing a 4-output-4-variable


combinational circuit is
a) 16
b) 8
c) 4
d) 1
10) The number of parity bits in a 12-bit Hamming code is
a) 4
b) 5
c) 6

d) 8

11) How many flip-flops are required to build a binary counter circuit to count
from 0 to 511 ?
a) 1
b) 2
c) 9
d) 8
12) What is the frequency of pulses at the output of the circuit ?
Clock signal (1 KHz)

Output (Q4)

a) 1 KHz
b) 500 Hz
c) 62.5 Hz
13) In a T flip-flop output frequency is
a) Same as the input clock frequency
b) One half its input clock frequency
c) Double the input clock frequency
d) None of the above

d) 31.5 Hz

14) A flip-flop has two outputs which are


a) Always zero
b) Always one
c) Always complementary
d) None of the above
15) In a SR flip-flop, the SR inputs must not be
a) S = R = 1
b) S = R = 0
c) S = 0, R = 1
16) Which one of the following is used as latch ?
a) JK flip-flop
b) T flip-flop
c) SR flip-flop

d) S = 1, R = 0
d) D flip-flop

17) A positive edge-triggered flip-flop changes state on the ___________ of the


clock pulse.
a) High to low transition
b) Low to high transition
c) Negative level
d) None of the above
18) The output Qn of JK flip-flop is 0. It changes to 1 when a clock pulse is
applied. The input Jn and Kn are respectively.
a) 1 and X
b) 0 and X
c) X and 0
d) X and 1
19) The characteristic equation of D flip-flop is given by
a) Q+ = DQ + DQ
b) Q+ = D
+
c) Q = Q
d) Q+ = DQ + Q
20) The number of flip-flops required to implement mod-6 counter is
a) 2
b) 3
c) 5
d) 4
______________

Set A

*SLRBB122*

-3-

SLR-BB 122

Seat
No.

S.E. (Part I) (Electronics Engg.) Examination, 2014


DIGITAL LOGIC DESIGN
Day and Date : Tuesday, 16-12-2014
Time : 10.00 a.m. to 1.00 p.m.

Marks : 80

Instructions : 1) All questions are compulsory.


2) Figures to right indicate full marks.
3) Assume suitable data whenever necessary.
SECTION I
2. Solve any four :

(44=16)

a) Differentiate between MUX and DE-MUX.


b) Design full subtractor using only 2-input NAND gate.
c) Explain static hazards.
d) Explain NMOS inverter and equivalent circuit of NMOS, when input voltage is
zero volt.
e) Do the following conversions :
i) (3A.2F)16 to decimal number
ii) (95.5)10 to hexadecimal number.
3. a) Explain the characteristics of TTL logic family with typical values.
b) Attempt any two :

(18=8)
(82=16)

i) Design and implement a 4-bit gray to binary converter.


ii) Reduce using mapping the expression m(0, 1, 2, 3, 5, 7, 8, 9, 10, 12, 13)
and implement it in universal logic.
iii) What is multiplexer, how it is differed with de-multiplexer, design 16 : 1
MUY using two 8 : 1 MUX and one 2 : 1 MUY.
Set A

SLR-BB 122

-4-

*SLRBB122*

SECTION II
4. Attempt any four :

(44=16)

a) Derive the state diagram for JK flip-flop.


b) Draw and explain ring counter.
c) What are different types of shift register ? Explain any one of it.
d) Derive the characteristic equation for SR flip-flop.
e) Write a short note on PLAs.
5. a) Design a BCD synchronous counter using T flip-flop. Draw neat circuit diagram
and waveforms.
b) Attempt any two :

(82=16)

i) Design a JK flip flop using a D flip-flop.


ii) Explain Melay machine with example. Also compare Moore and Melay
machine.
iii) Design a pulse train generator using a shift register for the following pulse
train :
.........................101110110.........................
_____________________

Set A

SLR-BB 123

*SLRBB123*
Seat
No.

Set

S.E. (Electronics) (Part I) Examination, 2014


DATA STRUCTURES
Day and Date : Thursday, 18-12-2014
Time : 10.00 a.m. to 1.00 p.m.
N. B. :

Max. Marks : 100

1) Assume suitable data if necessary.


2) Q. No. 1 is compulsory. It should be solved in first 30 minutes in
Answer Book Page No. 3. Each question carries one mark.
3) Answer MCQ/Objective type questions on Page No. 3 only.
Dont forget to mention, Q.P. Set (A/B/C/D) on Top of Page.
MCQ/Objective Type Questions

Duration : 30 Minutes

Marks : 20

1. Choose the correct answer :

20

1) Which data structure is needed to convert infix notation to postfix notation ?


a) Branch
b) Queue
c) Tree
d) Stack
2) The following sequence of operation is performed on stack : push (1), push (2),
pop, push (1), push (2), pop, pop, pop, push (2), pop. The sequence of popped out
values are
a) 2, 1, 2, 2, 2
b) 2, 2, 1, 2, 2
c) 2, 1, 2, 2, 1
d) 2, 2, 1, 1, 2
3) The postfix form of the expression (A + B) (C D E) F/G is
a) AB + CD E FG/
b) AB + CD E F G/
c) AB + CD E F G/
d) AB + CDE F G/
4) A linear list of elements in which deletion can be done from one end and insertion
can take place only at the other end is known as
a) Queue
b) Stack
c) Tree
d) Linked list
5) Consider linked list is used to implement the stack, then which of the following
node is considered as top of the stack ?
a) First node
b) Middle node
c) Last node
d) Any node
6) If in a linked list of integers, address of first node is 1020, then what will be the
address of node at 5th position ?
a) 1038
b) 1036
c) 1040
d) None of these
7) Recursion can be implemented by
a) Static storage allocation
c) Both a) and b)

b) Dynamic storage allocation


d) None of these

8) In a circular linked list organization, insertion of a record involves the


modification of
a) No pointer
b) 4 pointers
c) 2 pointers
d) 3 pointers
P.T.O.

SLR-BB 123

*SLRBB123*

-2-

9) A postfix expression is merely the reverse of the prefix expression.


a) True
b) False
10) Stack is useful for implementing
a) Breadth first search
c) Recursion

b) Depth first search


d) Both b) and c)

11) The complexity of Binary search algorithm is


a) O(n)
b) O(logn)
c) O(n2)

d) O(n log n)

12) The average case occur in linear search algorithm


a) When item is somewhere in the middle of the array
b) When item is not in the array at all
c) When item is the last element in the array
d) When item is the last element in the array or is not there at all
13) The number of swapping needed to sort numbers 8, 22, 7, 9, 31, 19, 5, 13 in
ascending order using bubble sort is
a) 11
b) 12
c) 13
d) 14
14) The number of binary tree with 3 nodes which when traversed in postorder gives
the sequence A, B, C is
a) 3
b) 9
c) 7
d) 5
15) A binary search tree is traversed in the following order recursively :
Right, root, left. The output sequence will be in
a) Ascending order
b) Bitomic sequence
c) Descending order
d) No specific order
16) For an undirected graph, with n vertices and e edges, the sum of the degree of
each vertex is equal to
a) 2n
b) (2n 1)/2
c) 2e
d) e2/2
17) Which of the following is useful in traversing a given graph by breadth first search ?
a) Stacks
b) Set
c) List
d) Queue
18) A binary tree of depth d is an almost complete binary tree if
a) each leaf in the tree is either at level d or at level d 1
b) for any node n in the tree with a right descendent at level d, all the left
descendents of n that are leaves, are also at level d
c) Both a) and b)
d) None of the above
19) The preorder and postorder traversal of a binary tree generates the same output.
The tree can have maximum
a) Three nodes
b) Two nodes
c) One node
d) Any number of nodes
20) A B-tree of order n is also called _____________ tree.
a) n
b) n + 1
c) n 1
______________

d) n (n 1)

Set A

*SLRBB123*

-3-

SLR-BB 123

Seat
No.

S.E. (Electronics) (Part I) Examination, 2014


DATA STRUCTURES
Day and Date : Thursday, 18-12-2014
Time : 10.00 a.m. to 1.00 p.m.
N. B. :

Marks : 80

1) All questions are compulsory.


2) Assume suitable data if necessary.

2. Attempt any four :

(45=20)

1) Explain push and pop operation of a stack using C routines.


2) Write a short note on priority queue.
3) Write a C program to find the greatest common divisor of two integers using
recursion.
4) Explain the algorithm for deletion of an element from a linked list.
5) Explain circular linked list for Josephus problem.
3. Attempt any two :

(210=20)

1) Write a C program to implement stack using linked list.


2) Write a C program to implement circular queue using array.
3) a) Illustrate addition of two polynomials using linked list 3x4 + 4x2 + 2x + 5 and
2x3 + 3x + 7.
b) Evaluate the following postfix expression using stack
396 62 / 5 + 73% .
4. Attempt any four :

(45=20)

1) Explain the various tree traversal methods with example.


2) Write a C program to search an element from an array using linear search.
3) Explain adjacency linked list representation of a graph with example.
4) Explain bubble sort. Sort the following using bubble sort 25, 30, 59, 10, 92, 85, 30.
Show the steps for each iteration.
5) Explain any three hashing functions commonly used with example.

Set A

SLR-BB 123

-4-

5. Attempt any two :

*SLRBB123*
(210=20)

1) Define the following with diagram :


1) Root
2) Leaf node
3) Siblings
4) Ancestor
5) Depth of the tree
6) Children
7) Binary search tree
8) Complete binary tree
9) Strictly binary tree
10) Parent.
2) What is graph traversal ? Explain breadth first traversal method of a graph with
flowchart and example.
3) What is hash collision ? Explain closed addressing technique to resolve hash
clashes.
_____________________

Set A

SLR-BB 124

*SLRBB124*
Seat
No.

Set

S.E. (Electronics) (Part II) (Old) Examination, 2014


ELECTRICAL MACHINES
Day and Date : Tuesday, 25-11-2014
Time : 3.00 p.m. to 6.00 p.m.

Max. Marks : 100

Instructions : 1) Q. No. 1 is compulsory. It should be solved in first 30


minutes in Answer book Page No. 3. Each question carries
one mark.
2) Figures to the right indicates full marks.
3) Assume suitable data whenever necessary.
4) Answer MCQ/Objective type questions on Page No. 3 only.
Dont forget to mention, Q.P. Set (A/B/C/D) on Top of Page.
MCQ/Objective Type Questions
Duration : 30 Minutes

Marks : 20

1. Choose correct option :

(201=20)

1) The speed of a dc motor is inversely proportional to


a) back emf and flux
b) back emf
c) flux
d) supply voltage
2) The resistance of the field regulator of a dc series motor is of the order of
b) 10
c) 2
d) 0.1
a) 100
3) If n is synchronous speed of three phase induction motor in rps and P are the
number of poles, then frequency is __________ Hz.
a) 0.5 Pn
b) Pn
c) Pn/12
d) Pn/60
4) Synchro is basically a __________
a) Transducer
b) Resister

c) Capacitor

d) Inductor

5) __________ is the starter used for 3 phase squirrel cage induction motor.
a) DOL starter
b) 3-point starter
c) 4-point starter
d) Rotor resistance starter
6) In case of DC series motor torque is proportional to
b) I2a
c) Ia/2
a) Ia

d) Va

7) The lagging power factor is due to __________ load.


a) Filament lamp b) Heater
c) Capacitive

d) Induction motor
P.T.O.

SLR-BB 124

*SLRBB124*

-2-

8) In case of three phase transformer hysteresis losses are reduced by using


a) Solid core
b) High silicon content steel core
c) Laminated core
d) None of the above
9) Which of the following connections is best suited for high voltage and small
size service ?
a) Delta-delta
b) V-V
c) Star-star
d) T-T
10) Universal motor runs satisfactorily on __________ supply.
a) a.c.
b) d.c.
c) a and b
d) None of the above
11) Power factor = __________
a) KW/KVAR
b) Z/R

c) VR/VL

d) None of the above

12) The back emf of a 220 V d.c. motor with armature resistance 2 and
Ia 20 (Amp) is
a) 200 V
b) 230 V
c) 260 V
d) 180 V
13) In case of slip ring induction motor
a) Only rotor is wound
c) Both a) and b)

b) Only stator is wound


d) None of the above

14) The stator loss of 3 ph. I.M. is 1 KW and rotor input is P KW then stator input is
a) P
b) P + 1
c) 0.5 P
d) None of the above
15) Swinburns test is used for
a) Checking breaks
c) For finding no load losses

b) Finding load capacity


d) None of the above

16) __________ motor should never be switched ON at no load.


a) The d.c. series
b) The d.c. shunt
c) Cumulative compound
d) Differential compound
17) Slip of three phase induction motor is given by
a) Ns-N/Ns
b) N-Ns/N
c) Ns-N/N

d) N-Ns/Ns

18) In case of d.c. motor at starting back emf is zero because


a) Ia is maximum b) Speed is zero c) Ra is small
d) All above
19) The shape of armature current Vs torque characteristic of dc series motor is
a) Linear
b) Straight line with X-intercept
c) Rectangular hyperbola
d) Parabola
20) The direction of rotation of a dc motor can be determined by
a) Flemings right hand rule
b) Flemings left hand rule
c) Lenz law
d) Amperes law
______________

Set A

*SLRBB124*

-3-

SLR-BB 124

Seat
No.

S.E. (Electronics) (Part II) (Old) Examination, 2014


ELECTRICAL MACHINES
Day and Date : Tuesday, 25-11-2014
Time : 3.00 p.m. to 6.00 p.m.

Marks : 80

Instructions : 1) Figures to the right indicates full marks.


2) Assume suitable data whenever necessary.
SECTION I
2. Attempt any four :

(64=24)

a) Explain with neat circuit diagram no load test on DC shunt motor.


b) A 220 (Volt) d.c. shunt motor runs at 800 (rpm) and if Ia is 50 (Amp). Calculate
the speed if torque is doubled. Take Ra = 0.2 . Assume flux and current
remains same.
c) Explain with neat diagram reluctance motor.
d) Explain with proper reason which d.c. motor is suitable for following
applications-lathe machine, traction and punching machine.
e) Explain significance of back emf in working of DC motor.
3. a) Explain with neat circuit diagram flux changing method to control the
speed d.c. shunt motor. Write its merits and demerits.

(82=16)

b) A 440 volt d.c. shunt motor takes 6 amp current when running on no load. If
Ra = 0.5 Rsh 220 . Find the efficiency of the motor when running at full
load and taking a current of 62 amps. Assume effect of temperature on
resistance is negligible.
OR
c) Explain with neat diagram following braking methods used for d.c. shunt
motor and d.c. series motor.
i) Dynamic braking
ii) Plugging.

Set A

SLR-BB 124

-4-

*SLRBB124*

SECTION II
4. Attempt any four :

(64=24)

a) Explain working principal of three phase induction motor.


b) Explain with neat diagram star-star connection used in case of three phase
transformer connection.
c) A rotor resistance of 3-phase, 4-pole, 50 Hz slip ring induction motor is
0.05 per phase and rotor reactance per phase at stand still is 0.2 .
Calculate :
i) Rotor impedance per phase at stand still.
ii) Slip and speed at maximum torque.
d) Define low power factor ? What are the different causes for this ?
e) A three phase, 8 pole, 50 (Hz) induction motor impedance Z = (0.001 + j 0.005)
per phase. Find the additional value of resistance to be added in the rotor
circuit for maximum starting torque and ratio of starting torque to maximum
torque.
5. Attempt any two :

(82=16)

a) Explain with neat diagram working of auto transformer and write equation of
copper saving.
b) A 50 Hz 8 pole three phase induction motor has full load slip of 4% the rotor
resistance per phase is 0.01 and standstill reactance per phase is 0.1 .
Find the ratio of TM/TF and the speed at which maximum torque occurs.
c) Explain with neat diagram slip torque characteristics of three phase squirrel
cage induction.
_____________________

Set A

SLR-BB 125

*SLRBB125*
Seat
No.

Set

S.E. (Electronics Engineering) (Part II) (Old) Examination, 2014


ELECTRONICS CIRCUIT ANALYSIS AND DESIGN II
Day and Date : Wednesday, 26-11-2014
Time : 3.00 p.m. to 6.00 p.m.
Instructions :

Total Marks : 100

1)
2)
3)
4)

Figures to the right indicate full marks.


Assume suitable data, if necessary.
Use of datasheet and calculator is allowed.
Q. No. 1 is compulsory. It should be solved in first 30 minutes in
Answer Book Page No. 3. Each question carries one mark.
5) Answer MCQ/Objective type questions on Page No. 3 only. Dont
forget to mention, Q.P. Set (A/B/C/D) on Top of Page.
MCQ/Objective Type Questions

Duration : 30 Minutes

Marks : 20

1. Multiple choice questions :


20
1) An amplifier circuit has an overall current gain of 100 and an input resistance of 10 K
with a load resistance of 1 K . The overall voltage gain of the amplifier is ___________
a) 5 dB
b) 10 dB
c) 20 dB
d) 40 dB
2) The phase shift oscillator can use either
a) Phase-lead or phase-lag RC sections
b) The combination of phase-lead and phase-lag RC sections
c) Only phase lead RC sections
d) Only phase lag RC sections
3) In common emitter amplifier resistor is added in emitter ___________
a) To increase input resistance
b) To increase voltage gain
c) To decrease stability
d) None of the above
4) The power dissipation of class C amplifier is normally ___________
a) Very low
b) Very high
c) Same as Class B
d) Same as Class A
5) Which one of the following is NOT true for direct coupled amplifiers ?
a) Low cost
b) Can amplify high frequency signals
c) Can amplify low frequency signals
d) Operating point shifts with temperature variations
6) The major advantage of dc amplifiers is that it
a) Uses less no. of components
b) Has very good temperature stability
c) Does not use frequency sensitive components
d) Can amplify direct current and low frequency signals
7) In a transistor push pull amplifier
a) There is no dc present in the output
b) There is no distortion present in the output
c) There are no even harmonics in the output
d) There are no odd harmonics in the output
P.T.O.

SLR-BB 125

*SLRBB125*

-2-

8) Which one of the following parameters is used for distinguishing between a small signal
and a large-signal amplifier ?
a) Voltage gain
b) Frequency response
c) Harmonic distortion
d) Input/output impedance
9) For a transconductance amplifier input and output resistances are respectively
a) Infinity and zero
b) Zero and Infinity c) Zero and zero
d) Infinity and Infinity
10) A cascaded amplifier comprises N identical non-interacting stages, each having a lower
3 dB frequency of fL. If fL* is the lower 3 dB frequency of the cascaded amplifier, then
which one of the following is correct ?
a) fL* = fL

c)

fL* =

fL

b) f * = f
L
L
1

2N 1

d) fL* = fL

2N 1

11) Reference voltage for adjustable voltage regulator using LM 317 is


a) 1.52 V
b) 0.125 V
c) 1.25 V
d) None of above
12) Pulse width value for IC 555 timer operates in monostable mode with 1.2 k resistance
and 0.01 F capacitor is
c) 132 sec
d) 13.2 n sec
a) 13.2 m sec
b) 13.2 sec
13) Monostable multivibrator circuit is used as
a) VCO
b) Timer
c) Flip flop
d) All of the above
14) IC 74121 is ___________, 74123 is ___________ monostable multivibrator respectively.
a) Nonretriggerable, retriggerable
b) Retriggerable, Retriggerable
c) Nonretriggerable, Nonretriggerable
d) Retriggerable, Nonretriggerable
15) The percentage change in output voltage for given change in input voltage is
a) Load regulation
b) Thermal regulation
c) Line regulation
d) Line and load regulation
16) The common-source JFET amplifier
a) Has a very low input impedance
b) Provides 180 phase shift
c) Used at front end of measuring instruments like CRO
d) None of above
17) Astable multivibrator can be used as
a) Sine wave converter
b) Square wave generator
c) Voltage to frequency converter
d) Both b) and c)
18) If charging time of astable multivibrator is 5 seconds and discharging time is 4 seconds
then duty cycle = ___________
a) 65.55%
b) 55.55%
c) 75%
d) 100%
19) An N channel D-MOSFET with VGS positive, it is operating in ___________ mode.
a) Enhancement
b) Depletion
c) Cutoff
d) Linear
20) JFET can be used a VVR in
a) Saturation region b) Ohmic region c) Breakdown region d) Both a) and c)
______________

Set A

*SLRBB125*

-3-

SLR-BB 125

Seat
No.

S.E. (Electronics Engineering) (Part II) (Old) Examination, 2014


ELECTRONICS CIRCUIT ANALYSIS AND DESIGN II
Day and Date : Wednesday, 26-11-2014
Time : 3.00 to 6.00 p.m.
Instructions : 1)
2)
3)
4)

Marks : 80

All the questions are compulsory.


Figures to the right indicate full marks.
Assume suitable data, if necessary.
Use of datasheet and calculator is allowed.
SECTION I

2. Answer the following (any four) :


(45=20)
1) Explain the difference between RC coupled amplifier and direct coupled amplifier.
2) An amplifier has voltage gain of 500. If a 10% negative feedback is employed into the
circuit
a) What will be the voltage gain with feedback ?
b) What should be the feedback in order to double the gain that existed in a) ?
3) Discuss the effect of negative feedback on gain stability of an amplifier with expressions.
4) Explain the essential conditions to have sustained oscillations.
5) Calculate the efficiency of class B amplifier for a supply voltage of Vcc = 24 V with peak
o/p voltage of :
a) VL(p) = 22 V
b) VL(p) = 6 V.
3. Answer the following (any two) :

(210=20)

1) For the two stage RC coupled amplifier shown below determine input resistance, individual
stage gains and overall voltage gain in dB. Also draw h-equivalent circuit

2) What is crossover distortion ? How it can be avoided ? Explain the operation of transformer
coupled pushpull amplifier with neat circuit diagram.
3) Design Wien Bridge oscillator to generate 25 KHz frequency with DC operating condition
20 V, 5 mA.
Set A

SLR-BB 125

-4-

*SLRBB125*

SECTION II
4. Solve any four questions :

(45=20)

1) Explain performance parameters of three terminal fixed IC voltage regulators.


2) Explain working of power on delay circuit.
3) Design constant current source that will deliver a 0.5 A current to 250 , 10 W load. Use
IC 7805.
4) Explain working of JFET as an amplifier.
5) Explain working of monostable Multivibrator.
5. Solve any two questions :

(102=20)

1) Explain current boosting circuit and foldback current limiting circuit for IC voltage
regulators.
2) Derive an expression for frequency and duty cycle of Astable Multivibrator. Design
square wave generator using IC 555 for VO = 8 V (amplitude) and f0 = 1 KHz and duty
cycle = 50% .
3) Explain construction, working and drain characteristics and transfer characteristics of N
channel depletion type MOSFET.
_____________________

Set A

SLR-BB 126

*SLRBB126*
Seat
No.

Set

S.E. (Electronics Engineering) (Old) (Part II) Examination, 2014


ANALOG COMMUNICATION
Day and Date : Thursday, 27-11-2014
Time : 3.00 p.m. to 6.00 p.m.

Max. Marks : 100

Instructions : 1)
2)
3)
4)

All questions are compulsory.


Figures to the right indicates full marks.
Assume suitable data if necessary.
Q. No. 1 is compulsory. It should be solved in first 30 minutes in
Answer book Page No. 3. Each question carries one mark.
5) Answer MCQ/Objective type questions on Page No. 3 only. Dont
forget to mention, Q.P. Set (A/B/C/D) on Top of Page.
MCQ/Objective Type Questions

Duration : 30 Minutes

Marks : 20

1. Select suitable option :

20

1) The advantage of modulation techniques is/are _________


a) Increase the range of communication
b) Reduces the height of antenna
c) Improves quality of reception
d) All of these
2) The noise level in the system is proportional to _________
a) Temperature

b) Bandwidth

c) Resistance

d) All of these

3) The noise figure of a communication system


a) Varies linearly as Teq
c) Varies linearly as I/Teq

b) Varies linearly as Teq2


d) Does not vary with Teq

4) When E1, E2, E3 and E4 are simultaneous modulating voltages, then the total modulating
voltage will be
a) E1 + E 2 + E 3 + E 4
c)

E1E2 + E3E4

b) (E1 + E 2 + E 3 + E 4 ) / 4

d)

E12 + E 22 + E 23 + E 24

5) One of the advantage of base modulation over collector modulation of a transistor class
C amplifier is
a) The lower modulating power required
b) Higher power output per transistors
c) Better efficiency
d) Better linearity
P.T.O.

SLR-BB 126

*SLRBB126*

-2-

6) The bandwidth required of AM wave is


a) 2fm where fm is the highest modulating frequency
b) fm where fm is the highest modulating frequency
c) 2nfm, where n is number of significant sidebands
d) fm + fc where fc is the carrier frequency
7) In conventional AM superhetrodye receivers, the detectors employed to extract intelligence
include
a) Peak detectors
b) Phase locked loops
c) Ratio detectors
d) Slope detectors
8) The IF amplifier in receiver is
a) Single stage single tuned amplifier
c) Double tuned amplifier

b) Two stage of single tuned amplifier


d) Class C amplifier

9) Double spotting in superhetrodyne receiver is caused by


a) Poor front end rejection
b) Misalignment of receiver
c) Detuning of one or more IF amplifier stages
d) Non functioning of ACG
10) The minimum value of noise factor is _________
a) Zero
b) One
c) 10

d) 100

11) Line of sight communication is not a factor in which frequency range


a) VHF
b) UHF
c) HF
d) Microwave
12) The type of radio wave responsible for long distance communication by multiple skips is
the
a) Ground wave
b) Sky wave
c) Surface wave
d) Space wave
13) What type of modulator plug is used on telephone cable ?
a) BNC
b) F-connector
c) RJ-II

d) RJ-45

14) Frequency of dial tone is


a) 133 Hz
b) 400 Hz

c) 33 Hz

d) 100 Hz

15) Over modulation occurs when


a) Vm > Vc
b) Vm < Vc

c) Vm = Vc

d) Vm = Vc = 0

16) A resonant antenna has a length which is


a) Half wavelength long
c) An exact multiple of half wavelength

b) Quarter-wavelength long
d) An exact multiple of quarter wavelength

17) The phenomenon of a strong FM signal dominating a weaker signal on common frequency
is referred to as the
a) Capture effect
b) Blot effect
c) Quieting factor
d) Dominating syndrome
18) A pre-emphasis circuit is a
a) Low pass filter
b) High pass filter

c) Phase shifter

d) Band pass filter

19) Tropospheric scatter is used with frequency in the following range


a) HF
b) VHF
c) UHF
d) VLF
20) When the modulating frequency is doubled, the modulation index is halved, and modulating
voltage remains constant. The modulation system is
a) Amplitude modulation
b) Phase modulation
c) Frequency modulation
d) Any one of the three

______________

Set A

*SLRBB126*

-3-

SLR-BB 126

Seat
No.

S.E. (Electronics Engineering) (Old) (Part II) Examination, 2014


ANALOG COMMUNICATION
Day and Date : Thursday, 27-11-2014
Time : 3.00 p.m. to 6.00 p.m.

Marks : 80

Instructions : 1) All questions are compulsory.


2) Figures to the right indicates full marks.
3) Assume suitable data if necessary.
SECTION I
2. Attempt any four :

16

a) With suitable diagram explain different elements of a communication system.


b) What are different external noises ?
c) Explain the effect of cascade connection on signal to noise ratio.
d) Derive an equation for total power in AM wave in terms of carrier power.
e) What is image frequency ? Why they occur ?
3. Solve any three :

24

a) The output voltage of a transmitter is given by 400 (1 + 0.4 sin 6280 t) sin 3.14 107t. This
voltage is fed to a load of 600 . Determine
a) Carrier frequency
b) Modulating frequency
c) Carrier power
d) Total power output.
e) Peak power output.
b) Draw and explain the two stage IF amplifier. State the factor consider in choice of
intermediate frequency.
c) What is Bandwidth requirement of AM wave ? Explain with suitable diagram and examples
of - Single Modulating Signal and Voice Modulating Signal.
d) Derive an expression for noise equivalent resistance due to cascading of many amplifiers.

Set A

SLR-BB 126

-4-

*SLRBB126*

SECTION II
4. Attempt any four :

(54=20)

a) Explain Sky wave and Space wave propagation.


b) Write a short on yagi antenna with its characteristics.
c) Define calling rate, holding time, grade of service and explain busy tone of an exchange.
d) What is a side tone in telephony ?
e) Explain in brief pre-emphasis and de-emphasis.
5. Attempt any two :

(102=20)

a) Write the sequence of operation in blocking and non blocking cross bar switch.
b) i) In an FM system, when the audio frequency is 500 Hz and AF voltage is 2.4v, the
deviation is 4.8Khz. If AF voltage is now increased to 7.2V, what is the new deviation ?
If any voltage is now raised to 10V while AF is dropped to 200 Hz, What is the deviation ?
Find modulation index in each case.
ii) Write a short note on horn antenna also list its characteristics.
c) Draw and explain the Armstrong method of FM generation with its block diagrams and
vector diagram.
_____________________

Set A

SLR-BB 127

*SLRBB127*
Seat
No.

Set

S.E. (Electronics) (Part II) Examination, 2014


LINEAR INTEGRATED CIRCUITS (Old)
Day and Date : Friday, 28-11-2014
Time : 3.00 p.m. to 6.00 p.m.

Max. Marks : 100

Instructions : 1) Q. No. 1 is compulsory. It should be solved in first 30 minutes


in Answer book Page No. 3. Each question carries one mark.
2) Answer MCQ/Objective type questions on Page No. 3 only.
Dont forget to mention, Q.P. Set (A/B/C/D) on Top of Page.
MCQ/Objective Type Questions
Duration : 30 Minutes

Marks : 20

1. Choose the correct answer :

20

1) Voltage series feed back amplifier is also called as


a) Noninverting amplifier with feed back
b) Inverting amplifier with feed back
c) Noninverting amplifier
d) None of above
2) Zero crossing detector is a special case of
a) Basic comparator
b) Peak detector
c) Schmitt trigger
d) Window detector
3) Differential amplifier with two op-amp has same characteristics as
a) Inverting amplifier
b) Noninverting amplifier
c) Comparator
d) Instrumentation amplifier
4) If VUT = 4V and VLT = 2V for schmitt trigger, then the hysteresis voltage is
a) 2V

b) 6V

5) For 741C op-amp, UGB is


a) 5Hz
b) 1Hz
6) For 741C, SVRR is
a) 150V/ v
b) 15 v /V
7) Noninverting buffer is nothing but
a) Comparator
c) Voltage follower

c) 8V

d) None of these

c) 1MHz

d) None of these

c) 155 v /V

d) 150 v /V

b) Differentiator
d) None of these

8) If square wave input is applied to a differrentiator, then out put will be


a) Inverted square wave
b) Cosine wave
c) Sine wave
d) Spikes
P.T.O.

SLR-BB 127

*SLRBB127*

-2-

9) Ideal value of CMRR of an op-amp should be


a) Infinite
b) zero
c) godB

d) None of these

10) What is the intermediate stage block in the block diagram of op-amp ?
a) DIBO differential amplifier
b) DIUO differential amplifier
c) Level shifting
d) Complementary-symmetry push-pull amplifier
11) For an oscillator, magnitude of loop gain must be
a) Zero
b) Infinite
c) 1
d) >1
12) Triangular waveform generator consists of
a) Comparator and differentiator
b) Integrator and comparator
c) Comparator and Integrator
d) None of above
13) Log amplifier uses in its feedback path
a) Transistor
b) Capacitor
c) Diode

d) Both a) and c)

14) A square wave generator is


a) Astable multivibrator
b) Monostable multivibrator
c) Bistable multivibrator
d) None of above
15) For an oscillator total phase shift of loop gain is
a) 90
b) 0
c) 270
d) 180
16) All pass filters are used to provide
a) filtering higher frequencies
c) Filtering lower frequencies

b) Predictable phase shift


d) None of above

17) For a phase shift oscillator, frequency is


a)

1
2 RC

b)

1
2 RC

c)

1.159
RC

d)

0.065
RC

18) VCO output frequency is controlled by


a) I/P frequency
b) RC component
c) Control voltage
d) None of above
19) A low pass filter has
a) Constant gain from fL
c) Constant gain from OHz to fH

b) Constant gain from fH


d) None of above

20) The lock range usually increases with


a) Increase in supply voltage
b) Increase in temperature
c) Increase in input voltage
d) None of above
______________
Set A

*SLRBB127*

-3-

SLR-BB 127

Seat
No.

S.E. (Electronics) (Part II) Examination, 2014


LINEAR INTEGRATED CIRCUITS (Old)
Day and Date : Friday, 28-11-2014
Time : 3.00 p.m. to 6.00 p.m.

Marks : 80

Instructions : 1) Figures to the right indicate full marks.


2) Assume suitable data whenever necessary
SECTION I
2. A) Solve any two :
(26=12)
1) What is clipper ? Explain positive clipper in detail.
2) Explain integrator using op-amp and draw frequency response of it.
3) The 741C op-amp having the following parameters are connected in voltage
follower. A = 200000, Ri = 2M , Ro = 75 , fo = 5Hz, supply voltage = 15V
output voltage swing = 13v
Calculate the values of AF, RiF, RoF, fF and VooT.
B) Solve all two :
1) Explain peak detector
2) Explain V to I converter with grounded load.

(24=8)

3. A) Solve any one :


(18=8)
1) Draw circuit diagram of closed loop inverting amplifier using op-amp and
derive the expression for voltage gain.
2) Explain instrumentation amplifier in detail.
B) Solve all three :
(34=12)
1) Design a differentiator to differentiate an input signal that varies in
frequency from 10 Hz to 1KHz (use practical circuit)
2) Explain DC analysis of DIBO differential amplifier.
3) Explain effect of slew rate on open loop noninverting amplifier using
op-amp.
Set A

SLR-BB 127

-4-

*SLRBB127*

SECTION II
4. Attempt any two :

(102=20)

a) Explain phase locked loop in detail with phase detector, LPF and VCO.
b) Explain triangular waveform generator. Derive equation of its frequency of
oscillations.
c) Design a wide band-pass filter with fL = 600 Hz and fH = 1 KHz and a passband
gain = 4. Construct circuit using HPF and LPF.
5. Solve any four :

(54=20)

a) Explain saw tooth wave generator.


b) Explain square wave generator.
c) Explain antilog amplifier using op-amp.
d) Design triangular waveform generator for fo = 5KHz, Vo(p p) = 8V.
e) Explain wein bridge oscillator.
_____________________

Set A

SLR-BB 128

*SLRBB128*
Seat
No.

Set

S.E. (Electronics) (Part II) (Old) Examination, 2014


SIGNALS AND SYSTEMS
Day and Date : Saturday, 29-11-2014
Time : 3.00 p.m. to 6.00 p.m.

Max. Marks : 100

Instructions : 1) Assume suitable data if necessary.


2) Q. No. 1 is compulsory. It should be solved in first 30 minutes in
Answer Book Page No. 3. Each question carries one mark.
3) Answer MCQ/Objective type questions on Page No. 3 only. Dont
forget to mention, Q.P. Set (A/B/C/D) on Top of Page.
MCQ/Objective Type Questions
Duration : 30 Minutes

Marks : 20

1. Choose the correct option :

(201=20)

1) If y(t) = x(t) x(t 20), the system is


a) Linear
c) Linear and causal

b) Causal
d) None of these

2) If x(n) = x(n) the signal is said to be


a) Linear
b) Even

c) Old

d) Causal

3) The LTI system can be completely characterised by its


a) Convolution property
b) Step response
c) Impulse response
d) Stability
4) The CT system is causal if
a) h(t) = 0 for t < 0
c) h(t) = 0 for t > 0

b) h(t) 0 for t < 0


d) h(t) 0 for t > 0

5) Majority of physical systems can be modelled by


a) Linear system
b) Recursive system
c) None recursive system
d) LIT system
6) Roles of input signal and impulse response are interchangeable, this is the ___________
property of convolution.
a) Associative
b) Distributive
c) Cumulative
d) Commutative
7) The impulse response of memoryless system is of the following form
a) u(n)
b) h(n) = k (n)
d) None of these
c) h(n) = (n + 1)
8) The signal x(n) = { 1, 2, 2} is equivalent to

a) u(n) u(n 2)
c) 2{u(n) u(n 2)}

b) (n) + 2 (n 1) + 2 (n 2)
d) None of these
P.T.O.

SLR-BB 128

*SLRBB128*

-2-

9) The impulse response of cascade combination of two systems with impulse response
h1(n) and h1(n) is
a) h1(n) + h2(n)
b) h1(n) h2(n)
c) h1(n) h2(n)
d) None of these
10) The signal x(t) = 2cos (0 t ) is
a) Periodic
b) Aperiodic

c) Causal

11) For x(n) = an u(n), ROC of X(z) is


a) Inside the unit circle
c) |z| > a

b) |z| < a
d) Circumference of circle with r = 1

d) None of these

12) If x(t) X(jw) then FT of x(t 2) is


a) ejw

b) e j2 X( jw)

13) The Fourier transform of 1 is


b) 2(w)
a) ( w)

c) e j 2 w X( jw)

d) None of these

c) j( w)

d) None of these

14) Interpolation using the impulse response of ideal lowpass filter is


a) Band limited interpolation
b) Zero order
c) Sample and hold interpolation
d) Initializing
15) In sampling the sampling period Ts should be
a) <

1
2Bm

b)

1
2Bm

c) > 12Bm

16) For m(t) = 2 sin 48 t + 3 sin 72 t, if Fs = 60 Hz, signal is


a) Under sampled
b) Over sampled
c) Both

d) 12Bm
d) None

17) The relation between trigonometric and exponential Fourier series can be given as
a) an = Cu C n
b) an = (Cu + C n)
c) an = j(Cu + C n)
d) None of these
18) Sampling process converts signals
a) Analog to digital
c) DT to CT

b) Continuous to discrete
d) All these

dX( t)
19) Ha F
is
dt
a) X(jw)

1
b) jw X( jw )

c) jw X(jw)

d) ejw X(jw)

20) For right sided infinite duration sequence the ROC is


a) Exterior to a circle
b) Interior of a circle
c) Annular ring
d) None of these
______________

Set A

*SLRBB128*

-3-

SLR-BB 128

Seat
No.

S.E. (Electronics) (Part II) (Old) Examination, 2014


SIGNALS AND SYSTEMS
Day and Date : Saturday, 29-11-2014
Time : 3.00 p.m. to 6.00 p.m.
Instructions :

Marks : 80

1) All questions are compulsory.


2) Assume suitable data if necessary.
SECTION I

2. Attempt any four :

(46=24)

a) Check whether following systems are linear or not.


i) y(t) = t x(t)

ii) y(t) = x2(t).

b) Consider signal x(n) = { 2, 2, 3, 1, 4} . Obtain the even and odd parts of x(n).

c) Consider x(t) as below

sketch following :
i) x(t + 2)

ii) x(t) u(t 1).

d) Determine the fundamental period of following signals :


i) x(t) = 2cos(10t + 1) sin(4t 1)
ii) x[n] = 1 + e j 4 n / 7 e j2 n / 5 .
e) Define singularity function. Find the integral and derivatives of singularity function.
3. Attempt any two :

(28=16)

a) Explain the methods of DT system realization using direct form I and direct form II with
suitable example.
b) Let x[n] = [n] + 2 [n 1] [n 3], h[n] = 2 [n + 1] + 2 [n 1] compute and plot.

i) x(n) h(n)
ii) x[n + 2] h(n)
c) Determine whether following systems are causal and/or stable. Justify your answers.
i) h(t) = e4t u(t 2)

ii) h[n] = u[n  2] u[n].

Set A

SLR-BB 128

-4-

*SLRBB128*

SECTION II
4. Attempt any four :

(46=24)

a) If x(t) = 4cos 100 t, calculate sampling rate for Fs = 200 Hz and 75 Hz. Obtain the DT
signal.
b) State and explain sampling theorem in frequency domain.
c) State and prove the convolution property of Fourier transform.
d) Find z transform of following sequences and sketch ROC.
i) x1(n) = 3 (n) + (n 2) + (n 4)
ii) x2(n) = (n +1) + 2 (n + 2).
e) Derive the relation between trigonometric and exponential Fourier series coefficients.
5. Attempt any two :

(28=16)

a) Draw the amplitude and phase spectrum of periodic time domain signal x(t) shown below.

b) Determine the DT signal associated with following :


i) X(z) =

1
1+

1 z 1
2
,|z
1 z 1
2

| > 1 is ROC use power series method.


2

ii) X(z) = 1 + z1 + z2.


c) Find the Fourier transform of signal x(t), using differentiation property.

_____________________

Set A

SLR-BB 129

*SLR-BB-129*
Seat
No.

Set

S.E. (Electronics) (Part II) Examination, 2014


ELECTRICAL MACHINES (New)
Day and Date : Tuesday, 25-11-2014
Time : 3.00 p.m. to 6.00 p.m.

Max. Marks : 100

Instructions : 1) Figures to the right indicates full marks.


2) Assume suitable data whenever necessary.
3) Q. No. 1 is compulsory. It should be solved in first 30 minutes in
Answer book Page No. 3. Each question carries one mark.
4) Answer MCQ/Objective type questions on Page No. 3 only.
Dont forget to mention, Q.P. Set (A/B/C/D) on Top of Page.
MCQ/Objective Type Questions

Duration : 30 Minutes

Marks : 20

1. Choose correct option :


(201=20)
1) The d.c. series motor should always be started with load because
a) at no load, it will rotate at dangerously high speed
b) it will fail to start
c) it will not develop high starting torque
d) all are true
2) The frequency of the rotor current in a 3 phase 50 Hz, 4 pole induction motor at full
load speed is about
a) 50 Hz.
b) 200 Hz.
c) 2 Hz.
d) Zero
3) In case of slip ring induction motor
a) rotor winding are connected in delta connection
b) rotor winding are connected in star connection
c) rotor is made of iron bars
d) rotor is made of copper bars
4) Auto transformer is __________
a) single winding transformer
b) single turn transformer
c) single core transformer
d) none of the above
5) The speed of a dc series motor is
a) directly proportional to back emf and flux
b) directly proportional to its back emf and inversely proportional to flux
c) inversely proportional to both back emf and flux
d) directly proportional to flux and inversely proportional to back emf.
6) Swinburn test is used for finding efficiency of
a) DC shunt motor
b) DC series motor
c) Both
d) None of the above
P.T.O.

SLR-BB 129

-2-

*SLR-BB-129*

7) The resistance of the shunt field winding is of DC shunt motor is _________


a) 0.01
b) 102
c) 0.1
d) 1
8) In case of slip ring induction motor
a) Rotor is wound
b) Stator is wound
c) Both a) and b)
d) None of the above
9) Which is the non-conventional type energy source ?
a) Wind
b) Solar
c) Both
d) None of the above
10) The generation voltage is usually
a) between 11 KV and 33 KV
b) between 132 KV and 400 KV
c) between 400 KV and 700 KV
d) none of the above
11) __________ is the starter not used for three phase induction motor.
a) DOL starter
b) Auto transformer starter
c) Star-delta starter
d) Delta-star starter
12) In case of d.c. series motor torque is proportional to
b) I2a
c) Ia/2
d) Va
a) Ia
13) The leading power factor is due to ______________ load.
a) filament lamp
b) heater
c) capacitive
d) induction motor
14) In case of three phase transformer eddy current losses are reduced by using
a) sold core
b) round core
c) laminated core
d) None of the above
15) Which of the following connections is best suited for high voltage and small size
service ?
a) delta-delta
b) V-V
c) star-star
d) T-T
16) The rotor input of 3 ph. I.M. is P Kw and slip is s, then per phase rotor copper loss
is
a) s P/3
b) 3 sP
c) sP
d) P/3 s
17) The direction of rotation of a dc motor can be determined by
a) Flemings right hand rule
b) Flemings left hand rule
c) Lenz law
d) Amperes law
18) DC _____________ motor is never used for any application.
a) shunt
b) series
c) cumulative compound
d) differential compound
19) In case two wattmeter method power factor is unity when
a) W1>W2
b) W1<W2
c) W1=W2
d) W1 or W2 becomes zero
20) Universal motor runs satisfactorily on ____________ supply.
a) a.c.
b) d.c.
c) a) and b)
d) none of the above
______________

Set A

*SLR-BB-129*

-3-

SLR-BB 129

Seat
No.

S.E. (Electronics) (Part II) Examination, 2014


ELECTRICAL MACHINES (New)
Day and Date : Tuesday, 25-11-2014
Time : 3.00 p.m. to 6.00 p.m.

Marks : 80

Instructions : 1) Figures to the right indicates full marks.


2) Assume suitable data whenever necessary.
SECTION I
2. Attempt any four :

(64=24)

a) Write short note on Swinburn Test used to test DC shunt motor.


b) Two wattmeter method is used to measure input power of 3-phase load and input
power measure is 10 KW. Calculate W1 and W2 if power factor is

i) 0.5

ii) 1.

c) Explain with neat diagram brushless dc motor.


d) A 2 pole d.c.series motor runs at 800 (rpm) when taking 100 (Amp) at 300 (Volt) and
with Rm = 0.1 . If speed is reduced to be 600(rpm) by inserting resistance in
series with motor. Calculate resistance if load current remains constant.
e) Compare three point and four point starter for field failure protection and application
for very high speed regulation.

3. a) Explain with neat circuit diagram flux changing method to control the speed
d.c. shunt motor. Write its merits and demerits.
(82=16)
b) A 230 (Volt) d.c. shunt motor runs at 1500 (rpm). Calculate the value of back emf
and armature current at staring., take Ra = 1 . Comment on this starting
current and explain necessity of starter.

OR
b) Explain with neat diagram following braking methods used for d.c. shunt motor and
d.c. series motor :

i) dynamic braking
ii) plugging.

Set A

SLR-BB 129

-4-

*SLR-BB-129*

SECTION II
4. Attempt any four :

(64=24)

a) Explain with neat diagram following 3-phase transformer connection

i) star-star
ii) delta-delta
iii) star-delta
b) 3-phase, 8-pole, 50 Hz slip ring induction motor is running at 740 rpm. Calculate
slip and rotor current frequency. If slip is 4% find speed of motor.
c) Explain working principal of three phase induction motor.
d) Draw neat diagram, explain features and write advantages of group drive.
e) Draw neat diagram and explain three phase to three phase transformation by using
Scott connection.
5. Attempt any two :

(82=16)

a) Define low power factor. What are the different causes for this ? Explain any one
method used for improvement of power factor.
b) Derive relation between starting torque, maximum torque and find condition for
maximum starting torque.
c) A three phase, 6 pole, 50 (Hz) running on full load with 4% slip develops a
torque of 149.3 NM. The friction and windage losses are 200 watts stator losses
are 1620 watts.
Find out put power, rotor copper loss and motor efficiency at full load.
_____________________

Set A

SLR-BB 13

*SLRBB13*
Seat
No.

Set

F.E. (Part II) Examination, 2014


ENGINEERING MATHEMATICS II
Day and Date : Tuesday, 25-11-2014
Time : 10.00 a.m. to 1.00 p.m.

Max. Marks : 100

N. B. : 1) Q. No. 1 is compulsory. It should be solved in first 30 minutes in


Answer book Page No. 3. Each question carries one mark.
2) Attempt any three questions from each Section.
3) Figures to the right indicate full marks.
4) Use of calculator is allowed.
5) Answer MCQ/Objective type questions on Page No. 3 only. Dont
forget to mention, Q.P. Set (A/B/C/D) on Top of Page.
MCQ/Objective Type Questions
Duration : 30 Minutes

Marks : 20

1. Tick (9) the correct alternative :

20

dy
= e x y is _____________
dx
b) x = ey + C
c) x = logy + C
d) ey = ex + C

1) The solution of differential equation


a) y = ex + C

2) The differential equation (3a2x2 + by cosx)dx + (2sinx 4ay3)dy = 0 is exact for _______
a) b = 3
b) b = 2
c) b = 4
d) b = 5
3) For the curve y = x(x2 1) the equation of the tangent at the origin is __________
a) y = x
b) y = x
c) x = 1
d) x = 1
4) The length of the normal to the curve y = f(x) is given by _________
y
a)

dy
dx

5) The IF of (x + 1)
a) x + 1

b) y

dy
dx

dy
c) y 1 +
dx

dx
d) y 1 +
dy

dy
y = e 3 x (x + 1)2 is ____________
dx
b) log(x + 1)

1
c) x + 1

d) ex + 1

dy y x
=
with y(0) = 1 and h = 0.02 then by Eulers method, the value of y(0.02) is
dx y + x
a) 1.02
b) 1
c) 1.04
d) 1

6) If

7) If

dy
= 1 + xy with y(0) = 0, then the Picards second approximation is
dx

2
3
a) x + x
2
3

3
b) x + x
3

2
c) x + x
2

3
2
d) x + x
3

P.T.O.

SLR-BB 13

*SLRBB13*

-2-

8) For fourth order Runge-Kutta method which of the following is wrong ?


K1

h
a) K 1 = hf (x 0 , y 0 )
b) K 2 = hf x 0 + 2 , y 0 + 2

c) K 3 = hf( x 0 + h, y 0 + K 2 )

d) K 4 = hf( x 0 + h, y 0 + K 3 )

9) Sterling formula is used to find the derivative at


a) The beginning
b) The central
c) The end

d) None of these

d2 y
10) In Newtons forward difference formula 2
= 12 D2 y 0 D 3 y 0 + K + .... the value
dx x = x 0 h

of K is __________
a)

11 4 y
0
12

11) The curve y =

11 4 y
b)
0
12

c)

12 4 y
0
11

d) None of these

x
is symmetrical
1 + x2

a) About X-axis

b) About Y-axis

c) In opposite quadrants d) None of these

12) A double point is called a node if the two tangents there at are
a) Coincident
b) Imaginary
c) Real and distinct d) None of these
13) The curve r = a sin 5 has _______
a) 4 loops
b) 5 loops

c) 10 loops

14) The process of finding the length of arc of a curve is called


a) Interpolation
b) Extrapolation
c) Rectification

15) If I(m) =

d) no loops
d) None of these

1 cos mx x
dI =
e dx then
_____
dm
x

1
c) m 2 1

m
d) m 2 1

16) The value of 0 is equal to


a) 0
b)

c) 1

d) None of these

17) For an integer m > 0, B(m, 1) = _____


1
a) m
b) m !

1
c) m + 1

d)

1
a) m2 + 1

18)
0

m
b) m 2 + 1

1
m

r cos drd = __________


0

1
1
1
b) 2
d)
c)
2
2 2
2
19) Evaluation of double integral by changing to polar co-ordinates is carried out by the substitution
a) x = cos , y = sin
b) x = rcos , y = rsin
c) x = rsin , y = rcos
d) x = rcos , y = rsin
a)

20) If the density at any point varies of the distance of the point from the x-axis, then is equal to
a) kxy
b) kx
c) ky
d) k(x2 + y2)
______________

Set A

*SLRBB13*

SLR-BB 13

-3-

Seat
No.

F.E. (Part II) Examination, 2014


ENGINEERING MATHEMATICS II
Day and Date : Tuesday, 25-11-2014
Time : 10.00 a.m. to 1.00 p.m.
N. B. : 1) Attempt any three questions from each Section.
2) Figures to the right indicate full marks.
3) Use of calculator is allowed.

Marks : 80

SECTION I
dy 6 x 2 y 7
=
.
dx
3x y + 4

dy
= y cos x + cos x sin2 x .
dx

c) Solve (xy 3 + y)dx + 2( x 2 y 2 + x + y 4 )dy = 0 .

2. a) Solve
b) Solve

3. a) Find the orthogonal trajectories of the family of curves rnsinn = an where a is the
parameter.
b) Prove that the tangent to the curve 8y = (x 2)2 at point (6, 8) is parallel to the curve

y = x + 3 at the point (1, 4).


x

2
x2 + y
= 1 where is parameter, is self orthogonal.
a 2 + b2 +
OR
c) When a resistance R ohms is connected in series with an inductance L henries with an

c) Show that the family

emf of E volts, the current i amperes at time t is given by L di + Ri = E . If E = 10 sint volts


dt
and i = 0, when t = 0, find i as a function of t.
4. a) Solve by Picards method

dy
= y k with y(0) = 2 upto y(5).
dx

b) Using Runge-Kutta method of fourth order find y(0.2), given that


y(0) = 1 in two steps.

6
dy
= xy + y 2 with
dx

5. a) Find first and second derivative at x = 1 and x = 8 for the following data :
x
y
b) Find

0
7

2
13

4
43

6
145

5
150

7
392

11
1452

13
2366

8
367

d2 y
dy
at x = 9 using the following data :
and
dx
dx 2

x
y

7
17
5202

Set A

SLR-BB 13

*SLRBB13*

-4-

SECTION II
6. a) Evaluate

x e

3x

dx .

b) Prove that

c) Evaluate

x sin5 x cos 4 x dx = 8 .
315

log(1 + ax 2 )
dx , (a > 0).
x2

7. a) Trace the curve r = a cos 2 .

b) Trace the curve x = a cos t + 1 a log tan2 t , y = a sin t .


2
2

c) Find the length of the arc of parabola y2 = 4ax, cut off by the line 3y = 8x.

8. a) Evaluate

log 2

b) Evaluate

x+y

e x + y + z dxdydz .

x(x y) dxdy where R is the triangle with vertices (0, 0), (1, 2), (0, 4).

c) Express the following integral in polar coordinates and evaluate

a
0

a2 x 2

ax x 2

dxdy

a2 x 2 y2

. 5

OR
2a x

c) Change the order of integration and evaluate


0


x2

xydxdy .

9. a) Find the area outside the circle r = a and inside the cardioid r = a(1 + cos ) by double integration. 4
2

x2 y
b) Find the mass of the lamina in the form of an ellipse 2 + 2 = 1 (a> b) if the density at any
a
b
point varies as the product of the distance from the axes of the ellipse.
c) Find by double integration the volume of the solid generated by revolving the region bounded
by y2 = 4x and y2 = 5 x about the Y-axis.

_____________________

Set A

SLR-BB 130

*SLRBB130*
Seat
No.

Set

S.E. (Electronics) (Part II) (New) Examination, 2014


ELECTRONIC CIRCUIT ANALYSIS AND DESIGN II
Day and Date : Wednesday, 26-11-2014
Time : 3.00 p.m. to 6.00 p.m.

Max. Marks : 100

Instructions : 1)
2)
3)
4)

Figures to the right indicates maximum marks.


Use of electronic component data sheet is allowed.
Assume suitable data if necessary.
Q. No. 1 is compulsory. It should be solved in first 30 minutes in
Answer book Page No. 3. Each question carries one mark.
5) Answer MCQ/Objective type questions on Page No. 3 only. Dont
forget to mention, Q.P. Set (A/B/C/D) on Top of Page.
MCQ/Objective Type Questions

Duration : 30 Minutes

Marks : 20

1. Choose the correct answer :


1) A multistage amplifier employs four stages each of which has voltage gain of 40, then
overall gain of amplifier is __________
a) 128.16dB
b) 1281.6dB
c) 12.81dB
d) 1.28 dB
2) A multistage amplifier consists of three similar stages. If each stage has RC = 2.4K ,
Rin = 1K and hfe = 60 then overall voltage gain in db is __________
a) 43.16
b) 12.95
c) 129.5
d) 1295
3) The efficiency of CLASS A transformer coupled power amplifier is
a) 78.5%
b) 0 to 25%
c) 50%
d) 78.5% and 100%
4) The upper 3 dB frequency for N identical stages is given by
a) fH(n) = fH

1
n
2

+1

b) fH(n) = fH

1
n
2

1 c) fH(n) = fH 1 2 n

d) fH(n) =

fH
1
n
2

+1
5) An amplifier has voltage gain of 400, fL = 50Hz. A resistive voltage negative feedback
is added such that k=0.01. Then lower cutoff frequency with feedback is
a) 10 Hz
b) 0.10 Hz
c) 20 Hz
d) 0.01 Hz

6) If D = 10%, Av = 40, K = 0.01. Distortion factor with feedback is __________


a) 7.4%
b) 7.01%
c) 7.00%
d) 7.14%
7) In voltage series feedback the feedback factor k is __________
a)

Rf1
Rf1 + Rf2

b)

Rf1
Rf2

8) Voltage shunt feedback is


a) Transresistance amplifier
c) Transconductance amplifier

c)

Rf1 + Rf2
Rf1

d)

Rf1
Rf1 Rf2

b) Current controlled voltage source


d) Both a) and b)
P.T.O.

SLR-BB 130

*SLRBB130*

-2-

9) Class AB power amplifier is generally used for _________


a) Good frequency response
b) To eliminate cross over distortion
c) Impedance matching
d) None of the above
10) Power amplifier is a power converter which converts __________ under control of input
signal.
a) AC power into DC power
b) DC power into AC power
c) DC power into DC power
d) AC power into AC power
11) The output voltage of LM 317 as adjustable regulator is

a) Vo = Vref 1 + R L + Iadj R L
R1

b) Vo = Vref 1 R L + Iadj R L
R1

R
c) Vo = Vref 1 L Iadj R L
R1

d) Vo = Vref 1 + R L Iadj R L

R 1

12) The percentage change in output voltage for given change in input voltage is
a) Load regulation
b) Thermal regulation
c) Line regulation
d) Line and load regulation
13) Astable multivibrator can be used as
a) Sine wave converter
c) Voltage to frequency converter

b) Square wave generator


d) Both b) and c)

14) If charging time of astable multivibrator is 5 seconds and discharging time is 4 seconds
then Duty cycle = ___________
a) 65.55%
b) 55.55%
c) 75%
d) 100%
15) In __________ frequency is determined by charging and discharging time constants during
exchange of energy.
a) Relaxation oscillator
b) RC phase shift oscillator
c) Wien bridge oscillator
d) Sinusoidal oscillator
16) The protection circuit which not only limits the output current, but gradually reduces to
zero if short circuit condition occur.
a) Simple current limiting circuit
b) Thermal shut down circuit
c) Foldback current protection circuit
d) Short circuit protection circuit
17) Pre regulator in voltage regulator acts as
a) Reference voltage source
c) Pre amplifier

b) Constant current source


d) All above

18) In Wein bridge oscillator if resistance in positive feedback circuit is increased, the frequency
_________
a) Increases
b) Decreases
c) Doubles
d) Remains same
19) For an Ideal voltage regulator, the ideal values of line regulation and load regulation are
a) 0, 0
b) , 0
c) ,
d) None of the above
20) For Wein bridge oscillator, R = 10K , find the range of capacitor required to generate
variable frequency of 1 KHz to 10 KHz.
a) 15.91nF-1.591nF
b) 15.91 F-1.591 F
d) None of these
c) 1.591 F-15.91 F

______________

Set A

*SLRBB130*

-3-

SLR-BB 130

Seat
No.

S.E. (Electronics) (Part II) (New) Examination, 2014


ELECTRONIC CIRCUIT ANALYSIS AND DESIGN II
Day and Date : Wednesday, 26-11-2014
Time : 3.00 p.m. to 6.00 p.m.

Marks : 80

Instructions : 1) Figures to the right indicates maximum marks.


2) Use of electronic component data sheet is allowed.
3) Assume suitable data if necessary.
SECTION I
2. Solve any four questions :

(45=20)

1) What is multistage amplifier circuit ? Discuss how to determine frequency response in


multistage amplifier.
2) Explain working of class AB power amplifier.
3) Derive an expression for Ai, Avf, Rif, Rof for Emitter follower circuit.
4) A class B push pull amplifier delivers 10W of audio power to load
a) If transformer is 80% efficient what is minimum power drain on power supply under
optimum condition.
b) What is minimum average dissipation rating required for each transistor ?
5) Compare the effect of voltage series, voltage shunt, current series and current shunt
feedback on following amplifier characteristics :
i) Voltage gain
ii) Input resistance
iii) Output resistance
iv) Bandwidth
3. Solve any two questions :

(102=20)

1) Explain working of class A transformer coupled power amplifier. Derive an expression for
its maximum efficiency and power dissipation.
2) Design a two stage current series feedback amplifier using 2N3904, Vcc=24V,
R(load) = 120k , fL=100Hz, hfe=100, hie=1k, Avf=1000 Rs = 600 .

Set A

SLR-BB 130

-4-

*SLRBB130*

3) Three common emitter RC coupled stages are cascaded with R = R1 || R2 = 1k ,


R1
= 5.2K , R2=1.24k , RE=1k , RC=2K CE1 = CE2 = CE3 = 50 f, CC1= CC2 = CC3
= CC4 = 1 f, hie = 1K , hfe = 50, Rs = 600 , VCC = 24V. Assume that all the three
stages are symmetric and consider R1, R2, RC and RE same for all three stages. For all
stages RE is bypassed. Draw suitable circuit diagram.
Calculate :
i) Overall voltage gain
ii) Overall current gain
iii) Power gain
iv) The overall value of fL for V0/Vs due to Cc alone
v) The overall value of fL for V0/Vs due to Ce alone.
SECTION II
4. Solve any four questions :

(45=20)

1) Explain working of a voltage to frequency converter using IC555.


2) Explain working of IC74121 with suitable circuit diagram.
3) Design constant current source to deliver 0.8A current to the load of 10 , 10W using
IC 7805.
4) Explain current boosting circuit for IC voltage regulators.
5) Explain working principle of oscillator circuit and Barkhausen criteria for oscillation.
5. Solve any two questions :

(102=20)

1) Explain working of Colpitts Oscillator. Derive an expression for frequency of oscillation


and minimum gain of sustained oscillation.
2) Derive an expression for frequency and duty cycle of astable multivibrator. Design
square wave generator using IC555 for frequency f0 = 1KHz and amplitude of output
voltage 5V.
3) Design a transistorized series voltage regulator using pre-regulator for 15V output voltage
at 100 mA. The unregulated power supply provides output of 24V. Use transistor
BC 147B PDmax = 250mW, hfe=240, ECN100 - PDmax = 5W, hfe = 50, PNP transistor
2N941 hfe = 10, ICmax = 50mA.
_____________________

Set A

SLR-BB 131

*SLR-BB-131*
Seat
No.

Set

S.E. (Electronics Engineering) (New) (Part II) Examination, 2014


ANALOG COMMUNICATION
Day and Date : Thursday, 27-11-2014
Time : 3.00 p.m.to 6.00 p.m.

Max. Marks : 100

Instructions : 1)
2)
3)
4)

All question are compulsory.


Figure to the right indicates full marks.
Assume suitable data if necessary.
Q. No. 1 is compulsory. It should be solved in first 30 minutes in
Answer book Page No. 3. . Each question carries one mark.
5) Answer MCQ/Objective type questions on Page No. 3 only.
Dont forget to mention, Q.P. Set (A/B/C/D) on Top of Page.
MCQ/Objective Type Questions

Duration : 30 Minutes

Marks : 20

1. Select suitable option :


1) Recovering information from a carrier is known as
a) Demultiplexing b) Modulation
c) Detection
d) Carrier recovery
2) Radio signals are made up to
a) Voltages and currents
b) Electric and magnetic fields
c) Electrons and protons
d) Noise and data
3) In commercial T.V. transmission in India, picture and speech signal are
modulated respectively as
a) VSB and VSB
b) VSB and SSB
c) VSB and FM
d) FM and VSB
4) The maximum useful power of an AM modulator is
a) 25%
b) 33%
c) 75%
d) 100%
5) Which of the following demodulator (s) can be used for demodulating the signal
x(t) = 5(1 + 2 cos 2000 t) cos 2000 t ?
a) envelope demodulator
b) square-law demodulator
c) synchronous demodulator
d) all of the above
6
3
6) u(t) = 5[cos (10 t) sin(10 t) sin(106 t] represents
a) DSB suppressed carrier signal
b) AM signal
c) SSB upper sideband signal
d) Narrowband FM signal
7) The image channel rejection in a superheterodyne receiver comes from
a) IF stages only
b) RF stages only
c) Detector and RF stages only
d) Detector RF and IF stages

20

P.T.O.

SLR-BB 131

-2-

*SLR-BB-131*

8) Single sideband system needs


a) more bandwidth
b) high power
c) complex receiver circuits, as compared to the other type system
d) none of the above
9) In a superheterodyne receiver, the IF is 455 kHz. If it is tuned to 1200 kHz, the
image frequency will be
a) 1655 kHz
b) 2110 kHz
c) 745 kHz
d) 910 kHz
10) In a low level modulation systems, the amplifier following the modulated stage
should be
a) only linear amplifier
b) only harmonic generator
c) only class A amplifier
d) none of the above
11) The output frequency of a frequency modulator is
a) constant
b) periodically varying
c) randomly varying
d) none of these
12) In phase modulation, the information signals modify the
a) phase angle
b) frequency
c) amplitude of the carrier
d) none of these
13) Which one of the following is an indirect way of generating FM ?
a) Maximum frequency deviation
b) Maximum permissible modulation index
c) Signal to noise voltage ratio
d) Minimum permissible modulation index
14) Which one of the following factors is limited in case of FM ?
a) Reactance FET modulator
b) Varactor diode modulator
c) Armstrong modulator
d) Reactance tube modulator
15) The noise figure of an amplifier is 3 dB. Its noise temperature will be about
a) 145 K
b) 290 K
c) 580 K
d) 870 K
16) Line of sight communication is not a factor in which frequency range.
a) VHF
b) HF
c) UHF
d) Microwave
17) A popular half wavelength antenna is the
a) Ground plane
b) Collinear
c) End fire
d) Dipole
18) What two sine wave frequencies are produced when the 8 key is pressed ?
a) 697 and 1477 Hz
b) 852 and 1330 Hz
c) 770 and 1209 Hz
d) 941 and 1336 Hz
19) The type of cable used between the telephone subscriber and central office is
a) Coaxial cable
b) Fiber optic cable
c) Twin lead
d) Twisted pair
20) The critical frequency of an ionosphere layer during the day time is
a) more than the night time
b) less than the night time
c) same as that for night time
d) none of these
______________
Set A

*SLR-BB-131*

-3-

SLR-BB 131

Seat
No.

S.E. (Electronics Engineering) (New) (Part II) Examination, 2014


ANALOG COMMUNICATION
Day and Date : Thursday, 27-11-2014

Marks : 80

Time : 3.00 p.m.to 6.00 p.m.


Instructions : 1) All question are compulsory.
2) Figure to the right indicates full marks.
3) Assume suitable data if necessary.
SECTION I
2. Attempt any four :

(44=16)

a) What is Electromagnetic Spectrum ? What is its importance ?


b) With suitable diagram explain different elements of a communication system.
c) Derive an equation for total power in AM wave in terms of carrier power.
d) Discuss advantages and disadvantages of SSB.
e) Draw and explain TRF radio receiver.
3. Solve any three :

(38=24)

a) With suitable diagram explain phase shift method for suppression of sideband.
b) Explain with suitable circuit diagram and waveforms A high level class C collector
modulator.
c) Derive an equation to show that an AM wave contains carrier, lower sideband and
upper sideband.
d) With suitable diagram explain third method for suppression of sideband.

Set A

SLR-BB 131

-4-

*SLR-BB-131*

SECTION II
4. Attempt any four :

(44=16)

a) With suitable waveform explain FM and PM.


b) Derive an expression for noise figure.
c) What is noise temperature ? What is its significance ?
d) Explain classification of switching system.
e) What are three different modes of wave propagation ? Explain its dependence of
frequency.
5. Solve any three :

(38=24)

a) With suitable circuit diagram explain a frequency modulation of a crystal


oscillator with a varactor diode.
b) Explain propagation of radio waves, polarization, isotropic radiator, power
density.
c) Explain a typical centralized SPC organization.
d) With suitable phase diagram and waveform explain Armstrong method.
_____________________

Set A

SLR-BB 132

*SLRBB132*
Seat
No.

Set

S.E. (Electronics Engineering) (New) (Part II) Examination, 2014


LINEAR INTEGRATED CIRCUITS
Day and Date : Friday, 28-11-2014
Time : 3.00 p.m. to 6.00 p.m.

Max. Marks : 100

Instructions : 1)
2)
3)
4)

All questions are compulsory.


Figures to the right indicate full marks.
Assume suitable data if necessary.
Q. No. 1 is compulsory. It should be solved in first 30 minutes in
Answer Book Page No. 3. Each question carries one mark.
5) Answer MCQ/Objective type questions on Page No. 3 only.
Dont forget to mention, Q.P. Set (A/B/C/D) on Top of Page.
MCQ/Objective Type Questions

Duration : 30 Minutes

Marks : 20

1. Select correct option :

(120=20)

1) An ideal filter has ____________ loss in pass band and ___________ loss in
stop band.
a) infinite, infinite
b) infinite, zero
c) zero, infinite
d) zero, zero
2) Voltage follower is a special case of
a) inverting amplifier
c) differential amplifier

b) non inverting amplifier


d) clipping circuit

3) The gain of the op amp decreases with frequency because of __________ within
op amp.
a) shunt resistance
b) capacitance
c) CMRR
d) level shifter
4) Which of below is not a part of analog multiplier ?
a) log amplifier
b) adder
c) level shifter

d) antilog amplifier

5) Which of below configuration of op amp has smallest bandwidth ?


a) open loop
b) non inverting c) inverting
d) differential
6) A scaling amplifier can be designed using __________ configuration.
a) inverting amplifier
b) non inverting amplifier
c) differential amplifier
d) all of above
7) ___________ is caused by current limiting and the saturation of internal stages of
Op Amp when large amplitude high frequency signal is applied.
a) Roll off
b) Thermal variation
c) Output offset voltage
d) Slew rate

P.T.O.

SLR-BB 132

*SLRBB132*

-2-

2R1 + R 2
, we are talking about
R

8) If we write, T = 2RC ln

a) square wave generator


c) second order high pass filter

b) third order low pass filter


d) no such circuit exists

9) For a first order low pass filter if fH is 1 KHz, passband gain is 2 and C = 0.01 F ,
value of R is _________ .
a) 1.59 K
b) 1 K
c) 11.9 K
d) none of these
10) To achieve a stable circuit loop gain must be _________ when its magnitude
reaches unity.
a) greater than 0
b) greater than 180
c) greater than 180
d) greater than 360
11) Higher the value of ___________ in dB, the lower is the change in input offset
voltage due to change in supply voltage.
a) CMRR
b) CVRR
c) voltage drift
d) thermal drift
12) Average of the two input bias currents of an Op amp is called
a) input current
b) offset current
c) input offset current
13) Output offset voltage changes with
a) temperature
b) time

c) supply voltage

14) Frequency of VCO NS/SE 566 is given by f0 =


a) supply voltage
c) modulating voltage

d) none of these
d) all of these

2(+ X VC )

. In this X is
R1C1(+ X )
b) reverence voltage
d) none of these

15) Which of below is not a mode of PLL ?


a) capture
b) stable
c) lock

d) free running

16) __________ filters have the property that they minimize the error between the
idealized and the actual filter characteristic over the range of the filter, but with
ripples in the passband.
a) Butterworth
b) Chebyshev
c) Cauer
d) Elliptical
1
, we are talking about
2RC
a) high pass filter b) low pass filter c) both a) and b)

17) If f =

d) none of a) and b)

18) Which of below application can be realized in open loop configuration of op amp ?
a) Schmit trigger
b) Peak detector
c) Zero crossing detector
d) Voltage follower
19) A negative half wave rectifier uses __________ in feedback loop.
a) resistor
b) capacitor
c) no component(short)
d) none of above
20) In a sample and hold circuit, op amp is used as
a) sample circuit
b) hold circuit
c) voltage follower
______________

d) peak detector
Set A

*SLRBB132*

-3-

SLR-BB 132

Seat
No.

S.E. (Electronics Engineering) (New) (Part II) Examination, 2014


LINEAR INTEGRATED CIRCUITS
Day and Date : Friday, 28-11-2014
Time : 3.00 p.m. to 6.00 p.m.

Marks : 80

Instructions : 1) All questions are compulsory.


2) Figures to the right indicate full marks.
3) Assume suitable data if necessary.
SECTION I
2. a) For non inverting amplifier derive expression for gain (exact and ideal). Also
derive expression for gain using rule-ideally differential voltage is zero.

b) Derive an expression for input offset voltage compensating network for Op


Amp without feedback. Also comment on component selection.

c) Derive an expression for closed loop voltage gain as a function of frequency. 8


OR
d) With suitable circuit diagram, explain instrumentation amplifier using Op Amp.
Explain its use for any physical parameter measurement.
3. Solve any four :

8
(44=16)

a) Explain voltage to current converter using Op Amps-with grounded load.


b) Why input bias current compensating network is not required for a differential
amplifier ?
c) Draw and explain (a general shape with salient features) frequency response
of 741.
d) With suitable circuit diagram, explain averaging amplifier.
e) Derive expression for output voltage of differential amplifier with one OPAMP.
Set A

SLR-BB 132

-4-

*SLRBB132*

SECTION II
4. a) Draw circuit for second order low pass Butterworth filter using OPAMP and find out
values of various components for a cut off frequency of 1 KHz.
Sketch (general) frequency response.

b) Draw and explain triangular and square wave generator. Discuss design
procedure and comment on component selection. What are its applications ?

c) Discuss features of PLL NE/SE 565. Discuss center frequency, lock range
and capture range. Comment on the component selection.

OR

d) Draw and explain dual slope ADC.


5. Solve any four :

8
(44=16)

a) Explain application of PLL NE/SE 565 as a FSK demodulator.


b) Explain R-2R network DAC.
c) Derive for condition required for oscillations.
d) Draw and explain all pass filter. Discuss design procedure and comment on
component selection. What are its applications ?
e) What is PLL ? Explain its basic working principle.
_____________________

Set A

SLR-BB 133

*SLRBB133*
Seat
No.

Set

S.E. (Electronics Engineering) (Part II) (New) Examination, 2014


SIGNALS AND SYSTEMS
Day and Date : Saturday, 29-11-2014
Time : 3.00 p.m. to 6.00 p.m.

Max. Marks : 100

Instructions : 1)
2)
3)
4)

Figures to the right indicate full marks.


Assume suitable data if necessary.
All questions are compulsory
Q. No. 1 is compulsory. It should be solved in first 30 minutes in
Answer book Page No. 3. Each question carries one mark.
5) Answer MCQ/Objective type questions on Page No. 3 only. Dont
forget to mention, Q.P. Set (A/B/C/D) on Top of Page.
MCQ/Objective Type Questions

Duration : 30 Minutes

Marks : 20

1. Select suitable option :


1) If the output of the signal is double to that of the input signal, then it is called _________
a) Time Shifting
b) Time Scaling
c) Time Reversal
d) Amplitude Scaling
dy(t )
+ 3 y(t ) = x(t ) is a
dt
a) Time invariant system
c) Partly (a) and partly (b)

2) The system

b) Time-variant system
d) None of these

3) The output of a system at any time depends only on the values of the input at the present
t time and in the past and not on future, such a system is known as ___________
a) Static
b) Dynamic
c) Anticipative
d) Non anticipative
4) The Fourier transform of a signal x(t) = e2t u( t) is
a)

1
2 jw

b)

2
1 jw

c)

1
j2 w

d)

2
j2 w

5) A discrete time LTI system is memoryless if


a) h[n] = 0 for n = 0
b) h[n] = 1 for n 0
c) h[n] = 0 for n 0
d) none of the above
6) In trigonometric Fourier series, an in terms of coefficients of exponential series is given
by
a) (Cn + Cn)
b) (Cn + jCn)
c) j(Cn C n)
d) j(Cn+Cn)
7) The unit step response is the output of an LTI system when input applied is
a) Unit ramp
b) Unit impulse
c) Unit step
d) Exponential signal
P.T.O.

SLR-BB 133

*SLRBB133*

-2-

8) The system y(t) = 3x(t) + 4 is


a) Linear system
c) Partly(a) and partly (b)

b) Non-linear system
d) None of these

9) A continuous-time periodic signal x(t), having a period T, is convolved with itself. The
resulting signal is
a) Not periodic
b) Periodic having a period T
c) Periodic having a period 2T
d) Periodic having a period T/2
10) The fundamental period T of the continuous time signal sin 50 t is
a)

1
sec
30

b)

1
sec
20

c)

1
sec
25

d)

1
sec
40

11) In order to get the original signal from sampled signal, it is necessary to use ______
a) Low pass filter
b) High pass filter c) Bandpass filter
d) Band stop filter
12) A signal of maximum frequency of 10 KHz is sampled at Nyquist rate. The time interval
between two successive samples is _________
a) 50 s
b) 100 s
c) 1000 s
d) 0.5 s
13) The range of Z for which x(Z) approaches to finite value is called ___________
a) Bilateral Z TF
b) Unilateral Z TF c) ROC
d) None of above
14) For independent events A and B, the conditional probability P(A/B) is _________
a) P(A)
b) P(B)
c) P(AB)
d) P(BA)
15) All odd moments of a gaussian distributed random variable with zero mean are ______
a) One

b) Zero

c) Infinity

d) Two

16) Variance of a random variable is _________ order of central moment.


a) First

b) Second

c) Third

d) Fourth

17) The cross correlation function is _________ function of time.


a) Odd

b) Even

c) Symmetry

d) None of above

18) The spectral density of a real valued random process has _________
a) No symmetry

b) Conjugate symmetry

c) Odd symmetry

d) Even symmetry

19) The Pdt is given by fx(x ) = Ke


a)

1
2

b)

x2

< x < the value of K should be _________


c)

1
2

d)

20) The Laplace transform of a unit same function starting at t = a is __________


a) 1(s + a )2

as
2
b) e (s + a )

as
c) e s 2

______________

d) a s2

Set A

*SLRBB133*

-3-

SLR-BB 133

Seat
No.

S.E. (Electronics Engineering)(Part II) (New) Examination, 2014


SIGNALS AND SYSTEMS
Day and Date : Saturday, 29-11-2014
Time : 3.00 p.m. to 6.00 p.m.

Marks : 80

Instructions : 1) Figures to the right indicate full marks.


2) Assume suitable data if necessary.
SECTION I
2. Attempt any four :
(46=24)
a) Draw and explain with necessary equation the relation between unit step and unit impulse.
b) Obtain the even and odd parts of a DT sequence given below.

c) Perform the transformation [x(t)+x(t)]u(t) for the following signal.

d) Explain the properties of convolution integral with example.


e) Find whether the following signals are periodic or not :
i) cos 60 t + sin 50 t
ii) 3cos 4t + 2 sin 2 t.
3. Attempt any two :

(28=16)

a) Obtain the convolution and sketch the result if


x(t ) = 1 , for 0 t 2
= 0 ,

otherwise

h(t) = 1 , for 0 t 3
= 0 ,

otherwise

Set A

SLR-BB 133

-4-

*SLRBB133*

b) Find the exponential fourier series and plot the magnitude and phase spectrum for the
periodic signal x(t).

c) Find the Fourier Transform of time domain non periodic signal x(t).

SECTION II
4. Attempt any four :
(45=20)
1) Explain how a signal reconstructed from its samples.
2) Define random variable and explain characteristic of random variable with properties.
3) An urn contains 5 white balls and 7 red balls, two balls are drawn at random from urn
without replacement.
a) What is probability that both balls are white ?
b) What is probability that second ball is Red ?
4) Find Z transform for the sinusoidal sequence x(n) = sin(n T) u(n)
5) Define spectral density of random process. It autocorrelation has given, explain method of
spectral density calculation.
5. Attempt any two :
(210=20)
1) Derive the expression for mean and variance for uniformly distributed random variable
as shown in fig.

2) Define and explain the properties


a) Auto correlation
b) Cross correlation
c) Power spectral density.
3) Find the output y(n) of a LTJ system specified by the equation.
y(n) 3/2y(n 1) + 1/2y(n 2) = 2x(n) + 3/2 x(n 1) when the initial conditions are
y(1) = 0, y(2) = 1 and x(n) = (1/4)nu(n)
_____________________

Set A

SLR-BB 134

*SLRBB134*
Seat
No.

Set

T.E. (Part I) (Electronics Engg.) (Old) Examination, 2014


CONTROL SYSTEM
Day and Date : Wednesday, 3-12-2014
Time : 3.00 p.m. to 6.00 p.m.

Max. Marks : 100

N.B. : 1)
2)
3)
4)

Figure to the right indicates full marks.


Assume suitable data, if necessary.
All questions are compulsory.
Q. No. 1 is compulsory. It should be solved in first 30 minutes in
Answer Book Page No. 3. Each question carries one mark.
5) Answer MCQ/Objective type questions on Page No. 3 only.
Dont forget to mention, Q.P. Set (A/B/C/D) on Top of Page.
MCQ/Objective Type Questions

Duration : 30 Minutes

Marks : 20

1. MCQ/Objective Type Questions :

(each one mark)

1) In force voltage analogy, Dashpot is analogous to ______________


a) Voltage
b) Current
c) Resistance
d) Inductance
2) Transfer function of the system is the ratio of output to input in ______________
a) z-transform
b) Laplace transform
c) Fourier transform d) none of these
3) In the Ist column of Routh array, Flowing elements are present 4, 3, 14/3, 5, This
system is ______________
a) Stable
b) Unstable with two roots lying on right of S-plane
c) Unstable with one root lying on right of S-plane
d) Stable because no imaginary roots
4) In closed loop control system ______________
a) Control action depends upon o/p
b) Control action does not depends upon o/p
c) No control action takes place
d) All above
5) When all the terms in the Ist column of routh array are positive then system is ____________
a) absolutely stable b) marginally stable c) conditionally stable d) unstable
6) The characteristics equation is 3s4 + 10s3 + 5s2 + 2 = 0, this system is __________
a) stable
b) unstable
c) marginally stable d) cant say
7) Unit impulse response of a system in Laplace transform gives
a) Transfer function b) System gain
c) Unit step function d) Unit ramp function
8)

can be replaced by ______________

a)

b)

c)

d)
P.T.O.

SLR-BB 134

*SLRBB134*

-2-

9) A stepper motor ______________


a) is a two phase induction motor
b) is a kind of rotating amplifier
c) is a electromagnetic transducer commonly used to convert an angular position of a
shaft into an electrical system
d) is an electro mechanical device which actuates a train of step angular movements in
response to a train of input pulses on one to one basis
10) An A.C. motor is used in servo applications is a ______________
a) single phase induction motor
b) two phase induction motor
c) three phase induction motor
d) synchronous motor
11) In root locus system, for transfer function G(s) =
are ______________
a) 1

b) 2

K
s( s +1) ( s + 2 ) ( s + 3 )

the numbers of branches

c) 3

d) 4

12) Time required for the response to reach 90% of steady state value for 1st time is called _________
a) Rise time
b) Settling time
c) Peak time
d) Delay time

13) Lag compensator reduces ______________


a) Band width
b) Rise time
c) Transient response
d) All of above
14) In bode plot, if gain margin is = zero and phase margin = zero then is the system is _______
a) Stable
b) Unstable
c) Marginally stable d) None of these
15) Peak time for 2nd order system is ______________
a)

b)

c)

d)

16) Using lag compensator the system becomes ______________


a) More sensitive
b) More stable
c) Faster

d) More bandwidth

17) The starting point of the root loci are the location of the pole when K =
a) 0
b) 1
c)
d)
18) The bandwidth in frequency domain analysis of 2nd order system is given by
1

2
2
4

a) b = n 1 2 + 2 4 + 4

c) b =

n2 1

2
b) b = n 1 2 + 2 4 2 + 4 4

2
2 + 2 4 + 4

d) b = n 2 1 2 2 + 2 4 + 4

1
2

19) In bode plot, if gc > pc the system is ______________

a) Stable

b) Unstable

c) Marginally stable d) None of these

20) When damping factor =1 the system is ______________


a) Over damped
b) Critically damped
c) Under damped
d) None of these
______________

Set A

*SLRBB134*

-3-

SLR-BB 134

Seat
No.

T.E. (Part I) (Electronics Engg.) (Old) Examination, 2014


CONTROL SYSTEM
Day and Date : Wednesday, 3-12-2014

Marks : 80

Time : 3.00 p.m. to 6.00 p.m.


N.B. : 1) Figure to the right indicates full marks.
2) Assume suitable data, if necessary.
3) All questions are compulsory.
SECTION I
2. Solve any four :

(64=24)

a) Explain position control system.


b) Derive the transfer function of R-L-C series circuit.
c) Find transfer function of following block diagram.

d) Write a short note on stepper motor.


e) Comment on stability using Rouths array for the given characteristics Eqn.
D(s) = s4 + 6s3 + 21s2 + 36s + 20 = 0.
3. a) Explain operation principle of A.C. servo motor. Derive the transfer function of
A.C. servo motor.
b) Solve any one :

8
8

i) Explain missile launching and guidance system in detail.


ii) Write a detail note on PID controller.
Set A

SLR-BB 134

*SLRBB134*

-4-

SECTION II
4. Solve any four :

(46=24)

a) Explain frequency domain specifications.


b) Explain lag-lead compensator.
c) Find open loop gain and time constants for following first system :
i) G(s) =

15
2 s +1

ii) G(s) =

15
6s+ 3

d) A unity feedback system with open transfer function as G(s) =

k
s ( s +10 )

. Determine the value

of K when = 0.5 .

e) Write a short note on PLC and its types.


5. Solve any two :

(28=16)

a) Explain rules for plotting root locus.


b) Sketch the bode plot and determine the gain cross over frequency and phase cross
over frequency for the system.

G(s) =

10
s(1+ 0.5 s ) (1+ 0,1s )

c) Determine the error constant for following transfer function and error to input r(t) =
G(s) =

5( s +4 )
s( s 3 + 6 s 2 + 8 s )

7 2
t
2

.
_________________

Set A

SLR-BB 135

*SLRBB135*
Seat
No.

Set

T.E. (Part I) (Electronics Engineering) (Old) Examination, 2014


DIGITAL SIGNAL PROCESSING
Day and Date : Friday, 5-12-2014
Time : 3.00 p.m. to 6.00 p.m.

Max. Marks : 100

Instructions : 1) Q. No. 1 is compulsory. It should be solved in first 30 minutes


in Answer Book Page No. 3. Each question carries one mark.
2) Answer MCQ/Objective type questions on Page No. 3 only.
Dont forget to mention, Q.P. Set (A/B/C/D) on Top of Page.
3) Assume suitable data if necessary
Duration : 30 Minutes

MCQ/Objective Type Questions

Marks : 20

1. Multiple choice questions. Choose the correct option :


(201=20)
1) DFT of a impulse function is
a) [n]
b) 1
c) [n 1]
d) none
2) Using long division, we divide numerator polynomial by a denominator
polynomial to obtain a power series in Z1 depending on whether the series to
be recovered is
a) anti-causal signal
b) causal signal
c) both sided signal
d) none
3) The system is linear if
a) It is a homogeneous
b) It is additive
c) It is a homogeneous or additive
d) It is a homogeneous and additive
4) Determine length of the convolution sum of sequences x(n) = {3, 2, 1, 2} and
h(n) = {1, 2, 1, 2} is
a) 7
b) 8
c) 4
d) 5
5) When length of DFT is 16 then real time multiplications and real time
additions for radix 2 algorithm are
a) 20, 148
b) 264, 1032
c) 88, 408
d) 24, 152
6) How many delay elements are available in direct form I realization of
Y(n) = 0.5y (n 1) 0.25y (n 2) + x(n) + 0.4 x(n 1)
a) 1
b) 2
c) 3
d) 4
7) The FFT algorithms
a) eliminate the redundant calculation and enable to analyze the spectral
properties of a signal
b) enable the redundant calculation and redundant to analyze the spectral
properties of a signal
c) a) and b)
d) none
P.T.O.

SLR-BB 135

*SLRBB135*

-2-

8) When a sequence is circularly shifted in time by 5 units, the magnitude response


a) increased by 5
b) remains unchanged
c) remains constant
d) shifts by 5 units
9) Which of the filter realization are not recommended for practical applications ?
a) direct form I and Direct form II
b) parallel form
c) cascaded form
d) lattice form
10) Z-Transform reduces to Fourier transform when it is evaluated on
a) A half circle
b) Z circle
c) Unit circle
d) Imaginary circle
11) The P-DSPs follow the __________ architecture.
a) Von Neumann
b) Harward
c) Modified Harward
d) VLIW
12) Which of the following features are required for efficient performance of DSP
operation ?
a) VLIW architecture
b) Multiple access memory
c) Multiplier and accumulator
d) All above
13) Butterworth filters have
a) Wide transition region
c) Oscillation in transition region

b) Sharp transition region


d) Maximally flat passband

14) The frequency mapping from s domain to z domain using Bilinear transformation is
a) one to one
b) Many to one c) One to many d) None of above
15) To convert the analog LPF with cutoff frequency c to LPF with cutoff
frequency c* we need to transform
c *
c
c c *
S c) S S c c * d) S
S
b) S
a) S
c
c *
S
16) The poles of the Butterworth LPF with cutoff frequency c
a) lie on the unit circle in s plane
b) lie on the RHS of s plane
c) lie on a circle of radius c
d) none of these
17) The approximate width of the main lobe in Bartlett window of length M is
a) 8 / M
b) 4 / M
c) 12 / M
d) 16 / M
18) Which of the following is true for FIR filters ?
a) They have linear phase
b) Are always stable
c) They are all zero filters
d) All above a), b) and c)
19) For getting the linear phase response in DSP we use
a) Butterworth filters b) IIR filters c) FIR filters
20) Robot vision is the ____________ application of DSP.
a) General
b) Image
c) Audio
______________

d) Convolution
d) Biomedical

Set A

*SLRBB135*

-3-

SLR-BB 135

Seat
No.

T.E. (Part I) (Electronics Engineering) (Old) Examination, 2014


DIGITAL SIGNAL PROCESSING
Day and Date : Friday, 5-12-2014
Time : 3.00 p.m. to 6.00 p.m.

Marks : 80

Instructions : 1) All questions are compulsory.


2) Assume suitable data if necessary
SECTION I
2. Solve any four of the following :

(45=20)

a) Obtain the cascade realization of the system characterized by transfer function


H(z) =

2(z + 2)
.
z(z 0.1) (z + 0.5) (z + 0.4)

b) Find ZT sketch ROC for x[n] = u[n].


c) State and explain properties of ROC of z transform.
d) Obtain the autocorrelation for DT sequence and also sketch the result :
X [ n] = { 0, 1, 2, 3, 4 }

e) Find the IDFT of X(k) = {10, 1 + 3j, 0, 1 3j}.


3. Solve any two :

(210=20)

a) Develop a DIF FFT algorithm. Hence determine X(k) of the sequence


x(n) = {1, 2, 3, 4, 4, 3, 2, 1}.
b) Obtain Direct form I, Direct form II, cascade and parallel realizations for the
system
y(n) = 0.1 y(n 1) + 0.2y(n 2) + 3x(n) + 3.6x(n 1) + 0.6 x(n 2).
c) Determine circular convolution of following using DFT and IDFT operations :
x1(n) = {3, 4, 2, 4} and x2(n) = {1, 2, 1, 2}.
Set A

SLR-BB 135

-4-

*SLRBB135*

SECTION II
4. Attempt any four :

(46=24)

a) Write the analog transfer function for Butterworth filter of order 2 and cutoff
frequency c = 1. Using frequency transformations convert above filter to
i) Low pass filter with cutoff frequency c = 0.786
ii) High pass filter with cutoff frequency c = 1.414 .
b) Describe the applications of DSP in image processing.
c) Draw and explain the architecture of TMS 320C54 DSP.
d) Give the transfer function of Butterworth LPF of order N. Explain in brief
how to compute order and cutoff frequency of the filter from the desired
specifications of filter.
e) Explain in detail the Multiply and Accumulate (MAC) unit of digital signal
processor.
f) Explain in brief windowing. Give mathematical expression for different windows.
Compare different window characteristics based on transition width and stop
band attenuation.
5. Attempt any two :

(28=16)

a) Explain the Bilinear transformation for digital filters in detail.


b) Design a digital Butterworth filter to meet the following constraints.
0.8 | H(e j ) | 1
| H(ej ) | 0.2

0 0.2
0.25

Using Bilinear transformation. Use T = 1 sec.


c) A low pass filter is to be designed with the desired frequency response as
e j2
Hd(e ) =
0
j

4 4
||
4

Determine the filter coefficients hd(n). Also determine H(e j ) using Hamming
window for M = 7.
_____________________

Set A

SLR-BB 136

*SLRBB136*
Seat
No.

Set

T.E. (Electronics) (Part I) (Old) Examination, 2014


MICROPROCESSORS AND INTERFACING
Day and Date : Monday, 8-12-2014
Time : 3.00 p.m. to 6.00 p.m.

Total Marks : 100

Instructions : 1) Figures to the right indicate full marks.


2) Assume suitable data wherever necessary.
3) Q. No. 1 is compulsory. It should be solved in first 30
minutes in Answer Book Page No. 3. Each question carries
one mark.
4) Answer MCQ/Objective type questions on Page No. 3
only. Dont forget to mention, Q.P. Set (A/B/C/D) on Top
of Page.
Duration : 30 Minutes

MCQ/Objective Type Questions

Marks : 20

1. Objective questions :
(201=20)
1) Shadow Address will exist in
a) Absolute decoding
b) Linear decoding
c) Partial decoding
d) None of the above
2) Single stepping is useful for
a) I/O interfacing
b) Debugging
c) Wait state
d) Slower memory interfacing
3) Which stack is used in 8085 ?
a) FIFO
b) LIFO
c) FILO
d) None of above
4) Addressing in which the instructions contains the data to be operated on is
known as
a) Immediate addressing
b) Implied addressing
c) Register addressing
d) Direct addressing
5) The number of machine cycles required to complete the instruction MOV D,
M are ________
a) 2
b) 3
c) 1
d) 4
6) In the following interrupts which is/are the vectored interrupt/s
a) TRAP
b) RST 7.5
c) RST 6.5
d) All of above
7) The memory map of a 4 KB memory begins at the location 4000 H. What is
the last location on the chip ?
a) 43FFH
b) 4FFFH
c) 47FFH
d) 7FFFH
P.T.O.

SLR-BB 136

-2-

*SLRBB136*

8) The synchronization between microprocessor and memory is done by


a) ALE signal
b) HOLD signal c) READY signal d) None of these
9) Any 8085 microprocessor based system should have at least one EPROM at the
a) Beginning of the memory address space
b) End of the memory address space
c) Middle of the memory address space
d) None of the above
10) 8085 has _________ software interrupts and __________ hardware interrupts.
a) 10, 5
b) 8, 4
c) 8, 5
d) 6, 6
11) IO mode, mode 0 uses ___________ for handshaking signals.
a) Port A
b) Port B
c) Port C
d) None of above
12) For bidirectional data transfer, 8255 can be used in
a) Mode 0
b) Mode 1
c) Mode 2
d) BSR mode
13) _________ can be used as an input port.
a) Buffer
b) Latch
c) Decoder
d) Encoder
14) For DAC 0808, the output _________ is proportional to input digital count.
a) Voltage
b) Current
c) Resistance
d) Voltage and Current
15) What is SIM ?
a) Set interrupt mask
b) Set interrupt memory
c) Set input memory
d) None of the above
16) Control register and Status register of 8251 are of ______ bits and are _______
bits respectively.
a) 8, 8
b) 16, 16
c) 16, 8
d) 8, 16
17) 8251 is a
a) UART
b) USART
c) Programmable Interrupt controller
d) Programmable interval timer/counter
18) The 8253 mode 3 is
a) Interrupt on terminal count
b) Programmable one shot
c) Rate generator
d) Square wave generator
19) For memory mapped I/O, address lines used are
a) 10 bit
b) 8 bit
c) 16 bit
d) 4 bit
20) RS 232 standard is defined for
a) Parallel communication
b) Serial communication
c) Both a) and b)
d) None of the above
______________
Set A

*SLRBB136*

-3-

SLR-BB 136

Seat
No.

T.E. (Electronics) (Part I) (Old) Examination, 2014


MICROPROCESSORS AND INTERFACING
Day and Date : Monday, 8-12-2014
Time : 3.00 p.m. to 6.00 p.m.

Marks : 80

Instructions : 1) Figures to the right indicate full marks.


2) Assume suitable data wherever necessary.
SECTION I
2. Answer any four :

(45=20)

a) Explain the addressing modes in 8085.


b) Explain the following instruction : ADD M, CMP, DCX H, LDA, and JNZ.
c) Write a program to count the number of 1s and 0s in a hex number.
d) What is stack ? Explain how stack can be used by programmer and processor.
e) Draw timing diagram of instruction stored at address 4567 : LXI H, 1234.
3. Answer any two :

(210=20)

a) Design a 8085 microprocessor based system with 8 KB EPROM having a


word length of 8-bits with the starting address of 0000H and two 2 KB RAMs
having word lengths of 4-bits each with starting address of 4000H.
b) What is interrupt ? Explain SIM and RIM instruction with their bit format.
c) Write a program
i) To convert 8 bit hex number to BCD.
ii) To convert 8 bit BCD number to hex.

Set A

SLR-BB 136

-4-

*SLRBB136*

SECTION II
4. Answer any four :

(45=20)

a) Interface two seven segment display to the 8085 through 8255 and write a
program to display 00 to 99.
b) Draw and explain functional block diagram of 8251.
c) Explain binary weighted register DAC.
d) Draw and explain interfacing of 8255 with 8085.
e) Explain Mode 2 and Mode 3 operations of 8253.
5. Answer any two :

(210=20)

a) Interface 3 8 keypad to 8085. Write a program to display the pressed key.


b) Interface ADC 0808 to 8085 and write a program for analog to digital
conversion.
c) Interface DAC to 8085 and write a program to generate a triangular wave.
_____________________

Set A

SLR-BB 137

*SLR-BB-137*
Seat
No.

Set

T.E. (Electronics Engineering) (Part I) (Old) Examination, 2014


ELECTROMAGNETIC ENGINEERING
Day and Date : Wednesday, 10-12-2014
Time : 3.00 p.m. to 6.00 p.m.

Max. Marks : 100

Instructions : 1)
2)
3)
4)

All questions are compulsory.


Figures to the right indicates full marks.
Assume suitable data if necessary.
Q. No. 1 is compulsory. It should be solved in first 30 minutes
in Answer book Page No. 3. Each question carries one mark.
5) Answer MCQ/Objective type questions on Page No. 3 only.
Dont forget to mention, Q.P. Set (A/B/C/D) on Top of Page.
MCQ/Objective Type Questions

Duration : 30 Minutes

Marks : 20

1. Select suitable option :


1) The operator del ( ) is a
a) Vector space function
c) Scalar space function
2) Unit of electric intensity is
a) joules/coulomb
c) volt/meter

20
b) Vector time function
d) Scalar time function
b) newton/coulomb
d) both b) and c)

3) The normal component of electric flux density across a dielectric-dielectric


boundary
a) are discontinuous
b) are continuous
c) depend on the magnitude of the surface charge density
d) depend on electric field intensity
4) Cylindrical coordinate r is related to the Cartesian coordinate as ___________
a) (x, y)
b) (x + y)
c) (x2/y2)
d) (x2 + y2)1/2
5) For transformation from the Cartesian coordinate system to Spherical
coordinate system, should be equal to ___________
a) cos
b) cos
c) sin
d) sin
6) Amperes Circuital Law is analogous to ___________ Law in electrostatics.
a) Lenzs
b) Gausss
c) Biot-Savarts d) Faradays
P.T.O.

SLR-BB 137

-2-

*SLR-BB-137*

7) Total flux passing through a closed surface held in a magnetic field is


___________
a) Infinity
b) Zero
c) Unity
d) None of these
8) The force between two point charges of 1 nC each with a 1 mm separation in air is
b) 9 106N
c) 9 109N
d) 9 1012N
a) 9 103N
9) Stokes Theorem relates, ___________ integral to a ___________ integral.
a) volume, surface b) volume, line c) line, surface
d) all of these
10) Maxwells equation in ___________ form give unformation at points of
discontinuity in electromagnetic fields.
a) Differential
b) Integral
c) Algebraic
d) None of these
11) An electric charge of Q coulombs is located at the origin. Consider electric
potential V and electric field intensity E at any point (x, y, z). Then
a) E and V are both scalars
b) E and V are both vectors
c) E is a scalar and V is a vector
d) E is a vector and V is a scalar
12) A dipole produces an electric field intensity of 1 m V/m at a distance of 2 km.
The field intensity at a distance of 4 km will be
a) 1 m V/m
b) 0.75 m V/m
c) 0.50 m V/m
d) 0.25 m V/m
13) Wave number has units of ___________
a) Radians
b) Meter
c) Radians/meter d) None of these
14) Phase velocity is given as ___________
b)
c) /
a)

d) None of these

15) For a good conductor ___________

=0
=
<< 1
>> 1
b)
c)
d)

A transmission line is specified in terms of ___________


a) R, G, L
b) G, L, C
c) R, G, L, C
d) None of these
A distortionless line has ___________
a) Non-vanishing attenuation constant
b) Constant velocity
c) Constant real characteristic impedance
d) All of these
Voltage reflection coefficient is the ratio of ___________ wave to ___________
wave.
a) Incident, reflected
b) Reflected, incident
c) Incident, absorbed
d) Absorbed, incident
Array factor depends upon ___________
a) Number of elements
b) Spacing among elements
c) Phase of the applied signal
d) All of these
If antenna directivity and antenna gain are equal, then antenna efficiency is
___________%
a) 20
b) 50
c) 75
d) 100
______________

a)

16)
17)
18)

19)
20)

Set A

*SLR-BB-137*

-3-

SLR-BB 137

Seat
No.

T.E. (Electronics Engineering) (Part I) (Old) Examination, 2014


ELECTROMAGNETIC ENGINEERING
Day and Date : Wednesday, 10-12-2014

Marks : 80

Time : 3.00 p.m. to 6.00 p.m.


Instructions : 1) All questions are compulsory.
2) Figures to the right indicates full marks.
3) Assume suitable data if necessary
SECTION I
2. Answer any four questions .

(44)

a) State and explain Culombs Law.


b) Uniform charge distribution 1 = 20 n c/m is distributed along x = 2m, y = 4m.
Find E at (2, 1, 4).

c) Given E = Em sin( wt z) a y in free space find B .
d) Transform a Cartesian vector A = 10ax 8ay + 6az to the spherical coordinates
at point P (10, 8, 6).
e) Prove that W = Q E dl .
3. Answer any three questions :

(83)

a) State and explain Divergence and strokes theorem.


b) Verify divergence theorem for

= 105 V A / m2, where

V = 10ex sin y ; 0 < x, y < 1.


- = 2.5 C
- 0, = 10 0. Determine whether or not following pair of equation
c) If = 0 , C
satisfy Maxwells equation. D = (Z + 6*107 t) a x, B = (754Z 4.52*1010t) a y.

d) Derive the expression for H due to finite length filament placed along z-axis
from z=z1 to z2.
Set A

SLR-BB 137

-4-

*SLR-BB-137*

SECTION II
4. Answer any four questions.

(44)

a) Explain the terms intrinsic impendence and velocity related to TEM wave.
b) Show that Z0 = ZscZoc .
c) A signal of 10 V is applied to a 50 coaxial transmission line terminated in
load 200 . Find v and reflected voltage.
d) Define Directive Gain and Directivity of antenna.
e) Explain Antenna Efficiency and give its significance.
5. Answer any three questions :

(83)

a) Derive the expression for radiation resistance of short dipole antenna.


b) A uniform transmission line has the constants R = 0.21 /m, L = 0.31 H/m,
C = 80pF/m and G = 10 /m. At 100 MHz determine :
i) Z0
ii) Propagation constant
iii) dB attenuation in 3m
iv) V/C ratio.
c) Derive the equation for reflection coefficient and transmission coefficient by
perfect dielectric medium for normal incidence.
d) Derive the wave equation for wave in dielectric medium.

_____________________

Set A

SLR-BB 138

*SLRBB138*
Seat
No.

Set

T.E. (Electronics) (Part I) Examination, 2014


INDUSTRIAL ECONOMICS AND MANAGEMENT (Old)
Day and Date : Friday, 12-12-2014
Time : 3.00 p.m. to 6.00 p.m.

Max. Marks : 100

Instructions : 1) Figures to the right indicate maximum marks.


2) Assume suitable data if necessary.
3) Q. No. 1 is compulsory. It should be solved in first 30 minutes
in Answer book Page No. 3. Each question carries one mark.
4) Answer MCQ/Objective type questions on Page No. 3 only.
Dont forget to mention, Q.P. Set (A/B/C/D) on Top of Page.
MCQ/Objective Type Questions
Duration : 30 Minutes

Marks : 20

1. Choose the correct answer :


(120=20)
1) Program benefits are created in ______________ phase of program creation.
a) Program Mandate b) Program brief c) Vision Statement d) Blue Print
2) Need to maximize utilization of resources ______________
a) Program manager
b) Project manager
c) Technical manager
d) Team leader
3) Merging of two organization is ______________
a) Infrastructure program
b) Business cycle program
c) Strategic program
d) Common program
4) The characteristics that distinguishing software project with other projects is
a) Controlling
b) Simplicity
c) Acceptability d) Conformity
5) Getting the project back to target is ______________
a) Dealing with project
b) Change control
c) Dealing with slippage
d) None of the above
6) Raw facts that describe a particular phenomenon is
a) information
b) data
c) technology
d) none of the above
7) Who knows how and when to apply technology ?
a) Technical person
b) Expert knowledge person
c) Technical literal knowledge person d) Technical Master person
8) The emphasis is now on the tasks at hand.
a) Forming
b) Norming
c) Storming

d) Performing
P.T.O.

SLR-BB 138
9) Step 3 of project planning shows
a) Identify project infrastructure
c) Analyse project characteristics

*SLRBB138*

-2-

b) Estimate effort
d) Allocate resource

10) A greater level of details is consider at each stage of the project in which model ?
a) Spiral model
b) U-process model
c) Waterfall model
d) None of the above
11) Product based approach used in
a) WBS
b) PRINCE2
c) ICT
d) None of the above
12) Where component tasks are identified and sized and these individual estimates
are aggregated ?
a) Bottom-up approach
b) Top-down approach
c) Parametric approach
d) Both a) and b)
13) Software effort estimation techniques which identifies the staff effort available
to do a project and uses that as the estimate.
a) Algorithmic models b) Parkinson c) Price to win
d) Top-down
14) PRINCE2 and stepwise will be used in
a) Activity based approach
b)
c) Product based approach
d)
15) Cumulative expenditure chart is used for
a) Cost monitoring
b)
c) Investment monitoring
d)

Hybrid approach
None of the above
Expenditure monitoring
All of the above

16) Power based on the personal attractiveness of the leader


a) Coercive power
b) Referent power
c) Legitimate power
d) Position power
17) The organization can run smoothly and gain competitive advantage in the
market place due to
a) Information
b) Resource
c) Environment d) Technology
18) All the requirements have been fulfilled is going to be check in
a) Qualification test b) Functional test c) Unit test
d) Acceptance test
19) A working model of one or more aspect of the project system is
a) SSADM
b) Prototype
c) Template
d) None of the above
20) The prototype is developed and modified until it is finally in a state where it
can become the operational system
a) Evolutionary prototype
b) Emerging prototype
c) Throw-away prototype
d) None of the above
______________

Set A

*SLRBB138*

-3-

SLR-BB 138

Seat
No.

T.E. (Electronics) (Part I) Examination, 2014


INDUSTRIAL ECONOMICS AND MANAGEMENT (Old)
Day and Date : Friday, 12-12-2014

Marks : 80

Time : 3.00 p.m. to 6.00 p.m.


Instructions : 1) Figures to the right indicate maximum marks.
2) Assume suitable data if necessary.
SECTION I
2. A) Attempt any three :

(63=18)

i) Describe distinguish characteristics of project.


ii) How cost-benefit analysis is being done in a project ? Explain.
iii) Explain Spiral-process model with neat diagram.
iv) Explain the difficulties that occur during effort estimation.
B) Attempt all :

(43=12)

i) Explain resource allocation within a program.


ii) Mention the problems to software projects concern with freshers.
iii) Explain how project have physical and technical dependency between
different activities with illustration.
3. Attempt any one :

(110=10)

i) Explain software prototyping and its types and also discuss the reasons
that lead to software prototyping.
ii) What are the different forms of program management explain each ?
Set A

SLR-BB 138

-4-

*SLRBB138*

SECTION II
4. A) Attempt any three :

(63=18)

i) Explain in brief people acts as a key resource in e-commerce.


ii) Draw and explain time line chart for project progress virtualization.
iii) How to enhance decision making ? Explain.
iv) What is risk reporting ? Explain.
B) Attempt all :

(43=12)

i) Describe the responsibilities for health and safety policy.


ii) Write a note on cost monitoring.
iii) Explain forward pass method.
5. Attempt any one :

(110=10)

i) A project specification table with estimated activity durations and


precedence requirement is shown below. Apply forward and backward pass
method to find critical path for this data :
Activity Duration Precedent
A

11

E, F

C, D

ii) What are the two types of data collection methods ? Explain in detail.
_____________________

Set A

SLR-BB 139

*SLRBB139*
Seat
No.

Set

T.E. (Electronics) (Part I) (New) Examination, 2014


CONTROL SYSTEMS
Day and Date : Wednesday, 3-12-2014
Time : 3.00 p.m. to 6.00 p.m.

Max. Marks : 100

N.B. : 1) Figures to the right indicate full marks.


2) Assume data, if necessary.
3) Q. No. 1 is compulsory. It should be solved in first 30 minutes in Answer
Book Page No. 3. Each question carries one mark.
4) Answer MCQ/Objective type questions on Page No. 3 only. Dont forget to
mention, Q.P. Set (A/B/C/D) on Top of Page.
MCQ/Objective Type Questions
Duration : 30 Minutes

Marks : 20

1. Choose the correct answer :

20

1) Transfer function of a system is the ratio of output to the input in ____________


a) Z-transform

b) Laplace transform c) Fourier transform

d) None of these

2) In closed loop control system ____________


a) Control action depends upon output
b) Control action does not depends upon output
c) No control action takes place
d) All above
3) Unit impulse response of a system in Laplace transform gives ____________
a) Transfer function

b) System gain

c) Unit step function

d) Unit ramp function

can be replaced by ____________

4)

a)

b)

c)

d)

5) ____________ is a electromechanical device which actuates a train of step angular movements


in response to a train of input pulses on one to one basis.
a) D.C. motor

b) A.C. 1 motor

c) A.C. 3 motor

d) Stepper motor

6) An A.C. motor used in servo applications is a ____________


a) 1 induction motor
b) 2 induction motor
c) 3 induction motor

d) Synchronous motor
P.T.O.

SLR-BB 139

*SLRBB139*

-2-

7) Time required for the response to reach 50% of steady state value for
Ist time is called ____________
a) Rise time
b) Settling time
c) Peak time

d) Delay time

8) Lead compensation is used to improve ____________


a) Transient response b) Gain
c) Phase margin

d) All above

9) f(t ) = A (t ) in this equation A stands for ____________


a) Function in time domain
b) Function strength
c) Deviation in time domain
d) All of above
10) The equation for ramp signal is given by ____________
a) f(t ) = A (t )

b) f( t ) = A u (t )

c) f(t ) = A t

d) f(t ) =

A 2
t
2

11) In bode plot, if gain margin is = zero and phase margin = zero then is the system is ____________
a) Stable
b) Unstable
c) Marginally stable
d) None of these
12) According to Hurwitz criterion the value of determinant D1 for following characteristic equation
2S3 + 12S2 + 21S + 8 = 0
a) 2
b) 12
c) 21
d) 8
13) Lag compensator reduces ____________
a) Band width
b) Rise time

c) Transient response d) All of above

14) In root locus system, for transfer function G(s) =


branches are ____________
a) 0
c) Average value of pole

K
the numbers of
s (s + 2.5) (s + 5) (s + 3.8)

b) k
d) 4

15) If nature of closed loop poles real and negative then the system is
a) Marginally stable
b) Unstable
c) Absolutely stable

d) None of these

16) Using lead compensator the system becomes ____________


a) More sensitive
b) Improve in steady state performance
c) Slower
d) More bandwidth
17) The starting point of the root loci are the location of the pole when K =
a) 0
b) 1
c)
18) If G(s) =

d)

s
then angle G(j ) is
(s + 3)

a) 90 tan1( / 3)

b) 90 tan1(3 / )

c) 180 tan1( / 3)

19) In bode plot, if gc > pc the system is ____________


a) Stable
b) Unstable
c) Marginally stable
20) The band width in frequency domain analysis of

c) b = n

d) None of these

order system is given by


1

2
2
4

a) b = n 1 2 + 2 4 + 4

2nd

d) 180 tan1(3 / )

2
b) b = n 1 2 + 2 4 2 + 4 4

1
1 2 2 + 2 4 2 + 4 4 2
d) b = n 2 1 2 2 + 2 4 + 4

______________

1
2

Set A

*SLRBB139*

-3-

SLR-BB 139

Seat
No.

T.E. (Electronics) (Part I) (New) Examination, 2014


CONTROL SYSTEMS
Day and Date : Wednesday, 3-12-2014
Time : 3.00 p.m. to 6.00 p.m.

Marks : 80

N.B. : 1) All questions are compulsory.


2) Figures to the right indicate full marks.
3) Assume data, if necessary.
SECTION I
2. Solve any four :
(64=24)
1) Explain advantages of feedback control system and explain pole zero and time constant form and
types of system.
2) Derive the transfer function of R-L-C series circuit.
3) Find transfer function for given block diagram :

4) Write a short note on stepper motor.


5) Determine position, velocity and acceleration constant of unity feed back control system with open loop
transfer function G(s) =

50
.
(1 + 0.15 s) (1 + 2 s)

3. a) Obtain over all transfer function

C
for given signal flow graph :
R

b) Solve any one :


i) Explain use of potentiometer as an error detector.
ii) Derive the relation for peak time of IInd order system.

8
Set A

SLR-BB 139

*SLRBB139*

-4-

SECTION II
4. Solve any four :

(46=24)

a) Explain frequency domain specifications.


b) Explain method to obtain gain margin and phase margin from polar plot.
c) Draw pole zero plot showing and determine centroid, asymptote lines for following system
G(s) H(s) =

K
2

s(s + 2 s + 32)

d) Determine stability of the system represented by following characteristic equation by using Rouths
criterion and show how many roots are in right half of s-plane S3 + 6S2 + 11S + 6 = 0.
e) Find forced sinusoidal response for following system G(s) =

10
2

s + 3 s + 10

with input signal

r(t) = 8 cos(2t + 30).


5. Solve any two :

(28=16)

a) What is compensator ? What is the need of compensator ? Explain different types in short.
b) Sketch the magnitude plot and determine the gain cross over frequency for the system
G(s) =

1
.
s (1 + 0.02 s) (1 + 0.04 s)

c) Sketch the root locus for the system G(s) H(s) =

K
.
s (s + 1) (s + 3)

_____________________

Set A

SLR-BB 14

*SLRBB14*
Seat
No.

Set

F.E. (Part II) Examination, 2014


BASIC CIVIL ENGINEERING
Day and Date : Wednesday, 26-11-2014
Time : 10.00 a.m. to 1.00 p.m.
Instructions :

Max. Marks : 100

1) Q. 2 and Q. 9 are compulsory; Attempt any two out of Q. 3 to Q. 5


and any two out of Q. 6 to Q. 8.
2) Assume suitable data if necessary but mention it clearly.
3) Q. No. 1 is compulsory. It should be solved in first 30 minutes in
Answer book Page No. 3. Each question carries one mark.
4) Answer MCQ/Objective type questions on Page No. 3 only. Dont
forget to mention, Q.P. Set (A/B/C/D) on Top of Page.
MCQ/Objective Type Questions

Duration : 30 Minutes

Marks : 20

1. Choose the correct answer :

(201=20)

1) Following is not the branch of Civil Engineering


a) Environmental
b) Irrigation
c) Transportation
d) Architecture
2) Height of Instrument Method provides check on
a) Back sight readings only
b) Intermediate sight readings only
c) Fore sight readings only
d) Both backsight and foresight readings
3) The length of a link in metric chain is
a) 10 c.m.
b) 20 c.m.

c) 30 c.m.

d) 100 c.m.

4) Which of the following lines are free from local attraction, if observed F. B and B. B are
a) 10, 100
b) 10, 190
c) 100, 190
d) 10, 170
5) What will be theoretical sum of included angles in a six sided closed traverse
a) 360
b) 540
c) 720
d) 1440
6) The longest chain line passing through the centre of the area is known as
a) Base line
b) Tie line
c) Check line
d) None of these
7) Method of irrigation in which land surrounded by natural or artificial banks is flooded is
known as
a) Broad irrigation
b) Natural irrigation
c) Basin irrigation
d) None of these
8) Immergency lane will be
a) Right of way
b) Shoulder

c) Kerb

d) Carriageway

P.T.O.

SLR-BB 14

*SLRBB14*

-2-

9) Camber is provided in
a) Longitudinal direction
b) Transverse direction
c) Both longitudinal and transverse direction
d) None of these
10) Contour Interval is
a) Vertical distance between two consecutive contours
b) Horizontal distance between two consecutive contours
c) Inclined distance between two consecutive contours
d) Vertical distance between any two contours
11) Superstructure consists of
a) Foundation and Plinth
c) Plinth only

b) Foundation only
d) Structure above plinth

12) Arrangement of door or windows on external wall, to reveal desirable and conceal
undesirable views is known as
a) Aspect
b) Prospect
c) Roominess
d) Grouping
13) The Chhajja is provided for
a) Protection from rain
c) Protection from sunrays

b) Protection from Dust


d) Protection to terrace users

14) Final setting time for cement should be


a) Greater than 10 Hours
c) Greater than 30 minutes

b) Less than 30 minutes


d) Lesser than 10 hours

15) Pile foundation is suitable for


a) Light to moderate loads
b) Heavy loads and weak strata at shallow depth
c) Strong strata at shallow depth
d) None of these
16) Height of building depends on
a) Front road width
c) Total built up area

b) FSI allowed
d) Plot area

17) If Floor Space Index is 1, then


a) Total plot area is 1
b) Total built up area is 1
c) Ratio of Total Plot area / Total built up area is 1
d) Ratio of Total built up area / Total plot area is 1
18) Active remote sensing system consists of energy from
a) Own source
b) Sunrays
c) Both own source and Sunrays
d) Either own source or Sunrays
19) Spout is used for
a) Removal of sanitary water
c) Removal of rainwater
20) GIS consist of
a) User
c) Software and hardware

b) Removal of dust
d) Filling water
b) Data
d) All of these
______________
Set A

*SLRBB14*

-3-

SLR-BB 14

Seat
No.
F.E. (Part II) Examination, 2014
BASIC CIVIL ENGINEERING
Day and Date : Wednesday, 26-11-2014
Time : 10.00 a.m. to 1.00 p.m.

Marks : 80

Instructions : 1) Q. 2 and Q. 9 are compulsory; Attempt any two out of Q. 3 to Q. 5


and any two out of Q. 6 to Q. 8.
2) Assume suitable data if necessary but mention it clearly.
SECTION I
2. Solve the following problems :
a) During leveling following staff readings are taken.
1.2, 2.3 (instrument shifted), 3.4, 1.9 (staff inverted), 2.7, 3.4
Enter the data in standard tabular form. Calculate R.L. of the last station if the R.L. of first
station is 100.000m. Use Rise and Fall method. Also apply usual arithmetic checks.

b) Following bearings were observed during a closed compass traverse run in clockwise
direction.

Observed
Station

F.B

B.B

10 20

130 0

100 40

190 0

240 0

280 40

310 20

60 20

Find :
i) Included angles
ii) Angular error
iii) Corrections at each station.

3. a) Explain briefly relevance of Civil engineering to Electrical Branch.

b) A 20m chain was checked before start of work and found to be 10 c.m. too short. After
measurement of 1000m it was tested again and found to be 20 c.m. too short. What will be
the correct distance of measurement.

c) Draw the sketch of Metric Chain and name its parts.

4
Set A

SLR-BB 14

-4-

*SLRBB14*

4. Write short notes on :


a) Errors in Chaining

b) Explain Types of offset taking instruments.

5. Write notes on :

12

a) Rain Water Harvesting


b) Difference between Rigid and Flexible pavement
c) Comparison of rail transport and highway transport.
SECTION II
6. a) Explain the functions of various components of building.
b) Explain Electromagnetic energy spectrum with sketch.
7. a) List ideal properties of concrete.
b) Differentiate between Brick Masonary and Stone Masonary.

8
4
8
4

8. Write short notes on :


a) Global Positioning System.

b) Ideal Engineering Properties of Cement.

9. Write notes on :
a) Types of Foundations

b) Building byelaws.

_____________________

Set A

SLR-BB 140

*SLRBB140*
Seat
No.

Set

T.E. (Electronics Engineering) (Part I) Examination, 2014


DIGITAL SIGNAL PROCESSING (New)
Day and Date : Friday, 5-12-2014
Time : 3.00 p.m. to 6.00 p.m.

Max. Marks : 100

Instructions : 1) Assume suitable data if necessary.


2) Q. No. 1 is compulsory. It should be solved in first 30 minutes
in Answer Book Page No. 3. Each question carries one mark.
3) Answer MCQ/Objective type questions on Page No. 3 only.
Dont forget to mention, Q.P. Set (A/B/C/D) on Top of Page.
MCQ/Objective Type Questions
Duration : 30 Minutes

Marks : 20

1. Choose the correct option :


1) Consider H(z) =

(201=20)

k
M
k = 0 bkz

. The number of memory locations for direct


1 + Nk = 1 akz k
form l realization of above is
a) M + N + 1
b) M + 1
c) M + N 1
d) M N
2) Which of the following is known as Moving Average system ?
a) FIR system
b) IIR system
c) LTI system
d) None of these
3) N point sequence is circularly even if
a) X(n) = x(n)
b) X(n) = x(n)
c) X(N n) = x(n)
d) X(N n) = x(n)
4) If the DFT of a real sequence x(n) is X(k) = {1, 2 + j, A, 5, 3 + j2, 2 j}, the
value of A is
a) 1 + j2
b) 3 + j2
c) 3 j2
d) 3 j2
5) Which of the following sequence is circularly odd ?
a) x(n) = {1, 2, 4, 4, 2}
b) x(n) = {0, 2, 4, 4, 2}
c) x(n) = {0, 2, 0, 4, 2}
d) x(n) = {1, 2, 4, 4, 2}
6) Number of complex additions required for computing M point DFT are
a) N2
b) M (M 1) c) 2 M 1
d) None of these
7) The twiddle factor W411 is
a) J
b) 1
8) The FFT algorithm calculates
a) DTFT
b) DCT

c) 1

d) j

c) IDFT

d) DST
P.T.O.

SLR-BB 140

-2-

*SLRBB140*

9) The range of values of a for which the system with impulse response
h(n) = an u(n) is stable is
a) |a| > 1
b) |a| < 1
c) a > 0
d) a < 0
10) Length of the convolution sum of sequences x(n) = {3, 2, 1, 2} and
h(n) = {1, 2, 1, 2} is
a) 7
b) 8
c) 4
d) 5
11) The P-DSPs follow the _________ architecture.
a) Von Neumann
b) Harward
c) Modified Harward
d) VLIW
12) TMS320C54 processor uses ___________ architecture.
a) Simple
b) Complex
c) Harward
d) Von Meumann
13) In impulse invariant transformation, relation between and is
a) = T
b) = /T
c) = tan( T)
d) = (T/2) tan( T/2)
14) Which of the following is not true for IIR filters ?
a) Impulse response duration is infinite
b) They have poles
c) They give linear phase response
d) All above
15) In impulse invariant technique the pole at s = a gets mapped in Z domain to
a) Z = aT
b) Z = epiT
c) Z = eaT
d) Z = e aT
16) The poles of the Butterworth LPF with cutoff frequency c
a) Lie on the unit circle in s plane
b) Lie on the RHS of s plane
d) None of these
c) Lie on a circle of radius c
17) Which of the following is true the FIR filters ?
a) They have linear phase
b) Are always stable
c) They are all zero filters
d) All above a, b and c
18) Which method of FIR filter design can lead to Linear phase characteristic
a) Windowing
b) Frequency sampling
c) Both a) and b)
d) None of these
19) Which of the following is known as a moving average filter ?
a) IIR filter
b) FIR filter
c) Butterworth filter
d) None of these
20) The hardware used in digital signal processor to scale down or scale up
operands is
a) Barrel shifter
b) MAC
c) Multiplier
d) ALU
______________
Set A

*SLRBB140*

-3-

SLR-BB 140

Seat
No.

T.E. (Electronics Engineering) (Part I) Examination, 2014


DIGITAL SIGNAL PROCESSING (New)
Day and Date : Friday, 5-12-2014
Time : 3.00 p.m. to 6.00 p.m.

Marks : 80

Instructions : 1) All questions are compulsory.


2) Assume suitable data if necessary.
SECTION I
2. Solve any four of the following :

(45=20)

a) Obtain the system function for the LTI system described by constant coefficient
difference equation. Also obtain the transfer function for all zero system and
all pole system. Why an all zero system is called as FIR system and all pole
system is called as IIR system ? Explain.
b) Obtain circular N point circular convolution of following sequences using
Time domain equation for (i) N = 4 and (ii) N = 6.
X1(n) = {2 4 5 3}
X2(n) = {3 5 1}
Comment on the result obtained.
c) What is transposed structure ? How to obtain transposed structure for a
given direct form structure ? Explain with example. Also comment on the
memory requirements and computations.
d) If X(k) = {22 2 + 2j 2 2 2j}. Calculate IDFT using DIT IDFT algorithm.
e) Explain the symmetry properties of DFT. If 8 point DFT of a real sequence
x(n) is X(k) = {36, 4 + 9.66j, 4 + 4j, 4 + 1.66j, 4, P, Q, R}. Obtain the
values of P, Q and R.
3. Solve any two of the following :

(210=20)

a) Explain how N point DFT can be calculated using N/2 point DFTs using
Decimation in Frequency with necessary equations. Compute DFT of
x(n) = {1, 4, 5, 4} using DIF FFT algorithm.
b) Obtain Direct form I, Direct form II, cascade and parallel realizations for the
system y(n) = y(n 1) 0.5y(n 2) + x(n) x(n 1) + x(n 2).
c) Determine circular convolution of following using DFT and IDFT operations.
x1(n) = {3, 4, 2, 4} and x2(n) = x1((1 n))4.
Set A

SLR-BB 140

-4-

*SLRBB140*

SECTION II
4. Attempt any four :

(46=24)

a) Convert the analog filter to digital filter whose system function is


H(s) =

s + 0.2
. The digital filter should have a resonant frequency of
(s + 0.2)2 + 16

wr = 4 . Use Bilinear transformation.

b) Describe the Biomedical applications of DSP.


c) Explain the FIR and IIR filters with respect to characteristics, advantages
and disadvantages.
d) Explain in brief windowing. Give mathematical expression for different
windows. Compare different window characteristics based on transition
width and stop band attenuation.
e) Draw and explain the structure for 4 4 Baron multiplier for unsigned numbers.
f) Explain in detail the following blocks of digital signal processor
i) Multiply and Accumulate (MAC)
ii) Barrel Shifter.
5. Attempt any two :

(28=16)

a) What are the requirements for converting a stable analog filter to a stable
digital filter ? Explain the impulse invariant technique in detail.
b) Explain the effects of finite word length on the design of IIR filters.
c) A low pass filter is to be designed with the desired frequency response as
e j3 4 4
Hd(e ) =
| |
0
4
Determine the filter coefficients hd(n).
j

Also determine H(e j ) using Hamming window for M = 7.


_____________________

Set A

SLR-BB 141

*SLRBB141*
Seat
No.

Set

T.E. (Electronics) (Part I) (New) Examination, 2014


MICROPROCESSORS AND INTERFACING
Day and Date : Monday, 8-12-2014
Time : 3.00 p.m. to 6.00 p.m.

Total Marks : 100

Instructions : 1) Figures to the right indicate full marks.


2) Assume suitable data wherever necessary.
3) Q. No. 1 is compulsory. It should be solved in first 30 minutes
in Answer Book Page No. 3. Each question carries one mark.
4) Answer MCQ/Objective type questions on Page No. 3 only.
Dont forget to mention, Q.P. Set (A/B/C/D) on Top of Page.
MCQ/Objective Type Questions
Duration : 30 Minutes

Marks : 20

1. Objective questions :
(201=20)
1) Single stepping is useful for
a) I/O interfacing
b) Debugging
c) Wait state
d) Slower memory interfacing
2) Which stack is used in 8085 ?
a) FIFO
b) LIFO
c) FILO
d) None of above
3) Addressing in which the instructions contains the address of the data to be
operated on is known as
a) immediate addressing
b) implied addressing
c) register addressing
d) direct addressing
4) What is the addressing mode used in instruction MOV M, C ?
a) Direct
b) Indirect
c) Indexed
d) Immediate
5) In the following interrupts which is the non-vectored interrupt
a) TRAP
b) INTR
c) RST 7.5
d) RST 6.5
6) Shadow Address will exist in
a) Absolute decoding
b) Linear decoding
c) Partial decoding
d) None of the above
7) The memory map of a 4KB memory begins at the location 4000 H. What is
the last location on the chip ?
a) 43FFH
b) 4FFFH
c) 47FFH
d) 7FFFH
8) The synchronization between microprocessor and memory is done by
a) ALE signal
b) HOLD signal
c) READY signal
d) None of these
P.T.O.

SLR-BB 141

*SLRBB141*

-2-

9) When READY pin of 8085 microprocessor is low ?


a) the processor will be ready to execute program
b) the processor will enter into wait state for one clock period
c) the processor will enter into wait state until the READY pin is made high
d) the processor will return back from its READY state
10) 8085 has _________ software interrupts and ________ hardware interrupts.
a) 10, 5
b) 8, 4
c) 8, 5
d) 6, 6
11) IO mode, mode 1 uses ____________ for handshaking signals.
a) Port A
b) Port B
c) Port C
d) Port D
12) For bidirectional data transfer, 8255 can be used in
a) Mode 0
b) Mode 1
c) Mode 2

d) BSR mode

13) ____________ can be used as an output port.


a) Buffer
b) Latch
c) Decoder

d) Encoder

14) For DAC 0808, the output ____________ is proportional to input digital count.
a) Voltage
b) Current
c) Resistance
d) Voltage and current
15) What is RIM ?
a) Read Interrupt Mask
c) Read Input Memory

b) Read Interrupt Memory


d) None of the above

16) Control register and status register of 8251 are having the ____________
address and are ____________ respectively.
a) Different, read and write
b) Different, write and read
c) Same, write and read
d) Same, read and write
17) 8251 is a
a) UART
b) USART
c) Programmable interrupt controller d) Programmable interval timer/counter
18) The 8253 mode 1 is
a) Interrupt on terminal count
c) Rate generator

b) Programmable one shot


d) Square wave generator

19) For memory mapped I/O, address lines used are


a) 10 bit
b) 8 bit
c) 16 bit

d) 4 bit

20) RS 232 standard is defined for


a) Parallel communication
b) Serial communication
c) Both a) and b)
d) None of the above
______________

Set A

*SLRBB141*

-3-

SLR-BB 141

Seat
No.

T.E. (Electronics) (Part I) (New) Examination, 2014


MICROPROCESSORS AND INTERFACING
Day and Date : Monday, 8-12-2014
Time : 3.00 p.m. to 6.00 p.m.

Marks : 80

Instructions : 1) Figures to the right indicate full marks.


2) Assume suitable data wherever necessary.
SECTION I
2. Answer any four :

(45=20)

a) Explain the function of READY, HOLD, HLDA, ALE and X1 and X2 pins of
8085.
b) Explain the following instruction :
DAA, LDA, SHLD, JNZ, CMP, XCHG.
c) Write a program to count the number of 1s and 0s in a hex number.
d) What is stack ? Explain how stack can be used by programmer and processor.
e) Draw timing diagram of instruction stored at address 4567:MVI A, 05.
3. Answer any two :

(210=20)

a) Design a 8085 microprocessor based system 8 KB EPROM having a word


length of 8-bits with the starting address of 0000H and two 2KB RAMs having
word lengths of 4-bits each with the starting address of 4000H.
b) What is interrupt ? Explain interrupt structure in 8085.
c) Write a program :
i) to convert 8 bit hex number to BCD.
ii) to convert 8 bit BCD number to hex.
Set A

SLR-BB 141

-4-

*SLRBB141*

SECTION II
4. Answer any four :

(45=20)

a) Interface one switch and one LED to the 8085 through 8255 and write a
program to turn on LED when switch is closed.
b) Draw and explain functional block diagram of 8254 PIT.
c) Explain binary weighted register DAC.
d) Explain SIM and RIM instructions.
e) Explain Mode 1 operation of 8255.
5. Answer any two :

(210=20)

a) Interface stepper motor to 8085. Write a program to run stepper motor. How
will you change the direction of motor ?
b) Interface ADC 0808 to 8085 and write a program for analog to digital conversion.
c) Interface two digit seven segment display to 8085 and write a program to
display BCD numbers from 00 to 99.
_____________________

Set A

SLR-BB 142

*SLRBB142*
Seat
No.

Set

T.E. (Electronics Engineering) (Part I) (New) Examination, 2014


ELECTROMAGNETIC ENGINEERING
Day and Date : Wednesday, 10-12-2014
Time : 3.00 p.m. to 6.00 p.m.

Max. Marks : 100

Instructions : 1) Figures to the right indicates full marks.


2) Assume suitable data if necessary.
3) Q. No. 1 is compulsory. It should be solved in first 30 minutes in
Answer Book Page No. 3. Each question carries one mark.
4) Answer MCQ/Objective type questions on Page No. 3 only.
Dont forget to mention, Q.P. Set (A/B/C/D) on Top of Page.
MCQ/Objective Type Questions
Duration : 30 Minutes

Marks : 20

1. Select suitable option :

20

1) A scalar quantity has ______________


a) Magnitude
b) Direction
c) Both a) and b)

d) None of these

2) Which of the following is not a scalar quantity ?


a) Entropy
b) Displacement c) Volume

d) Mass

3) A unit vector has ___________ direction to that of the main vector.


a) Same
b) Opposite
c) Normal upwards
d) Normal downwards

 



4) P + (Q + R) = (P + (Q) + R) demonstrates _____________
a) Commutative Law
b) Associative Law
c) Distributive Law
d) Law of Parallelogram
5) ____________ product is governed by the Right-Hand Screw Rule.
a) Vector
b) Scalar
c) Simple mathematical
d) None of these
6) Which of the following is/are incorrect ?
b) ay az = ax
c) az ax = ay
a) ax ay = az

d) none of these

7) The cross product of the same vector to itself is ____________


a) 0
b) 1
c)
d) 100
8) In Cylindrical coordinate systems, unit vector is _____________
a) Directed outwards radially
b) Normal to cylindrical surface at a point
c) Both a) and b)
d) None of these
P.T.O.

SLR-BB 142

*SLRBB142*

-2-

9) Line integral involves ______________ product.


a) Sscalar
b) Vector
c) Both a) and b)
10) Del operator is _______________
a) Same as the gradient operator
c) Both a) and b)

d) None of these

b) Vector differential operator


b) None of these

11) Electric dipole is a pair of ____________ and ____________ point charges.


a) Equal and same
b) Unequal and same
c) Equal and opposite
d) Unequal and opposite
12) Steady magnetic fields are governed by _____________ law.
a) Biot-Savarts
b) Amperes Circuital
c) Both a) and b)
d) None of these
13) There will be force of attraction between two current-carrying conductors if the
currents are in ____________ direction.
a) Same
b) Opposite
c) None of these d) Cannot say
14) Maxwells equations shelter on ____________ law(s).
a) Faradays
b) Gausss
c) Amperes

d) All of these

15) Wave speed in terms of frequency f and wavelength is expressed as


___________
a) f /
b) / f
c) f
d) ( + f )
16) For a lossy dielectric medium, ____________
a) = 0
b) 0
c) none of these

d) cannot say

17) A transmission line is specified in terms of ____________


a) R, G, L
b) G, L, C
c) R, G, L, C

d) none of these

18) Voltage reflection coefficient is the ratio of ____________ wave to __________


wave.
a) Incident, reflected
b) Reflected, incident
c) Incident, absorbed
d) Absorbed, incident
19) Antenna tuning is done by changing its _____________
a) Inductive reactances
b) Capacitive reactances
c) Both a) and b)
d) None of these
20) Resonant frequency of an antenna depends upon its __________
a) Physical length
b) Electrical length
c) None of these
d) Cannot say
______________

Set A

*SLRBB142*

-3-

SLR-BB 142

Seat
No.

T.E. (Electronics Engineering) (Part I) (New) Examination, 2014


ELECTROMAGNETIC ENGINEERING
Day and Date : Wednesday, 10-12-2014
Time : 3.00 p.m. to 6.00 p.m.

Marks : 80

Instructions : 1) All questions are compulsory.


2) Figures to the right indicates full marks.
3) Assume suitable data if necessary.
SECTION I
2. Answer any four questions :
(44)
a) Transform A = 3ax + 4ay + 5az in spherical coordinates.
b) A 2mC charge is located in vacuum at P1( 3, 7, 4) and 5m C charge at P2 (1, 4, 2),
find the vector force on 5mC charge.
c) Find the circumference and area of circular disc with radius r = 2m.
d) State and prove Gauss law.
e) Explain Boundary Conditions for Magnetic field.
3. Answer any three questions :

(83)

a) Derive the expression for magnetic field intensity at the centre of center carrying
loop.
b) A point charge of 6 C is located at origin a uniform line charge density of 180 nC/m
lies along x-axis and a uniform sheet charge equal to 25nC/m 2 lies in the
Z = 0 plane
Find :
1) D at A(0,0,4)
2) D at B(1, 2, 4)
3) Calculate total electric flux leaving the surface of a sphere at 4m radius centered
at origin.
c) Verify stoke theorem for the field H = 6xyax 3 y 2 ay C/m2 and the rectangular path
around the region, for 2 x 5,1 y 1, z = 0 .
d) Derive equation for electric potential due to electric dipole.

Set A

SLR-BB 142

-4-

*SLRBB142*

SECTION II
4. Answer any four questions :

(44)

a) Derive the wave equation for electric field and magnetic field in Conducting Medium.
b) Explain wave polarization.
c) Show that : Z0 = ZscZoc
d) Explain Antenna Efficiency and give its significance.
e) Define Directive Gain and Directivity of antenna.
5. Answer any three questions :

(83)

a) At a frequency of 100 MHz, the following values are appropriate for certain
transmission line L = 0.25 H / m, C = 80 pF/m, R = 0.15 /m and G = 8 /m.

Calculate values for , , , v and Z0.


b) Derive the equation for reflection coefficient and transmission coefficient by
perfect dielectric medium for normal incidence.
c) State and derive poynting theorem and give its significance.
d) If = 0, = 2.5 0 and = 10 0 . Determine whether or not following pair of fields
satisfy Maxwells equations.
D = (3 + 6 107 t )a x , B = ( 7543 4.52 1010 t )a y .
_____________________

Set A

SLR-BB 143

*SLRBB143*
S

T.E. (Electronics Engineering) (Part I) Examination, 2014


INFORMATION TECHNOLOGY AND MANAGEMENT (New)
Day and Date : Friday, 12-12-2014
Time : 3.00 p.m. to 6.00 p.m.

Max. Marks : 100

Instructions : 1) Assume suitable examples and case study to support your answers.
2) Q. No. 1 is compulsory. It should be solved in first 30
minutes in Answer Book Page No. 3. Each question carries
one mark.
3) Answer MCQ/Objective type questions on Page No. 3 only.
Dont forget to mention, Q.P. Set (A/B/C/D) on Top of Page.
Duration : 30 Minutes

MCQ/Objective Type Questions

Marks : 20

1. Choose the correct answer :


1) Which products are people most likely to be comfortable buying on the Internet ?
a) Books
b) PCs
c) CDs
d) All of the above
2) Increases exponentially with time
a) Man power
b) Money
c) Resources
d) Data
3) The same data are stored in many places relate to
a) Data redundancy
b) Data isolation
c) Data integrity
d) All of the above
4) Involve the ownership and value of information
a) Privacy Issue b) Accuracy Issue c) Property Issue d) None of the above
5) Allow people to compare information in quickly
a) Record
b) Table
c) Array
d) All of the above
6) The word process is
a) system in action
c) execution of tasks

b) execution of activities
d) none of the above

7) The objective of activity planning is


a) estimation
b) planning
c) co-ordination
8) PERT stands for
a) Process Evaluator and Return Technique
b) Process Evaluation and Review Technique
c) Program Evaluation and Review Technique
d) Project Estimation and Review Technique

d) all of the above

P.T.O.

SLR-BB 143

*SLRBB143*

-2-

9) CPM is a type of network planning model


a) activity on node
b) activity on arrow
c) activity on line
d) none
10) Activities can be organized in different ways and we can call these as
a) planning
b) process model c) SDLC
d) network
11) A relational database is usually called
a) File
b) Flat File
c) Table File
12) OLTP stands for
a) Online Time Processing
c) Online Typical Processing

d) None of the above

b) Online Top Processing


d) Online Transaction Processing

13) Data by itself is not useful unless


a) It is massive
b) It is processed to obtain information
c) It is collected from diverse sources d) It is properly states
14) For taking decisions data must be
a) Very accurate
c) Processed correctly

b) Massive
d) Collected from diverse sources

15) Because of Data Warehouse


a) User can access needed data quickly and easily
b) User can conduct analysis
c) User can have consolidate view of data
d) All of the above
16) Important criteria to become program manager
a) expert in technical skills
b) good in management skill
c) project management experience d) all the above
17) More striking way of showing whether or not targets have been met
a) Gantt chart
b) Slip chart
c) Ball chart
d) Timeline chart
18) The people who have interest in the project
a) Stakeholders
b) Stockholders c) Promoters

d) All of the above

19) When the requirements are well defined and development methods are well
understood _________ Model is used.
a) V process model
b) Water fall model
c) Spiral model
d) None of the above
20) __________ demonstrate how it can make a company more competitive.
a) Porters model
b) Newtons model
c) Petersons model
d) None of the above
______________
Set A

*SLRBB143*
S

-3-

SLR-BB 143

T.E. (Electronics Engineering) (Part I) Examination, 2014


INFORMATION TECHNOLOGY AND MANAGEMENT (New)
Day and Date : Friday, 12-12-2014

Marks : 80

Time : 3.00 p.m. to 6.00 p.m.


Instructions : 1) All questions are compulsory.
2) Assume suitable examples and case study to support your
answers.
SECTION I
2. Attempt any four :

(64=24)

1) Define e-commerce and distinguish it from e-business.


2) What is meant by information architecture ?
3) Describe entity-relationship modeling.
4) Write a short note on geographical information system.
5) Describe how forward and reverse auctions are used in B2B commerce.
3. Attempt any two :

(82=16)

1) Explain how operational, managerial and strategic activities are related to


various IT support systems.
2) What are the different components of DBMS explain ?
3) With respect to IT, describe issues of privacy and how it is tackle with code
of ethics ?

Set A

SLR-BB 143

-4-

*SLRBB143*

SECTION II
4. Attempt any four :

(64=24)

1) Define organization and explain features of an organization.


2) What is meant by digital economy and list its major characteristics ?
3) Explain, how does a time line chart gives the indication of slippage.
4) Differentiate between computers and information systems.
5) Write short note on stake holder.
5. Attempt any two :

(82=16)

1) Explain in detail various staged of software development life cycle.


2) What are the different knowledge areas required for the effective project
management ?
3) Explain how information system impact organizations and business firms.
_____________________

Set A

SLR-BB 145

*SLRBB145*
Seat
No.

Set

T.E. (Electronics) (Part II) Examination, 2014


OPERATING SYSTEM
Day and Date : Tuesday, 25-11-2014
Time : 10.00 a.m. to 1.00 p.m.

Total Marks : 100

Instructions : 1) All questions are compulsory.


2) Assume suitable data if necessary.
3) Q. No. 1 is compulsory. It should be solved in first 30 minutes in
Answer book Page No. 3. Each question carries one mark.
4) Answer MCQ/Objective type questions on Page No. 3 only.
Dont forget to mention, Q.P. Set (A/B/C/D) on Top of Page.
MCQ/Objective Type Questions
Duration : 30 Minutes

Marks : 20

1. Multiple Choice Questions :


20
1) What is operating system ?
a) collection of programs that manages hardware resources
b) system service provider to the application programs
c) link to interface the hardware and application programs
d) all of the mentioned
2) To access the services of operating system, the interface is provided by the
a) system calls
b) API
c) library
d) assembly instructions
3) A Process Control Block (PCB) does not contain which of the following
a) Program Counter
b) Process State
c) I/O status information
d) Bootstrap program
4) The number of processes completed per unit time is known as __________
a) Output
b) Throughput
c) Efficiency
d) Capacity
5) Which of the following do not belong to queues for processes ?
a) Job queue
b) PCB queue
c) Device queue d) Ready queue
6) When a process terminates
a) It is removed from all queues
b) It is removed from all, but the job queue
c) Its process control block is de-allocated d) Both a) and c)
7) In a time-sharing operating system, when the time slot given to a process is
completed, the process goes from the running state to the
a) Blocked state
b) Ready state
c) Suspended state d) Terminated state
8) Several processes access and manipulate the same data concurrently and the
outcome of the execution depends on the particular order in which the access
takes place, is called a(n) ___________
a) Shared memory segments
b) Entry section
c) Race condition
d) Process synchronization
P.T.O.

SLR-BB 145

-2-

*SLRBB145*

9) Which process can affect of be affected by other processes executing in the system ?
a) Cooperating process
b) Child process
c) Parent process
d) Init process
10) If a process is executing in its critical section, then no other processes can be
executing in their critical section. This condition is called
a) mutual exclusion
b) critical exclusion
c) synchronous exclusion
d) asynchronous exclusion
11) In UNIX, each process is identified by its
a) Process Control Block
b) Device Queue
c) Process Identifier
d) None of these
12) Message passing system allows processes to
a) communicate with one another without resorting to shared data
b) communicate with one another by resorting to shared data
c) share data
d) name the recipient or sender of the message
13) Which module gives control of the CPU to the process selected by the shortterm scheduler ?
a) dispatcher
b) interrupt
c) scheduler
d) none of the mentioned
14) In priority scheduling algorithm
a) CPU is allocated to the process with highest priority
b) CPU is allocated to the process with lowest priority
c) equal priority processes can not be scheduled
d) none of the mentioned
15) Concurrent access to shared data may result in
a) data consistency
b) data insecurity
c) data inconsistency
d) none of these
16) Which one of the following is the deadlock avoidance algorithm ?
a) bankers algorithm
b) round-robin algorithm
c) elevator algorithm
d) karns algorithm
17) What is compaction ?
a) a technique for overcoming internal fragmentation
b) a paging technique
c) a technique for overcoming external fragmentation
d) a technique for overcoming fatal error
18) The address of a page table in memory is pointed by
a) stack pointer
b) page table base register
c) page register
d) program counter
19) Physical memory is broken into fixed-sized blocks called _________
a) frames
b) pages
c) backing store
d) none of these
20) I/O hardware contains
a) bus
b) controller
c) I/O port and its registers
d) all of the mentioned
______________
Set A

*SLRBB145*

SLR-BB 145

-3-

Seat
No.

T.E. (Electronics) (Part II) Examination, 2014


OPERATING SYSTEM
Day and Date : Tuesday, 25-11-2014
Time : 10.00 a.m. to 1.00 p.m.

Marks : 80

Instructions : 1) All questions are compulsory.


2) Assume suitable data if necessary.
SECTION I
2. Attempt any four :

(64=24)

a) What is the important aspect of a Real-Time System or Mission Critical Systems ?


b) Explain system call in an operating system.
c) Write note on threads.
d) Explain first come first serve scheduling algorithm.
e) Explain bounded buffer problem as classical problems of synchronization.
3. Attempt any two :

(82=16)

a) Consider the following set of process and CPU burst times. Calculate the average
waiting time, average response time and average turn around time for FCFS, RR
(Time quantum = 3)
Process Name

Burst Time

Priority

P1

P2

12

P3

16

P4

18

P5

b) Explain message passing and shared memory access in co-operating process


environment.
c) Explain in detail different components of personal computing system.

Set A

SLR-BB 145

-4-

*SLRBB145*

SECTION II
4. Attempt any four :

(64=24)

a) What is thrashing ? What are the causes of thrashing ?


b) Describe concept of deadlock. What are its necessary conditions ?
c) Explain least recently used page replacement algorithm.
d) Explain logical Vs physical address mapping.
e) Describe application I/O interface.
5. Attempt any two :

(82=16)

a) What is segmentation ? Describe the concept with the help of segmentation


hardware and example.
b) Explain different deadlock prevention mechanisms in detail.
c) How many page fault occur for following reference string with 4 page frames
1, 2, 3, 4, 5, 3, 4, 1, 6, 7, 8, 7, 9, 7, 8, 9, 5, 4, 5
i) FIFO
ii) LRU
iii) Optimal
Which one having minimum number of page faults ? Draw appropriate diagrams.
_____________________

Set A

SLR-BB 146

*SLRBB146*
Seat
No.

Set

T.E. (Electronics Engineering) (Part II) Examination, 2014


DIGITAL COMMUNICATION
Day and Date : Wednesday, 26-11-2014
Time : 10.00 a.m. to 1.00 p.m.

Max. Marks : 100

Instructions : 1) Figures to the right indicates full marks.


2) Assume suitable data if necessary.
3) Q. No. 1 is compulsory. It should be solved in first 30 minutes
in Answer book Page No. 3. Each question carries one mark.
4) Answer MCQ/Objective type questions on Page No. 3
only. Dont forget to mention, Q.P. Set (A/B/C/D) on Top
of Page.
MCQ/Objective Type Questions
Duration : 30 Minutes

Marks : 20

1. Select suitable option :

(120=20)

1) Pick the odd man out : PAM, PWM, PTM, PCM


a) PAM
b) PWM
c) PTM

d) PCM

2) Average information per message is called


a) Mutual information
b) Joint information
c) Uncertainty
d) Entropy
3) 1 nat = _______ bits
a) 0.4232
b) 1.4426

c) 3.3219

d) 2.3026

4) __________ code is an example of prefix for frame synchronization.


a) PLL
b) Comma
c) PN
d) PIN
5) If the probability of a message is 1/4 then, the information in bits is
a) 1 bit
b) 2 bit
c) 4 bit
d) 8 bit
6) In a PCM system, number of quantization levels is increased from 2 to 8,
bandwidth
a) get doubled
b) get tripled
c) remains same
d) become eight times
7) Quantization noise occurs in
a) PWM
b) FDM

c) PCM

d) TDM
P.T.O.

SLR-BB 146

-2-

*SLRBB146*

8) The capacity of a channel is given by


a) number of digits used in coding
b) Volume of information it can take
c) Maximum information rate transmitted
d) Average entropy
9) Which of below gives maximum probability of error for data transmission ?
a) ASK
b) FSK
c) PSK
d) DPSK
10) Which of below may not be a block of a coherent BPSK detection ?
a) square law device
b) bandpass filter
c) integrator
d) envelope detector
11) Design principle of matched filter is to have maximum _________ at a sampling
time T.
a) Power
b) PSD
c) SNR
d) All of above
12) Which of below is a continuous phase modulation ?
a) MSK
b) DPSK
c) QPSK
d) QAM
13) Companding is used in PCM to
a) reduce bandwidth
b) reduce power
c) increase S/N ratio
d) get almost uniform S/N ratio
14) Total power in sidebands of ASK is
b) A2/4
c) A2/8
d) A2
a) A2/2
15) A given source will have maximum entropy if the messages produced are
a) Only two
b) Mutually exclusive
c) Statistically independent
d) Equiprobable
16) The problem with low varying signal in DM is called ___________
a) Slope overload
b) Granular noise
c) Low distortion
d) None of above
17) When logarithm is with base e in the calculation of information, unit of
information will be
a) bits
b) nats
c) decits
d) none of above
18) __________ uses 2 successive bits at receiver.
a) BPSK
b) BFSK
c) PSK
d) DPSK
19) Which of below is disadvantage of DPSK ?
a) probability of error is higher than BPSK
b) if first bit is in error next bit is also in error
c) noise interference is more
d) all of these
20) A single parity bit can detect ________ bit and correct _________ bit of error.
a) one, one
b) two, two
c) one, zero
d) zero, one
______________
Set A

*SLRBB146*

-3-

SLR-BB 146

Seat
No.

T.E. (Electronics Engineering) (Part II) Examination, 2014


DIGITAL COMMUNICATION
Day and Date : Wednesday, 26-11-2014
Time : 10.00 a.m. to 1.00 p.m.

Marks : 80

Instructions : 1) All questions are compulsory.


2) Figures to the right indicates full marks.
3) Assume suitable data if necessary.
SECTION I
2. Solve any three :

(83=24)

a) Explain PWM. Explain how IC 555 can be used to generate PWM.


b) With help of a block diagram, explain the TDM-PCM system.
c) With suitable block diagram explain DPCM transmitter and receiver.
d) What are the limitations of delta modulation ? How are they overcome in
adaptive delta modulation ?
3. Solve any four :

(44=16)

a) Explain sampling theorem and Nyquist rate. If a sine wave of frequency


10 KHz is sampled at Nyquist rate, draw the sampled signal. Comment on
recovery of the original signal.
b) Define entropy. Prove that the entropy is maximum when all the messages
are equiprobable.
c) Explain concept of rate of information. What is its significance ? Why it is
useful ?
d) With suitable diagram explain direct method of PWM and PPM generation.
e) Compare performance of PAM, PPM, PWM.
Set A

SLR-BB 146

-4-

*SLRBB146*

SECTION II
4. a) Why coding is required in communication systems ? What are different types
of coding ? What are their purposes ? What is source coding ? What is its
purpose ? What is channel coding ? What is its purpose ?

b) Apply Shannon Fano coding procedure for below messages. Also find
efficiency and redundancy.
[X] = [x1, x2, x3, x4, x5, x6, x7]
[P(X)] = [0.4, 0.2, 0.12, 0.08, 0.08, 0.08, 0.04]

c) With suitable diagram and example explain error correction in systematic


cyclic code.

OR
d) With suitable signal space representation explain QAM, QAM modulator and
demodulator.
5. Solve any four :

(44=16)

a) Explain incoherent FSK detection.


b) Explain how a carrier can be recovered from the received double band
modulated waveform ?
c) Explain generation of BASK without baseband signal.
d) Explain matched filters.
e) Explain frame synchronization.
_____________________

Set A

SLR-BB 147

*SLRBB147*
Seat
No.

Set

T.E. (Electronics) (Part II) Examination, 2014


MICROCONTROLLERS
Day and Date : Thursday, 27-11-2014
Time : 10.00 a.m. to 1.00 p.m.

Total Marks : 100

Note : 1) Assume suitable data wherever necessary.


2) Figures to the right indicate full marks.
4) Q. No. 1 is compulsory. It should be solved in first 30 minutes
in Answer Book Page No. 3. Each question carries one mark.
5) Answer MCQ/Objective type questions on Page No. 3 only.
Dont forget to mention, Q.P. Set (A/B/C/D) on Top of Page.
MCQ/Objective Type Questions
Duration : 30 Minutes

Marks : 20

1. Objective Questions (1 mark each) :


20
1) Of the lower 128 byte of RAM in the 8051, ______________ bytes are bit
addressable ?
a) 20-2F h
b) 10-1F h
c) 00-7F h
d) 00-1F h
2) In 8051 _______________ bit of ___________ register can be used to double
the baud rate.
a) SM0, SCON
b) SMOD, PCON
c) Gate, TMOD
d) SM0, PCON
3) In 8051 an Timer 1 interrupt vector address is of ______________
a) 000Bh
b) 001Bh
c) 0013h
d) 0023h
4) LCALL instruction of 8051 is ___________ byte instruction.
a) one
b) two
c) three
d) four
5) The 8051 microcontroller is of ___________ pin package as a _________
processor.
a) 30, 1 byte
b) 20, 1 byte
c) 40, 8 bit
d) 40, 8 byte
6) The SP in 8051 is of ________________ wide register. And this may be
defined anywhere in the ___________
a) 8 byte, on-chip upper 128 byte RAM
b) 8 bit, on chip 256 byte RAM
c) 16 bit, on-chip 128 byte ROM
d) 8 bit, on chip lower 128 byte RAM
P.T.O.

SLR-BB 147

-2-

*SLRBB147*

7) In 8051, serial communication mode 0, the baud rate will be __________


a) variable
b) fosc/164
c) fosc/32
d) fosc/12
8) Which of the following signal(s) of 8051 must be used in accessing external
data RAM ?
a) RD
b) WR
c) PSEN
d) Both a) and b)
9) ACALL instruction of 8051 is ___________ byte instruction.
a) one
b) two
c) three
d) four
10) In microcontroller and LCD interface which line will instruct the LCD that
microcontroller is sending the command ?
a) DB0
b) RW
c) EN
d) RS
11) In PIC to make Port B an input port, we must place ___________ in register
____________
a) 00 h, PORTB b) FF h, PORTB c) 00 h, TRISB
d) FF h, TRISB
12) In PIC 16F877 interrupt vector is at _______________ in program memory.
a) 0000 h
b) 0004 h
c) 0040 h
d) 0400 h
13) If IRP bit of STATUS register is set, then _____________ will gets
selected.
a) Bank 0
b) Bank 1
c) Bank 2 or 3
d) Bank 0 or 1
14) The PIC 16F877 has a ____________ level deep x _____________ bit wide
hardware stack.
a) 8, 13
b) 13, 8
c) 8, 12
d) 12, 8
15) Timer 2 of the PIC 16F877 is ________________ bit, accessed as ___________
a) 8, timer/counter
b) 16, timer/counter
c) 8, timer
d) 8, counter
16) In PIC 16F877 reset vector is at ___________ in program memory.
a) 0000 h
b) 0004 h
c) 0040 h
d) 0400 h
17) The PIC 16F877 has _______ bit, maximum ___________ analog inputs AD
converter module.
a) 8, 8
b) 10, 8
c) 8, 10
d) 10, 10
18) The PIC 16F877 MSSP module in I2C mode, implements ________ bit and
_________ bit addressing.
a) 7, 10
b) 10, 8
c) 7, 8
d) 11, 7
19) If IRP bit of STATUS register is cleared, then __________ will gets selected.
a) Bank 0
b) Bank 1
c) Bank 2 or 3
d) Bank 0 or 1
20) What is the address of the last location of on-chip flash program memory for
PIC 16F877 ?
a) 0FFF h
b) 1FFF h
c) FFFF h
d) 7FFF h
______________
Set A

*SLRBB147*

-3-

SLR-BB 147

Seat
No.

T.E. (Electronics) (Part II) Examination, 2014


MICROCONTROLLERS
Day and Date : Thursday, 27-11-2014

Marks : 80

Time : 10.00 a.m. to 1.00 p.m.


Note : 1) Assume suitable data wherever necessary.
2) Figures to the right indicate full marks.
SECTION I
2. Answer any four (5 marks each) :

20

a) Write a program for finding a largest number form a given array of 10


elements.
b) Draw and explain memory organization in 8051.
c) Write a short note on RS 232 bus standard.
d) Explain the interrupt sources of 8051 in detail.
e) Explain the different modes in serial interface of 8051.
3. Answer any two (10 marks each) :

20

a) Draw and explain interfacing of DAC to 8051. Write a program


i) To generate triangular wave
ii) To generate a saw tooth wave.
b) Draw and explain interfacing of 8K 8 Data ROM to 8051. Write a assembly
program to read 30 bytes starting at 1000h in external ROM.
c) Interface two seven segment display to 8051 and write a program to display
00 to 99.
Set A

SLR-BB 147

-4-

*SLRBB147*

SECTION II
4. Answer any four (5 marks each) :

20

a) Explain W, FSR, INDF, PCL, PCLATH registers in PIC.


b) Explain addressing modes in PIC 16F877.
c) Describe functioning of CCP1 unit for the capture mode.
d) Explain Serial Peripheral Interface in PIC.
e) What are the uses of Timer 2 in PIC 16F877 ? Explain its operation.
5. Answer any two (10 marks each) :

20

a) Interface 4 4 keypad to PIC 16F877 and explain it.


b) Draw and explain interfacing of LCD to PIC. Write a program to display NO
on LCD.
c) Interface eight LEDs to PORTB of PIC and write a program for running LEDs
continuously.
_____________________

Set A

SLR-BB 148

*SLRBB148*
Seat
No.

Set

T.E. (Electronics) (Part II) Examination, 2014


INDUSTRIAL ELECTRONICS
Day and Date : Friday, 28-11-2014
Time : 10.00 a.m. to 1.00 p.m.

Total Marks : 100

Instructions : 1) Q. No. 1 is compulsory. It should be solved in first 30 minutes in


Answer book Page No. 3. Each question carries one mark.
2) Answer MCQ/Objective type questions on Page No. 3 only.
Dont forget to mention, Q.P. Set (A/B/C/D) on Top of Page.
MCQ/Objective Type Questions

Duration : 30 Minutes

Marks : 20

1. Choose the correct answer :

20

1) In single phase fully controlled converter with freewheeling diode and inductive
load with firing angle of 60 and amplitude of input voltage is 150V, the dc output
voltage is
a)

225

b)

150

c)

50

d) 150

2) A UJT is used for triggering an SCR then the wave shape of the voltage obtained
from the UJT circuit is
a) Swatooth wave
b) Sine wave
c) Trapezoidal wave
d) Square wave
3) Thyristor can be termed as
a) AC switch
b) DC switch

c) Both a) and b)

d) Square wave switch

4) Which of the following PNPN device has two gate terminal ?


a) SCS
b) SUS
c) LASCR
d) PUT
5) A MOSFET switch in its ON state may be considered equivalent
a) Resistor
b) Capacitor
c) Inductor
d) Battery
6) For thyristor pulse triggering is preferred than DC triggering because
a) Gate dissipation is low
b) Pulse system is simpler
c) Triggering system is required for very short duration
d) All above
7) In 1 phase fully controlled converter with inductive load RMS value of output
voltage
a)

Vm
2

c) 0.707Vm

b) Same as RMS value of input


d) All above
P.T.O.

SLR-BB 148

*SLRBB148*

-2-

8) An UJT relaxation oscillator has RE = 60 K , C = 0.25 F and = 0.65, then


pulse repetition frequency is _________
a) 63.57Hz
b) 6.357Hz
c) 6357Hz
d) 630.57Hz
9) The conduction losses verses device current characteristics of power MOSFET is
best approximated by
a) a parabola
b) a straight line
c) a rectangular hyperbola
d) an exponentially decaying function
10) In mode __________, TRIAC operation is similar remote gate thyristor.
a) II
b) III
c) IV
d) Both b) and c)
11) An optocoupler is a combination of __________
a) Infrared light emitting diode and transistor
b) Infrared light emitting diode and photo SCR
c) Light emitting diode and transistor
d) Both a) and b)
12) Constant voltage transformer uses principle of
a) Ferro resonance
b) Parallel resonance
c) Series resonance
d) None of above
13) SMPS has better efficiency than of linear regulator because power transistor is
operated in __________ region.
a) Active
b) Saturation
c) Both a) and b) d) Saturation and cut off
14) In ladder diagram, limit level switches represented by
a)

b)

c)

d)

15) In isolated SMPS average O/P voltage increased by


a) Increasing duty cycle
b) Maintaining constant duty cycle
c) Decreasing duty cycle
d) Both b) and c)
16) In case of photo transistor ____________ junction is photosensitive.
a) Base emitter
b) Base collector c) Collector emitter d) Both b) and c)
17) To reduce the overshoot and bring the controlled variable to the set point quickly
__________ is used.
a) Derivative control
b) Proportional control
c) Integral control
d) On off controller
18) By narrowing the proportional band, a closed loop system
a) Become stable b) May oscillate c) Is unaffected

d) Both a) and c)

19) When photo diode is operated with reverse bias it operates as ________ device.
a) Photoconductive b) OFF
c) Photovoltaic
d) ON state
20) Modulated light means
a) Light supplied with regular interruption
b) It can be considered as flickering light
c) Continuous light
d) Both a) and b)
______________

Set A

*SLRBB148*

-3-

SLR-BB 148

Seat
No.

T.E. (Electronics) (Part II) Examination, 2014


INDUSTRIAL ELECTRONICS
Day and Date : Friday, 28-11-2014
Time : 10.00 a.m. to 1.00 p.m.

Marks : 80

Instructions : 1) All questions are compulsory.


2) Figures to the right indicate full marks.
SECTION I
2. Attempt any four :

(45=20)

1) Explain equivalent circuit and VI characteristics of SCS.


2) Explain Class B resonant pulse commutation. Sketch associated waveforms.
3) Explain parasitic model of MOSFET.
4) Compare Thyristor, IGBT, E-MOSFET and GTO.
5) A single phase controlled rectifier supplies dc voltage of 100 V. The output current
may be assumed constant at 20 A. The supply voltage is 230 V AC and source
inductance per phase is 5 mH. Calculate firing angle and overlap angle.
3. Attempt any two :

(210=20)

1) Explain switching characteristics of thyristor during turn on and turn off


process.
2) Explain VI characteristics TRIAC and explain switching action and different
triggering mode.
3) A single phase half wave controlled rectifier has purely resistive load of
R and delay angle is /2. Determine Rectification efficiency, Form factor and
Ripple factor.

In above circuit SCR is used to control the power of 1KW, 230 V, 50 Hz heater.
Determine heater power for firing angle of 45 and 90.

Set A

SLR-BB 148

-4-

*SLRBB148*

SECTION II
4. Attempt any four :

(45=20)

1) Explain different configuration of optocouplers.


2) A sensor output voltage ranging from 2.5V to 1.2V. For interface to ADC this
needs to be 0 to 2.5 V. Develop the required signal conditioning network.
3) Explain working of generalized data acquisition system.
4) Explain working of photo chopper circuit using photo conductive cell.
5) Develop the ladder diagram for AC motor with following :
NO start push button, NC stop push button, thermal overload switch opens at high
temperature, green light when running, red light for thermal overload.
5. Attempt any two :

(210=20)

1) Explain working of relay type voltage stabilizer and Tap changing voltage stabilizer
for boost action.
2) Design analog ON-OFF temperature controller with dead band to control
temperature in the range of 0 to 150C, set point is 60C and with dead band is
4C. Select sensor PT 100 at 0 resistance is 100 at 150 resistance is 150 .
3) Explain working of PID controller of robotic control system.
_____________________

Set A

SLR-BB-149

*SLRBB149*
S

T.E. (Electronics Engineering) (Part II) Examination, 2014


VLSI DESIGN
Day and Date : Saturday, 29-11-2014
Time : 10.00 a.m. to 1.00 p.m.

Max. Marks : 100

Instructions : 1) Figures at right indicate full marks.


2) Q. No. 1 is compulsory. It should be solved in first 30 minutes in
Answer Book Page No. 3. Each question carries one mark.
3) Answer MCQ/Objective type questions on Page No. 3 only. Dont
forget to mention, Q.P. Set (A/B/C/D) on Top of Page.
Duration : 30 Minutes

MCQ/Objective Type Questions

Marks : 20

1. Choose the correct option :

(201=20)

1) In CMOS gate, pull up network consist of _________


a) PMOS transistors
b) NMOS transistor
c) Both a) and b)
d) a) or b)
2) In a CMOS inverter with Vtn
n-device are
a) saturated, nonsaturated
c) nonsaturated, cutoff


Vin < VDD/2, the states of p-device and


b) nonsaturated, saturated
d) none of these

3) The design unit used for storage of common declarations are


a) configuration declarations
b) package body
c) package declarations
d) none of these
4) Process in VHDL becomes active, when
a) there is change in the value of signal in sensitivity list
b) change in clock statement
c) change in reset signal
d) none of the signal
5) The ratio Bn/Bp, for CMOS inverter that allows a capacitive load to charge and
discharge in equal times is
a) 10
b) 1
c) 0.1
d) None of these
6) DRAM uses _____________ transistors per bit of storage.
a) Two
b) Three
c) Four

d) One

7) _____________ is not a concurrent statement.


a) Generate
b) If
c) When else

d) With select

8) ___________ is almost a perfect switch when 1 is to be passed from input to output.


a) Buffer
b) N-switch
c) p-switch
d) None of these
9) Assuming the left operand as BIT vector, 1001010 rol 2 is
a) 0101010
b) 0101000
c) 0101011

d) None of these
P.T.O.

SLR-BB-149

*SLRBB146*

-2-

10) The number of transistors required to implement 4 input OR gate are


a) 3
b) 6
c) 8
d) 10
11) No. of flip-flops/CLB in FPGA XC4000 is
a) 4
b) 2

c) 6

d) 1

12) Consider the circuit

Fault C/1 is detected by test W1 W2 W3 =


a) 011
b) 010
c) 101

d) None of these

13) The difference between arrival times of clock at different devices is


a) clock difference
b) clock gating
c) clock arrival difference
d) clock skew
14) Microcells whose output are usable only internally are called _________ microcells.
a) feedback
b) internal
c) buried
d) none of these
15) Techniques used in Boolean optimization
a) Factoring and flattening
c) Translation and flattening

b) Translation and factoring


d) None of above

16) Struck at fault is generated when


a) An input to gate is incorrectly connected to power supply
b) If wire stuck at a particular value
c) Both a) and b)
d) None of above
17) DRAM uses __________ transistors per bit of storage.
a) Two
b) Three
c) Four

d) One

18) __________ attribute specifies the resistance of the driver, which controls how much
current it can source.
a) drive
b) source
c) load
d) none of these
19) The programmable unit in Xilinx XC 4000 family of FPGA is called
a) Programmable logic device
b) Look up table
c) Configurable logic block
d) None of these
20) Textual representation of interconnection of logic element is ___________
a) Netlist
b) Architecture
c) Entity
d) Package
______________

Set A

*SLRBB146*
S

-3-

SLR-BB-149

T.E. (Electronics Engineering) (Part II) Examination, 2014


VLSI DESIGN
Day and Date : Saturday, 29-11-2014

Marks : 80

Time : 10.00 a.m. to 1.00 p.m.


Instructions :

1) All questions are compulsory.


2) Figures at right indicate full marks.
SECTION I

2. Solve any four :

(44=16)

a) Explain static and dynamic power dissipation in CMOS.


b) Explain inertial delay model.
c) Write VHDL code for full adder.
d) Write a short note on CMOS logic.
e) Explain packages in VHDL with example.
f) Write a VHDL code for 4 : 1 multiplexer.
3. Solve any three :

(83=24)

a) What is CMOS inverter ? Explain CMOS Inverter DC transfer characteristics in detail.


b) Explain the following :
i) sequential statement
ii) signal assignment within sequential statement.
c) Explain variables, signals, data types, arrays related to VHDL.
d) Write a VHDL code for N-bit binary adder.
Set A

SLR-BB-149

-4-

*SLRBB146*

SECTION II
4. Attempt any four :

(46=24)

a) Write note on :Synchronization failure and metastability.


b) Draw and explain the IO block of CPLD family.
c) Explain the place and route process.
d) Write VHDL code for BCD up down counter.
e) Explain path sensitizing. Obtain the complete test set for circuit shown below using path
sensitizing.

f) Draw the circuit represented by following VHDL process.


process (clk)
begin
if clr = 1 then Q <= 0;
elsif clkevent and clk = 0 and CE = 1 then
if C = 0 then Q <= A and B;
else Q <= A or B;
end if;
end if;
end process;
Why is clr on the sensitivity but not C ?
5. Attempt any two :

(28=16)

a) Explain in detail the architecture of Function Block (FB) of CPLD Xilinx 9500.
b) Explain in detail VLSI design flow.
c) Derive the state table/diagram for Moore FSM that has an input w and an output z.
The machine has to generate z = 1 when the previous four values of w were 1001;
otherwise, z = 0. Overlapping input patterns are allowed. Write VHDL code for the FSM
described above.
_____________________

Set A

SLR-BB 15

*SLRBB15*
Seat
No.

Set

F.E. (Part II) Examination, 2014


BASIC ELECTRONICS AND COMPUTER PROGRAMMING
Day and Date : Thursday, 27-11-2014
Time : 10.00 a.m. to 1.00 p.m.

Max. Marks : 80

N. B. : 1) All questions are compulsory.


2) Figures to right indicates full marks.
3) Basic Electronics and computer programming should be written on
separate answer books.
4) Q. No. 1 is compulsory. It should be solved in first 30 minutes in
Answer book Page No. 3. Each question carries one mark.
5) Answer MCQ/Objective type questions on Page No. 3 only. Dont
forget to mention, Q.P. Set (A/B/C/D) on Top of Page.
MCQ/Objective Type Questions
Duration : 30 Minutes

Marks : 20
SECTION I
(Basic Electronics)

1. Multiple choice questions :

10

1) Knee voltage of germanium diode is _________


a) 0.3v
b) 0.7v
c) 1v

d) 0v

2) Color coding for given resistor is Blue-red-orange, then its value is ________
a) 62 kilo ohm
b) 6.2 kilo ohm
c) 6.2 meter ohm
d) 6200 ohm
3) Diode is ________ a device.
a) a non linear
c) an amplifying

b) a linear
d) none of the above

4) To display digit 9 in a seven segment display.

a) A segment off

b) B segment off

c) E segment off

d) All segments on

5) The base of transistor is ________ doped.


a) Heavily
b) Moderately
c) Lightly
d) Either lightly or heavily
6) The phase difference between the input and output voltages of a transistor connected in
common collector arrangement is
a) 180
b) 0
c) 90
d) 270
7) The smallest possible ripple factor we get in case of
a) Half wave rectifier
c) Center tap full wave rectifier with capacitor filter

b) Full wave bridge rectifier


d) None of the above

P.T.O.

SLR-BB 15

*SLRBB15*

-2-

8) Which of the following is active transducer


a) Thermocouple
c) Reluctance pickup
9) (A+B) = _________
a) A . B
b) A + B
10) (49.25)10 = ( ? )2
a) 11001.10
b) 110001.10

b) Linear Variable Differential Transformer


d) Load cell
c) A B

d) None of these

c) 110000.01

d) 110001.01

SECTION II
(Computer Programming)

10

11) _________ is user defined data type.


a) Structure
b) Union
c) Array
d) Both a) and b)
12) Syntax of declaring variable is _________
a) Variablename datatype;
b) datatype variablename;
c) variablename datatype
d) None
13) Actual parameters are passed at the time of _________
a) Function definition
b) Function calling
c) Function declaration
d) None of above
14) What is the output of the following program ?
int x = 10, y = 20;
x==20&&y! = 10 ? printf(True) : printf(False);
a) False
b) True
c) True False
d) None of above
15) Strut student
{
char name [20];
int rollno;
}; Size of structure is __________
a) 20
b) 40
c) 22
d) 24
16) Which of the following function is more appropriate for reading multi-word sting ?
a) printf
b) puts
c) scanf
d) gets
17) In an array int arr[12] the word arr represent _________ and 12 represent _________
a) datatype, size
b) size, datatype
c) name of array, size
d) size, name of array
18) Which of the following is the correct way of declaring a float pointer ?
a) float ptr ;
b) float *ptr;
c) *float ptr;
d) none of the above
19) What is the output of following program ?
int i=9, j=6, p;
float p, x=0. 5, y=0. 1;
char a=a, b=b;
p=(3*i-2*j)%(2*a b);
printf (%0.2f,p);
a) 96.00
b) 51,00
c) 0.15
d) 15.00
20) Which of the following are the pointer operators ?
a) &, %
b) &.*
c) *,$
d) none of the above
_________________

Set A

*SLRBB15*

-3-

SLR-BB 15

Seat
No.

F.E. (Part II) Examination, 2014


BASIC ELECTRONICS AND COMPUTER PROGRAMMING
Day and Date : Thursday, 27-11-2014
Time : 10.00 a.m. to 1.00 p.m.

Marks : 80

N. B. : 1) All questions are compulsory.


2) Figures to right indicates full marks.
3) Basic Electronics and computer programming should be written on
separate answer books.
SECTION I
(Basic Electronics)
2. Solve any four questions :

(45=20)

1) With the help of neat circuit diagram and waveforms, explain full wave bridge rectifier in
detail.
2) Compare LED with LCD display.
3) Perform the following subtraction using 2s complement method.
a) (23)10 (48)10
b) (48)10 (23)10
4) Define the following terms with respect to transducers.
a) Transducer
b) Linearity
c) Accuracy
5) Explain On Off, SPST and DPDT type of switches in detail.
6) Draw and explain input and output characteristics of common emitter amplifier.
3. Solve any two questions :

(210=20)

1) Explain the application of transistor as an amplifier and switch.


2) Explain thermocouple for the measurement of temperature in detail.
3) Prove the following Boolean expressions :
a) A + AB = A
b) (A+ B) (A + C) = A + BC
c) (A + B + AB) (A + B) (AB) = 0
d) A + AB + AB = A + B

Set A

SLR-BB 15

-4-

*SLRBB15*

SECTION II
(Computer Programming)
4. Attempt any 4 questions :

(45=20)

a) Write output of following program code and explain.


void main ( )
{
int x = 4, y = 0, z ;
while (x > = 0)
{
if (x = - y)
break ;
else
printf ( \n%d %d, x, y) ;
x;
y++;
}
}
b) Explain call by value and call by reference with example.
c) Write the program again by correcting the errors if any. Underline corrected errors.
And also write the output.
#include < stdio.h>
#include < string.h>
void main ( )
{
struct
{
Char bookname [20] ;
float price;
}
struct book b = {GO Embedded !, 240.00} ;
printf (%s\n%f, b.bookname, b.price)
}

Set A

SLR-BB 15

-5-

*SLRBB15*

d) Explain switch case statement with example.


e) Write the same program, indicate function definition, call and declaration.
What will be the output of the program.
#include<stdio.h>
void display ( ) ;
main ( )
{
Printf (Learn C\n) ;
display ( ) ;
}
void display ( )
{
printf (Followed by C++, c# and Java !) ;
}
f) Write note on :
1) Structure of C
2) Flowchart and algorithm
_____________________

Set A

Set A

SLR-BB 150

*SLRBB150*
S

B.E. (Electronics) (Part I) Examination, 2014


POWER ELECTRONICS
Day and Date : Tuesday, 2-12-2014
Time : 3.00 p.m. to 6.00 p.m.
Instructions :

Duration : 30 Minutes

Total Marks : 100

1) Assume suitable data if required.


2) Figures to right indicate maximum marks.
3) Q. No. 1 is compulsory. It should be solved in first 30 minutes in
Answer Book Page No. 3. Each question carries one mark.
4) Answer MCQ/Objective type questions on Page No. 3 only. Dont
forget to mention, Q.P. Set (A/B/C/D) on Top of Page.
MCQ/Objective Type Questions

Marks : 20

1. Choose the correct answer :


1) In three phase half controlled converter if firing angle is greater than 60 then converter
becomes __________________ converter.
a) 3 pulse
b) 6 pulse
c) 12 pulse
d) both a) and b)
2) In multiphase chopper (In phase operating mode)
a) choppers are ON and OFF at the same time
b) choppers are ON and OFF at the different time
c) choppers operates at the different frequency with proper phase shift
d) both b) and c)
3) Class C chopper
a) formed by combining Class A and B chopper
b) load current is always positive
c) load voltage is positive or negative
d) both b) and c)
4) In ________________ TON of chopper is varied keeping chopper frequency constant.
a) constant frequency system
b) pulse width modulation control
c) constant frequency TRC
d) all above
5) In _______________ commutation, charged capacitor is switched by an auxiliary SCR
to commutate main SCR.
a) impulse
b) current
c) load
d) both a) and b)
6) The input AC current of a three phase fully controlled converter contains only
______________ harmonics but no _______________ harmonics.
a) even, odd
b) odd, triplen
c) odd, even
d) trippler, odd

20

7) A three phase controlled converter operating with 3 440, 60Hz supply is feeding a resistive
load. Then PIV is _______________
a) 762
b) 254
c) 440
d) 622.2
8) Commutating overlap will cause
a) improved power factor
b) higher PIV
c) reduced output voltage
d) all above
= 60
9) A single phase fully controlled bridge converter supplies an inductive load. If
then average dc voltage is


a)
V

b)
2

c)

d) 0.95Vm

P.T.O.

SLR-BB 150

*SLRBB150*

-2-

10) A single phase controlled converter is charging a battery from an existing AC mains. It is
possible to feed DC power back to AC supply when
a) firing angle is in between 0 to 90 with same battery connection
b) firing angle is in between 90 to 180 with same battery connection
c) firing angle is in between 0 to 90 with reverse battery connection
d) firing angle is in between 90 to 180 with reverse battery connection
11) In three phase bridge inverter with fundamental output frequency of 50 Hz, if third harmonic
is eliminated, then frequencies of other components in the output voltage wave, in Hz,
would be
a) 50, 250, 350, 550, high frequencies
b) 50, 250, 350, 450
c) 50, 250, 350
d) 50 Hz
12) In voltage source inverter amplitude of output voltage _________________ and amplitude
of current _______________
a) depends upon load, does not depends upon load
b) does not depends upon load, depends upon load
c) independent of load, independent of load
d) remains constant, remains constant
13) In half bridge series resonant inverter
a) power flow from DC source is continuous
b) firing of one thyristor results turning off of other conducting thyristor
c) peak current rating of source is reduced
d) all above
14) In sinusoidal pulse width modulation if number of pulses per half cycle is 3 then
a) all harmonics up to 5th order are eliminated
b) all harmonics up to 7th order are eliminated
c) all harmonics up to 3rd order are eliminated
d) all above
15) Speed control of DC motor can be obtained from
a) dual converter
b) controlled rectifier
c) chopper
d) all above
16) Square wave PWM technique allows ___________
a) voltage and harmonics control
b) only harmonics control
c) only voltage control
d) only frequency control
17) In single phase bridge inverter, if input dc voltage is 100V then peak reverse blocking
voltage is
d) 50V
a) 200V
b) 100V
c)
18) For the large A.C. motor control shown below, if the firing angle of SCRs in the controlled
rectifier circuit is reduces then
1

a) motor speed will increases


b) motor speed will decrease
c) the speed will not be effected
d) frequency of inverter output will decrease
19) A three phase 3-pulse cycloconverter uses
a) 6 SCRs
b) 12SCRs
c) 24 SCRs
d) 18 SCRs
20) In a 3-pulse cycloconverter with intergroup reactor operating in circulating current mode,
both P and N converter groups synthesize the
a) same fundamental sine wave
b) different fundamental sine wave
c) same fundamental cosine wave
d) different fundamental cosinewave
______________
Set A

*SLRBB150*
S

SLR-BB 150

-3-

B.E. (Electronics) (Part I) Examination, 2014


POWER ELECTRONICS
Day and Date : Tuesday, 2-12-2014
Time : 3.00 p.m. to 6.00 p.m.
Instructions :

Marks : 80

1) All questions are compulsory.


2) Figures to right indicate maximum marks.
SECTION I

2. Attempt any four :

(46)

1) In single phase fully controlled thyristor bridge converter supplies a load consisting of RLE load.
The inductance in the circuit is so large that output current may be considered to virtually constant.
Assume SCR to be ideal with following data. RMS supply voltage = 230V, RL = 0.4
,
L = 12mH, Idc = 8A. Determine


a) Firing angle if Eb = 100V

b) Firing angle if Eb = 100V


c) Which source (AC or DC) is suppling power in a) and b)
d) Draw load voltage waveform for a) and b)
2) Derive an expression for average output voltage of three phase controlled rectifier with
highly inductive load. Sketch associated waveforms for
= 90.


3) Explain working of step down chopper and derive an expression for output voltage, duty
cycle and effective input resistance.
4) A three phase controlled bridge converter is connected to 440 (rms), 3 , 50Hz mains.
The load consists of 300V battery in series with resistor of 12 . The output current of
converter is 16 A ripple free. Calculate average output voltage of converter, delay angle
and ratings of SCRs.


5) Explain working multiphase chopper circuit.


3. Attempt any two :

(28)

1) Explain operation of dual converter with circulating current mode and non-circulating current
mode. Sketch associated waveforms.
2) Describe load commutated chopper with associated voltage and current waveform as a
function of time. Sketch associated waveforms.
3) Explain microcontroller based firing scheme for three phase controlled converters with
associated flow chart.

Set A

SLR-BB 150

-4-

*SLRBB150*

SECTION II
4. Attempt any four :

(46)

1) Explain operation of single phase full bridge IGBT based voltage source inverter with
resistive load and derive an expression of RMS output voltage. Sketch associated
waveforms.
2) Explain harmonics reduction techniques using stepped wave inverter.
3) Explain working of three phase cycloconverter drive.
4) What is power factor corrector ? Explain with neat diagram capacitive power factor
corrector.
5) Explain working of separately excited DC motor fed from two quadrant dc chopper.
5. Attempt any two :

(28)

1) Explain working of 120 conduction mode, three phase bridge inverter feeding star
connected purely resistive load. Draw associated line and phase voltage.
2) Explain block schematic of microcontroller based four quadrant AC drive with help of
flow chart.
3) Explain operation of single phase bridge type cycloconverter with inductive load.
The frequency is f0/fs = 1/7. Sketch associated waveforms.
_____________________

Set A

SLR-BB 151

*SLRBB151*
Seat
No.

Set

B.E. (Electronics Engg.) (Part I) Examination, 2014


COMPUTER NETWORKS
Day and Date : Thursday, 4-12-2014
Time : 3.00 p.m. to 6.00 p.m.

Max. Marks : 100

Instructions : 1) Figures to the right indicate full marks.


2) Assume suitable data if necessary.
3) Q. No. 1 is compulsory. It should be solved in first 30 minutes in
Answer book Page No. 3. Each question carries one mark.
4) Answer MCQ/Objective type questions on Page No. 3 only. Dont
forget to mention, Q.P. Set (A/B/C/D) on Top of Page.
MCQ/Objective Type Questions
Duration : 30 Minutes

Marks : 20

1. Choose the correct answer. (Each correct answer carries one mark).

20

1) Contention is
1) one or more conductors that serve as a common connection for a related group of
devices
2) the condition when two or more stations attempt to use the same channel at the same
time
3) a collection of interconnected functional units that provides a data communications
service among stations attached to the network
4) a continuous frequency capable of being modulated or impressed with a second signal
2) A distributed network configuration in which all data/information pass through a central
computer is
1) Bus topology
2) Star topology
3) Point to point topology
4) Ring topology
3) The slowest transmission speeds are those of
1) twisted-pair wire 2) coaxial cable
3) fiber-optic cable

4) microwaves

4) A noiseless 3 KHz channel transmits bits with binary level signals. What is the maximum
data rate ?
1) 3 KHz
2) 6 KHz
3) 12 KHz
4) 24 KHz
5) In CRC there is no error if the remainder at the receiver is ___________
1) equal to the remainder at the sender
2) zero
3) non zero
4) the quotient at the sender
6) Usually, it takes 10-bits to represent one character. How many characters can be
transmitted at a speed of 1200 BPS ?
1) 10
2) 12
3) 120
4) 1200
P.T.O.

SLR-BB 151

*SLRBB151*

-2-

7) Which of the following is required to communicate between two computers ?


1) communications software and hardware
2) protocol
3) access to transmission medium
4) all of the above
8) Three or more devices share a link in ___________ connection.
1) Unipoint
2) Multipoint
3) Point to point
4) None of the mentioned
9) The layer responsible for end to end delivery of the entire message is
1) Network layer
2) Session layer
3) Transport layer
4) Application layer
10) In ARQ, a NAK
1) is sent by the recipient if the message contains an error
2) means that the sender should continue with sending the next message
3) is sent by the recipient if the message was received without error
4) is sent by the sender at the same time as it sends a data packet
11) How many bits are used for addressing in the Ethernet ?
1) 64 bits
2) 48 bits
3) 32 bits

4) Variable bits

12) What is the name of routing type in which every incoming packet is sent to every neighbor
router except the one from which the packet came ?
1) distribution
2) multicast
3) flooding
4) shortest-path
13) Which of the following does not have a Data-Link Layer ?
1) Router
2) Gateway
3) Switch

4) Repeater

14) What is the purpose of preamble bits in an Ethernet frame ?


1) Pre-bit counting
2) Synchronization
3) Error checking
4) Destination address
15) What is the default mask of the gateway IP address 193.145.122.76 ?
1) 255.255.255.0
2) 255.255.255.255
3) 255.255.0.0
4) 255.0.0.0
16) What is the algorithm to handle collisions in Ethernet networks ?
1) Collision pooling
2) Time Division Mux
3) Exponential-backoff
4) CSMA-CD
17) Which of the following is most important disadvantage of bus networks ?
1) have many collisions
2) expensive cables used
3) one fault downs entire net
4) higher traffic
18) In IP networking, network and host addresses are separated using __________
1) ARP
2) TCP
3) netmask
4) separator
19) The IEEE 802.11 standard is known as __________
1) Token bus standard
2) Wireless LAN
3) CSMA/CD
4) Token ring standard
20) The minimum size of IP header is ____________
1) 10 bytes
2) 20 bytes
3) 32 bytes
______________

4) 64 bytes
Set A

*SLRBB151*

-3-

SLR-BB 151

Seat
No.

B.E. (Electronics Engg.) (Part I) Examination, 2014


COMPUTER NETWORKS
Day and Date : Thursday, 4-12-2014

Marks : 80

Time : 3.00 p.m. to 6.00 p.m.


Instructions : 1) Figures to the right indicate full marks.
2) Assume suitable data if necessary.
SECTION I
2. Answer any four :

(46=24)

1) What are the various network components ? Discuss different network categories and
types of communication.
2) What is layered reference model for communication ? Explain important issues to be
addressed in reliable communication.
3) Explain different types of MODEMS. What is the need of scrambler and de-scrambler in
the MODEM ?
4) Why flow control is essential ? Discuss stop_and_wait flow control protocol ? Which
signals are used for flow and error control in stop_and_wait flow control protocol ?
5) What are the assumptions about the carrier sense in dynamic channel allocation ? Explain
different CSMA methods.
3. Answer any two :

(28=16)

1) What is Hamming distance and Hamming codeword ? Generate Hamming codeword for
ascii character 10011011. Assume even parity during transmission.
2) Draw frame format of IEEE 802.3 LAN and explain each field is detail. Which CSMA
mechanism is adopted by IEEE 802.3 LAN during occurrence of collision ? Is efficiency
of IEEE 802.3 LAN is function of frame size ?
3) What are the different modes of data transfer in HDLC ? Draw control field format of I, S
and U frame in case of HDLC and describe it in detail.

Set A

SLR-BB 151

-4-

*SLRBB151*

SECTION II
4. Answer any four :

(46=24)

1) Draw TCP/IP reference model and describe each layers function in detail. What is
encapsulation and de-capsulation in TCP/IP ?
2) What are class full IP addresses ? Give address ranges of various IP addresses. Draw
special IP address formats and explain them in detail.
3) What is DNS ? Explain DNS name space and address mapping related to DNS.
4) What are the issues handled in network layer ? Differentiate between datagram and virtual
packet switching.
5) List different network devices used in computer communication. Explain the network
devices switch and bridges in detail.
5. Answer any two :

(28=16)

1) Draw the TCP header format and explain each field in detail.
2) What is principle of optimality and fairness in routing protocols ? Discuss shortest path
algorithm in detail.
3) What is congestion ? What is the effect of congestion in communication ? Discuss
leaky bucket algorithm in detail to control the congestion over network.
_____________________

Set A

SLR-BB 152

*SLRBB152*
Seat
No.

Set

B.E. (Electronics Engineering) (Part I) Examination, 2014


MOBILE TECHNOLOGY
Day and Date : Saturday, 6-12-2014
Time : 3.00 p.m. to 6.00 p.m.

Max. Marks : 100

Instructions : 1) Figures to the right indicate full marks.


2) Assume suitable data if necessary.
3) Q. No. 1 is compulsory. It should be solved in first 30 minutes
in Answer book Page No. 3. Each question carries one mark.
4) Answer MCQ/Objective type questions on Page No. 3 only.
Dont forget to mention, Q.P. Set (A/B/C/D) on Top of Page.
MCQ/Objective Type Questions
Duration : 30 Minutes

Marks : 20

1. Select suitable option :


20
1) What is the basic service unit of cellular telephony ?
a) location area
b) cell
c) PLMN service area
d) MSC/VLR service area
2) The ____________ provides authentication and encryption parameters that
verify the users identity and ensure the confidentiality of each call.
a) HLR
b) VLR
c) EIR
d) AUC
3) Modulation refers to ____________
a) the distance between the uplink and downlink frequencies
b) the separation between adjacent carrier frequencies
c) the process of changing the characteristics of a carrier frequency
d) the number of cycles per unit of time
4) The first cellular systems were ____________
a) analog
b) digital
c) semi analog
d) none of the above
5) The type of access used in GSM technology is
a) FDMA/TDMA
b) CDMA
c) OFDMA
d) None of the above
6) The core concept used in Cellular technology is
a) TDM
b) Frequency Reuse
c) Code Reuse
d) None of the above
7) The uplink frequency of P-GSM system is
a) 1850-1910 Mhz
b) 1710-1785 Mhz
c) 890-915 Mhz
d) none of the above
P.T.O.

SLR-BB 152

-2-

*SLRBB152*

8) The technique adopted to increase the system capacity and reduce


co-channel interference is
a) High power BTS
b) By installing the Omnidirectional antenna
c) Sectorisation
d) None of the above
9) The cell having the same number in the adjacent cluster using the same set
of RF channels are termed as
a) Adjacent cell
b) Co channel cell
c) Macro cell
d) Selective cell
10) The process of channel coding, Encryption, Multiplexing and modulation for
Trans direction and reverse for reception are to be carried out by
a) BTS
b) BSC
c) MSC
d) MS
11) In 802.11 network protocol stack, PMD and PLCP are the sublayers of _______
a) PHY
b) DLC
c) MAC
d) LLC
12) Which of the below is low power mode of Bluetooth device ?
a) park
b) hold
c) sniff
d) all of above
13) In a mobile IP, a tunnel usually ends at ____________
a) HA
b) FA
c) CN
d) Router
14) A DHCP protocol is based on ____________ model.
a) Agent discovery
b) Agent solicitation
c) Client server
d) All of above
15) Frame control filed in IEEE 802.11 MAC data frame is of _________ bytes.
a) 2
b) 1
c) 4
d) 6
16) An interconnected collection of piconet is called
a) scatternet
b) micronet
c) mininet
d) none of the mentioned
17) What is the Access Point (AP) in wireless LAN ?
a) device that allows wireless devices to connect to a wired network
b) wireless devices itself
c) both (a) and (b)
d) none of the mentioned
18) In wireless ad-hoc network
a) access point is not required
b) access point is must
c) nodes are not required
d) none of the mentioned
19) Which multiple access technique is used by IEEE 802.11 standard for wireless
LAN ?
a) CDMA
b) CSMA/CA
c) ALOHA
d) none of the mentioned
20) A wireless network interface controller can work in
a) infrastructure mode
b) ad-hoc mode
c) both (a) and (b)
d) none of the mentioned
______________
Set A

*SLRBB152*

-3-

SLR-BB 152

Seat
No.

B.E. (Electronics Engineering) (Part I) Examination, 2014


MOBILE TECHNOLOGY
Day and Date : Saturday, 6-12-2014
Time : 3.00 p.m. to 6.00 p.m.

Marks : 80

Instructions : 1) All questions are compulsory.


2) Figures to the right indicate full marks.
3) Assume suitable data if necessary.
SECTION I
2. Attempt any four :

20

a) With suitable example explain transmission, detection and interference range.


b) What is fading ? What are its types ? How it can be combated ?
c) Why cells are hexagonal ?
d) Discuss fixed channel assignment strategy with its pros and cons.
e) Derive an expression for traffic intensity per channel.
3. Attempt any two :

20

a) Explain wireless MAC Reservation ALOHA. What are its advantages ?


b) With suitable diagram explain different interfaces in different subsystems of
a GSM architecture.
c) Derive an expression for S/I ratio and N (cluster size). With suitable example
explain how this expression is used by a design engineer to plan cellular
architecture.
Set A

SLR-BB 152

-4-

*SLRBB152*

SECTION II
4. Attempt any four :

20

a) What are the requirements for a Mobile IP ?


b) With suitable example explain concept of Piconet and Scatternet.
c) What are salient features of HiperLAN2 ?
d) Describe ad-hoc architecture for WLAN.
e) What is tunneling ? Why it is required ?
5. Attempt any two :

20

a) Explain IEEE 802.11 protocol architecture in detail with scope.


b) Draw protocol stack and explain each component in brief for Bluetooth.
c) Draw and explain a typical WATM scenario Mobile ATM terminals and
switches.
_____________________

Set A

SLR-BB 153

*SLRBB153*
S

B.E. (Electronics Engineering) (Part I) Examination, 2014


ELECTRONIC INSTRUMENTATION
Day and Date : Tuesday, 9-12-2014
Time : 3.00 p.m. to 6.00 p.m.
Instructions :

Max. Marks : 100

1) Figures to the right indicate full marks.


2) Assume suitable data if necessary.
3) Q. No. 1 is compulsory. It should be solved in first 30 minutes in
Answer Book Page No. 3. Each question carries one mark.
4) Answer MCQ/Objective type questions on Page No. 3 only. Dont
forget to mention, Q.P. Set (A/B/C/D) on Top of Page.

Duration : 30 Minutes

MCQ/Objective Type Questions

Marks : 20

1. Select suitable option :

(201=20)

1) Which of the below is not a dynamic characteristic of an instrument ?


a) Speed of response
b) Fidelity
c) Lag
d) Sensitivity
2) Environmental errors are a type of __________
a) Gross error
b) Systematic error
c) Random error
d) None
3) Nixie tube is an example of __________
a) Digital display
c) Console display

b) Analog display
d) Both a) and b)

4) Semiconductor wafer is used in construction of __________


a) LCD
b) LED
c) CRT

d) None

5) The consistency in measurement is called


a) accuracy
b) resolution
c) precision

d) expected value

6) A voltage controlled oscillator is an instrument


a) whose frequency is dependent upon amplitude of input signal
b) whose frequency is independent of amplitude of input signal
c) whose frequency is dependent upon frequency of input signal
d) none
7) A random noise generator produces a signal
a) whose amplitude varies randomly
b) which has no periodic frequency
c) has an unpredictable power spectrum d) all of above
8) ____________ type of ohmmeter is used to measure law value resistance.
a) Series
b) Shunt
c) Both a) and b)
d) None
9) Intermodulation distortion
a) occurs due to nonlinearity of device
c) is same as signal top noise ratio

b) is same as harmonic distortion


d) cannot be measured
P.T.O.

SLR-BB 153

*SLRBB153*

-2-

10) A pulse with 25% duty cycle has ON time of 30


a) 95


sec

b) 90


sec

c) 85

sec. Its OFF time is




sec

d) 80


sec

11) A video frequency oscillator has a frequency range of


a) 20 Hz to 20 KHz
b) 20 KHz to 30 MHz
c) d.c. to 5 MHz
d) 2 MHz to 30 MHz
12) A digital generator does not require
a) D/A converter
b) clock

c) memory

d) address generator

13) To design a single pulse generator, we can use


a) astable multivibrator
b) bistable multivibrator
c) mono multivibrator
d) none
14) A filter bank spectrum analyzer with large frequency range and good resolution requires
a) only one filter and one detector
b) a large quantity of hardware
c) a large number of filters but only a single detector
d) a large number of detectors but only a single filter
15) The best analyzer for the vibration analysis is
a) spectrum analyzer
b) vector analyzer
c) FFT analyzer
d) DSO
16) Period measurement provides
a) low resolution at low signal frequencies
b) high resolution at low signal frequencies
c) measurements at only high signal frequencies
d) none
17) The purpose of ISO 9000 is to
a) harmonize a large number of international standards
b) install and maintain a quality system to strengthen competitiveness
c) establish quality system standards in place of product standards
d) all of the above
18) Standards specifically developed for testing and calibration are represented by
a) ISO 10011
b) ISO 17025
c) ISO 8402
d) ISO 10013
19) The goal of quality assurance of an industry is to develop its image and reputation with
regard to
a) product quality and reliability
b) price and delivery
c) customer care
d) all of the above
20) Quality planning covers
a) product planning
b) managerial and operation planning
c) preparation of quality plans and provisions for quality improvement
d) all of the above
______________

Set A

*SLRBB153*
S

SLR-BB 153

-3-

B.E. (Electronics Engineering) (Part I) Examination, 2014


ELECTRONIC INSTRUMENTATION
Day and Date : Tuesday, 9-12-2014

Marks : 80

Time : 3.00 p.m. to 6.00 p.m.


Instructions : 1) All questions are compulsory.
2) Figures to the right indicates full marks.
3) Assume suitable data if necessary.
SECTION I
2. Attempt any four :

(45=20)

a) Explain gas discharge plasma display with neat diagram.


b) Derive equation for shunt resistor used to design DC Ammeter. List disadvantages of
Aryton arrangement.
c) List and explain any five pulse parameters.
d) A 1 mA meter movement with an internal resistance of 50


is to be used in a 0 1V,

0 10V, 0 50V and 0 100V ranges in the arrangement of multirange voltmeter. Find
the value of required multiplier resistances.
e) Define error and derive equation for % error, % accuracy and precision.
3. a) Explain working of programmable decade indirect synthesizer with respect to resolution
of synthesized frequency.

10

b) With functional block diagram, explain data generators and its any five key characteristics. 10
OR
c) Give designing steps for series type ohmmeter. Derive equation for half scale deflection
current, R1 and R2. For a series type ohmmeter, a 1 mA meter movement with an internal
resistance of 50
be for 2500


is to be used. The battery voltage is 3V. Half scale deflection should

. Calculate values of R1 and R2.

10
Set A

SLR-BB 153

-4-

*SLRBB153*

SECTION II
4. Attempt any four :

(45=20)

a) Write short note on quality audits.


b) Draw the block diagram of SMPTE intermodulation analyser and explain its operation and
various features.
c) How is the target systems interfaced with the logic analyzer ?
d) Give relevant applications of DAS in detail.
e) Differentiate between logic analyzer and spectrum analyzer.
5. a) Explain the construction and operation of superheterodyne spectrum analyzer. Also
explain the features resolution, frequency stability and dynamic range.
b) Explain general block diagram of DAS. Why signal conditioning of inputs is necessary.

10
10

OR
c) What is quality ? Why quality management is required ? Give an overview of ISO 9000
quality management system.

10
_____________________

Set A

SLR-BB 154

*SLRBB154*
Seat
No.

Set

B.E. (Electronics Engineering) (Part I) Examination, 2014


MECHATRONICS (Elective I)
Day and Date : Thursday, 11-12-2014
Time : 3.00 p.m. to 6.00 p.m.

Max. Marks : 100

Instructions : 1) Figures of right hand side indicate maximum marks.


2) Q. No. 1 is compulsory. It should be solved in first 30 minutes
in Answer Book Page No. 3. Each question carries one mark.
3) Answer MCQ/Objective type questions on Page No. 3 only.
Dont forget to mention, Q.P. Set (A/B/C/D) on Top of Page.
MCQ/Objective Type Questions
Duration : 30 Minutes

Marks : 20

1. Choose the correct answer :

20

1) For electric traction which drive is most suitable ?


a) Two quadrant b) Four quadrant c) Single quadrant d) None of these
2) Actuators are interfaced with which card of PLC ?
a) Memory card
b) Output card c) Input card
d) Power supply
3) Which sensor is self generating type ?
a) Thermocouple b) RTD
c) PT100

d) Thermostat

4) Execution time of PLC depends on


a) Memory size
b) Ladder size
c) Power supply used for PLC
d) Number of I/O
5) The number of I/Os for micro PLC are
a) More than 8
b) Less than 4 c) More than 16 d) More than 32
6) The variable frequency drive is suitable for
a) DC servo motor
b) DC series motor
c) AC motor
d) DC shunt motor
7) The device which provides maximum isolation is
a) Pulse transformer
b) Normal transformer
c) Opto isolator
d) None of the above
8) In PID which parameter is responsible for Oscillation
a) KP
b) KD
c) KI
d) PO
9) The basic element of ON/OFF controller is
a) Amplifier
b) Comparator c) Oscillator

d) Differential amplifier
P.T.O.

SLR-BB 154

-2-

*SLRBB154*

10) In PID which parameter is responsible for maximum overshoot.


a) KP
b) KD
c) KI
d) PO
11) Proportional band of a controller is defined as the range of
a) Measured variable to the set variable
b) Air output as the measured variable varies from maximum to minimum
c) Measured variables through which the air output varies from maximum to
minimum
d) None of the above
12) Proportional band of the controller is expressed as
a) Gain
b) Ratio
c) Percentage d) Range of control variables
13) The term reset control refers to
a) Proportional
b) Integral
c) Derivative
d) None of the above
14) The integral control
a) Increases the steady state error
c) Increases the noise and stability

b) Decreases the steady state error


d) Decreases the damping coefficient

15) In a proportional temperature controller, if the quantity under the heater


increases the offset will
a) Increase
b) Reduce
c) Remain un effected
d) None of the above
16) When derivative action is included in a proportional controller, the proportional
band ?
a) Increases
b) Reduces
c) Remains unchanged
d) None of the above
17) The number of operational amplifiers require for designing of electronic PID
controller is
a) 1
b) 2
c) 3
d) 6
18) Which of the following system provides excellent transient and steady state
response ?
a) Proportional action
b) Proportional + Integral action
c) Proportional + Differential action d) Proportional + Integral + Differential action
19) In a PID controller, the offset has increased. The integral time constant has
to be _________ so as to reduce offset.
a) Reduced
b) Increased c) Reduced to zero
d) None of the above
20) In a PID controller, the overshoots has increased. The derivative time constant
has to be _________ so as to reduce the overshoots.
a) Increased
b) Reduced
c) Reduced to zero
d) None of the above
______________
Set A

*SLRBB154*

-3-

SLR-BB 154

Seat
No.

B.E. (Electronics Engineering) (Part I) Examination, 2014


MECHATRONICS (Elective I)
Day and Date : Thursday, 11-12-2014
Time : 3.00 p.m. to 6.00 p.m.

Marks : 80

Instructions : i) All questions are compulsory.


ii) Figures of right hand side indicate maximum marks.
SECTION I
2. Solve any three :

15

a) Draw neat diagram of PI controller. Explain each block in detail.


b) What are different methods of breaking systems ? Explain one with suitable
diagram and waveforms.
c) Distinguish between microprocessor and microcontroller based systems.
d) What are different blocks of PLC ? How output module of PLC works ?
3. Solve any two :

15

a) Explain with neat diagram How Hydraulic Power Systems Works ?


b) What is bipolar drive ? Explain working with neat diagram.
c) How SCR based drive works ? Explain with neat diagram and waveform.
4. Design analog proportional derivative controller for following specification.

10

i) Process variable range 0 to 600C


ii) Set point 200C to 250C
iii) Use R type thermocouple with sensitivity 40 micro V/C
iv) Proportional band 50%
v) Derivative constant 2.5
vi) Assume suitable data.
Set A

SLR-BB 154

-4-

*SLRBB154*

SECTION II
5. Write short note any three :

15

a) Range Sensor.
b) Magnetic Gripper.
c) Pneumatic System.
d) Interfacing of sensors with system.
6. Solve any two :

15

a) How touch sensor is manufactured ? Explain application of it.


b) What is digital sensor ? Why it is preferred over other sensors ? Explain with
example.
c) Compare with suitable example MEMS and Conventional system.
7. How robotic system works ? Explain components required for it.

10

_____________________

Set A

SLR-BB 155

*SLRBB155*
Seat
No.

Set

B.E. (Electronics Engg.) (Part I) Examination, 2014


IMAGE PROCESSING (Elective I)
Day and Date : Thursday, 11-12-2014
Time : 3.00 p.m. to 6.00 p.m.

Max. Marks : 100

Instructions : 1) Figures to the right indicates full marks.


2) Q. No. 1 is compulsory. It should be solved in first 30 minutes
in Answer Book Page No. 3. Each question carries one mark.
3) Answer MCQ/Objective type questions on Page No. 3 only.
Dont forget to mention, Q.P. Set (A/B/C/D) on Top of Page.
MCQ/Objective Type Questions
Duration : 30 Minutes

Marks : 20

1. Choose the correct answer :

(201=20)

1) The range of values spanned by the gray scale is called as ________ of an image.
a) Resolution
b) Histogram
c) Dynamic range
d) Contrast
2) The Euclidean distance between pixels p(x, y) and q(s, t) is defined as
1
a) | x s | + | y t|
b) [ (x s)2 + (y t)2 ] 2
c) | x + s | + | y + t|
d) [(x s)2 + (y t)2]
3) The output response of smoothing linear spatial filter is
a) Average of pixels in the neighborhood of a filter mask
b) Sum of pixels in the neighborhood of a filter mask
c) Median of pixels in the neighborhood of a filter mask
d) None of above
4) Which of the following is not an order statistic filter ?
a) Median
b) Mean
c) Max
5) Low frequencies in the DFT of an images are related to
a) Dark region in an image
b) Edges, noise present in an image
c) Sharp transitions in an intensity
d) Slowly varying intensity components in an image

d) Min

6) Ringing becomes significant in butterworth low pass filters of


a) Order 1
b) Order 2
c) Higher order
d) None of these
7) The salt and pepper noise is also known as _____________ noise.
a) Black and white
b) White and black
c) Impulse
d) Uniform
P.T.O.

SLR-BB 155

-2-

*SLRBB155*

8) An image of size 1024 1024 pixels in which the intensity of each pixel is an
8 bit quantity requires the storage space (if not compressed)
a) 1 KB
b) 1 MB
c) 2 KB
d) 2 MB
9) A Hotelling transform is also referred as
a) Eigenvector
b) Principal component
c) Discrete KL transform
d) All of these
10) Which of the following transform is separable and symmetric ?
a) DFT
b) DCT
c) Walsh
d) All of these a), b) and c)
11) ___________ are used to represent a boundary connected by a connected
sequence of straight line segments of specified length and direction.
a) Signatures
b) Skeletons
c) Chain codes
d) Boundary descriptors
12) Signatures can be normalized with respect to rotation by
a) Taking the first difference
b) Finding the variance of the region
c) Taking the same starting point to generate the signature
d) Computing the points equal in distance from centroid
13) Which of the following is useful descriptor of a boundary ?
a) Length of a boundary
b) Diameter of a boundary
c) Major axis of a boundary
d) All of these a), b) and c)
14) The shape of the boundary segments can be described quantitatively by using
a) Mean
b) Variance
c) Higher order moments
d) All of these a), b) and c)
15) Which of the following must be true for second derivative ?
a) Must be zero at the onset and end of intensity step
b) Must be nonzero along intensity ramp
c) Must be zero in the areas of constant intensity
d) All of these a), b) and c)
16) The sum of the coefficients of the derivative mask is
a) Zero
b) Positive
c) Negative
d) Nonzero
17) ____________ is the measure of information content in an image.
a) Entropy
b) Redundancy c) Probability
d) None of these
18) To remove the correlated image data which of the following image transform is used.
a) DCT
b) DFT
c) K L transform
d) Walsh
19) Hough transform is __________processing technique used for edge linking.
a) Local
b) Global
c) Point
d) Histogram
20) The selection of similarity criteria used in region growing depends upon
a) Problem under consideration
b) Type of image data
c) Both a) and b)
d) None of these
Set A
______________

*SLRBB155*

-3-

SLR-BB 155

Seat
No.

B.E. (Electronics Engg.) (Part I) Examination, 2014


IMAGE PROCESSING (Elective I)
Day and Date : Thursday, 11-12-2014
Time : 3.00 p.m. to 6.00 p.m.

Marks : 80

Instructions : 1) All questions are compulsory.


2) Assume suitable data if necessary.
3) Figures to the right indicates full marks.
SECTION I
2. Attempt any four :

(46=24)

a) Explain the meaning of bit plane slicing. Which of the bit planes contribute to
the subtle details of the image ?
b) Give the conditions under which the D4 distance between two pixels p and q
is equal to the shortest 4 path between these two points. Is this path unique ?
c) What are the sources of noise in image ? Give PDF along with sketches for
following :
I) Uniform noise
II) Gaussian noise
III) Exponential noise
d) What is Walsh transform ? Explain along with its properties.
e) Explain in how image smoothing is obtained in frequency domain.
f) Explain image enhancement using arithmetic and logical operations in brief,
3. Attempt any two :

(28=16)

a) Explain the following two noise removal techniques :


I) Neighbourhood averaging
II) Median filtering

Set A

SLR-BB 155

-4-

*SLRBB155*

Calculate the value of shaded pixel of the following image segment when the
above techniques are applied with 33 neighborhoods.

b) Explain geometric transformations. How these are useful for image registration ?
c) What is the discrete cosine transform ? Give 2 D kernels. Explain its
properties.
SECTION II
4. Attempt any four :

(46=24)

a) What is the difference between an isolated point, line and an edge ?


b) Explain the thresholding approach of segmentation of an image.
c) Explain entropy and its relation with image compression.
d) Show that the first difference of a chain code normalizes it to the rotation.
Compute the first difference of the code 01011022232331.
e) Write short note on Laplacian of Gaussian.
f) Write a note on :Vector Quantization.
5. Attempt any two :

(28=16)

a) Explain the procedures for following region based segmentation techniques.


I) Region growing
II) Region splitting and merging
b) Find an expression for signature of following boundaries and plot the signatures.
I) Circle

II) Square

c) Explain in detail how statistical moments are useful in describing the shape of
the boundary.
_____________________

Set A

SLR-BB 156

*SLRBB156*
Seat
No.

Set

B.E. (Electronics Engineering) (Part II) Examination, 2014


ADVANCED COMMUNICATION ENGINEERING
Day and Date : Tuesday, 25-11-2014
Time : 3.00 p.m. to 6.00 p.m.

Max. Marks : 100

Instructions : 1) Figures to the right indicates full marks.


2) Assume suitable data if necessary.
3) Q. No. 1 is compulsory. It should be solved in first 30 minutes
in Answer book Page No. 3. Each question carries one mark.
4) Answer MCQ/Objective type questions on Page No. 3 only.
Dont forget to mention, Q.P. Set (A/B/C/D) on Top of Page.
Duration : 30 Minutes

MCQ/Objective Type Questions

Marks : 20

1. Select suitable option :

20

1) The TEM wave are propagated equally in all the direction if the source is a
a) Line source
b) Point source c) Plane source d) None
2) E of an EM wave is in the vertical direction then the EM wave is said to
a) Vertically polarized
b) Horizontally polarized
c) Both a) and b)
d) None of these
3) Non directivity of directional coupler is measure of ______________
a) how much incident power is being sampled
b) how well directional coupler distinguishes between forward and reverse
travelling power
c) both a) and b)
d) none
4) In TE both electric and magnetic field are
a) E = 0 H = 0
b) E = 0 H 0 c) E 0 H = 0 d) E 0 H 0
5) In E-plane Tee port 1 and port 2 are the ______________ and port 3 is the
______________ respectively.
a) E-arm and collinear arms
b) Collinear arms and H-arm
c) Collinear arms and E-arm
d) Both a) and c)
6) Which of the following is application of PIN diode ?
a) PIN diode as a switch
b) As a phase shifter
c) As an amplitude modulator
d) All the above
7) Simple radar systems consist of transmitter and receiver, such a radar system
called ______________
a) Monostotic radar b) Pulse radar
c) Bistatic radar d) MTI radar
P.T.O.

SLR-BB 156

-2-

*SLRBB156*

8) ______________ is a example of transferred electronic device.


a) Gunn Diode
b) PIN Diode
c) IMPATT Diode d) TRAPATT Diode
9) In which of the following the part of satellite subsystem make a help to satellite
to move in orbit ?
a) Frequencies translator
b) Propulsion subsystem
c) Attitude control subsystem
d) Communication subsystem
10) In which of the following refers to satellite service that can be receive at
many unspecified location by relatively simple receiver only earth station ?
a) FSS
b) BSS
c) MSS
d) None
11) In which of the following is transponder used for communication ?
a) Single conversion
b) On-board controller
c) Double conversion
d) All
12) When transponders are required for short duration or occasionally then we use ?
a) DAMA
b) RMA
c) Both
d) None
13) In which of the following help for the determination of current orbit and
position of spacecraft take place ?
a) tracking
b) command system c) telemetry system d) none
14) Transponder converts ___________ frequency to __________ frequency.
a) Uplink, downlink b) Downlink, uplink c) Both a) and b) d) None of these
15) In a circular orbit the speed if revolution is ___________ while in elliptical
orbit it depends on ___________ of the satellite above he earth.
a) Variable, height b) Constant, width c) Constant, height d) Increases, height
16) MTI stands for
a) Moving Traffic Indicator
b) Mobile Target Indicator
c) Mobile Traffic Indicator
d) Moving Target Indicator
17) The time taken for a satellite to complete one orbit is called ___________ period.
a) Perigee
b) Apogee
c) Sideread
d) None of these
18) Waveguides are pressuised above normal atmospheric pressure for
a) increasing their power handling capacity
b) improving the conductivity of their walls
c) preventing higher order modes from propagating
d) varying the wave impedance
19) The angle b/w incident ray and normal to the plane is ___________
a) Angle of reflection
b) Angle of incident
c) Angle of coefficient
d) None of these
20) In second generation, wavelength of multimode fiber is ___________
a) 850 nm
b) 1310 nm
c) 1420 nm
d) 1550 nm
______________
Set A

*SLRBB156*

-3-

SLR-BB 156

Seat
No.

B.E. (Electronics Engineering) (Part II) Examination, 2014


ADVANCED COMMUNICATION ENGINEERING
Day and Date : Tuesday, 25-11-2014

Marks : 80

Time : 3.00 p.m. to 6.00 p.m.


Instructions : 1) All questions are compulsory.
2) Figures to the right indicates full marks.
3) Assume suitable data if necessary.
SECTION I
2. Attempt any three :

(38=24)

a) Explain construction and working of TWT. State its performance parameters.


b) Derive Radar range equation and explain factors effecting range of Radar.
c) Explain construction and working of -mode magnetron. State its performance
parameters.
d) Derive the wave equation for TE mode and obtain all the field components in
a rectangular waveguide.
3. Attempt any four :

(44=16)

a) With suitable diagram explain working of two hole directional coupler.


b) Compare CW Doppler radar and FM-CW radar.
c) Write a note on Radar Beacons.
d) Explain construction and working of Gunn diode.
e) Explain construction and working of Tunnel diode.
Set A

SLR-BB 156

-4-

*SLRBB156*

SECTION II
4. Attempt any three :

(38=24)

a) With suitable block diagrams explain Satellite subsystem.


b) What is system noise temperature ? How does it affect the C/N and G/T ratio ?
c) Explain construction and working of surface emitting LED.
d) Explain construction and working of Avalanche photo diode.
5. Attempt any four :

(44=16)

a) Explain perigee and apogee related to Satellite.


b) Draw and explain different types of optical fiber.
c) Write a note on DBS-TV uplink earth station.
d) A satellite is an elliptical orbit with a perigee of 1000 Km and apogee of
4000 Km. Using a mean earth of 6378.14 Km, find the period of the orbit in
hours, minutes, seconds and eccentricity of the orbit.
e) Derive numerical aperture for step index fiber.
_____________________

Set A

SLR-BB 157

*SLRBB157*
Seat
No.

Set

B.E. (Electronics Engineering) (Part II) Examination, 2014


AUDIO VIDEO ENGINEERING
Day and Date : Thursday, 27-11-2014
Time : 3.00 p.m. to 6.00 p.m.

Max. Marks : 100

Instructions : 1)
2)
3)
4)

All questions are compulsory.


Figures to the right indicates full marks.
Assume suitable data if necessary.
Q. No. 1 is compulsory. It should be solved in first 30
minutes in Answer book Page No. 3. Each question carries
one mark.
5) Answer MCQ/Objective type questions on Page No. 3 only.
Dont forget to mention, Q.P. Set (A/B/C/D) on Top of Page.
MCQ/Objective Type Questions

Duration : 30 Minutes

Marks : 20

1. Select suitable option :


1) Wow and flutter pertains to variation in __________
a) tape speed
b) a.c. bias
c) head gap

20
d) none

2) In lateral grooves, depth of the groove.


a) varies
b) remains constant
c) a) and b) both
d) none
3) The electronic tuner used in TV receiver uses __________
a) silicon diode
b) varactor diode
c) germanium diode
d) high switching diode
4) The typical input impedance of TV receiver is
b) 75
c) 175
a) 50

d) 300

5) Addition of two complementary equal intensity colours in television would


appear to human eye as
a) White
b) Black
c) Unsaturated white or black
d) Grey
6) Automatic Frequency Control (AFC) is required in ________ section.
a) Vertical
b) Horizontal
c) Both a) and b) d) None
7) Asper CCIR-B standard horizontal sweep oscillator frequency is equal to
a) 50 Hz
b) 15625 Hz
c) 25 Hz
d) 240 Hz

P.T.O.

SLR-BB 157

*SLRBB157*

-2-

8) Highest video frequency of scanning, for PAL system is


a) 625 MHz
b) 8 MHz
c) 4.75 MHz
d) 5 MHz
9) Process of demagnetizing then iron and steel parts of picture tube mounting
is known as
a) Convergence b) Purity
c) Degaussing
d) Pincushion
10) A colour sub-carrier is so chosen that,
a) It is the multiple of line frequency
b) It is an odd multiple of the half line frequency
c) It gives the least annoying interference dot pattern
d) b) and c)
11) In PAL system, phase of R-Y signal is changed every alternate line by ______
a) 90
b) 180
c) 270
d) 360
12) _______ polar modulation is used in PAL system.
a) AM
b) FM
c) Positive
13) Chroma signals in PAL consist of
a) I and Q signals
c) U and V signals

d) Negative

b) R-Y and B-Y signals


d) R and B signals

14) The two new signals generated in SECAM system are


a) (B-Y) and (R-Y) b) R and B
c) DB and DR

d) U and V

15) The aspect ratio used in HDTV system is


a) 4:3
b) 3:4
c) 1:2

d) 16:9

16) The type of AGC used in TV receiver is


a) Simple AGC
b) Delayed AGC c) Keyed AGC

d) b) and c) both

17) The problem of _______ errors has been successfully overcome in the PAL
system.
a) Contrast
b) Brightness
c) Colour
d) Differential phase
18) Slotted aperture mask is used in
a) Trintron
c) Shadow mask tube

b) PIL tubes
d) a) and b)

19) Ident pulses are generated in _______ system.


a) NTSC
b) PAL
c) SECAM

d) Both a) and b)

20) A trintron employs


a) Three in-line cathode beams
b) A common electron gun
c) Three separate electron guns
d) a) and b)
______________

Set A

*SLRBB157*

-3-

SLR-BB 157

Seat
No.

B.E. (Electronics Engineering) (Part II) Examination, 2014


AUDIO VIDEO ENGINEERING
Day and Date : Thursday, 27-11-2014
Time : 3.00 p.m. to 6.00 p.m.

Marks : 80

Instructions : 1) All questions are compulsory.


2) Figures to the right indicates full marks.
3) Assume suitable data if necessary.
SECTION I
2. Attempt any four :

(44=16)

a) Compare coarse-grooves and micro grooves.


b) Define multimedia. What are its elements ? What are its applications ?
c) What is interlaced scanning ? What are its advantages ?
d) With neat sketch, give details of video composite signal.
e) Why the colour difference signal (G-Y) is not suitable for transmission.
3. Solve any three :

(38=24)

a) Describe with the help of a neat diagram, tape transport mechanism. Explain
the function of capstan, pinch roller, tape guides and spools.
b) Explain compression technique for audio. List objectives of MPEG.
c) Explain with neat diagram working principle of I.O. camera tube. List the
characteristics of camera tube.
d) With block diagram explain working of low level transmitter used for television
transmission. What are its advantages ?
SECTION II
4. Attempt any four :

(44=16)

a) With circuit diagram explain working of degaussing coil used with picture
tube.
b) Compare delta gun and PIL picture tubes.

Set A

SLR-BB 157

-4-

*SLRBB157*

c) What are the drawbacks of NTSC system ? How those are removed in PAL
system ?
d) What is AGC ? What are different types of AGC ?
e) Explain working of video detector.
5. Attempt any three :

(38=24)

a) Explain the concept of frequency interleaving. How it is used to transmit


colour information in television system ?
b) With block diagram, explain in detail satellite TV system.
c) Explain with neat diagram each block of NTSC coder.
d) Give elements of cable TV network, explain block converter used in cable
TV.
_____________________

Set A

SLR-BB 158

*SLRBB158*
Seat
No.

Set

B.E. (Electronics) (Part II) Examination, 2014


EMBEDDED SYSTEM
Day and Date : Saturday, 29-11-2014
Time : 3.00 p.m. to 6.00 p.m.

Total Marks : 100

Instructions : 1) Figures to the right indicate full marks.


2) Assume suitable data whenever necessary.
3) Q. No. 1 is compulsory. It should be solved in first 30 minutes
in Answer book Page No. 3. Each question carries one mark.
4) Answer MCQ/Objective type questions on Page No. 3 only.
Dont forget to mention, Q.P. Set (A/B/C/D) on Top of Page.
MCQ/Objective Type Questions
Duration : 30 Minutes
1. Choose the correct answer :

Marks : 20
(201=20)

1) ______________ mode is used when the processor encounters an instruction


that is not supported by the implementation.
a) Supervisor
b) System
c) Undefined
d) Abort
2) The ARM register______________is used as stack pointer.
a) R12
b) R13
c) R14
d) R15
3) One important feature of the ARM is that input register can be preprocessed
in the ______________ before it enters ALU.
a) Address register
b) Barrel Shifter
c) Incrementer
d) None of the above
4) A ______________ processor has large number of general purpose registers.
a) CICS
b) RISC
c) Both a) and b)
d) None of the above
5) Let r0 = 002020202 and r1 = 000009000. What will be the content of r1
after execution of instruction LDR r0, [r1], #4 ?
a) 000009000
b) 002020202 c) 000009004
d) 000008ffb
6) ______________ exception is having the highest priority.
a) Reset
b) Supervisor
c) System
d) Interrupt request
7) In ARM7TDMI-S D stands for ______________
a) Debug
b) Divider
c) Difference
d) None of the above
P.T.O.

SLR-BB 158

-2-

*SLRBB158*

8) LPC2148 has on-chip___________KB flash memory and____________ KB SRAM.


a) 30,16
b) 128,32
c) 512,40
d) 256,40
9) The on-chip ADCs of LPC2148 has_______bit resolution and ________ conversion
time.
b) 10, 2.44 sec
a) 8, 2.44 sec
c) 12,1.44 sec
d) 10,1.44 sec
10) In LPC2148 _____________ pin select register is used to configure port pins
P0.0 to P0.15.
a) PINSEL1
b) PINSEL0
c) IODIR2
d) IOSET0
11) Information about a task is maintained in a _____________
a) Stack
b) Translation look aside buffer
c) Task control block
d) Task condition block
12) In _____________OS, the response time is very critical.
a) Multitasking
b) Batch
c) On-line
d) Real-time
13) Inter task communication can be done through _____________
a) Mailboxes
b) Queues
c) Pipes
d) All of above
14) I2C master can address_____________ other slaves at an instance.
a) 7
b) 11
c) 127
d) None of the above
15) The CAN protocol has the _____________ feature/s.
a) Multi-master
b) Message transmission
c) Communication speed
d) All of above
16) SDA and SCL signal lines are used in _____________communication.
a) SPI
b) CAN
c) MODBUS
d) I2C
17) The SPI bus can operate with a _____________ master device/s and with
_____________ slave device/s.
a) Single, one
b) Single, One or More
c) Two, Single
d) Two, Two
18) _____________ is a serial protocol used by ARM to send and receive debug
information between the processor core and test equipment.
a) Jazelle
b) JTAG
c) EmbeddedICE Macrocell
d) I2C
19) In LPC2148 slot 15 has _____________ priority.
a) Highest
b) Lowest
c) Middle
d) None of the above
20) The vector address of FIQ interrupt is_____________
a) 00000001C
b) 000000010 c) 000000018
______________

d) 000000014
Set A

*SLRBB158*

-3-

SLR-BB 158

Seat
No.

B.E. (Electronics) (Part II) Examination, 2014


EMBEDDED SYSTEM
Day and Date : Saturday, 29-11-2014

Marks : 80

Time : 3.00 p.m. to 6.00 p.m.


Instructions : 1) Figures to the right indicate full marks.
2) Assume suitable data whenever necessary.
SECTION I
2. Solve any four :

(46=24)

a) Explain LDM and STM instruction in detail with example.


b) Explain processor selection for an embedded system.
c) Write an ARM assembly program to add two 64 bit numbers.
d) Discuss recent trends in embedded systems.
e) Define embedded system. Explain its characteristics.
3. Solve any two :

(28=16)

a) Explain ARM programmers model in detail (Operating modes, Register model,


Program status registers, data types etc.)
b) List the various application areas of embedded systems and give examples
for each application area.
c) Explain how ARM responds to the exceptions with the vector table.

Set A

SLR-BB 158

-4-

*SLRBB158*

SECTION II
4. Solve any four :

(46=24)

a) Draw and explain interfacing of LCD with LPC2148.


b) Explain mailbox and pipe in RTOS.
c) Explain different addressing modes of ARM7 in detail.
d) Draw and explain bluetooth protocol stack.
e) Explain embedded communication using I2C.
5. Solve any two :

(28=16)

a) Interface 8 LEDs to LPC2148 port pins P0.2 to P0.9. Write an embedded C


program to blink them alternately with a neat circuit diagram.
b) Explain embedded system used in digital camera.
c) Explain round robin scheduling with example.

_____________________

Set A

SLR-BB 159

*SLRBB159*
Seat
No.

Set

B.E. (Electronics Engineering) (Part II) Examination, 2014


BROADBAND COMMUNICATION (Elective II)
Day and Date : Monday, 1-12-2014
Time : 3.00 p.m. to 6.00 p.m.

Max. Marks : 100

Instructions : 1) Figures to the right indicate full marks.


2) Assume suitable data if necessary.
3) Q. No. 1 is compulsory. It should be solved in first 30 minutes
in Answer book Page No. 3. Each question carries one mark.
4) Answer MCQ/Objective type questions on Page No. 3 only.
Dont forget to mention, Q.P. Set (A/B/C/D) on Top of Page.
Duration : 30 Minutes

MCQ/Objective Type Questions

Marks : 20

1. Choose the correct answer :


20
1) X.25, it was developed for computer connections used for
a) Timesharing connection
b) Terminal connection
c) Both a) and b)
d) None of these
2) In X.25 DTEs are not required to use the same line speed because of the
a) Storage in packet networks
b) Forward nature of packet switching
c) Excellent flow control
d) All of these
3) To enable control of 4096 logical channels in X.25, there are __________
channel groups.
a) 256
b) 16
c) 4096
d) 2556
4) X.25 protocol exchanged the data control information between
a) A node
b) A user device c) Both a) and b) d) None of these
5) The advantage of X.25 are
a) Was developed to recover errors
b) Packet switching eases compatibility problems in communications between
PCs
c) Packet switching cannot waste bandwidth
d) All of these
6) The device which splits data into frames as well as combines frames into
data is referred as
a) FRAD (Frame relay and disassembly)
b) FDLC
c) HDLC
d) DLCI
P.T.O.

SLR-BB 159

-2-

*SLRBB159*

7) BRA affords an ISDN user with simultaneous access to two __________


data channels.
a) 32 kbps
b) 64 kbps
c) 128 kbps
d) 256 kbps
8) ISDN connections may be seen as very __________ digital conduits.
a) High rate-of-error
b) Low rate-of-error
c) Both a) and b)
d) None of these
9) The total transmission rate of BRA workout to a combined total of
a) 192 kbit/s
b) 144 kbit/s
c) 146 kbit/s
d) 147 kbit/s
10) Equipment that controls the physical and electrical termination of ISDN at
the users premises is called _____________
a) NT1
b) NT2
c) NT3
d) NT4
11) Advantages of cell relay are
a) High-speed transmission
b) Multiplexing transmission
c) Both a) and b)
d) None of these
12) The part that corresponds to second layer in ATM is referred to as
a) DLC layer
b) ATM layer
c) STM layer
d) Protocol layer
13) VP is a bundle of
a) VCs
b) VCM
c) VCI
d) VIP
14) ATM is an
a) International Telecommunication
b) International Telecommunication-Union
c) International Telecommunication Union Telecommunication
Standardization Sector (ITU-T)
d) International Telecommunication Union Telecommunication
15) Data traffic in ATM tend to be
a) Continuous
b) Bursty
c) Discontinuous d) None of these
16) ATM cells coming from a user are guaranteed delivery at the other end with a
a) High probability b) Low delay
c) Both a) and b) d) None of these
17) The _________ service is suitable for customer who need real time video
transmission.
a) CBR
b) VBR
c) ABR
d) UBR
18) Which AAL type can best process a data stream having non-constant bit rate ?
a) AAL1
b) AAL2
c) AAL3/4
d) AAL5
19) In B-ISDN, when information is obtained from public centre, the service is
__________ service.
a) Conversational b) Messaging
c) Retrieval
d) Distributive
20) The optical link between STS multiplexer and regenerator is called _________
a) A section
b) A line
c) A path
d) None of above
______________
Set A

*SLRBB159*

-3-

SLR-BB 159

Seat
No.

B.E. (Electronics Engineering) (Part II) Examination, 2014


BROADBAND COMMUNICATION (Elective II)
Day and Date : Monday, 1-12-2014
Time : 3.00 p.m. to 6.00 p.m.

Marks : 80

Instructions : 1) Figures to the right indicate full marks.


2) Assume suitable data if necessary.
SECTION I
2. Attempt any four :

(45=20)

1) What is in-channel and out of channel control signaling ?


2) What is the maximum packet size supported by X.25 standard ? Can we
modify the packet size in middle of transmission ? Draw a packet format of
X.25 packet and explain in short.
3) What is intelligent network ? Explain various ISDN channel structures.
4) Compare between X.25 and frame relay. What is frame switching ? Define
various frame relay parameters.
5) Draw ISDN addressing structure. Explain multiplexing in ISDN.
3. Attempt any two :

(210=20)

1) Discuss LAPF core formats. How congestion is controlled in frame relay ?


Discuss congestion mechanism in detail.
2) Draw functional block diagram of UNI in ISDN. Which reference points are
used in ISDN ? Explain their significance.
3) Explain various broadband services.
Set A

SLR-BB 159

-4-

*SLRBB159*

SECTION II
4. Attempt any four :

(45=20)

1) Draw and explain ATM protocol architecture.


2) How ATM cells are transmitted on physical channel ? Discuss cell delineation
state diagram related to ATM receiver.
3) Draw and explain SONET system hierarchy.
4) Explain the functions of switch interface, concentrator, multiplexer in ATM
switch.
5) Explain in detail BISDN architecture and BISDN user interface.
5. 1) Draw and explain UNI and NNI interface ATM cell formats. How cells received
with error are handled by destination ?
10
2) Why buffers are required in ATM switches ? Categorize ATM switch elements
based on the buffer positions. Also mention the performance of switch based
on buffer position.
10
OR
2) Explain AAL 3/4 protocol in detail.

10

_____________________

Set A

SLR-BB 160

*SLRBB160*
Seat
No.

Set

B.E. (Electronics Engineering) (Part II) Examination, 2014


PLC AND INDUSTRIAL CONTROLLERS (Elective II)
Day and Date : Monday, 1-12-2014
Time : 3.00 p.m. to 6.00 p.m.
Instructions :

Max. Marks : 100

i) Figures of right hand side indicate maximum marks.


ii) Q. No. 1 is compulsory. It should be solved in first 30 minutes
in Answer book Page No. 3. Each question carries one mark.
iii) Answer MCQ/Objective type questions on Page No. 3 only.
Dont forget to mention, Q.P. Set (A/B/C/D) on Top of Page.
MCQ/Objective Type Questions

Duration : 30 Minutes

Marks : 20

1. Choose the correct answer :

20

1) The word address in PLC not includes


a) SMPS
b) INPUT
c) OUTPUT

d) Rack number

2) The device which provides maximum isolation is


a) Pulse transformer
b) Normal transformer
c) Opto isolator
d) None of the above
3) In PID which parameter is responsible for oscillation ?
a) KP
b) KD
c) KI

d) PO

4) The basic element of ON/OFF controller is


a) Amplifier
b) Comparator
c) Oscillator

d) Differential amplifier

5) Which sensor is semiconductor sensor ?


a) RTD
b) PT 100
c) LM 335

d) Thermocouple

6) Proportional band of a controller is defined as the range of


a) Measured variable to the set variable
b) Air output as the measured variable varies from maximum to minimum
c) Measured variables through which the air output varies from maximum to
minimum
d) None of the above
7) Proportional band of the controller is expressed as
a) Gain
b) Ratio
c) Percentage
d) Range of control variables
8) The term reset control refers to
a) Proportional
b) Integral

c) Derivative

d) None of the above

P.T.O.

SLR-BB 160

*SLRBB160*

-2-

9) The integral control


a) Increases the steady state error
c) Increases the noise and stability

b) Decreases the steady state error


d) Decreases the damping coefficient

10) In a proportional temperature controller, if the quantity under the heater increases
the offset will
a) Increase
b) Reduce
c) Remain uneffected
d) None of the above
11) When derivative action is included in a proportional controller, the proportional
band
a) Increases
b) Reduces
c) Remains unchanged
d) None of the above
12) The number of operational amplifiers require for designing of electronic PID
controller is
a) 1
b) 2
c) 3
d) 6
13) Which of the following system provides excellent transient and steady state
response ?
a) Proportional action
b) Proportional + Integral action
c) Proportional + Differential action
d) Proportional + Integral + Differential action
14) In a PID controller, the offset has increased. The integral time constant has to be
___________ so as to reduce offset.
a) Reduced
b) Increased
c) Reduced to zero
d) None of the above
15) In a PID controller, the overshoots has increased. The derivative time constant
has to be __________ so as to reduce the overshoots.
a) Increased
b) Reduced
c) Reduced to zero
d) None of the above
16) For electric traction which drive is most suitable ?
a) Two quadrant
b) Four quadrant
c) Single quadrant
d) None of these
17) Actuators are interfaced with which card of PLC ?
a) Memory card
b) Output card
c) Input card

d) Power supply

18) Which sensor is self generating type ?


a) Thermocouple b) RTD

c) PT 100

d) Thermostat

19) Execution time of PLC depends on


a) Memory size
c) Power supply used for PLC

b) Ladder size
d) Number of I/O

20) The number of I/Os for micro PLC are


a) More than 8
b) Less than 4
c) More than 16
______________

d) More than 32
Set A

*SLRBB160*

-3-

SLR-BB 160

Seat
No.

B.E. (Electronics Engineering) (Part II) Examination, 2014


PLC AND INDUSTRIAL CONTROLLERS (Elective II)
Day and Date : Monday, 1-12-2014

Marks : 80

Time : 3.00 p.m. to 6.00 p.m.


Instructions : i) All questions are compulsory.
ii) Figures of right hand side indicate maximum marks.
SECTION I
2. Solve any three :

15

a) What is SCAN in PLC ? Explain with suitable diagram.


b) Draw and explain current to voltage converter for grounded input.
c) What are different components in PLC ?
d) State advantages of PLCs over hard wired relay.
3. Solve any two :

15

a) Develop physical and programmable ladder for induction motor control with
following interlocks
i) Motor should run when START PB is pushed
ii) Motor should run in reverse direction when limit switch (LS1) becomes NC
iii) Over current trip with thermal overload.
b) Develop and draw ladder diagram for washing machine.
c) What are the different specifications of PLC ? Explain two in detail.
4. Develop a ladder diagram for controlling an Air Conditioner from any of the three
locations.

10

Set A

SLR-BB 160

-4-

*SLRBB160*

SECTION II
5. Solve any three :

15

a) Elaborate the selection of grounded load converter and floating load converter.
b) Design converter for conversion of 0V to 5V into 4MA to 20MA using floating load.
c) Explain Data Acquisition System (DAS) using microcontroller.
d) Explain with neat sketch why the current range is 4MA to 20MA. Why it is started
from 4MA instead of ZERO.
6. Solve any two :

15

a) What are different components of robotic system ?


b) Explain different types of flow sensors.
c) Design 2 Channel Data Acquisition System with following specification :
Channel 1 :
i) Temp. Range 0 to 200C
ii) Sensor RTD-PT 100
Channel 2 :
i) Temp. Range 0 to 200C
ii) Sensor LM35
7. Design analog proportional derivative controller for following specification :

10

i) Process variable range 0 to 600C


ii) Set point 200C to 250C
iii) Use R type thermocouple with sensitivity 40 micro V/C
iv) Proportional band 50%
v) Derivative constant 2.5
vi) Assume suitable data.
_____________________

Set A

SLR-BB 161

*SLRBB161*
Seat
No.

Set

S.E. (E and TC) (Part I) Examination, 2014


ENGINEERING MATHEMATICS III
Day and Date : Tuesday, 9-12-2014
Time : 10.00 a.m. to 1.00 p.m.

Max. Marks : 100

N.B. : 1)
2)
3)
4)

Attempt any three questions from each Section.


Figures to right indicate full marks.
Use of calculator is allowed
Q. No. 1 is compulsory. It should be solved in first 30 minutes in Answer
Book Page No. 3. Each question carries one mark.
5) Answer MCQ/Objective type questions on Page No. 3 only. Dont forget to
mention, Q.P. Set (A/B/C/D) on Top of Page.
MCQ/Objective Type Questions

Duration : 30 Minutes

Marks : 20

1. Choose the correct answer :


1) The particular integral of (D3 3D2 + 4) y = e2x is _______________
a)

x 2x
e
6

b)

x 2 2x
e
6

(201=20)

c)

x 3 2x
e
6

d) 0

c)

e 35
s

d) None of these

c)

t4
24

d)

t3
6

d)

2) L{( t 3)} = ___________


a) e35

b) e35

3) L1 1 = _____________
4
s
5
t
t3
b)
a)
5
3

4) L1
= ______________
s

a)

b)

c)

5) The general solution of partial differential equation pq = 3 is


a) z = ax + 3y + c
b) z = 3x + by + c
3
x + by + c
b

c) z =

6) If L{ J0 (t )} =

a)

1
s +1
2

d) None of these

, then L{J0(2t)} = ______________


b)

c)

s +4
s +1
1
7)
{ X} is equals to ___________
D+a

a)

X e

ax

dx

b)

X e

ax

dx

2
s +4
2

c) e a x eax X dx

d)

1
2

s +4
2

d) e a x X e ax dx
P.T.O.

SLR-BB 161

*SLRBB161*

-2-

8) Among the following, which equation is not a non linear partial differential equation ?
p + q =1

a)

b) y 2p + x 2 q = x 2 y 2 z

c) p 3 + q3 = 27z

d) pq = xy

9) L1 {1} = _____________
a) (t )

b) H( t )

c)

1
s

d) None of these

d2 y
10) If L{y(t)}= y (s), then L 2 = ____________
dt

b) s2 y (s) + sy(0) + y (0)

a) s y (s) y(0)

c) s2 y (s) s y (0) y (0)


d) s2 y (s) sy(0) y (0)
11) If f(x) = x is represented by Fourier series in ( , ) then ____________ is the constant term.

b)
2
12) The period of sin7x is ___________

a)

d) 2

c) 0

2
7
13) The speed of a particle moving along the curve x = 2 sin 3t, y = 2 cos 3t, z = 8t at time t = 0 is
___________
a) 8
b) 10
c) 14
d) 20

a) 7

b) 2

c)

d)

14) If r = xi + yj + zk and a is constant vector then (a r ) = _________


a) a
b) r
c) a r
d) 0
15) The unit tangent vector to the curve x = t2 + 1, y = 4t 3, z = 3t2 6t at t = 1 is
1

a)

(i + j + k)

b)

(i + 2 j + k)

c)

1
5

(i + 2 j)

d)

1
5

(i + 2k)

1 , 0 x < 1
16) The Fourier cosine transform of f(x) =
is _____________
0 , x > 1
2
s

a)

17) From the result

2
sin s

b)

c)

2 sin s
s

cos sx sin sk
, | x | < k
ds =
we can deduce that
s
0 , | x | > k

a) 0

2
s. sin s

sin ks
ds = ____
s

c) 2

c)

b) 1

d)

18) Inverse Fourier sine transform of Fs(s) is given by ___________


a)

1
Fs (s) cos sx ds
0

b)

1
2
Fs (s) cos sx ds c)
Fs (s) sin sx ds

0
0

19) If z {f(k)} = F (z) then z{f(k n)} = ______________


a) z k F(z)
b) zk F(z)
c) zn F(z)

d)

2
Fs (s) sin sx ds
0

d) zn F(z)

1 , k 0
20) If u(k) =
then z{u(k)} = _____________
0 , k < 0

a)

z
z 1

b)

1
z 1

c)

1
z 1

d)

z
z 1

______________
Set A

*SLRBB161*

-3-

SLR-BB-161

Seat
No.

S.E. (E and TC) (Part I) Examination, 2014


ENGINEERING MATHEMATICS III
Day and Date : Tuesday, 9-12-2014
Time : 10.00 a.m. to 1.00 p.m.

Marks : 80

N.B. : 1) Attempt any three questions from each Section.


2) Figures to right indicate full marks.
3) Use of calculator is allowed
SECTION I
2. A) Attempt any two :
i) Solve : (D2 + 2aD + a2) y = 8 cos (ax)
ii) Solve :(D2 + 6D + 9) y = 5x log 2 + e3x
iii) Solve : (5z 7y) p + (7x 3z) q = 3y 5x
B)

An electric circuit consists of an inductance L, capacitance C and an e.m.f. E0cos( t), so that the charge q
satisfies the differential equation.

E
d2q
1
+
q = 0 cos t . If 2 = 1 and q = q0, i = i0 when t = 0, then find charge q interms of t.
2
LC
L
dt
LC

3. a) Solve : z(p2 q2) = x y


b)
c)

Solve : p2 pq = 1 z2
Solve :

(D2

4) y =

e3xx2.

2s + 1
1
.
4. a) Find : L
2
(
s
3
)(
s
4
)

b)

Find : L e 2 t 1 + sin 4 t .

c)

Find Laplace transform of periodic function given by


f(t) = t
= t

0<t<

<t<2

where f(t) = f(t + 2 )

5. a) Find : L{ te 2 tH(t 3)} .

s2 a2
2
2
s b

b)

Find : L log

c)

The differential equation of a circuit containing resistance R, capacitance C is given by

dq 1
+ q = 40e 3 t + 20e 6t . If R = 20 , C = 0.01 farad and q(0) = 0, then using Laplace transform find the
dt C

charge q interms of time t.

Set A

SLR-BB 161

*SLRBB161*

-4-

SECTION II

6.

2 x 2
in ( , )

12 4

a)

Find the Fourier series for f(x ) =

b)

Show that the vector field F = (x 2 + xy 2 ) i + (y 2 + x 2 y )j is irrotational and find its scalar potential.

c)

Find the velocity and acceleration of a particle moving along the curve x = 2 sin 3t, y = 2 cos 3t, z = 8t. Also find
their magnitudes.

OR

7.

8.

9.

c)

Find the rate of change of = xyz in the direction normal to the surface x2y + y2 x + yz2 = 3 at the point (1, 1, 1).

a)

If a is a constant vector and r, r have usual meaning, prove that a =

b)

Obtain the Fourier expansion of x sin x (0 < x < 2 ).

a)

Find the inverse z-transform of F(z ) =

b)

Find z {K2 eak}.

c)

Find Fourier sine transform of f(x ) =

a)

Find the inverse z-transform of F(z ) =

b)

2
Express ex.cos x as Fourier cosine integral and show that ex.cos x =

1
r

a 3( a r ) r
+
.
r3
r5

6
7

3z 2 18z + 26
,3 < z < 4 .
(z 2)(z 3)(z 4)

5
5

sin x , 0 < x < 1


.
0 , x >1

z2
1
, | z |< .
1
1
5

z 4 z 5

hence evaluate

2 + 2
d .
( 4 + 4 )

2 + 2
cos x d and
( 4 + 4)
8

_____________________

Set A

SLR-BB 162

*SLR-BB-162*
Seat
No.

Set

S.E. (E & TC) (Part I) Examination, 2014


ELECTRONIC CIRCUIT ANALYSIS AND DESIGN I
Day and Date : Thursday, 11-12-2014
Time : 10.00 a.m. to 1.00 p.m.
Instructions :

Max. Marks : 100

1) Q. No. 1 is compulsory. It should be solved in first 30 minutes in


Answer Book Page No. 3. Each question carries one mark.
2) Question 1 carries 20 marks.
3) Assume suitable data wherever required.
4) Answer MCQ/Objective type questions on Page No. 3 only. Dont
forget to mention, Q.P. Set (A/B/C/D) on Top of Page.
MCQ/Objective Type Questions

Duration : 30 Minutes

Marks : 20

1. Choose the correct options :

20

1) The ripple factor is smaller for higher current in


c) L Filter
a) LC Filter
b) Filter

d) C Filter

2) The most significant component of a ripple voltage in a half wave rectifier is


a) DC component
b) Fundamental frequency
c) Second harmonic
d) None
3) In a PN diode for constant value of current at room temperature, dv/dt varies approximately
at the rate of
a) 25 mV/C
b) + 25 mV/C
c) 2.5 mV/C
d) + 2.5 mV/C
4) Zener breakdown results from
a) Strong electric field
c) Any other cause

b) High applied potential


d) None

5) When two zener diodes of each 10V and 15V are connected in series aiding, the
overall voltage between them when they are conducting is
a) 10.6 V
b) 15.6 V
c) 25 V
d) zero
6) The reverse saturation current for si diode is of the order of
a) 103 A
b) 106 A
c) 109 A

d) 10+6 A

7) The diode current equation for Germanium diode is


a) I = Io(e20v 1)
b) I = Io(e40v 1)
30v
c) I = Io(e
1)
d) None
8) The correct statement is
a) Ge diode is more popular than Si diode
b) Si diode is more popular than Ge diode
c) Both are more popular
d) Si diode is more sensitive to temperature

P.T.O.

SLR-BB 162

*SLR-BB-162*

-2-

9) Determine peak value of output waveform

a) 25 V

b) 15 V

c) 25V

d) 15 V

10) The circuit shown below is of ____________

a) Full wave rectifier b) Half wave rectifier


11) The base-emitter voltage is usually
a) less than base supply voltage
c) more than base voltage

c) Clipper

d) Clamper

b) equal to the base supply voltage


d) none of the above

12) If the base resistor is open, what is collector current ?


a) 0
b) 1 mA
c) 2 mA

d)

13) A small collector current with zero base current is caused by leakage current of
a) C-B junction
b) E-B junction
c) By both
d) None of the above
14) Early effect is caused, because of
a) increase in depletion width of C-B junction
b) increase in reverse bias supply voltage to the collector
c) by both a) and b)
d) forward biasing of B-E junction
15) Q-point stabilization is best against temperature and variation in the following circuit.
a) Collector to base bias
b) Self bias
c) Fixed bias
d) none of the above
16) Which of the following devices revolutionized the computer industry ?
a) JFET
b) D-MOSFET
c) E-MOSFET
d) Power FET
17) A D-MOSFET can be operated in the _____________
a) depletion mode only
b) enhancement mode only
c) depletion mode or enhancement mode d) low-impedance mode
18) The voltage across the load resistor of a capacitor coupled CE amplifier is _____________
a) DC and AC
b) DC only
c) AC only
d) Neither DC nor AC
19) DC load line is nothing but _____________
a) Collection of all possible Q-pts.
b) a line drawn on CE o/p characteristic
c) a line whose slope depends on Rc
d) all of the above
20) Current gain in CE configuration is _____________
a)

b)

c) 1+
______________

d)

Set A

*SLR-BB-162*

-3-

SLR-BB 162

Seat
No.

S.E. (E & TC) (Part I) Examination, 2014


ELECTRONIC CIRCUIT ANALYSIS AND DESIGN I
Day and Date : Thursday, 11-12-2014
Time : 10.00 a.m. to 1.00 p.m.
Instructions :

Marks : 80

Assume suitable data wherever required.


SECTION I

2. Solve any four questions :


(45=20)
a) Explain DC load line and Q point for diode circuit.
b) Explain capacitor filter with full wave rectifier. Also draw output waveforms.
c) Explain operation of circuit shown below for Vi = 30 sin wt. Draw output waveforms and
transfer function.

d) Draw and explain voltage doubler circuit.


e) Explain how zener diode can be used as voltage regulator and explain design of voltage
regulator.
f) What should be the value of inductance to use in inductor filter connected to FWR operating
at 50 Hz if ripple is not exceeding 5% for 15 load.
3. a) Explain operation of full wave rectifier. Give detailed analysis of following parameters :
Vorms, Iorms, Voavg, Ioavg, ripple factor ,

, TUF and PIV.

10

b) State necessary and sufficient conditions for a zener diode to work as regulator, for given
circuit find the range of RL and IL that will result in VRL = 10V.

10

OR
Design a stabilised power supply using L filter to give Vdc = 100 v at 300 load with
peak ripple voltage 10 V.
Set A

SLR-BB 162

*SLR-BB-162*

-4-

SECTION II
4. Solve any four :

(45=20)

1) Draw h-model of transistor and define the h-parameters. What are the advantages of
h-parameters.
2) Derive stability factor for collector to base bias circuit. Also explain how stability is achieved
in the circuit.
3) Frequency response of CE amplifier is affected by capacitors Justify.
4) Explain any one application of JFET.
5) Explain transistor as a switch and switching times of transistor.
6) Explain input and output characteristics of CB configuration.
5. Solve any two :

(210=20)

1) Design a single stage CE amplifier for peak o/p voltage of 3.5 V at 10 k . Lower cutoff
frequency is 40 Hz ; Vcc = 15V and Rs = 500 .
2) Explain drain characteristics and transfer characteristics of N-Channel JFET with
experimental setup. Also explain the important parameter of it.
3) Find Av, Ri, Ro and R i for the given circuit. Assume hie = 1.1k
hfe = 50
hoe = hre = 0

_____________________

Set A

SLR-BB 163

*SLRBB163*
Seat
No.

Set

S.E. (Part I) (Electronics and Telecommunication) Examination, 2014


CIRCUITS AND NETWORKS
Day and Date : Saturday, 13-12-2014
Time : 10.00 a.m. to 1.00 p.m.

Total Marks : 100

Instructions : 1) Figures to the right indicate full marks.


2) Assume suitable data if necessary.
3) Q. No. 1 is compulsory. It should be solved in first 30 minutes in Answer
Book Page No. 3. Each question carries one mark.
4) Answer MCQ/Objective type questions on Page No. 3 only. Dont forget to
mention, Q.P. Set (A/B/C/D) on Top of Page.
MCQ/Objective Type Questions
Duration : 30 Minutes

Marks : 20

1. Choose the correct answer :

(201=20)

1) In order to get maximum power transfer from a capacitive source the load must
a) have capacitive reactance equal to circuit resistance.
b) have an impedance that is the complex conjugate of the source impedance
c) be as capacitance as it is inductive
d) none of the above
2) What is the current through the capacitor in the given circuit ?

a) 3.5mA

b) 5.5mA

c) 9mA

d) 11.4mA

3) How much current will flow in the 100 Hz series RLC circuit if Vs = 20V, RT = 66 and XT = 47 ?
a) 1.05A
b) 240mA
c) 247mA
d) 252mA
4) Find the voltage VA in the circuit ?

a) 518mV
b) 5.18V
c) 9.56V
d) 956mV
5) A 26dBm output in watts equal to
a) 2.4W
b) 0.26W
c) 0.15W
d) 0.4W
6) What voltage will the capacitor charge up to in the given circuit for the single input pulse shown ?

a) 3.15V

b) 4.3V

c) 4.75V

d) 4.9V

7) If Is = 18mA source has an internal resistance Rs = 70 . If load resistance RL = 30 is connected


to current source the load power PL is
a) 476mW
b) 47.6mW
c) 4.76mW
d) 476MW
8) What is the range between f1 and f2 of an RLC circuit that resonates at 150 KHz and has Q of 30 ?
a) 100 KHz to 155 KHz
b) 147.5 KHz to 152.5 KHz
c) 4500 KHz to 155 KHz
d) 149,970 Hz to 150,030 Hz
P.T.O.

SLR-BB 163

*SLRBB163*

-2-

9) What is the voltage across the capacitor in the given circuit ?

a) 6.8V
b) 11.9V
c) 16.1V
10) Which of the following is true for the capacitor ?
a) a capacitor acts like a short to a instantaneous change in current
b) a capacitor voltage cannot change instantaneously
c) a capacitor acts like open for DC
d) all of the above

d) 22.9V

11) The characteristics impedance of a low pass filter in attenuation band is


a) purely imaginary
b) zero
c) complex quantity
d) real value
12) The Z parameters of the given T-network are given by

a) 5, 8, 12, 0

b) 13, 8, 8, 20

c) 13, 8, 8, 13

d) 0, 12, 8, 5

13) For a constant K-type high pass filter, characteristics impedance Z0 for f < fc is
a) resistive
b) inductive
c) capacitive
d) b) or c)
14) Asymmetrical two port networks have
a) Zsc1 = Zoc2
c) Zoc1 Zoc2

15) If (alpha) is attenuation in Neper then


a) attenuation in dB = alpha/0.8686
c) Attenuation in dB = 0.6 times of alpha

b) Zsc1 = Zsc2
d) Zoc1 Zoc2 and Zsc1 Zsc2
b) attenuation in dB = 0.8686 times of alpha
d) attenuation in dB = 0.01 times of alpha

16) For a two port reciprocal network the three transmission parameters are given by A = 4, B = 7 and
C = 5. The value of D equal to
a) 3
b) 2
c) 12
d) 9
17) The relation between R1 and R2 for the given symmetrical lattice attenuator is

a) R1 = R2 = R0

b) R1 =

R0 2

c) R1 =

R2

d) R1 =

R1

R0
R0
R2
18) In a parallel RLC circuit, which value may always be used as vector reference
a) current
b) reactance
c) resistance
d) voltage

19) The real part of propagation constant shows


a) variation of voltage and current on basic unit
b) variation of phase shift/position of voltage
c) variation in voltage, current values of signal amplitude
d) reduction of only voltage amplitude
20) Laplace transform of unit impulse function is
a) s
b) zero
c) es
______________

d) unity
Set A

*SLRBB163*

-3-

SLR-BB-163

Seat
No.
S.E. (Part I) (Electronics and Telecommunication) Examination, 2014
CIRCUITS AND NETWORKS
Day and Date : Saturday, 13-12-2014
Time : 10.00 a.m. to 1.00 p.m.

Marks : 80

Instructions : 1) Figures to the right indicate full marks.


2) Assume suitable data if necessary.
2. Attempt any four :

20

a) By using Nortons theorem, find the current in the load resistor RL for the circuit shown in Fig. a.

Fig. a
b) Determine the current in the 5 resistor for the circuit shown in Fig. b.

Fig. b.
c) For a series resonant circuit, R = 5 , L = 1H and C = 0.25 f . Find the resonance frequency and
band width.
d) Determine the ABCD parameters for the -network shown at Fig. c. Is this network bilateral or not
? Explain.

e) The Z parameters of the two port network are Z 11 = 10 , Z12 = 5 , Z21 = 5 and
Z22 = 15 . Find the equivalent T network and ABCD parameters.

3. a) Derive necessary and sufficient condition for maximum power transfer from a voltage source, with
source impedance Rs + J Xs to a load RL + jXL. What is the value of the
power transferred in this case.

10

OR

Set A

SLR-BB 163

*SLRBB163*

-4-

a) Find the z parameters of the given network. From the z parameters, find the h parameters
equivalent and the ABCD parameters equivalent.

10

b) A 100 mH inductor with 500 self-resistance is in parallel with a 5nF capacitor. Find the resonant
frequency of the combination. Find the impedance at resonance, quality factor of
the circuit and the half power bandwidth.

10

4. Attempt any four :

(45=20)

a) Define the unit step, ramp and impule function. Determine the Laplace transform for these functions.
b) Calculate the driving point impedance Z(s) of the network shown below. Plot the poles and zeros of
the driving point impedance function on the s-plane.

c) Design a Constant K Band Pass filter T-section having cut-off frequencies 2kHz and 5kHz and a
normal impedance of 600 . Draw the configuration of the filter.
d) Derive the relationships between Neper and Decibel units.
e) The series RLC circuit consists of R = 20 , L = 0.05H and C = 20 F with 100V constant source
when the switch is closed at t = 0. Find the current transient.
5. Attempt any two :

(210=20)

a) What is the significance of poles and zeros in network functions. What is the criteria of stability of
a network ? For the transform current I ( s) =

2s
(s + 1)(s + 2 )

, plot its poles and zeros in s-plane

and hence obtain the time domain response.


b) What are the advantages of the m-derived filter over K-type filter ? Derive the expressions for mderived T section and m-derived Section.
c) The series RLC circuit with elements R = 10 , L = 0.5H and C = 200 F has a sinusoidal
voltage v = 150 sin (200t + ). If the switch is closed when = 30, determine the current
equation.
_____________________

Set A

SLR-BB 164

*SLRBB164*
Seat
No.

Set

S.E. (E and TC) (Part I) Examination, 2014


DIGITAL TECHNIQUES
Day and Date : Tuesday, 16-12-2014
Time : 10.00 a.m. to 1.00 p.m.
Instructions :

Max. Marks : 100

1) Figures to right indicate full marks.


2) Assume suitable data wherever necessary.
3) Q. No. 1 is compulsory. It should be solved in first 30 minutes in
Answer Book Page No. 3. Each question carries one mark.
4) Answer MCQ/Objective type questions on Page No. 3 only. Dont
forget to mention, Q.P. Set (A/B/C/D) on Top of Page.
MCQ/Objective Type Questions

Duration : 30 Minutes

Marks : 20

1. Choose the correct answer :

20

1) The number of full adders required in a 4-bit parallel adder


a) one
b) two
c) four

d) eight

2) 1-Digit BCD adder requires ____________ numbers of full adders.


a) one
b) two
c) four
d) eight
3) Multiplexer is represented by
b) 2n n
a) 2n 1

c) n 2n

d) 1 2n

4) What is the number of inputs, outputs of a decoder that accepts 64 different combinations ?
a) 5
b) 6
c) 8
d) 64
5) A sequential circuit is one, whose output depends on __________
a) present state
b) present input
c) both a) and b)
d) none
6) The characteristics equation of d flip flop is
c) D Q
b) D
7) Which of the following are asynchronous inputs.
a) preset
b) clear
c) both a) and b)
a) D

d) D Q
d) none

8) IC 7448 is used for


a) common anode display
c) For both a) and b)

b) common cathode display


d) none

9) The following logic has the highest speed


a) TTL
b) CMOS

c) ECL

d) none

c) IC 7490

d) IC 7495

10) Following IC works as comparator


a) IC 7483
b) IC 7485

11) Which memory is non volatile and may be written only once ?
a) RAM
b) EE-PROM
c) EPROM

d) PROM

P.T.O.

SLR-BB 164

-2-

*SLRBB164*

12) The group of bits 11001 is serially shifted (right-most bit first) into a 5-bit parallel output
shift register with an initial state 01110. After three clock pulses, the register contains
____________
a) 01110
b) 00001
c) 00101
d) 00110
13) The difference between a PLA and a PAL is
a) The PLA has a programmable OR plane and a programmable AND plane, while the
PAL only has a programmable AND plane
b) The PAL has a programmable OR plane and a programmable AND plane, while the
PLA only has a programmable AND plane
c) The PAL has more possible product terms than the PLA
d) PALs and PLAs are the same thing
14) A modulus-12 ring counter requires a minimum of ___________
a) 10 flip-flops
b) 12 flip-flops
c) 6 flip-flops

d) 2 flip-flops

15) Assume that a 4-bit serial in/serial out shift register is initially clear. We wish to store the
nibble 1100. What will be the 4-bit pattern after the second clock pulse ? (Right-most bit
first)
a) 1100
b) 0011
c) 0000
d) 1111
16) Synchronous counters eliminate the delay problems encountered with asynchronous
counters because the
a) input clock pulses are applied only to the first and last stages
b) input clock pulses are applied only to the last stage
c) input clock pulses are not used to activate any of the counter stages
d) input clock pulses are applied simultaneously to each stage
17) With a 200 kHz clock frequency, eight bits can be serially entered into a shift register in
_______________
b) 40 s
c) 400 s
d) 40 ms
a) 4 s
18) How many clock pulses will be required to completely load serially a 5-bit shift register ?
a) 2
b) 3
c) 4
d) 5
19) The state diagram is the pictorial representation among ____________
a) present states
b) next states
c) Input and output
d) All of these
20) What is the difference between a ring shift counter and a Johnson shift counter ?
a) There is no difference
b) A ring is faster
c) The feedback is reversed
d) The Johnson is faster
______________

Set A

*SLRBB164*

-3-

SLR-BB 164

Seat
No.
S.E. (E and TC) (Part I) Examination, 2014
DIGITAL TECHNIQUES
Day and Date : Tuesday, 16-12-2014
Time : 10.00 a.m. to 1.00 p.m.
Instructions :

Marks : 80

1) All questions are compulsory.


2) Figures to right indicate full marks.
3) Assume suitable data wherever necessary.
SECTION I

2. Solve any four :

16

a) Explain SOP and POS.


b) Explain the concept of multiplexer tree.
c) Implement full substractor using Demultiplexer.
d) Explain the significance of TTL subfamilies.
e) What is race around condition ? How it is eliminated ?
3. Design 2-bit magnitude comparator.

4. Attempt any two :

16

a) Draw and explain a TTL gate with totem-pole output detail.


b) Explain characteristic table and Characteristic equation for SR, JK, D and T flip-flop.
c) Design and explain a 1-decimal digit BCD adder using IC7483.
SECTION II
5. Attempt any four :

(44=16)

a) Explain Parallel in Parallel out shift register.


b) Draw and explain 5 bit Ring counter.
c) Compare Asynchronous and Synchronous Sequential Circuit.
d) With the help of block diagram and waveform explain 3 bit Asynchronous Down Counter.
e) Design MOD-6 Asynchronous counter.
f) Explain with example implementation of a combinational circuit using ROM.
Set A

SLR-BB 164

-4-

*SLRBB164*

6. a) Design and explain 3 bit Synchronous Up Counter.


b) Attempt any two :

8
(82=16)

i) Implement a combinational logic function having 3 inputs and 4 outputs using PAL for
the following :
Y1 = m (1, 2, 4, 7)
Y2 = m (1, 2, 3, 6)
Y3 = m (0, 3, 5, 7)
Y4 = m (0, 2, 4, 5)
ii) Design a counter which counts in the given sequence (0, 1, 2, 4, 5, 0) using JK flip flop.
iii) Explain what are qualitative differences between parallel loading and serial loading in
shift register.
iv) Design a sequence detector for a following state diagram.

_____________________

Set A

SLR-BB 165

*SLRBB165*
Seat
No.

Set

S.E. (E & TC) (Part I) Examination, 2014


DATA STRUCTURES USING C
Day and Date : Thursday, 18-12-2014
Time : 10.00 a.m. to 1.00 p.m.

Max. Marks : 100

Instructions : 1) Figures to the right indicate full marks.


2) Assume suitable data if necessary.
3) Q. No. 1 is compulsory. It should be solved in first 30
minutes in Answer Book Page No. 3. Each question carries
one mark.
4) Answer MCQ/Objective type questions on Page No. 3
only. Dont forget to mention, Q.P. Set (A/B/C/D) on Top
of Page.
MCQ/Objective Type Questions
Duration : 30 Minutes

Marks : 20

1. Choose the correct answer :

20

1) We declare a function with ____________ if it does not have any return type.
a) long
b) double
c) void
d) int
2) In a circular linked list
a) components are all linked together in same sequential manner
b) there is no beginning and no end
c) components are arranged hierarchically
d) forward and backward traversal within the list is permitted
3) The data structure required to evaluate a postfix expression is
a) queue
b) stack
c) array
d) linked list
4) The postfix form of A * B + C/D is
a) AB * CD/+
b) *AB/CD+
c) A * BC+/D
d) ABCD+/*
5) In queue elements are added at
a) rear end
b) front end
c) top end
d) interior node
6) In this queue, the insertion of elements can takes place at one end only, but
the deletion of elements can be done at both the ends
a) Priority queue
b) Input restricted deque
c) Output restricted deque
d) Both a) and b)
P.T.O.

SLR-BB 165

*SLRBB165*

-2-

7) The prefix notation is also called as


a) polish notation
c) both a) and b)

b) reverse polish notation


d) none of above

8) The free( ) function is used to


a) to unlink the node
c) release the memory

b) to unlink the first and last node


d) none of above

9) The node of singly linked list contains


a) prev, info, next b) info, next
c) both a) and b) d) none of above
10) Due to the nature of a recursive definition, the implementation of a recursive
method involves
a) A while loop
b) A for loop
c) A static variable
d) An if statement
11) A tree travelled from root to left subtree and then right subtree it is _________
traversal method.
a) Pre-order
b) Post-order
c) In-order
d) None of above
12) A tree is called as binary tree, if each node have
a) max. 3 children
b) max. 2 children
c) max. 4 children
d) max. 1 children
13) The post-order traversal method travels first
a) left sub tree
b) right sub tree c) root
d) none of above
14) Graph containing only isolated node is called as
a) cyclic graph
b) a cyclic graph c) multigraph
15) The degree of isolated node is
a) 1
b) 2
c) 3

d) null graph
d) 0

16) The length of the longest path from root to any node is known as
a) Depth of tree
b) Length of tree
c) Width of tree
d) None of above
17) The nodes having same parents are called as
a) root node
b) interior node c) leaf node
d) siblings
18) Linear search is also called as
a) Sequential searching
b) Random searching
c) Hashing
d) None of above
19) Radix sort is based on
a) Positional value of actual digit
c) Number of digits

b) Value of actual digit


d) None of above

20) In insertion sort, records are inserted in


a) Unsorted file
b) Sorted file
c) Small file
______________

d) None of above
Set A

*SLRBB165*

-3-

SLR-BB 165

Seat
No.

S.E. (E & TC) (Part I) Examination, 2014


DATA STRUCTURES USING C
Day and Date : Thursday, 18-12-2014
Time : 10.00 a.m. to 1.00 p.m.

Marks : 80

Instructions : 1) All questions are compulsory.


2) Figures to the right indicate full marks.
3) Assume suitable data if necessary.
SECTION I
2. Answer any four :

(54=20)

1) Give difference between iterative function and recursive function.


2) Write different application of stack.
3) Differentiate between static memory allocation and dynamic memory allocation.
4) Evaluate the following postfix expression using stack.
4, 5, 4, 2, , +, , 2, 2, , 9, 3, /, ,
5) Write a note on double ended queue.
3. Answer any two :

(102=20)

1) Write a C program to implement linear queue using array and perform the
following operation
1) enqueue
2) dequeue
3) queue display
4) queue empty
5) queue full.
2) Define linklist and give its applications. Explain in detail doubly linked list
with neat diagram.
3) Write a recursive function and iterative function in C to find sequence of
Fibonacci numbers.
Set A

SLR-BB 165

-4-

*SLRBB165*

SECTION II
4. Answer any four :

(54=20)

a) Draw the binary search tree for following input


10, 15, 20, 30, 14, 40, 7, 9
b) Write a C code/program to implement binary search method.
c) Give difference between, linear search, binary search and hashing.
d) Consider the two sorted list given below, combine them to yorm third sorted
list using merge sort technique.
e) Explain isolated node with example and draw null graph with 6 nodes and find
its degree.
5. Answer any two :

(102=20)

a) Define tree and explain the following terms with example :


i) Siblings
ii) Ancestor node
iii) Depth of tree
iv) Descendent node
b) Represent the given graph using adjacent matrix and linked list AND also
find its degree.

c) Give difference between insertion sort and selection sort and write a
program to implement insertion sort technique.
_____________________

Set A

SLR-BB 166

*SLRBB166*
Seat
No.

Set

S.E. (E & TC) (Part II) Examination, 2014


ELECTRONIC CIRCUIT ANALYSIS AND DESIGN II (Old)
Day and Date : Tuesday, 25-11-2014
Time : 3.00 p.m. to 6.00 p.m.

Max. Marks : 100

N.B. : 1)
2)
3)
4)

All questions are compulsory.


Figures to the right indicates full marks.
Assume suitable data whenever required.
Q. No. 1 is compulsory. It should be solved in first 30 minutes in Answer Book
Page No. 3. Each question carries one mark.
5) Answer MCQ/Objective type questions on Page No. 3 only. Dont forget
to mention, Q.P. Set (A/B/C/D) on Top of Page.
MCQ/Objective Type Questions

Duration : 30 Minutes

Marks : 20

1. 1) For single stage RC coupled amplifier, output voltage is taken from emitter results to
a) current series feedback
b) voltage shunt feedback
c) voltage series feedback
d) none of the above
2) It is required to reduce the voltage gain of an amplifier from 100 to 80. What will be the feedback
factor ? If initial B.W. is 40 KHz what will be the new B.W. ?
a) 0.5%, 50 KHz
b) 0.25%, 50 KHz
c) 0.25%, 40 KHz
d) 0.5%, 100 KHz
3) For power amplifier circuit design with single transistor distortion levels are D2 = 10%, D3 = 20%,
D4 = 30%, D5 = 40%, D6 = 70%, D7 = 80%. If circuit is redesigned in push pull configuration the new
total distortion will be
a) 60%
b) 61.35%
c) 81.65%
d) 91.65%
4) For Colpitts oscillator tank circuit values are 0.02 F, 20 F and 10 mH. The frequency of oscillations is
a) 16.259 KHz
b) 1.529 KHz
c) 1.529 MHz
d) None of the above
5) The conversion efficiency of class B power amplifier is
a) 50%
b) 62.5%
c) 78.5%
d) 25%
6) For 3 stage RC coupled amplifier, highest value of individual power cutoff frequency is 50 Hz. The
over-all cutoff frequency for 3 stage amplifier is
a) 98.073 Hz
b) 88.05 Hz
c) 78.05 Hz
d) None of the above
7) Transistor with hFE =300, hie = 2.2 K , Rs = 100 , is used in the design of current series negative
feedback. What is the value of new input impedance with RE = 500 ?

a) 52.8 K
b) 100.8 K
c) 152.8 K
d) 52.8 K
8) Transformer coupled amplifier is preferred for
a) Good frequency response
b) Impedance matching
c) To get more output power
d) Both b) and c)
9) Drift is the serious problem in
a) RC coupled amplifier
b) Direct couple amplifier
c) Complementary symm. power amplifier
d) Transformer coupled amplifier
10) Effect of source resistance in amplifiers
a) Increases i/p resistance and volt. gain
b) Decreases i/p resistance and volt. gain
c) Increases input resistance and decreases volt. gain
d) No effect

P.T.O.

SLR-BB 166

*SLRBB166*

-2-

11) The internal circuitry of the timer consists of ____________, an RS FF, a transistor switch, an output
buffer amplifier and a voltage divider.
a) a comparator
b) a voltage amplifier
c) two comparators
d) peak detector
12) If a diode is connected across RB (positive end up) in the given figure, what is the new duty cycle of
the output waveform

a) 56%
b) 44%
c) 21.6%
d) 17.4%
13) The monostable multivibrator circuit is not an oscillator because
a) its output switches between two states
b) it requires a trigger to obtain an output voltage
c) it requires sine wave input signal
d) the circuit does not require a dc power supply
14) For N channel JFET IDSS = 8.7 mA, VP = 3V, VGS = 1 V, value of ID is __________
a) 3.866 mA
b) 38.66 mA
c) 3.86 A
d) none of these
15) The JFET is used as VVR the area of operation selected will be
a) saturation
b) cutoff
c) zero
d) ohmic
16) A N channel JFET has pinch off vtg. 4.5 V IDSS = 9 mA, the value of VGS for IDS = 3mA will be
a) 19.02V
b) 1.902 V
c) 1.902 V
d) none of these
17) Depletion type MOSFET can work in
a) Deletion mode only
b) Enhancement mode only
c) Depletion mode and enhancement mode
d) None of these
18) Output voltage of regular circuit shown below is

a) 10.805 V

b) 10.805

c) 1.08 V

d) none of these

19)

Assuming IQ 0, the value of IL for the above circuit will be


a) 0 A
b) 0.5 A
c) 5 A
d) none of these
20) The output current equation for IC regulator used as current source circuit is given by
a) IL =

VQ
RL

+I

b) IL = IQ

c) IL = VR R + IQ
______________

d) IL =

VR
R

+ IQ

Set A

*SLRBB166*

-3-

SLR-BB 166

Seat
No.

S.E. (E & TC) (Part II) Examination, 2014


ELECTRONIC CIRCUIT ANALYSIS AND DESIGN II (Old)
Day and Date : Tuesday, 25-11-2014

Marks : 80

Time : 3.00 p.m. to 6.00 p.m.


N.B. : 1) All questions are compulsory.
2) Figures to the right indicate full marks.
3) Assume suitable data wherever required.
2. Solve any four :

20

a) Derive the equation for a frequency of Weinbridge at which phase shift introduced is zero.
b) Justify h-parameter equivalent circuit fails at high frequency analysis of RC coupled amplifier and
analysis of power amplifiers.
c) Design tank circuits of Colpitts and Hartely oscillators for generating frequency 1 MHz.
d) Explain transformerless power amplifier.
e) Compare four types of negative feedback circuits.
f) Draw and explain equivalent circuit for a crystal and derive the equations for resonant frequencies.
3. a) Design class A single ended transformer coupled power amplifier for delivering 2 watts power to the
resistive load of 8 . Assume transformer efficiency 80%, VCC = 20 V.

10

b) Design RC phase shift oscillator for oscillating frequency 100 KHz using tran. BC 147. Use
VCC = 15 V. List out the demerits of RC phase shift oscillators.

10

OR
c) Design single stage RC coupled amplifier for voltage gain 100, load current 10 mA, Use VCC = 15 V.
If emitter bypass capacitor is removed, how much is the voltage gain of modified circuit.
4. Solve any four :

10
(45=20)

1) Design adjustable voltage regulator for the following specifications output voltage 5 V to 10 V,
output current 1.2 A.
2) What are advantages of IC vtg. regulators ? Draw and explain circuit for a + 12 V power supply for
input voltage 230 V, 50 Hz A-C.
Set A

SLR-BB 166

-4-

*SLRBB166*

3) Draw output characteristics of JFET and explain different regions of the output characteristics.
4) Compare between FET and BJT.
5) Determine TON, TOFF, T, f and % D for IC 555 based astable multivibrator output for C = 0.01 F,
RA = 2.2 K and RB = 3.901 K .
6) Design a power on delay circuit for 5 sec using IC 555.
5. Solve any two :

(210=20)

1) Draw and explain VI characteristics of JFFE (N channel). What are different applications of JFET ?
Discuss FET as VVR.
2) Prove that the frequency of oscillation for IC 555 based astable multivibrator is,
fo =

1.45
, suggest the modification for getting duty cycle less than 50%.
(R A +2RB )C

3) Design a dual tracking voltage regulator using three terminal adjustable IC for following
specification
1) Vo = 12 V to 24 V
2)

Vin = 30 2%

3) Io = 1.1 A.
_____________________

Set A

SLR-BB 167

*SLR-BB-167*
Seat
No.

Set
S.E. (E & TC) (Part II) (Old) Examination, 2014
ANALOG COMMUNICATION

Day and Date : Wednesday, 26-11-2014


Time : 3.00 p.m. to 6.00 p.m.

Max. Marks : 100

Instructions : 1)
2)
3)
4)

All questions are compulsory.


Figures to the right indicate full marks.
Assume suitable data if necessary.
Q. No. I is compulsory. It should be solved in first 30 minutes in
Answer Book Page No. 3. Each question carries one mark.
5) Answer MCQ/Objective type questions on Page No. 3 only.
Dont forget to mention, Q.P. Set (A/B/C/D) on Top of Page.
MCQ/Objective Type Questions

Duration : 30 Minutes
I.

Marks : 20

Choose one of the correct alternatives :


1) A 1000 kHz carrier is simultaneously modulated with 300 Hz, 800 Hz and 2 kHz audio
sine waves. What is the bandwidth of o/p AM signal ?
a) 600 Hz
b) 2000 kHz
c) 1600 Hz
d) 4 kHz
2) In AM, Modulation index is given by
a) V c/V m
b) Vm/Vc

c) Vm+Vc

d) V cV m

3) Good selectivity usually means ____________ Bandwidth.


a) Wide
b) Narrow
c) Phase
d) None of these
4) An amplitude modulation performs the mathematical operation of
a) addition
b) Multiplication c) Subtraction
d) Division
5) A 100 MHz carrier is deviated 50 kHz by 4 kHz signal. The modulation index is
a) 5
b) 8
c) 12.5
d) 20
6) The ionosphere causes radio signals to be
a) Diffused
b) Absorbed
c) Refracted

d) Reflected

7) Balanced modulator is used in ______________ generation


a) AM
b) DSBSC
c) SSB
d) b) and c)
8) Which one of the following is not type of telephone exchange ?
a) Strowger switched exchange
b) Crossbar switched exchange
c) Crowbar switched exchange
d) Electronic switched exchange
9) If modulation signal frequency is doubled, then maximum frequency deviation is
a) Doubled
b) Halved
c) Remains same
d) None of the above
P.T.O.

SLR-BB 167

*SLR-BB-167*

-2-

10) Radio communication is possible only due to _____________ transmission.


a) Single hop
b) Dual hop
c) Multi hop
d) None of above
11) A receiver that uses only tuned amplifiers and detector is known as _______
receiver.
a) Superheterodyne
b) TRF
c) Ratio
d) Armstrong
12) Perfect modulation occurs when _____________
b) Vm<Vc
c) Vm= Vc
a) Vm >Vc

d) Vm= Vc=0

13) In AM, the instantaneous _____________ of the carrier varies in accordance


with the information.
a) Frequency
b) Phase
c) Amplitude
d) All of these
14) In AM, modulation index is given by
a) Vc/Vm
b) Vm/V c

c) Vm+Vc

15) A pre-emphasis circuit is a


a) Low-pass filter b) Phase-shifter

c) High-pass filter d) Band pass filter

d) V cV m

16) Indicate which of the following circuits could not demodulate SSB
a) Balanced modulator
b) Product detector
c) BFO
d) Phase discriminator
17) A receiver has poor IF selectivity. It will therefore also have poor
a) Blocking
b) Double-spotting
c) Diversity reception
d) Sensitivity
18) A telephone is
a) Full-duplex

b) Half-duplex

c) Multiplex

19) _____________ antenna is not a wideband.


a) Discone
b) Folded dipole c) Helical
20) The FM produced by PM is called
a) FM
b) PM
c) Indirect FM
______________

d) All of the above


d) Marconi
d) Indirect PM

Set A

*SLR-BB-167*

-3-

SLR-BB 167

Seat
No.

S.E. (E & TC) (Part II) (Old) Examination, 2014


ANALOG COMMUNICATION
Day and Date : Wednesday, 26-11-2014
Time : 3.00 p.m. to 6.00 p.m.

Marks : 80

Instructions : 1) All questions are compulsory.


2) Figures to the right indicate full marks.
3) Assume suitable data if necessary.
SECTION I
II. Answer any four :

(54=20)

a) Define modulation. List and explain the different types of modulation in brief.
b) Describe Time Division Multiplexing (TDM) in detail.
c) Define Noise figure and derive its expression.
d) The antenna current of an AM broadcast transmitter, modulated to a depth of
40 percent by an audio sine wave, is 11A. It increases to 12 A as result of
simultaneous modulation by another audio sine wave. What is the modulation
index due to this second wave ?
e) Explain Independent Sideband System (ISB) in brief.
III. Answer any two :

(102=20)

a) i) Explain the different types of noises. Also discuss its causes and effects.
ii) Discuss the Third method for SSB generation.
b) With the help of block diagram. Explain the working of balanced modulator
using FET. Also give its mathematical analysis.
c) i) With the help of block diagram explain superheterodyne radio receiver.
ii) What is Automatic Gain Control (AGC). Explain its operation with neat
diagram.
Set A

SLR-BB 167

-4-

*SLR-BB-167*

SECTION II
IV. Answer any four :

(54=20)

a) Explain in brief pre-emphasis and de-emphasis.


b) Derive the formula for instantaneous value of an FM voltage and define modulation
index.
c) Explain multiple stage switching system used in telephony.
d) Write a short note on Yagi antenna with its characteristics.
e) Differentiate between narrow band and wide band FM.
V. Answer any two :

(102=20)

a) Explain Indirect method of FM generation with its block diagrams and vector
diagrams.
b) Explain Time Slot Interchange (TSI) system used in telephony with its diagram.
c)

i) Explain the ratio detector in detail.


ii) Define beam width, radiation resistance and antenna gain.
_____________________

Set A

SLR-BB 168

*SLRBB168*
Seat
No.

Set

S.E. (E & TC) (Part II) (Old) Examination, 2014


CONTROL SYSTEMS
Day and Date : Thursday, 27-11-2014
Time : 3.00 p.m. to 6.00 p.m.

Max. Marks : 100

Instructions : 1) Figure to the right indicate full marks.


2) Assume suitable data if necessary.
3) Q. No. 1 is compulsory. It should be solved in first 30 minutes in Answer book page
No. 3. Each question carries one mark.
4) Answer MCQ/Objective type questions on Page No. 3 only. Dont forget to
mention, Q.P. Set (A/B/C/D) on Top of Page.
MCQ/Objective Type Questions
Duration : 30 Minutes
1.

Marks : 20

Tick the correct alternative :


1) The output of the controller in a control system is given by
a) Amplifier
b) Comparator
c) Final control element
d) None of these
2) Knowledge of transfer function of a system is necessary for the calculation of
a) The time constant
b) The output for a given input
c) The steady state gain
d) The order of system
3) The resolution of a potentiometer should be
a) Zero
b) Low

c) High

d) Infinite

4) Principal drawback of a servomotor is that it


a) Has low reliability
b) Has low starting torque
c) Develops commutation problem
d) Can handle only light loads
5) The Routh array of a characteristic equation is given below. The number of roots lying on the RHS of S-plane is
S4

S 3
S1

S
a) 0

b) 1

c) 2

6) The liquid level control system, the desired liquid level H is __________
a) Controlled output
b) Reference input
c) Error signal

d) 4
d) None of these

7) The transfer function is defined for


a) Linear and time invariant system
b) Linear and time variant system
c) Non-linear time invariant system
d) Non-linear time variant system

P.T.O.

SLR-BB 168

*SLRBB168*

-2-

8) The block diagram is equivalent to

a)

b)

c)

d)

9) The number of individual loops in the SFG shown below are

a) 4

b) 5

c) 6

10) In force-voltage analogy, moment of inertia is analogous to


a) Capacitance
b) Inverse capacitance
c) Inductance

d) 7
d) Inverse inductance

11) The time required for the response to rise from 5% to 95% of the final value for critically damped system is called the
a) Rise time
b) Delay time
c) Peak time
d) Settling time
12) The root locus of a system has three asymptotes. The system may have
a) Three poles
b) Four poles and one zero
c) Five poles and two zeros

d) All of the above

13) The initial slope of bode plot for a transfer function having simple pole at origin is
a) 20 db/decade
b) 40 db/decade
c) 40 db / decade
d) 20 db / decade
14) When the gain margin is positive and the phase margin is negative, the system is
a) Stable
b) Unstable
c) Oscillatory
d) Highly stable
15) The characteristic equation of a system is S2 + 2S + 8. The damping factor will be
a) < 1
b) > 1
c) = 1
d) None
16) For a type-1 system, the steady state acceleration error constant is
a) Zero
b) Unity
c) Between zero and unity
17) The phase _____________compensator increases the band width mostly
a) Lag
b) Lead
c) Lag-lead

d) Infinite
d) None of these

18) A phase lead compensation is used for which one of the following ?
a) To increase rise time and decrease overshoot b) To decrease both rise time and overshoot
c) To increase both rise time and overshoot
d) Decrease rise time and increase overshoot
19) The break away points of the root locus occurs at
a) Real axis
b) Imaginary axis c) Multiple roots of characteristic equation

d) None of these

20) In the bode plot of a unity feedback control system, the value of phase of G(jw) at the gain cross over
frequency is 125. The phase margin of the system is
a) 125
b) 55
c) 55
d) 125

______________

Set A

*SLRBB168*

-3-

SLR-BB 168

Seat
No.

S.E. (E & TC) (Part II) (Old) Examination, 2014


CONTROL SYSTEMS
Day and Date : Thursday, 27-11-2014
Time : 3.00 p.m. to 6.00 p.m.
Instructions :

Marks : 80

1) Figure to the right indicate full marks.


2) Assume suitable data if necessary.
SECTION I

2. Solve any four :

20

a)

Derive the transfer function for open loop and close loop control systems.

b)

Explain mechanical rotational system element.

c)

Write a short note on tachogenerator.

d)

Explain advantage and limitation of Rouths criterion.

e)

Explain the terminology used in signal flow graph.

3. Explain Masons gain formula in detail. Find out the overall gain for given SFG.

10

4. Attempt any one :

10

a)

Explain potentiometer in detail. Also explain potentiometer as error detector with example.

b)

A unity feedback system has G(s) =

2
determine its relative stability about s = 1.
s ( s + 1) ( s + 2 )

Set A

SLR-BB 168

-4-

*SLRBB168*

SECTION II
5. Solve any four :

(45=20)

a)

Derive the expression and plot the response of undamped second order system when the input is unit step.

b)

Explain the following terms in reference to root locus with suitable example.
i) Break away and break in points
ii) Angle of departure and arrival

c)

What is a lead-lag compensator ? Obtain the transfer function of a lead-lag compensator and draw the polezero plot.

d)

Explain the steps to sketch the magnitude plot of bode plot with example.

e)

Draw the time domain response c(t) of a typical underdamped second- order system to a step input. Define
and indicate time domain specifications on the response.

6. Solve any two :


a)

(210=20)

The open loop transfer function of a unity feedback system is given by


k (s + 9 )
G(s) =

b)

s ( s 2 + 4 s + 11)

sketch the root locus for the system.

The open loop transfer function of a unity feedback system is given by


20
G(s) = s ( s + 2 ) ( s 2 + 2 s + 20 )
Find the static error constants and steady state error of the system when the inputs are :
i) 9

c)

iii) 17 t2

ii) 15 t

Sketch the bode plot for the transfer function


1000
G(s) = s (1+ 0.1s ) (1+ 0.001s )
Determine :
i) Gain crossover frequency
ii) Phase crossover frequency
iii) G. M. and P.M.

_____________________

Set A

SLR-BB 169

*SLRBB169*
Seat
No.

Set

S.E. (Electronics and Telecomm. Engg.) (Part II) Examination, 2014


LINEAR INTEGRATED CIRCUITS (Old)
Day and Date : Friday, 28-11-2014
Time : 3.00 p.m. to 6.00 p.m.

Max. Marks : 100

Instructions : 1) Q. No. 1 is compulsory. It should be solved in first 30 minutes


in Answer book Page No. 3. Each question carries one mark.
2) Answer MCQ/Objective type questions on Page No. 3 only.
Dont forget to mention, Q.P. Set (A/B/C/D) on Top of Page.
MCQ/Objective Type Questions
Duration : 30 Minutes

Marks : 20

1. Tick mark correct option :


1) Voltage gain of DIBO differential amplifier is
Rc

a) r
e

b) 2 Bacre

re

c) R
c

d)

2Rc
re

2) Tailored frequency response Op amps are Op amps with


a) Internal compensation
b) External compensation
c) Both a) and b)
d) None
3) Current missor is special case of
a) Constant voltage source
c) Variable current bias

b) Constant current bias


d) Variable voltage gain

4) Op amp is ____________amplifier.
a) RC coupled, low gain
c) Direct coupled high gain

b) RC coupled high gain


d) Direct coupled low gain

5) Output stage of Op amp provides


a) Low input resistance
c) Gain

b) Low output resistance


d) None

6) Slew rate is____________phenomenon.


a) Voltage
b) Current
c) Large signal

d) Small signal

7) Ideal value of CMRR should be


a) Zero
b) 1000

d) Infinite

c) 90 db

8) Open loop gain increases with increasing frequency


a) True
b) False
c) Partially true

d) None
P.T.O.

SLR-BB 169

*SLRBB169*

-2-

9) An averaging amplifier has fine inputs. The ratio Rf / RI must be


a) 5
b) 0.2
c) 0.5
d) 1
10) Window detector
a) Has only one usable threshold
c) Clamps the input positively

b) Uses hysteresis to speed up response


d) Detects input voltage between two limits

11) Square wave generator is also called as ____________


a) Free running multivibrator
b) Astable multivibrator
c) Bi-stable multivibrator
d) Both a) and b)
12) In triangular wave ____________
a) Rise time = Fall time
c) Rise time < Fall time

b) Rise time > Fall time


d) None of these

13) A circuit which generates a repetitive waveform of fixed amplitude and


frequency without any external input signal is called
a) Rectifier
b) Amplifier
c) Oscillator
d) None of these
14) Quadrature oscillator produces ____________
a) Square and triangular wave
b) Square and sawtooth wave
c) Triangular and sawtooth wave
d) Sine and cosine wave
15) The application of VCO is ____________
a) FM
b) Tone generators
c) FSK
d) All above
16) Log amplifier and antilog amplifiers are ____________ and ____________
applications of Op-amp respectively.
a) Linear linear
b) Non linear non linear
c) Linear non linear
d) Non linear linear
17) Multipliers are used in ____________
a) Amplitude modulation
b) Phase modulation
c) Frequency modulation
d) All of above
18) Which of the following function is done by PLL ?
a) Free running
b) Capture
c) Phase lock

d) All of above

19) Which controller having discontinuous mode of operation ?


a) ON-OFF controller
b) Proportional controller
c) PI controller
d) PD controller
20) OFF set error in proportional controller is also called as ____________
a) Steady state error
b) Reaction rate of controller
c) Set point of controller
d) None of these
______________
Set A

*SLRBB169*

-3-

SLR-BB 169

Seat
No.

S.E. (Electronics and Telecomm. Engg.) (Part II) Examination, 2014


LINEAR INTEGRATED CIRCUITS (Old)
Day and Date : Friday, 28-11-2014
Time : 3.00 p.m. to 6.00 p.m.

Marks : 80

SECTION I
2. Solve any four :

(45=20)

a) Draw and explain positive peak detector .


b) Define :
i) SVRR
iii) ViO

ii) SR
iv) Iio

v) I B
c) Explain sample and hold circuit with its application.
d) Explain current mirror circuit with its advantages.
e) Determine the input and output impedances of Op amp with given data in
NI mode.
Ri = 2M , RO = 75 , AOL = 2M, RI = 10 k , RF = 220 k
Also find closed loop voltage gain.
3. a) Explain how Op amp can be used as summing amplifier and how it can be
converted to averaging and scaling circuit.
(110=10)
b) Derive gain for dual input balanced output differential amplifier with necessary
diagrams.
(110=10)
OR
c) Explain each block of Op amp in detail with its contribution to electrical
parameter.

Set A

SLR-BB 169

-4-

*SLRBB169*

SECTION II
4. Solve any four :

(45=20)

1) Draw pin diagram of IC 565 PLL.


2) Draw and explain Wein bridge oscillator.
3) Explain antilog amplifier.
4) Draw and explain triangular wave generator.
5) Explain advantages and limitations of ON-OFF controller.
6) Draw and explain quadrature oscillator.
5. a) Draw and explain proportional temperature controller.
b) Solve any one :

(110=10)
(110=10)

1) Explain in detail full wave precision rectifier.


2) Draw neat circuit diagram and explain square wave generator using
Op-amp.
_____________________

Set A

SLR-BB 17

*SLRBB17*
S

F. E. (Part I) (New) Examination, 2014


ENGINEERING PHYSICS
Day and Date : Wednesday, 17-12-2014
Time : 10.00 a.m. to 1.00 p.m.

Max. Marks : 70

Instructions : 1) Make suitable assumptions, if necessary.


2) Figures to the right indicate full marks.
3) Q. No. 1 is compulsory. It should be solved in first 30
minutes in Answer Book Page No. 3. Each question carries
one mark.
4) Answer MCQ/Objective type questions on Page No. 3
only. Dont forget to mention, Q.P. Set (A/B/C/D) on Top
of Page.
Constants :

1) Avogadros no., N = 6.02 1026/ k.mol.


2) Velocity of light, c = 3 108 m/sec.
3) Charge of electron, e = 1.6 1019 C.

Duration : 30 Minutes

MCQ/Objective Type Questions

Marks : 14

SECTION I

1. Choose the correct answer.


1) In N-type semiconductor material the majority free charge carriers are ________
a) electrons
b) protons
c) neutrons
d) positive holes
2) The Hall coefficient (RH) is ___________ for P type semiconductor.
a) Negative
b) Positive
c) No sign
d) Any other
3) The co-ordination number in case of BCC structure is
a) 12
b) 6
c) 8
4) The Braggs equation is __________
a) 2d sin = /n
b) 2d sin
c) 2d sin = dn
d) 2 sin


=n
=n

d) 4

5) Magnetostriction method is used to produce ultrasonic waves of frequency


____________
a) 20 Hz to 20 kHz
b) less than 20 Hz
c) 20 Hz to 100 Hz
d) 20 kHz to 100 kHz
P.T.O.

SLR-BB 17

*SLRBB17*

-2-

6) The Einsteins mass energy relation is given by


a) M = Ec2

b) C = mE2

d) E = mc2

c) E = moc

7) The acoustic diffraction method is used to determine the


a) Wavelength of light
b) Resolving power of diffraction grating
c) Wavelength of ultrasonic waves
d) Velocity of light
SECTION II
8) The phenomenon causing polarization of light is
a) interference

b) diffraction

c) double refraction

d) refraction

9) The substances that rotate the plane of polarization are said to be


a) optically active

b) optically inactive

c) opaque

d) polaroid

10) In He-Ne laser, ____________ atoms are the active centers.


a) Cd

b) Al

c) He

d) Ne

11) Stimulated Emission process is represented as


a) A + h
c) A*

A*

A+h

b) A* + h
d) A* + h

A + 2h

A+h

12) In graded index optical fiber the refractive index of core is


a) uniform
b) increase towards the axis of core
c) decrease towards the axis of core
d) less than cladding refractive index
13) Energy radiated from sun is a result of __________
a) nuclear fission

b) combustion

c) cosmic radiation

d) nuclear fusion

14) Chirality of Zigzag CNT is _________


a) ( , 0)


b) ( ,


)
c) ( , b)
______________


d) (0, b)
Set A

*SLRBB17*
S

-3-

SLR-BB 17

F. E. (Part I) (New) Examination, 2014


ENGINEERING PHYSICS
Day and Date : Wednesday, 17-12-2014
Time : 10.00 a.m. to 1.00 p.m.

Marks : 56

Instructions : 1) Make suitable assumptions, if necessary.


2) Figures to the right indicate full mark.
Constants :

1) Avogadros no., N = 6.02 1026/ k.mol.


2) Velocity of light, c = 3 108 m/sec.
3) Charge of electron, e = 1.6 1019 C.
SECTION I

2. Attempt any five of the following :


15
a) Define valence band, conduction band and energy gap and show with neat
diagram.
b) Define :
i) Atomic radius
ii) Co-ordination number and
iii) Unit cell.
c) State the properties of ultrasonic waves.
d) Obtain the relativistic formula for the addition of velocities.
e) An n-type germanium sample has a donor density of 1021/m3. It is arranged
in a Hall experiment having magnetic field of 0.5 T and the current density
(J) is 500 A/m2. Find the Hall voltage if the sample is 0.003 m wide (w).
f) A classroom has dimensions 20 15 5m3. The reverberation time is 3.5 sec.
Calculate :
i) total absorption in the classroom and
ii) average absorption coefficient.
g) Calculate the mass of proton moving with a velocity 0.8 c. The rest mass of
a proton is 1.67 1027 Kg.
3. Deduce an expression for Lorentz transformations for space and time
co-ordinates.
OR

Discuss the various types of symmetry elements and symmetry operations


present in a cubic crystal.
Set A

SLR-BB 17

-4-

*SLRBB17*

4. Attempt any two of the following :

a) State the acoustic requirements of a good auditorium. Explain how these


requirements can be achieved.
b) Discuss the classification of solids into conductors, semiconductors and
insulators on the basis of energy band theory.
c) Define atomic packing factor. Calculate packing factor for SC, BCC and
FCC.
SECTION II
5. Attempt any five of the following :

15

a) Write applications of nanotechnology in different fields.


b) What are the characteristics of LASER ?
c) Nuclear reactor consumes 10.2 kg of U-235 in 600 hours of operation.
Calculate the power developed by the reactor.
Given : Energy released per fission = 200 MeV
d) Write advantages of optical fiber cable over conducting wires.
e) Explain :
i) Stimulated Emission
ii) Spontaneous Emission.
f) Explain with neat diagram Rayleighs criterion for resolution.
g) Calculate the specific rotation for sugar solution by using following data :
i) Length of the tube = 20 cm
ii) Volume of the solution = 80 cc
iii) Quantity of the sugar in solution = 5 gm
iv) Angle of rotation = 9


#

"

6. Derive an expression for acceptance angle, NA and fractional refractive index


change of an optical fiber.
7. Attempt any two of the following :
a) Nuclear fission can be taken as source of energy. Explain this statement
with reasons.

5
8

b) Write note on : Semiconductor diode laser


c) State and explain : Malus Law.
_____________________

Set A

SLR-BB 170

*SLRBB170*
S

S.E. (Electronics and Telecommunication) (Part II) (Old)


Examination, 2014
SIGNALS AND SYSTEMS
Day and Date : Saturday, 29-11-2014
Time : 3.00 p.m. to 6.00 p.m.

Max. Marks : 100

Instructions : 1) Figures to the right indicate max marks.


2) Assume suitable data if necessary.
3) Q. No. 1 is compulsory. It should be solved in first 30 minutes in
Answer book Page No. 3. Each question carries one mark.
4) Answer MCQ/Objective type questions on Page No. 3 only.
Dont forget to mention, Q.P. Set (A/B/C/D) on Top of Page.
Duration : 30 Minutes

MCQ/Objective Type Questions

Marks : 20

1. Choose the correct answer :


1) With

(201=20)

(t) as unit impulse response sequence, x(t)




(t t0) is __________

b) x(t0)
c) x(t0) (t t0)
2) Consider the signal x(t) = ejt cost, it is ______________
b) periodic with
a) periodic with period 2
c) a periodic
d) none of these
a) x(t)

(t)

d) x(t t0)

3) If x(t) is an odd signal then which of the following statement is true ?


a) x(t) = 0
b) x(t) = x(t)
c) x(0) = 0
d) none of these
4) If two LTI systems with impulse response h1(t) and h2(t) are connected in cascade then
impulse response is ______________
b) h1(t) h2(t)
c) h1(t)* h2(t)
d) none of these
a) h1(t) + h2(t)
5) Which of the following is best suited system for y(n) = cos (x(n)) ?
a) static, linear, time variant, stable
b) dynamic, linear, time invariant, causal, stable
c) static, non linear, time invariant, causal, stable
d) static, linear, time invariant, nonstable
6) A system processing a property of superposition is __________
a) Time varying
b) Time invariant
c) Non causal
d) Linear
7) The convolution in time domain is multiplication in ________________
a) Z domain
b) F domain
c) S domain
d) all
8) A ramp signal x(n) can be defined mathematically as __________
b) nu(n)
c) u(n)/n
d) n + u(n)
a) n (n)


P.T.O.

SLR-BB 170

*SLRBB170*

-2-

9) The series interconnection of two linear time invariant system is itself __________
a) linear, invariant
b) non linear, invariant
c) linear, variant
d) nonlinear, variant
10) The system y(t) = x(t 4) is invertible, the inverse system is ____________
a) y(t + 4)
b) y(t 4)
c) y(t2 + 2)
d) y(t2 2)
11) Antialiasing filter rejects
a) small frequency before sampling
c) small frequency after sampling

b) high frequency before sampling


d) high frequency after sampling

12) For a signal x(t) = 1 + cos 20 t + cos60


a) 20 Hz
b) 40Hz


t Niquisit rate is
c) 120 Hz

d) 60 Hz

13) The Fourier series expansion of an odd periodic function contain


a) only cosine terms
b) only sine terms
c) both cosine and sine
d) only sine term with constant
14) For convergence of Fourier series over any period, x(t) must be satisfying the following
condition
a)

b)


c)

d) none of these


6
J

15) If x(t) and y(t) are periodic with period T, then




a) x(t) y(t)

hk =

>

l


b) x(t) y(t)

hk =

c) x(t) y(t)


hk =

>

l


>

l


d) x(t) y(t)

hk =

>

16) The product of even and odd signal is


a) even signal
b) odd signal




c) even and odd signal

d) none of these

17) Aliasing occurs when a signal of band width W is sampled with sampling frequency F
a) Greater than W
b) Less than W
c) Greater than 2W d) Less than 2W
18) x(n)

X(z) then z [nkx(n)] is




a)

b)

d


c)

d)

d


20) If x(n)

a) entire z-plane
c) entire z, except z = 0, z =




X


, ROC is


d


a)

19) If

b) entire z-plane except z =


d) entire z-plane except z = 0


, then Fourier transform of

is

b)

c)
______________


d)

Set A

*SLRBB170*
S

SLR-BB 170

-3-

S.E. (Electronics and Telecommunication) (Part II) (Old)


Examination, 2014
SIGNALS AND SYSTEMS
Day and Date : Saturday, 29-11-2014
Time : 3.00 p.m. to 6.00 p.m.

Marks : 80

Instructions : 1) Figure to the right indicate max marks.


2) Assume suitable data if necessary.
SECTION I
2. Attempt any four :
(45=20)
a) Show that if x1(n) is an odd signal and x2(n) is even signal, then x1(n) x2(n) is an odd signal.
b) Find the even and odd components of each of the following signals.
1) X(t) = e2t cost
2) x(t) = cost + sint + sint cost.
c) Check whether following signals are Energy Signals or Power Signals
1) X(t) = e10t u(t)
2) x(t) = rect(t) cos 2 t.
d) Find which of the following signals are causal or non causal ?


1) x(t) = sinc(t)
2) x(n) = (1/2)n u(n + 3).
e) Find the step response of the following system where impulse response is given by
h(t) = e3t u(t) e2t u(t).
3. Attempt any two :

(210=20)
!

a) For a given system


@

"

@
@

Check whether system is


a) Static/dynamic
b) Linear/nonlinear
c) Causal/non causal
d) Time invariant/variant.
b) Obtain the convolution of two continuous time signals. Also sketch the result
x(t) = e2t u(t) and h(t) = u(t + 2).
c) Obtain the output of discrete time LTI system with input x(n) and unit impulse response
h(n) given below
x(n) = u(n + 1) and h(n) = (1/5)n u(n).
State whether above system is causal or not. Justify your answer.

Set A

SLR-BB 170

-4-

*SLRBB170*

SECTION II
4. Attempt any four :

(45=20)

a) i) Define Sampling theorem


ii) Aliasing error.
b) Fourier Transform of DT sequence is a special case of Z-transform. Explain.
c) Determine DT sequence for
ROC given as ; / Z/ > 1.

d) Find the FT for


x(t) = eat U (t).
Draw the magnitude spectrum.
e) Prove the convolution property of Fourier Transform.
5. Attempt any 2 :

(210=20)

a) State and explain the necessary and sufficient conditions of existence of Fourier Series
representation for a signal in detail.
b) For LTI system input and its impulse response is given by
x(t) = ebt U(t) ; b > 0
h(t) = eat U(t) ; a > 0; a


Find the response of LTI system both in frequency and time domain.
c) Find FT of
x(t) = ea/t/

a>0

Draw the magnitude spectrum.


_____________________

Set A

SLR-BB 171

*SLRBB171*
Seat
No.

Set

S.E. (Electronics and Telecommunication Engineering) (Part II) (New) Examination, 2014
ELECTRONICS CIRCUIT ANALYSIS AND DESIGN II
Day and Date : Tuesday, 25-11-2014
Time : 3.00 p.m. to 6.00 p.m.

Total Marks : 100

Instructions : 1)
2)
3)
4)

Figures to the right indicate full marks.


Assume suitable data if necessary.
Use of data sheet and calculator is allowed.
Q. No. 1 is compulsory. It should be solved in first 30 minutes in
Answer book Page No. 3. Each question carries one mark.
5) Answer MCQ/Objective type questions on Page No. 3 only. Dont
forget to mention, Q.P. Set (A/B/C/D) on Top of Page.
MCQ/Objective Type Questions

Duration : 30 Minutes

Marks : 20

1. Multiple choice questions :

20

1) An amplifier circuit has an overall current gain of 100 and an input resistance of 10 K
with a load resistance of 1 K . The overall voltage gain of the amplifier is _________
a) 5 dB
b) 10 dB
c) 20 dB
d) 40 dB
2) In common emitter amplifier resistor is added in emitter _________
a) To increase input resistance
b) To increase voltage gain
c) To decrease stability
d) None of the above
3) For a transconductance amplifier input and output resistances are respectively
a) Infinity and zero
b) Zero and infinity c) Zero and zero
d) Infinity and infinity
4) Calculate the harmonic distortion component for an o/p signal having a fundamental
amplitude of 3 V of a second harmonic amplitude of 0.25 V
a) 3.83%
b) 38.3%
c) 83.3%
d) 8.33%
5) Which amplifier is commonly used as a frequency multiplier ?
a) Class A
b) Class B
c) Class C

d) All of the above

6) A cascaded amplifier comprises N identical non-interacting stages, each having a lower


3 dB frequency of fL. If fL is the lower 3 dB frequency of the cascaded amplifier, then
which one of the following is correct ?

a) f L = f L

b) f L = f L 2 N 1

c) f L = f L

2N 1

d) f L = f L N

7) The maximum efficiency of the amplifier shown in figure is

a) about 25%

b) about 78%

c) 70%

d) 100%
P.T.O.

SLR-BB 171

*SLRBB171*

-2-

8) Which of the following is/are the result of negative feedback in a circuit ?


a) lower o/p impedance
b) reduced noise
c) more linear operation
d) all of the above
9) Crystal oscillators are often used for providing
a) Square wave output
b) Triangular wave output
c) Stable carrier wave in radio transmitter d) Frequency multiplication
10) Wien bridge oscillator is most often used whenever
a) Wide range of high purity sign waves is to be generated
b) High feedback ratio is needed
c) Square wave output waves are required
d) Extremely high resonant frequencies are required
11) IC 74123 is
a) Non retriggerable astable multivibrator b) Retriggerable astable multivibrator
c) Retriggerable monostable multivibrator d) None of above
12) Reference voltage for adjustable voltage regulator using LM 317 is
a) 1.52 V
b) 0.125 V
c) 1.25 V
d) None of above
13) The output voltage of LM 317 as adjustable regulator is
R

a) V0 = Vref 1 + 20 R + Iadj R 2
1

b) V0 = Vref 1 + 1 R + Iadj R 2

d) V0 = Vref 2 R + Iadj R 2

1
14) The percentage change in output voltage for given change in input voltage and variation
in load current respectively are
a) Load regulation, line regulation
b) Thermal regulation
c) Line regulation and load regulation
d) Line and line regulation
c) V0 = Vref (R 2 + R1 ) + Iadj R 2

15) Astable multivibrator can be used as


a) Square wave converter
c) Voltage to frequency converter

b) Square wave generator


d) Both b) and c)

16) Transistor used in series regulator has to be a power transistor because


a) It carries entire load current
b) It is connected in series with the load
c) Both a) and b)
d) None of the above
17) IC 555 can be used to design
a) Ramp generator
c) Voltage to frequency converter

b) Square wave generator


d) All of the above

18) Constant current source used in the design of series regulator is called
a) Pre regulator
b) Inverter
c) Booster
d) None of the above
19) Divider by n network operation is obtained from
a) Astable multivibrator
b) Monostable multivibrator
c) Bistable multivibrator
d) Schmitt trigger
20) Pulse width value for IC 555 timer operates in monostable mode with 2.2 k resistance
and 0.01 F capacitor is
b) 24.42 msec
c) 2.44 msec
d) None of the above
a) 24 sec
______________

Set A

*SLRBB171*

-3-

SLR-BB 171

Seat
No.

S.E. (Electronics and Telecommunication Engineering) (Part II) (New) Examination, 2014
ELECTRONICS CIRCUIT ANALYSIS AND DESIGN II
Day and Date : Tuesday, 25-11-2014
Time : 3.00 p.m. to 6.00 p.m.
Instructions : 1)
2)
3)
4)

Marks : 80

All the questions are compulsory.


Figures to the right indicate full marks.
Assume suitable data if necessary.
Use of data sheet and calculator is allowed.
SECTION I

2. Answer the following (any four) :


(45=20)
1) Explain the difference between RC coupled amplifier and direct coupled amplifier.
2) Input resistance increases and output resistance decreases for voltage series feedback
amplifier, Justify the statement.
3) Calculate Gmf, Rif, Rof for given circuit. Assume hie = 1 K, hfe = 50, hre = hoe = 0.

4) Explain the essential conditions to have sustained oscillations. Explain Colpitts oscillator.
5) Explain with suitable application Class C power amplifier.
3. Answer the following (any two) :

(210=20)

1) For given circuit determine input resistance, voltage gain and output resistance and
feedback factor. What type of feedback is present ? Assume hfe = 100, hie = 2K.

2) Explain class A power amplifier. Derive the efficiency of class A. How its efficiency can
be improved ?
3) Design RC phase shift oscillator for output frequency of oscillations of 10 KHz with load
impedance of 5 K and peak to peak output of 5 V. Use Vcc = 12 V.

Set A

SLR-BB 171

-4-

*SLRBB171*

SECTION II
4. Solve any four :

(45=20)

a) With suitable circuit diagram explain dual regulated power supply.


b) Design a circuit using transistor which will turn on and off a relay in regular interval
of 50 ms ?
c) With suitable waveform describe various time constants associated with transistor and
list out the specifications of switching transistor.
d) Derive the equation for pulse width of monostable multivibrator.
e) With suitable circuit diagram, explain the protection circuits for voltage regulator.
5. Solve any two :

(102=20)

a) With suitable circuit diagram explain the operation and design procedure of series
regulator circuit.
b) Design a circuit using IC 555 which will turn on LED for 50 ms and turn off for 15 ms
and repeating the sequence ?
c) Explain the following ICs by considering important features and pin configuration.
i) 78XX and 79XX
ii) 317
iii) 74121.
_____________________

Set A

SLR-BB 172

*SLRBB172*
Seat
No.

Set

S.E. (E&TC) (Part II) (New) Examination, 2014


ANALOG COMMUNICATION
Day and Date : Wednesday, 26-11-2014
Time : 3.00 p.m. to 6.00 p.m.

Total Marks : 100

Instructions : 1)
2)
3)
4)

All questions are compulsory.


Figures to the right indicate full marks.
Assume suitable data if necessary.
Q. No. I is compulsory. It should be solved in first 30 minutes
in Answer book Page No. 3. Each question carries one mark.
5) Answer MCQ/Objective type questions on Page No. 3
only. Dont forget to mention, Q.P. Set (A/B/C/D) on Top
of Page.
MCQ/Objective Type Questions

Duration : 30 Minutes

Marks : 20

I. Choose one of the correct alternatives :

20

1) An amplitude modulation performs the mathematical operation of


a) Addition
b) Multiplication c) Subtraction
d) Division
2) In AM, Modulation index is given by
a) Vc/Vm
b) Vm/Vc

c) Vm + Vc

d) Vc Vm

3) A 1000 KHz carrier is simultaneously modulated with 300 Hz, 800 Hz and
2 KHz audio sine waves. What is the bandwidth of o/p AM signal ?
a) 600 Hz
b) 2000 KHz
c) 1600 Hz
d) 4 KHz
4) Good selectivity usually means __________ Bandwidth.
a) Wide
b) Narrow
c) Phase
d) None of these
5) A 100 MHz carrier is deviated 50 KHz by 4 KHz signal. The modulation
index is
a) 5
b) 8
c) 12.5
d) 20
6) The ionosphere causes radio signals to be
a) Diffused
b) Absorbed
c) Refracted

d) Reflected

7) Radio communication is possible only due to __________ transmission.


a) Single hop
b) Dual hop
c) Multi hop
d) None of above

P.T.O.

SLR-BB 172

*SLRBB172*

-2-

8) Which one of the following is not type of telephone exchange ?


a) Stronger switched exchange
b) Crossbar switched exchange
c) Crowbar switched exchange
d) Electronic switched exchange
9) In FM, modulation signal frequency is doubled, then frequency deviation is
a) Doubled
b) Halved
c) Remains same
d) None of the above
10) Balanced modulator is used in __________ generation.
a) AM
b) DSBSC
c) SSB

d) b) and c)

11) A receiver that uses only tuned radio amplifiers and detector is known as
__________ receiver.
a) Superheterodyne
b) TRF
c) Ratio
d) Armstrong
12) Over modulation occurs when ___________
b) Vm < Vc
c) Vm = Vc
a) Vm > Vc

d) Vm = Vc = 0

13) In AM, the instantaneous __________ of the carrier varies in accordance


with the amplitude of modulating signal.
a) Frequency
b) Phase
c) Amplitude
d) All of these
14) In AM, modulation index is given by
b) Vm/Vc
a) Vc/Vm

c) Vm + Vc

15) A pre-emphasis circuit is a


a) Low-pass filter
c) High-pass filter

b) Phase-shifter
d) Band pass filter

16) A telephone is
a) Full-duplex

c) Multiplex

d) All of the above

c) Indirect FM

d) Indirect PM

b) Half-duplex

17) The FM produced by PM is called


a) FM
b) PM

d) Vc Vm

18) Indicate which of the following circuits could not demodulate SSB
a) Balanced modulator
b) Product detector
c) Balanced slope detector
d) Phase discriminator
19) A receiver has poor IF selectivity. It will therefore also have poor
a) Blocking
b) Double-spotting
c) Diversity reception
d) Sensitivity
20) _________ antenna is not a wideband.Set ASet A
a) Discone
b) Folded dipole c) Helical
______________

d) Marconi
Set A

*SLRBB172*

-3-

SLR-BB 172

Seat
No.

S.E. (E&TC) (Part II) (New) Examination, 2014


ANALOG COMMUNICATION
Day and Date : Wednesday, 26-11-2014
Time : 3.00 p.m. to 6.00 p.m.

Marks : 80

Instructions : 1) All questions are compulsory.


2) Figures to the right indicate full marks.
3) Assume suitable data if necessary.
SECTION I
II. Answer any four :

(54=20)

a) Define modulation. List and explain the different types of modulation in brief.
b) Describe Time Division Multiplexing (TDM) in detail.
c) Define Noise figure and derive its expression.
d) The antenna current of an AM broadcast transmitter, modulated to a depth of
40 percent by an audio sine wave, is 11 A. It increases to 12 A as result of
simultaneous modulation by another audio sine wave. What is the modulation
index due to this second wave ?
e) Explain Vestigial Sideband System (VSB) in brief.
III. Answer any two :

(102=20)

a) i) List and explain the different types of noises. Also discuss its causes and
effects.
ii) Discuss the phase shift method for SSB generation.
b) With the help of block diagram. Explain the working of balanced modulator
using FET. Also give its mathematical analysis.
c) i) List and describe the characteristics of an AM receiver.
ii) What is Automatic Gain Control (AGC) ? Explain its operation with neat
diagram.
Set A

SLR-BB 172

-4-

*SLRBB172*

SECTION II
IV. Answer any four :

(54=20)

a) Derive the formula for instantaneous value of an FM voltage and define


modulation index.
b) Explain in brief pre-emphasis and de-emphasis.
c) Explain multiple stage switching system used in telephony.
d) Write a short note on Yagi antenna with its characteristics.
e) Differentiate between narrow band and wide band FM.
V. Answer any two :

(102=20)

a) Draw and explain Armstrong method of FM generation with its block diagrams
and vector diagrams.
b) Explain Time Slot Interchange (TSI) system used in telephony. With its diagram.
c) i) Explain the phase discriminator in detail.
ii) Define beam width, radiation resistance and antenna gain.
_____________________

Set A

SLR-BB 173

*SLRBB173*
Seat
No.

Set

S.E. (E and TC) (Part II) (New) Examination, 2014


CONTROL SYSTEMS
Day and Date : Thursday, 27-11-2014
Time : 3.00 p.m. to 6.00 p.m.

Max. Marks : 100

Instructions : 1) Figures to right indicate full marks.


2) Assume suitable data whenever necessary.
3) Q. No. 1 is compulsory. It should be solved in first 30 minutes in Answer
Book Page No. 3. Each question carries one mark.
4) Answer MCQ/Objective type questions on Page No. 3 only. Dont
forget to mention, Q.P. Set (A/B/C/D) on Top of Page.
MCQ/Objective Type Questions
Duration : 30 Minutes

Marks : 20

1. Choose the correct answer :


1) For a system, if the initial conditions are zero it means that the system is
a) Working with zero reference input
b) Working but does not store energy
c) At rest but stores energy
d) At rest and has no energy stored in any of its parts
2) If a system has some poles lying on the imaginary axis, it is
a) Absolutely stable
b) Unstable
c) Conditionally stable
d) Marginally stable
3) The resolution of a potentiometer should be
a) Zero
b) Low
c) High

d) Infinite

4) Principal drawback of a servo motor is that it


a) has low reliability
b) has low starling torque
c) develops commutation problem
d) can handle only light loads
5) The transfer function of a system is defined as the ratio of output to input in
a) Z-transform
b) Fourier transform
c) Laplace transform
d) Single algebraic equation
6) A mathematical model is linear if the differentional equation describing it has coefficients
which are
a) Zero
b) Constant
c) Functions only of the independent variable
d) b) or c)
7) When all the terms in a row of Routh array are zero, the characteristic equation has
a) roots on positive real axis
b) roots on negative real axis
c) roots on imaginary real axis
d) none of these
8) The difference of the reference input and the actual output signal is called
a) error signal
b) controlling signal
c) actuating signal
d) none of these
9) The mass element has the transfer function
b) 1/ms
c) 1/m2s
a) 1/ms2

d) none of these
P.T.O.

SLR-BB 173

*SLRBB173*

-2-

10) The pitch and bank command are used in


a) Liquid level control system
b) Position control system
c) Automatic aircraft landing control system
d) None of these
11) A system has single pole at origin. Its impulse response will be __________
a) Constant
b) Ramp
c) Decaying exponential
d) Oscillatory
12) A feedback control system has a transfer function given by
F(s) =

6(s + 1) (s + 6)
3

. It is a ______________

s (s + 2) (s + 4)

a) Type 0 system

b) Type 2 system

c) Type 3 system

d) Type 5 system

13) The velocity error constant of a stable canonical feedback system is given by
______________

G(s)
a) slim
0

b) slim
s G(s)
0

c) slim
s2 G(s)
0

d) none

14) What is the center of asymptotes for a open loop transfer function given by ________
G(s) = 3(s+2) .
s2 ( s + 4 )
a) 1

b) 3

c) 4

d) 0

1000
15) The transfer function of a system is (1+0.1s) (1+0.01s ) . The corner frequencies are ___________
a) 0.1 and 0.01
b) 0.1 and 0.01 c) 10 and 100
d) none of these
16) The inductance is not being used to fabricate a lag network because ________
a) It has high reactance
b) Its size is big
c) It produces time delay and hysteresis loss
d) None of above
17) The lead lag compensation will improve __________
a) Transient response
b) Steady state response
c) transient and steady state response
d) None of above
18) The Bode plot of the transfer function G(s) = s is ____________
a) Zero magnitude and zero phase shift
b) 20 db/decade and phase shift

c) 20 db/decade and phase shift 2


d) 20 db/decade and phase shift 2
19) In terms of Bode plot, the system is stable if ___________
a) PM = GM
b) PM and GM both positive
c) PM negative but GM positive
d) PM and GM both negative
20) The number of poles is P and number of zeros in Z. In root locus, number of loci going to
infinity is ___________
a) P + Z

b) P Z

c) PZ
P
______________

d) P+ Z
PZ
Set A

*SLRBB173*

SLR-BB 173

-3-

Seat
No.

S.E. (E and TC) (Part II) (New) Examination, 2014


CONTROL SYSTEMS
Day and Date : Thursday, 27-11-2014
Time : 3.00 p.m. to 6.00 p.m.

Marks : 80

Instructions : 1) Figures to right indicate full marks.


2) Assume suitable data whenever necessary.
SECTION I
2. Solve any four :

20

a) Explain liquid level control system.


b) Explain concept of absolute stability and conditional stability.
c) Explain elements of mechanical systems.
d) Explain Masons Gain Formula.
e) Compare open loop control system with close loop control system.
3. By using block diagram reduction technique obtain :
i) C(s)/R(s)

ii) C(s)/X(s)

iii) C(s)/Y(s)

10
iv) Total output C(s).

4. Attempt any one :

10

a) Explain principle, construction, types and application of Tacho-generator.


b) For the system represented by the given equations find the transfer function x5/x1 by the
help of signal flow graph technique.
x2 = a12 x1 + a32 x3 + a42 x4 + a52 x5
x3 = a23 x2
x4 = a34
x5 = a35

x3 + a44
x3 + a45

x4
x4.
Set A

SLR-BB 173

*SLRBB173*

-4-

SECTION II
5. Attempt any four :

(45=20)

a) Explain Lag compensator. Also give limitations of lag compensator.


b) Explain following terms :
i) Asymptotes
ii) Centroid
iii) Break away point iv) Break in point
v) Angle of departure.
c) In frequency response using bode plot, the slope 6 db/octave and the slope 20 db/decade
are one and the same. Why ?
d) Explain the time response of first order system to unit step input.
e) For system shown in figure, determine KP and ess for unit step input.

6. Attempt any two :

(210=20)

a) Consider the unit step response of a unit feedback control system whose open loop transfer
1
function is G(s) = s(s+1) .
Obtain :
i) rise time

ii) peak time

b) Sketch root locus for G(s) H(s) =

iii) settling time

iv) maximum overshoot.

K
.
s( s + 2 ) ( s + 4 )

c) Draw bode polt for the transfer function G(s) =

16(1+0.5 s)
.
s) (1+0.1s)

s2 (1+0.125

From graph determine :


a) Phase crossover frequency
b) Gain crossover frequency
c) PM
d) GM
e) Comment on stability of system.
_____________________

Set A

SLR-BB 174

*SLRBB174*
S

S.E. (Electronics and Telecomm. Engineering) (Part II) (New)


Examination, 2014
LINEAR INTEGRATED CIRCUITS
Day and Date : Friday, 28-11-2014
Time : 3.00 p.m. to 6.00 p.m.
N. B. :

Max. Marks : 100

1) Figures to the right indicate full marks.


2) Assume suitable data wherever necessary.
3) Q. No. 1 is compulsory. It should be solved in first 30
minutes in Answer Book Page No. 3. Each question carries
one mark.
4) Answer MCQ/Objective type questions on Page No. 3
only. Dont forget to mention, Q.P. Set (A/B/C/D) on Top
of Page.

Duration : 30 Minutes

MCQ/Objective Type Questions

Marks : 20

1. Select correct option :


1) Output resistance of 741 C is __________ ohms.
a) zero
b) 50
c) 75

(120=20)
d) none

2) Which amplifier offers best immunity for induced noise signal ?


a) Differential amplifier
b) Non-inverting amplifier
c) Common mode
d) Inverting amplifier
3) For 741C maximum input bias current is _________
a) 500 nA
b) 50 nA
c) 5 nA
d) 50 A


4) The current mirror is used in differential amplifier to _________


a) Increase input resistance
b) Set up constant IE
c) Reduce loading effect
d) Increase current gain
5) Open loop gain of the OP-AMP is very ___________ and bandwidth is very
___________
a) high, high
b) small, high
c) high, small
d) small
6) The gain of the OP-AMP decreases with frequency because of _________
within OP-AMP.
a) Shunt resistance
b) Capacitance
c) CMRR
d) Level shifter
P.T.O.

SLR-BB 174

*SLRBB174*

-2-

7) Voltage follower is a special case of ____________


a) inverting amplifier
b) non-inverting amplifier
c) differential amplifier
d) clipping circuit
8) The type of feedback used in closed loop inverting amplifier is ___________
a) voltage series b) current series c) current shunt d) voltage shunt
9) A comparator followed by an integrator is a __________
a) Schmitt trigger
b) Log amplifier
c) Triangular wave generator
d) Square wave generator
10) I to V converter is special case of _________ amplifier.
a) Non-inverting b) Inverting
c) Differential
d) Scaling
11) Which of below application can be realized in open loop configuration of OP-AMP ?
a) Schmitt trigger
b) Peak detector
c) Zero crossing detector
d) Voltage follower
12) In a sample and hold circuit, OP-AMP is used as ___________
a) Sample circuit
b) Hold circuit
c) Voltage follower
d) Peak detector
13) For audio frequency oscillator components used are
a) crystal
b) LC
c) RC
d) LR
14) A duty cycle is ___________ in saw tooth wave generator.
a) 90%
b) 66%
c) other than 50%
d) 18%
15) A square wave generator is also called as
a) Astable multivibrator
b) Bistable multivibrator
c) Monostable multivibrator
d) None of above
16) Which of below is not a part of analog multiplier ?
a) log amplifier
b) adder
c) level shifter
d) antilog amplifier
17) Half wave precision rectifier can rectify input voltage __________
a) greater than 0.7 V
b) Less than 0.7 V
c) both a and b
d) none
18) PLL can be used as ___________
a) Analog multiplier
b) Frequency multiplier
c) Both a) and b)
d) None
19) An ideal filter has ___________ loss in pass band and _________ loss in
stop band.
a) infinite, infinite b) infinite, zero c) zero, infinite
d) zero, zero
20) For a first order low pass filter if fH is 1 KHz, passband gain is 2 and
C = 0.001 F value of R is ___________
a) 1.59 K
b) 1 K
c) 11.9 K
d) None of above
______________
Set A


*SLRBB174*
S

SLR-BB 174

-3-

S.E. (Electronics and Telecomm. Engineering) (Part II) (New)


Examination, 2014
LINEAR INTEGRATED CIRCUITS
Day and Date : Friday, 28-11-2014
Time : 3.00 p.m. to 6.00 p.m.

Marks : 80

N. B. : 1) Figures to the right indicate full marks.


2) Assume suitable data wherever necessary.
SECTION I
2. Solve any four :

(45=20)

a) Draw and explain equivalent circuit of OP-AMP.


b) Why balancing is required for OP-AMP ? How it is done ?
c) Explain V to I converter with floating load.
d) Explain AC amplifier using OP-AMP.
e) With suitable example, explain the concept of virtual ground.
3. a) Define electrical parameters of OP-AMP and give their typical value
for OP-AMP 741.
b) For voltage series feedback amplifier using OP-AMP 741 C with following
specifications.

10
10

, Rf = 20K , A = 2 105
Ri = 2 M , Ro = 75 , fo = 5Hz
Rl = 1 K

VCC = 15V, Output voltage swing =




13 V

Compute Af, Rif, Rof, fF and VOOT.


OR
c) Explain instrumentation amplifier using OP-AMP and derive expression for
its output voltage. How it can be used for measurement of temperature ?

10
Set A

SLR-BB 174

-4-

*SLRBB174*

SECTION II
4. Solve any four :

(45=20)

a) Explain schmitt trigger using OP-AMP.


b) Draw and explain wien bridge oscillator using OP-AMP.
c) Draw and explain precision half wave rectifier.
d) Explain second order low pass butterworth filter using OP-AMP.
e) Explain positive clipper using OP-AMP.
5. Solve any two :

(210=20)

a) Explain triangular wave generator using OP-AMP. How it can be converted


into sawtooth wave generator ?
b) Explain log amplifier and antilog amplifier using OP-AMP. How it can be used
as analog multiplier ?
c) Explain second order high-pass butter worth filter using OP-AMP. Explain
filter design steps.
_____________________

Set A

SLR-BB 175

*SLRBB175*
Seat
No.

Set

S.E. (E and TC) (Part II) (New) Examination, 2014


SIGNALS AND SYSTEMS
Day and Date : Saturday, 29-11-2014
Time : 3.00 p.m. to 6.00 p.m.
Instructions :

Max. Marks : 100

1) Figures to the right indicates max. marks.


2) Assume suitable data if necessary.
3) Q. No. 1 is compulsory. It should be solved in first 30 minutes in Answer
Book Page No. 3. Each question carries one mark.
4) Answer MCQ/Objective type questions on Page No. 3 only. Dont forget to
mention, Q.P. Set (A/B/C/D) on Top of Page.
MCQ/Objective Type Questions

Duration : 30 Minutes

Marks : 20

1. Choose the correct answer :

(120=20)

1) The series interconnection of two linear time invariant systems are itself _____________
a) Linear, invariant
b) Non linear, invariant
c) Linear, variant
d) Non linear, variant
2) Unit parabolic function is a integration of ___________
a) Unit step
b) Unit Ramp
c) Unit impulse

d) None of these

3) The sinc function oscillates with period _____________


a) 2

b)

4) The solution of

c) 2

d) 3

(t 3)2 (t 3) dt is ______________

a) One

b) Three

c) Zero

d) Infinity

5) Consider fig. shown y(t) and x(t) are related as

a) y(t) = x(t) + x(t+1)


c) y(t) = x(t 1) + x(t)

b) y (t) = x (2t)
d) y(t) = x(t) x (t 1)

6) If impulse response of LTI system is h(t) = (t 5) then output y(t) for input x(t) is _______________
a) x(5)

b) x (t + 5)

c) x (t 5)

7) If x(t) is an odd signal then which of the following statement is true


a) x(t) = 0
b) x(t) = x(t)
c) x(0) = 0

d) x (5)

(t 5)

d) None

8) A system processing a property of superposition, system is ___________


a) Time varying
b) Invariant
c) Non causal
d) Linear
9) If two system with impulse responses are
a) h1(t) * h2(t)
b) h1(t) + h2(t)

c) h1(t) h2(t)

d) h1(t) h2(t)
P.T.O.

SLR-BB 175

*SLRBB175*

-2-

10) Which of the following systems are time invariant ?


a) y(t) = x(2t)
b) y(t) = x(t) + x(t 1)

( )

c) y(t) = x t 2

d) y(t) = x( t)

11) FT of sampled signal is given as


a)

1
T

1
c) T


2m
j

1
b) T


2 m
j

T
m=


2m
j +

T
m =

1
d) T

m= 0

m= 0


2m
j +

12) For a signal n(t) = 1 + cos 10 t + Cos 40 t then the Nyquist rate is
a) 10Hz
b) 20Hz
c) 25Hz

d) 40Hz

13) Dirchlet conditions far periodic signals are in each period.


a) The function x(t) has only finite number of maxima and minima
b) The function x(t) has a finite number of discontinuities
c) The function x(t) is absolutely integrable over one period
d) All of these
14) A periodic signal is said to have a half-wave symmetry when
T

a) x(t) = x t
4

b) x (t) = x t +
4

15) For a continuous time, periodic signals,


a) True

1
T

c) x(t) = x t
2

| x(t ) |2 dt =

k =

d) x(t) = x t
2

| qk |2

b) False

16) If for LTI system, x(t) is i/p periodic signal, y(t) is the response of linear system, h(t) is impulse
response of system then y(t) is given as
a)

h(t ) * x(t ) = H( j) X( j)

b)

h(t ) x(t) = H( j) * X( j)

17) For a non causal signal of infinite duration ROC is


a) Exteriar to circle of radius b
b) Interior to circle of radius b
18)

x(n) X(z), then z nk x(n) is given by


d

a) Z X(z)
dz

b) Z X(z)
dz

d
c) Z
X (z )
dz

d
d) k(z)
dz

19) If x(z) = 1, 2, 1, 2, 3 ., ROC is

a) Entire z- plane
c) Entire z-plance except z = 0, z =

b) Entire z-plane except z =


d) Entire z-plance except z = 0

j 0 n

FT
x(n) is
20) If x(n) X(e j ) then FT e

j
a) X(e 0 )

b) X(e j( + 0 ) )

c) X(e j( 0 ) )

______________

d) X(e

j( 0 )

Set A

*SLRBB175*

SLR-BB 175

-3-

Seat
No.

S.E. (E and TC) (Part II) (New) Examination, 2014


SIGNALS AND SYSTEMS
Day and Date : Saturday, 29-11-2014
Time : 3.00 p.m. to 6.00 p.m.
Instructions :

Marks : 80

1) Figures to the right indicates max. marks.


2) Assume suitable data if necessary.
SECTION I

2. Attempt any four :

(45=20)

a) Show that the condition for stability of DT LTI system is

| h(n) |< .

n=

b) Explain property of invertibility and inverse system.


c) Find the even and odd components of each of the following signals.
1) x(t) = e2t cost
2) x(t) = cost + sint + sint cost
d) Check whether the following systems are linear or not
t

1) y(t ) =

x()d

2) y(n) = 2x(n) +

1
x(n 1)

e) Obtain the Block dia. realization for the system described by the difference equation
y(n) 5 y(n 1) + 1 y(n 2) = x(n) + 2 x(n 1) .
6
6

3. Attempt any two :

(210=20)

a) Consider x(t) and y(t) shown below in fig. and sketch carefully following signals.
1) x(t 1) y (t)
2) y(t) [u(t+1) u(t 1)]
3) x(t 1) u (t + 1)

b) Determine the convolution sum of following sequences.

( )

x(n) = cos n u(n) and h(n) = 1 u(n)


2

Set A

SLR-BB 175

-4-

*SLRBB175*

c) Determine the convolution of following signals and sketch result.


y(t) = 1 for 1 t 3
=0
elsewhere
&
x(t) = 1 + t
for 1 t 0
=1t
=0

for 0 t 1
elsewhere
SECTION II

4. Answer the following any 4 :

(45=20)

a) Explain the reconstruction of the signal in sampling technique.


b) Explain the properties of z transform.
c) Find the ZT and sketch ROC.
x(n) = (0.6)n (n) + (0.4)n (n)
d) Find the Fourier transform of gate transform (t).

e) Explain the convolution property of Fourier transform.


5. Attempt any 2 :

(210=20)

a) i) Determine the impulse response of DT LTI system given by y(n) 5y(n 1) x(n)
ii) What is aliasing error ? How it is taken care of ?
b) i) Find Z-transform of sequence given as x(n) = n2 (n).
ii) For a system to be stable what are the conditions required.

5
5
7
3

c) Find the exponential Fourier series and plot the magnitude and phase spectrum for periodic signal
m(t) given as

_____________________

Set A

SLR-BB 176

*SLRBB176*
Seat
No.

Set

T.E. (E & TC) (Part I) (Old) Examination, 2014


ELECTROMAGNETIC ENGG. AND RADIATING SYSTEM
Day and Date : Wednesday, 3-12-2014
Time : 3.00 p.m. to 6.00 p.m.
N.B. :

Max. Marks : 100

1) Q. No. 1 is compulsory. It should be solved in first


30 minutes in Answer Book Page No. 3. Each question
carries one mark.
2) Answer MCQ/Objective type questions on Page No. 3 only.
Dont forget to mention, Q.P. Set (A/B/C/D) on Top of Page.
MCQ/Objective Type Questions

Duration : 30 Minutes

Marks : 20

1. Choose the correct answer :


1) B is defined as ____________
a) Force per unit current
c) B = 0

20
b) Force per unit charge
d) None

2) Complete the statement H dl =


a) I

b) Q

c) v

d) Zero

3) If the two conductors carry current in opposite direction these will __________
a) Attract each other
b) Repel each other
c) No force
d) None
4) Identify the correct relationship of Gauss law __________
b) D = v

a) = Q
c)

D ds = v

v dv

d) None

5) Inside a conductor ______________


a) Vab is equal to infinite
b) Vab is equal to zero
c) Electric field intensity is zero
d) None
6) Energy stored in electric field is ___________
a)

1 E
2

b)

1 E2
2

c) E2

d) H2
P.T.O.

SLR-BB 176

*SLRBB176*

-2-

7) The electric field intensity of an infinite line charge varies ____________


a)

1
r

b)

1
r2

c)

1
r3

d)

8) The effect of dielectric is to ___________


a) Increase the capacitance
b) Decrease the capacitance
c) Work
d) None
9) The unit of electric flux density is ____________
b) c/m2
c) v/m
d) None
a) v/m2
10) When the divergence is negative, then that region is said to contain ___________
a) Vector
b) Scalar
c) Source
d) Sink
11) If E = 6V/m in free space then conduction current density is
a) 6 A/m2
b) 3 A/m2
c) 0
d) None
12) Gain and effective area of an antenna are ____________ to each other.
a) Proportional directly
b) Inversely proportional
c) Not related
d) None of these
13) In the near field power radiated by antenna ____________
a) Depends on distance
b) Independent of distance
c) Equal to distance
d) None
14) Which of the following is true ?
a) D = 0

b) B = 0

c) D = v

15) What frequency are 20 mm microwaves ?


a) 100 MHz
b) 400 MHz
c) 15 GHz

d) B = 0
d) 73 GHz

16) The directivity is related with beam solid angle ( A) as


a) D = 4 A

b) D =

A
4

c) D =

4
A

d) None of these

17) The radiation resistance of a / 2 dipole is approximately


b) 36.5
a) 37
18) The measure of solid angle is
a) Degrees
b) Radians
19) Unit of relative permeability is
a) Henry
c) Henry/2 meter
20) Poynting vector gives
a) Rate of energy flow
c) Intensity of electric field

c) 292

d) 73

c) Steradian

d) None of these

b) Henry/meter
d) It is dimensionless

b) Direction of polarization
d) Intensity of magnetic field
______________
Set A

*SLRBB176*

-3-

SLR-BB 176

Seat
No.

T.E. (E & TC) (Part I) (Old) Examination, 2014


ELECTROMAGNETIC ENGG. AND RADIATING SYSTEM
Day and Date : Wednesday, 3-12-2014
Time : 3.00 p.m. to 6.00 p.m.
N.B. :

Marks : 80

1) All questions are compulsory.


2) Figure to the right indicates full marks.
SECTION I

2. Attempt any four :

(45=20)

a) Given three points P(2, 3, 1), Q( 4, 2, 6) and R(1, 5, 3). Then find :
a) Unit vector of the vector P to R
b) Distance between P to Q.
b) Prove that E = v .
c) State and prove Gauss theorem.
d) Calculate electric field intensity at a point A (1, 2, 3) in free space caused by a
charge Q1 = 5 nc at point P(2, 3, 5) and another charge Q2 = 4nc at R(3, 0, 3).
e) A circular loop located on x2 + y2 = 16, z= 0 carries a direct current of 20A
along a direction . Find H at (0, 0, 6).
3. Attempt any two :

(210=20)

a) Evaluate both sides of divergence theorem for


D = x 2ax + y 2ay + z 2az over the cube 0 xyz 1.

b) Potential is given by
V = 2 (x + 1)2 (y + 2)2 (z + 3)2 v in free space then calculate
1) Potential
3) D

2) E
4) v at a point A (1,2,3)

c) a) Explain the boundary condition for magnetic field.


b) Explain the Amperes law and state its application.
Set A

SLR-BB 176

-4-

*SLRBB176*

SECTION II
4. Attempt any four :

(54=20)

a) Derive Helmholtz wave equation.


b) Given H = Hm e j( wt + z ) ax A/m in free space. Find E.
c) Explain the principle of radiation of antenna.
d) Write note on parabolic dish antenna.
e) Explain antenna field zones.
5. Solve any two questions :

(102=20)

a) Derive Maxwells equation in point and integral form for harmonically varying
fields. Also write its word statement.
b) Derive the equation for power radiated by short dipole antenna and its radiation
resistance.
Em
c) Do the fields E = Emsinxsint a y and H =
cos x cos t az satisfy Maxwells
0
equation.

_____________________

Set A

SLR-BB 177

*SLRBB177*
S

T.E. (E and TC) (Part I) Examination, 2014


DIGITAL COMMUNICATION (Old)
Day and Date : Friday, 5-12-2014
Time : 3.00 p.m. to 6.00 p.m.

Max. Marks : 100

Instructions : 1) Figures to the right indicate full marks.


2) Assume suitable data if necessary.
3) Q. No. 1 is compulsory. It should be solved in first 30 minutes in
Answer Book Page No. 3. Each question carries one mark.
4) Answer MCQ/Objective type questions on Page No. 3 only. Dont
forget to mention, Q.P. Set (A/B/C/D) on Top of Page.
Duration : 30 Minutes

MCQ/Objective Type Questions

Marks : 20

I. Choose one of the correct alternatives.


1) Redundancy is
a) not desirable in information transmission
b) sometimes deliberately introduced for privacy
c) sometimes deliberately introduced to reduce error
d) none of these
2) The main advantage of PCM is
a) lower b/w
b) lower power

c) lower noise

3) Which of the following method is employed in telephony ?


a) TDM
b) FDM
c) both a) and b)

d) none of these
d) none of the above

4) In all pulse communication system, carrier


a) Is necessarily a high repetition rate train of pulses
b) Is necessarily a high frequency continuous A.C. signal
c) Is either a train of pulses or a continuous A.C. wave
d) None of these
5) The entropy of message source generating 4 messages with probability 0.5, 0.25, 0.25 is
a) 1 bit/message
b) 2 bit/message
c) 1.5 bit/message d) 2.5 bit/message
6) Guard band increases the bandwidth for
a) FDM
b) TDM

c) Both a) and b)

d) None of these

7) In an ADM system, the output signal amplitude for 1s and 0s are


a) fixed and the repetition rate is also fixed
b) fixed but the repetition rate is variable
c) variable and the repetition rate is also variable
d) variable but the repetition rate is fixed
8) The PWM needs
a) more samples per second than PPM
c) more power than PPM

b) more bandwidth than PPM


d) none of these

P.T.O.

SLR-BB 177

*SLRBB177*

-2-

9) The _____________ of the system to timing error is revealed by the rate of closing of
the eye as sampling time is varied.
a) reliability
b) complexity
c) sensitivity
d) output
10) In _____________ equalizers, the error vector is continually estimated during the normal
course of data transmission.
a) Preset
b) Adaptive
c) Transversal
d) None of these
11) In QAM, both identities are varied
a) Amplitude and Phase
c) Bitrate and Phase

b) Frequency and Phase


d) Amplitude and Frequency

12) ___________ is the most affected by noise.


a) PSK
b) ASK
c) FSK

d) MSK

13) For a BPSK scheme, the bit error probability is given by __________
a)

erfc


b)
b

erfc


1
E

2
2
2

c)


erfc

d)

erfc


1
E

2
N

14) If carrier modulated by a digital bit stream had one of the possible phases 0, 90, 180 and
270, then modulation is called
a) BPSK
b) QPSK
c) QAM
d) MSK
15) A correlation receiver consists of
a) a multiplier and integrator
c) multiplier only

b) an integrator only
d) adder and integrator

16) The type of modulation used with direct sequence spread spectrum
a) PSK
b) ASK
c) FSK
d) MSK
17) Design principle of matched filter is to have maximum ___________ at a sampling time T.
a) power
b) PSD
c) SNR
d) all of the above
18) A specified sequence used for frame synchronization is called
a) comma
b) period
c) prefix

d) postfix

19) A major problem of multicarrier modulation is


a) Fading
b) Diversity
c) High peak to average power ratio
d) None of these
20) Synchronous demodulator is an example of ____________ detection.
a) coherent
b) non coherent
c) noisy
d) none of the above
______________

Set A

*SLRBB177*
S

SLR-BB 177

-3-

T.E. (E and TC) (Part I) Examination, 2014


DIGITAL COMMUNICATION (Old)
Day and Date : Friday, 5-12-2014

Marks : 80

Time : 3.00 p.m. to 6.00 p.m.


Instructions : 1) All questions are compulsory.
2) Figures to the right indicate full marks.
3) Assume suitable data if necessary.
SECTION I
II. a) Answer any three :

12

i) Define Rate of Information, Channel capacity, Entropy and Mutual Information.


ii) Explain with a block diagram of digital communication.
iii) Explain working of adaptive equalizer.
iv) Explain Bandwidth and S/N trade-off for channel capacity.
b) A DMS Source and their probabilities are as shown in table.
M

calculate the average code length, entropy and code efficiency by using Huffman coding
algorithm.
III. a) Answer any two :

12

i) With the help of circuit diagram explain PAM modulator and demodulator.
ii) Show that as the bandwidth approaches to


the maximum information rate

Rmax = S/N log2e = 1.44 S/N.


iii) With a block diagram explain DM and brief about noises in DM.
b) Explain TDM-PCM telephone system with a block diagram.

8
Set A

SLR-BB 177

-4-

*SLRBB177*

SECTION II
IV. a) Answer any three :

12

i) Explain with a block-diagram PSK modulation and demodulation method.


ii) What do you mean by M-ary orthogonal signals ?
iii) Write a note on integrate and dump type of coherent receiver.
iv) Explain in detail carrier recovery using Costas Loop method.
b) Write a note on QAM modulation and demodulation.

V. a) Answer any two :

12

i) What are different symbol synchronization methods ? Explain Early-Late gate method
in detail.
ii) Write a note on minimizing peak to average power ratio in multicarrier system.
iii) Derive the expression for probability of error in optimum receiver.
b) Write a note on comparison of digital modulation schemes with respect to bandwidth, power
and equipment complexity.

_____________________

Set A

SLR-BB 178

*SLRBB178*
Seat
No.

Set

T.E. (E&TC) (Part I) (Old) Examination, 2014


SOFTWARE ENGINEERING AND PROJECT MANAGEMENT SYSTEMS
Day and Date : Monday, 8-12-2014
Time : 3.00 p.m. to 6.00 p.m.

Max. Marks : 100

Instructions : 1) Q. No. 1 is compulsory. It should be solved in first 30 minutes


in Answer Book Page No. 3. Each question carries one mark.
2) Answer MCQ/Objective type questions on Page No. 3 only.
Dont forget to mention, Q.P. Set (A/B/C/D) on Top of Page.
Duration : 30 Minutes

MCQ/Objective Type Questions

Marks : 20

1. Choose the correct answer :


(120=20)
1) Which of the following is tool in design phase ?
a) Abstraction
b) Refinement
c) Information hiding
d) All above
2) Release testing is also known as
a) Functional testing
b) Regression testing
c) Alfa testing
d) None
3) Advantages of CBSE
a) Reduced software cost
b) Faster delivery
c) Reduced risk
d) All
4) Software specification is the process of
a) Understanding and defining the services which are required
b) Identifying constraints on system
c) Both a) and b)
d) None
5) The goals of software testing leads to
a) Defect testing
b) Violation testing
c) Both a) and b)
d) None
6) UML stands for
a) Unified Modified Language
b) Unified Monitoring Language
c) United Modeling Language
d) None of these
7) State-Transition diagram is
a) A stimulus-response model
b) Data-flow model
c) Architectural model
d) Composition model
P.T.O.

SLR-BB 178

-2-

*SLRBB178*

8) Data flow and state machine models are types of


a) Context models
b) Behavioural models
c) Data models
d) Object models
9) Functional requirement specification of a system should be
a) Complete
b) Consistent
c) Both a) and b) d) None of these
10) In object oriented design of software, objects have
a) Attributes and names
b) Attributes and operations
c) Attributes, names and operations d) None of these
11) Any delay in __________ activities will cause a delay in completion date of
the project.
a) free float
b) base lined
c) critical path
d) scheduled
12) Important criteria to become programme manager is
a) Expert in technical
b) Good at management skill
c) Experience as project manager
d) All
13) The highest level in a typical project reporting structure is
a) Project manager
b) Steering committee
c) Team leader
d) Human Resource Officer
14) _________ provides a more striking visual indication of those activities that
are not progressing to schedule.
a) Slip chart
b) Time line
c) Gantt chart
d) Ball chart
15) The person handles many projects simultaneously.
a) Programme manager
b) Project manager
c) Team leader
d) Software developer
16) __________ is carried out to calculate the latest dates on which each activity
may be started and completed.
a) backward pass b) forward pass c) end pass
d) none
17) Sponsoring group will get enough information from _________
a) Program mandate
b) Program brief
c) Blue print
d) Vision statement
18) The Gantt chart is used for
a) Tracking project progress
b) Knowing the activities
c) Knowing starting date of project d) Knowing end date of project
19) The base line budget is based on ______________
a) activity network
b) Cost monitoring
c) Proposed plan
d) None
20) The diagram showing for each product which other products are required as
input is called __________ diagram.
a) Product based b) Precedence c) Product driven d) Project process
______________
Set A

*SLRBB178*

-3-

SLR-BB 178

Seat
No.

T.E. (E&TC) (Part I) (Old) Examination, 2014


SOFTWARE ENGINEERING AND PROJECT MANAGEMENT SYSTEMS
Day and Date : Monday, 8-12-2014

Marks : 80

Time : 3.00 p.m. to 6.00 p.m.

SECTION I
2. Answer any four :

(45=20)

a) Write in briefly the inheritance model.


b) Give types of system models that you might create during analysis process.
c) List and explain characteristics of software process.
d) Explain object aggregation model.
e) Explain software validation.
3. a) Explain RUP (Rational Unified Process) in detail.

(18=8)

OR
a) Explain v model in detail.
b) Explain release testing in detail.

(18=8)

c) Explain path testing in short.

(14=4)

Set A

SLR-BB 178

-4-

*SLRBB178*

SECTION II
4. A) Attempt any two :

(26=12)

i) Explain product based approach for identifying activities.


ii) Write a short note on forward pass for activity planning.
iii) Explain Gantt chart to visualize progress of project.
B) Solve all :

(24=8)

i) What are different steps in constructing precedence network model ?


ii) What are steps for planning risk ?
5. A) Solve any one :

(18=8)

a) Draw and explain stepwise project planning.


b) What is change control ? Discuss the change control procedures.
B) Attempt all :

(34=12)

i) Define critical path and give its significance.


ii) Explain different cost benefit analysis methods.
iii) What is project ? What are different activities of management ?
_____________________

Set A

SLR-BB 179

*SLRBB179*
Seat
No.

Set

T.E. (Part I) (E & TC) (Old) Examination, 2014


DIGITAL SIGNAL PROCESSING
Day and Date : Wednesday, 10-12-2014
Time : 3.00 p.m. to 6.00 p.m.

Max. Marks : 100

Instructions : 1) Assume suitable data if necessary.


2) Q. No. 1 is compulsory. It should be solved in first 30 minutes
in Answer Book Page No. 3. Each question carries one mark.
3) Answer MCQ/Objective type questions on Page No. 3 only.
Dont forget to mention, Q.P. Set (A/B/C/D) on Top of Page.
MCQ/Objective Type Questions
Duration : 30 Minutes

Marks : 20

1. Multiple choice questions. Choose correct option :


(201=20)
1) A signal has
a) single value for single time instant
b) single value for multiple time instant
c) can have any number of values for any number of time instant
d) none
2) The range of values of a for which the system with impulse response
h(n) = an u(n) is stable is
a) |a| > 1
b) |a| < 1
c) a > 0
d) a < 0
3) Determine the convolution sum of two sequences x(n) = {3, 2, 1, 2} and
h(n) = {1, 2, 1, 2}.
a) y(n) = {3, 8, 8, 12, 9, 4, 4}
b) y(n) = {3, 0, 3, 12, 9, 4, 4}
c) y(n) = {3, 8, 8, 12, 9, 1, 4}
d) y(n) = {3, 8, 2, 1, 9, 4, 4}
4) The FFT algorithms
a) eliminate the redundant calculation and enable to analyze the spectral
properties of a signal
b) enable the redundant calculation and redundant to analyze the spectral
properties of a signal
c) a) and b)
d) none
5) Fast convolution techniques
a) overlap save
b) overlap add c) a) and b)
d) none of above
6) Calculate DFT of x(n) = {1, 0, 1, 0}.
a) x(k) = {2, 0, 2, 0}
b) x(k) = {1, 0, 1, 0}
c) x(k) = {2, 0, 1, 0}
d) none
P.T.O.

SLR-BB 179

-2-

*SLRBB179*

7) Transform domain is used as it


a) enhances the signal
b) compress the signal
c) reduces the data points
d) increases the data points
8) When length of DFT is 32 then real time multiplications and real time additions
for radix 2 algorithm are
a) 88,408
b) 24,152
c) 264,1302
d) 204,972
9) Number of delay elements are required for direct form I realization of
y(n) = 0.5y(n 1) 0.25y(n 2) + x(n) + 0.4x(n 1) are
a) 1
b) 2
c) 3
d) 4
10) Correlation is a measure of ________________ between two signal sequences.
a) similarity
b) difference
c) associativity d) several
11) VLIW architecture differs from conventional P-DSP in which of the following
aspects ?
a) instruction cache
b) number of functional units
c) use of piplelining
d) none of the above
12) The addressing mode that is convenient for FFT computation is
a) Indirect addressing
b) Circular mode addressing
c) Bit reversed addressing
d) Memory mapped addressing
13) Non linearity in the relationship between and is known as
a) Aliasing
b) Frequency warping
c) Unwarping
d) Frequency mixing
14) In Bilinear transformation the relation between and is
a) = T
b) = /T
c) = tan( T)
d) = (2/T) tan( /2)
15) The frequency mapping from s domain to z domain using impulse invariant
technique is
a) Many to many b) Many to one c) One to many d) None of above
16) In impulse invariant technique the pole at s = a gets mapped in Z domain to
a) Z = aT
b) Z = epiT
c) Z = eaT
d) Z = eaT
17) The approximate width of the main lobe in Hanning window of length M is
a) 8 /M
b) 4 /M
c) 12 /M
d) 16 /M
18) The FIR filter of order M gives linear phase response if
b) h(n) = h( n) c) h(n) = h(M 1 n) d) none of these
a) h(n) <
19) For getting the linear phase response in DSP we use
a) Butterworth filters b) IIR filters c) Convolution
d) FIR filters
20) To get the Linear phase characteristics from filters the poles must be
a) In reciprocal pair
b) Inside the unit circle in z plane
c) In complex conjugate pairs
d) None of these
______________
Set A

*SLRBB179*

-3-

SLR-BB 179

Seat
No.

T.E. (Part I) (E & TC) (Old) Examination, 2014


DIGITAL SIGNAL PROCESSING
Day and Date : Wednesday, 10-12-2014
Time : 3.00 p.m. to 6.00 p.m.

Marks : 80

Instructions : 1) All questions are compulsory.


2) Assume suitable data if necessary.
SECTION I
2. Solve any four of the followings :

(45=20)

a) Obtain the autocorrelation for DT sequence and also sketch the result
x[n] = {0, 1, 2, 3, 4}.

b) State and prove time reversal property of DFT of a DT signal.


c) Find N point DFT of DT signal given below :
x[n] = an u(n) for all n.
d) Draw the parallel realization for a filter characterized by transfer function
H[z] =

z1 + 4z2
.
5 2z1 + 0.15z2

e) State and explain properties of ROC of z transform.


3. Solve any two of the followings :

(210=20)

a) List out the similarities and differences between DIT and DIF algorithms. Find
IDFT of the sequence X(k) = {4, 1-j2.414, 0, 1-j0.414, 0, 1+j0.414, 1+j2.414}
using DIF.
b) Obtain Direct form I, Direct form II, cascade and parallel realizations for the
system y(n) = 0.1y(n 1) + 0.2y(n 2) + 3x(n) + 3.6x(n 1) + 0.6x(n 2).
c) Compute 8 point circular convolution of x1(n) = {1, 1, 1, 1, 0, 0, 0, 0} and
x2(n) = sin(3 n)/8 0 n 7.
Set A

SLR-BB 179

-4-

*SLRBB179*

SECTION II
4. Attempt any four :

(46=24)

a) Convert the analog filter to digital filter whose system function is


H(s) =

s + 0.1
(s + 0.1)2 + 9

The digital filter should have a resonant frequency of wr = /4. Use Bilinear
transformation.
b) Describe the applications of DSP in various fields.
c) Explain the Windowing technique for FIR filter design along with different
window functions.
d) Design the first order high pass filter with fc = 1 kHz and sampling frequency
fs = 10000 sps. Use bilinear transformation method.
e) Explain in brief the basic building blocks of digital signal processor.
5. Attempt any two :

(28=16)

a) Explain the impulse invariant technique in detail.


b) Explain the effects of finite word length on the design of IIR filters.
c) A low pass filter is to be designed with the desired frequency response as

( )

Hd e

e j3
=

0 / 2
.

Determine the filter coefficients h(n) for M = 7, using frequency sampling


method.
_____________________

Set A

SLR-BB 18

*SLRBB18*
Seat
No.

Set

F.E. (Part I) Examination, 2014


ENGINEERING CHEMISTRY (New)
Day and Date : Friday, 19-12-2014
Time : 10.00 a.m. to 1.00 p.m.

Total Marks : 70

Instructions : i) Q. No. 1 is compulsory. It should be solved in first 30 minutes


in Answer Book Page No. 3. Each question carries one mark.
ii) Answer MCQ/Objective type questions on Page No. 3 only.
Dont forget to mention, Q.P. Set (A/B/C/D) on Top of Page.
MCQ/Objective Type Questions
Duration : 30 Minutes

Marks : 14

1. Choose correct option :

14

1) Temporary hardness in water is caused by the presence of


A) Sodium chloride
B) Calcium sulphate
C) Magnesium sulphate
D) Calcium bicarbonate
2) The flow of water through a membrane from saline water into fresh water
take place when the applied pressure on saline water is
A) higher than osmotic pressure
B) lower than osmotic pressure
C) equal to osmotic pressure
D) zero
3) Gibbs phase rule is
A) F P = C + 2
C) F C = P + 2

B) F + P = C + 2
D) P + C = F + 2

4) When an oil is cooled and the temperature at which it ceases to flow is known
as
A) Flash point
B) Fire point
C) Cloud point
D) Pour point
5) Capacity of an oil to stick onto the surface of machine parts under
conditions of heavy load is called
A) Volatility
B) Oiliness
C) Acid value
D) Flash point
6) Electrical resistance of a corroding metallic wire
A) Increases
B) Decreases
C) Remains constant
D) None of these
P.T.O.

SLR-BB 18

*SLRBB18*

-2-

7) Viscosity of paint can be reduced by


A) Pigment

B) Drier

C) Fillers

D) Thinner

8) In the proximate of the coal, the moisture is determined at a temperature of


A) 50C
B) 105C110C
C) 200C

D) Above 200C

9) Natural rubber is basically a polymer of


A) Propylene
C) Ethylene

B) Isoprene
D) Propane

10) The fibre obtained by condensation of hexamethylene diamine and adipic


acid is
A) Dacron
C) Rayon

B) Nylon
D) Terylene

11) Maximum percentage of compound constituent in Portland cement is of


A) 3 CaO SiO2

B) 2 CaO SiO2

C) 3 CaO Al203

D) 4 CaO Al203 Fe203

12) A propellant used in rocket engine is


A) A fuel
B) An oxidising agent
C) Both A) and B)

D) None of these

13) Glass with low thermal co-efficient and high chemical resistance is
A) Soda glass
C) Flint glass

B) Hand glass
D) Boro silicate glass

14) The technique by which a mixture is separated into its constituents by a


moving gas phase passing over a stationary phase is
A) Column chromatography
B) Gas chromatography
C) Spectrometry
D) Ion-exchange chromatography
______________

Set A

*SLRBB18*

-3-

SLR-BB 18

Seat
No.

F.E. (Part I) Examination, 2014


ENGINEERING CHEMISTRY (New)
Day and Date : Friday, 19-12-2014
Time : 10.00 a.m. to 1.00 p.m.

Marks : 56

Instructions : i) Attempt all questions.


ii) Draw neat diagram wherever necessary.
iii) Figures to right indicate full marks.
SECTION I
2. A) Attempt any two :
a) Explain the Zeolite process for softening of hard water.
b) Describe water system with phase diagram.
c) A sample of water on analysis was found to contain following impurities
in ppm.
Impurities
Amount
Mole.wt
Ca(HCO3)2
8.5
162
Mg(HCO3)2
15.2
146
CaSO4
12.4
136
MgCl2
6.8
95
Calculate temporary, permanent and total hardness of water.
B) Attempt any two :

a) Explain thin film (boundary) lubrication.


b) Explain factors affecting rate of corrosion.
c) What is varnish ? Write the characteristics of good varnish.
3. A) i) How will you select lubricant for I C engine and delicate instruments ?

ii) Write a note on disinfection of water by chlorine.

B) Attempt any two :


i) Describe the weight loss and electrical resistance method of measurement
of rate of corrosion.
ii) Define :
a) Viscosity
b) Viscosity Index c) Aniline point
d) Oiliness.
iii) Define COD. Explain the experimental determination of COD.

4
4
4

Set A

SLR-BB 18

*SLRBB18*

-4-

SECTION II
4. A) Attempt any two :
i) Write a note on proximate analysis of a coal with its significance.
ii) Draw a neat labelled diagram of gas liquid chromatography.
iii) A bomb calorimeter experiment gave the following data :
1) Wt. of coal sample = 1.2 gm
2) Wt. of water taken = 1500 gm
3) Water equivalent of calorimeter = 500 gm
4) Observed rise the temperature = 2.40C
5) Cooling correction = 0.051C
6) Correction due to acid = 51.8 cal.
7) Fuse wire correction = 3.8 cal.
8) % H = 5.0%
9) Latent heat of steam = 587 Cal/gm.

8
4
4
4

Calculate higher and lower calorific value of coal sample.


B) Attempt any two :
Write a note on :
i) Low carbon and high carbon steel.
ii) Liquid and solid propellant.
iii) Chemical composition of Portland cement.

6
3
3
3

5. A) i) What is the wt. of MgCO3 required to prepare 0.15 N and 0.12 M solution
of 500 ml of Mg CO3 solution.
(Mol.wt. MgCO3 = 84).
ii) A polymer of poly styrene (Mol. wt. 25,000) was obtained from polymerization
of 104 gm of styrene. Calculate the degree of polymerization (DP)
CH=CH2
n

CH CH2

Styrene

n Polystyrene

OR
A) Write a note on :
a) Conducting polymery.
b) Epoxy Resin.

6
4
2

B) Attempt any two :


i) Write differences between natural and synthetic rubber.
ii) Write a note on safety glass and optical glass.
iii) Explain extrusion method for moulding of plastic into articles.
_____________________

8
4
4
4
Set A

SLR-BB 180

*SLRBB180*
Seat
No.

Set

T.E. (Electronics and Telecommunication Engg.) (Part I)


Examination, 2014
MICROPROCESSOR AND PERIPHERALS (Old)
Day and Date : Friday, 12-12-2014
Time : 3.00 p.m. to 6.00 p.m.

Max. Marks : 100

Instructions : 1) Figures to the right indicate full marks.


2) Draw neat diagrams where required.
3) Q. No. 1 is compulsory. It should be solved in first 30 minutes
in Answer Book Page No. 3. Each question carries one mark.
4) Answer MCQ/Objective type questions on Page No. 3 only.
Dont forget to mention, Q.P. Set (A/B/C/D) on Top of Page.
MCQ/Objective Type Questions
Duration : 30 Minutes

Marks : 20

1. Choose the correct answer :


20
1) The contents of accumulator are cleared with the instruction
a) CMA
b) SUB A
c) XRA A
d) Both b) and c)
2) The contents of accumulator after execution of the following program are
LXIH, C200H
MOV B, A
MOV A, H
HLT
a) XX H
b) 00H
c) C2H
d) Contents of B
3) The bits of SIM format required to generate square wave at SOD pins are
a) D7
b) D6
c) D4 and D3
d) Both a) and b)
4) To bring microprocessor out of halt state ___________
a) Reset the system
b) interrupt the system
c) Both a) and b)
d) Switch off the power supply
5) The user defined flags are
a) Zero
b) AC
c) Parity
d) Both a) and c)
6) RST 7.5 interrupt is having __________ priority than TRAP and __________
priority than __________ RST 5.5 respectively.
a) Less, less
b) More, more
c) Less, more
d) More, less
7) Control signals used in memory mapped I/O are
b) MEMR
c) MEMW
d) Both b) and c)
a) IOR
P.T.O.

SLR-BB 180
8) Match the following :
i) 8253
ii) 74138
iii) 74245
iv) 74244

9)
10)

11)

12)
13)
14)

15)
16)

17)
18)

19)

20)

-2-

*SLRBB180*

A) Unidirectional buffer
B) Timer
C) Bidirectional buffer
D) 3 : 8 decoder
E) 2 : 4 decoder
a) i B, ii E, iii C, iv A
b) i B, ii D, iii C, iv A
c) i A, ii E, iii C, iv B
d) i B, ii C, iii E, iv A
If crystal frequency of 8085 is 6 MHz, the operating frequency is
a) 6 MHz
b) 12 MHz
c) 3 MHz
d) 9 MHz
Control word format for initializing Port A and Port B as input and Port C as
output in mode 0 of 8255 is
a) 92H
b) 82H
c) 93H
d) None
Asynchronous and Synchronous baud rate for 8251 respectively are
a) DC to 11.5 Kbaud, DC to 64 Kbaud b) DC to 19.5 Kbaud, DC to 64 Kbaud
c) DC to 19.2 Kbaud, DC to 64 Kbaud d) DC to 11.5 Kbaud, DC to 34 Kbaud
Port A of 8255 can be configured in
a) Mode 0
b) BSR
c) Mode 1
d) Both a) and c)
Fastest ADC is
a) Dual slope
b) Single slope c) Flash
d) Counter type
Control word required for generating interrupt on terminal count using 8253
for counter 1, 8-bit count, BCD counting is
a) 52H
b) 51H
c) 41H
d) 43H
Number of address lines required to access counters of 8253 is
a) 2
b) 8
c) 16
d) Both b) and c)
Number of memory chips required to design a memory of capacity 4KB using
1K 4 chips are
a) 4
b) 2
c) 8
d) None
BSR format to send logic 1 at PC2 of 8255 is
a) 03H
b) 73H
c) 04H
d) Both a) and b)
2KB memory with starting address 3000H is interfaced with 8085, the last
memory address is
a) 37FFH
b) 47FFH
c) 57FFH
d) 4800H
The format 82H is used for 8255 which specifies
a) Port A and C input, B output in mode 0
b) Port A and C output, B input in mode 0
c) Port A and B output, C input in mode 0
d) Port B and C output, A input in mode 0
Select the invalid instruction
a) STAX D
b) RAL A
c) IN 2000H
d) Both b) and c)
______________
Set A

*SLRBB180*

-3-

SLR-BB 180

Seat
No.

T.E. (Electronics and Telecommunication Engg.) (Part I)


Examination, 2014
MICROPROCESSOR AND PERIPHERALS (Old)
Day and Date : Friday, 12-12-2014
Time : 3.00 p.m. to 6.00 p.m.

Marks : 80

Instructions : 1) All questions are compulsory.


2) Figures to the right indicate full marks.
3) Draw neat diagrams where required.
SECTION I
2. Solve any four :

(45=20)

a) Differentiate between memory mapped I/O and I/O mapped I/O by considering
minimum six parameters.
b) Explain STA 5000H instruction with suitable timing diagram. Clearly mention
the different machine cycles and T states.
c) With suitable flow chart explain EPROM programming method.
d) Explain SIM format of 8085 and write ALP for generating symmetrical square
wave at SOD pin.
e) Draw suitable hardware to demultiplex address data bus and to generate
different control signals for I/O devices and memory. State merits and
demerits of multiplexed address data bus.
3. A) Explain when processor enters into different states, what happens when it
remains in that state and explain the ways to take processor out of these
states. Draw required transition state diagram.
12
B) With suitable diagram explain interrupt structure of 8085. Write a program to
perform decimal addition of two numbers when RST 605 interrupt is enabled.

OR
B) Define addressing modes. With suitable examples, explain different addressing
modes. Write an ALP to perform subtraction of two decimal numbers available
at two memory locations.

Set A

SLR-BB 180

-4-

*SLRBB180*

SECTION II
4. Solve any four :

(45=20)

a) Explain control word format for 8253. Use counter 0 for 16-bit count, to produce
square wave at clock out pin. Use BCD counting.
b) Write an ALP to produce square wave of frequency 2KHz and duty cycle 50%
at PC2 using BSR mode.
c) Draw and explain block diagram of 8251.
d) List out the demerits of weighted resistor DAC. Explain R 2R ladder DAC.
e) Explain mode 1 (input) of 8255 with the help of timing diagrams.
f) Interface 2KB memory with 8085 with starting address 1000H.
5. A) Draw an interfacing circuit to interface 8-bit ADC with 8085 through 8255.
Write an ALP with flowchart to store the converted digital value of an analog
signal.
10
B) Interface 7-segment display with 8085 through 8255 for control word register
address 33H. Write an ALP with flow chart to display the numbers from 0 to 9.
Comment on the address map.
10
OR
B) Explain with suitable timing diagram.
i) Mode 1 of 8253
ii) Mode 2 of 8255.

10

_____________________

Set A

SLR-BB 181

*SLRBB181*
Seat
No.

Set

T.E. (E&TC) (Part I) (New) Examination, 2014


ELECTROMAGNETIC ENGG. AND RADIATING SYSTEM
Day and Date : Wednesday, 3-12-2014
Time : 3.00 p.m. to 6.00 p.m.
N. B. :

Max. Marks : 100

1) Figure indicates full marks.


2) Q. No. 1 is compulsory. It should be solved in first 30 minutes in
Answer Book Page No. 3. Each question carries one mark.
3) Answer MCQ/Objective type questions on Page No. 3 only.
Dont forget to mention, Q.P. Set (A/B/C/D) on Top of Page.
MCQ/Objective Type Questions

Duration : 30 Minutes

Marks : 20

1. Choose the correct answer :

20

1) Potential due to a charge at a point situated at infinity is __________


a) zero
b) infinity
c)
d) 1
2) Gradient of the potential and an equipotential surface ___________
a) have the same direction
b) have opposite direction
c) are orthogonal to each other
d) have no directional relation
3) If two vectors A = 2ax + kay + 4az and B = 6ax + 9ay + 12az have the same unit
vector, then the value of K is equal to __________
a) 3
b) 3
c) 4
d) zero
4) Electric fields of force do not cross each other because ___________
a) there cannot be two tangents at a point
b) the lines of force mutually repels
c) they are parallel
d) none of these
5) A particle of charge 3 109 C is acted upon a downward force 4.5 109 N in
an electric field. The strength of electric field is _______
a) 6 N
b) 1.5 N/C
c) 6 N-C
d) 1.5 N-C
6) If D = 30 xy 2 (z + 1)ax , then v at a point (1, 1, 0) is ___________
a) 30 c/m3
b) zero
c) 30 c/m2
d) 3 c/m2
7) Significance of Gauss law is _________
a) Non existence of monopole
b) Existence of source and sink
c) Both a) and b)
d) None
P.T.O.

SLR-BB 181

*SLRBB181*

-2-

8) When the distance and magnitude of two equal charges are decreased by half,
the force between them is __________
a) same as original
b) reduced to half
c) becomes double
d) none of these
9) A plane z = 0 carries a charge density 88.54 1012 C. The electric field at a
distance point is __________
a) 5 az v/m
b) 5 az v/m
c) zero
d) 2 az v/m
10) Curl (grade v) is equal to _________
a) scalar
b) vector

c) zero

d) none

11) Steady magnetic fields are governed by _________ law.


a) Biot-Savarts
b) Amperes circuital
c) Both a) and b)
d) None of these
12) Amperes circuital law can be applied ________ the conductor.
a) Inside
b) Outside
c) Both a) and b) d) None of these
13) Total flux passing through a closed surface held in a magnetic field is
________
a) Infinity
b) Zero
c) Unity
d) None of these
14) _________ gradient of magnetic scalar potential gives magnetic field intensity.
a) Positive
b) Negative
c) Double
d) Integral
15) If antenna directivity and antenna gain are equal, then antenna efficiency is ____%.
a) 20
b) 50
c) 75
d) 100
16) In _________ array, radiation is perpendicular to array orientation.
a) Broadside
b) End fire
c) None of these d) Cannot say
17) Which of the following statements is not true of waves in general ?
a) It may be a function of time only
b) It may be sinusoidal or cosinusoidal
c) It may be a function of time and space
d) For practical reasons, it must be finite in extent
18) The equation E =
a) Amperes Law

B
t is the generalization of
b) Faraday Law c) Gausss Law

19) Unit of relative permeability is


a) Henry
b) Henry/meter
20) Poynting vector gives
a) rate of energy flow
c) intensity of electric field

c) Henry/meter2

d) Biot-Savarts Law
d) It is dimensionless

b) direction of polarization
d) intensity of magnetic field
______________

Set A

*SLRBB181*

-3-

SLR-BB 181

Seat
No.

T.E. (E&TC) (Part I) (New) Examination, 2014


ELECTROMAGNETIC ENGG. AND RADIATING SYSTEM
Day and Date : Wednesday, 3-12-2014
Time : 3.00 p.m. to 6.00 p.m.

Marks : 80

N. B. : 1) All questions are compulsory.


2) Figure indicates full marks.
SECTION I
2. Attempt any four :
a) Explain in detail physical significance of curl, gradient and divergence.

(54=20)

b) Two infinite uniform sheet of charge 20 c / m2 are located at x = 1 m . Determine


E in all regions.
c) Transform the given vector A = 3ax 2ay 3az at point P(2, 3, 4) into spherical
coordinate system.
d) Three point charges Q1 = 106 C and Q2 = 106 C and Q3 = 0.5 106 C are
located at the corners of an equilateral triangle of side 50 cm. Determine the
magnitude and direction of force on Q3.
e) A diameter of a hollow metallic sphere is 60 cm and a sphere carries charge of
500 c . Find the electric field intensity
1) at the distance of 100 cm from the center of sphere.
2) at the surface of sphere.
f) State and prove point form of Gauss Law i.e. D = v .
3. Attempt any two :

(210=20)

a) 1) Derive the expression for electric field intensity due to infinite line charge using
Gauss Law.
2) An infinitely long uniform line charge of zonc/m is located at x = 2 m, y = 4m.
Determine E and D at P(2, 1, 4).
b) Evaluate both sides of divergence theorem for the field
D = (4x x 2 )ax 3y 2z 2ay (2y 3z 2 z)az in the region 0 x 1, 0 y 1, 0 z 1.
c) 1) Determine the work done in carrying a 2 c charge from P1(2, 1, 1) to
P2(8, 2, 1) in the field E = yax + xay along the parabola x = 2y2.
2) Find the potential at P(1, 2 , 3) with respect to Q(3, 4 , 2) , if the electric field is
E = 100 ar .
r

Set A

SLR-BB 181

*SLRBB181*

-4-

SECTION II
4. Attempt any four :

(54=20)

a) Derive the expression for magnetic field intensity on the axis of circular loop.
b) A current filament of 5 A in a y direction is parallel to Y-axis at X = 2, Z = 2 find H
at the origin.
c) Do the fields E = Em sin x sin t a y and H =

Em
cos x cos t az . Satisfy Maxwells
0

Equation.
d) What are antenna parameters and define any two parameters ?
e) Explain antenna field zones.
5. Solve any two questions :

(102=20)

Y 2 Z ax + 0.5Y 2 Z 2 az
a) Evaluate both sides of Stokes theorem for the field H =
X
X2
and find the current in the a y direction crossing the square surface in the plane
y = 2 bounded by x = z = 1 and x = z = 2.
b) State and prove poynting vector theorem and explain power flow associated with
poynting vector.
c) Derive the equation for E due to N-isotropic point sources of equal magnitude
and spacing arranged as a linear array.
_____________________

Set A

SLR-BB 182

*SLRBB182*
Seat
No.

Set

T.E. (E and TC) (Part I) Examination, 2014


PRINCIPLES OF DIGITAL COMMUNICATION (New)
Day and Date : Friday, 5-12-2014

Max. Marks : 100

Time : 3.00 p.m. to 6.00 p.m.


Instructions : 1) Figures to the right indicate full marks.
2) Assume suitable data if necessary.
3) Q. No. 1 is compulsory. It should be solved in first 30 minutes in Answer
Book Page No. 3. Each question carries one mark.
4) Answer MCQ/Objective type questions on Page No. 3 only. Dont
forget to mention, Q.P. Set (A/B/C/D) on Top of Page.
MCQ/Objective Type Questions
Duration : 30 Minutes

Marks : 20

1. Choose the correct answer :

20

1) Granular noise is associated with _____________


a) PCM

b) DM

c) DPCM

d) QAM

2) A scheme in which 1 is represented by a positive pulse for one half of symbol duration a negative
pulse for remaining half of symbl and for 0 the order is reversed is known as _____________
a) NRZ unipolar

b) Manchestor code

c) NRZ bipolar format

d) NRZ polar format

3) Which one of the following is not same ?


a) PWM
4) I (mk) 0 as
a) pk 1

b) PDM

c) PAM

d) PLM

b) Pk 1/4

c) Pk 0

d) pk 1/2

5) The overall S/N ratio of DM system is _____________ than overall S/N ratio of a PCM system
using the same bandwidth.
a) Higher

b) Lolwer

c) Very high

d) None of these

6) The dynamic range of PCM system can be materially improved by a process called _____________
a) Compressor

b) Expander

c) Compander

d) None of the above

7) Which of the following is digital in nature ?


a) PAM

b) PPM

c) DM

d) PWM

8) Equalizers are used for


a) Frame synchronization

b) Symbol synchronization

c) Carrier synchronization

d) Minimizing ISI
P.T.O.

SLR-BB 182

*SLRBB182*

-2-

9) Channel capacity
a) Restricts the rate at which information is transmitted
b) Gives the number of signals that can be transmitted simultaneously
c) Is not at all dependent upon the channel noise
d) None of these
10) If fm is modulating frequency than alliasing occurs, when Nyquist rate is
a) 2 fm

b) 3 fm

c) 2.5 fm

d) 1.2 fm

c) FSK

d) MSK

11) _____________ is most affected by noise.


a) PSK

b) ASK

12) Duobinary signaling makes use of


a) Two binary

b) A matched filter

c) Controlled ISI introduction

d) Correlation receiver

13) The minimum distance in two symbols in case of BPSK is


a) Eb

b) 2 Eb

c) EB

d) None of these

14) Consider the following statements comparing delta modulation with PCM systems DM requires
a) a lower sampling rate

b) a higher sampling rate

c) a large bandwidth

d) simpler hardware

15) Which of the techniques provide security in digital communication ?


a) Source coding

b) Multiplexing

c) Carrier modulation

d) Spread spectrum

16) In ADM system the output signal amplitude for 1s and 0s are
a) Fixed and the repetition rate is also fixed
b) Fixed but the repetition rate is variable
c) Variable and the repetition rate is also variable
d) Variable but the repetition rate is fixed
17) The non-coherent ASK receiver is _____________ than coherent receiver.
a) More complex

b) Much simpler

c) Less complex

d) None of these

18) A major problem of multicarrier modulation is


a) Fading

b) Diversity

c) High peak to average power ratio

d) None of these

19) According to Shannons theorem the output from any source of rate R can be coded and transmitted
over a channel of capacity C with the condition that
a) C < R

c) C < R2

b) C > R

d) C > R2

20) In QAM both, phase and _____________ of carrier frequency are varied.
a) Amplitude

b) Frequency

c) Bit rate

d) Baud rate

______________
Set A

*SLRBB182*

-3-

SLR-BB 182

Seat
No.

T.E. (E and TC) (Part I) Examination, 2014


PRINCIPLES OF DIGITAL COMMUNICATION (New)
Day and Date : Friday, 5-12-2014
Time : 3.00 p.m. to 6.00 p.m.

Marks : 80

Instructions : 1) Figures to the right indicate full marks.


2) All questions are compulsory.
3) Assume suitable data if necessary.
SECTION I
2. A) Answer any three :
a) An event has six possible outcomes with the probabilities 1/2, 1/4, 1/8, 1/16, 1/32 and 1/32
respectively. Find the entropy of the system. Also find the rate of information if there are 16
outcomes per second.
b) With suitable waveforms explain indirect method of generation of PWM and PPM signals.
c) With block diagram explain TDM-PCM telephone system.
d) Explain the properties of an eye diagram.
B) A discrete source transmits messages x1, x2 and x3 with the probabilities 0.3, 0.4 and
0.3. The source is connected to the channel given in fig. 1. Calculate all the entropies.

12

Fig. 1
3. A) Answer any two :
a) Draw data format for data 1101010010 with

i) RB
ii) Biphase Manchester.
b) With block diagram explain working of ADM.
c) Define pulse modulation. Explain how PAM are generated. What are its different types ?
B)

a) Explain any one :

1) Eye diagram
2) Equalization.
b) Apply the Shannon-Fano coding procedure for the following message ensemble. Calculate its
efficiency :
x2
x3
x4
x5
x6
x7
x8
[X] = [x1
[P] = [1/8
1/16
1/8
1/16
1/16
1/4
1/16 1/4]

Take M = 2.
Set A

SLR-BB 182

-4-

*SLRBB182*

SECTION II
4. A) Answer any three :

(34=12)

1) Explain what matched filter.


2) Draw block diagram of modulated signal and obtain equation of modulated signal.
3) Explain symbol synchronization technique.
4) Write a note on M-ary differential PSK system.
B) Write a note on FFT based multicarrier system.
5. A) Answer any three :

8
(34=12)

1) Compare BPSK and BFSK modulation schemes.


2) Explain DEPSK modulator and list disadvantages.
3) Draw the signal space representation of QPSK signals giving the relevant expressions.
4) Explain Costas loop method of carrier recovery.
B) Derive an expression for error probability of optimum filter.

_____________________

Set A

SLR-BB 183

*SLRBB183*
Seat
No.

Set

T.E. (Part I) (E & TC) (New) Examination, 2014


SOFTWARE ENGINEERING AND PROJECT MANAGEMENT SYSTEMS
Day and Date : Monday, 8-12-2014
Time : 3.00 p.m. to 6.00 p.m.

Max. Marks : 100

Instructions : 1) Q. No. 1 is compulsory. It should be solved in first 30


minutes in Answer Book Page No. 3. Each question carries
one mark.
2) Answer MCQ/Objective type questions on Page No. 3 only.
Dont forget to mention, Q.P. Set (A/B/C/D) on Top of Page.
MCQ/Objective Type Questions
Duration : 30 Minutes

Marks : 20

1. Choose the correct option :

(120=20)

1) The disadvantages of waterfall model is


a) Inflexible positioning
b) Complexity
c) Both
d) None
2) In object oriented design of software, objects have
a) Attributes and names
b) Attributes and operations
c) Attributes, names and operations d) None of these
3) Functional requirement should be
a) Complete
b) Universal

c) Consistent

d) Both a) and c)

4) Software specification is the process of


a) Understanding and defining the services which are required
b) Identifying constraints on system
c) Both a) and b)
d) None
5) The goals of software testing leads to
a) Defect testing
b) Violation testing
c) Both a) and b)
d) None
6) Advantages of CBSE
a) Reduced software cost
c) Reduced risk

b) Faster delivery
d) All

7) UML stands for


a) Unified Modified Language
c) Unified Modeling Language

b) Unified Monitoring Language


d) None of these
P.T.O.

SLR-BB 183

-2-

*SLRBB183*

8) A classification scheme that shows how object class is related to other classes
is known as
a) Relation
b) Taxonomy
c) Fixation
d) None
9) Data flow and state machine models are types of
a) Context models
b) Behavioral models
c) Data models
d) Object models
10) Path testing is _________ testing strategy.
a) Positional
b) Structural
c) Requirement based
d) All
11) _________ provides a more striking visual indication of those activities that
are not progressing to schedule.
a) Slip chart
b) Time line
c) Gantt chart
d) Ball chart
12) Following are the phases of RUP
a) Construction and transition
b) Inception and elaboration
c) Both a) and b)
d) None of these
13) The person handles many projects simultaneously
a) Programme manager
b) Project manager
c) Team leader
d) Software developer
14) Any delay in _________ activities will cause a delay in completion date of the
project.
a) Free float
b) Base lined
c) Critical path
d) Scheduled
15) The base line budget is based on
a) Activity network
b) Cost monitoring
c) Proposed plan
d) None
16) The Gantt chart is used for
a) Tracking project progress
b) Knowing the activities
c) Knowing starting date of project d) Knowing end date of project
17) Sponsoring group will get enough information from _________
a) Program mandate
b) Program brief
c) Blue print
d) Vision statement
18) The diagram showing for each product which other products are required as
input is called _________ diagram.
a) Product based
b) Precedence
c) Product driven
d) Project process
19) Which of below is true for risk ?
a) Relates to future
b) Involves cause and effect
c) Not a self contained topic within program manager
d) All
20) _________ is carried out to calculate the latest dates on which each activity
may be started and completed.
a) Backward pass b) Forward pass c) End pass
d) None
______________
Set A

*SLRBB183*

-3-

SLR-BB 183

Seat
No.

T.E. (Part I) (E & TC) (New) Examination, 2014


SOFTWARE ENGINEERING AND PROJECT MANAGEMENT SYSTEMS
Day and Date : Monday, 8-12-2014
Time : 3.00 p.m. to 6.00 p.m.

Marks : 80

SECTION I
2. Answer any four :

(45=20)

a) Write a short note on domain requirements.


b) Explain system validation.
c) Briefly explain types of non functional requirements.
d) What are key challenges facing software engineering ?
e) List various advantages of incremental development approach.
3. a) Explain context model along with the example of ATM.

(18=8)

OR
a) Explain in detail Rational Unified Process (RUP).
b) List two types of behavioral models. Explain any one in detail.

(18=8)

c) Explain briefly attributes of good software.

(14=4)

SECTION II
4. A) Attempt any two :

(26=12)

i) What are categories of risk ?


ii) Write a short note on forward pass for activity planning.
iii) Explain Gantt chart to visualize the progress of project.
B) Solve all :

(24=8)

i) Define critical path and give its significance.


ii) What are different categories of risk ?

Set A

SLR-BB 183

-4-

*SLRBB183*

5. A) Solve any one :

(18=8)

a) What are different methods to visualize the progress of a project ?


b) What is change control ? Discuss the change control procedures.
B) Attempt all :

(34=12)

i) Define critical path and give its significance.


ii) Explain Work Breakdown Structure with example.
iii) Describe forward pass for activity planning.

_____________________

Set A

SLR-BB 184

*SLRBB184*
Seat
No.

Set

T.E. (E & TC) (Part I) Examination, 2014


DIGITAL SIGNAL PROCESSING (New)
Day and Date : Wednesday, 10-12-2014
Time : 3.00 p.m. to 6.00 p.m.

Max. Marks : 100

Instructions : 1) Q. No. 1 is compulsory. It should be solved in first 30 minutes


in Answer book Page No. 3. Each question carries one mark.
2) Answer MCQ/Objective type questions on Page No. 3 only.
Dont forget to mention, Q.P. Set (A/B/C/D) on Top of Page.
MCQ/Objective Type Questions
Duration : 30 Minutes

Marks : 20

1. Choose the correct answer :

(120=20)

1) If the system describe by y(n) = 2nu ( n) is


a) Stable and causal
b) Causal and linear
c) Stable and non-causal
d) None
2) Additions are required to compute N point DFT
a) N
b) N(N 1)
c) N2

d) N3

3) X(0) value of 6 point sequence x(n) = {1 2 3 4 5 3} is


a) 18
b) 15
c) 17

d) None

4) If the sequence x(n) is real and even then its corresponding DFT values are
a) Imaginary and odd
b) Imaginary and even
c) Real and even
d) None
5) Circular time shift of a sequence is equivalent to ___________
a) Multiplication of the sequence x(n) with complex exponential
b) Multiplication of the sequence X(k) with complex exponential
c) Multiplication of the sequence x(n) with exponential factor
d) None
6) If we modify the circular sequence by multiplying it with e

j 4K
for a kth frequency
N

sample the DFT coefficient ___________


a) Remain unchanged
b) Shift by 4 unit
c) Shift by 8 unit
d) Shift by 2 unit

7) Circular convolution of the sequences x(n) = {1,2, 1} & h(n) = {1, 2, 2} is


a) {1,2,1}
b) {3,2,1}
c) {3,2,1}
d) None
8) Linear convolution of two real sequences with P & Q points, respectively, can be
converted to a circular convolution by appending extra zeros to every sequence
until its length is ___________
a) P + Q
b) P + Q + 1
c) P + Q 1
d) None
P.T.O.

SLR-BB 184

*SLRBB184*

-2-

9) DIT algorithm related to


a) X(k) shuffled
c) X(k) and x(n) shuffled

b) x(n) shuffled
d) None

10) Each butterfly operation involves ___________ complex multiplication.


a) One
b) Two
c) Three
d) None
11) The frequency sampling structures are efficient because
a) Sampled points are less
b) Parallel structures are efficient
c) Some sampled DFT coefficients are zero when filter is narrow band
d) Some sampled values can be assumed to be zero
12) FIR filter is always stable as
a) All its poles are at origin
c) h(n) = h(N 1 n)

b) All its zeros are at origin


d) None of these

13) The mapping at bilinear transformation is


a) One to one mapping
b) Many to one mapping
c) Many to many mapping
d) None of the above
14) A causal and stable IIR filter cannot have linear phase
a) True
b) False
15) To satisfy symmetric condition linear phase FIR filter should have
a) h(n) = h(N 1 n)
b) h(n) = h(N 1 n)
c) h(n) = 0
d) None
16) Quantisation noise can be reduced
a) By increasing step size
b) By reducing step size
c) By reducing number of bits used for quantization
d) None
17) Butterworth filter has
a) Constant response in pass band and decreasing response in stop band
b) Maximally flat response in pass band and monotonically decreasing response
in stop band
c) Ripple in pass band and ripple in stop band
d) None
18) What realization method minimizes the delay elements ?
a) Direct form I
b) Direct form II c) Cascade

d) Parallel

19) Condition for the phase for FIR filter is


a) () =

b) ( ) =

20) Gibbs phenomenon results in


a) FIR designing
b) IIR designing

c) ( ) = 0

d) ( ) =

c) Both a) and b)

d) None

______________
Set A

*SLRBB184*

-3-

SLR-BB 184

Seat
No.

T.E. (E & TC) (Part I) Examination, 2014


DIGITAL SIGNAL PROCESSING (New)
Day and Date : Wednesday, 10-12-2014
Time : 3.00 p.m. to 6.00 p.m.

Marks : 80

Instructions : 1) All questions are compulsory.


2) Figures to the right indicate maximum marks.
3) Assume suitable data if required.
SECTION I
2. Solve any four :

(54=20)

a) Find whether the following systems are linear OR nonlinear.


y(n) = x2(n)
y(n) = nx(n)
b) Determine cross correlation values of the two sequences
x(n) = {1,4,5,2,3} and h(n) = {1, 9, 5,7}
c) Establish the relation between DFT and Z transform.
d) Compute the 4 point DFT values for sequence given x(n) = {4,1,1,3} using linear
transformation matrix.
e) The first 5 point of the 8 point DFT of a real valued sequence are
{0.25, 0.125 j 0.3018, 0, 0.125 j 0.0518, 0}
Determine the remaining 3 points.
3. Solve any two :

(102=20)

a) List the properties of DFT and explain in detail circular convolution property of
DFT. Find the circular convolution of two finite duration sequence
x1(n) = {1, 1, _ 2, 3, 1} and x2(n) = {1,2,3,0,0}
b) Given 8 point DFT of the sequence x(n) = {1, 2, 3, 4, 4, 3, 2, 1}
Compute the DFT of the sequence
x1(n) = {2, 1, 1, 2, 3, 4, 4,3}
c) Explain overlap and save method for linear filtering of the given sequence. Let the
input sequence be
x(n) = {2, 3, 4, 5, 6, 7, 8, 2, 3, 4, 5, 1, 2, 3, 4, 5} and h(n) = {3, 2, 3, 1}
Use over lap save method.
d) Calculate DFT values of the sequence x(n) = {1, 1, 2, 4, 4, 2, 1, 1} using DIF, FFT
algorithm.
Set A

SLR-BB 184

-4-

*SLRBB184*

SECTION II
4. Answer any 4 :

(54=20)

a) Explain the design steps of digital butterworth low pass filter.


b) Explain windowing technique for designing FIR filter.
c) Explain application of DSP in image processing.
d) Obtain cascade realization of system function given as
H(z) = 1 +

5 1
z + 2 z2 + 2 z3
2

e) Explain the arrangement of zeros in FIR system.


5. Answer any 2:

(102=20)

a) i) Explain the necessary and sufficient condition for designing linear phase
FIR filter.

ii) Design a third order butterworth digital filter using impulse invariant
technique. Assume sampling period T = 1 sec.

b) i) Draw the parallel structure of IIR system.

ii) Realise the system y(n) + y (n 1) + ( 14) y (n 2) = x (n) in cascade form. 7


c) Using rectangular window technique design a LPF with pass band gain of unity,
cut off frequency of 1000 Hz an working at a sampling frequency of 5 kHz. Let the

length of impulse response be 7.

10

_____________________

Set A

SLR-BB 185

*SLRBB185*
Seat
No.

Set

T.E. (Part I) (Electronics and Telecommunication Engg.)


Examination, 2014
MICROPROCESSORS (New)
Day and Date : Friday, 12-12-2014
Time : 3.00 p.m. to 6.00 p.m.
Instructions :

Max. Marks : 100

1) Q. No. 1 is compulsory. It should be solved in first 30 minutes in


Answer Book Page No. 3. Each question carries one mark.
2) Answer MCQ/Objective type questions on Page No. 3 only. Dont
forget to mention, Q.P. Set (A/B/C/D) on Top of Page.
3) Figures to the right indicate full marks.
4) Draw neat diagram where required.
MCQ/Objective Type Questions

Duration : 30 Minutes

Marks : 20

1. Select the appropriate option :

20

1) The delay provided by the NOP instruction for 4 MHz crystal frequency is
______________
a) 1 s
b) 2 s
c) 3 s
d) None of these
2) Number of machine cycles and T states required to execute CALL Addr instruction are
a) M.C. = 5, T = 16
b) M.C. = 5, T = 18
c) M.C. = 3, T = 16
d) None of these
3) Instructions used for data transfer in I/O mapped I/O are
a) STA, LDA
b) IN, OUT
c) MOV, MVI

d) None of these

4) The first and last address in the address map of typical memory is 3000 H and 3FFFH
respectively, what is the memory capacity ?
a) 1 KB
b) 2 KB
c) 4 KB
d) None of these
5) Which of the following interrupt is level and edge trigger ?
a) RST 5.5
b) RST 6.5
c) RST 7.5

d) TRAP

6) Why 8085 processor is called an 8 bit processor ?


a) because 8085 processor has 8 bit ALU
b) because 8085 processor has 8 bit data bus
c) a) and b)
d) none of these
7) Specify the contents of accumulator and Cy flag when following instructions are executed
MVI A, 0FH
ORA A
RRC
a) A = 87H, CY =1
b) A = 87H, CY = 0
c) A = 1EH, CY = 1
d) A = 1E, CY = 0
P.T.O.

SLR-BB 185

*SLRBB185*

-2-

8) The Auxiliary carry flag is set when


a) The sum is more than 8 bit
b) Carry is generated out of D3 during arithmetic operation
c) Result of operation contains odd number of 1s
d) None of these
9) What is a maximum instruction length of 8085 ?
a) 1 byte
b) 2 byte
c) 3 byte
10) IC 74LS373 is a
a) Buffer

b) Latch

c) Encoder

d) 4 byte
d) Decoder

11) Basically 8251 does


a) Analog-to-digital conversion
b) Parallel-to-serial and serial-to-parallel conversion
c) Digital-to-analog conversion
d) None of the above
12) 8253, a programmable interval timer consists of
a) Three, 8-bit counters
b) Two, 16-bit counters
c) Three, 16-bit counters
d) Two, 8-bit counters
13) 8251 supports the baud rate for synchronous communication is
a) DC to 19.2 K band
b) DC to 64 K band
c) DC to 128 K band
d) Any baud rate
14) To generate interrupt on terminal count using timer IC for the requirements counter 1, 8
bit count, binary counting is
a) 60 H
b) 50 H
c) 70 H
d) None of the above
15) 8085 and peripherals are configured in I/O mapped I/O
a) Device address is 8 bits
b) Instructions In and OUT are used
c) For single operation 3 machine cycles are required
d) All of the above
16) Commonly used DAC techniques in DAC ICS is
a) Single slope DAC
b) Dual slope
c) R-2R ladder
d) Weighted resistor
17) Number of counters and the address lines for the timer IC 8253 respectively are
a) 3, 3
b) 3, 2
c) 3, 4
d) 2,3
18) The direction of EOC signal for ADC 0808 IC
a) Input
b) Output
c) Input and output

d) None

19) 8086 processor has ____________ address pins out of which _________ number of pins
are used as data pins.
a) 16, 8
b) 16, 14
c) 20, 16
d) 20, 8
20) In 8086 is a __________ bit microprocessor and is fabricated using _______ technology.
a) 16, CMOS
b) 8, HMOS
c) 20, NMOS
d) 16, HMOS

______________

Set A

*SLRBB185*

-3-

SLR-BB 185

Seat
No.

T.E. (Part I) (Electronics and Telecommunication Engg.)


Examination, 2014
MICROPROCESSORS (New)
Day and Date : Friday, 12-12-2014

Marks : 80

Time : 3.00 p.m. to 6.00 p.m.


Instructions :

1) All questions are compulsory.


2) Figures to the right indicate full marks.
3) Draw neat diagram where required.
SECTION I

2. Solve any four :

(45=20)

a) Draw and explain hardware required to generate various control signals for accessing
I/Os and memory.
b) Write an ALP for finding smallest element in a block of data.
c) Write an ALP for finding square of a number and result should be in BCD format.
d) Draw timing diagram of an instruction MOV R,M.
e) Explain function of following instructions of 8085 :
i) DAA
ii) PUSH Rp.
3. a) Draw and explain architecture of 8085.

10

b) The following memory devices are to be interfaced to 8085 :


i) 2K 8 EPROM
ii) 2K 8 RAM.
The EPROM address should start from 0000 H and RAM address from 8000H.
Draw the complete interfacing diagram.

10

OR
b) What are different hardware interrupts ? With suitable block diagram explain interrupt
structure of 8085. State the vector address and triggering levels of all interrupts.

10

Set A

SLR-BB 185

-4-

*SLRBB185*

SECTION II
4. Solve any four :

(45=20)

a) Write ALP to generate square wave on SOD pin with 50% duty cycle. (Assume Ton = 1 sec).
b) Write an ALP to perform the following operations :
i) Take any 8 bit number from Port B, add 01H number in it and store the result in any
memory location.
ii) Take any 8 bit number from Port A, subtract 01H number from it and send the result in
Port C.
c) Draw and explain block diagram of 8253.
d) With a neat diagram explain the working of successive approximation ADC.
e) Compare synchronous and asynchronous serial data communication.
5. Solve any two :

(210=20)

a) Interface DAC 0808 to 8085 using PPI in I/O mapped I/O mode. Write an ALP to
generate the following waves :
i) Ramp Wave
ii) Triangular wave
iii) Square wave Duty Cycle = 50%.
b) Explain IC 8251 by considering the following parameters :
i) Feature
ii) Internal Block diagram
iii) Mode word format.
c) List the features of 8086 microprocessor and also explain the register organization of
8086 microprocessor.
_____________________

Set A

SLR-BB 187

*SLRBB187*
Seat
No.

Set

T.E. (Electronics and Telecommunication Engg.) (Part II)


Examination, 2014
RADAR AND MICROWAVE ENGG.
Day and Date : Tuesday, 25-11-2014
Time : 10.00 a.m. to 1.00 p.m.

Total Marks : 100

Instructions : 1) Q. No. 1 is compulsory. It should be solved in first 30


minutes in Answer book Page No. 3. Each question carries
one mark.
2) Answer MCQ/Objective type questions on Page No. 3 only.
Dont forget to mention, Q.P. Set (A/B/C/D) on Top of Page.
MCQ/Objective Questions

Duration : 30 Minutes
Marks : 20
1. 1) __________ mode do not exist in rectangular waveguide.
a) TE10
b) TM10
c) TE11
d) TE21
2) Lossless Transmission line operating at 200 M rad/sec has L = 0.6 H,
C = 20 pF/m. Characteristic impedance of this transmission line is __________
a) 376.7
b) 173.20
c) 232.5
d) Zero
3) Circulator and Isolator works on the principle of __________
a) Faradays rod
b) Faradays Rotation
c) Reciprocity theorem
d) None of these
4) In case of Matched load __________
a) Transmission is zero
b) Reflection is zero
c) Reflection is unity
d) Transmission is equal to reflection
5) Wave in waveguide propogates __________
a) Without reflections
b) With refraction
c) With multiple reflections
d) None of these
6) The measurement of mismatch between load and transmission line is known
as __________
a) Directivity
b) Isolation
c) Refection coefficient
d) Transmission coefficient
7) 10dB directional coupler has provided with 3 watts then amount of power
coupled is __________
a) 0.6 watts
b) 100 watts
c) 1 watt
d) None of these
8) If power fed to E-arm of E plane TEE is 6 dB then power at each collinear arm
is __________
a) 6 dB
b) 12 dB
c) 3 dB
d) None of these
P.T.O.

SLR-BB 187

-2-

*SLRBB187*

9) An air filled rectangular waveguide having dimensions 5 cm 8 cm operates


in TE dominant mode. The cut-off wavelength at 12 GHz is __________
a) 10 m
b) 16 cm
c) 8 cm
d) None of these
10) Pure standing waves are formed if __________
a) Transmission line is characteristic impedance is equal to load impedance
b) Transmission line is open circuited
c) Transmission line is short circuited
d) None of these
11) In mode operation of magnetron, the spokes due to phase focusing effect
rotate at an angular velocity corresponding to
a) One pole/cycle
b) Two poles/cycle
c) Four poles/cycle
d) Six poles/cycle
12) The slow wave structure of a TWT is provided with an attenuator as
a) It prevents the RF power from reaching the output of the device when
beam is switched on.
b) It reduces the noise level of the device
c) It prevents oscillations in the device
d) It is used to broadband the device
13) The maximum unambiguous range in a system depends on
a) Maximum power of the transistor b) Sensitivity of the radar receiver
c) Width of the transmitted pulse
d) Pulse repetition frequency
14) Double minimum method is relevant to the measurement of
a) Low attenuation coefficient
b) High attenuation coefficient
c) High VSWR
d) Low VSWR
15) In microwave power measurements using bolometers, the principle of working
is the variation of
a) Resistance with the absorption of power
b) Inductance with the absorption of power
c) Capacitance with the absorption of power
d) Cavity dimensions with heat generated by the power
16) Which of the following is a M-type tube ?
a) Klystron
b) Reflex klystron c) TWT
d) Magnetron
17) A reflex klystron is often preferred to the cavity klystron oscillator because
the former is
a) Is more efficient
b) Has higher number of modes
c) Is easier to tune
d) Produces high output power
18) In microwave ovens the microwave oscillator used is
a) Two cavity klystron
b) Reflex klystron
c) Magnetron
d) TWT
19) PIN diode can be used as a
a) Switch
b) Amplitude modulator
c) Phase shifter d) All of above
20) Dipole domain formation occurs in
a) Gunn effect devices
b) IMPATT diodes
c) BARRITT diodes
d) All of the above
______________
Set A

*SLRBB187*

-3-

SLR-BB 187

Seat
No.

T.E. (Electronics and Telecommunication Engg.) (Part II)


Examination, 2014
RADAR AND MICROWAVE ENGG.
Day and Date : Tuesday, 25-11-2014
Time : 10.00 a.m. to 1.00 p.m.
Instructions : i)
ii)
iii)
iv)

Marks : 80

All questions are compulsory.


Assume suitable data if necessary.
Draw the neat figures wherever necessary.
Use of non-programmable calculator is allowed.
SECTION I

2. Attempt any four of the following :

(45=20)

a) Explain the working of Faradays rotation 4 port circulator.


b) List the properties of S matrix.
c) Derive transmission line equation from circuit equations.
d) What is hybrid TEE ?
e) A 35 dB directional coupler has 5 mW incident power and if back power
obtained is 2 nW find outputs at other ports of coupler.
3. a) A transmission line operating at 1 MHz having R = 19 ohm, L = 2.45 mH,
C = 7.9 nF, G = 3.32109 mho all per km.

10

Calculate :
1) Characteristic impedance
2) Propagation constant
3) Attenuation over 2km in dB
4) Phase velocity.
b) What is Smith Chart ? How it is useful to solve transmission line problems ? 10
OR
c) Derive the Transmission line equations. Also explain what are primary and
secondary constants of a transmission line.

10
Set A

SLR-BB 187

-4-

*SLRBB187*

SECTION II
4. Answer any four :

20

a) Explain velocity modulation in two cavity klystron with applegate diagram.


b) Explain IMPATT diode with negative resistance characteristics.
c) Explain the factors affecting radar range.
d) Explain the construction and working of microwave transistors.
e) Draw and explain the helix type TWT.
5. Attempt any two of the following :

20

a) A normal circular magnetron has the following parameters


Inner radius : Ra = 0.15 m
Outer radius Ro = 0.45 m
Magnetic flux density = 1.2 mWb/m2
i) Determine the Hull cot off voltage.
ii) Determine the cut off magnetic flux density if the beam voltage V0 as 6000 V.
iii) Determine the cyclotron frequency in GHz.
b) Explain MTI radars with its principle and block diagram.
c) Explain Gunn effect using the two valley theory.
_____________________

Set A

SLR-BB 188

*SLRBB188*
Seat
No.

Set

T.E. (Electronics and Telecommunication) (Part II) Examination, 2014


MICROCONTROLLER AND APPLICATIONS
Day and Date : Wednesday, 26-11-2014
Time : 10.00 a.m. to 1.00 p.m.

Total Marks : 100

Instructions : 1) All questions are compulsory.


2) Assume suitable data if necessary.
3) Q. No. 1 is compulsory. It should be solved in first 30
minutes in Answer book Page No. 3. Each question carries
one mark.
4) Answer MCQ/Objective type questions on Page No. 3 only.
Dont forget to mention, Q.P. Set (A/B/C/D) on Top of Page.
MCQ/Objective Type Questions
Duration : 30 Minutes

Marks : 20

1. 1) Which of the following activity does not take place during ISR ?
a) Enable interrupts during critical region
b) Save registers
c) Save interrupt structure
d) Disable interrupts during critical region
2) Which of the following instruction is to be used to set a BIT of SFR ?
a) MOV TCON,#55H
b) MOVX SCON,#10H
c) SETB TMOD
d) SETB PCON.1
3) Which of the following cannot be programmed in 8051 ?
a) Modes of operation
b) Pre-scalar
c) Start/stop
d) Preload
4) Which facility of the timer is used to measure the pulse width ?
a) Input capture
b) Output compare
c) Checking overflow
d) Programming preset value
5) For an 8-bit timer, 50 is loaded as preset input. After how many pulses all
outputs will become 1.
a) 255
b) 256
c) 205
d) 206
6) MOVC instruction uses following operands.
a) PC only b) A and A + PC
c) A and Ri + PC

d) A and Ri + DPTR
P.T.O.

SLR-BB 188

-2-

*SLRBB188*

7) MOVX instruction is used to transfer data between


a) Internal memory and accumulator
b) External data memory and accumulator
c) External code memory and accumulator
d) External data memory and general purpose register
8) If A = 05 and B = 64H then after MUL AB the SFRs A and B.
a) Do not change
b) A will have 01 and B will have F4H
c) A will have F4H and B will have 01 d) A will have 00H and B will have F4H
9) AC flag is affected in
a) Addition, subtraction, multiplies and divide
b) Multiply and divide
c) Addition and subtraction
d) Addition, subtraction and multiplication
10) The RS0 and RS1 bits are
a) Not available in PSW
b) Are bits of 3 and 4 of PSW
c) Are bits of 4 and 5 of PSW
d) In register bank itself
11) In PIC 16f877 Port A and D are _________ bits wide respectively.
a) 6, 3
b) 8, 8
c) 3, 8
d) 6, 8
12) Serial Data Line and Serial Clock Line can be used by
a) I2C
b) USB
c) SPI
d) SCI
13) CCP module is used with timer
a) Timer 1
b) Timer 0
c) Timer 2
d) Both b) and c)
14) Pins used in SPI mode are
a) SDO
b) SCK
c) SS
d) All
15) The CALL and GOTO instructions provides _________ bits address to allow
branching within any _________ program memory space.
a) 11, 2k
b) 13, 8k
c) 16, 64k
d) 2, 2k
16) Not writable bits in the status register of PIC 16f877 are
a) T0, PD
b) RP0, RP1
c) DC, C
d) T0, DC
17) DSI307 RTC ic supports
a) SPI
b) I2C
c) Both a) and b) d) USART
18) ADDLW K instruction may affect
a) C, AC, Z, DC b) C, DC, Z
c) C,DC
d) None of the above
19) ADFM bit of ADCON 1 register set 1 results in
a) Right justified
b) Left justified
c) A/D module interrupt
d) None of the above
20) PIC 16f877 consist of __________ channel and __________ bit ADC module.
a) 8, 8
b) 10, 8
c) 8, 10
d) All above
______________
Set A

*SLRBB188*

SLR-BB 188

-3-

Seat
No.

T.E. (Electronics and Telecommunication) (Part II) Examination, 2014


MICROCONTROLLER AND APPLICATIONS
Day and Date : Wednesday, 26-11-2014
Time : 10.00 a.m. to 1.00 p.m.

Marks : 80

Instructions : 1) All questions are compulsory.


2) Assume suitable data if necessary.
SECTION I
2. Attempt any four :

(45=20)

a) Assume that XTAL = 11.0592 MHz. What value do we need to load the timers
register if we want to have a time delay of 5 ms ? Show the program for timer
0 to create a pulse width of 5 ms on P2.3.
b) Define and explain PSW of 8051 with register format.
c) Draw the pin diagram of 8051 microcontroller. Explain briefly port 0 and
port 1 of 8051.
d) Write the difference between the following instructions :
i) SHLD and LHLD

ii) STAX and LDAX

iii) LDA and STA

e) Explain the internal memory structure of 8051.


3. a) Write a program using interrupts to simultaneously create 7 kHz and 500 Hz
square waves on P1.7 and P1.6.

10

b) With suitable examples, explain various addressing modes of 8051. Write an


ALP to add n bytes stored in external memory (Starting address 4000 and
number of bytes is 5) and store output at 3020H.

10

OR
c) Draw and explain interfacing circuit to interface DAC 0808 to 8051. Write
ALP when switch SW=0, the DAC output gives a staircase waveform. Connect
switch SW to pin P0.0.

10
Set A

SLR-BB 188

-4-

*SLRBB188*

SECTION II
4. Attempt any four :

(45=20)

1) Explain PWM in PIC 16f877.


2) Explain ADC module in PIC.
3) Write down the steps to read data to EEPROM.
4) The C and Z flag after execution of following code are
MOVLW FFH
ADDLW 01
5) Explain how relay is interfaced with PIC. Write program to toggle relay.
5. Attempt any two :

(210=20)

1) Draw interfacing diagram and write assembly language program for


interfacing LCD (162) to PIC and display SOLAPUR on LCD.
2) Explain different addressing modes of PIC 16f877. Write a program using
assembly for multiplication of two 8 bit numbers.
3) Explain following instructions :
a) BSF
b) SWAPF
c) RLF
d) BTFSS
e) XORLW.
_____________________

Set A

SLR-BB 189

*SLRBB189*
S

T.E. (E&TC) (Part II) Examination, 2014


INDUSTRIAL ELECTRONICS
Day and Date : Thursday, 27-11-2014
Time : 10.00 a.m. to 1.00 p.m.

Max. Marks : 100

Instructions : 1) Figures to the right indicate full marks.


2) Q. No. 1 is compulsory. It should be solved in first 30 minutes
in Answer Book Page No. 3. Each question carries one mark.
3) Answer MCQ/Objective type questions on Page No. 3
only. Dont forget to mention, Q.P. Set (A/B/C/D) on Top of
Page.
Duration : 30 Minutes

MCQ/Objective Type Questions

Marks : 20

1. Tick mark the correct answer :


1) The number of P-N junction in thyristor is
a) 1
b) 2
c) 3

d) 4

2) The function of snubber circuit connected across SCR is to


a) increase di/dt
b) increase dv/dt c) decrease dv/dt d) none
3) A thyristor may be termed as
a) DC switch
b) AC switch

c) Either a) or b)

d) None

4) A power MOSFET has three terminals, namely


a) Collector, emitter and base
b) Drain, source and base
c) Drain, source and gate
d) Collector, emitter and gate
5) Following semiconductor switch requires continuous signal to turn the switch on
a) SCR
b) MOSFET
c) IGBT
d) None
6) When cathode is more positive with respect to anode, the no. of blocked pn junction
in SCR is given as
a) 1
b) 2
c) 3
d) 4
7) Resonant pulse commutation is _______________ type commutation.
a) Class A
b) Class B
c) Class C
d) Class D
8) In a single phase full converter with resistive load and for a firing angle
load current is zero and non zero, respectively for
a)


b)


c)


d)


, the

9) For continuous conduction in a single phase semi converter each SCR conducts
for
a)
b)
c)
+
d)


P.T.O.

SLR-BB 189

*SLRBB189*

-2-

10) In three phase semi converter, the three SCRs are triggered at an interval of
a) 60
b) 90
c) 120
d) 180
11) The single phase half bridge inverter has resistive load of 10 and center tapped
dc input voltage is 96V. Then rms value of output voltage is
a) 67.88
b) 135.76
c) 48
d) 96


12) A single phase full bridge inverter has dc input voltage is 100 V, the PIV is
__________
a) 200

b) 100

c)


d)


13) In ________________ inverter output frequency is independent of commutating


component but depends upon triggering frequency of thyristors.
a) Parallel
b) Series
c) Series inverter with coupled inductors d) Both b) and c)
14) In Morgans chopper tON =
a)

b)

c)
?

d)


15) A step up chopper has input voltage of 200V and output voltage of 600 V, duty
cycle of chopper is _____________
a) 0.0667
b) 0.667
c) 66.7%
d) both b) and c)
16) A cycloconverter
a) Step down cycloconverter works on natural commutation
b) Step up cycloconverter require forced commutation
c) Cycloconverter is a frequency changer
d) All above
17) The RMS value of line voltage for 180 conduction mode in terms of Vdc is ________
a) 81.64%
b) 0.8164
c) 0.707
d) both a) and b)
18) A large capacitor, put across dc bus of a voltage source inverter, is intended to
a) allow a low impedance path to the high frequency component of dc link current
b) to minimize high frequency current ripple through the ideal dc source
c) to maintain a constant dc link current
d) to protect against switch failure
19) In pulse width modulation of chopper
a) T is kept constant and TON is varied
b) T is kept constant and TOFF is varied
c) By varying f with TON constant
d) All above
20) Choppers can be used in future electric automobiles
a) for speed control only
b) for braking only
c) for speed control and braking
d) none of above
______________

Set A

*SLRBB189*
S

-3-

SLR-BB 189

T.E. (E&TC) (Part II) Examination, 2014


INDUSTRIAL ELECTRONICS
Day and Date : Thursday, 27-11-2014

Marks : 80

Time : 10.00 a.m. to 1.00 p.m.


Instructions : 1) All questions are compulsory.
2) Figures to right indicate full marks.
SECTION I
2. Solve any four :

16

a) Explain any line commutation for SCR.


b) Write a short note on DIAC.
c) Draw and explain working of single phase half wave converter with RL load.
d) Explain over voltage protection for SCR.
e) Explain the effect of free-wheeling diode on single phase half wave converter
with RL load.
3. Solve any three :

24

a) Explain ratings of SCR.


b) Explain AC power control using TRIAC for fan regulator.
c) Explain single phase fully controlled bridge converter with diagrams and
waveforms for RL load.
d) Draw and explain the two transistor model of SCR. Derive the anode current
equation.

Set A

SLR-BB 189

*SLRBB189*

-4-

SECTION II
4. Attempt any four :

(44)

1) A step down chopper has resistive load of 10 and input voltage is 220V. When
converter switch remains on, its voltage drop is 2V and chopping frequency is
1 KHz. If duty cycle is 50% calculate


a> Average output voltage


b> RMS output voltage
c> Ripple factor
d> Form factor.
2) Explain working of single phase step up cycloconverter for
f

. Sketch

associated waveforms.
3) Explain working of static DC circuit breaker.
4) Explain working of single pulse width modulation techniques.
5) Explain the principle of dielectric heating.
5. Attempt any three :

(38)

1) Explain working of three phase bridge inverter with resistive load in 180 conduction
mode. Derive an expression for RMS value of line voltage and phase voltage.
Draw relevant line and phase voltage.
2) Explain working of Morgans chopper. Sketch associated voltage and current
waveforms.
3) With neat circuit diagram and appropriate waveforms explain working of a
single phase parallel inverter with inductive load.
4) Explain working of Class-C and Class-D chopper.
_____________________

Set A

SLR-BB 19

*SLRBB19*
Seat
No.

Set

S.E. (Civil) (Part I) Examination, 2014


CONCRETE TECHNOLOGY
Day and Date : Tuesday, 9-12-2014
Time : 10.00 a.m. to 1.00 p.m.

Max. Marks : 100

Instructions : 1) Q. No. 1 is compulsory. It should be solved in first 30 minutes


in Answer Book Page No. 3.
2) Answer MCQ/Objective type questions on Page No. 3 only.
Dont forget to mention, Q.P. Set (A/B/C/D) on Top of Page.
3) Solve any three questions from each Section.
4) Figures to right indicate full marks.
5) Use of non-programmable calculator is allowed.
6) Assume suitable data if necessary and mention it clearly.
MCQ/Objective Type Questions
Duration : 30 Minutes

Marks : 20

1. Choose the correct answer :


i) If 1500 g of water is required to have a cement paste 1875 g of normal
consistency, the percentage of water is
a) 15%
b) 20%
c) 25%
d) 30%
ii) For given water content, workability decreases if the concrete aggregates
contain an excess of
a) flat particles
b) elongated particles
c) flaky particles
d) all of the above
iii) While compacting the concrete by a mechanical vibrator, the slump should
not exceed
a) 2.5 cm
b) 5 cm
c) 8 cm
d) 10 cm

20
2

iv) The most commonly used admixture that prologs setting and hardening is
a) Calcium chloride
b) Sodium silicate
c) Gypsum
d) All of the above

v) Compacting factor of 0.87 indicates a mix of


a) medium workability
b) very low workability
c) low workability
d) high workability

P.T.O.

SLR-BB 19

*SLRBB19*

-2-

vi) Hydration of cement is due to chemical action of water with


a) Tricalcium silicate and dicalcium silicate
b) Dicalcium silicate and tricalcium aluminate
c) Tricalcium aluminate and tricalcium alumino ferrite
d) All the above

vii) As compared to Ordinary Portland Cement, High Alumina Cement has


a) Lower initial setting time but higher final setting time
b) Higher initial setting time but lower final setting time
c) Higher initial and final setting times
d) Lower initial and final setting times

viii) Bulking of sand is maximum if the moisture content is about


a) 2%
b) 6%
c) 4%
d) 10%

ix) The minimum water to cement ratio for hydration of cement concrete is
a) 0.65
b) 0.5
c) 0.35
d) 0.27

x) Volume of one bag of cement is taken as


a) 35 liters
b) 70 liters
c) 35 m3

2
d) 70 m3

xi) The characteristic strength of concrete is define as the strength of the concrete
below which not more than ____________ of the test result are expected to
fall.
a) 20%
b) 10%
c) 5%
d) 0%

xii) The strength of light-weight concrete depends upon


a) Density of concrete
b) Size of aggregates
c) Type of cement
d) Mix proportion

xiii) Which of the following is not a composition of cement ?


a) Tricalcium silicate
b) Dicalcium silicate
c) Tricalcium aluminate
d) Dicalcium aluminate

xiv) Air-entraining agents


a) Are used for entraining air in concrete
b) Contain wood resins, fats and lignosulphonates
c) Increase durability of concrete to frost-action
d) All of the above
______________

Set A

*SLRBB19*

-3-

SLR-BB 19

Seat
No.

S.E. (Civil) (Part I) Examination, 2014


CONCRETE TECHNOLOGY
Day and Date : Tuesday, 9-12-2014

Marks : 80

Time : 10.00 a.m. to 1.00 p.m.


Instructions : 1) Solve any three questions from each Section.
2) Figures to right indicate full marks.
3) Use of non-programmable calculator is allowed.
4) Assume suitable data if necessary and mention it clearly.
SECTION I
2. a) Write a short note on physical properties of cement and their significance.

b) Write a short note on accelerators and retarders in concrete.

c) Write about bulk density of sand.

3. Explain about the following (any three) :


a) Standard grading curves.

b) Bulking of sand.

c) Plasticizers and super plasticizers.

d) Effect of temperature on strength of concrete.

4. a) Write a short note on rapid hardening cement.

b) Explain in details about dry and wet process of manufacturing of cement with
neat sketches.
5. a) Explain various admixtures in concrete and their necessity.

9
6

b) Write about effect of shape of aggregates on performance of concrete.

c) Enlist methods of curing concrete.

3
Set A

SLR-BB 19

-4-

*SLRBB19*

SECTION II
6. a) Write in brief about elasticity of concrete.
b) Write a detail note on creep and shrinkage of concrete.

4
10

7. Write down the step by step procedure of concrete Mix design by revised
IS 10262 : 2009.

13

8. a) Explain the bleeding and segregation of concrete.


b) Write on joints in concrete. Explain any two.
9. a) Explain the field control of concrete.
b) Enlist various types of concrete and their suitability.

4
9
4
9

_____________________

Set A

SLR-BB 190

*SLRBB190*
Seat
No.

Set

T.E. (E&TC) (Part II) Examination, 2014


OPTICAL COMMUNICATION
Day and Date : Friday, 28-11-2014
Time : 10.00 a.m. to 1.00 p.m.

Max. Marks : 100

Instructions : 1) Figures to right indicate full marks.


2) Assume suitable data if necessary.
3) Q. No. 1 is compulsory. It should be solved in first 30 minutes in
Answer book Page No. 3. Each question carries one mark.
4) Answer MCQ/Objective type questions on Page No. 3 only. Dont
forget to mention, Q.P. Set (A/B/C/D) on Top of Page.
MCQ/Objective Type Questions
Duration : 30 Minutes

Marks : 20

1. Choose the correct options :

(201=20)

1) In optical communication system, the light detector is


a) Avalanche Photo Diode (APD)
b) Positive Intrinsic Negative (PIN) diode
c) Phototransistor
d) Either a) or b)
2) Light is confined within the core of a simple optical fiber by
a) Refraction
b) Total internal reflection at the outer edge of the cladding
c) Total internal reflection at the core cladding boundary
d) Reflection from the fibers plastic coating
3) What makes optical fibers immune to EMI ?
a) They transmit signals in as light rather than electric current
b) They are too small for magnetic fields to introduce current in them
c) Magnetic fields cannot penetrate the glass of the fiber
d) They are readily shielded by outer conductors in cable
4) Which is an advantage of optical communication links over using transmission lines or
waveguides ?
a) Small size
b) Extremely wide bandwidths
c) Immunity to Electromagnetic Interference (EMI)
d) All of the above
5) In fiber optics, the signal is __________ waves.
a) Light
b) Radio
c) Infrared

d) Very low frequency

6) In an optical fiber, the inner core is __________ the cladding.


a) Denser than
b) Less dense than
c) Both have same density
d) None of the above
7) When a beam of light travels through media of two different densities, if the angle of
incidence is greater than the critical angle ___________ occurs.
a) Reflection
b) Refraction
c) Incident
d) Criticism
P.T.O.

SLR-BB 190

-2-

*SLRBB190*

8) When the angle of incidence is ___________ the critical angle, the light beam bends
along the interface.
a) More than
b) Less than
c) Equal to
d) None of the above
9) In a fiber-optic cable, the signal is propagated along the inner core by ________
a) Reflection
b) Refraction
c) Modulation
d) None of the above
10) Multimode graded index fibers exhibit for less intermodal dispersion than multimode step
index fibers due to their
a) Structure
b) Acceptance angle
c) Refractive index profile
d) Multipath reflection
11) The absence of optical amplification through stimulated emission in the LED tends to limit
the __________
a) Internal quantum efficiency
b) External quantum efficiency
c) Both a) and b)
d) None of these
12) The planer and dome LEDs has application such as _________
a) Intruder alarm
b) TV channel changes
c) Industrial counting
d) All of these
13) Edge emitter LEDs has _________ modulation bandwidth than surface emitters with same
drive level.
a) Less
b) Better
c) One half
d) Same
14) _________ absorption requires assistance of photons so that momentum as well as energy
is conserved.
a) Direct
b) Direct but not indirect
c) Indirect
d) Both a) and b)
15) The main factors which limits the speed of response of a photodiode are
a) Drift time of carrier through the depletion region
b) Time constant incurred by capacitance of the photodiode with its load
c) Diffusion time of carriers generated outside the depletion region
d) All of these
16) The phototransistor provides internal gain of the photocurrent through __________
a) Avalanche multiplication
b) Transistor action
c) Both a) and b)
d) None of these
17) The phenomenon that leads to avalanche breakdown in ordinary reverse biased diode is
________
a) Zener breakdown
b) Impact ionization
c) Avalanche breakdown
d) Avalanche multiplication
18) The technique used for the fiber absorption loss measurement is __________
a) Cut-back method
b) Differential method
c) Calorimetric measurement
d) Both a) and b)
19) The common on-line measurement techniques uses fiber image projection (shadow
method) for the measurement of ___________
a) Numerical aperture
b) Core diameter
c) Outer diameter
d) Dispersion
20) _____________ are the fiber dispersion measurement techniques.
a) Time domain measurement
b) Frequency domain measurement
c) Interferometric method
d) Both a) and b)
______________

Set A

*SLRBB190*

-3-

SLR-BB 190

Seat
No.

T.E. (E&TC) (Part II) Examination, 2014


OPTICAL COMMUNICATION
Day and Date : Friday, 28-11-2014

Marks : 80

Time : 10.00 a.m. to 1.00 p.m.


Instructions : 1) All questions are compulsory.
2) Figures to right indicate full marks.
3) Assume suitable data if necessary.
SECTION I
2. A) Solve any three :

12

i) Define relative refractive index difference for an optical fiber and show how it may be
related to the numerical aperture.
ii) Explain about requirement of optical cable design.
iii) Explain vapour phase deposition technique for preparing fiber optics.
iv) List the characteristics of injection LASER.
B) A long single mode optical fiber has an attenuation of 0.5 dB/km when operating at a
wavelength of 1.3 m . The fiber core diameter is 6 m and the LASER source bandwidth
is 600 MHz. Compare the threshold optical powers for stimulated Brillouin and Raman
Scattering within the fiber at the wavelength specified.

3. A) Solve any two :

12

i) Briefly write about scattering losses observed in optical fiber.


ii) Explain any one type of non semiconductor LASER.
iii) Explain fiber alignment and joint loss.
B) A ruby LASER contains a crystal length 4 cm with a refractive index of 1.78. The peak
emission wavelength from the device is 0.55 m . Determine the number of longitudinal
modes and their frequency separation.

Set A

SLR-BB 190

-4-

*SLRBB190*

SECTION II
4. a) Solve any three questions :

12

i) Explain the characteristics of LED.


ii) Write a short note on principle of optical detection.
iii) Write a note on SONET.
iv) GaAs has a bandgap energy of 1.43 at 300 K. Determine the wavelength above which
an intrinsic photodetector fabricated from this material will cease to operate.
b) A photodiode has quantum efficiency of 65% when photons of energy 1.5 1019 J are
incident upon it.
i) At what wavelength is the photodiode operating ?
ii) Calculate the incident optical power required to obtain a photocurrent of 2.5 A .
When the photodiode is operating as described above.

5. a) Solve any two questions :

12

i) Define Dispersion. Explain the technique for Multimode Fiber Dispersion measurement
in the time domain.
ii) Explain the working of Avalanche photodiode with its benefits and drawbacks.
iii) Explain the structure of surface emitting LED.
b) Pulse dispersion measurements are taken over a 1.2 km length of partially graded multimode
fiber. The 3 dB widths of optical input pulses are 300 ps and the corresponding 3 dB
widths for the output pulses are found to be 12.6 ns. Assuming the pulse shapes and fiber
impulse response are Gaussian calculate :
i) The 3 dB pulse broadening for the fiber in ns km1;
ii) The fiber bandwidth length product.

_____________________

Set A

SLR-BB 191

*SLRBB191*
Seat
No.

Set

T.E. (E&TC) (Part II) Examination, 2014


ELECTRONIC SYSTEM DESIGN
Day and Date : Saturday, 29-11-2014
Time : 10.00 a.m. to 1.00 p.m.

Max. Marks : 100

Instructions : 1) Assume suitable data if required.


2) Q. No. 1 is compulsory. It should be solved in first 30 minutes
in Answer book Page No. 3. Each question carries one mark.
3) Answer MCQ/Objective type questions on Page No. 3 only.
Dont forget to mention, Q.P. Set (A/B/C/D) on Top of Page.
MCQ/Objective Type Questions
Duration : 30 Minutes

Marks : 20

1. Choose the correct option :


1) For input voltage range of 0 4 V, t2 = 4000, required size of DVM is
a) 14 bit
b) 16 bit
c) 17 bit
d) 19 bit

20

2) Selection of comparator depends on which of following characteristics


a) Maximum slew rate
b) Maximum input impedance
c) Zero output offset voltage
d) All of these
3) In CD4046, phase comparator I has signal with ________ for proper operation.
a) 50% duty cycle
b) Greater than 50% duty cycle
c) Less than 50% duty cycle
d) None of these
4) What is the reference signal value in frequency measurement for 0.01 Hz
resolution ?
a) 1 Hz
b) 0.1 Hz
c) 0.01 Hz
d) 10 Hz
5) What is input signal frequency required to generate 1 Hz sine wave using
EPROM look up table of 1 k Byte ?
a) 1000 Hz
b) 1008 Hz
c) 1024 Hz
d) 1012 Hz
6) IC XR2240 consist of
a) 8 bit counter
b) 8 digit counter c) 4 bit counter
d) 6 digit counter
7) A product specification depends on
a) Designers choice
c) Users choice

b) Manufacturers choice
d) Managers choice

8) A single IC LM2240 generate maximum delay upto few


a) Seconds
b) Hours
c) Days

d) Years
P.T.O.

SLR-BB 191
9) IC LM565 is
a) TTL PLL

*SLRBB191*

-2-

b) CMOS PLL

c) Counter

d) None of these

10) The number of digits required to count frequency of 1 MHz with 1 Hz


resolution is
a) 3
b) 4
c) 5
d) 6
11) Programmable Logic Controller (PLC) is a
a) Digital device
b) Analog device
c) Electromechanical device
d) All of the above
12) Which of the following is last cycle of washing machine ?
a) Rinse
b) Wash
c) Spin
d) Drain
13) The first instruction on the rung must always be an
a) Output
b) Input
c) Either a) or b) d) Both a) and b)
14) A typical heating control system requires
a) Direct acting controller
b) Inverse acting controller
c) Either a) or b)
d) None of the above
15) Error signal in controllers is given as
a) SP + PV
b) PV SP
c) SP PV
d) PV * SP
16) Which of the following voltage-to-current converter allows reduction of induced
noise at the receiver ?
a) Floating load
b) Grounded load
c) Either a) or b)
d) Both a) and b)
17) Beam forming is achieved by
a) Analog signal processing
c) Either a) or b)
18) ASM stands for
a) Automatic Simple Machine
c) Algorithmic State Machine

b) Digital signal processing


d) None of the above
b) Automatic Symbolic Machine
d) Asymptotic State Machine

19) Which of the following directly leads to a hardware realization ?


a) Flow chart
b) ASM chart
c) Both a) and b)
d) None of the above
20) In Microprocessor based controllers an ADC is placed at
a) Input
b) Output
c) Either a) or b) d) Both a) and b)
______________

Set A

*SLRBB191*

-3-

SLR-BB 191

Seat
No.

T.E. (E&TC) (Part II) Examination, 2014


ELECTRONIC SYSTEM DESIGN
Day and Date : Saturday, 29-11-2014

Marks : 80

Time : 10.00 a.m. to 1.00 p.m.


Instructions : 1) All questions are compulsory.
2) Assume suitable data if required.
SECTION I
2. Attempt any three :

(53=15)

a) Compare PLL565 and CD4046.


b) Design a timer using LM2240 for generation of 180 sec.
c) Explain FSK demodulator using PLL.
d) What are PCB design considerations for high frequency circuit ?
3. Attempt any one :

(101=10)

a) Design frequency ratio measurement set up to count frequency ratio upto 1 K


with 0.1 resolution. Use IC 74C926.
b) Design frequency synthesizer using CD4046 to obtain 1 KHz to 99 KHz
frequency. Crystal available is 1 MHz.
4. Design a DVM for measurement of 4 V input voltage. Explain its working.

15
Set A

SLR-BB 191

-4-

*SLRBB191*

SECTION II
5. Attempt any three :

(35=15)

a) Explain cold junction compensation in Thermocouples.


b) Explain MIMO.
c) Draw and explain PLC architecture.
d) Explain different temperature sensors.
6. Attempt any one :

10

a) Design microprocessor based proportional controller to control level with


level sensor output of 0 to 5 V for 0 to 5 m. Show all calculations and draw
flowchart.
b) Draw elevator control system and its ladder diagram.
7. a) Develop an ASM chart for an automated bank teller which will dispense. The
cash if the customer enters the correct account number and the correct amount.

b) Design current-to-voltage converter (floating load) for converting 4 mA to


20 mA to 0 5 V.

_____________________

Set A

SLR-BB 192

*SLRBB192*
Seat
No.

Set

B.E. (E & TC) (Part I) Examination, 2014


COMPUTER COMMUNICATION NETWORK
Day and Date : Tuesday, 2-12-2014
Time : 3.00 p.m. to 6.00 p.m.

Max. Marks : 100

Instructions : 1) Q. No. 1 is compulsory. It should be solved in first 30 minutes


in Answer Book Page No. 3. Each question carries one mark.
2) Answer MCQ/Objective type questions on Page No. 3 only.
Dont forget to mention, Q.P. Set (A/B/C/D) on Top of Page.
3) Figures to the right indicate full marks.
4) Assume suitable data if necessary.
MCQ/Objective Type Questions
Duration : 30 Minutes

Marks : 20

1. Choose the correct answer :


1) As the data packet moves from the upper to the lower layers, headers are
________
A) Added
B) Removed
C) Rearranged
D) Modified
2) Header of a frame generally contains
A) Synchronization bytes
B) Addresses
C) Frame identifier
D) All of the above
3) In stop and wait protocol the channel utilization is
A) Less
B) More
C) Sometimes less
D) Sometimes more
4) High-level Data Link Control (HDLC) is a ________ protocol for communication
over point-to-point and multipoint links.
A) bit-oriented
B) character-oriented
C) byte-oriented
D) none of these
5) In ______________, the station configuration is unbalanced. We have one
primary station and multiple secondary stations.
A) ABM
B) ARM
C) NBM
D) NRM
6) Why was the OSI model developed ?
A) manufacturers disliked the TCP/IP protocol suite
B) the rate of data transfer was increasing exponentially
C) standards were needed to allow any two systems to communicate
D) none of the above
P.T.O.

SLR-BB 192

-2-

*SLRBB192*

7) To deliver a message to the correct application program running on a host,


the ___________ address must be consulted.
A) Port
B) IP
C) Physical
D) None of the above
8) The __________ layer is responsible for the delivery of a message from one
process to another.
A) physical
B) network
C) transport
D) none of the above
9) Ethernet uses a ____________ physical address that is imprinted on network
interface card NIC.
A) 32 bit
B) 64 bit
C) 6 byte
D) none of the above
10) Identify the class of the following IP address : 229.1.2.3.
A) Class A
B) Class B
C) Class D
D) None of the above
11) An ARP reply is normally ___________
A) broadcast
B) unicast
C) multicast
D) none of these
12) IGMP is __________ protocol.
A) an error reporting
B) a group management
C) a transmission
D) none of these
13) A process called ___________ sends a multicast packet through WANs
that do not support physical multicast addressing.
A) delayed response
B) tunneling
C) jamming
D) none of these
14) The ports ranging from 0 to 1,023 are assigned and controlled by ICANN.
These are the ports.
A) well known
B) registered
C) dynamic
D) none of these
15) A mobile host normally uses two addresses. The host has its original address, called
the ________ address and a temporary address, called the ________ address.
A) care-of, home
B) home, care-of
C) home, foreigner
D) none of these
16) ICMP messages are divided into two broad categories
A) query and error reporting messages B) request and response messages
C) request and reply messages
D) none of these
17) One method do alert a source host of congestion is the ___________ message.
A) echo request
B) redirection
C) source-quench D) none of these
18) When the host is powered on station uses _______ client to get name of
configuration file from server.
A) DHCP
B) FTP
C) TFTP
D) None of these
19) HTTP uses services of ___________ on well known port __________
A) UDP, 81
B) TCP, 80
C) TCP, 90
D) None of these
20) HTTP functions like combination of ___________
A) TFTP and FTP
B) FTP and SNMP
C) FTP and SMTP
D) SNMP and SMTP
______________
Set A

*SLRBB192*

-3-

SLR-BB 192

Seat
No.

B.E. (E & TC) (Part I) Examination, 2014


COMPUTER COMMUNICATION NETWORK
Day and Date : Tuesday, 2-12-2014

Marks : 80

Time : 3.00 p.m. to 6.00 p.m.


Instructions : 1) Figures to the right indicate full marks.
2) Assume suitable data if necessary.
2. Answer any four questions :

(45=20)

1) Draw and explain 3 topologies along with its advantages and disadvantages.
2) Explain shortest path routing algorithm.
3) Write difference between TCP and UDP.
4) Draw and explain Rs. 232 interfacing sequences.
5) Explain in short Media Access Control.
3. Answer any two questions :

(210=20)

1) Explain with neat sketch different phases for data transfer between client and
server using TCP ?
2) Explain different framing techniques used at data link layer.
3) Explain in detail CRC method for error detection .

Set A

SLR-BB 192

-4-

*SLRBB192*

4. Answer any four questions :

(45=20)

1) Draw and explain mobile IP registration message.


2) Give methods of firewall monitoring.
3) Discuss in brief IGMP protocol.
4) Write a note on IPv6.
5) Explain the format for ICMP query messages.
5. Answer any two questions :

(210=20)

1) If message is transmitted as 1101011011 with generator polynomial


x4 + x2 + x + 1. What is the transmitted bit pattern ? (Assume leftmost bit is
MSB).
2) Explain the need of firewall. What are the packet filters and specifications of
packet filters ? Also explain access through firewall.
3) Explain various fields in header format of RTP.
_____________________

Set A

SLR-BB 193

*SLRBB193*
Seat
No.

Set

B.E. (E & TC) (Part I) Examination, 2014


VLSI DESIGN
Day and Date : Thursday, 4-12-2014
Time : 3.00 p.m. to 6.00 p.m.

Max. Marks : 100

Instructions : 1) Q. No. 1 is compulsory. It should be solved in first 30 minutes


in Answer Book Page No. 3. Each question carries one mark.
2) Answer MCQ/Objective type questions on Page No. 3 only.
Dont forget to mention, Q.P. Set (A/B/C/D) on Top of Page.
3) Figures to the right indicate maximum marks.
4) Assume suitable data if required.
Duration : 30 Minutes

MCQ/Objective Type Questions

Marks : 20

1. Choose the correct answer for following MCQs :


(120=20)
1) Process in VHDL becomes active, when
a) Change in clock statement
b) There is change in value of signal in sensitivity list
c) Change in reset signal
d) None of above
2) In an entity statement buffer is
a) Signal type
b) Signal port
c) Signal mode
d) None
3) In a VHDL code ** represents a
a) MOD operator
b) Reminder operator
c) Exponentiation
d) Absolute value
4) Variables in VHDL can be used in ______________
a) Process
b) Procedure
c) Function
d) All of the above
5) Which of the following is not a VHDL object data ?
a) Signal
b) Variable
c) Wire
d) Constant
6) Which VHDL data type can only have a value of 1 or 0 ?
a) Signal
b) Bit
c) Std-logic
d) Integer
7) The exclusive ORing of even number is always ___________
a) one
b) ten
c) zero
d) odd
8) A full adder circuit can be implemented by using
a) one suitable multiplexer
b) two suitable multiplexer
c) cannot be implemented by multiplexers
d) only decoders
P.T.O.

SLR-BB 193

*SLRBB193*

-2-

9) In Moore circuits, the outputs depends on ______________


a) Present state
b) Present state and past inputs
c) Inputs
d) None
10) A ripple counter is _______ sequential circuit.
a) Reset
b) Synchronous c) Asynchronous d) None
11) To implement 2 input AND gate using CMOS logic requires ______ no. of
MOS devices.
a) 4
b) 6
c) 8
d) none of the above
12) The TTL Fan out of capacity is ____________
a) 10
b) 20
c) 25
d) none
13) A logic signal experiences a delay in going through circuit. The two
propagation delay times are defined as
a) tPLH and tPHL
b) tDLH and tDHL
c) tHPL and tLPH
d) tLDH and tHDL
14) When VGS = 0 on an N-channel MOSFET switch, there is no ________ between
the source and the drain.
a) Voltage drop
b) Conductive channel
c) Capacitance
d) Inductance
15) A PLD has
a) Only PLAs
b) ROMs and PALs
c) Only PALs
d) None
16) The PLA has three input variables. The total number of AND gates in its
plane is ______________
a) 3
b) 6
c) 8
d) None
17) JTAG stands for
a) Joint Test Action Group
b) Joint Target Act Group
c) Joint Target Acted Group
d) None
18) Synthesis means
a) Checking correctness of design
b) Implement design into target technology
c) Conversion of design into circuit description
d) None
19) Netlist formats are
a) UCF file
b) EDIF file

c) XNF file

d) All of the above

20) The paths taken by the connection between components are decided during
a) Mapping
b) Placing
c) Routing
d) None
______________
Set A

*SLRBB193*

-3-

SLR-BB 193

Seat
No.

B.E. (E & TC) (Part I) Examination, 2014


VLSI DESIGN
Day and Date : Thursday, 4-12-2014

Marks : 80

Time : 3.00 p.m. to 6.00 p.m.


Instructions : 1) All questions are compulsory.
2) Figures to the right indicate maximum marks.
3) Assume suitable data if required.
SECTION I
2. Solve any four :

(44=16)

a) Write a short note on std_logic_1164 package.


b) What is overloading of operators ?
c) Write down the syntax of entity and architecture.
d) Write VHDL code for 1 : 4 demultiplexer.
e) Write a VHDL code for D flip flop.
3. Solve any three :

(83=24)

a) Writ syntax for :


i) Generate statement
ii) Case statement.
and explain it with example.
b) Write a VHDL code for 16 bit adder using structural modeling.
c) Explain delay models in VHDL.
d) Explain in detail simple processor design steps.

Set A

SLR-BB 193

-4-

*SLRBB193*

SECTION II
4. Solve any four :

(44=16)

a) Realize the function using PAL


F0 = m(0, 1, 4, 6)
F1 = m(2, 3, 4, 6, 7)
b) Compare CPLD and EPGA.
c) Design expression using CMOS f = ab + c .
d) Explain static load of MOS inverter in detail.
e) Write a short note on testability.
5. Solve any three :

(83=24)

a) Explain in detail about simulation process..


b) Explain flex 10k device structure along with block diagram.
c) Explain CMOS noise margin in detail.
d) Design and implement 16 : 1 multiplexer using PLA.
_____________________

Set A

SLR-BB 194

*SLRBB194*
Seat
No.

Set

B.E. (Electronics and Telecommunication) (Part I) Examination, 2014


MOBILE AND SATELLITE COMMUNICATION
Day and Date : Saturday, 6-12-2014
Time : 3.00 p.m. to 6.00 p.m.

Total Marks : 100

Instructions : 1) All questions are compulsory.


2) Assume suitable data if necessary.
3) Q. No. 1 is compulsory. It should be solved in first 30
minutes in Answer Book Page No. 3. Each question carries
one mark.
4) Answer MCQ/Objective type questions on Page No. 3
only. Dont forget to mention, Q.P. Set (A/B/C/D) on Top
of Page.
MCQ/Objective Type Questions
Duration : 30 Minutes

Marks : 20

1. MCQ/Objective type Questions :


1) In GSM the downlink frequency band is
a) 890 915 MHz
b) 890 935 MHz
c) 935 960 MHz
2) What is the data rate of TCH/F4.8 channel ?
a) 9600 bps
b) 4800 bps
c) 2400 bps
3) In GSM the interface between radio sub system and network sub system is
called
a) A bis Interface b) Um Interface c) A Interface
4) HiperLAN designed to operate in ___________ band.
a) 4 GHz
b) 5 GHz
c) 3 GHz
5) GSM multiframe consist of __________
a) 51 multiframe b) 13 multiframe c) 26 frame
6) In CDMA cellular system the chip rate is about
a) 1.22 MHz
b) 1.32 MHz
c) 1.34 MHz
7) HiperLAN is standardized by __________
a) IEEE
b) ETSI
c) GSM
P.T.O.

SLR-BB 194

-2-

*SLRBB194*

8) Wireless adhoc networks are


a) Infrastrure based
b) Wired and Infrastructure less
c) Infrastructure less and Wireless
9) The access point (AP) is the wireless equivalent of a wired LAN.
a) Switch
b) Repeater
c) Hub
10) The IEEE 802.11 standard designed to operate in __________ band.
a) 4.2 GHz
b) 2.4 GHz
c) 4.8 GHz
11) What is the typical battery voltages used in satellite ?
a) 10 20 V
b) 20 50 V
c) 50 80 V
12) Which of the following statements are true ?
a) In comparison to spin stabilized satellites, three axis stabilized satellites
have more power generation capability
b) Spin stabilized satellites are simpler in design and less expensive than
stabilized satellites
c) Both a) and b)
13) Bent pipe is as example of ___________
a) Orbit
b) Antenna
c) Transponder
14) The Broadcast Satellite Service (BSS) uses
a) One way implementation
b) Split two way implementation
c) Two way implementation
15) The first satellite launched was named
a) Sputnik
b) Sputnik 1
c) Explore
16) Sun Transit Outage occurs when
a) Satellite pass through earths shadow
b) During 11 year Sun spot cycle
c) Satellite pass directly in front of the sun
17) At GEO orbit altitude, the moons gravitational force is about twice as strong
as __________
a) Suns
b) Earths
c) Both a) and b)
18) The moons orbit is inclined to the equatorial plane by approximately ________
a) 3
b) 4
c) 5
19) On satellite wire antennas are used to provide communication for _______
a) Telephone
b) TTC and M
c) AOCS
20) Communication satellite are usually designed to have typical operating
life time.
a) 20 Years
b) 50 Years
c) 10 15 Years
______________
Set A

*SLRBB194*

-3-

SLR-BB 194

Seat
No.

B.E. (Electronics and Telecommunication) (Part I) Examination, 2014


MOBILE AND SATELLITE COMMUNICATION
Day and Date : Saturday, 6-12-2014
Time : 3.00 p.m. to 6.00 p.m.

Marks : 80

Instructions : 1) All questions are compulsory.


2) Assume suitable data if necessary.
SECTION I
2. Solve any four :

(44=16)

1) Explain the frequency reuse technique.


2) Explain GSM frame structure.
3) Explain the format of an IEEE 802.11 PHY frame using direct sequence spread
spectrum.
4) Explain advantages and disadvantages of WLAN.
5) Explain speech encoding and modulation in GSM.
3. Solve any three :

(38=24)

1) Explain forward link and reverse link structure in IS-95 CDMA system.
2) What is handover ? Explain handover scenario at cell boundary.
3) Explain GSM architecture.
4) Explain GSM common control and dedicated control channels.

Set A

SLR-BB 194

-4-

*SLRBB194*

SECTION II
4. Solve any four :

(44=16)

1) Short note on latitude, longitude and sub satellite point.


2) Explain in detail Molniya orbit.
3) Short note on Iridium.
4) Short note on communication sub system on satellite.
5) Explain in detail launch and launch vehicles.
5. Solve any three :

(38=24)

1) Explain Telemetry, Monitoring, Tracking and Command system.


2) Explain power subsystem on satellite.
3) Explain inclination changes effects of the sun and the moon.
4) A satellite is in an elliptical orbit with perigee of 1000 km and an apogee of
4000 km. using a mean earth radius of 6378.14 km. Find the period of the
orbit in hours, minutes and seconds. Also the eccentricity of the orbit.
_____________________

Set A

SLR-BB 195

*SLRBB195*
Seat
No.

Set

B.E. (E&TC) (Part I) Examination, 2014


RANDOM SIGNAL THEORY AND CODING
Day and Date : Tuesday, 9-12-2014
Time : 3.00 p.m. to 6.00 p.m.

Max. Marks : 100

Instructions : 1) Assume suitable data if necessary.


2) Q. No. 1 is compulsory. It should be solved in first 30 minutes in
Answer Book Page No. 3. Each question carries one mark.
3) Answer MCQ/Objective type questions on Page No. 3 only.
Dont forget to mention, Q.P. Set (A/B/C/D) on Top of Page.
Duration : 30 Minutes

MCQ/Objective Type Questions

Marks : 20

1. Multiple choice questions :


20
1) The linear block code is a subclass of
a) Cyclic code
b) Convolutional code
c) Parity code
d) Turbo code
k
2) All the 2 k-tuples of a co-set have the same syndrome and the syndromes of the
different co-sets are
a) Same
b) Different
c) Zero
d) One
3) A parity check code can
a) Detect a single bit error
b) Correct a single bit error
c) Detect two bit error
d) Correct a two bit error
4) If a valid codeword in non systematic cyclic code is 1101. Which of the following is
also a valid codeword ?
a) 1001
b) 0011
c) 0000
d) 1110
5) The _________ of errors is more difficult than __________
a) Correction, detection
b) Detection, correction
c) Creation, correction
d) Creation, detection
6) The minimum Hamming distance is ___________ minimum hamming weight.
a) Less than
b) Greater than
c) Not equal to
d) Equal to
7) Calculated syndrome is added to received vector to obtain valid code vector.
a) True
b) False
c) Can not be calculated
d) None of these
8) Which of the following is incorrect ?
a) P(S) = 1
b) P( A ) = P(A ) 1
c) 0 P(A ) 1
d) If A and B are mutually exclusive then P(A + B) = P(A) + P(B)
P.T.O.

SLR-BB 195

*SLRBB195*

-2-

9) Event A and B are statistically independent if,


a) A and B occurs simultaneously
b) A and B occurs at different times
c) Occurrence of A includes occurrence of B
d) None of these
10) A process of turbo-code decoding starts with the formation of ________ for each
data bit, which followed by choosing data bit value that corresponds to
____________ for that data bit.
a) MAP, APP
b) MAP, MAP
c) APP, MAP
d) None of these
11) In convolutional code, coding gain in dB is less than or equal to _________
a) r.df/3
b) r.df/2
c) r.df
d) None
12) If free distance of convolutional code is 7, number of bits corrected are
a) 2
b) 3
c) 4
d) 5
13) The Viterbi algorithm is an optimal ________ method for minimizing probability of
sequence error.
a) Encoding
b) Decoding
c) Both
d) None
14) Catastrophic error propagation occurs when all ones data state will have
a) Zero weight
b) Unity weight
c) Two weight
d) None of these
15) For a state diagram shown below what are generator sequences g1 and g2 of
convolutional encoder.

a) 11, 10

b) 10, 11

c) 11, 01

d) 01, 00

16) For above state diagram of convolutional encoder, what is the code rate and
constraint length
a) 1/2, 2
b) 1/2, 3
c) 1/3, 4
d) 1/3, 3
17) Branch metric is calculated in soft decision Viterbi decoding using
a) Hamming distance
b) Euclidean distance
c) Transfer function
d) None of these
18) MAP algorithm calculates state metric in
i) Forward direction
ii) Reverse direction
a) i
b) ii
c) Both
d) None
19) Catastrophic error occurs in
a) Systematic convolutional code
c) a) and b)

b) Non-systematic convolutional code


d) Does not depend on code type

20) The error correcting capability of convolutional code is


df 1
a) df
b) df 2
c)
2
2
2
______________

d) None

Set A

*SLRBB195*

SLR-BB 195

-3-

Seat
No.

B.E. (E&TC) (Part I) Examination, 2014


RANDOM SIGNAL THEORY AND CODING
Day and Date : Tuesday, 9-12-2014
Time : 3.00 p.m. to 6.00 p.m.

Marks : 80

Instructions : 1) All questions are compulsory.


2) Assume suitable data if necessary.
SECTION I
2. A) Attempt any three :
a) What is standard array ? How it is useful in decoding linear block codes ?
b) How error detection and correction is done in linear block code ? Explain it.
c) State and prove Bayes theorem.
d) Explain in brief statistical average and time average.

12

B) Design a feedback shift register encoder for an (8, 5) cyclic code with a generator
g(x) = 1 + x + x2 + x3. Use the encoder to find codeword for message 10101 in
systematic form.
3. A) Attempt any two :
a) The parity check matrix for (7, 4) block code is given by
1 0
H = 1 1
1 1

0
1

1
0

0
0

1
0

8
8

0
0
1

Draw the decoder.


b) The generator polynomial of (6, 3) cyclic code g(x) = 1 + x2. Find all codewords
using non systematic method.
c) Define :
i) Vector space
ii) Vector subspace.
B) a) The joint probability function of two random variables X and Y is given by
f(x, y) = c(x2 + 2y)
x = 0, 1, 2, y = 1, 2, 3, 4
=0
otherwise
Find :
i) Value of C
ii) P(X = 2, Y = 3) iii) P(X 1, Y > 2).
b) Define :
i) Standard deviation

ii)

Expectation.

8
4

Set A

SLR-BB 195

-4-

*SLRBB195*

SECTION II
4. A) Attempt any three :
12
a) What is maximum likelihood algorithm ? How it is used in Viterbi decoding ?
b) For convolutional encoder, whose generator sequences are g1 = 111, g2 = 101.
i) Draw state diagram representing starting state and ending state separately.
ii) Write state equations.
c) Define :
i) Coding gain
ii) Catastrophic error propagation.
d) Explain in brief Turbo decoder.
B) For the given convolutional encoder.

i) Find encoder output for an input 101011.


ii) Draw trellis diagram.
5. A) Attempt any two :
10
a) Explain add compare select computation used in decoder implementation of
convolutional codes.
b) Explain soft decision Viterbi decoding.
c) For recursive systematic convolutional code shown below.

Find encoded sequence for data 1110.

1
B) For a convolutional encoder rate = 3 , generator sequences are g1 = 111 and g2 = 101. 10
i) Draw encoder trellis diagram.
ii) If the received sequence is 11 01 01 10 01 ..... using viterbi decoding algorithm
find input message.
________________

Set A

SLR-BB 196

*SLRBB196*
Seat
No.

Set

B.E. (E and TC) (Part I) Examination, 2014


IMAGE PROCESSING (Elective I)
Day and Date : Thursday, 11-12-2014
Time : 3.00 p.m. to 6.00 p.m.

Max. Marks : 100

Instructions : 1) Q. No. 1 is compulsory. It should be solved in first 30


minutes in Answer Book Page No. 3. Each question carries
one mark.
2) Answer MCQ/Objective type questions on Page No. 3 only.
Dont forget to mention, Q.P. Set (A/B/C/D) on Top of Page.
MCQ/Objective Type Questions
Duration : 30 Minutes

Marks : 20

1. Choose the correct answer :

(120=20)

1) Digitizing the amplitude value of continuous image is called


a) Quantization
b) Sensing the image
c) Acquisition
d) Sampling
2) If no. of storage bits are 524288 for a 8-bit image then what will be the size of
the image ?
a) 128 128
b) 256 256
c) 512 512
d) 1024 1024
3) Two pixels p and q with values from set v are 8-adjacent if q is in the set
a) N4 (p)
b) N8 (p)
c) ND (p)
d) None
4) A measure of the degree to which a pure color is diluted by white light is
given by
a) Hue
b) Intensity
c) Cyan
d) Saturation
5) Principal component analysis is nothing but
a) K-L Transform b) Haar transform c) DCT

d) FFT

6) FFT accomplishes, for an image of size M N, on the order of


a) MN log2 MN operations
b) MN log 10 MN operations
c) MN operations
d) None
7) Following is not a gray level transformation
a) Image negative
b) Power law transformation
c) Log transformation
d) Histogram
P.T.O.

SLR-BB 196

*SLRBB196*

-2-

8) Histogram is narrow and centered towards the middle of gray scale


a) Low contrast image
b) High contrast image
c) Bright image
d) None
9) f Z9 Z5 + Z8 Z6 is
a) Sobel operator
c) Robert cross gradient operator

b) Prewitt operator
d) None

10) Image restoration process is ___________


a) Objective
b) Global
c) Subjective

d) Local

11) Image compression process relates to


a) Reduction in BW
b) Morphology
c) Enhancement
d) None
12) Blurring effect is observed in following filters
a) Arithmetic mean
b) Geometric mean
c) Harmonic mean
d) None
13) MPEG is ___________ standard.
a) Still image coding
c) Run length coding

b) Video coding
d) None

14) Erlang noise is also called as


a) Salt and pepper noise
c) Gaussian

b) Uniform noise
d) Gamma

15) Which method is not used for estimation of degradation function ?


a) By observation
b) By experimentation
c) Mathematical modeling
d) None
16) Reducing the object size is nothing but
a) Dilation
b) Erosion
c) Structuring element
d) None
17) In morphology closing process is denoted as
b) A B
c) A  B
a) A B

d) AB

18) In morphology Dilation process is denoted as


b) A B
c) A  B
a) A B

d) AB

19) SNR is related to which fidelity ?


a) Subjective
b) Objective

c) Error

d) None

20) Prewitt operator is used for


a) Line detection
c) Edge detection

b) Point detection
d) None
______________

Set A

*SLRBB196*

-3-

SLR-BB 196

Seat
No.

B.E. (E and TC) (Part I) Examination, 2014


IMAGE PROCESSING (Elective I)
Day and Date : Thursday, 11-12-2014
Time : 3.00 p.m. to 6.00 p.m.

Marks : 80

Instructions : 1) All questions are compulsory.


2) Figures to the right indicate full marks.
SECTION I
2. Solve any four :

20

i) What do you mean by CMYK color model ? Explain.


ii) Explain Fundamental steps Digital Image processing with block diagram.
iii) Explain Bit plan Slicing.
iv) Explain Model for Degradation/Restoration in image restoration.
v) What is image sharpening, explain any one method ?
3. i) Explain Harr and Hadmard transform in details.

10

ii) With reference to following example, Explain Histogram Equalization ?

10

OR
ii) Explain frequency domain filtering with example.

10
Set A

SLR-BB 196

-4-

*SLRBB196*

SECTION II
4. Solve any four from the following :

(54=20)

i) Explain Opening and Closing process with an example.


ii) Why need of compression, explain JPEG with block diagram ?
iii) Explain the least square filtering to image restoration.
iv) Explain Line detection in image.
v) How many types of estimation degradation function in image ? Explain any
one technique.
vi) Write short note on segmentation.
5. i) Explain the image Transform coding for using compression with its block
diagram.

10

ii) Explain of Edge Detection. Using Sobel, Prewwits, Robert Cross and Canny
Operator.
10
OR
ii) Explain different morphological operations in details with necessary diagrams. 10

_____________________

Set A

SLR-BB 197

*SLRBB197*
Seat
No.

Set

B.E. (E &TC) (Part II) Examination, 2014


BROADBAND COMMUNICATION
Day and Date : Tuesday, 25-11-2014
Time : 3.00 p.m. to 6.00 p.m.
N.B.:

Max. Marks : 100

1) Figures to the right indicate full marks.


2) Assume suitable data if necessary.
3) Q. No. 1 is compulsory. It should be solved in first 30 minutes in
Answer Book Page No. 3. Each question carries one mark.
4) Answer MCQ/Objective type questions on Page No. 3 only. Dont
forget to mention, Q.P. Set (A/B/C/D) on Top of Page.
MCQ/Objective Type Questions

Duration : 30 Minutes

Marks : 20

1. Tick mark the correct option :

20

1) The _____________ is absent in the LAPF core frame format.


a) Control field
b) User plane
c) Addressing
d) Extention Addressing Bit (EA)
2) Loopback testing is used in ___________ series for testing.
a) I 100
b) I 200
c) I 600
d) I 700
3) The 3 levels of protocols in X.25 are physical level, ___________ and packet
level respectively.
a) session level
b) link level
c) application level
d) transport level
4) At the datalink layer LAPD is used to provide data link control service with
a) Error control
b) Flow control
c) Both a) and b) d) None of these
5) The packet handling functions of _________ operates at layer three of the OSI model.
a) X.25
b) Frame relay
c) ATM
d) B-ISDN
6) Services in this category are referred to a broadcast services.
a) Messaging services
b) Retrieval services
c) Distribution services
d) All of these
7) The length of________ field and of DLCI is determined by the address extention bits.
a) Address
b) FCS
c) SAPI
d) DE
8) _______ series classifies services into lower level bearer services a higher level services.
a) I 100
b) I 300
c) I 200
d) I 400
P.T.O.

SLR-BB-197

*SLRBB197*

-2-

9) National destination code of ISDN address structure is


a) Of variable length and a portion of national ISDN number
b) Fixed length of national ISDN number
c) ISDN subscription number
d) None of these
10) Timing is major function of functional group.
a) TE
b) NT2
c) NT1

d) TA

11) Cell Loss Priority (CLP) bit is used to provide guidence to the network in the error of
a) Error
b) Congestion
c) Transmission
d) None of these
12) Identifier a particular VP link
a) Virtual channel link
c) Virtual path connection

b) Virtual path link


d) Virtual path identifier

13) In cell delineation state diagram in the ___________ state, a cell structure is
assumed.
a) Hunt
b) Presync
c) sync
d) HEC
14) The most recent addition to the AAL specification is the type___________ protocol.
a) 2
b) 3
c) 4
d) 5
15) ___________ service is perhaps the simplest service to define.
a) CBR
b) VBR
c) ABR
d) UBR
16) The peak cell rate defines___________ on the traffic that can be submitted by a
source on an ATM connection.
a) Lower bound
b) Upper bound
c) Intermediate bound
d) None of these
17) The SONETs ___________layer is responsible for end to end transport of data
at the appropriate signalling speed.
a) Path
b) Line
c) Section
d) Photonic
18) AAL type 1 operation deal with
a) Constant bit rate
c) Unspecified bit rate

b) Available bit rate


d) Real time variable bit rate

19) Reference point ___________ provides a non ISDN interface.


a) T
b) U
c) R
d) S
20) ___________ is the time it takes for a node to perform the necessary processing
as it switches data.
a) Node delay
b) Propagation delay
c) Transmission time
d) None of these

______________

Set A

*SLRBB197*

-3-

SLR-BB 197

Seat
No.

B.E. (E &TC) (Part II) Examination, 2014


BROADBAND COMMUNICATION
Day and Date : Tuesday, 25-11-2014

Marks : 80

Time : 3.00 p.m. to 6.00 p.m.


N.B. : 1) All questions are compulsory.
2) Figures to the right indicate full marks.
3) Assume suitable data if necessary.
SECTION I
2. Attempt any four :

20

a) Explain internal virtual circuit and datagram operation.


b) Briefly explain about the evolution of ISDN.
c) Describe functions of NT1 and NT2 functional groups of ISDN.
d) Explain about transmission structures used in ISDN. What are commonly used
channels of ISDN ?
e) With diagram explain ISDN protocols at the user-network interface.
3. A) i) With suitable sketch explain ISDN architecture.
ii) Describe possible configuration for ISDN user-network interfaces.
B) i) What is the importance of standards ? Explain structure of 1-series
recommendation.
ii) What are different types of frame relay access modes ?

4
6
8
2

OR
B) i) Explain LAPF core protocol format.
ii) List the services provided with ISDN.

8
2

Set A

SLR-BB-197

-4-

*SLRBB197*

SECTION II
4. Attempt any four :

20

a) Explain ATM protocol architecture.


b) With flow chart explain the effect of error in cell header.
c) Briefly write on working principle of ATM switch.
d) With diagram explain ATM cell format of network-network interface.
e) List the services provided with AAL.
5. Answer any two :

20

a) i) With diagram show ATM protocol hierarchy. How many functional levels
are these in physical layer ? Explain.

ii) Describe the types of buffers used in ATM switch.

b) i) With diagram explain HEC operation at receiver.

ii) Briefly write on SONET system hierarchy.

c) i) Explain interactive and distributive services provided with broad band ISDN.
ii) With flow chart explain call establishment using virtual paths.

6
4

_____________________

Set A

SLR-BB 198

*SLRBB198*
Seat
No.

Set

B.E. (E & TC) (Part II) Examination, 2014


AUDIO VIDEO ENGINEERING
Day and Date : Thursday, 27-11-2014
Time : 3.00 p.m. to 6.00 p.m.

Max. Marks : 100

Instructions : 1) Figures to the right indicate full marks.


2) Assume suitable data if necessary.
3) Q. No. 1 is compulsory. It should be solved in first 30 minutes
in Answer book Page No. 3. Each question carries one mark.
4) Answer MCQ/Objective type questions on Page No. 3 only.
Dont forget to mention, Q.P. Set (A/B/C/D) on Top of Page.
MCQ/Objective Type Questions
Duration : 30 Minutes

Marks : 20

1. 1) The laser beam scans the tracks from __________


a) Center to outer
b) At the same place
c) Outer to center
d) None of these
2) The technique of removing the repetition of pattern within each frame is ______
a) Temporal redundancy
b) Spatial redundancy
c) Video capture
d) Audio capture
3) Dolby methods gives improvement in SNR by ____________ dB.
a) 5
b) 10
c) 15
d) None of these
4) Playback discs are made by ____________ process.
a) Recording
b) Induction
c) Stamping

d) None of these

5) ___________ motor is used for tape transport.


a) Synchronous
b) Servo
c) Stepper

d) Asynchronous

6) Compact disc surface doesnt wear out because the sensor is ________
a) Mechanical
b) Electrical
c) Optical
d) Magnetic
7) The repetition rate for scanning raster is________________ frames/sec in
motion pictures.
a) 25
b) 24
c) 50
d) 1.5625

P.T.O.

SLR-BB 198

-2-

*SLRBB198*

8) The function of flywheel in tape is to reduce ________ noise.


a) Rumble
b) Wow
c) Flutter
d) Hissing
9) Final radiated picture signal for TV transmission has picture to sync ratio
equal to __________
a) 3/4
b) 10/4
c) 4/3
d) 4/10
10) Erasing frequency for tape is 5 times or more the ________ audio frequency.
a) Lowest
b) Highest
c) Normal
d) Average
11) Ident pulses are transmitted during ___________
a) Horizontal trace intervals
b) Horizontal blanking intervals
c) Vertical trace intervals
d) Vertical blanking intervals
12) I signal in NTSC colour TV system is located at 57 w.r. to _________
a) Colour burst
b) R Y
c) B Y
d) G Y
13) To avoid over modulation, colour difference signals are weighted as follows
a) 0.877 (R Y) b) 0.493 (B Y) c) Both a) and b) d) None of these
14) Colour burst is of 8 to 11 cycles of _________
a) Colour subcarrier
b) Picture subcarrier
c) Picture carrier
d) Sound carrier
15) In PAL colour TV system the phase of _________ subcarrier is reversed
from + 90 to 90 at the line frequency.
a) Q
b) I
c) V
d) U
16) The weighted colour difference signals are in SECAM are
a) 1.9 (R Y), 1.5 (B Y)
b) 1.9 (R Y), 1.5 (B Y)
c) 1.9 (R Y), 1.5 (B Y)
d) 1.9 (R Y), 1.5 (B Y)
17) The size of chromacity diagram reduces as ____________ reduces.
a) Frequency
b) Wavelength c) Brightness
d) None of these
18) The colour killer circuit is on for ________
a) Monochrome signal transmission b) Monochrome signal reception
c) Colour signal transmission
d) Colour signal reception
19) In colour TV intensity of colour produced depends on the intensity of ______
a) Beam currents b) Grid voltage c) Grid current
d) Mask voltage
20) Colour subcarrier is located in the same bandwidth of 7 MHz for 625 line
system by the use of __________
a) Phase alternation
b) Frequency inter leaving
c) Voltage doubling
d) Current boosting
______________
Set A

*SLRBB198*

-3-

SLR-BB 198

Seat
No.

B.E. (E & TC) (Part II) Examination, 2014


AUDIO VIDEO ENGINEERING
Day and Date : Thursday, 27-11-2014

Marks : 80

Time : 3.00 p.m. to 6.00 p.m.


Instructions : 1) All questions are compulsory.
2) Figures to the right indicate full marks.
3) Assume suitable data if necessary.
SECTION I
2. Attempt any four :
(45=20)
1) Define the term multimedia. List the elements of multimedia.
2) Explain principle of working of image orthicon camera tube.
3) Compare coarse and micro grooves.
4) What is the need for biasing in magnetic recording and reproduction ? Explain
how AC biasing is achieved.
5) With the help of neat diagram explain the use of front porch and back porch in
synchronizing pulses.
3. Attempt any two :

(210=20)

a) i) Draw and explain low level TV transmitter.


ii) Compare low level and high level TV transmitter.

6
4

b) i) What is the relation between gapsize, tape speed and frequency of audio
signal ? Derive the relation.

ii) Explain importance of relationship to get optimum output for audio


bandwidth.
c) i) Draw the block diagram of monochrome TV transmitter and explain each
block in detail.

4
6

ii) What are different types of cutter stylus and playback needles used in disc
recording and reproduction system.

Set A

SLR-BB 198

-4-

*SLRBB198*

SECTION II
4. Attempt any four :

(45=20)

1) Explain the terms degaussing, purity and convergence related to colour TV


receiver.
2) With neat block diagram explain PAL decoder.
3) State the requirements of RF tuner.
4) Briefly write about satellite TV system.
5) Prove the colour difference signal G Y is not suitable for transmission.
5. Attempt any two :

(210=20)

a) With the help of block diagram explain SECAM encoder and decoder.
b) Why is automatic gain control is required in TV receiver ? Explain how EHT
helps in proper working of TV ?
c) i) What is chromacity diagram ? Explain its significance.
ii) Explain the terms

6
4

1) Luminance
2) Chrominance
3) Hue
4) Saturation.
_____________________

Set A

SLR-BB 199

*SLRBB199*
Seat
No.

Set

B.E. (Electronics and Telecomm.) (Part II) Examination, 2014


EMBEDDED SYSTEMS
Day and Date : Saturday, 29-11-2014
Time : 3.00 p.m. to 6.00 p.m.

Max. Marks : 100

Instructions : 1) Q. No. 1 is compulsory. It should be solved in first 30 minutes


in Answer Book Page No. 3. Each question carries one mark.
2) Answer MCQ/Objective type questions on Page No. 3 only.
Dont forget to mention, Q.P. Set (A/B/C/D) on Top of Page.
3) Draw neat diagram whenever necessary.
4) Assume suitable data if required.
5) Figures to the right indicate full marks.
MCQ/Objective Type Questions
Duration : 30 Minutes

Marks : 20

1. Choose the correct answer :


(120=20)
1) The ARM register ______________ is called link register.
a) R12
b) R13
c) R14
d) R15
2) ARM uses the thumb _________ bit instruction set to improve code density.
a) 32
b) 8
c) 64
d) 16
3) In LPC 2148 _____________ pin select register is used to configure port pins
P1.1 to P1.31.
a) PINSEL2
b) PINSEL0
c) IODIR2
d) IOSET0
4) The force logic 1 on port pin P0.15, _________ register is used in LPC 2148.
a) IOSET0
b) IOSET1
c) IOCLR0
d) IODIR3
5) The carry Hag is set to ____________ it no borrow occurs and __________
if a borrow occurs.
a) zero, zero
b) zero, one
c) one, zero
d) none of the above
6) In ARMTTDMI-S S stands for
a) synthesizable
b) synthesis
c) small
d) none of the above
7) The on-chip ADCS of LPC 2148 has _____________ bit resolution and
_______ conversion time.
a) 8, 2.44 sec
b) 10, 2.44 sec c) 12, 1.14 sec d) 10, 1.44 sec
P.T.O.

SLR-BB 199

*SLRBB199*

-2-

8) In LPC 2148 slot 0 has __________ priority.


a) Highest
b) lowest
c) middle

d) none of the above

9) RTC is designed to provide a set of counters to measure a time when


a) user mode is selected
b) normal mode is selected
c) protected mode is selected
d) cannot say
10) An embedded system executes a specific program repeatedly called as
a) Reactive system
b) Tightly constrained
c) Single functioned
d) None of the above
11) Inter task communication can be done through
a) Mail boxes
b) Queues
c) Pipes

d) All of the above

12) Which command is used to set terminal IO characteristic ?


a) tty
b) ctty
c) ptty
d) stty
13) Which command is used to print a file ?
a) print
b) ptr
c) lpr

d) none of the above

14) Race condition can be avoided by using


a) semaphore
b) mutex
c) socket

d) both a) and b)

15) What do you mean by Linux utilities ?


a) System software
b) Predefined function
c) Application software
d) Both a) and b)
16) ___________ is a high level abstraction over semaphore.
a) Shared memory
b) Message passing
c) Monitor
d) Mutual exclusion
17) FIFO scheduling is
a) Preemptive scheduling
c) Deadline scheduling

b) Non Preemptive scheduling


d) Fair share scheduling

18) Two different users can share the same process.


a) False
b) True
c) Cannot say
19) A data structure is called as
a) Process control block
c) Memory control block

b) User control block


d) None of the above

20) What command do you use to create Linux file system ?


a) fdisk
b) mkfs
c) fsck
d) mount
______________
Set A

*SLRBB199*

-3-

SLR-BB 199

Seat
No.

B.E. (Electronics and Telecomm.) (Part II) Examination, 2014


EMBEDDED SYSTEMS
Day and Date : Saturday, 29-11-2014

Marks : 80

Time : 3.00 p.m. to 6.00 p.m.


Instructions : 1) All questions are compulsory.
2) Draw neat diagram whenever necessary
3) Assume suitable data if required.
4) Figures to the right indicate full marks.
SECTION I
2. Attempt any four :

(54=20)

1) Block diagram of Embedded system.


2) Design metrics used in designing Embedded system.
3) Memory map for LPC 2148.
4) On chip peripheral of LPC 2148 as timers.
5) Concept of PLL used on LPC 2148.
3. Attempt any two :

(102=20)

1) Draw and explain interfacing at RF modules with ARM processor.


2) Draw and describe data flow model of ARM core.
3) Explain with one application example of communication protocol CAN.

Set A

SLR-BB 199

-4-

*SLRBB199*

SECTION II
4. Attempt any four :

(54=20)

1) Kernel services in operating system.


2) Event functions used in Inter process communication.
3) Basic functions of RTOS.
4) Synchronization in UCOS II.
5) Configuring and Booting the Kernel in embedded Linux.
5. Attempt any two :

(102=20)

1) Explain the block diagram concept of round robin time slicing scheduling
model for R TOS. Explain with example.
2) Explain the concept of multithreading using P threads. Explain syslog and
strace debug technique used in Linux.
3) Explain the concept of semaphores with block diagram. Give the
significance of p and v semaphore. What is meant by critical section ?
Explain with example.
_____________________

Set A

SLR-BB 2

*SLRBB2*
Seat
No.

Set

F.E. (Part I) (New) Examination, 2014


BASIC MECHANICAL ENGINEERING
Day and Date : Wednesday, 10-12-2014
Time : 10.00 a.m. to 1.00 p.m.

Total Marks : 70

Instructions : 1) Neat diagrams must be drawn whenever necessary.


2) Figures to the right indicate full marks.
3) Make suitable assumptions if necessary and mention them
clearly.
4) Use of log tables and non programmable single memory
calculator is allowed.
5) Q. No. 1 is compulsory. It should be solved in first 30 minutes
in Answer Book Page No. 3. Each question carries one mark.
6) Answer MCQ/Objective type questions on Page No. 3 only.
Dont forget to mention, Q.P. Set (A/B/C/D) on Top of Page.
MCQ/Objective Type Questions
Duration : 30 Minutes

Marks : 14

1. 1) An isothermal process is governed by


a) Boyles Law
b) Charles Law
c) Joules Law
d) Gay Lussacs Law
2) Heat transfer during polytropic process is given by
a) ( n)/ 1 Work done
b) ( n)/ + 1 Work done
d) ( 1)/n 1 Work done
c) (n )/ 1 Work done
3) During a cycle the heat transfer are given as 120 KJ, 60 KJ, 48 KJ and
12 KJ the net work transfer of the cycle is
a) 60000 Nm
b) 24000 Nm
c) 12000 Nm
d) 4400 Nm
4) PMM of 1st kind is impossible according to
a) Zeroth law of thermodynamics
b) 1st law of thermodynamics
c) 2nd law of thermodynamics
d) 3rd law of thermodynamics
5) A compressor is used in
a) Gas turbine power plant
b) Starting and supercharging of IC engine
c) In pneumatic drills
d) All of above
P.T.O.

SLR-BB 2

-2-

*SLRBB2*

6) Which of the following is an impulse turbine ?


a) Pelton wheel
b) Francis turbine
c) Kaplan turbine
d) None of above
7) A surge tank is used to
a) Increase pressure energy of water
b) Increase kinetic energy of water
c) Prevent water hammer effect
d) None of above
8) In two stroke engine one power stroke is obtained in
a) One revolution of crankshaft
b) Two revolution of crankshaft
c) Four revolution of crankshaft
d) None
9) The body dimensions of a person are taken into consideration in
a) Ergonomic consideration
b) Thermodynamics
c) Air conditioning
d) Aesthetic consideration
10) Failure of component due to pushing action
a) Fatigue failure
b) Tensile failure
c) Compressive failure
d) Shear failure
11) Method of joining two work piece made of two dissimilar material above
450C is called
a) Welding
b) Brazing
c) Soldering
d) None
12) Drilling of work piece can be carried out by
a) Lathe
b) Drilling machine
c) Both
d) None
13) The gears used to contact two parallel shafts are known as
a) Spiral gears
b) Bevel gears
c) Spur gears
d) Rack and pinion
14) Which of the following is used when the shafts are separated by large
distance ?
a) Belts
b) Coupling
c) Gear
d) All of above
______________

Set A

*SLRBB2*

-3-

SLR-BB 2

Seat
No.

F.E. (Part I) (New) Examination, 2014


BASIC MECHANICAL ENGINEERING
Day and Date : Wednesday, 10-12-2014
Time : 10.00 a.m. to 1.00 p.m.

Marks : 56

Instructions : 1)
2)
3)
4)
5)
6)

All questions are compulsory.


Q. 2 and Q. 4 are Short Answer Type Questions.
Q. 3 and Q. 5 are Long Answer Type Questions.
Neat diagrams must be drawn whenever necessary.
Figures to the right indicate full marks.
Make suitable assumptions if necessary and mention them
clearly.
7) Use of log tables and non programmable single memory
calculator is allowed.
SECTION I

2. Attempt any five of the following :

15

a) State different statements of second law of thermodynamics.

b) State different types of work and explain any one of them.

c) Explain in brief point function and path function with suitable examples.

d) Derive an expression of work done for adiabatic process.

e) State applications of pumps and compressors.

f) Explain in brief working of double acting reciprocating pump.

g) Compare BWR and PWR.

3. Attempt any three of the following :

13

a) Steam enters a steam turbine with a velocity of 40 m/s and enthalpy of 2500 kJ/kg
and leaves with a velocity of 90 m/s and enthalpy of 2030 kJ/kg. Heat lost
from turbine to surrounding is 240 kJ/min. Find power developed by turbine if
steam flow rate is 6000 kg/hr.

Set A

SLR-BB 2

-4-

*SLRBB2*

b) A closed system undergoes a thermodynamic cycle consisting of 4 processes.


The following data gives the work and heat transfer for each of the process.
Process Heat transfer in kJ/min Work transfer in kJ/min
12
Nil
10000
23
10000
Nil
34
2000
14000
41
6000
2000
Show that the data is consistent with first law of thermodynamics and
determine internal energy change for each process.
c) 0.2 kg of air at 5 bar and 200C expands adiabatically to a final pressure of
1.2 bar. Find the work done and change in internal energy. Assume R = 287 J/kgK
and = 1.4 for air. Cp = 1.005 and Cv = 0.71 kJ/kg K.
d) Explain with neat sketch working of Francis turbine.
e) Draw a neat sketch of steam power plant. Explain function of air-pre heater,
economizer and super heater.

4
4
4

SECTION II
4. Attempt any five of the following :
a) Distinguish between 2-stroke engine and 4-stroke engine (any six points).
b) Write the comparison between Belt drive and Chain drive.
c) Enlist different types of gears. Explain any one of them.
d) Explain in short stress-strain diagram for ductile material.
e) Define the terms : Hardness, Toughness, Ductility.
f) Compare Electric Resistance Welding with Electric Arc Welding.
g) Explain in short Soldering process.

15
3
3
3
3
3
3
3

5. Attempt any three of the following :


13
a) Explain with neat sketch brazing process. State its advantages, limitations
and application.
4
b) Explain stepwise process of mechanical engineering design.
4
c) An engine works on Otto cycle. Initial pressure and temperature of air are 1 bar
and 40C. Heat supplied per kg of air is 825 kJ. Find the efficiency of cycle
and maximum cycle temperature.
4
d) Two pulleys, one 450 mm diameter and other 200 mm diameter are on parallel
shafts 1.95 m apart and connected by crossed belt. Find power transmitted
by the belt if larger pulley rotates at 200 rpm, maximum permissible tension
in belt is 1 kN and coefficient of friction is 0.25.
5
_____________________

Set A

SLR-BB 20

*SLRBB20*
S

S.E. (Civil) (Part I) Examination, 2014


STRUCTURAL MECHANICS I
Day and Date : Thursday, 11-12-2014
Time : 10.00 a.m. to 1.00 p.m.

Max. Marks : 100

Instructions : 1) Assume suitable data, if necessary.


2) Q. No. 1 is compulsory. It should be solved in first 30 minutes in
Answer Book Page No. 3. Each question carries one mark.
3) Answer MCQ/Objective type questions on Page No. 3 only. Dont
forget to mention, Q.P. Set (A/B/C/D) on Top of Page.
MCQ/Objective Type Questions

Duration : 30 Minutes

Marks : 20

1. Choose the correct answer.


1) The bending moment M at the fixed end of a cantilever beam subjected to a point load P
at the free end is
a) P

b) PL
L

c) P

d) P
L

2) A column subjected to an eccentric load is subject to


a) bending stresses only
b) direct stresses only
c) direct and bending stresses
d) none of these

3) A compound bar in a beam is a beam


a) with different dimensions of the same material
b) with different dimensions of the different materials
c) with same dimensions of the different materials
d) both b) and c)

4) The nature of stress on the top of a cantilever beam subjected to a downward load P at
its free end is
a) compressive
b) tensile
c) zero
d) cant say

5) The point of contraflexure in a beam is a point where


a) B. M. is max.
b) B. M. is zero
c) B. M. changes its sign
d) S. F. changes its sign

6) The thermal strain is given by

a)


tl

b)


c)


d)

7) Elastic materials show


a) sufficient warming before failure
c) almost no warning before failure

1
b) the deformations are negligible before failure
d) none of these

8) The shear force induces


a) Extension
b) Compression

c) Bending

d) None of these

1
P.T.O.

SLR-BB 20

*SLRBB20*

-2-

9) Poissons ratio ( ) is the ratio of

a)

b)
2

c)
s

d)
s

n
L

10) The pure bending in beams means


a) the bending stress is maximum
c) the bending stress is minimum

b) the bending stress is zero


d) the shear stress is zero

11) Section modulus of z1 is equal to


a)

b)
1

1
c)

1
O

d) Ixy


12) Flitched beam is type of


a) composite beam b) T-beam

c) fixed beam

d) cantilever beam

13) The strain energy stored by the body within elastic limit when loaded externally is called
a) Resilience
b) Proof resilience
c) Modulus of resilience
d) All above

14) In case of rectangular section

a)


= 1.5


b)


= 3.2


c)


= 0.5


d)


= 1.33


15) Torsional rigidity of a shaft is equal to


a) product of modulus of rigidity and polar M.I.
b) sum of modulus of rigidity and polar M.I.
c) difference of modulus of rigidity and polar M.I.
d) ration of modulus of rigidity and polar M.I.

16) The shear stress distribution across a circular section is


a) straight
b) parabolic
c) circular

d) cubic

17) The hoop stress in a thin spherical shell, when subjected to an internal pressure is equal to
a)
F

b)
@

"

c)
@

d)
@

&

18) Every cross section of a shaft, which is subjected to a twisting moment, is under
a) compressive stress
b) shear stress
c) tensile stress
d) bending stress

19) A cylindrical vessel is said to be thin if the ratio of its internal diameter to wall thickness is
a) less than 20
b) equal to 20
c) more than 20
d) none

20) The proof resilience is given by

a)


b)

c)

d)


______________

Set A

*SLRBB20*
S

SLR-BB 20

-3-

S.E. (Civil) (Part I) Examination, 2014


STRUCTURAL MECHANICS I
Day and Date : Thursday, 11-12-2014
Time : 10.00 a.m. to 1.00 p.m.

Marks : 80

Instructions : a) In Section I, solve any three questions.


b) In Section II, question 6 is compulsory and solve any two of the remaining.
c) Assume suitable data, if necessary.
SECTION I
2.

a) Draw the SFD and BMD for the beam shown below marking the values at salient points.
Locate the point of contraflexure, if any, and the point of maximum bending moment.

b) Explain Kern or core of a section.


3.

10

a) A short piece of steel pipe is to carry an axial compressive load of 1000 kN with a factor
th of its outside
of safety of 2 against yielding. If the thickness of the pipe is to be
diameter, find the minimum required outside diameter d. Given yield stress of steel
is 250 MPa.


b) Differentiate between :
i) stress and pressure
ii) stress and strain.

9
4

4. A prestressed concrete beam shown below carries a prestressing compressive force


270 kN as shown in fig. Calculate the stress at the top and bottom surfaces of the beam and
plot the variation of this stress.

(Fig. Not to Scale)


All dimensions are in mm

13

Set A

SLR-BB 20

-4-

*SLRBB20*

5. A beam of T section has a length of 2.5 m and is subjected to a point laod W at its free end as
shown below fig. (i) and its cross section is shown in fig (ii)

13

All dimensions are in mm


Calculate the value of W if the maximum bending stress (compression or tension) is limited
to 300 MPa.
SECTION II
6. Attempt the following questions.
a) Define polar modulus. Obtain the value for a solid shaft.

b) Define section modulus and explain its significance.

c) Give expression for strain energy due to axial force.

d) A thin cylindrical shell has an internal diameter 250 mm, and is 5 mm thick. It is subjected
to internal pressure 3
are closed.
m

. Estimate circumferential and longitudinal stresses if ends


N

4
m

7. A cast iron bracket, has a cross-section of I-shape with unequal flanges, as shown in figure.
If the section is subjected to a shear force of 1600 kN, draw the shear stress distribution over
depth of section, indicating the principal values.

Fig. No.

All dimensions are in mm

12

Set A

*SLRBB20*

-5-

SLR-BB 20

8. A timber beam 100 mm wide 150 mm deep is to be strengthened. Two steel plates 100 mm 12 mm
and 100 mm 6 mm are adequately secured to it, the thicker plate to the top surface and the
thinner to the lower surface, as shown in figure if maximum permissible stress in steel is 140
m

and the value of modular ratio m =

= 20, calculate the moment of resistance of the strengthened


I

section. Assume that the timber will not be over-stressed.

12

Fig. No.
All dimensions are in mm
9. A beam of uniform section is simply supported over a span of 5 meters and carries point load
50 kN at 1 m from right support. Determine the strain energy stored in beam.
Take : E = 2 105
N

I = 7.5

12

107 mm4

Fig. No. : Beam


_____________________

Set A

Set A

SLR-BB 200

*SLRBB200*
Seat
No.

Set

B.E. (E & TC) (Part II) Examination, 2014


PATTERN RECOGNITION (Elective II)
Day and Date : Monday, 1-12-2014
Time : 3.00 p.m. to 6.00 p.m.

Total Marks : 100

Instructions : 1) Q. No. 1 is compulsory. It should be solved in first 30 minutes


in Answer Book Page No. 3. Each question carries one mark.
2) Answer MCQ/Objective type questions on Page No. 3 only.
Dont forget to mention, Q.P. Set (A/B/C/D) on Top of Page.
3) Figures to the right indicate full marks.
4) Assume suitable data if necessary.
MCQ/Objective Type Questions
Duration : 30 Minutes

Marks : 20

1. Choose correct option :

20

1) The method in which parameters are random variable having some known
prior distribution is called
A) Maximum likelihood
B) Bayesian
C) Both A) and B)
D) None of these
2) The probability of emitting a particular visible state is denoted as
A) Aij
B) Bjk
C) Either A) or B)
D) None of these
3) A classifier that uses linear decriminant functions is called
A) Linear machine
B) Hidden Markov model
C) Minimum error rate classifier
D) None of these
4) The likelihood is given as
A) P(X|wj)
B) P(wj|X)

C) P(X)

D) None of these

5) To minimize the probability of error, the posterior probability be


A) Minimum
B) Zero
C) Maximum
D) None of the above
6) A minimum distance classifier uses
A) Euclidean distance
B) Mahalanobis distance
C) Absolute distance
D) All of these
7) Determining sequence of hidden states of given HMM is called
A) Evaluation problem
B) Decoding problem
C) Learning problem
D) All of these
8) Procedure that use labelled samples are said to be
A) Unsupervised B) Supervised
C) Both A) and B) D) None of these
P.T.O.

SLR-BB 200

-2-

*SLRBB200*

9) In general, features are _________ to translation, rotation and scale.


A) Invariant
B) Variant
C) Either A) or B) D) None of these
10) Which of the following is an input to the classifier ?
A) Camera image
B) Features
C) Pre-processed image
D) None of these
11) The density estimation in which the volume Vn is grown until it encloses
Kn = sqrt(n) neighbors is called
A) Parzen window method
B) Baysian estimation
C) Kn nearest neighbor
D) All of these
12) The orientation of decision surface represented by linear discriminant function
is given by
A) Weight vector
B) Threshold weight
C) Either A) or B)
D) Both A) and B)
13) Non negativity property of distance metric is given as
A) D(a,b)+D(b, c)>=D(a, c)
B) D(a, b)>=0
C) D(a,b)=D(b,a)
D) None of these
14) The solution vector is __________ of the solution region.
A) Inside
B) Outside
C) Middle
D) None of these
15) Minimum squared error procedure criterion function to solve linear inequality
involves
A) Misclassified samples
B) All samples
C) Few samples
D) All of these
16) If the distance between two samples is significantly large, they correspond to
A) Same cluster
B) Different cluster
C) Either A) or B)
D) None of these
17) Which of the following criterion function measures the clustering quality ?
A) Perception criterion function
B) Sum of squared error criterion function
C) Linear discriminant function
D) None of these
18) Complex decision boundaries lead to
A) Perfect classification
B) Wrong classification
C) Classification is not based on decision boundaries
D) None of the these
19) The value of discriminant function g(x)>0, for 2-category classifier indicates
A) Sample belongs to W1
B) Sample belongs to W2
C) Sample belongs to boundary
D) None of these
20) Parzen window classifier depends upon the choice of
A) No. of sample points
B) Window function
C) Both A) and B)
D) None of these
______________

Set A

*SLRBB200*

-3-

SLR-BB 200

Seat
No.

B.E. (E & TC) (Part II) Examination, 2014


PATTERN RECOGNITION (Elective II)
Day and Date : Monday, 1-12-2014

Marks : 80

Time : 3.00 p.m. to 6.00 p.m.


Instructions : 1) All questions are compulsory.
2) Figures to the right indicate full marks.
3) Assume suitable data if necessary.
SECTION I
2. Attempt any four :

16

a) What is supervised learning ?


b) Explain decision theory for discrete features.
c) Explain Bays decision rule.
d) What is mean and variance of normal density function ?
e) Define moments of random variables.
3. Attempt any two :

16

a) Compare maximum-likelihood and Baysian estimation methods.


b) Explain problems of dimensionality.
c) What is minimum distance classifier ?
4. Explain HMM decoding algorithm.

Set A

SLR-BB 200

-4-

*SLRBB200*

SECTION II
5. Attempt any four :

16

a) Explain parametric procedures for density estimation.


b) Compare unsupervised and supervised learning.
c) What is clustering ?
d) Explain Hierarchical clustering.
e) Write and explain Newton-descent algorithm.
6. Attempt any two :

16

a) Explain Kn-nearest neighbour procedure for parameter estimation.


b) Explain Parzen window procedure of parameter estimation.
c) Explain K-means clsutering.
7. Compare between Parzen window and Kn-nearest neighbour estimation.

_____________________

Set A

SLR-BB 201

*SLRBB201*
Seat
No.

Set

B.E. (E and TC) (Part II) Examination, 2014


FUZZY LOGIC (Elective II)
Day and Date : Monday, 1-12-2014
Time : 3.00 p.m. to 6.00 p.m.

Max. Marks : 100

Instructions : 1) Q. No. 1 is compulsory. It should be solved in first 30


minutes in Answer Book Page No. 3. Each question carries
one mark.
2) Answer MCQ/Objective type questions on Page No. 3 only.
Dont forget to mention, Q.P. Set (A/B/C/D) on Top of Page.
MCQ/Objective Type Questions
Duration : 30 Minutes

Marks : 20

1. Select correct option :


1) Fuzzy sets tend to capture vagueness exclusively via.
a) Membership function
b) Random function
c) Both a) and b)
d) None
2) A|B is defined as
a) Collection of all elements in universe that resides in A, but not in B
b) Collection of all elements in universe that resides in B, but not in A
c) Null set
d) None of above
3) Neither crisp nor fuzzy compositions are __________
a) Associative
b) Commutative c) Distributive
d) Transitive
4) A convex fuzzy set is described by membership value __________
a) Unity
b) Zero
c) Monotonically increasing or decreasing
d) Infinite
5) The process of production of new generation and its evaluation is repeated
until there is __________ within a generation.
a) Divergence
b) Convergence
c) Mutation
d) Cross over
6) [a, b] [c, d] =

a) [a, b] . 1 , 1
c d

c) [a, b] . 1 , d
d c

1
b) [c, d] . ,
a

1
b

d) None
P.T.O.

SLR-BB 201

-2-

*SLRBB201*

7) There is no need of min or max operation in _________ method.


a) Vertex
b) DSW algorithm
c) Restricted DSW Algorithm
d) None
8) Fuzzy C mean is also called as
a) HCM
b) Soft C mean
c) Sophisticated C mean
d) None
9) The contrast intensification operator on fuzzy set generates another fuzzy
set in which fuzziness is reduced by increasing value of membership
that are
a) Greater than 0.5
b) Equal to 0.5
c) Less than 0.5
d) None
10) Syntax analysis is also called as
a) Pattern recognition
b) Parsing
c) Labelling
d) None
11) Rank ordering are used for decisions when issues are
a) Deterministic b) Non specific
c) Specific
d) None
12) The uncertainty in rank arises because of _________
a) Random variable b) Ambiguity c) Impression
d) None
13) The third type of ranking involves the notion of _________
a) Ambiguity
b) Impression
c) Random
d) All of above
14) In relativity function F(x|y) is a measurement of membership value of
choosing _________ over _________
a) x, y
b) y, x
c) Both a) and b)
d) None
15) If the object of the control system is to maintain a physical variable at some
constant value in the presence of disturbance, then system is called a
_________ type of control.
a) Regulatory
b) Disturbance rejection
c) Both a) and b)
d) None
16) Following statement is correct for fuzzy control system.
a) Plant is observable and controllable
b) A solution exists
c) There exist body of knowledge comprised a set of linguistic rules
d) All of above
17) Following charts are used to represent attribute data
a) p
b) np
c) c, u
d) all of above
18) Conventional cognitive maps use _________ relationship.
a) Fuzzy
b) Binary
c) Both a) and b) d) None
19) A cognitive map transformed using matrix is called as _________ matrix.
a) Transpose
b) Adjacency
c) Inverse
d) Identity
20) Medical diagnosis using fuzzy, we require to identify _________
a) Patient
b) Report
c) Symptoms
d) b) and c)
______________
Set A

*SLRBB201*

SLR-BB 201

-3-

Seat
No.

B.E. (E and TC) (Part II) Examination, 2014


FUZZY LOGIC (Elective II)
Day and Date : Monday, 1-12-2014
Time : 3.00 p.m. to 6.00 p.m.

Marks : 80
SECTION I

2. Answer any four :


a) Explain fuzzy arithmetic with example.
b) What is fuzzification and defuzzification ?
c) Write a short note on fuzzy logic.
d) Write properties of fuzzy sets.
e) Differentiate between fuzzy set and crisp sets.

(45=20)

3. a) Fuzzy automata, generate fuzzy input and output sates under following
conditions.
(110=10)
y1
z1 1
z 0
Output relation R = 2
z3 0

z 4 0.5

y2
0
1
0

y3
0
0
1

1 0.3

and state transition relation


X1
z2 z3

z1

S = z1 0 0.4 0.2

0
z2 0.3 1
z 0.5 0
0
3
z 4 0
0
0

Initial fuzzy set C1 = [1

z4
1
0.2
1

0.8

X2

z4

z1 0
0 1 0

z 2 0.2 0 0 1
z3 0
0 0 1

z 4 1 0.3 0 0.6

0.6 0.4], input states are A1 = [0.2 0]


z1

z2

z3

A2 = [1, 0] A3 = [1, 0.4]


Set A

SLR-BB 201

-4-

*SLRBB201*

b) Solve the following fuzzy relation equation for max-min composition

(110=10)

P0 0.5 0 0.3 0
0.4 1 0.3 0
0.5 0.3 0.3 0.1
0 0.1 1 0.1 =
0.5 0.4 0.4 0.2
0.4 0.3 0.3 0.5

OR
b) Explain different defuzzification methods.
SECTION II
4. Write short note on any four :
a) Simple fuzzy control system
b) Measurement FSPC
c) Fuzzy expert system
d) Fuzzy linear programming
e) Multi criterion decision making.
5. a) To compare fidelity of various speakers design X = [X1 X2 X3 X4]

(45=20)

(110=10)

1 0.2 0.6 0.5


0.4 1
1 0.8

f=
0.1 0.3 1 0.9

0.5 1 0.1 1
Compute the comparison matrix using the pairwise comparison in
matrix above.

b) Determine average fuzziness, average certainty in R and distance to


~
type I consensus.
(110=10)
0 0.4 0.7 0.5
0.6 0 0.9 0.8

R
~ = 0.3 0.1 0 0.9

0.5 0.2 0.1 0


OR
b) Explain the following in detail :
a) Non transitive ranking
b) Preference and consensus.
_____________________

Set A

SLR-BB 202

*SLRBB202*
Seat
No.

Set

B.E. (Electronics and Telecommunication) (Part II) Examination, 2014


Elective II : DSP PROCESSORS AND APPLICATION
Day and Date : Monday, 1-12-2014
Time : 3.00 p.m. to 6.00 p.m.

Total Marks : 100

Instructions : 1) All questions are compulsory.


2) Assume suitable data if necessary.
3) Q. No. 1 is compulsory. It should be solved in first 30
minutes in Answer Book Page No. 3. Each question carries
one mark.
4) Answer MCQ/Objective type questions on Page No. 3 only.
Dont forget to mention, Q.P. Set (A/B/C/D) on Top of Page.
MCQ/Objective Type Questions
Duration : 30 Minutes

Marks : 20

1. Multiple choice questions :


1) The no. of registers which can be used for accessing data using indirect
addressing mode is __________
a) 16, 16
b) 16, 8
c) 8, 8
d) 8, 16
2) The register used for which holds the address of the current data memory
page is __________
a) DP
b) ARP
c) ARB
d) None
3) The AR ALU (ARAU) performs _________ arithmetic on _________ numbers.
a) Unsigned, 16
b) Signed, 16
c) Signed, 32
d) Unsigned, 32
4) The 320c54 DSP are said to have advanced Harvard architecture because
a) They have separate memory bus structure for data and program
b) They have instruction the enable data transfer between the program and
data
c) They have same memory bus structure for data and program
d) The contents of program memory cannot copy into data memory or vice
versa
5) The result of operation performed in central ALU are stored into _________
a) ACC
b) ACCB
c) TREG0
d) PREG
6) The __________ permits execution of logical operation on data without
affecting the contents of ACC.
a) Parallel logic unit
b) Auxiliary ALU
c) Central ALU
d) None
P.T.O.

SLR-BB 202

-2-

*SLRBB202*

7) __________ independent buses for dual data fetch, instruction fetch, and
nonintrusive I/O.
a) 4
b) 8
c) 6
d) 16
8) ADSP-21061 is __________
a) Harvard architecture
b) Advanced Harvard architecture
c) Super Harvard architecture
d) Both b) and c)
9) SRAM and integrated input output peripherals in ADSP-21061 are __________
a) Single ported
b) Dual ported
c) Single/dual ported
d) Both a) and c)
10) ADSP-21061 is __________ bit processor.
a) 8
b) 16
c) 32
d) 64
11) The C6X processors have an execution speed of __________ million
instructions per sec.
a) 100
b) 200
c) 1600
d) 6400
12) The no. of 32 bit general purpose registers in C6X is __________
a) 4
b) 8
c) 16
d) 32
13) The multipliers multiply __________ X __________ numbers.
a) 16, 16
b) 17, 17
c) 24, 24
d) 32, 32
14) The C6X pipeline requires __________ phases in decode stage.
a) 2
b) 24
c) 5
d) 6
15) C6X has __________ multipliers.
a) 2
b) 6
c) 4
d) 1
16) The use a fixed point DSP device efficiently, one must consider representing
filter coefficients and signal samples using __________
a) Fixed-point 2s complement representation
b) Fixed-point 1s complement representation
c) Fixed-point 2s and 1s complement representation
d) None
17) The TMS 320C6X generation of DSPs is based on __________ architecture.
a) VelociTI
b) Velocity
c) Velocit
d) None
18) The TMS 320C6X is __________ processor.
a) Floating point
b) Fixed point
c) Floating/fixed point
d) All above
19) In TMS320C6X internal memory is organized as __________
a) Separate program and data
b) Not separate program and data
c) Only program
d) Only data
20) DSP processors can be used to implement __________ application.
a) Image processing
b) Video processing
c) Signal processing
d) All above
______________
Set A

*SLRBB202*

-3-

SLR-BB 202

Seat
No.

B.E. (Electronics and Telecommunication) (Part II) Examination, 2014


Elective II : DSP PROCESSORS AND APPLICATION
Day and Date : Monday, 1-12-2014
Time : 3.00 p.m. to 6.00 p.m.

Marks : 80

Instructions : 1) All questions are compulsory.


2) Assume suitable data if necessary.
SECTION I
2. Attempt any four :
1) Explain about fixed point DSP processors.
2) Mention few applications of TI DSPs.
3) List the functional units in CALU of 5X and explain.
4) Explain the various register used with the ARAU.
5) Enlist the features of ADSP-21061.
3. Attempt any two :
1) Explain bus structure of 5X series DSP in detail.
2) Draw the architecture of ADSP-21061 and explain in detail.
3) List relative demerits and merits of RISC and CISC processors.

(54=20)

(102=20)

SECTION II
4. Attempt any four :
1) Explain addressing modes in 6X DSPs.
2) Explain FIR implementation using TMS 320C5X DSP processor.
3) Draw cache configuration register (CCFG).
4) Explain how the C6X architecture differs from C54X.
5) What is the function of EMIF in C6X ?

(54=20)

5. Attempt any two :


(102=20)
1) Draw and explain TMS320C6X CPU data paths.
2) Discuss the implementation of FFT algorithms using TMS 320C54X DSP
processor.
3) Explain internal architecture of TMS320C6x processor.
_____________________

Set A

Set A

SLR-BB 203

*SLR-BB-203*
Seat
No.

Set

S.E. (CSE) (Part I) Examination, 2014


APPLIED MATHEMATICS I
Day and Date : Tuesday, 9-12-2014
Time :10.00 a.m. to 1.00 p.m.

Max. Marks : 100

Instructions : 1) Q. No. 1 is compulsory. It should be solved in first 30 minutes in


Answer book Page No. 3. Each question carries one mark.
2) Attempt any three questions from each Section.
3) Figure to the right indicate full marks.
4) Use of non-programmable calculator is allowed.
5) Answer MCQ/Objective type questions on Page No. 3 only. Dont
forget to mention, Q.P. Set (A/B/C/D) on Top of Page.
MCQ/Objective Type Questions
Duration : 30 Minutes

Marks : 20

1. Choose the correct answer :


1)

1
x is equal to
D3
x 1
+
a)
3 9

(201=20)

b)

x 1

3 9

c)

x 1

3 9

d) None of these

2) The P.I of (D + 1)3 y = ex is

x 3 x
e
a)
3

ex
b)
8

x 3 x
e
c)
8

x 3 x
e
d)
6

3) The general solution of the diff. equation (D4 16)y = 0 is


a) C1 e4x + C2 e4x
b) C1 cos 4x + C2 sin 4x
c) C1 e2x + C2 e2x + C3 cos 2x + C4 sin 2x d) (C1 + C2x) e2x + (C3 + C4x)e2x

4) L cosh t dt = __________
0

s
1
b) 2
a) 2
s +1
s +1

d)

s
s 1

c) (s / a)

d)

1
(s / a)
a

c) t e3t

d) t2e3t

c)

1
s 1

5) If L {f(t)} = (s) then L{f(at)} = __________


a) (s/a)

b)

1 s

s a

1
= __________
2
(
s
+
3
)

e 3 t
a) t e3t
b)
t

6) L1

P.T.O.

SLR-BB 203

*SLR-BB-203*

-2-

7) If f(x) = x2 is expanded as cosine series in (0, ) then constant term is


a)

2
3

b)

3
3

c)

22
3

d)

32
2

8) The Fourier expansion in the interval [2,2] of the function


f(x) = x, 2 x 0
= x, 0 x 2 has
a) No cosine term
b) No sine term c) Both cosine and sine term

d) None of these

9) If Z{f(k)} = F(z) then Z {K f(k)} = _________


a)

d
F(z)
dz

b)

d
(ZF(z))
dz

c) z

1
= , k 0
K!

d
F(z )
dz

d)

1 d
F(z )
z dz

d)

1
1 z

10) z

a) ez

b) e

c)

1
z

11) The directional derivative of = x2 y2 + 2z2 at the point (1,2, 3) along z-axis is _________
a) 2
b) 4
c) 12
d) 0
12) If (x, y, z) = c represents a surface, then the unit normal vector to this surface is
a)

grad.
| grad. |

b) grad

c) div (grad. )

d) curl (grad )

13) If r = xi + yj + 3k and a is a constant vector then ( a . r ) = ______________


a) 2 a
b) a
c) r
d) 2 r
14) The lines of regression are given by 10y = x + 17, and x = 5y 7. Then x and y are
a) 1 and 3
b) 1 and 5
c) 2 and 3
d) 3 and 2
15) A continuous random variable has the following probability density function f(x)= kx(1x),
0 x 1 then k = ____________
a) 2
b) 3
c) 5
d) 6
16) If 10% pens are defective and if there are 10 pens in the box then the probability that there
is no defective pen in box is ____________
a) 0.25
b) 0.35
c) 0.45
d) 0
17) The mean and the standard deviation of a standard normal variate is __________
a) 1 and 0
b) 0 and 1
c) 1 and 1
d) None of these
18) In a M/M/1 queueing system, expected waiting time of unit that actually waits is given by
a)

( )

b)

c)

d)

( )
2

19) If average arrival rate in a queue is 6 per hour and the average service rate is 10 per hour.
Which one of the following is the average number of customers in the line including the
customer being served ?
a) 0.3
b) 0.6
c) 1.2
d) 1.5
20) If byx = 6/5 and bxy = 15/8 then r = ___________
a) 2.5
b) 1.5
c) 0.5
______________

d) 1

Set A

*SLR-BB-203*

-3-

SLR-BB 203

Seat
No.

S.E. (CSE) (Part I) Examination, 2014


APPLIED MATHEMATICS I
Day and Date : Tuesday, 9-12-2014
Time :10.00 a.m. to 1.00 p.m.

Marks : 80

Instructions : 1) Attempt any three questions from each Section.


2) Figure to the right indicate full marks.
3) Use of non-programmable calculator is allowed.
SECTION I
2. a) Solve the following diff. equations any two :
i)

(D2

+ 2D + 5)y =

sin2x

ii) (D3 3D2 + 3D 1)y = x ex + ex


iii) (D2 + 2D + 1)y = 4ex log x
b) Solve (D3 7D 6) y = (1 + x2) e2x.

sin2 2t
.
t

3. a) Find L

s + a .

t sin 3t dt by Laplace transform.

b) Find L1 tan

c) Evaluate

2 t 2

s+4
.
2
s(s 1)(s + 4)

d) Find L 1

4. a) Find the Fourier series of


f(x) = x,
1< x < 0
= x + 2, 0 < x < 1.

z2

1
1

<|z|< .
1
1
b) Find Z1
z z if 3
2
2
3

5. a) Find the half-range cosine series of f(x) = x ( x) in the interval (0, ).

1
4
c) Find z{cosh k}, k 0.

6
5

|k|

b) Find z{f(k)} if f(k) = for all k.

4
4
Set A

SLR-BB 203

-4-

*SLR-BB-203*

SECTION II
6. Attempt the following :
a) Prove that, (r nr ) = (n + 3)r n .

b) Find the directional derivative of = e2x cosyz at (0, 0,0) in the direction of the tangent to
the curve x = asint y = a cost, z = at t = .

c) Find the unit tangent vector at any point on the curve x = t2 + 2, y = 4t 5, z = 2t2 6t where
t is any variable. Also determine the unit tangent vector at the point t = 2.
7. Attempt the following :
a) A manufacturer finds that the average demand per day for the mechanic to repair his new
production is 1.5. over a period of one year the demand per day is distributed as Poisson
distribution. He employs two mechanics on how many days in one year i) both mechanics
would be free. ii) some demand is refused.
b) The size of hats is normally distributed with mean 18.5 cm and standard deviation of 2.5 cms.
How many hats in total of 2000 will have sizes between ?
i) 18 cms and 20 cms
ii) above 20 cms (Given : For S.N.V.z area between z = 0 to
z = 0.6 is 0.2257 and between z = 0 to z = 0.2 is 0.0793)
c) Fit a second degree parabolic curve to the following data estimate production in 1982.
Year
: 1974 75
76
77
78
79
80
81
Production : 12 14
26
42
40
50
52
53
8. Attempt the following :
a) The equations of two lines of regression are 6y = 5x + 90, and 15x = 8y + 130. Find the
mean of x and mean of y and the coefficient of correlation. If variance of x is 16, find the
standard deviation of y.
OR
a) The following table gives the number of motor-vehicles with licenses and the number of
motor-vehicles accidents in a city. Calculate the coefficient of correlation between the
number of motor-vehicles and the number of motor-vehicles accidents :
No. of vehicles in thousands : 2.6
2.8
2.9 3.1
3.2
2.3 2.5 1.8
No. of accidents in hundreds : 5.9
6.0
6.2 6.2
7.6
7.0 7.5 5.5
b) Fit a binomial distribution to the following data :
x : 0
1
2
3
4
5
6
f :
6
20
28
12
8
6
0
9. Attempt the following :
a) Customers arrive at a box office window, being manned by a single individuals according
to Poisson input process with mean rate of 20 per hour. Time required to serve a customer
has an exponential distribution with mean of 90 seconds. Find average waiting time of
customer. Also determine the average no. of customers in the system and average
queue length.
b) A warehouse has only one loading dock manned by a three person crew. Trucks arrive at
the loading dock at an average rate of 4 trucks per hour and the arrival rate is Poisson
distributed. The loading of a truck take 10 min. on an average and can be assumed to be
exponentially distributed. The operating cost of truck is Rs. 20 per hour and the members
of the loading crew are paid @ Rs. 6 per hour. Would you advise the truck owner to odd
another crew of three persons ?
_____________________

4
6

Set A

SLR-BB 204

*SLRBB204*
Seat
No.

Set

S.E. (CSE) (Part I) Examination, 2014


DISCRETE MATHEMATICAL STRUCTURES
Day and Date : Thursday, 11-12-2014
Time : 10.00 a.m. to 1.00 p.m.
Instructions :

Max. Marks : 100

1) Figures to the right indicate full marks.


2) Q. No. 1 is compulsory. It should be solved in first 30 minutes in
Answer Book Page No. 3. Each question carries one mark.
3) Answer MCQ/Objective type questions on Page No. 3 only. Dont
forget to mention, Q.P. Set (A/B/C/D) on Top of Page.
MCQ/Objective Type Questions

Duration : 30 Minutes

Marks : 20

1. Choose the correct options :

20

1) ~ (P Q) is equivalent to _______________
a) P ~ Q
b) ~ P Q
c) ~ P Q

d) P Q

2) The number of proper subsets of a set of order three is ___________


a) 3
b) 6
c) 8
d) 9
3) The relation which is reflexive, antisymmetric and transitive is called ____________
a) Equivalence relation
b) Partial order relation
c) Congruence relation
d) Function
4) The Boolean expression a * (a b) is equal to _________
b) a * b
c) a b
d) a + a * b
5) The function is said to be bijective (invertable) if and only if it is _____________
a) one-to-one function
b) onto function
c) Both one-to-one onto function
d) None of these
a) a

6) The dual of the statement formula (P Q) (~ P Q) T is ______________


a) (P Q) (~ P Q) F

b) (P Q) (~ P Q) T

c) (P Q) (~ P Q) T

d) (P Q) (~ P Q) F

7) Which of the following are not fucntionally complete set of connective ?


a) {, ~}

b) {}

c) {, ~}

d) { ,}

8) Given set S = {1, 2, .... 10} and a relation R on S where R = {<x, y> | x + y = 10}. Which of
the following is property of R ?
a) Symmetric
b) Reflexive
c) Transitive
d) Irreflexive

P.T.O.

SLR-BB 204

*SLRBB204*

-2-

9) If f : X Y is invertable function and I x : X X us ab identify function, which of the


following statement is wrong ?
b) f1of = Ix
a) f o f1 = Iy

c) f o Ix = f

d) f o Iy = f

10) Which of the following is true ? (Where A and B are the sets ?)
a) A + B = (A ~ B) (B ~ A )
b) A + B = (A B) (B A)
c) Both a) and b)
d) None of these
11) Boolean algebra is a _____________
a) Compound lattice
c) Complemented distributive lattice

b) Distributive lattice
d) Bounded lattice

12) The algebric system A,* which is closed, associative and having identity element under
* operation is called _____________
a) Semigroup
b) Monoid
c) Group
d) Subgroup
13) Which of the following is not partial order relation ?
a)
b)
c) =

d)

14) Which of the following suffix notation is valid ?


b) ab + d/
c) ab + cd *
a) a b * +

d) abc +

15) Let f : R R is defined by f(x) = x2 + 1 what is f1 (5) ?


a) {2, 2}

b) {3, 3}

16) An abelian group is ____________


a) Associative
b) Commutative

c) 27

d) None of these

c) Distributive

d) Both a) and b)

17) Which of the following can be tautologically implied by the statement formula
(P Q) (P R) (Q S) ?
a) S R

b) S R

c) Q S

d) P Q

18) Which of the following lattice in figures is not a distributive lattice ?

a)

b)

c)

19) The rank of well-formed formula is ____________


c) 1
a) 0
b) 1

d) All of these

d) None of these

20) Which of the following statement is false ?


a) Every chain is a distributive lattice
b) A semigroup with identity and ivnerset element is called group
c) Every lattice has a unique GLB and a unique upper bound
d) The reflexive, symmetric and transitive relation is an equivalence relation
______________

Set A

*SLRBB204*

-3-

SLR-BB 204

Seat
No.

S.E. (CSE) (Part I) Examination, 2014


DISCRETE MATHEMATICAL STRUCTURES
Day and Date : Thursday, 11-12-2014
Time : 10.00 a.m. to 1.00 p.m.
Instructions :

Marks : 80

i) Figures to the right indicate full marks.


ii) Assume suitable data if necessary.
SECTION I

2. Solve any four :


A) Show the following implications :

(54=20)

i) (P Q) (P Q)
ii) P (Q P) .
B) For any three sets A, B and C, (A B) C = (A C) (B C) where is a relative
complement operation.
C) Show that R (P Q) is a valid conclusion from the premises
P Q, Q R, Q R, R M and ~ M.
D) What is duality law ? Write the duals of :
i) (P Q) RV ( T P)
ii) (P Q) (~ P (~ P Q)) .
E) What is Cartesian product ? If A = {1}, B= {a, b} and C = {2, 3}, find A B C , B2 and B2 A.
F) Explain with example matrix and graph representation of relation.

3. Solve any one :

10

A) Explain the following terms :


i) Functionally complete set of connectives
ii) Tautology
iii) POSET
iv) Power set.
B) Let A be the set of factors of a particular positive integer m and let be the relation devides
i.e.
=
i)
ii)
iii)

{ < x, y >| x A Y A ( x divides y )} . Draw the Hasse diagram for


m = 12
m = 45
m = 30.
Set A

SLR-BB 204

-4-

*SLRBB204*

4. Obtain the PCNF and PDNF of the following statement formula.

10

i) (P Q) (~ P R) (Q R)
ii) (~ P R) (Q
P)
SECTION II
5. Solve any four :

(54=20)

A) Define inverse function and explain it with suitable example.


B) Let S be a nonempty set and P(S) be its power set. Show that the algebra P(s), is
monoid.
C) Explain with example group codes.
D) Obtain the sum-of-product canonical form of the following boolean expression :
i) x1 x 2
ii) x1 ( x 2 * x 31) .
E) Define semigroup and explain the homomorphism of semigroup.
F) Define Algebric system and state its properties.
6. Solve any one :

10

A) Describe Polish expressions and their compilation with the help of algorithm and
example.
B) Let Sn be the set of factors of a particular positive integer n and D be the division
operation on Sn. Draw the hasse diagram for Sn , D for n = 30 and for n = 45. Which of

these two lattices Sn , D for n = 30 and n = 45 are complemented ? Are these lattices
distributive ?
7. How many functions are there from the set X to the set Y given bellow ? Find also the number of
functions which are one-to-one, onto and on-to-one onto.
X = {1, 2, 3, 4}, Y = {a, b, c}.
10

_____________________

Set A

SLR-BB 205

*SLRBB205*
S

S.E. (C.S.E.) (Part I) Examination, 2014


ADVANCED C CONCEPTS
Day and Date : Saturday, 13-12-2014
Time : 10.00 a.m. to 1.00 p.m.

Max. Marks : 100

Instructions : 1)
2)
3)
4)
5)

All questions are compulsory.


Figures to the right indicate full marks.
Assume suitable data if necessary.
Draw neat diagrams.
Q. No. 1 is compulsory. It should be solved in first 30 minutes in
Answer Book Page No. 3. Each question carries one mark.
6) Answer MCQ/Objective type questions on Page No. 3 only. Dont
forget to mention, Q.P. Set (A/B/C/D) on Top of Page.

Duration : 30 Minutes

MCQ/Objective Type Questions

Marks : 20

1. Choose the correct answer :


1) Which of the following are incorrect declarations ?
a) extern int i;
b) auto int i;
c) static int i;

20
d) global int i;

2) Which is the correct statement of pointer definition and initialization ?


a) int a = 10; int *ptr = a;
b) int a = 10, *ptr; ptr = *a;
c) int a = 10, *ptr; *ptr = & a;
d) int a = 10, *ptr; ptr = &a;
3) Which of the following function adds one string to end of another ?
a) append ( ) ;
b) stringadd ();
c) strcat ( );
d) stradd ();
4) Which of the following statement is wrong with respect to a storage class ?
a) it specifies default initial value
b) it specifies life of a variables
c) by default a storage class is static
d) it specifies where variables is stored
5) What is the output of following code main ( ) ?
{
char name 1 [4] = {A, M, I, T};
char name 2 [5] = {A, M, I, T};
printf (%S, name 1);
printf (%S, name 2);
}
a) AMIT
b) AMIT AMIT c) AMIT Garbage value d) Garbage value AMIT
6) Which of the following library function gives current system time as integer ?
a) ftime
b) clock
c) time
d) current time
7) The term recursion refers to programming technique involving
a) FOR loops
b) WHILE loops
c) A method that calls itself
d) Nested IF statement
8) How would you implement an array of 3 function pointers where each function receives
two integers and return a float ?
a) float a[3] (int)
b) float (* a[3]) (int, int)
c) float (* a[3]) ( )
d) float (int, int) (*a[3])
P.T.O.

SLR-BB 205
9) What will be output of following code ?
void main ()
{
char c [ ] = gate 2014;
char * p = c;
printf (% S, p + p [3] p [1]);
}
a) gate
b) gate 20

*SLRBB205*

-2-

c) 2014

d) te 2014

10) Which of the following storage class have scope as global ?


a) static
b) external
c) automatic

d) register

11) The FILE structure is defined in which of the following ?


a) stdlib.h
b) stdio.c
c) io.h

d) stdio.h

12) A file created in text mode __________ for subsequent operations.


a) must always be opened in text mode
b) can be opened in any mode
c) must always be opened in binary mode d) none of the above
13) Contents of existing file are overwritten if the file is opened in
a) w + mode
b) a mode
c) a + mode

d) r mode

14) The best case running time for quick sort is


b) 0 (log2n)
c) 0 ((nlog2n)
a) 0 (n2)

d) 0 (log n)

15) The worst case in linear search algorithm occurs when


a) Item is in middle of the array
b) Item is not in array
c) Item is last element in array
d) Both b) and c)
16) If you have to sort a list consisting of a sorted list followed by a few random elements,
which of the following sorting method would be suitable especially for such a task
a) bubble sort
b) selection sort
c) quick sort
d) insertion sort
17) The complexity of the average case of an algorithm is
a) Much more complicated to analyze than worst case
b) Much more simpler to analyze than worst case
c) Some times complicated and some times simpler than that of worst case
d) None of above
18) Arrays are best data structures
a) for relatively permanent collection of data
b) The data and size of the structure are constantly changing
c) for both of above situation
d) for none of above situation
19) In a hash table if 89, 18, 49, 58, 69 keys are inserted using quadratic probing and
hash (K) = K mod 10 then the table index for key 58
a) T [0]
b) T [2]
c) T [8]
d) T [1]
20) Secondary clustering problem occurs in
a) Linear probing
b) Quadratic probing c) double hashing d) all the above
______________
Set A

*SLRBB205*
S

-3-

SLR-BB 205

S.E. (C.S.E.) (Part I) Examination, 2014


ADVANCED C CONCEPTS
Day and Date : Saturday, 13-12-2014
Time : 10.00 a.m. to 1.00 p.m.
Instructions : 1)
2)
3)
4)

Marks : 80

All questions are compulsory.


Figures to the right indicate full marks.
Assume suitable data if necessary.
Draw neat diagrams.
SECTION I

2. Attempt any four :

(54=20)

a) Define pointer. How pointer is declared and initialized ? Give advantage and disadvantage
of pointers.
b) List different character classification functions with their usage.
c) Write a short note on Dynamic Memory Allocation.
d) Write a C code to concatenate two strings without using string library functions.
e) Trace the output of following code :
(write in detail what each statement mean)
main ( )
{
int num [ ] = {10, 20, 30, 40, 50, 60};
show (& num [0], 6);
}
show (int *j, int n)
{
int i;
for (i = 0; i < n 1 ; i + +)
{
printf ( In Data = %d", *j);
j + +;
}
}
Set A

SLR-BB 205

-4-

*SLRBB205*

3. Attempt any two :

(210=20)

a) What is use of storage classes ? Explain all 4 types of storage class with help of following
points :
i) storage
ii) default initial value
iii) scope
iv) life.
b) How strings are declared and initialized ? Explain at least 4 string library functions with eg.
c) Define recursion. Give advantages and disadvantage of recursion. Explain with one
example and schematic stack representation, how recursion works.
SECTION II
4. Attempt any four :

(54=20)

a) Explain different file opening modes with example.


b) Use merge sort technique to sort following data in descending order
60, 10, 20, 15, 17, 21, 12, 79, 3, 5, 13.
c) Compare binary search and sequential search.
d) Explain concept of Big O and Omega notations.
e) Write C code to implement recursive binary search.
5. Attempt any two :

(102=20)

a) Sort the following numbers in ascending order using heap sort.


46, 25, 35, 49, 10, 92, 83, 32, 3.
b) What is hashing ? Explain need of hashing. Give techniques for resolving hash table
collision with suitable example.
c) Write C program to insert, delete and modify records in file. Give sample output.
_____________________

Set A

SLR-BB 206

*SLRBB206*
S

S.E. (Part I) (Computer Science and Engg.) Examination, 2014


DIGITAL TECHNIQUES
Day and Date : Tuesday, 16-12-2014
Time : 10.00 a.m. to 1.00 p.m.

Max. Marks : 100

Instructions : 1)
2)
3)
4)

All questions are compulsory.


Figures to right indicate full marks.
Assume suitable data whenever necessary.
Q. No. 1 is compulsory. It should be solved in first 30
minutes in Answer Book Page No. 3. Each question carries
one mark.
5) Answer MCQ/Objective type questions on Page No. 3
only. Dont forget to mention, Q.P. Set (A/B/C/D) on Top
of Page.

Duration : 30 Minutes

MCQ/Objective Type Questions

Marks : 20

1. Choose the correct answer :

20

1) Each term in the standard POS form is called as ____________


a) Minterm
b) Maxterm
c) Dont care
d) Lilteral
2) Which of the following input combinations is not allowed in an S-R flip-flop ?
a) S = 0, R = 0
b) S = 0, R = 1 c) S = 1, R = 0
d) S = 1, R = 1
3) Master slave configuration is used in flip-flops to ____________
a) increase the clocking rate
b) reduce power dissipation
c) eliminate race-around condition
d) improve its reliability
4) A flip-flop can store ____________
a) one bit of data
c) three bits of data

b) two bits of data


d) any number of bits of data

5) The Fan-out for TTL family is


a) 5
b) 10
c) 15
d) 20
6) In a counter circuit consisting of four J-K flip-flops, all the flip-flops gets
triggered simultaneously. This counter circuit
a) is a combinational circuit
b) is an asynchronous circuit
c) is a synchronous circuit
d) may be a combinational or a sequential circuit

P.T.O.

SLR-BB 206

*SLRBB206*

-2-

7) To perform multibit addition __________ circuit is used.


a) half adder
b) full adder
c) full subtractor
d) none of these
8) POS expression of F(A, B, C) = m (1, 3, 7) is
a) m (0, 2, 4, 5, 6)
b) M (1, 3, 7)
d) None of these
c) M (0, 2, 4, 5, 6)
9) A MOD-14 counter must have _________
a) 14 flip-flops
b) 3 flip-flops
c) 4 flip-flops
d) synchronous clocking
10) In which flip-flop race around condition occurs ___________
a) J-K flip-flop
b) S-R flip-flop c) T flip-flop
d) Flip-flop
11) For register level the information units are
a) Bits
b) Words
c) Vectors
d) Both b) and c)
12) A PLA can be used as ___________
a) A microprocessor
b) A dynamic memory
c) A sequential logic realization
d) A combinational logic realization
13) IC 74152 is a multiplexer having
a) 1 : 8 lines
b) 1 : 32 lines
c) 8 : 1 lines
d) 16 : 1 lines
14) For conserving power in multiplexed display in IC-7447 following pin is used
a) RBO
b) RBI
c) BI
d) LT
15) An EPROM is erased by _____________
a) Electrically
b) Ultraviolet radiation
c) Intent based radiation
d) None
16) A PAL is comprised of programmable __________ gates.
a) NAND
b) AND
c) NOR
d) NOT
17) A dynamic RAM is fabricated using ____________ technology.
a) MOS
b) NFET
c) FET
d) None
18) VHSIC stands for _________
a) Very Large Scale Integrated Circuit
b) Very High Speed Integrated Circuit
c) Very High Scale Integrated Circuit
d) Very Heavy Speed Integrated Circuit
19) ____________ specifies the behavior of a circuit to be designed.
a) System
b) Entity
c) Architecture body
d) Registers
20) An operator used for signal assignment is ________
a) >
b) > =
c) < =
d) <
______________
Set A


*SLRBB206*
S

SLR-BB 206

-3-

S.E. (Part I) (Computer Science and Engg.) Examination, 2014


DIGITAL TECHNIQUES
Day and Date : Tuesday, 16-12-2014
Time : 10.00 a.m. to 1.00 p.m.

Marks : 80

Instructions : 1) All questions are compulsory.


2) Figures to right indicate full marks.
3) Assume suitable data whenever necessary.
SECTION A
2. Solve any four :

(45=20)

a) Minimize the four variable logic function using K-map.


f(A, B, C, D) = AB
+

D+
)

BCD +
)

+
)

+A
+

+A C+
*

b) Draw and explain full adder using half adders.


c) Explain carry look ahead adder in detail.
d) Write a short note on comparator with diagram and truth table.
e) State the different types of shift register and draw output waveforms for
serial in serial out shift register with input sequence 10110.
3. Solve any two :

(210=20)

a) What is SOP equation ? Minimize the following logic function and realize
using universal gates.
f(A, B, C, D) =


m(1, 3, 5, 8, 9, 11, 15) + d(2, 13)

b) What is counter ? Design 3 bit synchronous counter using J-K flip-flop.


c) What is flip-flop ? Draw and explain how to construct JK flip-flop, D and T
flip-flop from SR flip-flop.

Set A

SLR-BB 206

-4-

*SLRBB206*

SECTION B
4. Answer any four :

(45=20)

a) Implement 16 : 1 MUX using 4 : 1 MUX.


b) Implement the following combinational logic circuit using a 4 to 16 line decoder.
F1 =
F2 =
F3 =
F4 =

m (4, 7, 8, 10, 14)




m (1, 5, 8, 13)
m (2, 3, 4, 7, 8, 11)
m (1, 8, 9, 10)

c) Explain timing characteristics of READ cycle.


d) Write short note on 74154.
e) Explain how MUX is used as a logic function with an example.
f) Write a VHDL code for half-adder.
5. Attempt any two :

(210=20)

a) Design and explain driver for 7-segment display.


b) Write a VHDL code for 4 : 1 multiplexer and 8 : 1 demux with output waveform.
c) Design a 1024 8 memory using 128 8 memory chip.
_____________________

Set A

SLR-BB 207

*SLRBB207*
Seat
No.

Set

S.E. (CSE) (Part I) Examination, 2014


COMPUTER GRAPHICS
Day and Date : Thursday, 18-12-2014
Time : 10.00 a.m. to 1.00 p.m.

Max. Marks : 100

N.B. : 1) Q. No. 1 is compulsory. It should be solved in first 30 minutes


in Answer Book Page No. 3. Each question carries one mark.
2) Answer MCQ/Objective type questions on Page No. 3 only.
Dont forget to mention, Q.P. Set (A/B/C/D) on Top of Page.
MCQ/Objective Type Questions
Duration : 30 Minutes

Marks : 20

1. Choose the correct answer :


1) The number of bytes per pixel is called as
a) Image depth
b) Bit depth
c) Byte depth
d) None of the above

20

2) RGB is an ______________
a) Additive model
b) Subtractive model
c) Multiplicative model
d) None of the above
3) Bresenhams circle generation algorithm, if (x, y) is the current pixel position
then the x-value of the next pixel position is
a) x
b) x 1
c) x + 1
d) x + 2
4) Run length encoding is used for
a) Image smoothening
c) Image coloring

b) Image compression
d) Image dithering

5) The property that adjacent pixel have same characteristics is called ________
a) Spatial coherence
b) Area coherence
c) Scan-line coherence
d) Pixel coherence
6) The equivalent representation of a two dimensional point (x, y) in homogeneous
co-ordinate system is
a) (x, y, 1)
b) (x, y, 0)
c) (x/w, y/w)
d) (x + y, x y)
7) During scaling, if scale factors are Sx = Sy = Sz = S > 1 then scaling is
called as
a) Magnification
b) Reduction
c) Rotation
d) None of the above
P.T.O.

SLR-BB 207

-2-

*SLRBB207*

8) In pure 3D reflection, the determinant of reflection matrices about any plane is


a) 1
b) 0
c) 1
d) 0.5
9) The technique of transforming one object into another is known as
a) Translation
b) Morphing
c) Scaling
d) None of above
10) An image is stored in a
a) CPU
b) Display file
c) Monitors
d) None of the above
11) _________ operation removes the segment from refresh cycle.
a) Posting
b) Appending
c) Unposting
d) None of these
12) __________ algorithm is used for clipping.
a) Sutherland Cohen
b) Fence
c) Warnock
d) Bresenhams
13) Unit of display file is __________
a) View port
b) Segment
c) Byte
d) Window
14) In quad tree data structure root of the tree is the display ________
a) View port
b) Unit
c) Segment
d) Window
15) _________ and depth buffers are used in Z-buffer algorithm.
a) Data buffer
b) Display buffer
c) Frame buffer
d) None of these
16) Curve is ___________ if it lies in its entirely in a single plane.
a) 3-D
b) 1-D
c) Single
d) 2-D
17) In Bezier curve ___________ functions are real.
a) Basis
b) Recursive
c) Global
d) None of these
18) A _____________ curve is determined by defining polygon.
a) Parametric
b) B-spline
c) Bezier
d) None of these
19) ___________ is effectively an infrastructure technology, characterized by
the ability to handle very diverse data types.
a) Software
b) Multimedia
c) GUI
d) None of these
20) __________ images which include photographs, halftones and single frames
from a video may be gray scale or color.
a) Continuous tone
b) Fattered tone
c) Non-continuous tone
d) All
______________

Set A

*SLRBB207*

-3-

SLR-BB 207

Seat
No.

S.E. (CSE) (Part I) Examination, 2014


COMPUTER GRAPHICS
Day and Date : Thursday, 18-12-2014

Marks : 80

Time : 10.00 a.m. to 1.00 p.m.


N.B. : 1) Each Section carries 40 marks.
2) Attempt all questions in each Section.
SECTION I
2. Solve any four (Each carries 5 marks) :

20

A) Explain the working of Refresh CRT with diagram.


B) Consider a line from (0, 0) to (6, 6) using simple DDA algorithm, rasterize the
line.
C) Explain edge fill algorithm in detail with an example.
D) Write a short note on homogeneous co-ordinate system.
E) Describe 3D Multiple Transformation.
F) Describe scan converting polygon.
3. Which are the different types of color models ? Explain any two of them in detail. 10
OR
Describe Bresenhams circle generation algorithm in detail along with flowchart.
4. Write the matrices and draw the diagrams for scaling, rotation of an 3D object.

10

Set A

SLR-BB 207

-4-

*SLRBB207*

SECTION II
5. Attempt any four (Each carries 5 marks) :

(54=20)

1) Explain display file compilation in detail.


2) Explain z-buffer algorithm along with its advantages and disadvantages.
3) Explain non-parametric curve representation.
4) What is an antialiasing ? Explain in detail.
5) What is multimedia ? Also describe all the elements of a multimedia system.
6) Write note on Image Capture.
6. Explain working of Cohen-Sutherland line clipping algorithm.

10

7. Explain Bezier curve along with its all properties.

10

_____________________

Set A

SLR-BB 208

*SLRBB208*
Seat
No.

Set

S.E. (CSE) (Part II) Examination, 2014


APPLIED MATHEMATICS II (Old)
Day and Date : Tuesday, 25-11-2014
Time : 3.00 p.m. to 6.00 p.m.

Max. Marks : 100

Instructions : 1) Q. No. 1 is compulsory. It should be solved in first 30 minutes in


Answer book Page No. 3. Each question carries one mark.
2) Answer MCQ/Objective type questions on Page No. 3 only. Dont
forget to mention, Q.P. Set (A/B/C/D) on Top of Page.
MCQ/Objective Type Questions
Duration : 30 Minutes

Marks : 20

1. Tick the correct alternatives :

(201=20)

1) Which of the following methods is used to verify the optimality of the current solution of
transportation problem ?
a) Least cost entry method
b) Hungerian method
c) Modified distribution method
d) All of the above
2) In assignment problem, multiple zeros in columns and rows are all indicates
a) multiple optimal solution
b) improvement in present solution
c) non-occurrence of optimal solution
d) degeneracy in optimal solution
3) If there are n workers and n-jobs then there would be
a) (n 1) ! solutions b) n solutions
c) n ! solutions

d) n 1 solutions

4) In optimal simplex table Cj Zj = 0 value indicate


a) Unbounded solution
b) Alternate solution
c) Cycling
d) Infeasible solution
5) In queueing theory the traffic intensity is given by
a) 1

b) 2

c)

1
d) 1

6) Which of the following is false ?


a) Unbalanced transportation problem can be balanced
b) Every optimal solution of transportation problem is feasible
c) In North-West corner method the transportation cost is taken into account
d) All of the above
7) Which of the following characteristics apply to queueing system ?
a) Customer population
b) Arrival process
c) Both a) and b)
d) None of these
8) The slack variables are called as
a) basic variables
c) non-basic variables

b) special variables
d) None of these
P.T.O.

SLR-BB 208

*SLRBB208*

-2-

9) If Poisson distribution is such that P(x = 2) = P(x = 3) then mean of distribution is m =


a) 2
b) 3
c) 4
d) 5
10) For the binomial distribution which of following is true ?
a) mean variance
b) variance mean
d) All of the above
c) p 1 n
11) Which of the following law is violated for Fuzzy sets ?
a) Law of absorption
b) Law of identity
c) Law of contradiction
d) None of these
12) The set of all levels [0 , 1] that represents distinct -cuts of a given fuzzy set A is
called
a) Support of A
b) Level set of A
c) Strong -cut of A
d) None of these
13) State which of the following is true ?
a) Support of fuzzy set contains an element whose membership grade is zero
b) If height of fuzzy set is 1, then it is called subnormal
c) Support of A is equal to strong -cut of A for = 0
d) 0-cut of A is called core of A
14) If A = [4, 10] and B = [1, 2] then A/B = __________
a) [6, 8]
b) [5, 12]
c) [2, 10]

d) None of these

15) If A = [2 1, 3 2] and B = [2 + 1, 5 2] then for (0, 1] , ( A + B ) = ___________


a) [4, 8 4]

b) [4 6, 2 4 ]

16) -cut of a fuzzy set is


a) Fuzzy set
b) Crisp set

c) [4, 4]

d) None of these

c) Void set

d) None of these

17) If A = 0a.2 + 0b.7 + 0c.3 + 0d.0 is a fuzzy set, then scalar cardinality of A is
a) 0.12
b) 0
c) 1
d) 1.2
18) One cut of A i.e. 1A is called
a) Support of A
c) Both a) and b)

b) Core of A
d) None of these

19) S(A, B) = ______________


a)

|A B|
|A|

|A|
b) | A B |

c)

|A B|
|B |

|B|
d) | A B |

20) Which of the following is true ?


a) Fuzzy complement is cutworthy.
b) A B iff B A .
c) B A iff + A +B .
d) Fuzzy complement is not strong cutworthy.
______________

Set A

*SLRBB208*

SLR-BB 208

-3-

Seat
No.

S.E. (CSE) (Part II) Examination, 2014


APPLIED MATHEMATICS II (Old)
Day and Date : Tuesday, 25-11-2014
Time : 3.00 p.m. to 6.00 p.m.
Instructions : i)
ii)
iii)
iv)

Marks : 80

Attempt any three questions from each Section.


Figures to the right indicate full marks.
Use of programmable calculator is allowed.
Use of statistical table is allowed.
SECTION I

2. a) Solve the following assignment problem of minimization :


A
B
C
D
P
9
7
6
12
Q
11
13
6
10
R
10
8
9
11
S
9
11
11
12
b) Find optimal solution to following transportation problem to minimize the cost.
S1

S2

S3

S4

S5

Supply

O1

11

150

O2

200

O3

10

14

125

Sales

80

100

75

45

125

3. a) Using Simplex Method :


Maximize Z = 4x1 + 10x2
Subject to
2x1 + x2 50
2x1 + 5x2 100
2x1 + 3x2 90
x1, x2 0.
b) A bank has only one typist. Since the typing work varies in length (number of pages to be
typed) the typing rate is randomly distributed approximating a Poisson distribution with
mean service rate of 8 letters per hour. The letters arrive at a rate of 5 letters per hour,
during 8 hour work duty. If the typewriter is valued at Rs. 1.50 per hour, determine
i) Equipment utilization
ii) The percent time typewriter remains busy
iii) Average system time
iv) Average cost due to waiting on the part of typewriter.
4. a) A manufacturer of airmail envelopes knows from experience that weight of envelopes is
normally distributed with mean 1.95 gms and standard deviation 0.05 gms, about how
many envelopes weighing
i) 2 gms or more
ii) 2.05 gms or more can be expected
iii) given packets of 100 envelopes.
b) The probability density function f(x) of continuous random variable is given by
f( x ) = y o e |x| < x < prove that y o = 12 .

5
4

Set A

SLR-BB 208

5.

*SLRBB208*

-4-

c) A person repairing radios find that, the time spent on the radio set has been exponential
distribution with mean 20 minutes. If the radios are repaired in the order in which they
come in and their arrival is approximately Poisson with the average rate of 15 for 8 hours
a day, what is the repairmans expected idle time each day ? How many jobs are ahead of
the average set just brought in ?
a) The manufacturer of pins knows that on an average 5% of production is defective. He
sales pins in boxes of 100 and guarantees that not more than 4 pins will be defective in how
many boxes out of 1000 he will meet a guaranteed quantity ?
b) Maximize Z = 4x1 + 5x2
Subject to
x1 2x2 2
2x1 + x2 6
x1 + 2x2 2
x1, x2 0.
c) Find Initial basic feasible solution by VAM (Vogels Approximation Method) for the following
transportation problem to minimize the cost
O1
O2
O3
Demand

D1
2
1
5
7

D2
3
o
8
5

D3
11
6
15
3

D4
7
1
9
2

5
4

Supply
6
1
10
17

SECTION II
6. a) Verify law of absorption for fuzzy sets.

b) Let A and B are two fuzzy sets defined by A = 01.5 + 02.6 + 03.9 + 41 + 05.2 , B = 03.3 + 04.4 + 05.7 + 61 + 07.3
Let the function F : X X X be defined as F(x1, x2) = 2x1 + x2, find F(A, B).
c) Define Fuzzy number.

7
3

0.2 0.4 0.6 0.8


7. a) Find special fuzzy sets of A where A = x + x 2 + x 3 + x4 + x15 .
1

b) Let A and B are the Fuzzy numbers as below : A = 0.2 + 0.6 + 0.8 + 0.7 + 1 + 0.5 + 0.1
[0, 1)

[1, 2)

[ 2, 3)

[3, 4)

(4, 5]

and B = 0.1 + 0.2 + 0.6 + 0.7 + 0.8 + 0.9 + 61 + 0.5 + 0.4 + 0.2 + 0.1
[ 0, 1) [1, 2) [ 2, 3) [ 3, 4) [ 4, 5) [ 5, 6)
(6, 7] (7, 8] (8, 9] (9, 10]
Solve the fuzzy equation A + X = B.

(5, 6]

8. a) Represent the interconnections among modules of Fuzzy controller by using block diagram. 4
b) Define Fuzzy cardinality for fuzzy set. Consider fuzzy sets A and B whose membership
functions are defined by formulae A(x ) = x and B( x) = 1 x for all x {0, 1, 2, ..., 10} = X .
x +1
10
Calculate i) Scalar and Fuzzy cardinality of A and B. ii) Degrees of subsethood

9.

S (| A |, | B |) and S (| B |, | A |).
a) Find -cut and strong -cut for = 0.1, 0.5, 0.3 for the following fuzzy sets :
A = 0x.6 + 0x.5 + 0x.7 + 0x.3 + x1 , B = 1 + 110 x where x {0, 1, 2, ..., 10}.
2
3
4
5
1

b) Explain in short indirect method with one expert for constructing membership function.

_____________________

Set A

SLR-BB 209

*SLRBB209*
Seat
No.

Set

S.E. (CSE) (Part II) (Old) Examination, 2014


FORMAL SYSTEMS AND AUTOMATA
Day and Date : Wednesday, 26-11-2014
Time : 3.00 p.m. to 6.00 p.m.

Max. Marks : 100

Instructions : 1) Q. No. 1 is compulsory. It should be solved in first 30 minutes


in Answer Book Page No. 3. Each question carries one mark.
2) Answer MCQ/Objective type questions on Page No. 3 only.
Dont forget to mention, Q.P. Set (A/B/C/D) on Top of Page.
MCQ/Objective Type Questions
Duration : 30 Minutes

Marks : 20

1. Select correct alternative :


20
1) The context free grammar is ambiguous if
a) the grammar contains useless non-terminals
b) it produces more than one parse tree from some sentence
c) some production has two non-terminals side by side on the right hand side
d) none of the above
2) (P + Q)* = ?
a) (P* + Q*)
b) P* + Q*
c) (P* Q*)*
d) both a) and c)
3) Which string can be generated by S aS / bA, A d / ccA ?
a) aabccd
b) adabcca
c) abcca
d) abababd
4) The intersection of CFL and regular languages
a) is always regular
b) is always context-free
c) both option 1 and option 2 above d) need not be regular
5) Power of
a) DFSM and NDFSM are same
b) DFSM and NDFSM are different
c) DPDM and NDPDM are different d) Both a) and c)
6) Which of the following pairs of regular expressions are equivalent ?
a) 1(01)* and (10)*1
b) x(xx)* and (xx)* x
+
+
+
d) all of these
c) x and x x*
7) If two finite state machines are equivalent, they should have the same number of
a) states
b) edges
c) states and edges d) none of these
8) For which of the following applications regular expressions can be used ?
a) Designing compilers
b) Developing text editors
c) Simulating sequential circuits
d) All of these
P.T.O.

SLR-BB 209

-2-

*SLRBB209*

9) L = {ap | p ; } is prime. Then L is


a) regular
b) not regular
c) accepted by DFA
d) accepted by PDA
10) Which of the following are not regular ?
a) String of 0s whose length is a perfect square
b) Set of all palindromes made up of 0s and 1s
c) Strings of 0s, whose length is a prime number
d) All of these
11) We can access any element randomly from the input tape in the model of
________________
a) FA
b) PDA
c) TM
d) None of these
12) If q0 is the initial state and x is a string to be accepted by the TM, then the
initial configuration of TM is represented by _____________
a) ( qo , x )
b) ( qo , x )
c) ( x, qo )
d) None of these
13) The * for the TM is defined by ______________
a) * : Q X (U { }) Q U{ha , hr } X ( U { }) X {L, R, S }
b) * : Q X Q U {ha , hr } X (U { }) X {L, R, S}
c) * : Q X ( U { }) Q X (U { }) X {L, R, S}
d) * : Q X U { } Q U {h a , h r } X ( U { })
14) TM is called as __________________
a) Acceptor
b) Calculator
c) Both a) and b) d) None of these
15) In the Universal TM, the nonhalting states of a TM T1 are encoded as ________
a) s(qi) = 0i + 1
b) e(qi) = 0i + 2 c) s(qi) = 0i
d) s(qi) = 0i + 2
16) The TM can have strong computing power because its R/W head on the input tape
a) Move in forward direction
b) Move in backward direction
c) Remain stationary
d) All of these
17) In the parse tree, the leaf nodes corresponds to the __________
a) Variables that appears in the derivation
b) Terminals that appears in the derivation
c) Start symbol of the CFG
d) None of these
18) Which of the PDA is called shift-reduce parser ?
a) Top-down PDA
b) Deterministic bottom-up PDA
c) Non-deterministic bottom-up PDA d) NPDA
19) PDA uses _____________ as its internal memory.
a) Queue
b) Linked list
c) Stack
d) None of these
20) The class of CFL is not closed under ____________
a) Concatenation b) Intersection c) Complement d) Both b) and c)
Set A
______________

*SLRBB209*

-3-

SLR-BB 209

Seat
No.

S.E. (CSE) (Part II) (Old) Examination, 2014


FORMAL SYSTEMS AND AUTOMATA
Day and Date : Wednesday, 26-11-2014
Time : 3.00 p.m. to 6.00 p.m.

Marks : 80

2. Answer any four questions :


a) Prove that if either of a and b is even number the a * b is even number.
b) Find regular expression for :
i) Only two 0 and 1 in any order. ii) String doesnt end with 11.
c) Draw FA to recognize the following languages defined over (0, 1)*.
i) (0 + 1) * (110) ii) Language containing string of exactly two zeros

20

d) Define NFA and * for NFA.


e) Find languages corresponding to following CFG production.
i) S aSa | bSb |^
ii) S aSb | bSa |^
3. Answer any one :

10

1) Find minimal FA for following FA :


Q = {1,2,3,4,5,6} A = {3,6} and q0 = 1
Transition Table :
State input a input b
1
2
6
2
1
3
3
2
4
4
4
2
5
4
5
6
5
4
2) Given that :
L1 = { x (0, 1) | ends with 01}
L 2 = { x (0, 1) | ends with 11}

Give FA for L1, L2 and L1  L2.

Set A

SLR-BB 209

-4-

*SLRBB209*

4. Let M = (Q, , q0, 6, A ) where Q = {a, b, c, d), q0 = a and A = {d} and is given
as follows :
10
State input 0
input 1
a
{b, d}
{c, d}
b
{b}
{d}
c
{d}
{c}
d

Give transition diagram for above NFA and find whether string 100101 will be
accepted by it or not.
SECTION II
5. Attempt any four :

(45=20)

A) Explain the working of PDA with example.


B) State and explain the pumping lemma for CFL.
C) Explain the types of acceptance by PDA.
D) Design and explain Turing Machine for computing the function f(x) = x + 3,
where x is any natural number.
E) Explain the closure properties of context-free languages.
F) What is parsing ? Explain the shift-reduce parsing with example.
6. Attempt any one :

10

A) Explain in detail how Turing machines are combined.


B) Define Top-Down PDA corresponding to CFG. Draw the Top-Down PDA
corresponding to the following CFG G :
E E+ T| T
T T a | a

Explain how it works to simulate the leftmost derivation of the string


a + a* a L(G).
7. Define Turing Machine for computing numeric function. Draw and explain the
Turing Machine to compute the 2s complement of binary number.

10

_____________________

Set A

SLR-BB 21

*SLRBB21*
Seat
No.

Set

S.E. (Civil) (Part I) Examination, 2014


SURVEYING I
Day and Date : Saturday, 13-12-2014
Time : 10.00 a.m. to 1.00 p.m.

Total Marks : 100

Instructions : 1) Q. No. 1 is compulsory. It should be solved in first 30 minutes


in Answer Book Page No. 3. Each question carries one mark.
2) Answer MCQ/Objective type questions on Page No. 3 only.
Dont forget to mention, Q.P. Set (A/B/C/D) on Top of Page.
MCQ/Objective Type Questions
Duration : 30 Minutes
1. Choose the correct answer.

Marks : 20
(201=20)

1) If during first permanent adjustment of dumpy level, after turning telescope through
180, bubble goes out of center by six divisions, then the correction to be applied
by capstan headed nuts will be
a) Four divisions
b) Three divisions c) Two divisions d) One division
2) In tilting level
a) Vertical axis need not be truly vertical
b) Line of collimation need not be horizontal
c) Axis of telescope need not be horizontal
d) Levelling staff is held tilted
3) If distance between staff and dumpy level is D meter, the curvature correction
(in meter) for staff reading will be
a) + 0.0785 (D/1000)2
b) 0.0785 (D/1000)2
c) + 0.0673(D/1000)2
d) 0.0673(D/1000)2
4) If true bearing of a survey line is 34 30 and the magnetic declination is 3 30
West, the magnetic bearing of the line will be
a) 38
b) 31
c) 9.86
d) 120.75
5) During the repetition method, the face left and face right observations are taken
a) In clockwise direction only
b) In anticlockwise direction only
c) In clockwise and anticlockwise directions respectively
d) In anticlockwise and clockwise directions respectively
6) With theodolite set at T to 0 vertical angle, the staff reading on a staff held on
ABM with 100 m RL is 1m. If vertical angle to S (bottom of spout) is +(plus) 45
and horizontal distance T to S is 10 m, the R.L. of S will be
a) 1m
b) 101m
c) 111m
d) 91m
P.T.O.

SLR-BB 21

*SLRBB21*

-2-

7) If angular measurements are more precise than linear measurements the closed
theodolite traverse is balanced by
a) Transit rule
b) Bow Ditchs rule c) Graphical method d)None of these
8) The process of rotating telescope through 180 in vertical plane is called as
a) Lining in
b) Balancing in
c) Transiting
d) Swinging
9) The plotting of sounding can be done with the solution of three point problem if
the sounding point have been located by
a) Two angles from the boat
b) Two angles from the shore
c) One angle each from the boat and the shore
d) Range and one angle
10) Velocity of sound in sea water directly depends on
a) Surface temperature of water
b) Mean sea level
c) Depth of sea water
d) Distance from the sea shore
11) In case of plane table surveying inaccessible points may be located by the
a) Resection method
b) Intersection method
c) Radiation method
d) None of these
12) The principle of plane table surveying is
a) Parallelism
b) Triangulation c) Traversing

d) None of these

13) The working edge of the alidade is known as the


a) Fiducial edge
b) Bevelled edge c) Parallel edge

d) None of these

14) When contour lines touch one another at a particular zone, it indicates a
a) Level surface
b) Vertical cliff c) Horizontal surface d) All of the above
15) The contour interval is inversely proportional to the
a) Steepness of the area
b) Extent of the area
c) Scale of the map
d) None of above
16) The value of the planimeter constant (C) is added only when
a) The anchor point is inside the figure
b) The anchor point is outside the figure
c) The anchor point is just on the boundary line
d) None of these
17) Irregular area of a closed figure may be computed by an instrument known as
a) Pantograph
b) Planimeter
c) Passometer
d) None of above
18) With notations carrying their usual meanings, the cross sectional area of an
embankment is given by _________________
d) (2b + 4sh) h
a) (b + 2sh)h
b) (b +sh) h
c) (b + sh)h2
19) Geodimeter is based on principle of ______________
a) Propagation of modulated light waves b) Propagation of infrared radiation
c) High frequency radio waves
d) None of these
20) One hectare of an area is equivalent to _____________
a) 102m2
b) 104m2
c) 106m2

______________

d) 109m2

Set A

*SLRBB21*

SLR-BB 21

-3-

Seat
No.

S.E. (Civil) (Part I) Examination, 2014


SURVEYING I
Day and Date : Saturday, 13-12-2014
Time : 10.00 a.m. to 1.00 p.m.

Marks : 80

Instructions : 1) Q. 2 and Q. 6 are compulsory. Attempt any two out of


Q. 3 to Q. 5 and any two out of Q. 7 to Q. 9.
2) Assume suitable data if necessary but mention it clearly.
SECTION I
2. a) During determination of sensitivity of bubble tube following readings were
observed
Bubble at

Staff reading in meter

Center

1.500

1 div out of center

1.490

2 div out of center

1.480

3 div out of center

1.465

Remark
Dist. of staff
from the
instrument
= 50 meter

What will be the


i) Angular value of one division of the bubble tube and
ii) Radius of curvature of the bubble tube ?

b) Explain the construction of automatic level with suitable sketch. What are its
temporary adjustments ?

c) A tip of lighthouse of 85m height above MSL (Mean Sea Level), is just visible to a
person standing on the deck of ship with eye level 6m above MSL. Determine the
distance between the lighthouse and ship assuming the radius of earth as 6370 km.

Set A

SLR-BB 21

*SLRBB21*

-4-

3. a) Following are the readings taken during a closed theodolite traverse :

Consecutive Coordinates
Station

Latitude

Departure

Northing Southing
P

23

34

12

44

Independent
Coordinates

Easting Westing Northing

Easting

46

50

50

57
32
43

Calculate :
i) The length of closing error and its bearing.
ii) Corrections to consecutive coordinates using transit rule.
iii) Corrected consecutive coordinates.
iv) Independent coordinates for remaining stations.

b) Compare the repetition method and reiteration method for measurement of


horizontal angle indicating errors eliminated with each method.

4. a) During a closed compass traverse following readings were omitted. Determine


the omitted measurements.

Line

Length (m)

Corrected Whole Circle Bearing

AB

55

BC

100

325

CD

100

235

DA

145

Set A

*SLRBB21*

SLR-BB 21

-5-

b) Explain Trigonometric leveling Object inaccessible case and both instrument


stations and object is in different vertical planes with suitable sketch.

5. a) Explain method of locating the sounding.

b) Explain construction and use of Fathometer, Ghat tracer and proportional


compass.

SECTION II
6. a) Explain the construction and working of distometer ?

b) The following offsets are taken from a survey line to a curved boundary line.

Distance (m)
Offset (m)
Distance (m)
Offset (m)

10

15

20

2.50

3.80

4.60

5.20

6.10

30

40

60

80

4.70

5.80

3.90

2.20

Find the area between the survey line, the curved boundary line and first and last
offset by
i) The Trapezoidal rule
ii) The Simpsons rule
7. a) Explain the procedure for solving Two point problem, with neat sketch.
b) What are advantages and limitations of plane table survey ?
8. a) What are the various methods for measuring volumes of earth work ? Explain
the method of cross section.
b) Write short note on ,Co-ordinate method.

9
8
5

8
5

Set A

SLR-BB 21

*SLRBB21*

-6-

9. a) Calculate the area of the zero circle with the following data :

I.R.

6.520

1.222

F.R.

2.724

7.720

Position of
anchor point

Remarks

Outside the figure

Zero of counting disc


cross the fixed index
mark once in the
clockwise direction

Inside the figure

Zero of counting disc


crossed the fixed index
mark twice in the
anticlockwise direction

Assume that the tracing arm of the planimeter was so set that one revolution
of the measuring wheel measures 100 cm2 on the paper.
b) What are the characteristics of contours ?

_____________________

Set A

SLR-BB 210

*SLRBB210*
Seat
No.

Set

S.E. (Part II) (Old) (Computer Science and Engineering)


Examination, 2014
MICROPROCESSORS
Day and Date : Thursday, 27-11-2014
Time : 3.00 p.m. to 6.00 p.m.

Total Marks : 100

Instructions : 1) All questions are compulsory.


2) Figures to the right indicate full marks.
3) Q. No. 1 is compulsory. It should be solved in first 30 minutes
in Answer book Page No. 3. Each question carries one mark.
4) Answer MCQ/Objective type questions on Page No. 3 only.
Dont forget to mention, Q.P. Set (A/B/C/D) on Top of Page.
MCQ/Objective Type Questions
Duration : 30 Minutes

Marks : 20

1. Choose the correct option.

20

1) In a microprocessor, the resister which holds address of the next instruction to be


fetched is
a) Accumulator
b) Program counter
c) Stack pointer
d) Instructor register
2) In a microprocessor, WAIT states are used to
a) Make the processor wait during a DMA operation
b) Make the processor wait during a power interrupt processing
c) Make the processor wait during the power shutdown
d) Interface slow peripherals to the processor
3) The invalid instruction from the given options are
a) STAXH
b) MOV A,B
c) MOVB, 10

d) Both a) and c)

4) LHLD 3000 is ___________ byte instruction and requires __________ machine


cycles for its execution.
a) 3, 4
b) 3, 5
c) 3, 2
d) 3, 1
5) Match the following :
I. MVI, A, B
A.
II. STA 6000
B.
III. CALL 2000
C.
IV. HLT
D.
a) I-C, II-D, III-B, IV-A
c) I-D, II-A, III-B, IV-C

5
3
2
1
b) I-D, II-B, III-A, IV-C
d) None
P.T.O.

SLR-BB 210

-2-

*SLRBB210*

6) The construct 2k 8 memory using 1k 4 memory, number of chips


required are
a) 4
b) 3
c) 2
d) 8
7) AD7-0 bus is _________ and A15-8 bus is _________ respectively.
a) Both bidirectional
b) Both unidirectional
c) Bidirectional, unidirectional
d) Unidirectional, bidirectional
8) Instruction LDA 5000H and OUT 05H respectively represents
a) Both memory mapped
b) Both IO mapped
c) Memory mapped IO, IO mapped IO
d) IO mapped IO, memory mapped IO
9) When HLT instruction of a 8085 microprocessor is executed the
microprocessor ?
a) Is disconnected from the system bus till the reset is pressed
b) Halts execution of the program and returns to monitor
c) Enters into a halt state and the buses are tri-stated
d) Reloads the program form the locations 0024 and 0025 H
10) Ready and Reset in pins of 8085 respectively are
a) Both output pins
b) Both input pins
c) Input, output
d) Output, input
11) Disable interrupt
a) is used to disable all maskable interrupts
b) is a single byte instruction
c) it requires only one machine cycle
d) all of the above
12) Maskable interrupt
a) is used to interface peripherals
b) it is at lower priority
c) response time is high
d) all of the above
13) ISR address for TRAP interrupt is
a) 0024H
b) 003CH
c) 0034H
d) 002CH
14) Which of the following IC is Programmable Interrupt Controller ?
a) 8259
b) 8255
c) 8253
d) 8251
15) Which of the following power is selected if pins of 8255 are CS = 0, A1 = 0 and
A0 = 1 ?
a) Port B
b) Port A
c) Port C
d) Control Register
16) Which of the following is a type of digital to analog converter ?
a) Current output
b) Voltage output c) Multiplying type d) All of the above
17) To reset bit PC7 of 8255, what is the control word format of 8255 in BSR
mode ?
a) 0FH
b) OEH
c) 07H
d) 06H
18) Which of the following IC is the Programmable Peripheral Interface ?
a) 8255
b) 8253
c) 8251
d) 8259
19) If the data is transmitted in both the direction but in one way at a time is called
a) Half duplex
b) Simplex
c) Full duplex
d) None of the above
20) Serial communication occurs in
a) Synchronous format
b) Asynchronous format
c) Both a) and b)
d) None of the above
Set A
______________

*SLRBB210*

SLR-BB 210

-3-

Seat
No.

S.E. (Part II) (Old) (Computer Science and Engineering)


Examination, 2014
MICROPROCESSORS
Day and Date : Thursday, 27-11-2014
Time : 3.00 p.m. to 6.00 p.m.

Marks : 80

Instructions : 1) All questions are compulsory.


2) Figures to the right indicate full marks.
SECTION I
2. Solve any four :

(44=16)

a) Describe the methods of demultiplexing of address and data bus of 8085.


b) Draw the timing diagram of instruction MVI 8-bit address.
c) Distinguish between single stepping and single cycle execution of an instruction.
d) Write an ALP to separate positive and negative numbers from the series of
10 numbers.
e) Describe various addressing modes with examples.
3. Solve any three :

(38=24)

a) What are various transition states of 8085 ? Draw and explain complete transition
state diagram.
b) Define the following terms related with semiconductor memories :
i) Memory access time
ii) Volatile memory
iii) Memory address
iv) Dynamic memory
c) Explain the following instructions by considering addressing mode, byte length,
machine cycles, flags affected after execution.
i) OUT 20H
ii) NOP
iii) RAR
iv) JNZ 1000
d) Explain the following pins of 8085.
i) Ready
iii) IO/ M

ii) S1and S0
iv) Reset OUT
Set A

SLR-BB 210

-4-

*SLRBB210*

SECTION II
4. Attempt any 4 :

(44=16)

1) Draw the format of SIM and RIM instruction.


2) What are the functions of following blocks in case of programmable interrupt
controller ?
a) Interrupt Request Register (IRR)
b) In-Service Register (ISR)
c) Interrupt Mask Register (IMR)
d) Priority resolver.
3) Write a note on control word register format of 8253.
4) With neat diagram, explain weighted resister DAC.
5) Write the features of 8251.
5. Attempt any 3 :

(38=24)

1) With neat diagram explain the use of HOLD and HLDA pins for DMA transfer.
2) Explain the concept of interfacing thumbwheel switches using 8255 with neat
diagram.
3) With neat circuit diagram explain successive approximation and dual slope ADC.
4) Explain the three I/O modes of 8255.
_____________________

Set A

SLR-BB 211

*SLRBB211*
Seat
No.

Set

S.E. (Computer Science and Engineering) (Part II) (Old) Examination, 2014
DATA STRUCTURE II
Day and Date : Friday, 28-11-2014
Time : 3.00 p.m. to 6.00 p.m.

Max. Marks : 100

Instructions : 1) All questions are compulsory.


2) Figures to right indicate full marks.
3) Q. No. 1 is compulsory. It should be solved in first 30 minutes
in Answer book Page No. 3. Each question carries one mark.
4) Answer MCQ/Objective type questions on Page No. 3 only.
Dont forget to mention, Q.P. Set (A/B/C/D) on Top of Page.
MCQ/Objective Type Questions
Duration : 30 Minutes

Marks : 20

1. Choose correct alternatives :


1) Two main measures of the efficiency of an algorithm are
A) Processor and Memory
B) Time and Space
C) Complexity
D) Date and Time

20

2) Rehashing is used for


A) To avoid clustering
B) To increase collison
C) To avoid traversal
D) To avoid space wastage
3) The complexity of bubble sort algorithm
A) 0(n)
B) 0(log n)
C) 0(n2)
D) 0(n log n)
4) A tree node that has no children is called a _____________
A) Leaf node
B) Root node
C) Internal node D) Loop node
5) The method in which a node to be deleted is replaced a by a rightmost node
in its left subtree or leftmost node in its right subtree is
A) Delete a node with single child in BST
B) Delete a node with both left and right child in BST
C) Delete a left node in BST
D) None of above
6) The tree traversal technique in which the root is traversed before its children
is known as
A) Post order traversal
B) Pre order traversal
C) In order traversal
D) None of above
P.T.O.

SLR-BB 211

-2-

*SLRBB211*

7) This sort does not use divide and conquers methodology.


A) Merge sort
B) Quick sort
C) Bubble sort
D) None of these
8) In AVL tree balance factor can be ____________
A) 1, 0, 1
B) 2, 1, 2
C) 1, 0, 2
D) None of above
9) The worst case complexity of heap sort is
A) O(nlogn)
B) O(logn)
C) O(n)
D) O(n2)
10) Any complete binary tree with n internal nodes has _____________ external
nodes.
A) n
B) 2 n
C) 2n + 1
D) n + 1
11) If all the leaves are present at same level in binary tree, then it is called as
______________
A) Binary search tree
B) Strictly BT
C) Complete BT
D) None of these
12) Hashing is an ______________
A) Searching technique
B) Sorting technique
C) Both A) and B)
D) None of these
13) A Btree having order of 5 can have maximum of _____________ number of
keys.
A) 1
B) 4
C) 5
D) 6
14) In a graph if a node is having indegree as 1 and outdegree as 0 then it is
called ______________
A) Child
B) Pendant Vertex
C) Isolated vertex
D) None of these
15) A Graph is a forest.
A) True
B) False
16) If an edge whose start and end vertices are same then it is called _________
A) Isolated vertex B) Self loop
C) Pendent vertex D) Weighted graph
17) In B-tree of order m, internal nodes contain at most ____________ keys.
A) m/2
B) m 1
C) m
D) m + 12
18) In a multiway tree, each node can have more than _____________ keys.
A) One
B) Two
C) Three
D) All the above
19) In ____________ all leaves are at same level.
A) B + Tree
B) B Tree
C) Both A) and B) D) Neither A) nor B)
20) A strictly binary tree with n leaves always contains ____________ nodes.
A) 2n 1
B) n 1
C) n + 1
D) 2n + 1
______________
Set A

*SLRBB211*

-3-

SLR-BB 211

Seat
No.

S.E. (Computer Science and Engineering) (Part II) (Old) Examination, 2014
DATA STRUCTURE II
Day and Date : Friday, 28-11-2014

Marks : 80

Time : 3.00 p.m. to 6.00 p.m.


Instructions : 1) All questions are compulsory.
2) Figures to right indicate full marks.
SECTION I
2. Solve any four :

(54=20)

1) Write a algorithm for searching a node in a binary search tree.


2) What is a Binary tree ? Mention the properties of a binary tree.
3) Differentiate between linear and binary search.
4) Perform bubble sort (ascending) of following numbers-show every iteration
7, 3, 11, 5, 1, 19, 17
5) Explain the concept of asymptotic notations with corresponding graphs.
6) Give any three Hashing functions with examples.
3. What is threaded-Binary tree ? Explain with example.

10

a) Left-in threaded Binary Tree


b) Right-in threaded Binary Tree
c) Fully threaded Binary Tree.
4. Write a C program for implementing Quick sort using recursive technique.

10

OR
4. What do you mean by Hashing ? Explain different open addressing techniques to
resolve collision in detail by giving suitable examples.
Set A

SLR-BB 211

-4-

*SLRBB211*

SECTION II
5. Solve any four :

(54=20)

1) Define the Graph and explain adjacency list and adjacency matrix
representation of Graph.
2) Explain deletion operation in multi-way search tree.
3) Define AVL tree and give one example.
4) Compare between tree and graph.
5) Give the Depth first traversal algorithm for graph. Also explain with example.
6) What is Fibonacci tree ? Explain with example.
6. Explain with example and algorithm the topological ordering of graph.

10

7. Explain finding the shortest path in a given graph using Dijkstra algorithm, (state
and assume suitable data).
10
OR
7. Explain in details Balanced tree, also give suitable example for insertion and
deletion from balanced tree.
_____________________

Set A

SLR-BB 212

*SLR-BB-212*
Seat
No.

Set

S.E. (CSE) (Part II) (Old) Examination, 2014


COMPUTER NETWORKS I
Day and Date : Saturday, 29-11-2014
Time : 3.00 p.m. to 6.00 p.m.

Max. Marks : 100

Instructions : 1) Q. No. 1 is compulsory. It should be solved in first 30 minutes


in Answer Book Page No. 3. . Each question carries one mark.
2) All questions are compulsory.
3) Figures to the right indicate full marks.
4) Answer MCQ/Objective type questions on Page No. 3 only.
Dont forget to mention, Q.P. Set (A/B/C/D) on Top of Page.
MCQ/Objective Type Questions
Duration : 30 Minutes

Marks : 20

1. Choose correct alternatives :


1) Which of the following is not a scrambling technique ?
a) 08ZS
b) HDB3
c) Bipolar-AMI

20
d) All

2) ____________ are privately-owned networks within a campus.


a) WAN
b) LAN
c) MAN
d) WWW
3) The strength of a signal falls off with distance is ___________
a) Noise
b) Attenuation
c) Collision
d) Delay distortion
4) Guided medium is ___________ if it provides a direct link between
two-devices.
a) point-to-point
b) multipoint
c) multicasting
d) all
5) In ATM reference model, __________ layer deals with cells and cells
transport.
a) Physical
b) AAL
c) ATM
d) None of above
6) In OSI, __________ layer is controlling the operations of subnet.
a) DLL
b) Network
c) Transport
d) MAC
7) Which of the following is not an example of analog data ?
a) Audio
b) Text
c) Video
d) None of above
8) DQDB is used in __________
a) LAN
b) MAN

c) WAN

d) WWW

P.T.O.

SLR-BB 212

-2-

*SLR-BB-212*

9) In CRC, if generator is n-bits long, then __________ zero bits are appended
to the original message of m-bits.
a) n 1
b) m 1
c) n
d) none of the above
10) Protocols in which sender sends one frame and then waits for an
acknowledgment before proceeding are __________protocols.
a) Sliding window b) Stop-and-wait c) Simplex
d) All
11) Which of the following is a static channel allocation method ?
a) CSMA
b) TDM
c) CSMA/CD
d) Bit-map
12) Pure ALOHA uses __________ time.
a) continuous
b) slotted
c) carries sense d) all
13) Protocols in which stations listen for a carrier and act accordingly are
called __________ protocol.
a) Carrier sense b) Contention
c) Reservation
d) OSI
14) The bits in each stations address are BOOLEAN ORed in __________
a) Bit-map
b) CSMA/CD
c) Binary countdown
d) None
15) IEEE standard 802.4 is known as __________
a) Ethernet
b) Token Ring
c) Token Bus
d) Ring
16) Congestion control is __________ in Datagram subnet.
a) Easy
b) Difficult
c) Simple

d) Cant say

17) Flow-based routing is __________ routing algorithm.


a) Static
b) Dynamic
c) Nondeterminastic
d) All
18) Count-to-infinity problem comes under __________ routing.
a) Flooding
b) Distance vector
c) Link state
d) Flow-based
19) Packets are fragmented and reassembled at each subsequent network in
__________ fragmentation.
a) Transparent
b) Non-transparent
c) Both
d) None
20) __________ solves the problem of finding out Ethernet address corresponds
to a given IP address.
a) ARP
b) RARP
c) BOOTP
d) None
______________

Set A

*SLR-BB-212*

-3-

SLR-BB 212

Seat
No.

S.E. (CSE) (Part II) (Old) Examination, 2014


COMPUTER NETWORKS I
Day and Date : Saturday, 29-11-2014

Marks : 80

Time : 3.00 p.m. to 6.00 p.m.


Instructions : 1) All questions are compulsory.
2) Figures to the right indicate full marks.
SECTION I
2. Solve any four (each carries 5 marks) :

20

A) Compare between analog and digital transmission.


B) Describe physical description and applications of coaxial cable.
C) Convert 1010111001 in Manchester and differential Manchester encoding
format.
D) Describe need of protocol hierarchies.
E) Explain ATM reference model.
F) What is framing ? Explain types of framing methods.
3. Describe all three transmission impairments in detail.

10

OR
Describe OSI reference model in detail.
4. Derive transmitted data for 10101101 using :

10
10

i) Hamming code
ii) CRC (Consider 101 as generator).

Set A

SLR-BB 212

-4-

*SLR-BB-212*

SECTION II
5. Solve any four (each carries 5 marks) :

20

A) Describe CSMA/CD in detail.


B) Draw and explain frame format of IEEE std. 802.5.
C) Describe flow-based routing with example.
D) Explain Leaky bucket algorithm.
E) What is IP address ? Describe classes of IP address.
F) Write a note on Bridges.
6. Write and explain shortest path routing with Dijakstra algorithm and example.

10

OR
Describe following in detail :

10

i) ARP
ii) RARP
7. Describe following Collision-free protocols.

10

i) A bit-map protocol
ii) Binary count down
_____________________

Set A

SLR-BB 213

*SLR-BB-213*
Seat
No.

Set

S.E. (CSE) (Part II) (New) Examination, 2014


APPLIED MATHEMATICS II
Day and Date : Tuesday, 25-11-2014
Time : 3.00 p.m. to 6.00 p.m.

Max. Marks : 100

N.B. : 1) Question No. 1 is compulsory. It should be solved in first 30 minutes


in Answer book Page No. 3. Each question carries one mark.
2) Use of scientific calculator is allowed.
3) Answer MCQ/Objective type questions on Page No. 3 only. Dont
forget to mention, Q.P. Set (A/B/C/D) on Top of Page.
MCQ/Objective Type Questions
Duration : 30 Minutes

Marks : 20

1. Tick (9)mark the correct alternative (one mark each) :


1) Newtons iterative formula to find approximate value of
number is _____________
a) xn+1 =

1
2

(x

N
xn

b) xn+1 =

1
2

(x

1
Nx n

20
1
N

where N is a positive real

c) xn+1 = xn (2 Nxn) d) xn+1 = 2 x n +

1
Nx n

2) Every equation of the nth degree has _________ roots.


a) n
b) n + 1
c) n 1
d) 1
3) The order of convergence of Regula falsi method for finding root of equation f(x) = 0 is
a) Second order
b) Cubic order
c) First order
d) Very slow
4) In solving simultaneous linear equations which of the following method is iterative method ?
a) Factorization
b) Jacobis
c) Elimination
d) Jordan
5) The power method is used to find __________
a) smallest eigen value of the matrix
b) only integer valued eigen value
c) largest eigen value of the matrix
d) none
6) When Gauss-Elimination method is used to solve set of equation AX=B, matrix A is
transformed to
a) upper triangular matrix
b) Diagonal matrix
c) Identity matrix
d) Scalar matrix

7) To apply Simpsons 13 rd rule the number of sub-intervals must be


a) odd
b) multiple of 3
c) any

d) even

8) Which of the following is the Gaussian two-point formula ?


1

a)

c)

1
1

+ f
f(x) dx = f
3
3

b)

1
1

+ f
3
3

f(x) dx = f
0

1
1

+ f
3
3

f(x) dx = f

d) None

9) The rule of integration which is most accurate is


a) Trapezoidal

3
b) Simpsons
8

th

1
c) Simpsons
3

rd

d) Weddles rule
P.T.O.

SLR-BB 213

*SLR-BB-213*

-2b

10) If I1 and I2 represent approximate value of integration I = f(x)dx , obtained by trapezoidal


a

h
then by Rombergs method I is obtained by
rule by taking the spacing for x as h and
2
I I
a) I = I2 2 1
3

I + I
b) I = I2 + 2 1
3

I I
c) I = I2 + 2 1
3

d) None

11) For any fuzzy set A defined on universal set X, strong 1 cut of A is
a) a closed interval b) an open interval c) empty set
d) none of the above
12) The subtraction of closed interval [2, 4] [3, 6] =
a) [5, 2]
b) [5, 2]
c) [5, 2]
13) If ((x) =

1
and D(x) =
1 + 5x

a) C D

d) [8, 1]

, x 0 then ordering of fuzzy sets is


1 + 5x

b) D C

d) C = D

c) C = D

14) Consider the fuzzy sets


A=

0.4 0.6 1 0.34 0.12


+
+ +
+
x
y
z
u
v

B=

0.3 0.5 0.67 0.9 0.8 0.32


+
+
+
+
+
x
y
z
u
v
w

then the scalar cardinality | A B | =


a) 4.02

b) 3.7

c) 5.95

d) 5.63

x
, 0 x < then the level set of A(x) is the interval
x +1
b) [0, 1]
c) [1,1]
d) [0, 1)

15) Consider the fuzzy set A(x) =


a) (0, 1)

16) Let A be a fuzzy set, then the fuzzy complement of A is defined as


a) A (X) = A(X)

b) A (X) = A1(X)

c) A (X) = 1 A(X)

1
d) A (X) = A X

17) The fuzzy proposition P, taking the form P : If X is A the Y is B is


a) conditional and qualified
b) conditional and unqualified
c) unconditional and qualified
d) unconditional and unqualified
18) The crisp rule p q ; q R p R is known as
a) disjunctive syllogism
b) hypothetical syllogism
c) modus ponens
d) modus tollens

19) Using Lukasiewicz implication j 0, 1 =


2
1
a) 0
b) 2

c) 1.5

d) 1

tan x for 0 x
cos x 0 x / 2
20) Consider (i) A(x)
(ii) B( x)
otherwise
otherwise
0
0
then the fuzzy number is
a) only (i)

b) only (ii)
c) both (i) and (ii)
______________

d) None of (i) and (ii)

Set A

*SLR-BB-213*

SLR-BB 213

-3-

Seat
No.

S.E. (CSE) (Part II) (New) Examination, 2014


APPLIED MATHEMATICS II
Day and Date : Tuesday, 25-11-2014
Time : 3.00 p.m. to 6.00 p.m.

Marks : 80

Instructions : 1) Attempt any three questions from each Section.


2) Figures to the right indicate full marks.
3) Use of calculator is allowed.
SECTION I
2. a) Find the positive real root of the equation cosx 3x + 1 = 0 using Regula falsi method.

b) Perform two iterations of the Newton-Raphson method to solve non-linear equations


x2 + y2 = x and y = x2 y2 starting with initial conditions as (0.8, 0.4).

c) Solve the system of equations by Gauss-Jordan method :


2x 3y + 4z = 13
x+y+z=9
3x + 4y + 5z = 40
3. a)

1 1
i) Find complex eigen value for the matrix
.
0 .4 0 .6

5 4
ii) Determine the largest eigen value and corresponding eigen vector of the matrix
.
1 2
1
Take X0 = using power method.
0

b) Solve the system of equations


27x + 6y z = 85
x + y + 54z = 110
6x + 15y + 2z = 72
by using Gauss-Jocobis and Gauss Seidal method.
0 .5

4.

a) Evaluate the integral

sin x dx using Rombergs method correct to three decimal

places taking h = 0.25, 0.125, 0.0625.


rd

b)

1
rule to evaluate the integral
Apply Simpsons
3

c) Using N-R method find an approximate value of

6
2 .6 4 .4

12 .

dx dy
xy .

4
3

Set A

SLR-BB 213

*SLR-BB-213*

-4-

th

3
5. a) Using Simpsons
rule evaluate
8

x dx
5 + 2x , n = 8.

4
3

b) Using Gaussian quadrature three point formula evaluate

cos 2x
dx.
1 + sin x

c) Using Mullers method find an approximate root of the equation cosx = x ex take x0 = 1,
x1 = 0, x2 = 1 (perform 1 iteration).

OR
c) Using Bisection method find an approximate root of the equation x sin x = 1 (perform 6
iterations).

SECTION II
6.

a) Describe the concept of fuzzy set in your own words. Give some examples of fuzzy
variables from daily life.

b) Prove that F(x) : fuzzy power set of sets defined on the universal set X forms de-Morgans
algebra.

OR
b) Prove the following properties of fuzzy sets.
i) If then A A and + A
+
A

iii) ( A B ) = A B

6
ii) ( A B ) = A B
iv) A = (1 )+ A .

c) If A and B are fuzzy sets defined on universal set X = set of integers given by

A =
B=

0 . 5 1 0 .5 0 . 3
+ +
+
1 0
1
2
0.5 1 0.5 0.3
+ +
+
2
3
4
5

A function f : X X X defined by f(x1, x2) = 2x1 + x2 find f(A, B).

7. a) Let

(x + 2) / 2 for

A(x) = (2 x) / 2 for
0

(x 6) / 2 for

C(x ) = (10 x) / 2 for


0

( x 2) / 2

B( x) = (6 x) / 2
0

2< x 0
0<x2
otherwise
6<x8
8 < x 10
otherwise

for

2<x4

for

4<x6
otherwise

Solve for X the equation i) A + X = B and ii) B.X = C.

Set A

*SLR-BB-213*

-5-

SLR-BB 213

x
x
B(x) = 1
x {0, 1, 2, 3, ...,10} find fuzzy
x +1
10
~
~
~
~
cardinality of A and B i.e. | A | and | B | also find degree of subsethood s(| A | , | B |) and
~
~
S (| B |, | A |).

b) Consider the fuzzy sets A(x) =

8. a) Explain briefly :
i) Conditional and unqualified
ii) Conditional and qualified fuzzy proposition. Discuss their forms.

b) Find -cuts and strong -cuts for each of the following fuzzy sets for values of = 0.2, 0.5, 0.8, 1.
A(x) =

x
1
B( x) = 2 x C( x) =
where x {0, 1, 2, 3, 4}.
x+2
1 + 10( x 2)2

c) Find the following for the two given fuzzy sets.


i) height of A and B
ii) Are A and B normal
iii) Sketch the graphs of A and B.
A(x ) x + 2

= 2x

for 2 < x 0
for

0<x2
otherwise

B( x) = (x 2) / 2

(6 x ) / 2
0

2<x4
0<x6
otherwise

9. a) Let A and B be two fuzzy no. whose membership functions are given by
x + 2
3

A( x ) = 4 x
3

0
x 1

B( x) = 3 x
0

find the two fuzzy

2 < x 1
1< x 4
otherwise
1< x 2
2<x3
otherwise
numbers MAX (A,B) (x) MIN (A,B) (x).

Set A

SLR-BB 213

-6-

b) Complete the following table where :

*SLR-BB-213*
5

a b = min (a, b)
a b = max (a, b)
a b = min (a, 1 a + b)
a b = 1 | a b|
a

1
2
1

0
1
2
1
2
1
2
1
1
1

ab ab a b a b

0
1
2
1
0
1
2
1
_____________________

Set A

SLR-BB 214

*SLRBB214*
Seat
No.

Set

S.E. (CSE) (Part II) Examination, 2014


THEORY OF COMPUTATION (New)
Day and Date : Wednesday, 26-11-2014
Time : 3.00 p.m. to 6.00 p.m.

Max. Marks : 100

Instructions : 1) Q. No. 1 is compulsory. It should be solved in first 30 minutes in


Answer book Page No. 3. Each question carries one mark.
2) Answer MCQ/Objective type questions on Page No. 3 only. Dont
forget to mention, Q.P. Set (A/B/C/D) on Top of Page.
MCQ/Objective Type Questions
Duration : 30 Minutes

Marks : 20

1. Choose the correct alternative/s :


1) The string 1101 does not belong to the set represented by
A) 110*(0 + 1)
B) 1(0 + 1)*101
C) (00 + (11)*01)*
D) (10)*(01)*(00 + 11)*
2) The given CFG indicate strings s as | bx | x | a | b x as | a
A) That do not contain bbb
B) With aa or bb, but not both
C) That has no consecutive bs but as can be consecutive
D) Strings containing atmost two as given by
3) Strings containing atmost two as given by
A) b*ab*a
B) b*(a + )b*(a + )
C) b* + b*ab + b*ab*ab
D) All of above
4) The statement, if R1 and R2 is regular expression then R1+R2 is regular expression
A) True
B) False
C) Cant say
D) None of the above
5) Which of the following regular exp. identity is true ?
A) r(*) = r*
B) (r*s*)* = (r + s)*
C) (r + s)* = r* + s*
D) r*s* = r*+ s*
6) Which of the following string is not generated by following grammar ?
S SaSbSl
A) aabb

B) abab

C) aababb

D) aaabb

7) A language accepted by DFA is


A) Regular
B) Non regular

C) Strings

D) Binary search

8) (a + b)* a(a + b)* gives


A) Odd length strings
C) Even length

B) String containing a and b


D) Strings with atleast one a

9) Ambiguous grammer produces


A) More parse tree
C) Graph

B) No parse tree
D) All of the above
P.T.O.

SLR-BB 214

*SLRBB214*

-2-

10) DFA have _____________ memory.


A) Finite
B) Infinite

C) Undertermined

11) PDA is the machine format of


A) Type language
C) Type 2 language

B) Type 1 language
D) Type 3 language

D) None

12) Which is not true for mechanical diagram of PDA ?


A) PDA contains a stack
B) The head reads as well as writes
C) The head moves from left to right
D) Input string is surrounded by infinite number of blanks in both side
13) In PDA transitional function is in the form
A) Q ( {}) (Q, )
B) Q Q
C) Q Q
D) Q Q
14) What can be said about a regular languages L over {a} whose minimal finite state
automation has two states ?
A) L must be {an|n is odd}
B) L must be {an| n is even}
C) L must be {an|n> }
D) Either A or B
15) Let L be a language recognizable by a finite automation. The language
REVERSE(L) = {W, such that w is the reverse of V L} is a
A) Regular language
B) Context free language
C) Context sensitive language
D) Recursive enumerable language
16) The logic of pumping lemma is a good example of
A) Pigeon hole principle
B) Divide and conquer Technique
C) Recursion
D) Iteration
17) Which is not a part of the mechanical diagram of Turing machine ?
A) Input tape
B) Read-write head
C) Finite control
D) Stack
18) Difference between turing machine and two way FA is in
A) Input tape
B) Read-write head
C) Finite control
D) All of these
19) Next move function of a turing machine M = {, , , , q0 , B, F} is a mapping
B) : Q Q {L, R}
A) : Q Q
C) : Q Q {L,R}
D) : Q Q {L, R}
20) If L1 = { | is a palindrome in {0 + 1} *},
L2 = {letter (letter + digit)*}
L3 = {0n 1n 2n | n >1} and
L4 = {am bn am + n | m, n > 1}, then which of the following is incorrect ?
A) L1, is a CFL and L3 is CSL
B) L2 is regular and L4 is not a CFL
C) Both L1 and L2 are regular sets
D) Both L3 and L4 are CSL

______________

Set A

*SLRBB214*

-3-

SLR-BB 214

Seat
No.

S.E. (CSE) (Part II) Examination, 2014


THEORY OF COMPUTATION (New)
Day and Date : Wednesday, 26-11-2014
Time : 3.00 p.m. to 6.00 p.m.

Marks : 80

SECTION I
2. Solve any four :

20

1) Construct finite automation for the following regular expression (0+1)*(11+110)(0+1)*

(54)

2) Find the regular expression for


1) The lang. of strings consist of odd numbers of 0s.
2) The lang. of strings containing atleast two 0s .
3) Compare between NFA and DFA.
4) Convert CFG to CNF
S aC | SB
B aSB | bBC
C aBC | ad
5) For the following regular expression, draw an NFA , with the help of Kleenes
theorem (0+1) (01)*(011)*
3. For the following NFA- , draw an equivalent NFA and FA accepting same.

10

OR
3. Define the grammer rules for the following Regular expression (0+1)*(01+110).

10

Set A

SLR-BB 214

-4-

*SLRBB214*

4. A transition table is given for an NFA- with Seven states.


q

d(q, a)

{2}

{3}

{5}

{4}

{4}

{1}

{6, 7}

{5}

10

d(q, b) d(q, )

Find
1) d*(1, ba)
3) d*(1, ababa)
5) ({1}

2) d*(1, ab)
4) ({2, 3})
SECTION II

5. Answer any four questions :


(45=20)
i) Define PDA with diagram and differentiate DPDA and NPDA.
ii) Construct an NPDA for the language L = {w {a, b}* : na(w) = nb(w)}
iii) Find a context free grammar that generates the language accepted by the NPDA
transitions.
(q0, a, z) = (q0, Az)
(q0, a, A) = (q0, A)
(q0, b, A) = (q1, )
(q1, , z) = (q2, )
iv) Show that the language L = {anbn : n 0, n is not a multiple.
v) Design a Turing machine that accepts L = {anbncn : n 1}.
vi) Write a short note on combining turing machines for complicated tasks.
6. Prove that if L = L(M) for some NPDA M, then L is a context-free language.

10

OR
Prove that for any context-free language L, there exists an NPDA M such that L = L(M).
7. Construct Turing machine that will accept the following languages on {a, b}.
i) L = {anbman+m : n , m 1}

10

ii) L = {anbnanbn : n }

OR
Write short notes on the following :
i) Turing machine with semi-infinite tape ii) Turing machine with multi tape
iii) Off-line Turing machine.
_____________________

Set A

SLR-BB 215

*SLRBB215*
Seat
No.

Set

S.E. (CSE) (Part II) (New) Examination, 2014


MICROPROCESSORS
Day and Date : Thursday, 27-11-2014
Time : 3.00 p.m. to 6.00 p.m.

Max. Marks : 100

Instructions : 1) Q. No. 1 is compulsory. It should be solved in first 30


minutes in Answer book Page No. 3. Each question carries
one mark.
2) Answer MCQ/Objective type questions on Page No. 3 only.
Dont forget to mention, Q.P. Set (A/B/C/D) on Top of Page.
MCQ/Objective Type Questions
Duration : 30 Minutes
1. Multiple choice questions :

Marks : 20
(201=20)

1) Which of the following is 32 bit processor ?


a) 8086
b) 80286
c) 80386
d) Pentium 4
2) The microprocessor
a) Reads instruction from memory
b) Communicates with all peripherals using the system bus
c) Controls the timing of information flow
d) All of the above
3) Specify the contents of memory location 8000H after the following instructions
are executed
MVI B, 05H
MOV A, B
SUB B
STA 8000H
HLT
a) 05H
b) 00H
c) FFH
d) 10H
4) Which one of the following is the control signal ?
a) RD
b) S1
c) S2
d) IO/ M
5) Which of the following option has the same timing diagrams ?
a) DAA. NOP, EI, DI
b) DAA, NOP, ADI data, ACI data
c) DAA, NOP, ORI data, XRI data
d) DAA, NOP, SUI data, SBI data
6) Which of the following instruction requires idle machine cycle ?
a) MOV R1, R2
b) MVI A, 35 H c) STAX, Rp
d) DAD Rp
P.T.O.

SLR-BB 215

-2-

*SLRBB215*

7) In HALT state
a) The execution of processor is stopped
b) Address buses are tristated
c) Data and control buses are tristated
d) All of the above
8) The mechanical switches of keyboard can be arranged in
a) Matrix form only
b) Non-matrix form only
c) Both matrix and non-matrix
d) None of the above
9) The hexadecimal key board consists of 16 keys. The keys are arranged as
4 4 matrix, hence the number of I/O lines required to connect this keyboard
are
a) 4
b) 8
c) 16
d) 32
10) Which of the following pin of 8085 is output pin ?
a) READY
b) HOLD
c) INTR
d) HLDA
11) 8255 operates in _________
a) Simple I/O
b) Strobed I/O
c) Stobed unidirectional I/O
d) All of above
12) EU means
a) Execution unit b) Executed unit c) Execute unit
d) None of above
13) Which of the following is not a level triggered interrupt logic ?
a) TRAP
b) RST 7.5
c) RST 6.5
d) INTR
14) Which of the following is the mode of 8257 ?
a) I/O mapped I/O
b) Memory mapped I/O
c) Fixed priority mode
d) Strobed I/O mode
15) Synchronous Baud rate of 8251 is
a) 64k baud
b) 19.2k baud
c) 68k baud
d) 1.5k baud
16) The process of digitalizing an analog value is called
a) Optimization
b) Digitalization c) Coupling
d) Quantization
17) BIU is
a) Bus Interface Unit
b) Bus Interaction Unit
c) Basic Interface Unit
d) Bidirectional Unit
18) EIA developed standard for transferring the signal and Handshake signal
between DTE and DCE are
a) RS 232C
b) BISYNC Protocol
c) SDLC
d) HDLC
19) For serial communication interface which IC is used
a) 8251
b) 8085
c) 8259
d) 8253
20) How many interrupt levels are handled by 8259 ?
a) 64
b) 8
c) 16
d) None
______________
Set A

*SLRBB215*

-3-

SLR-BB 215

Seat
No.

S.E. (CSE) (Part II) (New) Examination, 2014


MICROPROCESSORS
Day and Date : Thursday, 27-11-2014
Time : 3.00 p.m. to 6.00 p.m.

Marks : 80

SECTION I
2. Attempt any four :

(44=16)

1) Write a note on any four externally initiated signals.


2) Draw the timing diagram of OPCODE fetch cycle.
3) Exchange the contents of H and L with D and E without using XCHG instruction.
4) Write an assembly language program to load the data byte A8H in register C,
mask the higher order bits (D7-D4) and display the low-order bits (D3-D0) at
an output port.
5) Interface 1K8 memory to 8085 using 1K4 memory chips interface it at
address C400H.
3. Attempt any three :

(83=24)

1) Draw the pin diagram of 8085 and list the 8085 features.
2) With an example, explain the following instructions PUSH, POP, XTHL and
SPHL.
3) Interface 6K 8 memory to 8085 using 2K 8 memory chips. Select starting
address 8000H. Give the address range of each chip used.
4) What is T state and machine cycle. Represent the following signals in time
domain-clock signal, single line signal, multiple line signal, multiple lines with
high impedance, single line and single effect signal, multiple line and single
effect signal.
Set A

SLR-BB 215

-4-

*SLRBB215*

SECTION II
4. Attempt any four :

(44=16)

1) Explain memory segmentation in 8086 microprocessor.


2) Explain interrupt logic control.
3) What are the operating modes of 8257 ?
4) What is BSR mode ? Write a program to set bit 4 and bit 1 of port c.
5) Explain DAS (Data Acquisition System) in detail.
5. Attempt any three :

(83=24)

1) What is DAC ? Explain Weighted register DAC.


2) Explain data communication over long distance and explain how the
integrity is maintained.
3) Draw and explain block dig of 8257.
4) Draw and explain Architecture of 80386.
_____________________

Set A

SLR-BB 216

*SLR-BB-216*
Seat
No.

Set

S.E. (CSE) (Part II) Examination, 2014


DATA STRUCTURE (New)
Day and Date : Friday, 28-11-2014
Time : 3.00 p.m. to 6.00 p.m.

Max. Marks : 100

Instructions : 1) Q. No. 1 is compulsory. It should be solved in first 30 minutes in


Answer book Page No. 3. . Each question carries one mark.
2) Answer MCQ/Objective type questions on Page No. 3 only.
Dont forget to mention, Q.P. Set (A/B/C/D) on Top of Page.

MCQ/Objective Type Questions


Duration : 30 Minutes

Marks : 20

1. Tick mark the correct alternative :

(120=20)

1) Stack cannot be used to


a) Allocating resources and scheduling
b) Implementation of recursion
c) Reverse string
d) Postfix expression evaluation
2) The postfix form of A*B+C/D is
a) AB*CD/+
b) A*BCD/+

c) ABCD*/+

d) None

3) A ____________ queue is a queue in which the element next to the last element
is the first element.
a) Dequeue
b) Circular
c) Linked list
d) Heap
4) Ascending priority queue is one is which the item removed is
a) The smallest item
b) The largest item
c) Any item
d) None
5) Direct or random access of elements is not possible in
a) String
b) Array
c) Linked list

d) None of these

6) The situation when in a linked list START=NULL is


a) Saturated
b) Overflow
c) Houseful

d) Underflow

7) If a node having two children is deleted from a binary tree, it is replaced by its
a) Postorder predecessor
b) Inorder successor
c) Preorder predecessor
d) None
8) A binary search tree in which the nodes have been inserted in following order : 25,
15, 35, 10, 20, 30, 40. Which of the following is the result of a postorder traversal of
the original tree.
a) 10 15 20 30 35 40 25
b) 10 20 15 30 40 35 25
c) 10 15 20 25 30 35 40
d) 40 35 30 25 20 15 10
P.T.O.

SLR-BB 216

*SLR-BB-216*

-2-

9) In Red Black Tree root node is always _____________


a) Red
b) Green
c) Black

d) Blue

10) If only left NULL pointers are used as threads then the binary tree is called as
a) left in-threaded binary tree
b) Right in-threaded binary tree
c) Fully threaded binary tree
d) None
11) How does a B Tree grow ?
a) At its root
c) At both root as well as leaves

b) At its leaves
d) None

12) A binary tree in which, all its levels except possibly the last, have the maximum
number of nodes and all the nodes at the last level appear as far left as possible,
is known as
a) Full binary tree
b) 2 tree
c) Almost complete binary tree
d) extended binary tree
13) A complete binary tree of level 6 (considering root at level 1) has how many
nodes ?
a) 15
b) 31
c) 25
d) 63
14) Which of the following statement is false ?
a) a tree with n nodes contains n-1 edges
b) a tree contains a cycle
c) a tree is a connected graph
d) none of above
15) Graph which has self loop, parallel edges or both is called as
a) simple graph
b) regular graph c) multi graph
d) connected graph
16) Data structure used for Breadth First Search Traversal is
a) link list
b) stack
c) queue

d) tree

17) If balance factor of a node in binary search tree is __________, then it is not an
AVL Tree.
a) 1
b) +1
c) 0
d) 2
18) If number nodes in a graph are n, then numbers of edges in a complete directed
graph are
a) n(n1)/2
b) n(n1)
c) n(n2)/2
d) n(n2)
19) If unbalance occurred due to insertion in the right subtree of a left child of a node in
an AVL tree, then to rebalance it ____________ rotation is required.
a) LR
b) RR
c) RL
d) LL
20) _____________ stores all data in the lowest level of the tree.
a) B Tree
b) AVL Tree
c) B+ Tree
d) Red Black Tree

______________

Set A

*SLR-BB-216*

-3-

SLR-BB 216

Seat
No.

S.E. (CSE) (Part II) Examination, 2014


DATA STRUCTURE (New)
Day and Date : Friday, 28-11-2014

Marks : 80

Time : 3.00 p.m. to 6.00 p.m.

SECTION I
2. Answer following questions (any four) :

(45=20)

a) Evaluate following postfix expression 123*+ 4 , show stepwise evaluation using


stack.
b) Write a C function for PUSH ( ) and POP ( ) operations of a stack.
c) What are the applications of the linked list.
d) What is a Binary tree ? Mention the properties of a binary tree.
e) Write a short note on Red Black Tree.
3. Solve any one :
a) What is Threaded Binary Tree ? Explain with example.

10
10

a) Left-in Threaded Binary Tree.


b) Right-in Threaded Binary Tree.
c) Fully Threaded Binary tree.
b) Write a C program for addition of two polynomials
4. Write a C program to implement stack using linked list.

10
10

Set A

SLR-BB 216

-4-

*SLR-BB-216*

SECTION II
5. Solve any four questions :

(45=20)

a) What is M-way search tree. Give one example of M-way search tree of order 4.
b) Discuss B+ Tree with an example. What is difference between B Tree and
B+ Tree.
c) Construct a B Tree of order 5 for following keys with explanation for every step.
78, 21, 14, 11, 97, 85, 74, 63, 45, 42, 57, 20, 16, 19, 52, 30, 21.
d) If following are the Preorder and Inorder traversals, construct a Binary tree with
stepwise explanation.
Preorder F, A, E, K, C, D, H, G, B
Inorder E, A, C, K, F, H, D, B, G
e) What are different ways of representing graph in computer memory ?
6. Define graph. Which data structure is used for Depth First Search traversal ? Explain
the procedure for traversal using this data structure with algorithm and one example.
OR
What is purpose of Dijkstras algorithm ? Explain it with the help of an example.

10

7. What is AVL tree ? Explain different cases of rotations with examples in order to balance
the AVL tree during insertion.
10

_____________________

Set A

SLR-BB 217

*SLRBB217*
Seat
No.

Set

S.E. (Computer Sci. and Engg.) (Part II) (New) Examination, 2014
DATA COMMUNICATION
Day and Date : Saturday, 29-11-2014
Time : 3.00 p.m. to 6.00 p.m.

Total Marks : 100

Instructions : 1) Figures to the right indicate full marks.


2) Q. No. 1 is compulsory. It should be solved in first 30 minutes
in Answer Book Page No. 3. Each question carries one mark.
3) Answer MCQ/Objective type questions on Page No. 3 only.
Dont forget to mention, Q.P. Set (A/B/C/D) on Top of Page.

MCQ/Objective Type Questions


Duration : 30 Minutes

Marks : 20

1. Choose the correct answer :


1. A ________ connection is one in which more than two specific devices share
a single link.
A) Point-to-Point
B) Multi point
C) Both A) and B)
D) None of above
2. In which topology, each device has a dedicated point to point connection only
with two devices on either side of it ?
A) Ring
B) Bus
C) Star
D) None of above
3. An ________ signal has infinitely many levels of intensity over a period of time.
A) Analog
B) Digital
C) Both A) and B) D) None of above
4. _____________ encoding uses an inversion at the middle of each bit interval
for synchronization and bit representation.
A) NRZ
B) NRZ-L
C) Manchester
D) Both A) and B)
5. The rules and conventions used is _____________
A) Layer
B) Token
C) Protocol

D) None of above

6. Which of following service is provided by DLL to network ?


A) Unacknowledged Connectionless
B) Acknowledged Connectionless
C) Acknowledged Connection Oriented D) All above
7. In error-correcting codes, the number of bit positions in which two code words
differ is known as _____________
A) Check bits
B) Hamming distance
C) Redundant bits
D) Both A) and C)
P.T.O.

SLR-BB 217

-2-

*SLRBB217*

8. Protocols in which the sender sends one frame and then waits for
acknowledgment before proceeding are _____________
A) Selective repeat
B) Go back n
C) Stop and wait
D) All
9. Which of the following is main function of transport layer ?
A) Node to node delivery
B) Process to process communication
C) Synchronization
D) None of above
10. An n-bit unit containing data and check bits is called as _____________
A) Word
B) Codeword
C) Checksum
D) Parity
11. _________ standard defines MAC layer for bus network that use token passing
mechanism.
A) 802.3
B) 803.6
C) 802.4
D) 802.5
12. 10Base5 cabling is also called as _____________
A) Thin Ethernet B) Thick Ethernet C) Base Ethernet D) None of above
13. _____________ is static routing algorithm.
A) Distance vector routing
B) Link state routing
C) Flooding
D) None of above
14. Open loop technique attempts to solve problem by good design before occurring
congestion in network.
A) True
B) False
15. In connectionless network packets are called as _____________
A) Datagram
B) Frames
C) Segment
D) Data
16. _____________ IP addresses are called as reserved IP addresses.
A) Class B
B) Class C
C) Class D
D) Class E
17. In load shedding oldest packet is preferred to drop than newer; this is also
called as _____________
A) Wine
B) Milk
C) Water
D) None of above
18. Count-to-infinity problem can occur in
A) Link state routing
B) Distance vector routing
C) Shortest path routing
D) Both A) and B)
19. The distance vector routing algorithm is also called as
A) Static routing algorithm
B) Shortest path algorithm
C) Dijkstras algorithm
D) Bellman-Ford routing algorithm
20. According to ____________ if router J is on the minimum path from router I to
router K, then the minimum path from J to K also falls along the same route.
A) Bellman-Ford principle
B) Ford-Fulkerson principle
C) Optimality principle
D) None of above
______________
Set A

SLRBB217

-3-

SLR-BB 217

Seat
No.

S.E. (Computer Sci. and Engg.) (Part II) (New) Examination, 2014
DATA COMMUNICATION
Day and Date : Saturday, 29-11-2014
Time : 3.00 p.m. to 6.00 p.m.

Marks : 80

Instructions : 1) All questions are compulsory.


2) Figures to the right indicate full marks.

SECTION I

2. Attempt any four (Each carries 5 marks) :

20

A) Describe the uses of computer network.


B) Explain a simplex stop and wait protocol.
C) List different framing methods. Explain one of the framing method.
D) Describe hamming code with example.
E) Explain TCP/IP protocol.
F) Explain GO-Back-N protocol.
3. Describe design issues of data link layer in detail.

10

OR
Explain attenuation and delay distortion.
4. Explain ATM model in detail.

10
10

Set A

SLR-BB 217

-4-

*SLRBB217*

SECTION II

5. Attempt any four (Each carries 5 marks) :

20

A) Explain ALOHA protocol and its types.


B) Draw and explain IEEE 802.3 packet format.
C) Describe desirable properties of routing algorithm.
D) Write a note on load shedding.
E) Write a note on NAT.
F) Describe IPv4 addresses and different classes of IP addresses.
6. Explain leaky bucket and token bucket algorithm.

10

OR
Explain collision free protocols.
7. Explain distance vector routing. What is count to infinity problem ?
How it is solved ?

10

_________________

Set A

SLR-BB 218

*SLRBB218*
Seat
No.

Set

T.E. (Part I) (CSE) Examination, 2014


COMPUTER NETWORK II (Old)
Day and Date : Wednesday, 3-12-2014
Time : 3.00 p.m. to 6.00 p.m.

Max. Marks : 100

Instructions : 1) Q. No. 1 is compulsory. It should be solved in first 30


minutes in Answer Book Page No. 3. Each question carries
one mark.
2) Answer MCQ/Objective type questions on Page No. 3 only.
Dont forget to mention, Q.P. Set (A/B/C/D) on Top of Page.
MCQ/Objective Type Questions
Duration : 30 Minutes

Marks : 20

1. 1) Which of the following functions does UDP perform ?


A) Process-to-process communication
B) Host-to-process communication
C) Host-to-host communication
D) End-to-end reliable data delivery

20

2) TCP unlike UDP is a ___________ protocol.


A) Byte oriented
B) Bit oriented
C) Stream oriented
D) Both A) and C)
3) Connection establishment in TCP is called ___________
A) Two way handshaking
B) Three way handshaking
C) Both A) and B)
D) None of these
4) TELNET stands for ___________
A) TELephone NETwork
B) TELemart NETwork
D) TELex NETwork
C) TErminal NETwork
5) The ports ranging from 1,024 to 49,151 are called ___________
A) Dynamic
B) Static
C) Well-known
D) Registered
6) In the ___________ algorithm the size of the congestion window increases
exponentially until it reaches a threshold.
A) Congestion avoidance
B) Congestion detection
C) Slow start
D) None of the choices are correct
7) If DHCP client and server are on different networks, there is a need for an
intermediary called a ___________
A) Second client B) Primary server C) Relay agent D) None of these
P.T.O.

SLR-BB 218

-2-

*SLRBB218*

8) NVT uses two sets of characters, one for _________ and one for __________
A) Sending; receiving
B) Request; reply
C) Data; control
D) None of these
9) After sending the DHCPDISCOVER message, the client goes to the
__________ state.
A) selecting
B) init
C) requesting
D) none of these
10) In the DNS, the names are defined in __________ structure.
A) A linear list
B) An inverted tree
C) A graph
D) None of these
11) For control congestion, a FTP uses the __________ character set.
A) Regular ASCII B) EBCDIC
C) NVT ASCII
D) None of these
12) __________ is more powerful and complex than __________
A) POP3; IMAP4
B) IMAP4; POP3
C) SMTP; POP3
D) None of these
13) When sender is connected to mail server via a LAN or WAN we need only
__________
A) Two MTA
B) Two UAs and two pairs of MTAs
C) Two UAs and a pair of MTAs
D) None of these
14) A special segment called a probe is sent by a sending TCP when the
__________ timer goes off.
A) Transmission B) Persistence C) Keep alive
D) None of these
15) Information in a computer is stored in __________ byte order.
A) Host
B) Network
C) Server
D) None of these
16) An interface is a set of __________ designed to facilitate interaction between
two entities.
A) Programs
B) Instructions C) Rules
D) None of these
17) In TFTP if ACK message for a packet is not lost but delayed, then every
succeeding block is sent twice and every acknowledgement is received twice.
This is called
A) Bed bug
B) Sorcerers apprentice bug
C) Interactive bug
D) All of these
18) The beginning tag has an important attribute __________ which defines the
address (URL) of the linked document.
A) SRC
B) ALIGN
C) HREF
D) None of these
19) A __________ consists of a controller, client programs and interpreters.
A) Server
B) Applet
C) Browser
D) All of these
20) MIME stands for __________
A) Multiprogram International Mail Extensions
B) Multitasking International Mail Extensions
C) Multipurpose Internet Magical Extensions
D) Multipurpose Internet Mail Extensions
______________
Set A

*SLRBB218*

-3-

SLR-BB 218

Seat
No.

T.E. (Part I) (CSE) Examination, 2014


COMPUTER NETWORK II (Old)
Day and Date : Wednesday, 3-12-2014
Time : 3.00 p.m. to 6.00 p.m.

Marks : 80

Instructions : 1) All questions are compulsory.


2) Figures to right indicate full marks.
SECTION I
2. Attempt any four (each carries 5 marks) :

20

A) Explain different UDP operations.


B) Explain BOOTP. What are the operations of BOOTP ?
C) List different timers used in TCP. Explain retransmission timer.
D) Explain the concept of connection oriented concurrent server.
E) Describe the following briefly.
i) Bind
ii) Accept
iii) Listen
iv) Connect.
3. With suitable diagram explain UDP client-server program.

10

OR
Draw TCP segment format. Explain each field in detail.
4. Explain connection oriented concurrent server in detail.

10

SECTION II
5. Attempt any four (each carries 5 marks) :

20

A) What is name space ? Explain Hierarchical name space.


B) Explain different options provided by TELNET.
C) Describe the concept of TFTP.
Set A

SLR-BB 218

-4-

*SLRBB218*

D) Describe the following briefly.


i) Label
ii) Domain name
iii) Domain
iv) Zone
v) Root server
E) What are cookies ? Explain their use.
6. Attempt any one :

10

A) Explain the following file systems in detail.


i) FAT-16
ii) FAT-32
iii) NTFS
B) With help of neat diagrams explain architecture of E-mail.
7. Describe in detail the Hypertext Transfer Protocol (HTTP).

10

_____________________

Set A

SLR-BB 219

*SLRBB219*
Seat
No.

Set

T.E. (CSE) (Part I) Examination, 2014


COMPUTER ORGANIZATION (Old)
Day and Date : Friday, 5-12-2014
Time : 3.00 p.m. to 6.00 p.m.

Max. Marks : 100

Instructions : 1) Q. No. 1 is compulsory. It should be solved in first 30 minutes


in Answer book Page No. 3. Each question carries one mark.
2) Answer MCQ/Objective type questions on Page No. 3 only.
Dont forget to mention, Q.P. Set (A/B/C/D) on Top of Page.
MCQ/Objective Type Questions
Duration : 30 Minutes

Marks : 20

1. Objective questions. Each question carry one mark.

20

1) Von Neumann was introduced new computer for its precedecessors stored and
processed numbers in binary or base 2 form called as
a) ENIAC
b) EDVAC
c) IBM
d) IAS
2) The most of the RISC processors use ___________ instructions.
a) 16 bit
b) 32 bit
c) 64 bit
d) 8 bit
3) Integrated circuits and operating system were developed in
a) First generation
b) Second generation
c) Third generation
d) Fourth generation
4) Fixed length block that can be assigned to fixed regions is called as
a) Segments
b) Pages
c) Virtual address
d) None of these
5) The average time required to reach a storage location in memory and obtain its
contents
a) Latency time
b) Response time
c) Access time
d) Turn around time
6) MOV CX, 5[BX] is a
a) One address machine
c) Relative based addressing

b) Two address machine


d) Register indirect addressing

7) The offset for a memory operand is called the operands


a) Offset address
b) Memory address
c) Effective address
d) Effective offset address
8) In full adder sum and carry output of each stage cannot be produced until the input
carry occurs this leads to a time delay in the addition process known as
a) Gate delay
b) Gate propagation delay
c) Carry delay
d) Carry propagation delay
P.T.O.

SLR-BB 219

*SLRBB219*

-2-

9) What is the control units function in the CPU ?


a) To transfer data to primary storage
b) To store program instruction
c) To perform logic operation
d) To decode program instruction
10) Memory Access in RISC architecture is limited to instructions ?
a) CALL and RET
b) PUSH and POP
c) STA and LDA
d) MOV and JMP
11) The goal of replacement policy is
a) To maximize the hit ratio
b) To minimize the number of times a referenced block is not in memory M
c) Both a) and b)
d) None
12) Which replacement policy selects the block which is entered first ?
a) FIFO
b) LRU
c) OPT
d) Stack

13) Let N1 and N2 denote the number of references to M1 and M2 respectively, in


the block address stream. Them the block hit ratio H* is defined by
a) H =

N1
N1 + M2

b) H =

N1 + N2
N1

c) H =

N1 N2
N1

d) H =

N1 + N2
N2

14) The major variables on which H(hit) depends are


a) The address stream
b) The average block size
c) Capacity of memory
d) All
15) Vector processor supports for manipulations of
a) One data item
b) Two data item
c) Many data item
d) None
16) Example of loosely coupled multiprocessor architecture is
a) Cm*
b) Cyber 170
c) Both a) and b)

d) Honeywell 60/66

17) PMIN connects


a) Processor to Input Output Channels b) Processor to Memory Modules
c) Memory to Input Output Channels
d) None
18) PMIN supports for
a) Single stage network
c) Multistage network

b) Two stage network


d) None

19) Hazards _________ the performance from the ideal speed-up gained by pipelining.
a) Reduces
b) Increases
c) Do not change d) None
20) Classification of hazards are done depending on the order of _________ accesses
in the instructions.
a) Read only
b) Write only
c) Read and write d) None
______________
Set A

*SLRBB219*

-3-

SLR-BB 219

Seat
No.

T.E. (CSE) (Part I) Examination, 2014


COMPUTER ORGANIZATION (Old)
Day and Date : Friday, 5-12-2014
Time : 3.00 p.m. to 6.00 p.m.

Marks : 80

SECTION I
2. Attempt any four :

(45=20)

1) Perform the operation of y = a + b c using one and two address machine


instruction.
2) Explain with example Babbages difference engine.
3) Draw and explain control unit organisation.
4) Perform (10)10 (4)10 multiplication using Booths algorithm.
5) Explain the concept of paging.
6) Explain advantages of RISC processor over CISC processor.
3. A) Explain state table method of 2s complement multiplier in detail.
B) Design carry look ahead adder for

10
10

i) A 32 bit adder composed of 8 bit adders


ii) A 16 bit adder composed of 4 bit adders.
OR
B) Suppose there are three available regions and available space list as follows :
Region address

Size (words)

50

300

400

800

200

If two new blocks, whose sizes are 100 and 250 words respectively then show the
memory allocation for new blocks by first fit and best fit method and also state
advantages of each method.
10
Set A

SLR-BB 219

-4-

*SLRBB219*

SECTION II
4. Attempt any four :

(45=20)

1) What is the goal of every memory system ? How the different memory types varies
in access time with respective to cost ?
2) How the speed of memory can be increased ?
3) What are the different component of computer model ? How non hierarchical loosely
coupled multiprocessor system works ?
4) What are the different types of interconnection networks ? How the communication
between processor and memory takes place through PMIN ?
5) What is hazard ? Classify the different hazards.
6) Explain the role of bubbles in pipeline.
5. A) Consider a paging system in which memory M has the capacity of five pages. The page
address stream formed is

Show how these pages are assigned to memory M using stack replacement policy.
How many lists were occurred ?
10
B) With neat diagram, explain the cache execution of read operation.

10

OR
Differentiate linear pipeline and non linear pipeline with neat diagram.

10

_____________________

Set A

SLR-BB 22

*SLRBB22*
Seat
No.

Set

S.E. (Civil) (Part I) Examination, 2014


BUILDING CONSTRUCTION AND DRAWING
Day and Date : Tuesday, 16-12-2014
Time : 10.00 a.m. to 2.00 p.m.

Total Marks : 100

Instructions : 1) Figures to the right indicate full marks.


2) Assume suitable data wherever necessary and mention it
clearly.
3) Q. No. 1 is compulsory. It should be solved in first 30
minutes in Answer Book Page No. 3. Each question carries
one mark.
4) Answer MCQ/Objective type questions on Page No. 3
only. Dont forget to mention, Q.P. Set (A/B/C/D) on Top
of Page.
MCQ/Objective Type Questions
Duration : 30 Minutes

Marks : 20

1. State whether following statement is correct or incorrect in the given :

20

1) Horn is extension of post of frame below finished floor level.


2) Baluster is element of staircase.
3) IPS can be top layer of B. B. Coba.
4) M M Tiles are factory manufactured vitrified tiles used for flooring.
5) Double bull nose bricks are used for coping of parapet wall.
6) Length of through stone should be equal to thickness of wall.
7) PCC is provided below footing as a leveling course.
8) Foundation distributes the load of the super structure to the smallest area.
9) A good quality brick should have minimum crushing strength of 3.5 N/mm2.
10) Basalt is the best metamorphic rock for CRS.
P.T.O.

SLR-BB 22

-2-

*SLRBB22*

11) ISMB500 means Indain standard medium beam having depth 500 m.
12) A well planned building gives maximum comfort and convenience to the
occupants.
13) Spraying of pesticides and insecticide are essential to control the termit.
14) Circular arch, semicircular arch, segmental arch, horse-shoe arch have of
two centers.
15) P.V.C. tiles are more durable than flagstone.
16) Red colour to the brick is due to presence of Iron oxide.
17) Minimum thickness of single flemish bond is 1 brick thick.
18) Isolated footing is type of open foundation.
19) Width of landing should be equal to height of head room.
20) Pitched roofs are suitable for buildings in hot regions where rainfall is moderate.

______________

Set A

*SLRBB22*

-3-

SLR-BB 22

Seat
No.

S.E. (Civil) (Part I) Examination, 2014


BUILDING CONSTRUCTION AND DRAWING
Day and Date : Tuesday, 16-12-2014
Time : 10.00 a.m. to 2.00 p.m.

Marks : 80

Instructions : 1) All questions are compulsory.


2) Section I to be written in answer book.
3) Section II to be drawn on half imperial drawing sheet.
Use both sides of the sheet.
4) Figures to the right indicate full marks.
5) Assume suitable data wherever necessary and mention it
clearly.
6) Retain all projection/construction lines on drawing sheet.
SECTION I
2. Attempt any five questions :

(85=40)

a) Draw neat and proportionate sketch isometric view of Full brick, Queen closer,
King closer, Beveled bat.
b) Draw a neat sketch for fixing CGI sheet to angle purlin also draw details of
bolts.
c) Draw neat labeled sketch of four paneled framed door for clear opening of
1000 2100 mm assume and mention wood sections to be used (Use answer
sheet).
d) Compare load bearing structure with framed structure.
e) What are the requirements of good building stones ?
f) Explain with sketch Brick Bat Coba for terrace water-proofing.
g) Define foundation. Draw and write in brief about any three types of spread
footings.
h) Explain with sketch requirements of good staircase.
Set A

SLR-BB 22

*SLRBB22*

-4-

SECTION II
3. A) Draw to a scale of 1 : 10, detailed section, front elevation and sectional
elevation for fully paneled T. W. window. Use following
data (All dimensions are in mm)

20

a) Clear operating = 10001200


b) Wood section for frame = 10060
c) Vertical styles = 100 35
d) Top rails = 100 35
e) Bottom rail = 100 35
f) Panels = 2 numbers, 25 mm thick, raised type
g) Show various fixtures at proper location.
OR
Draw to scale 1 : 10 L shaped portion of brick masonry of thickness 1 brick
and 11/2 brick with English bond. Use nominal size of brick. Draw elevation
and plan for 1, 3, 5 ... and 2, 4, 6, courses.
B) Design and draw to scale 1 : 30, plan and vertical section for Quarter turn
R. C. C. staircase for a mezzanine floor of a shop. Use following data and
dimension.

20

a) Height to be climbed = 2400 mm


b) Width of flight = 1000 mm
c) Railing 80 mm diameter S. S. pipe railing
d) Reinforcement details are not necessary
e) One winder is provided at landing
(Write step by step calculation on sheet with pencil only).
OR
Draw to scale 1 : 10 :
a) Vertical section

b) Front elevation

For solid core flush door of overall size of 1000 mm 2100 mm.
Use following data :
1) Wood section for frame 75 mm 120 mm
2) Shutter : solid core factory manufactured flush door shutter 35 mm thick
with 4 mm face veneer on both sides.
Set A
_____________________

SLR-BB 220

*SLRBB220*
Seat
No.

Set

T.E. (CSE) (Part I) (Old) Examination, 2014


SYSTEM PROGRAMMING
Day and Date : Monday, 8-12-2014
Time : 3.00 p.m. to 6.00 p.m.

Max. Marks : 100

Instructions : 1) Q. No. 1 is compulsory. It should be solved in first 30


minutes in Answer Book Page No. 3. Each question carries
one mark.
2) Answer MCQ/Objective type questions on Page No. 3
only. Dont forget to mention, Q.P. Set (A/B/C/D) on Top of
Page.
MCQ/Objective Type Questions
Duration : 30 Minutes

Marks : 20

1. Choose the correct option :

20

1) Program generator introduces a new domain between


a) Application domain and Execution domain
b) Application domain and Target PL domain
c) Application domain and PL domain
d) None of these
2) A ______________ is a software which bridges a specification or execution
gap.
a) Semantic gap
b) Linker
c) Language Processor
d) Loader
3) All errors are reported against the erroneous statement itself, if error reports are
produced in ______________
a) pass I
b) pass II
c) a) and b) both
d) not possible in both
4) Which analysis builds a descriptor, called a token ?
a) Lexical
b) Syntax
c) Semantic

d) None of these

5) Input specification of LEX ______________


a) Lexical units
b) Specification of semantic actions
c) Both a) and b)
d) None of these
6) ______________ are context sensitive grammars.
a) Type 0
b) Type 1
c) Type 2

d) Type 3

7) A ______________ is an operand with the syntax = <value>.


a) Code
b) Literal
c) Constant
d) Assembler
P.T.O.

SLR-BB 220

*SLRBB220*

-2-

8) Symbolic references in source statement are not processed at all during


Pass I in
a) Variant I
b) Variant II
c) Variant III
d) All above
9) <macro name> appears in the mnemonic field of
a) Model statements
b) Prototype statement
c) Assembly statement
d) Preprocessor statement
10) Which of the following is/are assembler directive(S) ?
a) STOP
b) READ
c) EQU
d) None of the above
11) To compile a reference to an element of a data structure, the compiler must develop
a) Memory mapping
b) Address resolution technique
c) Address calculation technique
d) None of above
12) The control structure of a language consists of
a) Conditional transfer of control
b) Conditional execution
c) Iteration control and procedure calls
d) All of above
13) In parsing action T4*T5 T6, the superscript of an NT shows
a) Register descriptor
b) Operand descriptor
c) Both a) and b)
d) None of above
14) Which is not a PL feature that contributes to semantic gap between PL domain
and execution domain ?
a) Control structures
b) Scope rules
c) Data types
d) IC generation
15) Which is not a component of object module of program ?
a) LINKTAB
b) Header
c) Program
16) Overlay structures are also called as
a) Absolute loading
c) Relocation loading

d) NTAB

b) Dynamic loading
d) Direct linking loading

17) Resolution of externally defined symbols is performed by


a) Linker
b) Loader
c) Compiler

d) Assembler

18) DLL uses ______________ to perform linking of external references.


a) Relocation bits
b) Transfer vector
c) Flipper
d) None of above
19) Which among the following is not valid database for loaders ?
a) GEST
b) IPLA
c) LESA
d) NMT
20) In loaders, location and length of each address constant is maintained by
a) ESD
b) TXT
c) RLD
d) END
______________

Set A

*SLRBB220*

-3-

SLR-BB 220

Seat
No.

T.E. (CSE) (Part I) (Old) Examination, 2014


SYSTEM PROGRAMMING
Day and Date : Monday, 8-12-2014
Time : 3.00 p.m. to 6.00 p.m.

Marks : 80

SECTION I
2. Attempt any four :

20

a) Explain language processor development tool.


b) Define IR. Prepare an IR by making frond end analysis of following program
P, N:Integer;
I, R:real
I=P*N*R/100;
c) Explain pass structure of assemblers.
d) List and explain different types of assembly language statements with example.
e) Describe analytic and synthetic operators.
f) Explain macro definition and call. Write macro expansion algorithm.
3. Give four step approach to develop a design specification for an assembler.
Explain synthesis phase and analysis phase of an assembler.

10

OR
Explain in detail the language processing activities.
4. Explain following assembler directives with example :

10

I) EQU and PURGE


II) SEGMENT, ENDS and ASSUME
III) PROC, ENDP, NEAR and FAR
IV) PUBLIC and EXTRN.

Set A

SLR-BB 220

-4-

*SLRBB220*

SECTION II
5. Attempt any four :

20

a) Explain different intermediate code forms for expressions used by compiler.


b) Explain how availability of expression is checked in data flow analysis ?
c) Describe object module of a program with example.
d) What is overlay structured program ? Explain linking of overlays with example.
e) Explain general loading scheme. Give advantages and disadvantages of absolute
loader.
f) Write steps followed in designing a direct linking loader. Explain problem
specification in detail.
6. Describe BSS loader with 360 RX instructions format. Explain how relocation bits
and transfer vector are used by BSS loader with example.

10

OR
Define local optimization and explain using value numbers.
7. List and explain pass I algorithm for direct linking loader with example. Draw flowchart
for same.
10
_____________________

Set A

SLR-BB 221

*SLRBB221*
S

T.E. (Computer Sci. and Engg.) (Part I) (Old) Examination, 2014


OPERATING SYSTEM I
Day and Date : Wednesday, 10-12-2014
Time : 3.00 p.m. to 6.00 p.m.

Max. Marks : 100

Instructions : 1) Q. No. 1 is compulsory. It should be solved in first 30


minutes in Answer Book Page No. 3. Each question carries
one mark.
2) Answer MCQ/Objective type questions on Page No. 3
only. Dont forget to mention, Q.P. Set (A/B/C/D) on Top
of Page.
Duration : 30 Minutes

MCQ/Objective Type Questions

Marks : 20

1. Choose correct alternatives.


20
1) Systems that control scientific experiments, medical imaging and industrial
control systems are the examples of ___________
A) Parallel
B) Distributed
C) Real-time
D) None of above
2) Systems Calls provides the interface between __________ and __________
A) Hardware, Software
B) Process, OS
C) Hardware, OS
D) None of above
3) Process Control Block (PCB) consist of which of the following field ?
A) Process State
B) CPU registers
C) Program Counter
D) All above
4) The __________ scheduler is also known as Job Scheduler.
A) Short-term
B) Long-term
C) Medium-term D) None of above
5) The advantage of allowing cooperating processes is __________
A) Information sharing
B) Computation Speedup
C) Modularity
D) All above
6) ____________ overlaps the I/O of one job with computation of another job.
A) Scheduling
B) Context Switch
C) Spooling
D) All above
7) In preemptive scheduling, once the CPU has been allocated to the process,
the process keeps the CPU until it releases.
A) True
B) False
C) Cant say
D) None of above
8) ___________ time is the sum of period spends in the ready queue.
A) Response
B) CPU utilization
C) Throughput
D) Waiting
P.T.O.

SLR-BB 221

*SLRBB221*

-2-

9) A major problem with ___________ scheduling algorithm is indefinite blocking


or starvation.
A) FCFS
B) SJF
C) Priority
D) None of above
10) In RR scheduling, if time quantum is very small then it is called as _________
A) SJF
B) Preemptive
C) FCFS
D) Processor sharing
11) If process Pi is executing in its critical section, then no other processes can
be executing in their critical section is ___________ condition.
A) Critical Section
B) Progress
C) Mutual exclusion
D) Bounded waiting
12) A semaphore uses two standard operations wait and signal which must be
___________
A) Dependent
B) In dependent C) Atomic
D) None of above
13) The ___________ readers writers problem requires that, once a writer is
ready, that writer performs its write as soon as possible.
A) First
B) Second
C) Third
D) None of above
14) Which is not the necessary condition for a deadlock situation ?
A) Mutual exclusion
B) Hold and wait
C) Progress
D) No preemption
15) In Resource Allocation Graph, a directed edge from Rj
Pi is called _________
A) Assignment Edge
B) Request Edge
C) Both A) and B)
D) None of above
16) Bankers Algorithm is use by ____________ method.
A) Deadlock Avoidance
B) Deadlock Recovery
C) Deadlock Detection
D) Deadlock Prevention


17) Addresses seen by MMU and is loaded into the memory address register are
commonly referred as ______________ addresses.
A) Physical
B) Logical
C) Virtual
D) None of above
18) In memory allocation, allocate the smallest hole that is big enough among all
the holes is __________
A) Best fit
B) Worst fit
C) First fit
D) None of above
19) The percentage of times that a page is found in the associative registers is
called ___________
A) hit ratio
B) hit miss
C) both A) and B) D) hit access time
20) A process is thrashing if it is spending more time in paging than executing
A) True
B) False
C) Cant say
D) None of above
______________
Set A

*SLRBB221*
S

-3-

SLR-BB 221

T.E. (Computer Sci. and Engg.) (Part I) (Old) Examination, 2014


OPERATING SYSTEM I
Day and Date : Wednesday, 10-12-2014
Time : 3.00 p.m. to 6.00 p.m.

Marks : 80

Instruction : All questions are compulsory.


SECTION I
2. Attempt any four (each carries 5 marks) :

20

A) Describe time-sharing system.


B) Explain real-time system.
C) Explain the functions of medium-term scheduler.
D) What is co-operating process ? Explain with example.
E) List and explain different scheduling criteria.
F) Describe counting and binary semaphores.
3. Describe readers-writers problem and provide solution for same using
semaphores.

10
OR

What is process ? Describe with diagram different states of process and PCB.
4. Describe following scheduling algorithms with the help of examples :

10
10

i) SJF
ii) Priority.
Set A

SLR-BB 221

-4-

*SLRBB221*

SECTION II
5. Attempt any four (each carries 5 marks) :

20

A) Describe how to avoid circular wait condition in deadlock prevention method.


B) Explain meaning of safe and unsafe states.
C) Explain Logical Vs Physical address space.
D) Draw and explain paging hardware.
E) Describe concept of Thrashing.
6. Explain Deadlock detection methods in detail.

10

OR
What is segmentation ? Explain basic method, hardware and example of
segmentation.

10

7. Describe following page replacement algorithms with the help of example :

10

i) FIFO
ii) Optimal
iii) LRU.
_____________________

Set A

SLR-BB 222

*SLRBB222*
Seat
No.

Set

T.E. (CSE) (Part I) Examination, 2014


DESIGN AND ANALYSIS OF ALGORITHM (Old)
Day and Date : Friday, 12-12-2014
Time : 3.00 p.m. to 6.00 p.m.

Max. Marks : 100

Instructions : 1) Q. No. 1 is compulsory. It should be solved in first 30 minutes


in Answer Book Page No. 3. Each question carries one mark.
2) Answer MCQ/Objective type questions on Page No. 3 only.
Dont forget to mention, Q.P. Set (A/B/C/D) on Top of Page.
MCQ/Objective Type Questions
Duration : 30 Minutes

Marks : 20

1. Choose the correct answer :


1) In average case, Quicksort is
a) 0 (1)
b) 0 (log n)

20
c) 0 (n logn)

d) None of the above

2) A recursive case running time of (n + T (n + 2)) indicate___________time.


a) Constant
b) nlogn
c) Linear
d) None of the above
3) A simple approach, that is efficient when it works, but does not always work, is
a) Divide and conquer
b) Greedy algorithm
c) Brute force
d) Dynamic programming
4) Building a spanning tree by finding at each step the minimal edge adjacent to a
partial subtree is an instance of___________algorithm.
a) Brute force
b) Divide and conquer
c) Greedy algorithm
d) Dynamic Pg
5) All problem with greedy solution have
a) Recursive structure
c) The optimal substructure properly

b) Many local optima


d) None of the above

6) Huffman algorithm is
a) Divide and conquer
c) Brute force

b) Greedy algorithm
d) Dynamic programming

7) Algorithm A is said to be _____________ recursive if it call another algorithm


which in turn calls A.
a) Indirect
b) Direct
c) Pointed
d) Related
8) Once an algorithm devised, it is necessary to that it computes correct answer for
all possible legal input is
a) Algorithm validation
b) Program proving
c) Program verification
d) Profiling
P.T.O.

SLR-BB 222

*SLRBB222*

-2-

9) A procedure whose flow of control is clear but whose primary operations are
specified by other procedure is
a) Algorithm
b) Function
c) Flowchart
d) Control abstraction
10) In unsuccessful binary search, the best case complexity is
a) (logn)
b) 0 (logn)
c) (logn)
d) (n)
11) To determine shortest path from each vertex to all other vertices is known as
a) All-pair shortest-path
b) Single-source shortest-path
c) Dijkstras algorithm
d) Multistage shortest-path
12) Cost function for binary search tree for successful search from the internal node ai
a) p(i) * level (Ei)
b) p(i) * level (ai)
c) p(i) * (level (Ei) 1)
d) p(i) * (level (ai) 1)
13) The Hamiltonian cycle is used for
a) Spanning tree
c) Traveling sales person problem

b) Minimum cost spanning tree


d) None of these

14) In traveling sales person problem the path from vertex K to vertex 1 goes through
each vertex.
a) Exactly once
b) Exactly twice
c) More than twice
d) None of these
15) When the search necessarily involves the examination of every vertex in the object
being searched, is called a
a) Search
b) Traversal
c) BFS
d) DFS
16) The root node of a depth first spanning tree is an articulation point iff it has
a) At least two children
b) Exactly two children
c) At least one children
d) Exactly one children
17) Which one of the following design techniques is used in finding all pairs shortest
path in graph ?
a) Back tracking
b) Dynamic programming
c) Greedy method
d) Divide and conquer
18) ____________, are used to kill live nodes without generating all their children.
a) Backtracking
b) Dead node
c) Bounding function
d) None of these
19) ____________are those problem states S for which the path from the root to S
defines a tuple in the solution space.
a) Answer states
b) Solution states
c) State space tree
d) State space
20) The tree organization of the solution space is referred to as the
a) Static tree
b) Dynamic tree
c) State space tree
d) Space tree

______________
Set A

*SLRBB222*

-3-

SLR-BB 222

Seat
No.

T.E. (CSE) (Part I) Examination, 2014


DESIGN AND ANALYSIS OF ALGORITHM (Old)
Day and Date : Friday, 12-12-2014
Time : 3.00 p.m. to 6.00 p.m.

Marks : 80

Instructions : 1) All questions are compulsory.


2) Figures to right indicate full marks.
3) Assume suitable data if necessary.
SECTION I
2. Solve any four :

(54=20)

1) Explain performance analysis.


2) Define asymptotic notations.
3) Explain divide and conquer with e.g.
4) Explain optimal storage on tape.
5) Explain recurrence relation.
3. Sort the following element using quicksort 5,4,3,2,5,8,9.

10

OR
3. Compare Prims and Kruskals algorithm. Find minimum cost spanning tree using
Kruskals method.

10

4. Find the complexity of Merge sort, Binary search.

10

Set A

SLR-BB 222

-4-

*SLRBB222*

SECTION II
5. Attempt any four :

(54=20)

a) Generate the sets Si, 0 i 4, when (W1, W2, W3 W4) = (10, 15, 6, 9) and
(P1, P2 , P3 , P4) = (2, 5, 8, 1).
b) Explain flow shop scheduling.
c) Explain following terms with example :
i) Articulation point
ii) Biconnected component

iii) DFN

d) Explain techniques for graph with example.


e) Write an algorithm for N-queen problem.
f) Draw the binary tree for which preorder traversal sequence is ABDFGHIEC and
inorder traversal sequence is FDHGIBEAC.
6. a) Write an algorithm for sum of subset problem. Apply an algorithm for n = 6,
m = 30, W [1 : 6] = 5, 10, 12, 13, 15, 18. Draw the portion of state space tree
that is generated.
10
OR
b) Let n = 4 and (a1, a2, a3, a4) = (do, if, int,, while). Let P (1:4) = (3, 3, 1, 1) and
q (0 : 4) = (2, 3, 1, 1, 1). Using r (i, j)s construct optimal binary search tree.
7. Write an algorithm for multistage graph corresponding to the forward approach
and apply the same on following graph to find minimum cost path from s to t.

_____________________

10
10

Set A

SLR-BB 223

*SLRBB223*
Seat
No.

Set

T.E. (CSE) (New) (Part I) Examination, 2014


COMPUTER NETWORKS
Day and Date : Wednesday, 3-12-2014
Time : 3.00 p.m. to 6.00 p.m.

Max. Marks : 100

Instructions : 1) Figures to the right indicate full marks.


2) Q. No. 1 is compulsory. It should be solved in first 30 minutes
in Answer Book Page No. 3. Each question carries one mark.
3) Answer MCQ/Objective type questions on Page No. 3 only.
Dont forget to mention, Q.P. Set (A/B/C/D) on Top of Page.
MCQ/Objective Type Questions
Duration : 30 Minutes
1. Choose the correct answer :

Marks : 20
(120=20)

1) In the _________ algorithm the size of the congestion window increases


exponentially until it reaches a threshold value.
a) Congestion avoidance
b) Congestion detection
c) Slow start
d) None of these
2) A special segment called the probe is sent by the sending TCP when the
_______ timer goes off.
a) Transmission b) Persistence c) Keep alive
d) None of these
3) The ports ranging from 1024 to 49151 are called the _________ ports.
a) Well known
b) Registered
c) Dynamic
d) None of these
4) As the data packet moves from the upper to the lower layers, headers are
__________
a) Added
b) Removed
c) Rearranged
d) Modified
5) To deliver a message to the correct application program running on a host,
the __________ address must be consulted.
a) Port
b) IP
c) Physical
d) None of the above
6) ___________ provides full transport layer services to applications.
a) TCP
b) UDP
c) ARP
d) None of these
7) The ___________ address uniquely defines a host on the internet.
a) Physical
b) IP
c) Port
d) Specific
P.T.O.

SLR-BB 223

-2-

*SLRBB223*

8) The ___________ address identifies the process on the host.


a) Physical
b) IP
c) Port
d) Specific
9) Mail services are available to network users through the __________ layer.
a) Data link
b) Physical
c) Transport
d) Application
10) The layer responsible for moving frames from one hop (node) to the next is
a) Physical
b) Data link
c) Transport
d) None of the above
11) In TFTP block of less than __________ signals the end of file.
a) 256 bytes
b) 512 bytes
c) 512 bits
d) 256 bits
12) DNS can use the services of UDP or TCP, using the well-known port ______
a) 53
b) 35
c) 27
d) 63
13) If label ends with null string it is called as __________
a) FQDN
b) PQDN
c) SQDN

d) None of above

14) _________ name spaces cannot be used for large systems like internet etc.
a) Hierarchical
b) Flat
c) Both a) and b) d) None of these
15) The role of _________ is to compose, read, reply, forwards mails and manages
mailboxes.
a) UA
b) MTA
c) MAA
d) Mail gateway
16) In internet, the address consists of two parts : a local part and _________
a) Receiver name b) Sender name c) Domain name d) All
17) POP3 has two modes : the delete mode and _________ mode.
a) Alive
b) Create
c) Keep
d) Remove
18) _____________ is a small piece of text stored as a computer by web browser.
a) URL
b) Path
c) Cookies
d) Crawling
19) POP3 uses _________ port number for communication.
a) 101
b) 80
c) 53
d) 110
20) To distinguish between data and control character, each sequence of character
is preceded by special control character called as ____________
a) IC
b) IAC
c) ICC
d) NVT
______________

Set A

*SLRBB223*

-3-

SLR-BB 223

Seat
No.

T.E. (CSE) (New) (Part I) Examination, 2014


COMPUTER NETWORKS
Day and Date : Wednesday, 3-12-2014

Marks : 80

Time : 3.00 p.m. to 6.00 p.m.


Instructions : 1) All questions are compulsory.
2) Figures to the right indicate full marks.
SECTION I
2. Attempt any four of the following :

(45=20)

a) Explain the TCP Timers in detail.


b) Explain the OSI reference model with a neat sketch.
c) Explain the use of sequence numbers in error control.
d) Provide a list of well known ports used with UDP and give their description.
e) Write a note on SCTP Association.
f) Explain bind and connect system call.
3. Attempt any one of the following :

(110=10)

a) What is UDP ? Discuss in detail the UDP feature, their advantages and
disadvantages.
b) Draw TCP segment format. Explain each field in detail.
4. Attempt any one of the following :

(110=10)

a) Explain the services and functions of Application and Transport Layer.


b) Explain the TCP with a neat transition diagram in detail.
Set A

SLR-BB 223

-4-

*SLRBB223*

SECTION II
5. Attempt the following (any four) :

20

A) Describe flat and hierarchical name space.


B) Write a note on POP3.
C) Write a note on IMAP.
D) Describe SSH and explain need of SSH.
E) Explain out of band signaling and escape character concepts of TELNET.
F) Write a note on TFTP.
6. Describe architecture of Email by considering four different scenarios.

10

OR
Explain different types of messages used in TFTP communication.
7. Describe DNS in internet. What are different types of domain ? List the
characteristics of each type of domain.

10

_____________________

Set A

SLR-BB 224

*SLRBB224*
Seat
No.

Set

T.E. (CSE) (Part I) (New) (Computer Science and Engineering) Examination, 2014
COMPUTER ORGANIZATION
Day and Date : Friday, 5-12-2014
Time : 3.00 p.m. to 6.00 p.m.

Max. Marks : 100

Instructions : 1) Q. No. 1 is compulsory. It should be solved in first 30 minutes in


Answer Book Page No. 3. Each question carries one mark.
2) Answer MCQ/Objective type questions on Page No. 3 only.
Dont forget to mention, Q.P. Set (A/B/C/D) on Top of Page.
MCQ/Objective Type Questions
Duration : 30 Minutes

Marks : 20

1. Objective questions. Each question carry one mark :

20

1) Ferrite cores becoming the dominant technology for main memories in __________
generation.
a) First
b) Second
c) Third
d) None of these
2) ______________ method uses counters for designing hardwired control unit.
a) Sequence counter method
b) Delay element method
c) State table method
d) All of the above
3) Limitation of Babbages difference engine is
a) It performs only subtraction operation
c) It performs only multiplication operation

b) It performs only addition operation


d) It performs only division operation

4) ______________ addressing mode is most suitable to change the normal


sequence of execution of instructions.
a) Relative
b) Indirect
c) Immediate
d) Index with offset
5) RISC have major attribute such as
a) Variable and easily decoded instruction formats
b) Micro programmed control
c) Fixed and easily decoded instruction formats
d) Multicycle instruction execution
6) Push B is a
a) Two address instruction
c) Zero address instruction

b) One address instruction


d) Three address instruction

7) The different way in which the Location of an operand is specified in an instruction


are referred to as
a) Register Modes b) Assembly codes c) Instructions d) Addressing modes
8) OP A is a
a) EDVAC one address information format b) ENIVAC one address information format
c) IAS one address information format
d) None of these
P.T.O.

SLR-BB 224

*SLRBB224*

-2-

9) It is a collection of functional units capable of performing certain operations on data,


a) Control unit
b) Memory unit
c) Data processing unit
d) All of the above
10) Floating point representation used to store
a) Boolean values b) Whole numbers

c) Real integers

d) Integers

11) A single instruction with multiple scalar instructions is characteristic of which processor ?
a) Multi-processor
b) Uniprocessor
c) Concurrent processor
d) Vector processor
12) The most signification bit of address of memory that are used for comparison are
called ?
a) Index
b) Offset
c) Tag
d) Word
13) What are the advantages of the write through cache ?
a) Memory bandwidth in high
b) Retain consistent image of program activity in memory
c) Dirty bit is set
d) None of above
14) In which process the original program is decomposed into basic sub program
a) Scheduling of task
b) Partitioning
c) Synchronization
d) None of above
15) The address register for memory operation is called
a) Memory address register
b) Storage register
c) Instruction register
d) Micro instruction register
16) Which register holds the instruction being interpreted or executed ?
a) Instruction register
b) Memory address register
c) Stack register
d) Memory instruction register
17) A pipe lined processor is assumed to operate under full capacity, which of the
following is not one of the constraint ?
a) One instruction is decoded in each cycle
b) Instruction are decoded in the sequence they are stored in memory
c) Instruction are decoded in the sequence they appear in program representation
d) None of above
18) Virtual memory is implemented through
a) Segmentation
b) Paging
c) Demand paging
19) Cycle stealing is/are used in which concept ?
a) Programmed I/O b) DMA
c) Interrupts

d) None of above
d) Memory mapped I/O

20) In memory-mapped I/O ______________


a) The I/O devices and the memory share the same address space
b) The I/O devices have a separate address space
c) The memory and I/O devices have an associated address space
d) A part of the memory is specifically set aside for the I/O operation
______________
Set A

*SLRBB224*

-3-

SLR-BB 224

Seat
No.

T.E. (CSE) (Part I) (New) (Computer Science and Engineering) Examination, 2014
COMPUTER ORGANIZATION
Day and Date : Friday, 5-12-2014

Marks : 80

Time : 3.00 p.m. to 6.00 p.m.

SECTION I
2. Attempt any four :

(45=20)

1) Write a short note on Babbages difference engine and state its drawbacks.
2) Evaluate the equation using zero address and one address instruction.
X = 2*(ab).
3) Draw and explain hardwired control unit Organization.
4) What are the major attributes of RISC and CISC processors ?
5) Write a short note on IEEE floating point standard of Double precision format.
6) Explain how ripple Carry adder works ? State its drawbacks.
3. A) Perform 105 15 using Restoring and Non-Restoring division algorithm.
B) Explain with neat diagram sequence counter method of hardwired control.

10
10

OR
C) Explain Bus Hierarchical Architecture in detail.

10

Set A

SLR-BB 224

-4-

*SLRBB224*

SECTION II
4. Attempt any 4 :

(45=20)

1) Explain DMA block diagram.


2) Write short note on high speed memory.
3) Explain logical circuit of associative memory cell.
4) Explain I/O processor interconnection network.
5) Explain architecture of pipeline processor.
6) Explain multiprocessor parallelism UMA, NUMA.
5. Attempt :

10

1) Consider a paging system in which m1 has capacity of three pages. The page
address stream formed is 2 3 4 1 5 2 4 1 3 4 4 1.
6. Attempt any 1 :

10

1) With neat diagram explain architecture of vector super computer.


2) Explain look aside and look through system organization.
_____________________

Set A

SLR-BB 225

*SLRBB225*
Seat
No.

Set

T.E. (CSE) (Part I) (New) Examination, 2014


SYSTEM PROGRAMMING
Day and Date : Monday, 8-12-2014
Time : 3.00 p.m. to 6.00 p.m.

Max. Marks : 100

Instructions : 1) Q. No. 1 is compulsory. It should be solved in first 30 minutes in Answer Book


Page No. 3. Each question carries one mark.
2) Answer MCQ/Objective type questions on Page No. 3 only. Dont forget to mention,
Q.P. Set (A/B/C/D) on Top of Page.
MCQ/Objective Type Questions
Duration : 30 Minutes
1.

Marks : 20

Select correct alternative :


1) Execution gap is bridged by ______________
a) Software development team
c) Programmer

(201=20)
b) Designer of program language processor
d) All above

2) Specification language of PL domain is ______________


a) PL itslef
b) Machine language c) Assembly language

d) None of above

3) Which of the following statement NOT TRUE ?


a) Problem oriented languages have very large execution gap
b) Problem oriented languages have very small execution gap
c) Problem oriented languages have small specification gap
d) None of above
4) Which one of the following is a operator grammar ?
a) A :: = CD + E|a
b) A :: = CAD | C + D

c) A :: = C + D | C D| a d) None of above

5) Consider the following assembly code


START
100
MOVER
AREG, A
ADD
AREG, B
MOVEM
AREG, C
STOP
A
DC
5
B
DC
8
C
DS
1
END
The content of symbol table for above code will be

a)

c)

Symbol Address Length


A
103
1
B
104
1
C
105
1
Symbol Address Length
A
105
1
B
106
1
C
107
1

b)

d)

Symbol Address Length


A
104
1
B
105
1
C
106
1
Symbol Address Length
A
104
1
B
106
1
C
105
1
P.T.O.

SLR-BB 225

*SLRBB225*

-2-

6) Consider the following assembly code


START
100
MOVER
AREG, =5
ADD
AREG, B
MOVEM
AREG, C
STOP
B
DC
8
C
DS
1
END
What will be the variant-II intermediate code for above code
a) (AD, 01) (C, 100)
b) (AD, 01) (C, 100)
c)
(IS, 04) AREG, (L, 01)
(IS, 03) AREG, (L, 01)
(IS, 01) AREG, B
(IS, 02) AREG, B
(IS, 05) AREG, C
(IS, 05) AREG, C
(IS, 00)
(IS, 00)
(DL, 02) (C, 8)
(DL, 01) (C, 8)
(DL, 01) (C, 1)
(DL, 02) (C, 1)
7) By default all the literals are stored
a) At the end of program
b) Start of program

?
(AD, 01) (C, 100)
d) (AD, 01) (C, 100)
(IS, 04) AREG, (L, 01)
(IS, 04) AREG, (L, 01)
(IS, 01) AREG, B
(IS, 01) AREG, B
(IS, 05) AREG, C
(IS, 05) AREG, C
(IS, 00)
(IS, 01)
(DL, 01) (C, 8)
(DL, 01) (C, 8)
(DL, 02) (C, 1)
(DL, 02) (C, 1)

c) After LTORG

d) Middle of the program

8) Expansion of nested macro calls follow the ___________


a) FIFO
b) LIFO
c) Circular Queue

d) None of above

9) Keyword parameters are passed to macro as


a) ='&Variable Name'
b) &Variable Name

d) All of above

c) =Variable Name

10) Consider the macro definition


MACRO
CONTANTS
LCL
&A
&A
SET
1
DB
&A
&A
SET
&A + 1
DB
&A
MEND
In above macro definition, which instruction is used for defining local expansion variable ?
a) SET
b) LCL
c) DB
d) MACRO
11) A Block in a Program is a ___________ that all may contain data declaration.
a) Function
b) Procedure
c) Code of Unit

d) All the above

12) Binding of external references to the link time address is called _________
a) Translation
b) Loading
c) Linking

d) None

13) Binary symbolic subroutine loader is example of


a) Absolute loader
b) Compiler-and-go

c) Relocating loader

d) Direct linking loader

14) Code optimization is ____________


a) Improve the execution efficiency of a program
c) Rearrangement of computations in a program

b) Eliminate the redundancies in a program


d) All of above

15) Object module of program unit consist of __________


a) RELOCTAB
b) LINKTAB
c) Header

d) All

16) Loading of operating system is performed by a special purpose loader called _________
a) Absolute loader
b) Relocation loader
c) Bootstrap loader
d) Compiler-and-Go loader
17) Compilers and Interpreters are themselves
a) High level language
b) Codes

c) Mnemonics

d) Programs

18) The relocation factor is difference between ________ and ___________


a) Translated and linked origin
b) Translated origin and public definition
c) Linked origin and public definition
d) None
19) Compiler bridges a Semantic gap between which domain
a) Application and execution
b) Application and PL
c) PL and Execution
d) None
20) LINKTAB contains information concerning
a) Public definition
b) External references c) a) and b) both
______________

d) Internal references

Set A

*SLRBB225*

-3-

SLR-BB 225

Seat
No.
T.E. (CSE) (Part I) (New) Examination, 2014
SYSTEM PROGRAMMING
Day and Date : Monday, 8-12-2014
Time : 3.00 p.m. to 6.00 p.m.

Marks : 80

SECTION I
2.

Solve any four :

(54=20)

a) Explain the concept of derivation, reduction and parse tree with example.
b) Explain program interpretation and program execution.
c) Explain the tables generated by pass-I of two pass assembler with example.
d) Explain following assembler directives with example.
i) EQU and PURGE
ii) SEGMENT, ENDS and ASSUME.
e) Consider an assembly language program :
START

100

MOVER

AREG, A

ADD

AREG, B

MOVEM

AREG, C

STOP
A

DC

DC

DS

END
Display the content of different data structure after pass-I and show the intermediate code generated for the
program.
3.

Solve any one :

10

a) Explain in detail following language processing activities.


i) Program generation activity
ii) Program execution activity.
b) Explain design of macro pre-processor.
4.

Write and explain pass-I algorithm of two pass assembler.

10

Set A

SLR-BB 225

-4-

*SLRBB225*

SECTION II
5.

Attempt any four :

(54=20)

a) Give aspects of compilation and explain role of data types and scope rules.
b) Explain operand descriptors and register descriptors.
c) Explain object module with example.
d) How external references are resolved in linker ? Explain with example.
e) Explain absolute loading with example.
f) What is the order of ID number on the ESD cards ? Why is it not needed for normal symbols ?
6.

Write and explain pass-I algorithm for direct linking loader with example. Draw flowchart for same.

10

OR
Explain binary program. Write program relocation algorithm and explain with example.
7.

Explain control flow analysis and data flow analysis.

10

_____________________

Set A

SLR-BB 226

*SLRBB226*
Seat
No.

Set

T.E. (Computer Sci. and Engg.) (Part I) (New) Examination, 2014


OPERATING SYSTEM CONCEPTS
Day and Date : Wednesday, 10-12-2014
Time : 3.00 p.m. to 6.00 p.m.

Max. Marks : 100

Instructions : 1) Figures to right indicate full marks.


2) Q. No. 1 is compulsory. It should be solved in first 30 minutes in
Answer book Page No. 3. Each question carries one mark.
3) Answer MCQ/Objective type questions on Page No. 3 only. Dont
forget to mention, Q.P. Set (A/B/C/D) on Top of Page.
MCQ/Objective Type Questions
Duration : 30 Minutes

Marks : 20

1. Choose the correct alternatives.

20

1) Which of the following system is used when there are rigid time requirements on the
operation of processor or the flow of data ?
A) Time-sharing
B) Parallel
C) Distributed
D) None of above
2) In which state, instructions are being executed
A) waiting
B) ready
C) running

D) none of above

3) A Process Control Block consist of which of the following field.


A) Program Counter
B) Memory management information
C) CPU Scheduling Information
D) None of the above
4) In __________ scheduling, once the CPU has been allocated to the process, the process
keeps the CPU until it releases.
A) Pre-emptive
B) Non-pre-emptive
C) Round robin
D) All
5) Which one of the following scheduler present in the operating system ?
A) Large Scheduler
B) Small Scheduler
C) Medium-term Scheduler
D) Sequencing Scheduler
6) ____________ is the segment of code in which the process is changing common variables
or data.
A) Critical Section
B) Remainder Section
C) Real-time code
D) None of above
7) A solution to the Critical Section problem must satisfy which of the following condition.
A) Progress
B) Circular waits
C) Speed up
D) All above
8) ___________ time is a pure overhead, because system does not do any useful work.
A) Scheduling
B) Context Switch
C) Process Running
D) All above
P.T.O.

SLR-BB 226

*SLRBB226*

-2-

9) If multiple CPUs are available, the scheduling problem is correspondingly more complex
as compared with a system with single CPU.
A) True
B) False
C) Cant say
D) None of above
10) A Semaphore is ____________, when its integer value ranges over an unrestricted domain.
A) Binary
B) Counting
C) Running
D) None of above
11) ___________ is not essential condition for occurrence of deadlock.
A) Circular wait
B) Hold and wait
C) No preemption of resource
D) None
12) Unsafe state indicates _____________
A) Possibility of deadlock occurrence in future
B) Guarantee of presence of deadlock in future
C) Increased processor utilization
D) None
13) Ri > Pj Edge in resource allocation graph (from resource Ri to process Pj) indicates
___________
A) Process Pj has requested resource Ri and Ri not allocated yet to Pj
B) Ri resource is allocated to process Pj
C) Process Pj has completed utilization of resource Ri
D) Resource Ri is not be allocated to process Pj
14) ___________ is a non contiguous way of memory allocation.
A) Paging
B) Segmentation
C) Both A) and B)

D) None

15) Best fit has ___________ and worst fit has _____________ advantage.
A) Reduced searching time, efficient memory utilization
B) Efficient memory utilization, reduced searching time
C) Efficient memory utilization, dynamic expansion
D) Reduced searching time, dynamic expansion
16) Logical page number is held in _____________ register before translating it to physical
address.
A) Base register
B) Memory access register
C) Page map table base register
D) Instruction register
17) Which algorithm chooses the page that has not been used for the longest period of time
whenever the page required to be replaced ?
A) first in first out algorithm
B) additional reference bit algorithm
C) least recently used algorithm
D) counting based page replacement algorithm
18) __________ page replacement policy generates minimum number of page fault.
A) LRU
B) LFU
C) FIFO
D) Optimal
19) Thrashing leads to ___________
A) Excessive page transitions between primary and secondary memory
B) Decreased processor utilization
C) More I/O
D) All
20) ____________ is a set of wires and a rigidly defined protocol that specifies a set of
messages that can be sent on wires.
A) BIOS
B) PORT
C) Bus
D) Controller
______________
Set A

*SLRBB226*

-3-

SLR-BB 226

Seat
No.

T.E. (Computer Sci. and Engg.) (Part I) (New) Examination, 2014


OPERATING SYSTEM CONCEPTS
Day and Date : Wednesday, 10-12-2014
Time : 3.00 p.m. to 6.00 p.m.

Marks : 80

Instructions : 1) All questions are compulsory.


2) Figures to right indicate full marks.
SECTION I
2. Attempt any four (each carries 5 marks) :

20

A) Describe the use of time-sharing system.


B) What is system call ? List and explain at least four process control system calls.
C) Describe PCB with diagram.
D) Explain the concept of Cooperating Process.
E) Write a note on Multiple Processor Scheduling.
3. Describe the dinning philosophers problem of synchronization. Provide a solution to this
problem using monitors/semaphore.

10

OR
Describe the concept of medium term scheduler with diagram. Compare between long-term
scheduler, short term scheduler and medium-term scheduler.
4. Describe the Shortest Job First (preemptive and non-preemptive) Scheduling and Priority
Scheduling (preemptive and nonpreeemptive) with suitable examples.

10

10

SECTION II
5. Attempt any four :

(54=20)

A) Define deadlock. Explain how four conditions necessary for occurrence of deadlock.
B) Explain with example notation for RAG and how it is used for deadlock detection.
C) What is fixed partitioning ? State and explain its drawback.
D) Explain TLB mechanism in paging.
E) Write a note on thrashing.
F) Explain Most Frequently Used page replacement with appropriate example.
Set A

SLR-BB 226

-4-

*SLRBB226*

6. Explain page replacement algorithm LRU, FIFO algorithm following reference string as example
with memory for 3 frames is available. Determine number of page faults in each case and
state which is closer to optimal in given context.

10

7, 0, 1, 2, 0, 3, 0, 4, 2, 3, 0, 3, 2, 1, 2, 0, 1, 7, 0, 1
OR
Explain the concept of segmentation with paging with the help of diagram. Also show address
translation.

10

7. Explain bankers algorithm. Solve following example with bankers algorithm and determine all
possible safe sequences if exist. Assume p1, p2, p3 as process ids and R1, R2, R3 as
resource ids.

10

Max = 7
9
11

10
8
12

Alloc = 4
6

Available = 20

12
7
7

20

17

In a special scenario if P1 is granted 3 resources of type R1, check after allocation safe
sequence exist or not.
_____________________

Set A

SLR-BB 227

*SLRBB227*
Seat
No.

Set

T.E. (CSE) (Part I) (New) Examination, 2014


DESIGN AND ANALYSIS OF ALGORITHM
Day and Date : Friday, 12-12-2014
Time : 3.00 p.m. to 6.00 p.m.

Max. Marks : 100

N.B. : 1) Q. No. 1 is compulsory. It should be solved in first 30 minutes


in Answer Book Page No. 3. Each question carries one mark.
2) Answer MCQ/Objective type questions on Page No. 3 only.
Dont forget to mention, Q.P. Set (A/B/C/D) on Top of Page.
3) Figures to right indicate marks.
MCQ/Objective Type Questions
Duration : 30 Minutes

Marks : 20

1. Choose the correct alternatives :


1) In dynamic programming, the output to stage n becomes the input to
a) stage n 1
b) stage n itself
c) stage n + 1
d) stage n 2

20

2) Which of the following function has the largest growth rate ?


n

a) n 2
b) n100
c) 2 2
d) 2n !
3) The running time of quicksort depends on most heavily on the selection of
a) Number of input
b) Arrangement of element in array
c) Size of element
d) Pivot element
4) The minimum spanning trees in connected graph with n node is
1

c) 2
d) 1
b) n 2
5) How many colours are need to colour the graph show in fig. ?
a) n 1

a) 3

b) 4

c) 5

d) 6
P.T.O.

SLR-BB 227

-2-

*SLRBB227*

6) Algorithm which solve All-pair shortest path problem is


a) Dijkstras
b) Floyds
c) Prims
d) Warshall
7) For merging two sorted list of sizes m into a sorted list of size m + n, we
require comparison of
a) O(m)
b) O(n)
c) O(m + n)
d) O(logn + logm)
8) Tree organizations that problem instant dependent are called _________ trees
a) instance
b) heap
c) dynamic
d) graph
9) The node which has been generated and all of whose children have not yet
been generated is called ______________ node.
a) live
b) dead
c) state
d) next
10) __________ algorithm has the property that result of every operation is uniquely
defined.
a) Deterministic
b) Nondeterministic
c) Polynomial
d) Non-polynomial
11) An algorithm A is of polynomial complexity if there exists a polynomial P( )
such that computing time of A is O(p(n)) for every input size of n
b) O(np)
c) O(p(n))
d) O(p)
a) O(n2)
12) The number of iteration in fining maximum and minimum using divide and
conquer method
n

13)
14)

15)
16)

a) n 2
b) 3n 2
c) 3n 3
d) 3 2 2
Five files with sizes (20, 30, 10, 5, 30), with greedy rule, no. of record move is
a) 205
b) 95
c) 40
d) 105
Algorithm A is said to be __________ recursive if it calls another algorithm
which inturn call A.
a) direct
b) indirect
c) recursive
d) loop
An algorithm must terminate after a finite no. of steps is known as
a) definiteness
b) finiteness
c) effectiveness d) correctness
A feasible solution that either maximizes or minimizes a given objective
function is called
a) optimal solution
b) local solution
c) exact solution
d) correct solution

17 20 Match the pair :


Algorithm
17) Binary search
18) Merge sort
19) Quick sort
20) Selection

Woost Case Complexity


a) O(n)
b) O(n2)
c) O(n logn)
d) (log n)
______________
Set A

*SLRBB227*

SLR-BB 227

-3-

Seat
No.

T.E. (CSE) (Part I) (New) Examination, 2014


DESIGN AND ANALYSIS OF ALGORITHM
Day and Date : Friday, 12-12-2014
Time : 3.00 p.m. to 6.00 p.m.

Marks : 80

N.B. : 1) All questions are compulsory.


2) Figures to right indicate marks.
SECTION I
2. Solve any four :

(54=20)
n

a) Compare the two functions n2 and 2 4 for various values of n. Determine


with proper values.
b) Determine the frequency count for all statements in the following alg :
Alg Mult (a, b, c, n)
{
for i = 1 to n do
for j = 1 to n do
{
c[i, j] =0 ;
for k = 1 to n do
c[i, j] = c [i, j] + a[i, k] * b [k, j] ;
}
}
c) Differentiate between greedy algorithm and dynamic programming.
d) Write : Control abstraction for divide and conquer.
e) Solve :
a
n = 1, a cons tant

T(n) =
2 T(n / 2) + cn n > 1, c cons tan t

f) What is the solution generated by the function job scheduling when n = 7 ?


(P1, P2, .... P7) = (3, 5, 20, 18, 1, 6, 30)
(d1, d2, ... d7) = (1, 3, 4, 3, 2, 1, 2).
Set A

SLR-BB 227

*SLRBB227*

-4-

3. A) Show that following equalities are correct :


a) 5n2 6n = (n2 )

10

b) n ! = O(nn).

OR
A) Devise a binary search algorithm that splits set not into two sets of (almost)
equal sizes but into two sets, one of which is twice the size of the other.
10
4. Obtain a set of optimal Huffman codes for the messages (M1, ....M7) with
relative frequencies (q1, ... q7) = (4, 5, 7, 8, 10, 12, 20). Draw the decode tree.

10

SECTION II
5. Solve any four :

(54=20)

a) Explain principal of optimality with example.


b) Write a short note on : Reliability design.
c) Draw tree organisation for 4 queens solution space and explain.
d) Define Hamilton cycle. With one example explain.
e) What are P, NP, NP complete and NP-Hard ?
f) Consider the Knapsack instance n = 3
(w1, w2, w3) = (2, 3, 4)
(P1, P2, P3) = (1, 2, 5) and m = 6
solve using DP.
6. A) Two jobs have to be scheduled on three processors. The task time are given
by the Matrix J
2 0
J = 3 3
5 2

Draw two possible schedule and find MFT.

10

OR
A) Explain Cooks theorem.

10

7. Let w = [5, 7, 10, 12, 15, 18, 20} and m = 35, find all possible subset of w that
sum of m. Draw the portion of state space tree that is generated.

10

_____________________

Set A

SLR-BB 229

*SLRBB229*
Seat
No.

Set

T.E. (CSE) (Part II) Examination, 2014


DATABASE ENGINEERING
Day and Date : Tuesday, 25-11-2014
Time : 10.00 a.m. to 1.00 p.m.

Max. Marks : 100

Instructions : 1) All questions are compulsory.


2) Figure to right indicates marks.
3) Answer MCQ/Objective type questions on Page No. 3 only.
Dont forget to mention, Q.P. Set (A/B/C/D) on Top of Page.
4) Q. No. 1 is compulsory. It should be solved in first 30 minutes
in Answer book Page No. 3. Each question carries one mark.
MCQ/Objective Type Questions
Duration : 30 Minutes

Marks : 20

1. Tick mark the correct answer :


20
1) The SQL WHERE clause
a) Limits the column data that are returned b) Limits the row data are returned
c) Both a) and b) are correct
d) Neither a) nor b) are correct
2) Architecture of the database can be viewed as
a) Two levels
b) Four levels
c) Three levels
d) One level
3) Normalization ____________ data duplication.
a) Eliminates
b) Reduces
c) Increases
d) Maximizes
4) A relation is said to be in 2 NF if
i) it is in 1 NF
ii) non-key attributes dependent on key attribute
iii) non-key attributes are independent of one another
iv) if it has a composite key, no non-key attribute should be dependent on
part of the composite key
a) i, ii, iii
b) i and ii
c) i, ii, iv
d) i, iv
5) You can add a row using SQL in a database with which of the following ?
a) ADD
b) CREATE
c) INSERT
d) MAKE
6) AS clause is used in SQL for \n
a) Selection operation
b) Rename operation
c) Join operation
d) Projection operation
7) A data dictionary is a special file that contains: \n
a) The name of all fields in all files
b) The width of all fields in all files
c) The data type of all fields in all files d) All
8) Relational calculus is a
a) Procedural language
b) Non-Procedural language
c) Data definition language
d) High level language

P.T.O.

SLR-BB 229

-2-

*SLRBB229*

9) In the architecture of a database system external level is the


a) Physical level b) Logical level c) Conceptual level d) View level
10) The natural join is equal to
a) Cartesian product
b) Combination of Union and Cartesian product
c) Combination of Selection and Cartesian product
d) Combination of Projection and Cartesian product
11) Locking may cause which of the following problems
a) Erroneous updates
b) Deadlock
c) Versioning d) All of the above
12) Which of the following is the preferred way to recover a database after a
transaction in progress terminates abnormally ?
a) Rollback
b) Rollforward
c) Switch to duplicate database
d) Reprocess transactions
13) Concurrency control is important for which of the following reasons ?
a) To ensure data integrity when updates occur to the database in a multiuser
environment
b) To ensure data integrity when updates occur to the database in a single-user
environment
c) To ensure data integrity while reading data occurs to the database in a
multiuser environment
d) To ensure data integrity while reading data occurs to the database in a
single-user environment
14) Indices whose search key specifies an order different from the sequential
order of the files are called ____________
a) Nonclustering indices
b) Clustering indices
c) Hash indices
d) None
15) Which is duplication of computer operations and routine backups to combat
any unforeseen problems ?
a) Concurrency
b) Deadlock
c) Backup
d) Recovery
16) Which are types of recovery control techniques ?
a) Deferred update
b) Immediate update
c) Both
d) None
17) The ___________ indexes are built based on the same key by which the data
is ordered on the disk.
a) Non-clustered b) Clustered
c) Hashed
d) Primary
18) A clustering index is defined on the fields which are of type
a) =Non-key and ordering
b) Non-key and non-ordering
c) Key and ordering
d) Key and non-ordering
19) Cascading rollback is avoided in all protocol except
a) Strict two-phase locking protocol b) Tree locking protocol
c) Two-phase locking protocol
d) Validation based protocol
20) Check points are a part of
a) Recovery measures
b) Security measures
c) Concurrency measures
d) Authorization measures
______________
Set A

*SLRBB229*

-3-

SLR-BB 229

Seat
No.

T.E. (CSE) (Part II) Examination, 2014


DATABASE ENGINEERING
Day and Date : Tuesday, 25-11-2014

Marks : 80

Time : 10.00 a.m. to 1.00 p.m.


Instructions : 1) All questions are compulsory.
2) Figure to right indicates marks.
SECTION I
2. Attempt any four :

(45=20)

a) Draw and explain 3 schema architecture.


b) Explain database users and database administrator.
c) List and explain relational algebra operation and additional extended operation.
d) Explain domain relational calculus.
e) Explain decomposition using functional dependency.
3. Attempt any one :

10

a) Define and explain 2NF, 3NF, BCNF.


b) Explain in detail domain constraints and referential integrity.
4. Explain modification of databases and NULL values.

10

Set A

SLR-BB 229

-4-

*SLRBB229*

SECTION II
5. Solve any four :

(45=20)

a) Explain in brief immediate database modifications.


b) Explain the difference between three storage types volatile, non volatile and
stable in terms of I/O cost.
c) Explain insertion of a node into a B+ tree.
d) Explain extendable hash structure in dynamic hashing.
e) Write a short note on locks.
f) Write a short note on validation based protocol.
6. Solve any one :

10

a) Explain the concept of ordered indices with an example.


b) Discuss log based recovery in detail.
7. Explain concept of conflict and view serializability.

10

_____________________

Set A

SLR-BB 23

*SLRBB23*
Seat
No.

Set

S.E. (Civil) (Part I) Examination, 2014


FLUID MECHANICS I
Day and Date : Thursday, 18-12-2014
Time : 10.00 a.m. to 1.00 p.m.

Max. Marks : 100

Instructions : 1) Q. No. 1 is compulsory. It should be solved in first 30


minutes in Answer Book Page No. 3.
2) Answer MCQ/Objective type questions on Page No. 3 only.
Dont forget to mention, Q.P. Set (A/B/C/D) on Top of Page.
MCQ/Objective Type Questions
Duration : 30 Minutes

Marks : 20

1. Choose the correct answer :


1) Newtons law of viscosity is given by the relation.
2
a) =

du
dy

b) =

du
dy

c) =

du
dy

1
d) =

du
dy

2) Rise of sap in tree is an example of


1
a) Surface tension
b) Pressure
c) Compressibility
d) Capillarity
3) For a submerged body, if centre of buoyancy is above than C. G. of hte body,
the body will remain in the state of
1
a) Stable equilibrium
b) Neutral equilibrium
c) Unstable equilibrium
d) Any of the above
4) The path followed by the fluid particle in motion is called as
a) Streak line
b) Stream line
c) Path line
d) None of the above

5) If the Reynolds No. is > 4000, the flow in a pipe is called as


a) Laminar
b) Turbulent
c) Transitional
d) None of the above
6) Inclined single column manometer is useful for the measurement of
___________pressures.
a) Small
b) Medium
c) High
d) Negative

7) Surface tension is expressed in


a) N/m
b) N/m2

c)

N2/m

d)

N/m3

P.T.O.

SLR-BB 23

*SLRBB23*

-2-

8) The continuity equation is based on the principle of


a) Conservation of momentum
b) Conservation of mass
c) Conservation of energy
d) None of the above

9) If the liquid is having sp.gr. of 0.7, then its specific weight is


2
3
3
3
a) 6867 N/m
b) 700 N/m
c) 9810 N/m
d) None of the above
10) The total head in a flow is the sum of
a) Piezometric head and datum head
b) Piezometric head and pressure head
c) Piezometric head and velocity head
d) Piezometric head velocity head and datum head
11) Piezometers measure ____________ pressure only.
a) Absolute
b) Gauge
c) Atmospheric
d) Any of the above

12) The discharge of a fluid through circular pipe of dia 20 cm having maximum
velocity 1.5 m/sec. is (in m3/sec.)
a) 0.047
b) 0.031
c) 0.023
d) 0.31

13) For laminar flow through circular tube, the average velocity at a section is
1
a) 1/3 V max
b) Vmax/2
c) 2/3 Vmax
d) None
14) In terms of roughness Reynold number boundary is smooth when (u*k)/ is 1
a) < 0.25
b) < 4.0
c) < 6.0
d) 2.5 to 6.0
15) The loss of head at the entrance to a pipe is given by
a) V2/2g
b) V21 V22) / 2g
c) 0.5 V2/2g

d)

0.5V2/4g

16) The parameter k/d of a pipe is called


1
a) Absolute roughness
b) Average roughness
c) Relative roughness
d) Equivalent roughness
17) For a sudden enlargement in a certain pipe velocity changes from 7.96 m/s to
1.99 m/s. What will be head loss due to sudden enlargement ?
1
a) 35.64 m
b) 3.63 m
c) 1.81m
d) 0.30 m
18) The boundary layer takes place for
a) Ideal fluids
c) Real fluids

1
b) Pipe flow only
d) Flow over flat plate only

19) Boundary layer separation is caused by


a) Negative pressure gradent
b) Positive pressure gradent
c) Zero pressure gradent
d) None of the above

______________
Set A

*SLRBB23*

-3-

SLR-BB 23

Seat
No.

S.E. (Civil) (Part I) Examination, 2014


FLUID MECHANICS I
Day and Date : Thursday, 18-12-2014
Time : 10.00 a.m. to 1.00 p.m.

Marks : 80

Instructions : 1) Q. 2 and Q. 6 are compulsory. Attempt any two questions


from remaining questions in Section I and II respectively.
2) Figures to right indicate full marks.
3) Assume correct data wherever necessary.
4) Use of non-programmable calculator is allowed.
SECTION I
2. a) State and prove Pascals law.

b) A plate having an area of 0.6 m2 is sliding down the inclined plane at 30 to


the horizontal with a velocity of 0.36 m/s. There is a fluid of thickness 1.8 mm
between the plane and the plate. Find the viscosity of fluid if the weight of
plate is 280 N.

c) The diameter of a pipe at the Section 11 and 2 2 are 200 mm and 300 mm resp.
If the velocity of water flowing through the pipe at Section 1 1 is 4 m/s, find :
i) Discharge through the pipe
ii) Velocity of water at Section 2 2.
4
3. a) Derive the expression for total pressure and centre of pressure for submerged
vertical plane surface.
b) An isosceles triangular plate of base 3 m and altitude 3 m is immersed vertically
in an oil of specific gravity 0.8. The base of the plate coincides with free
surface of oil.
Determine : i) Total pressure on the plate
ii) Centre of plate
4. a) A wooden block of specific gravity 0.7 floats in water. Determine the
metacentric height of the block if its size is 2 m1 m0.8 m.

b) If for a two dimensional potential flow, the velocity potential is given by


= x (2y 1). Determine the velocity at the point P(4, 5). Determine also the
value of stream function at the point P.

5. Attempt any three questions :

6
12

a) Derive the expression for capillary rise of water in a glass tube.


b) Define :
i) Ideal fluids

ii) Real fluids

iii) Steady flow iv) Uniform flow


Set A

SLR-BB 23
-4*SLRBB23*
c) The right limb of a simple U-tube manometer containing mercury is open to
atmosphere while left limb is connected to a pipe in which a fluid of sp.gr. 0.9
is flowing. The centre of pipe is 12 cm below the level of mercury in the right
limb. Find the pressure if fluid in the pipe if difference of mercury level in the
two limbs is 20 cm.
d) Following are the velocity components for steady incompressible flow :
i) u = 2x 3y, v = x 2y, w = 0
ii) u = 2x2 xy + z2, v = x2 4xy + y2 , w = 2xy yz + y2
For above cases, check whether the flow is continuous or not.
SECTION II
6. a) A certain pipe carries water at point A diameters is 20 cm and at point B it is
10 cm. Pipe carries water 35 lit/sec. The point A is 6 m above the ground and
point B is 4m above the ground. If pressure at point A is 39.24 N/cm2, what
will be the pressure at B.

b) Derive the expression for drop of pressure for given length (L) of a pipe for
laminar viscous flow.

c) A certain pipe carries water and having average height of irregularity


0.15 mm if shear stress developed is 4.9 N/m and viscosity of water is
0.01 104 m2/s. What type of boundary is it ?

7. a) Derive the expression for rate of flow through venture meter.

b) A circular tank of diameter 1.25 m contains water upto height of 5 m. Find the
height of water after 1.5 minutes if orifice is used to remove the water having
50 mm diameter and Cd = 0.62.
8. a) On the basis of Nikuradse experiment and Reynolds number how will you
classify the type of boundary.

6
4

b) Certain pipe is enlarged from diameter 200 mm to 400 mm if discharge through


pipe is 250 lit/sec. What will be the loss of head due to enlargement ?

c) What is siphon explain with its uses ?

9. a) Write a note on :

1) Displacement thickness 2) Momentum thickness 3) Energy thickness.


b) Velocity distribution in boundary layer is given by :

u
y y
= 2
U

Calculate : 1) Displacement thickness

2) Momentum thickness.

_____________________

Set A

SLR-BB 230

*SLRBB230*
Seat
No.

Set

T.E. (CSE) (Part II) Examination, 2014


COMPILER CONSTRUCTION
Day and Date : Wednesday, 26-11-2014
Time : 10.00 a.m. to 1.00 p.m.

Max. Marks : 100

Instructions : 1) Q. No. 1 is compulsory. It should be solved in first 30 minutes

in Answer book Page No. 3. Each question carries one mark.


2) Answer MCQ/Objective type questions on Page No. 3 only. Dont
forget to mention, Q.P. Set (A/B/C/D) on Top of Page.
MCQ/Objective Type Questions
Duration : 30 Minutes
1. Select correct alternative :

Marks : 20

1) Certain variables might be used before being defined is an example of


a) Structure editor
b) Pretty printers
c) Static checker
d) Interpreter
2) Output of loader is
a) Source program
c) Target assembly program
3)

b) Relocatable machine code


d) Absolute machine code

- closure (T) is
a) Set of NFA states reachable from some NFA state s in T on - transition alone
b) Set of NFA states reachable from NFA state on - transition alone
c) Set of NFA states
d) None

4) LL(k) grammar
a) Has to be CFG
c) Cannot have left recursive non-terminals

b) Has to be unambiguous
d) None of above

5) Which of the following is not example of top-down parser ?


a) Recursive descent parser
b) Predictive parsing
c) Shift-reduce parser
d) Non-recursive predictive parsing
6) A grammar G is said to be operator precedence if it posses
a) No production on the right side is
b) No production on the right side has 2 adjacent non-terminals
c) Both a) and b)
d) None of above
7) The grammar E E+E|E*E|a is
a) Ambiguous
c) No, depends on sentence

b) Unambiguous
d) ND
P.T.O.

SLR-BB 230
8) Consider the grammar
Ad |
S Aa|bAc|Bc
FIRST(S) is
a) {a,b, }

*SLRBB230*

-2-

Bd

b) {a, b,d}

c) {a,b,d, }

9) ___________syntax of mkleaf is
a) mkleaf (id, num)
b) mkleaf (num, val)

d) None of the above

c) mkleaf (id, entry) d) Both b) and c)

10) Value-number method for constructing node in DAG uses


a) Index search techniques
b) Sequential search
c) Hash function
d) Both b) and c)
11) Which of the following is needed to refer to the non-local data held in other activation
record ?
a) Control link
b) Access link c) Temporaries d) Saved machine status
12) In which storage allocation strategy size is required at compiler time ?
a) Static allocation b) Dynamic allocation
c) Stack allocation
13) Which of the following are parameter passing method ?
a) Call by value
b) Call by reference
c) Call by restore

d)All
d) All

14) The cost of instruction depends on


a) Cost associated with speed of execution
b) Cost associated with instruction length
c) One plus cost associated with source and destination source address
d) All
15) The quality of generated code is determined, depends on the which of the following factors
a) Uniformity, completeness
b) Speed, size
c) Machine idioms, uniformity
d) None
16) Code improvement phase consists of ___________ followed by the application
of transformations.
a) Data flow analysis
b) Control flow analysis
c) Both a) and b)
d) None of the above
17) A transformation of a program is called ___________ if it can be performed by
looking only at the statements in basic block.
a) Local
b) Global
c) Both a) and b) d) None of the above
18) The technique(s) important for loop optimization is/are
a) Code motion
b) Induction variable elimination
c) Reduction in strength
d) All of the above
19) A method for trying to improve the performance of the target program by examining
a short sequence of target instructions and replacing these instructions by a shorter
or faster sequence is
a) Peephole optimization
b) Code motion
c) Reduction in strength
d) All of the above

20) Which the following are three address code representations ?


a) Indirect triple

b) Triple

c) Quadruple

______________

d) All of above

Set A

*SLRBB230*

-3-

SLR-BB 230

Seat
No.

T.E. (CSE) (Part II) Examination, 2014


COMPILER CONSTRUCTION
Day and Date : Wednesday, 26-11-2014
Time : 10.00 a.m. to 1.00 p.m.
2. Attempt any four :

Marks : 80

(54=20)

a) Describe different phases of a compiler with help of block diagram.


b) Explain shift reduce-parsing using stack with example.
c) Consider the grammar
S (L) | a
L L,S|S
1) What is language generated by above grammar ?
2) Is the grammar ambiguous ? Justify.
d) Explain error recovery strategies in parser.
e) Explain syntax directed definition for constructing syntax tree.
3. Attempt any one :

10

a) Explain left factoring and elimination of left recursion with example. Eliminate left
recursion from the following grammar.
S Aa|b
A Ac|Sd|
b) Define bottom up parsing. Explain LR parsing algorithm with suitable example.
4. Explain top down translation of L-attributed definitions.

10

Set A

SLR-BB 230

-4-

*SLRBB230*

5. Attempt any four questions :

(54=20)

a) Explain the stack and heap storage allocation strategies.


b) Discuss issues in design of code generator.
c) Explain backpatching.
d) Explain the concept of copy propagation with example.
e) Explain dynamic storage allocation.
6. Attempt any one :

10

a) Write and explain syntax directed definition for flow of control statements.
b) Explain different loop optimization techniques.
7. Explain peephole optimization techniques.

10

_____________________

Set A

SLR-BB 231

*SLR-BB-231*
Seat
No.

Set

T.E. (CSE) (Part II) Examination, 2014


OPERATING SYSTEM II
Day and Date : Thursday, 27-11-2014
Time : 10.00 a.m. to 1.00 p.m.

Max. Marks : 100

Instructions : 1)
2)
3)
4)

Attempt all questions.


Assume data if necessary.
Figures to the right indicate full marks.
Q. No. 1 is compulsory. It should be solved in first 30 minutes in
Answer book Page No. 3. Each question carries one mark.
5) Answer MCQ/Objective type questions on Page No. 3 only.
Dont forget to mention, Q.P. Set (A/B/C/D) on Top of Page.
MCQ/Objective Type Questions

Duration : 30 Minutes

Marks : 20

1. Attempt the following multiple choice questions :

20

1) Which of the following term most correctly describes the Unix Kernel ?
A) Set of interrupts and handlers
B) Set of system calls and their internal algorithm
C) Set of tools and utility software
D) Central processing module
2) Following system call is used to create a new process.
A) fork( )
B) child( )
C) create( )

D) exec ( )

3) Which of the following is not a Unix operating system kernel services ?


A) Process execution control
B) I/O and memory management
C) Secondary memory management
D) Authentication and authorization
4) The ____________ identifies all open files for a process.
A) User file descriptor table
B) File table
C) Inode table
D) File allocation table
5) Unix kernel minimizes the frequency of disk access by use of ____________
A) scheduling
B) buffer cache
C) cache memory
D) I/O redirection
6) Unix kernel caches the data in buffer pool according to a _____________ algorithm.
A) most recently used
B) last in first out
C) most significantly used
D) least recently used
7) Which of the following algorithm is used to convert a user-level path name to an inode ?
A) openfile
B) namei
C) mapi
D) falloc
P.T.O.

SLR-BB 231

*SLR-BB-231*

-2-

8) Every directory contains the file names dot and dot-dot(.and ..) whose inode numbers
are _____________
A) of another directories
B) address to the file allocation table
C) those of directory and its parent respectively
D) pointers to the last and first inode entry
9) I seek file system call in Unix is used for _______________
A) positioning the I/O and allowing random access
B) seeking the last position in inode table
C) adjusting the position of offset in file
D) providing the last accessed file descriptor
10) Processes use the _______________ system call for named pipes and _______________
system call to create unnamed pipes.
A) open,dup
B) open,pipe
C) pipe,open
D) dup,open
11) Each process contains a private per process region table called a _______________
A) Pregion memory
B) Virtual memory
C) Logical memory
D) None of these
12) A kernel allocates a new region during.
A) fork
B) exec

C) shmget

D) all of these

13) A _______________ is an Array where each entry consists of an address of an


allocatable resource and the number of resource units available there.
A) map
B) list
C) table
D) page table
14) The process is ready to run, but _______________ must swap the process into main
memory before execution.
A) Kernel
B) Swapper
C) Scheduler
D) Process
15) ____________ is commonly called in it because it is responsible for initialization of new
process.
A) Process 0
B) Process 1
C) Process 2
D) None
16) Algorithm Xalloc is used to ____________
A) Allocate disk block
B) Allocate inode
C) Allocate data region
D) Allocate text region
17) A setuid program is an executable file that has the ____________ bit set in its permission
mode filed.
A) Setupid
B) Setuid
C) Both
D) ssetuid
18) When a process accesses a page that is not a part of its working set, it incurs
____________ page fault.
A) validity
B) invalid
C) modification
D) recent
19) Moving between kernel and user mode is called as ____________
A) Change in mode B) Context switch C) Kernel running
D) None of above
20) ____________ translates a file system address, consisting of a logical device number
and block number, to a particular sector on the disk.
A) Stream
B) Disk Driver
C) Strategy Interface D) Kernal
______________

Set A

*SLR-BB-231*

-3-

SLR-BB 231

Seat
No.

T.E. (CSE) (Part II) Examination, 2014


OPERATING SYSTEM II
Day and Date : Thursday, 27-11-2014
Time : 10.00 a.m. to 1.00 p.m.

Marks : 80

Instructions : 1) Attempt all questions.


2) Assume data if necessary.
3) Figures to the right indicate full marks.
SECTION I
2. Explain following terms in the light of hardware assumption for Unix OS :

10

A) Kernel mode
B) User mode
C) Interrupts
D) Exception
E) Execution levels.
3. Explain what are inodes ? Describe in-core inodes and disk inodes.

10

OR
Explain in details allocation of disk blocks (alloc algorithm) with neat diagram and example.
4. Write a short note on (any four) :

(45=20)

A) iget system call


B) Super block of file system
C) History of Unix OS
D) Creation of Special File
E) Advantages and disadvantages of cache
F) File system maintenance.
Set A

SLR-BB 231

-4-

*SLR-BB-231*

SECTION II
5. Explain process State transition diagram in detail.

10

6. How process synchronize its execution with termination of child process ? Explain with
algorithm.

10

OR
Explain how Kernel manages space on swap device in detail.
7. Answer any four :

(45=20)

A) Explain operations performed by kernel in handling interrupts and exceptions.


B) Write and explain allocreg algorithm.
C) What is signal ? How are they classified in Unix System V.
D) With example explain Fair-Share Scheduler.
E) What is terminal driver ? List functions of line discipline.
F) Write short note on exit system call.
_____________________

Set A

SLR-BB 232

*SLRBB232*
Seat
No.

Set
T.E. (C.S.E.) (Part II) Examination, 2014
SOFTWARE ENGINEERING

Day and Date : Friday, 28-11-2014


Time : 10.00 a.m. to 1.00 p.m.

Max. Marks : 100

Instructions : 1) Figures to the right indicate full marks.


2) Q. No. 1 is compulsory. It should be solved in first 30 minutes in
Answer book Page No. 3. Each question carries one mark.
3) Answer MCQ/Objective type questions on Page No. 3 only.
Dont forget to mention, Q.P. Set (A/B/C/D) on Top of Page.
MCQ/Objective Type Questions
Duration : 30 Minutes

Marks : 20

1. Choose the correct alternative :


1) KPA stands for ____________
a) Key principal area
c) Key product area
2) COCOMO stands for ___________
a) Cost control mode
c) Concurrent control model

(201=20)
1
b) Key process area
d) None of these
1
b) Constructive cost model
d) None of these

3) The major verification and validation activation for software development are
a) Inspection
b) Reviews
c) Testing
d) All of the above
4) ____________ is a permanent repository of the process performance data from
projects which is used for project planning, estimation etc.
a) Process database
b) Product database
c) Both
d) None of these

5) _____________ essentially provide the structure of document in which the


output of a process or step can be captured.
a) Process assets b) Templates
c) Defect
d) None of these

6) ____________ is the number of defects per unit size in the delivered software.
a) Quality
b) Delivered defect density
c) Both
d) None

7) _______________ are those events or conditions that may occur and if occurred
has negative effect on project.
a) Risk
b) Failure
c) UPS
d) None
8) SDLC framework for ______________
a) Requirement
b) Design
c) Code and test

1
1

d) All of above
P.T.O.

SLR-BB 232

*SLRBB232*

-2-

9) _____________ between modules is the strength of interconnections between


Modules.
a) Coupling
b) Cohesion
c) Both
d) None of these
10) Which phase is not available ?
a) Abstraction
b) Coding

1
1

c) Testing

11) A ____________ defines a possible set of objects.


a) Class
b) Object
c) Template

d) Maintenance
1
d) Variable

12) __________ and ___________ are the basic building blocks of an object oriented
design (OOD).
a) Identity and state
b) State and behaviour
c) Classes and objects
d) Classes and data
13) __________ polymorphism in which wherever an object of a superclass can
be used, objects of subclasses can be used.
a) Object polymorphism
b) Data polymorphism
c) Dynamic polymorphism
d) All of above

14) _____________ is responsible for coordinating all the process activities, including
process definition, process improvement and process deployment.
a) Software Engineering process Group (SEPG)
b) Software technology
c) Quality manager
d) None of these
15) ____________ are the most effective and commonly used method for
identifying defects.
a) Reviews
b) Identity
c) Polymorphism d) Development

18) UML stands for _____________


a) Unified make up language
c) Both a) and b)

b) Unified modeling language


d) None of these

19) DFD shows ____________


a) System flow
c) Information flow

b) Interface flow
d) Architecture design

16) Reviewers use ______________ to record all defects or issues found during their
independent reviews.
a) Self-preparation log
b) Group meeting log
c) Normal log
d) Identity log
17) _____________ log, issues are recorded as they arise, along with related
information.
a) Issue log
b) System log
c) Project log
d) Channel log

1
1

20) LOC stands for ______________


a) Line of code
b) Line of compiler
c) Line of composite
d) None of these
______________

Set A

*SLRBB232*

-3-

SLR-BB 232

Seat
No.

T.E. (C.S.E.) (Part II) Examination, 2014


SOFTWARE ENGINEERING
Day and Date : Friday, 28-11-2014

Marks : 80

Time : 10.00 a.m. to 1.00 p.m.


Instruction : Figures to the right indicate full marks.
SECTION I
2. Attempt any four :

(45=20)

a) Explain process assets with body for knowledge system.

b) What is meant by process database and explain contents of PDB.

c) Explain in detail spiral model.

d) What is software process and explain components of software process.

e) Explain in detail abstraction.

f) Explain on defect prevention planning.

3. Attempt any one :


a) What is SRS ? Explain characteristics of SRS and components of SRS.

10

OR
b) What is risk ? Explain risk management in detail.
4. Write short note on (any two) :

10
(25=10)

1) COCOMO model.

2) Validation for SRS.

3) The process capability Baseline.

5
Set A

SLR-BB 232

-4-

*SLRBB232*

SECTION II

5. Attempt any four :

(45=20)

a) Explain in detail project tracking ?

b) Explain object oriented analysis and object oriented design in detail.

c) Explain coupling and cohesion for design concepts.

d) Explain data collection in detail.

e) Explain milestone analysis in project monitoring and control.

f) What is cause-effect graphing technique ? Explain it with suitable example.

6. Attempt any one :


a) Explain configuration management process in detail.

10

OR
b) Explain Black-box testing in detail.

10

7. Write short note on (any 2) :


a) Unit testing

b) SEPG

c) Process monitoring and Audit.

_____________________

Set A

SLR-BB 233

*SLR-BB-233*
Seat
No.

Set

T.E. (CSE) (Part II) Examination, 2014


ARTIFICIAL INTELLIGENCE
Day and Date : Saturday, 29-11-2014
Time : 10.00 a.m. to 1.00 p.m.

Max. Marks : 100

Instructions : 1) Q. No. 1 is compulsory. It should be solved in first 30 minutes


in Answer book Page No. 3. Each question carries one mark.
2) Answer MCQ/Objective type questions on Page No. 3 only.
Dont forget to mention, Q.P. Set (A/B/C/D) on Top of Page.
MCQ/Objective Type Questions
Duration : 30 Minutes

Marks : 20

1. Choose the correct alternative :


1) Knowledge does not possesses following property
a) Hard to characterize accurately
b) Constantly changing
c) Voluminous
d) It does not differs from data by being organized in a way that it will be
used

20

2) _____________ provides a way of solving complex problem by exploiting the


structure of the objects that are involved.
a) Technique
b) Abstraction
c) Knowledge
d) JAVA
3) Conversion from informal description to formal description with program is
known as
a) formulation
b) operationalization
c) language mapping
d) none of above
4) Following is one of the partially commutative nonmonotonic system
a) Theorem proving
b) Robot navigation
c) Chemical synthesis
d) Bridge
5) Depth-first search tree with backtracking technique is used to implement
a) Simple Hill Climbing
b) Steepest-Ascent Hill Climbing
c) Generate and Test
d) Simulated Annealing
6) _____________ is useful for representing the solution of problem that can be
solved by decomposing them into set of smaller problem.
a) OR-Graph
b) AND-Graph
c) AND-OR Graph
d) None of above
P.T.O.

SLR-BB 233
7) Hill Climbing is the variant of
a) Generate and test
c) Best first search

*SLR-BB-233*

-2-

b) Depth-first search
d) Means-ends analysis

8) The problem of representing the facts that changes as well as those that do
not is known as
a) Means-ends analysis
b) Frame problem
c) Semantic net
d) Procedural knowledge
9) Rules that describe actions that should be taken if value ever become known.
These rules are called
a) backward
b) if-needed rule c) forward
d) none of above
10) ____________ level at which facts are described
a) Knowledge
b) Symbol
c) Both a) and b) d) View level
11) A script consists of ____________
a) Slots
b) Classes

c) Variables

d) All

12) Default reasoning is also called as


a) monotonic
b) complex
c) nonmonotonic d) none of these
13) In conceptual dependency, representation of actions are built from a set of
____________
a) secondary acts b) primitive acts c) non primitive d) none of these
14) An ____________ attributes are used in predicate logic.
a) Instance and isa
b) Facts and isa
c) Objects and is
d) None of these
15) Experts system use ____________ representation for knowledge.
a) architectural
b) structural
c) symbolic
d) formal
16) ____________ is structured set of closely related knowledge.
a) Networks
b) Scripts
c) Frames
d) Nodes
17) Frame is a collection of ____________ and associated values that describe
some entity in the world.
a) attributes
b) knowledge
c) nodes
d) none of these
18) In semantic net information is represented as a set of ____________ connected
to each other.
a) nodes
b) files
c) links
d) all of these
19) ____________ is a very large knowledge base project aimed at capturing
human commonsense knowledge.
a) Scripts
b) Slot
c) CYC
d) None of these
20) A ____________ consists of set of slots.
a) Frame
b) CYC
c) Script
______________

d) None
Set A

*SLR-BB-233*

-3-

SLR-BB 233

Seat
No.

T.E. (CSE) (Part II) Examination, 2014


ARTIFICIAL INTELLIGENCE
Day and Date : Saturday, 29-11-2014

Marks : 80

Time : 10.00 a.m. to 1.00 p.m.

SECTION I
2. Attempt any four :

(45=20)

1) What question arises to study AI problem ? Explain physical symbol system


hypothesis.
2) Write short note on AI Technique.
3) Explain the Best-First search algorithm with example.
4) Write short note on hill climbing with their variation.
5) Write short note on production systems.
6) Explain the inferential knowledge and procedural knowledge with example.
3. Solve any one :

10

1) Explain A* Algorithm in detail.


2) List out issues related to knowledge representation. Explain in detail
relationships among attributes.
4. Explain water jug problem as a state space search.

10

Set A

SLR-BB 233

-4-

*SLR-BB-233*

SECTION II
5. Attempt any four of the following :

(45=20)

1) Describe in detail with different examples, representation of simple facts in


predicate logic.
2) Write a note on monotonic reasoning.
3) Write a note on conceptual dependency in detail.
4) Explain characteristics of expert system in detail.
5) Explain slot and filler structures.
6. Attempt any two of the following :

(102=20)

1) Explain Bayesian Network along with its conditional probabilities.


2) What is IsA relationship ? Explain with example.
3) What factors determine the selection of forward or backward reasoning
approach for an AI problem ?
_____________________

Set A

SLR-BB 234

*SLRBB234*
Seat
No.

Set

B.E. (CSE) (Part I) Examination, 2014


ADVANCED COMPUTER ARCHITECTURE
Day and Date : Tuesday, 2-12-2014
Time : 3.00 p.m. to 6.00 p.m.

Max. Marks : 100

Instructions : 1)
2)
3)
4)

Question 1 is objective type.


All questions are compulsory.
Figures to the right indicate full marks.
Answers to Section I and II should be written in the same answer
book.
5) Q. No. 1 is compulsory. It should be solved in first 30 minutes in
Answer book Page No. 3. Each question carries one mark.
6) Answer MCQ/Objective type questions on Page No. 3 only. Dont
forget to mention, Q.P. Set (A/B/C/D) on Top of Page.
MCQ/Objective Type Questions

Duration : 30 Minutes

Marks : 20

1. Choose the correct answer :

20

1) There are a number of techniques for converting loop-level parallelism into instruction
level parallelism, such techniques work by
a) Statically by the compiler
b) Dynamically by the hardware
c) Both a) and b)
d) None of the above
2) The simplest and most common way to increase the ILP is to exploit parallelism among
iterations of a loop. This type of parallelism is often called
a) Loop level parallelism
b) Instruction level parallelism
c) Both a) and b)
d) None of the above
3) Antidependence is a type of
a) Control dependence
c) Name dependence

b) Data dependence
d) True data dependence

4) Which of the following is not a hazard ?


a) RAW
b) WAW

c) WAR

d) RAR

5) The type of limit to the gains that can be achieved by loop unrolling is
a) A decrease in the amount of overhead amortized with each unroll
b) Code size limitations
c) Compiler limitations
d) All of the above
6) A compilers ability to perform basic pipeline scheduling depends on
a) The amount of ILP available in the program
b) The latencies of the functional units in the pipeline
c) Both a) and b)
d) None of the above
P.T.O.

SLR-BB 234

*SLRBB234*

-2-

7) Which of the following issue structure is concerned to VLIW ?


a) Static
b) Dynamic
c) Primarily static
d) Both static and dynamic
8) The element of vector, may be a
a) Floating point number
c) A logical value

b) An integer
d) All of the above

9) __________ is needed to determine the termination of a vector instruction.


a) Vector length
b) Address offset c) Base address
d) Operation code
10) In which architecture operands and results are retrieved indirectly from the main memory
through the use of a large number of vector or scalar registers ?
a) Register to register architecture
b) Memory to memory architecture
c) Both a) and b)
d) None of the above
11) Multiprocessors can be grossly-characterized by which of the following attribute ?
a) Multiprocessor is a single computer that includes multiple processors.
b) Processors may communicate and cooperate at different levels in solving a given
problem
c) The communication may occur by sending messages from one processor to the
other or by sharing a common memory
d) All of the above
12) Which of the following is an interface to other computer modules in case of a non
hierarchical loosely coupled multiprocessor system ?
a) Processor
b) I/O
c) Local memory
d) Channel and arbiter switch
13) Which of the following is a component of the Kmap in Cm* ?
a) Kbus
b) Linc
c) Pmap

d) All of the above

14) Which of the following bus connects two different Slocal switch ?
a) Intercluster bus
b) Local bus
c) Map bus
d) All of the above
15) Which of the following section delivers operation packets from the memory section to the
processing section in case of static data flow computer architecture ?
a) Arbitration network
b) Control network
c) Distribution network
d) None of the above
16) Which of the following is required to construct the network ?
a) Switches
b) Arbiters
c) Distributors
d) All of these
17) In dynamic machines, data tokens are _________ to allow multiple tokens to appear
simultaneously on any input arc of an operator node.
a) Tagged
b) Labelled
c) Colored
d) All of the above
18) Which of the following aspect is concerned to data flow computers ?
a) Projected performance
b) Matching technology
c) Programming productivity
d) All of the above
19) Uniform memory access model is a type of
a) Shared memory multiprocessors
b) Distributed memory architecture
c) Both a) and b)
d) Vector super computer
20) BBN Butterfly is a type of _________ model.
a) NUMA
b) CDMA
c) UMA

______________

d) All of these

Set A

*SLRBB234*

-3-

SLR-BB 234

Seat
No.

B.E. (CSE) (Part I) Examination, 2014


ADVANCED COMPUTER ARCHITECTURE
Day and Date : Tuesday, 2-12-2014
Time : 3.00 p.m. to 6.00 p.m.

Marks : 80

Instructions : 1) Figures to the right indicate full marks.


2) Answers to Section I and II should be written in the same answer
book.
SECTION I
2. Attempt any four :
1) What is instruction level parallelism ? Explain the possible data hazards.
2) What is name dependences ? Explain antidependence and output dependence.
3) Write a short note on loop unrolling.
4) Write a note on basic VLIW approach.
5) Explain the following terms vector length and vector stride.

(54=20)

3. Attempt any one :

10

1) Draw and explain basic vector architecture.


2) Explain the basic structure of FP unit using Tomasulos algorithm and extended to handle
speculation.
4. Write a note on effectiveness of compiler vectorization.

10

SECTION II
5. Attempt any four :

(54=20)

1) Explain with neat diagram about how communication between processes in a


multiprocessor environment takes place ?
2) Write a note on Processor Memory Interconnection Network (PMIN) and IO/P
Interconnection Network (IOPIN)
3) Write a note on static data flow computer architecture with neat diagram.
4) Mention any five important technical problems that remain to be solved in case of data
flow computer.
5) Draw and briefly explain distributed memory architecture.

Set A

SLR-BB 234

-4-

*SLRBB234*

6. Attempt any one :

10

1) Explain the different steps in an intercluster memory access with neat diagram.
2) List the useful properties of data flow language and draw a data flow graph for the
computation of
U = f(A, B) = (A (A + B) (A + B) B) (A (A + B) (A + B)).
7. With neat diagram explain the components of the Kmap in Cm*.

10

_____________________

Set A

SLR-BB 235

*SLRBB235*
Seat
No.

Set

B.E. (CSE) (Part I) Examination, 2014


NETWORK SECURITY
Day and Date : Thursday, 4-12-2014
Time : 3.00 p.m. to 6.00 p.m.

Max. Marks : 100

N. B. : 1) Figures to right indicate full marks.


2) Q. No. 1 is compulsory. It should be solved in first 30 minutes
in Answer Book Page No. 3. Each question carries one mark.
3) Answer MCQ/Objective type questions on Page No. 3 only.
Dont forget to mention, Q.P. Set (A/B/C/D) on Top of Page.
MCQ/Objective Type Questions
Duration : 30 Minutes

Marks : 20

1. Choose correct alternatives :


20
1) A process that is designed to detect, prevent or recover from security attack is
a) Security
b) Security Mechanism
c) Security Services
d) Threat
2) OSI security architecture focuses on
a) Security Attacks
b) Security Mechanism
c) Security Services
d) All the above
3) The area of Cryptography and Cryptanalysis together are called
a) Cryptology
b) Security
c) Cipher system d) All of above
4) _________ enables selection of particular physically secure routes for certain
data.
a) Encipherment
b) Digital signature
c) Access control
d) Routing control
5) Playfair cipher is __________ cipher.
a) Monoalphabetic
b) Polyalphabetic
c) Multiletter
d) Blinding
6) Trying every possible key until an intelligible translation of the cipher text
into plaintext is obtained is called
a) Transposition
b) Substitution
c) Brute force attack
d) Notarization
7) In block cipher, the impact of larger block size is
a) Greater Security
b) Decrease encryption/decryption speed
c) Easy for hacker to hack
d) Both a) and b)
P.T.O.

SLR-BB 235

-2-

*SLRBB235*

8) Symmetric encryption is also known as


a) Single-key Encryption
b) Conventional Encryption
c) Secrete-key Encryption
d) All of above
9) Rail fence technique is
a) Substitution technique
b) Transposition technique
c) Mono alphabetic cipher
d) None of these
10) In __________ mode of operation, the input to the encryption is the XOR of
current plaintext block and the preceding cipher text block.
a) Cipher Block Chaining
b) Cipher Feedback Mode
c) Electronic Code Book
d) None of above
11) We trust a digital certificate because it contains
a) Owners Public Key
b) Owners Signature
c) CAs Public Key
d) CAs Signature
12) PGP (Pretty Good Privacy) provides
a) Confidentiality b) Authentication c) Both a) and b) d) None of above
13) Triple DES algorithm uses the key size of
a) 56 bits
b) 64 bits
c) 168 bits
d) 256 bits
14) Anderson identified three classes of intruder namely Masquerader, Clandestine
user and ____________
a) Suspicious user
b) Misfeasor
c) Cliff Stoll
d) None of above
15) Simple Mail Transfer Protocol is an example of
a) One-way Authentication
b) Two-way Authentication
c) Three-way Authentication
d) None of these
16) In __________ the statistical structure of the plain text is dissipated into
long-range statistics of the cipher text.
a) Confusion
b) Diffusion
c) Substitution
d) Permutation
17) Password selection strategies uses which of the following technique ?
a) Computer-generated passwords b) Reactive password checking
c) Proactive password checking
d) User Education
18) __________ algorithm is used for key exchange only.
a) DSS
b) RSA
c) Diffie Hellman d) ECC
19) Diffie Hellman Protocol for key exchange is insecure from __________
a) Timing attack
b) Man-in-middle attack
c) Brute force attack
d) All above
20) In Kerberos to provide more secured authentication dialogue _______ is used.
a) Ticket grant server
b) Authentication server
c) Server
d) Hash value
______________
Set A

*SLRBB235*

-3-

SLR-BB 235

Seat
No.

B.E. (CSE) (Part I) Examination, 2014


NETWORK SECURITY
Day and Date : Thursday, 4-12-2014

Marks : 80

Time : 3.00 p.m. to 6.00 p.m.


N. B. : 1) All questions are compulsory.
2) Figures to right indicate full marks.
SECTION I
2. Attempt any four (each carries 5 marks) :

20

a) Explain and compare between stream and block cipher.


b) What is the purpose of S-boxes in DES algorithm.
c) Describe the Cipher Block Chaining (CBC) mode of block cipher.
d) Explain Playfair cipher.
e) Write a note on : DSS.
3. Attempt any one :

10

a) Describe the list all types of active attacks with the help of diagram.
b) Explain the RSA algorithm with the help of suitable example.
4. Describe with the help of diagram three basic uses of Message Authentication
Code.

10

Set A

SLR-BB 235

-4-

*SLRBB235*

SECTION II
5. Attempt any four (Each carries 5 marks) :

20

a) Describe and explain IP Sec Security Architecture.


b) Elaborate the transmission and reception of PGP Messages with diagram.
c) Draw and explain the IP Sec Authentication Header.
d) Describe privacy in Semantic Web.
e) Draw and explain fields of ESP of IP Sec.
6. Attempt any one :

10

A) Describe X.509 Authentication Service. Draw and explain X.509 formats.


B) What is SET ? Describe the key features of SET and SET participant with
diagram.
7. What is Intrusion Detection ? Explain different approaches to intrusion detection. 10
_____________________

Set A

SLR-BB 236

*SLRBB236*
Seat
No.

Set

B.E. (Computer Science and Engg.) (Part I) Examination, 2014


DISTRIBUTED SYSTEMS
Day and Date : Saturday, 6-12-2014
Time : 3.00 p.m. to 6.00 p.m.
Instructions :

Max. Marks : 100

1) Q. No. 1 is compulsory. It should be solved in first 30 minutes


in Answer Book Page No. 3. Each question carries one mark.
2) Answer MCQ/Objective type questions on Page No. 3 only.
Dont forget to mention, Q.P. Set (A/B/C/D) on Top of Page.
MCQ/Objective Type Questions

Duration : 30 Minutes
I.

Marks : 20

Choose correct alternative :

20

1) In ________ model there is no concept of home machine.


a) Workstation model
b) Workstation server model
c) Processor-Pool model
d) HYBRID model
2) Hybrid model combines the advantages of _______ and ___________
a) Workstation and workstation server models
b) Workstation and Processor models
c) Workstation-server and processor pool models
d) Workstation, workstation server and processor pool models
3) Workstation server model is popular because it is used for
a) Email
b) Editing jobs
c) Executing small programs
d) All
4) ___________model is used for massive computations.
a) Workstation model
b) Workstation server model
c) Processor-Pool model
d) HYBRID model
5) The performance metrics of distributed computing systems are
a) Response time and throughput of user processes
b) Space complexity and response time
c) Throughput of user processes and space complexity
d) Response time, throughput of user processes and space complexity
6) ____________ transparency means that users should not be able to know whether the
resource they need is local or remote.
a) Location Transparency
b) Access Transparency
c) Resource Transparency
d) Object Transparency
7) If the system is to be designed to tolerate k Byzantine failures, a minimum of
_____________ replicas are needed.
a) k
b) k + 1
c) k + 2
d) 2k + 1
P.T.O.

SLR-BB 236

*SLRBB236*

-2-

8) ______________ servers make crash recovery very easy.


a) Stateless servers
b) Stateful server
c) Neither a) nor b)
d) Both a) and b)
9) __________ feature of message-passing system ensures that messages will be delivered
correctly despite partial failure of process, machines or communication links.
a) Correctness
b) Reliability
c) Efficiency
d) Flexibility
10) In primitive Receive_any(process_id,message), the process_id belongs to
a) A specific sender process
b) Any sender process
c) Receiver process that initiates this primitive
d) Any receiver process
11) ___________ is an idempotent operation.
a) Getsqrt(64)
c) Increment(varname)

b) Read_next_record(filename)
d) Append_record(filename, record)

12) ____________ semantics should be used for application making a request to a booking
server to cancel an already booked seat.
a) At-least-once
b) Exactly-once
c) Last-one
d) None of the above
13) An efficient mechanism used to cope with the problem of lost and out-of-sequence
packets in multidatagram messages is
a) Stop-and-wait
b) BLAST protocol
c) Selective repeat
d) CBCAST protocol
14) _____________ semantics is used in application where response message from caller is
not important for the caller and it operates in LAN where there is high probability of
successful transmission of messages.
a) May-be Call
b) Last-one call
c) At-Least-Once
d) Exactly-Once
15) Indirect call method is used in __________
a) Binding at compile time
b) Binding At Link time
c) Binding at call time
d) All the above
16) For computer clocks based on quartz crystals, the drift rate is approximately equal to
_____________
a) 10 6
b) 10 8
c) 10 4
d) 106
17) For perfect clocks, the drift rate is ___________
a) dc/dt<1
b) dc/dt=1
c) dc/dt>1

d) dc/dt=0

18) Berkley algorithm is used for _____________


a) Active time server centralized algorithm
b) Passive time server centralized algorithm
c) Global averaging distributed algorithm
d) Localized averaging distributed algorithm
19) Happened before relation is
a) Reflexive
b) Irreflexive

c) Not transitive

20) Two events a and b are said to be concurrent if


a) a->b
b) b->a
c) a->c and c->b

d) Commutative
d) none of the above

______________
Set A

*SLRBB236*

-3-

SLR-BB 236

Seat
No.

B.E. (Computer Science and Engg.) (Part I) Examination, 2014


DISTRIBUTED SYSTEMS
Day and Date : Saturday, 6-12-2014

Marks : 80

Time : 3.00 p.m. to 6.00 p.m.

SECTION I
II. Answer any four :

20

a) Describe workstation server model with diagram.


b) What is buffering ? Explain Finite buffering strategy.
c) Explain server implementation of RPC mechanism.
d) What is causal ordering of messages ? Explain this concept with diagram.
e) Explain with example how to keep track of lost and out of sequence packets in a
multidatagram message transmission.
III. Answer any one :

10

a) Explain in detail different communication protocols used for RPC.


b) Explain the structure of message with diagram and explain the nonblocking send
and nonblocking receive primitives in message passing.
IV. What is group communication ? Explain following with respect to one-to-many
communication :

10

i) Group addressing.
ii) Message delivery to receiver processes.
iii) Buffered and unbuffered multicast.

Set A

SLR-BB 236

-4-

*SLRBB236*

SECTION II
V. Answer any four :

20

a) Explain advantages of process migration.


b) What is thread scheduling ? Explain different types of thread scheduling.
c) Explain Global averaging distributed algorithms for clock synchronization.
d) Explain the immutable-file sharing and the session semantics in distributed file
systems.
e) What is freezing time ? Explain the handling of co-processes in process
migration mechanism.
VI. Answer any one :

10

a) Explain in detail file-caching schemes in distributed file systems.


b) What are the design goals in MACH ? Sketch and explain the MACH system
architecture.
VII. Explain the different methods of address space transfer mechanism in

10

process Migration.
_____________________

Set A

SLR-BB 237-B

*SLRBB237B*
Seat
No.

Set

B.E. (Computer Science Engg.) (Part I) Examination, 2014


INFORMATION RETRIEVAL (Elective I)
Day and Date : Tuesday, 9-12-2014
Time : 3.00 p.m. to 6.00 p.m.

Max. Marks : 100

Instructions : 1)
2)
3)
4)

Figures to right indicate full marks.


All questions are compulsory.
Assume data wherever necessary.
Q. No. 1 is compulsory. It should be solved in first 30
minutes in Answer Book Page No. 3. Each question carries
one mark.
5) Answer MCQ/Objective type questions on Page No. 3 only.
Dont forget to mention, Q.P. Set (A/B/C/D) on Top of Page.
MCQ/Objective Type Questions

Duration : 30 Minutes

Marks : 20

1. Choose correct alternative(s) :


1) ________ model considers that index terms are present or absent in document.
a) Boolean
b) Vector
c) Probability
d) Fuzzy
2) The Boolean model is a simple retrieval model based on ________
a) Set theory
b) Boolean algebra
c) Both a) and b)
d) None of the above
3) In ________ retrieval, single erroneous object among thousands, means total
failure.
a) Information
b) Data
c) Multimedia
d) All of the above
4) ________ is the function of the relevant document which has been retrieved.
a) Retrieval
b) Precision
c) Recall
d) All of the above
5) In vector model, inverse document frequency of Ki be given by
a) fi,j
b) freqi,j / maxlfreql,j
c) logni/N
d) log N/ni
6) In evaluating retrieval performance of interactive session ________ is the
critical aspects.
a) User effort
b) Characteristics of interface design
c) Guidance provided by the system d) All of the above
7) The keyword based queries are popular because they are ________
a) Intuitive
b) Easy to express
c) Allow for fast ranking
d) All of the above

P.T.O.

SLR-BB 237-B

-2-

*SLRBB237B*

8) Index terms are generally ________


a) Nouns
b) Adverbs
c) Adjectives
d) Connectives
9) A document can also have information about itself is called ________
a) Self data
b) Self information
c) Metadata
d) None of the above
10) ________ specifies two dimensional data interchange standard which allows
graphical data to be stored and exchanged.
a) MPEG
b) JPEG
c) VRML
d) CGM
11) ________ provides a indexing mechanism and query interface to the data.
a) Gatherer
b) Broker
c) Object cache
d) Replication manager
12) Which of the following term does not represent a precise value ?
a) Normal
b) Typical
c) Unacceptable
d) All of the above
13) Birkhoffs theory claims that interesting signals, such as musical score and
other works of art, consist of ________ noise.
a) Brown
b) Black
c) Pink
d) Both a) and b)
14) ________ provides security and can respond to a repository access protocol
in digital library.
a) Digital objects b) Handles
c) Repositories d) Both a) and c)
15) In multos models, ________ allows semantic oriented description of a
document content.
a) Logical
b) Layout
c) Conceptual
d) None of the above
16) To solve Harvest problems, Harvest introduced some main elements that
are ________
a) Gatherer
b) Brokers
c) Both a) and b)
d) None of the above
17) Image data type of store images is supported by ________
a) ORACLE
b) ACCESS
c) Sybase SQL d) SQL3
18) ________ clause allows one to express structural predicate in MULTOS query
language.
a) Scope
b) Type
c) Condition
d) With
19) In Hot Bot, sequence of words is a reference to WebPages.
a) Having all words
b) Having at least one of those words
c) None of them
20) In skewed energy spectrum O(Fb) ________ noise represents successfully
stock movements and exchange rates.
a) Brown
b) Black
c) Pink
d) White
______________
Set A

*SLRBB237B*

-3-

SLR-BB 237-B

Seat
No.

B.E. (Computer Science Engg.) (Part I) Examination, 2014


INFORMATION RETRIEVAL (Elective I)
Day and Date : Tuesday, 9-12-2014
Time : 3.00 p.m. to 6.00 p.m.

Marks : 80

Instructions : 1) Figures to right indicate full marks.


2) All questions are compulsory.
3) Assume data wherever necessary.
SECTION I
2. Attempt any four :

(45=20)

1) List the difference between data retrieval and information retrieval.


2) Explain the formal characterization of IR model.
3) Explain model based on non overlapping list.
4) Explain information retrieval process with a neat sketch.
5) Explain sequential searching algorithm based on bit parallelism.
6) Write a short note on information theory.
3. Attempt any one :

10

1) What are different steps of suffix array construction for large texts ?
2) Explain 3 main types of structures found in text database.
4. How BM algorithm works for following text and pattern ?

10

Text : bacbabababacaca
Pattern : ababaca.
SECTION II
5. Attempt any four :

(45=20)

1) Explain MULTOS query format.


2) What type of document should be supported by digital library ?
3) Write a short ntoe on cross-talk problem.
Set A

SLR-BB 237-B

-4-

*SLRBB237B*

4) Explain Biokleisli system architecture.


5) Explain Crawler-index architecture.
6) Explain different multimedia query predicates with example.
6. Attempt any one :

10

1) What is predicate ? Explain different types of predicates used in multimedia


IR system with example.
2) Explain two main tasks required in multimedia IR system w.r.t. data
modelling issue.
7. Explain architectural issues of Digital Library in detail.

10

_____________________

Set A

*SLRBB237A*

SLR-BB 237-A

Seat
No.

Set

B.E. (Computer Sci. and Engg.) (Part I) Examination, 2014


OBJECT ORIENTED MODELING AND DESIGN (Elective I)
Day and Date : Tuesday, 9-12-2014
Time : 3.00 p.m. to 6.00 p.m.

Max. Marks : 100

Instructions : 1) Figures to right indicate full marks.


2) Q. No. 1 is compulsory. It should be solved in first 30 minutes in Answer
Book Page No. 3. Each question carries one mark.
3) Answer MCQ/Objective type questions on Page No. 3 only. Dont forget
to mention, Q.P. Set (A/B/C/D) on Top of Page.
MCQ/Objective Type Questions
Duration : 30 Minutes
Marks : 20
1. Solve the following MCQ :
1) Inheritance is ___________
A) The use of multiple instances of a class
B) The sharing of hierarchical structure between classes
C) The sharing of attributes and operations among classes
D) Not a hierarchical relationship among classes
2) Which one is correct match ?
a) Problem statement
i) Implementation
b) Architecture
ii) Object design
c) Design model
iii) System design
d) Programming language
iv) Analysis
A) a ii, b iv, c i, d iii
B) a i, b iv, c iii, d ii
C) a iv, b iii, c ii, d i
D) a iii, b ii, c i, d iv
3) An instance diagram describes ____________
A) Object classes
B) Object instances
C) Functional aspect
D) Events within objects
4) Aggregation is ___________ relationship between object classes.
A) Part whole
B) Qualified
C) Unqualified
D) Hierarchical
5) A ______ is a sequence of events that occurs during one particular execution of a system.
A) State transition
B) Scenario
C) Trigger
D) Qualifier
6) I) In dynamic model aggregation implies absence of concurrency.
II) The aggregate state correspond to the combined states of all the component diagrams.
A) Only II is true
B) Only I is true
C) Both are true
D) Both are false
7) Data flow diagram is part of ___________
A) Object model
B) Functional model
C) Dynamic model
D) All of them
8) Which of the following drives the data flow graph by producing or consuming values ?
A) Event
B) Processes
C) Actor
D) Triggers
9) Which one of the following should be supported by programming language for OMT ?
i) Data structure
ii) Dynamic flow of control
iii) Functional transformation
A) Only i
B) Both i and ii
C) All of them
D) None
10) An activity is associated with __________
A) Event
B) Condition
C) Transaction
D) State
11) For understanding control mechanisms, such as user interfaces and device controllers, which one
of the following UML artifacts is the MOST useful ?
A) Interaction Diagrams
B) Activity Diagrams
C) Package Diagrams
D) State Diagrams
12) Which one of the following highlights the roles each object plays in an interaction model ?
A) Sequence Diagrams
B) Collaboration Diagrams
C) Deployment Diagrams
D) Package Diagrams

20

P.T.O.

SLR-BB 237-A

-2-

*SLRBB237A*

13) Which one of the following is true ?


A) In the Dynamic Model, every trigger must map to an operation in the interface of a class.
B) Every arrow incident on an object in an Interaction Model represents an operation that must be
in the interface of a class
C) Both A and B are true
D) Both A and B are false
14) What methods must be implemented by the CreditProcessor class in the payment sequence diagram ?
A) checkCredit, generateConfirmationCode, displayConfirmation
B) checkCredit, generateConfirmationCode
C) checkCredit, generateConfirmationCode, reserveSeat
D) checkCredit, reserveSeat, displayConfirmation

15) Consider the following statements for UML _________


S1 : Interface cannot have direct instances.
S2 : One can also associate signals with an interface.
A) Only S1 is TRUE
B) Only S2 is TRUE
C) All are FALSE
D) All are TRUE
16) In UML one can indicate that there are more attributes or properties than shown by ending each list
with an __________
A) Ellipsis
B) Comma
C) Dollar
D) Exclamation
17) A _________ is an interaction diagram that emphasizes the structural organization of the object that
send and receive messages.
A) Sequence diagram
B) Activity diagram
C) Communication diagram
D) State diagram
18) For the Classes in UML, qualified name is the class name __________
A) Prefixed by the name of the owner of that class
B) Prefixed by the name of the super class
C) Prefixed by the name of the package in which that class lives
D) Prefixed by the name of the instance of that class.
19) Classification in OOD means ___________
A) Data is quantized into discrete, distinguishable entities
B) Object with same structure and behavior are grouped together
C) Ability to behave differently on different classes
D) None of the above
20) A ___________ is a diagram that shows the configuration of run time processing nodes and the
artifacts that live on them.
A) Collaboration diagram
B) Artifact diagram
C) Communication diagram
D) Deployment diagram

______________

Set A

*SLRBB237A*

-3-

SLR-BB 237-A

Seat
No.

B.E. (Computer Sci. and Engg.) (Part I) Examination, 2014


OBJECT ORIENTED MODELING AND DESIGN (Elective I)
Day and Date : Tuesday, 9-12-2014
Time : 3.00 p.m. to 6.00 p.m.
Instructions :

Marks : 80

1) All questions are compulsory.


2) Figures to right indicate full marks.
SECTION I

2. Solve the following questions (any 4) :

(54=20)

A) Explain the object oriented language features.


B) Explain in details how OMT model can be implemented using programming language and database
systems.
C) How operations are specified in functional modeling ?
D) Explain with respect to dynamic model.
1) Entry and exit action
2) Internal action.
E) What is metadata in object model ? Explain.
F) Explain the grouping constructs in object model.
3. Explain with neat diagram the analysis for object diagram compiler (Draw basic models and
explain).

10
OR

3. Explain the concept of state and event. Also wish suitable example explain state diagram.
4. Explain with suitable example modeling as a design technique.

10

Set A

SLR-BB 237-A

-4-

*SLRBB237A*

SECTION II
5. Solve any four :

(54=20)

A) Explain in brief Modeling flow of control.


B) Explain the concept of package. How it is represented graphically ?
C) What are building Blocks of UML ?
D) How do we Model the Distribution of Responsibilities in a System ?
E) Explain with example modeling a logical database schema.
F) Explain Organizing Attributes, Operations and Responsibilities.
6. What are interface, type and role ? How static and dynamic types are modeled ?

10

OR
6. Explain how components, classes and interfaces are modeled in architectural model.
7. Explain in details with suitable example and diagram Kinds of Events in Behavioral model.

10

_____________________

Set A

SLR-BB 237-C

*SLRBB237C*
S

B.E. (Computer Science and Engineering) (Part I) Examination, 2014


Elective I : HUMAN COMPUTER INTERFACE
Day and Date : Tuesday, 9-12-2014
Time : 3.00 p.m. to 6.00 p.m.

Max. Marks : 100

Instructions : 1) Q. No. 1 is compulsory. It should be solved in first 30 minutes in


Answer Book Page No. 3. Each question carries one mark.
2) Answer MCQ/Objective type questions on Page No. 3 only. Dont
forget to mention, Q.P. Set (A/B/C/D) on Top of Page.
MCQ/Objective Type Questions
Duration : 30 Minutes

Marks : 20

1. MCQ (Each 1 mark).

20

1) Which one of the following is not the element of the WIMP interface ?
a) Icons
b) Menus
c) Pointers
d) Buttons
2) What is the study of the physical characteristics of the interaction ?
a) Interaction framework
b) Ergonomics
c) Interaction domain
d) None
3) Which is/are goal/s of evaluation ?
a) To assess the extent and accessibility of the systems functionality
b) To assess users experience of the interaction
c) To identify any specific problem with the systems
d) All above
4) Which one of the following is not Evaluation through expert analysis ?
a) Cognitive Walkthrough
b) Experimental Evaluation
c) Heuristic Evaluation
d) Model-based Evaluation
5) Which one of the following is Evaluation through user participation ?
a) Query techniques
b) Heuristic Evaluation
c) Model-based Evaluation
d) All above
6) Learnability includes
a) Observability

b) Customizability

c) Predictability

d) None

7) Which refers to the users ability to adjust the form of input and output ?
a) Adaptability
b) Predictability
c) Customizability
d) None
8) What can be measured using photometer ?
a) Hue
b) Luminance
c) Whiteness

d) None
P.T.O.

SLR-BB 237-C

*SLRBB237C*

-2-

9) In what memory information remains very briefly, in the order of 0.5 seconds ?
a) Short term memory
b) Long term memory
c) Iconic memory
d) None
10) Human can hear frequencies from
a) 20 Hz to 15 KHz
c) 30 KHz to 50 KHz

b) 20 KHz to 50 KHz
d) 30 Hz to 50 KHz

11) Pitch is frequency


a) Sound

c) Intensity

b) Light

d) Noise

12) In memory, recall is


a) Self generated
c) Information from memory

b) Information from brain


d) None of the above

13) Reasoning types


a) Reason, understand, inference
c) Cumulative, assimilative, conclusive

b) Deductive, inductive, abductive


d) None of the above

14) A persona in the context of goal-oriented interaction design


a) Description of an example user
b) Represents a sedimentary type of user
c) Is a real person
d) Should represent an average user
15) Colour is regarded to be made of three components
a) Red, Green, Blue
b) Cyan, Magenta, Yellow
c) Hue, Intensity, Saturation
d) None of the above
16) Affordables are
a) The cost of buying a user interface component
b) Possibilities for using an object or interface
c) User settable preferences
d) The completion times for a typical task
17) Waterfall model, in which each step must be completed before the next step can be
started is a type of ?
a) Incremental model
b) Linear model
c) Iterative model
d) Analytical model
18) Which of the following is NOT a computer output device ?
a) Joystick
b) Monitor
c) Head Mounted Display
d) Speakers
19) Design phase usually
a) Top-down
b) Bottom-up

c) Random

20) The goals of HCl are


a) Usability and user experience
c) Usability experience and goals

b) Task and goals


d) None of the above

d) None of the above

______________

Set A

*SLRBB237C*
S

-3-

SLR-BB 237-C

B.E. (Computer Science and Engineering) (Part I) Examination, 2014


Elective I : HUMAN COMPUTER INTERFACE
Day and Date : Tuesday, 9-12-2014

Marks : 80

Time : 3.00 p.m. to 6.00 p.m.

SECTION I
2. Answer the following (any five) :

(58=40)

1) What is ubiquitous computing in HCl ?


2) What is interaction framework ?
3) Explain the process of design.
4) What are limitations on interactive performance ?
5) State and explain scenarios.
6) Explain design space analysis.
SECTION II
3. Answer the following (any five) :

(58=40)

1) Explain programming the application.


2) What is User Interface Management System (UIMS) ?
3) Explain 3 state models.
4) State and explain Normans seven principles for transforming difficult task into simple one.
5) What is gesture recognition ?
6) Explain methodology for choosing an evaluation method.
_____________________

Set A

Set A

SLR-BB 24

*SLRBB24*
Seat
No.

Set

S.E. (Civil) (Part I) Examination, 2014


ENGINEERING GEOLOGY
Day and Date : Saturday, 20-12-2014
Time : 10.00 a.m. to 1.00 p.m.

Max. Marks : 100

Instructions : 1) Q. No. 1 is compulsory. It should be solved in first 30 minutes


in Answer Book Page No. 3. Each question carries one mark.
2) Answer MCQ/Objective type questions on Page No. 3 only.
Dont forget to mention, Q.P. Set (A/B/C/D) on Top of Page.
MCQ/Objective Type Questions
Duration : 30 Minutes

Marks : 20

1. Choose the correct answer :

20

1) The vesicles of volcanic rocks filled by the secondary minerals are called as
____________ structure.
a) Vesicular
b) Xenolithic
c) Amygdaloidal d) Flow
2) The maximum content of volcanic gases is
a) Carbon dioxide
b) Carbon monoxide
c) Steam
d) Nitrogen
3) The chemical composition of inner core is
a) Nickel and Cadmium
b) Iron and Nickel
c) Silica and Aluminium
d) Magnesium and Silica
4) ____________ is the depositional features of the old river.
a) Deltas
b) Waterfall
c) Potholes
d) Mesa and Butte
5) The average density of the Earth is ___________
a) 3.3 g/cc
b) 5.517 g/cc
c) 4.3 g/cc

d) 5.37 g/cc

6) Axial plane in the symmetrical fold is ___________


a) Horizontal
b) Vertical
c) Inclined

d) Perforated

7) Columnar joints are found in ___________ rocks.


a) Basalt
b) Slate
c) Granite

d) Sandstone

8) Maximum angle of inclination of layer of a rock with the horizontal is called


as ________
a) Apparent dip
b) Low dip
c) True dip
d) Dip
P.T.O.

SLR-BB 24

-2-

*SLRBB24*

9) Sedimentary rocks shows _________ texture.


a) Porphyritic
b) Granitic
c) Gneisses

d) Clastic

10) The hardness of orthoclase mineral is ___________


a) Five
b) Six
c) One

d) Seven

11) _________ is the dam, the major part of the thrust forces are transmitted on
to the abutment.
a) Gravity dam
b) Arch dam
c) Embarkment dam
d) Earth fill dam
12) __________ are the lines which joins equal intensity of earthquake waves.
a) Seismic lines
b) P-wave lines
c) S-wave lines
d) Isoseismal lines
13) Rock formation which is porus, permeable and yield sufficient quantity of
water is called
a) Aquifer
b) Aquiclude
c) Aquitard
d) Aquifuse
14) Which one of the following is the oldest type of drilling ?
a) Rotatory drilling
b) Auger drilling
c) Calyx drilling
d) C-Auger drilling
15) The excessive quantity of rock broken down beyond the perimeter of tunnel
is known as __________
a) Caving
b) Excavation
c) Overbreak
d) None of these
16) The earthquake epicentre will be at the point of intersection of the _________
circles.
a) Two
b) Three
c) Four
d) Five
17) Landslide involving a continuous and gentle downward creep of soil, sand,
gravel etc., is known as __________
a) Rock slip
b) Collapsing
c) Soil slip
d) Overbreak
18) Artificially created water storage basins are called as __________
a) Dams
b) Tank
c) Earthern Dam d) Reservoir
19) The lithology of subsurface is identified with the help of _________ instrument.
a) Magnetometer
b) Tilt meter
c) Electric resistivity meter
d) None of these
20) The spring which discharge hot water is called __________ spring.
a) Mineral
b) Medium
c) Karst
d) Thermal
______________

Set A

*SLRBB24*

-3-

SLR-BB 24

Seat
No.

S.E. (Civil) (Part I) Examination, 2014


ENGINEERING GEOLOGY
Day and Date : Saturday, 20-12-2014
Time : 10.00 a.m. to 1.00 p.m.

Marks : 80

Instructions : 1) From Section I attempt any two questions from Q. No. 2,


Q. No. 3 and Q. No. 4.
2) From Section II attempt any two questions from Q. No. 6,
Q. No. 7 and Q. No. 8.
3) Q. No. 5 and Q. No. 9 are compulsory.
4) Draw neat figures wherever necessary.
5) Figures to the right side indicate full marks.
SECTION I
2. a) Explain external and internal forces modifying the earth surface.
b) Explain in detail internal structure of the Earth.
3. a) What are faults ? Explain with neat sketches types of faults.
b) Describe any four structures of Igneous rocks.
4. a) Give a full account of Geological work carried out by a River.
b) Products of volcanoes.
5. Write notes on any four of the following :

7
7
7
7
7
7
12

a) Subdivisions of Geology
b) Waterfall
c) Fissure type of volcanoes
d) Chemical deposits
e) Dip and strike.
Set A

SLR-BB 24

-4-

*SLRBB24*

SECTION II
6. a) Explain the effects and precautions of Earthquake.

b) Define Aquifer. Describe different types of Aquifer.

7. a) Describe requirement of good building stones.


b) Describe dams on sedimentary rocks and folded strata.
8. a) What are landslides ? Explain effects and disaster management.
b) Explain types of bridges with examples.
9. Write notes on any four of the following :

7
7
7
7
12

a) Perched water table


b) Seismic waves
c) Core logging
d) Building stones in India
e) Requirement of good dam site.
_____________________

Set A

SLR-BB 241

*SLRBB241*
Seat
No.

Set

B.E. (Computer Science and Engg.) (Part II) Examination, 2014


ADVANCED DATABASE SYSTEMS
Day and Date : Tuesday, 25-11-2014
Time : 3.00 p.m. to 6.00 p.m.

Max. Marks : 100

Instructions : 1) Q. No. 1 is compulsory. It should be solved in first 30


minutes in Answer book Page No. 3. Each question carries one mark.
2) Answer MCQ/Objective type questions on Page No. 3 only. Dont
forget to mention, Q.P. Set (A/B/C/D) on Top of Page.
MCQ/Objective Type Questions
Duration : 30 Minutes
I.

Marks : 20

Multiple choice questions :

20

1) The read and write quorum must satisfy the condition


a) Qr + Qw = S
b) Qr + Qw < S
c) Qr + Qw > S

d) Qr = Qw

2) Todays General purpose computer systems provide


a) Coarse-Granularity Parallelism
b) Fine-Granularity Parallelism
c) Both a) and b)
d) Neither a) nor b)
3) In path expressions, for object references, the symbol used is
c) >
a) *
b)

d) :

4) In __________ parallelism, transaction throughput is increased.


a) Interquery
b) Intraquery
c) Intraoperation

d) All of the above

5) OMG Stands for


a) Operational Management Group
c) Operational Manipulation Group

b) Objective Management Group


d) Object Management Group

6) The operation of changing dimensions used in a cross-tab is called


a) Slicing
b) Dicing
c) Pivoting
d) Roll up
7) The projection operation distributes over ______________ operation.
a) Union
b) Set difference
c) Intersection
d) All the above
8) Some of the columns of a relation are at different sites in which of the following ?
a) Horizontal fragmentation
b) Horizontal and vertical fragmentation
c) Vertical fragmentation
d) Data replication
9) A homogenous distributed database is which of the following ?
a) The same DBMS is used at each location and data are not distributed across all nodes
b) The same DBMS is used at each location and data are distributed across all nodes
c) A different DBMS is used at each location and data are not distributed across all
nodes
d) A different DBMS is used at each location and data are distributed across all nodes
P.T.O.

SLR-BB 241

*SLRBB241*

-2-

10) Parallel Execution of multiple individual operations within a given query is called
____________ parallelism.
a) Interoperation
b) Interquery
c) Intraoperation
d) Intraquery
11) The task of correcting and preprocessing the data is called
a) Data warehousing
b) Data cleansing
c) Data mining
d) None of these
12) The task of deduplication is called as
a) Householding
c) View-maintenance

b) Merge-purge
d) Redundancy

13) TPC D Benchmark is used


a) To test the performance of the core database system
b) To test the performance of the database systems on decision-support queries
c) To simulate a typical bank application
d) To concentrate on order-entry environment
14) Tables containing multidimensional data are called
a) Super Table
b) Fact Tables
c) Star Table
d) Sub Table
15) ______________ is a measure of fraction of population of how often the consequent is
true when antecedent is true.
a) Support
b) Population
c) Confidence
d) None of these
16) The quality of classification can be measured using
a) Accuracy
b) Recall
c) Precision
d) All the above
17) For n relations, there are ____________ different join orders.
a) 2n
c) 2n

b) (2(n 1)) ! /(n 1)!


d) 2(n 1)

18) ______________ are standardized sets of tasks that help to characterize the performance
of database systems.
a) Legacy systems
b) Benchmarks
c) Bottlenecks
d) Materialized views
19) Bayesian and support vector machine are two types of
a) Classification algorithms
b) Hierarchical clustering algorithms
c) Association rule mining algorithms
d) Collaborative filtering algorithms
20) Which architecture of workflow management system has one scheduler instantiated for
each workflow ?
a) Centralized Architecture
b) Fully Distributed Architecture
c) Partially Distributed Architecture
d) None of the above
______________
Set A

*SLRBB241*

-3-

SLR-BB 241

Seat
No.
B.E. (Computer Science and Engg.) (Part II) Examination, 2014
ADVANCED DATABASE SYSTEMS
Day and Date : Tuesday, 25-11-2014
Time : 3.00 p.m. to 6.00 p.m.

Marks : 80

SECTION I
II. Answer any 4 :

(45=20)

1) Explain the features of parallel systems.


2) Explain parallel external sort merge algorithm.
3) A car-rental company maintains a vehicle database for all the vehicles in its current fleet.
For all vehicles, it includes the vehicle identification number, license number, manufacturer,
model, date of purchase and color. Special data are included for certain types of vehicles

Trucks : cargo capacity

Sports cars : horsepower, renter age requirement

Vans : number of passengers.

Construct an SQL : 1999 schema definition for this database. Use inheritance
where appropriate.
4) Write the steps of semi join operation.
5) Describe the benefits and drawbacks of pipelined parallelism.
III. Answer any one :

10

1) Explain 3 partitioning techniques and compare them.


2) What is the objective of 2-phase commit protocol ? Explain its working and its ability to
handle failure of participating site.
IV. Explain two approaches for distributed data storage along with their advantages and
disadvantages.

10
Set A

SLR-BB 241

-4-

*SLRBB241*

SECTION II
V. Answer any 4 :

(45=20)

1) Write the queries for the operations :


a) cube
b) rollup
c) rank() and dense rank()
d) ntile()
e) windowing
2) Explain three architectural approaches to workflow management system.
3) Explain block nested-loop join and its cost analysis.
4) List five important features of main-memory databases.
5) Explain the following search algorithms with their cost estimates.
A2 (primary index, equality on key)
A3 (Primary index, Equality on nonkey)
VI. Answer any one :

10

1) What are decision support systems ? Explain several issues of decision support systems.
2) Explain real time transaction systems with its key properties.
VII. Draw the architecture of data were house and explain the data warehouse schemas.

10

_____________________

Set A

SLR-BB 242

*SLRBB242*
Seat
No.

Set

B.E. (C.S.E.) (Part II) Examination, 2014


MOBILE COMPUTING
Day and Date : Thursday, 27-11-2014
Time : 3.00 p.m. to 6.00 p.m.

Max. Marks : 100

Instructions : 1)
2)
3)
4)

Q. 1 is objective type.
All questions are compulsory.
Figures to the right indicate full marks.
Answer to the two Sections should be written in the same
answer book.
5) Q. No. 1 is compulsory. It should be solved in first 30
minutes in Answer book Page No. 3. Each question carries
one mark.
6) Answer MCQ/Objective type questions on Page No. 3 only.
Dont forget to mention, Q.P. Set (A/B/C/D) on Top of Page.
MCQ/Objective Type Questions

Duration : 30 Minutes

Marks : 20

1. Choose the correct answer :


20
1) The Marconi antenna is mounted on the roof of a car, the __________ length
is efficient.
b) /2
c)
d) None of above
a) /4
2) Orthogonal codes are used as __________ spaces in code division
multiplexing.
a) Guard
b) Small
c) Gap
d) None of above
3) GSM uses the __________ for internal unique identification of a subscriber.
a) TMSI
b) MSRN
c) ISDN
d) IMSI
4) A __________ comprises all radio equipment i.e. antennas, signal processing,
amplifiers necessary for radio transmission.
a) BTS
b) BSS
c) BSC
d) All of above
5) The heart of the GSM system is formed by the __________ subsystem.
a) Radio
b) Network and switching
c) Operation
d) All of above
6) __________ security algorithm is used for encryption.
a) A3
b) A8
c) A5
d) None of above
7) Call management layer contains foll entities.
a) Call Control (CC)
b) Short Message Service (SMS)
c) Supplementary Service (SS)
d) All of above
P.T.O.

SLR-BB 242

-2-

*SLRBB242*

8) Downlink is from base station to mobile station or from satellite to ground


control.
a) True
b) False
9) Sensing the carrier and accessing the medium only if the carrier is idle, and
decreases the probability of a collision, this scheme is called as __________
a) CSMA
b) CDMA
c) DAMA
d) PRMA
10) If the size of an obstacle is in the order of the wavelength or less, then waves
can be __________
a) Scattered
b) Diffracted
c) Reflected
d) Blocked
11) __________ technology uses diffuse light reflected at walls and furniture.
a) Radio transmission
b) Infra red
c) Directed light
d) Laser diodes
12) A waiting time between __________ and __________ is used for a timebounded service.
a) IDFS and ISFS
b) DIFS and SIFS
c) FIDS and FISS
d) None of above
13) The __________ contains a timestamp and other management information
used for power management and roaming.
a) BSS
b) Station
c) Beacon
d) None of above
14) The functions of the transport layer in the internet.
a) Checksumming over user data
b) Multiplexing of data
c) Demultiplexing of data
d) All of above
15) In traditional TCP, the sender always calculates a __________ window for a
receiver.
a) Contraction
b) Small
c) Congestion
d) None of above
16) __________ bits are used for synchronization field of an IEEE 802.11 PHY
frame format by using DSSS.
a) 128
b) 80
c) 16
d) 8
17) Which is a typical wireless ad-hoc network ?
a) IEEE 802.11
b) Bluetooth
c) HIPERLAN
d) None of above
18) The main tasks of the PHY management include __________
a) Encoding of signal
b) Decoding of signal
c) Channel tuning
d) All of above
19) A __________ is an end-system or router that can change its point of
attachment to the internet using mobile IP.
a) MN
b) HA
c) FA
d) None of above
20) __________ denotes the validity of the registration in seconds.
a) Code
b) Identification c) Home agent
d) None of above
______________
Set A

*SLRBB242*

-3-

SLR-BB 242

Seat
No.

B.E. (C.S.E.) (Part II) Examination, 2014


MOBILE COMPUTING
Day and Date : Thursday, 27-11-2014
Time : 3.00 p.m. to 6.00 p.m.

Marks : 80

Instructions : 1) All questions are compulsory.


2) Figures to the right indicate full marks.
3) Answer to the two Sections should be written in
same answer book.
SECTION I
2. Solve any four short notes :

(45=20)

1) Frequency division multiple access


2) Radio subsystem
3) Radiation pattern of a simple dipole
4) Spread spectrum
5) Advantages and disadvantages of cellular system.
3. Explain with an example the packet reservation multiple access and compare
it with reservation TDMA.

10

OR
Explain with neat diagram the different algorithms used to provide security
services in GSM.
4. What is signal propagation ? Explain in detail the different signal propagation
effects.

10
Set A

SLR-BB 242

-4-

*SLRBB242*

SECTION II
5. Solve any four :

(45=20)

1) Explain transaction oriented TCP.


2) Write a short note on PICO NET.
3) Explain generic WATM reference model.
4) Explain design goals of WLAN.
5) Explain IEEE 802.11 MAC packet structure.
6. Why encapsulation is needed ? Explain generic routing encapsulation in mobile
network layer.

10
OR

Explain HIPERIAN 1 with different phases of EY-NPMA access scheme.


7. Explain with neat diagram the system architecture for infrastructure and ad-hoc
based IEEE 802.11.

10
_____________________

Set A

SLR-BB 243

*SLRBB243*
Seat
No.

Set

B.E. (CSE) (Part II) Examination, 2014


PRINCIPLES OF MANAGEMENT AND ENGINEERING ECONOMICS
Day and Date : Saturday, 29-11-2014
Time : 3.00 p.m. to 6.00 p.m.

Max. Marks : 100

Instructions : 1)
2)
3)
4)

Figures to the right indicate full marks.


First question is objective type.
All questions are compulsory.
Q. No. 1 is compulsory. It should be solved in first 30 minutes in
Answer book Page No. 3. Each question carries one mark.
5) Answer MCQ/Objective type questions on Page No. 3 only. Dont
forget to mention, Q.P. Set (A/B/C/D) on Top of Page.
MCQ/Objective Type Questions

Duration : 30 Minutes

Marks : 20

1. Choose the correct answer :


1) The oldest form of organization, also called the traditional organization is
a) Functional Organization
b) Line, Military or Scalar Organization
c) Product Organization
d) Project Organization
2) According to Henry Fayol, Authority is defined as
a) The right to give orders and the power to demand obedience
b) The right to give orders and the control by dictatorship
c) The right to give orders without listening to their queries
d) None of these
3) The number of persons who directly report to an executive is called
a) Span of employees
b) Span of industry
c) Span of control
d) None of these
4) In ___________ Leadership style, the leader assumes all power and decision.
a) Democratic or participative
b) Laissez Faire or Free Rein
c) Authoritative or Autocratic
d) None of these
5) In the Maslows Hierarchy of needs theory the first and foremost need is
a) Esteem needs
b) Social needs
c) Safety needs
d) Physiological needs
6) The theory of X and theory Y of motivation was suggested by
a) Hertzberg
b) McGregor
c) Maslow

d) McClelland

7) PERT stands for


a) Process Evaluation and Resale Technique
b) Process Evaluation and Resend Technique
c) Process Evaluation and Recording Technique
d) Process Evaluation and Review Technique
P.T.O.

SLR-BB 243

*SLRBB243*

-2-

8) Partners who just invest money and do not take part in the management are called
a) General partners b) Nominal partners c) Sleeping partners d) Secret partners
9) Placements of employees is a function of
a) Staffing
b) Authority

c) Responsibility

10) _________ is called as the father of scientific management.


a) Henry Fayol
b) F. W. Taylor
c) F. B. Gilbert

d) None of these
d) Koontz

11) Which of the following defines marginal utility ?


a) The change in total utility divided by the price of a product
b) The maximum amount of satisfaction from consuming a product
c) The total satisfaction received from consuming as much of the product that is available
for consumption
d) The additional satisfaction received from consuming one more unit of a product
12) When marginal utility is decreasing but positive, total utility is
a) Increasing at a decreasing rate
b) Decreasing at a decreasing rate
c) Increasing at an increasing rate
d) Decreasing at an increasing rate
13) Scarcity guarantees that
a) Wants will exceed demands
c) Demands will exceed wants

b) Demands will be equal to wants


d) Most demands will be satisfied

14) The law of demand states that the quantity of a good demanded varies
a) Inversely with its price
b) Directly with population
c) Directly with income
d) Inversely with the price substitute goods
15) Inferior goods are those for which demand increases as
a) Income decreases
b) Income increases
c) The price of a substitute rises
d) The price of a substitute falls
16) If supply is the inelastic and demand deceases, the total revenue of sellers will be
a) Increase
b) Decrease
c) Decrease only if demand is elastic
d) Increase only if demand is inelastic
17) If a decrease in the price of gasoline increases the demand for large cars, then
a) Gasoline and large cars are complement in consumption
b) Large cars are an inferior good
c) Gasoline is an inferior good
d) Gasoline and large cars are substitutes in consumption
18) If the price elasticity is between 0 and 1, demand is _________
a) Inelastic
b) Elastic
c) Perfectly elastic

d) Unit elastic

19) The cost incurred even when no output is produced are called
a) Fixed costs
b) External costs c) Variable costs

d) Marginal costs

20) Luxury goods are usually


a) Price inelastic
c) Income elastic

b) Income inelastic
d) Goods with negative income elasticity

______________

Set A

*SLRBB243*

-3-

SLR-BB 243

Seat
No.

B.E. (CSE) (Part II) Examination, 2014


PRINCIPLES OF MANAGEMENT AND ENGINEERING ECONOMICS
Day and Date : Saturday, 29-11-2014
Time : 3.00 p.m. to 6.00 p.m.

Marks : 80

Instructions : 1) All questions are compulsory.


2) Figures to the right indicate full marks.
SECTION I
2. Attempt any four of the following :

(45=20)

i) What are the types of business organizations ? Explain Committee Organization with its
advantages and disadvantages.
ii) Explain Maslows Hierarchy of needs theory with the diagram of pyramid.
iii) What is Leadership ? Explain any one Leadership style in detail.
iv) Define planning ? What are the various steps involved in planning ?
v) Explain the theory X and theory Y by bringing out the differences between them.
3. Attempt any two of the following :

(25=10)

i) Define the following terms and briefly explain :


a) Group dynamics
b) Authority
c) Organization chart
d) Span of control
e) Leadership.
ii) Differentiate between Product/Project Organization and Matrix or grid Organization.
iii) Explain PERT charts by taking an example.
4. Attempt any two of the following :

(25=10)

i) Define Staffing. Explain the various steps involved in staffing.


ii) Define Planning. Explain the various steps involved in Planning.
iii) Explain the three levels of management viz., Top Level, Middle Level, and Lower or
supervisory with their functions.

Set A

SLR-BB 243

-4-

*SLRBB243*

SECTION II
5. Attempt any 4 of the following :
1) What is demand ? Explain Price Elasticity of demand with example.
2) Explain factors of production.
3) Explain graphical method for break even point with example.
4) Write 3 advantages and 3 disadvantages of break even charts ?
5) Explain demand schedule, law of demand and demand curve with diagram and example.

20

6. Write a short note on any 2 :


1) Functions of Bank
2) Wants
3) Capital.

10

OR
6. Attempt the following :
Kirati Company submitted following information :
1) Selling price Rs. P.U 20
2) Trade discount 5%
3) Material P.U Rs. 6
4) Wages P.U. Rs. 4
5) Variable overhead 100% of labour
6) Fixed cost 20,000
7) Sold out units 1,00000
Find out :
i) BEP (Break Even Point)
ii) Profit Volume ratio
iii) Margin of safety.

10

7. Attempt the following :


1) Direct wages per unit Rs. 1.50
2) Direct material per unit Rs. 2.25
3) Direct Expenses per unit Rs. 0.65
4) Fixed Cost :
Supervisors salary Rs. 10,000
Deprecation Rs. 4,000
Other overhead Rs .16,000
5) Selling price per unit Rs. 8
Calculate sales at break even point and profit if :
i) 30,000 units are sold and ii) 20,000 units are sold.

10

_____________________

Set A

SLR-BB 244

*SLRBB244*
Seat
No.

Set

B.E. (CSE) (Part II) Examination, 2014


Elective II : DATA MINING
Day and Date : Monday, 1-12-2014
Time : 3.00 p.m. to 6.00 p.m.

Total Marks : 100

Instructions : 1) Q. No. 1 is compulsory. It should be solved in first 30 minutes


in Answer book Page No. 3. Each question carries one mark.
2) Answer MCQ/Objective type questions on Page No. 3 only.
Dont forget to mention, Q.P. Set (A/B/C/D) on Top of Page.
MCQ/Objective Type Questions
Duration : 30 Minutes

Marks : 20

1. Choose the correct answer :

20

1) Data mining is used to aid in


a) Operational management
b) Analysing past decision made by managers
c) Detecting patterns in operational data
d) Retrieving archival data
2) A data warehouse is which of the following ?
a) Can be updated by end users
b) Contains numerous naming conventions and formats
c) Organized around important subject areas
d) Contains only current data
3) A goal of data mining includes which of the following ?
a) To explain some observed event or condition
b) To confirm that data exists
c) To analyze data for expected relationships
d) To create a new data warehouse
4) Which among the following is not an example of spatial database ?
a) Stock exchange database
b) GIS database
c) VLSI database
d) CAS database
5) Which among following is not a method for data normalization ?
a) Max-min normalization
b) Z-score normalization
c) Normalization by decimal scaling
d) Min-max normalization
6) ___________ is not a pattern detection operation in temporal mining.
a) Match
b) Update
c) Insert
d) Delete
7) __________ is not a approach to handle missing data.
a) Ignore the missing value
b) Assume a value for missing data
c) Both a) and b)
d) None of above
P.T.O.

SLR-BB 244

-2-

*SLRBB244*

8) Classification is the mapping from the ________ to the set of classes.


a) Data
b) Database
c) Information
d) Knowledge
9) ___________ of an item is the percentage of transaction in which that item occurs.
a) Confidence
b) Support
c) Strength
d) None of above
10) __________ and prediction may be viewed as types of classification.
a) Regression
b) Estimation
c) Cleaning
d) Correlation
11) Pages contains links to many relevant pages is called
a) Seed pages
b) Authoritative pages
c) Hub pages
d) Distiller pages
12) Regression problem deals with estimation of an output value based on _______
a) Previous output b) Input value
c) Both a) and b)
d) None of above
13) Which association algorithm uses Any subset of large item set must be large
property of large item set ?
a) Partitioning
b) Sampling
c) Apriori
d) None of above
14) A __________ is a directed graph<V, A> with vertices representing states
V = {v1, v2, ..., vn) and arcs A = {<i, j> | vi, vj V}, showing transitions between
states.
a) Finite State Recognizer
b) Markov Model
c) Finite State Machine
d) None
15) Time series analysis may be viewed as finding patters in the data which are of
type __________
a) Trends
b) Seasonal
c) Cycles
d) All the above
16) The long form of STING
a) Sequential Time Information in Neural Grid
b) Sequential Information Grid-Based Method
c) Sequential Time Information Grid-Based Method
d) Statistical Information Grid-Based Method
17) A __________ was designed to index multi-attribute data.
a) R Tree
b) Quad tree
c) k-D tree
d) None of the above
18) _________ algorithm is used in the database design area to determine how to
group data and place on a disk.
a) PAM
b) Nearest neighbor
c) K-Means
d) Bond energy
19) Any subset of a large itemset is ____________
a) Large
b) Small
c) Less than threshold
d) None of above
20) The difference between the Quad tree and R-tree is ____________
a) In R-Tree partitioning with MBRs whereas in Quad Tree partitioning with
quadrants
b) In R-Tree partitioning with quadrants whereas in Quad Tree partitioning with
MBRs
c) There is no difference
d) None of the above
______________
Set A

*SLRBB244*

-3-

SLR-BB 244

Seat
No.

B.E. (CSE) (Part II) Examination, 2014


Elective II : DATA MINING
Day and Date : Monday, 1-12-2014

Marks : 80

Time : 3.00 p.m. to 6.00 p.m.


SECTION I
2. Solve any four :

(45=20)

a) Explain the major issues in data mining.


b) Which tools are used for data mining ?
c) State the major tasks required in data preprocessing.
d) What are outliers ? How might you determine the outliers in the data ?
e) Discuss the classification of association rule mining based on various criteria.
3. With diagram explain the KDD process in detail.

10

OR
3. With suitable example explain the statistical based techniques used for regression
problem.
4. Explain the K nearest neighbors classification algorithm with suitable example.

10
10

Set A

SLR-BB 244

-4-

*SLRBB244*

SECTION II
5. Solve any four :

(45=20)

a) Write an agglomerative clustering algorithm.


b) If it is given the following items to cluster, using K-means clustering algorithm show
the iterations in clustering till convergence. Assume k=2.
Items to cluster = {2, 4, 10, 12, 3, 20, 30, 11, 25}.
c) Explain support and confidence with example.
d) Write a note on trend analysis in time series databases.
e) Explain different spatial data mining primitives.
6. Explain web structure mining in detail.

10

7. Write short notes on :

10

a) Difference between temporal and traditional database.


b) Quad tree.
_____________________

Set A

SLR-BB 245

*SLRBB245*
S

B.E. (CSE) (Part II) Examination, 2014


SOFTWARE TESTING AND QUALITY ASSURANCE (Elective II)
Day and Date : Monday, 1-12-2014
Time : 3.00 p.m. to 6.00 p.m.

Max. Marks : 100

Instructions : 1) Q. No. 1 is compulsory. It should be solved in first 30


minutes in Answer Book Page No. 3. Each question carries
one mark.
2) Answer MCQ/Objective type questions on Page No. 3
only. Dont forget to mention, Q.P. Set (A/B/C/D) on Top
of Page.
Duration : 30 Minutes

MCQ/Objective Type Questions

Marks : 20

1. Choose the correct option.

20

1) In ___________ view, quality is viewed as tied to the inherent characteristics


of the product.
a) Product
b) User
c) Manufacturing
d) Transcendental
2) ___________ analysis of a software system involves actual program execution
in order to expose possible program failures.
a) Static
b) Dynamic
c) Structural
d) None of these
3) ___________ testing is performed whenever a component of the system is
modified.
a) Unit
b) System
c) Regression
d) Integration
4) White Box testing is also called as ___________ testing.
a) architectural
b) functional
c) logical
d) structural
5) ___________ are collection of software related activities.
a) Processes
b) Product
c) Resources
d) Specification
6) ___________ attributes of a product, process or resource are those that can
be measured purely in term of the product, process or resource itself.
a) External
b) Internal
c) Linear
d) All
7) Usability and reliability are the ___________ attributes of product, process
and resources.
a) architectural
b) structural
c) external
d) internal
P.T.O.

SLR-BB 245

-2-

*SLRBB245*

8) The ___________ of a module is the extent to which its individual components


are needed to perform the same task.
a) coupling
b) binding
c) performance d) cohesion
9) A ___________ is a contiguous sequence of program statements bounded
by boundary elements, having an aggregate identifier.
a) module
b) segment
c) block
d) none of these
10) ___________ is a physical size of software product.
a) Functionality
b) Length
c) Complexity
d) Reuse
11) Sandwich and Big bang approaches are the ___________ testing techniques.
a) Control flow
b) Unit
c) System integration
d) Domain
12) ___________ test determine how well the system recovers from various
input errors and other failure situations.
a) Scalability
b) Performance c) Stress
d) Robustness
13) The purpose of ___________ plan is to get ready and organize for the test
execution.
a) System test
b) Unit test
c) Integration test d) All
14) In ___________ representation operational profile is represented as a tree
structure consisting of nodes and branches.
a) tabular
b) graphical
c) structural
d) cohesion
15) ___________ model is one of the type of reliability model.
a) Basic
b) Quality
c) Logarithmic
d) Reliability
16) ___________ is the first step of effective recruiting process of test engineer.
a) Job specification
b) Job validation
c) Job confirmation
d) Job requisition
17) Correctness is quality factor of ___________ category.
a) product operation
b) product revision
c) product transition
d) none of these
18) ___________ McCalls quality factor, effort required to couple one system
with another.
a) Flexibility
b) Interoperability
c) Reusability
d) Integrity
19) ___________ quality characteristics of ISO 9126 quality standard.
a) Accuracy
b) Stability
c) Functionality d) Security
20) Quality factors are ___________ attributes of software system.
a) basic
b) internal
c) logical
d) external
______________
Set A

*SLRBB245*
S

-3-

SLR-BB 245

B.E. (CSE) (Part II) Examination, 2014


SOFTWARE TESTING AND QUALITY ASSURANCE (Elective II)
Day and Date : Monday, 1-12-2014

Marks : 80

Time : 3.00 p.m. to 6.00 p.m.

SECTION I
2. Attempt any four (each carries 5 marks) :

20

1) Explain quality revolution in detail.


2) Explain concept of complete testing and central issues of testing.
3) Explain overview of statistical tests in software measurements and metrics.
4) Explain reuse of software in software engineering measurement along with
examples.
5) Explain control flow structure in software engineering measurement.
6) Explain object oriented metrics in software engineering measurements.
3. Explain how different frameworks applying in software measurement.

10

OR
3. What is Test case ? Explain along with an examples. Also explain different
objectives of testing. Describe briefly White-Box Testing.

10

4. Explain how functionality used in software engineering measurement by Albrechts


approach, COCOMO 2.0 approach and Demarcos approach.

10

Set A

SLR-BB 245

-4-

*SLRBB245*

SECTION II
5. Attempt any four (each carries 5 marks) :

20

1) Explain concept of integration testing.


2) Explain test automation infrastructure along with its components.
3) What are the different applications of software reliability ?
4) Explain different reliability models in detail.
5) What are the five views of software quality ?
6) Explain different McCalls quality factors in detail.
6. Explain ISO 9000 : 2000 Software Quality standard in detail.

10

OR
6. Explain different types of interfaces and interface errors in system integration
testing.

10

7. What are the six phases of effective recruiting process of recruiting test
engineers ?

10

_____________________

Set A

SLR-BB 246

*SLRBB246*
Seat
No.

Set

B.E. (CSE) (Part II) Examination, 2014


Elective II : MOBILE APPLICATION DEVELOPMENT
Day and Date : Monday, 1-12-2014
Time : 3.00 p.m. to 6.00 p.m.
Instructions :

Total Marks : 100

1) Figures to the right indicate full marks.


2) Assume data if necessary.
3) Q. No. 1 is compulsory. It should be solved in first 30 minutes in
Answer Book Page No. 3. Each question carries one mark.
4) Answer MCQ/Objective type questions on Page No. 3 only. Dont
forget to mention, Q.P. Set (A/B/C/D) on Top of Page.
MCQ/Objective Type Questions

Duration : 30 Minutes

Marks : 20

1. Objective type questions. Choose the correct option :

20

1) A device with Android installed is needed to develop apps for Android.


A) True
B) False
2) What part of the Android platform is open source ?
A) Low-level Linux modules
B) Native libraries
C) Application frame work
D) All of the above
3) Which piece of code used in Android is not open source ?
A) Keypad Driver
B) WiFi ? driver
C) Audio driver
D) Power management
4) Android is built upon the Java Micro Edition (J2ME) version of Java.
A) True
B) False
5) Which among these are NOT a part of Androids native libraries ?
A) Webkit
B) Dalvik
C) OpenGL
D) SQLite
6) Android is based on Linux for the following reason.
A) Security
B) Portability
C) Networking

D) All of these

7) What operating system is used as the base of the Android stack ?


A) Linux
B) Windows
C) Java
D) XML
8) What is a key difference with the distribution of apps for Android based devices than
other mobile device platform applications ?
A) Applications are distributed by Apple App Store only
B) Applications are distributed by multiple vendors with different policies on applications
C) Applications are distributed by multiple vendors with the exact same policies on
applications
D) Applications are distributed by the Android Market only
P.T.O.

SLR-BB 246

*SLRBB246*

-2-

9) When developing for the Android OS, Java byte code is compiled into what ?
A) Java source code
B) Dalvik application code
C) Dalvik byte code
D) C source code
10) What does the .apk extension stand for ?
A) Application Package
C) Android Proprietary Kit

B) Application Program Kit


D) Android Package

11) When you distribute your application commercially, youll want to sign it with your
own key ?
A) True
B) False
12) How does Google check for malicious software in the Android Market ?
A) Every new app is scanned by a virus scanner
B) Users report malicious software to Google
C) Google employees verify each new app
D) A separate company monitors the Android Market for Google
13) Which of these are not one of the three main components of the APK ?
A) Dalvik Executable
B) Resources
C) Native Libraries
D) Webkit
14) What is the name of the program that converts Java byte code into Dalvik byte code ?
A) Android Interpretive Compiler (AIC)
B) Dalvik Converter
C) Dex Compiler
D) Mobile Interpretive Compiler (MIC)
15) What was the main reason for replacing the Java VM with the Dalvik VM when the
project began ?
A) There was not enough memory capability
B) Java virtual machine was not free
C) Java VM was too complicated to configure
D) Java VM ran too slow
16) The R. Java file is where you edit the resources for your project.
A) True
B) False
17) What is contained within the manifest xml file ?
A) The permissions the app requires
B) The list of strings used in the app
C) The source code
D) All other choices
18) What is contained within the Layout xml file ?
A) Orientations and layouts that specify what the display looks like.
B) The permissions required by the app
C) The strings used in the app
D) The code which is compiled to run the app
19) Which of the following is NOT a state in the life cycle of a service ?
A) Starting
B) Running
C) Destroyed
D) Paused
20) In an explicit intent, the sender specifies the type of receiver.
A) True
B) False
______________
Set A

*SLRBB246*

-3-

SLR-BB 246

Seat
No.
B.E. (CSE) (Part II) Examination, 2014
Elective II : MOBILE APPLICATION DEVELOPMENT
Day and Date : Monday, 1-12-2014

Marks : 80

Time : 3.00 p.m. to 6.00 p.m.


Instructions :

1) All questions from Section I and II are compulsory.


2) Figures to the right indicate full marks.
3) Assume data if necessary.
SECTION I

2. Attempt any four :

(54=20)

1) What is Android Intent ? What is explicit intent ? What is implicit intent ?


2) What is Android Content Provider ? Give simple example that would help you understand
Content Provider.
3) State different features of an Android.
4) What is Layout ? Draw and state different types of layouts in android.
5) What are the components contain in ADT (Android Development Tools) Bundle ?
3. a) Draw and explain the diagram of the development process for an Android App.
b) Illustrate the benefits of Async Task over Threads for implementing long-

Running tasks.

4. What are the different roles of the following with respect to application ? Framework ?

5
10

a) Activities
b) Services
c) View System
d) Resource Manager
e) Notification Manager.
Set A

SLR-BB 246

-4-

*SLRBB246*

SECTION II
5. Attempt any four of the following :

(54=20)

a) Name some commonly used in-built content providers.


b) What are the key graphics capabilities of an Android platform ?
c) What is drawable Animation ? Write a Java code to apply drawable animation to a view.
d) Outline the features of Location Services.
e) Illustrate an example of MPEG-4 container with encoded audio and video elements.

6. What is the use of Shared Preference in Android ? Explain two components that the Share
Preference app will contain. What are the methods offered by editor to save data in Shared
Preferences ?

10

7. Define the categories of sensors available in android devices. How a developer can identify the
sensors present in a device and put them to use in an app ? Write a Java code for the same.

10

_____________________

Set A

SLR-BB 247

*SLR-BB-247*
Seat
No.

Set

S.E. (IT) (Part I) Examination, 2014


APPLIED MATHEMATICS I
Day and Date : Tuesday, 9-12-2014
Time :10.00 a.m. to 1.00 p.m.

Max. Marks : 100

Instructions : 1) Q. No. 1 is compulsory. It should be solved in first 30 minutes in


Answer book Page No. 3. Each question carries one mark.
2) Attempt any three questions from each Section.
3) Figure to the right indicate full marks.
4) Use of non-programmable calculator is allowed.
5) Answer MCQ/Objective type questions on Page No. 3 only. Dont
forget to mention, Q.P. Set (A/B/C/D) on Top of Page.
MCQ/Objective Type Questions
Duration : 30 Minutes

Marks : 20

1. Choose the correct answer :


1)

1
x is equal to
D3
x 1
+
a)
3 9

(201=20)

b)

x 1

3 9

c)

x 1

3 9

d) None of these

2) The P.I of (D + 1)3 y = ex is

x 3 x
e
a)
3

ex
b)
8

x 3 x
e
c)
8

x 3 x
e
d)
6

3) The general solution of the diff. equation (D4 16)y = 0 is


a) C1 e4x + C2 e4x
b) C1 cos 4x + C2 sin 4x
c) C1 e2x + C2 e2x + C3 cos 2x + C4 sin 2x d) (C1 + C2x) e2x + (C3 + C4x)e2x

4) L cosh t dt = __________
0

s
1
b) 2
a) 2
s +1
s +1

d)

s
s 1

c) (s / a)

d)

1
(s / a)
a

c) t e3t

d) t2e3t

c)

1
s 1

5) If L {f(t)} = (s) then L{f(at)} = __________


a) (s/a)

b)

1 s

s a

1
= __________
2
(
s
+
3
)

e 3 t
a) t e3t
b)
t

6) L1

P.T.O.

SLR-BB 247

*SLR-BB-247*

-2-

7) If f(x) = x2 is expanded as cosine series in (0, ) then constant term is


a)

2
3

b)

3
3

c)

22
3

d)

32
2

8) The Fourier expansion in the interval [2,2] of the function


f(x) = x, 2 x 0
= x, 0 x 2 has
a) No cosine term
b) No sine term c) Both cosine and sine term

d) None of these

9) If Z{f(k)} = F(z) then Z {K f(k)} = _________


a)

d
F(z)
dz

b)

d
(ZF(z))
dz

c) z

1
= , k 0
K!

d
F(z )
dz

d)

1 d
F(z )
z dz

d)

1
1 z

10) z

a) ez

b) e

c)

1
z

11) The directional derivative of = x2 y2 + 2z2 at the point (1,2, 3) along z-axis is _________
a) 2
b) 4
c) 12
d) 0
12) If (x, y, z) = c represents a surface, then the unit normal vector to this surface is
a)

grad.
| grad. |

b) grad

c) div (grad. )

d) curl (grad )

13) If r = xi + yj + 3k and a is a constant vector then ( a . r ) = ______________


a) 2 a
b) a
c) r
d) 2 r
14) The lines of regression are given by 10y = x + 17, and x = 5y 7. Then x and y are
a) 1 and 3
b) 1 and 5
c) 2 and 3
d) 3 and 2
15) A continuous random variable has the following probability density function f(x)= kx(1x),
0 x 1 then k = ____________
a) 2
b) 3
c) 5
d) 6
16) If 10% pens are defective and if there are 10 pens in the box then the probability that there
is no defective pen in box is ____________
a) 0.25
b) 0.35
c) 0.45
d) 0
17) The mean and the standard deviation of a standard normal variate is __________
a) 1 and 0
b) 0 and 1
c) 1 and 1
d) None of these
18) In a M/M/1 queueing system, expected waiting time of unit that actually waits is given by
a)

( )

b)

c)

d)

( )
2

19) If average arrival rate in a queue is 6 per hour and the average service rate is 10 per hour.
Which one of the following is the average number of customers in the line including the
customer being served ?
a) 0.3
b) 0.6
c) 1.2
d) 1.5
20) If byx = 6/5 and bxy = 15/8 then r = ___________
a) 2.5
b) 1.5
c) 0.5
______________

d) 1

Set A

*SLR-BB-247*

-3-

SLR-BB 247

Seat
No.

S.E. (IT) (Part I) Examination, 2014


APPLIED MATHEMATICS I
Day and Date : Tuesday, 9-12-2014
Time :10.00 a.m. to 1.00 p.m.

Marks : 80

Instructions : 1) Attempt any three questions from each Section.


2) Figure to the right indicate full marks.
3) Use of non-programmable calculator is allowed.
SECTION I
2. a) Solve the following diff. equations any two :
i)

(D2

+ 2D + 5)y =

sin2x

ii) (D3 3D2 + 3D 1)y = x ex + ex


iii) (D2 + 2D + 1)y = 4ex log x
b) Solve (D3 7D 6) y = (1 + x2) e2x.

sin2 2t
.
t

3. a) Find L

s + a .

t sin 3t dt by Laplace transform.

b) Find L1 tan

c) Evaluate

2 t 2

s+4
.
2
s(s 1)(s + 4)

d) Find L 1

4. a) Find the Fourier series of


f(x) = x,
1< x < 0
= x + 2, 0 < x < 1.

z2

1
1

<|z|< .
1
1
b) Find Z1
z z if 3
2
2
3

5. a) Find the half-range cosine series of f(x) = x ( x) in the interval (0, ).

1
4
c) Find z{cosh k}, k 0.

6
5

|k|

b) Find z{f(k)} if f(k) = for all k.

4
4
Set A

SLR-BB 247

-4-

*SLR-BB-247*

SECTION II
6. Attempt the following :
a) Prove that, (r nr ) = (n + 3)r n .

b) Find the directional derivative of = e2x cosyz at (0, 0,0) in the direction of the tangent to
the curve x = asint y = a cost, z = at t = .

c) Find the unit tangent vector at any point on the curve x = t2 + 2, y = 4t 5, z = 2t2 6t where
t is any variable. Also determine the unit tangent vector at the point t = 2.
7. Attempt the following :
a) A manufacturer finds that the average demand per day for the mechanic to repair his new
production is 1.5. over a period of one year the demand per day is distributed as Poisson
distribution. He employs two mechanics on how many days in one year i) both mechanics
would be free. ii) some demand is refused.
b) The size of hats is normally distributed with mean 18.5 cm and standard deviation of 2.5 cms.
How many hats in total of 2000 will have sizes between ?
i) 18 cms and 20 cms
ii) above 20 cms (Given : For S.N.V.z area between z = 0 to
z = 0.6 is 0.2257 and between z = 0 to z = 0.2 is 0.0793)
c) Fit a second degree parabolic curve to the following data estimate production in 1982.
Year
: 1974 75
76
77
78
79
80
81
Production : 12 14
26
42
40
50
52
53
8. Attempt the following :
a) The equations of two lines of regression are 6y = 5x + 90, and 15x = 8y + 130. Find the
mean of x and mean of y and the coefficient of correlation. If variance of x is 16, find the
standard deviation of y.
OR
a) The following table gives the number of motor-vehicles with licenses and the number of
motor-vehicles accidents in a city. Calculate the coefficient of correlation between the
number of motor-vehicles and the number of motor-vehicles accidents :
No. of vehicles in thousands : 2.6
2.8
2.9 3.1
3.2
2.3 2.5 1.8
No. of accidents in hundreds : 5.9
6.0
6.2 6.2
7.6
7.0 7.5 5.5
b) Fit a binomial distribution to the following data :
x : 0
1
2
3
4
5
6
f :
6
20
28
12
8
6
0
9. Attempt the following :
a) Customers arrive at a box office window, being manned by a single individuals according
to Poisson input process with mean rate of 20 per hour. Time required to serve a customer
has an exponential distribution with mean of 90 seconds. Find average waiting time of
customer. Also determine the average no. of customers in the system and average
queue length.
b) A warehouse has only one loading dock manned by a three person crew. Trucks arrive at
the loading dock at an average rate of 4 trucks per hour and the arrival rate is Poisson
distributed. The loading of a truck take 10 min. on an average and can be assumed to be
exponentially distributed. The operating cost of truck is Rs. 20 per hour and the members
of the loading crew are paid @ Rs. 6 per hour. Would you advise the truck owner to odd
another crew of three persons ?
_____________________

4
6

Set A

SLR-BB 248

*SLRBB248*
Seat
No.

Set

S.E. (Information Technology) (Part I) Examination, 2014


DISCRETE MATHEMATICAL STRUCTURE
Day and Date : Thursday, 11-12-2014
Time : 10.00 a.m. to 1.00 p.m.

Max. Marks : 100

Instructions : 1) Q. No. 1 is compulsory. It should be solved in first 30 minutes in


Answer Book Page No. 3. Each question carries one mark.
2) Answer MCQ/Objective type questions on Page No. 3 only. Dont
forget to mention, Q.P. Set (A/B/C/D) on Top of Page.
MCQ/Objective Type Questions
Duration : 30 Minutes

Marks : 20

1. Choose the correct answer :


1) Consider the statement form p q where p = If Tom is Janes father then Jane is Bills
niece and q = Bill is Toms brother. Which of the following statements is equivalent to
this statement ?
a) If Bill is Toms brother, then Tom is Janes father and Jane is not Bills niece
b) If Bill is not Toms brother, then Tom is Janes father and Jane is not Bills niece
c) If Bill is not Toms brother, then Tom is Janes father or Jane is Bills niece
d) If Bill is Toms brother, then Tom is Janes father and Jane is Bills niece
2) The statement form (p r ) (q r ) is equivalent to
a) [(~ p r ) (p ~ r )] ~ [(~ q r ) (q ~ r )]
b) ~ [(~ p r ) (p ~ r )] [(~ q r ) (q ~ r )]

c) [(~ p r ) (p ~ r )] [(~ q r ) (q ~ r )]
d) ~ [(~ p r ) (p ~ r )] [(~ q r ) (q ~ r )]
3) Which of the following statements is NOT equivalent to the statement, There exists
either a computer scientist or a mathematician who knows both discrete math and Java.
a) There exists a person who is a computer scientist and who knows both discrete math
and Java or there exists a person who is a mathematical and who knows both discrete
math and java
b) There exists a person who is a computer scientist or there exists a person who is a
mathematician who knows discrete math or who knows Java
c) There exists a person who is a computer scientist and who knows both discrete math
and Java or there exists a mathematician who knows both discrete math and Java
d) There exists a computer scientist who knows both discrete math and Java or there
exists a person who is a mathematician who knows both discrete math and Java
P.T.O.

SLR-BB 248

*SLRBB248*

-2-

4) What is the meaning of complement in set theory ?


a) All the elements in the universal set that are not identical
b) A set of elements that work well with a given set
c) All the elements of a universal set that do not belong to a subset of it
d) All the elements that are the opposite of the elements in a given set
5) What is the meaning of disjoint in set theory ?
a) Two or more sets having no elements in common b) Two or more sets that do not match
c) Sets that are in different universal sets
d) Sets that contain no elements

6) Insert or
/ in the blank to make the statement true.
{4, 20, 25} _________ {3, 20, 25, 35}
c)
a) /
b)
7) Use the union rule to answer the question.
If n(A) = 24, n(B) = 69, and n(A B) = 81; what is n(A B) ?
a) 36
b) 12
c) 6
8) Let S = {x, y, z} covering sets are
a) {{x, y}, {y, z}}
b) {{x}, {x, y}, {x, z}}

c) both

9) The contra positive of P Q is equivalent to


b) P Q
c) P Q
a) P Q

d)
\

d) 14
d) none of the above
d) Q P

10) Complemented and distributive lattice is called


a) Complemented lattice b) Boolean function c) Boolean algebra d) Bounded lattice
11) If < G, *, e g > and < S, ,e s > are the given two monoids and direct product of the two
is given by < S G, > then the identity element of the direct product is
b) es
c) < eg , e s >
d) < e s , eg >
a) eg
12) Homomorphism g : X Y is called epimorphism if
a) g is one to one
b) g is onto
c) g is one to one onto d) none of the above
13) In the group G = {2, 4, 6, 8} under multiplication mod 10 the identity element is
a) 6
b) 8
c) 4
d) 2
14) Permutation is ___________ type of mapping.
a) One to one onto
b) One to one
c) Onto
d) None of the above
15) Hasse diagrams are drawn for
a) Boolean algebra b) POSET
c) Lattices
d) For all a, b, c, d
16) If f: X Y is invertible then f1 f is equal to
b) lx
c) f
d) None of the above
a) ly
2
17) An element a G satisfying a = a is called
a) Unity
b) Inverse
c) Identity
d) Idempotent
18) The algebraic system can be represented by
a) Matrix
b) Graph
c) Composition table
d) None of the above
19) <1, + > is the given algebraic system where I = {0, 1, 2, 3} and + is the binary operation.
The <1, +> is
a) Semigroup
b) Groups
c) Monoid
d) None of the above
20) In the lattice given below, how many complements does the element e have ?

a) 0

b) 3

c) 2

______________

d) 1

Set A

*SLRBB248*

-3-

SLR-BB 248

Seat
No.

S.E. (Information Technology) (Part I) Examination, 2014


DISCRETE MATHEMATICAL STRUCTURE
Day and Date : Thursday, 11-12-2014
Time : 10.00 a.m. to 1.00 p.m.

Marks : 80

SECTION I
2. Solve any four :
(54=20)
a) Show that ( P ( Q R) (Q R) (P R) R.
b) Describe the Hasse diagram ? Give the procedure to draw the Hasse diagram with example.
c) A B A B = A prove.
d) What is well formed formula ? Explain with example.
e) What is indirect method of proof explain by proof of contradiction.
3. Attempt any two :

(102=20)

a) ((x+y) 2) + ((x 4)/3) taking above example explain completely parantheized infix and
polish notations.

10

b) Define relation and explain all properties of the relation.

10

c) Explain POSET with its representation. Give some partial ordered relations which are
frequently used ?

10

SECTION II
4. Solve any four :

20

a) Explain Boolean algebra.


b) Explain complemented lattice with example.
c) Let <G, *> be the given algebraic system, where G be the set of non-zero real numbers
and * is the binary operation on G defined by
a*b = ab/4
Show that <G, *> is an abelian group.
d) Show that inverse of an element a in a group is unique.
e) Explain error recovery in group codes.
5. Attempt any one :

10

a) Write a short note on Peano axioms. And show that 2n < n! for all nN.
b) Write a algorithm for converting infix expression containing parenthesis to reverse
polish notation using statck.
6. What do you mean by algebraic system ? What are the different properties of the algebraic
system ?

10

_____________________

Set A

Set A

SLR-BB 249

*SLRBB249*
Seat
No.

Set

S.E. (I.T.) (Part I) Examination, 2014


ADVANCED C CONCEPTS
Day and Date : Saturday, 13-12-2014
Time : 10.00 a.m. to 1.00 p.m.

Max. Marks : 100

Instructions : 1) Q. No. 1 is compulsory. It should be solved in first 30


minutes in Answer book Page No. 3. Each question
carries one mark.
2) Answer MCQ/Objective type questions on Page No. 3 only.
Dont forget to mention, Q.P. Set (A/B/C/D) on Top of Page.
MCQ/Objective Type Questions
Duration : 30 Minutes

Marks : 20

1. Choose the correct alternative :


20
1) Divide and conquer strategy is used in
a) Merge sort
b) Quick sort
c) Both a) and b) d) Neither a) nor b)
2) What is the use of getchar ( ) ?
a) The next input character each time it is called
b) EOF when it encounters end of file
c) Both a) and b)
d) None of the mentioned
3) How to call a function without using the function name to send parameters ?
a) typedets
b) function pointer c) both a) and b)
d) none of above
4) User defined datatype can be derived by ______________
a) struct
b) enum
c) typedef
d) all of the above
5) An array of similar data types which themselves are collection of dissimilar
data type are
a) Pointer
b) Array of structure c) Array of pointers
d) Tree
6) Which operator connects the structure name to its member name ?
a)
b) <
c) Dot
d) Both b) and c)
7) Which of the following cannot be a structure member ?
a) Another structure b) Function c) Array
d) None of above
8) The return type used in string operations are
a) void only
b) void and (char*) only
c) void and int only
d) void, int and (char*) only
P.T.O.

SLR-BB 249

*SLRBB249*

-2-

9) Which of the following function compares two strings with case insensitivity ?
a) strcmp (s, t)
b) strcmpcase (s, t) c) strcase cmp (s, t) d) strchr (s, t)
10) FILE is of type ______________ ?
a) int type
b) char* type

c) struct type

11) What is meant by a in the following operation ?


fp = fopen (Random.txt, a) ;
a) Attach
b) Append
c) Apprehend

d) none of above

d) Add

12) Which of the following cannot be used inside size of ( ) ?


a) Pointers
b) Functions c) Macro definition d) None of above
13) The memory address of the first element of an array is called ___________
a) floor address
b) foundation address
c) first address
d) base address
14) The time complexity of linear search algorithm over an array of n elements is
______________
b) 0(n)
c) 0(n log2 n)
d) 0(n2)
a) 0(log2n)
15) Which of the following sorting algorithm does not have a worst case running
time of 0(n2) ?
a) Insertion sort
b) Merge sort
c) Quick sort
d) Bubble sort
16) A search begins the search with the element that is located in the middle of
the array
a) serial
b) random
c) parallel
d) binary
17) An advantage of chained hash table over the open addressing scheme is
a) worst case complexity of search operation is less
b) space used is less
c) deletion is easier
d) none of above
18) The average search time of hashing with linear probing will be less if the load factor
a) Is far less than one
b) Equal one
c) Is far greater than one
d) None of these
19) Array sizes are optional during array declaration by using ________ key word.
a) auto
b) static
c) extern
d) register
20) Which of the following is used to write the string into file ?
a) fputs ( )
b) fgets ( )
c) gets ( )
d) puts ( )
______________

Set A

*SLRBB249*

-3-

SLR-BB 249

Seat
No.

S.E. (I.T.) (Part I) Examination, 2014


ADVANCED C CONCEPTS
Day and Date : Saturday, 13-12-2014
Time : 10.00 a.m. to 1.00 p.m.

Marks : 80

Instructions : 1) Each Section carries 40 marks.


2) Attempt all questions in each Section.
SECTION I
2. Attempt any four :

(54=20)

a) What are the differences between array of pointers and pointers to array ?
b) What is recursion ? What are the advantages of using recursion ? Give one
example.
c) Write a program to compare two strings irrespective of the case.
d) What are different types of storage classes in C ? Explain in detail.
e) Write a program to sort a set of names stored in any array in alphabetical
order.
3. Attempt any one :

10

a) Solve the Towers of Hanoi problem taking 04 number of disks. Also write a
recursive C program for Towers of Hanoi problem.
b) Explain array of structure and pointer to structure with the help of an example.
4. Attempt any one :

10

a) What is dynamic memory allocation ? Explain the following with the help of
an example :
i) size of ( )
ii) malloc ( )
iii) calloc ( ).
b) Write a recursive C program to generate Fibonacci series.
Set A

SLR-BB 249

-4-

*SLRBB249*

SECTION II
5. Attempt any 4 :

(54=20)

a) Explain the Big O Notation and the O mega Notation clearly.


b) Write a recursive program for Binary Search (Assuming elements are in
ascending order).
c) What is a Hash function ? Explain with an example.
d) Sort the following numbers using insertion sort. Show all passes :
5, 12, 87, 25, 09, 65, 98, 35
e) Explain stream oriented and system oriented files.
6. Attempt any one :

10

a) Explain different types of file operations (any five).


b) Write a program to sort elements using insertion sort. Write time space
complexity of the same.
7. Attempt any one :

10

a) Explain Heap sort briefly. Construct Heap sort of the following numbers :
40, 56, 28, 79, 20, 18, 67, 56
b) Explain Open Addressing and Chaining techniques in Hashing.
_____________________

Set A

SLR-BB 25

*SLRBB25*
S eat
No.

Set

S.E. (Civil) (Part II) Examination, 2014


STRUCTURAL MECHANICS II (Old)
Day and Date : Tuesday, 25-11-2014
Time : 3.00 p.m. to 6.00 p.m.
Instructions :

Total Marks : 100

i) Q. No. 1 is compulsory. It should be solved in first 30 minutes


in Answer book Page No. 3. Each question carries one mark.
ii) Answer MCQ/Objective type questions on Page No. 3 only.
Dont forget to mention, Q.P. Set (A/B/C/D) on Top of Page.
MCQ/Objective Type Questions

Duration : 30 Minutes
1. Tick mark the correct alternative :

Marks : 20
(201=20)

1) Principal planes carry only


a) Normal stress
b) Shear stress

c) Tensile stress

d) Compressive stress

2) Range within which the load can be applied so as not to produce any tensile
stress, is within_____________of the base of any section.
a) First half
b) Second half
c) Middle third
d) Last third
3) If the force of friction is less than the force due to water pressure, the dam will fail due to
a) Overturning
b) Sliding
c) Cracking
d) Crushing
4
d y

4) The expression EI.


a) Slope

gives value of

dx 4
b) Bending moment

c) Shearing force

d) Rate of loading

5) Free end in actual beam is equivalent to ____________ end in conjugate beam.


a) Free
b) Fixed
c) Simply supported
d) Roller
6) A column subjected to external load, will tend to buckle in the direction of ________moment
of inertia.
a) Least
b) Maximum
c) Either a) or b)
d) None
7) Slenderness ratio is the ratio of
a) Effective length to least radius of gyration
b) Actual length to least radius of gyration
c) Radius of gyration to effective length
d) Radius of gyration to actual length
8) Strut is a member which carries only_____________load.
a) Tensile
b) Compressive
c) Shear

d) Torsional

P.T.O.

SLR-BB 25

*SLRBB25*

-2-

9) The ratio of moment of inertia about the neutral axis to the distance of the extreme
fibre from the neutral axis is called
a) Moment of inertia
b) Section modulus
c) Polar moment of inertia
d) Modulus of rigidity
10) Bending strength of a section depends upon
a) Section modulus
b) Bending moment
c) Shear force
d) Support reaction
11) The shear stress on an oblique plane at an angle to the cross section of a body
which is subjected to a direct tensile stress ( ) is equal to

a)

.sin 2
2

b) . cos

c) . cos2

d) . sin2

12) The buckling load will be maximum for a column, if


a) One end of the column is fixed and other free
b) Both ends of the column are fixed
c) Both ends are hinged
d) One end is hinged, other is fixed
13) Slope at the free end of a cantilever subjected to a point load of W at the free end will be
Wl
Wl 2
Wl 2
WL3

a)

EI

b)

c)

d)

EI
2EI
EI
14) Deflection at the centre of a simply supported beam subjected to a udl of W over
the entire span will be
5 Wl 4
5Wl 3
5 Wl 4
5Wl 4

a)

374EI

b)

308 EI

c)

384 EI

d)

384 EI

15) Slopes and deflections in beams with varying moment of inertia, are found by using
a) Double integration method
b) Moment area method
c) Macaulays method
d) Conjugate beam method
16) Influence line diagrams are useful in analysis of
a) Beams in residential struct
b) Beams in water tanks
c) Bridge girders
d) None
17) The first theorem of castigliano gives value of __________in a structural member.
a) Slope
b) Deflection
c) Bending moment d) Shear force
18) Coefficient of wind resistance for a rectangular chimney is
a) 1
b) 2
c) 2
3

1
d) 3
19) For a member subjected to direct stress in one plane, max. shear stress is carried
by a plane which is inclined at __________to the normal section.
a) 25
b) 40
c) 45
d) 60
20) One Pascal means
a) 1 N/mm2
b) 1 kN/m2

c) 1 N/m2

______________

d) 1 kN/mm2

Set A

*SLRBB25*

-3-

SLR-BB 25

Seat
No.

S.E. (Civil) (Part II) Examination, 2014


STRUCTURAL MECHANICS II (Old)
Day and Date : Tuesday, 25-11-2014
Time : 3.00 p.m. to 6.00 p.m.
Instructions :

Marks : 80

i) Q. No. 1 is compulsory in Section I.


ii) Solve any two questions from Q. 2. to Q. 4 in Section I.
iii) Q.No. 5 is compulsory in Section II. Solve any two questions
from Q. 6 to Q. 8.
iv) Assume necessary data if required and mention it clearly.
SECTION I

1. a) Draw neat sketches of core of the following cross-sections.

i) Circle with 200 mm diameter.


ii) Rectangle 300 mm 400 mm
b) A masonry dam, trapezoidal in cross section, 4 m high, 1 m wide at its top and 3 m
wide at its bottom, retains water on its vertical face to a maximum height of 3.5 m
from its base. Determine the maximum and minimum stresses at the base when
reservoir is full. Take unit weight of masonry as
19.62 kN/m3.
10
2. a) State assumptions made in Eulers column theory.
b) A timber column 150 mm 200 mm is 6 m long both ends being hinged. If the
Youngs modulus for timber = 17.5 kN/mm2. Determine :

3
10

i) Crippling load and


ii) Safe load for the column if factor of safety = 3.
3. a) Define:
i) Principal planes

3
ii) Principal stresses.

b) At a point in a strained material, the principal tensile stresses across two


perpendicular planes, are 80 N/mm2 and 40 N/mm2. Determine normal stress,
shear stress and the resultant stress on a plane inclined at 20 with the major
principal plane. Determine also the obliquity. What will be the intensity of stress,
which acting alone will produce the same maximum strain if Poissons
1.
ratio is 4
10

Set A

SLR-BB 25

-4-

*SLRBB25*

4. a) Define :

i) Strength of a shaft

ii) Torsional rigidity.

b) The maximum allowable shear stress in a hollow shaft of external diameter equal
to twice the internal diameter, is 80 N/mm2. Determine the diameter of the shaft if it
is subjected to a torque of 4000 kN mm and a bending moment of
3000 kN mm.
10
SECTION II
5. a) State relations between actual beam and conjugate beam for different end
conditions in a tabular form.

b) A simply supported beam AB of span 4 m carries a point load of 100 kN at its


centre C. The value of I for left half is 1 108 mm4 and for the right half portion is
2108 mm4. Find the slope at support A and deflection under the
load. Take E = 200 GN/m2.
10

6. a) Obtain an expression for strain energy stored due to bending.

b) A simply supported beam AB is subjected to an eccentric load p at a distance of


a from end A and at a distance of b from end B. If span of the beam is L,
find deflection ( ) of the beam under load by using castiglions first theorem.
7. a) State the uses of influence line diagrams.
b) A udl of 60kN/m (Intensity of loading) and length 5 m moves on a girder of
span 16 m. Find :

8
4
9

i) The max. + ve and ve S.F. at a section 5 m from left end.


ii) The max. bending moment at the centre of the girder.
8. a) State maximum principal stress theory.

b) The principal stresses at a point in an elastic material are 80 N/mm2 (tensile),


60 N/mm2 (tensile) and 40 N/mm2 (comp.). If the stress at the elastic limit in simple
tension is 120 N/mm2, determine about possibility of occurrence of failure according
to maximum principal stress theory. If not, then determine
the factor of safety.
10

_____________________
Set A

SLR-BB 250

*SLRBB250*
Seat
No.

Set

S.E. (Information Technology) (Part I) Examination, 2014


DIGITAL TECHNIQUES
Day and Date : Tuesday, 16-12-2014
Time : 10.00 a.m. to 1.00 p.m.

Max. Marks : 100

Instructions : 1) Q. No. 1 is compulsory. It should be solved in first 30


minutes in Answer Book Page No. 3. Each question carries
one mark.
2) Answer MCQ/Objective type questions on Page No. 3 only.
Dont forget to mention, Q.P. Set (A/B/C/D) on Top of Page.
MCQ/Objective Type Questions
Duration : 30 Minutes

Marks : 20

1. Tick the correct answer :

(201=20)

1) Signed 2s complement representation of (15)10 is


a) 11111
b) 10001
c) 01111

d) 10000

2) Which of the following respectively represent commutative law, Associative law


and Distributive law ?
I) A . (B C) = (A . B) . C
II) A . (B + C) = A . B + A . C
III) A + B = B + A
a) I, III, and II
b) II, I and III
c) III, II and I
d) III, I and II
3) A four variable K-map has
a) 8 min terms
b) 12 min terms

c) 32 min terms

d) None of these

4) With regard to a D latch, ______________


a) The Q output follows the D input when EN is LOW
b) The Q output is opposite the D input when EN is LOW
c) The Q output follows the D input when EN is HIGH
d) The Q output is HIGH regardless of ENs input state
5) A full adder circuit has both the inputs 1 and carry-in is also 1. Its sum and carry
outputs will be ______________ and ______________ respectively.
a) 1, 1
b) 0, 1
c) 1, 0
d) 0, 0
6) A mod-5 synchronous counter is designed by using J-K flip flops, the number of
counts skipped by it, will be
a) 2
b) 3
c) 5
d) 0
7) A synchronous sequential circuit is designed to detect a bit sequence 0101
(overlapping sequence include). Every time this sequence is detected, the circuit
produces output of 1. What is the minimum number of states circuit must have ?
a) 4
b) 5
c) 6
d) 7
P.T.O.

SLR-BB 250

*SLRBB250*

-2-

8) A flip flop is popularly known as


a) Astable multivibrator
c) Mono stable multivibrator

b) Bistable multivibrator
d) None of these

9) The terminal count of a modulus-11 binary counter is ______________


a) 1010
b) 1000
c) 1001
d) 1100
10) In a positive edge triggered JK flip-flop, a low J and a low K produce the
______________ state A high J and a high K mean that the output will
______________ on the rising edge of the clock.
a) Active....Race
b) Inactive ..... dead
c) Inactive ..... toggle
d) Active ..... constant
11) The number of control lines for a 8 to 1 multiplexer is
a) 2
b) 3
c) 4

d) 5

12) The device which changes from serial data to parallel data is
a) Counter
b) Multiplexer
c) Demultiplexer d) Flip-flop
13) A device which converts BCD to Seven Segment is called
a) Encoder
b) Decoder
c) Multiplexer

d) Demultiplexer

14) The commercially available 8-input multiplexer integrated circuit in the TTL
family is
a) 7495
b) 74153
c) 74154
d) 74151
15) Which of following cannot be accessed randomly ?
a) DRAM
b) SRAM
c) ROM

d) Magnetic tape.

16) How many address bits are needed to select all memory locations in the
16 K 1 RAM ?
a) 8
b) 10
c) 14
d) 16
17) The storage element for a static RAM is the ______________
a) Diode
b) Resistor
c) Capacitor
d) Flip-flop
18) Which is/are the basic refresh mode(s) for dynamic RAM ?
a) Burst refresh
b) Distributed refresh
c) Open fresh
d) Burst refresh and distributed refresh
19) The format used to present the logic output for the various combinations of logic
inputs to a gate is called a(n)
a) Truth table
b) Input logic function
c) Boolean constant
d) Boolean variable
20) In VHDL, the mode of a port does not define
a) An input
b) An output
c) Both an input and an output
d) The TYPE of the bit
______________
Set A

*SLRBB250*

-3-

SLR-BB 250

Seat
No.

S.E. (Information Technology) (Part I) Examination, 2014


DIGITAL TECHNIQUES
Day and Date : Tuesday, 16-12-2014
Time : 10.00 a.m. to 1.00 p.m.

Marks : 80

SECTION I
2. Solve any four :
a) i) Convert the following expression in to Canonical SOP form

(45=20)

f = AB + A C + C + AD + A B C + ABC .
ii) Minimise expression f of (i) and realise the minimised expression using NAND
gates only.
b) Minimise the following functions and realise using minimum number of
gates :

m (0, 3, 5, 6 9, 10, 12, 15)


f2 = m (0, 1, 2, 3 11, 12, 14, 15) .

i) f1 =
ii)

c) Explain Arithmetic Logic Unit using 74181 IC.


d) What is Flip-Flop ? How S-R Flip Flop is converted to J-K Flip-Flop ?
e) What are the different modes of Operation of Shift register ? Explain any
one of it.
3. Solve any two :

(210=20)

a) Draw and explain operation to carry look ahead adder. Obtain the expression for
carry at each stage for 4 bit adder.
b) What are the Advantages of Synchronous counter over asynchronous counter ?
With the help of neat diagram and wave forms explain 3 bit synchronous counter.
c) What is SOP and POS Expression ? Minimise the following expressions using
K-map and realize using universal gates
f (A, B, C, D) =

m (0, 3, 5, 6, 9, 10, 12, 15)

Set A

SLR-BB 250

*SLRBB250*

-4-

SECTION II
4. Attempt any four of the following :

(45=20)

a) Implement the given Boolean expression F (A, B, C) =


using 4 : 1 MUX.

m (0, 2, 3, 6, 7)

b) Explain the working of decoder with an example.


c) Explain the decoder/driver for 7 segment display using 7447.
d) Explain the timing characteristics of Write cycle.
e) Write a short note on Word capacity.
f) Explain entity and architecture declaration of VHDL with example.
5. Attempt any two of the following :

(102=20)

a) Explain multiplexer tree concept with example and realize 32 : 1 multiplexer


using two 16 : 1 multiplexer and one 2 : 1 multiplexer.
b) Explain pin configuration of IC 74154 and implement the following Boolean
function using IC 74154,

m (0, 1, 2, 4, 6, 8, 11, 13, 15),


= m (0, 1, 4, 7, 11, 15) .

F1 =
F3

F2 =

m (2, 3, 5, 7, 9, 12, 14) and

c) Design a 2048 8 memory using 256 8 memory chip.


d) Explain entity and architecture of VHDL and write a VHDL program to implement
8 : 1 multiplexer.
_____________________

Set A

SLR-BB 251

*SLR-BB-251*
Seat
No.

Set

S.E. (IT) (Part I) Examination, 2014


COMPUTER GRAPHICS
Day and Date : Thursday, 18-12-2014
Time :10.00 a.m. to 1.00 p.m.

Max. Marks : 100

Instructions : 1) All questions are compulsory.


2) Q. No. 1 is compulsory. It should be solved in first 30 minutes
in Answer Book Page No. 3. Each question carries one mark.
3) Answer MCQ/Objective type questions on Page No. 3 only.
Dont forget to mention, Q.P. Set (A/B/C/D) on Top of Page.
MCQ/Objective Type Questions
Duration : 30 Minutes

Marks : 20

1. Choose correct alternative :


20
1) The amount of time the phosphor produce light or shine is controlled by
chemical composition of the phosphor, this is known as
a) Persistence
b) Resistance
c) Generators
d) None
2) Refresh CRT consist of
a) Glass wrapper
b) The phosphor viewing surface
c) The electron gun assembly
d) All of above
3) Reflection about x-axis means ______________
a) x=0
b) y=0
c) y=x
d) y=x
4) The technique to minimizing aliasing are
a) Increased no of resolution
b) Modify pixel intensities
c) Super sampling
d) All of these
5) The transformation in which an object is moved in a minimum distance path
from one position to another is called
a) Translation
b) Scaling
c) Rotation
d) Reflection
6) The selection and separation of a part of text or image for further operation
are called
a) Translation
b) Shear
c) Reflection
d) Clipping
7) If initial value x of a co-ordinate changes to half of its value then scaling
factor is ______________
a) 2
b) 1
c) 0.2
d) 0.5
P.T.O.

SLR-BB 251

-2-

*SLR-BB-251*

8) In reflection which of the following line passes through origin ?


a) x=0
b) y=0
c) y=x
d) all of these
9) Deletion of any segment is much ______________ than creation of any new
segment.
a) Easier
b) Difficult
c) Higher
d) None
10) The transformation that disturbs the shape of an object are called
a) Reflection
b) Shear
c) Rotation
d) Scaling
11) In Warnock algorithm the polygon is totally outside the window is
______________
a) contained
b) intersects
c) surrounds
d) disjoint
12) The process of mapping a world window in world coordinate system to viewport
are called
a) Transformation viewing
b) View Port
c) Clipping window
d) Screen coordinate system
13) The object space in which the application model is defined
a) Screen coordinate system
b) Clipping window or world window
c) World coordinate system
d) None of these
14) User can make any change on image with the use of
a) Non-interactive graphics
b) Interactive graphics
c) Both a) and b)
d) None of these
15) A Bezier curve is a polynomial of degree ______________ the no of control
points used.
a) One more than b) One less than c) Two less than d) None of these
16) The no. of control points in a Bezier curve ensures the
a) Jaggies of curve
b) Smoothness of curve
c) Straightness of curve
d) None of these
17) Why we need removal of hidden surface ?
a) for displaying realistic view
b) for determining the closest visible surface
c) both a) and b)
d) none of these
18) Which surface algorithm is based on perspective depth ?
a) Depth comparison
b) Z-buffer or depth-buffer algorithm
c) Subdivision method
d) Back-face removal
19) The dynamic effect of an image is called
a) Video
b) Animation
c) Super sampling d) None of these
20) One of the disadvantages of multimedia is
a) cost
b) adaptability
c) usability
d) relativity
______________
Set A

*SLR-BB-251*

-3-

SLR-BB 251

Seat
No.

S.E. (IT) (Part I) Examination, 2014


COMPUTER GRAPHICS
Day and Date : Thursday, 18-12-2014
Time :10.00 a.m. to 1.00 p.m.

Marks : 80

SECTION I
2. Attempt any four :

(45=20)

A) Explain RLE in detail


B) Explain working of refresh CRT.
C) Explain 2D translation, 2D scaling.
D) Explain rotation in 3D
E) Explain Edge flag algorithm.
3. Explain Bresenhams circle generation algorithm in detail.

10

4. Equation of line is y = 12 (x + 4). ABC is a triangle having vertices with position


vectors as A[2, 4, 1], B[4, 6, 1], C[2, 6, 1]. Then find out the reflection of
triangle ABC with given line.
10
SECTION II
5. Attempt any four :

(45=20)

A) Explain viewing transformation.


B) Explain Z-buffer algorithm in detail.
C) Explain B spline curves.
D) Explain image and compression.
E) Explain display file compilation.
6. Explain antialiasing and halftoning in detail with e.g.

10

7. Explain Sutherland cohen line clipping algorithm with end point code and simple
visibility test.
10
_____________________

Set A

Set A

SLR-BB 252

*SLRBB252*
Seat
No.

Set

S.E. (IT) (Part II) Examination, 2014


APPLIED MATHEMATICS II (Old)
Day and Date : Tuesday, 25-11-2014
Time : 3.00 p.m. to 6.00 p.m.

Max. Marks : 100

Instructions : 1) Q. No. 1 is compulsory. It should be solved in first 30 minutes in


Answer book Page No. 3. Each question carries one mark.
2) Answer MCQ/Objective type questions on Page No. 3 only. Dont
forget to mention, Q.P. Set (A/B/C/D) on Top of Page.
MCQ/Objective Type Questions
Duration : 30 Minutes

Marks : 20

1. Tick the correct alternatives :

(201=20)

1) Which of the following methods is used to verify the optimality of the current solution of
transportation problem ?
a) Least cost entry method
b) Hungerian method
c) Modified distribution method
d) All of the above
2) In assignment problem, multiple zeros in columns and rows are all indicates
a) multiple optimal solution
b) improvement in present solution
c) non-occurrence of optimal solution
d) degeneracy in optimal solution
3) If there are n workers and n-jobs then there would be
a) (n 1) ! solutions b) n solutions
c) n ! solutions

d) n 1 solutions

4) In optimal simplex table Cj Zj = 0 value indicate


a) Unbounded solution
b) Alternate solution
c) Cycling
d) Infeasible solution
5) In queueing theory the traffic intensity is given by
a) 1

b) 2

c)

1
d) 1

6) Which of the following is false ?


a) Unbalanced transportation problem can be balanced
b) Every optimal solution of transportation problem is feasible
c) In North-West corner method the transportation cost is taken into account
d) All of the above
7) Which of the following characteristics apply to queueing system ?
a) Customer population
b) Arrival process
c) Both a) and b)
d) None of these
8) The slack variables are called as
a) basic variables
c) non-basic variables

b) special variables
d) None of these
P.T.O.

SLR-BB 252

*SLRBB252*

-2-

9) If Poisson distribution is such that P(x = 2) = P(x = 3) then mean of distribution is m =


a) 2
b) 3
c) 4
d) 5
10) For the binomial distribution which of following is true ?
a) mean variance
b) variance mean
d) All of the above
c) p 1 n
11) Which of the following law is violated for Fuzzy sets ?
a) Law of absorption
b) Law of identity
c) Law of contradiction
d) None of these
12) The set of all levels [0 , 1] that represents distinct -cuts of a given fuzzy set A is
called
a) Support of A
b) Level set of A
c) Strong -cut of A
d) None of these
13) State which of the following is true ?
a) Support of fuzzy set contains an element whose membership grade is zero
b) If height of fuzzy set is 1, then it is called subnormal
c) Support of A is equal to strong -cut of A for = 0
d) 0-cut of A is called core of A
14) If A = [4, 10] and B = [1, 2] then A/B = __________
a) [6, 8]
b) [5, 12]
c) [2, 10]

d) None of these

15) If A = [2 1, 3 2] and B = [2 + 1, 5 2] then for (0, 1] , ( A + B ) = ___________


a) [4, 8 4]

b) [4 6, 2 4 ]

16) -cut of a fuzzy set is


a) Fuzzy set
b) Crisp set

c) [4, 4]

d) None of these

c) Void set

d) None of these

17) If A = 0a.2 + 0b.7 + 0c.3 + 0d.0 is a fuzzy set, then scalar cardinality of A is
a) 0.12
b) 0
c) 1
d) 1.2
18) One cut of A i.e. 1A is called
a) Support of A
c) Both a) and b)

b) Core of A
d) None of these

19) S(A, B) = ______________


a)

|A B|
|A|

|A|
b) | A B |

c)

|A B|
|B |

|B|
d) | A B |

20) Which of the following is true ?


a) Fuzzy complement is cutworthy.
b) A B iff B A .
c) B A iff + A +B .
d) Fuzzy complement is not strong cutworthy.
______________

Set A

*SLRBB252*

SLR-BB 252

-3-

Seat
No.

S.E. (IT) (Part II) Examination, 2014


APPLIED MATHEMATICS II (Old)
Day and Date : Tuesday, 25-11-2014
Time : 3.00 p.m. to 6.00 p.m.
Instructions : i)
ii)
iii)
iv)

Marks : 80

Attempt any three questions from each Section.


Figures to the right indicate full marks.
Use of programmable calculator is allowed.
Use of statistical table is allowed.
SECTION I

2. a) Solve the following assignment problem of minimization :


A
B
C
D
P
9
7
6
12
Q
11
13
6
10
R
10
8
9
11
S
9
11
11
12
b) Find optimal solution to following transportation problem to minimize the cost.
S1

S2

S3

S4

S5

Supply

O1

11

150

O2

200

O3

10

14

125

Sales

80

100

75

45

125

3. a) Using Simplex Method :


Maximize Z = 4x1 + 10x2
Subject to
2x1 + x2 50
2x1 + 5x2 100
2x1 + 3x2 90
x1, x2 0.
b) A bank has only one typist. Since the typing work varies in length (number of pages to be
typed) the typing rate is randomly distributed approximating a Poisson distribution with
mean service rate of 8 letters per hour. The letters arrive at a rate of 5 letters per hour,
during 8 hour work duty. If the typewriter is valued at Rs. 1.50 per hour, determine
i) Equipment utilization
ii) The percent time typewriter remains busy
iii) Average system time
iv) Average cost due to waiting on the part of typewriter.
4. a) A manufacturer of airmail envelopes knows from experience that weight of envelopes is
normally distributed with mean 1.95 gms and standard deviation 0.05 gms, about how
many envelopes weighing
i) 2 gms or more
ii) 2.05 gms or more can be expected
iii) given packets of 100 envelopes.
b) The probability density function f(x) of continuous random variable is given by
f( x ) = y o e |x| < x < prove that y o = 12 .

5
4

Set A

SLR-BB 252

5.

*SLRBB252*

-4-

c) A person repairing radios find that, the time spent on the radio set has been exponential
distribution with mean 20 minutes. If the radios are repaired in the order in which they
come in and their arrival is approximately Poisson with the average rate of 15 for 8 hours
a day, what is the repairmans expected idle time each day ? How many jobs are ahead of
the average set just brought in ?
a) The manufacturer of pins knows that on an average 5% of production is defective. He
sales pins in boxes of 100 and guarantees that not more than 4 pins will be defective in how
many boxes out of 1000 he will meet a guaranteed quantity ?
b) Maximize Z = 4x1 + 5x2
Subject to
x1 2x2 2
2x1 + x2 6
x1 + 2x2 2
x1, x2 0.
c) Find Initial basic feasible solution by VAM (Vogels Approximation Method) for the following
transportation problem to minimize the cost
O1
O2
O3
Demand

D1
2
1
5
7

D2
3
o
8
5

D3
11
6
15
3

D4
7
1
9
2

5
4

Supply
6
1
10
17

SECTION II
6. a) Verify law of absorption for fuzzy sets.

b) Let A and B are two fuzzy sets defined by A = 01.5 + 02.6 + 03.9 + 41 + 05.2 , B = 03.3 + 04.4 + 05.7 + 61 + 07.3
Let the function F : X X X be defined as F(x1, x2) = 2x1 + x2, find F(A, B).
c) Define Fuzzy number.

7
3

0.2 0.4 0.6 0.8


7. a) Find special fuzzy sets of A where A = x + x 2 + x 3 + x4 + x15 .
1

b) Let A and B are the Fuzzy numbers as below : A = 0.2 + 0.6 + 0.8 + 0.7 + 1 + 0.5 + 0.1
[0, 1)

[1, 2)

[ 2, 3)

[3, 4)

(4, 5]

and B = 0.1 + 0.2 + 0.6 + 0.7 + 0.8 + 0.9 + 61 + 0.5 + 0.4 + 0.2 + 0.1
[ 0, 1) [1, 2) [ 2, 3) [ 3, 4) [ 4, 5) [ 5, 6)
(6, 7] (7, 8] (8, 9] (9, 10]
Solve the fuzzy equation A + X = B.

(5, 6]

8. a) Represent the interconnections among modules of Fuzzy controller by using block diagram. 4
b) Define Fuzzy cardinality for fuzzy set. Consider fuzzy sets A and B whose membership
functions are defined by formulae A(x ) = x and B( x) = 1 x for all x {0, 1, 2, ..., 10} = X .
x +1
10
Calculate i) Scalar and Fuzzy cardinality of A and B. ii) Degrees of subsethood

9.

S (| A |, | B |) and S (| B |, | A |).
a) Find -cut and strong -cut for = 0.1, 0.5, 0.3 for the following fuzzy sets :
A = 0x.6 + 0x.5 + 0x.7 + 0x.3 + x1 , B = 1 + 110 x where x {0, 1, 2, ..., 10}.
2
3
4
5
1

b) Explain in short indirect method with one expert for constructing membership function.

_____________________

Set A

SLR-BB 253

*SLRBB253*
Seat
No.

Set

S.E. (IT) (Part II) (Old) Examination, 2014


FORMAL SYSTEMS AND AUTOMATA
Day and Date : Wednesday, 26-11-2014
Time : 3.00 p.m. to 6.00 p.m.

Max. Marks : 100

Instructions : 1) Figures to right indicate full marks.


2) Q. No. 1 is compulsory. It should be solved in first 30 minutes
in Answer book Page No. 3. Each question carries one mark.
3) Answer MCQ/Objective type questions on Page No. 3
only. Dont forget to mention, Q.P. Set (A/B/C/D) on Top
of Page.
MCQ/Objective Type Questions
Duration : 30 Minutes
1. Tick the correct alternative :
1) Tape symbols are a part of
a) DFA
c) PDA
2) Ambiguity in grammar is due to
a) Symbolic manipulation
c) Conjunctions
3) A multitape TM is equivalent to a
a) FA
c) Single tape TM

Marks : 20
20
b) NFA
d) TM
b) Parse trees
d) Disjunctions
b) PDA
d) NFA

4) L + L = L is _____________ low of union


a) Distributive
b) Closure
c) Idempotent
d) Annihitators
5) ATM is ____________ powerful than DFA.
a) Less
b) More
c) Very less
d) Not as much
6) PDA uses a _____________
a) Tree
b) Queue
c) Link list
d) Stack
7) A regular expression can be developed __________
a) Exponentially
b) Annually
c) Recursively
d) Normally
P.T.O.

SLR-BB 253

*SLRBB253*1

-2-

8) A final state in NFA ____________ be a start state.


a) Cannot
b) Can
c) Never can
d) Always can
9) L1 L 2 is ___________ of two languages.
a) Union
b) Intersection
c) Closure
d) Product
10) PDA accepts strings by empty stack and _____________
a) Empty stack
b) Empty input
c) Both a) and b)
d) None of these
11) Pumping Lemma is used to prove ______________ of languge.
a) Regularity
b) Irregularity
c) Context free
d) Context sensitive
12) (a + b)* a (a + b)* are
a) Odd length strings
b) Even strings
c) All odd strings
d) Strings that have atleast one a
13) |uv| < n is a condition for
a) Pigeonhole
c) Pumping Lemma

b) FA
d) Moore

14) is a
a) Regular expression
c) Empty move

b) Non regular expression


d) Exponent

15) A is accepted by DFA.


a) Regular string
c) None

b) Non regular string


d) Empty string

Say true or false for statements 16 to 20 :


16) NFA is equivalent to DFA.
17) Both acceptances for PDA are equivalent.
18) Every CFL is non regular
19) CFG is equivalent to context sensitive grammar.
20) Type 2 grammar is context free.
______________

Set A

*SLRBB253*

-3-

SLR-BB 253

Seat
No.

S.E. (IT) (Part II) (Old) Examination, 2014


FORMAL SYSTEMS AND AUTOMATA
Day and Date : Wednesday, 26-11-2014
Time : 3.00 p.m. to 6.00 p.m.

Marks : 80

Instructions : 1) All questions are compulsory.


2) Figures to right indicate full marks.
SECTION I
2. Attempt any four :
a) Define regular expressions giving examples.
b) If L1 and L2 are CFL prove that L1 L 2 is also a CFL.
c) Draw NFA for 01* + 1 (0 + 1)*.
d) Define type 0 and type 1 grammars. Give examples.
e) Where are DFA and NFA used ?

(45=20)

3. Attempt any one :


10
a) What is a CNF. Explain the steps for converting a CFG to CNF.
OR
b) How is ambiguity in a grammar dealt with ? Give the steps to remove ambiguity.
4. What is mathematical Induction ? Where is it applied ?

10

SECTION II
5. Answer any four:
a) What is the application of Push down automata ?
b) List the different variations in Turing machine.
c) State Pumping Lemma. Prove anbncn | n 1 is not a CFL.
d) Compare PDA with TM.
e) Design a TM to compute x + y where x and y are binary numbers.

(45=20)

6. Answer any one :


10
a) For an TM, generate a minimum state TM and draw its state transition diagram.
(elaborate all steps)
OR
b) Explain the steps carried out in top-down parsing.
7. What is an encoding function ? Explain it with examples.

10

_____________________

Set A

Set A

SLR-BB 254

*SLRBB254*
Seat
No.

Set

S.E. (IT) (Part II) (Old) Examination, 2014


MICROPROCESSORS
Day and Date : Thursday, 27-11-2014
Time : 3.00 p.m. to 6.00 p.m.

Max. Marks : 100

Instructions : 1) Q. No. 1 is compulsory. It should be solved in first 30 minutes


in Answer book Page No. 3. Each question carries one mark.
2) Answer MCQ/Objective type questions on Page No. 3 only.
Dont forget to mention, Q.P. Set (A/B/C/D) on Top of Page.
MCQ/Objective Type Questions
Duration : 30 Minutes

Marks : 20

1. 1) ____________ register deals with sequencing the execution of instructions.


a) Stack pointer
b) Program counter
c) Accumulator
d) Flag
2) ____________ instruction is used to call a procedure.
a) CALL
b) RET
c) PROC
d) NEAR
3) One of the following addressing modes is not possible in 8085.
a) Indexed addressing
b) Indirect addressing
c) Direct addressing
d) Indirect register address
4) Which of the following is does NOT belong to 8085 flag register ?
a) Carry flag
b) Sign flag
c) Zero flag
d) One flag
5) __________ and __________ instructions are specifically used to change
the status of carry flag.
a) STI, CLI
b) STC, CMC
c) MOV, LEA
d) DAA, XCHG
6) __________ IC is designed specifically to work with 8085 as programmable
interrupt controller, it contains __________ number of pins.
a) 8259, 28
b) 8269, 28
c) 8279, 26
d) 8299, 26
7) Which of the following is programmable IO port is ?
a) 8255
b) 8251
c) 8085
d) none of these
8) Which of the instruction is used to selectively mask an interrupt ?
a) DI
b) EI
c) SIM
d) RIM
9) Which register is not available for user in microprocessor ?
a) W
b) Z
c) B
d) Both a) and b)
P.T.O.

SLR-BB 254

-2-

*SLRBB254*

10) In Memory Mapped I/O the number of Input and Output Address will not be
256 instead whole 64K is __________ by Input, Output, Memory.
a) Attached
b) Connected
c) Shared
d) Both b) and c)
11) In Slave mode of 8257, DMA is ________ and Microprocessor is ________
a) Slave, Master b) Slave, Slave c) Master, Slave d) Master, Master
12) Which of the following is compulsory Initialization Command Word ?
a) ICW1 and ICW3
b) ICW2 and ICW3
c) OCW3
d) ICW1 and ICW2
13) Which of the following are operating modes of 8255 PPI ?
a) BSR Mode
b) I/O Mode
c) Both BSR and I/O Mode
d) None of these
14) Which one of the following instruction may be used for clearing accumulator
content ?
a) XRA A
b) SUB A
c) MVI A, 00H
d) All of these
15) How many counters are there is 8253 ?
a) One
b) Two
c) Three
d) Four
16) When S1 = 1, S0 = 1 then following operation is done in Micro Processor ?
a) Opcode Fetch
b) Memory Read
c) Memory Write
d) Halt
17) The maximum number of I/O devices which can be interfaced in memory
mapped I/O technique are
a) 256
b) 128
c) 65536
d) 32768
18) Which of the following is an 8085 hardware interrupt ?
a) TRAP
b) RST6.5
c) RST7.5
d) All the above
19) 8257 provides onchip __________
a) External Peripheral
b) Status Signal
c) Control Register
d) Priority Resolver
20) How many T-states are required for LHLD address instruction ?
a) 16
b) 14
c) 18
d) 10
______________

Set A

*SLRBB254*

-3-

SLR-BB 254

Seat
No.

S.E. (IT) (Part II) (Old) Examination, 2014


MICROPROCESSORS
Day and Date : Thursday, 27-11-2014
Time : 3.00 p.m. to 6.00 p.m.

Marks : 80

SECTION I
2. Attempt any four :

(44=16)

a) Write the 8085 features and explain what is addressing mode.


b) Load the data byte ABH in register C. Mark the high order bits (D7 D4) and
display the low order (D3 D0) at an output port.
c) Draw timing diagram for Logical memory related instructions XRA M.
d) Differentiate memory mapped I/O and I/O mapped I/O.
e) How demultiplexing of address and data bus takes ?
3. Attempt any three :

(83=24)

a) Interface 4K 8 bit memory of 8085 using 1K 8 bit ROM memory chips.


Give address range of each chip.
b) Explain BRANCH group of instructions with example.
c) Interface 6K 8 memory to 8085 using 2K 8 memory chips. Select starting
address 8000H. Give the address range of each chip used.
d) Compare following :
a) RRC and RLC
b) RAL and RAR
c) CMA and CMC
d) PUSH Rp and POP Rp
Set A

SLR-BB 254

-4-

*SLRBB254*

SECTION II
4. Attempt any four :

(44=16)

a) Explain in detail operation of R-2R ladder type of DAC.


b) Explain features of 8255.
c) Explain ICW1 and ICW2 of 8259 PIC in detail.
d) Draw and explain SIM instruction bit format.
e) Explain mode 0 of 8253.
5. Attempt any three :

(83=24)

a) Explain interrupt structure of 8085 with neat diagram.


b) Explain 8251 with block diagram.
c) What is DMA ? Explain operation of data transfer with help of Block Diagram.
d) Interface ADC 0809 to 8085 using 8255.
_____________________

Set A

SLR-BB 255

*SLRBB255*
Seat
No.

Set

S.E. (I.T.) (Part II) Examination, 2014


DATA STRUCTURE II (Old)
Day and Date : Friday, 28-11-2014
Time : 3.00 p.m. to 6.00 p.m.

Total Marks : 100

Instructions : 1)
2)
3)
4)

All questions from Section I and II are compulsory.


Figures to the right indicate full marks.
Assume data if necessary.
Q. No. 1 is compulsory. It should be solved in first 30 minutes in
Answer book Page No. 3. Each question carries one mark.
5) Answer MCQ/Objective type questions on Page No. 3 only. Dont
forget to mention, Q.P. Set (A/B/C/D) on Top of Page.
MCQ/Objective Type Questions

Duration : 30 Minutes
1. Objective type questions. Choose the correct option.

Marks : 20
20

1) Two main measures for the efficiency of an algorithm are


a) Processor and memory
b) Complexity and capacity
c) Time and space
d) Data and space
2) The time factor when determining the efficiency of algorithm is measured by
a) Counting micro seconds
b) Counting the number of key operations
c) Counting the number of statements
d) Counting the kilobytes of algorithm
3) The space factor when determining the efficiency of algorithm is measured by
a) Counting the maximum memory needed by the algorithm
b) Counting the minimum memory needed by the algorithm
c) Counting the average memory needed by the algorithm
d) Counting the maximum disk space needed by the algorithm
4) Which of the following case does not exist in complexity theory ?
a) Best case
b) Worst case
c) Average case
d) Null case
5) The Worst case occur in linear search algorithm when
a) Item is somewhere in the middle of the array
b) Item is not in the array at all
c) Item is the last element in the array
d) Item is the last element in the array or is not there at all
6) The Average case occur in linear search algorithm
a) When Item is somewhere in the middle of the array
b) When Item is not in the array at all
c) When Item is the last element in the array
d) When Item is the last element in the array or is not there at all
P.T.O.

SLR-BB 255

*SLRBB255*

-2-

7) The complexity of the average case of an algorithm is


a) Much more complicated to analyze than that of worst case
b) Much more simpler to analyze than that of worst case
c) Sometimes more complicated and some other times simpler than that of worst case
d) None or above
8) The complexity of linear search algorithm is
a) O(n)
b) O(log n)
c) O(n2)

d) O(n log n)

9) The complexity of Binary search algorithm is


a) O(n)
b) O(log)
c) O(n2)

d) O(n log n)

10) The complexity of Bubble sort algorithm is


a) O(n)
b) O(log n)

c) O(n2)

d) O(n log n)

11) The complexity of merge sort algorithm is


a) O(n)
b) O(log n)

c) O(n2)

d) O(n log n)

12) The indirect change of the values of a variable in one module by another module is called
a) Internal change
b) Inter-module change
c) Side effect
d) Side-module update
13) Which of the following data structure is not linear data structure ?
a) Arrays
b) Linked lists
c) Both of above
d) None of above
14) Which of the following data structure is linear data structure ?
a) Trees
b) Graphs
c) Arrays

d) None of above

15) The operation of processing each element in the list is known as


a) Sorting
b) Merging
c) Inserting
d) Traversal
16) Finding the location of the element with a given value is
a) Traversal
b) Search
c) Sort

d) None of above

17) Arrays are best data structures


a) For relatively permanent collections of data
b) For the size of the structure and the data in the structure are constantly changing
c) For both of above situation
d) For none of above situation
18) Linked lists are best suited
a) For relatively permanent collections of data
b) For the size of the structure and the data in the structure are constantly changing
c) For both of above situation
d) For none of above situation
19) Each array declaration need not give, implicity or explicity, the information about
a) The name of array
b) The data type of array
c) The first data from the set to be stored d) The index set of the array
20) The elements of an array are stored successively in memory cells because
a) By this way computer can keep track only the address of the first element and the
addresses of other elements can be calculated
b) The architecture of computer memory does not allow arrays to store other than serially
c) Both of above
d) None of above

______________

Set A

*SLRBB255*

-3-

SLR-BB 255

Seat
No.

S.E. (I.T.) (Part II) Examination, 2014


DATA STRUCTURE II (Old)
Day and Date : Friday, 28-11-2014
Time : 3.00 p.m. to 6.00 p.m.

Marks : 80

Instructions : 1) All questions from Section I and II are compulsory.


2) Figures to the right indicate full marks.
3) Assume data if necessary.
SECTION I
2. Attempt any four of the following :

(54=20)

a) Explain asymptotic notation in detail.


b) Explain Quick sort. Also write an algorithm.
c) Illustrate the operation of heap sort algorithm on the array A = 5, 13, 2, 25, 7, 17, 20, 8, 4
also specify the worst case time Complexity.
d) What is hashing ? List important features of a good hashing algorithm.
e) Explain the advantage of Quadratic probing over linear probing.
f) What is sequential search ? Explain with an example.
3. Consider the array of elements [1....10] and elements are
310, 285, 179, 625, 351, 423, 861, 254, 450, 520. Apply merge sort and show the
complete trace. To get the sorted output discuss the time complexity.

10

4. Write an algorithm of an insertion sort. Illustrate analysis of an insertion sort


with an example.

10

Set A

SLR-BB 255

-4-

*SLRBB255*

SECTION II
5. Attempt any four of the following :

(54=20)

a) What is multiway search tree ? Explain with an example.


b) Define Height balance AVL Tree explain LL Rotation, RR Rotation, LR Rotation, RL
Rotation.
c) What is binary tree ? Mention the properties of a binary tree.
d) What is graph ? Explain two types of Representation of graphs.
e) Draw a graph of five vertices each of degree 4.
6. Construct the tree if preorder and inorder traversal of binary tree is given.

10

Preorder : A B C D F H J M K E G I L N
Inorder : A

D J M H K F C

N L G E B

7. Write an algorithm for breadth first search (BFS). Write the time and space

10

requirement of the algorithm.

_____________________

Set A

SLR-BB 256

*SLRBB256*
Seat
No.

Set

S.E. (Information Tech.) (Part II) Examination, 2014


COMPUTER NETWORKS I (Old)
Day and Date : Saturday, 29-11-2014
Time : 3.00 p.m. to 6.00 p.m.

Max. Marks : 100

Instructions : 1) Q. No. 1 is compulsory. It should be solved in first 30


minutes in Answer book Page No. 3. Each question carries
one mark.
2) Answer MCQ/Objective type questions on Page No. 3 only.
Dont forget to mention, Q.P. Set (A/B/C/D) on Top of Page.
MCQ/Objective Type Questions
Duration : 30 Minutes

Marks : 20

1. Choose correct alternatives :


1) ______________ model does not have session or presentation layers.
a) OSI
b) TCP/IP
c) ISO
d) Both a) and c)
2) In OSI, ______________layer is concerned with controlling the operation of
the subnet.
a) DLL
b) MAC
c) Network
d) Transport
3) ______________uses DQDB.
a) LAN
b) MAN
c) WAN
d) None of above
4) Transmitting data or packet to a subset of the machines is known as_________
a) Broadcasting
b) Multicasting
c) Point-to-point
d) All
5) In CRC, if generator is n-bits long, then ______________zero bits are
appended to the original message of m-bits.
a) n
b) n 1
c) m 1
d) None of the above
6) Protocols in which sender sends one frame and then waits for an
acknowledgment before proceeding are called ______________ protocols.
a) Sliding window b) Stop and wait c) Simplex
d) None
7) Which of the following is not a framing method used by DLL ?
a) Character count
b) NRZ
c) Physical layer coding violation
d) Starting and ending flags, with bit stuffing
P.T.O.

SLR-BB 256

-2-

*SLRBB256*

8) ______________ of a signal is the range of frequencies that it contains


a) Bandwidth
b) Spectrum
c) Absolute bandwidth
d) Phase
9) A signal with a component of zero frequency is ______________
a) dc component
b) ac component
c) digital signal
d) all
10) Which of the following is an example of analog data ?
a) Audio
b) Video
c) Text
d) None of the above
11) In sliding window protocol, the receiver simply discard all subsequent frames,
sending no acknowledgement for the discarded frame is______________
a) Pipeline
b) Selective repeat
c) Go back n
d) None
12) ARP solves the problem of finding out which ______________ address
corresponds to a given IP address.
a) Ethernet
b) Network
c) Host
d) None
13) ______________is a process that extracts the address of the physical
network from an IP address.
a) Masking
b) Subnetting
c) Supernetting d) Classing
14) In 802.3, 10 Base 5 cabling is called ______________
a) Ethernet
b) Thick ethernet
c) Thin ethernet
d) All
15) MAC address is of ______________bits.
a) 32
b) 64
c) 48

d) 6

16) Which policy of network layer affects congestion ?


a) Flow control
b) ACK policy
c) Packet discard policy
d) All
17) TCP uses ______________type of forwarding to transmit message from
source to destination.
a) Connection less
b) Connection oriented
c) Both
d) None
18) In IP header format ______________ field is a counter used to limit packet
life times.
a) Time to live
b) IHL
c) DF
d) MH
19) The technique in which incoming packet is sent on these lines that are going
approximately in the right direction is ______________
a) Flooding
b) Flow-based routing
c) Selective flooding
d) Symmetric flooding
20) ______________is inter domain routing protocol.
a) OSPF
b) IP
c) BGP
d) All
______________
Set A

*SLRBB256*

-3-

SLR-BB 256

Seat
No.

S.E. (Information Tech.) (Part II) Examination, 2014


COMPUTER NETWORKS I (Old)
Day and Date : Saturday, 29-11-2014
Time : 3.00 p.m. to 6.00 p.m.

Marks : 80

Instruction : All questions are compulsory.


SECTION I
2. Solve any four (each carries 5 marks) :

20

A) What is framing ? Explain different types of framing methods.


B) Describe one-bit sliding window protocol.
C) List and explain uses of computer network.
D) Describe LAN and MAN.
E) Explain Bipolar-AMI and pseudoternary.
F) State advantages of digital data transmission.
3. Describe twisted pair with the help of its physical description, transmission
characteristics and applications.

10

OR
Describe following in detail.
i) TCP/IP protocol

10
ii) ATM reference model

4. How the message 10101101 is transmitted using :

10

i) Hamming code
ii) CRC (consider generator is 1011)
Set A

SLR-BB 256

-4-

*SLRBB256*

SECTION II
5. Attempt any four :

(45=20)

1) What is routing ? Explain flooding in detail.


2) What is medium access control ? Explain CSMA/CD in detail.
3) Write RARP in detail.
4) Discuss network layer design issues.
5) Write a note on subnetting and supernetting.
6. Explain all collision free protocols.

10

OR
Enlist inter networking devices. What is bridge ? Explain transparent bridge and
source routing bridge in detail.
7. Write short note on :

10

1) Leaky bucket algorithm.


2) OSPF

_____________________

Set A

SLR-BB 257

*SLR-BB-257*
Seat
No.

Set

S.E. (IT) (Part II) (New) Examination, 2014


APPLIED MATHEMATICS II
Day and Date : Tuesday, 25-11-2014
Time : 3.00 p.m. to 6.00 p.m.

Max. Marks : 100

N.B. : 1) Question No. 1 is compulsory. It should be solved in first 30 minutes


in Answer book Page No. 3. Each question carries one mark.
2) Use of scientific calculator is allowed.
3) Answer MCQ/Objective type questions on Page No. 3 only. Dont
forget to mention, Q.P. Set (A/B/C/D) on Top of Page.
MCQ/Objective Type Questions
Duration : 30 Minutes

Marks : 20

1. Tick (9)mark the correct alternative (one mark each) :


1) Newtons iterative formula to find approximate value of
number is _____________
a) xn+1 =

1
2

(x

N
xn

b) xn+1 =

1
2

(x

1
Nx n

20
1
N

where N is a positive real

c) xn+1 = xn (2 Nxn) d) xn+1 = 2 x n +

1
Nx n

2) Every equation of the nth degree has _________ roots.


a) n
b) n + 1
c) n 1
d) 1
3) The order of convergence of Regula falsi method for finding root of equation f(x) = 0 is
a) Second order
b) Cubic order
c) First order
d) Very slow
4) In solving simultaneous linear equations which of the following method is iterative method ?
a) Factorization
b) Jacobis
c) Elimination
d) Jordan
5) The power method is used to find __________
a) smallest eigen value of the matrix
b) only integer valued eigen value
c) largest eigen value of the matrix
d) none
6) When Gauss-Elimination method is used to solve set of equation AX=B, matrix A is
transformed to
a) upper triangular matrix
b) Diagonal matrix
c) Identity matrix
d) Scalar matrix

7) To apply Simpsons 13 rd rule the number of sub-intervals must be


a) odd
b) multiple of 3
c) any

d) even

8) Which of the following is the Gaussian two-point formula ?


1

a)

c)

1
1

+ f
f(x) dx = f
3
3

b)

1
1

+ f
3
3

f(x) dx = f
0

1
1

+ f
3
3

f(x) dx = f

d) None

9) The rule of integration which is most accurate is


a) Trapezoidal

3
b) Simpsons
8

th

1
c) Simpsons
3

rd

d) Weddles rule
P.T.O.

SLR-BB 257

*SLR-BB-257*

-2b

10) If I1 and I2 represent approximate value of integration I = f(x)dx , obtained by trapezoidal


a

h
then by Rombergs method I is obtained by
rule by taking the spacing for x as h and
2
I I
a) I = I2 2 1
3

I + I
b) I = I2 + 2 1
3

I I
c) I = I2 + 2 1
3

d) None

11) For any fuzzy set A defined on universal set X, strong 1 cut of A is
a) a closed interval b) an open interval c) empty set
d) none of the above
12) The subtraction of closed interval [2, 4] [3, 6] =
a) [5, 2]
b) [5, 2]
c) [5, 2]
13) If ((x) =

1
and D(x) =
1 + 5x

a) C D

d) [8, 1]

, x 0 then ordering of fuzzy sets is


1 + 5x

b) D C

d) C = D

c) C = D

14) Consider the fuzzy sets


A=

0.4 0.6 1 0.34 0.12


+
+ +
+
x
y
z
u
v

B=

0.3 0.5 0.67 0.9 0.8 0.32


+
+
+
+
+
x
y
z
u
v
w

then the scalar cardinality | A B | =


a) 4.02

b) 3.7

c) 5.95

d) 5.63

x
, 0 x < then the level set of A(x) is the interval
x +1
b) [0, 1]
c) [1,1]
d) [0, 1)

15) Consider the fuzzy set A(x) =


a) (0, 1)

16) Let A be a fuzzy set, then the fuzzy complement of A is defined as


a) A (X) = A(X)

b) A (X) = A1(X)

c) A (X) = 1 A(X)

1
d) A (X) = A X

17) The fuzzy proposition P, taking the form P : If X is A the Y is B is


a) conditional and qualified
b) conditional and unqualified
c) unconditional and qualified
d) unconditional and unqualified
18) The crisp rule p q ; q R p R is known as
a) disjunctive syllogism
b) hypothetical syllogism
c) modus ponens
d) modus tollens

19) Using Lukasiewicz implication j 0, 1 =


2
1
a) 0
b) 2

c) 1.5

d) 1

tan x for 0 x
cos x 0 x / 2
20) Consider (i) A(x)
(ii) B( x)
otherwise
otherwise
0
0
then the fuzzy number is
a) only (i)

b) only (ii)
c) both (i) and (ii)
______________

d) None of (i) and (ii)

Set A

*SLR-BB-257*

SLR-BB 257

-3-

Seat
No.

S.E. (IT) (Part II) (New) Examination, 2014


APPLIED MATHEMATICS II
Day and Date : Tuesday, 25-11-2014
Time : 3.00 p.m. to 6.00 p.m.

Marks : 80

Instructions : 1) Attempt any three questions from each Section.


2) Figures to the right indicate full marks.
3) Use of calculator is allowed.
SECTION I
2. a) Find the positive real root of the equation cosx 3x + 1 = 0 using Regula falsi method.

b) Perform two iterations of the Newton-Raphson method to solve non-linear equations


x2 + y2 = x and y = x2 y2 starting with initial conditions as (0.8, 0.4).

c) Solve the system of equations by Gauss-Jordan method :


2x 3y + 4z = 13
x+y+z=9
3x + 4y + 5z = 40
3. a)

1 1
i) Find complex eigen value for the matrix
.
0 .4 0 .6

5 4
ii) Determine the largest eigen value and corresponding eigen vector of the matrix
.
1 2
1
Take X0 = using power method.
0

b) Solve the system of equations


27x + 6y z = 85
x + y + 54z = 110
6x + 15y + 2z = 72
by using Gauss-Jocobis and Gauss Seidal method.
0 .5

4.

a) Evaluate the integral

sin x dx using Rombergs method correct to three decimal

places taking h = 0.25, 0.125, 0.0625.


rd

b)

1
rule to evaluate the integral
Apply Simpsons
3

c) Using N-R method find an approximate value of

6
2 .6 4 .4

12 .

dx dy
xy .

4
3

Set A

SLR-BB 257

*SLR-BB-257*

-4-

th

3
5. a) Using Simpsons
rule evaluate
8

x dx
5 + 2x , n = 8.

4
3

b) Using Gaussian quadrature three point formula evaluate

cos 2x
dx.
1 + sin x

c) Using Mullers method find an approximate root of the equation cosx = x ex take x0 = 1,
x1 = 0, x2 = 1 (perform 1 iteration).

OR
c) Using Bisection method find an approximate root of the equation x sin x = 1 (perform 6
iterations).

SECTION II
6.

a) Describe the concept of fuzzy set in your own words. Give some examples of fuzzy
variables from daily life.

b) Prove that F(x) : fuzzy power set of sets defined on the universal set X forms de-Morgans
algebra.

OR
b) Prove the following properties of fuzzy sets.
i) If then A A and + A
+
A

iii) ( A B ) = A B

6
ii) ( A B ) = A B
iv) A = (1 )+ A .

c) If A and B are fuzzy sets defined on universal set X = set of integers given by

A =
B=

0 . 5 1 0 .5 0 . 3
+ +
+
1 0
1
2
0.5 1 0.5 0.3
+ +
+
2
3
4
5

A function f : X X X defined by f(x1, x2) = 2x1 + x2 find f(A, B).

7. a) Let

(x + 2) / 2 for

A(x) = (2 x) / 2 for
0

(x 6) / 2 for

C(x ) = (10 x) / 2 for


0

( x 2) / 2

B( x) = (6 x) / 2
0

2< x 0
0<x2
otherwise
6<x8
8 < x 10
otherwise

for

2<x4

for

4<x6
otherwise

Solve for X the equation i) A + X = B and ii) B.X = C.

Set A

*SLR-BB-257*

-5-

SLR-BB 257

x
x
B(x) = 1
x {0, 1, 2, 3, ...,10} find fuzzy
x +1
10
~
~
~
~
cardinality of A and B i.e. | A | and | B | also find degree of subsethood s(| A | , | B |) and
~
~
S (| B |, | A |).

b) Consider the fuzzy sets A(x) =

8. a) Explain briefly :
i) Conditional and unqualified
ii) Conditional and qualified fuzzy proposition. Discuss their forms.

b) Find -cuts and strong -cuts for each of the following fuzzy sets for values of = 0.2, 0.5, 0.8, 1.
A(x) =

x
1
B( x) = 2 x C( x) =
where x {0, 1, 2, 3, 4}.
x+2
1 + 10( x 2)2

c) Find the following for the two given fuzzy sets.


i) height of A and B
ii) Are A and B normal
iii) Sketch the graphs of A and B.
A(x ) x + 2

= 2x

for 2 < x 0
for

0<x2
otherwise

B( x) = (x 2) / 2

(6 x ) / 2
0

2<x4
0<x6
otherwise

9. a) Let A and B be two fuzzy no. whose membership functions are given by
x + 2
3

A( x ) = 4 x
3

0
x 1

B( x) = 3 x
0

find the two fuzzy

2 < x 1
1< x 4
otherwise
1< x 2
2<x3
otherwise
numbers MAX (A,B) (x) MIN (A,B) (x).

Set A

SLR-BB 257

-6-

b) Complete the following table where :

*SLR-BB-257*
5

a b = min (a, b)
a b = max (a, b)
a b = min (a, 1 a + b)
a b = 1 | a b|
a

1
2
1

0
1
2
1
2
1
2
1
1
1

ab ab a b a b

0
1
2
1
0
1
2
1
_____________________

Set A

SLR-BB 258

*SLRBB258*
S

S.E. (Information Technology) (Part II) (New) Examination, 2014


THEORY OF COMPUTATION
Day and Date : Wednesday, 26-11-2014
Time : 3.00 p.m. to 6.00 p.m.
Instructions :

Total Marks : 100

1) Figures to the right indicate marks to a question.


2) Assume suitable data wherever necessary.
3) Q. No. 1 is compulsory. It should be solved in first 30 minutes in
Answer book Page No. 3. Each question carries one mark.
4) Answer MCQ/Objective type questions on Page No. 3 only. Dont
forget to mention, Q.P. Set (A/B/C/D) on Top of Page.
MCQ/Objective Type Questions

Duration : 30 Minutes

Marks : 20

1. Choose the correct alternative(s) :

20

1) Find the equivalent regular expressions ?


i) (00)*(^ + 0)

ii) (00)*

iii) (0)*
a) i) and iv)

iv) 0(00)*
b) ii) and iii)

c) i) and iii)

d) iii) and iv)

2) One language can be expressed by more than one FA. This statement is ____________
a) True
b) False
c) Sometimes true and sometimes false
d) None
3) Regular expression recognizing language of strings containing substring 00 is
b) (1 + 01)*

a) (1 + 10)*

c) (01 + 10)*

d) (10)*

4) If Automata M1 has 3 states and M2 has 4 states then automata accepting language
L1
L2 has ______________ states.
a) 12
b) 7
c) 4
d) 3


5) What is true about finite automata ?


a) Acceptor
c) Pattern matcher

b) Recognizer
d) All

6) The language of all words (made up of as and bs) with at least two as cannot be described
by the regular expression.
a) a(a + b) a (a + b) (a + b) ab
b) (a + b) aba (a + b)
c) baba (a + b)
d) None of these
7) Transition function for NFA is ____________ where for (Q,
a)
c)

:Q
:Q(


Q
U {^})


2Q

b)
d)


:Q
:Q

, q0,





Q*
2Q

, A).

P.T.O.

SLR-BB 258

*SLRBB258*

-2-

8) Which of the following is NOT a regular language ?


a) string of 0s whose length is a perfect square
b) set of all palindromes made up of 0s and 1s
c) string of 0s whose length is a prime number
d) all of the above
9) L = {anbn | n = 1, 2, ...., 100} is a
a) CFL but nor Regular
b) Regular
c) Non-regular
d) None of these
, S1, P1) and G2 = (V2,
, S2, P2) are CFG generating L1 and L2
10) If G1 = (V1,
respectively then grammar generating L1L2 has productions.
a) P1
P2
{S
S1| S2}
b) P1
P2
{S
S1S2}
P2
{S
S1S | ^}
d) P1
P2
c) P1
11) Which of the following string can be obtained by the language L = {aib2i/i > = 1} ?
a) aaabbbbbb
b) aabbb
c) abbabbba
d) aaaabbbabb


12) Which of the production is in CNF where a


, A, B, C are nonterminals ?
a) A
BC
b) A
B
c) A
^
d) A
aA


13) Which of the following language is accepted by deterministic PDA where w is string ?
b) {wwR where w {a, b, c}*}
a) {wcwR where w {a, b}*}
0}
d) {w | w is palindrome over {a, b, c}}
c) {anbncn, n
Which of the following is the initial stack symbol of PDA ?
b) S
c) Z 0
d) A
a) Q 0
A CFG is said to be ambiguous if it has
a) W L (G) which has at least two leftmost derivation trees
b) W L(G) which has at least two rightmost derivation trees
c) Either a) or b)
d) Both a) and b)
Which statement is true ?
a) The tape of turing machine is infinite
b) The tape of turing machine is finite
c) The tape of turing machine is infinite when the language is regular
d) The tape of turing machine is finite when the language is nonregular
In the following production which is nullable variable ?
S
ABCD
A
CD
B
Cb
C
a|^
D
bD | ^
a) S
b) A
c) B
d) None of these
In turing machine, for (q, X) = (r, Y, D), where D is
a) Left
b) Right
c) Stationary
d) Move to which square
If a language is accepted by PDA, it ______________ be accepted by TM.
a) can
b) cannot
c) may
d) may not


14)
15)

16)

17)

18)
19)

20) Turing machine accepts ____________ language.


a) Regular
b) Nonregular
c) CFL
______________

d) All

Set A

*SLRBB258*
S

SLR-BB 258

-3-

S.E. (Information Technology) (Part II) (New) Examination, 2014


THEORY OF COMPUTATION
Day and Date : Wednesday, 26-11-2014
Time : 3.00 p.m. to 6.00 p.m.

Marks : 80

Instructions : 1) Figures to the right indicate marks to a question.


2) Assume suitable data wherever necessary.
SECTION I
2. Attempt any four :
1) Consider two regular expressions
R = 0* + 1*

20

S = 01* + 10* + 1*0 + (0*1)*

1) Find a string corresponding to R but not S.


2) Find a string corresponding to S but not R.
3) Find a string corresponding to both.
2) Draw NFA-^ for the regular expression (0 + 1) (01)*(011)*.
3) Define Regular Grammar. Find Regular Grammar for the given automata.

4) Define context free grammar and context free language. Find CFG for the language over
alphabet {0, 1} where string is of even length.
5) If L1 and L2 are context free languages then L1
L2 is also context free language.
6) State the differences between FA, NFA and NFA-^.


3. Attempt any one :

10

1) Find minimum state automata for the given NFA-^.

2) Prove that the language accepted by any automata is regular.

Set A

SLR-BB 258

*SLRBB258*

-4-

4. Convert following grammar in Chomsky Normal Form.


AB | ABC

S


BA | BC | ^ | a

A
B
C

10

AC | CB | ^ | b


BC | AB | A | c
SECTION II

5. Attempt any four :

20

1) How the Turing machines are combined ? Why it is needed ?


2) Draw Turing which accepts the language of string containing substring 010.
3) Explain how to obtain CFG from PDA.
4) State the difference between Pushdown Automata and Turing machine.
5) Write a short note on Multitape Turing Machine.
6) State and explain pumping lemma for context free language.
6. Attempt any one :

10

1) Give the transition table for PDA recognizing the language {x




{a, b}* | na (x) > nb (x)}.

na(x) number of as in string x


nb (x) number of bs in string x
Show working of this PDA with example.
2) Prove that if L1 and L2 are CFLs then L1
CFL.


L2 is not a CFL and if L is CFL then L is not

7. Why Universal Turing machine is introduced ? Define the encoding function and explain with
example.

10

_____________________

Set A

SLR-BB 259

*SLRBB259*
Seat
No.

Set

S.E. (I.T.) (Part II) (New) Examination, 2014


MICROPROCESSORS
Day and Date : Thursday, 27-11-2014
Time : 3.00 p.m. to 6.00 p.m.

Max. Marks : 100

Instructions : 1) Q. No. 1 is compulsory. It should be solved in first 30


minutes in Answer book Page No. 3. Each question carries
one mark.
2) Answer MCQ/Objective type questions on Page No. 3 only.
Dont forget to mention, Q.P. Set (A/B/C/D) on Top of Page.
MCQ/Objective Type Questions
Duration : 30 Minutes

Marks : 20

1. Objective type questions :


1) Among these which one of the following is the one byte instruction ?
a) MVI A, 28H
b) LHLD 2500H c) STAX B
d) IN 29H
2) In the Compare instruction of 8085, if accumulator > data then
a) Carry flag is set
b) Zero and Carry flags are reset
c) Zero flag is set
d) Carry flag is reset
3) In the instruction DCR and INR, which of the following flag bit is not affected ?
a) Zero
b) Sign
c) Parity
d) Carry
4) What is the instruction XTHL mean ?
a) Exchange HL pair with DE pair
b) Exchange HL pair with BC pair
c) Exchange BC Pair with DE pair
d) Exchange H and L with op of Stack
5) Which of the following instructions requires highest number of T States for
execution ?
a) XTHL
b) CALL
c) STAX D
d) DAD H
6) Which Instruction will Shift D7 bit in D0 bit of accumulator as well as in CY
flag bit ?
a) RLC
b) RAL
c) RRC
d) RAR
7) RIM is used to check whether ______________
a) The write operation is done or not
b) The interrupt is masked or not
c) Read the status of interrupts and read serial data input bit
d) None of the above
P.T.O.

SLR-BB 259

-2-

*SLRBB259*

8) What is CP 2500H Instruction ?


a) Call on Positive S = 0
b) Call on Negative S = 0
c) Call on Minus S = 1
d) None of these
9) What is next Operation Perform by 8085 in response of INTR ?
a) Generates INTA Signal
b) Generates READY Signal
c) Generates HOLD Signal
d) Generates HLDA Signal
10) Which of the Instruction is used to selectively mask an interrupt ?
a) DI
b) EI
c) SIM
d) RIM
11) Which register is not available for user in microprocessor ?
a) W
b) Z
c) B
d) Both a) and b)
12) Which Subroutine is used for handling the interrupts in 8085 ?
a) CWR
b) ISR
c) CWR
d) None of these
13) The simplest form of Output Port is ______________
a) Latch
b) Microprocessor
c) Buffer
d) Keyboard
14) In Memory Mapped I/O the number of Input and Output Address will not be
256 instead whole 64K is _____________ by Input, Output, Memory.
a) Attached
b) Connected
c) Shared
d) Both b) and c)
15) Which of the following is DMA control signal ?
a) SID
b) HOLD
c) Reset
d) RD
16) In Master mode of 8257, DMA is ______________ and Microprocessor is
________________
a) Slave, Master b) Slave, Slave c) Master, Slave d) Master, Master
17) Which of the following is compulsory Initialization Command Word ?
a) ICW1 and ICW3
b) ICW2 and ICW3
c) OCW3
d) ICW1 and ICW2
18) Which is 16 bit microprocessor ?
a) 8088
b) 8086
c) 8085
d) All of these
19) Who was introduced the 80286 microprocessor updated on 8086, in 1983 ?
a) Intel
b) Motorola
c) Fairchild
d) None of these
20) What are features of 80386 Microprocessor ?
a) Flexible 32 Bit MP
b) 8 general purpose 32 bit register
c) Can address up to 64 terabytes of memory
d) All of these
______________
Set A

*SLRBB259*

-3-

SLR-BB 259

Seat
No.

S.E. (I.T.) (Part II) (New) Examination, 2014


MICROPROCESSORS
Day and Date : Thursday, 27-11-2014
Time : 3.00 p.m. to 6.00 p.m.

Marks : 80

SECTION I
2. Attempt any four :

(44=16)

a) Write the 8085 features and explain what is addressing mode.


b) Load the data byte ABH in register C. Mark the high order bits (D7 D4) and
display the low order (D3 D0) at an output port.
c) Describe the Operation of RESET OUT, SID, READY, and CLK OUT.
d) Explain Instruction fetching and execution Operation of Microprocessor.
e) Explain Arithmetic group of instruction with example.
3. Attempt any three :

(83=24)

a) Explain timing diagram of IN and OUT instruction in detail.


b) Write a program for Block transfer 10 bytes of data from Memory location
8000H > 8009H to 9000H > 9009H with Overlapping, Without Overlapping
and also reverse back.
c) Explain branch instruction with example.
d) Compare following :
a) RAL and RRC
b) XCHG and XTHL
c) LDA and LDAX Rp
d) PUSH Rp and POP Rp.
Set A

SLR-BB 259

-4-

*SLRBB259*

SECTION II
4. Attempt any four :

(44=16)

a) What is DMA ? Explain with help of Diagram.


b) What is BSR mode ? Write a program to set D5 bit of port C.
c) Explain difference between RIM and SIM.
d) What is OCW1 of 8259 PIC ?
e) Explain the segmentation of memory used with 8086.
5. Attempt any three :

(83=24)

a) Explain interrupt structure of 8085 with neat diagram.


b) Draw and explain interfacting of Thumb wheel switch.
c) Explain the features and architecture of 80286 in detail.
d) Explain feature and Block diagram for 8251.

_____________________

Set A

SLR-BB 260

*SLRBB260*
Seat
No.

Set

S.E. (Information Technology) (Part II) Examination, 2014


DATA STRUCTURES (New)
Day and Date : Friday, 28-11-2014
Time : 3.00 p.m. to 6.00 p.m.
N.B. :

Max. Marks : 100

1) Q. No. 1 is compulsory. It should be solved in first 30 minutes in Answer


Book Page No. 3. Each question carries one mark.
2) Answer MCQ/Objective type questions on Page No. 3 only. Dont forget to
mention, Q.P. Set (A/B/C/D) on Top of Page.
MCQ/Objective Type Questions

Duration : 30 Minutes

Marks : 20

1. Answer all questions : Choose the correct option :

20

1) Which of the following is the correct way of declaring a float pointer ?


a) float ptr;
b) float*ptr;
c) *float ptr;
2) Which of the following is the feature of stack ?
a) All operations are at one end
c) All elements are of different data types

d) none of the above

b) It cannot reuse its memory


d) Any element can be accessed from it directly

3) A link field in a structure in C is a place holder for


a) Memory Address
b) Old value of variable
c) A static component
d) None of above
4) The number of element that it first moved from stack is from the
a) Bottom of the stack
b) Top of stack
c) Middle of stack
d) None of these
5) Which of the following stack operation could result in stack underflow ?
a) PUSH
b) POP
c) PEEP

d) None of these

6) What is the output of following code ?


void main ()
{
int a=10, b=20;
char x=1, y=0;
if (a, b, x, y)
{
printf(EXAM);
}
}
a) XAM is printed
b) Exam is printed

d) Nothing is printed

c) Compiler error

7) C programs are converted into machine language with the help of


a) interpreter
b) compiler
c) operating system

d) microprocessor

8) Inserting a node in a doubly linked list after a given node requires


a) one pointer change
b) four pointer change
c) two pointer change
d) none
9) Another name for prefix notation is
a) Reverse Polish
b) Polish

c) Infix

d) Inverse Polish
P.T.O.

SLR-BB 260
10) What is result of following program ?
main ()
{
main ();
}
a) Runtime Error: Stack Overflow
c) No Error

*SLRBB260*

-2-

b) Output will be blank


d) Linker Error

11) Which data structure used in traversing a given graph by depth first search
a) Stack
b) Set
c) List
d) Queue
12) The efficiency of Breadth First Traversal for adjacency list graph representation is
___________ (e-edges n-vertices)
a) O(n+e)
b) O(e)
c) O(n2)
d) O(nlogn)
13) For an AVL tree the balance factor is calculated as _____________
a) BF = (Height of right subtree Height of left subtree)
b) BF = (Height of left subtree Height of right subtree)
c) BF = Height of left subtree
d) BF = Height of tree
14) Number of nodes in a complete binary tree of depth k is
b) 2k 1
c) 2k
a) 2k

d) none of above

15) Maximum possible height of an AVL tree with 7 nodes is


a) 3
b) 5
c) 4

d) none of the above

16) In an multiway tree each node can have more than __________ keys.
a) one
b) two
c) three

d) all the above

17) Suppose you have a directed graph representing all the heights that an airline flies. What algorithm
might be used to find the best sequence of connection from one city to another ?
a) DFS
b) BFS
c) Shortest path algorithm
d) Cycle-finding algorithm
18) When degree of a node is zero, then the node is called ___________
a) isolated node
b) distinct node
c) group node

d) single node

19) Which is adjacency matrix representation for given below graph ?

a)

b)

c)

d)

20) Which of the following statement is true in view of multiway search tree ? If a node has
a) 4 subtrees, it contains 3 keys
b) 5 keys, it has 7 subtrees
c) 6 subtrees, it contains 6 keys
d) none of above
______________

Set A

*SLRBB260*

-3-

SLR-BB 260

Seat
No.
S.E. (Information Technology) (Part II) Examination, 2014
DATA STRUCTURES (New)
Day and Date : Friday, 28-11-2014
Time : 3.00 p.m. to 6.00 p.m.

Marks : 80

N.B. : All questions are compulsory.


SECTION I
2. Answer any four questions of the following :
1) Define Linked List. Explain any one application of Linked List.
2) Explain the concept of inserting a node at particular position in singly linked list.
3) Explain the traversing technique in doubly linked list.
4) Define Tree and explain the types of binary tree.
5) Write algorithm for converting following Expression to postfix with help of stack
representation
((A + (B C) * D) ^ E + F).
3. Answer any two questions of the following :

(45=20)

(210=20)

1) Write a note linked list and types and operations performed on linked list.
2) Write an algorithm to convert to an infix expression into prefix. Explain the method with
suitable example.
3) Write a program to implement two queues in one array where first queue will start from
0th position and second queue will start from last position of array.

SECTION II
4. Answer any four questions of the following :

(45=20)

1) What are different representations of graph ? Write both representation for the following
graph ?

Set A

SLR-BB 260

-4-

*SLRBB260*

2) Construct an AVL search tree by inserting following elements in order of their occurrence
11, 12, 13, 14, 15, 16
Write number of total rotations required.
3) Explain difference between B and B+ tree with an example.
4) Write an algorithm for depth first search traversal of graph.
5) Explain how AVL tree is balanced using single rotation and double rotation examples.
6) Define the following terms :
i) Graph
ii) Directed graph
iii) Weighted graph
iv) Disjoint graph
v) Connected graph.
5. Answer any two questions of the following :

(210=20)

1) Write pseudo code of Dijkstras algorithm for finding shortest path. Explain algorithm
with example showing each step with corresponding graph ?
2) What is an AVL tree ? Explain Insertion and Deletion of node in AVL tree in detail with
example.
3) Construct a B-tree of order 3 by inserting following keys in the order
M, Q, A, N, P, W, X, T, G, E, J
_____________________

Set A

SLR-BB 261

*SLRBB261*
Seat
No.

Set

S.E. (IT) (Part II) (New) Examination, 2014


DATA COMMUNICATION
Day and Date : Saturday, 29-11-2014
Time : 3.00 p.m. to 6.00 p.m.

Max. Marks : 100

Instructions : 1) Q. No. 1 is compulsory. It should be solved in first 30 minutes


in Answer Book Page No. 3. Each question carries one mark.
2) Answer MCQ/Objective type questions on Page No. 3 only.
Dont forget to mention, Q.P. Set (A/B/C/D) on Top of Page.
MCQ/Objective Type Questions
Duration : 30 Minutes

Marks : 20

1. Write the correct answer from the options given below :

20

1) ___________ layer translates, encrypt the data between communicating


systems ?
a) Transport
b) Presentation
c) Application
d) Physical
2) Protocols are
a) Agreements on how communication components and DTEs are to
communicate
b) Logical communication channels used for transferring data
c) Physical communication channels used for transferring data
d) None of the above
3) The method of communication in which transmission takes place in both directions,
but to in one direction at a time is called
a) Simplex
b) Four wire circuit
c) Full duplex
d) Half duplex
4) Error detection at the data link level is achieved by
a) Bit stuffing
b) Cyclic redundancy codes
c) Hamming codes
d) Equalization
5) Start and stop bits are used in serial communication for
a) Error detection
b) Error correction
c) Synchronization
d) Slowing down the communication
6) Manchester code is a
a) Bi-polar code
c) Polar code

b) None return to zero code


d) None of the above

7) _________ layer uses physical addresses to route the data to destination hosts.
a) Network layer
b) Data link layer
c) Transport layer
d) All of above
P.T.O.

SLR-BB 261

*SLRBB261*

-2-

8) Your company has a LAN in its downtown office and has now set up a LAN in the
manufacturing plant in border. To enable everyone to share data and resources
between the two LANs, what type of device(s) are needed to connect them ? Choose
the most correct answer.
a) Modem
b) Cable
c) Hub
d) Router
9) In CRC there is no error if the remainder at the receiver is _________
a) Equal to the remainder at the sender
b) Zero
c) Nonzero
d) The quotient at the sender
10) Which of the following summation operation is performed on the bits the check an
error-detecting code ?
a) Codec
b) Coder-decoder
c) Checksum d) Attenuation
11) IP addresses 10000001 00001011 00001011 11101111 from binary notation to
dotted notation will be
a) 81.11.11.255
b) 129.131.27.255
c) 81.11.11.241 d) 129.131.27.241
12) 134.12.14.87 is in _________
a) Class A
b) Class B

c) Class C

d) Class D

13) Class _________ is reserved for future purpose.


a) Class A
b) Class B
c) Class E

d) Class D

14) _________ routing is/are dynamic algorithm.


a) Distance vector b) Link state
c) Both above

d) None

15) In distance vector routing distance to each of the neighbour is _________


a) Estimate of time
b) Distance to destination
c) Hops
d) None
16) Congestion occurs due to _________
a) Slow processors
b) No free buffer

c) Both

d) None

17) Two or more network forms _________


a) LAN
b) Internet
c) Intranet

d) None

18) _________ are store and forward device in data link layer.
a) Bridge
b) Repeaters
c) Router
d) None
19) IPV6Address is
a) 8 bit

b) 16 bit

c) 32 bit

d) 128 bit

20) ATM (Asynchronous Transfer Mode) is fundamentally a _________ technology.


a) Circuit switching
b) Packet switching

c) Narrow band

d) None of these
______________

Set A

*SLRBB261*

-3-

SLR-BB 261

Seat
No.

S.E. (IT) (Part II) (New) Examination, 2014


DATA COMMUNICATION
Day and Date : Saturday, 29-11-2014
Time : 3.00 p.m. to 6.00 p.m.

Marks : 80
SECTION I

2. Answer any four of the following :


a) Write a short note on Transmission Impairments.
b) Differentiate between OSI and TCP model.
c) Explain analog and digital data transmission.
d) Describe optical fibres with the help of its physical description, transmission
characteristics and application.
e) Explain use of computer network in your college.

(45)

3. Answer any one of the following :


a) Explain a protocol using go back n.
b) Explain a protocol one bit sliding protocol.

10

4. Answer the following :


a) What is difference in CRC and Hamming ? Elaborate with example.

10

SECTION II
5. Answer any four of the following :
a) Explain dynamic channel allocation in LAN and MAN.
b) What are advantage of slotted aloha over pure aloha.
c) Discuss load shedding in congestion control.
d) Write down detail about 802.3 IEEE standard.
e) Explain classless protocol.
6. Attempt any one of the following :

(45)

10

a) With example explain distance vector routing algorithm.


b) What is dynamic routing algorithm ? Explain link state routing with example.
7. Answer the following :
a) Explain collision free protocol with both method.
_____________________

10

Set A

Set A

SLR-BB 262

*SLRBB262*
Seat
No.

Set

T.E. (Information Technology) (Old) (Part I) Examination, 2014


COMPUTER NETWORK II
Day and Date : Wednesday, 3-12-2014
Time : 3.00 p.m. to 6.00 p.m.

Max. Marks : 100

N.B. : 1) All questions are compulsory.


2) Q. No. 1 is compulsory. It should be solved in first 30 minutes
in Answer Book Page No. 3. Each question carries one mark.
3) Answer MCQ/Objective type questions on Page No. 3 only.
Dont forget to mention, Q.P. Set (A/B/C/D) on Top of Page.
MCQ/Objective Type Questions
Duration : 30 Minutes

Marks : 20

1. Choose the correct answer :


1) The ports ranging from 1,024 to 49, 151 are called __________ ports.
A) Well-known

B) Registered

C) Dynamic

D) None of these

2) Which of the following functions does UDP perform ?


A) Process-to-process communication

B) Host-to-host communication

C) End-to-end reliable data delivery

D) None of the choices are correct

3) When the IP layer of a receiving host receives a datagram, __________


A) Delivery is complete

B) A transport layer protocol takes over

C) A header is added

D) None of the choices are correct

4) UDP and TCP are both __________ layer protocols.


A) Network

B) Data link layer

C) Transport

D) None of the choices are correct

5) In TCP, the size of the send window is the __________ of rwnd and cwnd.
A) Maximum

B) Sum of

C) Minimum

D) None of the choices are correct

6) Which of the following does UDP guarantee ?


A) Flow control

B) Connection-oriented delivery

C) Error control

D) None

7) In the __________ algorithm the size of the congestion window increases


exponentially until it reaches a threshold.
A) Congestion avoidance

B) Congestion detection

C) Slow start

D) None of the choices are correct


P.T.O.

SLR-BB 262

*SLRBB262*

-2-

8) A special segment called a probe is sent by a sending TCP when the


__________ timer goes off.
A) Transmission

B) Persistence

C) Keep alive

D) None of these

9) Information in a computer is stored in __________ byte order.


A) Host

B) Network

C) Server

D) None of these

10) An interface is a set of __________ designed to facilitate interaction between


two entities.
A) Programs

B) Instructions

C) Rules

D) None of these

11) __________ transfers email across the internet, LAN or WAN.


A) MAA

B) MTA

C) UA

D) None

12) In a static web document contents are fixed and stored


A) At client side

B) In a server

C) Both

D) None

13) In control communication of NVT, the file type, data structure, transmission mode
are defined by
A) Server

B) Client

C) Both

D) None

14) Inverse domain finds a


A) Domain name for given IP

B) IP for given domain

C) Both

D) None

15) The DNS is divided into __________ sections.


A) 3

B) 2

C) 5

D) 4

16) MIME allows __________ data to be sent through SMTP.


A) Image

B) Text

C) Audio

D) All of these

17) TFTP is an application layer protocol which uses __________ for its transport
mechanism ?
A) TCP

B) UDP

C) Both

D) None

18) TELNET uses an interface called universal __________ character set.


A) Network Virtual Terminal

B) Network Virtual Tool

C) Network Verified Tool

D) None of these

19) A __________ is used as an intermediately between DHCP client and server.


A) Relay agent

B) Broadcast agent

C) Discover agent

D) None of these

20) To distinguish data from control characters, each sequence of control characters
is preceded by a special control character called __________
A) ICA

B) IAC

C) AIC

______________

D) None of these

Set A

*SLRBB262*

-3-

SLR-BB 262

Seat
No.

T.E. (Information Technology) (Old) (Part I) Examination, 2014


COMPUTER NETWORK II
Day and Date : Wednesday, 3-12-2014
Time : 3.00 p.m. to 6.00 p.m.

Marks : 80
SECTION I

1. Answer any four from the following questions each carries 5 marks :
a) Explain what is DHCP and DHCP packet format and operations of DHCP ?
b) Explain the concept of connection oriented concurrent server.
c) Explain three-way handshaking in TCP connection.
d) Describe DNS Query and DNS response message in detail.
e) Describe following system calls
a) Bind
b)
Accept
c) Listen
d) Connect
2. Answer any one from the following questions carries 10 marks :
a) What is DHCP ? What is need of it ? Explain concept of DHCP with transition
diagram.
b) Draw TCP segment format. Explain each field in detail.
3. Answer the following questions carries 10 marks :
What is congestion window ? Explain all congestion control policies used by TCP.
SECTION II
1. Answer any four from the following questions each carries 5 marks :
a) What is MIME ? Explain its application in detail ?
b) Explain terms GET and POST in HTTP.
c) Describe NVT character set.
d) Explain the use of cookies.
e) Describe overview of Windows NT.
2. Answer any two from the following each questions carries 10 marks :
a) Explain architecture of electronic mail with 4 scenarios.

b) Discuss the architecture of WWW and describe concept of HTTP.


c) Define the list and fields in request and response message.
_____________________

Set A

Set A

SLR-BB 263

*SLRBB263*
Seat
No.

Set

T.E. (I.T.) (Part I) Examination, 2014


COMPUTER ORGANIZATION (Old)
Day and Date : Friday, 5-12-2014
Time : 3.00 p.m. to 6.00 p.m.

Max. Marks : 100

Instructions : 1) Q. No. 1 is compulsory. It should be solved in first 30 minutes


in Answer Book Page No. 3. Each question carries one mark.
2) Answer MCQ/Objective type questions on Page No. 3 only.
Dont forget to mention, Q.P. Set (A/B/C/D) on Top of Page.
MCQ/Objective Type Questions
Duration : 30 Minutes
1. Objective questions. Each question carry one mark.

Marks : 20
(120=20)

1) Assembly language came in to use in


a) First generation
b) Third generation
c) Second generation d)Fourth generation
2) Processors of all computers, whether main frame, micro, mini must have
a) ALU
b) Primary storage
c) Control unit
d) All of above
3) The drawback of building a large memory with DRAM is
a) The large cost factor
b) The inefficient memory organization
c) The slow speed of operation
d) All of the above
4) The next level of memory hierarchy after the L2 cache is _____________
a) Secondary storage
b) TLB
c) Main memory
d) Register
5) The reason for the implementation of the cache memory is
a) To increase the internal memory of the system
b) The difference in speeds of operation of the processor and memory
c) To reduce the memory access and cycle time
d) All of the above
6) A stack computer evaluates arithmetic and other expressions using a format known
as
a) Stack notation
b) Polish notation
c) Either a) or b)
d) None of these
7) _____________ register keeps the track of instructions stored in memory.
a) AR (Address Register)
b) IR (Index Register)
c) PC (Program Counter)
d) AC (Accumulator)
P.T.O.

SLR-BB 263

*SLRBB263*

-2-

8) Process of converting logical address in to linear address known as


a) Page translation
b) Segment translation
c) Two address translation
d) None of these
9) In a virtual memory system the address space specified by the address lines of
the CPU must be _____________than the physical memory and _________then
the secondary storage size.
a) Smaller, smaller
b) Larger, smaller
c) Smaller, Larger
d) Larger, Larger
10) Property of locality reference may fail if a program has
a) Many conditional jumps
b) Many unconditional jumps
c) Many operands
d) Many operators
11) How many RAM chips of size (256 K 1 bit) are required to build 1MB memory ?
a) 8
b) 12
c) 24
d) 32
12) Which of the following is true ?
a) ROM is read/write memory
b) PC points to the last instruction that was executed
c) Stack works on the principle of LIFO
d) All of the above
13) Desirable characteristics of a memory system is (are)
a) Speed and reliability
b) Low power consumption
c) Durability and compactness
d) All of these
14) The situation where in the data of operands are not available is called
_____________
a) Data hazard
b) Stack
c) Dead lock
d) Structural hazard
15) Pipe lining is a unique feature of ___________
a) RISC
b) CISC
c) ISA

d) IANA

16) The periods of time when the unit is idle is called as ___________
a) Stalls
b) Bubbles
c) Hazards
d) Both a) and b)
17) Vector computers appears as
a) SISD
b) SIMD

c) MIMD

d) MISD

18) Manipulation of one or two data items at a time is through


a) Scalar processor
b) Vector processor
c) Both a) and b)
d) None of these
19) An effective to introduce parallelism in memory access is by___________
a) Memory interleaving
b) TLB
c) Pages
d) Frames
20) During write operation if the required block is not present in the cache
then___________ occurs.
a) Write latency
b) Write hit
c) Write delay
d) Write miss

______________

Set A

*SLRBB263*

-3-

SLR-BB 263

Seat
No.

T.E. (I.T.) (Part I) Examination, 2014


COMPUTER ORGANIZATION (Old)
Day and Date : Friday, 5-12-2014
Time : 3.00 p.m. to 6.00 p.m.

Marks : 80

Instruction : All questions are compulsory.


SECTION I
2. Attempt any 4 :

(45=20)

a) Compare RISC and CISC.


b) Describe the concept of memory hierarchy.
c) Briefly explain design methods of hard wired control unit.
d) List and explain different instruction types.
e) Explain 4 bit carry look ahead adder.
3. Attempt any 2 :

(210=20)

a) Design a multiplier control unit using sequence counter method.


b) Explain Booths algorithm with example.
c) Explain First-Fit and Best-Fit memory allocations with diagrams.

Set A

SLR-BB 263

-4-

*SLRBB263*

SECTION II
4. Attempts any 4 :

(45=20)

a) Write a note on uniprocessors.


b) Explain on brief concept of interleaved memories.
c) What is need of replacement algorithm ? Give the algorithm for LRU replacement.
d) Explain linear pipeline with neat diagrams.
e) Write a note on loosely coupled system.
5. Attempts any 2 :

(210=20)

a) Explain different pipeline hazards in detail with proper examples.


b) Explain the structure of associative memory. Give the logic diagram of 1 bit
associative memory cell.
c) What are the different Flynns classifications of computers ? Explain with diagram
and give example of each.

_____________________

Set A

SLR-BB 264

*SLRBB264*
Seat
No.

Set
T.E. (Information Technology) (Part I) Examination, 2014
SYSTEM PROGRAMMING (Old)

Day and Date : Monday, 8-12-2014


Time : 3.00 p.m. to 6.00 p.m.

Max. Marks : 100

Instructions : 1) Figures to the right indicate full marks.


2) Q. No. 1 is compulsory. It should be solved in first 30 minutes in
Answer Book Page No. 3. Each question carries one mark.
3) Answer MCQ/Objective type questions on Page No. 3 only.
Dont forget to mention, Q.P. Set (A/B/C/D) on Top of Page.
MCQ/Objective Type Questions
Duration : 30 Minutes

Marks : 20

1. Choose the correct alternative :


1) Symbolic references in source statement are not processed at all during Pass I in

a) Variant I

b) Variant II

c) Variant III

d) All of these

c) END

d) Both a) and b)

2) Which directive sets the LC ?

a) START

b) ORIGIN

3) Macro prototype statement declares

a) Name of macro
c) Both a) and b)

b) Name and Parameters


d) None

4) An ideal compiler should

a) Detect error
c) Detect, repair and correct error

b) Detect and report error


d) None

5) An optimizing compiler

a)
b)
c)
d)

None of these
Is optimized to occupy less space
Is optimized to take less time for execution
Optimizes the code

6) A basic block can be analyzed by

a) DAG
c) Graph having cycles

b) Flow graph
d) None

7) If translated and linked origin are different then relocation is performed by

a) Linker

b) Loader

c) Translator

d) All

8) Relocation bits are used in

a) Absolute loader
c) Direct linking loader

b) Relocating loader
d) Dynamic loader
P.T.O.

SLR-BB 264

*SLRBB264*

-2-

9) File with .EXE extension contains

a) Non-relocated object program


c) Linked program

b) Relocated program
d) None

10) Overlays are used to

a) Reduce execution time


c) Reduce memory requirement

b) Reduce code
d) Reduce linking requirement

11) The most general phrase structured grammar is

a) Regular grammar
c) Context-sensitive

b) CFG
d) None

12) Whether a given pattern constitutes of token or not depends on

a) Source language
c) Compiler

b) Target language
d) All of these

13) Symbol table can be used for

a)
b)
c)
d)

Checking type compatibility


Suppressing duplication of error messages
Storage allocation
All of these

14) Compiler can diagnose

a) Grammatical errors
c) Both a) and b)

b) Logical errors
d) None

15) Pseudo-code is a

a) Machine language
c) High-level language

b) Assembly language
d) None

16) Resolution of an externally defined symbol is performed by

a) Linker

b) Loader

c) Compiler

d) Assembler

c) Programmer

d) Assembler

17) In absolute loader linking is done by

a) Linker

b) Loader

18) In direct linking loader which among the following is not related to object program

a) ESD

b) RLD

c) BSS

d) TXT

19) EXE2BIN creates

a) System file

b) Executable file c) Object file

d) None

20) Binary symbolic subroutine loader is an example of

a) Absolute loader
c) Relocating loader

b) Compile-go loader
d) Direct linking loader
______________

Set A

*SLRBB264*

-3-

SLR-BB 264

Seat
No.

T.E. (Information Technology) (Part I) Examination, 2014


SYSTEM PROGRAMMING (Old)
Day and Date : Monday, 8-12-2014

Marks : 80

Time : 3.00 p.m. to 6.00 p.m.


Instructions : 1) All questions are compulsory.
2) Figures to the right indicate full marks.
SECTION I
2. Attempt any four the following briefly :

(45=20)

a) Give the structure of a LEX specification file.


b) What are imperative statements ? Give examples.
c) Draw the architecture of a single pass assembler.
d) What are forward references ? How are they dealt with ?
e) What is a macro ? What are its requirements ?
3. Develop a simple assembly scheme for understanding working of an assembler.

10

OR
With neat flowcharts explain the analysis and synthesis phases of two-pass assembler.
4. Explain any four of the following terms with suitable illustrations.

10

i) Semantic gap
ii) Specification Gap
iii) Execution Gap
iv) Programming Language domain
v) Application domain
vi) Execution domain.
Set A

SLR-BB 264

-4-

*SLRBB264*

SECTION II
5. Answer any four of the following briefly :

(45=20)

a) What is strength reduction ? Give examples.


b) Define relocation factor. How is it used ?
c) What are the basic steps in program execution ?
d) What are overlays ? How are they handled ?
e) What is local optimization ?
6. State the different code optimization techniques. Explain for local and Global code
scenarios.

10

OR
What are the benefits achieved by code optimization ? Explain the scenarios at compile
and runtime.
7. State and illustrate any four with diagrams the following loader schemes :

10

i) Compile and Go
ii) General
iii) Absolute
iv) Relocating
v) Direct linking
vi) Dynamic.

_____________________

Set A

SLR-BB 265

*SLRBB265*
Seat
No.

Set

T.E. (I.T.) (Part I) Examination, 2014


OPERATING SYSTEM I (Old)
Day and Date : Wednesday, 10-12-2014
Time : 3.00 p.m. to 6.00 p.m.

Max. Marks : 100

Instructions : 1)
2)
3)
4)

All questions are compulsory.


Figures to the right indicate full marks.
Assume suitable data if necessary.
Q. No. 1 is compulsory. It should be solved in first 30 minutes in
Answer book Page No. 3. Each question carries one mark.
5) Answer MCQ/Objective type questions on Page No. 3 only. Dont
forget to mention, Q.P. Set (A/B/C/D) on Top of Page.

Duration : 30 Minutes

MCQ/Objective Type Questions

1. Multiple choice questions :

Marks : 20
(201=20)

1) What is the ready state of a process ?


a) when process is scheduled to run after some execution
b) when process is unable to run until some task has been completed
c) when process is using the CPU
d) none of the mentioned
2) A single thread of control allows the process to perform
a) only one task at a time
b) multiple tasks at a time
c) all of these
d) none of these
3) Process synchronization can be done on
a) hardware level
b) software level
c) both a) and b)
d) none of the mentioned
4) Cascading termination refers to termination of all child processes before the parent
terminates ______________
a) Normally
b) Abnormally
c) Normally or abnormally
d) None of these
5) Message passing system allows processes to
a) communicate with one another without resorting to shared data
b) communicate with one another by resorting to shared data
c) share data
d) name the recipient or sender of the message
6) The interval from the time of submission of a process to the time of completion is termed as
a) waiting time
b) turnaround time
c) response time
d) throughput
7) Scheduling is
a) allowing a job to use the processor
b) making proper use of processor
c) both a) and b)
d) none of these
8) Which of the following statements are true ?
i) Shortest remaining time first scheduling may cause starvation.
ii) Preemptive scheduling may cause starvation.
iii) Round robin is better than FCFS in terns of response time.
a) i) only
b) i) and iii) only
c) ii) and iii) only
d) i), ii) and iii)
P.T.O.

SLR-BB 265

*SLRBB265*

-2-

9) The segment of code in which the process may change common variables, update
tables, write into files is known as
a) program
b) critical section
c) non-critical section
d) synchronizing
10) Bounded waiting implies that there exists a bound on the number of times a process is
allowed to enter its critical section
a) after a process has made a request to enter its critical section and before the request
is granted
b) when another process is in its critical section
c) before a process has made a request to enter its critical section
d) none of these
11) For effective operating system, when to check for deadlock ?
a) every time a resource request is made b) at fixed time intervals
c) both a) and b)
d) none of the mentioned
12) The solution to starvation is
a) the number of rollbacks must be included in the cost factor
b) the number of resources must be included in resource preemption
c) resource preemption be done instead
d) all of these
13) Dynamic loading is
a) loading multiple routines dynamically
c) loading multiple routines randomly

b) loading a routine only when it is called


d) none of these

14) The run time mapping from virtual to physical addresses is done by a hardware device
called the
a) Virtual to physical mapper
b) Memory management unit
c) Memory mapping unit
d) None of these
15) The major part of swap time is ______________ time.
a) waiting
b) transfer
c) execution

d) none of these

16) The ______________ table contains the base address of each page in physical memory.
a) process
b) memory
c) page
d) frame
17) An ______________ uniquely identifies processes and is used to provide address space
protection for that process.
a) address space locator
b) address space identifier
c) address process identifier
d) none of these
18) Memory protection in a paged environment is accomplished by
a) protection algorithm with each page
b) restricted access rights to users
c) restriction on page visibility
d) protection bit with each page
19) The CPU hardware has a wire called ______________ that the CPU senses after executing
every instruction.
a) interrupt request line b) interrupt bus c) interrupt receive line d) interrupt sense line
20) The ______________ are reserved for events such as unrecoverable memory errors.
a) non-maskable interrupts
b) blocked interrupts
c) maskable interrupts
d) none of these
______________

Set A

*SLRBB265*

-3-

SLR-BB 265

Seat
No.

T.E. (I.T.) (Part I) Examination, 2014


OPERATING SYSTEM I (Old)
Day and Date : Wednesday, 10-12-2014

Marks : 80

Time : 3.00 p.m. to 6.00 p.m.


Instructions :

1) All questions are compulsory.


2) Figures to the right indicate full marks.
3) Assume suitable data if necessary.
SECTION I

2. Attempt any four :

(45=20)

a) Explain with figure computer system architecture.


b) Explain all the operations on process.
c) Explain multilevel queue scheduling algorithm.
d) Explain semaphore with its usage and implementation.
e) Explain interprocess communication.
3. Explain the critical section problem with two process solution algorithm.

10

OR
Explain with example priority scheduling and round robin scheduling algorithm.
4. Write short note (any two) :

(25=10)

1) Schedulers
2) Client server computing
3) Readers and writers problem.

Set A

SLR-BB 265

-4-

*SLRBB265*

SECTION II
5. Attempt any four :

(45=20)

a) Explain with example the Bankers algorithm.


b) What is paging ? Explain with figure the paging model of logical and physical memory.
c) Explain in detail the segmentation.
d) Explain with figure the concept of demand paging.
e) Explain with figure the I/O hardware.
6. Explain in detail Kernel I/O sub-system.

10

OR
Explain all page replacement algorithms with example.
7. Solve any two :

(25=10)

1) Structure of a page table


2) Deadlock avoidance
3) Thrashing with working set model.
_________________

Set A

SLR-BB 266

*SLRBB266*
Seat
No.

Set

T.E. (Information Technology) (Part I) (Old) Examination, 2014


DESIGN AND ANALYSIS OF ALGORITHMS
Day and Date : Friday, 12-12-2014
Time : 3.00 p.m. to 6.00 p.m.

Total Marks : 100

Instructions : 1) Figures to the right indicate marks to a question.


2) Q. No. 1 is compulsory. It should be solved in first 30 minutes
in Answer Book Page No. 3. Each question carries one mark.
3) Answer MCQ/Objective type questions on Page No. 3 only.
Dont forget to mention, Q.P. Set (A/B/C/D) on Top of Page.
MCQ/Objective Type Questions
Duration : 30 Minutes

Marks : 20

1. Choose the correct alternative(s) :

20

1) Which of the following statement is true ?


b) 10n2 + 9 = O(n)
a) n! = O(nn)
c) n2 + logn = (n2)
d) n2/log n = (n2)
2) O(1) means computing time is ____________
a) Linear
b) Constant
c) Quadratic

d) Exponential

3) Average number of comparisons in quick sort is ____________


a) O(n2)
b) O(nlogn)
c) O(n)
d) O(logn)
4) Algorithm space1(a,b,c,d)
{
return a+b/c+d*(a+b+a)
}
Space complexity of above algorithm is
a) n words
b) 4 words
c) 1 word
d) None
5) In optimal storage on tape problem, if (I1, I2, I3) = (23, 17, 34) then optimal
MRT is obtained by the ordering ____________
a) 1, 2, 3
b) 3, 2, 1
c) 2, 1, 3
d) 1, 3, 2
6) Dijkstras algorithm uses ____________ method.
a) Divide and conquer
b) Greedy method
c) Dynamic programming
d) Backtracking
7) Recurrence relation of MinMax algorithm using divide and conquer method is
____________
a) T(n/2) + 2
b) T(n/2) + 4
c) 2T(n/2) + 2
d) 2T(n/2) + 4
P.T.O.

SLR-BB 266

-2-

*SLRBB266*

8) In Kruskals algorithm, costs of edges of a graph are stored in ____________


a) Linked list
b) Array
c) Heap
d) Stack
9) Average number of comparisons required in binary search is ____________
a) O(n2)
b) O(nlogn)
c) O(n)
d) O(logn)
10) In job sequencing with deadlines problem, if n = 3, (p1, p2, p3) = 100, 10, 15)
and (d1, d2, d3) = (1, 1, 1), then how many jobs will get completed ?
a) 1
b) 2
c) 3
d) no job
11) Dynamic programming is based on principle of
a) Optimality
b) Heruquisics c) Regularity
d) None of these
12) The multistage graph can be solved by using
a) Forward approach
b) Backward approach
c) Forward and backward approach d) None
13) If a1, a2, a3 = {do, if, while} is an identifier set then how many combination
will exists ?
a) 3
b) 9
c) 6
d) 5
14) In flow shop scheduling OFT stands for
a) Optimal Find Time
b) Optimal Flow Time
c) Optimal Finish Time
d) None of these
15) _____________ is a generated node which is to be expanded further or all of
whose children have been generated.
a) Live node
b) E-node
c) Dead node
d) Root node
16) In DFS of graph which of the following data structure is used ?
a) Stack
b) Linked list
c) Queue
d) None
17) Every biconnected component of a connected graph G contains at least
________ vertices.
a) One
b) Three
c) Two
d) Four
18) Solid edges of depth first spanning tree are called
a) Back edges
b) Tree edges
c) Broken edges d) None
19) The smallest number of colors needed to color a graph G is called its
___________
a) Face number
b) Chromatic number
c) Edges number
d) Vertex number
20) Total time required for m-coloring algorithm is ____________ where m=no.
of colours, n=no. of nodes.
a) O(nm)
b) O(nmn)
c) O(m.nn)
d) O(mn)
______________

Set A

*SLRBB266*

-3-

SLR-BB 266

Seat
No.

T.E. (Information Technology) (Part I) (Old) Examination, 2014


DESIGN AND ANALYSIS OF ALGORITHMS
Day and Date : Friday, 12-12-2014
Time : 3.00 p.m. to 6.00 p.m.

Marks : 80

Instructions : 1) All questions are compulsory.


2) Figures to the right indicate marks to a question.
SECTION I
2. Attempt any four :
20
1) Find time and space complexity of the following algorithm.
Algorithm abc
{
i:=1;
while (i n) do
{
x:=x+1;
i=i+1;
}
}
2) Write a short note on Greedy method.
3) Search the elements 85, 48, 50 from the following array using binary search.
45, 50, 55, 60, 65, 70, 80, 85, 90.
4) Find the feasible solutions for job sequencing with deadlines,
n = 4, (p1, p2, p3, p4) = (100, 25, 15, 17) and deadlines (d1, d2, d3, d4) = (2, 1, 2, 1).
5) Give the criteria that should be satisfied by all algorithms.
6) Solve the recurrence relation using substitution method
T(n) = 2T(n/2) + cn.
3. Attempt any one :
1) Find the shortest path from vertex 1.

2) Prove that average time for Quick sort is O(nlogn).

10

Set A

SLR-BB 266

-4-

4. Sort the following array using merge sort.


Draw recursive calls for merge sort and merge.
3, 4, 1, 5, 9, 2, 6, 5, 3, 9.

*SLRBB266*
10

SECTION II
5. Attempt any four :
(54=20)
a) Explain multistage graphs with example.
b) Explain flow shop scheduling with example.
c) Compare between Breadth First Search and Depth First Search.
d) Explain Bi-connected components with example.
e) State Graph coloring problem. Define and explain following terms with suitable
example.
i) Chromatic number.
ii) Planar Graph.
f) Explain Hamilton Cycle with example.
6. Attempt the following :
10
a) State All pairs shortest path problem find all pairs shortest path using dynamic
programming for following diagram :

b) Explain backtracking solution for knapsack problem.

10

OR
c) Write a short note on :
a) Sum of subsets
b) Hamilton cycle.
_____________________

Set A

SLR-BB 267

*SLRBB267*
S

T.E. (IT) (New) (Part I) Examination, 2014


NETWORK MANAGEMENT
Day and Date : Wednesday, 3-12-2014
Time : 3.00 p.m. to 6.00 p.m.

Max. Marks : 100

Instructions : 1) Figures to the right indicate full marks.


2) Q. No. 1 is compulsory. It should be solved in first 30 minutes in
Answer Book Page No. 3. Each question carries one mark.
3) Answer MCQ/Objective type questions on Page No. 3 only. Dont
forget to mention, Q.P. Set (A/B/C/D) on Top of Page.
Duration : 30 Minutes

MCQ/Objective Type Questions

Marks : 20

1. Choose the correct answer :

(120=20)

1) A TCP/IP like a Network Layer provides two types of services. They are
a) Connection terminated and connection non terminated
b) Connection wait and connection acknowledged
c) Connectionless and connection oriented
d) Connection hold and connection release
2) UDP is a
a) connectionless, unreliable transport protocol
b) connection oriented, unreliable transport protocol
c) connectionless, reliable transport protocol
d) none of these
3) Which of the following is true ?
a) UDP length = IP length + IP headers length
b) UDP length = IP length * IP headers length
c) UDP length = IP length/IP headers length
d) UDP length = IP length IP headers length
4) Which of the following is true ?
a) Frame (Collection of bits/bytes), Packet (Collection of Frames)
b) Frame (Collection of Packets), Packet (Collection of Frames)
c) Frame (Collection of checksum bits), Packet (Collection of ACKs)
d) None of these
5) The DHCP server issues a passive open command on UDP port number
___________ and waits for a client.
a) 65
b) 66
c) 67
d) 68
6) Which of the following is not a networking device ?
a) gateway
b) linux
c) routers

d) bridges

7) Which of the following can be a software ?


a) routers
b) modems
c) gateways

d) firewalls
P.T.O.

SLR-BB 267

*SLRBB267*

-2-

8) The supplementary protocol that allows non ASCII data to be sent through E-Mail is
a) FTP (File Transfer Protocol)
b) SMTP (Simple Mail Transfer Protocol)
c) MIME (Multipurpose Internet Mail Extensions)
d) IPV6
9) Each IP packet must contain
a) Source Address
c) Source and destination address

b) Destination Address
d) None of these

10) TELNET solves the problem and allows heterogeneous systems communicate with each
other by defining a universal interface called as the
a) Network Universal Terminal (NUT)
b) Network Realistic Terminal (NRT)
c) Network Virtual Terminal (NVT)
d) None of the above
11) _____________ messages goes across a network between a manager and an agent.
a) SMTP
b) HTTP
c) SNMP
d) None
12) In SNMP a tool __________ going across LAN in monitoring system.
a) SNMP
b) Sniffs every packet
c) Both
d) None
13) Remote Network Monitoring (RMON)
a) Remote Monitoring Network
c) Recent Monitoring Network

b) Remote Network Monitoring


d) None

14) Rmon acts as a network


a) Manager
b) Agent

c) Both

d) None

15) SNMP manager contains additional object than SNMP agent is


a) DLC
b) Trap
c) Management data
d) None
16) RFC 1155 describes SNMP
a) Architecture
c) Management information

b) Structure
d) None

17) MIB view


a) Network element compare managed objects
b) Network elements inspected
c) Both
d) None
18) Network management is based on knowledge of
a) Network topology
b) Network address
c) Network LAN
d) None
19) Fault in network is normally associated with failure of
a) Network component
b) Subsequent loss of connectivity
c) Both
d) None
20) The response time of network depends on
a) Throughput
b) Application
c) Both
______________

d) None

Set A

*SLRBB267*
S

-3-

SLR-BB 267

T.E. (IT) (New) (Part I) Examination, 2014


NETWORK MANAGEMENT
Day and Date : Wednesday, 3-12-2014

Marks : 80

Time : 3.00 p.m. to 6.00 p.m.


Instruction : Figures to the right indicate full marks.
SECTION I
2. Attempt any four of the following :

(45=20)

a) Explain the concept of connection oriented concurrent server.


b) Explain the three way handshaking in TCP connection.
c) Explain DHCP packet format.
d) Explain DNS query and DNS response message in detail.
e) Write a note on TFTP.
f) Explain the different UDP operations.
3. Attempt any one of the following :

(110=10)

a) Explain the concept of connectionless and connection oriented services of the transport
layer protocol.
b) Explain the architecture of E-mail along with neat diagrams of all scenarios.
4. Attempt any one of the following :

(110=10)

a) Explain clearly TELNET and the Network Virtual Terminal associated with TELNET.
b) Describe the Hyper Text transfer protocol in detail.

Set A

SLR-BB 267

-4-

*SLRBB267*

SECTION II
5. Answer the following any two :

(72)

I) Draw and explain SNMP network management architecture.


II) Explain SNMP three tier organization models.
III) Explain proxy server organization model with diagram.
6. Answer the following any two :

(72)

I) With all 5 steps explain fault management.


II) What is the role of network topology in configuration management ?
III) Explain network provisioning.
7. Write short notes :

(62)

I) SNMP operations and functional model.


II) SNMP communication model architecture.
_____________________

Set A

SLR-BB 268

*SLRBB268*
Seat
No.

Set

T.E. (IT) (New) (Part I) Examination, 2014


COMPUTER ORGANIZATION AND ARCHITECTURE
Day and Date : Friday, 5-12-2014
Time : 3.00 p.m. to 6.00 p.m.

Max. Marks : 100

Instructions : 1) Figures to the right indicate full marks.


2) Q. No. 1 is compulsory. It should be solved in first 30 minutes in
Answer Book Page No. 3. Each question carries one mark.
3) Answer MCQ/Objective type questions on Page No. 3 only. Dont
forget to mention, Q.P. Set (A/B/C/D) on Top of Page.
MCQ/Objective Type Questions
Duration : 30 Minutes

Marks : 20

1. Choose the correct answer :

(120=20)

1) Vacuum Tubes and Tunnel Diodes formed devices of which generation ?


a) First generation
b) Second generation
c) Third generation
d) Fourth generation
2) The ability to use all addressing modes in a uniform and consistent way with all opcodes
of the instruction set of an assembly language is called
a) Orthogonality
b) Symmetry
c) Uniformity
d) None of these
3) CISC stand for
a) Complex Instruction Set Computer
c) Complex Information Solution Computer

b) Complex Information Set Computer


d) None of these

4) Which of the following relations hold good between memory levels Mi and Mi + 1 where C
is cost per bit, tA is the Access time and S is the storage capacity ?
a) Ci < Ci + 1, tAi < ti + 1, Si < Si + 1
b) Ci > Ci + 1, tAi > ti + 1, Si < Si + 1
c) Ci > Ci + 1, tAi < ti + 1, Si < Si + 1
d) None of these
5) HDL stands for
a) Hardware Definition Language
c) Hardware Domain Language

b) Hardware Description Language


d) None of these

6) Which of the following statement is true ?


a) Multiplication ends with a single length product and division begins with double length
dividend
b) Multiplication ends with a double length product and division ends with single length
dividend
c) Multiplication ends with a double length product and division begins with double length
dividend
d) None of these
7) When a single instruction is required by the CPU, it is efficient to transfer a block of
consecutive words. This is called
a) Segment
b) Virtual address space
c) Page
d) Page Frame
P.T.O.

SLR-BB 268

*SLRBB268*

-2-

8) 8085 Assembly Languages uses _______ for specifying addressing modes.


a) Operand fields
b) Op codes
c) Special bits
d) None of these
9) If an instruction contains the address W of a storage location which in turn contains the
address X of the desired operand value V(X) then it is called
a) Direct Addressing
b) Auto Indexing
c) Register to Register Addressing
d) Indirect Addressing
10) ______ Register contains the address of the instruction to be executed next.
a) Instruction Register
b) Program Counter
c) Instruction Counter
d) All of these
11) Pipeline branch hazards can be handled by
a) Forwarding branch
b) Delayed branch
c) Both a) and b)
d) None of these
12) The MIPS pipeline with the appropriate registers, called pipeline registers or pipeline latches,
between each
a) Pipeline instructions
b) Pipeline stage
c) Clock cycles
d) None of these
13) _________ arise when an instruction depends on the results of a previous instruction in
a way that is exposed by the overlapping of instructions in the pipeline.
a) Structural hazards
b) Control hazards
c) Data hazards
d) None of these
14) Tomasulo scheme operates on
a) Renaming
c) Common data bus

b) Reservation stations
d) All of the above

15) The important feature of the VLIW is _______


a) ILP
b) Cost effectiveness
c) Performance
d) None of the mentioned
16) ___________ has been developed specifically for pipelined systems.
a) Utility softwares
b) Speed up utilities
c) Optimizing compilers
d) None of the mentioned
17) MIMD Computers exploit _____________
a) Data level parallelism
c) Control level parallelism

b) Thread level parallelism


d) None of these

18) Snooping protocols is called a ________ because it invalidates other copies on a write.
a) Read invalidate protocol
b) Write invalidate protocol
c) Write validate protocol
d) Read and write protocol
19) __________ defines the behavior of reads and writes with respect to accesses to other
memory locations.
a) Consistency
b) Coherence
c) Replication
d) Migration
20) Let T1 be the time taken for a single instruction on a pipelined CPU and T2 be the time
take for a single instruction on a non-pipelined but identical CPU. Comparing T1 and T2
we can say that
a) T1 = T2 + the time taken for one instruction fetch cycle
b) T1 < t2
c) T1 T2
d) T1 T2
______________

Set A

*SLRBB268*

-3-

SLR-BB 268

Seat
No.

T.E. (IT) (New) (Part I) Examination, 2014


COMPUTER ORGANIZATION AND ARCHITECTURE
Day and Date : Friday, 5-12-2014

Marks : 80

Time : 3.00 p.m. to 6.00 p.m.


Instructions : 1) All questions are compulsory.
2) Figures to the right indicate full marks.
SECTION I
2. Attempt any four of the following :

(45=20)

a) Give the salient features of five generation of computers.


b) List and explain different instruction types in ARC-A RISC computer.
c) Explain concept of pseudo operations.
d) Describe the IEEE format for floating point numbers.
e) Explain the concept of cache memory.
f) Explain the different rules of delay element method.
3. Attempt any one of the following :

(110=10)

a) Explain any two design methods of hardwired control unit.


b) Explain the Booths Algorithm clearly and multiply 12 and 3 to find the product.
4. Attempt any one of the following :

(110=10)

a) Explain the virtual memory with an example.


b) Explain the Non-Restoring Division Algorithm by performing 17 10.

Set A

SLR-BB 268

-4-

*SLRBB268*

SECTION II
5. Attempt any four :

(45=20)

1) Define Computer Architecture. Illustrate the seven dimensions of an ISA.


2) Explain different techniques to overcome data hazards.
3) Explain correlating branch prediction technique.
4) What is dependability ? Explain two main measures of dependability.
5) Explain the directory based coherence for a distributed memory multiprocessor system.
6) Explain the basic schemes for enforcing Coherence in a shared memory multiprocessor
system.
6. Attempt any one :

(110=10)

1) With a neat diagram explain the classic five stage pipeline for a RISC Processor.
2) Explain the basic VLIW approach for exploiting ILP, using multiple issues.
7. Attempt any one :

(110=10)

1) With a neat diagram give the basic structure of Tomasulo based MIPS - FP unit and
explain the various fields of reservation stations.
2) What are the key issues in implementing advanced speculation techniques ? Explain
them in detail.
_____________________

Set A

SLR-BB 269

*SLRBB269*
S

T.E. (IT) (New) (Part I) Examination, 2014


SYSTEM SOFTWARE
Day and Date : Monday, 8-12-2014
Time : 3.00 p.m. to 6.00 p.m.

Max. Marks : 100

Instructions : 1) Figures to the right indicate full marks.


2) Q. No. 1 is compulsory. It should be solved in first 30 minutes
in Answer Book Page No. 3. Each question carries one mark.
3) Answer MCQ/Objective type questions on Page No. 3 only.
Dont forget to mention, Q.P. Set (A/B/C/D) on Top of Page.
Duration : 30 Minutes

MCQ/Objective Type Questions

Marks : 20

1. Choose the correct option.

20

1) The gap between Application and PL domain is called


a) Semantic gap
b) Specification gap
c) Execution gap
d) None of these
2) Specification to LEX is in the form of
a) Alphabets
b) Productions
c) Regular expressions
d) None of these
3) Language processor which bridges an execution gap but is not a language
processor is
a) De-translator
b) Migrator
c) Translator
d) Preprocessor
4) Syntax errors are identified by
a) Scanner
b) Parser
c) Semantic analyzer
d) None of these
5) Input buffering is used in _________ phase of compiler.
a) Syntax analysis
b) Lexical analysis
c) Semantic analysis
d) Both a) and b)
6) An ideal compiler should
a) Detect error
c) Detect, repair and correct error

b) Detect and repair error


d) None of these

7) Which parser uses parsing table ?


a) Operator-precedence
c) Recursive descent

b) SR parser
d) LR parser

P.T.O.

SLR-BB 269

*SLRBB269*

-2-

8) In machine instruction format, number of bits used by opcode field is


a) 1 bit
b) 2 bits
c) 3 bits
d) None of these
9) Symbol table in assembler is created by
a) Analysis phase
b) Synthesis phase
c) Both a) and b)
d) None of these
10) In which variant of the intermediate code memory is balanced
a) Variant I
b) Variant II
c) Both a) and b)
d) None of these
11) Instruction cost of following instruction is
SUB 4(R0), *12(R1)
a) 1
b) 2
c) 3
d) 4
12) Which of the following is not Basic block transformation ?
a) Common sub-expression elimination
b) Dead-code elimination
c) Renaming of temporary variables
d) None of these
13) Graph-coloring is a technique for
a) Register allocation
b) Basic block transformation
c) Next-use information
d) None of these
14) Which of the following is not peephole optimization ?
a) Unreachable code
b) Reduction in strength
c) Use of machine idioms
d) None of these
15) Linking table LINKTAB contains information of
a) Object modules
b) Public definitions
c) External references
d) Both b) and c)
16) Overlays are used for
a) Increasing the speed of program b) Reducing main memory requirement
c) Linking external references
d) None of these
17) Which type of the following programs uses relocating logic ?
a) Non-relocating
b) Self relocating
c) Relocating
d) Both a) and c)
18) Type 1 Grammar is
a)
b)


c)
A

19) In absolute loader, linking is done by


a) Programmer
b) Assembler
c) Loader

d)


d) Linker

20) A card used in direct-linking loader for resolving external references is


a) TXT
b) ESD
c) RLD
d) END
______________
Set A

*SLRBB269*
S

-3-

SLR-BB 269

T.E. (IT) (New) (Part I) Examination, 2014


SYSTEM SOFTWARE
Day and Date : Monday, 8-12-2014
Time : 3.00 p.m. to 6.00 p.m.

Marks : 80

Instructions : 1) All questions are compulsory.


2) Figures to the right indicate full marks.
SECTION I
2. Attempt any four :

(54=20)

a) What is an intermediate representation ?


b) State the characteristics of an interpreter.
c) What is the output of lexical analysis ?
d) What is the input and output of an assembler ?
e) What are the advantages of assembly language ?
3. Attempt any one :

10

a) What are the programming language grammars ? State their roles in language
specification.
b) What is the task accomplished by a lexical analyzer ?
4. Attempt any one :

10

a) What is LR parsers ? Illustrate the development of an LR parser for an


illustrative grammar and parse string.
b) How is IC for imperative statements developed ? State the variants involved.
Set A

SLR-BB 269

-4-

*SLRBB269*

SECTION II
5. Attempt any four :

(54=20)

a) List the issues in design of code generator.


b) Define basic block and illustrate the same.
c) How is code generated from DAGS ? Illustrate.
d) What are self relocating programs ?
e) Demonstrate the working of a general loader scheme with a block diagram.
6. Attempt any one :

10

a) What are relocating loaders ? Explain working of a BSS loader.


b) What are subroutine linkages ? How are they carried out ?
7. Attempt any one :

10

a) What concepts are required during the design of a linker ? Give a basic
linking algorithm.
b) Elaborate on the function preserving transformations used in code optimization.
_____________________

Set A

SLR-BB 27

*SLRBB27*
Seat
No.

Set

S.E. (Civil) (Part II) Examination, 2014


BUILDING CONSTRUCTION AND DESIGN (Old)
Day and Date : Thursday, 27-11-2014
Time : 3.00 p.m. to 7.00 p.m.

Total Marks : 100

Instructions : 1) Q. No. 1 is compulsory. It should be solved in first 30


minutes in Answer book Page No. 3. Each question
carries one mark.
2) All questions are compulsory from Section I and II.
3) Figures to the right indicate full marks.
4) Use answer book for Section II.
5) Use of both sides of full imperial sheet is permitted.
6) Answer MCQ/Objective type questions on Page No. 3
only. Dont forget to mention, Q.P. Set (A/B/C/D) on Top
of Page.
MCQ/Objective Type Questions
Duration : 30 Minutes
1. State whether following statements are correct or incorrect :

Marks : 20
(120=20)

1) Semi-detached houses has one common wall to both units.


2) For comfortable working a temperature range of 18C to 28C is ideal.
3) Orientation of building depends on climatic conditions.
4) Rental value and byelaws govern planning of the building.
5) Grouping means locating different rooms (such as kitchen and dining) away
from each other.
6) Privacy is very important for bedrooms.
7) Plenum is type of ventilation system.
8) Warm air is light (less in weight as compared to cool air).
9) Site plan shows positions of door and window.
P.T.O.

SLR-BB 27

-2-

*SLRBB27*

10) Bathroom require Power Point for water heater.


11) Permission drawing is submitted to Municipal Corporation.
12) FSI can be calculated from Block Plan.
13) Driers reduces the elasticity of the paint.
14) Sewer trap is also known as intercepting trap.
15) Anglo-Indian type is a type of WC pan.
16) Eastern aspect is preferred for kitchen.
17) Priming coat is required for adhesion of the paint to surface.
18) Location plan is not a part of permission drawing.
19) Artificial ventilation involves providing doors and windows.
20) Air changes means ventilation.

______________

Set A

*SLRBB27*

-3-

SLR-BB 27

Seat
No.

S.E. (Civil) (Part II) Examination, 2014


BUILDING CONSTRUCTION AND DESIGN (Old)
Day and Date : Thursday, 27-11-2014
Time : 3.00 p.m. to 7.00 p.m.

Marks : 80

Instructions : 1) All questions are compulsory from Section I and II.


2) Figures on right indicate full marks.
3) Use of both sides of full imperial sheet is permitted.
4) Use answer book for Section II.
5) Assume suitable data wherever required and state it clearly.
SECTION I

40

2. Mr. Joshi and Mr. Patil, working in a private company, want to construct a twin
bungalow. Both of them have common requirements as given below regarding
their dream home.
i) Living room 20 m2 1 no.
ii) Kitchen cum dining 15 m2 1 no.
iii) Bedrooms 12 m2 2 nos.
iv) W. C. and Bath of suitable size 1 each.
v) L shaped folded staircase located in a living room.
Draw to a scale 1 : 50
i) Detailed plan of twin bungalows.

20

ii) Elevation of twin bungalows.

20
Set A

SLR-BB 27

-4-

*SLRBB27*

SECTION II
3. Answer any five questions :

40
(58=40)

a) Explain methods of thermal insulation for residential buildings.


b) Explain procedure for painting on new wood work.
c) Enlist all types of plaster finishes and explain any four in detail.
d) Explain importance of septic tank with the sketch (Draw plan and section).
e) With the help of line plans or block diagrams explain difference between
detached, semi-detached and row house bungalows.
f) Explain byelaws for size and height of rooms.
g) Explain defects in Painting.
_____________________

Set A

SLR-BB 270

*SLRBB270*
Seat
No.

Set

T.E. (IT) (Part I) (New) Examination, 2014


PRINCIPLES OF OPERATING SYSTEMS
Day and Date : Wednesday, 10-12-2014
Time : 3.00 p.m. to 6.00 p.m.
Instructions :

Max. Marks : 100

1) Q. No. 1 is compulsory. It should be solved in first 30 minutes in


Answer Book Page No. 3. Each question carries one mark.
2) Answer MCQ/Objective type questions on Page No. 3 only. Dont
forget to mention, Q.P. Set (A/B/C/D) on Top of Page.
MCQ/Objective Type Questions

Duration : 30 Minutes

Marks : 20

1. Choose the correct answer :


1) Operating system is a ______________
a) Hardware program
c) Both a) and b)

b) Software program
d) None of above

2) ______________ provides basis for system and application programs.


a) System call
b) Users
c) Operating system
d) None of above
3) Which of the following is not the state of process ?
a) Blocked
b) Running
c) Ready

d) Privileged

4) To get process identification number, we use ______________ system call.


a) Fork
b) Exit
c) Abort
d) None of the above
5) Process state is defined as ______________
a) Current activity of program
b) Current activity of process
c) Current activity of project
d) All of above
6) Waiting state is also called as ______________
a) Blocked state
b) Terminated state
c) Ready state
d) Running state
7) The number of processes completed per unit time is known as ______________
a) Output
b) Efficiency
c) Throughput
d) Capacity
8) The solution to critical section problem should satisfy mutual exclusion, progress and
bounded waiting.
a) The statement is false
b) The statement is true
c) The statement is contradictory
d) None of above
9) SJF is ______________
a) Pre-emptive
b) Non-preemptive

c) Both a) and b)

d) None of above

10) Which term describes the data structure used during the backing up the environment
data of a process when it is interrupted ?
a) Context switching
b) PCB
c) Spooling
d) Single tasking
P.T.O.

SLR-BB 270

-2-

*SLRBB270*

11) If we pre-empt a resource from a process, the process cannot continue with its normal
execution and it must be
a) Aborted
b) Rolled back
c) Terminated
d) Queued
12) If the resources are always preempted from the same process, ______can occur.
a) Deadlock
b) System crash
c) Aging
d) Starvation
13) An edge from process Pi to Pj in a wait for graph indicates that
a) Pi is waiting for Pj to release a resource that Pi needs
b) Pj is waiting for Pi to release a resource that Pj needs
c) Pi is waiting for Pj to leave the system
d) Pj is waiting for Pi to leave the system
14) A computer system has 6 tape drives, with n processes competing for them. Each
process may need 3 tape drives. The maximum value of n for which the system is
guaranteed to be deadlock free is
a) 2
b) 3
c) 4
d) 1
15) A multilevel page table is preferred in comparison to a single level page table for translating
virtual address to physical address because
a) It reduces the memory access time to read or write a memory location
b) It helps to reduce the size of page table needed to implement the virtual address
space of a process
c) It is required by the translation look aside buffer
d) It helps to reduce the number of page faults in page replacement algorithms
16) To obtain better memory utilization, dynamic loading is used. With dynamic loading,
a routine is not loaded until it is called. For implementing dynamic loading.
a) Special support from hardware is required
b) Special support from operating system is essential
c) Special support from both hardware and operating system is essential
d) User programs can implement dynamic loading without any special support from
hardware or operating system
17) Segment replacement algorithms are more complex than page replacement algorithms
because
a) Segments are better than pages
b) Pages are better than segments
c) Segments have variable sizes
d) Segments have fixed sizes
18) A process refers to 5 pages, A, B, C, D, E in the order : A, B, C, D, A, B, E, A, B, C, D, E.
If the page replacement algorithm is FIFO, the number of page transfers with an empty
internal store of 3 frames is
a) 8
b) 10
c) 9
d) 7
19) In the working set model, for :
2 6 1 5 7 7 7 7 5 1 6 2 3 4 1 2 3 4 4 4 3 4 3 4 4 4 1 3 2 3 if DELTA = 10, then the working
set at time t1( .... 7 5 1) is :
a) {1, 2, 4, 5, 6}
b) {2, 1, 6, 7, 3}
c) {1, 6, 5, 7, 2}
d) {1, 2, 3, 4, 5}
20) An I / O port typically consists of four registers status, control, ________and ________registers.
a) System in, system out
b) Data in, data out
c) Flow in, flow out
d) Input, output

______________

Set A

*SLRBB270*

-3-

SLR-BB 270

Seat
No.

T.E. (IT) (Part I) (New) Examination, 2014


PRINCIPLES OF OPERATING SYSTEMS
Day and Date : Wednesday, 10-12-2014

Marks : 80

Time : 3.00 p.m. to 6.00 p.m.


Instructions :

1) All questions are compulsory.


2) Figures to the right indicate full marks.
SECTION I

2. Explain IPC in detail.

10
OR

What is process state ? Explain different types of states with the help of process state transition
and real life example.
3. With the help of schematic, structure and example explain the concept of monitor.

10

OR
What is semaphore ? Explain its characteristics, advantages and disadvantages in detail.
4. Write a short note on (any four) :

(45=20)

a) Memory management
b) Medium term scheduler
c) Context switch
d) Swap instruction
e) Computer system architecture
f)

RR scheduling.

Set A

SLR-BB 270

-4-

*SLRBB270*

SECTION II
5. Attempt any five :

(54=20)

a) Explain with example paging hardware and basic method with 32 byte memory with
4-byte pages.
b) Explain with example the single and multiple instance resource type.
c) Explain the fork() in paging with example.
d) Explain with example all the strategies of allocations of frames.
e) Explain the difference between internal and external fragmentation.
f)

Explain the kernel I/O subsystem with example.

g) Explain with figure interrupt driven I/O cycle.


6. State and explain the operating system device driver interface.

10

OR
Explain the need for page replacement with first three page replacement algorithm.
7. Attempt any two :

(25=10)

a) State and explain the necessary conditions for preventing the deadlock.
b) Explain the memory protection in paged environment.
c) Explain the paging hardware support with TLB.

_____________________

Set A

SLR-BB 271

*SLRBB271*
Seat
No.

Set

T.E. (Information Technology) (Part I) (New) Examination, 2014


DESIGN AND ANALYSIS OF ALGORITHMS
Day and Date : Friday, 12-12-2014
Time : 3.00 p.m. to 6.00 p.m.

Total Marks : 100

Instructions : 1) Figures to the right indicate marks to a question.


2) Q. No. 1 is compulsory. It should be solved in first 30 minutes in
Answer Book Page No. 3. Each question carries one mark.
3) Answer MCQ/Objective type questions on Page No. 3 only.
Dont forget to mention, Q.P. Set (A/B/C/D) on Top of Page.
MCQ/Objective Type Questions
Duration : 30 Minutes

Marks : 20

1. Choose correct alternative(s) :

20

1) Example of O(n2) algorithm is (are)


a) Initializing all elements in 2-dimensional array
b) Printing all elements in 2-dimensional array
c) Sorting array in ascending order which is in decreasing order using quick sort
d) All of the above
2) Sequential search requires array in __________ order.
a) Increasing
b) Decreasing
c) Random

d) Any one of these

3) O(n) means computing time is ____________


a) Linear
b) Constant
c) Quadratic

d) Exponential

4) Which of the following statement is false ?


a) n! = O(nn)
b) 10n2 + 9 = O(n)
c) n2 + logn = O(n2)
d) n2/log n = O(n2)
5) Best case complexity for successful search in Binary search is _________
a) (1)
b) (nlogn)
c) (n)
d) (logn)
6) If a = 1, b = 2 and f(n) = c then time complexity of T(n) will be ___________
a) (1)
b) (nlogn)
c) (n)
d) (logn)
7) Kruskals algorithm is for finding _________
a) All pairs shortest path
b) Single source shortest path
c) Minimum cost spanning tree
d) Minimum cost tour
8) In job sequencing with deadlines problem, if n = 3, (p1, p2, p3) = (10, 100, 15) and
(d1, d2, d3) = (1, 1, 1), then maximum profit will be
a) 100
b) 110
c) 125
d) 10
P.T.O.

SLR-BB 271

-2-

*SLRBB271*

9) Which of the following statement is correct ?


a) Greedy method uses principle of optimality
b) Greedy method has backward and forward approach
c) Greedy method generates only one sequence of decision
d) Knapsack problem is an example of ordering paradigm
10) If x1, x2, x3 are three sorted files of length 30, 20, 10 records respectively.
Total number of record moves using greedy method to merge all files is
a) 90
b) 110
c) 60
d) None
11) Which of the following technique is used in finding all pairs shortest path ?
a) Dynamic programming
b) Backtracking
c) Greedy Technique
d) None
12) Multistage graph G = (V, E) is a _____________
a) Directed graph
b) Undirected graph
c) Both a) and b)
d) None
13) Which one of the following does not hold principal of optimality ?
a) Knapsack problem
b) Shortest path
c) Multistage graphs
d) Longest path
14) A ___________ schedule is a schedule in which the processing of a task on any
processor is not terminated until the task is complete.
a) Nonpreemptive schedule
b) Preemptive
c) Both a) and b)
d) Flow shop
15) _________ to use dynamic programming to solve a problem with multiplicative
optimization function.
a) Flow shop
b) Optimal binary search trees
c) Reliability design
d) Multistage graphs
16) The name backtrack was first coined by ________ in the year 1950.
a) L. Baumert
b) R. J. Walker
c) S. Golomb
d) D. H. Lehmer
17) A graph is said to be __________ iff it can be drawn in a place in such way that no
two edges cross each other.
a) Direct
b) Colored
c) Planar
d) Cross
18) Any problem we solve using backtracking satisfy a constraint.
a) Implicit
b) Explicit
c) Both a) and b) d) Either a) or b)
19) The following are the statements regarding the NP problems. Choose the right
option from the following options :
i) All NP-complete problems are not NP Hard.
ii) Some NP hard problems are not known to be NP-complete.
a) Both (i) and (ii) are true
b) Both (i) and (ii) are false
c) Only (i) is true
d) Only (ii) is true
20) Which design strategy stops the execution when it finds the solution otherwise
starts the problem from top ?
a) Backtracking
b) Branch and Bound
c) Dynamic programming
d) None
______________

Set A

*SLRBB271*

-3-

SLR-BB 271

Seat
No.

T.E. (Information Technology) (Part I) (New) Examination, 2014


DESIGN AND ANALYSIS OF ALGORITHMS
Day and Date : Friday, 12-12-2014
Time : 3.00 p.m. to 6.00 p.m.

Marks : 80

Instructions : 1) All questions are compulsory.


2) Figures to the right indicate marks to a question.
SECTION I
2. Attempt any four :

20

1) How to validate and analyze algorithms ?


2) Devise an algorithm that inputs three integers and outputs them in nondecreasing
order.
3) Write recurrence relation for merge sort and solve using substitution method.
4) Explain how array is partitioned in Quick sort with example.
5) Find the feasible solutions for job sequencing with deadlines,
n = 6, (p1, p2, p3, p4, p5, p6) = (10, 34, 67, 45, 23, 99) and deadlines (d1, d2, d3,
d4, d5, d6) = (2, 3, 1, 4, 5, 3).
6) Define spanning tree. Give the applications of spanning tree.
3. Attempt any one :

10

1) Devise a binary search algorithm that splits the set not into two sets of (almost)
equal sizes but into two sets, one of which is twice of the other. Draw binary decision
tree for given array using the algorithm
10

20

30

40

50

60

70

80

90

2) Find optimal solution to knapsack problem n = 7, m = 15, (p1, ...., p7) = (10, 5, 15,
7, 6, 18, 3) and (w1, ...., w7) = (2, 3, 5, 7, 1, 4, 1).
4. Write the problem statement of optimal storage on tapes. Find all possible ordering for
the instance n = 3, (I1, I2, I3) = (8, 10, 12). Find minimum MRT.
10

Set A

SLR-BB 271

-4-

*SLRBB271*

SECTION II
5. Attempt any four :

(54=20)

a) Write an algorithm for all pair shortest paths.


b) Write a short note on Reliability design.
c) Explain flow shop scheduling with example.
d) Write an algorithm for recursive backtracking for sum of subsets problem.
e) Explain graph coloring with example.
f) Write a short note on efficiency considerations for branch and bound method.
6. Attempt the following :
a) Solve below problem by multistage graph method and find minimum cost
from s to t.

10

b) Explain 8-queen problem with example.

10

OR
c) Explain 0/1 knapsack method with respect to branch and bound method.
_____________________

Set A

SLR-BB 273

*SLRBB273*
S

T.E. (IT) (Part II) Examination, 2014


DATA BASE ENGINEERING
Day and Date : Tuesday, 25-11-2014
Time : 10.00 a.m. to 1.00 p.m.
Instructions :

Max. Marks : 100

1) Q. No. 1 is compulsory. It should be solved in first 30 minutes in


Answer book Page No. 3. Each question carries one mark.
2) Wherever necessary support your answers with neat figures or block
diagrams.
3) Answer any three full questions from Section I and
Section II.
4) Answer MCQ/Objective type questions on Page No. 3 only. Dont
forget to mention, Q.P. Set (A/B/C/D) on Top of Page.

Duration : 30 Minutes

MCQ/Objective Type Questions

1. Tick the correct alternative :

Marks : 20
20

1) ___________ represents multivalued attributes.


A) Double lines
B) Double ellipses
C) Dashed ellipses
D) Double rectangles
2) Which of the following are database design methods ?
A) Normalization and entity relationship diagram
B) Tuple relational calculus and relational algebra
C) Object based method and semi structured method
D) Data mining method and analysis method
3) Set difference operation allow us to find tuples that
A) are in two different relations
B) are in one relation but also in other relation
C) are in one relation but are not in another relation
D) are in one relation but present differently in other relation
4) One of the following is NOT a valid E-R schema constraint
A) Cardinality constraint
B) Key constraint
C) Participation constraint
D) Isolation constraint
5) SQL stands for
A) Sequential Query Language
C) Structured Query Language

B) Structured Queing Language


D) Simple Query Language

6) Following are not disadvantages of file processing system ?


A) Data redundancy and inconsistency
B) Serializability and isolation of transaction
C) Difficulty in accessing data and atomicity problem
D) Security problem

P.T.O.

SLR-BB 273

-2-

*SLRBB273*

7) Following is NOT a valid SQL operation


A) DROP
B) ALTER
C) EXCEPT
D) SET
8) Telephone number is following type of attribute
A) Composite
B) Derived
C) Multivalued
D) Nullable
9) The relational algebra is a _______________
A) Procedural language
B) Non-procedural language
C) Both (A) and (B)
D) None of the above
10) A domain is atomic, if elements of the domain are considered to be indivisible units.
A) True
B) False
11) A transaction that completes its execution successfully is said to be
A) Aborted
B) Rolled back
C) Committed
D) Compensating
12) Transaction is a
A) Unit of program execution that accesses and possibly updates various data item
B) Special relation in database
C) Transfer of money from one account to another via network
D) One type of integrity constraint on relation
13) Deadlock in concurrency control is a situation where
A) All locks in data item are in dead state
B) All locks are consumed and system is a dead state
C) No transaction can ever proceed with its aborted state
D) No transaction can ever proceed with its normal execution
14) Checkpoints are used for
A) Transaction recovery
B) Transaction concurrency control
C) Designing serializable schedule
D) Designing optimized query
15) Time stamp based protocol is
A) Transaction isolation protocol
B) Concurrency control protocol
C) Transaction management protocol
D) Recovery system protocol
16) Consider two statements for two-phase locking protocol.
S1 : In shrinking phase a transaction may release locks.
S2 : In shrinking phase a transaction may not obtain any new lock.
A) Both S1 and S2 are false
B) Only S1 is true
C) Only S2 is true
D) Both S1 and S2 are true
17) Which of the following is reason for transaction failure ?
A) Logical error and system error
B) Atomicity and isolation
C) Presence of cycle in wait-for-graph
D) None of the above
18) Each leaf in a B+ tree with n pointers can hold upto ____________ values.
A) n
B) n + 1
C) n 1
D) None of these
19) ACID property of a transaction are
A) atomicity, coherency, integrity, durability
B) atomicity, concurrency, integrity, durability
C) atomicity, concurrency, inheritance, durability
D) atomicity, consistency, isolation, durability
20) Clustering indices are also called as ______________
A) Secondary indices
B) Ordered indices
C) Primary indices
D) Hash indices
______________

Set A

*SLRBB273*
S

-3-

SLR-BB 273

T.E. (IT) (Part II) Examination, 2014


DATA BASE ENGINEERING
Day and Date : Tuesday, 25-11-2014

Marks : 80

Time : 10.00 a.m. to 1.00 p.m.


Instructions :

1) Wherever necessary support your answers with neat figures or


block diagrams.
2) Answer any three full questions from Section I and
Section II.
SECTION I

2. Answer the following :

20

A) Explain with neat diagram the database architecture.


B) Explain the concept of keys with type and examples.
C) Explain the concept of functional dependency with example.
D) Explain the following terms with example :
a) Entity
b) Relation
c) Attribute
d) Key.
3. Explain all types of join operations with example.

10

OR
Explain concept of lose less decomposition and dependency preservation with example.
4. Construct E-R diagram for a car insurance company that has a set of customers, each of
whom owns one or more cars. Each car has associated with a zero to any number of recorded
accidents.

10

Set A

SLR-BB 273

-4-

*SLRBB273*

SECTION II
5. Answer any four :

20

A) What is multilevel indexing ?


B) Discuss advantages and disadvantages of two-phase locking protocol.
C) Discuss ACID properties.
D) Explain difference between static and dynamic hashing.
E) Explain B+ tree file organization.
6. Discuss insertion of a node in B+ tree in detail.

10

OR
Explain time-stamp protocol in detail.
7. Explain the concept of view serializability.

10

_____________________

Set A

SLR-BB 274

*SLRBB274*
Seat
No.

Set

T.E. (IT) (Part II) Examination, 2014


COMPILER CONSTRUCTION
Day and Date : Wednesday, 26-11-2014
Time : 10.00 a.m. to 1.00 p.m.

Max. Marks : 100

Instructions : 1) Q. No. 1 is compulsory. It should be solved in first 30 minutes


in Answer Book Page No. 3. Each question carries one mark.
2) Answer MCQ/Objective type questions on Page No. 3
only. Dont forget to mention, Q.P. Set (A/B/C/D) on Top
of Page.
Duration : 30 Minutes

MCQ/Objective Type Questions

Marks : 20

1. Choose the correct alternative :


20
1) Identify the compiler construction tool
a) Syntax analyzer
b) Lexical analyzer
c) Scanner
d) Dataflow engine
2) For the conversion of NFA to DFA which of the following algorithm is used by
the lexical analyzer ?
a) DFA simulation
b) Subset construction
c) Lexical analysis
d) NFA simulation algorithm
3) Which of the following is the left-recursive grammar ?
a) E > E + E
b) E > F + E
c) E > F + F
d) All
4) The ambiguous grammar produces one or more __________
a) Left most derivation
b) Right most derivation
c) Both
d) None
5) Backtracking is used in ___________
a) Predictive parser
b) LR parser
c) Recursive-descent parser
d) LALR parser
6) Which of the following type of graph shows evaluation order ?
a) Dependency graph
b) Flow graph
c) Basic block graph
d) Recursion graph
7) Which of the following is not included in an Activation record ?
a) Access link
b) Dynamic link c) Control link
d) None
8) Which of the following is not an addressing mode ?
a) Indirect
b) Absolute
c) Indirect-Absolute
d) Register
P.T.O.

SLR-BB 274

-2-

*SLRBB274*

9) Which of the following phase of the compilation is an optional phase ?


a) Lexical
b) Semantic
c) Code optimization
d) Code generation
10) A synthesized attribute is an attribute whose value at a parse tree node
depends on _____________
a) Attributes at siblings only
b) Parent node only
c) Attribute at a children node only d) All
11) A basic block is a block of _______________
a) Non consecutive statements
b) Consecutive statements which may be entered at the beginning and when
entered are executed in sequence without halt
c) Consecutive statements which may be entered at the beginning and when
entered are executed in sequence with halt
d) All
12) Which of the following is not a Peephole Optimization ?
a) Removal of unreachable codes
b) Elimination of multiple jumps
c) Elimination of loop invariant computation
d) All of the above
13) YACC is a ________________
a) Lexical analyzer
c) Parser generator

b) Lexical analyzer generator


d) All

14) The Three-Address-Code for the statement p = q + r by IC generator is


a) t1 = q + r and p = t1
b) p = q + r
d) All
c) t1 = q + r and t2 = p
15) Which of the following information is not required in code generation ?
a) Flowgraphs
b) Next-use info
c) Register descriptor
d) Parameter descriptor
16) Output of semantic analyzer is syntax tree. (State True/False)
17) The left factored grammar can be recognized by predictive parser.
(State True/False)
18) The FOLLOW function can be used only for terminal symbols.
(State True/False)
19) An induction variable in a loop every time changes its value.
(State True/False)
20) Constant folding is not the function preserving transformation.
(State True/False)
______________
Set A

*SLRBB274*

-3-

SLR-BB 274

Seat
No.

T.E. (IT) (Part II) Examination, 2014


COMPILER CONSTRUCTION
Day and Date : Wednesday, 26-11-2014

Marks : 80

Time : 10.00 a.m. to 1.00 p.m.


SECTION I
2. Write short notes (Attempt any 4) :

20

a) Front end of a compiler


b) L-attributed definition
c) Left factoring
d) Syntax definition for a simple one pass compiler
e) LALR parsing.
3. Attempt any one :

10

What is the role of transition diagram in recognizing tokens ? Explain with the
transition diagram for relational operators.
OR
How the SLR parsing table is constructed ? Explain with example.
4. What is Syntax Directed definition ? Explain it with an mathematical expression
example.

10

Set A

SLR-BB 274

-4-

*SLRBB274*

SECTION II
5. Write short notes (Attempt any 4) :

20

a) Activation tree
b) Heap allocation strategy
c) Types of three address code statements
d) Symbol table organization
e) Loop optimization
6. Attempt any one :

10

How the three address statements are implemented using quadruples, triples
and indirect triples ? Elaborate with examples.
OR
What is code generation ? List and explain the issues in the design of a code generator.
7. What is optimization ? Explain optimization of basic blocks with example.

10

_____________________

Set A

SLR-BB 275

*SLRBB275*
Seat
No.

Set

T.E. (I.T.) (Part II) Examination, 2014


OPERATING SYSTEM II
Day and Date : Thursday, 27-11-2014
Time : 10.00 a.m. to 1.00 p.m.

Total Marks : 100

Instructions : 1) Q. No. 1 is compulsory. It should be solved in first 30 minutes


in Answer book Page No. 3. Each question carries one mark.
2) Answer MCQ/Objective type questions on Page No. 3 only.
Dont forget to mention, Q.P. Set (A/B/C/D) on Top of Page.
MCQ/Objective Type Questions
Duration : 30 Minutes

Marks : 20

1. Multiple Choice Questions :


(120=20)
1) ________ is having highest priority according to processor execution levels.
a) Disk
b) Network devices
c) Clock
d) Machine errors
2) Process control subsystem is not responsible for
a) Process synchronization
b) Memory management
c) Interprocess communication
d) File handling
3) Race for locked buffer occurs in
a) Second scenario
b) Fifth scenario
c) Third scenario
d) Fourth scenario
4) The kernal finds the block on the hash queue, but its buffer is currently busy,
this statement indicates
a) Second scenario
b) Fifth scenario
c) First scenario
d) Fourth scenario
5) Kernel uses ______________ algorithm to convert pathname to inode.
a) ialloc
b) iput
c) iget
d) namei
6) Kernel uses _____________ algo. For allocating disk blocks.
a) iget
b) alloc
c) ialloc
d) getblk
7) Which field is present in U area ?
a) State
c) Process id

b) Real and effective ids


d) Timers

8) All system calls return _____________ on failure.


a) 1
b) 1
c) 0

d) 2
P.T.O.

SLR-BB 275

-2-

*SLRBB275*

9) The permission modes on newly created file will be


a) Octal
b) Hexadecimal c) Decimal
d) Binary
10) The kernel must write the buffer contents to disk before reassigning the
buffer; this condition is known as
a) Asynchronous write
b) Delayed write
c) Synchronous write
d) None
11) What is a context switch ?
a) Kernel switches from executing one process to another
b) Process switches from kernel mode to user mode
c) Process switches from user mode to kernel mode
d) None of the above
12) How do you get parent process identification number ?
a) waitpid
b) getpid()
c) getppid()
d) parentid()
13) If the fork () system call returns 1, then it means
a) No new child process is created
b) The child process is an orphan
c) The child process is in Zombie
d) A system has 512MB of physical memory
14) Which among the following is not a suitable virtual memory size for this
system architecture ?
a) 512MB
b) 256M
c) 4GB
d) None of the above
15) Which signal is sent when the child process terminates ?
a) SIGINIT
b) SIGKILL
c) SIGSTOP
d) SIGCHLD
16) The major number identifies the _______ associated with the device.
a) driver
b) protocol
c) port
d) none of the mentioned
17) When a child process exits before the parent process exits, which of the
following is true ?
a) the child process becomes defunct
b) the parent process becomes defunct
c) if the parent process does not handle SIGCHLD, the child process becomes
a zombie
d) none of the above
18) Which of the following commands can be used to copy files across systems ?
a) ssh
b) telnet
c) rsh
d) ftp
19) Setting the execute bit on has no meaning, if the file is a
a) Directory
b) Shell script
c) Source code d) Symbol table
20) In which section of a process the information about the arguments to the
program are available ?
a) data
b) text
c) stack
d) user block
Set A
______________

*SLRBB275*

-3-

SLR-BB 275

Seat
No.

T.E. (I.T.) (Part II) Examination, 2014


OPERATING SYSTEM II
Day and Date : Thursday, 27-11-2014
Time : 10.00 a.m. to 1.00 p.m.

Marks : 80

Instructions : 1) All questions are compulsory.


2) Figures to the right indicate marks.
3) Assume suitable data if necessary.
SECTION I
2. Attempt any four :

(45=20)

1) Explain the process environment and building block primitives.


2) Explain the working of pipe() system call.
3) Explain the third and fourth scenario for buffer allocation.
4) Explain ialloc algorithm.
5) Explain the algorithm for making new node.
3. A) Write and explain algorithm for buffer allocation.

10

OR
B) Write and explain algorithm to convert pathname into an inode.
4. What are the data structures of UNIX kernel for processes ?

10

Set A

SLR-BB 275

-4-

*SLRBB275*

SECTION II
5. Solve any four :

(45=20)

a) Explain with figure the complete process state transition diagram.


b) What is region ? Explain it with example of mapping of virtual address to
physical address ?
c) Explain the algorithm of process creation.
d) State and explain the functions of the clock interrupt handler.
e) Explain with figure the fork in a paging system.
6. Explain the line disciplines with removing and placing characters on clist.

10

OR
Explain all parts of swapping algorithm with example.
7. Solve any two :

(25=10)

1) Components of the context of a process.


2) Signals and algorithm for handling signals.
3) Fair share scheduler.
_____________________

Set A

SLR-BB 276

*SLRBB276*
S

T.E. (Information Technology) (Part II) Examination, 2014


SOFTWARE ENGINEERING
Day and Date : Friday, 28-11-2014
Time : 10.00 a.m. to 1.00 p.m.

Max. Marks : 100

Instructions : 1) Q. No. 1 is compulsory. It should be solved in first 30 minutes


in Answer book Page No. 3. Each question carries one mark.
2) Answer MCQ/Objective type questions on Page No. 3 only.
Dont forget to mention, Q.P. Set (A/B/C/D) on Top of Page.

Duration : 30 Minutes

MCQ/Objective Type Questions

Marks : 20

1. 1) The agent that performs the transformation of data from one state to another is
called
a) A process (or bubble)
b) A program
c) An entity
d) None of these
2) DFDs which are hierarchically organized and helps in progressively partitioning
and analyzing large problems is called
a) A leveled DFD set
b) Unlevelled DFD set
c) Normalized DFD set
d) Simple DFD set
3) There are two approaches to prototyping
a) Conventional and contemporary
b) Throwaway and evolutionary
c) Standard and normal
d) None of these
4) One of the characteristics of SRS is
a) Ambiguous
b) Consistent

c) Desirability

d) Avoidability

5) Four most common types of errors are


a) Omission, system fault, system malfunction, low memory
b) Omission, incomplete, no scalability, low memory
c) Inconsistency, incorrect fact and ambiguity
d) None of these
6) A system is considered to be ______________ if it consists of discreet components
which can be implemented separately and a change to one component has minimal
impact on another component.
a) Singular
b) Biased
c) Modular
d) Conventional
7) Coupling between modules is the measure of ______________ among modules.
a) Metrics
b) Space
c) Processing speed
d) Interdependence
P.T.O.

SLR-BB 276

-2-

*SLRBB276*

8) ______________ is the relation between classes that allows for definition and
implementation of one class bases on the definition of existing classes.
a) Polymorphism b) Aggregation
c) Binding
d) Inheritance
9) The types of inheritance when a subclass inherits from many super classes is called
a) Multiple inheritance
b) Single inheritance
c) Multilevel inheritance
d) None of these
10) The code associated with a given procedure call is not known until the moment of
the call is called
a) Static binding
b) Memory binding
c) Resource binding
d) Dynamic
11) In a class diagram the generalization specialization relationship is specified by
having __________________ coming from the subclass to the super class.
a) triangle
b) rectangle
c) arrow
d) circle
12) In ________________ testing the structure of the program is not considered.
a) Black box testing
b) Pair wise testing
c) State based testing
d) None of these
13) For some systems when same inputs provide same outputs, then the
appropriate testing used is ____________
a) White box testing
b) Black box testing
c) State based testing
d) None of these
14) SPC stands for _____________
a) Statistical Process Control
b) Status Product Control
c) Single Process Control
d) Similar Process Control
15) LOC stands for __________________
a) Line of Compiler
b) Line of Code
c) Line of Composite
d) None of these
16) UML stands for
a) Unified Modeling Language
b) Unified Model Language
c) Unified Markup Language
d) None of these
17) OOD stands for _______________
a) Object Oriented Development
b) Object Oriented Design
c) Both a) and b)
d) None of these
18) _____________ values can be used to specify additional properties of the elements
to which they are attached.
a) Tagged values
b) Untagged values
c) Random values
d) None of these
19) DD stands for
a) Data directory
b) Data dictionary
c) Data on demand
d) None of these
20) A tool that permits a designer to consider a component at a basic level is
a) Non abstraction
b) Composite
c) Abstraction
d) None of these
______________

Set A

*SLRBB276*
S

-3-

SLR-BB 276

T.E. (Information Technology) (Part II) Examination, 2014


SOFTWARE ENGINEERING
Day and Date : Friday, 28-11-2014

Marks : 80

Time : 10.00 a.m. to 1.00 p.m.


Instructions : 1) All questions are compulsory.
2) Figures to the right indicate full marks.
3) Assume suitable data if necessary.
SECTION I
2. Write short notes on any four :

(45=20)

a) What are the desired characteristics of software process ? Explain any two in detail.
b) With a neat schematic explain the waterfall model.
c) What is a SRS (Software Requirement Specification) ? Explain any five
characteristics of SRS.
d) Explain what software metrics are.
e) Explain in detail the COCOMO model.
3. With a neat figure give a detail description of the CMM.

10

OR
Explain the top down and bottom up estimation approach.
4. Write short notes on any two :

10
(25=10)

a) The Spiral Model.


b) Object Oriented Modelling.
c) Components of an SRS.
d) Data Flow Modelling.

Set A

SLR-BB 276

-4-

*SLRBB276*

SECTION II
5. Attempt any four :

(45=20)

a) Explain Object Oriented Analysis and Object Oriented Design.


b) Explain Coupling and Cohesion in brief.
c) Explain in detail the Review Process.
d) Explain how Defect Analysis is done and prevented.
e) Explain in detail the Unit Testing.
6. A) Explain Project Monitoring and Tracking in detail.

10

OR
B) Explain the White Box testing in detail.
7. Write short notes on any two :

10
(25=10)

a) Test cases and Test plans


b) Causal analysis
c) Activity level analysis using SPC.
_____________________

Set A

SLR-BB 277

*SLRBB277*
S

T.E. (IT) (Part II) Examination, 2014


ARTIFICIAL INTELLIGENCE
Day and Date : Saturday, 29-11-2014
Time : 10.00 a.m. to 1.00 p.m.

Max. Marks : 100

Instructions : 1) Figures to the right indicate full marks.


2) Q. No. 1 is compulsory. It should be solved in first 30 minutes in
Answer book Page No. 3. Each question carries one mark.
3) Answer MCQ/Objective type questions on Page No. 3 only. Dont
forget to mention, Q.P. Set (A/B/C/D) on Top of Page.
Duration : 30 Minutes

MCQ/Objective Type Questions

1. MCQ :
1) A perceptron is a _________________
a) Feed-forward neural network
c) Back-tracking algorithm

Marks : 20
(201=20)

b) Back-propagation algorithm
d) None of these

2) Which is true ?
a) Not all formal languages are context-free
b) All formal languages are context free
c) All formal languages are like natural language
d) None of these
3) Which is not true ?
a) The union and concatenation of two context-free languages is context-free
b) The reverse of a context-free language is context-free, but the complement need not be
c) Every regular language is context-free because it can be described by a regular grammar
d) None of these
4) What is the goal of artificial intelligence ?
a) To solve real-world problems
b) To solve artificial problems
c) To explain various sorts of intelligence d) None of these
5) An algorithm is complete if
a) It terminates with a solution when one exists
b) It starts with a solution
c) It does not terminate with a solution
d) None of these
6) Which is true regarding BFS ?
a) BFS will get trapped exploring a single path
b) The entire tree so far been generated must be stored in BFS
c) BFS is not guaranteed to find a solution, if exists
d) None of these
7) What is a heuristic function ?
a) A function to solve mathematical problems
b) A function which takes parameters of type string and returns an integer value
c) A function whose return type is nothing
d) None of these
P.T.O.

SLR-BB 277

*SLRBB277*

-2-

8) The traveling salesman problem involves n cities with paths connecting the cities. The
time taken for traversing through all the cities, without knowing in advance the length of a
minimum tour, is
c) O(n!)
d) None of these
a) O(n)
b) O(n2)
9) The problem space of means-end analysis has
a) An initial state and one or more goal states
b) One or more initial states and one goal state
c) One or more initial states and one or more goal state
d) None of these
10) An algorithm A is admissible if
a) It is not guaranteed to return an optimal solution when one exists
b) It is guaranteed to return an optimal solution when one exists
c) It returns more solutions, but not an optimal one
d) None of these
11) Knowledge may be
I) Declarative
II) Procedural
III) Non-procedural.
a) Only (I) above

b) Only (II) above

c) Only (III) above

d) None of these

12) In Bayes theorem, what is the meant by P (Hi|E) ?


a) The probability that hypotheses Hi is true given evidence E
b) The probability that hypotheses Hi is false given evidence E
c) The probability that hypotheses Hi is true given false evidence E
d) None of these
13) Default reasoning is another type of
a) Monotonic reasoning
c) Bitonic reasoning

b) Analogical reasoning
d) None of these

14) How many proposition symbols are there in artificial intelligence ?


a) 1
b) 2
c) 3
d) 4
15) How many types of agents are there in artificial intelligence ?
a) 1
b) 2
c) 3
16) Semantic Networks is
a) A way of representing knowledge
c) Data Type

d) 4

b) Data structure
d) None of the mentioned

17) How many logical connectives are there in artificial intelligence ?


a) 2
b) 3
c) 4
d) 5
18) Perception involves
a) Sights, sounds, smell and touch
c) Boxing

b) Hitting
d) None of these

19) Expert systems use ________________ representation for knowledge.


a) Symbolic
b) Mathematical
c) Conventional
d) None of these
20) ____________ is a theory of how to represent the kind of knowledge about events that is
usually contained in natural language sentences.
a) Acquisitional knowledge
b) Referential knowledge
c) Conceptual dependency
d) Frames
______________

Set A

*SLRBB277*
S

-3-

SLR-BB 277

T.E. (IT) (Part II) Examination, 2014


ARTIFICIAL INTELLIGENCE
Day and Date : Saturday, 29-11-2014

Marks : 80

Time : 10.00 a.m. to 1.00 p.m.


Instructions : 1) All questions are compulsory.
2) Figures to the right indicate full marks.
SECTION I
2. Attempt any four :

(54=20)

a) What is a frame problem ?


b) Give the advantages and disadvantages of Generate and Test method of search.
c) How is the level of a model computed ?
d) What is Means-ends analysis ?
e) State the characteristics of AI problems.
3. Attempt any two :

10

a) How is knowledge represented ? Illustrate the different approaches of knowledge


representation.
b) Compare between A* and AO* algorithms.
4. Attempt any one :

10

a) What is an AI Technique ? Illustrate the steps involved.


b) Illustrate Best First Search algorithm with an example.

Set A

SLR-BB 277

-4-

*SLRBB277*

SECTION II
5. Attempt any four :

(54=20)

a) Compare Logic and PROLOG representations.


b) Give the CD representation of Mahesh cut fruits with a knife.
c) What is global ontology ? Illustrate.
d) What are the implementation issues when dealing with non-monotonic reasoning ?
e) What is CYC ? What is it based on ?
6. Attempt any one :

10

a) What is natural deduction ? Illustrate the procedure used.


b) What are Semantic Networks ? How are they built ?
7. Attempt any one :

10

a) What is matching process ? Elaborate on the methods used for matching.


b) Compare between Predicate and Propositional Logic.
_____________________

Set A

SLR-BB 278

*SLRBB278*
S

B.E. (Information Technology) (Part I) Examination, 2014


ADVANCED COMPUTER ARCHITECTURE
Day and Date : Tuesday, 2-12-2014
Time : 3.00 p.m. to 6.00 p.m.

Max. Marks : 100

Instructions : 1) Q. No. 1 is compulsory. It should be solved in first 30 minutes in


Answer Book Page No. 3.
2) Answer MCQ/Objective type questions on Page No. 3 only. Dont
forget to mention, Q.P. Set (A/B/C/D) on Top of Page.
Duration : 30 Minutes

MCQ/Objective Type Questions

Marks : 20

1. Choose the correct answer :


1) MIP is
a) Multiple Instruction Per Second
c) Million Instruction Processor

b) Multiple Instruction Processor


d) Million Instruction Per Second

2) In a load-store architecture the effective address and _________ can be


combined into a single clock cycle.
a) execution cycles
b) fetching cycle
c) write back
d) none

3) Consider the unpipelined processor in the previous section. Assume that it has a 1 ns
clock cycle and that it uses 4 cycles for ALU operations and branches and 5 cycles for
memory operations. Assume that the relative frequencies of these operations are 40%,
20% and 40% respectively. Suppose that due to clock skew and setup, pipelining the
processor adds 0.2 ns of overhead to the clock. Find Average instruction execution time
a) 4.1 ns
b) 4.4 ns
c) 5 ns
d) none
4) ___________ arise when an instruction depends on the results of a previous instruction
in a way that is exposed by the overlapping of instructions in the pipeline.
a) Structural hazards b) Data hazards
c) Control hazards d) None
5) Pipeline branch hazards can be handled by
a) delayed branch
c) both

b) forwarding branch
d) none

1
1

6) The limitations on delayed-branch scheduling arise from


a) the restrictions on the instructions that are scheduled into the delay slots
b) our ability to predict at compile time whether a branch is likely to be taken or not
c) both
d) none

7) Tamasulo scheme operates on


a) Renaming
c) common data bus

b) Reservation stations
d) all of the above

P.T.O.

SLR-BB 278

*SLRBB278*

-2-

8) Software pipelining
a) rearranging loops

b) replicating loop

9) Vector processor includes


a) vector register
c) vector load and store

c) both

d) none

b) vector function unit


d) all of the above

1
1

10) Loosely coupled multiprocessor are efficient when


1
a) interaction between task are minimal
b) interaction between task are maximum
c) both
d) none
11) Arbiter switch in multiprocessor responsible for
1
a) one of the simultaneous request
b) communication by exchanging message
c) both
d) none
12) Kmap processor responsible for
a) routing data
b) mapping address

c) both

13) PMIN, ISIN, IOPIN units consists in


a) Loosely coupled multiprocessor
c) Both

b) Tightly coupled multiprocessor


d) None

d) none

1
1

14) In multiprocessor ISIN is used for


a) interconnection between processor and memory
b) interconnection between processor
c) each processor to direct interrupt to any processor
d) none

15) Load balancing characteristics in multiprocessor is handle by


a) last recently used b) daisy chain
c) rotating daisy chain

d) all above

16) In data flow machine tagged data tokens are used for
a) static data flow
b) dynamic data flow c) both

d) none

17) The concepts of control flow and data flow computing is based on
a) control of algorithmic sequences
b) control of computation sequences
c) control of logical sequences
d) control of turing sequences

18)

in data this symbol is used for

a) merger

b) decider

c) boolean link

d) data link

19) Dead lock detection algorithm uses the information contents in a __________ of the
system to decide whether or not a dead lock exists.
a) path
b) link
c) state
d) output
______________

Set A

*SLRBB278*
S

-3-

SLR-BB 278

B.E. (Information Technology) (Part I) Examination, 2014


ADVANCED COMPUTER ARCHITECTURE
Day and Date : Tuesday, 2-12-2014

Marks : 80

Time : 3.00 p.m. to 6.00 p.m.

SECTION I
2. Answer any two of the following.

14

a) How to overcome Data hazard with dynamic scheduling ?


b) What is loop unrolling explain with example ?
c) How RAW hazard can remove in ILP ?
3. Answer any two of the following.

14

a) What is VLIW, explain in detail ?


b) How compiler support for ILP ?
c) What is 2 bit Dynamic branch prediction ?
4. Answer the following.

12

a) Vector stride
b) Vector components.
SECTION II
5. Answer any two of the following.

14

a) Compare loosely coupled and tightly coupled multiprocessors.


b) Compare multiprocessor with time shared bus with multiport memory.
c) Draw and explain an 23 23 delta network.
Set A

SLR-BB 278

-4-

*SLRBB278*

6. Attempt any two of the following.

14

a) Explain the concept of dataflow computing and also state potential problems.
b) Explain static dataflow architectures.
c) Explain ring structure data flow.
7. Write short note :

12

a) Multiprocessor synchronization
b) Distributed shared memory multiprocessor architecture.
_____________________

Set A

SLR-BB 279

*SLRBB279*
Seat
No.

Set

B.E. (Information Technology) (Part I) Examination, 2014


MANAGEMENT INFORMATION SYSTEM
Day and Date : Thursday, 4-12-2014
Time : 3.00 p.m. to 6.00 p.m.

Max. Marks : 100

Instructions : 1) Q. No. 1 is compulsory. It should be solved in first 30 minutes


in Answer book Page No. 3. Each question carries one mark.
2) Answer MCQ/Objective type questions on Page No. 3 only.
Dont forget to mention, Q.P. Set (A/B/C/D) on Top of Page.
MCQ/Objective Type Questions
Duration : 30 Minutes

Marks : 20

1. Choose the correct alternative :

20

1) A ____________ is one in which nearly all the organizations significant business


relationships with customers, suppliers and employees are digitally enabled.
A) Digital firm
B) Digitization
C) Digital organization
D) None of these
2) ____________ supports non-routine decision making for middle management.
A) DSS
B) ESS
C) OSS
D) None of these
3) ____________ can be defined technically as a set of interrelated components
that collect, process, store and distribute information.
A) Information System
B) DBMS
C) Database
D) Digital data
4) BPR stands for____________
A) Business Product Restructure
C) Business Process Revaluation

B) Business Process Reengineering


D) None of these

5) According to ____________ , the firm is viewed as a nexus of contracts among


self-interested individuals rather than as a unified, profit-maximizing entity.
A) Transaction Cost Theory
B) Agency Theory
C) Collaboration
D) None of these
6) A/An____________ is a stable, formal social structure that takes resources from
the environment and processes them to produce outputs.
A) Organization
B) Institute
C) Business
D) Infrastructure
7) ____________ involves comparing the efficiency and effectiveness of your
business against standards and measuring performance against those standards.
A) Bench marking
B) Reengineering
C) Quality control
D) Quality measure
8) A____________ grants the owner an exclusive monopoly on the ideas behind an
invention for 20 years.
A) Patent
B) Copyright
C) Both A) and B) D) None of these
P.T.O.

SLR-BB 279

-2-

*SLRBB279*

9) ____________installs itself secretly on an internet users computer by piggy


backing on larger applications.
A) Software
B) Spyware
C) Adware
D) Middleware
10) ____________involves connecting geographically remote computers into a single
network to create a virtual super computer.
A) Grid computing
B) Utility computing
C) Geo computing
D) Autonomic computing
11) ____________consists of a set of physical devices and software applications that
are required to operate the enterprise.
A) IT infrastructure
B) IT business
C) Information technology
D) All of these
12) ____________ is a software that connects two otherwise separate applications,
enabling them to communicate with each other.
A) Firmware
B) Multiware
C) Middleware
D) ERP
13) A ____________ is a database that stores current and historical data of
potential interest to decision makers throughout the company.
A) DBMS
B) Data mine
C) Data warehouse
D) Data mart
14) ____________ refers to firms off-loading peak demand for computing power to
remote, large scale data processing center.
A) On-demand computing
B) Grid computing
C) Cloud computing
D) Edge computing
15) An/A____________ is a box consisting of a radio receiver/transmitter and antennas
that links to a wired network, routes or hub.
A) Ariel
B) Access point C) Backbone
D) Hotpsot
16) ____________involves setting up fake websites on sending e-mails that look like
those of legimate business to ask user for confidential personal data.
A) Phishing
B) Spoofing
C) Sniffing
D) Identify theft
17) A software vendor created small pieces of software called____________ to repair
the flows without disturbing the proper operation of the software.
A) Virus
B) Worm
C) Operating system
D) Patches
18) A ____________ is one that is stored in more than one physical location.
A) Distributed database
B) Parallel database
C) Data mart
D) None of these
19) ____________involves retailing products and services to individual shopper.
A) B2B
B) B2C
C) C2C
D) All of these
20) ____________ uses online alalytical processing and data mining to analyze large
pools of date.
A) DSS
B) ESS
C) OSS
D) FSS

______________
Set A

*SLRBB279*

-3-

SLR-BB 279

Seat
No.

B.E. (Information Technology) (Part I) Examination, 2014


MANAGEMENT INFORMATION SYSTEM
ay and Date : Thursday, 4-12-2014

Marks : 80

Time : 3.00 p.m. to 6.00 p.m.


Instruction : All questions are compulsory.
SECTION I
2. Attempt any four of the following :

(64=24)

a) Explain the globalization challenges and opportunities.


b) What are the contemporary approaches to information systems ?
c) Explain the organizational and behavioural impacts.
d) Explain the model for thinking about ethical, social and political issues.
e) Write a short note on data mining.

3. Attempt any two of the following :

(82=16)

a) Discuss the Porters competitive forces model.


b) Explain the quality of life in terms of equity, access and boundaries.
c) Write a short note on wireless computer networks and internet access.

Set A

SLR-BB 279

-4-

*SLRBB279*

SECTION II
4. Attempt any four of the following :

(64=24)

a) What are Enterprise Systems ? Explain the business value of enterprise systems.
b) Explain the Supply Chain Management Systems with neat diagram.
c) Explain the requirement of the decision-making process in the organization.
d) Explain the systems development and organizational change.
e) What are the challenges and obstacles to global business systems ?
5. Attempt any two of the following :

(82=16)

a) Explain the M-commerce services and applications.


b) Explain the types of knowledge management systems.
c) Explain the importance of project management.

_____________________

Set A

SLR-BB 28

*SLR-BB-28*
Seat
No.

Set

S.E. (Civil) (Part II) (Old) Examination, 2014


FLUID MECHANICS II
Day and Date : Friday, 28-11-2014
Time : 3.00 p.m. to 6.00 p.m.
Instructions :

Max. Marks : 100

1) Q. 2 and Q. 6 are compulsory. Solve any two questions from


remaining Section.
2) Q. No. 1 is compulsory. It should be solved in first 30 minutes in
Answer Book Page No. 3. . Each question carries one mark.
3) Answer MCQ/Objective type questions on Page No. 3 only. Dont
forget to mention, Q.P. Set (A/B/C/D) on Top of Page.
MCQ/Objective Type Questions

Duration : 30 Minutes

Marks : 20

1. Choose the correct option :


1) Which of the following is the most essential condition for a hydraulic jump to form ?
a) The constancy of specific energy
b) The existence of subcritical flow before the jump
c) The existence of supercritical flow before the jump
d) None of the above
, in the case of uniform flow in the channel, is equal to
2) The water surface dy
dx
a) When the channel slope is equal to the normal slope
b) When the pressure force and the change of momentum are different from each other
c) When the force causing the force is not equal to the resistance force
d) When there is equilibrium between the force causing the flow and those opposing it
3) In open channel, the alternate depths of flow are the depths
a) Which occur at the same specific energy
b) At which total energies are same
c) For the same specific force
d) None of the above
4) The critical depth is the depth of flow at which
a) The Froude number is less than unity
b) The specific energy is minimum
c) The specific energy is maximum
d) The unit discharge is minimum
5) In open channels, the specific energy is
a) The total energy unit discharge
b) The total energy measured above is a horizontal datum
c) The total energy measured with respect to the channel bottom which is taken as
datum
d) The kinetic energy plotted above the free-surface
P.T.O.

SLR-BB 28

*SLR-BB-28*

-2-

6) The discharge over a rectangular weir, considering velocity of approach, is given by


a) Q = 2 CdL 2gH3 / 2
b) Q = 2 C dL 2g [(H + Ha )3 / 2 (Ha )3 / 2 ]
3
3
1
1
2
c) Q = C dL. 2g [(H + Ha )2 (Ha )2 ]
d) None of these
3
7) Franciss formula for a rectangular weir with n end contractions is given by
3
2
a) Q = C d . (L 0.1 nH) 2g . H2
3

3
b) Q = 3 C d . (L 0.1 nH) 2g . H2
2

c) Q = Cd . (L 0.1 nH) 2g . H32

d) none of these

8) Maximum discharge over a broad-crested weir is given by


a) Q = Cd L . H32

b) Q = 0.5 Cd L . H52

c) Q = 1.705 Cd L . H32 d) None of these

9) Which of the following turbines is suitable for specific speed ranging from 300 to 1000 and
head below 30 m;
a) Francis
b) Kaplan
c) Propeller
d) Pelton
10) Specific speed of an impulse turbine (Pelton wheel) ranges from
a) 12 to 70
b) 80 to 400
c) 300 to 1000
d) 1000 to 1200
11) Cavitation can take place in case of
a) Pelton Wheel
b) Francis Turbine c) Centrifugal Pump d) Both b) and c)
12) A Kaplan turbine is
a) An inward flow impulse turbine
b) Low head axial flow turbine
c) High head axial flow turbine
d) High head mixed flow turbine
13) The specific speed of a pump is defined as the speed of unit of such a size that it
a) Produces unit power with unit head available
b) Delivers unit discharge unit head
c) Requires unit power to develop unit head
d) Delivers unit discharge at unit power
14) Which of the following statement pertaining to a given centrifugal pump is correct ?
a) Discharge varies as the square of speed b) Power varies as the square of speed
c) Discharge varies directly as speed
d) Head varies inversely as speed
15) Which of the following is not a dimensionless parameter ?
a) Friction factor
b) Specific speed
c) Thomas cavitation parameter
d) Pressure co-efficient
16) In centrifugal pumps, cavitation is reduced by
a) Increasing the floe velocity
b) Reducing the discharge
c) Throttling the discharge
d) Reducing the suction head
17) In a centrifugal pump the sum of suction head and delivery head is known as
a) Manometer
b) Total head
c) Static head
d) None of the above
18) Dynamic similarity between the model and prototype is the
a) Similarity of motion b) Similarity of lengths c) Similarity of forces d) None of the above
19) ___________ is equal to the product of shear stress due to viscosity and surface area of
flow.
a) Viscous force
b) Inertia force
c) Pressure force
d) Gravity force
20) ___________ is the square root of the ratio of the inertia force to the pressure force.
a) Froudes number b) Reynolds number c) Eulers number
d) Machs number
______________
Set A

*SLR-BB-28*

-3-

SLR-BB 28

Seat
No.

S.E. (Civil) (Part II) (Old) Examination, 2014


FLUID MECHANICS II
Day and Date : Friday, 28-11-2014
Time : 3.00 p.m. to 6.00 p.m.

Marks : 80

Instruction : Q. 2 and Q. 6 are compulsory. Solve any two questions from


remaining Section.
SECTION I
2. a) Derive the expression for Gradually Varied Flow (GVF) in open channel and state the
assumptions in deriving the equation.

b) Water issues out under a sluice gale, the opening of which is 250 mm in a rectangular
channel, through which a discharge of 1-1 m3/sec/m is passing if cc (coefficient of
contraction) is 0.6, determine, if a hydraulic jump is possible if yes. Calculate the loss of
energy head.

3. a) Derive the expression for theoretical discharge through triangular notch, draw neat figure.

b) A wide rectangular channel carries a discharge of 5m3/s/m width of channel with a bed
slope of 1 in 3600 and n = 0.02. If the depth at the section is 3.5 m, determine how far
upstream or down stream of the section, the depth of flow will be within 5% of the normal
depth. Use step method and take two steps.

4. a) Derive the expression for time required to empty the tank by rectangular notch.

b) A broad crested heir 50 m long has 50 cm height of water above crest. Find the maximum
discharge, take cd = 0.62 and neglect velocity of approach, also if the velocity of approach
is taken into consideration find maximum discharge when the channel has cross sectional
area of 50 m2 on upstream side.
5. Attempt any three :

6
12

a) Sketch M2 and S1 type of water surface profile.


b) Classify hydraulic jump with froudes number.
c) Find the time required to lower the water level from 3m to 2m in a reservoir of dimension
80 m 80 m ; by a rectangular notch of length 2m Take cd = 0.62.
d) The head of water over a triangular notch of angle 60 is 50 cm and cd = 0.62. The flow is
measured within accuracy of 1.5% up or down. Find the limiting values of the head.

Set A

SLR-BB 28

-4-

*SLR-BB-28*

SECTION II
6. a) State Buckingham theorem and explain procedure for determining functional relationship. 6
b) Derive an expression for the force exerted by the jet on a stationary inclined flat plate.

c) Differentiate between centrifugal pump and reciprocating pump.

7. a) Draw a neat sketch of Pelton wheel turbine and name at each part.

b) A Kaplan turbine develops 24647.6 kW power at an average head of 39 m. Assuming a


speed ratio of 2, flow ratio of 0.6, diameter of the boss equal to 0.35 times the diameter of
the runner and an overall efficiency of 90%.
Calculate the diameter, speed and specific speed of the turbine.
8. a) Draw neat sketch of centrifugal pump and state the advantages of centrifugal pump.

b) The resistance force F is a function of its length L, velocity V, acceleration due to gravity g,
and fluid properties like and viscosity . Write the relationship in a dimensionless form.
Use Buckingham Method.
9. Solve any three :

7
12

a) Derive Reynolds no. and Froude no.


b) Explain similitude in detail.
c) Define surge tank and explain its working with figure.
d) Define draft tube and explain its function
e) Common pump troubles and remedies.
_____________________

Set A

SLR-BB 280

*SLRBB280*
Seat
No.

Set

B.E. (Information Technology) (Part I) Examination, 2014


ADVANCED DATABASE SYSTEMS
Day and Date : Saturday, 6-12-2014
Time : 3.00 p.m. to 6.00 p.m.

Total Marks : 100

Instructions : 1) All questions are compulsory.


2) Figures to the right indicate marks to a question.
3) Q. No. 1 is compulsory. It should be solved in first 30 minutes
in Answer book Page No. 3. Each question carries one mark.
4) Answer MCQ/Objective type questions on Page No. 3 only.
Dont forget to mention, Q.P. Set (A/B/C/D) on Top of Page.
Duration : 30 Minutes

MCQ/Objective Type Questions

Marks : 20

1. Choose correct alternative(s) :


20
1) Machines with fine-granularity parallelism have ______________
a) Few no of processors
b) Large no of processors
c) Single task, single processor
d) All of the above
2) All of the above Execution of single query on multiple processor is _________
a) Interopertion parallelism
b) Intraopertion parallelism
c) Intraquery parallelism
d) Interquery parallelism
3) In distributed database, transaction manager is responsible for ______________
a) starting the execution of transaction
b) breaking transaction into number of sub-transactions
c) coordinating termination of transaction
d) maintaining log for recovery purpose
4) ______________ objects vanish after program termination.
a) Transient
b) Persistent
c) Both
d) None of the above
5) The operation of moving from coarser granularity data to a finer granularity
is called ______________
a) rollup
b) drilldown
c) slicing
d) dicing
6) With n relations, there are ______________ different join orderings.
c) 2*(n 1)!/n!
d) 2*(n 1)!/(n 1)!
a) n
b) n2
7) Administrator can tune database system at three levels. Middle level is ________
a) Hardware
b) Buffer size and check pointing interval
c) Schema and transactions d) None of the above
P.T.O.

SLR-BB 280

-2-

*SLRBB280*

8) The _____ benchmark prohibits materialised views and other redundant information.
a) TCP-C
b) TCP-D
c) TCP-H
d) TCP-W
9) __________ standards are dominant products which are generally accepted
as standards without any formal process of recognition.
a) Formal
b) De facto
c) Anticipatory
d) Reactionary
10) Semijoin strategy is useful when to relations are present on the same site.
Statement is ______________
a) True
b) False
11)
a)
12) A heterogeneous distributed database is which of the following ?
a) The same DBMS is used at each location and data are not distributed
across all nodes
b) The same DBMS is used at each location and data are distributed
across all nodes
c) A different DBMS is used at each location and data are not distributed
across all nodes
d) A different DBMS is used at each location and data are distributed
across all nodes
13) Data Warehouse is about taking or collecting data from different ________ sources.
a) Harmonized
b) Identical
c) Homogeneous d) Heterogeneous
14) SQL query with location transparency needs to specify ______________
a) Fragments
b) Locations
c) Local formats d) All
15) The cost of query evaluation can be measured in terms of ______________
a) Number of different resources
b) Disk Access
c) CPU time
d) All
16 20. Match correctly :

1) SQL/CLI
2) SQL/PSM

a) Overview of standards
b) Application program interface

3) SQL/OLB

c) Types, Schemas, Tables

4) SQL/Framework
5) SQL/Foundation
a) 1-e, 2-b, 3-c, 4-a, 5-d
c) 1-b, 2-e, 3-d, 4-c, 5-a

d) Embedding SQL in Java


e) Extensions to SQL
b) 1-b, 2-e, 3-d, 4-a, 5-c
d) 1-e, 2-b, 3-d, 4-c, 5-a
______________
Set A

*SLRBB280*

-3-

SLR-BB 280

Seat
No.

B.E. (Information Technology) (Part I) Examination, 2014


ADVANCED DATABASE SYSTEMS
Day and Date : Saturday, 6-12-2014
Time : 3.00 p.m. to 6.00 p.m.

Marks : 80

Instructions : 1) All questions are compulsory.


2) Figures to the right indicate marks to a question.
SECTION I
2. Attempt any four :

20

1) Explain bully algorithm.


2) Explain working of time stamp protocol in distributed database.
3) Explain interoperation parallelism with example.
4) Explain transaction servers and data servers.
5) Write a short note on persistent programming language.
6) Explain the working of three phase commit protocol.
3. Attempt any one:

10

1) Explain structured type, Type inheritance, Array and Multiset Types in SQL
with example.
2) Consider a relation that is fragmented horizontally by plant_number with no
replica and stored at two sites New York and Sanfrancisco :
Employee (name, address, salary, plant_number).
There are only two values for plant_number. Find how the following queries
are executed in fragmentation transparency ?
1) Find *from employee.
2) Find low paid employee in the company.
4. Explain Fragment and Replicate join with neat diagram.

10
Set A

SLR-BB 280

-4-

*SLRBB280*

SECTION II
5. Attempt any four :

20

1) Explain A5 algorithm for selection operation.


2) Explain Process-per-client model of TP monitor with its disadvantages.
3) How hardware is tuned in performance tuning ?
4) Write a short note on Main memory databases.
5) Explain star schema and snowflake schema.
6) What is OLAP ? Explain measure attributes, dimension attributes and
multidimensional data.
6. Attempt any one :

10

1) Consider following relation scheme


Instructor (id, name dept_name, salary)
Teaches (id, course_id, sec_id, semester, year)
Course (course_id, title, dept_name, credits)
Find the equivalent expression tree and transformed expression tree of the
following query Find the names of all instructors in the Music department
together with course title of all courses that the instructors teach.
2) Discuss the issues to be addressed in building data warehouse.
7. List and explain are key properties of long duration transaction.

10

_____________________

Set A

SLR-BB 281

*SLRBB281*
Seat
No.

Set

B.E. (I.T.) (Part I) Examination, 2014


OBJECT ORIENTED MODELLING AND DESIGN (Elective I)
Day and Date : Tuesday, 9-12-2014
Time : 3.00 p.m. to 6.00 p.m.
Instructions :

Max. Marks : 100

1) Q. No. 1 is compulsory. It should be solved in first 30 minutes in


Answer Book Page No. 3. Each question carries one mark.
2) Answer MCQ/Objective type questions on Page No. 3 only. Dont
forget to mention, Q.P. Set (A/B/C/D) on Top of Page.
3) Figures drawn by pencil, ruler only indicate full marks.
4) Do not use pen to draw and label the diagrams.
MCQ/Objective Type Questions

Duration : 30 Minutes

Marks : 20

1. Choose the correct answer :

20

1) The __________ model describes the aspects of a system that change over time.
a) Static
b) OMT
c) Dynamic
d) Functional
2) The link attribute is a property of the links in an _________
a) association
b) binary association
c) ternary association
d) n-ary association
3) Which of the following statement is true concerning objects and/or classes ?
a) An object is an instance of a class
b) A class is an instance of an object
c) An object includes encapsulates only data
d) A class includes encapsulates only data
4) In an activity diagram, when an activity state is completed, processing moves to another
state________________ are used to mark this movement.
a) Dashed Arrow
b) Links
c) Transition
d) All of the above
5) In UML, structural and architectural diagram does not include which of the following ?
a) Deployment diagram, component diagram b) Object diagram, class diagram
c) Use case diagram
d) None
6) The deployment diagram shows
a) Objects of a system
b) Distribution of components on the nodes in a system
c) Functions of a system
d) Distribution of nodes
7) An object can have which of the following multiplicities ?
a) zero
b) one
c) more than one

d) all of the above

8) What is the relationship between these two use cases ?

a) include

b) extension

c) generalization

d) association
P.T.O.

SLR-BB 281

*SLRBB281*

-2-

9) In order to model the relationship a course is composed of 5 to 20 students and one or


more instructors, you could use
a) Aggregation
b) Association
c) Composition
d) Realization
10) Use cases are used in the ___________ phase of software development to articulate the
high-level requirements of the system.
a) analysis
b) design
c) documentation
d) development
11) Unified Process is a software development methodology which is
a) Component-driven
b) Iterative and incremental
c) Related to Extreme Programming
d) Done in only one iteration
12) An abstract class is which of the following ?
a) A class that has direct instances, but whose descendants may have direct instances
b) A class that has no direct instances, but whose descendants may have direct instances
c) A class that has direct instances, but whose descendants may not have direct
instances
d) A class that has no direct instances, but whose descendants may not have direct
instances
13) The fact that the same operation may apply to two or more classes is called what ?
a) Inheritance
b) Polymorphism
c) Encapsulation
d) Multiple classification
14) The activity diagram
a) Models the interaction between objects
b) Models the external events stimulating one object
c) Focuses on the transitions between states of a particular object
d) Focuses on flows driven by internal processing
15) An interface is
a) A set of classes used on a collaboration
b) A set of operations used to specify a service of a class or component
c) A set of attributes used on an operation
d) A set of objects used to provide a specific behaviour
16) A _____________ is a sequence of events that occurs during one particular execution of
a system.
a) use case
b) scenario
c) both a) and b)
d) activity
17) Composition is a stronger form of which of the following ?
a) Aggregation
b) Encapsulation
c) Inheritance

d) All of the above

18) Which of the following is a technique for hiding the internal implementation details of an
object ?
a) Encapsulation
b) Polymorphism c) Inheritance
d) All of the above
19) The __________ notation is useful for graphically depicting an object-oriented analysis or
design model.
a) UML
b) OMT
c) Both a) and b)
d) DFD
20) A state chart diagram describes
a) Attributes of objects
c) Operations executed on a thread

b) Nodes of the system


d) Events triggered by an object

______________
Set A

*SLRBB281*

-3-

SLR-BB 281

Seat
No.

B.E. (I.T.) (Part I) Examination, 2014


OBJECT ORIENTED MODELLING AND DESIGN (Elective I)
Day and Date : Tuesday, 9-12-2014
Time : 3.00 p.m. to 6.00 p.m.
Instructions :

Marks : 80

1) All questions are compulsory.


2) Figures drawn by pencil, ruler only indicate full marks.
3) Do not use pen to draw and label the diagrams.
SECTION I

2. Attempt any four :

(45=20)

a) Prepare a list of objects that you would expect each of the following systems to handle :
i) A program for laying out a newspaper
ii) A program to compute and store bowling scores
iii) A telephone answering machine
iv) A controller for a video cassette recorder
v) A catalog store order entry system.
b) Explain the three views of modeling systems.
c) Explain features of object oriented languages.
d) Explain data flow diagram with example.
e) Explain Link attribute with notation.
3. Draw any one :

10

Prepare an instance diagram for an imaginary round trip you took last weekend to London.
Include at least one instance of each object class. Fortunately, direct flights on a hypersonic
plane were available. A friend of yours went with you but decided to stay a while and is still
there. Captain Johnson was your pilot on both flights you had different seats each way but
you noticed it was on the same plane because of a distinctive dent in the tail section.
OR
Prepare a class diagram for a school using below mentioned classes showing at least 10
relationships among the following classes. Include association, aggregation and generalization.
Use qualified associations and show multiplicity balls in your diagrams. Use association names
where needed. As you prepare the diagrams you may add additional object classes. [School,
playground, principal, schoolboard, classroom, book, student, teacher, cafeteria, rest room,
computer, desk, chair, ruler, door, swing.]
4. Draw a functional model of Flight Simulator.

10

Set A

SLR-BB 281

-4-

*SLRBB281*

SECTION II
5. Attempt any four :

(45=20)

a) What are the different common mechanisms of UML ?


b) Give UML notations for the following :
i) Generalization
ii) Association
iii) Aggregation
iv) Dependencies.
c) Explain aggregation and composition with example.
d) What is a sequence diagram. Draw the graphical notations used in sequence diagram.
i) Object
ii) Actor
iii) Lifeline
iv) Activation
v) Messages.
e) Draw class diagram for software written to control a coffee vending machine.
6. Draw any one :

10

Draw a collaboration diagram for a scenario of Conduct Interviews in an IT industry.


OR
Draw Use case diagram for a Software Project Management Systems. The designed software
helps in software project planning, estimation, project monitoring and project reporting. Consider
key/major activities under each of above subsystems.
7. Explain Development diagram with its graphical notations. Also draw a labelled deployment
diagram showing the relationships among software and hardware components involved in
real estate transactions using the mentioned nodes :
[Bank Server, Real Estate server, a PC] with connectors and interface.
10
_____________________

Set A

SLR-BB 282

*SLRBB282*
Seat
No.

Set

B.E. (Information Technology) (Part II) Examination, 2014


INFORMATION RETRIEVAL
Day and Date : Tuesday, 25-11-2014
Time : 3.00 p.m. to 6.00 p.m.

Max. Marks : 100

Instructions : 1) Q. No. 1 is compulsory. It should be solved in first 30


minutes in Answer book Page No. 3. Each question carries
one mark.
2) Figures to the right indicate marks to a question.
3) Answer MCQ/Objective type questions on Page No. 3
only. Dont forget to mention, Q.P. Set (A/B/C/D) on Top
of Page.
MCQ/Objective Type Questions
Duration : 30 Minutes

Marks : 20

1. A) Choose correct alternative(s) :

10

1) Information retrieval deals with __________ of information.


a) Representation

b) Storage

c) Organization

d) All of the above

2) One needs to determine what are the features which better describe the
objects in the set, This set of features provides quantification of _______
a) Intracluster similarity

b) Intercluster dissimilarity

c) Intercluster similarity

d) None of the above

3) In automation of approximate string matching algorithm, vertical arrow


represents __________ of a character.
a) Match

b) Insertion

c) Deletion

d) Replacement

4) In __________ retrieval, object might be inaccurate.


a) Information

b) Data

c) Both a) and b)

d) None of the above


P.T.O.

SLR-BB 282

*SLRBB282*

-2-

5) ___________ increase user base scalability in Harvest architecture.


a) Gatherer

b) Broker

c) Object cache
d) Replication manager
6) Data blades are supported by __________
a) ORACLE

b) Illustra

c) Sybase SQL

d) SQL3

7) In MULTOs model, ___________ phase deals with image interpretation.


a) Low level analysis

b) High level analysis

8) is contained in is a example of ___________ predicate.


a) Semantic
b) Temporal
c) Spatial
d) Attribute
9) ___________ theorem states that DFT preserves the energy of a signal as
well as distances between two signals.
a) Zipfs

b) Parsevals

c) Heaps

d) None of the above

10) ___________ is a meta search engine.


a) Google

b) Ask

c) MSN

d) All of them

B) Match correctly :

1) Webdirectory

a) SavvySearch

2) Combining browsing with searching

b) FLORID

3) Web Query Language

c) Yahoo

4) Metasearcher

d) Alexa

5) Web Navigation Service

e) WebGlimpse

C) State whether the following statement is true or false :

1) Data retrieval system deals with data that has a well defined structure
and semantics
2) KMP algorithm is based on bit parallelism.
3) Google uses Pagerank algorithm for ranking.
4) Use of Spatial Access method achieves faster result than sequential
searching.
5) Boolean model allows partial matching.
______________
Set A

*SLRBB282*

SLR-BB 282

-3-

Seat
No.

B.E. (Information Technology) (Part II) Examination, 2014


INFORMATION RETRIEVAL
Day and Date : Tuesday, 25-11-2014
Time : 3.00 p.m. to 6.00 p.m.

Marks : 80

Instruction : Figures to the right indicate marks to a question.


SECTION I
2. Attempt any four :

20

1) What are the different evaluation measures of retrieval ?


2) Show how pattern day will be in the text Saturday using brute force approach.
3) How to find weights of index terms in vector model ?
4) Explain Hytime Architecture.
5) Explain context queries with suitable example.
6) State the differences in information retrieval and data retrieval.
3. Attempt any one :

10

1) Plot graph of precision vs recall.


Ranking for query : * indicates relevant document
1) d588*

2) d589

3) d576

4) d590*

5) d986

6) d592*

7) d984

8) d988*

9) d578

10) d985

11) d103

13) d772*

14) d990

12) d591

Set A

SLR-BB 282

*SLRBB282*

-4-

2) If bit masks are as follows then find signature for following text which is
divided in four blocks and show how word database is searched in it ?
h(Data)

= 000101

h(database)

= 110000

h(management)

= 1000100

h(system)

= 001100

h(Oracle)

= 100001

This is database

Data is stored in database

Oracle is example of database

Management system.

4. Explain different strategies for finding precision as single value summary.

10

SECTION II
5. Attempt any four :

20

1) Which different documents are considered in digital library ?


2) Explain working of Webglimpse.
3) How multimedia data is supported in commercial database ?
4) Give the types of query predicate with example.
5) Explain different types of multimedia queries.
6) Explain how multimedia data are represented inside the system.
6. Attempt any one :

10

1) Explain architectural issues in digital library.


2) How GEMINI approach is applied to two dimensional color images within
QBIC ?
7. What do you mean by ranking ? Explain different ranking algorithms.

10

_____________________

Set A

SLR-BB 283

*SLRBB283*
S

B.E. (Part II) (Information Technology) Examination, 2014


MOBILE COMPUTING
Day and Date : Thursday, 27-11-2014
Time : 3.00 p.m. to 6.00 p.m.

Max. Marks : 100

N. B. : 1) Assume any suitable data.


2) Q. No. 1 is compulsory. It should be solved in first 30 minutes
in Answer book Page No. 3. Each question carries one mark.
3) Answer MCQ/Objective type questions on Page No. 3 only.
Dont forget to mention, Q.P. Set (A/B/C/D) on Top of Page.
Duration : 30 Minutes

MCQ/Objective Type Questions

Marks : 20

1. Multiple choice questions :


(201=20)
1) The space between the interference range is called
a) Frequency division
b) Space
c) Guard space
d) All of the above
2) The two binary values 1 and 0 are represented by two different amplitudes in
case of
a) PSK
b) TDM
c) ASK
d) FSK
3) Example of explicit reservation is
a) PRMA
b) DAMA
c) Reservation TDMA
d) CSMA
4) __________ is centralized scheme with one master of several slaves.
a) Polling
b) Slotted Aloha c) PRMA
d) None of these
5) Spread spectrum is resistant to ___________ interference.
a) Narrow band
b) Broad band
c) Wide band
d) All of the above
6) ___________ CSMA station senses the carrier and start sending immediately
if the medium is idle.
a) I-persistent CSMA
b) Non-persistent CSMA
c) P-persistent CSMA
d) O-persistent CSMA
7) ___________ range is used for communication under water.
a) Infrared (IR)
b) Low Frequency (LF)
c) Medium Frequency (MF)
d) Super High Frequency (SHF)
8) Number portability call forwarding three way calling are features of _________
a) CDMA
b) SS7
c) Both
d) None

P.T.O.

SLR-BB 283

-2-

*SLRBB283*

9) The ___________ has additional connections to other fixed networks, such


as PSTN and ISDN.
a) GMSC
b) MSC
c) BSC
d) BTS
10) The A3 algorithm in GSM is used for ___________
a) Encryption
b) Authentication
c) Generation of cipher key
d) All
11) ___________ is a collection of blue tooth devices which are synchronized to
same hopping sequence.
a) Piconet
b) Ad-hoc
c) Standby
d) None
12) ___________ supports the application of COA for mobile nodes, in wireless
communication.
a) BOOTP
b) Network
c) DHCP
d) None
13) The stations and access points which are within the same radio coverage
form a ___________
a) BSS
b) IBSS
c) ESS
d) None
14) DSSS is spread spectrum method separated by
a) Code
b) Frequency
c) Time

d) None

15) ___________ segments is combination of fixed part and wireless part.


a) Driect TCP
b) Indirect TCP c) Both
d) None
16) CN is network layer is ___________ which can be fix for mobile.
a) Common Network
b) Correspondent Node
c) Correspondent Network
d) None
17) ___________ avoid slow start after roaming.
a) Snooping TCP b) ITCP
c) MTCP

d) None of these

18) ___________ is broadcasted by client.


a) DHCPACK
b) DHCP DISCOVER
c) Both
d) None of the above
19) Foreign agents and home agents advertise their presence periodically using
special ___________ messages.
a) Agent advertisement
b) User advertisement
c) Case of address
d) None of these
20) To remain transparent in snooping TCP, the foreign agent must not
acknowledge data to the ___________
a) Mobile host
b) Correspondent host
c) Both
d) None of the above
______________

Set A

*SLRBB283*
S

-3-

SLR-BB 283

B.E. (Part II) (Information Technology) Examination, 2014


MOBILE COMPUTING
Day and Date : Thursday, 27-11-2014
Time : 3.00 p.m. to 6.00 p.m.

Marks : 80

N. B. : 1) All questions are compulsory.


2) Assume any suitable data.
SECTION I
1. a) Mention the applications of wireless communications and briefly explain each. 5
b) Explain in detail the system architecture of GSM.

15

OR
2. a) Explain with neat diagram the DSSS technique.
b) How is hidden and exposed terminal problem overcome by MACA ?

10
10

3. Explain with neat diagram the Frequency Shift Keying (FSK) modulation
technique.

10
OR

Explain security authentication and encryption in GSM.


4. Write short notes on any one :

10
10

a) Hand over in GSM.


b) Multipath propagation and inter symbol interference.
Set A

SLR-BB 283

-4-

*SLRBB283*

SECTION II
5. a) Describe the basic DHCP configuration.
b) Discuss MAC frame in WLAN IEEE 802.11 in detail.

10
10

OR
a) What are advantages and disadvantages of wireless LAN ?

b) Explain different entities and terminologies used in mobile network.

c) Explain transaction oriented TCP.

6. Explain agent discovery and advertisement in mobile network layer.

10

OR
Explain Tunnelling and encapsulation in mobile network layer.
7. Write short notes on any one.

10
10

a) HIPERLAN
b) PICONET.
_____________________

Set A

SLR-BB 284

*SLRBB284*
Seat
No.

Set

B.E. (I.T.) (Part II) Examination, 2014


NETWORK SECURITY
Day and Date : Saturday, 29-11-2014
Time : 3.00 p.m. to 6.00 p.m.

Total Marks : 100

Instructions : 1) Q. No. 1 is compulsory. It should be solved in first 30 minutes


in Answer Book Page No. 3. Each question carries one mark.
2) Answer MCQ/Objective type questions on Page No. 3 only.
Dont forget to mention, Q.P. Set (A/B/C/D) on Top of Page.
MCQ/Objective Type Questions
Duration : 30 Minutes

Marks : 20

1. 1) Programs that come into a computer system disguised as something else


are called
a) Spoofers
b) Loggers
c) Defacers
d) Trojan horses
2) Which of the following algorithms belong to symmetric encryption ?
a) 3DES (Triple DES) and IDEA
b) RSA
c) RC6
d) All
3) A(n) __________ function creates a message digest out of a message.
a) encryption
b) decryption
c) hash
d) none of the above
4) Digital signature cannot provide ___________ for the message.
a) Integrity
b) Confidentiality
c) Non repudation
d) Authentication
5) A(n) _________ is a hierarchical system that answers queries about key
certification.
a) KDC
b) PKI
c) CA
d) None of the above
6) __________ is the science and art of transforming messages to make them secure
and immune to attacks.
a) Cryptography
b) Crypto analysis
c) Either a) or b)
d) Neither a) nor b)
7) In an asymmetric-key cipher, the receiver uses the __________ key.
a) Private
b) Public
c) Either a) or b)
d) Neither a) nor b)
8) _________ ciphers can be categorized into two broad categories : monoalphabetic
and polyalphabetic.
a) Substitution
b) Transposition
c) Either a) or b)
d) Product composition
P.T.O.

SLR-BB 284

*SLRBB284*

-2-

9) The ___________ attack can endanger the security of the Diffie-Hellman method
if two parties are not authenticated to each other.
a) Man-in-the-middle
b) Ciphertext attack
c) Plaintext attack
d) None of the above
10) SSL provides ___________
a) Message integrity
c) Compression

b) Confidentiality
d) All of the above

11) One security protocol for the e-mail system is __________


a) IPSEc
b) SSL
c) PGP

d) None of the above

12) The __________ method provides a one-time session key for two parties.
a) Diffie-Hellman b) RSA
c) DES
d) AES
13) Kerberos is an encryption-based system that uses
a) Secret key encryption
b) Public key encryption
c) Both a) and d)
d) Data key encryption
14) By symmetric key encryption we mean
a) one private key is used for both encryption and decryption
b) private and public key used are symmetric
c) only public keys are used for encryption
d) only symmetric key is used for encryption
15) A firewall may be implemented in
a) routers which connect intranet to internet
b) bridges used in an intranet
c) expensive modem
d) users application programs
16) Message __________ means that the receiver is ensured that the message is
coming from the intended sender, not an imposter.
a) Confidentiality b) Integrity
c) Authentication d) None of the above
17) __________ was invented by Phil Zimmerman.
a) IPSec
b) SSL
c) PGP

d) None of the above

18) Which one of the following algorithm is not used in asymmetric-key cryptography ?
a) RSA algorithm
b) diffie-hellman algorithm
c) electronic code book algorithm
d) none of the mentioned
19) Hashed message is signed by a sender using
a) his public key
b) his private key
c) receivers public key
d) receivers private key
20) A computer virus is
a) software written with malicious intent to cause annoyance or damage
b) a process of scrambling a message so that it cannot be read until it is
unscrambled
c) a small device that scans the number off credit cards
d) an ID-checking tool used to monitor who is logged onto a network
______________
Set A

*SLRBB284*

-3-

SLR-BB 284

Seat
No.

B.E. (I.T.) (Part II) Examination, 2014


NETWORK SECURITY
Day and Date : Saturday, 29-11-2014
Time : 3.00 p.m. to 6.00 p.m.

Marks : 80

SECTION I
2. Write short notes on (any four) :

20

a) Network security model


b) Active attack and its types
c) MAC and requirements of MAC
d) CFB
e) Differential and linear cryptanalysis
f) Digital signature.
3. Explain Feistel Cipher structure in detail. Which parameters and design choices
determine the actual algorithm of a Feistel Cipher.

10

OR
Explain Diffie-Hellman key exchange algorithm with example.
4. What is DES Encryption ? Explain its general depiction with its strengths and
weakness.

10

Set A

SLR-BB 284

-4-

*SLRBB284*

SECTION II
5. Write short notes on (any four) :

20

a) Tunnel Mode Vs Transport Mode Encryption


b) RFC 822
c) Kerberos 4 Vs Kerberos 5
d) PGP services
e) Honey pots
f) Virus and four phases of virus.
6. What is X.509 authentication service ? Explain in detail X.509 certificate.

10

OR
What is password management ? Explain password protection and password selection
strategies in detail.
7. What is firewall ? State the characteristics of firewall and its types with diagram.

10

_____________________

Set A

SLR-BB 285

*SLRBB285*
S

B.E. (I.T.) (Part II) Examination, 2014


Elective II : SOFTWARE TESTING AND QUALITY ASSURANCE
Day and Date : Monday, 1-12-2014
Time : 3.00 p.m. to 6.00 p.m.

Max. Marks : 100

Instructions : 1) Figures to the right indicate full marks.


2) Q. No. 1 is compulsory. It should be solved in first 30 minutes in
Answer Book Page No. 3. Each question carries one mark.
3) Answer MCQ/Objective type questions on Page No. 3 only. Dont
forget to mention, Q.P. Set (A/B/C/D) on Top of Page.
Duration : 30 Minutes
1.

MCQ/Objective Type Questions

Marks : 20

1) Which term defines the process of project compliance with policies and
procedures ?
a) Quality control
b) Quality assurances
c) Quality audits
d) Quality control management
2) Which process does QA provide and umbrella for ?
a) Continuous process improvement
b) Quality assurance
c) Project management plan
d) Quality baseline
3) Which of the processes includes all activities of the performing organization that determines
policies and responsibilities of a project ?
a) Performance quality control
b) Project quality management
c) Performance plan management
d) Quality control management
4) Which of the following are negative possibilities if quality requirements are not met ?
a) Overworking team members and poor product quality
b) Customer complaints and late product
c) Poor quality
d) Termination
5) Which of these terms apply to identifying quality standards and how to satisfy them ?
a) Quality projections
b) Quality management
c) Quality planning
d) Quality overview
6) Which of the following definitions apply to the cost of quality ?
a) Left over product
b) Total cost of all project efforts
c) Customer satisfaction
d) Redoing entire project
7) A regression test
a) Will always be automated
b) Will help ensure unchanged areas of the software that have not been affected
c) Will help ensure changed areas of the software that have not been affected
d) Can only be run during user acceptance testing
P.T.O.

SLR-BB 285

-2-

*SLRBB285*

8) With thorough testing it is possible to remove all defects from a program prior to
delivery to the customer
a) True
b) False
9) Which of the following are characteristics of testable software ?
a) observability
b) simplicity
c) stability

d) all of the above

10) The testing technique that requires devising test cases to demonstrate that each program
function is operational is called
a) black-box testing
b) glass-box testing
c) grey-box testing
d) white-box testing
11) The testing technique that requires devising test cases to exercise the internal logic of a
software module is called
a) behavioral testing
b) black-box testing
c) grey-box testing
d) white-box testing
12) What types of errors are missed by black-box testing and can be uncovered by white-box
testing ?
a) behavioral errors
b) performance errors
c) typographical errors
d) both b) and d)
13) Program flow graphs are identical to program flowcharts
a) True
b) False
14) Condition testing is a control structure testing technique where the criteria used to design
test cases is that they
a) rely on basis path testing
b) exercise the logical conditions in a program module
c) select test paths based on the locations and uses of variables
d) focus on testing the validity of loop constructs
15) Which of the following is not a characteristic for testability ?
a) Operability
b) Observability
c) Simplicity

d) Robustness

16) Which one of the following views expresses the requirement of a system ?
a) Use case
b) Design
c) Process
d) Implementation
17) A mechanism which describes structural and behavioral features is known as
a) Class
b) Classifier
c) Object
d) Entity
18) Graph-based testing methods can only be used for object-oriented systems
a) True
b) False
19) Boundary value analysis can only be used to do white-box testing
a) True
b) False
20) Testing OO class operations is made more difficult by
a) encapsulation
b) inheritance
c) polymorphism
______________

d) both b) and c)

Set A

*SLRBB285*
S

-3-

SLR-BB 285

B.E. (I.T.) (Part II) Examination, 2014


Elective II : SOFTWARE TESTING AND QUALITY ASSURANCE
Day and Date : Monday, 1-12-2014

Marks : 80

Time : 3.00 p.m. to 6.00 p.m.


Instructions : 1) All questions from Section I and II are compulsory.
2) Figures to the right indicate full marks.
SECTION I
2. Answer briefly :

(210=20)

1) What is Software Verification and Validation ?


2) What is fault, failure, error and defect ?
3) What is the concept of complete testing ?
4) What are the object oriented metrics ?
5) What are the measurement in software engineering ?
6) What is verification and validation ?
7) What is control flow structure ?
8) What are the central issues in testing ?
9) What is Box plot ?
10) What is scatter plot ?
3. What is a program flow graph ? Draw various flow graphs used in programming language.

10

4. How do we apply the framework in software development process ?

10
Set A

SLR-BB 285

-4-

*SLRBB285*

SECTION II
5. Answer briefly.

(210=20)

1) List various types of testing.


2) What is Acceptance Testing ?
3) What is software reliability ?
4) How do we perform System Test Planning and Automation ?
5) What are the views of software quality ?
6) What is Software Quality Assurance ?
7) List McCalls Quality Factors.
8) What are the factors influencing of software reliability ?
9) How do we apply system test execution with example ?
10) What is functional testing ?
6. Explain in detail ISO 9126 Quality Characteristics.

10

7. What is Software Quality Assurance Groups ? Explain Software Quality Assurance Groups
activities.

10
_____________________

Set A

SLR-BB 286

*SLRBB286*
Seat
No.

Set

B.E. (I.T.) (Part II) Examination, 2014


Elective II : MOBILE APPLICATION DEVELOPMENT
Day and Date : Monday, 1-12-2014
Time : 3.00 p.m. to 6.00 p.m.
Instructions :

Total Marks : 100

1) Figures to the right indicate full marks.


2) Assume data if necessary.
3) Q. No. 1 is compulsory. It should be solved in first 30 minutes in
Answer Book Page No. 3. Each question carries one mark.
4) Answer MCQ/Objective type questions on Page No. 3 only. Dont
forget to mention, Q.P. Set (A/B/C/D) on Top of Page.
MCQ/Objective Type Questions

Duration : 30 Minutes

Marks : 20

1. Objective type questions. Choose the correct option :

20

1) A device with Android installed is needed to develop apps for Android.


A) True
B) False
2) What part of the Android platform is open source ?
A) Low-level Linux modules
B) Native libraries
C) Application frame work
D) All of the above
3) Which piece of code used in Android is not open source ?
A) Keypad Driver
B) WiFi ? driver
C) Audio driver
D) Power management
4) Android is built upon the Java Micro Edition (J2ME) version of Java.
A) True
B) False
5) Which among these are NOT a part of Androids native libraries ?
A) Webkit
B) Dalvik
C) OpenGL
D) SQLite
6) Android is based on Linux for the following reason.
A) Security
B) Portability
C) Networking

D) All of these

7) What operating system is used as the base of the Android stack ?


A) Linux
B) Windows
C) Java
D) XML
8) What is a key difference with the distribution of apps for Android based devices than
other mobile device platform applications ?
A) Applications are distributed by Apple App Store only
B) Applications are distributed by multiple vendors with different policies on applications
C) Applications are distributed by multiple vendors with the exact same policies on
applications
D) Applications are distributed by the Android Market only
P.T.O.

SLR-BB 286

*SLRBB286*

-2-

9) When developing for the Android OS, Java byte code is compiled into what ?
A) Java source code
B) Dalvik application code
C) Dalvik byte code
D) C source code
10) What does the .apk extension stand for ?
A) Application Package
C) Android Proprietary Kit

B) Application Program Kit


D) Android Package

11) When you distribute your application commercially, youll want to sign it with your
own key ?
A) True
B) False
12) How does Google check for malicious software in the Android Market ?
A) Every new app is scanned by a virus scanner
B) Users report malicious software to Google
C) Google employees verify each new app
D) A separate company monitors the Android Market for Google
13) Which of these are not one of the three main components of the APK ?
A) Dalvik Executable
B) Resources
C) Native Libraries
D) Webkit
14) What is the name of the program that converts Java byte code into Dalvik byte code ?
A) Android Interpretive Compiler (AIC)
B) Dalvik Converter
C) Dex Compiler
D) Mobile Interpretive Compiler (MIC)
15) What was the main reason for replacing the Java VM with the Dalvik VM when the
project began ?
A) There was not enough memory capability
B) Java virtual machine was not free
C) Java VM was too complicated to configure
D) Java VM ran too slow
16) The R. Java file is where you edit the resources for your project.
A) True
B) False
17) What is contained within the manifest xml file ?
A) The permissions the app requires
B) The list of strings used in the app
C) The source code
D) All other choices
18) What is contained within the Layout xml file ?
A) Orientations and layouts that specify what the display looks like.
B) The permissions required by the app
C) The strings used in the app
D) The code which is compiled to run the app
19) Which of the following is NOT a state in the life cycle of a service ?
A) Starting
B) Running
C) Destroyed
D) Paused
20) In an explicit intent, the sender specifies the type of receiver.
A) True
B) False
______________
Set A

*SLRBB286*

-3-

SLR-BB 286

Seat
No.
B.E. (I.T.) (Part II) Examination, 2014
Elective II : MOBILE APPLICATION DEVELOPMENT
Day and Date : Monday, 1-12-2014

Marks : 80

Time : 3.00 p.m. to 6.00 p.m.


Instructions :

1) All questions from Section I and II are compulsory.


2) Figures to the right indicate full marks.
3) Assume data if necessary.
SECTION I

2. Attempt any four :

(54=20)

1) What is Android Intent ? What is explicit intent ? What is implicit intent ?


2) What is Android Content Provider ? Give simple example that would help you understand
Content Provider.
3) State different features of an Android.
4) What is Layout ? Draw and state different types of layouts in android.
5) What are the components contain in ADT (Android Development Tools) Bundle ?
3. a) Draw and explain the diagram of the development process for an Android App.
b) Illustrate the benefits of Async Task over Threads for implementing long-Running tasks.
4. What are the different roles of the following with respect to application ? Framework ?

5
5
10

a) Activities
b) Services
c) View System
d) Resource Manager
e) Notification Manager.
Set A

SLR-BB 286

-4-

*SLRBB286*

SECTION II
5. Attempt any four of the following :

(54=20)

a) Name some commonly used in-built content providers.


b) What are the key graphics capabilities of an Android platform ?
c) What is drawable Animation ? Write a Java code to apply drawable animation to a view.
d) Outline the features of Location Services.
e) Illustrate an example of MPEG-4 container with encoded audio and video elements.

6. What is the use of Shared Preference in Android ? Explain two components that the Share
Preference app will contain. What are the methods offered by editor to save data in Shared
Preferences ?

10

7. Define the categories of sensors available in android devices. How a developer can identify the
sensors present in a device and put them to use in an app ? Write a Java code for the same.

10

_____________________

Set A

SLR-BB 287

*SLRBB287*
Seat
No.

Set

S.E. (Electrical) (Part I) Examination, 2014


ENGINEERING MATHEMATICS III
Day and Date : Tuesday, 9-12-2014
Time : 10.00 a.m. to 1.00 p.m.
Instructions :

Max. Marks : 100

1) Q. No. 1 is compulsory. It should be solved in first 30 minutes in Answer


Book Page No. 3. Each question carries one mark.
2) Answer MCQ/Objective type questions on Page No. 3 only. Dont forget to
mention, Q.P. Set (A/B/C/D) on Top of Page.
MCQ/Objective Type Questions

Duration : 30 Minutes

Marks : 20

1. Choose the correct answer :

(201=20)

1) The complete solution of equation (D3 + 3D2 + 3D + 1) y = 0 is


a) y = (C1 + C2 x + C3) x

b) y = (C1 + C2 x + C3 x2) x

c) y = (C1 + C2 x + C3 x2) ex

d) y = (C1 + C2 x + C3 x2) ex

2) PI of

D2 + 9

cos 3x =

a) 3 cos 3x

b) 3 sin 3x

c) 6 sin 3x

d)

3) The PI of the differential equation (D2 + 1) y = x2 is


a) x2 2
b) x2 + 2
c) x2
4) The solution for (1 + x)2

x
6 sin 3x

d) 2x

d y
dy
+ (1 + x) dx + y = 0 is
dx 2

a) y = C1 cosx + C2 sinx

b) y = C1 log (1 + x) + C2 log (1 + x)

c) y = C1 cos (log (1 + x) )+ C2 sin (log (1+ x)) d) y = [C1 + C2 log (1 + x)] (1 + x)


5) If Z {xk} = F(Z) then Z{ak xk} is

()

()

(z )

a) F a
b) a F a
c) a F a
z
z
6) Which of the following is not correct ? For k 0
1

(z )

d) F a
z ( z sin )

b) z{sin k} = z 2 2z cos + 1

a) z {1} = z 1
z ( z cos )

d) z{k k} = ( z )2

c) z{cos k} = z 2 2z cos + 1
7) The solution of z px qy = 2 pq is
a) z = ax + by 2 ab
8) The solution of q = e
a) z = ax + e

b) z = ax + by + 2 ab

y + c

c) z = bx + ay 2 ba d) z = ax by 2 ba

is
b) z = ax + e

y + c

c) z = ax +

a y+c
a
d) z = ax y + c

P.T.O.

SLR-BB 287

*SLRBB287*

-2-

9) The solution of px+ qy = z is


a) (x + y, y +z) = 0
10) z1

{ } for 2 < |z| is

b) (xy, zx) = 0

1
z 2

a) 2k k 1

k 1

b) 2

k 1

11) Fourier series of f(x) = sinx in ( , ) will be


a) 0
b) sinx terms only

(, )

c) xy

y2
z

k1

k 1

c) 2

=0

( )
y

d) xy , z = 0

d) 2

c) cosine terms only

k 1

d) None of these

12) Half range cosine series of f(x) defined in the interval (O, L) will be
a) f(x) =
c) f(x) =

ao
2
ao

( )

+ a n cos nx
L
n=1

b) f(x) =

ao
2

( )

+ a n sin nx
L
n=1

( )

+ b n cos nx
L
n=1

d) None of the above


2
13) If f(x) is odd and defined in ( a, a) then Fourier coefficient bn =

( )

2
nx
a) f ( x ) cos a dx

( )

2
n x
b) f ( x ) sin a dx

( )

2
nx
c) f ( x ) sin a dx
0

d) 0

14) If L{f(t)} = F(s) then L{ f (u) du} = ___________________


0

a) SL{ f(t)}

b)

L { f ( t )}
S

c) SL{f(t)} f(0)

d) ds L { f ( t )}

15) By convolution theorem, with usual notations L1 { 1( s) 2 ( s )} = _______________


t

a) f1 (t) f2 (t u) du
0

b) f1 (u) f2 (u t) du
0

c) f1 (u) f2 (t u) du

0
1 + t
The function f(t) = 1 t

for 2 t 1
for 1 t 0
16)
for 0 t 1 is
for 1 t 2
a) Odd function
b) Even function
c) Periodic function


17) If f is a vector point function then f is called solenoidal if
 

 
a) f = f
b) f = 0
c) f = f


18) Directional derivative of f in the direction of a is

a


a) f a
b) f a
c) f
a

d) None of the above

( )

d) None of these

d) f = 0


d)

( f ) aa

19) If f is a scalar point function then f = _______________


a) 0

b) 2 f

c) 1

20) The unit vector normal to surface x2+ y2 + z2 = 1 at


a)

1 ( i j )
2

b)

1 ( i + j )
2

c)

1 , 1 , 0 is
2
2
1 ( i + j )
2


d) r

d) None of these

______________
Set A

*SLRBB287*

SLR-BB 287

-3-

Seat
No.

S.E. (Electrical) (Part I) Examination, 2014


ENGINEERING MATHEMATICS III
Day and Date : Tuesday, 9-12-2014
Time : 10.00 a.m. to 1.00 p.m.
Instructions :

Marks : 80

1) Solve any three questions from each Section.


2) Figures to the right indicate full marks.
3) Use of non-programmable calculator is allowed.
SECTION I

2. a) Solve

(D4

+ 10

D2

+ 9) y = sin 2x sinx.

b) Solve (1 +

x)2

d y
dx

+ (1 + x)

dy
dx

+ y = 2 sin [log (1 + x) ].

c) Using method of separation of variables solve 3 x

z
x

5y

z
y

=0.

3. a) Solve p2x2 = z(z qy).


b) Solve

(x2

y2

yz) p +

4
(4+1=5)

(x2

y2

zx) q = z(x y).

c) Solve p2q3 = x2y3.

4. a) Find inverse z-transform of

z
( z 2 ) ( z 3)

| z | < 2.

b) Find z {(k + 1) ak} , k 0.

c) Find z {cosh k} , k 0.

5. a) Solve (D2 4D + 3) y = (x2 ex)2.


b) Find z

{ }

2k 3k
, k 0.
k

OR
b) Solve p(1 + q) = qz.

c) The charge q on the plate of a condenser of capacity C charged through a resistance R by a


steady voltage V satisfies the differential equation R

dq q
+ = V . If q = 0 at t = 0. Find q and i.
dt c

Set A

SLR-BB 287

*SLRBB287*

-4-

SECTION II
6. a) Find L

{ ( )}

log

s+a
s+b

5
1 cos t
.
t

b) Find Laplace transform of

c) Evaluate

3 t

t sint dt.

 

 
7. a) If a , b are constants and r = a cosnt + b sinnt, prove that

 dr
 
= n ( a b)
i) r
dt

ii)

2
dr

dt

2 
+ n r = 0.

b) Find the directional derivative of = x2y + y2z + z2x at p(1,2,1) in the direction of normal to the
surface x2 + y2 z2 x = 1 at Q (1, 1,1).

c) If F = ( x + 3 y ) i + ( y 2z )j + ( az + x ) k is solenoidal, then find the value of a.
8. a) Find Fourier series of

f( x ) =

5
3
8

<x<0
0<x<

b) Find half range sine series of

x 0 < x 2

x 2 < x <

f(x ) =

9. a) Obtained Half range cosine series of f(x) = x in (0, 1).


t

b) Find Laplace transform of

e 3u sin2u
du .
u

c) Find L

.
( s + 1) ( s 2) ( s 3)
2

2s 4

OR
c)


r
r

2
r

_____________________

Set A

SLR-BB 288

*SLRBB288*
Seat
No.

Set

S.E. (Electrical Engineering) (Part I) Examination, 2014


ELECTRICAL MACHINES I
Day and Date : Thursday, 11-12-2014
Time : 10.00 a.m. to 1.00 p.m.

Max. Marks : 100

Instructions : 1) Figure right to indicates full marks.


2) Assume suitable data if necessary.
3) Q. No. 1 is compulsory. It should be solved in first 30 minutes
in Answer Book Page No. 3. Each question carries one mark.
4) Answer MCQ/Objective type questions on Page No. 3 only.
Dont forget to mention, Q.P. Set (A/B/C/D) on Top of Page.
MCQ/Objective Type Questions
Duration : 30 Minutes
1. Choose the correct answer :

Marks : 20
(120=20)

1) The nature of current flowing in the armature of the DC generator is


a) ac
b) dc
c) pulsating
d) dc superimposed on ac
2) The e.m.f. generated in the armature of d.c. generator is directly proportional to
a) number of poles
b) speed of armature
c) flux/pole
d) all of the above
3) In DC generators, armature reaction is produced actually by
a) Its field current
b) Armature conductors
c) Load current in armature
d) Field pole winding
4) A 4 pole lap wound dc shunt motor rotates at the speed of 1500 rpm, has a
flux of 0.4 mWb and the total number of conductors are 1000. What is the
value of emf ?
a) 100 Volts.
b) 0.1 Volts.
c) 1 Volts.
d) 10 Volts.
5) In a shunt generator the voltage build up is restricted by
a) speed limitations
b) armature heating
c) insulation restrictions
d) saturation of iron
6) Which of the following law/rule can be used to determine the direction of
rotation of dc motor ?
a) lenzs law
b) Faradays law
c) Coloumbs law
d) Flemings left hand rule
7) In which braking back emf exceeds supply voltage ?
a) plugging
b) regenerative braking
c) dynamic braking
d) none of these

P.T.O.

SLR-BB 288

-2-

*SLRBB288*

8) In a d.c. series motor the electromagnetic torque developed is proportional to


a) Ia
b) Ia2
c) 1/Ia
d) 1/Ia2
9) Which breaking is not possible in series motor ?
a) counter current braking
b) regenerative braking
c) dynamic braking
d) none of these
10) What is the current drawn by a 220 V d.c. motor of armature resistance 0.5
and back e.m.f. 200 V ?
a) 4A
b) 20A
c) 40A
d) 110A
11) Why HV winding is always placed above the LV winding in a core type
transformer ?
a) to reduce copper cost
b) to provide better cooling
c) to reduce insulation cost
d) to provide more mechanical strength
12) In a single phase transformer at full load Cu loss is 80 W, at half load it is
a) 50 W
b) 25 W
c) 100 W
d) 20 W
13) A transformer has R1 primary resistance, k is its transformation ratio, this
resistance referred to secondary will be ?
b) k2*R1
c) k*R1
d) k*R12
a) R1/k2
14) During testing of transformer
a) both sc and oc are performed at rated current
b) both sc and oc are performed at rated voltage
c) oc test is performed at rated current
d) sc test is performed at rated current
15) The energy efficiency is the term related to
a) Distribution transformer
b) Electrical power transformer
c) Current transformer
d) Voltage transformer
16) If a transformer primary is energised from a square wave voltage source, its
output voltage will be
a) A square wave
b) A sine wave
c) A triangular wave
d) A pulse wave
17) The phase difference between primary and secondary voltages in an ideal
transformer is generally
a) 180
b) 90
c) 0
d) 45
18) In a transformer the voltage regulation will be zero when it operates at
a) unity p.f.
b) leading p.f.
c) lagging p.f.
d) zero p.f. leading
19) The emf induced in the primary of a transformer
a) is in phase with the flux
b) lags behind the flux by 90 degree
c) leads the flux by 90 degree
d) is in phase opposition to that of flux
20) Distribution transformer is _____________ 3-phase transformer.
a) Y-Y
b) -
c) -Y
d) Y-
______________
Set A

*SLRBB288*

SLR-BB 288

-3-

Seat
No.

S.E. (Electrical Engineering) (Part I) Examination, 2014


ELECTRICAL MACHINES I
Day and Date : Thursday, 11-12-2014
Time : 10.00 a.m. to 1.00 p.m.

Marks : 80

Instructions : 1) All questions are compulsory.


2) Figure right to indicates full marks.
3) Assume suitable data if necessary.
SECTION I
2. Attempt any four :
(45=20)
1) Derive expression for cross-magnetizing effect of armature reaction.
2) Write short note on simplex lap and wave winding.
3) An 8 pole lap wound generator armature has 960 conductors, a flux of 40 mwb
and speed of 400 rpm. Calculate the emf generated on open circuit. If the
same armature has wave wound, at what speed must it be driven to generate
400 Volts.
4) Derive torque equation of dc motor.
5) In a brake test the effective load on the branch pulley was 38.1 kg, the
effective diameter of the pulley 63.5 cm and speed 12 rps. The motor took
49 A and 220 V. Calculate output power and the efficiency at this load.
6) Explain dynamic and regenerative braking for dc motor.
3. Attempt any two :
(210=20)
a) A 240 V dc shunt motor has an armature resistance of 0.4 ohm and is running
at the full-load speed of 600 r.p.m. with a full load current of 25A. The field
current is constant; also a resistance of 1 ohm is added in series with
the armature. Find the speed (i) at the full-load torque and (ii) at twice the
full-load torque.
b) Explain different methods of speed control of dc motor.
c) The O.C.C. of dc generator driven at 600 rpm is given below :
Field current 1.4

2.4

3.6

4.0

4.8

Induced volts 125 188 238 250 270


Calculate,
i) the voltage that can be developed with a field resistance of 70 .
ii) the field resistance required to develop 250 Volts.
iii) the critical resistance.
Set A

SLR-BB 288

-4-

*SLRBB288*

SECTION II
4. Attempt any four :

(45=20)

1) Explain why transformer is called constant flux machine.


2) Explain oc and sc test conducted on transformer.
3) Explain different losses in transformer.
4) A single phase transformer with ratio of 440/110 V takes a no load current of
5 A at 0.2 P.F. lagging. If the secondary supplies a current of 120 A at a p.f. of
0.8 lagging estimate the current taken by the primary.
5) Write short note on open delta connection for transformer.
6) Write short note on Equivalent circuit of transformer.
5. Attempt any two :

(210=20)

a) A 30 KVA, 2400/120, 50 Hz transformer has a high voltage winding resistance


of 0.1 and a leakage reactance of 0.22 . The low voltage winding resistance
is 0.035 and the leakage reactance is 0.012 . Find the equivalent winding
resistance, reactance and impedance referred to (i) high voltage side (ii) low
voltage side.
b) What are the different connections used for three phase transformer ? Explain
each in short with sketches.
c) Write short note on auto-transformer. Derive the expression for cu saving in
auto-transformer.
_____________________

Set A

SLR-BB 289

*SLRBB289*
Seat
No.

Set

S.E. (Electrical) (Part I) Examination, 2014


ELECTRONIC DEVICES AND CIRCUITS
Day and Date : Saturday, 13-12-2014
Time : 10.00 a.m. to 1.00 p.m.

Max. Marks : 100

Note : 1) Figures to the right indicate full marks.


2) Q. No. 1 is compulsory. It should be solved in first 30 minutes
in Answer Book Page No. 3. Each question carries one mark.
3) Answer MCQ/Objective type questions on Page No. 3 only.
Dont forget to mention, Q.P. Set (A/B/C/D) on Top of Page.
MCQ/Objective Type Questions
Duration : 30 Minutes
1. Select the appropriate option :

Marks : 20
(120=20)

1) The low frequency response of an transistor amplifier affects due to


a) C junction
b) C in and C out
c) C emitter
d) Both b) and c)
2) At pinch off voltage, for JFET drain current is
a) Maximum
b) Minimum
c) Zero
d) Infinity
3) Self destruction of transistor due to over heating is called as _________
a) Thermal runaway
b) Early effect
c) Bias compensation
d) None of these
4) The feedback factor at critical frequency of oscillation of wein bridge
oscillator is __________
a) 1/3
b) 3
c) 29
d) 1/29
5) The forward current gain for CE amplifier is given by
a) hie
b) hfe
c) hfb
d) hic
6) Transistor can be used as faithful amplifier in __________ region.
a) Active
b) Cutoff
c) Saturation
d) None
7) The capacitor filter is mostly used for __________ load resistance.
a) Heavy
b) Low
c) Medium
d) None
8) Negative feedback amplifiers gain due to negative feedback.
a) Decreases
b) Increases
c) Equal to unity
d) None
P.T.O.

SLR-BB 289

-2-

*SLRBB289*

9) Voltage series feedback amplifier is called as


a) Voltage amplifier
b) Current amplifier
c) Tran resistance amplifier
d) Tran conductance amplifier
10) Doping concentration in transistor regions is
a) E < B < C
b) E < C < B
c) E > C > B
d) E = C > B
11) In CE configuration collector current at IB = 0 will be
a) ICBO
b) ICEO
c) Zero
d) IE
12) Series inductor filter
a) Allow DC blocks AC
b) Block DC and AC
c) Allow DC and AC
d) Allows ac and blocks DC
13) Stability factor for ___________ bias is (1 + ).
a) Voltage divider
b) Fixed
c) Emitter bias
d) None
14) Cross over distortion is drawback of ___________ amplifier.
a) RC coupled
b) Xr coupled
c) Class A
d) Class B
15) To operate transistor as a switch operating regions are restricted to
a) Active only
b) Saturation and active
c) Saturation and cut off
d) Cutoff only
16) Small signal amplifier is also called as _______________ amplifiers.
a) Power
b) Feedback
c) Voltage
d) None
17) Oscillator circuit using a quartz crystal has __________ frequency stability.
a) High
b) Low
c) Medium
d) None
18) An oscillator basically is an amplifier with a product of A .
a) zero
b) one
c) infinite
d) low
19) For JFET
b) =RD + gm
a) =RD/gm
c) =RD*gm
d) =RD gm
20) Efficiency of class B power amplifier is _____________ %.
a) 25
b) 78.5
c) 50.5
d) 100
______________
Set A

*SLRBB289*

-3-

SLR-BB 289

Seat
No.

S.E. (Electrical) (Part I) Examination, 2014


ELECTRONIC DEVICES AND CIRCUITS
Day and Date : Saturday, 13-12-2014
Time : 10.00 a.m. to 1.00 p.m.

Marks : 80

Note : 1) All questions are compulsory.


2) Figures to the right indicate full marks.
SECTION I
2. Solve any four :

(44=16)

1) Compare FET and BJT.


2) Explain need of bias compensation.
3) Explain direct coupling and its advantages for BJT amplifier.
4) Explain the effect of coupling capacitors on frequency response.
5) Draw AC and DC equivalent circuit for common emitter BJT amplifier.
3. Solve any three :

(83=24)

1) Explain different biasing techniques in detail.


2) Design single stage RC coupled amplifier for the given data BC 147
(transistor), hfe = 350, hie = 2.5 kohm, Vcc = 15 V, ICQ = 4 mA, VCEQ = 7V,
VBE = 0.6 V, S = 5. Voltage gain (Av) = 4, Assume data if required.
3) Draw frequency response of BJT amplifier. Define bandwidth. Why it is drawn
on semi log paper ?
4) Explain construction and operation with transfer characteristics of depletion
type MOSFET and enhancement type MOSFET.
Set A

SLR-BB 289

-4-

*SLRBB289*

SECTION II
4. Solve any four :

(44=16)

1) Compare capacitor and inductor filter.


2) Explain Barkhausens criterion for sustained oscillations.
3) What are the advantages and disadvantages of negative feedback in amplifiers ?
4) Define cross over distortion and % conversion efficiency of power amplifier.
5) Compare voltage amplifier with power amplifier.
5. Solve any three :

(83=24)

1) Draw and explain wein bridge oscillator. Derive equation for the frequency of
oscillation.
2) Explain effect of negative feedback on output impedance, input impedance,
gain and bandwidth of voltage shunt feedback amplifier.
3) Derive the ripple factor of inductive filter with FWR.
4) Derive the expression of efficiency for class B power amplifier and explain
harmonic distortion.
_____________________

Set A

SLR-BB 29

*SLRBB29*
Seat
No.

Set

S.E. (Civil) (Part II) Examination, 2014


WATER RESOURCES ENGG. I (Old)
Day and Date : Saturday, 29-11-2014
Time : 3.00 p.m. to 6.00 p.m.
N.B. :

Max. Marks : 100

1) Figures to the right indicate full marks.


2) Use of electronic non-programmable calculators is allowed.
3) Q. No. 1 is compulsory. It should be solved in first 30 minutes in
Answer Book Page No. 3. Each question carries one mark.
4) Answer MCQ/Objective type questions on Page No. 3 only. Dont
forget to mention, Q.P. Set (A/B/C/D) on Top of Page.
MCQ/Objective Type Questions

Duration : 30 Minutes
1.

i) The average annual rainfall over the whole of India is estimated as


a) 51 cm
b) 143 cm
c) 119 cm
d) 167 cm

Marks : 20
(120=20)

ii) The normal date for onset of monsoon rains in India is


a) Early June at Kerala and Assam
b) Early June at Bombay
c) Early June at Delhi
d) Late November at Kerala
iii) In filtration capacity
a) is a constant factor
b) changes with time
c) changes with location
d) changes both with time as well as location
iv) A catchment is located in hilly and rugged area. The area of the catchment is about
8000 sq km. The method of calculating the mean rainfall over this catchment would
preferably be
a) Arithmetic mean method
b) Thiessen polygon method
c) Isohyetal method
d) Geometrical mean method
v) Excess or effective rainfall may be defined as the
a) Amount of rainfall which produces equal runoff
b) Total rainfall-infiltration-losses
c) Water depth collected during the net supply interval
d) All the above
vi) A lag time of a basin is the time interval between
a) The beginning and end of the direct runoff
b) The centroid of the rainfall excess and the peak of the runoff hydrograph
c) The beginning of rainfall and the peak of runoff hydrograph
d) The end of rainfall and end of runoff
P.T.O.

SLR-BB 29

-2-

*SLRBB29*

vii) The rotational formula is restricted to the catchments of size less than
a) 500 ha
b) 5000 ha
c) 50000 ha
d) 500000 ha
viii) Dickens formula predicts the maximum flood discharge Q in terms of the area A and the
coefficient C as Q = C. An. The value of n is
a) 0.25
b) 0.50
c) 0.67
d) 0.75
ix) Specific capacity of a well as
a) Constant over time after commissioning of the well
b) Increases with time after start of pumping
c) Decreases with time after start of pumping
d) May increase or decrease depending upon particular aquifer
x) Time of concentration is the
a) Time for which the rain remains concentrated on a basin
b) Time of maximum precipitation that may ever concentrate and fall over a given basin
c) Time taken by the rain water to flow to an existing defined drain in a basin
d) The maximum time taken by the rain water to reach the outlet of the basin
xi) Rain water harvesting is required for _____________
a) Ground water table recharge
b) Bore well recharge
c) Storage for future use
d) All of the above
xii) The most economical method of soil conservation is to
a) Construct check dams
b) Drain the soil
c) Construct the contour bunds
d) A forest the soil
xiii) The method of growing the crops on ridges, running on sides of water ditches is
known as
a) Flood irrigation
b) Furrow irrigation c) Check irrigation d) None of them
xiv) The crop amongst the following, which is expected to have maximum duty is
a) Wheat
b) Rice
c) Sugarcane
d) Cotton
xv) The moisture held by the well-drained soil against gravity drainage by the force of surface
tension between the soil drains and water drops is called
a) Field capacity water
b) Hygroscopic water
c) Capillary water
d) Water of adhesion
xvi) Crops grown during monsoon month and harvested in October are called as
_____________ crops.
a) Rabbi
b) Kharif
c) Rabbi and Kharif d) Hot weather
xvii) If the duty for a crop with base period 120 days is 1500 Hectares/cumec then it delta is
_____________ cm.
a) 69.12
b) 108
c) 0.6912
d) none of these
xviii) If the intensity of irrigation for Kharif is 45% and that for Rabi is 60%, then the annual
intensity of irrigation, is
a) 60%
b) 100%
c) 105%
d) none of them
xix) In a mildly water scarce area, the drip irrigation could be preferred for growing
a) Wheat
b) Fodder
c) Rice
d) Fruits and vegetables
xx) Permanent wilting point moisture content for a crop represents the
a) Hygroscopic water
b) Capillary water
c) Field capacity water
d) None of the above
______________

Set A

*SLRBB29*

SLR-BB 29

-3-

Seat
No.

S.E. (Civil) (Part II) Examination, 2014


WATER RESOURCES ENGG. I (Old)
Day and Date : Saturday, 29-11-2014
Time : 3.00 p.m. to 6.00 p.m.

Marks : 80

N.B. : 1) Attempt any three questions from questions 2, 3, 4 and 5.


Attempt any three questions from questions 6, 7, 8, and 9.
2) Figures to the right indicate full marks.
3) Use of electronic non-programmable calculators is allowed.
SECTION I
2.

3.

a) What is runoff ? What are the factors that affect the runoff from a catchman area ?
b) Precipitation station X was inoperative for part of a month during which a storm occurred.
The respective storm totals at three surrounding stations A, B, C were 107, 89 and
122 mm. The normal annual precipitations at storms X, A, B and C are 978, 1120, 935 and
1200 mm respectively.
Estimate the storm precipitation for station X.
a) A storm with 15.0 cm precipitation produced direct runoff of 8.7 cm. The time distribution
of the storm was as following :
Time (hr) 1
Rainfall
0.6
cm

1.35 2.25 3.45

2.7

2.4

1.5

0.75

7
6

Estimate the index of the storm.

b) Write down the most common empirical formula used to calculate evaporation. What are
the factors affecting evaporation ? Describe any one practical method of determining
evaporation.
4. a) The ordinates of 3 hour unit hydrograph are given below :
Time (hr)
Ordinates
(m3/sec)

12

15

18

21 24

10

25

20

16

12

7
8

27

30

For the ordinates of a 6 hour unit hydrograph for the same basin, analytically.
What is the peak value of discharge in this unit hydrograph ?
b) Explain the following terms related to flood frequency studies :
i) Flood frequency
ii) Recurrence interval
iii) Return period
iv) Probability of occurrence.
5. Write short notes on :
i) S-curve hydrograph.
ii) Stream gauging.
iii) Dupuits theory of ground water flow in unconfined aquifer.

4
4
5

Set A

SLR-BB 29

-4-

*SLRBB29*

SECTION II
6.

a) Discuss the terms field capacity, wilting point and optimum water content.

b) The irrigation canal has GCA of 80000 hectares out of which 85% is culturable, the intensity
of irrigation for Kharif season is 30% and for Rabbi season 60%. Find the discharge required
at the head of the canal. If the duty at its head is 800 hectare/cumec for Kharif and
1700 hectares/cumec for Rabbi season.

7. a) Discuss the conditions favouring adoption of sprinkler irrigation method. Also discuss the
limitations of this method.

b) Discuss the various types of soils in India with reference to their suitability for irrigation.

8. a) Discuss the factors affecting the duty of water in a canal system. What methods will you
suggest to improve the duty ?

b) Discuss different techniques of rain water harvesting and ground water harvesting.
9. a) Write a note on any two :

7
8

i) Percolation Tank
ii) Warabandi System
iii) Rainwater Management.
b) Describe with a neat sketch, general layout of a lift irrigation scheme. Briefly explain the
Jack Well.

_____________________

Set A

SLR-BB 290

*SLR-BB-290*
Seat
No.

Set

S.E. (Part I) (Electrical Engineering) Examination, 2014


ELECTRICAL MEASUREMENTS AND INSTRUMENTATION
Day and Date : Tuesday, 16-12-2014
Time : 10.00 a.m. to 1.00 p.m.

Max. Marks : 100

N.B. : 1)
2)
3)
4)

Solve all questions.


Figures to the right indicate full marks.
Make suitable assumptions if any.
Q. No. 1 is compulsory. It should be solved in first 30 minutes in
Answer Book Page No. 3. Each question carries one mark.
5) Answer MCQ/Objective type questions on Page No. 3 only. Dont
forget to mention, Q.P. Set (A/B/C/D) on Top of Page.
MCQ/Objective Type Questions

Duration : 30 Minutes

Marks : 20

1. Choose the correct options :

(201=20)

1) The usage of electronic instruments is becoming more extensive because they have
a) A high sensitivity and reliability
b) A fast response and compatibility with digital computers
c) The capability to respond to signals from remote place
d) All of the above
2) The input resistance of a cathode ray oscilloscope is of the order of
a) Tens of ohms
b) Mega ohm
c) Kilo ohms

d) fraction of ohm

3) A watt meter has a full scale range of 2500 W. It has an error of 1% of true value. What would be
the range of reading if the true value is 1250 W ?
a) 1225 W 1275 W
b) 1245 W 1255 W c) 1200 W 1330 W
d) 1237.5 W 1262.5 W
4) A quantity whose magnitude has a definite repeating time cycle is called as
a) Transient
b) Steady state periodic
c) Steady state a periodic
d) Transient state periodic
5) The symbol nm stands for
a) Newton meter
b) Nano meter

c) Nono milli

d) Number of moles

6) In order that the value of an inductance standard should remain constant irrespective of the value of
current carried by its coil.
a) Cores of ferromagnetic materials should be used
b) Air cored coils should be used
c) Solid conductors should be used
d) All of the above
7) Permanent magnets used in instruments are hard cored materials because
a) They have broad hysteresis loop
b) The energy density is high
c) They have a hig BH max product
d) All of the above
8) A vibration galvanometer is to be tuned to a frequency of 50 Hz. The ratio of its control constant to
intertial constant should be
a) 98696
b) 2500
c) 10.132 106
d) None of the above
P.T.O.

SLR-BB 290

*SLR-BB-290*

-2-

9) In spring controlled moving iron instruments, the scale is


a) uniform
b) cramped at lower end and expanded at upper end
c) expanded at lower and cramped at upper end
d) cramped at both lower and upper end
10) An astatic movements in an electrodynamometer type of instrument is used for
a) eliminating errors on account for external magnetic field
b) increasing the operating torque of the instrument
c) providing eddy current damping
d) none of the above
11) The burden of current transformers is expressed in terms of
a) a secondary winding current
b) VA rating of the transformers
c) voltage, current and PF of the secondary winding circuit
d) none of the above
12) In electrodynamometer type wattmeter, current coils designed for carrying heavy currents are standard
wires or laminated conductors
a) to reduce the iron losses
b) to reduce the hysteresis losses
c) to reduce the eddy current losses in conductors
d) all of the above
13) In household single phase induction type wattmeter, the meter can be reversed by
a) reversing the supply terminals
b) reversing the load terminals
c) opening the meter connections and reversing either the potential coil terminals or current coil terminals
d) reversing connections of both current and potential coil circuits
14) Equal resistances of 100 each are connected in each arm of Wheatstone bridge which is supplied
by a current of one A. The smallest value of resistance that can be measured is
a) 20 m

b) 2

c) 20

d) none of the above

15) Standardization of potentiometer is done in order that they become


a) accurate
b) precise
c) accurate and direct reading
d) accurate and precise
16) Frequency can be measured by using
a) Maxwells bridge
c) Heaviside Compbell bridge

b) Schering bridge
d) Wiens bridge

17) Permanent magnets are tested by


a) Ballastic methods
b) Using an electric circuit having a mutual inductance
c) Potentiometric methods
d) Betteridge apparatus
18) The optical spectrum have wavelengths ranging from
a) 10 nm to 106 nm
b) 10 um to 770 nm c) 370 nm to 770 nm

d) 370 nm to 106 nm

19) Electronic voltmeters which use rectifier employ negative feedback. This is done
a) to increase the overall gain
b) to improve stability
c) to overcome non linearity of diodes
d) none of the above
20) The deflection of an electronic beam on a CRT screen is 10 mm, suppose the pre-accelerating
anode voltage is halved and potential between deflecting plates is doubled, the deflection of electrons
beam will be
a) 80 mm
b) 40 mm
c) 20 mm
d) 10 mm

______________

Set A

*SLR-BB-290*

-3-

SLR-BB 290

Seat
No.

S.E. (Part I) (Electrical Engineering) Examination, 2014


ELECTRICAL MEASUREMENTS AND INSTRUMENTATION
Day and Date : Tuesday, 16-12-2014
Time : 10.00 a.m. to 1.00 p.m.

Marks : 80

N.B. : 1) Solve all questions.


2) Figures to the right indicate full marks.
3) Make suitable assumptions if any.
SECTION I
2. Solve any four :
a) What is synchronization ? Explain Weston synchroscope.
b) Explain MI type of instrument. Explain types of MI instruments.
c) Explain classification of standards of measurements.

(45=20)

d) Explain working of 1- energy meter with the help of neat sketch.


e) Define the following :
i) Accuracy
ii) Sensitivity
iii) Dead zone
iv) True value
v) Linearity.
f) A watthout meter is used to measure power in the circuit with the help of following
equation : P = E2R where the limiting values of voltage and res. are E = 200 V 1% and
R = 100 5%
Calculate :
i) The nominal power consumed
ii) The limiting error of power in percent and watts.
3. a) Explain two wattmeter method to measure 3 with the help of neat diagrams. Desire the
equation for calculation of load power factor.

10

b) Explain Ayrton shunt and galvanometer shunts with neat sketches. Derive expression for
the shunt res. and shunt current.

10

OR
b) Explain De-Sauty bridge. Also derive the equation for unknown values.

10

Set A

SLR-BB 290

-4-

*SLR-BB-290*

SECTION II
4. Solve any four :

(45=20)

a) Draw the typical phasor diagram of potential transformer. Derive equation for actual
transformation ratio.
b) What are the sources of errors in 1- induction type energy meter, also explain the
adjustments made for compensating these errors.
c) Explain measurement of time period using electronic counter (Digital).
d) Explain the procedure of standardization of a dc potentiometer.
e) Draw block diagram of general purpose CRO and explain controls enlisted below :
i) Intensity control
ii) Focus
iii) Horizontal and vertical control.
f) With neat diagram, explain the working of Drysdale potentiometer.
5. Solve any two :

(102=20)

a) Explain with a neat block diagram working of a digital multimeter.


b) Explain Kelvins double bridge, derive the equation of unknown resistance, state the
advantage of Kelvins bridge over Maxwells bridge.
c) Four arms of Hay bridge are arranged as follows
AD- is coil of unknown impedance Z,
DC-is non-inductive res. of 1k
CB-is non inductive res. of 800 in series with standard capacitor of 2 F
BA- is non inductive res. of 16500 , if the supply frequency is 50 Hz. Calculate the value
of L and R at balance.
_____________________

Set A

SLR-BB 291

*SLRBB291*
Seat
No.

Set

S.E. (Part I) (Electrical Engg.) Examination, 2014


POWER PLANT ENGINEERING
Day and Date : Thursday, 18-12-2014
Time : 10.00 a.m. to 1.00 p.m.

Max. Marks : 100

Instructions : 1) Q. No. 1 is compulsory. It should be solved in first 30 minutes


in Answer book Page No. 3. Each question carries one mark.
2) Answer MCQ/Objective type questions on Page No. 3 only.
Dont forget to mention, Q.P. Set (A/B/C/D) on Top of Page.
MCQ/Objective Type Questions
Duration : 30 Minutes

Marks : 20

1. Choose the correct answer :


1) Which engine has the highest air fuel ratio ?
A) Petrol engine
B) Gas engine
C) Diesel engine

20
D) Gas turbine

2) In a power plant, coal is carried from storage place to boilers generally by means
of
A) Bucket
B) V-belts
C) Trolleys
D) Manually
3) Live storage of coal in a power plant means
A) Coal ready for combustion
B) Preheated coal
C) Storage of coal sufficient to meet
D) Coal in transit
4) What is the maximum size of steam turbine usually being installed, for thermal
power plants ?
A) 120 MW
B) 250 MW
C) 500 MW
D) 1000 MW
5) Which variety of coal has lowest calorific value ?
A) Steam-coal
B) Bituminous coal
C) Lignite
D) Anthracite
6) An impulse turbine
A) Always operates submerged
B) Makes use of a draft tube
C) Is most suited for low head installations
D) Operates by initial complete conversion to kinetic energy
7) In a diesel engine fuel is injected at a pressure of
A) 10 kg/cm2
B) 20 30 kg/cm2
C) 60 80 kg/cm2
D) 90 130 kg/cm2
8) Diesel engines for power plants are usually
A) Horizontal
B) Slow speed
C) Supercharged
D) Air cooled
P.T.O.

SLR-BB 291

*SLRBB291*

-2-

9) A two stroke engine may be identified by


A) Piston size
B) Absence of valves
C) Cooling system
D) Lubrication system
10) Specific gravity of diesel oil is
A) 0.84
B) 1.14

C) 2.7

D) 3.8

11) Out of the following diesel engines, the minimum air consumption per BHP will be
in
A) 4 stroke, mechanical injection
B) 4 stroke, air injection
C) 2 stroke, air injection
D) 2 stroke, mech. injection, pump scavenging
12) In a steam power plant water is used for cooling purposes in
A) Boiler
B) Economizer
C) Condenser
D) Super-heaters
13) Even with best possible arrangements, the thermal efficiency of a gas turbine
cycle is always below
A) 10%
B) 20%
C) 25%
D) 40%
14) The path followed by the gases discharged from chimney called the plume,
depends on
A) Thermal properties of gases
B) Dynamic properties of gases
C) Wind direction
D) All of the above
15) Dissolved solids in water can be removed/reduced by
A) Distillation
B) Demineralization
C) Softening
D) Any of the above
16) Small domestic electric power generators are usually of capacity around
A) 10 kVA
B) 5 kVA
C) 1 kVA
D) 100 VA
17) In which of the following power plant the availability of power is least reliable ?
A) Solar power plant
B) Wind energy
C) Tidal power plant
D) Geothermal power plant
18) In solar thermal conversion systems the solar heat is transferred to
A) Water-steam
B) Liquid metals C) Molten salts
D) Any of the above
19) Winds caused by greater solar heating of the earths surface near the equator
than near the northern or southern poles, are known as
A) Local winds
B) Equatorial winds
C) Planetary winds
D) Trade winds
20) The total power of a wind stream is proportional to
A) Velocity of stream
B) (Velocity of stream)2
C) (velocity of stream)3
D) 1/(velocity of stream)
______________

Set A

*SLRBB291*

-3-

SLR-BB 291

Seat
No.

S.E. (Part I) (Electrical Engg.) Examination, 2014


POWER PLANT ENGINEERING
Day and Date : Thursday, 18-12-2014
Time : 10.00 a.m. 1.00 p.m.

Marks : 80

SECTION I
2. Solve any four (5 marks each) :
A) With a neat sketch explain the arrangement of hydraulic power plant.
B) Define :
a) Demand factor
b) Diversity factor
c) Average load
d) Plant capacity factor
e) Plant use factor
C) A generating station has connected load of 43 MW and maximum demand of
20 MW the units generated being 61.5 106 per annum. Calculate :
1) Demand factor
2) Load factor
D) Write short notes on :
a) Precipitation
b) Evaporation
c) Storage
d) Pondage.
E) Write a short note on fluidized bed combustion.
F) Give the general layout of ash handing and dust collection system.
3. Solve any two (10 marks each) :
A) Write short notes on :
a) Power factor tariff
b) Three part tariff
c) Two part tariff
d) Straight meter rate
e) Block meter rate
f) Flat rate
g) Block rate.
B) Explain with neat sketch diesel engine power plant.
C) Classify reactors and also explain.

Set A

SLR-BB 291

-4-

*SLRBB291*

SECTION II
4. Solve any four (5 marks each) :
A) Explain different types of wind power plants.
B) What is geothermal energy ? How it can be used for power generation.
C) Explain fission and fusion with neat sketch. Explain working of moderator in nuclear
power plant.
D) Write a short note on biomass power generation.
E) Write a short note on tidal power generation.
F) Explain with neat sketch pelton wheel turbine.
5. Solve any two (10 marks each) :
A) Explain :
i) Horizontal axis wind mills
ii) Vertical axis wind mills with neat sketches.
B) Explain with neat sketch open cycle and closed cycle gas turbine Power Plant.
C) Explain with neat sketch typical layout of solar thermal power plant.

_____________________

Set A

SLR-BB 292

*SLRBB292*
Seat
No.

Set

S.E. (Part II) (Electrical) Examination, 2014


LINEAR ALGEBRA (Old)
Day and Date : Tuesday, 2-12-2014
Time : 3.00 p.m. to 6.00 p.m.

Max. Marks : 100

Instructions : 1) Attempt any three questions from each Section.


2) Q. No. 1 is compulsory. It should be solved in first 30 minutes
in Answer Book Page No. 3. Each question carries one mark.
3) Answer MCQ/Objective type questions on Page No. 3 only.
Dont forget to mention, Q.P. Set (A/B/C/D) on Top of Page.
MCQ/Objective Type Questions
Duration : 30 Minutes

Marks : 20

1. Choose the correct alternative :

20
2

3 2 5
1) The eigen value of 0 1 6 are

0 0 3

a) 3, 1, 3

b) 1, 1, 3

c) 9, 1, 9

d)

1
1
, 1,
3
3

2) The columns of Am n span Rm if


a)
b)
c)
d)

For each b Rn, the equation AX = b has solution


A has pivot element in each column
A has pivot element in each row
For each b Rn, the equation AX = b has no solution

3) The system of equations AX = b where A is a non singular matrix is consistent if the rank
of A is _________
a) Less than the rank of [A, b]
b) Greater than the rank of [A, b]
c) Is equal to the rank of [A, b]
d) None of these
12

4) Rank of the matrix 2 2


0

a) 1

22
32
0

32

4 2 is
0

b) 4

c) 3

d) 2

Rn Rm

5) Let T :
be a linear transformation and let A be the standard matrix for T.
Then T is one to one if and only if
a) Columns of A span Rm
b) Columns of A are linearly
c) Rows of A span Rm
d) Rows of A are linearly
x1
x 2
6) If T : R2 R2 by T(X) =
, where X = x ,then standard matrix of T is
2
x1
0 1
a)

1 0

0 1
b)

1 0

0 1
c)

1 1

0 1
d)

1 0

P.T.O.

SLR-BB 292

*SLRBB292*

-2-

7) In LU factorization of a matrix A
a) L is echelon form of A
c) Both L, U are echelon form of A

b) U is echelon form of A
d) None of L, U are in echelon form of A

8) A basis for a subspace H of Rn is


a) A linearly independent set in H that spans H
b) A linearly dependent set in H that spans H
c) A linearly independent set
d) None of these
9) The pivot column of a matrix A form
a) Basis for column space of A
c) Basis for row space of A
10) If A is a m n matrix, then Nul A is =
a) {X : X is in Rn and AX = I}
c) {X : X is in Rn and A1 X = 0}

b) Basis for null space of A


d) None of these
b) { X : X is in Rn and AX = 0}
d) None of these

11) An n n matrix A is orthogonally diagonalizable if and only if


a) A is skew symmetric b) A is diagonal
c) A is symmetric

d) None of these

12) Let u be a non zero vector. Then a unit vector in the direction of u is equal to
u
u
d) None of these
b) u
a)
c) u
u
13) The function f(z) = ez is analytic
a) Only at (0, 0)
b) No where

c) Every where

14) If f(z) = u (x, y) + iv (x, y) then f1(z) = _______


b) uy + vy
c) ux ivx
a) ux + vx

d) Except at (0, 0)
d) ux + ivx

5z 2
is bilinear transformation, then the value of determinant of this transformation
g + 3z
is equal to
a) 51
b) 3
c) 33
d) 39

15) If w =

16) The harmonic conjugate of excosy is


a) exsiny + c
b) ex + c
17) The transformation w =
a) Translation

1
is
z
b) Inversion

c) excosy + c

d) None of these

c) Rotation and magnification d) None of these

18) If x = 70, y = 149, the coefficient of regression of y on x = 0.7. Then equation of line of
regression of y on x is
a) y = 0.5 x + 60
b) y = 0.6x + 80
c) y = 0.8x + 120
d) y = 0.7x + 100
19) If b1 and b2 are two regression coefficients then coefficient of correlation is
b1
b2
d)
a) b1 b 2
b) b1 b 2
c)
b2
b1
20) The equation of the line of regression of x on y is
b) x x = byx ( y y )
a) x x = byx (y y )

c) y y = bxy (x x )

d) x x = bxy ( y y )

______________
Set A

*SLRBB292*

-3-

SLR-BB 292

Seat
No.

S.E. (Part II) (Electrical) Examination, 2014


LINEAR ALGEBRA (Old)
Day and Date : Tuesday, 2-12-2014
Time : 3.00 p.m. to 6.00 p.m.

Marks : 80

Instruction : Attempt any three questions from each Section.


SECTION I
2 2 4
2. a) Find LU fabrication of A = 1 3 1
3 7 5
b) Write the matrix equation that determines the loop currents.

c) Solve :
x1 3x3 = 8
2x1 + 2x2 + 9x3 = 7

x2 + 5x3 = 2

(6+4+4=14)

3. a) Find the co-ordinate vector [X]B of X relative to the given basis B = {b1, b2, b3 ] where
1
8
3
2
1

b1 = , b2 = 4 , b3 = 2 , X = 9 .


3
6
9
4

Set A

SLR-BB 292

*SLRBB292*

-4-

2
6 12
b) Let A =
and W = 1 . Determine if W is in Col A. Is W in Nul A ?

3 6
c) Find the general flow pattern of the network shown in Figure. Assuming that the flows are
all non-negative. What is the largest possible value of x3 ?

(4+3+6=13)

4. a) If A~B find bases for Col A and Nul A.


4 1
9
1 3
1

2 + 6 6 1 10
0
,B =
Where A =
3
9 6 6 3
0

4
9
0
3 9
0

0
0

2
0

3
0

8
5

b) Find basis for the space span by the vectors.


1

0 ,
0

1

2

1 ,
1

1

6

1 ,
2

1

5

3 ,
3

4

0

3
1

1

(6+7=13)

5. a) Apply power method to find dominant eigen values and eigen vectors for
2 1
1
A=
, X0 = .
4 5
0

b) Diagonalize the following matrix


3
3
1

A = 3 5 3
.
3
3
1

(6+7=13)
SECTION II

1
1
6. a) Let A =
1
1

1 0
7


1 0
2
3
1 0
, b =
0 1
6
5
0 1


0 1
4

Find a least-square solution of Ax = b.

Set A

*SLRBB292*

SLR-BB 292

-5-

b) If possible diagonalize, the matrix


8
3
0

A = 8
0 2
3 2
0
7. a) Find K such that
b) Show that u =

7
1
log (x2 + y2) + itan1
2

Kx
is analytic.
y

1
log (x 2 + y 2 ) is harmonic and find its harmonic conjugate.
2

8. a) Find the coefficient of correlation between x and y from the following data n = 25,
x = 100, x2 = 950, y = 110 y2 = 850, xy = 800.

b) The equation of lines of regression are x + 2y = 5 and 2x + 3y = 8 find x, y and r


(coefficient of correlation).
c) Given

X series

Y series

Mean

18

100

Standard Deviation

14

20

Coefficient of correlation between X and Y is + 0.8. Find most probable value of


Y if X is 70 and the most probable value of X if Y is 90.
2
9. a) Show that {u1, u2, u3} is orthogonal set where u1= 7 , u2 =

1

b) Evaluate

3
6
3 , u3= 1 .


1
9

e 2z
(z 1) (z 2) dz , where C is the circle z = 3.

2
c) Let Q(x) = x12 8x1x 2 5 x 22 . Compute the value of Q (x) for x = .
2

_____________________

Set A

Set A

SLR-BB 293

*SLRBB293*
Seat
No.

Set

S.E. (Electrical) (Part II) Examination, 2014


ELECTRIC MEASUREMENT (Old)
Day and Date : Wednesday, 26-11-2014
Time : 3.00 p.m. to 6.00 p.m.

Max. Marks : 100

Instructions : 1) Q. No. 1 is compulsory. It should be solved in first 30 minutes in Answer


book Page No. 3. Each question carries one mark.
2) Figure to right indicates full marks.
3) Make suitable assumptions if necessary.
4) Answer MCQ/Objective type questions on Page No. 3 only. Dont forget
to mention, Q.P. Set (A/B/C/D) on Top of Page.
MCQ/Objective Type Questions
Duration : 30 Minutes

Marks : 20

1. MCQ/Objective type questions :

20

1) The reason why eddy current damping cannot be used in moving iron instrument is __________
a) they have a strong operating field
b) they are not normally used in vertical position
c) they need a large damping force which can be provided by air friction
d) they have a weak operating magnetic field
2) The power in a 3 , 4 wire circuit can be measured by using ___________
a) 2-wattmeters
b) 4-wattmeters
c) 3-wattmeters
d) 1-wattmeters
3) The controlling torque in a megger is provided by __________
a) springs
b) weights attached to the moving system
c) it does not need any controlling torque
d) none of the above
4) Creeping in a 1 induction type energy meter may be due to
a) over compensation for friction
b) over voltage
c) vibrations
d) all of the above
5) In present day measurement system,
a) direct methods are commonly used
b) use of direct methods is limited but indirect methods are commonly used
c) both direct and indirect methods are commonly used
d) all of the above
6) Air friction damping should not be used where the deflecting torque in the instrument is produced
due to _________
a) magnetic field
b) electrostatic field c) thermoelectric emf d) none of the above
7) Which of the meter has the highest accuracy in the prescribed limit of frequency range ?
a) PMMC
b) moving iron
c) EDM
d) rectifier
8) A moving iron instrument can be used for current and voltage measurements _________
a) in a.c. circuits only
b) in d.c. circuits only
c) in both d.c. and a.c. circuits for any value of frequency (in case of a.c.)
d) in both a.c. and d.c. circuits for frequencies upto about 125 Hz (in a.c. ckts)
P.T.O.

SLR-BB 293

-2-

*SLRBB293*

9) A PMMC instrument has an internal resistance 200 and current required for its full scale deflection
is 50 A . The meter is capable of measuring on its own a maximum voltage of
a) 5 mV
b) 10 mV
c) 5 V
d) 10 V
10) In 1 induction motor, in order to obtain true value of energy, the shunt magnet flux should lag
behind the applied voltage by _________
a) 90
b) 0
c) 45
d) none of the above
11) Standardisation of potentiometer is done in order that, they become __________
a) accurate
b) precise
c) accurate and direct reading
d) accurate and precise
12) When a potentiometer is used for measurement of voltage of an unknown source, the power consumed
in the circuit of the unknown source under null conditions __________
a) is very high
b) is high
c) is small
d) is ideally zero
13) The advantage of Hays bridge over Maxwells inductance capacitance bridge is
a) its equations for balance do not contain any frequency terms
b) it can be used for measurement of inductance of high Q-coils
c) it can be used for measurement of inductance of low Q-coils
d) none of the above
14) The disadvantages of shunts for use of high currents are
a) it is difficult to achieve good accuracy with shunts
b) power consumption of the shunts is large
c) the metering circuit is not electrically isolated from the power circuit
d) all of the above
15) In circuits of RF voltmeters
a) vacuum tube diodes are used
b) conventional PN junction diodes are used
c) schott key barrier or point contact type diodes are used
d) all of the above
16) The power consumption of a d.c. voltmeter using a direct coupled amplifier when measuring a
voltage 0.5 V is of the order of _________
a) few watt
b) few milliwatt
c) few microwatt
d) a few nanowatt
17) A thin aluminium film is usually deposited on the non-viewing side of the phosphor because
a) it acts as a heat sink and prevents phosphor burn
b) the light scatter from the phosphor is reduced
c) it does not allow the screen to be negatively charged
d) all of the above
18) The horizontal amplifier should be designed for
a) high frequency signal with a fast rise time
b) high amplitude signal with slow rise time
c) high amplitude signal with a fast rise time
d) low amplitude signal with a fast rise time
19) The nominal ratio of a current transformer is
a) primary winding current/secondary winding current
b) rated primary winding current/rated secondary winding current
c) no. of secondary winding turns/no. of primary winding turns
d) all of the above
20) A voltmeter with broad bandwidth has
a) low noise level and high sensitivity
b) high noise level and high sensitivity
c) high noise level and low sensitivity
d) low noise level and low sensitivity
______________

Set A

*SLRBB293*

-3-

SLR-BB 293

Seat
No.
S.E. (Electrical) (Part II) Examination, 2014
ELECTRIC MEASUREMENT (Old)
Day and Date : Wednesday, 26-11-2014
Time : 3.00 p.m. to 6.00 p.m.

Marks : 80

Instructions : 1) All questions are compulsory.


2) Figure to right indicates full marks.
3) Make suitable assumptions if necessary.
SECTION I
2. Solve any four :

(45=20)

a) With the help of neat block diagram explain generalised measurement system.
b) Explain the working of repulsion type moving iron instrument. Derive the general torque equation.
c) A moving coil instrument gives a full scale deflection of 10 mA when a potential difference across its
terminals is 100 mV calculate :
a) the shunt resistance for a full scale deflection corresponding to 100 A.
b) the series resistance for full scale reading with 100 V.
d) A wattmeter has a current coil of 0.03 resistance and pressure coil of 6000 resistance
calculate the % error if the wattmeter is so connected that
a) current coil is on load side
b) pressure coil is on load side, when input is 3000 W and current is 12 A at 250 V.
e) With the help of neat diagram, explain working of electrical resonance type frequency meter. State
the limitations of the meter.
f) What are three essential torques acting on the indicating instruments for its satisfactory operation ?
Explain the function of each in brief.
3. Solve any two :

(210=20)

a) Explain how the megger is used for insulation resistance testing.


b) An energy meter is designed to make 200 revolutions of the disc for one unit of energy. Calculate
the no. of revolutions made by it when connected to a load carrying 50 A at 230 V and 0.6 power
factor for an hour. If it actually makes 360 revolutions. Find % error.
c) Explain Weston frequency meter with a neat diagram.

Set A

SLR-BB 293

-4-

*SLRBB293*

SECTION II
4. Solve any four :

(45=20)

a) Explain sharing bridge for capacitance measurement. Draw phasor diagram.


b) Draw the block diagram of integrating type DVM. Explain working of each block.
c) Explain procedure of standardisation of d.c. potentiometer.
d) Define the following terms of instrument transformer :
i) Burdon of an instrument transformer
ii) Actual transformation ratio
iii) Nominal transformation ratio
iv) Turns ratio.
e) Explain Drysdale A.C. potentiometer with a neat diagram.
f) With a neat block diagram, explain digital multimeter.
5. Solve any two :

(210=20)

a) Explain with neat block diagram the working of general purpose CRO.
b) Draw the equivalent circuit and phasor diagram of a potential transformer.
c) A maxwells inductance comparison bridge is shown in fig. Arm AB consists of a coil with inductance L1
and resistance r1 in series with a non-inductive resistance R. Arm BC and CD are each a
non-inductive resistance of 100 . Arm AD consists of standard variable inductor L of resistance 32.7 .
Balance is obtained when L2 = 47.8 mH and R = 1.36 . Find resistance and inductance of the coil
in arm AB.

_____________________

Set A

SLR-BB 294

*SLRBB294*
Seat
No.

Set

S.E. (Part II) (Electrical Engg.) Examination, 2014


ELECTRICAL MACHINES II (Old)
Day and Date : Thursday, 27-11-2014
Time : 3.00 p.m. to 6.00 p.m.

Max. Marks : 100

Instructions : 1) Q. No. 1 is compulsory. It should be solved in first 30 minutes


in Answer Book Page No. 3. Each question carries one mark.
5) Answer MCQ/Objective type questions on Page No. 3 only.
Dont forget to mention, Q.P. Set (A/B/C/D) on Top of Page.
MCQ/Objective Type Questions
Duration : 30 Minutes

Marks : 20

1. Attempt all :

(120=20)

1) A 3-phase 50 Hz induction motor runs at speed of 940 RPM; the speed of the
rotating magnetic field will be
a) 940 RPM
b) 1000 RPM
c) 1050 RPM
d) 1100 RPM
2) Iron losses in the rotor of a 3-phase induction motor are negligible, because
a) Frequency of rotor emf is too low
b) Flux linking the rotor is of constant magnitude
c) Flux density in the rotor parts is too low
d) None of the above
3) The term cogging is associated with
a) Transformers
c) DC series Motor

b) Compound generator
d) Induction motor

4) The no-load current of a 3-phase induction motor in terms of its full-load current is
of the order of
a) 5%
b) 10%
c) 25%
d) 50%
5) A capacitor start capacitor run single phase induction motor will usually have
a) Low p.f.
b) High p.f.
c) Low efficiency
d) High starting torque
6) The torque developed by a split phase motor is proportional to
a) Sine of angle between lm and ls
b) Cosine of angle between lm and Is
c) Main winding current, Im
d) Auxiliary winding current, Is
7) Large air-gap in 3-phase induction motor
a) increases pf
b) decreases pf
c) unaffected on pf
d) none of the above
P.T.O.

SLR-BB 294

*SLRBB294*

-2-

8) Which of the following quantity in squirrel cage induction motor does not depends
on its slip ?
a) Reactance
b) Resistance
c) EMF
d) Frequency
9) For high starting torque, the most 3-phase induction motor is
a) Squirrel-cage type
b) Slip-ring type
c) Deep bar squirrel-cage type
d) Double-cage induction motor
10) The crawling in the induction motor is caused by
a) Improper design of the machine
b) Low supply voltage
c) High loads
d) Harmonics developed in the motor
11) 4 pole, 1500 RPM alternator generates A.C. Voltage of frequency
a) 50 Hz
b) 60 Hz
c) 80 Hz
d) 100 Hz
12) Rating of alternator is expressed in
a) KW
b) HP

c) KVA

13) Alternator field is excited by


a) AC supply
c) Pulse supply

b) DC supply
d) Triangular wave supply

d) KVAR

14) KV A rating of alternator delivering a load of 100 KW at 0.8 p.f is


a) 80 KVA
b) 100 KVA
c) 200 KVA
d) 0 KVA
15) The synchronous motor is operating at 0.5 p.f leading. The excitation is called
a) Over
b) Under
c) Normal
d) Super
16) Magnetizing current drawn by induction motor is the cause of their _________
power factor.
a) zero
b) unity
c) lagging
d) leading
17) Salient poles are generally used on
a) high speed prime movers only
c) low speed prime movers only

b) medium speed prime movers only


d) low and medium speed prime movers

18) Magnitude of EMF generated by an alternator depends upon


a) Nos. of poles
b) Rotor speed
c) Flux per pole
d) All of the above
19) Syn. motor is working on over excited condition, then machine acts as
a) Capacitor
b) Inductor
c) Resistor
d) None
20) The power factor of an alternator depends on
a) Load
b) Speed of rotor
c) Core losses
d) Armature losses
______________

Set A

*SLRBB294*

-3-

SLR-BB 294

Seat
No.

S.E. (Part II) (Electrical Engg.) Examination, 2014


ELECTRICAL MACHINES II (Old)
Day and Date : Thursday, 27-11-2014
Time : 3.00 p.m. to 6.00 p.m.

Marks : 80

SECTION I
2. Attempt any four :

(45=20)

a) What is armature reaction in alternators and what effect will it produce for
R-C load ?
b) Explain parallel operation of two alternators.
c) Write the industrial applications of synchronous motor.
d) Derive the power equation for smooth cylindrical rotor alternator.
e) Derive EMF equation of alternator with short pitched coils and distributed
winding.
3. Attempt any two :

(210=20)

a) With neat vector diagram explain V and inverted V curves of synchronous


motor. Draw both the curves.
b) Draw neat vector diagram of salient pole alternator and derive expression
for power generated in alternator and draw P Vs characteristics.
c) Find the no load line voltage of a star connected 4 pole, 50 hz alternator
from the following data :
Flux per pole = 0.12 Wb; No. of slots/pole = 4; conductors/slot = 4; two
layer winding with coil span 150 degrees.

Set A

SLR-BB 294

-4-

*SLRBB294*

SECTION II
4. Attempt any four :

(45=20)

a) Derive the condition for maximum running torque of an induction motor.


b) An 8-pole, 3-phase, 50 Hz induction motor running with a slip of 4% is taking
20 kW. Stator losses amount to 0.5 kW. If the mechanical torque lost in
friction is 16.25 N-m, find
i) BHP
ii) Efficiency.
c) What is the necessity of starters ? Explain about star-delta starter with neat
diagram. Derive the relation between the Tst and Tfl.
d) Derive Pi : Pc : Pm = 1 : S : (1-S). Where Pi = Rotor input; Pc = Rotor cu loss
and Pm = Mechanical power developed.
e) Draw slip-torque characteristics of 3-phase induction motor when rotor
resistance R2 is varied. Explain the shapes of the curve why are modified in
such shapes.
5. Attempt any two :

(210=20)

a) Give the constructional details of 3 phase induction motor with suitable


sketches.
b) A 100 kW, 3300 V, 50 Hz, 3-phase, Y connected induction motor has a
Ns = 500 rpm. The full-load slip is 1.8% and full load p.f 0.85. Stator
Cu loss = 2440 W. Iron loss = 3500 W. Rotational losses = 1200 W.
Calculate
i) The rotor Cu loss
ii) The line current.
c) Explain how rotating magnetic field is created when 3-phase supply voltage
is given to the stator winding. Give the important assumptions in this case.
_____________________

Set A

SLR-BB 295

*SLRBB295*
Seat
No.

Set

S.E. (Part II) (Electrical) (Old) Examination, 2014


SIGNALS AND SYSTEM
Day and Date : Friday, 28-11-2014
Time : 3.00 p.m. to 6.00 p.m.

Max. Marks : 100

Instructions : 1) Q. No. 1 is compulsory. It should be solved in first 30 minutes in


Answer Book Page No. 3. Each question carries one mark.
2) Answer MCQ/Objective type questions on Page No. 3 only.
Dont forget to mention, Q.P. Set (A/B/C/D) on Top of Page.
MCQ/Objective Type Questions
Duration : 30 Minutes

Marks : 20

1. Choose the correct answer :

20

1) The signal which exists only at t = 0, whose area is referred as continuous time
unit ____________
a) Impulse
b) Time
c) Series
d) Level
2) L{cos(at)} =
s
a) s 2 + a 2

a
b) s 2 + a 2

c)

s2
s + a2
2

d)

a2
s + a2
2

3) The convolution of two DT sequence x[n] = {1, 1, 1} and h[n] = {1, 0, 1}


a) {1, 1, 0, 1, 1}
b) {1, 1, 2, 1, 1}
c) {2, 0, 1, 2, 1}
d) {0, 1, 1, 1, 1}
4) For a sinusoidal signal the value of cos t is equal to
e jt e jt
e jt + e jt
e jt + e jt
e jt e jt
a)
b)
c)
d)
2j
2j
2
2
5) Mathematically discrete time signal can be obtained as
a) [n] = u[n]
b) [n] = u[n] u[n 1]
c) [n] = u[n] + u[n + 1]
d) [n] = u[n] u[n 2]
6) The fundamental frequency of x(n) = e j6 n is,
a) w 0 = 2
b) w 0 = 3
c) w 0 = 2

d) w 0 = 6

7) The energy and power of signal x(n) = 1 u(n) is,


a) E =

9
;P=0
8

b) E =

8) The given system F(s) =


a) Stable
c) Marginally stable

7
;P=0
8

1
(s + 2) (s + 3)

c) P = 98 ; E = 0

d) P = 78 ; E = 0

is
b) Unstable
d) None of above
P.T.O.

SLR-BB 295

*SLRBB295*

-2-

9) In a system, if present output depends only on present and past inputs, such systems
is called as,
a) Non causal
b) Anticipatory
c) Causal
d) Static
10) The system y(n) = x(n) is
a) Time-variant
b) Time-invariant c) Both a) and b)

d) None of above

11) ROC is defined as


a) Range of values for which Z-transform converges
b) Range of values for which Z-transform diverges
c) Range of values of n for which series converges
d) Range of values of n for which series diverges
12) ROC of causal LTI system is
a) Exterior of the circle in the z-plane
c) On the circle in the z-plane

b) Interior of the circle in the z-plane


d) Both a) and b)

13) The sampling theorem is applicable to continuous time


a) Band limited signal
b) Band undefined
c) Any signal
d) Stochastic signal
14) The region of convergence of the z-transform of a unit step function is
a) |z| > 1
b) |z| < 1
c) (Real part of z) > 0
d) (Real part of z) < 0
15) The Nyquist sampling rate for the signal g(t ) = 10 cos( 50 t) cos 2 (150 t ) where t
is in seconds is
a) 150 samples/sec
b) 200 samples/sec
c) 300 samples/sec
d) 350 samples/sec
16) Zero padding are
a) Zero appearing in the X[k] sequence
c) Dummy sample added with value of 0 in X[k]

b) Value of X[k] is zero


d) Both a) and b)

17) If x[n] is real and odd, then its discrete Fourier series coefficient ck will be
a) Real
b) Odd
c) Imaginary
d) Both a) and b)
18) Fourier transform of x[n] is
a) X( )
b) X( )

c) X( )

d) Zero

19) If the output of discrete time LTI system is always identical to the input signal then
the impulse response h(n) is
a) Unit step
b) Unit impulse
c) All one
d) Ramp
20) The number of complex multiplication required to calculate N-point DFT using
radix-2 DIT-FFT algorithm is
a) N log2 N

N
b) N log10
c) N log10 N
2
______________

d) N log2 N
2

Set A

*SLRBB295*

-3-

SLR-BB 295

Seat
No.

S.E. (Part II) (Electrical) (Old) Examination, 2014


SIGNALS AND SYSTEM
Day and Date : Friday, 28-11-2014
Time : 3.00 p.m. to 6.00 p.m.

Marks : 80

SECTION I
2. Attempt any four :

(45=20)

1) Find whether following systems are periodic or not :


a) x(t) = sin2t
b) x[n] = cos 0.5 n.
2) Determine even and odd component of x(t) = cos t + sin t.
3) Determine given signal is energy or power signal.
a) f(t) = e3|t|
b) f(t) = e3t
4) For the waveform shown find Laplace transform.

5) Sketch the following signals :


1) u(t + 1)
2) 2u (t 1)
3. a) Determine convolution sum of two sequence x(n) = {1, 2, 3, 4} and h(n) = {1, 0, 1, 1}. 10
b) Plot the poles and zeros for F(s) = 4

(s + 1) (s + 3)
and comment on stability.
( s + 2 ) ( s + 4)

10

OR
b) State and prove differentiation property of Laplace transform.

10

Set A

SLR-BB 295

-4-

*SLRBB295*

SECTION II
4. Solve any four :

(45=20)

1) Determine the Nyquist sampling rate and Nyquist sampling interval for the signal
sin c2(200 t).
2) Find z-transform of x(n) = u(n) u(n 1).
3) Explain sampling theorem and Nyquist rate.
4) Explain properties of region of convergence for z plane.
5) Find inverse Fourier transform of x(n) = (a)|n|.
5. Solve any two :

(210=20)

1) Determine exponential Fourier series representation of full wave rectified sine


wave also plot line spectrum.
2) Find Fourier transform of following signals.

3) Plot the pole zero pattern and determine stability of the given system.
1) y(n) = y(n 1) 0.5y(n 2) + x(n) + x(n 1)
2) y(n) = 1.8y(n 1) 0.72y(n 2) + x(n) + 0.5x(n 1).
_____________________

Set A

SLR-BB 296

*SLRBB296*
Seat
No.

Set

S.E. (Electrical) (Part II) (Old) Examination, 2014


BASIC CIRCUIT THEORY
Day and Date : Saturday, 29-11-2014
Time : 3.00 p.m. to 6.00 p.m.
Instructions :

Max. Marks : 100

1) Q. No. 1 is compulsory. It should be solved in first 30 minutes in


Answer book Page No. 3. Each question carries one mark.
2) Answer MCQ/Objective type questions on Page No. 3 only. Dont
forget to mention, Q.P. Set (A/B/C/D) on Top of Page.
MCQ/Objective Type Questions

Duration : 30 Minutes

Marks : 20

1. Choose the correct option :

(201=20)

1) Resistance across A and B in the circuit shown in figure below is

a) R

b) 3R

c) 4R

d) 5R

2) The nodal method of circuit analysis is based on


a) KVL and ohms law
b) KCL and ohms law
c) KCL and KVL
d) KCL, KVL and ohms law
3) The number of independent loops for a network with n nodes and b branches is
a) n 1
b) b n
c) b n + 1
d) Independent of the number of nodes
4) A circuit contains two unequal resistances in parallel
a) Current is same in both
b) Large current flows in larger resistor
c) Potential difference across each is same
d) Smaller resistance has smaller conductance
5) Resistance of a wire is r ohms. The wire is stretched to double its length, the
resistance in ohms is
a) r/2
b) 4r
c) 2r
d) r/4
6) Two resistances R1 and R2 give combined resistance of 4.5 ohms when in series and
1 ohm when in parallel. The resistances are
a) 3 ohms and 6 ohms
b) 3 ohms and 9 ohms
c) 1.5 ohms and 3 ohms
d) 1.5 ohms and 0.5 ohms

P.T.O.

SLR-BB 296

*SLRBB296*

-2-

7) Three 3 ohm resistors are connected to form a triangle. What is the resistance between
any two of the corners ?
a) 3/4 ohm
b) 3 ohm
c) 2 ohm
d) 4/3 ohm
8) Two two-port networks are connected in cascade. The combination is to be represented
as a single two-port network. The parameters are obtained by multiplying the individual.
a) z-parameter matrix
b) h-parameter matrix
c) y-parameter matrix
d) ABCD parameter matrix
9) A two-port network is defined by the following pair of equations I1 = 2V1+V2 and I2 = V1 + V2.
Its impedance parameters (Z11, Z12, Z21, Z22) are given by
a) 2, 1, 1, 1
b) 1, 1, 1, 2
c) 1, 1, 1, 2
d) 2, 1, 1, 1
10) If one of the resistors in a parallel circuit is removed, what happens to the total resistance ?
a) Decreases
b) Increases
c) Remain constant d) Exactly double
11) Four equal value resistors are connected in parallel. 5V voltage is applied across the
parallel circuit, and 2.5 mA current are measured from the source. What is the value of
each resistor ?
a) 4
b) 8
c) 2.5
d) 5
12) The number of branches in a tree is __________ number of branches in a graph.
a) Less than
b) More than
c) equal to
d) Half
13) Inductor does not allow sudden changes in
a) Currents
b) Voltages
c) Both a) and b)
d) None of the above
14) The time constant of a series RL circuit is
a) LR
b) L/R

c) R/L

d) e R/L

15) The time constant of a series RC circuit is


a) 1/RC
b) R/C

c) RC

d) e RC

16) Which parameters are widely used in transmission line theory ?


a) Z
b) Y
c) ABCD

d) h

17) Superposition theorem is not applicable to network containing.


a) Nonlinear elements
b) Dependent voltage source
c) Dependent current source
d) Transformers
18) The reciprocity theorem is applicable to
a) Linear networks only
c) Linear or bilateral networks

b) Bilateral networks only


d) None of these

19) When the superposition theorem is applied to any circuit, the dependent voltage source in
that circuit is always
a) Opened
b) Shorted
c) Active
d) None of the above
20) While applying thevenins theorem, the thevenins voltage is equal to
a) Short circuit voltage at the terminals
b) Open circuit voltage at the terminals
c) Voltage of the source
d) Total voltage available in the circuit

______________

Set A

*SLRBB296*

SLR-BB 296

-3-

Seat
No.

S.E. (Electrical) (Part II) (Old) Examination, 2014


BASIC CIRCUIT THEORY
Day and Date : Saturday, 29-11-2014
Time : 3.00 p.m. to 6.00 p.m.

Marks : 80
SECTION I

2. Solve any four :


a) Find the current through 3+j4 impedance.

(54=20)

b) Find voltage Vx.

c) Obtain thevenions equivalent n/w for the fig. show.

d) Explain Incidence matrix and cutset matrix.


e) Find the duality of n/w.

3. a) In the n/w show in fig. a steady state is reached with the switch open at t = 0
switch is closed. Determine Vc(0), i1(0+), i2(0+),

di1 +
di2 +
(0 ) and
(0 ).
dt
dt

10

Set A

SLR-BB 296

-4-

*SLRBB296*

b) For the n/w shown in fig. obtain equilibrium equation on node basis.

10

OR
b) Find Nortons equivalent n/w and find current in 10 resistor.

SECTION II
4. Solve any four of the following :
a) Write reciprocity and symmetry condin. in transmission parameter and prove it.
b) Find Z parameters for the n/w shown in fig.

10

(54=20)

c) Determine Y parameters for the n/w shown in fig.

d) In the n/w of fig. switch is moved from a to b at t = 0 steady state conditions having been
reached at position a) Determine i(t) in s domain.

e) Explain Y parameter interms of transmission parameters.


5. Solve any two :
(102=20)
a) For the n/w shown in fig., determine the current i(t) when the switch is closed at t = 0
with zero initial condition.

b) Find Z and n parameters for the n/w shown in fig.

c) In the n/w of fig. switch is moved from 1 to 2 at t = 0 determine i(t).

Set A
_____________________

SLR-BB 297

*SLRBB297*
S

S.E. (Electrical) (Part II) Examination, 2014


COMPUTER PROGRAMMING C++ (Old)
Day and Date : Monday, 1-12-2014
Time : 3.00 p.m. to 6.00 p.m.

Max. Marks : 100

Instructions : 1) Q. No. 1 is compulsory. It should be solved in first 30


minutes in Answer Book Page No. 3. Each question carries
one mark.
2) Answer MCQ/Objective type questions on Page No. 3
only. Dont forget to mention, Q.P. Set (A/B/C/D) on Top
of Page.
Duration : 30 Minutes

MCQ/Objective Type Questions

Marks : 20

1. Choose correct alternatives :


1) Which of the following is a relational operator ?
A) = =
B) ! =
C) <

20
D) All above

2) _________ provides an alternative name for standard data types.


A) enum
B) typedef
C) preperative
D) all
3) Which can be passed as an argument to a function ?
A) expression B) another function C) constant
D) all of above
4) A copy constructor takes __________
A) one argument B) no argument C) two arguments D) any no. of arguments
5) A white space in C++ is
A) blank space
B) new line
C) tab
6) What will be the result of expression 13 and 25 ?
A) 25
B) 9
C) 12

D) all of above
D) 38

7) Which is the correct way to access a data member using this pointer ?
A) this.X
B) * this.X
C) (* this).X
D) * (this.X)
8) What is the error in the following code ?
class t
{
virtual void print ( );
}
A) print ( ) should be declared static B) No error
C) Class t definition wrong
D) all above

P.T.O.

SLR-BB 297

*SLRBB297*

-2-

9) A pure virtual function is a virtual function that


A) has no body
B) return nothing C) cant say

D) none of above

10) An array element is accessed using ___________


A) an index number
B) the operator
C) a name of member
D) none of above
11) A library function exit ( ), does following thing
A) exit from loop
B) exit from function
C) exit from program
D) none of above
12) By default, the member of a class is
A) public
B) protected
C) private
D) all
13) Which operator is not used with pointer ?
A) : :
B) &
C) *
D)
14) In multiple inheritance, can not have virtual classes
A) True
B) False
C) Cant say
D) None of above
15) We can declare as a friend
A) a member function
B) a global function
C) a class
D) all above


16) Which of the following cannot be passed as parameter to function ?


A) header file
B) structure
C) variable
D) constant
17) Both function and operator overloading are type of
A) compile-time polymorphism
B) data hiding
C) run-time polymorphism
D) inheritance
18) The object which is currently invoking the member function can be pointed by
A) base class
B) this pointer
C) pointer to pointer
D) all
19) _________ cannot return a value.
A) Constructor
B) Destructor

C) Function

D) Both A) and B)

20) A virtual class is the same as


A) abstract class
B) class with virtual function
C) a base class
D) none of above
______________

Set A

*SLRBB297*
S

-3-

SLR-BB 297

S.E. (Electrical) (Part II) Examination, 2014


COMPUTER PROGRAMMING C++ (Old)
Day and Date : Monday, 1-12-2014
Time : 3.00 p.m. to 6.00 p.m.

Marks : 80

Instruction : All questions are compulsory.


SECTION I
2. Attempt any four :

(45=20)

A) Explain the applications and benefits of OOP.


B) Define and explain encapsulation and data abstraction in OOP.
C) Define structure. Write the syntax for specifying the structure and
initializing its members with example.
D) Explain object as function argument with example.
E) What is inline function ? Explain the advantage of using inline function in
program.
F) Explain while and do while loop statement in C++.
3. Attempt any one :

10

A) Explain the concept of constructor and destructor in detail.


B) What is function overloading ? Write a program to overload volume ( ) function
to find volume of sphere, cylinder.
4. Write a program to create a class time with data members : minutes, seconds
and member functions : To read the time from user, To add the times and to
display the time.
Also write the main function to test the program.

10

Set A

SLR-BB 297

-4-

*SLRBB297*

SECTION II
5. Attempt any four :

(45=20)

A) What is inheritance ? Explain its types.


B) Explain the concept of this pointer.
C) Explain the concept of array of objects.
D) Explain the syntax of defining derived constructor with suitable example.
E) WAP to find largest from array of 5 elements using pointer.
F) Explain the concept of pointer to derived classes with example.
6. Attempt any one :

10

A) Explain polymorphism in terms of virtual function with example.


B) WAP to overload + operator to add two complex objects with data
members : real, image.
7. Using Friend Function add ( ) write a program to add two objects of class
distance. The data members are feet, inch. The member functions are
get Distance( ), putDistance( ).

10

_____________________

Set A

SLR-BB 298

*SLRBB298*
Seat
No.

Set

S.E. (Elect.) (Part II) (New) Examination, 2014


NUMERICAL METHODS AND COMPUTER PROGRAMMING
Day and Date : Tuesday, 25-11-2014
Time : 3.00 p.m. to 6.00 p.m.

Max. Marks : 100

N.B. : 1) Q. No. 1 is compulsory. It should be solved in first 30 minutes in


Answer book Page No. 3. Each question carries one mark.
2) Attempt any three questions from each Section.
3) Use of non-programmable calculator is allowed.
4) Use suitable data, if required.
5) Answer MCQ/Objective type questions on Page No. 3 only. Dont
forget to mention, Q.P. Set (A/B/C/D) on Top of Page.
MCQ/Objective Type Questions
Duration : 30 Minutes

Marks : 20

1. Choose the correct alternative :


1) Lin-Bairstows method is used to find
a) Linear Factor
b) Cubic Factor

20
c) Quadratic Factor

d) None of these

2) In case of bisection method, the convergence is


a) Quadratic
b) Linear
c) Very fast

d) Very slow

3) The Newton-Raphson method fails when


a) f (x) < 0
b) f (x) = 0

d) never fails

c) f (x) is too large

4) Newtons iterative formula to find approximate value of N where N is a positive real


number is

a) xn+1 = 2 xn +
x

n
c) xn+1 = xn (2 Nxn)

N
xn +

xn
2
1
1

d) xn+1 = xn +
2
Nxn
b) xn+1 =

5) When Gauss-elimination method is used to solve set of equation AX = B, matrix A is


transformed to
a) Diagonal matrix
b) Identity matrix
c) Upper triangular matrix
d) None of these
6) In factorization method of solving simultaneous linear equations coefficient matrix is
expressed as
a) Product of lower and upper triangular matrices
b) Sum of lower and upper triangular matrices
c) Quotient of lower and upper triangular matrices
d) None of these
7) If y = 2x + 5 is the best fit for 8 pairs of values of (x, y) by the method of least squares and
y = 120 then x = _____________
a) 35
b) 40

c) 45

d) 30

P.T.O.

SLR-BB 298

*SLRBB298*

-2-

8) Newtons divided difference formula is used to find polynomial, if given values of x are
a) Equally spaced
b) Odd numbers
c) Unequally spaced
d) Even numbers
9) If y = x2 + 1, then the first divided difference of the argument 1 and 2 is
a) 1
b) 2
c) 1
d) 3
10) To fit a polynomial a0 + a1x + a2x2 + a3x3 to the given data of n observations we require
_____________ number of normal equations.
a) 3
b) 2
c) 4
d) n
11) For the given data

1
0 0.5 1
.The value of I = y dx by trapezoidal rule is
1 0.8 0.5
0

x:
y:

a) 0.275

b) 0.575

c) 0.755

d) 0.775

c)

d) none of these

1 1 i
12) Eigen values of the matrix
are
1 + i 1
a) 1 + 2 i, 1 2 i

b) 1 + 2, 1 2

13) Rombergs method is used to solve


a) Ordinary differential equation
c) Integration
14) To apply Simpsons

3
8

a) Even

2 i, 2 i

b) Partial differential equation


d) None of these

th
rule the number of sub-intervals must be
b) Odd

c) Multiple of 3

d) Any

15) If any of the Gerschgorin circle is isolated then it contains


a) All eigen values
b) Two eigen values
c) No eigen values
d) Exactly one eigen value
16) While finding eigen values of matrix A. Which of the following method consists of
diagonalization of A ?
a) Power method
b) Jacobis method
c) Gerschgorin method
d) None of these
17) The equation ut = 2 u xx is
a) Elliptic
b) Hyperbolic

c) Parabolic

d) Poisson

18) Wave equation is represented by


b) utt c2 uxx = 0
a) utt = uxx

c) ut c2 uxx = 0

d) None of these

19) The partial differential equation


a) Heat equation
c) Laplaces equation

2
2
u u
+
= f ( x, y ) is called
2
2
x
y
b) Wave equation
d) Poissons equation

20) Which of the following is not the method of numerical solution of ordinary differential
equation ?
a) Taylors series method
b) Crank-Nicolson method
c) Runge-Kutta method
d) Picards method
______________
Set A

*SLRBB298*

-3-

SLR-BB 298

Seat
No.

S.E. (Elect.) (Part II) (New) Examination, 2014


NUMERICAL METHODS AND COMPUTER PROGRAMMING
Day and Date : Tuesday, 25-11-2014
Time : 3.00 p.m. to 6.00 p.m.

Marks : 80

N.B. : 1) Attempt any three questions from each Section.


2) Use of non-programmable calculator is allowed.
3) Use suitable data, if required.
SECTION I
2. a) Find a real root of the equation x log10 x = 1.2 by regula falsi method correct to four
decimal places. (Take (2, 3) as search interval).
b) Find by Newtons method, the real root of the equation 3x = cos x + 1 taking x0 = 0.6 as an
initial approximation.
c) Find a root of f (x) = ex 3x = 0 by using Bisection method. (perform five Iterations).
3. a) Apply Gauss-Jordan method to solve the equations
x + y + z = 9, 2x 3y + 4z = 13, 3x + 4y + 5z = 40.
b) Solve the equations 5x + 2y + z = 12, x + 4y + 2z = 15, x + 2y + 5z = 20 by Jacobis
method (perform four iterations only).
c) Use LU factorization method to solve 2x + 3y + z = 9, x + 2y + 3z = 6, 3x + y + 2z = 8.
x:
61 26
7
2.6
.
y : 350 400 500 600
b) Use Lagranges interpolation formula to find the value of y when x = 10 if the following
x : 5 6 9 11
.
values of x and y are given
y : 12 13 14 16
c) Use Newtons divided difference formula to find f (x) from the following data :

4. a) Fit a curve y = axb to the following data

x:
f (x) :

0
1

1
14

2
15

4
5

5
6

4
4
5
4
4
6
4

6
.
19

OR
c) The result of measurement of electric resistance R of a copper bar at various temperatures
t C are listed below :

t : 19 25 30 36 40 45 50
.
R : 76 77 79 80 82 83 85
Find a relation R = a + bt where a and b are constants to be determined.
5. a) Find the cubic splines for the following table of values :
x: 1 2
3
.
y : 6 1 16
b) Find the approximate quadratic factor of polynomial f (x) = x3 2x2 + x 2 using
Lin-Bairstows method. (Perform only one iteration).

7
Set A

SLR-BB 298

*SLRBB298*

-4-

SECTION II
1

6.

a) Use Rombergs method to compute

dx

1+ x

(Take h = 0.5, 0.25 and 0.125).

b) For the following values of x and y find f (2 ) and f (2.04 )


1.96
1.98
x:
f (x) : 0.7825 0.7739
7.

2
2.02
2.04
0.7651 0.7563 0.7473

a) Determine the largest eigen value and its corresponding eigen vector of the matrix A using

1
2 1 0

power method A = 1 2 1 (Take 0 as an initial approximation).

0
0 1 2

b) Using Jacobis method, find all the eigen values and eigen vectors of the matrix

1
2

2
8.

a) Solve the equation 2u = 0 for the following mesh by using Leibmanns method.
(Perform Four Iterations) :
0
0
500 1000 500

1000
2000
1000

1000
2000
1000
0

500 1000

500

b) Apply Runge-Kutta method to find approximate value of y for x = 0.2 in steps of 0.1 if
dy
2
= x + y given that y = 1 when x = 0.
dx
9. a) Using Eulers method solve for y at x = 0.1 from

dy
dx

= x + y + xy , y (0) = 1 taking h = 0.25. 4

th
dx
3
b) Evaluate 1 + x taking 7 ordinates by applying Simpsons
rule.
0
8
c) Find the bounds of eigen values of the following matrix using Gerschgorin circle

1 2 1

theorem 1 1 1 .
1 3 1

OR
c) Define eigen values and eigen vectors of a matrix.

_____________________

Set A

SLR-BB 299

*SLRBB299*
Seat
No.

Set

S.E. (Part II) (Electrical) (New) Examination, 2014


ELECTRICAL MACHINES II
Day and Date : Wednesday, 26-11-2014
Time : 3.00 p.m. to 6.00 p.m.

Max. Marks : 100

N. B. : 1) Assume suitably the missing data.


2) Q. No. I is compulsory. It should be solved in first 30 minutes
in Answer book Page No. 3. Each question carries one mark.
3) Answer MCQ/Objective type questions on Page No. 3 only.
Dont forget to mention, Q.P. Set (A/B/C/D) on Top of Page.

MCQ/Objective Type Questions


Duration : 30 Minutes
I.

Marks : 20

Tick the correct answer :


(120=20)
1) Regarding skewing of motor bars in a squirrel cage induction motor, which
statement is false ?
a) It prevents cogging
b) It increases starting torque
c) It produces more uniform torque
d) It reduces motor hum during its operation
2) The magnetising current drawn by transformers and induction motors is the cause
of their ____________ power factor.
a) zero
b) unity
c) lagging
d) leading
3) The effect of increasing the length of air-gap in an induction motor will be to increase
the
a) power factor
b) speed
c) magnetising current
d) air gap flux
4) In a 3- induction motor, the relative speed of stator flux with respect to ______ is
zero.
a) stator winding
b) rotor
c) rotor flux
d) space
5) An eight-pole wound rotor induction motor operating on 60 Hz supply is driven at
1800 rpm by a prime mover in the opposite direction of revolving magnetic field.
The frequency of rotor current is
a) 60 Hz
b) 120 Hz
c) 180 Hz
d) None of the above
6) Irrespective of the supply frequency, the torque developed by a squirrel cage
induction motor is the same whenever ____________ is the same.
a) supply voltage
b) external load
c) rotor resistance
d) slip speed
7) The member of stator poles produced in the rotating magnetic field of a 3- induction
motor having 3 slots per pole per phase is
a) 3
b) 6
c) 2
d) 12
P.T.O.

SLR-BB 299

-2-

*SLRBB299*

8) The power factor of a squirrel cage induction motor is


a) Low at light loads
b) Low of heavy loads
c) Low at light and heavy loads
d) Low at rated load
9) Which of the following rotor quantity in a squirrel cage induction motor does not
depend on slip ?
a) reactance
b) speed
c) induced emf
d) frequency
10) The efficiency of a 3- induction motor is approximately proportional to
a) (1-S)
b) S
c) N
d) N S
11) In the circle diagram for a 3- induction motor, the diameter of the circle is
determined by
a) rotor current
b) exciting current
c) total stator current
d) rotor current referred to stator
12) In the circle diagram of an induction motor, point of maximum input lies on the
tangent drawn parallel to
a) output line
b) torque line
c) vertical axis
d) horizontal axis
13) If stator voltage of a squirrel cage induction motor is reduced to 50 percent of its
rated value. Torque developed is reduced by ______________ percent of its
full load value.
a) 50
b) 25
c) 75
d) 57.7
14) In a double cage induction motor outer cage is made of high resistance metal bars
primarily for the purpose of increasing its.
a) speed regulation b) starting torque c) efficiency
d) string current
15) A squirrel cage induction motor with 36 slots stator has two separate windings :
one with 3 coil groups/phase/pole and the other with 2 coil groups/phase/pole.
The obtainable two motor speeds would be in the ratio of
a) 3 : 2
b) 2 : 3
c) 2 : 1
d) 1 : 2
16) One of the characteristics of a 1- motor is that it
a) is self starting
b) is not self starting
c) requires only one winding
d) can rotate in one direction only
17) After the starting winding of a 1- induction motor is disconnected from supply,
it continues to run only on ___________ winding.
a) rotor
b) compensating c) field
d) running
18) If starting winding of a 1- induction motor is left in the circuit it will
a) draw excessive current and overheat
b) run slower
c) run faster
d) spark of light loads
19) If a 1- induction motor runs slower than normal, the more likely defect is
a) improper fuses
b) shorted running winding
c) open starting winding
d) worn bearings
20) The starting torque of a capacitor-start induction motor is directly related to the
angle between its tow winding currents by the relation.
a) cos
b) sin
c) tan
d) sin /2

______________

Set A

*SLRBB299*

-3-

SLR-BB 299

Seat
No.

S.E. (Part II) (Electrical) (New) Examination, 2014


ELECTRICAL MACHINES II
Day and Date : Wednesday, 26-11-2014
Time : 3.00 p.m. to 6.00 p.m.

Marks : 80

N. B. : Assume suitably the missing data.

SECTION I
II. Answer any four questions :

(54=20)

1) With a neat sketch explain the constructional details of 3- induction motor.


2) Why does the rotor of an induction motor rotate explain briefly ?
3) Calculate the torque exerted by an 8 pole, 50 Hz, 3- induction motor operating
with 4% slip which develops a maximum torque of 150 kg. mt. at a speed of
660 rpm the resistance per phase of the rotor is 0.5 .
4) The power input to the rotor of a 440 V, 50 Hz, 6 pole 3- induction motor is
100 kW. The rotor electromotive force is observed to make 120 cycles per minute,
calculate : (i) the slip (ii) the rotor speed (iii) mechanical power developed (iv) the
rotor cu loss per phase and (v) speed of stator field with respect to rotor.
5) What are the different methods of speed control of induction motors explain with
neat diagram the rotor rheostat control method ?
III. Answer any two questions :

(210=20)

1) With a neat vector diagrams explain the production of rotating magnetic field in
3- induction motor.
2) Derive an expressions for starting, torque, running torque and maximum torque.
3) The rotor resistance and stand still reactance of a 3- induction motor are
respectively 0.015 and 0.09 per phase at normal voltage, the full load slip is
3% estimate the percentage reduction in stator voltage to develop full load torque
at half full load speed also calculate the power factor.
Set A

SLR-BB 299

-4-

*SLRBB299*

SECTION II
IV. Answer any four questions :

(54=20)

1) Draw the approximate equivalent circuit diagram in order to draw a circle diagram
and explain briefly.
2) With a neat sketch explain the DOL stater.
3) A squirrel cage type induction motor when started by means of a star/Delta starter
takes 180% of full load line current and develops 35% of full load torque at starting
calculate the starting torque and current in terms of full load values, if an
autotransformer with 75% tapping were employed.
4) With a neat sketch explain the double field revolving theory in 1- induction motor.
5) With a neat diagram explain how the 1- induction motor is self starting.
V. Answer any two questions :

(210=20)

1) With a neat sketches explain the constructional details of a circle diagram.


2) Draw the circle diagram of a 7.46 kW, 200 V, 50 Hz, 3- slip ring induction motor
with a star connected stator and rotor, a winding ratio of unity, a stator resistance
of 0.38 /phase and a rotor resistance of 0.24 /phase the following are the
readings :
No. load : 200 V, 7.7 A, cos O = 0.195
Short circuit : 100 V, 47.6 A, cos SC = 0.454.
Find :
i) Starting torque
ii) Maximum torque both in synchronous watts
iii) The maximum power factor
iv) The slip for maximum torque
v) The maximum out put.
3) Draw a equivalent circuit diagrams of a 1- induction motor with and without core
loss.
_____________________

Set A

SLR-BB 3

*SLRBB3*
Seat
No.

Set

F.E. (Part I) (New) Examination, 2014


BASIC ELECTRICAL ENGINEERING
Day and Date : Friday, 12-12-2014
Time : 10.00 a.m. to 1.00 p.m.

Max. Marks : 70

Instructions : 1) Q. No. I is compulsory. It should be solved in first 30


minutes in Answer Book Page No. 3. Each question carries
one mark.
2) Answer MCQ/Objective type questions on Page No. 3 only.
Dont forget to mention, Q.P. Set (A/B/C/D) on Top of Page.
MCQ/Objective Type Questions
Duration : 30 Minutes

Marks : 14

I. Choose the correct answer :


1) Thevenin equivalent circuit is
a) Current source in series with equivalent resistance
b) Current source in parallel with equivalent resistance
c) Voltage source in series with equivalent resistance
d) Voltage source in parallel with equivalent resistance
2) Three 3 ohms resistors are connected in the form of equilateral triangle. The
total resistance between any two corners is
a) 2 ohms

b) 6 ohms

c) 3 ohms

d) 4/3 ohms

3) The equivalent of 1 joule is __________


a) Watt-hour

b) Kwh

c) Watt-sec

d) Joule-sec

4) The rating of electric lamp is 220V, 100 W. If it is operated at 110 V, the


power consumed by it will be
a) 50 W

b) 75 W

c) 90 W

d) 25 W

5) Coefficient of coupling K for magnetic circuit is given by


a) L/ M1 M 2

b) L/M

c) M L1 L 2

d) M/ L1 L 2

6) Self-inductance of coil is proportional to


a) Current
c) Power

b) Voltage
d) Number of turns
P.T.O.

SLR-BB 3

*SLRBB3*

-2-

7) An alternating current is given by i = 14.14 sin 377t. Its time period is


a) 20 ms
b) 16.67 ms
c) 2.65 ms
d) 5.3 ms
8) Ohms law for AC circuit is given by
a) V = IR
b) V = I/Z
c) V = Z/I

d) V = IZ

9) For addition or subtraction of phasors, we use ____________form.


a) Polar
b) Rectangular
c) Trigonometrical

d) None of above

10) An open circuit test on a transformer is performed basically to determine


a) Regulation

b) Copper losses

c) Iron losses

d) Leakage impedance

11) When a 400 Hz transformer is operated at 50 Hz its KVA rating is


a) Reduced to 1/8
c) Increased 8 times

b) Reduced to 50 Hz
d) Unaffected

12) Three identical resistances connected in star consume 4000W. If these three
resistances are connected in delta across the same supply, the power
consumed will be
a) 4000W
b) 6000W
c) 8000W
d) 12000W
13) The phase voltage of delta connected load is 100V then the line voltage is
a) 173 V
b) 73V
c) 100V
d) None
14) To make single phase induction motor self-starting an auxiliary winding is
connected
a) In series with main winding
b) In parallel with main winding
c) It is connected in series with main winding only for starting period
d) In parallel with main winding only for starting period
______________

Set A

*SLRBB3*

SLR-BB 3

-3-

Seat
No.

F.E. (Part I) (New) Examination, 2014


BASIC ELECTRICAL ENGINEERING
Day and Date : Friday, 12-12-2014
Time : 10.00 a.m. to 1.00 p.m.

Marks : 56

Instructions : 1) All questions are compulsory.


2) Assume suitable data if necessary and mention it clearly.
3) Figures to the right indicate full marks.
SECTION I
II. Attempt any four :

(44=16)

a) State and explain Kirchhoffs laws, with neat diagram.


b) A piece of silver wire has resistance of 1 . What will be the resistance of
manganin wire of one-third length and one-third diameter, if specific resistance
of manganin is 30 times that of silver.
c) If resistance of 100 is connected to 200V supply for 10 hours. Calculate:
i) Power

ii) Energy consumed

iii) Cost of energy using it for 10 hours at Rs. 2 per unit.


d) Explain terms-useful flux, magnetic leakage, leakage factor and fringing.
e) Define following terms :
i) Cycle

ii) Time period

iii) Frequency

iv) Wave form

v) Phase difference vi) Form factor vii) Peak factor


f) A 50 resistor is in parallel with 100 resistor. Current in 50 resistor is
7.2 A. How will you add a third resistor and what will be its value if the total
current is to be kept 12.1 A ?
III. Attempt any two :

(62=12)

a) Determine the current in each branch of the network shown in figure.

Set A

SLR-BB 3

-4-

*SLRBB3*

b) An iron ring 15 cm in diameter and 10cm2 in cross section is wound with 200
turns for a flux density of 1 Wb/m2. Assume r = 500 for iron and neglect
leakage and fringing. Find :
i) Reluctance of ring

ii) Flux in ring

iii) MMF required for iron ring

iv) Exciting current

c) Four voltages are represented by e1 = 10 sin ( t), e2 = 8 sin ( t + /3),


e3 = 4 sin ( t /6) and e4 = 6 sin ( t + 3 /4) volts act together in a circuit.
Find an expression for the resultant voltage represented by (e1 e2 + e3 e4).
Draw phasor diagram for (e1 e2 + e3 e4).
SECTION II
IV. Attempt any four :

(44=16)

a) Define and explain :


a) Impedance

b) Admittance

c) Reactance

b) In R-C series circuit, voltage across the combination is given by


40 sin (2000t + 45), R = 10 ohms. The current leads the voltage by /3
radians. Find the value of C. Also find the expression for current.
c) Derive the relation between line and phase voltages and currents in balanced
star connected 3 -phase load.
d) Derive an expression for induced emf in a transformer in terms of frequency,
maximum flux and the number of turns on the winding.
e) A 230/2300 V transformer takes no load current of 5 A at 0.25 power factor
lagging. Find core loss component and magnetising current.
f) Write short notes on transformer losses.
V. Attempt any two :

(62=12)

a) A 100 KVA single phase transformer has iron loss of 400 W. The full load
copper loss is 690 watts. Calculate efficiency at full load and half load at :
i) Unity power factor

ii) 0.8 lagging power factor

b) Explain the working principle of three phase induction motor and its applications.
c) The two impedances (14 + j5) and (18 + j 10) are connected in parallel
across a 200 V, 50 Hz supply. Find :
i) Admittance in each branch
ii) Total admittance of circuit
iii) Total current and power factor of circuit iv) Total power consumed.
_____________________

Set A

SLR-BB 30

*SLRBB30*
S

S.E. (Civil) (Part II) Examination, 2014


CONCRETE TECHNOLOGY (Old)
Day and Date : Tuesday, 2-12-2014
Time : 3.00 p.m. to 5.00 p.m.

Max. Marks : 50

N. B. : 1) Draw neat sketches wherever necessary.


2) Q. No. I is compulsory. It should be solved in first 15 minutes
in Answer book Page No. 3. Each question carries one mark.
3) Answer MCQ/Objective type questions on Page No. 3 only.
Dont forget to mention, Q.P. Set (A/B/C/D) on Top of Page.
Duration : 15 Minutes

MCQ/Objective Type Questions

Marks : 10

I. Choose the most appropriate answer :

10

1) Recommended value for compacting factor of medium workable concrete is


____________
a) 0.78
b) 0.85
c) 0.92
d) 0.96
2) As per BIS OPC grade 43 shall conform to _____________
a) IS : 269
b) IS : 8112
c) IS : 12269

d) IS : 1343

3) Initial setting time of OPC shall not be less than ____________ minutes.
a) 15

b) 20

c) 30

d) 600

4) Gypsum is added to concrete for _________ setting time.


a) Increasing

b) Decreasing

5) The porosity of concrete depends largely upon ____________


a) Grading of aggregate
b) Degree of compaction
c) Type of cement
d) None of the above

P.T.O.

SLR-BB 30

*SLRBB30*

-2-

6) FM for coarse sand in between ____________


a) 2.9 to 3.2

b) 2.3 to 2.6

c) 2.6 to 2.9

d) None

7) Silt content in sand shall be less than _______________


a) 2%

b) 3%

d) 4%

d) 5%

8) Segregation of concrete means ____________


a) Separation of constituents of a homogeneous mixture of concrete.
b) Accumulation of water at the surface, separating itself from the solid
constituents.
9) Flexural strength of concrete as per IS : 456 2000 is __________
a) 0.7

b) 0.5

c) 0.75

d) 0.6

10) The aggregate cement ratio depends up on


a) Type and grading of aggregate
b) Water cement ratio
c) Maximum size of aggregate
d) All a), b) and c)
______________

Set A

*SLRBB30*
S

-3-

SLR-BB 30

S.E. (Civil) (Part II) Examination, 2014


CONCRETE TECHNOLOGY (Old)
Day and Date : Tuesday, 2-12-2014
Time : 3.00 p.m. to 5.00 p.m.

Marks : 40

N. B. : Draw neat sketches wherever necessary.


II. Write a note on standard grading curves and their use in mix design.
III. Write notes on any five :

10
30

a) High strength concrete


b) Bulking of sand
c) Segregation and bleeding of concrete
d) Types of cement discuss any one in detail.
e) Types of concrete discuss any one in detail.
f) Use of admixtures in concrete.
g) Enumerate various steps in ACI method of mix design.
_____________________

Set A

Set A

SLR-BB 300

*SLRBB300*
Seat
No.

Set
S.E. (Part II) Examination, 2014
ELEMENTS OF POWER SYSTEM (New)

Day and Date : Thursday, 27-11-2014


Time : 3.00 p.m. to 6.00 p.m.

Max. Marks : 100

Instructions : 1) Figures to right side shows full marks for that question.
2) Assume data, if necessary with explanation.
3) Q. No. 1 is compulsory. It should be solved in first 30 minutes in
Answer book Page No. 3. Each question carries one mark.
4) Answer MCQ/Objective type questions on Page No. 3 only. Dont
forget to mention, Q.P. Set (A/B/C/D) on Top of Page.
MCQ/Objective Type Questions
Duration : 30 Minutes

Marks : 20

1. Choose the correct answer :

(120=20)

1) Single phase A.C. power


a) is constant
b) pulsates around an average power at double the supply frequency
c) pulsate around the average value of supply frequency
2) The conductor connecting consumers terminals to the distributor is called as
a) feeder
b) distributor
c) service mains
d) none of above
3) In a transmission system the weight of copper used in proportional to
b) E
c) 1 2
d) 1E
a) E 2
E
4) If 3 MW power is to be transmitted over a distance of 30 kms, the desirable transmission
voltage will be ________
a) 11 kv
b) 33 kv
c) 66 kv
d) 3.3 kv
5) In India for distribution of electric power we generally use
a) Wooden pole
b) Fabricated steel pole
c) RCC
d) Both b) and c)
6) The steel core used in ACSR conductor is usually of
a) alloy steel
b) galvanised steel c) stainless steel

d) high speed steel

7) Between two supports, due to sag the conductor takes the form of
a) semi-circle
b) centenary
c) hyperbola
d) any of above
e) none of above
8) Galloping in transmission line conductors arises due to
a) asymmetrical layers of ice formation
b) vertex phenomenon in light winds
c) heavy weight of line conductors
d) adoption of horizontal conductor configuration
9) The voltages across the various discs of a string of suspension insulators having identical
discs is different and it is due to
a) surface leakage current
b) series capacitance
c) shunt capacitance to ground
d) series and shunt capacitors
P.T.O.

SLR-BB 300

*SLRBB300*

-2-

10) If the frequency of a transmission system is changed from 50 c/s to 100 c/s, the string
efficiency will
a) increase
b) decrease
c) remain unchanged
11) The string efficiency of insulators can be increased by
a) reducing the number of insulators
b) increasing number of insulators
c) correct grading of insulators of various capacitances
d) changing the orientation of string
12) Corona is accompanied by
a) violet visible glow
c) vibrations
e) ratio interference

b) hissing noise
d) power loss
f) ozone

g) all of above

13) Corona can be reduced


a) increasing the operating voltage
b) reducing the spacing between conductors
c) increasing the effective conductor diameter
d) none of above
14) The skin effect depends upon
a) cross-section of a conductor
c) permeability of conductor material

b) supply frequency
d) all of above

15) The inductance of an overhead line of single phase two-wire is given by [D is distance
between conductors and r is the radius of conductor]
a) 0.5 loge

D
mH/km
r

b) 0.55 loge

D
mH/km
r

c) 0.4 loge

r
mH/km
D

d) 0.55 loge

r
mH/km
D

16) Capacitance of line with increase in length


a) increases
b) decreases

c) remains same

17) The ABCD constants of 3 ph. line with linear and passive elements
a) are always equal
b) never equal
c) A and D are equal
d) B and C are equal
18) The power transmitted will be maximum when
a) line reactance is high
b) corona losses are minimum
c) sending end voltage is more
d) receiving end voltage is more
19) Paper is used as an insulating material is usually treated with only compound because it
a) is hygroscopic
b) gets electrostatically charged at high voltage
c) is porous
d) all of above
20) In compressed gas insulated cable, SF6 has the gas pressure in the range of
a) 10 20 mmHg
b) 80 100 mmHg c) 3 5 kg/cm2
d) 40 50 kg/cm2
______________

Set A

*SLRBB300*

-3-

SLR-BB 300

Seat
No.

S.E. (Part II) Examination, 2014


ELEMENTS OF POWER SYSTEM (New)
Day and Date : Thursday, 27-11-2014
Time : 3.00 p.m. to 6.00 p.m.

Marks : 80

Instructions : 1) All questions are compulsory.


2) Draw neat diagram wherever necessary.
3) Assumptions if made, must be clearly mentioned.
SECTION 1
2. Solve any four questions :

(45=20)

1) Derive both expressions for volume of conductor material required in 3 phase 3 wire
system and 2 wire d.c. system and by comparing.
2) Explain skin effect.
3) Describe Bundled conductors.
4) Explain giving 3 equations and causes for selecting high voltage for transmission lines to
transmit power.
5) Define and describe
i) Corona
ii) Disruptive critical voltage
iii) Visual critical voltage with equations.
3. Solve any two :

(210=20)

1) A transmission line has a span of 150 m between level supports. The conductor has a
cross-sectional area of 2 cm2. The tension in conductor is 2000 kg. If specific gravity of
conductor material is 9.9 gm/cm3 and wind pressure is 1.5 kg/m length. Calculate the sag
and also its vertical sag.
2) A 3 phase, 220 kv, 50 Hz transmission line consists of 1.5 cm radius conductor spaced 2
meters apart in equilateral triangular formation. If the temperature is 40C and atmospheric
pressure is 76 cm. Calculate the corona loss per km of line. Take m0 = 0.85.
3) An insulator string consists of 3 units and each having a safe working voltage of 15 kv.
The ratio of self capacitance to shunt capacitance of each unit is 8 : 1. Find the maximum
safe working voltage of the string. Also find the string efficiency.

Set A

SLR-BB 300

-4-

*SLRBB300*

SECTION 2
4. Solve any four :

(45=20)

1) Draw circuit diagram and vector diagram of short transmission line and derive % V.R., p.f.
[cos s ] , and transmission efficiency.
2) Draw and describe the general construction, functions of 3-conductor cable.
3) Draw neat diagram of 3 ph screened cables (both) along with their application.
4) Derive an expression of capacitance of single core cable.
5) Define capacitance grading and derive expression for potential difference between core
and earth sheet in case of using 3 layers of different dielectrics.
5. Solve any two :

(210=20)

1) Draw analysis of long transmission line by (Rigorous method) for sending end voltage in
terms of receiving end voltage and current.
2) A 100 km long, 50 Hz transmission line has following line constants Resistance/ph/km
= 0.1 , Reactance/ph/km = 0.5 , susceptance/ph/km = 10 106 s. If the line
supplies a load of 20 MW at 0.9 p.f. lagging at 66 kv at the receiving end, calculate by
Nominal -method.
i) Sending end p.f.
ii) % V.R.
iii) Transmission efficiency.
3) A single-phase A.C. distributor AB 300 meter long is fed from end A and is loaded as
under
i) 100 Amp at 0.707 p.f. lagging 200 m from point A.
ii) 200 Amps at 0.8 p.f. lagging 300 m from point A. The load resistance and reactance of
distributor is 0.2 and 0.1 per km. Calculate the voltage drop in the distributor.
The load p.f. refers to the voltage at the far end.
________________

Set A

SLR-BB 301

*SLRBB301*
Seat
No.

Set

S.E. (Electrical Engineering) (Part II) Examination, 2014


ANALOG AND DIGITAL INTEGRATED CIRCUITS (New)
Day and Date : Friday, 28-11-2014
Time : 3.00 p.m. to 6.00 p.m.
Instructions :

Total Marks : 100

1) Q. No. 1 is compulsory. It should be solved in first 30 minutes in


Answer book Page No. 3. Each question carries one mark.
2) Figures to the right indicate full marks.
3) Use of calculator is allowed.
4) Answer MCQ/Objective type questions on Page No. 3 only. Dont
forget to mention, Q.P. Set (A/B/C/D) on Top of Page.
MCQ/Objective Type Questions

Duration : 30 Minutes

Marks : 20

1. Choose the correct option :

20

1) The change in OPAMPs input offset voltage caused by variations in supply voltage is
called as
a) Output offset voltage
b) Offset voltage adjustment range
c) SVRR
d) Power consumption
2) Open loop bandwidth of IC 741 is approximately __________
a) 1 KHz
b) 1 MHz
c) 10 KHz

d) 5 Hz

3) Output offset voltage is caused by ___________


a) Input bias current
b) Input offset voltage
c) Both a) and b)
d) None of these
4) A summing amplifier can be realized with
a) Inverting amplifier
c) Differential amplifier

b) Non inverting amplifier


d) All of the above

5) For a voltage follower which of the following is true ?


a) Non inverting amplifier with feedback
b) Output = input
c) Gain = 1
d) All of the above
6) In the circuit shown, the value of output, Vo is ________

a) + 3 V

b) 3 V

c) 7 V

d) + 7 V
P.T.O.

SLR-BB 301

*SLRBB301*

-2-

7) A differential amplifier is invariably used in the input stage of all OPAMPs. This is done
basically to provide the OPAMP with a very high ___________
a) CMRR
b) Bandwidth
c) Slew rate
d) Open loop gain
8) Rif = Ri(1+A ) represents
a) Input resistance of non inverting closed loop amplifier
b) Input resistance of inverting closed loop amplifier
c) Input resistance of OPAMP with open loop
d) None of these
9) Level shifter is _________ circuit.
a) Collector biased b) Base biased
c) Emitter follower d) Voltage divider
10) For step input output of an integrator is __________
a) Triangular
b) Ramp
c) Square
d) Sine
11) A ring counter consisting of five Flip-Flops will have
a) 5 states
b) 10 states
c) 32 states
d) Infinite states
12) If the input to T-flipflop is 100 Hz signal, the final output of the three T-flip flops in cascade is
a) 1000 Hz
b) 500 Hz
c) 333 Hz
d) 12.5 Hz
13) A 4-bit synchronous counter uses flip-flops with propagation delay times of 15 ns each.
The maximum possible time required for change of state will be
a) 15 ns.
b) 30 ns.
c) 45 ns.
d) 60 ns.
14) The commercially available 8-input multiplexer integrated circuit in the TTL family is
a) 7495
b) 74153
c) 74154
d) 74151
15) The K-map given below represents

a) OR logic

b) AND logic

c) XNOR logic

d) XOR logic

16) TTL to CMOS interfacing is possible.


a) True
b) False
17) If the output of a shift register is fed back to serial input, then the shift register can be used
as a
a) Multiplexer
b) Demultiplexer
c) Ring Counter
d) Encoder
18) The diagram given below represents a

a) SR Latch

b) 1 bit memory cell c) D flip flop

d) Both a) and b)

19) A 1 hz signal can be obtained from a 60 Hz signal by using


a) Three 7490 ICs
b) One 7490 IC
c) Two 7490 ICs

d) Both a) and b)

20) IC 74139 is a
a) 2 : 1 MUX

d) D Flip flop

b) 2 : 4 DEMUX

c) 8 : 1 MUX

______________
Set A

*SLRBB301*

-3-

SLR-BB 301

Seat
No.

S.E. (Electrical Engineering) (Part II) Examination, 2014


ANALOG AND DIGITAL INTEGRATED CIRCUITS (New)
Day and Date : Friday, 28-11-2014
Time : 3.00 p.m. to 6.00 p.m.

Marks : 80

SECTION I
2. Answer the following (any four) :

(45=20)

1) Explain closed loop inverting amplifier using OPAMP for AC input with circuit diagram and
waveforms.
2) Derive an expression for closed loop gain, input resistance and output resistance for
voltage series feedback amplifier.
3) Explain differential amplifier and derive equation for gain.
4) Explain block diagram of OPAMP.
5) Explain I to V converter.
3. Answer the following (any two) :

(210=20)

1) Explain instrumentation amplifier and derive expression for gain.


2) Explain characteristics of ideal OPAMP also explain internal equivalent circuit of OPAMP.
3) Explain the following terms supply voltage rejection ratio, output voltage swing, output
resistance, slew rate, gain bandwidth product.
SECTION II
4. Answer any four :

(54=20)

a) Design a mod 25 counter using IC 7490.


b) Determine the number of flip flops required to construct a register capable of storing.
i) 6 bit binary number
ii) Decimal numbers up to 32
iii) Hexadecimal numbers up to F
iv) Octal numbers up to 10.
c) Draw and explain the working of a serial in serial out shift register.
d) Explain the differences between TTL and CMOS logic families.
e) Explain Multiplexing and Demultiplexing. Name the ICs that are used as Multiplexers and
Demultiplexers. Obtain an XOR gate using 2 : 1 MUX.

Set A

SLR-BB 301

-4-

*SLRBB301*

5. Attempt any two :

(102=20)

a) Design a pulse train generator circuit using shift register for the pulse train .......
1000110 ...........
b) Draw the internal architecture of IC 7490. Design a down counter counting states from
1101 to 0011 using 4 bit synchronous counter IC 74LS191.
c) Given the following four four-variable functions :
W(A, B, C, D) = m(0, 2, 4, 5, 8, 10, 12, 13)
Y(A, B, C, D) = m(4, 5, 6, 8, 10, 12, 13, 14)
Use K-maps to independently minimize the functions.
_____________________

Set A

SLR-BB 302

*SLRBB302*
Seat
No.

Set

S.E. (Electrical) (Part II) (New) Examination, 2014


NETWORK ANALYSIS
Day and Date : Saturday, 29-11-2014
Time : 3.00 p.m. to 6.00 p.m.

Max. Marks : 100

Instructions : 1) Q. No. 1 is compulsory. It should be solved in first 30 minutes in


Answer Book Page No. 3. Each question carries one mark.
2) Answer MCQ/Objective type questions on Page No. 3 only. Dont
forget to mention, Q.P. Set (A/B/C/D) on Top of Page.
MCQ/Objective Type Questions
Duration : 30 Minutes

Marks : 20

1. Choose the correct answer :

(120=20)

1) When n resistances each of value r are connected in parallel, then resultant is x. When
these n resistances are connected in series, total resistance is
a) nx
b) r nx
c) x/n
d) n2x
2) A circuit contains two unequal resistances in parallel
a) current is same in both
b) large current flows in larger resistor
c) potential difference across each is same
d) smaller resistance has smaller conductance
3) Four identical resistors are first connected in parallel and then in series. The resultant
resistance of the first combination to the second will be
a) 1/16 times
b) 1/4 times
c) 4 times
d) 16 times
4) Three resistance of 6 ohm each are connected as shown in fig. below. The equivalent
resistance between X1 and X2 is

a) 2 ohm

b) 4 ohm

c) 8 ohm

d) 12 ohm

5) The nodal method of circuit analysis is based on


a) KVL and ohms law
b) KCL and ohms law law
c) KCL and KVL
d) KCL, KVL and ohms law
6) Superposition theorem is not applicable to network containing
a) nonlinear elements
b) dependent voltage source
c) dependent current source
d) transformers
7) A network has 7 nodes and 5 independent loops. The number of branched in the network is
a) 13
b) 12
c) 11
d) 10
P.T.O.

SLR-BB 302

*SLRBB302*

-2-

8) When a series RL circuit is connected to a voltage source V at t = 0, the current passing


through the inductor L at t = 0+ is
a) V/R
b) Infinite
c) zero
d) V/L
9) A capacitor does not allow sudden changes in
a) currents
b) voltages
c) both a) and b)

d) none of the above

10) If one of the resistors in a parallel circuit is removed, what happens to the total resistance ?
a) decreases
b) increases
c) remain constant d) exactly doubles
11) Four equal value resistors are connected in parallel. 5V voltage is applied across the
parallel circuit, and 2.5 mA current are measured from the source. What is the value of
each resistor ?
a) 4
b) 8
c) 2.5
d) 5
12) The number of branches in a tree is ___________ the number of branches in a graph.
a) less than
b) more than
c) equal to
d) half
13) The reciprocity theorem is applicable to
a) linear networks only
c) linear or bilateral networks

b) bilateral networks only


d) none of these

14) Nortons equivalent circuit consist of


a) voltage source in parallel with resistance
b) voltage source in series with resistance
c) current source in series with resistance
d) current source in parallel with resistance
15) Indicate the dual of series network consists of voltage source, capacitance, inductance in
a) parallel combination of resistance, capacitance and inductance
b) series combination of current source, capacitance and inductance
c) parallel combination of current source, inductance and capacitance
d) none of the above
16) The Nortons theorem is valid
a) only for a.c. circuits
c) for both a.c. and d.c. circuits

b) only for d.c. circuits


d) none of these

17) In a complex impedance circuit, the maximum power transfer occurs when the load
impedance is equal to
a) complex conjugate of source impedance
b) source impedance
c) complex impedance
d) none of the above
18) Two two-port networks are connected in cascade. The combination is to be represented
as a single two-port network. The parameters are obtained by multiplying the individual
a) z-parameter matrix
b) h-parameter matrix
c) y-parameter matrix
d) ABCD parameter matrix
19) A two-port network is defined by the following pair of equations I1 = 2V1 + V2 and I2 = V1 + V2.
Its impedance parameters (Z11, Z12, Z21, Z22) are given by
a) 2, 1, 1, 1
b) 1, 1, 1, 2
c) 1, 1, 1, 2
d) 2, 1, 1, 1
20) Which parameters are widely used in transmission line theory ?
a) Z
b) Y
c) ABCD

d) h

______________
Set A

*SLRBB302*

-3-

SLR-BB 302

Seat
No.

S.E. (Electrical) (Part II) (New) Examination, 2014


NETWORK ANALYSIS
Day and Date : Saturday, 29-11-2014
Time : 3.00 p.m. to 6.00 p.m.

Marks : 80

2. Solve any four :

(45=20)
SECTION I

a) Replace the network shown below with single voltage source and a resistor :

b) Draw the dual of the network shown in figure below :

c) For the graph shown in figure, write :


i) incidence matrix
ii) Tieset matrix
iii) Cutset matrix

Set A

SLR-BB 302

*SLRBB302*

-4-

d) Obtain Thevenins equivalent network for the circuit shown below :


10 0

e) Find the current through 8 resistor using Nortons theorem for the ckt shown below :

3. Solve any two :

(210=20)

a) State and explain Maximum Power Transfer Theorem. For the fig. shown below,
what will be the value of RL to get maximum power delivered to it ? What is the
value of this power ?

b) State and explain Super position theorem. Determine the voltage across the (2 + j5)
impedance in fig. shown below using Super position theorem.

30A

c) Write the formulae for Star to Delta and Delta to Star transformation. Determine the power
supplied to network shown below :

Set A

*SLRBB302*

-5-

SLR-BB 302

SECTION II

4. Solve any four :

(45=20)

a) Derive h parameter in terms of Z-parameter.


b) Find Z parameter for the network shown below :

c) Obtain Y parameter of the network shown below :

d) In the network shown below, at t = 0 the switch is opened. Calculate V, dv/dt,


d2V/dt2 at t = 0+.

e) The switch in the network shown in figure below is closed at t = 0. Determine the
voltage across the capacitor using Laplace Transform :

Set A

SLR-BB 302

-6-

5. Solve any two :

*SLRBB302*
(210=20)

a) Find Z and h parameter for the network shown below :

b) In the network shown below, at switch closes at t = 0. The capacitor is initially uncharged.
Find VC(t) and iC(t).

c) Determine ABCD parameters for the ladder network shown below :

_____________________

Set A

SLR-BB 303

*SLRBB303*
Seat
No.

Set

T.E. (Part I) (Old) (Electrical Engineering) Examination, 2014


POWER SYSTEM I
Day and Date : Wednesday, 3-12-2014
Time : 3.00 p.m. to 6.00 p.m.

Max. Marks : 100

Instructions : i) Figure to right indicate full marks.


ii) Assume suitable data if necessary.
iii) Q. No. 1 is compulsory. It should be solved in first 30 minutes in
Answer Book Page No. 3. Each question carries one mark.
iv) Answer MCQ/Objective type questions on Page No. 3 only. Dont
forget to mention, Q.P. Set (A/B/C/D) on Top of Page.
MCQ/Objective Type Questions
Duration : 30 Minutes

Marks : 20

1. Solve the following :

(120=20)

1) Which of the following is not the distribution system normally used


A) 3 phase-4 wire
B) 3 phase-3 wire
C) Single phase-3 wire
D) Single phase-4 wire
2) Skin effect results in
A) reduced effective resistance but increased effective internal reactance of the conductor
B) increased effective resistance but reduced effective internal reactance of the conductor
C) reduced effective resistance as well as effective internal reactance
D) increased effective resistance as well as effective internal reactance
3) In overhead transmission lines the effect of capacitance can be neglected when the length
of line is less than
A) 200 km
B) 160 km
C) 100 km
D) 80 km
4) Ten discs usually suggest that the transmission line voltage is
A) 11 kV
B) 33 kV
C) 66 kV

D) 132 kV

5) Presence of ozone as a result of corona is harmful because


A) it gives bad odor
B) it corrodes the material
C) it transfers energy to the ground
D) reduces power factor
6) Which of the following statement is correct ?
A) Wind pressure reduces corona effects
B) Ice on conductors improves power factor
C) Wind pressure is taken to act in a direction at right angles to that for ice
D) Wind pressure and ice on conductors together improve regulation of power transmitted
7) For improving life, steel poles are galvanized. Galvanizing is the process of applying a
layer of
A) paint
B) varnish
C) tar coal
D) zinc
P.T.O.

SLR-BB 303

*SLRBB303*

-2-

8) For transmission lines the standing wave ratio is the ratio of


A) maximum voltage to minimum voltage B) maximum current to minimum voltage
C) peak voltage to rms voltage
D) maximum reactance to minimum reactance
9) When the power is to be transmitted over a distance of 500 km, the transmission voltage
should be in the range
A) 33 kV 66 kV
B) 66 kV 100 kV
C) 110 kV 150 kV
D) 150 kV 220 kV
10) Neglecting losses in a transmission system, if the voltage is doubled, for the same power
transmission, the weight of conductor material required will be
A) four times
B) double
C) half
D) one fourth
11) The dielectric strength of air at 25C and 76 cm/Hg is
A) 1 kV/cm
B) 1 kV/mm
C) 3 kV/cm
12) The chances of corona are maximum in
A) domestic wiring
C) transmission lines

D) 30 kV/cm

B) distribution lines
D) all of the above

13) Minimum horizontal clearance of a low voltage transmission line from residential
buildings must be
A) 11/2 feet
B) 3 feet
C) 4 feet
D) 8 feet
14) For 11 kV line the inductance per km per phase will be of the order of
A) 1 H
B) 0.1 H
C) 1 mH
D) 0.1 mH
15) The most economic voltage for transmitting given power over a known distance by
overhead transmission line is approximately
A) 3.6 kV/km
B) 1.6 kV/km
C) 2.6 kV/km
D) 3.6 kV/km
16) A 70/6 ACSR conduction is an aluminium conductor steel reinforced, having
A) cross sectional area of aluminium as 70 mm2 and the cross-sectional area of steel as
6 mm2
B) cross-sectional area of steel as 70 mm2 and the cross-sections area of aluminium as
6 mm2
C) 70 aluminium conductors and 6 steel conductors
D) 80 steel conductors and 6 aluminium conductors
17) Out of the following systems of distribution, which system offers the best economy ?
A) Direct current system
B) AC single phase system
C) AC 3 phase 3 wire system
D) AC 3 phase 4 wire system
18) The main advantage of a.c. transmission system over d.c. transmission system is
A) Easy transformation
B) Less losses in transmission over long distances
C) Less insulation problems
D) Less problem of instability
19) Material generally used for bus bar is
A) copper
B) aluminium

C) steel

D) tungsten

20) A 10 MVA generator has power factor 0.866 lagging. The reactive power produced will be
A) 10 MVA
B) 8 MVA
C) 5 MVA
D) 1.34 MVA
______________
Set A

*SLRBB303*

-3-

SLR-BB 303

Seat
No.

T.E. (Part I) (Old) (Electrical Engineering) Examination, 2014


POWER SYSTEM I
Day and Date : Wednesday, 3-12-2014
Time : 3.00 p.m. to 6.00 p.m.
Instructions :

Marks : 80

i) All questions are compulsory.


ii) Figure to right indicate full marks.
iii) Assume suitable data if necessary.
SECTION I

2. Solve any four of the following :


(54=20)
a) What is corona ? Describe the merits and demerits of corona.
b) Derive expression for voltage distribution in 3 insulators in a string.
c) Derive an expression for conductor material required in two wire dc systems with one
conductor earthed and in two wire dc system with midpoint earth with comparison.
d) Explain the bundled conductors.
e) Explain the grading of cables.
3. Solve any two of the following :
a) An overhead transmission line at river crossing is supported from 2 towers at heights of
25 m and 75 m above the water level. The horizontal distance between the tower is 250 m.
If the required clearance between the conductor and the water midway between the towers
is 45 m and if the both the towers are on the same side of the point of maximum sag of the
parabolic configuration, find the stringing tension in the conductor. The weight of the
conductor is 0.7 kg/m.
b) A part of the transposition cycle of a 3 phase double circuit line is as shown in figure.
Radius of each conductor is 0.9 cm. The conductors are solid copper. Find the inductance
per phase per km of the line.

10

c) A string of six insulators is to be graded to obtain uniform distribution of voltages across


the string. If the capacitance of the top unit is 10 times the capacitance to ground of earth
unit determine the capacitance to ground of each unit, determine the capacitance of the
remaining five units.

10

10

Set A

SLR-BB 303

-4-

*SLRBB303*

SECTION II
4. Solve any four of the following :

(54=20)

a) Explain the different types of AC and DC distribution systems.


b) Derive an expression for dielectric strength of 1 phase underground cable.
c) Explain the different method of voltage control in a power system network.
d) Draw and describe both types of pressure cables.
e) Explain different types of tariffs. Which is best suited for the residential purpose ? Prove
with example.
5. Solve any two of the following :

(102=20)

a) A 3 phase transmission line 50 km long consists of 3 hard drawn copper conductors in a


1.2 m delta. Load conditions at receiving end 33,000 Volts, 50 Hz. Line is designed so that
transmission loss is approximately 10%. Find
i) the sending end voltage and power factor
ii) efficiency and
iii) regulation.
b) The sending end voltage of a 3 phase overhead line with a resistance and reactance per
phase of 5 ohm and 20 ohm respectively is w 46.85 kv while the receiving end voltage is
33 kv. Determine the power output at 0.8 pf lag and the sending end pf.
c) An 800 m distributor fed from both ends A and B is loaded uniformly at the rate of 1.2 A per
meter run, the resistance of each conductor being 0.05 ohm per km. Determine the minimum
voltage and the point where it occurs if feeling points A and B are maintained at 255 V and
250 V respectively. Find the current supplied from feeding points A and B.
_____________________

Set A

SLR-BB 304

*SLRBB304*
Seat
No.

Set

T.E. (Electrical Engg.) (Part I) (Old) Examination, 2014


LINEAR INTEGRATED CIRCUITS
Day and Date : Friday, 5-12-2014
Time : 3.00 p.m. to 6.00 p.m.

Max. Marks : 100

Instructions : 1) Q. No. 1 is compulsory. It should be solved in first 30


minutes in Answer book Page No. 3. Each question carries
one mark.
2) Answer MCQ/Objective type questions on Page No. 3
only. Dont forget to mention, Q.P. Set (A/B/C/D) on Top
of Page.
Duration : 30 Minutes

MCQ/Objective Type Questions

Marks : 20

1. Choose the correct answer :


1) Window comparator has ______________ thresholds.
a) one
b) two
c) three
d) none
2) In scaling amplifier input resistors are
a) of same value
b) of different values
c) proportional to weight of its input d) scaled by two
3) A differential amplifier ________________
a) is a part of OPAMP
b) has one input and one output
c) has two output
d) both a) and c)
4) Common mode gain is ______________
a) very high
b) very low
c) always unity
d) unpredictable
5) IC 741C has a unity gain frequency of
a) 10Hz
b) 100 kHz
c) 1MHz
d) 75Hz
6) Typical value of cmRR for 741 is ______________ db.
a) 90
b) 120
c) 20
d) 20
7) Gain of non inverting amplifier is always greater than 1.
a) True
b) False
c) Neither true nor false
d) partially true
8) In a differentiator, the feedback element is a _________________
a) voltage divider b) capacitor
c) zener diode
d) resistor
9) The zero level detector is one application of a ____________
a) comparator
b) differentiator
c) summing amplifier
d) diode

20

P.T.O.

SLR-BB 304

-2-

*SLRBB304*

10) Which of the following characteristic doesnot necessarily apply to an op-amp ?


a) high gain
b) low power
c) high input impedance
d) low output impedance
11) A two pole high pass active filter would have a roll-off rate of
a) 40 dB/decade
b) 40 dB/decade
c) 20 dB/decade
d) 20 dB/decade
12) The center frequency of a band-pass filter is always equal to the
a) bandwidth
b) 3dB frequency
c) geometric average of the critical frequencies
d) None of the above
13) What is the purpose of the op-amp within an active filter ?
a) impedance matching
b) to alleviate filter losses
c) easy adjustment over a wide frequency range
d) all of the above
14) A feedback oscillators action is best described as
a) an amplification of a specific frequency from a given input
b) a pulse generating circuit
c) a controlling of amplitude
d) signal generation from an amplifier with positive frequency selective
feedback
15) The comparator output of a triangular-wave oscillator is a
a) triangle wave
b) sawtooth wave
c) square wave
d) none
16) The 555 timer can be configured as
a) a voltage controlled square-wave oscillator
b) a triangular-wave oscillator
c) a low-distortion sine-wave oscillator
d) a ramp voltage generator
17) The output of a variable IC regulator such as the LM 317 is controlled by
a) external capacitances
b) external inductances
c) external resistances
d) the LM317 is not an adjustable regulator
18) The range of frequencies that a PLL circuit will track is called
a) capture range b) track range
c) bandwidth
d) lock range
19) The actual device that produces the pulse of a voltage-to-frequency converter is
a) comparator
b) integrator
c) capacitor
d) one-shot multivibrator
20) Sawtooth waveform can be obtained from _____________ circuit.
a) ramp generator
b) triangle waveform generator
c) square waveform generator
d) none of the above
______________
Set A

*SLRBB304*

-3-

SLR-BB 304

Seat
No.

T.E. (Electrical Engg.) (Part I) (Old) Examination, 2014


LINEAR INTEGRATED CIRCUITS
Day and Date : Friday, 5-12-2014
Time : 3.00 p.m. to 6.00 p.m.

Marks : 80
SECTION I

2. Attempt any four :


(45=20)
a) Explain ideal characteristics of op-amp.
b) Explain in short input offset voltage and input offset current.
c) Explain half wave precision rectifier in detail.
d) Draw the block diagram of differential amplifier and explain in brief.
e) What is virtual ground ? Derive the gain for inverting amplifier using virtual
ground concept.
3. a) Explain with circuit diagram V to I and I to V converter.

(110=10)

b) Explain op-amp as non-inverting amplifier and derive the gain in closed loop.
(110=10)
OR
b) Explain with neat diagram log and antilog amplifier.
SECTION II
4. Solve any four :
20
a) Differentiate in detail P, PI, PID controller.
b) Calculate the minimum and maximum output voltage for adjustable voltage
regulator using IC LM317 for R1 = 240 R2 = 4.7 (Pot), ladj = 50 A.
c) Explain first order high pass filter with its frequency response.
d) Explain with suitable waveform square wave generator.
e) Explain block diagram and operating principal of PLL.
5. Solve any two :
20
a) Draw and explain triangular wave generator. Derive the expression for
oscillation frequency of triangular wave.
b) Design an astable multivibrator using IC 555 for maximum output frequency
of 5 kHz with duty cycle 25%, also calculate Vcc required for 5V output.
c) Explain Functional Block Diagram of IC 723 with its features ? Also explain
low voltage high current boosting circuit.
_____________________

Set A

Set A

SLR-BB 305

*SLRBB305*
Seat
No.

Set

T.E. (Electrical) (Part I) (Old) Examination, 2014


ELECTROMAGNETIC ENGINEERING
Day and Date : Monday, 8-12-2014
Time : 3.00 p.m. to 6.00 p.m.
Instructions :

Max. Marks : 100

1) Q. No. 1 is compulsory. It should be solved in first 30 minutes in


Answer Book Page No. 3. Each question carries one mark.
2) Answer MCQ/Objective type questions on Page No. 3 only. Dont
forget to mention, Q.P. Set (A/B/C/D) on Top of Page.
MCQ/Objective Type Questions

Duration : 30 Minutes

Marks : 20

1. Choose the correct option :


1) The value of ax ax is
a) 1
b) 1

c) Zero

2) dv = rdrd dz is the differential volume in


a) Cartesian co-ordinate system
c) Spherical co-ordinate system

b) Cylindrical co-ordinate system


d) None of the above

3) The value of unit vector is


a) Zero
b) 1

c) 1

4) If dot product of two vectors is zero then vectors are


a) Perpendicular
b) Parallel
c) Oblique

d) 2

d) 2
d) None of these

5) dsz = rdrd is the differential surface area in


a) Cylindrical co-ordinate
b) Spherical co-ordinate
c) Cartesian co-ordinate
d) None
Q1Q 2
is
4 r 2
a) Faradays law

6) F =

b) Coulombs law

H
is
t
a) Amperes law
c) Ohms law

c) Ohms law

d) Gausss law

7) E =

8) Acceleration is a
a) Scalar

b) Faradays law
d) Gausss law
b) Unit vector

9) Unit of electric field intensity is


a) Amp/meter
c) Volts/meter

c) Vector

d) None

b) Coulomb/meter
d) Newton/meter
P.T.O.

SLR-BB 305

*SLRBB305*

-2-

10) A co-ordinate in spherical system is defined in terms of Cartesian system as


z

a) = cos 1z

b) = cos 1

c) cos 1 x 2 + y 2 + z 2

z
d) = sin 1
2
x + y 2 + z2

x 2 + y 2 + z2

11) Energy density of magnetic field is


a)

1 2
H
2

b) H

c) 2H

d)

1 2
H
2

12) Velocity of EM wave in free space is


a) 10 108 m s

b) 3 108 m Hour

c) 6.8 108 m s

d) 3 108 m s

13) Self inductance L is given by


a)

NI

b)

I
N

c)

N
I

d)

N
I

14) Displacement current density is


a) D

c) D t

b) J

d) J t

15) Scalar magnetic potential exists where J is present


a) Yes
b) No
c) Cant determine
d) None of these
16) The unit of magnetic susceptibility is
a) Nil
b) Amp

c) H/m

d) Wb

17) A is
a) H

c) J

d) 0

b) B

18) If a charge, 1 c is moving with a velocity 2ax in a magnetic field of B = 1a y , the force on the
charge is
b) 1a y

c) 2 a x

d) 1a z

19) Intrinsic impedance of free space is


b) 3
a) 2

c) 100

d) 120

20) H = ( + jWE )E is for


a) Time varying field
c) Harmonically varying field

b) Static field
d) None-time varying field

a) 2 a z

______________
Set A

*SLRBB305*

-3-

SLR-BB 305

Seat
No.

T.E. (Electrical) (Part I) (Old) Examination, 2014


ELECTROMAGNETIC ENGINEERING
Day and Date : Monday, 8-12-2014
Time : 3.00 p.m. to 6.00 p.m.

Marks : 80

SECTION 1
2. Solve any four :

(54=20)

a) Transform the given vector A = 3 a x 2 a y 3 a z at P( 2, 3, 4) into spherical coordinate


system.
b) Four identical point charges of 8 uc are placed at four corners of square 4m in size. Find
out the total vector force on charge Q4.
c) Two infinite uniform sheets of charge of 20 uc/m2 are located at X = 1m. Determine E
in all regions.
d) State Gauss Law and show that the divergence of electric field and that of electric flux
density in charge free region is zero.
2

e) If E = 8xy ax 4x ay + az v/m. Find the work done carrying a 6 coulomb charge from
A(1, 8, 5) to B(2, 18, 6) along the path y = 3x + 2, z = x + 4.
3. Solve any two :

(102=20)

a) Derive the boundary conditions for E and D at the interface between two dielectrics.
b) Obtain Poissons equation and from this obtain Laplaces equation stating necessary
conditions.
c) State and derive the uniqueness theorem.

Set A

SLR-BB 305

-4-

*SLRBB305*

SECTION 2
4. Solve any four :

(54=20)

a) State and explain Biot-Savart Law.


b) A current filament of 5 amp in a y direction is parallel to y axis at x = 2m and z = 2m.
Find H at the origin.
c) An area of 0.65 m2 in the z = 0 plane is enclosed by a filamentary conductor. Find the

induced voltage if the applied magnetic flux density is B = 0.05 cos 10 3 t ay + az

2 T.

d) Calculate the current density everywhere due to a current filament placed along z axis
with current I amp in the az direction.
e) Explain scalar and vector magnetic potential.
5. Solve any two :

(102=20)

a) Given H =6r sin


/ a r + 18r sin a
/ , evaluate both sides of stokes theorem for the
portion of the cone = 0.1 bounded by r = 2,
/ = 0 and
/ = 0.3 consider direction

of ds be a .
b) Write down the maxwells equations for time periodic fields and show that divergence
equations follow from curl equations and the continuity equations.
c) Derive the propagation characteristics of EM waves in space for conducting medium.
_____________________

Set A

SLR-BB 306

*SLRBB306*
Seat
No.

Set

T.E. Electrical (Part I) (Old) Examination, 2014


CONTROL SYSTEM I
Day and Date : Wednesday, 10-12-2014
Time : 3.00 p.m. to 6.00 p.m.

Max. Marks : 100

Instructions : 1) Assume suitable data if necessary.


2) Q. No. 1 is compulsory. It should be solved in first 30 minutes in Answer
Book Page No. 3. Each question carries one mark.
3) Answer MCQ/Objective type questions on Page No. 3 only. Dont forget to
mention, Q.P. Set (A/B/C/D) on Top of Page.
MCQ/Objective Type Questions
Duration : 30 Minutes

Marks : 20

1. Choose the correct answer :


1) An automatic toaster is a _____________ lop control system.
a) Open

b) Closed

c) Partialy closed

d) Any of the above

2) Which one of the following statements is not true ?


a) Feedback does not reduce the sensitivity of the control system to variations in its parameters.
b) Sensors in the feedback path may introduce noise in the system
c) Feedback may lead to instability in the closed-loop system even if the open-loop system is
stable
d) Feedback increases the overall gain of the system
3) Under force-voltage analogy, reciprocal of capacitance is analogous to
a) Spring stiffness

b) Coefficient of friction

c) Rate of change of velocity

d) Rate of change of displacement

4) ___________ has tendency to oscilate.


a) Open lop system

b) Closed lop system

c) Both a) and b)

d) Neither a) nor b)

5) In an automatic control system which of the following elements is not used ?


a) Error detector

b) Final control element

c) Sensor

d) Oscillator

6) The transfer function of tachometer is of the form


a) Ks

b) K/s

c) k/s+1

d) K/s(s+1)

7) Despite the presence of negative feedback, control system still have problems of Instability because the
a) Components used have nonlinearities
b) Dynamic equations of sub system are not known exactly
c) Mathematical analysis involves approximation
d) System has large negative phase angles at high frequency
P.T.O.

SLR-BB 306

*SLRBB306*

-2-

8) Signal flow graph is used to find


a) Stability of system

b) Controllability of system

c) Transfer function of system

d) Poles of the system

9) A temperature control system is known as


a) Process control system

b) Servo mechanism

c) Cascade control system

d) None of above

10) Mass in mechanical system is analogous to ___________ in electrical system.


a) Resistance

b) Inductance

c) Capacitance

d) Conductance

11) Which of the following is the time domain method of determining stability of a control system ?
a) Bode plot

b) Nyquist plot

c) Nicholos chart

d) Root locus technique

12) The valid relation between setting time Ts and rise time Tr is
a) Tr > Ts

b) Ts > Tr

c) Ts = Tr

d) None of these

13) Flat frequency response means that the magnitude ratio of output to input over the bandwidth is
a) Variable

b) Zero

c) Constant

d) None of above

14) The unit-impulse response of a system starting from rest is given by C(t) = 1 e2t for t > = 0. The
transfer function of the system is
a) 1/(1 + 2s)

b) 2/(2 + 2s)

c) 2/s(s + 2)

d) 1/(s + 2)

15) For a type one system, the steady state error due to step input is equal to
a) Infinity

b) 0

c) 0.5

d) 0.25

16) A system with gain margin close to unity or a phase margin close to zero is
a) Highly stable

b) Oscillatory

c) Relatively stable

d) Unstable

17) The major disadvantage of a feedback system may be


a) Inaccuracy

b) Inefficiency

c) Unreliability

d) Instability

18) A good factor for Mp should be


a) less than 1

b) lying between 1.1 and 1.5

c) more than 2.2

d) zero

19) If the transfer function of a first-order system is G(s) = 10/(1 + 2s) then the time constant of the
system is
a) 10 sec

b) 1/10 sec

c) 2 sec

d) 1/2 sec

20) The initial response when the output is not equal to input is called
a) Transient Response
c) Error response

b) Dynamic Response
d) Either of the above
______________

Set A

*SLRBB306*

-3-

SLR-BB 306

Seat
No.

T.E. Electrical (Part I) (Old) Examination, 2014


CONTROL SYSTEM I
Day and Date : Wednesday, 10-12-2014
Time : 3.00 p.m. to 6.00 p.m.

Marks : 80

Instruction : Assume suitable data if necessary.


SECTION I
2. Solve any four :

(45=20)

a) Define open loop and closed loop control systems giving suitable examples.
b) Determine the TF of the armature controlled DC servomotor.
c) Derive the TF of the system shown :

d) Obtain the TF C(s)/R(s) for the system below :

e) Find C(s)/R(s) by Masons gain formula.

3. Solve any two :

(210=20)

a) Draw the signal flow graph and obtain the TF of the system shown below :

R (s)

Set A

SLR-BB 306

*SLRBB306*

-4-

b) Obtain C(s)/R(s)

c) Explain in detail force-voltage analogy for the systems and various sensors used in control system.
SECTION II
4. Solve any four :

(45=20)

a) Explain breakaway and breakin points for Root Locus.


b) Explain all pass and minimum phase systems.
c) Define and explain the following :
i) Asymptote
ii) Centroid
iii) Angle of departure and angle of arrival.
d) The forward path transfer function of a unity feedback system is given by
G(S) =

2
obtain expression for unit step response.
S(S + 3)

e) The response of servomechanism is c(t) = 1 + 0.2 e60t 1.2e 10t when subjected to unit step input.
Obtain the closed loop transfer function hence determine natural frequency and damping ratio.
5. Solve any two :

(210=20)

a) Draw the root locus for the unity feedback system whose Open Loop Transfer Function is
G(s) =

k(s + 2)(s + 3)
comment on stability.
s(s + 1)

b) A unity feedback control system has G( s) =

k(s + 2 )
3

s(s + 7s + 12 s )

. Determine the steady state error

for unit ramp, unit impulse and unit step input. Also determine the TYPE of system.
80
. From this determine :
s(s + 20)(s + 2)
i) Gain and phase crossover frequencies
ii) Gain and phase margins
iii) Comment on stability.

c) Construct BODE plot of G(s) =

_____________________

Set A

SLR-BB 307

*SLRBB307*
Seat
No.

Set

T.E. (Electrical) (Part I) Examination, 2014


INSTRUMENTATION TECHNIQUES (Old)
Day and Date : Friday, 12-12-2014
Time : 3.00 p.m. to 6.00 p.m.

Max. Marks : 100

Instructions : 1) Make suitable assumptions wherever necessary.


2) Draw neat sketch whenever necessary.
3) Q. No. 1 is compulsory. It should be solved in first 30 minutes in
Answer book Page No. 3. Each question carries one mark.
4) Answer MCQ/Objective type questions on Page No. 3 only.
Dont forget to mention, Q.P. Set (A/B/C/D) on Top of Page.
MCQ/Objective Type Questions
Duration : 30 Minutes

Marks : 20

1. Choose the correct answer :

20

1) In a CRO which of the following is not a part of electron gun ?


a) cathode
b) grid
c) accelerating anode
d) X-Y plates
2) In a strain measuring device using a strain gauge, the output quantity is
a) Voltage
b) Resistance
c) Impedance
d) Either a) or b)
3) A capacitance transducer can be used to measure
a) thickness of sheet b) displacement
c) level of fluids
4) LVDT is a
a) capacitive transducer
c) inductive transducer

b) resistive transducer
d) none of them

5) The resistive transducer is basically a


a) active transducer
c) potentiometer

b) passive transducer
d) none of the above

d) all of the above

6) The closed loop gain of an op-amp is dependent upon whether the op-amp is used
a) In inverting mode
b) In non-inverting mode
c) Is independent of the connection of I/P
d) Is dependent upon the connection of I/P
7) A buffer amplifier has a gain of
a) infinity
c) unity

b) zero
d) dependent on the circuit parameter

8) The Q-meter works on the principle of


a) Self inductance
c) Series resonance

b) Mutual inductance
d) Parallel resonance
P.T.O.

SLR-BB 307
9) The rotor of a synchro-transmitter is
a) Salient pole type
c) Either a) or b)

*SLRBB307*

-2-

b) Cylindrical pole type


d) None of the above

10) Filters that transmit all frequencies above a defined cut off frequency are known as
a) Low pass filter b) High pass filter
c) Band pass filter d) Band reject filter
11) A vertical amplifier for a CRO can be designed for
a) Only a high gain
b) Only a broad bandwidth
c) A constant gain times bandwidth product
d) All of the above
12) Post acceleration needed in CRO if frequency of signal is
a) less than 1 MHz
b) more than 1 MHz
c) more than 10 MHz
d) more than 10 Hz
13) Time division multiplexing requires
a) constant data transmission
b) transmission of data samples
c) transmission of data at random
d) transmission of data of only one measured
14) In FM systems operate at
a) VHF and UHF b) Only VHF

c) Only MF

d) MF and HF

15) The advantages of F. M. magnetic tape recording are


a) it can record from d.c. to several kHz
b) it is free from drop out effect
c) it is independent of amplitude variations and accurately reproduces the waveform
of input signal
d) all of above
16) Period measurement is done in frequency meters for achieving high accuracy in
the case of
a) high frequencies
b) medium frequencies
c) d.c.
d) low frequencies
17) An aquadag is used in a CRO to collect
a) primary electrons
b) secondary emission electrons
c) both primary and secondary emission electrons
d) none of above
18) Filters that transmit all frequencies below a defined cut-off frequency are known as
a) Low-pass filters b) High-pass filters
c) Band-pass filters d) Any of these
19) Magnetic tape recorders have ___________
a) A wide frequency range
b) A wide dynamic range
c) A low distortion
d) All of the above
20) Digital instruments have a input impedance of the order of ____________
a)
b) k
c) M
d) m
______________
Set A

*SLRBB307*

-3-

SLR-BB 307

Seat
No.

T.E. (Electrical) (Part I) Examination, 2014


INSTRUMENTATION TECHNIQUES (Old)
Day and Date : Friday, 12-12-2014
Time : 3.00 p.m. to 6.00 p.m.

Marks : 80

Instructions : 1) All questions are compulsory.


2) Make suitable assumptions wherever necessary.
3) Draw neat sketch whenever necessary.
SECTION I
2. Solve any four :

(45=20)

a) Write a short note on sample and hold circuit.


b) Explain the different characteristics while selecting a transducer for any application.
c) Write a short note on electromagnetic flow meter.
d) Define and classify errors in an instrument.
e) Explain the method of measurement of displacement using capacitive transducer.
3. Solve the following :

(210=20)

a) Explain the function of active filters. Draw the neat diagram of high pass butterworth
filter and explain its function.
b) Classify different types of analog to digital converter. Explain the dual slope
integrator method in detail.
OR
b) Explain the principle of resistive transducer and derive the expression for gauge
factor.

Set A

SLR-BB 307

-4-

*SLRBB307*

SECTION II
4. Solve any four :

(45=20)

a) Explain in detail the block diagram of telemetry system.


b) Explain with block diagram dual slope integration type analog to digital converter.
c) Explain the digital method for measurement of speed.
d) Describe the types of oscilloscopes.
e) Explain different types of CRO.
5. Solve any two :

(210=20)

a) Explain the following with detail illustration :


1) Amplitude modulation
2) Frequency modulation
b) Explain ladder diagram construction of PLC system.
c) Explain the working of strip chart recorder.
_____________________

Set A

SLR-BB 308

*SLRBB308*
Seat
No.

Set

T.E. (Electrical) (Part I) (Old) Examination, 2014


ENERGY MANAGEMENT SYSTEM
Day and Date : Monday, 15-12-2014
Time : 3.00 p.m. to 5.00 p.m.
N.B. :

Total Marks : 50

1) Q. No. 1 is compulsory. It should be solved in first


20 minutes in Answer Book Page No. 3. Each question
carries one mark.
2) Answer MCQ/Objective type questions on Page No. 3 only.
Dont forget to mention, Q.P. Set (A/B/C/D) on Top of Page.
MCQ/Objective Type Questions

Duration : 20 Minutes

Marks : 10

1. Choose the correct answer :


1) Which of the following has the highest unit size in India ?
a) Diesel plant
b) Nuclear plant
c) Hydro-electric plant
d) Steam plant
2) Which of the following has maximum number of auxiliaries ?
a) Hydro-electric plant
b) Diesel plant
c) Nuclear plant
d) Steam plant
3) Block meter rate is generally used for
a) Industrial consumer only
b) Residential consumers
c) Commercial consumers only
d) Industrial and commercial consumers
4) The capacity factor of a plant is equal to
a) Maximum load / Average load
b) Average load / Maximum load
c) Maximum load / Plant capacity
d) Average load / Plant capacity
P.T.O.

SLR-BB 308

*SLRBB308*

-2-

5) In rural parts of India how much population lacked electricity in the year
2010.
1
a) 4

1
b) 3

1
c) 2

1
d) 5

6) Which of the following power plants has a minimum operating cost ?


a) Diesel plants

b) Nuclear plants

c) Hydro-electric plants

d) Steam plants

7) A power system needs injection of vars


a) At peak load
b) At off peak load
c) Both a) and b)
d) When load is neither too high nor too low
8) An industrial installation has a power factor of 0.8 lagging it would be
economical to improve the power factor to
a) Unity

b) About 0.95 lagging

c) About 0.8 leading

d) About 0.9 leading

9) The power which must be available even under emergency conditions is


known as
a) Firm reserve

b) Spinning reserve

c) Cold reserve

d) Hot reserve

10) The ash content in Indian coal is as high as


a) 20% to 25%

b) 35% to 40%

c) 5% to 10%

d) 10% to 15 %

________________

Set A

*SLRBB308*

-3-

SLR-BB 308

Seat
No.

T.E. (Electrical) (Part I) (Old) Examination, 2014


ENERGY MANAGEMENT SYSTEM
Day and Date : Monday, 15-12-2014
Time : 3.00 p.m. to 5.00 p.m.

Marks : 40

N.B. : 1) All the questions are compulsory.


2) Figures to the right indicate full marks.
2. Explain commercial energy production for different sector in India since 1950.

10

3. Explain the different aspects of demand side management of electrical energy.

10

OR
3. What are the different types of energy audit and explain the procedure for the
different audits.

10

4. Write short notes on any four :

20

a) Impact of tariff on energy management.


b) Types of instruments used for energy audit.
c) Indian electricity Act, 2003.
d) Various forms of energy.
e) Energy consumption production relationship.
f) Commercial and non-commercial energy.

_____________________

Set A

Set A

SLR-BB 309

*SLRBB309*
Seat
No.

Set

T.E. (Electrical Engineering) (Part I) (New) Examination, 2014


POWER SYSTEM ANALYSIS
Day and Date : Wednesday, 3-12-2014
Time : 3.00 p.m. to 6.00 p.m.
Instructions :

Max. Marks : 100

1) Q. No. 1 is compulsory. It should be solved in first 30 minutes in


Answer book Page No. 3. Each question carries one mark.
2) Answer MCQ/Objective type questions on Page No. 3 only. Dont
forget to mention, Q.P. Set (A/B/C/D) on Top of Page.
MCQ/Objective Type Questions

Duration : 30 Minutes
I

Marks : 20

Choose the correct answer :

20

1) In Gauss Seidel method of power flow problem, the number of iterations may be reduced
if correction in voltage at each bus is multiplied by
a) Gauss constant
b) Acceleration constant
c) Blocking factor
d) Deceleration constant
2) Load flow study involve solving simultaneous
a) linear algebraic equations
b) Non-linear algebraic equations
c) Linear differential equations
d) Non-linear differential equations
3) Primitive Y matrix is _____________ matrix.
a) Singular
b) Null
c) Diagonal

d) Sparse

4) An acceleration factor is used in load flow studies using G-S method to


a) To increase the number of iterations
b) To increase the speed of calculations
c) Both a) and b)
d) None of above
5) Per unit value is defined as
a) Base value/actual value
c) Old value/new value

b) New value/old value


d) Actual value/base value

6) At slack bus ______________ are specified.


b) P, Q
c) P, |V|
a) |V|,

d) Q, |V|

7) When line to ground fault occurs, the current in faulted phase is 100A. The zero sequence
current in this case will be
a) Zero
b) 33.3A
c) 66.66
d) 100 A
8) Which of the following results in a symmetrical fault ?
a) Single line to earth
b) Phase to phase
c) All the three phases to earth
d) Two phases to earth
9) The per unit value a 2 ohm resistor at 100 MVA base and 10 KV base voltage is
a) 4pu
b) 2pu
c) 0.5pu
d) 0.2pu
P.T.O.

SLR-BB 309

*SLRBB309*

-2-

10) Load flow study is carried for


a) Load frequency control
c) Fault calculations

b) Planning of power system


d) Study of stability of the system

11) Fault level means


a) Voltage at fault point
c) Fault power factor

b) Fault current
d) Fault MVA

12) The magnitude of the fault current depends upon _____________


a) Total impedance up to fault
b) Voltage at the fault point
c) Load current being supplied before occurrence of fault
d) Both a) and b)
13) Negative sequence reactance of a transformer is _____________
a) Equal to the positive sequence reactance
b) Larger than the positive sequence reactance
c) Smaller than positive sequence reactance
d) None of the above
14) In a star connected system without neutral grounding, zero sequence currents are
a) Zero
b) Phasor sum of phase currents
c) Same as rms value of phase currents
d) Same as peace value of phase currents
15) The value of the expression 1 + a + a2 is
a) Zero
b) 1
16) For stable operation, the normal value of
a) 0 to 30
b) 0 to 90

c) 1

d) 2

lies between
c) 0 to 60

d) 0 to 180

17) In which type of fault listed below, the positive and negative sequence voltages are equal
(Va1 = Va2)
a) Line to line fault
b) Double line to ground fault
c) Single line to ground fault
d) None of the above
18) The symmetrical components are used in the fault analysis because
a) Number equations become smaller
b) The sequence network do not have mutual couplings
c) The results are required in terms of symmetrical components
d) None of above
19) The positive sequence current of the transmission line is
a) Always zero
b) 1/3 of negative sequence current
c) Equal to the negative sequence current
d) Three time negative sequence current
20) By using the method of equal area criterion, we get the information about
a) Swing equation
b) Stability region c) relative stability d) Absolute stability
______________
Set A

*SLRBB309*

SLR-BB-309

-3-

Seat
No.
T.E. (Electrical Engineering) (Part I) (New) Examination, 2014
POWER SYSTEM ANALYSIS
Day and Date : Wednesday, 3-12-2014
Time : 3.00 p.m. to 6.00 p.m.

Marks : 80
SECTION I

II. Attempt any four :


(45=20)
1) What is per unit system ? How the base quantities are selected ?
2) Describe the various types of buses and their significance in detail.
3) Write in matrix form node equations to solve for voltages of numbered buses. The network
is equivalent to fig. shown below. The emfs are Ea = 1.5 < 0, Eb = 1.5 < 36.87,
Ec = 1.5 < 0 all in p.u.

4) Determine Ybus for simple four bus system shown in fig. Neglect shunt capacitance of the line.
Line
12
13
14
23
34

Impedance (p.u.)
0.25 + j1.0
0.20 + j0.8
0.30 + j1.2
0.20 + j0.8
0.15 + j0.6

5) Explain the short circuit of synchronous machine on load.


III.

Attempt following any two :


(102)
1) Write computational algorithm of Gauss Seidel method for load flow analysis.
2) For the sample system generators are connected to all the three buses, while load are at
bus 2 and 3. Values of reactive and real powers are listed in table. All the buses other than
slack are PQ. Assuming flat voltage start, find voltage and bus angle at three buses at the
end of first GS iteration
Bus
1
2
3
4

Pi, pu

0.5
1.0
0.3

i, pu

0.2
0.5
0.1

Vi, pu
1.04 at 0

Remark
Slack bus
PQ bus
PQ bus
PQ bus

The calculated Ybus for above sample system is


0
2 + j6
1+ 3j
3 j9
2 + j6 3.666 j11 0.666 + j2 1 + j3
1 + j3 0.666 + j2 3.666 j11 2 + j6

3 j9
1 + j3
2 + j6
0

Set A

SLR-BB 309

*SLRBB309*

-4-

3) 100 MVA, 33KV, 3ph generator has sub-transient reactance of 15%. The generator is
connected to motor through a transmission line and transformer. The motor have rated
input of 30 MVA, 20 MVA and 50 MVA at 30 KV with 20% sub transient reactance. 3ph T/F
are rated at 110 MVA, 32/11 KV with leakage reactance of 8%. The line has reactance of
50 ohm. Selecting the generator rating as the base quantities in the other parts of the
system and evaluate the corresponding pu values.
X

SECTION II
IV. Attempt any four :
(45=20)
1) Give the difference between symmetrical and unsymmetrical faults.
2) Explain travelling waves on transmission line.
3) Explain the short circuit of synchronous machine on no load.
4) The phase voltages across a certain unbalanced load are given as
Er = 176 j132, Ey = 128 j96, Eb = 160 + j100. Find symmetrical components.
5) Write a short note on transient stability and steady state stability.
V. Attempt any two :
(102)
1) Draw a diagram showing interconnection of sequence network for single line to ground
fault. Derive equation for sequence currents.
2) Derive the swing equation for synchronous machine.
3) A delta connected balanced resistive load is connected across a balanced three phase
supply as shown in fig. With current in line A and B specified, find the symmetrical
component of line currents. Also find the symmetrical component of delta currents. Do
you notice any relationship between symmetrical components of line and delta currents ?
Comment.

_____________________

Set A

SLR-BB 31

*SLRBB31*
Seat
No.

Set

S.E. (Civil) (Part II) Examination, 2014


STRUCTURAL MECHANICS II (New)
Day and Date : Tuesday, 25-11-2014
Time : 3.00 p.m. to 6.00 p.m.

Total Marks : 100

Instructions : 1)
2)
3)
4)

Figures right to indicate full marks.


Use of non-programmable Calculator is allowed.
Assume suitable data if necessary.
Q. No. 1 is compulsory. It should be solved in first 30 minutes in
Answer book Page No. 3.
5) Answer MCQ/Objective type questions on Page No. 3 only. Dont
forget to mention, Q.P. Set (A/B/C/D) on Top of Page.

MCQ/Objective Type Questions


Duration : 30 Minutes

Marks : 20

1. Choose the most appropriate answer :


1) At a certain cross section of a shaft 180 mm in dia. there is a bending moment of 5 kNm
and twisting moment of 12 kNm. The maximum direct stress induced
in the section will be
a) 16 N/mm2
b) 160 N/mm2 c) 18 N/mm2
d) 180 N/mm2
2) The equivalent length is equal to half the actual length of a column with
a) One end fixed and other end free
b) Both ends fixed
c) Both ends hinged
d) One end fixed and other end hinged
3) The strength of column with both ends fixed is ___________ times the strength of column
when its one end is fixed and other end free other parameters remaining same.
a) 2
b) 4
c) 16
d) 18
4) Maximum principal stress theory was postulated by
a) st Venant
b) Rankine
c) Mohr
d) Tresca
5) If 1and 2 are two normal stress on two mutually perpendicular plane then
the radius of Mohrs circle is ______________
a) (1 2 ) / 2

b) (1 2 ) / 4

c) ( 2 1 ) / 2

1
1

1
1

d) None of the above

6) Three hinged arch is statically ___________


a) Determinate
b) Indeterminate
c) Indeterminate with 2 degree
d) None of the above
7) The equivalent bending moment M under action of bending moment and torque T is
2
2
c) 1 / 2( M2 + T 2 ) d) 1/ 2(M M + T )
b) M M2 + T 2
M2 + T 2
8) An arch acts as inverted cable so it receives
a) Tension
b) Compression
c) Bending moment d) All of the above

a)

P.T.O.

SLR-BB 31

*SLRBB31*

-2-

9) Maximum shear stress on principal plane is


a) Zero
b) Twice that the Major principal stress
c) One half that of Minor principal stress
d) None of the above
10) Max shear stress is given by expression _____________
a) max = (1 2 ) / 2

b) max = (1 2 ) / 4

c) max = (2 1) / 2

d) max = ( 2 1) / 4

11) A simply supported beam of a rectangular section carries a uniformly distributed load
over the whole span. The deflection at the centre is y. If the depth of the beam is doubled,
the deflection at the centre would be

y
y
d)
2
8
The slope at the free end of a cantilever of length 1m is 1. If the cantilever carries a
uniformly distributed load over the whole length, then the deflection at the free end will be
a) 10 mm
b) 13.09 mm
c) 26.18 mm
d) 39.27 mm
Castiglianos second theorem is applicable for
a) Determinate structures
b) Indeterminate structures
c) Both a) and b)
d) None
The maximum strain energy stored in a body without exceeding the limit of
proportionality is
a) Proof resilience
b) Modulus of Rigidity
c) Resilience
d) Modulus of Resilience
Influence line diagram for reaction at support B for following simply supported
a) 2y

12)

13)

14)

15)

b) 4y

c)

beam subjected to unit rolling load is

2
1

a)

b)

c)

d)

16) Muller Breslaus principle can be applied to


a) Determinate structures only
b) Indeterminate structures only
c) Both a) and b)
d) None
17) Influence line diagram is a curve or graph that represents a function like a
a) Reaction at support
b) Shear force at a section
c) Bending moment at a section
d) All of these

______________
Set A

*SLRBB31*

-3-

SLR-BB 31

Seat
No.
S.E. (Civil) (Part II) Examination, 2014
STRUCTURAL MECHANICS II (New)
Day and Date : Tuesday, 25-11-2014
Time : 3.00 p.m. to 6.00 p.m.
Instructions :

1)
2)
3)
4)
5)
6)

Marks : 80

Figures right indicates full marks.


Use of non-programmable calculators is permissible.
Assume suitable data if necessary and mentioned it clearly.
Solve any three questions from Section I.
Solve any three questions from Section II.
Draw neat sketches wherever necessary.

SECTION I
2. a) State assumption made in Eulars theory.
b) Using Eulers theory find the crippling load of a both ends hinged 8m high column of
uniform cross-section of dia 500mm. Hence find the safe load the column can carry if
E = 2 105 N/mm2, Take factor of safety as 2.

3. a) Write note on Mohrs Circle of stress.

b) Find the principal stresses if the bi-axial stress system at a point in a strained material is
x = 150 MPa, y = 75 MPa, xy = 50 MPa . Also Locate principal planes.

10

4. a) Define equivalent BM and Equivalent Torque.


b) A solid circular shaft of dia. 100 mm carries B.M. and T.M. of magnitudes 40KN-m and
20 KN-m respectively. Find the major principal stress induced in the section. Where does it
develop ?

5. a) State and explain Max. Shear Stress Theory.


b) The three-hinged arch bridge has a parabolic shape and supports the uniform load.
Assume the load is uniformly transmitted to the arch ribs. Show that the parabolic arch
is subjected only to axial compression at an intermediate point such as point D.

Fig. No. 1

Set A

SLR-BB 31

-4-

*SLRBB31*

SECTION II
6. A cantilever beam PQ of length 2m carries a point load of 10 kN at Q. The moment of inertia
for the right half of the cantilever is 108 mm4 whereas that for the left half is 2108 mm4. If E =
2108 kN/m2, find the slope and deflection at the
free end of the cantilever, using conjugate beam method.

14

7. Find the central deflection of the uniform bend ABCDEFG as shown in fig. using
strain energy method.

13

8. For the span shown in the sketch below, obtain the bending moment at a section
P, 20m from A due to the loads in the position indicated

13

Also determine the position of the loads for the maximum bending moment of section P and
the value of maximum moment.
9. a) State effect of rolling loads on structures.

b) A uniformly distributed load of 5kN/m, covering a length of 15m, crosses a girder of span
50m. Find the values of maximum S.F. and B.M. at a section
10m from the left hand support.

11

_____________________

Set A

SLR-BB 310

*SLRBB310*
Seat
No.

Set

T.E. (Electricals) (Part I) (New) Examination, 2014


ENGINEERING ECONOMICS AND INDUSTRIAL MANAGEMENT
Day and Date : Friday, 5-12-2014
Time : 3.00 p.m. to 6.00 p.m.
Instructions :

Max. Marks : 100

1) Figures to the right indicate full marks.


2) Q. No. 1 is compulsory. It should be solved in first 30 minutes in
Answer book Page No. 3. Each question carries one mark.
3) Answer MCQ/Objective type questions on Page No. 3 only. Dont
forget to mention, Q.P. Set (A/B/C/D) on Top of Page.
MCQ/Objective Type Questions

Duration : 30 Minutes

Marks : 20

1. Choose the correct answer :


1) Trade cycles normally occurs in
1) Socialist Economy
3) Capitalist Economy

20
2) Mixed Economy
4) Does not occur

2) Trade cycle is an outcome of generous money supply by the banks is an opinion of


1) F. A. Hayak
2) Joseph Schumpeter
3) R. G. Hawtrey
4) None of them
3) Economic Order Quantity (EOQ) is
1) Base Minimum Quantity
3) Reasonable Quantity

2) Maximum Quantity
4) None of the above

4) Cost control and cost reduction are


1) One and the same
3) Unrelated to each other

2) Different concepts
4) Related to each other

5) Partnership consists of minimum 2 members and maximum no limit


1) This statement is true
2) This statement is wrong
3) Statement is neither right nor wrong
4) No any relation
6) Proprietorship is nothing but a company formed by private person with sole ownership
1) The statement is true
2) The statement is false
3) Statement neither true nor false
4) No such companies formed in India
7) Perpetual succession is an essential characteristics of one of the followings
1) Partnership
2) Proprietorship
3) Public Ltd. company
4) None of the above
8) Which of the following is having limited liability of members ?
1) Partnership
2) Proprietorship
3) Public Ltd. Company
4) None of above
9) Infrastructure means
1) Machines
3) Capital

2) Manpower
4) Transport communication facilities

P.T.O.

SLR-BB 310

*SLRBB310*

-2-

10) Power in our context means


1) Political power
3) Energy

2) Govt. Authority
4) None of these

11) Development can be measured through


1) Quantitative parameters
3) By both above

2) Qualitative parameters
4) By none above

12) Thermal energy source


1) Exhausts with its use
3) Both above

2) Can be renewed as and when wants


4) None of above

13) Management is nothing but what a manager does. This is contention of


1) Peter Druker
2) Ralph Davis
3) Louis Allen
4) Ross Moore
14) Management means Decision Making according to
1) Louis Allen
2) Harold Koozntz
3) Ross Moore
4) Stanley Vance
15) Management is
1) One time activity
3) Both above

2) Continuous process
4) None of the above

16) Project Planning planning of project within specific time limit irrespective of cost element.
1) This statement is absolutely correct
2) The statement is totally false
3) Though time is important but cost factor is also equally important
4) None of the above
17) Small scale industries were accorded least importance in
1) All five year plans
2) Only 2nd five year plan
3) Both above
4) None of the above
18) Self employment necessarily means
1) Earn your livelihood at your own
2) Absorbing in the job by Govt.
3) Giving employment to the needy people
4) None of the above
19) Entrepreneurship development means and includes
1) Creation of jobs for unemployed
2) Starting new venture
3) Giving opportunities and facilities to entrepreneurs
4) Imposing many new taxes on entrepreneurs
20) Electricity acts
1) Empowers electrical inspectors to carry out inspections entering the premises

2) Prevents wrongful and illegal activities being carried by the users


3) Both above
4) None of the above

______________

Set A

*SLRBB310*

-3-

SLR-BB 310

Seat
No.

T.E. (Electricals) (Part I) (New) Examination, 2014


ENGINEERING ECONOMICS AND INDUSTRIAL MANAGEMENT
Day and Date : Friday, 5-12-2014
Time : 3.00 p.m. to 6.00 p.m.

Marks : 80

Instruction : Figures to the right indicate full marks.

SECTION I
2. Explain the concepts micro economics and macro economics. How theories in Economics
are guiding decision making ?

10

3. Explain different types of business organizations and state the difference between line and
staff organisation.

10

OR
3. Explain in details the structure and characteristics of Public Ltd. Company.

10

4. Write short notes on any four :

20

1) ABC Analysis
2) Characteristics of Private Company
3) Role of infrastructure in development
4) Agriculture sector in India
5) Estimates of energy requirements
6) Joint stock company.

SECTION II
5. Explain in details PERT and CPM methods, with suitable diagrams, in project implementation.

10

6. Explain different functions of management in details.

10

OR
6. What do you mean by project planning ? Explain the role of cost factor in planning and
implementation of a project ?

10

Set A

SLR-BB 310

-4-

7. Write short notes on any four :

*SLRBB310*
20

1) Monitoring and control of project cost


2) Steps for setting small scale industry
3) Management information system
4) Entrepreneurship development
5) Welfare provisions in Factory Act
6) Industrial safety and its advantages.

_____________________

Set A

SLR-BB 311

*SLR-BB-311*
Seat
No.

Set

T.E. (Part I) Electrical Engineering (New) Examination, 2014


ELECTROMAGNETIC ENGINEERING
Day and Date : Monday, 8-12-2014
Time :3.00 p.m. to 6.00 p.m.

Max. Marks : 100

Instructions : 1) Q. No. 1 is compulsory. It should be solved in first 30 minutes


in Answer Book Page No. 3. Each question carries one mark.
2) Answer MCQ/Objective type questions on Page No. 3 only.
Dont forget to mention, Q.P. Set (A/B/C/D) on Top of Page.
MCQ/Objective Type Questions
Duration : 30 Minutes

Marks : 20

1. Solve all of the following :


1) If dot product of two vectors is zero then vectors are
a) Perpendicular b) Parallel
c) Oblique

(120=20)
d) None of these

2) Vectors A = 3ax + 2ay + az and B = 5ax + 6ay + 3az are _____________


a) Parallel
b) Perpendicular c) At an angle 35 d) None of these
3) The operator del ( ) is a
a) Vector space function
c) Scalar space function

b) Vector time function


d) Scalar time function

4) dsz = r dr d is the differential surface area in


a) Cylindrical co-ordinate
b) Spherical co-ordinate
c) Cartesian co-ordinate
d) None of these
5) Electric Flux Lines passing through Gaussian surface
a) Parallel
b) Perpendicular c) At Oblique
d) None of these
6) The surface over which Gausss law is applied must be
a) Open
b) Closed
c) Ps
d) None of these
7) Work done in electric field is
a) W=Q/V
b) W= V/Q
c) W = QV
d) None of the above
8) Relation between electric field intensity and permittivity is given by
a) D= E
b) E= D
c) D = E/
d) E= /d
9) Electric dipole moment is given by
a) P=Qd2
b) P= d2

c) P= Q d

d) None
P.T.O.

SLR-BB 311

*SLR-BB-311*

-2-

10) The line integral of the vector potential A around the boundary of a source S
represent which of the following.
a) Flux through the surface S
b) Flux density in the surface S
c) Magnetic field intensity
d) Current density
11) Maxwells equation give the relations between
a) Different field
b) Different sources
c) Different boundary conditions
d) None of above
12) For static magnetic field
a) B=
b) B= J
c) . B= 0J
d) B=0
13) If a charge, 1 c is moving with a velocity 2ax in a magnetic field of B = 1ay,
the force on the charge is
a) 2az
b) 2ax
c) 1ay
d) 1az
14) x A = ______________
a) H
b) B

c) J

15) Lorentz force equation is


a) F=Q [E+ V B]
c) F = Q [V + E B]

b) F = Q [E + V B]
d) F = Q [B + V E]

d) None of these

16) The product of no. of turns and the flux is called


a) Field linkage
b) Magnetic field c) Flux linkage
d) Area linkage
17) Magnetic boundary condition is that the normal component of magnetic flux
density is ____________ across the boundary.
a) Unity
b) Zero
c) Non-continuous d) Continuous
18) x H = J is known as
a) Maxwells equation
c) Ohms law

b) Amperes law
d) Gausss law

D
is
t
a) Maxwells equation
b) Amperes law
c) Ohms law
d) Gausss law
20) If N is total Nos. of turns, I is the current passing through the coil, L is the
mean length of the path then H is given by

19) x H = J +

a) H=NIL

b) H=

NI
NL
c) H=
L
I
______________

d) H=

I
NL

Set A

*SLR-BB-311*

-3-

SLR-BB 311

Seat
No.

T.E. (Part I) Electrical Engineering (New) Examination, 2014


ELECTROMAGNETIC ENGINEERING
Day and Date : Monday, 8-12-2014
Time :3.00 p.m. to 6.00 p.m.

Max. Marks : 80

SECTION I
2. Solve any four from the following :

(54=20)

1) Transform the given vector A = 2xyz ax 5(x+y+z) az at (2,60,3) into


cylindrical coordinate system.
2) Find the total charge inside the volume having volume charge density as
10Z2e0.1x sin y C/m2. the volume is defined between 2 x 2, 0 y 1,
3 z 4.
3) Two infinite uniform sheets of charge of 20 nc/m2 are located at X = 1m.
determine E in all regions.
4) State gauss law and show that the divergence of electric field and that of
electric flux density in charge free region is zero.
5) If E = 8 xy ax 4x2 ay + az v/m. Find the work done carrying a 6 coulomb
charge from A(1, 8, 5) to B(2, 18, 6) along the path y = 3x + 2, z = x + 4.
3. Solve any two from the following :

(102=20)

1) Derive the boundary conditions for E and D at the interface between two
dielectrics.
2) Given that D = 5x3/2 ax C/m2, evaluate both sides of divergence theorem
for the volume of a cube 1 m on an edge, centered at origin and with edges
parallel to the axis.
3) State and derive the uniqueness theorem.

Set A

SLR-BB 311

-4-

*SLR-BB-311*

SECTION II
4. Solve any four from the following :

(54=20)

1) Derive the equation for vector magnetic potential in terms of current density J.
2) Derive the Maxwells equation in point and integral form from Faradays law
for static field.
3) State and prove Strokes Theorem.
4) Current filaments of 3 ax A lies along the X axis. Find H at P(1, 3, 2).
5) Derive the expression for Energy density in magnetic field.
5. Solve any two from the following :

(102=20)

1) Determine the Amperes law to differential element.


2) Derive the Maxwells equation in point and integral form from Amperes law
for time varying field.
3) Let the permeability be 5 H/m in region A where x < 0 and 20 H/m in the
region B where x > 0. If there is surface current density is 150 ax 400 ay +
500 az A/m, at x = 0 and if HA = 300 ax 400 ay + 500 az A/m. Find
i) 1 HtA 1
ii) 1 HnA 1
iii) 1 HtB 1
iv) 1 HnB 1
_____________________

Set A

SLR-BB 312

*SLR-BB-312*
Seat
No.

Set

T.E. (Electrical Engineering) (Part I) Examination, 2014


CONTROL SYSTEMS I (New)
Day and Date : Wednesday, 10-12-2014
Time : 3.00 p.m. to 6.00 p.m.

Max. Marks : 100

Instructions : 1)
2)
3)
4)

All questions are compulsory.


Assume suitable data wherever necessary.
Figures to the right indicates full marks.
Q. No. 1 is compulsory. It should be solved in first 30 minutes in
Answer Book Page No. 3. Each question carries one mark.
5) Answer MCQ/Objective type questions on Page No. 3 only.
Dont forget to mention, Q.P. Set (A/B/C/D) on Top of Page.
MCQ/Objective Type Questions

Duration : 30 Minutes

Marks : 20

1. Choose the correct options :

(120=20)

1) If the gain of the critical damped system is increased it will behave as


a) oscillatory
b) critically damped c) overdamped
d) underdamped
2) A.C. servomotor is basically a
a) universal motor
c) two phase induction motor

b) single phase induction motor


d) three phase induction motor

3) Signal Flow Graph (SFG) is a


a) polar graph
b) semi log graph
c) logloggraph
d) a topological representation of a set of differential equations
4) Pick up false statement. Routh-Hurwitz criterion
a) is used for determining stability of a system
b) is an algebraic procedure
c) gives the exact location of roots of the characteristic equation
d) does not indicate relative degree of stability or instability
5) A control system in which the control action is somehow dependent on the
output is known as
a) Closed loop system
b) Semiclosed loop system
c) Open system
d) None of the above
6) A good control system has all the following features except
a) good stability
b) slow response
c) good accuracy
d) sufficient power handling capacity
P.T.O.

SLR-BB 312

-2-

7) Regenerative feedback implies feedback with


a) oscillations
b) step input
c) negative sign

*SLR-BB-312*
d) positive sign

8) Transfer function of a system is used to calculate which of the following ?


a) The order of the system
b) The time constant
c) The output for any given input
d) The steady state gain
9) In an automatic control system which of the following elements is not used ?
a) Error detector
b) Final control element
c) Sensor
d) Oscillator
10) In force-voltage analogy, velocity is analogous to
a) current
b) charge
c) inductance

d) capacitance

11) Static error co-efficients are used as a measure of the effectiveness of closed loop
systems for specified ____________ input signal.
a) acceleration
b) velocity
c) position
d) all of the above
12) The type 0 system has ____________ at the origin.
a) no pole
b) net pole
c) simple pole

d) two poles

13) Phase margin of a system is used to specify which of the following ?


a) Frequency response
b) Absolute stability
c) Relative stability
d) Time response
14) ____________ technique gives quick transient and stability response.
a) Root locus
b) Bode
c) Nyquist
d) Nichols
15) Which of the following is exhibited by Root locus diagrams ?
a) The poles of the transfer function for a set of parameter values
b) The bandwidth of the system
c) The response of a system to a step input
d) The frequency response of a system
k
16) Given a unity feedback control system with G(s) =
, the value of K for a
s(s + 4)
damping ratio of 0.5 is
a) 1
b) 16
c) 32
d) 64
17) A system with gain margin close to unity or a phase margin close to zero is
a) highly stable
b) oscillatory
c) relatively stable d) unstable
18) An increase in gain, most systems, leads to
a) smaller damping ratio
b) larger damping ratio
c) constant damping ratio
d) none of the above
19) Velocity error constant of a system is measured when the input to the system is
unit ___________ function.
a) parabolic
b) ramp
c) impulse
d) step
20) Due to which of the following reasons excessive bond width in control systems
should be avoided ?
a) It leads to slow speed of response b) It leads to low relative stability
c) Noise is proportional to bandwidth d) None of the above
______________
Set A

*SLR-BB-312*

-3-

SLR-BB 312

Seat
No.

T.E. (Electrical Engineering) (Part I) Examination, 2014


CONTROL SYSTEMS I (New)
Day and Date : Wednesday, 10-12-2014
Time : 3.00 p.m. to 6.00 p.m.

Marks : 80

Instructions : 1) All questions are compulsory.


2) Assume suitable data wherever necessary.
3) Figures to the right indicates full marks.
SECTION I
2. Solve any four :
a) Explain Masons Gain formula of signal flow graph.
b) Find the transfer function of the network below :

(45=20)

c) Draw analogous network of mechanical system below by F-V analogy.

Set A

SLR-BB 312

-4-

*SLR-BB-312*

d) Derive the transfer function of a simple closed loop control system.


e) Find the T/F of block diagram below :

3. a) Fig. shows the block diagram of system with two inputs

1) The set point R(s) and


2) The load disturbance D(s).
Show by the principle of superposition the effect on the output of system.

b) Explain the rules for, block diagram reduction techniques.

10

10

OR

Set A

*SLR-BB-312*

-5-

SLR-BB 312

b) Convert the block diagram to signal flow graph and determine the closed loop
transfer function by masons gain formula.

10

SECTION II
4. Solve any four :

(45=20)

1) How stability is related to location of pole ? Explain in detail for different case of
impulse input.
2) The open loop TF is given as G(S) =

100
find :
s + 15s + 100
2

a) Peak time
b) Percent overshoot
c) Settling time
d) Rise time.
3) Find the range of k to make the system stable for the unity feedback system using
Routh-Hurwitz criteria.
G(s) =

k(s + 20)
.
s(s + 2)(s + 3)

4) Explain the frequency domain specifications.


5) The open loop transfer function of a unity feedback system is given by
1
. Sketch the polar plot and determine the gain margin and
s (1 + s)(1 + 2s)
phase margin.

G(s) =

Set A

SLR-BB 312

-6-

*SLR-BB-312*

5. Solve any two :

(210=20)

1) Sketch the root locus of the unity feedback system with OLTF
G(s) =

k
.
s(s + 2)(s + 4)

10
. Determine
s(1 + 0.4s)(1 + 0.1s)
a) gain cross over frequency b) the phase crossover frequency.

2) Draw the Bode plot for the system having, G(s) =

3) A unity feedback control system has an amplifier with gain KA = 10 and gain
ratio, G(s) = 1/s(s + 2) in the feed forward path. A derivative feedback, H(s) = sK0
is introduced as a minor loop around G(s). Determine the derivative feedback
constant, K0 so that the system damping factor is 0.6.

_____________________

Set A

SLR-BB 313

*SLRBB313*
Seat
No.

Set

T.E. (Electrical Engineering) (Part I) Examination, 2014


ELECTRICAL MACHINES III (New)
Day and Date : Friday, 12-12-2014
Time : 3.00 p.m. to 6.00 p.m.

Max. Marks : 100

Instructions : 1) Q. No. 1 is compulsory. It should be solved in first 30 minutes


in Answer book Page No. 3. Each question carries one mark.
2) Answer MCQ/Objective type questions on Page No. 3 only.
Dont forget to mention, Q.P. Set (A/B/C/D) on Top of Page.
MCQ/Objective Type Questions
Duration : 30 Minutes

Marks : 20

1. Choose the correct answer :

(120=20)

1) The slot angle = ______________ if S = No. of slots per pole.


a) 180 + S
b) S/180
c) 180 S
d) 180/S
2) In alternator electrical power is given to ______________ winding.
a) Armature
b) Field
c) Damper
d) Compensation
3) The synchronous reactance is the
a) Reactance due to armature reaction of the machine
b) Reactance due to leakage flux
c) Combined reactance due to leakage flux and armature reaction
d) Reactance either due to leakage flux or armature reaction
4) Armature winding of alternator are generally
a) Closed winding
b) Open winding
c) Partly open and partly closed
d) None
5) The effect of armature reaction mainly depends on
a) Load current
b) Load PF
c) Load voltage

d) Both a) and b)

6) A 3ph alternator has 3 slots per pole. The distribution factor of the winding is
a) 0.866
b) 0.5
c) 1
d) None of the above
7) For a machine on infinite bus active power can be varied by
a) Changing field excitation
b) Changing of prime mover speed
c) Both a) and b)
d) None of the above
P.T.O.

SLR-BB 313

-2-

*SLRBB313*

8) Two alternators are running in parallel. The excitation of one of the


alternator is increased. The result will be
a) Machine with excess excitation will burn
b) Both machines will start vibrating
c) Power output will decrease
d) Watt less component will increase
9) The potier triangle separates
a) Stator and root losses
b) Fixed and variable losses
c) Armature voltage and field voltage
d) Armature leakage reactance and armature reaction MMF
10) In case of alternators, the dark and bright lamp method is used for
a) Phase sequence b) Load balancing c) Synchronizing d) Load transfer
11) In synchronous motor the magnitude of stator back emf depends on
a) Speed of motor
b) Load on motor
c) Both speed and rotor flux
d) DC excitation only
12) V curves of synchronous motor are drawn between
a) Ia and V
b) Ia and If
c) Ia and PF
d) Ia and Eb
13) Damper winding in a synchronous motor is on ______________
a) Salient pole
b) Non-salient pole
c) Identical to that of a DC motor
d) None of the above
14) When the synchronous motor is on no load the load angle is about
a) 45 electrical
b) 90 electrical c) 30 electrical d) 0 electrical
15) The synchronous motor is operating at 0.5 pF leading. The excitation is called
a) Under
b) Normal
c) Over
d) Super
16) The hunting in a synchronous motor takes place when
a) Friction in bearings is more
b) Airgap is less
c) Load is variable
d) Load is constant
17) Which synchronous motor will be smallest in size ?
a) 5 HP, 500 rpm b) 5 HP, 375 rpm c) 10 HP, 500 rpm d) 10 HP, 375 rpm
18) The most popular types of stepper motors are ______________
a) Shunt and series
b) Shunt and compound
c) PM and thin disc rotor
d) PM and VR
19) The stepping angle for a 3ph, 24 pole PM stepper motor is ______________
a) 5/step
b) 10/step
c) 15/step
d) 2.5/step
20) One of the basic requirements of a servomotor is that it must produce high
torque at all
a) Loads
b) Frequencies c) Speeds
d) Voltages
______________
Set A

*SLRBB313*

SLR-BB 313

-3-

Seat
No.

T.E. (Electrical Engineering) (Part I) Examination, 2014


ELECTRICAL MACHINES III (New)
Day and Date : Friday, 12-12-2014
Time : 3.00 p.m. to 6.00 p.m.

Marks : 80

2. Solve any four :


(54=20)
a) Define pitch factor. Derive expression for distribution factor.
b) Derive output power equation of smooth cylindrical rotor alternator.
c) What is armature reaction in alternator and what effect will produce for lagging
and leading load ?
d) Explain two reaction theory for salient pole synchronous machines.
e) A 60 KVA, 400V, 50Hz, 3 ph Alternator has effective resistance 0.016 ohm
and an armature reactance of 0.07 ohm. Find the voltage induced in the
armature when the alternator is delivering rated current at a load PF of
i) Unity

ii) 0.7 lagging.

3. Solve any two :


(102=20)
a) What are the necessary conditions for parallel operation of alternator ? Explain
synchronisation methods.
b) 3 phase, 50 Hz, star connected 2000 KVA, 2300 V alternator gives short
circuit current of 600 Amps for certain field excitation. With the same excitation,
open circuit voltage was 900 V. The resistance per phase of armature is
0.6 ohm. Find the full load regulation at
i) UPF

ii) 0.8 PF lag by EMF method.

c) The open circuit and short circuit test results for a 3 phase star connected
alternator 1000 KVA, 1905 V, 50 Hz are given below :
Open circuit terminal voltage
Short circuit current
Field current

760 1500

1700

1905 2300 2600

220

335

10

20

25

30

40

50

The armature resistance per phase is 0.2 ohm. Find the regulation of alternator
on full load 0.8 lagging PF by MMF method.
Set A

SLR-BB 313

-4-

*SLRBB313*

4. Solve any four :

(54=20)

a) Explain construction and working of permanent magnet DC motor.


b) Explain about hysteresis motor.
c) Explain the principle of operation of synchronous motor.
d) Draw and explain equivalent diagram and vector diagram of synchronous
motor on per phase basis.
e) Explain construction and working of switched reluctance motor.
5. Solve any two :

(102=20)

a) Explain the operation of synchronous motor with constant load and following
different excitation with phasor diagram :
i) Normal excitation
ii) Under excitation
iii) Over excitation.
b) A 400 V, 10 HP, 3 phase star connected synchronous motor has negligible
armature resistance and has synchronous reactance of 10 ohm/ph. Find the
minimum current and the corresponding emf induced for full load conditions.
Assume efficiency of 85%.
c) Define servomechanism and explain in detail DC and AC servomotors.
_____________________

Set A

SLR-BB 315

*SLRBB315*
Seat
No.

Set

T.E. (Electrical) (Part II) Examination, 2014


POWER SYSTEM II
Day and Date : Tuesday, 25-11-2014
Max. Marks : 100
Time : 10.00 a.m. to 1.00 p.m.
Instructions : 1) Q. No. 1 is compulsory. It should be solved in first 30 minutes in Answer
book Page No. 3. Each question carries one mark.
2) Assume suitable data if necessary.
3) Answer MCQ/Objective type questions on Page No. 3 only. Dont forget
to mention, Q.P. Set (A/B/C/D) on Top of Page.
MCQ/Objective Type Questions
Duration : 30 Minutes
1. Objective questions :

Marks : 20
(201=20)

1) Which of the following results in symmetrical faults ?


a) Single phase to earth
b) Phase to phase
c) All the three phase to earth
d) Two phase to earth
2) For a faults at the terminals of synchronous generator, the faults current is maximum for a
a) 3-phase fault
b) 3-phase to ground fault
c) line-to-ground fault
d) line-to-line fault
3) An equipment has a per unit impedance of 0.9 pu to a base of 20 MVA, 33 kv. The pu impedance to
the base of 50 MVA and 11 kv will be
a) 4.7
b) 20.25
c) 0.9
d) None of the above
4) Four alternators, each rated at 5 MVA, 11 kv with 20% reactance are working in parallel. The
short-circuit level at bus-bars is
a) 6.25 MVA
b) 20 MVA
c) 25 MVA
d) 100 MVA
5) Zero sequence components in 3-phase voltage of delta is
a) Line voltage
b) Zero
c) Line voltages

d) Line voltage/3

6) The positive-sequence current of a transmission line is


a) Always zero
b) One third of negative-sequence current
c) Equal to negative-sequence current
d) Three-times the negative sequence current
7) Zero-sequence currents can flow form a line to transfer bank if the winding are in
a) Grounded star/delta
b) Delta/star
c) Star/grounded star
d) Delta/delta
8) For the transformer connection shown below

The correct zero sequence equivalent network is


a)

b)

c)

d)
P.T.O.

SLR-BB 315

*SLRBB315*

-2-

9) When a line-to-ground fault occurs, the current in a faulted phase is 100 A. The zero sequence
current in this case will be
a) zero
b) 33.3
c) 66.6
d) 100 A
10) Steady-states stability of a power system is the ability of the power system to
a) Maintain voltage at the rated voltage level
b) Maintain frequency exactly at 50 Hz
c) Maintain a spinning reserve margin at all times
d) Maintain synchronism between machines and on external tie lines
11) A 100 MVA, 11 kv, 3-phase, 50 Hz, 8-pole synchronous generator has an intertia constant H of
equal to 4 seconds. The energy stored in the rotor of the generator at synchronous speed will be
a) 100 MJ
b) 400 MJ
c) 800 MJ
d) 12.5 MJ
12) The equal area criterion of stability is used for
a) No load on the bus-bar
b) One machine and infinite bus-bar
c) More than one machine and infinite bus-bar
d) None of the above
13) Load-flow study is carried out for
a) Load-frequency control
c) Fault calculations

b) Planning of power system


d) Study of stability of the system

14) Load bus is specified by


a) P and V
b) P and

c) P and |V|

d) P and Q

15) The voltage of particular bus can be controlled by controlling the


a) Active power of the bus
b) Reactive power of the bus
c) Phase angle
d) Both a) and b)
16) Gauss-Saidal interactive method can be used for solving a set of
a) Linear differential equation only
b) Linear algebraic equation only
c) Both Linear and non-linear algebraic equation
d) Both linear and non-linear differential equation
17) An acceleration factor is used in ________ method of load flow studies.
a) Gauss-Seidal
b) Newton-Raphon
c) De-coupled

d) All of the above

18) For accurate load-flow calculation on large power systems __________ method is best one.
a) G-S
b) N R
c) De-coupled
d) None
19) A star-connected 3-phase 11kv, 25 MVA alternator with its neutral grounded through a 0.033 pu
reactance (based on the alternator rating) has positive, negative and zero sequence reactance of
0.2 pu, 0.1 pu and 0.1 pu respectively. A single line to ground fault on one of its terminals would
result in a fault MVA of
a) 150 MVA
b) 125 MVA
c) 100 MVA
d) 50 MVA
20) The zero-sequence fault current are absent when the fault is
a) Single line to ground
b) Line to line
c) Double line to ground
d) None of the above
______________

Set A

*SLRBB315*

-3-

SLR-BB 315

Seat
No.
T.E. (Electrical) (Part II) Examination, 2014
POWER SYSTEM II
Day and Date : Tuesday, 25-11-2014
Time : 10.00 a.m. to 1.00 p.m.

Marks : 80

Instructions : 1) All questions are compulsory.


2) Assume suitable data if necessary.
SECTION I
2. Solve any four :
(46=24)
1) Explain the terms base KVA, Base KV and percentage reactance, State the relation between them.
2) Draw the PV impedance diagram for the power system shown in figure. Use base of 100MVA,
220KV in 50 line. The rating of the generator, motor and transformers are
G 40MVA, 25KV,X=20%
M 50MVA, 11KV, X=30%
YYT/F 40MVA, 33Y 220YKV, X=15%
Y T/F 30MVA, 11 220YKV, X=15%

3) Discuss the method of formation of Ybus.


4) Develop the static load flow equations (SLFE).
5) A sample power system has the following line data form the bus admittance matrix for this system.
Bus code
Series impedance
12
0.025 + j0.1
13
0.1 + j0.4
24
0.04 + j0.16
34
0.05 + j0.2
32
0.02 + j0.08
6) Discuss the various types of buses used in the flow studies.
3. 1) A three bus system is shown in figure, line impedance are marked in pu. Treat bus 1 as slack bus
with E1 = 1.05 < 0 pu and bus 2 as PQ bus with P2 + jQ2 = 5.96 + j1.46 pu and Bus 3 as PV bus
with E3 = 1.02 pu. Calculate the voltage of bus 2 at the end of first iteration. Use the Gauss-Seidal
method with acceleration factor 1.4.
0

Assume E 2 = 1.0 < 0 pu and E 03 = 1.02 < 0 pu.


2) With a flow chart explain the Newton Raphson iterative method for solving load flow problems.

8
8

OR
2) For the system shown in figure determine the generator voltage.

Set A

SLR-BB 315

-4-

*SLRBB315*

SECTION II
4. Solve any four :

(46=24)

1) Explain the time varying reactances of the synchronous machine when its terminals are short circuited.
2) Two 3-phase 11kv generators of capacities 8 MVA and 4 MVA and sub transient reactances of 8%
and 4% respectively operate in parallel. The generating station is connected to a transmission line of
200km length, through a step-up transformer of capacity 4MVA and having percentage reactance of
3.5%. The transmission line impedance is (0.025+j0.1) /km and it operate at 66kv. Calculate the
short circuit MVA for a three phase symmetrical fault at the receiving end of the transmission line.
3) Explain what is the difference between the symmetrical components of positive, negative and zero
phase sequence.
4) What are causes of unsymmetrical faults in the power system ? Derive the expression for the fault
current for a single line to ground fault.
5) Determine symmetrical components of currents in line a, the original phases of which are Ia = 12 + j6,
Ib = 12 j12, Ic = 15 + j10.
6) Draw the zero sequence networks for the system shown in figure.

5. Solve any two :


(82=16)
1) A double line fault occurs between the two phases of the transmission line with impedance Zf
between them. Find the interconnection of the three sequence networks and hence determine the
fault current.
2) For the given system find fault current for a line to line fault on phase b and c at point q. Assume
1.0 pu prefault voltage.

Element
Positive Seq X1
Negative Seq X2
G
0.1
0.12
M1
0.05
0.06
M2
0.05
0.06
T1
0.07
0.07
T2
0.08
0.08
Line
0.1
0.1
Generator grounding reactance is 0.5 pv.
3) Find the tree seuqence networks of the transmission lines.
_____________________

Zero Seq X3
0.05
0.025
0.025
0.07
0.08
0.1

Set A

SLR-BB 316

*SLRBB316*
Seat
No.

Set

T.E. (Electrical) (Part II) Examination, 2014


CONTROL SYSTEM II
Day and Date : Wednesday, 26-11-2014
Time : 10.00 a.m. to 1.00 p.m.
Instructions :

Max. Marks : 100

i)
ii)
iii)
iv)

All questions are compulsory.


Assume suitable data, if necessary.
Use of no-programmable calculator is allowed.
Q. No. 1 is compulsory. It should be solved in first 30 minutes in
Answer book Page No. 3. Each question carries one mark.
v) Answer MCQ/Objective type questions on Page No. 3 only. Dont
forget to mention, Q.P. Set (A/B/C/D) on Top of Page.
MCQ/Objective Type Questions

Duration : 30 Minutes

Marks : 20

1. Tick mark the correct answer :


(201=20)
1) In order to increase the damping of a badly underdamped system which of following
compensator may be used
a) Phase-Lead
b) Phase-Lag
c) Both a) and b)
d) None of the above
2) A differentiator is usually not a part of control system because it
a) Reduces damping
b) Reduces the gain margin
c) Increases input noise
d) Increase error
3) In a control system integral error compensation ____________ steady state error.
a) Increases
b) Minimizes
c) Does not have any effect
d) None of the above
4) A property of phase lead compensation is that the
a) Overshoot is increased
b) Bandwidth of CL system is reduced
c) Rise-time of CL system is reduced
d) Gain margin is reduced
5) The TF of a phase-lead compensator is given by GC (s) =
maximum phase shift provided by such a compensator is

1 + 3 Ts
where T > 0. The
1 + Ts

b) 3
c) 4
d) 6
a) 2
6) The phase lag produced by the transportation relays
a) Is independent of frequency
b) Is inversely proportional to frequency
c) Increases linearly with frequency
d) Decreases linearly with frequency
7) Gain margin is the factor by which the gain of the system is increased to make it
a) Stable
b) Unstable
c) Oscillatory
d) Exponential
8) The effect of error rate damping is
a) Reduced steady state error
b) Faster response
c) Larger settling time
d) Both a) and b)
P.T.O.

SLR-BB 316

-2-

*SLRBB316*

9) Due to which of the following reasons excessive bandwidth in control system should be
avoided ?
a) It leads to slow speed of response
b) It leads to low relative stability
c) Noise is proportional to bandwidth
d) None of the above
10) The eigen values of a linear system are the locations of
a) Finite poles
b) Poles of the system
c) Zeros of the system
d) None of the above
 = AX , where X is the
11) The state variable description of a linear autonomous system is X

12)

13)
14)

15)

16)
17)

0 2
two dimensional state vector and A is the system matrix given by A =
. The roots of
2 0
the characteristic equation are
a) 2 and + 2
b) j2 and + j2
c) 2 and 2
d) + 2 and + 2
The state variable approach can be applied to ____________ system(s).
a) Time variant
b) Non-linear
c) Linear and time-invariant
d) All of the above
For an nth order system the state equations will be of the order of
a) n
b) 1
c) n/2
d) (n + 1)/2
Which of the following is an example of intentional nonlinear system ?
a) Contractor serves
b) Nonlinear pitch or dampers in aircraft control
c) On-off transducers
d) All of the above
For which of the following reasons nonlinearities are introduced ?
a) To simplify the system
b) To achieve special characteristics
c) Both a) and b)
d) None of the above
____________ is the nonlinearity caused by servomotor.
a) Dead space
b) Saturation
c) Backlash
d) Static friction
If u(t) is the unit step and (t ) is the unit impulse function, the inverse
z-transform of F(z) =

1
for k > 0 is
z+1

a) (1)k (k)
d) u(k) (1)k
b) (k) ( 1)k
c) (1)ku(k)
18) The right hand plane of s-plane, when mapped into z-plane, when the direction of contour
is anticlockwise ?
a) Covers the entire portion of inside of the unit circle
b) Covers the entire portion of outside of the unit circle
c) It falls on the unit circle
d) It covers the entire portion except the unit circle
19) The characteristic equation of second-order sampled data system is given by
F(z)
= a2z2 + a1z + a0 = 0, a2 > 0. What are the stability constraints for this system ?
1) a2 + a1 + a0 > 0
2) a2 a1 + a0 > 0 3) |a0| < a2
4) |a0| > a2
5) |a1| < a2
Select the correct answer using the code given below :
a) Only 1, 2, and 3
b) Only 1, 2, and 4 c) Only 1, 3, and 5 d) Only 2, 3, and 5
20) The output of first order hold between two consecutive sampling instants is a
a) Constant
b) Quadratic function
c) Ramp function
d) Exponential function

______________
Set A

*SLRBB316*

-3-

SLR-B-316

Seat
No.

T.E. (Electrical) (Part II) Examination, 2014


CONTROL SYSTEM II
Day and Date : Wednesday, 26-11-2014
Time : 10.00 a.m. to 1.00 p.m.
Instructions :

Marks : 80

i) All questions are compulsory


ii) Assume suitable data. if necessary.
iii) Use of non-programmable calculator is allowed.

SECTION I
2. Solve any four :
(54=20)
a) Write the steps to design lag compensator using root locus technique.
b) Derive the realization of lead compensator circuit.
c) Explain the procedure to design lag-lead compensator using frequency response.
d) Consider a matrix A given below find the eigen values and eigen vector and model matrix
0 1 0
A = 3 0 2

12 7 6
x 1 2 3 x1 3
e) Consider a system having state model =
+ u (t) : Y = [1 1]
x 2 4 2 x 2 5
with D = 0. Obtain the Transfer Function.
3. a) Design a suitable lag compensator for a system whose OLTF is GC(s) =

x1
x
2

K
s(s + 1)(s + 4)

to meet the following specifications :


i) Damping ratio = 0.5
ii) Settling time = 10 sec,
iii) Kv 5 sec1.

10

b) The TF of a certain unity negative feedback control system is given by


G(s) =

K
. Design a lead compensator to satisfy the following specifications :
s(s + 2)(s + 30 )

10

a) PM 35
b) GM 20 dB
c) Steady state error for unit ramp input 25.

OR
b) The state variable model of a continuous LTI system is
x 1 0 2 x 1 0
x = 3 5 x + 1 u ( t) . Calculate the time solution of the state vector.
2
2

10

Set A

SLR-BB 316

-4-

*SLRBB316*

SECTION II
4. Solve any four :

(54=20)

a) Explain isocline method for construction of phase trajectories.


b) Explain in short friction and dead-zone nonlinearity.
c) Derive the transfer function of zero order hold device.
d) Determine the z-domain transfer fucntion for the following s-domain transfer functions
H(s) =

a
.
(s + a )2

e) Determine the inverse z-transform of z-domain function F(z) =

3z 2 + 2z + 1
.
z 2 3z + 2

5. Solve any two :

(102=20)

a) Derive the describing function of dead-zone and saturation nonlinearity.


b) Check the stability of the sampled data control system represented by the equation
z3 0.2z2 0.25z + 0.05 = 0.

c) What is stability criterion for sampled data control system ? Explain any one method for
stability in detail.
_____________________

Set A

SLR-BB 317

*SLRBB317*
Seat
No.

Set

T.E. (Electrical) (Part II) Examination, 2014


MICROPROCESSOR AND APPLICATIONS
Day and Date : Thursday, 27-11-2014
Time : 10.00 a.m. to 1.00 p.m.

Max. Marks : 100

Instructions : 1) Q. No. 1 is compulsory. It should be solved in first 30


minutes in Answer Book Page No. 3. Each question carries
one mark.
2) Answer MCQ/Objective type questions on Page No. 3 only.
Dont forget to mention, Q.P. Set (A/B/C/D) on Top of Page.
MCQ/Objective Type Questions
Duration : 30 Minutes

Marks : 20

1. Choose the correct alternative :


1) Special purpose register is __________
a) Accumulator
b) Flag register
c) Program counter
d) W Register
2) 8085 access __________
a) 256 input ports
c) 256 I/O ports

b) 256 output ports


d) 64 I/O ports

3) TRAP is __________ where as RST 6.5, RST 5.5, RST 7.5 are __________
a) Maskable, non maskable
b) Maskable, maskable
c) Non maskable, nonmaskable
d) Nonmaskable, maskable
4) The width of the stack pointer in 8085 is __________
a) 8 bit
b) 32 bit
c) 16 bit

d) 64 bit

5) JN2 8000 is how many byte instruction ?


a) 4
b) 3
c) 2

d) 1

6) What happens when HLT instruction is executed in microprocessor ?


a) Microprocessor enters into wait state
b) Microprocessor enters into halt state and buses are tristated
c) Microprocessor restart its execution by making PC = 0000
d) Microprocessor performs NOP
7) Which of the following is the example of direct addressing mode ?
a) STA C200
b) mov A, B
c) mvIA, 08H
d) RAL

P.T.O.

SLR-BB 317

*SLRBB317*

-2-

8) The instruction used for data transfer in I/O mapped I/O are _______
a) IN, OUT
b) LDA address
c) STA address
d) LDA addr, STA addr
9) INTA pin is the acknowledgement signal for _______
a) RST 5.5
b) RST 6.5
c) TRAP

d) INTR

10) If the contents of PC are 8007 H, then after execution of JMP 8010 instruction,
the contents of PC will be
a) 8008
b) 8010
c) 8009
d) 800 A
11) When A1 = 1, A0 = 1 which port of 8255 is selected ?
a) Port A
b) Port B
c) Port C

d) CWR

12) How many modes are there in 8253 ?


a) One
b) Two
c) Three

d) Six

13) Which IC is used for USART ?


a) 8279
b) 8255

d) 8051

c) 8251

14) 8255 has ___________ programmable I/O pins arranged in a three 8 bit
ports.
a) 12
b) 24
c) 32
d) 6
15) How many interrupt levels are handled by 8259 ?
a) 64
b) 128
c) 16
16) Mode 0 of 8253 is ________
a) Interrupt on terminal count
c) Rate generator

d) None of these

b) Programmable one short


d) Square wave generator

17) Vector address of RST 6.5 Interrupt is __________


a) 0034 H
b) 0024 H
c) 002C H

d) 003C H

18) SID and SOD are of following types of signals.


a) Reset signals
b) Serial I/O signals
c) DMA signals
d) Status signals
19) Which of following is not conditional jump instruction ?
a) JC
b) JZ
c) JD

d) JP

20) Clock frequency of 8085 microprocessor is __________


a) 1 MHz
b) 2 MHz
c) 3 MHz
d) 4 MHz
______________

Set A

*SLRBB317*

-3-

SLR-BB 317

Seat
No.

T.E. (Electrical) (Part II) Examination, 2014


MICROPROCESSOR AND APPLICATIONS
Day and Date : Thursday, 27-11-2014
Time : 10.00 a.m. to 1.00 p.m.

Marks : 80

SECTION I
2. Solve any four :

(45=20)

1) Discuss brief function of following pins :


a) READY
b) IolM
c) X1, X2
d) RESET OUT
e) SO, S1.
2) Draw the flow chart and write an assembly language program to move a
block of 8 decimal numbers from one memory location to the other and arrange
them in reverse order.
3) Draw and timing diagram for memory read machine cycle with introduction of
two wait states.
4) Write down the addressing modes and explain the following instructions.
a) LDAX B
b) MVI A, XXH c) JNZ 4000 H
5) Write a short note on memory mapped I/O.
3. a) What are hardware and software interrupts ? With suitable circuit or internal
block diagram explain interrupt structure of 8085. Highlight triggering levels
and ISR addresses of different interrupts.
10
b) Design a scheme to interface :
i) 2 KB ROM in the address range 0000H to 07FFH.
ii) 2KB RAM in the address range 0800 H to 0FFFH.

10

OR
Interface 16kx8 EPROM to 8085. The available memory chip is 4kx4. The
address should start from 0000H onwards. Give the detailed interfacing
schematic and address range for each chip used.
10
Set A

SLR-BB 317

-4-

*SLRBB317*

SECTION II
4. Solve any four :

(54=20)

a) With a neat diagram explain the working of dual slope ADC.


b) Draw and explain the block diagram of 8253.
c) What is BSR mode in 8255 ? Write a program to set D5 bit of port C.
d) Write an assembly language program to generate triangular wave at the output
of 0808 DAC. Draw the interfacing diagram.
e) Explain mode 2 of 8255 PPI.
5. Solve any two :

(102=20)

a) Describe various features of 8259 PIC. Draw the functional block diagram
of 8259 PIC and explain its working in detail.
b) With the help of neat schematic describe stepper motor interfacing.
c) Draw and explain 8251 USART in detail. Explain the synchronous mode
operation for 8251 USART.
_____________________

Set A

SLR-BB 318

*SLRBB318*
S

T.E. (Electrical) (Part II) Examination, 2014


POWER ELECTRONICS
Day and Date : Friday, 28-11-2014
Time : 10.00 a.m. to 1.00 p.m.

Max. Marks : 100

Instructions : 1) Q. No. 1 is compulsory. It should be solved in first 30 minutes in


Answer book Page No. 3. Each question carries one mark.
2) Answer MCQ/Objective type questions on Page No. 3 only. Dont
forget to mention, Q.P. Set (A/B/C/D) on Top of Page.
Duration : 30 Minutes

MCQ/Objective Type Questions

1. Solve all :

Marks : 20
(201=20)

1) The function of Snubber circuit connected across the SCR is to


a) Suppress dv/dt
b) Increase dv/dt
c) Decrease dv/dt
d) Decrease di/dt
2) A four quadrant operation require
a) Two full converters in series
b) Two full converters connected back to back
c) Two full converters connected in parallel
d) Two semi conductors connected back to back
3) In a circulating-current type of dual converter, the nature of the voltage across the reactor is
a) Alternating
b) Pulsating
c) Direct
d) Triangular
4) A single-phase full bridge inverter can operate in load-commutation mode in case load
consists of
a) RL load
b) RLC under damped
c) RLC damped
d) RLC critically damped
5) In thyristor, holding current is
a) more than the latching current
c) equal to latching current

b) less than the latching current


d) none of the above

6) When a thyristor turn on, the gate drive


a) can be turned down but thyristor remains in on position
b) cannot be turned down as thyristor will be turned off
c) gate drive has no impact on starting and turning off the thyristor
d) none of the above
7) During forward blocking state, a thyristor is associated with
a) large current and low voltage
b) low current and large voltage
c) medium current and large voltage
d) none of the above
8) Chopper control for DC motor provides variation in
a) input voltage
b) frequency
c) both (a) and (b)
d) none of the above

P.T.O.

SLR-BB 318

*SLRBB318*

-2-

9) The efficiency of the chopper can be expected in the range


a) 50% to 55%
b) 65% to 72%
c) 82% to 87%
10) Power MOSFET has
a) fast switching
c) high input resistance

b) low on resistance drop


d) all of the above

11) The voltage given G(s) =


a)

c)
K

1
1

for buck boost regulator is given by

b)
1

d) 92% to 99%

d)
K

12) When anode is possible with respect to cathode, in SCR the No. of blocked PN Junctions
are
a) 1
b) 2
c) 3
d) 4
13) In a thyristor anode current is made up of
a) Electrons only
c) Electrons or holes

b) Electron and holes


d) Holes only

14) Power MOSFET is


a) Bipolar device
c) Voltage controlled device

b) Unipolar device
d) Both b) and c)

15) In a GTO, anode current begins to fall when gate current


a) Is negative peaks at time t = 0
b) Is negative peak at t = storage period ts
c) Just begins to become negative and t = 0
d) None of these
16) A PWM switching scheme is used with a three phase inverter to
a) Reduce the total distortion with modest filtering
b) Minimize the load on DC side
c) Increase the life of the batteries
d) Reduce low order harmonics and increase high order harmonics
17) The uncontrolled electronic switch employed in power-electronic converters is
a) SCR
b) BJT
c) Diode
d) MOSFET
18) Which semiconductor power device out of the following is not a current triggered
device
a) SCR
b) GTO
c) TRIAC
d) MOSFET
19) In step-down chopper output voltage is given by
a) Vo =


Vs

b)
8

c)


Vs


d) (1


)Vs

20) If the chopper frequency is 200 Hz and ton time is 2 ms, the duty cycle is
a) 0.4
b) 0.8
c) 0.6
d) None of the above
______________
Set A

*SLRBB318*
S

SLR-BB 318

-3-

T.E. (Electrical) (Part II) Examination, 2014


POWER ELECTRONICS
Day and Date : Friday, 28-11-2014

Marks : 80

Time : 10.00 a.m. to 1.00 p.m.


SECTION I
2. Solve any four :

(45=20)

a) Draw gate characteristics of SCR and explain.


b) Explain the TRIAC, V-I characteristics and its application.
c) Draw a single-phase controlled bridge rectifier and corresponding waveforms. Derive
the equation for average o/p voltage.
d) Derive the performance parameters of converter.
e) If the half-wave controlled rectifier has purely resistive load of R and the delay angle is


=


. Determine :

a) Rectification efficiency
b) Form factor
c) Ripple factor
d) PIV of SCR T1.
3. Solve any two :

(210=20)

a) Draw the construction diagram of depletion type MOSFET and explain its output and
transfer characteristics.
b) Draw the circuit for fully controlled rectifier and explain its two quadrant operation. Derive
the formula used for explanation.
c) A single phase fully controlled bridge is operated with a resistive load R = 10


, the input

voltage to the bridge is 230 V. Calculate the following :


i) Average load voltage
ii) RMS load voltage
iii) Average and RMS load current
iv) Form factor and ripple factor. The firing angle


= 60.
Set A

SLR-BB 318

-4-

*SLRBB318*

SECTION II
4. Solve any four :

(45=20)

a) Describe the working of type E chopper with relevant circuit diagram and its operation in
all four quadrants.
b) What are the types of UPS systems ? Explain the NO. Break UPS.
c) Derive the performance parameters of Inverter.
d) A single-phase half bridge inverter has a resistive load of R = 3ohms and the DC input
voltage Edc = 24V. Determine :
a) Total Harmonic Distortion THD
b) The DF
c) The harmonic factor and the distortion factor of the lowest order harmonic.
e) Input to the step up chopper is 200 V. The output required is 600 V. If the conducting time
of thyristor is 200 sec. Compute :


i) Chopping frequency
ii) If the pulse width is halved for constant frequency of operation, find the new output
voltage.
5. Solve any two :

(210=20)

a) Discuss the principle of working of 3-phase bridge inverter with appropriate wave form
and circuit diagram on assumption that each thyristor conduct for 180 degree and resistive
load is star connected.
b) What are the types of SMPS ? Explain about Fly back SMPS.
c) Explain the operation of BUCK-BOOST converter with neat diagram and wave forms.
Derive the formula for peak-to-peak ripple current and peak-to-peak capacitor voltage.
_____________________

Set A

SLR-BB 319

*SLRBB319*
Seat
No.

Set

T.E. (Electrical) (Part II) Examination, 2014


ENGINEERING ECONOMICS AND INDUSTRIAL MANAGEMENT
Day and Date : Saturday, 29-11-2014
Time : 10.00 a.m. to 1.00 p.m.

Max. Marks : 100

N. B. : 1) Figures to the right indicate full marks.


2) Q. No. 1 is compulsory. It should be solved in first 30 minutes in
Answer book Page No. 3. Each question carries one mark.
3) Answer MCQ/Objective type questions on Page No. 3 only.
Dont forget to mention, Q.P. Set (A/B/C/D) on Top of Page.
MCQ/Objective Type Questions
Duration : 30 Minutes

Marks : 20

1. Choose the correct answers from the multiple choice given :


1) Partnership has been defined by Indian Partnership Act in
a) 1932
b) 1931
c) 1942

20
d) 1941

2) Which of the following indicators are considered for measuring organisational


effectiveness ?
a) Ability to produce useful results
b) Productivity
c) Quality
d) All of the above
3) Working hours for adults in a factory is
a) 30 hours in a week
c) 48 hours in a week

b) 36 hours in a week
d) 56 hours in a week

4) Reasons for entrepreneurial failure is


a) Financial problem
c) Marketing problem

b) Production problem
d) All of the above

5) Managing change is an integral part of


a) Every managers job
b) The first line managers job
c) Middle level managers job
d) Top management job
6) Value is equal to
a) Utility/cost

b) Cost/utility

c) Utility

d) Cost

7) In which type of company secrecy can be preserve


a) Proprietorship
b) Partnership
c) Joint Stock Company
d) Both a) and b)
8) The Indian Electricity Act was first made in
a) 1902
b) 1920
c) 1903

d) 1917
P.T.O.

SLR-BB 319

*SLRBB319*

-2-

9) The oldest and simplest form of organisation is


a) Partnership
b) Joint stock company
c) Proprietorship
d) Public company
10) The minimum wage bill was introduced in the central legislative in
a) March 1946
b) April 1946
c) June 1947
d) April 1946
11) No women shall be required to work in factory except between the hours of
_________ a.m. to _________ p.m.
a) 8 to 5
b) 7 to 6
c) 6 to 7
d) 10 to 5
12) Govt. of India notified it Rural Electrification policy in
a) April 2006
b) March 2006
c) April 2007

d) March 2007

13) Managers need to make the employees abilities


a) Are being maximized
b) Match the job requirement
c) Are not affected by their stress
d) Are not causing them stress
14) Engineering economics deals with
a) Economic theory to business management
b) Economic theory to financial management
c) Economic theory to organised behaviour
d) Economic theory to statistic and management
15) Manager who acts as crises manager is playing which of the roles ?
a) Resource allocator
b) Disturbance handler
c) Entrepreneur
d) Negotiator
16) Military organisation is known as
a) Matrix organisation
c) Fundamental organisation

b) Line and staff organisation


d) Line organisation

17) The time taken by a trained worker to perform an operation, while working a steady
pace is known as
a) Standard time
b) Normal time
c) Representative time
d) None of these
18) The nature and engineering economics is
a) Micro
b) Macro
c) Normative and micro
d) Only normative
19) Function of staffing is
a) Placement
c) Integrating employees

b) Development of personnel
d) All of the above

20) The time elapsed between the placing of an order and its arrival is called as
a) Cycle time
b) Lead time
c) Work station process time
d) None of the above
______________

Set A

*SLRBB319*

-3-

SLR-BB 319

Seat
No.

T.E. (Electrical) (Part II) Examination, 2014


ENGINEERING ECONOMICS AND INDUSTRIAL MANAGEMENT
Day and Date : Saturday, 29-11-2014
Time : 10.00 a.m. to 1.00 p.m.
N. B. :

Marks : 80

1) All the questions are compulsory.


2) Figures to the right indicate full marks.
SECTION I

2. Explain the importance of value engineering and how it is implemented.

10

3. Describe proprietorship and partnership organisation with their advantages and


disadvantages.

10
OR

Brief policies of Indian Govt. for industrial and agricultural sector.


4. Write short notes on any four :

20

1) Micro and Macro economics


2) Importance of payback period
3) Joint stock companies
4) Line organisation
5) Organisational structure
6) Infrastructure of Indian Economy.

Set A

SLR-BB 319

-4-

*SLRBB319*

SECTION II
5. Explain in detail planning, directing and controlling functions.

10

6. Discuss the methods of reducing project cost for hydro power plants.

10

OR
Brief the role of small sectors in development of Indian Economy.
7. Write short notes on any four :

20

1) Importance of safety in industries


2) Entrepreneurship development
3) Advantages of small scale industry
4) Self employment for electrical engineers
5) Methods for reducing project cost
6) Health provisions in factory act.
_____________________

Set A

SLR-BB 32

*SLRBB32*
S eat
No.

Set

S.E. (Civil) (Part II) Examination, 2014

SURVEYING II (New)
Day and Date : Wednesday, 26-11-2014
Time : 3.00 p.m. to 6.00 p.m.
Instructions :

Max. Marks : 100

1) Q. No. 2 and Q. No. 6 are compulsory. Attempt any two out of Q. No. 3 to
Q. No. 5 (Section I) and out of Q. No. 7 to Q. No. 9 (Section II).
2) Assume suitable data, if necessary but mention it clearly.
3) Figures to right indicate full marks.
4) Q. No. 1 is compulsory. It should be solved in first 30 minutes in
Answer book Page No. 3. Each question carries one mark.
5) Answer MCQ/Objective type questions on Page No. 3 only. Dont
forget to mention, Q.P. Set (A/B/C/D) on Top of Page.
MCQ/Objective Type Questions

Duration : 30 Minutes

Marks : 20

1. Multiple choice questions :


1) Horizontal distances obtained by tacheometric observations
a) Require slope corrections
b) Require pull corrections
c) Require slope and pull corrections
d) Do not require slope and pull corrections
2) The purpose of an anallatic lens in a tacheometer is to
a) Increase magnification
b) Reduce effective length of the telescope
c) To eliminate multiplying constant
d) Make staff intercept proportional to its distance from the tacheometer
3) Substense bar is used for
a) Levelling
b) Measurement of horizontal distances in plane areas
c) Measurement of horizontal distance in undulated areas
d) Measurement of angles
4) In tangential tacheometry, an ordinary level staff is used which
a) Leans towards the instrument for inclined sight upwards
b) Leans away from the instrument for inclined sight downwards
c) Is vertical in all cases
d) Is equipped with two targets a fixed distance apart
5) The long chord and tangent length of a circular curve of radius R, will be equal if the angle
of deflection is
a) 30
b) 60
c) 120
d) 150
6) The radius of a simple circular curve is 30 m and the length of the specified chord is 30 m.
The degree of the curve is
a) 57.29
b) 3.70
c) 55.60
d) 37.03

P.T.O.

SLR-BB 32

-2-

*SLRBB32*

7) A parabola is best suited for a vertical curve since ____________


a) It provides a longer sight distance
b) Rate of change of grade is uniform throughout
c) Smooth riding condition are provided
d) All of the above
8) The range measurements in GPS are made with
a) 3 L-Band Frequencies
b) 2 L-Band Frequencies
c) 2 C-Band Frequencies
d) 3 C-Band Frequencies
9) Geostationary satellites have
a) Same distance from earth surface
b) Same speed as earths rotation
c) Same mass as global weight
d) Same angle with geodetic stations
10) For air-borne applications and materialization of GPS receiver and easy construction,
which is the most frequently used antenna ?
a) Microstrip
b) Micropole
c) Spiral helix
d) Choke ring
11) Which of the following coincides with the point of intersection of X, and Y, axes of an
aerial photograph ?
a) Nadir point
b) Principal point
c) ISO centre
d) Air pollution
12) If the net length covered by each photograph is 1.2 km and the length of strip is 20 km,
the number of air photographs required are
a) 16
b) 17
c) 18
d) 24
13) The object of photo interpretation is
a) Identification of objects
b) Recognition of objects
c) Judging the significance of objects
d) All the above

14) The difference of parallax for a given difference in elevation is independent of


a) Focal length of camera
b) Percentage of overlap
c) Over all size of the photograph
d) All of the above
15) Ground control points are established in aerial photogrammetry to control
a) Height distortion
b) Tilt distortion
c) Relief displacement
d) Scale
16) The graphical elements of vector data structures are
a) Point
b) Arc
c) Area
d) All of these
17) In a raster overlay, a point is represented by a
a) String of cells
b) Group of cells
c) Single cell
d) All of these
18) In designing of culverts, the centre lines of the water body and road and the points of their
intersections are considered analogous to
a) The axes of co-ordinates and the origin
b) The base lines and the origin
c) The axes and the critical point
d) None of the above
19) Setting out bridges involves
a) Determination of the length of centre line and the height of piers
b) Determination of the direction of centre line and the height of piers
c) Determination of the length of centre line and the position of piers
d) None of the above
20) Absolute accuracy in setting out of tunnels is stressed because
a) A slight error can lead to the collapse of the tunnel
b) Tunnels are strategically very important
c) Even a small error costs a lot to rectify
d) Are provided in hills

______________

1
1

1
1
1

Set A

*SLRBB32*

SLR-BB 32

-3-

Seat
No.

S.E. (Civil) (Part II) Examination, 2014


SURVEYING II (New)
Day and Date : Wednesday, 26-11-2014
Time : 3.00 p.m. to 6.00 p.m.
Instructions :

Marks : 80

1) Q. No. 2 and Q. No. 6 are compulsory. Attempt any two out of Q. No. 3 to
Q. No. 5 (Section I) and out of Q. No. 7 to Q. No. 9 (Section II).
2) Assume suitable data, if necessary but mention it clearly.
3) Figures to right indicate full marks.
SECTION I

2. a) Explain the procedure to determine the constants of tacheometer in fixed hair


method.

b) Determine the gradient from a point A, to point B. From the following observations made
with a tacheometer fitted with an anallatic lens, the constant of the instrument was 100
and the staff was held vertically.

Inst. station

Staff Point

Bearing

Vertical angle

Staff readings

134o

+10o32'

1.360, 1.915, 2.470

224o

+5o6'

1.065, 1.885, 2.705

3. a) Explain methods for determining length of transition curve.

b) Two straights AB and BC intersect at a chainage of 4242.0 m. The angle of intersection


is140; it is required to setout a 5 simple circular curve to connect the straights. Calculate
all the data necessary to setout the curve by the method of offsets from the chord produced
with an interval of 30 m.
4. a) Describe briefly the various methods of GPS surveying. Also give a brief
account of the applicability and limitations of each techniques.

8
5

b) A parabolic vertical curve of length L is formed at a summit below on uphill gradient of a%


and a downhill gradient of b%, as a part of road improvement, the uphill gradient is
reduced to c% and the downhill gradient is increased to d% but as far as possible, the
original curve is retained. Show that, the length of the new
vertical curve is, L

( c + d)
(a + b)

5. Write short note on following :


a) GPS segment with sketch.
b) Types of vertical curve with neat sketch.
c) GPS receivers and its types.

5
4

4
Set A

SLR-BB 32

-4-

*SLRBB32*

SECTION II
6. Write short note on :
a) Electromagnetic spectrum with neat sketch.

b) Parallax bar.

c) Image processing.

d) Giving centre line for columns in building line out.

7. a) Explain project survey to set out tunnel with neat sketch.

b) Define remote sensing. State how it differs from photogrammetry. Also name
the systems of remote sensing.

8. a) An area of 150 km 105 km is to be surveyed using aerial photogrammetry. From the data
given below determine number of photographs required to cover complete area, height of
flight, spacing of the flight lines, ground exposure distance and exposure interval.
Size of photograph = 23 cm 23 cm
Average scale of photograph = 1 : 25000
Average elevation of terrain = 335 m
Longitudinal overlap = 65%
Side overlap = 28%
Ground speed of air craft = 270 km/hr
Focal length of camera = 200 mm
Least count of intervalometer = 0.5 sec.
b) Explain procedure to set out culvert with neat sketch.
9. a) Explain the concept of atmospheric influence with respect to remote sensing
techniques.
b) Distinguish between Raster and Vector data modelling systems.

8
4

6
6

_____________________

Set A

SLR-BB 320

*SLRBB320*
Seat
No.

Set

T.E. (Electrical) (Part II) Examination, 2014


UTILIZATION OF ELECTRICAL ENERGY AND ENERGY CONSERVATION
Day and Date : Monday, 1-12-2014
Time : 10.00 a.m. to 1.00 p.m.

Max. Marks : 100

Instructions : 1) Figure to right indicate full marks.


2) Assume suitable data if necessary.
3) Q. No. 1 is compulsory. It should be solved in first 30 minutes in Answer
Book Page No. 3. Each question carries one mark.
4) Answer MCQ/Objective type questions on Page No. 3 only. Dont
forget to mention, Q.P. Set (A/B/C/D) on Top of Page.
MCQ/Objective Type Questions
Duration : 30 Minutes

Marks : 20

1. Multiple choice questions :

(120=20)

1) Which of the following methods of heating is not dependent on the frequency of supply ?
a) Induction heating
b) Dielectric heating
c) Electric resistance heating
d) All of the above
2) The material of the heating element for a furnace should have
a) Lower melting point
b) Higher temperature coefficient
c) High specific resistance
d) All of the above
3) For the transmission of heat from one body to another.
a) Temperature of the two bodies must be different
b) Both bodies must be solids
c) At least one of the bodies must have some source of heating
d) Both bodies must be in contact
4) Which of the following will happen if the thickness of refractory wall of furnace is increased ?
a) Heat loss through furnace wall will increase
b) Temperature inside the furnace will fall
c) Energy consumption will increase
d) Temperature on the outer surface of furnace walls will drop
5) When a body reflects entire radiation incident on it, then it is known as
a) White body
b) Grey body
c) Transparent body
d) Black body
6) Overall efficiency of steam locomotive system is close to
a) 5 to 10%
b) 25 to 30%
c) 55% to 60%

d) 75 to 80%

7) In a steam locomotive electric power is provided through


a) Battery system
b) Diesel engine generator
c) Overhead wire
d) Small turbo generator
8) Maximum horse power of steam locomotive is
a) 100
b) 500
c) 1500

d) 2500
P.T.O.

SLR-BB 320

*SLRBB320*

-2-

9) The pressure of steam in a locomotive is


a) 10 15 kg/cm2
b) 20 30 kg/cm2

c) 40 50 kg/cm2

d) 80 90 kg/cm2

10) The efficiency of diesel locomotives is nearly


a) 20 25 percent
b) 35 40 percent c) 50 55 percent

d) 70 75 percent

11) The unit of luminous flux is


a) Steradian
b) Candela

d) Lux

c) Lumen

12) The illumination is directly proportional to the cosine of the angle made by the normal to
the illuminated surface with the direction of the incident flux. Above statement is associated
with
a) Plancks law
b) Macbeths law of illumination
c) Bunsens law of illumination
d) Lamberts cosine law
13) Which curve represents life of the lamp ?

a) Curve A

b) Curve B

c) Curve C

d) Curve D

14) Illumination level required for precision work is around


a) 50 lm/m2
b) 100 lm/m2
c) 200 lm/m2

d) 500 lm/m2

15) Which of the following will need the highest level of illumination ?
a) Proof reading
b) Bed rooms
c) Hospital wards

d) Railway platforms

16) A submarine while moving under water, is provided driving power through
a) Diesel engines
b) Steam turbine
c) Gas turbine
d) Batteries
17) Overload capacity of diesel engines is usually restricted to
a) 1%
b) 10%
c) 25%

d) 50%

18) Which locomotive has the highest operational availability ?


a) Diesel
b) Electric
c) Steam
d) All have same availability
19) Which motor is used in tramways ?
a) AC single phase capacitor start motor
c) DC series motor

b) AC three phase motor


d) DC shunt motor

20) A drive suitable for mines where explosive gas exist, is


a) Diesel engine
b) Steam engine c) Battery locomotive
______________

d) Any of the above

Set A

*SLRBB320*

-3-

SLR-BB 320

Seat
No.

T.E. (Electrical) (Part II) Examination, 2014


UTILIZATION OF ELECTRICAL ENERGY AND ENERGY CONSERVATION
Day and Date : Monday, 1-12-2014
Time : 10.00 a.m. to 1.00 p.m.

Marks : 80

Instructions : 1) All questions are compulsory.


2) Figure to right indicate full marks.
3) Assume suitable data if necessary.
SECTION I
2. Solve any four :

(54=20)

1) Write a note on motor selection in rolling machines.


2) Explain high frequency eddy current heating.
3) Define and explain the following terms :
a) Candle power
b) Reduction factor
c) Utilization factor
d) Maintenance factor
e) Glare.
4) A 15 kw, 220V, single phase resistance oven employs nickel-chrome wire for its heating
elements. If the wire temperature is not to exceed 1000C and the temperature of the
charge is to be 600C. Calculate the diameter and length of the wire. Assume radiating
efficiency to be 0.6 and emissivity as 0.9, for nickel chrome resistivity is 1.016106 m.
5) Write short note on integrating sphere.
3. Solve :

(102=20)

a) Explain types of lighting schemes. What are the objectives of lighting scheme ?
Explain the factors to be considered while designing the lighting schemes.
b) With neat diagram explain spot welding and butt welding.
OR
b) Explain the classification of Electric heating. What are the advantages of electric
heating.

Set A

SLR-BB 320

*SLRBB320*

-4-

SECTION II
4. Solve any four :

(54=20)

1) Draw a typical speed-time curve for train movement, and explain


a) Acceleration

b) Free running

c) Coasting

d) Retardation

2) Write a short note on energy conservation planning.


3) An electric train is to have a braking retardation of 3.2 km/h/s. If the ratio of maximum
speed to average speed is 1.3 the time for stop is 26 seconds and acceleration is 0.8 km/h/s,
find its schedule speed for a run of 1.5 km. Assume trapezoidal speed time curve.
4) How can energy conservation be done in power plant ?
5) What is specific energy consumption of a train ? Discuss various factors affecting it.
5. Solve any two :

(102=20)

a) What is meant by energy conservation ? Explain maximum energy efficiency principle


and maximum cost effectiveness in energy use.
b) Explain Electric breaking in traction system.
c) Define adhesive weight, dead weight and tractive effort. Derive an expression for simplified
Trapezoidal Speed time curve.

_____________________

Set A

SLR-BB 321

*SLRBB321*
Seat
No.

Set

B.E. (Elect.) (Part I) Examination, 2014


INDUSTRIAL DRIVES AND CONTROL
Day and Date : Tuesday, 2-12-2014
Time : 3.00 p.m. to 6.00 p.m.

Max. Marks : 100

N.B. : 1) Q. No. 1 is compulsory. It should be solved in first 30


minutes in Answer Book Page No. 3. Each question carries
one mark.
2) Answer MCQ/Objective type questions on Page No. 3 only.
Dont forget to mention, Q.P. Set (A/B/C/D) on Top of Page.
MCQ/Objective Type Questions
Duration : 30 Minutes

Marks : 20

1. Choose the correct answer :


1) The motor commonly used in computers and digital system is
a) D.C. shunt motor
b) Induction motor
c) Stepper motor
d) Synchronous motor
2) In three phase induction motor mechanical power developed is
(1 S)
S
2
2
a) Pm = I2 R 2
b) Pm = I2 R2
S
1.S
(1 S )
c) Pm = I22 R 2 S(1 S)
d) Pm = 2
I2 R 2 S
3) For variable speed control suitable motor is
a) Sq. cage IM
b) Slip ring induction motor
c) D.C. motor
d) a) and c)
4) PWM technique in inverter is used for
a) Harmonic reduction
b) Higher output frequency
c) Improving efficiency
d) All of these
5) Variable frequency supply to IM for speed control can be made available using
a) VSI
b) CSI
c) Cyclo converter d) All of these
6) What are the advantages of A.C. drives ?
a) High cost
b) More sparking effect
c) No communication effect
d) None of these
7) Torque of separately excited d.c. motor is directly proportional to
a) I a

b) I2a

c)

Ia

d) Ia

P.T.O.

SLR-BB 321

*SLRBB321*

-2-

8) D.C. shunt motor speed variation from no load to full load is about
a) 25%
b) 20%
c) 5%
d) 15%
9) Torque in d.c. shunt motor is

I
d) K I

a
10) By field control the speed of d.c. motor can be
a) Below base speed
b) Above base speed
c) Equal to base speed
d) None of these
11) In Class A chopper reversing motor is
a) Possible
b) Drive will have both direction
c) Not possible
d) None of these

a) K Ia

c) K

b) KIa

12) In step down chopper the output voltage is


a) Vo =

Vs

b) Vo = Vs

c) Vo = 2Vs

d) 3Vs

13) In step up chopper the output voltage is


a) Vo =

Vs
1

b) Vo =

Vs
1+

c) Vo =

Vs
1 + 2

d) Vo = (1+ ) Vs

14) Full load slip of 3 phase induction motor is in the range of


a) 0.1 to 0.2
b) 0.8 to 0.1
c) 0.7 to 0.8
d) 0.03 to 0.04
15) In static resistance control of wound 3 IM effective resistance is given by
a) R

b) (1 + ) R

c) (1 ) R

d) 1

16) If S is the forward slip then backward slip in case of single phase induction motor is
a) 2S
b) (2 S)
c) (3 S)
d) (1 S)
17) If rotor resistance is increased then starting torque
a) Reduces
b) Unaffected
c) Increases
d) None of these
18) Starters are required to start d.c. motor to limit
a) Power
b) Voltage
c) Current

d) Torque

19) For small values of slip, the torque slip characteristic is


a) Hyperbola
b) Straight line c) Parabola
d) Ellipse
20) Mechanical power developed in 3 IM is given by
2

a) PM = I2 R 2
c) IR

(1 S)
S

(1 S)
S

b) PM = I2 R 2
2
d) I2 R 2

______________

S
1 S

(1+ S)
S

Set A

*SLRBB321*

-3-

SLR-BB 321

Seat
No.

B.E. (Elect.) (Part I) Examination, 2014


INDUSTRIAL DRIVES AND CONTROL
Day and Date : Tuesday, 2-12-2014
Time : 3.00 p.m. to 6.00 p.m.

Marks : 80

N.B. : 1) Marks are shown to the right of each question.


2) Missing data may be suitably assumed.
3) Solve all five questions.
SECTION I
2. Solve any four questions :

(45=20)

a) Draw constant torque and constant power characteristics of d.c. motor.


Explain.
b) Write factors on which choice of electric drive depend.
c) Write the advantages of electric drive.
d) 220 V, 500A, 600 rpm separately excited d.c. motor has armature and field
resistances are 0.02 and 10 respectively load torque is given by
TL = (2000 2N) Newton. Calculate motor terminal voltage and armature
current when the speed is 450 rpm.
e) 200 V 875V, 150 A separately excited d.c. motor has armature resistance
of 0.06. It is fed from single phase fully controlled rectifier with a source
voltage of 220 V 50Hz. Assuming continuous conduction calculate firing angle
for rated torque and 750 rpm.
3. Solve any two questions :

(210=20)

a) 230 V, 960 rpm and 200 A separately excited d.c. motor has an armature
resistance of 0.02 . The motor is fed from chopper. The source has voltage
of 230 V assuming continuous conduction calculate :
i) duty ratio for 350 rpm at rated torque.
b) Derive equivalent values of drive parameters for loads with rotational motion
and translational motor.

Set A

SLR-BB 321

-4-

*SLRBB321*

c) 220V, 70 A series motor has combined armature and field resistance is 0.12 .
This motor is controlled in regenerative braking by chopper with source voltage
of 220V. Calculate duty ratio of chopper for motor speed of 600 RPM braking
torque is twice the rated torque what will be motor speed for duty ratio of 0.5
and motor torque is twice rated torque.
SECTION II
4. Solve any four :

(45=20)

a) Draw speed load and torque load characteristic for d.c. series motor and
explain.
b) Draw the circuit, wave shapes for single phase fully controlled rectifier drive
for d.c. series motor.
c) Block diagram for closed loop d.c. drive system for above base speed and
below base speed.
d) Draw the circuit diagram and explain brush less d.c. motor drive system.
e) With neat diagram explain the operation of stepper motor.
5. Solve any two questions :

(210=20)

a) With neat block diagram explain closed loop speed control of load commutated
inverter synchronous motor drive system.
b) A 440V, 50 Hz 4 pole 1420 rpm data connected squirrel cage induction motor
has the following parameters.
Rs = 0.35 Rr = 0 .4 X S = 0 .7 Xr = 0.8 the motor is fed from a voltage
V
control upto 50 Hz
f
and at the rated voltage above 50 Hz. Calculate brake down torque for
frequency of 75 Hz both for motoring and braking operation.

source inverter the drive is operated with a constant

c) 1000 kW 3 phase 6.6 kV 50 Hz 6 pole delta connected unity power factor


synchronous motor has following parameters XS = 40 Rs = 0 rated field
current 5A machine is controlled by variable frequency control at a constant
ratio calculate torque and field current for rated armature current 500 rpm
and unit power factor.
V

_____________________

Set A

SLR-BB 322

*SLR-BB-322*
Seat
No.

Set

B.E. (Electrical Engg.) (Part I) Examination, 2014


MICROCONTROLLER AND APPLICATIONS
Day and Date : Thursday, 4-12-2014
Time : 3.00 p.m. to 6.00 p.m.

Max. Marks : 100

Instructions : 1) Q. No. 1 is compulsory. It should be solved in first 30


minutes in Answer Book Page No. 3. Each question carries
one mark.
2) Answer MCQ/Objective type questions on Page No. 3 only.
Dont forget to mention, Q.P. Set (A/B/C/D) on Top of Page.
MCQ/Objective Type Questions
Duration : 30 Minutes

Marks : 20

1. Select the appropriate option :


1) On power-up, the 8051 uses bank _____________ for registers R0 ..... R7.
a) 8
b) 16
c) 32
d) 64

20

2) Which register bank is selected with the instruction SETB PSW.3 ?


a) bank 0
b) bank 1
c) bank 2
d) bank 3
3) JNC HERE is a _______________ byte instruction.
a) 1
b) 2
c) 3
d) none
4) In the 8051 control can be transferred any where within the ___________ K
bytes of, code space if using the LCALL instruction.
a) 8
b) 16
c) 32
d) 64
5) In 8051, SP register is _____________ bit wide.
a) 8
b) 16
c) 32
6) Upon reset SP starts from _____________
a) 07
b) 08
c) 09

d) None
d) 0A

7) To assign the highest priority to INT1 the instruction used is _____________


a) MOV IP, # 0000, 0100B
b) SETB IP.2
c) Both (a) and (b) are correct
d) None
8) MCS-51 has upto _____________ clock frequency.
a) 12 MHz
b) 14 MHz
c) 24 MHz
9) LCALL instruction is _____________ byte instruction.
a) 3
b) 2
c) 4

d) 36 MHz
d) 1
P.T.O.

SLR-BB 322

*SLR-BB-322*

-2-

10) Vector location of INTO interrupt is ______________


a) 0003 H
b) 000B H
c) 0013 H

d) 001B H

11) 8051 has ____________ bidirectional input/output lines.


a) 8
b) 16
c) 32
d) 64
12) ____________ register is used for timer select.
a) TCON
b) TMOD
c) SCON

d) None

13) Flash program memory, data memory (RAM) and data memory (ROM) for
PIC 16F877 are
a) 8K 8, 368 8, 256 8
b) 8K 14, 368 16, 256 8
c) 14K 8, 14K 8, 256 8
d) 8K 14, 368 8, 256 8
14) To transfer the data serially, it must be placed in the register ____________
a) SBUF
b) SMOD
c) SCON
d) None
15) For ADC 0808, pin EOC is _____________ pin.
a) Input
b) Output
c) Sometimes input
d) None
16) ADDL WK instruction may affect
a) C, AC, Z, DC b) C, DC, Z

c) C, DC

17) DS 1307 RTC supports


a) SPI
b) I2C

c) Both a) and b) d) None

d) None

18) PIC microcontroller use ____________ architecture.


a) Hardvard
b) Neumann
c) Both a) and b) d) None
19) In PIC to make PORT B as output port, we must place ____________ in
register ____________
a) 00H, PORTB
b) FFH, PORTB c) 00H, TRISB
d) FFH, TRISB
20) The PIC 16F877 has interrupt capability up to ___________ sources.
a) 11
b) 13
c) 14
d) 16
______________

Set A

*SLRBB322*

-3-

SLR-BB 322

Seat
No.

B.E. (Electrical Engg.) (Part I) Examination, 2014


MICROCONTROLLER AND APPLICATIONS
Day and Date : Thursday, 4-12-2014
Time : 3.00 p.m. to 6.00 p.m.

Marks : 80

SECTION I
2. Solve any four :

(45=20)

a) Explain different addressing modes of 8051 with examples.


b) Explain timer mode 0, mode 1, mode 2 operations.
c) Explain interrupt structure of MCS 51.
d) A switch is connected to pin P2.0. Write a program to monitor the status of
SW and perform the following :
i) If SW = 0, the stepper motor moves clockwise direction.
ii) If SW = 1, the stepper motor moves counter clockwise direction.
e) What will be the accumulator contents after execution of the following code
and status of carry flag ?
MOV A, #99H
MOV R1, #07H
ADD A, R1
DAA
3. Solve any two :

(210=20)

a) Explain the function of all pins of 8051.


b) Interface 4 4 matrix keyboard with 8051. Write an ALP to display any key
value pressed from matrix keyboard on to Port P2.
c) Design 8051 system with 16 KB of program ROM and 16 KB of data RAM.
Set A

SLR-BB 322

-4-

*SLR-BB-322*

SECTION II
4. Solve any four :

(45=20)

a) List out the features of PIC family (16F877).


b) Explain the details of PIC 16F877 interrupts.
c) Write short note on SFRs of 8051 and explain any one in details.
d) Find the settings of TH1, in timer mode 2 to generate a baud rate of 9600 Hz,
if the serial port is in mode 1 and an 11.0592 MHz crystal is in use. Find the
settings for both values of SMOD.
e) Give the significance of every bit of IE register.
5. Attempt any two :

(210=20)

a) Draw and explain the architecture of 16F877.


b) Explain SCON register format. Write a program to transfer message Yes
serially at 9600 band rate, 8 bit data, one start bit, one stop bit framing).
c) Explain internal RAM structure of 8051 microcontroller.
_____________________

Set A

SLR-BB 323

*SLRBB323*
Seat
No.

Set

B.E. (Elect.) (Part I) Examination, 2014


ELECTRICAL MACHINE DESIGN
Day and Date : Saturday, 6-12-2014
Time : 3.00 p.m. to 6.00 p.m.
N. B. :

Max. Marks : 100

i)
ii)
iii)
iv)

Use of non-programmable calculator is allowed.


Assume suitably missing data.
Marks to each question are shown to the right of question.
Q. No. 1 is compulsory. It should be solved in first 30 minutes
in Answer Book Page No. 3. Each question carries one mark.
v) Answer MCQ/Objective type questions on Page No. 3 only.
Dont forget to mention, Q.P. Set (A/B/C/D) on Top of Page.
MCQ/Objective Type Questions

Duration : 30 Minutes

Marks : 20

1. Solve all 20 questions :


1) Air gap of three phase induction motor is kept small to
a) reduce noise
b) obtain high torque
c) reduce magnetising current
d) reduce crawling
2) L = design gives
a) lower cost
c) good power factor

(120=20)

b) high efficiency
d) over all good performance

3) Specific loading (magnetic) i3 3 induction motor is


P
P
2P
a)
b)
c)
2DL
DL
DL
4) Specific electric loading is given by in 3 IM
6I T
8I T
I T
a) ph ph
b) ph ph
c) ph ph
D
D
6DL
5) If large value of Bav. is selected then power factor
a) increases
b) unaffected
c) reduces
6) For minimum cost in I.M. L ratio is
a) 1
b) 1.5
c) 2 to 3
7) Stacking factor is of the order of
a) 0.9
b) 0.8
c) 0.7

d)

DL
P

d)

IphTph
3DL

d) none of the above


d) 1.5 to 2
d) 0.6
P.T.O.

SLR-BB 323
8) Conductors are stranded because
a) it reduces cost
c) less I2R loss

*SLRBB323*

-2-

b) easy to handle
d) both b) and c)

9) Cooling of transformer has following effect on KVA rating of transformer


a) decreases
b) increases
c) no effect
d) a) and b)
10) E t = K S where S is KVA rating and K is constant then K is given by
a)

4.44 f 103

c) K = 4.44 fr 103

b) K = 4.44 r 10 3
d) none of these

11) Permissible value of flux density in core of a distribution transformer is


a) 1.1 to 1.4 web/m2
b) 1.2 to 1.5 web/m2
c) 3.2 to 3.8 web/m2
d) 4.1 to 4.5 web/m2
12) For large power transformer current density is given by
a) 5.4 to 6.3 A/mm2
b) 2.4 to 3.4 A/mm2
c) 1.2 to 1.5 A/mm2
d) 4.1 to 4.5 A/mm2
13) Window space factor KW depends on
a) KA
b) KV
c) KW
d) KWh
14) For 3 core type transformer overall width W is
a) W = 2a + D
b) a + 2D
c) D + a
d) 2D + a
15) Rotor of single phase induction motor is of
a) Wound type
b) Slip ring type
c) Squirrel cage type
d) Salient pole type
16) For 750 watts rating of single phase induction motor power factor is
a) 0.72
b) 0.38
c) 0.5
d) 0.6
17) Which property of copper enables it to be drawn as thin wires ?
a) Toughness
b) Elasticity
c) Tenacity
d) Ductility
18) Speed of synchronous motor
a) Increases with load
b) Decreases with increase in load
c) Remains constant always
d) Varies with power factor
19) Large airgap in synchronous motor lead to
a) increase in noise
b) reduces unbalanced magnetic pull
c) poor voltage regulation
d) lower limit of stability
20) Rotor of turbo machine is usually made up of
a) cast iron
b) forged steel c) CROS
______________

d) dynamo grade steel

Set A

*SLRBB323*

-3-

SLR-BB 323

Seat
No.

B.E. (Elect.) (Part I) Examination, 2014


ELECTRICAL MACHINE DESIGN
Day and Date : Saturday, 6-12-2014
Time : 3.00 p.m. to 6.00 p.m.
N. B. :

Marks : 80

i) Use of non-programmable calculator is allowed.


ii) Assume suitably missing data.
iii) Marks to each question are shown to the right of question.

2. Solve any 4 questions :

(45=20)

a) Distinguish between core type and shell type transformer.


b) For transformer show that E t = K S .
c) Find the maximum power factor in terms of dispersion co-efficient.
d) Determine main dimension of core and yoke for 100 KVA 50 Hz single phase
core type transformer with square core distance between adjacent limbs is
1.6 times width of laminations Et =14, Bm = 1.1 web/m2 window space factor
0.32 current density 3 A/mm2 staking factor 0.9 flux density in yoke is 80% of
flux density in core.
e) Estimate the main dimensions for 3 phase 20 H.P., 400 V 6 pole 970 RPM,
L
Bav = 0.45 ac = 23000 ac/m C
= 0.85, = 0.88, P.f. = 0.89.

3. Solve any two questions :

(210=20)

a) Estimate the main dimensions for 50 Hz 3 phase 200 KVA 6600/500 volts
star/delta core type transformer use the following data core should be 4
stepped area factor is 0.62 window space factor = 0.27.
b) 15 H.D., 400 V, 1430 RPM 3 phase induction motor has following data = 80%,
P.f = 0.81, D = 30 cm, L = 12 cm. Estimate D and L for 50 HP 406 V, 4 pole
50 Hz induction motor to be designed for 84% efficiency P.f. = 85% assuming
same specific loading as the previous motor.
c) Derive the output equation for 3 phase core type transformer with usual
notations.
Set A

SLR-BB 323

-4-

*SLRBB323*

4. Solve any four :

(45=20)

a) Why single phase induction motor only with one winding is not self starting ?
b) Determine main dimensions of 12 MVA, 13.8 KV 50 Hz 1500 RPM 3 phase
star connected alternator having following data Bav = 0.6 web/m2 ac = 42000
ac peritorial speed = 80 m/sec.
c) Explain capacitor split phase single phase induction motor with circuit and
vector diagram.
d) Explain double revolving field theory of single phase induction motor.
e) Design 41 HP 230 V, 50 Hz 1430 RPM split phase motor with data = 61%
P.f. = 59%.
5. Solve any two questions :

(210=20)

a) Derive an expression for power in smooth cylindrical synchronous machine


b) Design the suitable values of diameter and length of a 75 MVA, 11 KV 50 Hz,
3000 RPM 3 ph. Y connected alternator.
Bav = 0.6 web/m2
ac = 50,000
Speed = 180 m/sec
Window factor = 0.95
Current density = 6 A/mm2.
c) Draw the equivalent circuit diagram of single phase induction motor on the
basis of double revolving field theory.
_____________________

Set A

SLR-BB 324

*SLRBB324*
Seat
No.

Set

B.E. (Electrical) (Part I) Examination, 2014


POWER PLANT ENGINEERING
Day and Date : Tuesday, 9-12-2014
Time : 3.00 p.m. to 6.00 p.m.

Max. Marks : 100

Instructions : 1)
2)
3)
4)

All questions are compulsory.


Figure to right indicate full marks.
Assume suitable data if necessary.
Q. No. 1 is compulsory. It should be solved in first 30 minutes
in Answer Book Page No. 3. Each question carries one mark.
5) Answer MCQ/Objective type questions on Page No. 3 only.
Dont forget to mention, Q.P. Set (A/B/C/D) on Top of Page.
MCQ/Objective Type Questions

Duration : 30 Minutes

Marks : 20

1. Objective questions :

(120)

1) Power output (in kW) of a hydro-power station is equal to (Q = discharge in m 3/s,


= overall efficiency of the plant, h = head in, w = density of water).
b) 0.736/75 Qwh
a) 75/0.736 Qwh/
c) 750/0.736 Qw /h
d) 0.736/750 Qh /w
2) A mass curve can be plotted from
a) Load duration curve
b) Chronological load curve
c) Energy load curve
d) Both load duration curve and chronological and curve
3) IN Francis turbine runner, the number of blades is usually of the order of
a) 16 24
b) 8 10
c) 6 8
d) 3 6
4) Water is supplied to the boiler at
a) Atmospheric pressure
b) More than steam pressure in the boiler
c) Slightly more than atmospheric pressure
d) Any pressure
5) The main function of economizer of a boiler plant is to
a) Increase steam production
b) Reduce fuel consumption
c) Increase steam pressure
d) Increase life of the boiler
6) For the same power, the size of a turbine
a) Increase with speed
c) Decrease with speed

b) Remains with speed


d) None of the above

7) Coal rank classifies coal as per its


a) Specific gravity
c) Carbon percentage

b) Degree of metamorphism
d) Ash content
P.T.O.

SLR-BB 324

*SLRBB324*

-2-

8) Flyash generally results from


a) Fluidized bed boilers
c) Diesel engines

b) Pulverised coal boilers


d) Gas turbines

9) Nuclear reactions usually employ


a) Fission
c) Both Fission and Fusion

b) Fusion
d) None of the above

10) Which of the following materials cannot be used as a moderator ?


a) Deuterium
b) Graphite
c) Heavy water
d) Beryllium
11) The function of reflector in a nuclear reactor is to
a) Bounce back most of the neutrons that escape from the fuel core
b) Reduce the speed of the neutrons
c) Stop the chain reaction
d) All of the above
12) Gas cooled reactor
a) Uses CO2 or helium as coolant and graphite as the moderator.
b) Uses coolant at pressure of 14-28 kg/cm2 and temperature of 700-800c.
c) Uses heavy water as coolant as well as moderator
d) Both a) and b)
13) Air standard efficiency of a diesel engine depends upon
a) Expansion ratio
b) Compression ratio
c) Fuel used
d) None of the above
14) Which of the following diesel engines have minimum air consumption per bhp ?
a) 2 stroke injection
b) 4 stroke injection
c) 4 stroke mechanical injection
d) All of the above almost equal air consumption
15) In a two stroke engine, the four operations (suction, compression, expansion, and exhaust)
are completed in
a) One revolution of crack shaft
b) Two revolution of crack shaft
c) Four revolution of crack shaft
d) Eight revolution of crack shaft
16) Air filter is used in
a) Stream power plant
c) Hydro power plant

b) Nuclear power plant


d) Diesel power plant

17) The pressure ratio in a gas turbine power plant is


a) 5 to 6
b) 9 to 12
c) 2 to 3

d) None of above

18) Coal and peat is not generally used in gas turbines as fuel because of
a) Coal handling and ash handling problem
b) Their higher cost
c) Their non-availability
d) All of the above
19) In an MHD generator the conductor is made of
a) Aluminium
b) Copper
c) Nichrome
20) Biogas consists of
a) Only methane
c) Only ethane

d) None of them

b) Methane and carbon dioxide with some impurities


d) A special organic gas

______________

Set A

*SLRBB324*

-3-

SLR-BB 324

Seat
No.

B.E. (Electrical) (Part I) Examination, 2014


POWER PLANT ENGINEERING
Day and Date : Tuesday, 9-12-2014
Time : 3.00 p.m. to 6.00 p.m.

Marks : 80

Instructions : 1) All questions are compulsory.


2) Figure to right indicate full marks.
3) Assume suitable data if necessary.
SECTION I
2. Answer any four questions :

(45=20)

1) Explain with a neat plots the thermodynamics cycle related to power plant ?
2) Why is the overall efficiency of a steam power station is very low ?
3) What do you understand by load curve ? What informations are conveyed by a load
curve ?
4) What are the factors affecting the power plant design ?
5) Explain with a neat sketch the operation of a fuel cell for the generation of electrical energy ?
3. Answer the questions :

(102=20)

1) Write short notes on following :


1) Simple tariff
2) Flat rate tariff
3) Black rate tariff
4) Two part tariff
5) Max-demand tariff
6) Power factor tariff.
2) With neat sketches explain briefly the Tidal power plant and Geo thermal power plant ?
OR
2) Draw a neat schematic diagram of a HEPP and explain the function of various components.

Set A

SLR-BB 324

-4-

*SLRBB324*

SECTION II
4. Answer any four questions.

(45=20)

1) Why hot gas at high pressure and not hot gas at atmospheric pressure is used in gas
turbine power plants ?
2) Briefly discuss the steam generating equipments and its types in thermal power plant ?
3) What are the safety measure are required for nuclear power plants ?
4) Discuss the harmful effect of emission and steps taken for and their impact.
5) What are the factors influencing co-generating choice ?
5. Answer any two questions :

(102=20)

1) Draw the schematic diagram of a nuclear power station and discuss its operation.
2) Explain the working of a gas turbine power plant with a neat diagram.
3) With a neat diagram explain CANDU type reactor and gas cooled reactor.
_____________________

Set A

SLR-BB 325-A

*SLRBB325A*
S

B.E. (Electrical) (Part I) Examination, 2014


EXTRA HIGH VOLTAGE AC TRANSMISSION (Elective I)
Day and Date : Thursday, 11-12-2014
Time : 3.00 p.m. to 6.00 p.m.

Max. Marks : 100

Instructions : 1) Q. No. 1 is compulsory. It should be solved in first 30 minutes in


Answer Book Page No. 3. Each question carries one mark.
2) Answer MCQ/Objective type questions on Page No. 3 only. Dont
forget to mention, Q.P. Set (A/B/C/D) on Top of Page.
Duration : 30 Minutes

MCQ/Objective Type Questions

Marks : 20

1. Choose the correct answer.

20

1) The positive sequence reactance per phase in ohms/km in 1000 kV transmission line is
a) 0.272
b) 0.231
c) 0.227
d) 0.213
2) The Maxwells potential co-efficient depends on only
a) Dimensions of line conductors configuration on the tower
b) System voltage
c) Quantum of power transmitted
d) None of these
3) The percentage power loss in 1200 kV transmission line is nearly equal to
a) 0.584
b) 0.78
c) 2.5
d) None of these
4) The average value of resistance per phase in ohm/km for 750 kV transmission line is
a) 0.031
b) 0.0136
c) 0.032
d) None of these
5) The I2R loss in EHV lines for 1000 kV system and 2000 MW power transmitted nearly
equals to
a) 7.8 MW
b) 50 MW
c) 96.5 MW
d) 15.6 MW
6) Corona loss in transmission line is dependent on
a) diameter of conductor
b) material of conductor
c) height of conductor
d) all of above
7) First mode of propagation is called as
a) Line-to-ground
b) Line-to-line

c) Homopolar

d) Both a) and c)

8) Corona occurs between two transmission conductors when they


a) have high potential difference
b) are closely spaced
c) carry dc power
d) both a) and b)
9) The current drawn by the line owing to corona loss is
a) Non-sinusoidal
b) sinusoidal
c) dc
10) The surface voltage gradient on conductors in bundle governs
a) Generation of corona on the line
b) Audiable noise
c) Radio interference
d) None of these

d) square

P.T.O.

SLR-BB 325-A

*SLRBB325A*

-2-

11) The conductor used in EHV transmission in recent development is


a) ACSR
b) ACAR
c) AAAC
d) All of the above
12) The main drawback of overhead system over underground system is
a) underground system is more flexible than overhead system
b) higher charging current
c) surge problem
d) high initial cost
13) Operating 750 KV line gives AN at a level of
a) 50 dB
b) 52 dB
c) 55.4 dB

d) 58.5 dB

14) The AN level at 30 m from the line center for 1150 KV line in USA is fixed to
a) 50 dB
b) 52 dB
c) 55 dB
d) 57 dB
15) When surge propagates on transmission line they suffer attenuation or decrease in amplitude
due to energy loss in the
a) conductor resistance
b) ground resistance
c) corona
d) all of the above
16) In general method of Laplace transform the series and shunt impedance operator per
unit length of line is
b) y(s) = g + cs
c) Both a) and b)
d) None of these
a) z(s) = r + l s
17) The peak value of current related to pulse properties during positive and negative pulse
is respectively
a) 100 mA, 10 mA
b) 100 mA, 100 mA
c) 10 mA, 10 mA
d) none of the above
18) The type of EHV cable is
a) high pressure oil filled
c) gas insulated lines

b) cross linked polythene


d) all of the above

19) The measurement of electrostatic field of an e.h.v. line is done by


a) Dipole
b) Parallel plates
c) Spherical Dipole
d) All of the above
20) The most sever over voltages online which the insulation must with stand are caused by
a) Energizing an open ended line
b) Re-energizing an open ended line
c) Both a) and b)
d) None of these
______________

Set A

*SLRBB325A*
S

-3-

SLR-BB 325-A

B.E. (Electrical) (Part I) Examination, 2014


EXTRA HIGH VOLTAGE AC TRANSMISSION (Elective I)
Day and Date : Thursday, 11-12-2014

Marks : 80

Time : 3.00 p.m. to 6.00 p.m.


Instructions :

1) Programmable calculator is not allowed.


2) Solve five full questions including MCQ.
3) Marks are indicated to the right of question.
SECTION I

2. Attempt any 4 questions :

(5 marks each)

a) Explain engineering aspect and growth of EHV AC transmission line.


b) Explain charge-potential relation for multi-conductor lines.
c) Explain limits for radio interference.
d) Explain Reflection and Refraction coefficient of transmission line.
e) Tower footing resistance.
f) Explain lighting stroke of transmission line.
3. Solve any two :

(10 marks each)

a) Derive and explain modes of propagation.


b) Derive and explain distribution of voltage gradient on sub conductor of bundle.
c) Explain resistance and inductance of ground return.
SECTION II
4. Write short notes on any four :

(5 marks each)

a) Expression for generalized constants.


b) Reduction of switch surge over voltages.
Set A

SLR-BB 325-A

-4-

*SLRBB325A*

c) What are the factors under steady state in design of EHV lines ?
d) Describe the insulation co-ordination and over voltage protection based on lighting.
e) Static reactive compensating system.
f) Sources of overvoltages.
5. Solve any two :

(10 marks each)

a) Brief the sinusoidal excitation-lumped parameter circuits.


b) Power circle diagram and its use.
c) 100 MVA 230 KV 50 Hz transformer has Xt = 12% and is connected to a line 200 Km long
which has an inductance of 1 mH/Km. The filter connected to the L.V. side 33 KV of the
transformer, is required to suppress the 5th harmonic generated by the TCR to 1% of in
calculate the value of filter capacitor if the filter inductance used in 2 mH.

_____________________

Set A

SLR-BB 325-B

*SLRBB325B*
Seat
No.

Set

B.E. (Electrical) (Part I) Examination, 2014


HIGH VOLTAGE ENGINEERING (Elective I)
Day and Date : Thursday, 11-12-2014
Time : 3.00 p.m. to 6.00 p.m.

Max. Marks : 100

Instructions : 1) Figure to right indicate full marks.


2) Assume suitable data if necessary.
3) Q. No. 1 is compulsory. It should be solved in first 30 minutes in
Answer book Page No. 3. Each question carries one mark.
4) Answer MCQ/Objective type questions on Page No. 3 only. Dont
forget to mention, Q.P. Set (A/B/C/D) on Top of Page.
MCQ/Objective Type Questions
Duration : 30 Minutes

Marks : 20

1. Solve the following :


(120=20)
1) An electronegative gas is one in which
a) positive ions are formed along with electrons
b) the gas has inherent negative charge
c) gas is ionized due to electron bombardment
d) the gases in which electron gets attached to form negative ion
2) SF6 is a
a) neutral gas
b) electronegative gas
c) ionizes easily to form ions
d) non-attaching gas
3) Ionization coefficients , are functions of
a) applied voltage
b) pressure and temperature
c) electric field
d) ratio of electric field to pressure
4) Stressed oil volume theory is applicable when
a) Small volume of liquid is involved
b) Large volume of liquid is involved
c) Large gap distance is involved
d) Pure liquids are involved
5) The parameters that affect the breakdown strength of liquids is
a) Hydrostatic pressure and temperature
b) Dissolved impurities
c) Dielectric constant
d) Pressure, temperature, dissolved impurities and suspended particles
6) Which of the following liquids has highest breakdown strength ?
a) Mineral oils
b) Silicone oil
c) Chlorinated hydrocarbon oils
d) Polyolefin or esters
7) Which of the following property is important for a liquid to be used both for electrical
insulation and cooling purposes ?
a) Thermal conductivity
b) Viscosity
c) Viscosity temperature characteristics d) Breakdown strength
8) The intrinsic breakdown strength of solid dielectrics is about
a) 50 to 100 kV/mm b) 500 to 1000 kV/mm
c) 5 to 10 kV/mm
d) 1 to 5 kV/mm
P.T.O.

SLR-BB 325-B

-2-

*SLRBB325B*

9) The usual mechanism of breakdown in solid dielectrics is


a) Intrinsic breakdown
b) Electromechanical breakdown
c) Thermal breakdown
d) Chemical breakdown
10) Long-term deterioration and breakdown occurs in solid dielectrics due to
a) thermal phenomenon
b) surface discharge
c) internal discharge
d) treeing phenomenon
11) Aging in electrical insulating materials under an electric field means
a) Gradual reduction in dielectric strength which may lead to breakdown
b) Decrease in insulation resistance of material
c) Progressive building up of disruptive discharges inside the material
d) None of the above
12) In an impulse current generator the capacitors are connected in
a) Series
b) Parallel
c) Connected in parallel while charging and in series while discharging
d) Connected in series while charging and in parallel while discharging
13) Voltage stabilizers used for regulating high dc voltages are
a) series type
b) shunt
c) series and shunt type
d) shunt or series degenerative
14) A trigetron gap is used with
a) cascade transformer units
b) impulse current generator
c) impulse voltage generator
d) dc voltage double units
15) Sphere gap measurement is linear and valid for gap spacing less than or equal to
a) Radius of sphere
b) Diameter of the sphere
c) Half the radius of sphere
d) Two times the diameter of sphere
16) The main factors that affect the sparkover voltage of sphere gaps are
a) Humidity and waveform
b) Nearby earthed objects and atmospheric conditions
c) Diameter of the sphere
d) Gap spacing, diameter and waveform
17) In wet flashover tests, the conductivity of water used is
b) 100 15 siemens at ambient temperature
a) 10 1.5 siemens
c) 45 10 siemens at room temperature d) < 1.0 siemens at 27C
18) Impulse testing of transformers indicates
a) Winding to ground insulation strength
b) Winding to winding insulation strength
c) die-electric strength, quantity of insulation and processing
d) Induced voltages in other windings during transients
19) Most important tests conducted on isolators and circuit breakers are
a) voltage withstand tests
b) short circuit tests
c) high current tests
d) temperature rise tests
20) C-tan test on electric bushing is done using
a) impulse generator
b) high voltage schering bridge
c) power frequency cascade transformer unit
d) resonant transformer
______________
Set A

*SLRBB325B*

-3-

SLR-BB 325-B

Seat
No.

B.E. (Electrical) (Part I) Examination, 2014


HIGH VOLTAGE ENGINEERING (Elective I)
Day and Date : Thursday, 11-12-2014
Time : 3.00 p.m. to 6.00 p.m.

Marks : 80

Instructions : 1) All questions are compulsory.


2) Figure to right indicate full marks.
3) Assume suitable data if necessary.
SECTION I
2. Solve any four questions :

(45=20)

a) Explain the breakdown in solid dielectrics due to chemical and electrochemical deterioration.
b) What are commercial liquid dielectrics and how are they different from pure liquid dielectric ?
c) Write a note on methods of purification of liquid dielectrics.
d) Explain post breakdown phenomenon and its application.
e) Explain the phenomenon of corona. What are the factors affecting it ? State the expression
for corona power loss.
f) Explain the term Electron attachment. Why are electrons attaching gases useful for
practical use as insulants, when compared to non-attaching gases ?
3. Attempt following :

(210=20)

a) Define Townsends first and second ionization coefficients. How is the condition for
breakdown in a Townsend discharge ? State limitations of Townsend discharge theory
and hence explain the streamer theory of breakdown.
b) State and explain the different mechanisms by which breakdown occurs in solid dielectrics.
State the applications of solid dielectrics materials.
OR
c) Explain pure liquid dielectrics and commercial liquid dielectrics. Explain various theories
of breakdown in commercial liquid dielectrics.
Set A

SLR-BB 325-B

-4-

*SLRBB325B*

SECTION II
4. Solve any four questions :

(45=20)

a) Define the front and tail times of an impulse wave. What are the tolerances allowed as per
the specifications ?
b) What is surge diverter ? Explain its function as a shunt protective device.
c) State and explain various HV tests performed on bushings.
d) Describe the generating voltmeters used for measurements. How does it compare with
potential divider ?
e) Mention the different electrical tests done on isolators and circuit breaker.
f) Give the Marxs circuit arrangement for multistage impulse generators. How is the basic
arrangement for multistage impulse generators ? How is the basic arrangement modified
to accommodate the wave time control resistances ?
5. Solve any two :

(210=20)

a) With reference to high voltage testing, explain the following terms :


i) Withstand voltage
ii) Flashover voltage
iii) 50% flashover voltage.
b) With the help of a circuit diagram explain the methods of producing impulse wave and
derive the expression
= o [exp ( t ) exp ( )]
c) Explain the power frequency voltage tests and impulse voltage tests on insulators,
bushing and circuit breakers.
_____________________

Set A

SLR-BB 326

*SLRBB326*
Seat
No.

Set

B.E. (Part II) (Electrical Engineering) Examination, 2014


FLEXIBLE AC TRANSMISSION SYSTEMS (FACTS)
Day and Date : Tuesday, 25-11-2014
Time : 3.00 p.m. to 6.00 p.m.
Instructions :

Max. Marks : 100

i)
ii)
iii)
iv)

All questions are compulsory.


Figure to right indicate full marks.
Assume suitable data if necessary.
Q. No. 1 is compulsory. It should be solved in first 30 minutes in
Answer book Page No. 3. Each question carries one mark.
v) Answer MCQ/Objective type questions on Page No. 3 only.
Dont forget to mention, Q.P. Set (A/B/C/D) on Top of Page.
MCQ/Objective Type Questions

Duration : 30 Minutes

Marks : 20

1. Solve the following :


(120=20)
1) In dynamic stability can sedaration current flow phaser is _________
a) Parallel to driving vtg
b) Series to driving vtg
c) Perpendicular to driving vtg
d) In phase to driving vtg
2) A shunt connected, thyristor switches inductor whose effective reactance is varied
in a __________
a) stepwise manner
b) continuous manner
c) linear manner
d) none of above
3) _________ in which a dc voltage always has a one polarity.
a) CSC
b) VSC
c) Both CSC and VSC
d) None of above
4) In single phase full converter, device 1 and 2 on, 3 and 4 off give __________
a) +ve vtg, ve current
b) +ve vtg, +ve current
c) ve vtg, +ve current
d) ve vtg, ve current
5) In mid-point compensation Vsm and Vmr are __________
a) Same
b) Different
c) Unity
d) None of above
6) In SVC the Ism and Isr ___________
a) Same
b) Different
c) Unity
d) None of above
7) The area A margin between __________ and _________ represent transient
stability margin of the system.
a) Delta1 and delta 2 b) Delta 2 and /2 c) Delta C to pi/2 d) None of the above
8) In case of an __________, any minor disturbance can cause the machine angle
to oscillate around its steady state at natural frequency.
a) Over damped P S
b) Under damped P S
c) Excited damped P S
d) None of above
P.T.O.

SLR-BB 326

*SLRBB326*

-2-

9) IPFC provides capability of directly Xer ___________ power between compensated


line.
a) Active and phase angle
b) Reactive and phase angle
c) Active and reactive power
d) None of the above
10) Reactive power a capacitive compensation is a _____________
a) Q = 2v2/X (1-cos delta)
b) Q = v2/X sin delta/2
c) Q = E1 E2/X sin delta
d) Q = 2v2x/XL (1-sin delta/2)
11) FACTS provides ___________
a) Power transfer capability and controllability
b) Stability and transient response
c) Phase sequence and comparatively
d) None of above
12) In SVC the thyristor _____________
a) With GTO capability
c) With T off capability

b) Without GTO capability


d) None of these

13) UPFC is an example of ___________


a) Combined series shunt controller
b) Combined series series controller
c) Series controller
d) Shunt controller
14) Sustained oscillations below the fundamental frequency is called as ___________
a) Subsynchronous oscillations
b) Damped oscillations
c) Power oscillations
d) None
15) K is ___________
a) X/Xc

b) 1/Xc

c) X

d) Xc/X

16) For better series compensating K should be using ___________


a) 1 < = k < D
b) 0 < = K < 1
c) 1 < = K < = 1 d) 1 >
17) As the degree of series compensation increases, the __________ increase.
a) Voltage stability
b) Power factor
c) Transmission line reactance
d) All of these
18) The fastest operation of compensator is ___________
a) TSSC
b) TCSC
c) GCSC

d) None

19) At high degree of compensation and degree subsynchronous resonance present,


the __________ compensation method is out used.
a) TSSC
b) GCSC
c) TCSC
d) All the above
20) TCSC is a combination of _____________
a) Capacitor and thyristor valves
b) Thyristor valve and TCR
c) Capacitor and TCR
d) Either capacitor or TCR
______________
Set A

*SLRBB326*

-3-

SLR-BB 326

Seat
No.

B.E. (Part II) (Electrical Engineering) Examination, 2014


FLEXIBLE AC TRANSMISSION SYSTEMS (FACTS)
Day and Date : Tuesday, 25-11-2014
Time : 3.00 p.m. to 6.00 p.m.
Instructions :

Marks : 80

i) All questions are compulsory.


ii) Figure to right indicate full marks.
iii) Assume suitable data if necessary.
SECTION I

2. Solve any four of the following :

(54=20)

a) What are the limits of loading stability ?


b) Describe the characteristics of TSC with block diagram.
c) Discuss the needs of FACTS devices for transmission interconnection.
d) Describe the harmonic reduction method in TCR bank.
e) Explain the reactive shunt compensation helps to damp out power oscillations in
power system.
3. a) Explain switching converter type VAR generator with basic operating principle
and control approach.

20

b) Explain power flow controlled in HVDC transmission line and power flow control
using FACTS controller, distinguish both the methods with their merits and demerits.
OR
b) Compare FC-TCR and converter based static VAR generator on points :
i) Type
ii) Maximum compensation current
iii) Losses at zero output and
iv) Harmonic.

Set A

SLR-BB 326

-4-

*SLRBB326*

SECTION II
4. Solve any four of the following :

(54=20)

a) Compare UPFC and IPFC.


b) Compare TCVC and TCPAR.
c) Explain the merits and demerits of SSSC.
d) Explain the power flow control by voltage regulator.
e) Explain the hybrid phase angle regulator with TCSC.
5. Solve any two :

(102=20)

a) Draw loss Vs VAR characteristics of :


i) STATCOM
ii) STATCOM + TSC
iii) TSC and
iv) TSC + FC.
b) Explain SSSC in full.
c) Compare in between TCSC, SSSC, STATCOM, TCVR and TCPAR.
_____________________

Set A

SLR-BB 327

*SLRBB327*
Seat
No.

Set

B.E. (Part II) (Electrical Engineering) Examination, 2014


SWITCHGEAR AND PROTECTION
Day and Date : Thursday, 27-11-2014
Time : 3.00 p.m. to 6.00 p.m.
Instructions :

Max. Marks : 100

1)
2)
3)
4)

Figures to right indicates full marks.


Assume suitable data, if necessary.
Draw vector diagram, circuit diagram wherever necessary.
Q. No. 1 is compulsory. It should be solved in first 30 minutes in
Answer book Page No. 3. Each question carries one mark.
5) Answer MCQ/Objective type questions on Page No. 3 only. Dont
forget to mention, Q.P. Set (A/B/C/D) on Top of Page.
MCQ/Objective Type Questions

Duration : 30 Minutes

Marks : 20

1. Objective Questions :

(120=20)

1) Circuit breakers usually operate under


a) Steady state short-circuit current
b) Sub-transient state of short circuit current
c) Transient state of short-circuit current
d) None of the above
2) The contact resistance of a circuit breaker is about
b) 2
c) 20m
a) 20

d) 20

3) In a circuit breaker, ionization is facilitated by


a) Increase in field strength
b) Increase in mean free path
c) High temperature of surrounding medium
d) All of above
4) Resistance switching is normally employed in
a) All breakers
b) Bulk oil breakers
c) Minimum oil breakers
d) Air-blast circuit breakers
5) Main purpose of oil in OCB is to
a) Provide insulation
c) Quenching arc

b) Provide cooling contacts


d) None of above

6) SF6 gas has excellent heat transfer properties because of


a) Low gaseous viscosity
b) High dielectric strength
c) Higher molecular weight
d) Both a) and b)
e) Both a) and c)
7) Which of the following circuit breakers has high reliability and negligible maintenance ?
a) Air-blast
b) SF6
c) Oil
d) Vacuum
P.T.O.

SLR-BB 327
8) Arc-chutes are used in
a) Oil C.B.
b) Vacuum C.B.

*SLRBB327*

-2-

c) SF-6 C.B.

d) Air-blast C.B.

9) For high voltage C.B. the rated short-circuit current is passed for
a) 15 sec.
b) 3 sec.
c) 10 sec.
d) 0.2 sec.
10) The actuating quantity for relay may be
a) Magnitude
b) Frequency

c) Phase angle

d) Any of the above

11) The operating speed of induction disc relay depends upon


a) Rate of flux built-up
b) Armature core air-gap
c) Spring tension
d) All of above
12) If fault current is 2000 Amp, the relay setting is 50% and C.T. ratio is 400/5, PSM will be
a) 25
b) 15
c) 50
d) 10
13) Earth fault relays are
a) Directional relays
c) Short operating time relay

b) Non-directional relays
d) None of above

14) The relay best suited for phase fault relaying for medium transmission line is
a) mho relay
b) reactance relay
c) Impedance relay
d) None of the above
15) For the protection of EHV/UHV transmission line we use
a) O/C relay
b) Plain impedance relay
c) mho relay
d) Unit protection
16) The protection for negative sequence current is provided for
a) Transformer
b) Transmission line
c) Motors
d) Generator
17) The relay with inverse-time characteristic will operate within
a) 10 milisec.
b) 5 to 10 milisec.
c) 10 to 20 milisec.
d) Above 25 sec.
18) Purpose of back-up protection is
a) To increase the speed
b) To increase the reach
c) To leave to blind spot
d) To guard against failure of primary protection
19) Magnetising in-rush current in a transformer in rich is
a) 3rd harmonics
b) 5th harmonics
nd
c) 2 harmonics
d) None of above
20) Problems associated with differential protection is/are
a) Magnetizing in-rush current
b) Mismatching characteristics of C.T.S.
c) Change of ratio due to tapping
d) All of above
______________

Set A

*SLRBB327*

-3-

SLR-BB 327

Seat
No.
B.E. (Part II) (Electrical Engineering) Examination, 2014
SWITCHGEAR AND PROTECTION
Day and Date : Thursday, 27-11-2014
Time : 3.00 p.m. to 6.00 p.m.
Instructions :

Marks : 80

I) All questions are compulsory.


II) Figures to right indicates full marks.
III) Assume data, if necessary.
SECTION I

2. Attempt any four questions :


(45=20)
1) Draw neat diagram and describe construction, operation, application of Air-blast
(both types) circuit breakers.
2) Draw neat diagram and describe construction, operation, application of minimum oil circuit
breakers.
3) Draw circuit diagram and explain synthetic (type) testing of circuit breaker for short-circuit
current test.
4) Write short note on Insulation co-ordination with explaining voltage Vs time characteristic.
Also explain basic impulse insulation level (B.I.L.)
5) Draw the figure and explain construction, operation of High Rupturing Capacity cartiage
fuse (HRC).
3. Answer any two questions :

(210=20)

1) Draw diagram and explain both phenomenon of :


i) Low inductive current chopping and
ii) Interruption of capacitive current.
2) What is Resistance switching ? Derive an expression for critical resistance (R) in terms
of induction and capacitance for three conditions of transient oscillations.
OR
3) Define restriking and Recovery voltage. Drive an expression for Restriking voltage and
RRRV in terms of system inductance and capacitance.
SECTION II
4. Answer any four questions :

(45=20)

1) An IDMT over current relay to protect a feeder thr. C.T. ratio 500/1A PSM = 125% TMS = 0.3.
Find the actual time of operation of relay, if fault current is 5000A. Relay characteristic is
as follows :
i) PSM
2
3
5
8
10
15
ii) Time of operation
10
6
4.5
3.2
3
2.5
Set A

SLR-BB 327

-4-

*SLRBB327*

2) Draw circuit diagram and describe directional induction disc type over-current relay. Also
explain the concept of direction showing vector diagram.
3) Draw block diagram and explain inverse-time over-current static relay.
4) Describe main basic requirement of protection system.
5) Explain three zone impedance relay characteristic alongwith three stepped distance
protection scheme using Time Vs distance diagram.
5. Answer two questions :

(210=20)

1) Describe by drawing circuit diagram of static MHO relay using Amplitude comparator
and phase comparator (both), also draw connection of MHO relay and its characteristic
diagram.
2) Derive a generalised mathematic equation of distance relay for microprocessor based
relays to realise MHO, off-set MHO and impedance relay.
3) Draw circuit diagram of percentage (biased) differential protection scheme of
transformer. Explain the construction, function of operations. Further mention the
importance of biased winding.
_____________________

Set A

SLR-BB 328

*SLR-BB-328*
Seat
No.

Set
B.E. (Electrical) (Part II) Examination, 2014
DIGITAL SIGNAL PROCESSING

Day and Date : Saturday, 29-11-2014


Time : 3.00 p.m. to 6.00 p.m.

Max. Marks : 100

N.B. : 1) All questions are compulsory.


2) Assume suitable data if necessary.
3) Q. No. 1 is compulsory. It should be solved in first 30 minutes in
Answer Book Page No. 3. Each question carries one mark.
4) Answer MCQ/Objective type questions on Page No. 3 only. Dont
forget to mention, Q.P. Set (A/B/C/D) on Top of Page.
MCQ/Objective Type Questions

Duration : 30 Minutes

Marks : 20

1. Choose the correct option :

(201=20)

1) When sequence is circularly shifted by 5 units, the magnitude response


_____________
a) Increased by 5
b) Decrease by 5
c) Remains unchanged
d) Shifts by 5 units
2) If DFT [x(n)] = X(k) then DFT[X*(n)] = _____________
a) X*(k)
b) X*(Nk)
c) (X*(C+k))N

d) None of above

3) Circular convolution of x(n) ={0, 1, 2} and h(n) = {1, 1, 0} is _____________


a) {0, 2, 3}
b) {1, 2, 3}
c) {1, 1, 1}
d) {2, 1, 3}
4) Decimation time FFT decimates
a) DFT coefficient
c) Both sequence and DFT

b) Input sequence
d) None

5) The number of multiplications in FIR can be reduced using _____________


a) Finite impulse response
b) Symmetric nature of impulse response
c) Infinite impulse response
d) None of above
6) The discrete time signal x[n] is defined as
1 , n=1

x[n] = 1 , n = 1
0 , n = 0 & |n| >1

If y(n) is defined by y[n] = x[n] + x[n] then value of y[n] is


a) 0
b) 1
c) 2

d)
7) The number of butterfly operations required in each stage of decimation is
___________
a) N

b) N 2

c) 2N

d) N 2

8) The number of memory locations required for direct form I realization.


a) M + N
b) M + N + 1
c) M + N 1
d) M N
P.T.O.

SLR-BB 328

*SLR-BB-328*

-2-

9) Linear convolution of two real sequences with P and Q points, respectively, can be
converted to circular convolution by appending extra zeros to every sequence until its
length is ____________
a) P + Q
b) P + Q + 1
c) P + Q 1
d) P Q 1
10) Fast algorithms for calculation of predictor coefficients use __________
a) Cascade form realisation
b) Parallel form realization
c) Direct form realization
d) Lattice structure realization
11) The impulse response of ideal filter is
a) Causal
c) Non-causal and finite

b) Non-causal
d) None

12) Convolution of ideal filter response and sync function results in


a) Side lobe oscillations
b) Main lobe oscillations
c) Sharpening of transition width
d) Decrease in main lobe width
13) When an impulse response of the ideal filter is truncated, in Fourier domain there is
a) Multiplication of filter with rectangular window
b) Multiplication of two transforms
c) Convolution of two transforms
d) Convolution of filter with rectangular window
14) For BLT method, we execute prewarping using the equation
WD T
WD T
1 WD T
a) WA = tan 2 b) WA = tan 4 c) WA = tan 2

15) IIR filter design is based on


a) DT filter design
c) Analog filter design

WD T
d) jWA = tan 2

b) Digital filter design


d) None

16) To avoid the overflow of adders, scaling is added


a) At the output of the adder
b) At the input of first adder
c) At the input of the second adder
d) At the output of second adder
17) The quantization noise can be reduced
a) by increasing step size
b) by reducing step size
c) by reducing number of bits used for quantization
d) none
18) At low frequencies, frequency resolution is better and at higher frequencies
a) time resolution is not good
b) time resolution is made half
c) time resolution is doubled
d) time resolution is better
19) Harvard architecture has
a) separate program memory
c) common program and data memory

b) separate data memory


d) separate and data memory

20) Finite word length effect gives


a) Improvement in performance of filter
b) Increased accuracy
c) Degradation in performance of filter
d) None of above
______________

Set A

*SLR-BB-328*

-3-

SLR-BB 328

Seat
No.
B.E. (Electrical) (Part II) Examination, 2014
DIGITAL SIGNAL PROCESSING
Day and Date : Saturday, 29-11-2014
Time : 3.00 p.m. to 6.00 p.m.

Marks : 80

N.B. : 1) All questions are compulsory.


2) Assume suitable data if necessary.
SECTION I
2. Attempt any four :

20

a) Define DFT, IDFT, twiddle factor.


b) First five points of 8 point DFT of a real valued sequence are given by
{0, 2 + j2, j4, 2 j2, 0}. Find remaining points.
c) Find IDFT using DIF FFT algorithm. X (K) = {2, 1 + j, 0, 1 j }
d) Find cross correlation of two sequences x(n) = {1,2,1,1} and y(n) = {1,1,2,1}
e) Realize following difference equation in cascade form.
y(n) = x(n) +

5
1
x (n 1) + x (n 2) .

3. a) Find circular convolution using DFT and IDFT method.

10

x1(n) = {1, 2, 2, 1} x2 (n) = {1, 2, 3, 1}.


OR
a) Realize equation y(n) = 0.1 y(n 1) + 0.72 y (n 2) + 0.7 x(n) 0.252 x (n 2)
in parallel form.

10

b) Compute 8-point DFT of sequence using DIT FFT algorithm.


x(n) = {0.5, 0.5, 0.5, 0.5, 0, 0, ,0, 0}.

10

Set A

SLR-BB 328

*SLR-BB-328*

-4-

SECTION II
4. Attempt any four :

(45=20)

a) Explain brief windowing technique and also write advantages and disadvantages of window
method.
b) Find H(z) using IIT method at 5Hz sampling frequency from H(s) as given below :
H(s) =

2
.
(s + 1) (s + 2)

c) Find H (z) using bilinear transformation if Ha(s) =

1
and Ts = 0.1 sec.
(s + 1)2

d) Explain the applications of DSP for data compression.


e) Explain the addressing modes for digital signal processors.
5. Attempt any two :

(210=20)

a) Explain fixed point and floating point digital signal processors.


b) Compare the performance or characteristics of FIR and IIR filters.
c)

i) Explain FIR implementation technique

ii) Explain the MAC unit in DSP processors.

_____________________

Set A

SLR-BB 329

*SLRBB329*
Seat
No.

Set

B.E. (Electrical Engineering) (Part II) Examination, 2014


HVDC TRANSMISSION (Elective II)
Day and Date : Monday, 1-12-2014
Time : 3.00 p.m. to 6.00 p.m.

Total Marks : 100

Instructions : 1) Figures to right indicate full marks.


2) Assume suitable data if necessary.
3) Q. No. 1 is compulsory. It should be solved in first 30 minutes in
Answer book Page No. 3. Each question carries one mark.
4) Answer MCQ/Objective type questions on Page No. 3 only.
Dont forget to mention, Q.P. Set (A/B/C/D) on Top of Page.
MCQ/Objective Type Questions
Duration : 30 Minutes

Marks : 20

1. Solve the following :

(120=20)

1) Bipolar link has two conductors, _____________


a) one positive and other negative
b) one positive and other positive
c) one negative and other negative
d) both are negative
2) The AC system is said to week if SCR is _____________
a) equal to 3
b) less than 3
c) greater than 3
3) CCC stands for _____________
a) Capacitor Compressed Converter
c) Capacitor Commutated Converter

d) none of the above

b) Capacitor Commutator Converter


d) None of the above

4) UHV DC transmission is being considered more than _____________


a) 500 KV
b) 800 KV
c) 800 MV
d) 800 V
5) Individual operation of firing pulses for each valve is the feature of _________
a) IPC
b) EPC
c) CCC
d) VGC
6) The main advantage of inverse cosine control scheme is that _____________
a) avg. dc voltage across bridge varies linearly with the control voltage
b) avg. dc voltage across bridge becomes equal to the control voltage
c) the delay angle is nominally proportional to the inverse cosine of the control voltage
d) b) and c) only
7) Pulse Phase Control (PPC) is the variations of _____________
a) IPC
b) EPC
c) CC
d) VGC
8) Harmonic instability problem particularly with low short circuit ratios, is the drawback
of _____________
a) IPC scheme
b) EPC scheme
c) Current controller
d) Current Extinguish Angle Controller
P.T.O.

SLR-BB 329

*SLRBB329*

-2-

9) The bypassing of the bridge can be done with the help of _____________
a) Replacing a single valve in the arm
b) Replacing both valve in same arm
c) Activating a bypass pair in the bridge
d) Deactivating some arm from converter set
10) Short gate pulse or long gate pulse used for _____________
a) to start AC link
b) To stop AC link
c) to start dc link
d) to stop dc link
11) Misfire occurs _____________
a) the presence of an unwanted signals
b) because of duplicated converter controls
c) the required gate pulse is missing
d) both (a) and (b)
12) Current extinction can occur when current flowing through valve is less than ________
a) latching current b) holding current c) rated current
d) None of the above
13) Tuned filters are used to filter out _____________
a) characteristic harmonics
b) non-characteristic harmonics
c) both (a) and (b)
d) transients only
14) Passive filters acts as source of _____________
a) active power
b) reactive power c) apparent power d) only (a) and (c)
15) TIF is _____________

m
2
a) ( lnFn )
b) n = 2 lnZn E1 * 100
n = 1

1
m

c) n = 1(lnZnFn ) 2 E
d) (1nZn/E1)*100

16) For damped filter, the quality factor (Q) is _____________

a)

x0
R

b)

1
GX0

c)

x0
X

d)

x0
Z

17) During switching in, the filter currents can have magnitudes ranging _________
times the harmonics current in normal operation.
a) 10 to 20
b) 20 to 40
c) 20 to 100
d) 20 to 1000
18) For slow variations in the load _____________ provide control.
a) Switched capacitors
b) Inverters
c) TCR
d) All of the above
19) In HVDC converter stations, the provision of SVC mainly helps to overcome ________
a) Voltage instability
b) Under voltages
c) Resonance frequency
d) Harmonics
20) SVC is a _____________ device.
a) Variable reactance
b) Variable impedance
c) Only reactive
d) All of the above
______________
Set A

*SLRBB329*

-3-

SLR-BB 329

Seat
No.

B.E. (Electrical Engineering) (Part II) Examination, 2014


HVDC TRANSMISSION (Elective II)
Day and Date : Monday, 1-12-2014
Time : 3.00 p.m. to 6.00 p.m.

Marks : 80

Instructions : 1) All questions are compulsory.


2) Figures to right indicate full marks.
3) Assume suitable data if necessary.

SECTION I
2. Solve any four :

(54=20)

a) Explain the special feature of Converter transformers.


b) Write short note on Surge arrestors.
c) Explain the function BI-polar DC link.
d) Explain the IPC scheme of firing angle generation.
e) Explain different modes of operation for converter.
3. a) Why HVDC controls is necessary and explain various portions of HVDC
control characteristics ?
b) Discuss the different factors that favor DC transmission.

10
10

OR
b) Discuss about modern trends in HVDC technology.

10

Set A

SLR-BB 329

-4-

*SLRBB329*

SECTION II
4. Solve any four :

(54=20)

a) Explain the following in detail :


i) Arc backs
ii) Arc through.
b) Explain non-characteristic harmonics and also explain effect of unbalanced
voltages.
c) Explain protection against over currents in converter stations.
d) State necessity of filter and also explain types of filter.
e) Explain conventional control strategies of reactive power requirements in steady
state condition.
5. Solve any two :

(102=20)

a) Give the comparison between series and parallel MTDC system in detail.
b) Explain the following :
i) Harmonic distortion
ii) THF
iii) THFF
iv) IT product.
c) Explain the static VAR system.
_____________________

Set A

SLR-BB 33

*SLRBB33*
S

S.E. (Civil) (Part II) (New) Examination, 2014


BUILDING CONSTRUCTION AND DESIGN
Day and Date : Thursday, 27-11-2014
Time : 3.00 p.m. to 7.00 p.m.

Max. Marks : 100

Instructions : 1) All questions are compulsory.


2) Figures on right indicate full marks.
3) Use full size drawing sheet for Question 2 and use of both
sides of drawing sheet permitted.
4) Assume suitable data as per requirement and mention it.
5) Q. No. 1 is compulsory. It should be solved in first 30
minutes in Answer book Page No. 3. Each question
carries one mark.
6) Answer MCQ/Objective type questions on Page No. 3
only. Dont forget to mention, Q.P. Set (A/B/C/D) on Top
of Page.
MCQ/Objective Type Questions

Duration : 30 Minutes
Marks : 20
1. Objective type questions :
1) For bed room aspect it needs at _____________
a) NE-E aspect
b) N aspect
c) S aspect
d) SW-W aspect
2) Maximum permissible built up area for plot of size between 500m2 to 1000m2
a) 60%
b) 50%
c) 30%
d) 40%
3) Village road building line is actual limited in urban area located at distance of
____________
a) 30 m
b) 12 m
c) 15 m
d) 9 m
4) Window opening in Wet hot climate should be ________________ of the floor
area.
b) 1/10th
c) 1/8th
d) 1/12th
a) 1/6th
5) For issuing ___________ certificate, side/front/rear margins/distances from
plot boundary are checked.
a) Occupancy certificate
b) Commencement certificate
c) Plinth checking certificate
d) Completion certificate
6) ______________ means planning a room or rooms in such a way which,
though originally designed for a specific purpose, may be used to serve other
overlapping purposes also, as and when desired.
a) Sanitation
b) Flexibility
c) Roominess
d) Grouping
P.T.O.

SLR-BB 33

-2-

*SLRBB33*

7) For water closet the minimum area should be


a) 2.1 Sq. m.
b) 9.5 Sq. m.
c) 1.8 Sq. m.
d) 1.1 Sq. m.
8) Minimum Height of individual rooms for kitchen specified by National Building
Code of India is _______________
a) 2200 mm
b) 2750 mm
c) 2500 mm
d) 3000 mm
9) The maximum height generally limited on the basis of the width of street, if
width of street is up to 8m is
a) 1.5 times width of street
b) 1.2 times width of street
c) 1.1 times width of street
d) 1.3 times width of street
10) ________________ means the movement space provided on the same floor
either between the rooms or within the rooms.
a) Prospect
b) Circulation
c) Grouping
d) Flexibility
11) For _________________ ventilation, the position of outlets should be just
opposite to inlets.
a) Lateral
b) Diagonal
c) Cross
d) Indirect
12) In a residential building level rooms and bed rooms require minimum
_______________ air change per hour.
a) One
b) Two
c) Three
d) Four
13) _____________ arising from temperature of vapour pressure difference or
both, between inside and outside of the room and the difference of height
between the outlet and inlet openings.
a) Wind effects
b) Stack effects c) Vapour effects d) None of these
14) The requirements for fitments for drainage and sanitation are given in
a) IS : 1172 : 1971
b) IS : 774 : 1971
c) IS : 456 : 1972
d) IS : 962 : 1989
15) System of air conditioning is ______________
a) Central system
b) Self contained system
c) Combined system
d) All of the above
16) Driers reduces the ________________ of the paint.
a) Thickness
b) Elasticity
c) Spread
d) Workability
17) The drier an oil paint should not be more than ______________
a) 5%
b) 10%
c) 15%
d) 20%
18) Minimum sizes of the drain pipes may be adopted for individual kitchen sinks
GI pipe.
a) 5cm dia
b) 6cm dia
c) 4cm dia
d) 10cm dia
19) ______________ are fittings to prevent the entry of foul sewer and drain
gases into the houses.
a) Inspection Chambers
b) Bath Tubs
c) Water Closets
d) Traps
20) Switch board and bell push should be provide at height of _____________
from floor finish level.
a) 1.35 m
b) 1.5 m
c) 0.9 m
d) 2.25 m
______________
Set A

*SLRBB33*
S

-3-

SLR-BB 33

S.E. (Civil) (Part II) (New) Examination, 2014


BUILDING CONSTRUCTION AND DESIGN
Day and Date : Thursday, 27-11-2014
Time : 3.00 p.m. to 7.00 p.m.

Marks : 80

Instructions : 1) All questions are compulsory.


2) Figures on right indicate full marks.
3) Use full size drawing sheet for Question 2 and use of both
sides of drawing sheet permitted.
4) Assume suitable data as per requirement and mention it.
SECTION I
2. Design and draw a Semi Detached house for following data :
1) A living room
2) A kitchen
3) One bed room
4) Toilet block adequate size.
Draw and show all details :
a) Typical plan.

25

b) Sectional elevation for above mentioned plan passing from toilet block.

15

SECTION II
3. Write any five of the following :
a) Write in brief about Site selection criteria.
b) Write the procedure of Building Permission.
c) Write a note on Maintenance, Repairs, Rehabilitation.
d) Explain concept of Earthing.
e) Explain in brief about systems and components of the air conditioning.
f) Draw sketches of plan and sectional elevation of typical septic tank for
detached house.
g) Write a note on various defects in painting.
h) Explain systems of Air-Conditioning in summer.
_____________________

40

Set A

Set A

SLR-BB 330

*SLRBB330*
Seat
No.

Set

S.E. (Part I) (Bio-Medical Engg.) Examination, 2014


ENGINEERING MATHEMATICS III
Day and Date : Tuesday, 9-12-2014
Time : 10.00 a.m. to 1.00 p.m.

Max. Marks : 100

Instructions : 1) Figures to the right indicate full marks.


2) Use of non-programmable calculator is allowed.
3) Q. No. 1 is compulsory. It should be solved in first 30 minutes in Answer book
Page No. 3. Each question carries one mark.
4) Answer MCQ/Objective type questions on Page No. 3 only. Dont forget
to mention, Q.P. Set (A/B/C/D) on Top of Page.
MCQ/Objective Type Questions
Duration : 30 Minutes

Marks : 20

1. Choose the correct answer :

1
1) L1
(s 3)2 + 16

a)

20

=?

e 3t
sin 4t
4

b)

e 3 t
sin 4t
4

c) e3t cos 4t

d) e3t cos 4t

1 1
2) L 3 2 = ?
s

a)

2 t

sa
1
3) L
(s a )2 b 2

a) eat cosh bt

b) 2

c)

2t

d) None of these

=?

b) eat sinhbt

c) eat cosh bt

d) eat sinbt

4) L{H(t)} = ?
a)

1
s

b)

1
s

c)

2
s2

d) None of these

1
5) L t 2 = ?

a)

b)

6) If L { f (t ) } = (s ) , then L f (u) du = ?
0

1
1
(s )
a) (s )
b)
s
s

c)

d) None of these

c) s (s)

d) s (s)
P.T.O.

SLR-BB 330

*SLRBB330*

-2-

7) L {sin2t} = ?
2
a) 2
s +4

b)

4
2

ss +4

c)

ss +4

8) If f(z) = u + iv is analytic function, then f(z ) equals to


a) ux iuy
b) ux ivx
c) vy ivx

d)

4
2

s +4

d) None of these

ebx

9) If u =
cos 5y is harmonic function, then the value of b is
a) 5
b) 2
c) 0

d) 3

10) The Cauchy-Riemann equations for f(z) = u + iv are


a) ux = vy, uy = vx
b) ux = vy, uy = vx
c) ux = vy, uy = vx

d) ux = vy, uy = vx

11) The Fourier series of f(x) = | x | in < x < contains __________


a) only sine term
b) only cosine term
c) both sine and cosine term
d) neither sine nor cosine term
12) In half range cosine series f(x) = (x 1)2 in 0 x 1, a0 = __________
a)

1
3

b)

1
3

c) 0

13) Which of the following is not Dirchlets condition ?


a) f(x) is periodic
b) f(x) is single valued

d) 1

c) f(x) is finite

d) f(x) is even function

14) If f(x) = x2 in the expansion as a cosine series in (0, ) then constant term = __________
a)

2
3

b)

2 2
3

c)

3 2
2

d)

3
3

15) The Fourier expansion in the interval (2, 2) of the function f (x ) = x, 2 x 0


= x
0 < x 2 has ________
a) No sine term

b) No cosine term

c) Both sine and cosine term

d) None of these

16) If in the mapping w = cz where c is complex constant, a triangle is mapped into ________
a) square
b) a straight line
c) a circle
d) a triangle
17) In bilinear transformation w =
___________
a) ad + bc = 0
18) The value of

az + b
where a, b, c, d are complex constant and the condition is
cz + d

b) ad bc = 0

c) ab dc = 0

z dz from z = p to q is __________

d) ab + dc = 0

1 2
q p2
d) q + p
2
19) If f(z) is analytic in a simply connected domain D and if C be any closed continuous rectifiable curve
in D then ___________

b) q2 p2

a) q p

a)

f (z) dz 0

20) The value of


a) 0

b)

f (z) dz = 0

c)

c)

f (z ) dz = 2i

dz
where c is | z | = 1 is ___________
z2
b) i
c) 2i
______________

d)

f (z) dz =

d) 4i

Set A

*SLRBB330*

SLR-BB 330

-3-

Seat
No.

S.E. (Part I) (Bio-Medical Engg.) Examination, 2014


ENGINEERING MATHEMATICS III
Day and Date : Tuesday, 9-12-2014
Time : 10.00 a.m. to 1.00 p.m.

Marks : 80

Instructions : 1) Attempt any three questions from each Section.


2) Figures to the right indicate full marks.
3) Use of non-programmable calculator is allowed.
SECTION I
2. a) Find Laplace transform of teatsinat.

4 t
b) Find L e

u sin 3u du .

0
t

c) Find Laplace transform of

(s + 3 )2

3. a) Find L1
s 2 + 6s + 13

1
1 e 2t .
t

using convolution theorem.


2

b) Solve by using Laplace transform

d2y
dt 2

+4

dy
+ 8y = 1, where y(0) = 0 and y (0 ) = 1.
dt

) (

4. a) Prove that f (z ) = x 3 3xy 2 + 2xy + i 3x 2 y x 2 + y 2 y 3 is analytic and find f(z ) , f(z) in


terms of z.
b) Show that u = cosx coshy is a harmonic function. Find its harmonic conjugate and corresponding
analytic function.
5. a) Find Laplace transform of f (t ) = K
3s 7
1
b) Find L 2
s 6s +

t
for 0 < t < T and f(t) = f(t + T).
T

.
8

c) Find p in f(z) = r2 cos2 + i r2 sinp is analytic.

6
7

4
5

OR
c) Find an analytic function f(z) = u + iv where u + v = ex(cosy + siny).

5
Set A

SLR-BB 330

*SLRBB330*

-4-

SECTION II

6. a) Evaluate

e 3z
dz
(z 1) (z 2 ) where c is the circle | z | = 3.

1+ i

b) Evaluate

2
(x iy ) dz alonge
0

i) y = x
ii) y = x2.

7. a) Find the bilinear transformation which maps z = 2, 1, 0 onto w = 1, 0, i.


b) Find the image of z 2i = 2 under the transformation w =

1
.
z

8. a) Find the Fourier series for f(x) in (0, 2)


f (x ) =

7
6
8

x
, 0x
2 x < x 2

b) Find the Fourier expansion of f (x ) = sin x in the interval x .

9. a) Evaluate

cos z
z3

dz where c is | z | = 3.

x
b) Obtain half range sine series of f (x ) =
in the interval 0 x .
2

c) Find the image | z | = 2 under the transformation w = z + 3 + 2i.

OR
Find the image | z | = K under the transformation w = z + 4 + 3i.
_____________________

Set A

SLR-BB 331

*SLR-BB-331*
Seat
No.

Set

S.E. (Biomedical Engineering) (Part I) Examination, 2014


HUMAN ANATOMY AND PHYSIOLOGY
Day and Date : Thursday, 11-12-2014
Time : 10.00 a.m. to 1.00 p.m.

Max. Marks : 100

Instructions : 1) Q. No. 1 is compulsory. It should be solved in first 30 minutes


in Answer book Page No. 3. Each question carries one mark.
2) Answer MCQ/Objective type questions on Page No. 3 only.
Dont forget to mention, Q.P. Set (A/B/C/D) on Top of Page.
MCQ/Objective Type Questions
Duration : 30 Minutes

Marks : 20

1. Fill in the blanks :


1) ____________ is the only artery that carries deoxygenated blood.
a) Pulmonary artery
b) Aorta
c) Superior venacava
d) Inferior Venacava

20

2) The total period of 1 Cardiac cycle is ____________ seconds.


a) 0.8
b) 1.5
c) 0.2
d) 0.6
3) ____________ is the pacemaker of the heart.
a) A.V.Node
b) S.A.Node
c) Bundle of HIS d) AV Bundle
4) ____________ is the power house of the cell.
a) Nucleus
b) Centrosome
c) Endoplasmi reticulum
d) Mitochondria
5) There are ____________ of ribs found in the thoracic cavity.
a) 18 pairs
b) 12 pairs
c) 16 pairs
d) 5 pairs
6) ____________ cavity is the largest in the body.
a) Cranial
b) Thoracic
c) Abdominal
d) Pelvic
7) Blood makes upto ____________ of body weight.
a) 10%
b) 7%
c) 50%

d) 25%

8) The life span of RBC is ____________ days.


a) 120
b) 90
c) 100
d) 50
9) Blood group ____________ is known as Universal donor.
a) O
b) AB
c) B
d) A

P.T.O.

SLR-BB 331

*SLR-BB-331*

-2-

10) There are 31 pairs of ____________


a) Cranial nerves b) Spinal Nerves c) Both of them

d) None of them

11) The average respiration rate is ____________ breaths per minute.


a) 12 to 15
b) 20 to 25
c) 18 to 22
d) 10 to 11
12) ____________ is both an endocrine gland as well as exocrine gland.
a) Liver
b) Stomach
c) Pancreas
d) Hypothalamus
13) The main function of kidney is formation and secretion of ____________
a) Bile
b) Pancreatic juice
c) Urine
d) Gastric Juice
14) The kidney is composed of ____________ number of functional units called
Nephons.
a) 10 millions
b) 1 million
c) 100 millions
d) 1000 millions
15) ____________is the largest organ of the body.
a) Skin
b) Liver
c) Intestine
d) Stomach
16) Spermatogenies takes place at temperature about ____________ below normal
temperature.
a) 3C
b) 10C
c) 5C
d) 12C
17) The amount of air passing into and out of lungs during each cycle of breathing
is called
a) Inspiratory reserve volume
b) Tidal volume
c) Vital capacity
d) Functional Residual capacity
18) ____________ is essential for the formation of thyroid glands hormones.
a) Vitamin A
b) Iodine
c) Calcium
d) Potassium
19) The fibrous layer of tissue that lies between duramater and piamater is
____________
a) CSF
b) Arachnoid mater
c) Meninges
d) All of the above
20) Blood is a ____________ tissue.
a) Fibrous
b) Connective

c) Epithelial

d) Cardiac

______________

Set A

*SLR-BB-331*

-3-

SLR-BB 331

Seat
No.

S.E. (Biomedical Engineering) (Part I) Examination, 2014


HUMAN ANATOMY AND PHYSIOLOGY
Day and Date : Thursday, 11-12-2014

Marks : 80

Time : 10.00 a.m. to 1.00 p.m.

SECTION I
2. Attempt any four out of six :

20

1) Describe in detail the conducting system of the heart.


2) Explain Peristalsis with the help of a neat diagram.
3) Classify different types of tissues with examples.
4) What are the functions of Pancreatic juice ?
5) Define Cardiac output and explain Cardiac Cycle.
6) Write a short note on ECG.
3. Attempt any two out of three :

20

1) Draw and label a typical Human cell and explain the functions of any four
important organelles.
2) Describe the various respiratory volumes and capacities.
3) Describe the anatomy of small intestine. What are the functions of the Liver ?

Set A

SLR-BB 331

-4-

*SLR-BB-331*

SECTION II
4. Attempt any four out of six :

20

1) Differentiate between axons and dendrites.


2) What is meant by spermatogenesis ?
3) What are the functions of CSF ?
4) Describe Rods and Cones.
5) Describe the structure of Spinal Cord.
6) What are the functions of the skin ?
5. A) Explain the formation of urine in detail.

10

B) Attempt any one :

10

1) Describe the overian cycle.


2) Draw a labelled diagram of the middle ear and inner ear. And explain the
mechanism of hearing.
_____________________

Set A

SLR-BB 332

*SLR-BB-332*
Seat
No.

Set

S.E. (Biomedical Engineering) (Part I) Examination, 2014


BIOMATERIAL
Day and Date : Saturday, 13-12-2014
Time : 10.00 a.m. to 1.00 p.m.

Max. Marks : 100

Instructions : 1) Q. No. 1 is compulsory. It should be solved in first 30 minutes in


Answer Book Page No. 3. Each question carries one mark.
2) Answer MCQ/Objective type questions on Page No. 3 only.
Dont forget to mention, Q.P. Set (A/B/C/D) on Top of Page.
MCQ/Objective Type Questions
Duration : 30 Minutes

Marks : 20

1. Choose the correct answer :

20

1) _____________ property below would you NOT associated with the metals used
to make the ball section of the hip joint.
a) Wear resistance
b) High ductility
c) High tensile strength
d) Sturdiness
2) _________ of these materials is NOT used to build the ball section of the hip joint.
a) Chromium
b) Cobalt
c) Silicon
d) Titanium
3) Examples of medical uses for bioceramics are in _____________
a) Pacemakers
b) Kidney dialysis machines
c) Respirators
d) All the above
4) The second group of vascular prostheses involves the synthetic grafts made of
_____________
a) Dacron and Teflon b) Teflon
c) Dacron
d) Cellophane
5) The most important requirement for the blood interfacing implants is
_____________
a) blood volume
b) blood cells
c) blood compatibility
d) erythrocytes
6) _____________ property of polymeric sutures helps them keep the wound closed.
a) Low coff. of friction
b) High tensile strength
c) Low melting point
d) High thermoplasticity
7) _____________ material coating helps the body to accept titanium implants.
a) Hydroginated fat b) Hydrochloric acid c) Hydroxyapatite d) Hydranged
8) The internal diameter of aorta is measured using _____________
a) Ultrasonic or radiographic techniques b) Dialysis
c) Hemodialysis
d) MRI scan
P.T.O.

SLR-BB 332

-2-

*SLR-BB-332*

9) Teflon is used as coating on mechanical heart values _____________


a) to prevent wear resistant
b) for very low friction
c) increased strength
d) enhanced rigidity
10) _____________ is a primary advantage of using biological heart valve over
mechanical one.
a) Ability to self repair b) Cost
c) Reliability
d) Durability
11) Interstitial defects _____________
a) come from impurity atoms on lattice sites
b) are usually associated with vacant lattice sites
c) are found between lattice sites
d) none of these
12) An alloy is a _____________
a) pure metal
b) mixture of a metal and another element (metal or non-metal)
c) mixture of a metal and another metal only
d) mixture of a metal and non-metal only
13) _____________ is the purpose of the stem cell attracting coating inside an
artificial artery.
a) To reject foreign materials
b) To help form a natural smooth coating
c) To provide tensile strength
d) To prevent infection
14) _____________ is used as tear substitutes.
a) Dimethyl silicone
b) Dimethyl siloxane
c) Methyl cellulose
d) Poly vinyl alcohol
15) _____________ metal offers very good bio-compatibility properties.
a) Stainless steel
b) Cobalt chromium c) Gold
d) Titanium
16) _____________ is the Atomic Packing Fraction (APF) for a Hexagonal ClosePacked (HCP) structure.
a) 0.63
b) 0.74
c) 0.96
d) 1.00
17) The directional atomic bonds include _____________
a) ionic bonds
b) covalent bonds c) metallic bonds

d) All of them

18) _____________ is not a property of bio ceramics


a) Long lasting
b) Structural failure resistant
c) Corrosion resistant
d) Have a high Youngs modulus
19) _____________ ceramics mainly used in patients suffering from anaemia and
similar disease.
a) FECAP
b) ZCAP
c) ZSCAP
d) ALCAP
20) Full form of PMMA is _____________
a) Polymethane Methacrylate
b) Polymethyle Methacrylate
c) Polymethyle Methane Acrylate
d) Polymethane Methane Acrylate
______________
Set A

*SLR-BB-332*

-3-

SLR-BB 332

Seat
No.

S.E. (Biomedical Engineering) (Part I) Examination, 2014


BIOMATERIAL
Day and Date : Saturday, 13-12-2014

Marks : 80

Time : 10.00 a.m. to 1.00 p.m.

SECTION I
2. Solve any four :

20

1) Define biomaterials. Classify it broadly.


2) What are the applications of bioglass ?
3) Discuss the applications of PMMA.
4) Explain the classification of bio ceramics.
5) Explain the classification of polymers.
6) Explain absorbable and porous biomaterial with example.
3. Solve any two :

20

1) Explain the properties and applications of stainless steel.


2) Explain the properties and applications of titanium and its alloy.
3) Explain which material is suitable as bone cement, give its properties.
SECTION II
4. Solve any four questions.

20

1) What are the various types of leathers used in prosthesis and orthotic devices ?
2) What are the materials used for soft tissue implants ?

Set A

SLR-BB 332

-4-

*SLR-BB-332*

3) Define biocompatiability. Explain its importance.


4) What are thermoplastic and thermosetting resins ?
5) Explain types and applications of rubber in prosthesis.
6) Explain the role of wood and binding material in prosthesis with one example.
5. a) Explain any 2 techniques in details for measurement of surface properties of

biomaterials.

10

b) Solve any one :

10

1) Explain the various methods used for biological testing of biomaterials.


2) Explain different types of corrosion which occur in metallic implants.
_____________________

Set A

SLR-BB 333

*SLRBB333*
Seat
No.

Set

S.E. (Biomedical Engineering) (Part I) Examination, 2014


ELECTRONIC CIRCUITS ANALYSIS AND DESIGN I (New)
Day and Date : Tuesday, 16-12-2014
Time : 10.00 a.m. to 1.00 p.m.

Max. Marks : 100

Instructions : 1) Q. No. 1 is compulsory. It should be solved in first 30 minutes


in Answer Book Page No. 3. Each question carries one mark.
2) Answer MCQ/Objective type questions on Page No. 3 only.
Dont forget to mention, Q.P. Set (A/B/C/D) on Top of Page.
MCQ/Objective Type Questions
Duration : 30 Minutes

Marks : 20

1. Choose the correct answer :


(201=20)
1) The B.E. junction as well as the collector junction must be forward biased to
operate the transistor in __________ region.
a) active
b) cut-off
c) saturation
d) forward
2) ______________ is a process of applying a external voltage to the transistor.
a) Biasing
b) Stabilizing
c) Operating
d) Power suppling
3) FET acts as a ____________ in ohmic region.
a) voltage variable resistor
b) voltage controlled device
c) constant current source
d) None of the above
4) In low frequency region of an amplifier, the gain and output voltage remains
____________
a) high
b) constant
c) low
d) very high
5) In both types of JFET, the gate current is equal to ___________
a) infinity
b) less than 1
c) zero
d) one
6) ______________ is an another type of FET.
a) Cmos
b) MOSFET
c) BJT
d) None of the above
7) In order to use transistor as an amplifier, it must be operated in its ________
mode.
a) saturation
b) forward active
c) cut-off
d) none of the above
8) _________ is a graph of amplifier output versus the frequency of input signal.
a) Frequency response
b) Bode plot
c) Time response
d) Stability response
P.T.O.

SLR-BB 333

-2-

*SLRBB333*

9) ____________ are used to remove a part of inout waveform without distorting


the remaining waveform.
a) Clampers
b) Diodes
c) Transistors
d) Clippers
10) The application of transistor in ce configuration is ____________
a) Pre amplifier
b) Amplifier
c) Audio amplifier d) Gain amplifier
11) ____________ is the most widely used configuration in biasing circuits.
a) Voltage divider ckt
b) Fixed biased ckt
c) Collector to base bias ckt
d) CE configuration ckt
12) The Q pt is also called _______________ of a transistor.
a) Dc load line
b) Cut off point
c) Operating point
d) Saturation point
13) The gate source junction is always _____________
a) reverse biased
b) Forward biased
c) not biased
d) partially biased
14) MOSFETs have a _____________ of resistivity.
a) negative temp. coefficient
b) no temp. coefficient
c) less temp. coefficient
d) positive temp. coefficient
15) BJT is _____________ device.
a) unipolar
b) transistor
c) bipolar
d) amplifier
16) In order to use JFET as an amplifier, it must be operated in its _____________
a) Saturation
b) Ohmic
c) Cut-off
d) Active
17) The cascading of CE stage with the CB stage is known as ____________
a) cascade configuration
b) cascode configuration
c) darlington configuration
d) amplifier configuration
18) The transistor acts as a ________ in saturation region.
a) open switch
b) switch
c) close switch
d) amplifier
19) JFET cannot be used as a ____________
a) current amplifier
b) switch
c) amplifier
d) voltage amplifier
20) In active region, the transistor is said to operate as a ________
a) constant voltage source
b) switch
c) constant current source
d) amplifier
______________

Set A

*SLRBB333*

-3-

SLR-BB 333

Seat
No.

S.E. (Biomedical Engineering) (Part I) Examination, 2014


ELECTRONIC CIRCUITS ANALYSIS AND DESIGN I (New)
Day and Date : Tuesday, 16-12-2014
Time : 10.00 a.m. to 1.00 p.m.

Marks : 80

SECTION 1
2. Solve any four :
20
a) For the given collector to base bias circuit, find the values of Rb and Rc. (any 4).
+ VCC = 12V

(IC + IB )

RC
RB C
IB

b)
c)
d)
e)
f)

dc = 120
VCEQ

VCEO = 5V
ICQ = 5mA

Explain with help of suitable diagram the Enhancement MOSFET.


Compare CE, CB and CC amplifiers.
Short note on Zero temperature drift in FET.
Short note on Thermal runway.
For the circuit shown, find the Q point.

VDD = 20 V
R D = 3 . 3k
R S = 1k

Set A

SLR-BB 333

*SLRBB333*

-4-

3. Solve any 2 :

20

a) Draw various biasing schemes used for BJTs. Compare them in terms of
stability factor.
b) Obtain the values of Vdsq, Vgsq and Idq for the voltage divider circuit shown.
Given : Idss : 6 mA, Vp = 4V
+ 12V
100k
VD
47 k

RD = 1.5 K

1.5 k
ID

IDSS = 6 mA

IDss = 6mA
VP = 4V
VS
1K

VDD = +12 V

VP = 4V
R1 = 100 k
R 2 = 47 k
R s = 1 k

c) Design a single stage RC coupled CE amplifier using the transistor. The


following specifications of amplifier are :
hie = 2.7 k hfe = 220, hoe = hre = 0
Av 180, Vo = 3 V, Vcc = 18 V, fl 20Hz, S 10 .
SECTION 2
4. Solve any four :

20

a) Write a short notes on Cascode amplifier.


b) Determine the values of Icq and Vceq for the circuit shown :

Set A

*SLRBB333*

SLR-BB 333

-5-

c) Write a short note on clamper and its applications.


d) Write a short note on common drain amplifier circuit and its applications.
e) Explain the working of two way clipper with neat circuit diagram.
f) For the circuit shown, calculate the values of Idsq, Vdsq.
Given : Id(on) = 6mA, Vgs(on) = 8 V and Vgs(th) = 3V.

VDD = 12 V

2k
Vo

5. Solve any 2 :

20

a) Derive the expression for the stability factor S of the voltage divider circuit and
also comment on the result.
b) For CE amplifier with bypassed Re, obtain the expressions for Av, Ai, Ri and Ro.
c) For the circuit shown, calculate the values of Zi, Zo, Av.
Given gm = 2mS and rd = 20 k .

+ VDD = 12 V
V1

Z1

0.47F

1.5M

0.47F
V0
1K

____________________

Z0

Set A

Set A

SLR-BB 334

*SLRBB334*
Seat
No.

Set
S.E. (Biomedical Engineering) (Part I) Examination, 2014
LINEAR CIRCUITS ANALYSIS

Day and Date : Thursday, 18-12-2014


Time : 10.00 a.m. to 1.00 p.m.
Instructions :

Max. Marks : 100

1) Q. No. 1 is compulsory. It should be solved in first 30 minutes in


Answer Book Page No. 3. Each question carries one mark.
2) Answer MCQ/Objective type questions on Page No. 3 only.
Dont forget to mention, Q.P. Set (A/B/C/D) on Top of Page.
MCQ/Objective Type Questions

Duration : 30 Minutes
I.

Marks : 20

Choose the correct answer :

20

1) Identify passive element from the following


a) voltage source
b) current source c) inductor

d) transistor

2) The power dissipation to each of the three parallel branches is 1 watt, what is the total
power dissipation of the circuit
a) 1 W
b) 4 W
c) 3 W
d) 0 W
3) The inductor doesnt allow sudden changes
a) in currents
b) in voltages
c) both a) and b)

d) none

4) The tie -set schedule gives the relationship between


a) Branch currents and link currents
b) Branch voltages and link currents
c) Branch currents and link voltages
d) None of the above
5) Mesh analysis is based on
a) KCL
b) KVL

c) Both

d) None

6) The maximum power is transferred when the load impedance is


a) equal to source resistance
b) equal to half of the source resistance
c) equal to zero
d) none
7) The Nortons current in the circuit shown below is

a) infinite

b) zero

c) 25i

d) none

8) In order to tune a parallel resonant circuit to a lower frequency, the capacitance must
a) be increased
b) be decreased
c) be zero
d) remain the same
P.T.O.

SLR-BB 334

*SLRBB334*

-2-

9) What is the total reactance of a series RLC circuit at resonance ?


a) Equal to XL
b) Equal to XC
c) Equal to R
d) Equal to zero
10) In a certain series resonant circuit, VC = 150 V VL = 150 V VR = 50. What is the value of
source voltage ?
a) zero
b) 50 V
c) 150 V
d) 200 V
11) For a two port network to be reciprocal
a) z11 = z22
b) y21 = y22

c) h21 = h21
d) AD BC = 0
12) A band pass filter may be obtained using a high pass filter followed by a low pass filter.
a) True
b) False
c) Invalid
d) None
13) The characteristic impedance Zo of T type filter network is _____________
a) Zo = ZOC ZSC

b) Zo = ZOC ZSC

c) Z o = ZSC Z OC

d) Z o = Z OC Z SC

14) The R1 of T-pad attenuator which gives an attenuation of 60 dB (N = 1000) and to work in
line with 500 impedance
a) 50
b) 500
c) 1
d) 499

15) For a symmetric lattice network the value of the series impedance is 3 and that of the
diagonal impedance is 5 , then the z parameters of the network are given by
b) Z11 = Z 22 = 4

a) Z11 = Z 22 = 2

Z12 = Z 21 = 1

Z12 = Z 21 = 1/ 2
c) Z11 = Z 22 = 8

d) Z11 = Z 22 = 2

Z12 = Z 21 = 2

Z12 = Z 21 = 8

16) The driving point impedance of a two port network is given by


a) Z(s) = V(s)/I(s)
b) Z(s) = I(s)/V(s)
c) Z(t) = V(t)/I(t)
d) Z(t) = I(t)/V(t)
17) If the poles and zeros are repeated then the function is said to be having ________
a) Simple poles
b) multiple poles
c) multiple zeros
d) both b) and c)
18) The necessary condition for the transfer function is that
a) The coefficients in the polynomial P(s) and Q(s) must be real
b) The coefficient in Q(s) must be negative
c) Complex or imaginary poles and zeros may not conjugate
d) If the real part of the pole is zero, then that pole must be multiple
19) The L-type attenuator to operate into load resistance of 500 , attenuation of 20 dB then
R2 is
a) 22.22
b) 33.33
c) 55.55
d) 66.66
20) In K-type low pass filter the value of K (in both T or ) is given by
a)

LC

b)

L
C

c)

C
L

d)

1
LC

______________
Set A

*SLRBB334*

-3-

SLR-BB 334

Seat
No.
S.E. (Biomedical Engineering) (Part I) Examination, 2014
LINEAR CIRCUITS ANALYSIS
Day and Date : Thursday, 18-12-2014
Time : 10.00 a.m. to 1.00 p.m.

Marks : 80

Instructions : 1) Figures to the right indicate full marks.


2) Assume suitable data if necessary.
SECTION I
II. Solve any four :

(54=20)

1) Determine the voltage drop across the 10 resistance in the circuit as shown in figure
below

2) Determine the mesh current I1 in the circuit shown in figure below :

3) Determine Nortons equivalent circuit for the circuit shown in figure below :

4) Derive and explain the AC response of RL series circuit with necessary phaser diagrams
showing voltage current and power waveforms.
Set A

SLR-BB 334

-4-

*SLRBB334*

5) For the circuit shown in figure. Determine the value of Q at resonance and bandwidth of
the circuit.

III. Answer any two :

(102=20)

1) Find the voltage across the 2 resistors in figure by using the super position theorem.

2) Derive the expression for bandwidth, Q, power (at f1 and f2) of a series RLC resonance
circuit.
3) a) The series RC circuit consists of resistor of 10 and a capacitor of 0.1 F as shown in
the figure. A constant voltage of 20 V is applied to the circuit at t = 0. Obtain the current
equation; determine the voltage across the resistance and capacitance.

b) Draw the graph corresponding to the given incidence matrix :

Set A

*SLRBB334*

-5-

SLR-BB 334

SECTION II
IV. Answer any four :

(54=20)

1) Find Z-parameter for the following circuit (derivation).


2) Classify filter with proper waveforms and define the terms napher, decibel, propagation
constant ( ) and Zo.
3) Classify different types of attenuator and explain type attenuator.
4) Design a symmetrical bridge T-type attenuator with attenuation of 20 dB and terminated
into load of 500 .
5) List the properties of transfer functions of a system.
V. Answer any two :

(102=20)

1) Find Y parameter for the network shown.

2) a) Design K-type Low pass filter having cutoff frequency of 2kHz into operate with
terminal load resistance of 500 .
b) Explain the band elimination filter and give Zo and expression with necessary
diagrams.
3) For the given denominator polynomial of a network function determine the value of k for
which the network to be stable
Q(s) = S3 + 2S2 + 4S + k.
_____________________

Set A

Set A

SLR-BB 335

*SLR-BB-335*
Seat
No.

Set

S.E. (Biomedical Engg.) (Part II) Examination, 2014


TRANSDUCER IN BIOMEDICAL INSTRUMENTATION (Old)
Day and Date : Tuesday, 25-11-2014
Time :3.00 p.m.to 6.00 p.m.

Max. Marks : 100

Instructions : 1) Q. No. 1 is compulsory. It should be solved in first 30 minutes


in Answer book Page No. 3. Each question carries one mark.
2) Figures to the right indicate full marks.
3) Assume suitable data wherever required.
4) Answer MCQ/Objective type questions on Page No. 3 only.
Dont forget to mention, Q.P. Set (A/B/C/D) on Top of Page.
MCQ/Objective Type Questions
Duration : 30 Minutes

Marks : 20

1. Answer the following questions :


(201=20)
1) Total Hemoglobin count is inversely proportional to percentage of ____________
b) O2
c) PO2
d) HbO2
a) SO2
2) ____________ sensors are sensitive to ambient temperature.
a) Pressure
b) Light
c) Optical
d) radiation
3) ____________ utilizes micro miniature sensors.
a) Transducer
b) LVDT
c) ISFETs
d) Piezoelectric
4) The silver chloride electrode is ____________ electrode.
a) polarized
b) nonpolarized c) charged
d) discharged
5) In ____________ fluids the concentration of sodium ions is relatively high.
a) CSF
b) Biological
c) Hormone
d) Saliva
6) Electric ____________ of epidermal layer behaves as parallel RL circuit.
a) Impedance
b) Conductance c) Current
d) None of above
7) ____________ tip of the electrode of metal microelectrode remains
uninsulated.
a) Covered
b) Extreme
c) Open
d) None of above
8) ____________ is the smallest incremental quantity that can be measured
with certainty.
a) Resolution
b) Linearity
c) Error
d) Micro

P.T.O.

SLR-BB 335

-2-

*SLR-BB-335*

9) The relationship between input and output is called as ____________ function.


a) Transfer
b) Nonlinear
c) linear
d) complex
10) A linear potentiometer is a example of a ____________ instrument.
a) First order
b) Second order c) Third order
d) Zero order
11) __________ low pass filter circuit is an example of first order instrument.
a) RLC
b) RL
c) LC
d) RC
12) ____________ measurement with thermocouple is based on Seebeck effect.
a) Current
b) Force
c) Pressure
d) Temperature
13) Thermistors are conductors made of ____________ material.
a) copper
b) aluminium
c) ceramic
d) none of above
14) The ability of an instrument to give the ____________ output for equal inputs
applied over some period is called linearity.
a) different
b) same
c) zero
d) compactiable
15) A mechanical pressure measuring instrument illustrates the properties of
____________ order system.
a) first order
b) second order c) third order
d) zero order
16) The ____________ are used to measure displacement.
a) thermostat
b) piezoelectric c) strain gauge

d) none of above

17) The ____________ circuit is ideal for measuring small changes in resistance.
a) Amplifier
b) Wheatstone-bridge
c) Oscillator
d) Buffer
18) When ____________ electrode is in contact with an electrolyte a double
layer of charge forms at the interface ?
a) polarizable
b) nonpolarizable
c) disk
d) gel
19) Percutaneous electrodes are used to detect ____________ with the body.
a) fluid
b) biopotential
c) potential
d) current
20) ____________ micropipette microelectrodes are fabricated from glass
capillaries.
a) Glass
b) Metal
c) Ceramic
d) Copper
______________

Set A

*SLR-BB-335*

-3-

SLR-BB 335

Seat
No.

S.E. (Biomedical Engg.) (Part II) Examination, 2014


TRANSDUCER IN BIOMEDICAL INSTRUMENTATION (Old)
Day and Date : Tuesday, 25-11-2014

Marks : 80

Time :3.00 p.m.to 6.00 p.m.


Instructions : 1) Figures to the right indicate full marks.
2) Assume suitable data wherever required.
SECTION I
2. Attempt any four :

(45=20)

a) Differentiate between transducers and sensors. Give example of each type.


b) Draw and explain bonded and unbonded strain gages.
c) Describe first and second order characteristics of transducer.
d) What is a linear and nonlinear characteristic ? Explain thermistor linearization.
e) Define and explain gauge factor. Derive an expression for the same.
f) Distinguish linear and nonlinear characteristic of transducers. Define any two
of each characteristic.
3. Attempt any two :

(210=20)

a) Explain construction and working principle of LVDT. Mention its any two
biomedical applications.
b) Define thermocouples. State the laws governing the working of the same.
c) List and explain various factors that should be taken into consideration for
selecting a transducer for biomedical applications.

Set A

SLR-BB 335

-4-

*SLR-BB-335*

SECTION II
4. Attempt any four :

(45=20)

a) Describe the equivalent circuit model for electrode skin interface.


b) What is the nature of Enzyme electrode ? State its applications.
c) Which type of electrode is used to measure cell action potential ? Describe
its construction.
d) Which radiation sensors are used to detect temperature ?
e) Explain the construction and working principle of pCO2 electrode.
f) What is a radiation thermopile ? State its application in Biomedical field.
5. Attempt any two :

(210=20)

a) Explain different types of biopotential electrodes with suitable sketch.


b) Explain working principle of pO2 Electrode and Fiber optic temperature
sensor in detail.
c) Explain the significance of pH of a solution ? Explain with the constructional
details working of combinational glass electrode with its reactions.
_____________________

Set A

SLR-BB 337

*SLRBB337*
Seat
No.

Set

S.E. (Part II) (Biomedical) (Old) Examination, 2014


ELECTRONIC INSTRUMENTATION
Day and Date : Thursday, 27-11-2014
Time : 3.00 p.m. to 6.00 p.m.

Max. Marks : 100

Instructions : 1) Q. No. 1 is compulsory. It should be solved in first 30


minutes in Answer book Page No. 3. Each question carries
one mark.
2) Answer MCQ/Objective type questions on Page No. 3 only.
Dont forget to mention, Q.P. Set (A/B/C/D) on Top of Page.
MCQ/Objective Type Questions
Duration : 30 Minutes

Marks : 20

1. 1) DVMS have input resistance typically _________ and input capacitance


_________
b) 10M and 40pF
a) 20M and 20pF
d) None
c) 12M and 30pF
2) Sensitivity of the digital voltmeter is _________
a) S=fs*R
b) S=fs/R
c) S=fs+R
d) None
3) The parallel resistor is used with the meter movement in ammeter is known
as _________
a) Multiplier
b) Shunt
c) Divider
d) None
4) If very little current flows through the shunt in aryton shunt voltmeter then
sensitivity _________
a) Increases
b) Decreases
c) Remains constant
d) None
5) In average responding voltmeters the diodes acts as switches to maintain
_________ current.
a) Unidirectional b) Bidirectional c) a) and b)
d) None
6) Absolute accuracy of DVMS as high as _________ of the reading.
a) 0.009%
b) 0.005%
c) 1.00%
d) 0.008%
7) In an over damped damping ratio ( ) systems is
a) > 1
b) < 1
c) = 1
d) None
8) An example for second order system is _________
a) Linear potentiometer
b) Series R-C circuit
c) Mass damper system
d) None
P.T.O.

SLR-BB 337

*SLRBB337*

-2-

9) The resolution of a 31/2 digit display on 10V range is _________


a) 0.001

b) 0.01

c) 0.1

d) 0.2

10) The power requirements of D Arsonval movement are from _________


a) 50-100 W

b) 25-200 W

c) 100-200 W

d) 50-10 W

11) In dual beam CRO, there is a common _________ is used.


a) Time base

b) Amplifier

c) Integrator

d) None

12) _________ is considered the best for photographing from CRT screen.
a) P5

b) P11

c) P2

d) P1

13) DAS should not have a down time greater than _________
a) 0.1%

b) 0.2%

c) 0.3%

d) 0.8%

14) The oscilloscope is basically an electron beam _________


a) Ammeter

b) Voltmeter

c) Ohm meter

d) None

15) The turn ON and OFF times of LCD are of the order of _________
a) 1 S

b) 1 mS

c) 10 mS

d) 10 nS

16) In function generator frequency is controlled by varying the magnitude of


_________ which drives the integrator.
a) Current

b) Voltage

c) Resistance

d) None

17) A square wave generator has a _________ duty cycle.


a) 40%

b) 50%

c) 60%

d) 80%

18) A magnetic tape recorder consists of complements _________


a) Recording head
c) Conditioning devices

b) Magnetic head
d) All

19) An example for active transducer is _________


a) Piezo electric crystal
b) Strain guage
c) Thermistor

d) None

20) The use of _________ provides a isolation between master oscillator and
power amplifier in signal generator.
a) Power amplifier

b) Buffer amplifier

c) Filter

d) None
______________
Set A

*SLRBB337*

-3-

SLR-BB 337

Seat
No.

S.E. (Part II) (Biomedical) (Old) Examination, 2014


ELECTRONIC INSTRUMENTATION
Day and Date : Thursday, 27-11-2014
Time : 3.00 p.m. to 6.00 p.m.

Marks : 80

SECTION I
2. Answer the following (any four) :

(45=20)

1) Draw and explain block diagram of generalized measurement system.


2) Define dead time element. Why dead time elements are used in the systems.
3) Draw and explain Ramp type DVM.
4) Explain Digital Phase Meter.
5) What is DArsonval movement and explain how this principle is used in the
implementation of an ammeter ?
3. Answer the following (any two) :

(210=20)

1) Explain Average and Peak Responding Voltmeters.


2) Explain with neat block diagram digital Frequency meter.
3) Discuss the working principle with block diagram of Successive Approximation
voltmeter and mention the importance of SAR.
SECTION II
4. Answer the following (any four) :

(45=20)

1) What is CRO ? Explain the basic principle of an oscilloscope.


2) Explain generalized DAS.
3) Explain magnetic recorder.
Set A

SLR-BB 337

-4-

*SLRBB337*

4) What is CRT ? Explain its features.


5) What are lissajous patterns ? How lissajous patterns are used to measure
frequency ?
5. Answer the following (any two) :

(210=20)

1) Explain AF Sine and square wave generator.


2) Explain multi channel DAS.
3) Explain :
a) LCD display system
b) Touch screen display system.
_____________________

Set A

SLR-BB 338

*SLRBB338*
Seat
No.

Set

S.E. (Biomedical Engineering) (Part II) Examination, 2014


DIGITAL DESIGN (Old)
Day and Date : Friday, 28-11-2014
Time : 3.00 p.m. to 6.00 p.m.

Max. Marks : 100

Instructions : 1) Q. No.1 is compulsory. It should be solved in first


30 minutes in Answer Book Page No. 3. Each question
carries one mark.
2) Answer MCQ/Objective type questions on Page No. 3
only. Dont forget to mention, Q.P. Set (A/B/C/D) on Top
of Page.
MCQ/Objective Type Questions
Duration : 30 Minutes

Marks : 20

1. Choose the correct answer :

(201=20)

1) The NAND gate output will be low if the two inputs are ____________
a) 00
b) 01
c) 10
d) 11
2) How many flip flops are required for mod 10 counter ?
a) 5
b) 6
c) 3

d) 4

3) The Gray code foe decimal number 6 is equivalent to ____________


a) 1100
b) 1001
c) 0101
d) 0110
4) ____________ consumes minimum power.
a) TTL
b) CMOS
c) DTL

d) RTL

5) ____________memory is a volatile memory.


a) ROM
b) RAM
c) PROM

d) EEPROM

6) ____________device changes serial data to parallel data.


a) Counter
b) Multiplexer c) Demultiplexer d) Flip flop
7) 8 is equal to signed binary number ____________
a) 10001000
b) 00001000
c) 10000000
d) 11000000
8) ____________ logic family has the lowest propagation delay time.
a) ECL
b) TTL
c) CMOS
d) PMOS
P.T.O.

SLR-BB 338

*SLRBB338*

-2-

9) ____________ memory stores the most number of bits.


a) 5M*8
b) 1M*16
c) 5M*4
d) 1M*12
10) The number of control lines for 16 to 1 multiplexer is ____________
a) 2
b) 4
c) 3
d) 5
11) How many two input AND and OR gates are required to realize Y = CD + EF + G
a) 2,2
b) 2,3
c) 3,3
d) None of above
12) The output of SR flip flop when S = 1, R = 0 is
a) 1
b) 0
c) No change
d) High impedance
13) The number of flip flops contained in IC 7490 is ____________
a) 2
b) 3
c) 4
d) 10
14) The octal equivalent of (247)10 is __________
a) (252)8
b) (350)8
c) (367)8

d) (400)8

15) In a RAM information can be stored by ____________


a) The manufacturer
b) The user
c) Microcontroller
d) None of above
16) The A/D converter whose conversion time is independent of the number of
bits is ____________
a) Dual slope
b) Counter type
c) Parallel conversion
d) None of above
17) The access time of ROM using bipolar transistor is ____________
a) 1 sec
b) 1msec
c) 1nsec
d) 1 Microsec
18) The hexadecimal number A0 has the decimal value equivalent to __________
a) 80
b) 256
c) 100
d) 160
19) EPROM contents can be erased by exposing it to ____________
a) U.V. rays
b) I.R. rays
c) Microwaves
d) Heart radiations
20) When the set of input data to an even parity generator is 0111, the output
will be ____________
a) 1
b) 0
c) Unpredictable
d) None of above
______________

Set A

*SLRBB338*

-3-

SLR-BB 338

Seat
No.

S.E. (Biomedical Engineering) (Part II) Examination, 2014


DIGITAL DESIGN (Old)
Day and Date : Friday, 28-11-2014
Time : 3.00 p.m. to 6.00 p.m.

Marks : 80

SECTION I
2. Solve any four :

20

1) Show that A B + AC = AB + A C + BC .
2) Minimize following Boolean expression using K-map and realize it using basic
gates Y = m(1, 3, 5, 9, 11, 13).
3) State and prove De Morgans theorem.
4) Obtain 8:1 multiplexer using 4:1 multiplexer.
5) Explain the conversion of D flip-flop to T flip-flop and vice versa.
6) Define and explain hamming code with one example.
3. Solve any two :

(20 Marks each)

1) What is race around condition in JK flip flop. Explain master slave JK flip flop
with avoidance of race around condition.
2) Minimize the given expression using K-map
Y = m(1, 5, 6, 7, 11, 12, 13, 15)
3) Implement the expression Y = A B + CD+ AB C+ B C D using
a) Single 8:1 mux and gates
b) 4 lines to 16 lines decoder and gates

Set A

SLR-BB 338

-4-

*SLRBB338*

SECTION II
4. Solve any four questions.

20

1) Perform the following :


a) (653)8 (177)8
b) (A)16 (8)16
2) Realize full adder with NAND gates.
3) Multiply (72)16 (39)16
4) Divide (24)8 (4)8
5) Define a counter and mention different types of counter with four
applications.
6) Differentiate between synchronous and asynchronous counter with example
of each.
5. a) Compare TTL and CMOS logic families. Draw the TTL NAND gate and
explain the totem pole output.
b) Solve any one :

10
10

1) Draw and explain 4 bit ring counter with correct waveforms.


2) Design asynchronous mod 4 up down counter using JK flip flop.
_____________________

Set A

SLR-BB 339

*SLRBB339*
Seat
No.

Set

S.E. (Bio-medical Engineering) (Part II) Examination, 2014


ELECTRONIC CIRCUITS ANALYSIS AND DESIGN II (Old)
Day and Date : Saturday, 29-11-2014
Time : 3.00 p.m. to 6.00 p.m.
N.B. :

Max. Marks : 100

1)
2)
3)
4)
5)

All questions are compulsory.


Figures to right indicating full marks.
Use of non-programmable calculator is allowed.
Use of data sheets is allowed.
Q. No. 1 is compulsory. It should be solved in first 30 minutes in
Answer Book Page No. 3. Each question carries one mark.
6) Answer MCQ/Objective type questions on Page No. 3 only. Dont
forget to mention, Q.P. Set (A/B/C/D) on Top of Page.
MCQ/Objective Type Questions

Duration : 30 Minutes

Marks : 20

1. Objective type questions (one mark each) :


1) The unit of hfe is
a) ohm
b) mho
c) none
d) dimensionless
2) Graphically hfe is calculated by using-characteristics.
a) CB output characteristics
b) CC output characteristics
c) CE input characteristics
d) CE output characteristics

20

3) Current gain Az is given as ___________


a) A z =

hf
z

1 + ho L

b) A z =

hf
hf
c) A z =
1+ ho zo
1 + zL / ho

4) The configuration have very low current gain


a) CE
b) CC
c) CB

d) A z =

hf
1 + zL / ho

d) None

5) The number N of decibels by which the power P2 exceeds the power P1 is defined as
a) N = 20 log

P2
P1

b) N = 10 log

P1
P2

c) N = 20 log

P1
P2

d) N = 10 log

P2
P1

6) The transistor T1 and T2 has 1 and 2 current gain respectively, when connected in
darlington configuration then current gain is
a) 1 +2

b) (1 +2 )2

c) 12 2

d) 1 2

7) _________ stabilizes current series feedback amplifier.


a) Avf
b) amf
c) Aif

d) Rmf

8) The f/B improves transconductance amplification.


a) Voltage series
b) Current series c) Current shunt

d) Voltage shunt
P.T.O.

SLR-BB 339

*SLRBB339*

-2-

9) Second harmonic distortion is __________


B
B
a) D = B2
b) D2 = 2
c) D2 = 1
2
B2
B1
B1

d) D =

B1
B2

10) The advantage of push-pull system is


a) No odd harmonics are present
b) No even harmonics are present
c) More power efficiency
d) Used as power amplifier
11) The ability of difference amplifier to reject common signal is known as
a) CMRR
b) SVRR
c) Voltage swing
d) Slew rate
12) Slew rate is only consider for
a) AC applications
b) DC applications c) a) and b)
d) None
13) Ideal op-amp has __________ bandwidth.
a) 130 MHz
b) 500 MHz
c) Defined
d) Infinite
14) Voltage shunt feedback is used in
a) Inverting amplifier b) Non-inverting amplifier c) Emitter follower
d) None
15) Thermal drift is occur in op-amp due to
a) Change in temperature b) Change in supply voltage c) Time
d) All above
16) The circuit is

a) Peak detector
b) Rectifier
17) The o/p current of following circuit is

a) io =

vo
Rca

b) io =

vo
RL

c) Integrator

c) io =

d) Peaking amplifier

vo
Ri

d) v o =

vo
R total

18) fH is the high cut-off frequency of the first order low pass Butterworth filter, the value is
calculated as
b) fH= 1
c) f H= 1
d) fH = 1
a) fH = 1
2C
2f
2 RC
2 fC
19) The gain of second order high pass butterworth filter is
Av
Av
Av
b) Avf=
c) Avf=
d) None
a) Avf=
1 + j( f / fH )
1 + ( f ( f )2
1 + ( f / f )2
H

20) Weinbridge oscillator the gain of op-amp must be


a) 29
b) 3
c) 1.414

______________

d) A B =1

Set A

*SLRBB339*

-3-

SLR-BB 339

Seat
No.

S.E. (Bio-medical Engineering) (Part II) Examination, 2014


ELECTRONIC CIRCUITS ANALYSIS AND DESIGN II (Old)
Day and Date : Saturday, 29-11-2014
Time : 3.00 p.m. to 6.00 p.m.
N.B. :

1)
2)
3)
4)

Marks : 80

All questions are compulsory.


Figures to right indicating full marks.
Use of non-programmable calculator is allowed.
Use of data sheets is allowed.
SECTION I

2. Attempt any four :

(45=20)

1) Explain in detail h-parameter analysis of transistor. Derive the expression for AV, AI.
2) What is feedback amplifier and effect of feedback on stability and bandwidth ?
3) Classify the power amplifier.
4) Why power amplifier is last stage of amplification system ?
5) Show that the negative feedback amplifier reduces gain of two stage RC coupled amplifier.
3. Attempt any two :

(210=20)

1) Design RC coupled amplifier to amplify input signal of 10 mv, 25 Hz to 10 v output signal.


The range of frequency is 20 Hz 20 KHz.
2) Design RC phase shift oscillator to provide stabilized oscillations of 100 KHz.
3) Design the complementary system power amplifier to drive 8 , 500 W speaker.
SECTION II
4. Attempt any four :

(45=20)

1) Define the terms CMRR, SVRR , voltage swing and slew rate.
2) Explain the effect of variation in power supply voltage on offset voltage.

Set A

SLR-BB 339

-4-

*SLRBB339*

3) Calculate the gain and output voltage of following fig.

4) What is frequency rolloff and how to overcome roll off effect ?


5) Explain with waveforms Weinbridge oscillators.
5. Attempt any two :

(210=20)

1) Design inverting and non-inverting amplifier with the gain of 100, 200 respectively.
2) Design high pass second order Butterworth filter with cut-off frequency of 100 KHz.

3)

Calculate the output and draw O/P waveforms.


_____________________

Set A

SLR-BB 34

*SLRBB34*
Seat
No.

Set

S.E. (Civil) (Part II) (New) Examination, 2014


FLUID MECHANICS II
Day and Date : Friday, 28-11-2014
Time : 3.00 p.m. to 6.00 p.m.
Instructions :

Max. Marks : 100

i) Q. No. 1 is compulsory. It should be solved in first 30 minutes in


Answer Book Page No. 3. Each question carries one mark.
ii) Answer MCQ/Objective type questions on Page No. 3 only. Dont
forget to mention, Q.P. Set (A/B/C/D) on Top of Page.
iii) Q. No. 2 from Section I and Q. No. 6 from Section II are compulsory.
iv) Solve any two questions from questions 3, 4 and 5 from Section I.
v) Solve any two questions from questions 7, 8 and 9 from
Section II.
MCQ/Objective Type Questions

Duration : 30 Minutes

Marks : 20

1. Choose the correct answer :

20

1) The Kinetic energy correction factor is given as ___________

dA

dA

c)

vdA

d) None of above
v A
v A
vA
2) On specific energy curve, above the critical depth _________ flow exist.
a) critical
b) subcritical
c) super critical
d) uniform
a)

b)

3) Critical depth in channel is expressed by __________


Q2A3
=1
d)
9T3

Q2T
QT 2
Q2 T2
=
1
=
1
=1
a)
b)
c)
9A 3
9A 3
9A 3
4) For oscillating jump, Froude No. is ____________
a) 2.5 to 4.5
b) 4.5 to 9
c) 1.0 to 1.7

d) 1.7 to 2.5

5) The direct step method is used for finding ___________


a) flow profile height
b) length of flow profile
c) conjugate depth of jump
d) none of above
6) Hydraulic jump is used for ______________
a) Mixing of chemicals
b) Protection of hydraulic structure
c) Energy dissipation
d) All of above
7) An error of 1% in measurement of H will produce ___________ error in the measurement
of discharge over a triangular notch.
a) 1%
b) 1.5%
c) 2%
d) 2.5%
8) Maximum discharge for broad crested weir is given as _____________
a) Q = Cd LH

b) Q = Cd LH

c) Q = 0 .5 Cd LH

d) Q = 1 .705 Cd H

P.T.O.

SLR-BB 34

*SLRBB34*

-2-

9) For broad crested weir, its crest width (B) is related to height of weir (H) as _____
a) B 2.5 H

c) 0.1

b) B 10 H

H
0.4
B

d) All of above

10) The flow in open channel is turbulent if Reynolds No. is _____________


a) 2000
b) More than 2000
c) More than 4000
d) Less than 4000
11) Work done by jet on flat inclined plate moving in direction of jet is _________
a) au( v u)2 sin 2

b) au( v u)3 sin 2

c) au( v u) 2

d) a( v u)2 sin

12) For turbine, mechanical efficiency is given as ____________


a)

Shaft power
Runner power

b)

Runner power
Water power

13) Kaplan turbine is ___________


a) Impulse turbine
c) Axial flow reaction turbine

c)

Runner power
Shaft power

d)

Water power
Shaft power

b) Radial flow impulse turbine


d) Radial flow reaction turbine

14) Benefit of connecting pump in parallel is ___________


a) Increase in discharge
b) Increase in head
c) Both a) and b)
d) None of above
15) Specific speed of pump is given by ___________
a) NS =

N P
Hm

5/4

b) NS =

N Q
Hm

5/4

c) NS =

N P
Hm

3/4

d) None of above

16) A centrifugal pump is said to be low head pump if head is _________


a) up to 15 m
b) between 15 to 40 m
c) above 40 m
d) none of above
17) Dimensions of dynamic viscosity are _____________

a) L2 T 1

b) M1L2 T 2

18) Prototype is ____________


a) Actual machine
c) Large replica machine

c) L2 T 1

d) M1L1T 1

b) Small replica machine


d) Both b) and c)

19) For dynamic similarity to exist ______________


a) Geometric similarity is must
b) Kinematic similarity is must
c) Both a) and b)
d) None of above
20) Kinematic similarity between model and prototype exists if their __________
a) Linear dimensions are same
b) Velocity and acceleration of corresponding point are same
c) Forces acting at corresponding point are same
d) None of above
______________
Set A

*SLRBB34*

SLR-BB 34

-3-

Seat
No.
S.E. (Civil) (Part II) (New) Examination, 2014
FLUID MECHANICS II
Day and Date : Friday, 28-11-2014
Time : 3.00 p.m. to 6.00 p.m.
Instructions :

Marks : 80

i) Q. No. 2 from Section I and Q. No. 6 from Section II are


compulsory.
ii) Solve any two questions from questions 3, 4 and 5 from Section I.
iii) Solve any two questions from questions 7, 8 and 9 from
Section II.
SECTION I

2. A) Explain the term :


i) Conveyance factor of channel
ii) Momentum correction factor.

B) Draw the sketches of M3, S1 water surface profiles and give the conditions.

C) Find the time required to lower the water level from 5m to 4m in a reservoir of size
90 m 90 m by
i) Rectangular notch of 1.0 m and
ii) Triangular notch. Take Cd = 0.62.

3. A) For hydraulic jump show that energy loss E =

(y 2 y1 )3
4y1y 2

B) A rectangular channel of width 4 m is having bed slope of 1 in 1500. Find maximum


discharge through the channel. Take C = 50.

4. A) Derive the expression of most economical channel section for trapezoidal channel section. 7
B) Explain different types of nappe with neat sketches.

C) What do you mean by ventilation of weirs ? What is its necessity ?

5. A) Show that for rectangular notch,

dQ
dH
= 1.5
.
Q
H

B) The depth of flow at a certain section of rectangular channel of 4m wide is 0.5 m. The
discharge through channel is 16 m3/sec. If the hydraulic jump takes place on down
stream side, find the depth of flow after the jump and height of jump, energy loss per kg
of water.

Set A

SLR-BB 34

-4-

*SLRBB34*

SECTION II
6. A) Derive an expression for force exerted by jet on stationary curved plate when jet striking
at centre of symmetrical curved plate.

B) With neat sketch explain operating characteristics of centrifugal pump.

C) Explain Reynolds and Froudes model laws.

7. A) Differentiate between impulse and reaction turbine.

B) Find the force exerted by jet of water of diameter 70 mm on stationary flat plate when
the jet strikes the plate normally with velocity of 25 m/sec.
8. A) Define :
i) Static head
ii) Manometric head
iii) Specific speed of centrifugal pump.

7
6

B) A centrifugal pump has an impeller 0.5 m out diameter and when running at 600 rpm
discharges water at rate of 8000 lit./min. against a head of 8.5 m. The water enters the
impeller without whirl and shock. The inner diameter is 0.25 m and the vanes are set
back at outlet at an angle of 45 and the area of flow which is constant from inlet to
outlet of the impeller is 0.06 m2. Determine :
i) Manometric efficiency and
ii) Vane angle at inlet.
9. A) What is mean by dimensionless number ? Explain any two dimensionless numbers.

7
6

B) Show that frictional torque (T) of a disc of diameter (D) at a speed N is a fluid of
viscosity and density in a turbulent flow is given by T = D5N2

[ D N] .
2

_____________________

Set A

SLR-BB 340

*SLRBB340*
Seat
No.

Set

S.E. (Biomedical Engg.) (Part II) Examination, 2014


TRANSDUCER IN BIOMEDICAL INSTRUMENTATION (New)
Day and Date : Tuesday, 25-11-2014
Time : 3.00 p.m. to 6.00 p.m.

Total Marks : 100

Instructions : 1) Q. No. 1 is compulsory. It should be solved in first 30 minutes


in Answer book Page No. 3. Each question carries one mark.
2) Figures to the right indicate full marks.
3) Assume suitable data wherever required.
4) Answer MCQ/Objective type questions on Page No. 3
only. Dont forget to mention, Q.P. Set (A/B/C/D) on Top
of Page.
MCQ/Objective Type Questions
Duration : 30 Minutes
1. Answer the following questions :

Marks : 20
(201=20)

1) Total Hemoglobin count in inversely proportional to percentage of _____


a) So2
b) O2
c) Po2
d) HbO2
2) _____________ sensors are sensitive to ambient temperature.
a) Pressure
b) Light
c) Optical
d) Radiation
3) ___________ utilizes micro miniature sensors.
a) Transducer
b) LVDT
c) ISFETs

d) Piezoelectric

4) The silver chloride electrode is ____________ electrode.


a) polarized
b) nonpolarized c) charged
d) discharged
5) In _________________ fluids the concentration of sodium ions is relatively
high.
a) CSF
b) Biological
c) Hormone
d) Saliva
6) Electric ____________ of epidermal layer behaves as parallel RL circuit.
a) impedance
b) conductance c) current
d) none of above
7) ___________ tip of the electrode of metal microelectrode remains uninsulated.
a) Covered
b) Extreme
c) Open
d) None of above
P.T.O.

SLR-BB 340

-2-

*SLRBB340*

8) ___________ is the smallest incremental quantity that can be measured with


certainty.
a) Resolution
b) Linearity
c) Error
d) Micro
9) The relationship between input and output is called as ____________ function.
a) transfer
b) nonlinear
c) linear
d) complex
10) A linear potentiometer is a example of a ____________ instrument.
a) first order
b) second order c) third order
d) zero order
11) _____________ low pass filter circuit is an example of first order instrument.
a) RLC
b) RL
c) LC
d) RC
12) _____________ measurement with thermocouple is based on Seebeck effect.
a) Current
b) Force
c) Pressure
d) Temperature
13) Thermistors are conductors made of ______________ material.
a) copper
b) aluminium
c) ceramic
d) none of above
14) The ability of an instrument to give the ___________ output for equal inputs
applied over some period is called linearity.
a) different
b) same
c) zero
d) compactiable
15) A mechanical pressure measuring instrument illustrates the properties of
_________ order system.
a) first order
b) second order c) third order
d) zero order
16) The ________ are used to measure displacement.
a) thermostate
b) piezoelectric c) strain gauge

d) none of above

17) The _______________ circuit is ideal for measuring small changes in


resistance.
a) amplifier
b) wheatstone-bridge
c) oscillator
d) buffer
18) When _____________ electrode is in contact with an electrolyte a double
layer of charge forms at the interface.
a) polarizable
b) non-polarizable
c) disk
d) gel
19) Percutaneous electrodes are used to detect ___________ within the body.
a) fluid
b) biopotential
c) potential
d) current
20) ________________ micropipette microelectrodes are fabricated from glass
capillaries.
a) Glass
b) Metal
c) Ceramic
d) Copper
______________
Set A

*SLRBB340*

-3-

SLR-BB 340

Seat
No.

S.E. (Biomedical Engg.) (Part II) Examination, 2014


TRANSDUCER IN BIOMEDICAL INSTRUMENTATION (New)
Day and Date : Tuesday, 25-11-2014

Marks : 80

Time : 3.00 p.m. to 6.00 p.m.


Instructions : 1) Figures to the right indicate full marks.
2) Assume suitable data wherever required.
SECTION I
2. Attempt any four :

(45=20)

1) Explain the concept of electrode electrolyte interface with relevant diagram.


2) Draw and explain bonded and unbonded strain gages.
3) Describe first and second order characteristics of transducer.
4) What is motion artifact ? How it is minimized ?
5) Define and explain gauge factor ? Derive an expression for the same.
3. Attempt any two :

(210=20)

1) Explain construction and working principle of LVDT. Mention its any two
biomedical applications.
2) Draw and explain electrical equivalent circuit for electrode skin interface.
3) List and explain various factors that should be taken into consideration for
selecting a transducer for biomedical applications.

Set A

SLR-BB 340

-4-

*SLRBB340*

SECTION II
4. Attempt any four :

(45=20)

1) Describe the equivalent circuit model for electrode skin interface.


2) Explain the working principle of ISFET with necessary diagram.
3) Explain the use of fiber optic in chemical sensors.
4) State various radiation sensors that are used to detect temperature in detail.
5) Explain the construction and working principle of pCO2 electrode.
5. Attempt any two :

(210=20)

1) Define radiation sensor and explain it with any two examples.


2) Explain different types of biopotential electrodes with suitable sketch.
3) Define Immuno sensor. Explain the working of any one immunosensor with
relevant diagram.
_____________________

Set A

SLR-BB 341

*SLRBB341*
S

S.E. (Biomedical Engg.) (Part II) Examination, 2014


BIOMEDICAL PROSTHETIC AND ORTHOTICS (New)
Day and Date : Wednesday, 26-11-2014
Time : 3.00 p.m. to 6.00 p.m.

Total Marks : 100

Instructions : 1) Figures to the right indicate full marks.


2) Assume suitable data wherever required.
3) Q. No. 1 is compulsory. It should be solved in first 30
minutes in Answer Book Page No. 3. Each question
carries one mark.
4) Answer MCQ/Objective type questions on Page No. 3
only. Dont forget to mention, Q.P. Set (A/B/C/D) on Top
of Page.
Duration : 30 Minutes

MCQ/Objective Type Questions

Marks : 20

1. Answer the following questions :


(201=20)
1) Bounding training and depth jumps are the best method to enhance explosive
strength in ____________ extremities.
a) lower
b) upper
c) middle
d) none of above
2) Kinematics is the branch of mechanics that describes the cause of __________
a) pressure
b) force
c) biomechanics d) stress
3) In isometric ______________, the muscle shortens.
a) shortens
b) resting
c) relaxation
d) contraction
4) ______________ is the branch of physics that deals with bodies in motion.
a) Prosthesis
b) Orthosis
c) Mechanics
d) None of the above
5) Sideways bending of trunk is an example of movement in _____________
a) frontal plane and sagittal axis
b) sagittal plane and sagittal axis
c) frontal plane and transverse axis d) sagittal plane and lateral axis
6) Newtons second law of motion is also called ___________
a) law of action and reaction
b) law of inertia
c) law of gravity
d) law of acceleration
7) Lever system prevalent in human arm is ______________
a) Class III
b) Class II
c) Class I
d) None of the above
P.T.O.

SLR-BB 341

*SLRBB341*

-2-

8) First law of motion is also called ___________


a) Law of action and reaction
b) Law of conservation of energy
c) Law of inertia
d) Law of transference of momentum
9) Imaginary line passing laterally from one side to other is called _____________
a) sagittal axis
b) sagittal plane
c) vertical axis
d) lateral axis
10) ______________ is measure of time.
a) Pressure
b) Displacement c) Force

d) Duration

11) Fast twitch type of muscles is capable of resisting fatigue in ____________


duration activity.
a) very less
b) short
c) long
d) both (a) and (b)
12) Muscles which cause the joints to bend are called ___________
a) flexors
b) extensors
c) abductors
d) adductors
13) Duration is measure of ___________
a) distance
b) displacement c) force

d) time

14) Speed is indicated in _______________


b) Cm/hour
c) Newton
a) Km/sec2

d) Km/hr

15) ________________ joint is slightly movable.


a) Synovial
b) Wrist
c) Both (a) and (b)
d) None of the above
16) The terms anterior and posterior are synonymous with ___________
a) frontal and back
b) verbal and dorsal
c) lateral and medial
d) None of the above
17) The production of bioartificial skin grafts requires that both __________ and
keratinocytes.
a) Fibroblasts
b) Cellulose
c) Cells
d) Dermis
18) Long type of bone performs the function of providing ___________
a) stress
b) force
c) support
d) strength
19) ____________ joint is the example of Hinge joint.
a) Pivot joint
b) Carpal
c) Middle
d) Ball and socket joint
20) Vertical planes of the body divides it into upper and ___________ parts.
a) joint
b) bone
c) middle
d) lower
______________

Set A

*SLRBB341*
S

-3-

SLR-BB 341

S.E. (Biomedical Engg.) (Part II) Examination, 2014


BIOMEDICAL PROSTHETIC AND ORTHOTICS (New)
Day and Date : Wednesday, 26-11-2014

Marks : 80

Time : 3.00 p.m. to 6.00 p.m.


Instructions : 1) Figures to the right indicate full marks.
2) Assume suitable data wherever required.
SECTION I
2. Attempt any four :

(45=20)

1) Draw and explain stress stain curve with nomenclatures.


2) List different types of synovial joints and explain one of it in detail.
3) Explain with a neat diagram anisotropic property of bone.
4) Define and classify force system.
5) Write a short note on Goniometer.
3. Attempt any two :

(210=20)

1) Define lever. Explain different types of lever with two anatomical examples.
2) Explain Motion Analysis in detail.
3) What is the significance of gait cycle analysis ? Explain the steps involved in
gait analysis with neat diagram.

Set A

SLR-BB 341

-4-

*SLRBB341*

SECTION II
4. Attempt any four :

(45=20)

1) Draw and explain structure and function of Jaipur foot.


2) Explain the significance of patient rehabilitation in detail.
3) Discuss the purpose for providing prosthesis and orthosis with each example.
4) Write a note on SACH foot.
5) Explain three point pressure principles. How it can be applied in various
prosthetic devices ?
5. Attempt any two :

(210=20)

1) List upper and lower extremity orthoses and prostheses and explain any
one of each with neat diagram.
2) Explain properties and applications of biomaterials used for soft tissue
replacement.
3) Explain total PTB contact prosthesis using suitable diagram.
_____________________

Set A

SLR-BB 342

*SLRBB342*
Seat
No.

Set

S.E. (Bio-Medical Engineering) (Part II) (New) Examination, 2014


ELECTRONIC INSTRUMENTATION
Day and Date : Thursday, 27-11-2014
Time : 3.00 p.m. to 6.00 p.m.

Max. Marks : 100

Instructions : 1) Assume suitable data if necessary.


2) Figure to right indicates full marks.
3) Q. No. 1 is compulsory. It should be solved in first 30 minutes in Answer
Book Page No. 3. Each question carries one mark.
4) Answer MCQ/Objective type questions on Page No. 3 only. Dont
forget to mention, Q.P. Set (A/B/C/D) on Top of Page.
MCQ/Objective Type Questions
Duration : 30 Minutes

Marks : 20

1. Select the correct answer :

(120=20)
SECTION I

1) In measurement systems, which of the following static characteristics are desirable


a) Accuracy
b) Sensitivity
c) Reproducibility d) All of the above
2) In measurement systems, which of the following are undesirable static characteristics
a) Accuracy and sensitivity
b) Drift, static error and dead zone
c) Reproducibility and non-linearity
d) Drift, static error, dead zone and non-linearity
3) A reading is record as 23.90 C. The reading has
a) three significant figures
b) five significant figures
c) four significant figures
d) none of the above
4) The power can be controlled in a circuit by
a) controlling the voltage applied to load
c) using pulse modulation of power

b) controlling the current level in the circuit


d) all of the above

5) A quantity whose magnitude has a definite repeating time cycle is called a __________
a) transient
b) steady state periodic
c) steady state aperiodic
d) transient state periodic
6) Dynamic response consists of _________________
a) two parts, one steady state and the other transient state response
b) only transient state response
c) only steady state response
d) none of these
7) In a second order system, the peak overshoot is 100%. The value of damping factor is
a) 1
b) 0.707
c) 0
d) 0.5
8) Bandwidth a frequency domain concept, is indicative of __________
a) rise time in time domain
b) setting time in time domain
c) steady state error in the domain
d) none of these
P.T.O.

SLR-BB 342

*SLRBB342*

-2-

9) In present day measurement systems ______________


a) direct methods are commonly used
b) use of direct methods is limited but indirect methods are commonly used
c) both direct indirect methods are commonly used
d) all of the above
10) Purely mechanical instruments cannot be used for dynamic measurements because they
have _____
a) high inertia
b) large time constant
c) higher response time
d) all of the above
SECTION II
11) The source of Emission of Electron in a CRT is
a) PN function diode
b) A barium and strontium oxide coated cathode
c) Accelerating anodes
d) Post accelerating anodes
12) In a CRT the focussing anode is located
a) Between pre-accelerating and accelerating anodes
b) After accelerating anode
c) Before accelearting anode
d) None of the above
13) The deflection of an electron beam on a CRT screen is 10 mm suppose the preaccelerating
anode voltage is Halved and the potential between deflecting plates is doubled the deflection
of the electron beam will be
a) 80 mm
b) 40 mm
c) 20 mm
d) 10 m
14) If the distance of an screen from a CRT to centre of deflection plates is 15 cm the length of
deflection plates is 2 cm the distance between plates is 1 cm and the accelerating voltage
is 500 V the deflection sensitivity is
a) 33.2 V/cm
b) 0.03 cm/V
c) 66.4 V/cm
d) 0.015 cm/V
15) Post acceleration is needed in a CRO if the frequency of the signal is
a) less than 1 MHz
b) more than 1 MHz
c) more than 10 MHz
d) more than 10 Hz
16) The desirable characteristics of a signal source are
a) the frequency of the signal should be well known and stable
b) the amplitude of a signal sources should be controllable from very small to relatively
large values
c) the output signal should be free from distortion
d) all the above
17) In signal generator
a) energy is created
b) energy is generated
c) energy is converted from simple D.C. source into a.c. energy at some specific frequency
d) all the above
18) A video frequency oscillator has a frequency range of
a) 20 Hz to 20 KHz
b) 20 KHz to 30 KHz c) D.C. to 5 MHz

d) 2 MHz to 30 MHz

19) Feedback oscillator have a closed loop gain of


G
G
H
b) 1 + GH
c) 1 GH
d) 1 + GH
a) 1 GGH
20) In an RC phase shift oscillator having three stages which have equalivalued resistor and capacitor
the amplifier in order to satisfy Brakhausen criterion should have a minimum gain of
a) 6
b) 9
c) 31
d) none of the above
______________
Set A

*SLRBB342*

-3-

SLR-BB 342

Seat
No.

S.E. (Bio-Medical Engineering) (Part II) (New) Examination, 2014


ELECTRONIC INSTRUMENTATION
Day and Date : Thursday, 27-11-2014

Marks : 80

Time : 3.00 p.m. to 6.00 p.m.


Instructions : 1) All questions are compulsory.
2) Assume suitable data if necessary.
3) Figure to right indicate full marks.

SECTION I
2. Solve any four :

(45=20)

a) What do you mean by order of a system ? Explain with example.


b) Explain the factors involved in selection of Voltmeter.
c) Explain principle of operation :
1) Ammeter
2) Ohmmeter
d) Explain methods of analog to digital conversion.
e) Draw and explain a block diagram of digital multimeter.
3. Solve any two :

(210=20)

a) With the neat diagram explain true RMS responding voltmeter.


b) Which are the standard test signals ? Explain the time of response of first, second order
system.
c) Differentiate between analog and digital frequency meter. Explain resolution and sensitivity
of digital voltmeter.

Set A

SLR-BB 342

-4-

*SLRBB342*

SECTION II
4. Answer any four questions :

(45=20)

1) Explain about the requirement of a good laboratory type signed generator.


2) Explain about the double beam for oscilloscopes.
3) Explain about the multichannel DAS.
4) Explain about the oscilloscope for biomedical measurement.
5) Explain about the Non-Fade display system.
5. Answer any two questions :

(210=20)

1) Draw the block diagram of an oscilloscope and explain the function of each block.
2) Give the construction features of strip-chart recorder sketch its various application.
3) Explain about the single and multichannel display.
_____________________

Set A

SLR-BB 343

*SLRBB343*
Seat
No.

Set

S.E. (Bio-Medical Engineering) (Part II) (New) Examination, 2014


DIGITAL DESIGN
Day and Date : Friday, 28-11-2014
Time : 3.00 p.m. to 6.00 p.m.
Instructions :

Max. Marks : 100

1) Assume suitable data if necessary.


2) Figures to right indicate full marks.
3) Q. No. 1 is compulsory. It should be solved in first 30 minutes in Answer book
Page No. 3. Each question carries one mark.
4) Answer MCQ/Objective type questions on Page No. 3 only. Dont forget to
mention, Q.P. Set (A/B/C/D) on Top of Page.
MCQ/Objective Type Questions

Duration : 30 Minutes

Marks : 20

1. Select the correct answer :

(201=20)

1) The difference between a PLA and a PAL is


a) The PLA has a programmable OR plane and a programmable AND plane, while the PAL only
has a programmable AND plane
b) The PAL has a programmable OR plane and a programmable AND plane, while the PLA only
has a programmable AND plane
c) The PAL has more possible product terms than the PLA
d) PALs and PLAs are the same thing
2) On the fifth clock pulse, a 4-bit Johnson sequence is Q0 = 0, Q1 = 1, Q2 = 1, and Q3 = 1. On the
sixth clock pulse, the sequence is ___________
a) Q0 = 1, Q1 = 0, Q2 = 0, Q3 = 0
b) Q0 = 1, Q1 = 1, Q2 = 1, Q3 = 0
c) Q0 = 0, Q1 = 0, Q2 = 1, Q3 = 1
d) Q0 = 0, Q1 = 0, Q2 = 0, Q3 = 1
3) What is the difference between a ring shift counter and a Johnson shift counter ?
a) There is no difference
b) A ring is faster
c) The feedback is reversed
d) The Johnson is faster
4) Three cascaded modulus-5 counters have an overall modulus of __________
a) 5
b) 25
c) 125
d) 500
5) A correct output is achieved from a master-slave J-K flip-flop only if its inputs are stable while the
a) Clock is LOW
b) Slave is transferring
c) Flip-flop is reset
d) Clock is HIGH
6) The simplest equation which implements the K-map shown below is
AB
AB
AB
AB

C
0
1
1
0

C
0
1
1
1

a) X = AC + B

b) X = A B

c) X = AB C + ABC + A B C

d) X = AB + A B
P.T.O.

SLR-BB 343

*SLRBB343*

-2-

7) The output 0 and 1 level for TTL logic family is approximate


a) 0.1 V and 5 V
b) 0.6 V and 3.5 V
c) 0.9 V and 1.15 V

d) 0 V and 5 V

8) The value of x, y, z, w that satisfies following equations


x + xy = 0 , xy = xz, xy + xz + zw = zw is
a) x = 1 y = 0 z = 0 w = 1
c) x = 1 y = 0 z = 1 w = 1

b) x = 1
d) x = 1

y=1
y=0

z=0
z=0

w=0
w=0

9) The output z of the following circuit is

a) 0
b) 1
c) x
d) x
10) Convert the following SOP expression to an equivalent POS expression.
ABC+AB C+ABC+ABC+A BC
a) ( A + B + C ) ( A + B + C ) ( A + B + C)

b) (A + B + C) (A + B + C) (A + B + C )

c) (A + B + C ) (A + B + C) (A + B + C)

d) (A + B + C) (A + B + C ) (A + B + C)

11) Why is a refresh cycle necessary for a dynamic RAM ?


a) To clear the flip-flops
b) To set the flip-flops
c) The refresh cycle discharges the capacitor cells
d) The refresh cycle keeps the charge on the capacitor cells
12) Which type of ROM can be erased by UV light ?
a) ROM
b) Masked ROM
c) EPROM

d) EEPROM

13) In general, the _______ have the smallest bit size and the _________ have the largest.
a) SRAM, mask ROM
b) mask ROM, SRAM
c) EEPROMs, Flash
d) DRAM, PROM
14) The 2s complement of the number 1101101 is
a) 0101110
b) 0111110

c) 0110010

d) 0010011

15) The A/D converter whose conversion time is independent of the number of bits is
a) Dual slope
b) Counter type
c) Parallel conversion
d) Successive approximation
16) When signed numbers are used in binary arithmetic, then which one of the following notations would
have unique representation for zero.
a) Sign-magnitude
b) 1s complement
c) 2s complement
d) 9s complement
17) When comparing the conversions from digital-to-analog and analog-to-digital, the A/D conversion is
generally
a) less complicated but more time consuming than the D/A conversion
b) more complicated and more time consuming than the D/A conversion
c) less complicated and less time consuming than the D/A conversion
d) more complicated but less time consuming than the D/A conversion
18) The decimal equivalent of hex number 1A53 is
a) 6793
b) 6739

c) 6973

d) 6379

19) An analog quantity varies from 0 7 V and is input to a 6-bit A/D converter. What analog value is
represented by each step on the digital output ?
a) 1.17 V
b) 0.109 V
c) 0.111 V
d) 0.857 V
20) The Gray code for decimal number 6 is equivalent to
a) 1100
b) 1001
c) 0101
______________

d) 0110

Set A

*SLRBB343*

-3-

SLR-BB 343

Seat
No.

S.E. (Bio-Medical Engineering) (Part II) (New) Examination, 2014


DIGITAL DESIGN
Day and Date : Friday, 28-11-2014
Time : 3.00 p.m. to 6.00 p.m.
Instructions :

Marks : 80

1) All questions are compulsory.


2) Assume suitable data if necessary.
3) Figures to right indicate full marks.
SECTION I

2. Solve any four :

(44=16)

1) What are the advantages of CMOS logic and explain CMOS Inverter with the help of a neat circuit
diagram ?
2) What is a Shift Register ? What are its various types ? Explain any one with waveform.
3) Design a 32 : 1 multiplexer using two 16 : 1 multiplexers and a 2 : 1 multiplexer.
4) For F = A B C + B C D + A B C , write the truth table. Simplify using Karnaugh map and realize
the function using NAND gates only.
5) Draw a logic diagram, truth table and output waveforms for a 4-bit ripple counter.
3. Solve any three :

(38=24)

1) What is a flip-flop ? Write the truth table for a clocked J-K flip-flop that is triggered by the positivegoing edge of the clock signal. Explain the operation of this flip-flop for the following conditions.
Initially all inputs are zero and assume the Q output to be 1.

2) A staircase light is controlled by two switches one at the top of the stairs and another at the bottom
of stairs.
i) Make a truth table for this system.
ii) Write the logic equation is SOP form.
iii) Realize the circuit using AND-OR gates.

Set A

SLR-BB 343

-4-

*SLRBB343*

3) A three input 4 output combinational circuit has following output functions. Implement the circuit
using suitable PAL.
A(x, y, z) = m(1, 2, 4, 6)
B(x, y, z) = m(0, 1, 3, 6, 7)
C(x, y, z) = m(1, 2, 4, 6, 7)
D(x, y, z) = m(1, 2, 3, 5, 7)

4) What are synchronous counters ? Design a Mod-5 synchronous counter using J-K flip-flops.
SECTION II
4. Solve any four :

(44=16)

1) Distinguish between ROM, PROM, EPROM, EEPROM.


2) Explain Arithmetic and Logic Unit (ALU) in detail.
3) What is a half-adder ? Explain a half-adder with the help of truth-table and logic diagram.
4) A 6-bit R-2R ladder D/A converter has a reference voltage of 6.5 V. It meets standard linearity.
Find :
i) The Resolution in Percent.
ii) The output voltage for the word 011100.
5) Prove the following equations using the Boolean algebraic theorems :
i) A + A B + A B = A + B
ii) ABC + A B C + AB C + ABC = AB + BC + AC .
5. Solve any three :

(38=24)

1) With the help of a neat diagram, explain the working of a successive approximation A/D converter.
2) Perform the operation using 2s complement of the two numbers.
(+187)10 (+41)10
Convert following no. to Hexadecimal, octal and Binary equivalent (125)10.
3) With the help of R-2R binary network, explain the working of a 4 bit D/A converter and derive an
expression for the output voltage.
4) Difference between static and dynamic RAM. Draw the circuits of one cell of each and explain its
working.
_____________________

Set A

SLR-BB 344

*SLRBB344*
Seat
No.

Set

S.E. (Bio-Medical Engg.) (Part II) Examination, 2014


ELECTRONIC CIRCUIT ANALYSIS AND DESIGN II (New)
Day and Date : Saturday, 29-11-2014
Time : 3.00 p.m. to 6.00 p.m.

Max. Marks : 100

Instructions : 1) Use of data sheet and calculator is allowed.


2) Figures to the right indicate full marks.
3) Q. No. 1 is compulsory. It should be solved in first 30 minutes
in Answer Book Page No. 3. Each question carries one mark.
4) Answer MCQ/Objective type questions on Page No. 3 only.
Dont forget to mention, Q.P. Set (A/B/C/D) on Top of Page.
MCQ/Objective Type Questions
Duration : 30 Minutes

Marks : 20

1. Choose the correct option :


(120=20)
1) Two stages of a multistage amplifier have a gains of 50 and 20. The dB
voltage gain is __________
a) 3
b) 60
c) 300
d) 1000
2) The resistance-capacitance coupling is popular in audio amplifiers because
__________
a) it provides an output signal in phase with the input signal
b) it needs low collector supply voltage
c) it has better audio frequency response
d) none of these
3) A transformer coupled amplifier would give
a) Maximum voltage gain
b) Impedance matching
c) Maximum current gain
d) larger bandwidth
4) In an amplifier if current series feedback is employed then input resistance
__________
a) increases
b) decreases
c) no effect
d) none of these
5) A feedback amplifier has a closed gain of 200. It should not vary more
than 50% despite 25% variation in amplifier gain A without feedback. The
value of A is __________
a) 800
b) 800
c) 1000
d) 1000
6) The maximum overall efficiency of a transformer coupled class-A amplifier
is __________
a) 78.5%
b) 25%
c) 50%
d) 85%
P.T.O.

SLR-BB 344

-2-

*SLRBB344*

7) Cross over distortion occurs in __________ amplifiers.


a) Push-pull
b) Class A
c) Class B
d) Class AB
8) Class AB operation is often used in power amplifiers in order to
a) get maximum efficiency
b) remove even harmonics
c) overcome cross-over distortion
d) reduce collector dissipation
9) For sustained oscillation loop gain must be __________
a) greater than 1 b) less than 1
c) equal to 1
d) none of these
10) Oscillators must employ __________
a) negative feedback
b) positive feedback
c) neither negative nor positive
d) both positive and negative
11) Common mode gain should be
a) Very low
b) Very high
c) Always unity d) Unpredictable
12) The voltage gain of voltage follower is
a) Unity
b) Less than unity
c) Greater than unity
d) Variable
13) The voltage gain of non inverting amplifier
a) Unity
b) Less than unity
c) Greater than unity
d) Both a) and c)
14) A differential amplifier __________
a) is a part of an OP-amp
b) has one input and one output
c) has two outputs
d) both a) and c) answers
15) Higher order active filters are used for
a) Bandwidth
b) Gain in passband
c) Impedance
d) Higher roll off rate
16) Common mode gain is
a) Very high
b) Very low
c) Unity
d) Unpredictable
17) The input stage of an op-amp is usually
a) Differential amplifier
b) Class B push pull amplifier
c) CE amplifier
d) Any one of these
18) Current cannot flow to ground through __________
a) a mechanical ground
b) an a.c. ground
c) a virtual ground
d) an ordinary ground
19) The output of a particular OP-amp increases 8 V in 12 s. The slew rate is
__________
b) 0.67 V/ s
c) 1.5 V/ s
d) none of these
a) 90 V/ s
20) The OP-amp can amplify __________
a) a.c. signals only
b) d.c. signals only
c) both a.c. and d.c. signals
d) neither d.c. nor a.c. signals
______________
Set A

*SLRBB344*

SLR-BB 344

-3-

Seat
No.

S.E. (Bio-Medical Engg.) (Part II) Examination, 2014


ELECTRONIC CIRCUIT ANALYSIS AND DESIGN II (New)
Day and Date : Saturday, 29-11-2014
Time : 3.00 p.m. to 6.00 p.m.

Marks : 80

Instructions : 1) Use of data sheet and calculator is allowed.


2) Figures to the right indicate full marks.
SECTION I
2. Answer any four :

(45=20)

1) Explain different types of coupling in detail.


2) What are the advantages offered by negative feedback ?
3) Explain Barkhausens criteria to have oscillations. Explain RC phase shift
oscillator.
4) Explain Class A operation and derive the expression for collector efficiency.
5) What is cross-over distortion ? How it can be avoided ?
3. Answer any two :

(210=20)

1) For the circuit shown below, determine Ri, Ro, Av and draw h-equivalent
circuit. Assume hie = 2k and hfe = 50.

1K

2) Design Wein bridge oscillator, to provide output signal with 5V peak and
frequency of 500 kHz.
3) Explain with suitable application Class C power amplifier. Explain Class C
operation in detail.
Set A

SLR-BB 344

-4-

*SLRBB344*

SECTION II
4. Solve any four :

(45=20)

1) Explain concept of virtual ground concept.


2) Explain effect of variation of power supply on offset voltage.
3) Draw and explain instrumentation amplifier.
4) Find the output voltage for the inverting amplifier with open loop configuration
for :
a) Vin = 20 mV DC
b) Vin = 50 V peak sine wave.
5) Explain working of ECG filter.
6) Explain working of subtracter using op-amp.
5. Solve any two :

(210=20)

1) Design a second order low pass filter at a high cutoff frequency of 1kHz and
draw the frequency response of filter designed.
2) What is shield drive and explain EGG application using op-amp ?
3) Define the following parameters of OPAMP and write their ideal and practical
values.
a) Input offset voltage
b) CMRR
c) Slew rate
d) SVRR
e) Output voltage swing
_____________________

Set A

SLR-BB 346

*SLRBB346*
Seat
No.

Set

T.E. (Biomedical Engg.) (Part I) Examination, 2014


BIOLOGICAL MODELING AND SIMULATION (Old)
Day and Date : Friday, 5-12-2014
Time : 3.00 p.m. to 6.00 p.m.

Total Marks : 100

Instructions : 1) Figures to the right indicate full marks.


2) Assume suitable data wherever required.
3) Q. No. 1 is compulsory. It should be solved in first 30
minutes in Answer Book Page No. 3. Each question carries
one mark.
4) Answer MCQ/Objective type questions on Page No. 3 only.
Dont forget to mention, Q.P. Set (A/B/C/D) on Top of Page.
Duration : 30 Minutes

Marks : 20

1. Answer the following questions :

(201=20)

1) Define modeling.
2) Mention Nernst equation.
3) Cable equation defines ________
a) Voltage clamp
c) Membrane voltage

b) Membrane current
d) Ion pump

4) Ficks law states ________ of ions.


a) Diffusion
c) Nernst potential

b) Drift
d) Resistance

5) Define action potential.


6) Express Donnans expression.
7) State Einsteins relationship.
8) Mention the significance of forming a model of a cardio muscular system.
9) Define voltage clamp mechanism.
10) __________ equation is called two ion equilibrium equations.
a) Einstein
b) Nernst
c) Donnan
d) Fick
P.T.O.

SLR-BB 346

*SLRBB346*

-2-

11) Define saccadic eye movement.


12) Agonist and ________ are part of stretch reflex.
a) Antagonist

b) Cell

c) Neuron

d) None of above

13) Define spindle receptor function.


14) Define Golgi tendons function.
15) ________ control mechanism is called as closed loop system.
a) One

b) Three

c) Four

d) Two

16) Space suit is a example of ________ system.


a) Neuromuscular

b) Action potential

c) Thermoregulatory

d) None

17) Define polysynaptic reflex.


18) Explain the significance of reciprocal innervations model.
19) Define smooth pursuit eye movement.
20) Give any 3 symptoms of Parkinsons syndrome.
______________

Set A

*SLRBB346*

-3-

SLR-BB 346

Seat
No.

T.E. (Biomedical Engg.) (Part I) Examination, 2014


BIOLOGICAL MODELING AND SIMULATION (Old)
Day and Date : Friday, 5-12-2014
Time : 3.00 p.m. to 6.00 p.m.

Marks : 80

Instructions : 1) Figures to the right indicate full marks.


2) Assume suitable data wherever required.
SECTION I
2. Attempt any four :
1) Explain the steps of physiological modeling.
2) Explain the significance of ion pumps.
3) What are the assumptions made for Goldmans equation concept ?
4) Draw and explain electrode electrolyte model.
5) Draw and explain micro electrode model in detail.
3. Attempt any two :

(45=20)

(210=20)

1) Derive the expression for Nernst potential of a divalent ion.


2) With the neat diagram explain voltage clamp experiment. What were the
assumptions made for voltage clamp experiment ? Discuss results of the
experiments.
3) State and explain biophysics tools for modeling with their mathematical
expression.
SECTION II
4. Attempt any four :

(45=20)

1) Explain the significance of Golgi tendon and Spindle receptor in neuromuscular


system.
2) Explain different types of eye movements with each example.
3) Write a note on Parkinsons syndrome in detail.
4) Explain any one industrial application of thermoregulatory system in detail.
5) Explain different physiological mechanisms of temperature control.
Set A

SLR-BB 346

-4-

5. Attempt any two :

*SLRBB346*
(210=20)

1) Explain using suitable diagram of plant model of thermoregulatory system.


2) Draw and explain model of drug delivery system.
3) Explain closed loop neuromuscular control system showing anatomical
connections between physiological components that participate in stretch
reflex.
_____________________

Set A

SLR-BB 347

*SLRBB347*
Seat
No.

Set

T.E. (Bio-medical Engg.) (Part I) (Old) Examination, 2014


MICROPROCESSORS AND PERIPHERALS
Day and Date : Monday, 8-12-2014
Time : 3.00 p.m. to 6.00 p.m.
Instructions :

Max. Marks : 100

1) Figures to the right indicate full marks.


2) Assume suitable data if necessary.
3) Q. No. I is compulsory. It should be solved in first 30
minutes in Answer book Page No. 3. Each question carries
one mark.
4) Answer MCQ/Objective type questions on Page No. 3 only. Dont
forget to mention, Q.P. Set (A/B/C/D) on Top of Page.
MCQ/Objective Type Questions

Duration : 30 Minutes
I.

Marks : 20

Choose the correct answer :

(201=20)

1) The __________ type of memory in which the data stored in the form of charge in capacitor.
a) Dynamic RAM
b) Static RAM
c) Both a) and b)
d) None
2) The contents of ___________ memory can be erased by exposing the memory to the
ultra-violet light.
a) EPROM
b) Dynamic RAM c) Static RAM
d) Both a) and b)
3) In 8085 contains _____________ no. of 8 bit register to store data.
a) 6-Register
b) 8
c) 16
d) 10
4) The flag register contains ___________ no. of flags.
a) 5
b) 6
c) 8

d) 9

5) Microprocessor 8085 has ___________ bit data bus and it is ___________ directional.
a) 8 and bidirectional
b) 16 and bidirectional
c) 8 and unidirectional
d) 16 and unidirectional
6) What is the o/p of the following :
MVI A, 55H
STA 2000H
END
a) 55H is stored in the accumulator
c) 55H is stored in Reg. B

b) 55H is stored in memlocation 2000H


d) None

7) The IN-Instruction ____________ byte Instruction.


a) 1-byte
b) 2-byte
c) 3-byte

d) none

8) The Instruction JPO (16 bit) is jump __________ instruction.


a) Jumps instruction to 16 bit address
b) Checks parity and jumps to 16 bit address
c) Jumps on ODD-parity to 16 bit address
d) Jumps on plus to 16 bit address
P.T.O.

SLR-BB 347
9) Which is not a three byte instruction ?
a) LXI
b) JMP

*SLRBB347*

-2-

c) CALL

d) ORI

10) The XRI 8 bit instruction is used for


a) Logically EX-OR with accumulator
b) EX-OR immediate with accumulator
c) Exchange the data
d) EX-OR the second byte with content of accumulator
11) The successive approximation A/D converter includes three elements SAR, DAC
______________
a) Comparator
b) Sampler
c) Integrator
d) Differentiator
12) The p interfacing devices uses the following Handshake signals _________
a) STB
b) 1BF & 0BF
c) ACK
d) All above
13) The 8255A is programmable _________ device.
a) Parallel I/O
b) Series I/O
c) Parallel I / serial O/P
d) Serial I/Parallel O/P
14) The 8255A has ___________ I/O pins.
a) 24
b) 16

c) 8

d) 18

15) The bidirection communication/data transfer between two microcomputers can be


accomplished using 8255 PPI.
a) Mod-0
b) Mod-1
c) Mod-2
d) BSR
16) The IC 8253 can operate with clock frequency DC to ____________
a) 2 MHz
b) 4 MHz
c) 6 MHz
d) None
17) The 8254 includes ___________ identical 16-bit counters.
a) 3
b) 2
c) 4

d) 1

18) The 8254 can operates in _____________ modes.


a) 2
b) 8
c) 4

d) 6

19) The rate at which the bits are transmitted bits/sec is called
a) BAUD
b) Data speed
c) Both a) and b)

d) Simplex mode

20) The TXD means ___________


a) Transmitted data-serial bits are transmitted
b) Transmitted data-parallel bits
c) Both a) and b)
d) None
______________

Set A

*SLRBB347*

-3-

SLR-BB 347

Seat
No.
T.E. (Bio-medical Engg.) (Part I) (Old) Examination, 2014
MICROPROCESSORS AND PERIPHERALS
Day and Date : Monday, 8-12-2014

Marks : 80

Time : 3.00 p.m. to 6.00 p.m.


Instructions :

1) All questions are compulsory.


2) Figures to the right indicate full marks.
3) Assume suitable data if necessary.
SECTION I

II. Attempt any four :

(45=20)

1) Give the classification of memory ? Compare static RAM and dynamic RAM.
2) What are the different types of data transfer operations in p , explain with example.

3) Explain the immediate addressing and register addressing with example of p 8085.
4) Explain the Program Counter (PC) and Stack Pointer (SP).
5) Explain and classify the different types of branch instructions with examples.
III.

Answer any two :

(102=20)

1) What is microprocessor ? Draw the architecture of 8085 ? Explain its various functional
blocks.
2) What is Interrupt ? Explain the different types of maskable and non-maskable interrupts
with examples.
3) Explain the basic concept of memory interfacing.

Set A

SLR-BB 347

-4-

*SLRBB347*

SECTION II
IV. Answer any four :

(45=20)

1) Compare memory mapped I/O and I/O mapped I/O.


2) Draw internal block diagram of 8255 and explain the different functional blocks.
3) Explain the function of control signals, MEMR , MEMW , RD , WR and IO/ M .
4) Write a program to configure 8253 in timer mode-O; mode 2, counting in decimal load
16 bit value.
5) Write short note on logic analyzer.
V. Answer any two :

(102=20)

1) Discuss the mode instruction formal of 8251 for asynchronous data transfer.
2) Draw the Schematic interface of DAC 0808 with 8085 through 8255, write an assembly
language programme to generate square wave.
3) Draw the block diagram of 8251 and explain the different functional blocks.

_____________________

Set A

SLR-BB 348

*SLRBB348*
Seat
No.

Set

T.E. (Biomedical) (Part I) Examination, 2014


PRINCIPLES OF COMMUNICATION (Old)
Day and Date : Wednesday, 10-12-2014
Time : 3.00 p.m. to 6.00 p.m.

Max. Marks : 100

Instructions : 1) Q. No. 1 is compulsory. It should be solved in first 30


minutes in Answer Book Page No. 3. Each question carries
one mark.
2) Answer MCQ/Objective type questions on Page No. 3 only.
Dont forget to mention, Q.P. Set (A/B/C/D) on Top of Page.
MCQ/Objective Type Questions
Duration : 30 Minutes

Marks : 20

1. Choose the correct option :

20

1) A DSB-SC signal can be demodulated using


a) A low pass filter
b) A synchronous filter
c) A phase shifter discriminator
d) An envelope detector
2) The modulation index of an AM wave is changed from 0 to 1. The transmitted
power is
a) Unchanged
b) Halved
c) Increased by 50%
d) Quadrupled
3) An FM signal can be detected by using
a) an LPF
b) a PLL
c) a discriminator
d) an average detector
4) A narrowband noise shows
a) Amplitude modulation only
c) Both AM and FM

b) Frequency modulation only


d) None of these

5) Following is not advantage of FM over AM


a) Noise immunity
b) Fidelity
c) Capture effect
d) Sputtering effect
6) A PAM signal can be detected by using
a) An ADC
b) An integrator
c) A bandpas filter
d) A highpass filter
P.T.O.

SLR-BB 348

*SLRBB348*

-2-

7) The main advantage of TDM over FDM is that it


a) Needs less power
b) Needs less B/W
c) Needs simple ckt
d) Gives better S/N ratio
8) The PWM needs
a) More power than PPM
c) More B/W than PPM

b) More samples per second


d) None of these

9) Flat top sampling leads to


a) An aperture effect
c) Loss of signal

b) Aliasing
d) None of these

10) Which of the following modulation is digital in nature ?


a) PAM
b) PPM
c) DM

d) None of these

11) Quantization noise occurs in


a) PAM
b) PWM

d) None of these

c) DM

12) Which of the following modulation is analog in nature ?


a) PCM
b) DPCM
c) DM

d) None of these

13) In PCM , the quantization noise depends on


a) Sampling rate
b) Number of quantization levels
c) Signal power
d) None of these
14) Which of the following gives maximum probability of error ?
a) ASK
b) FSK
c) PSK
d) DPSK
15) Probability of error in DPSK is
a) Always less than ASK
c) Always greater than PSK

b) Always less than PSK


d) None of these

16) Amplitude shift keying is also known as


a) On-off keying
b) Off-on keying
c) Analog shift keying
d) None of these
17) Which of the following gives minimum probability of error ?
a) ASK
b) PSK
c) FSK
d) None of these
18) _________ is based on binary division.
a) Checksum error detection
b) Cyclic redundancy check (CRC)
c) Parity check
d) None of these
19) If carrier modulated by a digital bit stream had one of the possible phases of
0, 90, 180, 270 degrees, then modulation is called
a) BPSK
b) DPSK
c) QAM
d) MSK
20) In QAM, both identifies are varied
a) Amplitude and phase
b) Frequency and phase
c) Bit rate and phase
d) Boud rate and phase
______________

Set A

*SLRBB348*

-3-

SLR-BB 348

Seat
No.

T.E. (Biomedical) (Part I) Examination, 2014


PRINCIPLES OF COMMUNICATION (Old)
Day and Date : Wednesday, 10-12-2014
Time : 3.00 p.m. to 6.00 p.m.

Marks : 80

SECTION I
2. Solve any four :
a) Differentiate AM, FM and PM.
b) Draw and explain the block schematic of communication system.
c) What is modulation and explain its necessity ?
d) What are different types of noise and what is noise figure ?
e) What is pre-emphasis and de-emphasis ?

(45=20)

3. Solve any two :


(210=20)
a) Explain in brief ground, space and sky wave propagation.
b) A transmitter radiates 10 kW power with the carrier unmodulated, and
10.5 kW when the carrier is modulated by one sinusoidal signal. Calculate
the modulation index. If another modulating signal corresponding to 30%
modulation is transmitted simultaneously, determine the total radiated power.
c) Which methods are used to suppress the unwanted sideband ? Explain any two.
SECTION II
4. Attempt any four :
1) Explain sampling theorem and its types.
2) State difference between TDM and FDM.
3) Explain PAM demodulator in detail.
4) Explain in brief unipolar, bipolar RZ and NRZ terms.
5. Attempt any two :
1) Explain hamming and convolutional codes in detail.
2) Explain PTM modulation for PTM signal by using direct and indirect
method.
3) Explain source coding. Also explain types of quantisation in detail.
_____________________

(45=20)

(210=20)

Set A

Set A

SLR-BB 349

*SLRBB349*
Seat
No.

Set

T.E. (Part I) (Biomedical Engg.) Examination, 2014


SIGNALS AND SYSTEMS (Old)
Day and Date : Friday, 12-12-2014
Time : 3.00 p.m. to 6.00 p.m.
Instructions :

Max. Marks : 100

1) Figures to the right Question indicates full marks.


2) Assume data where ever necessary.
3) Q. No. I is compulsory. It should be solved in first 30 minutes in Answer
Book Page No. 3. Each question carries one mark.
4) Answer MCQ/Objective type questions on Page No. 3 only. Dont
forget to mention, Q.P. Set (A/B/C/D) on Top of Page.
MCQ/Objective Type Questions

Duration : 30 Minutes
I.

Marks : 20

Choose the correct answer :

(120=20)

1) Convolution Integral can be applied to signal of


a) Continuous time
b) Discrete times
c) Digital type
d) All
2) Condition for Nyquist rate is
a) Fs Fm
b) Fs > 2Fm

c) Fs 2Fm

d) Fs = 2Fm

3) A signal having period of 12 Sec. will have angular frequency.


a)

b) 2

c) 2

d) 6

4) The inverse Z-transform of X(Z) is


a) X(n) =

c) x(n) =

1
2
1
2 J

CX( Z )Z

X( Z)Z

n 1

dZ

n 1

dZ

b) x(n) =
d) x(n) =

2 J
1
2

CX( Z )Z dZ

X( Z)Z

5) Bilateral Z-transformation can be described with


b) > n
c) < n <
a) n >

n+ 1

dz

d) None

6) The Fourier transform of (n)


a) 1

b) e J

7) If X(n) = (n) its ROC is


a) |Z| < 1
b) |Z| > 1

c) 1e

d) e J

c) Entire Z plane

d) |Z| = 1

P.T.O.

SLR-BB 349

*SLRBB349*

-2-

8) x(n) = {3, 2, 1, 4, 1} then ROC of X(z) is


a) Entire Z plane
b) Entire Z plane except at Z = 0 and Z =
c) Entire Z plane except at Z = 0
d) Entire Z plane except at Z =
9) The convolution in time domain is multiplication in
a) Z domain
b) S domain
c) Z plane
d) Frequency domain
10) If a system is giving bounded output for bounded input system will be
a) Causal
b) Stable
c) None causal
d) Periodic
11) If x(t) is time shifted with x(t + t0) the signal will shift towards.
a) Right side
b) Left side
c) Upward

d) Downward

12) Convolution sum can be applied to signal of


a) Continuous time b) Discrete time
c) Digital type

d) All

13) If the total energy of signal satisfies the condition 0 < E < is called
a) Power signal
b) Energy signal
c) Primary signal
d) None
14) x(t) = x (t) is a condition necessary for a signal to be
a) Periodic
b) Linear
c) Even

d) Odd

15) Assymetrical signal is called as


a) Odd signal
b) Even signal

c) Stable signal

d) unstable signal

c) Static

d) Causal

16) The system Y(t) =


a) DT dynamic

d
x( t) is
dt
b) CT dynamic

17) Which of the following system is non causal


n

x(K)

a) Y(n) = x(n) = x(n 1)

b) Y(n) =

c) Y(n) = a X (n)

d) Y(n) = x (n)

K =

18) Nyquist sampling rate for x(J ) = Sin c(x)


a) 120 Hz
b) 200 Hz
c) 100 Hz

d) 50 Hz

19) A signal having period of 4 seconds will have angular frequency


a)

b) 2

c) 2

d) 4

20) The term an in Fourier series expansion gives amplitude of


a) Sine components
b) Cosine components
c) D.C. components
d) None
______________

Set A

*SLRBB349*

-3-

SLR-BB 349

Seat
No.
T.E. (Part I) (Biomedical Engg.) Examination, 2014
SIGNALS AND SYSTEMS (Old)
Day and Date : Friday, 12-12-2014
Time : 3.00 p.m. to 6.00 p.m.
Instructions :

Marks : 80

1) Figures to the right Question indicates full marks.


2) Assume data whereever necessary.
SECTION I

II. Answer any four questions :


1) Define a signal ? What are the types of signal ? Explain any two.
2) Explain Aliasing Effect. What is the effect of under sampling.
3) What do you mean by periodic and Aperiodic signal ? Explain with example.
4) For the signal x(t) shown in figure. Find the signals :
a) x(t 2)
b) x (2t + 3).

(54=20)

5) Obtain the convolution sum of D.T. signal if


x(n) = 1

for 0 n 2

= 1

for 3 n 5

=0

elsewhere

y(n) = 1 for 0 n 2
=0

elsewhere

Also sketch the result.


III.

Answer any two question :

(102=20)

1) Verify the general properties of the system by Y(n) x(n). u(n).


2) The impulse response of CT LTI system is h(t) = e2t u(t 1)
Determine causality and stability of system.
3) Define Sampling theorem and sampling Intervals. Explain the method to reconstruct the
signals from its sampling by using interpolation.
Set A

SLR-BB 349

-4-

*SLRBB349*

SECTION II
IV. Answer any four questions :

(54=20)

1) Define Z-transform. State and explain the linearity and time shifting properties.
2) Find the Fourier transform of
a) Unit step
b) Impulse function.
3) Determine the Z-transform ROC and pole zero location of x(Z) for
x(n) =

( ) ( )

2 n + 1 n un
3
2

4) Find the Fourier transform of


x(t) = e(t)

1 t 1

for

=0

otherwise

5) Find the inverse of Z-transform of given signal


x(z) =

z +1
z 3z + 2
2

V. Answer any two questions :

(102=20)

1) Find the cosine Fourier series of an half wave rectified sine function.

2) Find the Fourier transform of given rectangular pulsar signal in fig. draw the amplitude
and phase spectrum.

( T)

3) Prove that convolution of two sequence in time domain is equivalent to multiplication of


same sequence in transformed Z-domain.
_____________________

Set A

SLR-BB 35

*SLRBB35*
Seat
No.

Set

S.E. (Civil) (Part II) (New) Examination, 2014


WATER RESOURCES ENGINEERING I
Day and Date : Saturday, 29-11-2014
Time : 3.00 p.m. to 6.00 p.m.

Max. Marks : 100

Instructions : 1) Attempt any three questions from each Section.


2) Assume suitable data if necessary and state it clearly.
3) Q. No. 1 is compulsory. It should be solved in first 30 minutes in
Answer Book Page No. 3. Each question carries one mark.
4) Answer MCQ/Objective type questions on Page No. 3 only. Dont
forget to mention, Q.P. Set (A/B/C/D) on Top of Page.
Duration : 30 Minutes

MCQ/Objective Type Questions

1. Choose the correct alternative :

Marks : 20
(120=20)

1) An isohyet is line joining points having


a) Equal evaporation value
b) Equal barometric pressure
c) Equal height above the M.S.L.
d) Equal rainfall depth in a given duration
2) In a cyclones of the northern hemisphere, wind blows
a) Clockwise inward
b) Anticlockwise inward
c) Clockwise outward
d) Anticlockwise outward
3) The convective precipitation is due to
a) Vertical instability of moist air produced by surface heating
b) A disturbance on air front develops into cyclone
c) The colder air mass forms a wedge and lifts the warm air mass
d) None of these
4) Rainfall mass curve is variation of
a) Rainfall intensity with time
b) Rainfall intensity with cumulative rainfall
c) Rainfall excess with time
d) Cumulative rainfall with time
5) The most accurate method of finding the average depth of rainfall over area is
a) Isohytel method
b) Arithmetic mean method
c) Thiessen polygon method
d) None of these
6) A unit hydrograph has one unit of
a) Peak discharge
b) Rainfall duration
c) Direct runoff
d) The time base of direct runoff
7) The recession limb of a flood hydrograph can be expressed with positive values of
coefficients as Qr/Qo =
a) kc at
b) aktat
c) aat
d) eat2
8) Radius of influence is the horizontal distance between centre of pumped well and
a) Point on cone of depression of maximum drawdown
b) Point on cone of depression of zero drawdown
c) The first observation well
d) Second observation well
P.T.O.

SLR-BB 35

*SLRBB35*

-2-

9) Permeability of aquifer
a) Increases with increase in temperature b) Increases with decrease in temperature
c) Independent of temperature
d) Decreases with decrease in temperature
10) The dimensions of the transmissibility of aquifer T are
b) L2 T1
c) L2 T2
a) LT1

d) L2 T2

11) The water requirement in terms of delta is maximum for


a) Rice
b) Tobacco
c) Potatoes

d) Sugar cane

12) As per National water policy of India, the government gives top priority to
a) Drinking water
b) Hydropower
c) Irrigation water
d) None of the above
13) Trap efficiency of a storage reservoir is defined as
Total annual sediment inflow
a)
Reservoir capacity
Total sediments deposited in a given period
b)
Total sediment inflow in that period
Total annual sediments deposited in the reservoir
c)
Dead storage capacity of the reservoir
d) None of the above
14) Flow duration curve is a plot of
a) Accumulated flow against time
b) Discharge against time in chronological order
c) The base flow against the percentage of times the flow is exceeded
d) The stream discharge against the percentage of times the flow is equalled or exceeded
15) Unit hydrograph theory is suitable for catchments having areas upto
a) 1000 sqkm
b) 2500 sqkm
c) 5000 sqkm
d) 10000 sqkm
16) Which of the following pairs is not correctly matched ?
a) Waterloss Evaporation
b) Runoff Stream flow
c) Percolation Soil erosion
d) Storm Precipitation
17) Streams intersecting the water table and receiving flow from the ground water are known as
a) Influent stream
b) Effluent streams
c) Ephemeral streams
d) Intermittent streams
18) Which of the following statements about design period with respect to water resources
project are true ?
i) It is concerned with economy of investments.
ii) It takes into account, aspects like life, durability and ability for capacity enhancement.
iii) It is concerned with estimating future requirements.
iv) It considers the frequencies of extreme river flow.
a) (i) and (ii) are true
b) (ii) and (iii) are true
c) (i) and (iii) are true
d) None of the above
19) The discharge per unit drawdown for a well is known as
a) Specific capacity b) Safe yield
c) Specific stage

d) None of the above

20) According to Ryves formula for estimating floods, the peak discharge is proportional to
a) A
b) A1/2
c) A2/3
d) A3/4
______________

Set A

*SLRBB35*

SLR-BB 35

-3-

Seat
No.

S.E. (Civil) (Part II) (New) Examination, 2014


WATER RESOURCES ENGINEERING I
Day and Date : Saturday, 29-11-2014
Time : 3.00 p.m. to 6.00 p.m.

Marks : 80

Instructions : 1) Attempt any three questions from each Section.


2) Assume suitable data if necessary and state it clearly.
SECTION I
2. a) What are the different types of rain-gauges used for measurement of precipitation ? Explain
any one of them with a neat sketch.

b) Thiessen polygons constructed for a network of ten rain gauges in a river basin yielded
Thiessen weights of 0.1, 0.16, 0.12, 0.11, 0.09, 0.08, 0.07, 0.11, 0.06 and 0.1. If the
rainfalls recorded at these gauges during a cyclonic storm are 132, 114, 162, 138, 207,
156, 135, 158, 168 and 150 mm respectively. Determine the average depth of rainfall by
Thiessen mean method and Arithmetic mean method. Also determine the volume of
surface runoff at the basin outlet if 35% of the rainfall is lost as infiltration. Take area of
the basin as 580 km2.

3. a) Explain the concept of Unit Hydrograph. State the assumptions and limitations of Unit
Hydrograph theory.

b) Given the ordinates of a 4 hour unit hydrograph as below, derive the ordinates of a
12 hour unit hydrograph for the same catchment.
Time (hr)

12

16

Ordinates of 4-hour unit


hydrograph (m3/sec)

20

75 125 150

20

24 28

32 36 40 44

130 80 55 25 15

4. a) Enlist different methods of stream gauging and explain briefly the area velocity method of
stream gauging.
b) Compute the stream flow for the measurement data given below :
Distance
0
0.6 1.2 1.8 2.4
3
3.6 4.2 4.8 5.4
6
6.6
(m)
Depth
0
0.3 1.29 2.16 2.55 2.22 1.68 1.41 1.05 0.63 0.42 0
(m)
Velocity
at 0.2 d
0 0.42 0.57 0.78 0.87 0.81 0.75 0.69 0.63 0.54 0.45 0
(m/s)
Velocity
at 0.8 d
0 0.21 0.36 0.54 0.6 0.3 0.51 0.44 0.39 0.33 0.3
0
(m/s)

6
7

Set A

SLR-BB 35

-4-

*SLRBB35*

5. a) Enlist, classify and discuss in brief, geological formations where ground water occurs.

b) A well with radius of 0.25 m, completely penetrates into confined aquifer of thickness 20 m
and hydraulic conductivity 8.2 104 m/s. What is the maximum yield expected from this
well, if the drawdown in the well is not to exceed 3 m. The radius of influence may be taken
as 260 m.

SECTION II
6. a) Define irrigation and explain its necessity in a tropical country like India. What are the
advantages and ill effects of assured irrigation ?

b) Enlist the proposed inter basin water transfer links proposed for Himalayan Rivers,
Development component of National perspective plan. State the advantages of Himalayan
component.

7. a) The following data pertains to the healthy growth of a crop


i) Field capacity of soil = 30%
ii) Permanent wilting point = 11%
iii) Density of soil = 1300 kg/m3
iv) Effective depth of root zone = 700 mm
v) Daily consumptive use of water = 12 mm
For healthy growth, moisture content must not fall below 25% of the water holding capacity
and the permanent wilting point. Determine the watering interval in days.

b) Describe with the help of a diagram, various forms of a soil moisture. What do you
understand by the term available moisture ?

8. a) Discuss the various soil types observed in India. Discuss suitability of each soil type for
crops and irrigation.

b) What do you understand by contour forming ? Compare it with the wild flooding method of
irrigation.

9. a) List the various soil conservation measured adopted for watershed erosion control.
b) Write a note on role of formers cooperatives in irrigation water management.

6
8

_____________________

Set A

SLR-BB 350

*SLRBB350*
Seat
No.

Set

T.E. (Part I) (Biomedical Engg.) (New) Examination, 2014


BIOMEDICAL INSTRUMENTATION I
Day and Date : Wednesday, 3-12-2014
Time : 3.00 p.m. to 6.00 p.m.

Max. Marks : 100

Instructions : 1) Figures to the right indicate full marks.


2) Use legible handwriting, use blue/black only.
3) Q. No. 1 is compulsory. It should be solved in first 30 minutes in
Answer Book Page No. 3. Each question carries one mark.
4) Answer MCQ/Objective type questions on Page No. 3 only.
Dont forget to mention, Q.P. Set (A/B/C/D) on Top of Page.
MCQ/Objective Type Questions
Duration : 30 Minutes

Marks : 20

I. Choose the correct answer :

(201=20)

1) According to Beer-Lambert Law P =


a) Po 10 bc

b) bc 10

Po

c) Po 10 bc

d) bc 10Po

2) In multichannel colorimeter ____________ channel serve as reference.


a) 1 st
b) 9th
c) 15th
d) 25th
3) Relative Centrifugal Force RCF =

rw 2
a)
g

w2
b)
rg

rw
c) 2
g

r 2w
d)
g

4) The inert carrier gas is used as _____________ phase of Gas-Liquid


Chromatograph.
a) Stationary
b) Mobile
c) Neutral
d) None
5) In pH meter ___________ electrode is used as reference electrode.
a) Silver-Silver Chloride Electrode
b) Calomel Electrode
c) Both a) and b)
d) None
6) The percentage of cells in the blood is called ____________ volume.
a) Mean cell
b) Packed cell
c) Mean Platelet d) None
7) In pCO2 electrode _____________ membrane is permeable to CO2.
a) Nylon
b) Plastic
c) Teflon
d) None of the above
8) Increase in the concentration of hydrogen ions in blood makes solution more
______________
a) Acidic
b) Alkaline
c) Neutral
d) No change
P.T.O.

SLR-BB 350

-2-

*SLRBB350*

9) The magnitude of voltage produced across Electromagnetic flow meter is e = CHVd;


where d is ______________
a) Density of blood
b) Diameter of Blood Vessel
c) Distance
d) None
10) The Ultrasonic Blood Flow Meter is based on _____________ principle.
a) Change is magnetic field strength
b) Change is amplitude of signal
c) Doppler frequency shift
d) All of the above
11) Fox and Wood suggested ___________ Dye substance used for Dye Dilution
Method.
a) Indocyanine Green
b) Indocyanine Blue
c) Indocyanine Red
d) Indocyanine Yellow
12) The output across differential manometer is measured using __________ law.
a) Ohms Law
b) Poiseuille Law
c) Beer-Lambert law
d) None
13) In infrared gas analyzer the chopper rotates at speed of ___________ rpm.
a) 500
b) 1000
c) 2000
d) 3000
14) At 805 nm wavelength, the molecular extinction coefficient for HbO2 and Hb are
_______________
a) 1.0 m
b) 1.5 m
c) 2.0 m
d) 2.5 m
15) A __________ m long flexible fiber is used for ear oximeter.
a) 1.0 m
b) 1.5 m
c) 375 kPa
d) 300 kPa
16) The maximum pressure present at gas cylinder of anesthesia machine is
____________
a) 200 kPa
b) 275 kPa
c) 375 kPa
d) 300 kPa
17) The inspiratory flow is represent in graph as ___________
a) Positive flow
b) Negative flow
c) Both Positive and Negative Flow
d) None
18) Using Heart Lung Machine without hyperthermia a person time is limit of operation
is up to ___________ minutes.
a) 1 minute
b) 5 minutes
c) 10 minutes
d) 15 minutes
19) The heart pumps about ____________ liters of blood per minute.
a) 2 liters
b) 3 liters
c) 4 liters
d) 5 liters
20) The audio response recorded using audiometer is drawn on ___________ graph.
a) Normal Graph
b) Semi-log graph c) Both
d) None
______________

Set A

*SLRBB350*

-3-

SLR-BB 350

Seat
No.

T.E. (Part I) (Biomedical Engg.) (New) Examination, 2014


BIOMEDICAL INSTRUMENTATION I
Day and Date : Wednesday, 3-12-2014

Marks : 80

Time : 3.00 p.m. to 6.00 p.m.


Instructions : 1) Figures to the right indicate full marks.
2) Draw diagrams/sketches wherever necessary.
3) Use legible handwriting, use blue/black only.
SECTION I

II. Solve any four :

(45=20)

1) With neat figure explain chromatography briefly.


2) Explain principle and working of coulter counters.
3) Briefly explain principle of electrophoresis.
4) Define blood pH. Draw constructional diagram of micro-capillary blood
pH electrode.
5) Briefly explain thermal dilution techniques.

III. Solve any two :

(210=20)

1) Briefly explain measurement of blood pCO2 with labeled figure ? Give calculations
for bicarbonate, Total CO2 and base excess.
2) Draw the circuit diagram of bicarbonate (HCO3), Total CO2 and base excess ?
Briefly explain.
3) a) Draw and explain working principle of electromagnetic blood flow meter.
b) Explain LASER Doppler blood flow meter with neat diagram.

Set A

SLR-BB 350

-4-

*SLRBB350*

SECTION II

IV. Solve any four :

(45=20)

1) Define :
a) Tidal Volume (TV)
b) Vital Capacity (VC)
c) Inspiratory Reserve Volume (IRV)
d) Total Lung Capacity (TLC)
e) Minute Volume (MV).

2) Classify Pneumotachometers. Briefly explain Fleish Pneumotachometer.


3) List types of respiratory Gas Analyzers ? Briefly explain Infrared Gas
Analysers.
4) Write a short note on mechanical functions of heart.
5) List differences between pure tone and speech audiometer.
V. Solve any two :

(210=20)

1) Briefly explain Pulse Oximeter with the help of neat labeled diagram.
2) Briefly explain classification of ventilators.
3) Explain the principle and working of heart lung machine with neat figure.
_____________________

Set A

SLR-BB 351

*SLRBB351*
Seat
No.

Set

T.E. (Part I) (Biomedical) (New) Examination, 2014


BIOLOGICAL MODELING AND SIMULATION
Day and Date : Friday, 5-12-2014
Time : 3.00 p.m. to 6.00 p.m.

Max. Marks : 100

Instructions : 1)
2)
3)
4)

Figures to the right indicate full marks.


Assume data wherever necessary.
Use legible handwriting, use blue/black only.
Q. No. 1 is compulsory. It should be solved in first 30 minutes in
Answer book Page No. 3. Each question carries one mark.
5) Answer MCQ/Objective type questions on Page No. 3 only.
Dont forget to mention, Q.P. Set (A/B/C/D) on Top of Page.
MCQ/Objective Type Questions

Duration : 30 Minutes

Marks : 20

1. Attempt all :

(120=20)

1) The modeling which can be represented with the help of graphs is called as
___________
a) Graphical modeling
b) Pictorial modeling
c) Mathematical modeling
d) None
2) The modeling which can represent distributed model is known as __________
a) Compartmental modeling
b) Distributed parameter modeling
c) Lumped parameter modeling
d) None
3) In _________, each compartment depends on the relation with the compartment
present either and before.
a) Parallel model
b) Reducible model
c) Circulatory model
d) None
4) Distributed parameter is used in ___________
a) High frequency
c) Both low frequency and high frequency

b) Low frequency
d) None

5) Example of distributed parameter is ____________


a) Rigid bodies
b) Transmission line
c) Both rigid bodies and transmission line
d) None
6) ___________ states that drift of positively charged particles takes place down the
electric potential gradient and is everywhere directly proportional to the magnitude
of that gradient.
a) Ohms law
b) Ficks law
c) Einstein relationship
d) None
P.T.O.

SLR-BB 351

*SLRBB351*

-2-

7) If we apply external or internal stimulus or signal to a membrane then membrane


gets imbalanced called __________
a) Active transport
b) Action potential
c) Donnan equilibrium
d) None
d[G]
is known as __________
dx
b) Space charge neutrality
c) Ficks law

8) The formula J0 (x ) = D 0
a) Ohms law
9) The formula

C + = A

d) None

shows _________

a) Ohms law
c) Space charge neutrality

b) Ficks law
d) None

10) ___________ is the relation between diffusion and drift process.


a) Ohms law
b) Einstein relationship
c) Ficks law
d) None
11) Controlling of load dynamics with the muscular action by using neural network is
called as _____________
a) Neuro muscular control system
b) Stretch reflex
c) Golgi tendon organ
d) None
12) ___________ is a central nervous system disease affecting elderly persons.
a) Stretch reflex
b) Parkinsons syndrome
c) Open loop control strategy
d) None
13) __________ are used when looking between near and distant objects.
a) Saccadic eye movement
b) Vestibular ocular movements
c) Vergence eye movements
d) None
14) __________ are used when tracking moving objects.
a) Smooth pursuit eye movements
b) Saccadic eye movements
c) Vestibular ocular movements
d) None
15) Exercise produces __________
a) 100 W
b) 10000 W

c) 1000 W

d) None

16) Shivering produces ___________


a) 100 W
b) 10000 W

c) 1000 W

d) None

17) Lymphoid organ is known as ____________


a) RBC
b) WBC
c) Lymph

d) None

18) T cell mature in __________


a) Thymus
b) Plasma

d) None

c) Lymp node

19) _________ are simplified representation of objects and system.


a) Modeling
b) Simulation
c) Models
d) None
20) __________ means constructing a model that explains a physical system.
a) Modeling
b) Simulation
c) Models
d) None
______________
Set A

*SLRBB351*

-3-

SLR-BB 351

Seat
No.

T.E. (Part I) (Biomedical) (New) Examination, 2014


BIOLOGICAL MODELING AND SIMULATION
Day and Date : Friday, 5-12-2014

Marks : 80

Time : 3.00 p.m. to 6.00 p.m.


Instructions : 1) Figures to the right indicates full marks.
2) Assume data wherever necessary.
3) Draw diagrams/sketches wherever necessary.
4) Use legible handwriting, use blue/black only.
SECTION I
2. Attempt any four :

(45=20)

1) Differentiate between lumped parameter and distributed parameter models.


2) Explain in brief mathematical modeling of valve or Wind Kesel model.
3) Explain in brief Drift of charged particles in aqueous solutions.
4) Derive Donnan equilibrium.
5) Explain in brief voltage clamp.
3. Attempt any two :

(210=20)

1) Derive the Nernst equation.


2) What is active transport ? Explain ion pump with the help of neat figure
3) Draw electrical equivalent model of biological membrane and explain it.

Set A

SLR-BB 351

-4-

*SLRBB351*

SECTION II
4. Attempt any four :

(45=20)

1) Explain in brief experimental validation of the model with respect to neuromuscular


system.
2) Explain in brief controller model for thermoregulation.
3) Explain in brief behaviour of the immune system.
4) Differentiate between one control and two control mechanism of neuromuscular
system.
5) Explain in brief :
i) model
ii) modeling
iii) simulation
iv) stretch reflex.
5. Attempt any two :

(210=20)

1) Explain in detail Golgi Tendon Organ with neat block diagram.


2) Explain thermoregulation system with neat block diagram.
3) Explain in detail :
i) Validity criteria for eye movement model
ii) Parkinson syndrome
_____________________

Set A

SLR-BB 352

*SLRBB352*
Seat
No.

Set

T.E. (Biomedical) (Part I) Examination, 2014


MICROPROCESSOR AND MICROCONTROLLER (New)
Day and Date : Monday, 8-12-2014
Time : 3.00 p.m. to 6.00 p.m.

Max. Marks : 100

Instructions : 1) Q. No. I is compulsory. It should be solved in first 30


minutes in Answer Book Page No. 3. Each question
carries one mark.
2) Answer MCQ/Objective type questions on Page No. 3
only. Dont forget to mention, Q.P. Set (A/B/C/D) on
Top of Page.
MCQ/Objective Type Questions
Duration : 30 Minutes

Marks : 20

I. Choose the right answer :

20

1) Intel 8085 is a _______________ bits microprocessor.


a) 4-bits
b) 8-bits
c) 16-bits

d) 32-bits

2) The number of status of flags in 8085 are


a) 5
b) 6
c) 8
d) 7
3) Microprocessor speed depends on
a) Clock frequency
b) Size of register
c) Address bus width
d) None
4) The status of S0 and S1 pins for memory read is _______________
a) 0, 0
b) 0,1
c) 1, 0
d) 1,1
5) Which of the following is a one byte instruction ?
a) MVI B, 05
b) LDA 2589 H c) IN 02
d) RAR
6) How many transistors are required to store a bit in SRAM ?
a) 3
b) 4
c) 5
d) 6
7) What are level triggered interrupts ?
a) RST 6.5 and RST 5.5
b) RST 7.5 and RST 5.5
c) RST 6.5 and RST 7.5
d) INTR and RST 7.5
8) What is SIM ?
a) Select interrupt mask
c) Set interrupt mask

b) Sorting interrupt mask


d) Softer interrupt mask
P.T.O.

SLR-BB 352

-2-

*SLRBB352*

9) The address bus flow is _______________


a) Bidirectional
b) Unidirectional
c) Multidirectional
d) Circular
10) Execution of two or more programms by single processor is known as
_______________
a) Multiprocessing
b) Time sharing
c) Multiprogramming
d) None
11) The length of stack pointer is _______________
a) 8 bits
b) 16 bits
c) 32 bits

d) 64 bits

12) The advantage of memory mapped i/o over i/o mapped i/o is _______________
a) Faster
b) Many instructions support memory mapped i/o
c) Require a bigger address decoder
d) All of the above
13) The instruction used for storing the contents of H and L registers in to
2050 H and 2051 H respectively is _______________
a) SPHL 2050 H
b) SPHL 2051 H
c) SHLD 2050 H
d) STAX 2050 H
14) The non-maskable interrupt is the _______________
a) TRAP
b) RST 5.5
c) RST 6.5

d) INTR

15) The instruction MOV A, 81 H uses _______________ addressing mode.


a) Immediate
b) Register
c) Direct
d) None of these
16) Which of the following register is bit addressable ?
a) SBUF
b) PCON
c) PSW

d) None of these

17) Which of the 8051 I/O port is not used for dual purpose ?
a) Port 1
b) Port 2
c) Port 3
d) None of these
18) In 8051 which interrupt has highest priority ?
a) IE 1
b) TF 0
c) IE0

d) TF1

19) What is the address of PSW special function register ?


a) 88
b) 0A
c) 0B

d) D0

20) Which timer mode is the auto reload timer ?


a) Timer mode 0
b) Timer mode 1
c) Timer mode 2
d) Timer mode 3
______________
Set A

*SLRBB352*

-3-

SLR-BB 352

Seat
No.

T.E. (Biomedical) (Part I) Examination, 2014


MICROPROCESSOR AND MICROCONTROLLER (New)
Day and Date : Monday, 8-12-2014
Time : 3.00 p.m. to 6.00 p.m.

Marks : 80

SECTION I
II. Attempt any four :

(45=20)

1) Compare SRAM and DRAM.


2) Explain the following instructions :
i) DAA
ii) CMP M
iii) RAL
iv) MOV A, M
v) INX H.
3) Draw a timing diagram for STA 8500 H.
4) Define interrupt. Give an interrupt priority table and vector address.
5) Explain different addressing modes in 8085.
III. Attempt any two :

(210=20)

1) Draw the architecture of 8085 and explain it various functional blocks.


2) Compare the following :
a) Software and Hardware interrupt
b) Maskable and Non-Maskable interrupt.
3) Write an assembly language program to sort given 8 bytes of data in
ascending order.

Set A

SLR-BB 352

-4-

*SLRBB352*

SECTION II
IV. Attempt any four :

(45=20)

1) Explain IN and OUT peripheral I/O instructions.


2) Explain with neat diagram timer mode 1 operation.
3) Explain Serial Port Control (SCON) special function register.
4) List out difference between microcontroller and microprocessor.
5) Write a microcontroller program to multiply two 8-bit numbers and store
16-bit result in 8050 and 8051.
V. Attempt any two :

(210=20)

1) Compare peripheral I/O and memory mapped I/O.


2) Draw programming model of 8051.
3) With a neat diagram explain how matrix keyboard of size (4 * 4) interfaced
with microcontroller.
_____________________

Set A

SLR-BB 353

*SLRBB353*
Seat
No.

Set

T.E. (Biomedical Engg.) (Part I) Examination, 2014


PRINCIPLES OF COMMUNICATION (New)
Day and Date : Wednesday, 10-12-2014
Time : 3.00 p.m. to 6.00 p.m.

Max. Marks : 100

Instructions : 1) Q. No. 1 is compulsory. It should be solved in first 30 minutes


in Answer Book Page No. 3. Each question carries one mark.
2) Answer MCQ/Objective type questions on Page No. 3 only.
Dont forget to mention, Q.P. Set (A/B/C/D) on Top of Page.
MCQ/Objective Type Questions
Duration : 30 Minutes
1. Choose the correct answer :
1) An FM signal can be detected by using
a) an LPF
b) a PPL
c) a discriminator
d) an average detector

Marks : 20
(201=20)

2) A narrowband noise shows


a) amplitude modulation only
b) frequency modulation only
c) both AM and FM
d) none of these
3) Following is not advantage of FM over AM
a) noise immunity
b) fidelity
c) capture effect
d) sputtering effect
4) A DSB-SC signal can be demodulated using
a) a low pass filter
b) a synchronous detector
c) a phase shift discriminator
d) an envelop detector
5) The modulation index of an AM wave is changed from 0 to 1. The transmitted
power is
a) unchanged
b) halved
c) increased by 50%
d) quadrupled
6) A PAM signal can be detected by using
a) an ADC
b) an integrator
c) a band pass filter
d) a high pass filter
7) The main advantage of TDM over FDM is that it
a) needs less power
b) needs less B/W
c) needs simple circuit
d) gives better S/N ration
P.T.O.

SLR-BB 353

*SLRBB353*

-2-

8) The PWM needs


a) more power than PPM
c) more B/W than PPM

b) more samples per second


d) none of these

9) Flat top sampling leads to


a) an aperture effect
c) loss of signal

b) aliasing
d) none of these

10) Which of the following modulation is digital in nature ?


a) PAM
b) PPM
c) DM
11) Quantization noise occurs in
a) PAM
b) PWM
c) DM
12) Which of the following modulation is analog in nature ?
a) PCM
b) DPCM
c) DM

d) None of these
d) None of these
d) None of these

13) In PCM, quantization noise depends on


a) sampling rate
b) number of quantization levels
c) signal power
d) none of these
14) Which of the following gives maximum probability of error ?
a) ASK
b) FSK
c) PSK
d) DPSK
15) Probability of error in DPSK is
a) Always less than ASK
c) Always greater than PSK

b) Always less than PSK


d) None of these

16) Amplitude shift keying is also known as


a) ON-OFF keying
b) OFF-ON keying
c) Analog shift keying
d) None of these
17) Which of the following gives minimum probability of error ?
a) ASK
b) PSK
c) FSK
d) None of these
18) ________________ is based on binary division.
a) Check sum error detection
b) Cyclic redundancy check
c) Parity check
d) None of these
19) If carrier modulated by a digital bit stream had one of the possible phases of
0, 90, 180, 270 degrees, then modulation is called
a) BPSK
b) DPSK
c) QAM
d) MSK
20) In QAM, both identities are varied
a) amplitude and phase
b) frequency and phase
c) bit rate and phase
d) baud rate and phase
______________

Set A

*SLRBB353*

-3-

SLR-BB 353

Seat
No.

T.E. (Biomedical Engg.) (Part I) Examination, 2014


PRINCIPLES OF COMMUNICATION (New)
Day and Date : Wednesday, 10-12-2014

Marks : 80

Time : 3.00 p.m. to 6.00 p.m.


SECTION I
2. Solve any four :

(45=20)

a) Draw and explain the block schematic of communication system.


b) What is modulation and explain its necessity.
c) What are different types of noise and what is noise figure ?
d) What is pre emphasis and de-emphasis ?
e) Differentiate AM, FM and PM.
3. Solve any two :

(210=20)

a) Explain in brief ground, space and sky wave propagation.


b) Which methods are used to suppress the unwanted side bands ? Explain any
two.
c) A transmitter radiates 10 kw power with the carrier unmodulated and 10.5 kw
when the carrier is modulated by one sinusoidal signal. Calculate the
modulation index. If another modulating signal corresponding to 30% modulation
is transmitted simultaneously, determine the total radiated power.

Set A

SLR-BB 353

-4-

*SLRBB353*

SECTION II
4. Solve any four :

(45=20)

a) Compare ASK, FSK and PSK.


b) Explain Uniform and Non-uniform quantization.
c) Explain the generation of PTM signals.
d) Compare FDM and TDM.
e) Explain natural and flat top sampling.
5. Write a note on (Solve any four) :

(45=20)

a) QAM
b) PCM
c) DM
d) PWM
e) DPSK.
_____________________

Set A

SLR-BB 354

*SLRBB354*
S

T. E. (Bio-Medical) (Part I) Examination, 2014


SIGNAL AND SYSTEM (New)
Day and Date : Friday, 12-12-2014
Time : 3.00 p.m. to 6.00 p.m.

Max. Marks : 100

Instructions : 1) Figures to right indicate full marks.


2) Assume suitable data if necessary.
3) Q. No. 1 is compulsory. It should be solved in first 30 minutes in
Answer Book Page No. 3. Each question carries one mark.
4) Answer MCQ/Objective type questions on Page No. 3 only. Dont
forget to mention, Q.P. Set (A/B/C/D) on Top of Page.
MCQ/Objective Type Questions

Duration : 30 Minutes

1. Choose the correct option.


1) Which of the following signal is periodic ?
a) s(t) = cos 2t + cos 3t + cos 5t
c) s(t) = cos 2t cost u(t)

Marks : 20
20

b) s(t) = e7t sin 10 t


d) None of above


2) Represent following signal shown in figure A as sum of singularity function

Fig. A
a) r(t) + 2r(t) u(t 1)
b) r(t + 1) + r (t 1) + 2u (t 2) r (t 2)
c) r (t + 1) u (t) + 2r (t 1) r (t 2) u(t 2)
d) r (t + 1) u(t) + r(t 1) r (t 2) u(t 2)
3) If x1(t) = 2 sin t + cos 4 t and x2 (t) = sin 5 t + 3 sin 3 t then
a) x1 and x2 both are periodic
b) x1 and x2 both are A periodic
c) x1 is periodic but x is not
d) x1 is not periodic, but x2 is periodic
4) The system y(n + 2) + y(n + 1) = x(n + 2) is _________
a) causal and memory less
b) causal and has memory
c) is causal
d) is non causal


5) For the system shown below which of system are linear


a)
@

+ y2(t) = f(t)
O

b)
@

+ 2y(t) = f(t)
O

c)
@

+ 2y(t) = f(t)
O

@
@

6) The system y(t) = ex(t) is __________


a) stable, causal
b) noncausal, stable

d) None of above

c) unstable, causal

d) unstable, noncausal

7) Which of the following is response y(t) of causal LTI system describe by


H(s) =
I

for given input x(t) = et u(t) ?




a) y(t) = et sin t u(t)


c) y(t) = sin (t 1) u (t 1)

b) y(t) = e(t 1) sin (t 1) u(t 1)


d) y(t) = et cost u(t)

P.T.O.

SLR-BB 354

*SLRBB354*

-2-

8) A signal x(t) = 6 cos 10 t is sampled at the rate of 14 Hz. To recover the original signal,
the cut off frequency Fc of ideal low pass filter should be
a) 75 Hz < fc < 9 Hz
b) 9 Hz
c) 10 Hz
d) 14 Hz
9) The Nyquist sampling frequency (in Hz) of a signal given by 6 104 sin c (400 t) 106
sin c (100 t) is
a) 200
b) 300
c) 500
d) 1000
10) The trigonometric Fourier series of an even function of time does not have
a) Dc term
b) Cosine term
c) Sine term
d) Odd Harmonic terms
11) If x(t) is odd, than its Fourier series coefficients must be
a) Real and odd
b) Imaginary and odd c) Real and even d) imaginary and even
12) Which of the following cannot be the Fourier series expansion of periodic signals ?
a) x(t) = 2 cost
b) x(t) = 2 cos t + 7 cost c) x(t) = cost + 0.5 d) None of above
13) If the Fourier series coefficient of x(t) are cn, then the Fourier series coefficients of x(t) is
given by
a) c n *
b) c n
c) c* n
d) c n
14) If x(t) and y(t) are two periodic signals with Fourier series coefficients cn and dn, then the
Fourier coefficients z(t) = x(t) * y(t) are
a)
cn dn
b) T cn dn
c) cn dn
d) cn * dn


15) The Fourier transform of a function x(t) is x(f). The Fourier transform of
@

a)
@


N

b) j2 t fx(f)


will be


d) None of above

16) Let F (j

) be the Fourier transform of a function f(t) then F (0) is




a)

c) jfx (f)

|f(t)|2 dt


f(t) dt


b)


|t f(t)|2 dt


c)


d)

t f(t) dt


17) The Fourier transform of the signal x(t) = e3+2 is of the following form where A and B are
constant
2
a) A eB|f|
b) A eBf
c) A + B |f |2
d) A eBf


18) The Fourier transform of even part of signal x(t) =

is


a)

b)


c) 2

d)



A

19) The impulse response of a discrete time system with a pole is shown in figure (B). The
pole of system must be located on the

Fig. (B)

a) Real axis at z = 1
b) Real axis between z = 0 and z = 1
c) Imaginary axis at z = j
d) Imaginary axis between z = 0 and z = j
20) The z transform of f(z) of function f(nT) = anT is
a)
z

b)
z

a
z

c)
z

______________
z




d)
z

Set A

*SLRBB354*
S

SLR-BB 354

-3-

T. E. (Bio-Medical) (Part I) Examination, 2014


SIGNAL AND SYSTEM (New)
Day and Date : Friday, 12-12-2014
Time : 3.00 p.m. to 6.00 p.m.

Marks : 80

Instructions : 1) All questions are compulsory.


2) Figures to right indicate full marks.
3) Assume suitable data if necessary.
SECTION I
2. Attempt any four :

(45=20)

a) Determine whether the following signal are periodic or not. If periodic find the fundamental
period x (t) = 2 cos




b) Write a note on singularity function.


c) Check the following system for linearity y(t) = 5x (t) + 3
@

d) Explain in detail reconstruction of signal from its sample using interpolation.


e) Find the Nyquist rate and internal for x(t) = 10 cos [1500 t] cos [3000


3. Attempt any two :

t]
(210=20)

a) Determine whether the following system is linear. If not given the suitable condition for
linearity y(t) =

x(t)


.
#

b) i) Referring to figure c) find odd and even part of DT sequence

Fig. C
ii) Construct the block diagram of the system described by
@

O


"


O


#


Set A

SLR-BB 354

*SLRBB354*

-4-

c) i) Determine the power and energy for the signal given below x[n] =

. u(n)




ii) A signal x(t) = 4 cos 60 t + 5 sin 300 t cos 120 t is given then find Nyquist rate
of sampling. If the signal is sampled with that rate, then what is the discrete time signal
obtained after sampling.
SECTION II


4. Attempt any four :

(54=20)

a) State and prove time scaling properties of Fourier transform.


b) Determine z transform and there Roc of the following signal x(n) = an u(n) bn u( n 1).
c) State and prove multiplication property of Fourier transform.
d) Find the Fourier series coefficients for continuous time periodic signal
x(t)


with fundamental frequency




e) Sketch the sequence x(n) =

(n 3k) and find its discrete Fourier series.




5. Attempt any two :

(102=20)

a) Determine the sequence x[n] associated with z-transform given below using Partial Fraction
Expansion method (PFE). x[z] =


; [Right sided sequence]


z

2


b) Determine trigonometric Fourier series of the following wave from shown in figure D.

Fig. D
c) Find the Fourier transform of following :
i) x(t) = 3 sin2 (5t)
ii) x(t) = sin


t. u(t).
_____________________

Set A

SLR-BB 356

*SLRBB356*
Seat
No.

Set

T.E. (Biomedical Engineering) (Part II) Examination, 2014


BIOMEDICAL INSTRUMENTATION II
Day and Date : Tuesday, 25-11-2014
Time : 10.00 a.m. to 1.00 p.m.

Max. Marks : 100

Instructions : 1) Figures to the right indicate full marks.


2) Assume reasonable data wherever necessary.
3) Q. No. 1 is compulsory. It should be solved in first 30 minutes
in Answer book Page No. 3. Each question carries one mark.
4) Answer MCQ/Objective type questions on Page No. 3 only.
Dont forget to mention, Q.P. Set (A/B/C/D) on Top of Page.
MCQ/Objective Type Questions
Duration : 30 Minutes

Marks : 20

1. Write the correct answer :


1) Which bio signal is not periodic ?
a) BP
b) EEG

20
c) Mean BP

d) Respiration

2) What is band width of a normal ECG signal ?


a) 100 Hz
b) < 10 Hz
c) > 10 Hz
d) None of the above
3) What part of the ECG signal is the most useful for determining heart rate ?
a) P wave
b) QRS complex
c) T wave
d) None of the above
4) QRS complex of ECG wave corresponds to action potential ___________
a) De-polarization of ventricles
b) Re-polarization of the ventricles
c) Travel time between SA node and AV node
d) De-polarization of atria
5) Pulse Oximetry measures ______________
a) CO2 saturation in the arterial blood
b) Oxidation reaction
c) Oxygen saturation in the arterial blood
d) Pulse rate
6) Apnea is a (an) _____________
a) Sleep abnormality
b) Cessation of breathing
c) Nervous disorder
d) Infection
P.T.O.

SLR-BB 356

-2-

*SLRBB356*

7) When a mechanical ventilator is switched to a mode enabling the patient to


attempt to breathe by him or herself, the mode is called _____________
a) Controlled
b) Assist
c) Functional
d) Time limited
8) Lung volume is measured by ____________
a) Ohm meter
b) Cardio meter c) Oximeter

d) Spyrometer

9) Maximum peak amplitude of the EEG waveforms typically are __________


a) 120 mV
b) 100 mV
c) 100 micro volt d) 10 micro volt
10) When attaching a surface electrode you may squeeze a drop of electrode gel
on to the surface of the skin or on to the electrode for ____________
a) Removing hair for skin
b) Amplifying the EMG signal
c) Making good electrical contact
d) None of the above
11) The lowest frequency signal in an EEG waveform occurs in the ___________
a) Alpha wave
b) Beta wave
c) Theta wave
d) Delta wave
12) The length of a transmitting antenna in a bedside monitor depends upon the _________
a) Transmitted power
b) Received power
c) Transmitting frequency
d) None of the above
13) In what parts of the hospital are microshock hazards likely to exist ?
a) X-ray area
b) Cardiac catheterization lab
c) Operating room
d) None of the above
14) What is the maximum allowable resistance for the ground wire connecting
the receptacles to prevent exceeding the safe current limit for microshock in
the patient ?
a) 75 ohm
b) 300 ohm
c) 0.5 ohm
d) None of the above
15) 50 to 60 Hz noise in ECG machine can be reduced by ___________
a) Right leg drive
b) CMRR of instrumentation amplifier
c) Notch filter
d) All of the above
16) The AV delay is about ____________ms.
a) 110
b) 40
c) 60
d) None of the above
17) Ground fault indicator devices are usually used in the _____________
a) Operating room
b) EEG laboratory
c) Hemodialyses ward
d) None of the above
18) The graphical recording of heart sounds is called ____________
a) Phonocardiography
b) ECG
c) EOG
d) VCG
19) The frequency response of a diagnostic ECG machine is from 0.05 to _________ Hz.
a) 50
b) 40
c) 100
d) None of the above
20) In a 5 patient electrode ECG front end, a bank of differential amplifier is often
used to derive the ______________ standard leads from the Wilson network.
a) 12
b) 6
c) 9
d) 3
______________
Set A

SLRBB356*

-3-

SLR-BB 356

Seat
No.

T.E. (Biomedical Engineering) (Part II) Examination, 2014


BIOMEDICAL INSTRUMENTATION II
Day and Date : Tuesday, 25-11-2014
Time : 10.00 a.m. to 1.00 p.m.

Marks : 80

Instructions : 1) Figures to the right indicate full marks.


2) Assume reasonable data wherever necessary.
SECTION I
2. Attempt any four :

(45=20)

a) What are bio electric potentials ? Discuss the frequency and voltage range of
ECG, EEG, EMG and EOG signals.
b) Explain how a differential amplifier can amplify the ECG signal while not
amplifying the noise sketch how the patient is connected to the differential
amplifier.
c) Compare amplifiers in terms of gain, band width, noise requirements in ECG,
EEG and EMG measurements.
d) Draw the block diagram of an EEG unit and explain the different blocks in it.
e) How EMG measurements be made ? Describe the recording setup used in
EMG.
f) Explain the origin of various heart sounds.
3. a) Explain how the respiratory rate can be measured .

10

b) Attempt any one :

10

i) Explain the significance of P, Q, R, S, T wave of heart. With the help of


functional block diagram explain the working of a typical ECG machine.
ii) Explain with the help of neat diagram differential asculatory technique of
BP measurement.
Set A

SLR-BB 356

-4-

*SLRBB356*

SECTION II
4. Attempt any four :

(45=20)

a) What are the various waves by which macro shocks can be induced ?
b) Explain the working of a ground fault interrupter.
c) Define bio telemetry. Explain the importance of bio telemetry in the modern
world.
d) Draw a block diagram of telemetry circuit to transmit ECG signals in a reliable
manner.
e) Explain infant incubator.
f) Explain methods of monitoring fetal heart rate.
5. a) Show how a single electric instrument can at the same time be the path for
microshock current flowing both to and from the patient. Use complete diagrams
and do a sample calculation.
10
b) Attempt any one :

10

i) Explain fetal scalp pH measurement.


ii) What is labour activity ? Explain risk factors present in fetal HR monitoring.

_____________________

Set A

SLR-BB 357

*SLRBB357*
Seat
No.

Set

T.E. Biomedical Engg. (Part II) Examination, 2014


MEDICAL IMAGING I
Day and Date : Wednesday, 26-11-2014
Time : 10.00 a.m. to 1.00 p.m.

Max. Marks : 100

Instructions : 1) Figures to the right indicate full marks.


2) Assume suitable data wherever required.
3) Q. No. 1 is compulsory. It should be solved in first 30 minutes
in Answer book Page No. 3. Each question carries one mark.
4) Answer MCQ/Objective type questions on Page No. 3 only.
Dont forget to mention, Q.P. Set (A/B/C/D) on Top of Page.
MCQ/Objective Type Questions
Duration : 30 Minutes

Marks : 20

1. Fill in the blanks :

20

1) In X ray tube _____________ moves from cathode to anode.


a) photons
b) positrons
c) electrons
d) neutrons
2) Heat quantity in _____________ machine is measured in Joules.
a) thermography
b) X-ray
c) image intensifier
d) CT
3) Clinical application frequency of ultrasound machine lies in the range of
_____________
a) 1-15MHz
b) 1-15KHz
c) 1-15Hz
d) 100MHz
4) X ray ___________ rating are done in ampere.
a) tube
b) gantry
c) cathode
d) none of above
5) ___________ anode is used for heat dissipation.
a) stationary
b) rotating
c) platinum

d) copper

6) KVp is a measurement of ___________ in X Ray.


a) current
b) induction
c) capacitance
d) voltage
P.T.O.

SLR-BB 357

*SLRBB357*

-2-

7) The___________ effect is a change in the perceived frequency.


a) Doppler
b) Peltier
c) Seebek
d) Snels
8) Piezoelectric transducer is a part of ___________
a) Fluoroscopy
b) Image intensifier
c) Ultrasound
d) X-ray
9) In X-ray ___________ works as filters.
a) grids
b) collimators
c) table
10) Quartz is a material in ___________
a) Piezo electric b) NaI
c) Collimators

d) oil
d) Grids

11) Heating is not a part of ___________ steps.


a) image intensifying
b) thermographic
c) filtering
d) film processing
12) Ultrasound Doppler works on the principle of ___________
a) Doppler shift
b) Reciprocal innervation
c) Conduction
d) None of above
13) Ultrasound waves need following medium to travel ___________
a) air
b) water
c) solvent
d) gas
14) In rectifier circuit ___________ type of semiconductor is used.
a) Z
b) P
c) N
d) Q
15) Ultrasound Doppler uses ___________ transducer.
a) Piezocrystal
b) Sodium
c) Na-I

d) Active

16) Endoscope consists of ___________


a) Lasers
b) Piezocrystal c) NaI

d) None of above

17) KVp stands for ___________


a) kilo volts peak
c) kinetic voltage peak

b) kilo volt pressure


d) kilo volts

18) Complete the above equation, E = ___________


a) h
b) h
c) h

d) None of above

19) ___________ is used to lower the temperature of X-ray tube.


a) Oil
b) Water
c) Platinum
d) Copper
20) Ultrasound Doppler measures ___________
a) Heart sound
b) Heart rate
c) ECG
______________

d) Pulse rate

Set A

*SLRBB357*

-3-

SLR-BB 357

Seat
No.

T.E. (Biomedical Engg.) (Part II) Examination, 2014


MEDICAL IMAGING I
Day and Date : Wednesday, 26-11-2014
Time : 10.00 a.m. to 1.00 p.m.

Marks : 80

Instructions : 1) Figures to the right indicate full marks.


2) Assume suitable data wherever required.

SECTION I
2. Attempt any four :

(45=20)

1) Define and explain Doppler shift principle in detail.


2) Explain any 2 applications of image intensifier.
3) Differentiate between stationary anode and rotating anode in X-ray machine.
4) Mention any 5 front panel controls of ultrasound machine.
5) Draw detailed constructional diagram of ultrasound transducer.
3. Attempt any two :

(210=20)

1) Write a short note on : X-ray tube, collimators, X-ray tube, grids.


2) Explain any two specifications and characteristics of ultrasound machine.
3) Define : Q factor, Doppler shift in the ultrasound machine.

Set A

SLR-BB 357

-4-

*SLRBB357*

SECTION II
4. Attempt any four :

(45=20)

1) Mentions parts of mammography and explain any two with necessary diagram.
2) Mention any five applications of thermography.
3) Explain the term angiography with its application.
4) Draw construction of fluoroscopy with proper naming.
5) Differentiate between analog and digital radiography with example of each.
5. Attempt any two :

(210=20)

1) Define and explain endoscopy procedure with any one example in detail.
2) Explain how tumor/cancer can be detected using thermography with necessary
diagram.
3) What is the significance of radiation protection ? How is it achieved ?
_____________________

Set A

SLR-BB 358

*SLRBB358*
Seat
No.

Set

T.E. (Bio-Medical) (Part II) Examination, 2014


BIO-STATISTICS
Day and Date : Thursday, 27-11-2014
Time : 10.00 a.m. to 1.00 p.m.

Max. Marks : 100

Instructions : 1)
2)
3)
4)

Attempt any three questions from each Section.


Figures to the right indicate full marks.
Use of non-programmable calculator is allowed.
Q. No. 1 is compulsory. It should be solved in first 30 minutes in
Answer Book Page No. 3. Each question carries one mark.
5) Answer MCQ/Objective type questions on Page No. 3 only. Dont
forget to mention, Q.P. Set (A/B/C/D) on Top of Page.
MCQ/Objective Type Questions

Duration : 30 Minutes
Marks : 20
1. Tick the correct alternative (each one mark) :
20
5
15
1) If byx = /18, bxy = /8, then r = __________
a) 2.5
b) 1/2
c) 2/3
d) 3/2
2) If r = 1 then correlation is _______
a) perfect positive
b) low +ve
c) low ve
d) perfect negative
3) If two regression coefficients are 0.5 and 0.9 then value of correlation is _______
a) 0.6708
b) 0.4500
c) 0.7000
d) 0.0450
4) For the Poisson distribution _________
a) mean = variance
b) mean variance
c) mean variance
d) none of these
5) If a random variable X has a Poisson distribution with mean 3, then P(X = 4) = _______
a) 0.11
b) 0.13
c) 0.15
d) 0.17
6) If z = x 315 is a standard normal variate then the mean and standard deviation of x are
__________
a) 3, 15
b) 15, 3
c) 4, 3
d) 3, 18
7) If byx = 1.4 and bxy = 0.5 then r = ___________
a) 0.84
b) 0.74
c) 0.84

d) 0.74

8) The range of r12.3 is ___________


a) 0 to 1
b) 1 to 0

d) 0 to 2

c) 1 to 1

9) If the correlation is perfect, then coefficient of correlation r = ___________


a) 1, 1
b) (0, 1)
c) (1, 1)
d) (1, 0)
10) If the sum of square of deviation of 15 observation from their mean 20 is 240 then s.d. is
___________
a) 2
b) 4
c) 6
d) 8
P.T.O.

SLR-BB 358

*SLRBB358*

-2-

11) For the distribution of 5 observation 7, 9, 2, 5, 10 then median is


a) 2
b) 5
c) 7

d) 9

12) Variance ration (F) is ___________


a)

2
Se
S2
t

b)

S2
t

c)

S2
e

Se
St

d)

St
Se

13) Standard deviation is ___________ square root of variance.


a) positive
b) negative
c) both a) and b)

d) none of these

14) If cov(x, y) = 0, then r = ___________


a) 1
b) 1

d) 0

c) 0.5

15) The statistics of Chi-square test is ___________


0 E
a)

0 E
b)

c)

(0 E)

d) none of these

16) For a single tailed test H0 is rejected at 5% level of significant if | 2 | > ___________
a) 1.96
b) 2.58
c) 1.645
d) 2.33
17) 95% confidence limit for population mean are ___________
a) x

1.96
n

b) x

2.56
n

c) Both a) and b)

d) None of these

18) Which of the following statements is not true about Poisson probability distribution with
parameter ?
a) the mean of distribution is
b) the variance of distribution is
c) the coefficient of variation is 1
d) the parameter must be greater than zero
19) Student t statistics is defined as ___________
a)

b)

x
s

c)

+x

d)

+x

20) Which of the following values are correct for (byx, bxy) respectively ?
a) (1, 1.1)
b) (1.1, 1)
c) (0.1, 8)
d) (0.5, 4)
______________

SET A

*SLRBB358*

SLR-BB 358

-3-

Seat
No.

T.E. (Bio-Medical) (Part II) Examination, 2014


BIO-STATISTICS
Day and Date : Thursday, 27-11-2014
Time : 10.00 a.m. to 1.00 p.m.

Marks : 80

Instructions : 1) Attempt any three questions from each Section.


2) Figures to the right indicate full marks.
3) Use of non-programmable calculator is allowed.
SECTION I
2. Attempt any two :

(72=14)

a) Calculate S.D. and C.V. of the following :


Class : 0 10
10 20
20 30 30 40
Freqn :
7
13
12
20
b) Find the missing frequence of the following data :
Class : 300 325 325 350 350 375
Freqn :
5
17
8

375 400
......

40 50
25

50 60
10

60 70
3

400 425 425 450 450 475 475 500


326
......
88
9

c) Calculate mean, median and mode of the following data :


Age above
0
10
20
30
40
50
60
70
80
No. of Person
125
115
110
100
75
53
30
15
0
3. a) A company wishes to test the average life of each of the four brands of paracetamol
tablets of 500 mg. The company uses all brands on randomly selected tablets the records
showing life of tablet are given in months.
Brand 1
Brand 2
Brand 3
Brand 4
20
19
21
15
23
15
19
17
18
17
20
16
17
20
17
18
16
16
Test the hypothesis that the average life for each brand of paracetamol tablet is the
same. Assume = 0.01 (Table value of F( 1 = 3, 2 = 14) is 5.56.
b) Monthly salaries of 1000 worker have a normal distribution with mean of Rs. 575 and S.D.
of Rs. 75. Find the number of worker having salaries betn Rs. 500 and Rs. 625. Also find
the minimum salary of the highest paid 200 worker.
(SNV z = 0 to z = 1 is 0.3413
z = 0 to z = 0.67 is 0.2486
z = 0 to z = 0.84 is 0.3)
4. a) Find the probability that at most 4 defective bulbs will be found in a box of 400 bulbs if it is
know that 1 percent of the bulbs are defective.
b) For a binomial distribution mean is 2 and S.D. of 1 find all the constant of distribution.
c) In a Poisson distribution P(X=2) = 23 P(X=1). Find P(X = 0) and P(X = 3).

5
4
4
5

5. a) A soap manufacturing company was distributing a particular brand of soap through a


large number of retail shops. Before a heavy advertisement campaign the mean sales per
week per shop was 140 dozen. After the campaign, a sample of 26 shops was taken and
the mean sales was found to be 147 dozens with standard deviation 16. Can you consider
4
the advertisement effective (Table value t25 (0.05) for right test is 1.798).
SET A

SLR-BB 358

-4-

*SLRBB358*

b) Certain pesticide is packed into bags by a machine. A random sample of 10 bags is drawn
and their contents are found to weigh (in kg.) as follows :
50, 49, 52, 44, 45, 48, 46, 45, 49, 45.
Test if the average packing can be taken to be 50 kg.
Table value t0.05 for g.d.f. is 2.262)
c) Find the C.V. of the following data :
50, 55, 40, 45, 20, 22, 25, 40, 48, 50.

SECTION II
6. Attempt any two :
a) Calculate Spearmans rank correlation coefficient of the following data :
X:
45
35
45
40
35
20
21
50
48
Y:
24
25
12
12
20
21
26
28
5
b) Fit a second degree curve to the following data :
Year
:
2000 2001 2002 2003 2004 2005 2006
Production (m. ton) :
40
50
51
48
35
20
25
c) Find the line of regression of two lines :
X:
10
12
14
16
18
20
Y:
18
12
24
8
20
24

(72=14)
10
7

7. a) If 12 = 0.59, 13 = 0.46, 23 = 0.77


Calculate :
ii) 23.1
iii) R1.23 and R2.13.
i) 12.3
b) A machine puts out 16 imperfect articles in a sample of 500. After the machine is overhauled,
it puts out 3 imperfect article in a batch of 100. Has the machine improved ?
Use 5% and 1% level of significant.
8. a) Given the following information relating to two places A and B, test whether there is any
significant difference between their mean wages.
A
B
Mean wages (Rs.) 47
49
S.D. (Rs.)
28
40
Number of workers 1000
1500
Use 1% level of significant.
2
b) Prove that R1.23 = b12.3 12

2
1

+ b13.2 13

3
1

8
5

5
5

c) The lines of regression are 4x 5y + 33 = 0 and 20x 9y 107 = 0 find, X, Y and .

9. a) The theory predicts the proportion of beans in the four group A, B, C and D should be
9 : 3: 3 : 1. In an experiment among 1600 beans the number in the four group were 822,
313, 287 and 118. Does the experimental result support the theory ?.
(Table value of 20.05 for 3 d.f. is 7.81).
b) A sample of 400 students of under-graduate and 400 students of post-graduate classes
was taken to know their opinion about autonomous college. 290 of the under-graduate and
310 of the post-graduate student favoured the autonomous status. Present these facts in
the form of a table and test at 5% level, that the opinion regarding autonomous status of
college is independent of the level of classes of student.
2
(Table value of at 5% level is 3.84 for 1 d.f.)
c) Fit a St. line to the following data :
x:
0
1
2
3
4
y:
1
1.8
3.3
4.5
6.3

_____________________

SET A

SLR-BB 359

*SLRBB359*
Seat
No.

Set

T.E. (Bio-Medical Engineering) (Part II) (New) Examination, 2014


DIGITAL SIGNAL PROCESSING
Day and Date : Friday, 28-11-2014
Time : 10.00 a.m. to 1.00 p.m.

Total Marks : 100

Instructions : 1) Assume suitable data if necessary.


2) Q. No. 1 is compulsory. It should be solved in first 30 minutes
in Answer book Page No. 3. Each question carries one mark.
3) Answer MCQ/Objective type questions on Page No. 3 only.
Dont forget to mention, Q.P. Set (A/B/C/D) on Top of Page.
MCQ/Objective Type Questions
Duration : 30 Minutes

Marks : 20

1. Multiple choice questions.

20

1) The methods to calculate filtering of long duration sequences is


a) Overlap save
b) Overlap add
c) FIR and IIR
d) Both a) and b)
2) Circular convolution of the sequences x(n) = {1, 2, 1} and h(n) = {1, 2, 3} is
a) {1, 2, 1}
b) {3, 2, 1}
c) {3, 2, 1}
d) {5, 3, 0}
3) IDFT of Y(k) = {1, 0, 1, 0} is
a) {0.5, 1, 0.5, 0}
c) {0.5, 0, 0.5, 1}

b) {0.5, 0, 0.5, 0}
d) {0.5, 1, 0.5, 1}

4) The direct evaluation DFT requires _________ complex additions.


a) N(N + 1)

b) N2

c) N(N 1)

d)

N(N 1)
2

5) DFT is
a) Sampled version of Fourier series
b) Sampled version of Fourier transform
c) Sampled version of fast Fourier transform
d) None of above
6) The DIT or DIF FFT is an algorithm used to compute the
a) DFT
b) FT
c) Convolution
d) Circular convolution
P.T.O.

SLR-BB 359

*SLRBB359*

-2-

7) When the system has poles inside the unit circle in Z-domain ?
a) The system is stable and its impulse response is a decaying function
b) Time domain behavior will be exponentially rising signal
c) The system is unstable
d) The impulse response is marginally constant
8) If X(k) is N-point DFT of finite duration sequence x(n), then
a) x(n+N) = x(n)
b) x(nN) = x(n) c) x(n+N) = x(n)

d) None of the above

9) The direct form II realization requires ___________ no. of additions.


a) M N + 1
b) M + N + 1
c) M + N
d) M N
10) Main advantage of cascade form realization
a) Minimizes quantization error
b) Requires more memory
c) Sensitive to error
d) All above
11) FIR filter design is based on ____________ filter design.
a) Analog
b) Digital
c) DT

d) None of above

12) Aliasing is present in which filter technique


a) Impulse invariant technique
b) Bilinear transformation
c) Approximation of derivatives
d) The matched Z-transform
13) FIR filters are __________ type.
a) Non recursive
b) Recursive

c) Both a) and b)

d) None

14) Using impulse invariant technique which type of filters are not possible to design ?
a) High pass filters
b) LPF
c) BPF
d) Both LPF and BPF
15) The mapping for BLT method is
a) One to one
b) Many to one

c) Many to many

d) None of above

16) Memory requirement and execution time required for FIR filter is
a) High
b) Low
c) Very high
d) Very low
17) The disadvantage of bilinear transformation technique is
a) The mapping is highly linear
b) The mapping is highly non linear
c) Both a) and b)
d) The mapping is highly co linear
18) The transition band is ___________ in Butterworth filter to chebyshev filter.
a) More
b) Less
c) Twice
d) Three times
19) A digital filter is causal if its impulse response ___________
a) H(n) = 0 for n > 0
b) H(n) = 1 for n > 0
c) H(n) = 0 for n < 0
d) H(n) = 1 for n < 0
20) A digital filter is stable if its impulse response is _____________
a) Absolutely differentiable
b) Absolutely integrable
c) Absolutely summable
d) Both b) and c)
______________
Set A

*SLRBB359*

-3-

SLR-BB 359

Seat
No.

T.E. (Bio-Medical Engineering) (Part II) (New) Examination, 2014


DIGITAL SIGNAL PROCESSING
Day and Date : Friday, 28-11-2014
Time : 10.00 a.m. to 1.00 p.m.

Marks : 80

Instructions : 1) All questions are compulsory.


2) Assume suitable data if necessary.
SECTION I
2. Attempt any four :

(45=20)

1) Explain the advantages of FFT.


2) Perform circular convolution using concentric circle method for x1(n) = {1, 2, 3, 1} and
x2(n) = {4, 3, 2, 2}.
3) Differentiate between overlap save and overlap add methods.
4) Compute four point DFT of a sequence x(n) = {0, 1, 2, 3} using DIF FFT algorithm.
5) Find DFT of x(n) = {1, 1, 0, 0}.
3. Attempt any two :

(210=20)

1) Find circular convolution of the following sequences x(n) = {1, 1, 1, 2}; y(n) = {1, 2, 3, 2}
DFT and IDFT method.
2) Perform linear convolution of finite duration sequences h(n) = {1, 1, 2, 1} and
x(n) = {1, 1, 1, 2, 1, 0, 1, 4, 3, 2, 1, 0, 1, 1} by
a) overlap save
b) overlap add method.
3) Obtain the cascade realization with minimum number of multipliers
1
1
1 1

H(z ) = + z 1 + z 2 + z 3
4
2
2 4

Set A

SLR-BB 359

*SLRBB359*

-4-

SECTION II
4. Attempt any four :

(45=20)

1) What are properties of FIR filter ?

( )


2
2

= 0 for

2

2) Design an ideal LPF with a frequency response Hd e j = 1 for

Find values of h(n) for N = 4 using hanning window.


3) Explain LMS filter.
4) By BLT method obtain the digital filter transfer function of analog filter H(s) =

1
s+1

5) What are the properties that are maintained same in the transfer of analog filter into digital
filter ?
5. Attempt any two :

(210=20)

1) Explain in detail adaptive noise cancellation in detail.

2) Apply impulse invariant method and find H(z) for H(s) =

10
2

s + 7 s + 10

with T = 0.2 sec.

( ) = 1 for

3) Design an ideal HPF with a frequency response Hd e

/ /
4

= 0 for / /
4

Find values of h(n) for N = 11 plot frequency response.


_____________________

Set A

SLR-BB 36

*SLRBB36*
Seat
No.

Set

S.E. (Civil) (Part II) (New) Examination, 2014


ENGINEERING MATHEMATICS III
Day and Date : Monday, 1-12-2014
Time : 3.00 p.m. to 6.00 p.m.

Max. Marks : 100

Instructions : 1)
2)
3)
4)

All questions carry equal marks.


Figures to the right indicate full marks.
Use of non-programmable calculator is allowed.
Q. No. 1 is compulsory. It should be solved in first 30 minutes in
Answer Book Page No. 3. Each question carries one mark.
5) Answer MCQ/Objective type questions on Page No. 3 only.
Dont forget to mention, Q.P. Set (A/B/C/D) on Top of Page.
MCQ/Objective Type Questions

Duration : 30 Minutes

Marks : 20

1. Choose the correct answer :

20

1) L { Sin2 3t} is
a)

b)

S + 36

6
2

S S + 36

c)

18
2

S S + 36

d)

(S

18
2

+ 36

S 1

1
2) L 2
= ?
S 2S + 5

a) et Sin 2t

b) et Cos 2t

c) et Sin 2t

d) et Cos 2t

3) Among the following, which equation is not a non-linear partial differential equation ?
a)

p + q =1

b) pq = xy

c) p3 + q3 = 27z

d) y2p + x2q = x2y2z

c) (xy, yz) = 0

d) (x2, y2) = 0

4) The general solution of equation xp + yq = z is


a) (x, y) = 0

x y
b) , = 0
y z

5) The solution of pq = 1 is
a) z = ax

y
+c
a

b) z =

x
by + c
b

6) The P.I. of (x2D2 + xD + 1) y = 2 Sin (log x) is


a) log x Cos (log x)
c) x Cos x

c) z = ax +

d) none of these

b) log x Cos (log x)


d) x Cos x

2
7) On putting x = ez the transformed differential equation of x

a) (D2 3D + 4) y = 2e2z
c) (D2 3D + 4) y = 2x2

y
+c
a

d2 y
2

3x

dy
+ 4y = 2x 2 is
dx

dx
b) (D2 4D + 4) y = 2x2
d) (D2 4D + 4) y = 2e2z

P.T.O.

SLR-BB 36

*SLRBB36*

-2-

8) The general solution of (3x + 2 )2


a) y = c1 (3x + 2)

1
3

c) y = c1 (3x + 2)

1
3

d2 y

+ 5 (3x + 2)

dx 2

+ c2 (3x + 2)1

dy
3y = 0 is
dx

b) y = c1 (3x + 2)

+ c2 (3x + 2)1

d) y = c1 (3x + 2)

1
3
1
2

+ c2 (3x + 2)
+ c2 (3x + 2)1

1
9) D a X is equal to
ax
a) e

ax

X dx

ax
b) e

ax

X dx

10) The P.I. of the equation (D2 + 1) y = x2 is


a) x
b) x2

ax
c) e

X dx

c) x2 + 2

ax
d) e

X dx

d) x2 2

11) The conditions of expansions of functions in Fourier series are known as


a) Harmonic
b) Periodic
c) Riemanns conditions
d) Dirichlets conditions
12) If r = ae3t + be2t, then at t = 0,
a) a

dr
=
dt

b) b

c) 2b + 3a

d) 2a + 3b

13) Curl (xyi + yzj + zxk) = ____________ at (1, 1, 1).


a) i + j + k

b) 0

c)

d) (i + j + k)

14) If f(x) = x2 is expanded as Fourier series in the interval (0, 2 ) then the constant term is
a)

2
3

b)

42
3

c)

15) For a Poisson distribution


a) Mean = variance
c) Mean > variance

32
2

d) none

b) Mean variance
d) Mean < variance

16) Fourier series of even function in (1, 1) contains


a) Only sine series
b) Only cosine series
c) Both sine and cosine series
d) None
17) If y = x + 1 and x = 3y 7 are the two lines of regression then r =
a)

b) 3

c) 2

d)

18) The value of Karl Pearsons coefficient of correlation r lies between


a) 2 and 1
b) 1 and 1
c) 1 and 2

d) 2 and 3

19) A vector function F is called solenoidal if


a) div F = 0

b) curl F = 0

c) grad F = 0

20) In a lot of 50 items the probability of a defective item is


distribution is
a) 2

b) 8

c) 10

d) div F = 0

1
, then the mean of the Binomial
5

d) 20

______________
Set A

*SLRBB36*

-3-

SLR-BB 36

Seat
No.

S.E. (Civil) (Part II) (New) Examination, 2014


ENGINEERING MATHEMATICS III
Day and Date : Monday, 1-12-2014
Time : 3.00 p.m. to 6.00 p.m.
Instructions : 1)
2)
3)
4)

Marks : 80

All questions are compulsory.


Attempt any three questions from each Section.
Figures to the right indicate full marks.
Use of non-programmable calculator is allowed.
SECTION I

2. a) Solve, (D 3 + 1) y = e 2 Sin

3
x.
2

b) Solve, (D2 + 5D + 4) y = 3 2x.

c) Solve, 2p (1 + q) = 3qz.

z z
+
= 3z by the method of separation of variables.
x y
b) Solve, (2z y) p + (x + z) q = (2x + y).

3. a) Solve the partial differential equation 4

c) Solve,

(D2

c) Solve,

(D2

+ 3D + 2) y = sin

5
4

ex.

OR
+ 2D + 1) y = x cos x.

1
2
2
4. a) Solve xD + D y = 9x .
x

b) Find L

s2

s 2 + 4 s 2 + 9

)(

by using convolution theorem.

c) Using Laplace transform evaluate

2t

t sin2 t dt .

5. a) The deflection of a strut with one end built in and the other end supported and subjected to end thrust
dy
a 2R
.
Given
that
= 0, y = 0 when x = 0 and y = 0
(
l

x
)
dx
P
dx 2
R sin ax

l cos ax + l x where al = tanal and l is length of strut.


when x = l. Prove that y =
P a

P satisfies the equation

d2 y

+ a2y =

b) Solve, (D2 + 4D + 13) y = et sint where y = 0,

dy
= 0 at t = 0 by using Laplace transform method.
dx

Set A

SLR-BB 36

*SLRBB36*

-4-

SECTION II
6. a) If f(x) = x
= (2 x)

0 x 1
1 x 2

with period 2. Find Fourier series and hence deduce that

1
12

1
32

+ ..... =

2
.
8

b) Prove that F = (6xy + z3) i + (3x2 z) j + (3xz2 y) k is irrotational. Find the function such that
F = .

7. a) Find the equations of lines of regression and also the coefficient of correlation from the following
data :
x:

57

42

38

42

45

44

40

46

44

43

40

y:

10

26

41

29

27

19

18

19

31

29

33

b) In a large cosignment of electric bulbs, 10% are defective. A random sample of 20 is taken for
inspection. Using Binomial distribution. Find the probability that :
i) All are good bulbs
ii) Exactly three are defective bulbs.

8. a) Find the angle between the normal to the surfaces x2y + z = 3 and x log z y2 + 4 = 0 at (1, 2, 1).
1 8

b) Prove that r 4 = 5 .
r r

4
4

OR
b) If u = x2y i + y2x3j 3x2z2k and v = 2xz2i yzj + x2y3k. Find (u v ) at (1, 2, 1).

c) Fit a first degree curve to the following :

x:

10

20

30

40

50

60

70

80

y:

10

9. a) Find half range sine series for f(x) = x x2 in (0, ).

b) The marks of 1000 students in a university are found to be normally distributed with mean 70 and
standard deviation 5. Estimate the number of students whose marks will be :
i) between 60 and 75
ii) more than 75
(For S.N.V. z area under the curve between z = 0 and z = 1 is 0.3413 and between
z = 0 and z = 2 is 0.4772).
c) In a Poisson distribution P (x = 2) =

2
P (x = 1).
3

Find :
i) P (x = 0)
ii) P (x = 1).

3
_____________________

Set A

SLR-BB 360

*SLRBB360*
Seat
No.

Set

T.E. (Biomedical) (Part II) Examination, 2014


EMBEDDED SYSTEM
Day and Date : Saturday, 29-11-2014
Time : 10.00 a.m. to 1.00 p.m.

Max. Marks : 100

Instructions : 1) Q. No. I is compulsory. It should be solved in first 30 minutes


in Answer book Page No. 3. Each question carries one mark.
2) Answer MCQ/Objective type questions on Page No. 3 only.
Dont forget to mention, Q.P. Set (A/B/C/D) on Top of Page.
MCQ/Objective Type Questions
Duration : 30 Minutes

Marks : 20

I. Choose the correct answer :


1) DMA stands for
a) Direct Memory Access
c) Dual Memory Access

(201=20)
b) Download Memory Access
d) None

2) Microcontroller is a _________ processor.


a) 16-bit
b) 8-bit
c) 32-bit

d) 64-bit

3) The instruction MOV A, R0 uses _________ addressing mode.


a) Immediate
b) Register
c) Accumulator d) None
4) Indicate which of the following register is not a bit addressable ?
a) A
b) B
c) R4
d) PSW
5) Which of 8051 ports need pull-up register to function as I/O port ?
a) Port 1
b) Port 2
c) Port 0
d) Port 3
6) For 8051 based system if crystal frequency is 16 MHz then period of machine
cycle in micro second will be _________
a) 0.60
b) 0.75
c) 1.08
d) 0.050
7) In timer mode 2 the counter rolls over when it goes from
a) FF-00
b) FFFF-0000
c) 1FFF-1000
d) None
8) In microcontroller and LCD interface which line will instruct the LCD that
microcontroller is sending data ?
a) DB0
b) RW
c) RS
d) EN
P.T.O.

SLR-BB 360

*SLRBB360*

-2-

9) The way in which LSB of a word is written into the lower addressed byte and
the other bytes are written in increasing order of significance is called
a) Big-endian system
b) Medium-endian system
c) Little-endian system
d) None
10) What is the address for TCON, SCON, SBUF and PCON ?
a) 98 H, 99 H, 87 H, 88 H
b) 88 H, 98 H, 99 H, 87 H
c) 99 H, 98 H, 88 H, 87 H
d) 87 H, 88 H, 98 H, 99 H
11) Which bit of TMOD will exactly configure timer/counter as a timer or counter ?
i) TMOD.6 of C/T for Timer 1
ii) TMOD.6 of C/T for Timer 0
iii) TMOD.2 of C/T for Timer 0
iv) TMOD.2 of C/T for Timer 1
a) i, ii
b) ii, iv
c) i, iii
d) iii, iv
12) What are software interrupts ?
a) RST 0 7
b) RST 5.5 7.5 c) INTR, TRAP

d) None

13) Which is the nonvolatile memory ?


a) RAM
b) ROM

d) None

c) SRAM

14) The 8085 processor can address up to __________ bytes of memory.


a) 64 K
b) 32 K
c) 256 K
d) 128 K
15) The ALE pin stands for ___________
a) Address latch enable
b) Address low edge
c) Address line edge
d) None
16) Which one of the following is a serial bus ?
a) PCI
b) I2C
c) AMBA

d) All

17) One direction data communication is called


a) Half duplex
b) Full duplex
c) a) and b)

d) None

18) SFR stands for


a) Special Function Register
c) Significant Function Register

b) Sign Flag Register


d) None

19) Which bus is used in Automobile electronics ?


a) I2C
b) PCI
c) SPI

d) CAN

20) TMOD stands for


a) Timer Mode Control Register
b) Time Multiplexing of Data
c) a) and b)
d) None
______________
Set A

*SLRBB360*

-3-

SLR-BB 360

Seat
No.

T.E. (Biomedical) (Part II) Examination, 2014


EMBEDDED SYSTEM
Day and Date : Saturday, 29-11-2014

Marks : 80

Time : 10.00 a.m. to 1.00 p.m.


SECTION I
II. Answer any 4 of the following :

(54=20)

1) Explain the following instructions for MCS-51 family.


i) CJNE
ii) CPL bit
iii) SJMP 8-bit Address
iv) MOVX
v) SWAP A
2) Explain with diagram for timer mode 1.
3) Explain about the RTC.
4) Draw and explain PORT1 structure of 8051.
5) Write a program for addition of two 16 bit numbers with expected result of
16-bits.
III. Answer any 2 of the following :

(102=20)

1) Explain the interfacing of DAC0808 to 8051 and write a program to generate


saw tooth waveform.
2) Draw and explain interfacing of matrix key board of size 4*4 to micro controller
and write an assembly language program to display key code on LCD display.
3) Explain about the relay and LCD display.
Set A

SLR-BB 360

-4-

*SLRBB360*

SECTION II
IV. Answer any 4 of the following :

(54=20)

1) Write any 10 applications of Embedded system.


2) Explain about the memory selection for embedded system.
3) Explain about the timer and counting for device networks.
4) Explain the memory map of embedded system.
5) Explain about the serial and parallel communication buses.
V. Answer any 2 of the following :

(102=20)

1) Explain the characteristics and typical hardware components for embedded


systems.
2) Explain about the memories and I/O devices.
3) Explain serial communication using I2C protocol with master and slave mode
for transmission.
_____________________

Set A

SLR-BB 361

*SLRBB361*
S

B.E. (Biomedical Engg.) (Part I) Examination, 2014


NUCLEAR MEDICINE
Day and Date : Tuesday, 2-12-2014
Time : 3.00 p.m. to 6.00 p.m.

Total Marks : 100

Instructions : 1) Figures to the right indicate full marks.


2) Assume suitable data wherever required.
3) Q. No. 1 is compulsory. It should be solved in first 30
minutes in Answer Book Page No. 3. Each question carries
one mark.
4) Answer MCQ/Objective type questions on Page No. 3
only. Dont forget to mention, Q.P. Set (A/B/C/D) on Top
of Page.
Duration : 30 Minutes

MCQ/Objective Type Questions

Marks : 20

1. Choose the correct answer :

(201=20)

1) Radioactive __________ are called as tracers.


a) material
b) radioisotopes c) chemical

d) transmitters

2) A nuclide in a metastable state decays to a stable atom through a process


called _________
a) ionization
b) radiation
c) isometric transition
d) none of above
3) _________ particles constitute electromagnetic radiation.
a) Isometric
b) Gamma
c) Beta
d) None of above


4)

a) 0.693

b) 0.6

c) 0.678

5) The unit of radioactivity is ___________


a) zeta
b) curie
c) ml

d) 0.345
d) mmHg

6) A scintillator is a substances that produces minute flashes of _________


a) heat
b) energy
c) light
d) none of above
P.T.O.

SLR-BB 361

*SLRBB361*

-2-

7) The gamma camera is a ___________ imaging device.


a) rotatory
b) stationary
c) vertical

d) horizontal

8) A ___________ scanner is a rotatory imaging device.


a) NaI
b) Rectilinear
c) Collimator

d) Horizontal

9) ___________ detects only radionuclides that produces a cascaded emission


of single photons.
a) PET
b) ECT
c) SPECT
d) RIA
10) ___________ is used to obtain in vivo cross sectional images.
a) PET
b) ECT
c) SPECT
d) RIA
11) ___________ emissions are less penetrating power.
a) Alpha
b) Beta
c) Gamma

d) Photons

12) ___________ crystals are having high density.


a) KI
b) CaI
c) NaI

d) None of above

13) N =
a) No

b) N

c) NAI

d)


14) ___________ techniques is used for the detection of HIV.


a) PET
b) SPECT
c) Ionization
d) RIA
15) The ___________ is the principle type of accelerator used in radionuclide
production.
a) Cyclotron
b) Reactors
c) Radionuclide d) Radiotracers
16) ___________ are made up of lead.
a) Collimators
b) Analyzers
c) RIA
d) Gantry
17) Resolution is the power of the system to _________ two points on the plane.
a) substract
b) add
c) integrate
d) differentiate
18) The purpose of pulse height analyzer is to exclude ___________ radiation.
a) gamma ray
b) scattered
c) collimator
d) pulse height analyzer
19) Positrons emission in PET modality produces __________ 511 KeV.
a) one
b) two
c) three
d) zero
20) ___________ is a unit of energy deposition that is not restricted to air.
a) Rontgen
b) Rad
c) Rem
d) Curie
______________

Set A

*SLRBB361*
S

-3-

SLR-BB 361

B.E. (Biomedical Engg.) (Part I) Examination, 2014


NUCLEAR MEDICINE
Day and Date : Tuesday, 2-12-2014

Marks : 80

Time : 3.00 p.m. to 6.00 p.m.


Instructions : 1) Figures to the right indicate full marks.
2) Assume suitable data wherever required.
SECTION I
2. Attempt any four :

(45=20)

1) Explain isometric transition with necessary diagram.


2) List ideal characteristics of a radiopharmaceutical.
3) State a law of radioactive decay and derive expression for half life of a
radionuclide.
4) Explain the working of pulse height analyzer with neat diagram.
5) Draw and explain working of Gas filled detector in detail.
3. Attempt any two :

(210=20)

1) Explain using suitable block diagram working of Gamma Camera system


in detail.
2) Explain in detail principle and working of rectilinear scanner.
3) Discuss various quality control tests to be carried out to assess the
performance of Gamma camera.

Set A

SLR-BB 361

-4-

*SLRBB361*

SECTION II
4. Attempt any four :

(45=20)

1) What are the various biological effects of internal and external radiation ?
2) State various applications of RIA system.
3) Differentiate between PET and SPECT modality.
4) Discuss various positrons emitting radionuclide.
5) Explain the process of positron annihilation process in PET with necessary
diagram.
5. Attempt any two :

(210=20)

1) Mention various reconstruction techniques and explain filtered back projection


technique in SPECT.
2) What is the principle of radiation safety ? How it is achieved ?
3) Define and explain function of RIA technique.
_____________________

Set A

SLR-BB 362

*SLRBB362*
S

B.E. (Biomedical Eng.) (Part I) Examination, 2014


MEDICAL INFORMATICS
Day and Date : Thursday, 4-12-2014
Time : 3.00 p.m. to 6.00 p.m.

Max. Marks : 100

Instructions : 1) Figures to the right indicate full marks.


2) Assume suitable data wherever required.
3) Q. No. 1 is compulsory. It should be solved in first 30
minutes in Answer Book Page No. 3. Each question carries
one mark.
4) Answer MCQ/Objective type questions on Page No. 3
only. Dont forget to mention, Q.P. Set (A/B/C/D) on Top
of Page.
Duration : 30 Minutes

MCQ/Objective Type Questions

Marks : 20

1. Answer the following questions.


(201=20)
1) The applications of Internet to ____________ is intended to enhance
communication among various hospitals.
a) HMIS
b) CAPE
c) VE
d) CPR
2) ____________ is that branch of computer science which focuses on the
development of computer system.
a) HMIS
b) AI
c) VE
d) CPR
3) Expert systems generate alerts and ____________ assist in diagnosis.
a) help
b) information
c) reminders
d) none of above
4) Blood donors come under ____________ module.
a) pathology
b) pharmacy
c) operation theater
d) blood bank
5) ____________ information module maintain database of the all the categories
of employees.
a) Employee
b) Administration
c) Operation theater
d) Personnel
6) ____________ consists of real time video conferencing.
a) Tele monitoring
b) Tele education
c) Visualizer
d) None of above
P.T.O.

SLR-BB 362

-2-

*SLRBB362*

7) CAME stand for _____________


a) Computer Assisted Matching Education
b) Committee Assisted Matching Education
c) Computer Assisted Medical Information
d) None of above
8) A majority of the ___________ programmers are related to the management
of chronic diseases.
a) CAS
b) CPR
c) VE
d) CAPE
9) In __________ surgeons concomitant global view of the related CT/MRI slice.
a) VE
b) CAPE
c) CPR
d) CAS
10) CAS provides _____________ for the surgeon.
a) Images
b) Image guidance
c) Image slices
d) None of above
11) _____________ enhances patient access to care, encompasses remote
consultation on a doctor to doctor level.
a) VE
b) CAPE
c) Telemedicine d) CAS
12) Telemedicine includes _____________ image transmission via telephone.
a) analog
b) matrix
c) gray
d) digital
13) The best telemedicine tool available on the internet is _____________
a) email
b) intranet
c) GPS
d) None of above
14) _____________ uses the protocols of the www to share the information
resources within the organization.
a) Networking
b) Intranet
c) Internet
d) email
15) PEM stands for _____________
a) Permanent Enhancement Mail
b) Privacy Enhanced Mail
c) Privacy Emitted Mail
d) None of above
16) Diet for patients come under _____________ module.
a) procedure
b) pathology
c) diet planning d) patient care
17) _____________ request and result entry for special tests like TFT, TSH.
a) Medical Research Module
b) Internet
c) Intranet
d) Health Module
18) VC stands for _____________
a) Virtual Instrumentation
b) Virtual Endoscopy
c) Virtual Council
d) None of above
19) ___________ and MRI imaging offer spatial information on a patients diseases.
a) PET
b) X-ray
c) CT
d) PET/CT
20) Static imaging _____________ is useful for rendering diagnostic opinions.
a) tele medicine b) tele radiology c) internet
d) tele pathology
______________
Set A

*SLRBB362*
S

-3-

SLR-BB 362

B.E. (Biomedical Eng.) (Part I) Examination, 2014


MEDICAL INFORMATICS
Day and Date : Thursday, 4-12-2014
Time : 3.00 p.m. to 6.00 p.m.

Marks : 80

Instructions : 1) Figures to the right indicate full marks.


2) Assume suitable data wherever required.
SECTION I
2. Attempt any four :

(45=20)

1) Define and describe the prospective of medical informatics.


2) What is the role of medical research in hospital information system ?
3) Define and differentiate between OPD/Consultant module and polyclinic module.
4) State and define the advantages of online learning.
5) Explain the role of blood bank module and pathology laboratory module.
3. Attempt any two :

(210=20)

1) Write a short note on :


a) diet planning module
b) radiology module.
2) Explain the significance of virtual environment technology and state its
applications in detail.
3) Describe different methods used in surgical simulation.

Set A

SLR-BB 362

-4-

*SLRBB362*

SECTION II
4. Attempt any four :

(45=20)

1) Define expert system and state its applications.


2) State and explain the significance of intranet.
3) List and explain any 5 applications of telemedicine.
4) Differentiate between telesurgery and robotic surgery.
5) Explain the need of CPR in detail.
5. Attempt any two :

(210=20)

1) With the help of diagram explain how satellite transmission wothOP2000/


SILONET takes place.
2) Define CAS. Explain its functioning and disadvantages.
3) Write a short note on :
a) Tele education
b) CAPE.
_____________________

Set A

SLR-BB 363

*SLRBB363*
Seat
No.

Set

B.E. Biomedical Engineering (Part I) Examination, 2014


BIOMEDICAL INSTRUMENTATION III
Day and Date : Saturday, 6-12-2014
Time : 3.00 p.m. to 6.00 p.m.

Max. Marks : 100

Instructions : 1) Figures to the right indicate full marks.


2) Assume reasonable data wherever necessary.
3) Q. No. 1 is compulsory. It should be solved in first 30 minutes in
Answer Book Page No. 3. Each question carries one mark.
4) Answer MCQ/Objective type questions on Page No. 3 only. Dont
forget to mention, Q.P. Set (A/B/C/D) on Top of Page.
MCQ/Objective Type Questions
Duration : 30 Minutes

Marks : 20

1. Write the correct answer :

20

1) A defibrillator uses electrical current discharge to defibrillate the ___________


a) Lungs
b) Heart
c) Kidneys
d) Patient
2) Defibrillators discharge a strong electrical current thought the patient heart and attempt to
shock the ___________ to resume its control
a) AV node
b) Bundle of HIS
c) SA node
d) Purkinje fibers
3) The implantable defibrillator can protect patients from severe ____________
a) Ventricular tachycardia
b) Bradycardia
c) Mitral valve prolapse
d) None of the above
4) Blood is diverted to a machine for oxygenation that is called a(n) __________
a) Dialysis machine
b) Intro-aortic balloon pump device
c) Heart lung machine
d) Apheresis machine
5) The proper conductivity of dialysis machine is in the range of
a) 5 ms to 8 ms
b) 12 ms to 16 ms
c) 1 ms to 5 ms
d) None of the above
6) Laser light is mono chromatic that means it is ____________
a) Of single frequency
b) Of single color
c) Both a) and b)
d) Either a) or b)
7) A laser beam can be used to ___________ bleeding wounds and ulcers
a) Cut out
b) Cauterize
c) Shock
d) Weld
8) One of the main duties of the surgical technologist is ___________
a) To detect odors
b) To sterilize instruments
c) To manipulate surgical instruments
d) To monitor safely in the OR

P.T.O.

SLR-BB 363

-2-

*SLRBB363*

9) A hazardous vaporized tissue plume in the OR is removed by ___________


a) Smoke evacuators
b) A humidifier
c) A suction pump
d) All of the above
10) Fibrillation refers to ______________
a) Asynchronous skeletal contractions
b) Synchronous heart wave
c) Asynchronous heart contractions
d) All of the above
11) A Cardioverter is a ____________ that is synchronized to discharge only on the patients
R wave.
a) Pacemaker
b) Diathermy machine
c) Stimulator
d) Defibrillator
12) The AV synchronized pacemaker responds to the ECG __________
a) P wave
b) R-R interval
c) T wave
d) None of the above
13) Negative feedback loops exist amount lungs, heart, and brain centers to maintain
constant _____________
a) Blood pH
b) BP
c) HR
d) Sugar
14) Anesthesia is used to manage ___________ in surgical procedures.
a) Pain
b) BP
c) HR
d) Blood flow
15) _______________ is the process of killing micro organisms on tools, instruments and
other objects used in surgery.
a) Coagulation
b) Sterilization
c) Anesthesia
d) All of the above
16) Aseptic techniques and working practices are design to reduce the incidence of
_____________
a) Sterilization
b) Anesthesia
c) Infection
d) All of the above
17) Hemodylesis is a process that involves the removal of chemical substances from the
blood by passing it through tubes surrounded by ____________
a) Dialyser
b) Semi permeable membrane
c) Dialyser coil
d) All of the above
18) Electrical safety on ____________ machine is very important because the entire system
involves wet components, direct patient bloodstream connection, and electrically operated
devices.
a) Pacemaker
b) Defibrillator
c) Hemodilaysis
d) All of the above
19) The cardiotacometer is a _____________ meter
a) BP
b) HR
c) Respiration rate

d) None of the above

20) Upon failure of the biological kidney, the patients blood must be treated on the hemodialysis
for _____________
a) 10 12 hr per treatment, once in a month
b) 4 8 hr per treatment, once in a three month
c) 4 5 hr per treatment, 3 day a week
d) 2 4 hr per treatment, twice in a year
______________
Set A

*SLRBB363*

-3-

SLR-BB 363

Seat
No.

B.E. Biomedical Engineering (Part I) Examination, 2014


BIOMEDICAL INSTRUMENTATION III
Day and Date : Saturday, 6-12-2014

Marks : 80

Time : 3.00 p.m. to 6.00 p.m.


Instructions : 1) Figures to the right indicate full marks.
2) Assume reasonable data wherever necessary.
SECTION I
2. Attempt any four :

(45=20)

a) Write technical specifications of a pace maker.


b) Which are the components of a pace maker ? Draw block diagram of a pace maker.
c) Discuss the different types of stimulator wave forms. How will you select the wave form
which would not produce any ionization.
d) Explain the function of microwave, short wave and surgical diathermy.
e) Describe the cardiac pace maker wave forms and explain their importance.
f) Discuss the different modes of operation of cardiac pace maker.
3. a) Explain various types of voltage sources by which the implanted pacemaker circuit can
be operated.

10

b) Attempt any one :

10

i) Give a note on different techniques involved in electro surgery.


ii) List four factors affecting the interaction between ultra sound waves and biological
tissue. Define the meaning of ultra sound.

Set A

SLR-BB 363

-4-

*SLRBB363*

SECTION II
4. Attempt any four :

(45=20)

a) List four DC defibrillator waveforms.


b) What precaution should a clinician take when it is necessary to defibrillate a patient who
has an implanted pacemaker of either the synchronous or the asynchronous type ?
c) Explain therapeutic application of the LASER.
d) Why is electrical safety important in hemodylesis machine ?
e) Define hemodylesis including high flux-high efficiency types.
f) What is heart rate variability ? How its measurement is carried out ? State applications of
the same.
5. a) List eight subsystems of a hemodylesis machine explain with a block diagram.

10

b) Attempt any one :

10

i) Explain the difference between photo coagulating and photo vaporizing tissue using a
laser beam and focusing it to achieve high power densities. How does a lens help in
focusing.
ii) Compare and contrast the 3 types of laser technology in terms of device, technique,
cost, applications CO2, Argon, Nd-YAG laser.
_____________________

Set A

SLR-BB 364

*SLRBB364*
Seat
No.

Set

B.E. (Part I) (Bio-Medical Engineering) Examination, 2014


PRINCIPLES OF IMAGE PROCESSING
Day and Date : Tuesday, 9-12-2014
Time : 3.00 p.m. to 6.00 p.m.

Max. Marks : 100

Instructions : 1) Q. No. 1 is compulsory. It should be solved in first 30 minutes


in Answer Book Page No. 3. Each question carries one mark.
2) Answer MCQ/Objective type questions on Page No. 3 only.
Dont forget to mention, Q.P. Set (A/B/C/D) on Top of Page.
3) Figures to right indicate full marks.
4) Use of non-programmable calculator is allowed.
MCQ/Objective Type Questions
Duration : 30 Minutes
1. Choose the correct answer :
1) Digitizing amplitude value is called _____________
a) Quantization
b) Sampling
c) Both a) and b)
d) None

Marks : 20
(201=20)

2) If number of storage bits are 524288 for 8 bit image then size of the image
will be __________
a) 256 256
b) 128 128
c) 512 512
d) 1024 1024
3) The Extension of DFT are ____________
a) periodic
b) non periodic c) symmetric
d) none
4) Segmentation should stop when _________ of interest have isolated.
a) subject
b) point
c) edge
d) object
5) Histogram are useful in ______________
a) image enhancement
b) image compression
c) segmentation
d) all
6) Fourier spectrum value ranging from 0 to very high value to display it in 8 bit
system requires i.e. compresses dynamic range
a) Inverse transform
b) Power law transform
c) Log transform
d) None
7) Prewitt operator is used for ____________
a) Point detection
b) Line detection
c) Diagonal Edge detection
d) Both b) and c)
P.T.O.

SLR-BB 364

*SLRBB364*

-2-

8) The color at point of equal energy is _________


a) White
b) Black
c) Red

d) Green

9) (Z7 + Z8 + Z9) (Z1 + Z2 + Z3) represents operator


a) Robert cross gradient
b) Prewitt
c) Sobel
d) Laplacian
10) Following is not a gray level transformation.
a) image negative
b) power law
c) log
d) histogram
11) Hit and Miss transform is used for detection of
a) Shape
b) Edge
c) Boundary

d) Skeleton

12) The Haar transform has poor energy ___________ for image.
a) expansion
b) dilation
c) compaction
d) none of above
13) Multiple opening or closing of a set have no effect after operator has been
applied once.
a) True
b) False
c) Partially true
d) Neither true nor false
14) Channel Encoder in compression model reduces
a) Redundancy
b) Noise
c) Both a) and b) d) None
15) SNR is related to which fidelity ____________
a) Subjective
b) Objective
c) Error

d) None

16) In Morphological process closing is denoted by ______________


a) A B
b) A B
c) A  B
d) A B
17) _____________ is responsible for inter pixel redundancy.
a) Channel Encoder
b) Source Encoder
c) Source Decoder
d) All
18) Hough transform is used for ___________
a) Edge linking
b) Point detection
c) Line detection
d) Region growing
19) The chain code of boundary depends on ________ point.
a) starting
b) end
c) middle
d) none of these
20) DCT is used in ____________
a) Image segmentation
b) Morphological processing
c) Image compression
d) Image restoration
______________

Set A

*SLRBB364*

-3-

SLR-BB 364

Seat
No.

B.E. (Part I) (Bio-Medical Engineering) Examination, 2014


PRINCIPLES OF IMAGE PROCESSING
Day and Date : Tuesday, 9-12-2014
Time : 3.00 p.m. to 6.00 p.m.

Marks : 80

Instructions : 1) All questions are compulsory.


2) Figures to right indicate full marks.
3) Use of non-programmable calculator is allowed.
SECTION I
2. Attempt any four (five marks each) :
1) Write a short note on image sensing and acquisition.
2) Write a short note on segmentation in image processing.
3) What is Histogram ? Explain equalization.
4) Explain what is gradient operator in edge detection.
5) What is Homomorphic filter in image enhancement ?

(45=20)

3. Attempt any two (ten marks each) :


(210=20)
1) What do you mean by sampling and quantization ? Explain in detail.
2) What is meant by detection of discontinuities ? Explain point and line detection.
3) Explain properties of 2D-DFT in detail ?
SECTION II
4. Attempt any four (five marks each) :

(45=20)

1) Explain operation of dilation and erosion in morphological operation.


2) What are different lossy compression methods ? Explain any one.
3) Drive 4 4 Haar matrix.
4) What is DCT ? Explain with one example.
5) Explain Boundary Extraction by contour following.
5. Attempt any two (ten marks each) :

(210=20)

1) Explain LZW coding for image compression with example.


2) What are different method of feature extraction ? Explain in detail.
3) Explain DFT, FFT and KL transform in detail.
_____________________

Set A

Set A

SLR-BB 365

*SLRBB365*
Seat
No.

Set

B.E. (Biomedical Engineering) (Part I) Examination, 2014


HOSPITAL MANAGEMENT
Day and Date : Thursday, 11-12-2014
Time : 3.00 p.m. to 6.00 p.m.

Max. Marks : 100

Instructions : 1) Figures to the right indicate full marks.


2) Assume reasonable data wherever necessary.
3) Q. No. 1 is compulsory. It should be solved in first 30 minutes
in Answer Book Page No. 3. Each question carries one mark.
4) Answer MCQ/Objective type questions on Page No. 3 only.
Dont forget to mention, Q.P. Set (A/B/C/D) on Top of Page.
MCQ/Objective Type Questions
Duration : 30 Minutes

Marks : 20

1. Write the correct answer :

20

1) Organization effectiveness comprises all except


a) Productivity power
b) Adaptability to change
c) Flexibility in structure and strategy
d) Rigidity in structure
2) Which is the skeleton of organization ?
a) Organizational function
b) Organizational structure
c) Decentralization
d) Coordination
3) Which services are not a part of the hospital services ?
a) Essential services
b) Additional services
c) Utility services
d) Administrative services
4) All except one is the output indicator of hospitals system
a) Patient satisfaction
b) Public relation
c) Quality of care
d) Machines
5) Supportive services of the hospital includes all except
a) Pharmacy services
b) Laboratory services
c) Home keeping services
d) Laundry services
6) Which is not the basic of classification of hospital
a) Specialty
b) Functional
c) Size
d) Shape
7) Concept behind changing the role of hospital from indoor care includes all except
a) Rising cost of hospital care
b) Shortage of hospital bed
c) Economic importance
d) Increase of hospital bed
P.T.O.

SLR-BB 365

-2-

*SLRBB365*

8) The function of the OPD include __________


a) Promotion of health
b) Training of medical and nursing personal
c) Social search
d) All of the above
9) Shopping window of a hospital is __________
a) Operation theater
b) ICU
c) OPD
d) Cafeteria services
10) Which of the following is a basic of classification of hospital ?
a) Hospital practice
b) Length of stay of patient
c) Educational purpose
d) Medical staff
11) Which of the following is a function of emergency care
a) First aid
b) Immediate resuscitation
c) Dietary management
d) Hospitalization
12) Which of the following facility is not needed in emergency department
a) Examination room
b) Treatment room
c) Observation area
d) Cafeteria
13) What are the primary objectives of human resource management in hospital
services ?
a) To motivate the employees
b) To create good organization relation
c) Coordination
d) Contribution of services
14) Which of the following is a function of nursing supervisor ?
a) Administrative and coordinative
b) As a leader
c) Supervisor
d) None of the above
15) Which is the nurse-patient ratio in general wards within a hospital
a) 2 : 10
b) 1 : 3
c) 2 : 6
d) 1 : 5
16) Functional nursing is a method of providing
a) Equipment care
b) Care of records
c) Care management
d) Patient care
17) Which of the following is not an element of progressive patient care ?
a) Intensive care
b) Constant care
c) Intermediate care
d) Community care
18) Which of the following should be included in efficient material management in
hospital ?
a) A list materials
b) Procurement
c) Taking an inventory
d) All of the above
19) Which of the following is not a material used in hospital and community ?
a) Drugs and medicine
b) Transport
c) Supplies
d) Equipment and instrument
20) Which one of the following is the element of directing in hospital ?
a) Evaluation
b) Discipline
c) Planning
d) Delegation

______________

Set A

*SLRBB365*

-3-

SLR-BB 365

Seat
No.

B.E. (Biomedical Engineering) (Part I) Examination, 2014


HOSPITAL MANAGEMENT
Day and Date : Thursday, 11-12-2014
Time : 3.00 p.m. to 6.00 p.m.

Marks : 80

Instructions : 1) Figures to the right indicate full marks.


2) Assume reasonable data wherever necessary.
SECTION I
2. Attempt any four :
(45=20)
a) Which are the activities of medium sized and large sized hospitals ? State the
issues involved in hospital management.
b) Draw a flow chart representing stages in promoting and building a new hospital.
c) Explain staff, space and facilities requirement for biomedical department in hospital.
d) Explain in brief management financing techniques used for hospitals.
e) Discuss different types of approaches in medical ethics.
f) List advantages of having efficient hospital information system.
3. a) Prepare job description for housekeeping manager of a hospital.
10
b) Attempt any one :
10
i) If you were a quality manager of a hospital how would you go about in
implementingquality ?
ii) Define cost accounting. How does is different from management accounting.
SECTION II
4. Attempt any four :
(45=20)
a) Explain the different methods of communication used in health sectors.
b) What are the medico legal procedures in medical records management ?
c) Mention the types of drugs and physical facilities, storage processes in pharmacy
department.
d) What are the objectives of nursing services ?
e) What are the emergency equipments to be maintained in an operation theater ?
f) Write about the equipments used and the applications of various imaging
processes.
5. a) Describe the various speciality services to be maintained in a general hospital.
b) Attempt any one :
i) Describe the principles, objectives and the needs of medical records.
ii) Write in detail about the hospital waste management.
_____________________

10
10

Set A

Set A

SLR-BB 366

*SLRBB366*
Seat
No.

Set

B.E. (Part II) (Biomedical Engg.) Examination, 2014


MEDICAL IMAGING II
Day and Date : Tuesday, 25-11-2014
Time : 3.00 p.m. to 6.00 p.m.

Max. Marks : 100

Instructions : 1)
2)
3)
4)
5)

Assume data wherever necessary.


Figures on the right indicate full marks.
Draw diagram/sketches wherever necessary.
Use legible handwriting, use blue/black ink only.
Q. No. 1 is compulsory. It should be solved in first 30 minutes
in Answer book Page No. 3. Each question carries one mark.
6) Answer MCQ/Objective type questions on Page No. 3 only.
Dont forget to mention, Q.P. Set (A/B/C/D) on Top of Page.
MCQ/Objective Type Questions

Duration : 30 Minutes

Marks : 20

1. Select correct alternative :

20

1) When y-gradient coils are on, the protons near the front of patient will precess
__________
a) Slower
b) Faster
c) Medium
d) None of these
2) The slice selection z-gradient (G/z) is applied slightly before the ______ RF
pulse.
a) 90
b) 180
c) 60
d) 30
3) A spin echo sequence with a short TR will produce a ______ weighted image.
a) T1
b) T2
c) T3
d) T4
4) There is a way to acquire all the slices at one time called _______
a) Multislice imaging
b) Single slice imaging
c) Both
d) None of these
5) The number of slices per rotation is equal to the number of _________
a) Collimator
b) Detector
c) Filter
d) X-ray tube
6) An image reconstructor receives sampled and digitized X-ray data from ______
a) DAS
b) Slip ring
c) Detector
d) Filter
7) In the PRESS method, the RF pulses have flip angles of _________
a) 90 180 180
b) 180 90 180
c) 90 90 180
d) 180 180 180
P.T.O.

SLR-BB 366

*SLRBB366*

-2-

8) Protons in the antiparallel orientation are said to be in a __________


a) Low energy state
b) High energy state
c) Both
d) None of these
9) CT-angiography should be avoided in patients with ___________ disease.
a) Kidney
b) Lung
c) Heart
d) Liver
10) The multidetector computer tomography (MDCT) is similar to __________
generation.
a) First
b) Second
c) Third
d) Fourth
11) ___________ is closely related to the FID.
a) T1
b) T2
c) TE

d) TR

12) The superconducting magnets made using _______ material.


a) Lead
b) Copper
c) Silver
d) Niobium tantalum alloy
13) The bandwidth of the RF pulse selects the _________ of the slice.
a) Location
b) Thickness
c) Both
d) None of these
14) The frequency of precession called _______
a) Larmor frequency
b) Magnetic Diapole Moment (MDM)
c) T1
d) T2
15) When 90 RF pulse ON then ________ gradient ON.
a) Z
b) Y
c) X

d) All of above

16) A long _________ means long time to acquire an image.


a) TR
b) TE
c) T1
d) T2
17) T1 is the time for _________ percent of the nuclei to return to its lower energy
state.
a) 50
b) 40
c) 63
d) 70
18) In rotate-fixed scanner detectors are __________
a) Fixed
b) Rotate
c) Both

d) None of these

19) The original EMI scanner was designed specifically for evaluation of the _____
a) Heart
b) Lung
c) Liver
d) Brain
20) The expression for T1 is given by _________
b) Mz = Mz (1 e 2 t T1 )
a) Mz = Mz (1 e t T1 )
c) Mz = Mz (1 e 3 t T1 )

d) Mz = Mz (1 e 4 t T1 )
______________
Set A

*SLRBB366*

-3-

SLR-BB 366

Seat
No.

B.E. (Part II) (Biomedical Engg.) Examination, 2014


MEDICAL IMAGING II
Day and Date : Tuesday, 25-11-2014

Marks : 80

Time : 3.00 p.m. to 6.00 p.m.


Instructions : 1) Assume data wherever necessary.
2) Figures on the right indicate full marks.
3) Draw diagram/sketches wherever necessary.
4) Use legible handwriting, use blue/black ink only.
SECTION I
2. Attempt any four :

20

1) Write note on CT scanner parameters.

2) Draw and explain Xenon gas detector.

3) Explain first generation of CT scan with the help of diagram.

4) What is MRS ? Explain different applications of MRS.

5) Explain Fourier reconstruction technique with the help of diagram.

3. Attempt any two :

20

1) Draw and explain working of CT gantry in detail.

10

2) Explain PRESS sequence and STEAM sequence with the help of diagram.

10

3) Explain spiral CT in detail with the help of diagram.

10
Set A

SLR-BB 366

-4-

*SLRBB366*

SECTION II
4. Attempt any four :

20

1) Explain advantages, disadvantages and applications of CT-angiography.

2) Draw only diagram for multiecho imaging and multislice imaging.

3) Explain T1 and T2 relaxation parameters used in MRI.

4) Which factors affecting signal-to-noise ratio ?

5) Write note on RF coils.

5. Attempt any two :

20

1) Draw and explain spin echo pulse sequence in detail.

10

2) Explain working principle of MDCT and also explain advantages, disadvantages


and applications.

10

3) Write down different applications of MRI in detail.

10

_____________________

Set A

SLR-BB 367

*SLRBB367*
Seat
No.

Set

B.E. (Part II) (Biomedical Engg.) Examination, 2014


INSTALLATION, MAINTENANCE AND SERVICING
Day and Date : Thursday, 27-11-2014
Time : 3.00 p.m. to 6.00 p.m.

Max. Marks : 100

Instructions : 1) Q. No. 1 is compulsory. It should be solved in first


30 minutes in Answer book Page No. 3. Each question carries
one mark.
2) Answer MCQ/Objective type questions on Page No. 3 only.
Dont forget to mention, Q.P. Set (A/B/C/D) on Top of Page.
MCQ/Objective Type Questions
Duration : 30 Minutes

Marks : 20

1. Choose the correct answer :

(201=20)

1) A ______________ calibration is performed shortly after power on or reset of


the electrolyte analyzer.
a) 1 point
b) Automatic
c) Manual
d) 2 point
2) In the synchronized shock mode of the defibrillator the LED blinks during the
detection of each.
a) P wave
b) R wave
c) T wave
d) All
3) In the defibrillator when the internal paddles are used, the default energy
level on power up is set to
a) 10 J
b) 15 J
c) 20 J
d) 25 J
4) CPAP stands for
a) Coronary Positive Airway Pressure
b) Continuous Pulmonary Airway Pressure
c) Continuous Positive Airway Pressure
d) Cardiac Positive Airway Pressure
5) Which one of the following is not an operational mode of ventilator ?
a) M mode
b) CP AP Mode c) S mode
d) T mode
6) In the synchrony ventilator system the level of pressure support delivered is
determined by
a) PS = EP AP IP AP

b) PS = IP AP + EP AP

c) PS = IP AP EP AP

d) PS = IP AP /EP AP
P.T.O.

SLR-BB 367

-2-

7) The battery used in patient monitoring system is of


a) 3 volts
b) 6 volts
c) 9 volts

*SLRBB367*
d) 12 volts

8) The upper inlet in the ventilator is used for__________ pressure gas.


a) Low
b) High
c) Moderate
d) All
9) In the ventilator the gas flows through __________ filter.
a) Bacteria
b) Fiber
c) Optical
d) Liquid
10) The flow transducer on __________side measures the gas flow to the patient.
a) Expiration
b) Inspiration
c) a) and b)
d) None
11) The clause for ISO 9001-2008 is given as
a) Quality management system
b) Quantity management system
c) Product realization
d) a) and c)
12) The sample used for the calibration of auto analyzer must have the HCT
value to be within
a) 35.5% 55.5% b) 40% 60% c) 60% 80%
d) 25% 50%
13) The lower inlet is used in ventilator for pressure gas.
a) Low
b) High
c) Moderate
d) All
14) In the ventilator____________ shows the working pressure.
a) Thermometer b) Barometer
c) Spirometer
d) Manometer
15) In the ventilator a safety valve opens, if the pressure exceeds approximately
___________ of H2O.
a) 102 cm
b) 201 cm
c) 303 cm
d) 120 cm
16) The clause for NABH is given as
a) Information to patient
b) Rights of patient and family
c) Quality investigation
d) All
17) In an electrolyte analyzer, the___________ measuring chain within the
electrode determines the difference between the two potential values on either
side of the membrane.
a) Galvanic
b) Faradic
c) Ohmic
d) b) and c)
18) In an electrolyte analyzer the difference in ion concentration between the
inner electrolyte and the sample causes an electro chemical potential to form
across the membrane of the ___________electrode.
a) Clark
b) Active
c) Mercurous
d) All
19) The peristaltic pump of the electrolyte analyzer transports all samples and
operating fluids instrument.
a) Inside
b) Outside
c) Upside
d) Downside
20) The X-ray was discovered by
a) Ruther ford
b) Venkat Raman
c) Rontgen
d) Newton
______________
Set A

*SLRBB367*

-3-

SLR-BB 367

Seat
No.

B.E. (Part II) (Biomedical Engg.) Examination, 2014


INSTALLATION, MAINTENANCE AND SERVICING
Day and Date : Thursday, 27-11-2014
Time : 3.00 p.m. to 6.00 p.m.

Marks : 80

II. Answer any four of the following :

(45=20)

1) What are the different medical gases ? Explain the use of these gases in the
hospital.
2) What kind of tests and checkups must be performed before the final acceptance
of the instrument.
3) Give the chart of daily, weekly and monthly maintenance of the auto analyzer.
4) Give the flow chart for the trouble shooting of the following ECG machine
components.
a) Stylus heat control
b) Battery status indication
5) Explain why a trained biomedical engineer is necessary for the servicing and
maintenance of the biomedical equipments.
III. Answer any 2 of the following :

(210=20)

1) Draw the front panel of the pulse oximeter showing the different parameters
and components.
2) Give the trouble shooting for the following problems of pulse oximeter.
a) Pulse rate and /or pleth bar vary significantly from the palpated pulse rate.
b) Displayed values and/or pleth bar do not correlate with the physiological
condition of the patient.
c) Tones do not sound during start up.
d) Display is blank wtih a continuous sound.
e) Monitor will not power ON.
Set A

SLR-BB 367

-4-

*SLRBB367*

3) Define the following :


a) MTTF

b) MTBF

c) MTTR

d) MTTRS

e) Calibration and maintenance


IV. Answer any four of the following :

(45=20)

1) Explain the trouble shooting techniques.


2) Give the recommendations and practices for the optimum use of X-ray machine
and to reduce radiation exposure.
3) Give the battery installation steps for the defibrillator.
4) Give the SPO2 trouble shooting of the patient monitoring system.
5) Explain the design and planning of OT and list the different OT equipments.
V. Answer any 2 of the following :

(210=20)

1) Draw a neat labeled figure of the front panel of the defibrillator.


2) Draw a neat labeled figure of the control panel of the electrosurgical unit and
specify different controls.
3) Write short notes on the following :
a) ISO and NABH certification
b) What is an incubator ? Explain its significance.
_____________________

Set A

SLR-BB 368

*SLRBB368*
Seat
No.

Set

B.E. (Part II) (Biomedical Engg.) Examination, 2014


BIOMEDICAL MICROSYSTEMS
Day and Date : Saturday, 29-11-2014
Time : 3.00 p.m. to 6.00 p.m.

Max. Marks : 100

Instructions : 1) Q. No. I is compulsory. It should be solved in first 30 minutes


in Answer book Page No. 3. Each question carries one mark.
2) Answer MCQ/Objective type questions on Page No. 3 only.
Dont forget to mention, Q.P. Set (A/B/C/D) on Top of Page.
Duration : 30 Minutes

MCQ/Objective Type Questions

Marks : 20

I. Choose the correct answer :


20
1) The microsystems contains ___________ sized devices.
a) m
b) cm
c) mm
d) m
2) One of the principle use of SiO2 is as ________
a) Mask
b) Etchant
c) Active substrate
d) None
3) In miller indices the orientation of right face plane of silicon crystal is
represented as ___________
d) 001
a) (010)
b) (001)
c) 010
4) LPCVD is __________
a) Low Pressure Chemical Vapor Deposition
b) Large Pressure Chemical Vapor Deposition
c) Less Parameter Chemical Vapor Deposition
d) None
5) ____________ photoresist become more soluble under shadow of UV light.
a) Positive
b) Negative
c) Both
d) Zero
6) For special application the polymer are made as electrically ________ material.
a) Insulator
b) Conducting
c) Semiconductor d) None
7) The German term Abformung means _________
a) Lithography
b) Electroforming c) Molding
d) All
8) In _________ method single crystal substrate is extended by growing a film
of same single crystal material.
a) Epitaxy
b) Chemical Vapor Deposition
c) Physical Vapor Deposition
d) Sputtering
P.T.O.

SLR-BB 368

-2-

*SLRBB368*

9) _________ etching is known as orientation dependent etching.


a) An isotropic
b) Isotropic
c) Dry
d) None
10) To protect masking material from x-rays application __________ layer is
used in LIGA process.
a) Golden Plated Region
b) Metal layer
c) Solid Region
d) All
11) ___________ sensor is used to sense a particular chemical compound.
a) Pressure
b) Chemical
c) Acoustic
d) Thermal
12) In cantilever based biosensor the mechanism of transduction is recognized
by ___________
a) Micro Mechanical Twist
b) Micro Mechanical Balance
c) Micro Mechanical Rotation
d) Micro Mechanical Bending
13) The biocompatibility of Quantum Dots (QD) is increased by covering with
_______ molecules.
a) Live Cells
b) Poly Ethylene Glycol (PEG)
c) Glucose
d) All of the above
14) In micro valve Type A of TAS; restriction is _________ to fluid flow.
a) Parallel
b) Perpendicular
c) Is formed by frozen part of liquid d) None
15) The microfluidic unit of TAS is used to ____________
a) For sampling
b) For sensing
c) To control fluid flow
d) For analysis
16) The fluid flow rate in piezoelectric actuation of TAS is ________ ml/min.
a) 0.5
b) 2
c) 5
d) 7
17) ISFET means ___________
a) Ion Sensitive Field Effect Transistor
b) Indian Society for Field Effect Transistor
c) Indian Standards of Field Effect Transistor
d) None
18) In injection compression molding the injection is stopped ________ mm before
mold is closed completely.
a) 0.5
b) 0.75
c) 1
d) 1.5
19) The soft lithography process is capable of generating structures as small as
_____________
a) 10 nm
b) 20 nm
c) 30 nm
d) 40 nm
20) For type A diabetes patients drug delivery system is used to deliver _________
a) Glucose
b) Insulin
c) Both a) and b) d) None
______________
Set A

*SLRBB368*

-3-

SLR-BB 368

Seat
No.

B.E. (Part II) (Biomedical Engg.) Examination, 2014


BIOMEDICAL MICROSYSTEMS
Day and Date : Saturday, 29-11-2014

Marks : 80

Time : 3.00 p.m. to 6.00 p.m.

SECTION I
II. Solve any four questions :

(54=20)

1) List and explain typical MEMS and microsystem products.


2) Explain Czochralski method for growing single crystal silicon.
3) Explain chemical reactions in thin film deposition over silicon substrate in
CVD.
4) Differentiate between wet etching and dry etching.
5) Write a short note on piezoelectric crystal.
III. Solve any two questions :

(102=20)

1) Explain LIGA process in general.


2) Classify and explain different deposition techniques.
3) Explain miller indices. Give mechanical properties of silicon.

Set A

SLR-BB 368

-4-

*SLRBB368*

SECTION II
IV. Solve any four questions :

(45=20)

1) Classify different types of biosensors classified based on transduction and


explain.
2) Explain cantilever based biosensors.
3) Define pressure sensor. Explain packaged pressure sensors.
4) With the help of neat diagram explain piezoelectric actuation of TAS.
5) Explain micro-transfer molding.
V. Solve any two questions :

(102=20)

1) Explain fabrication of microstructures on a processed Si wafer using


compression molding technique.
2) List types of microvalves of TAS. Explain each one with neat diagram.
3) Write a short note on :
i) Application of Biosensor in glucose detection Invivo.
ii) Application of Biosensor in bacterial urinary tract infections.
_____________________

Set A

SLR-BB 369

*SLRBB369*
Seat
No.

Set

B.E. (Part II) (Biomedical) Examination, 2014


TISSUE ENGINEERING
Day and Date : Monday, 1-12-2014
Time : 3.00 p.m. to 6.00 p.m.

Max. Marks : 100

Instructions : 1) Q. No. 1 is compulsory. It should be solved in first


30 minutes in Answer Book Page No. 3. Each question
carries one mark.
2) Answer MCQ/Objective type questions on Page No. 3 only.
Dont forget to mention, Q.P. Set (A/B/C/D) on Top of Page.
MCQ/Objective Type Questions
Duration : 30 Minutes

Marks : 20

1. Choose the correct answer :


1) _____________ are responsible for the synthesis of many GAGs and for the
deposition and organization of collagens.
a) Pluripotent
b) Fibroblast
c) Myoblast
d) None
2) The directional translocation of cells in concentration gradient of some chemo
attractant or chemo repellent substance.
a) Chemotaxis
b) Desmotaxis
c) Diataxis
d) None
3) A blanket term that covers all aspects of movement actively generated by a cell
termed _____________
a) Cell translocation
b) Cell motility
c) Cell grow
d) None
4) Fluid shear stress expressed in _____________
a) Viscosity
b) Dynes/cm2
c) Hgmm

d) None

5) _____________ refers to the relative parallel motion between adjacent fluid


planes during flow.
a) Invitro
b) Invivo
c) Shear
d) None
6) Satellite cells are quiescent myogenic stem cells located between__________
and the _____________
a) Basal lamina and Sarcolemma
b) Mc Neil and Khakee
c) Myosin and Actin
d) None
7) _____________ number of skeletal muscle are present in the human body.
a) 660
b) 720
c) 560
d) None
8) If the force developed by the muscle is less than the load termed as_____________
a) Miometric
b) Isometric
c) Mobility
d) None
9) When the force developed by the muscle is equal to load termed as_____________
a) Miometric
b) Isometric
c) Philometric
d) None
P.T.O.

SLR-BB 369

-2-

*SLRBB369*

10) Cartilage is an Avascular tissue that contains only one cell type termed as
a) Collage
b) Chondrocyte
c) Gene therapy
d) None
11) Contraction is define as the activation of muscle fibres with a tendency of fibres to
a) Elongation
b) Shorten
c) Instability
d) None
12) When the force developed by the muscle is greater than the load on the muscle
termed ____________
a) Miometric
b) Isometric
c) Immobility
d) None
13) The blood cell population is divided into two major groups.
a) Myeloid and Lymphoids
b) Heamodynamic and hydrodynamic
c) Invivo and invitro
d) None of the above
14) Which type of tissue facilitates movement of the skeleton ?
a) Epithelial
b) Muscular
c) Connective

d) Nervous

15) Cells in which tissue type are specialized to transmit electrical impulses from one
body region to another.
a) Muscle
b) Nervous
c) Connective
d) Epithelial
16) Aging affect connective tissue in which of the following way
a) Loss of phability and Resiliency
b) Joint pains and brittle bones
c) Decrease efficiency in tissue repair and wound
d) All of the above
17) Mesenchyme means
a) Gives rise to all other connective tissue
b) Is the first connective tissue in embryo
c) Persists as stem cells
d) All the above
18) Application of reconstituted Ex-Vivo hemotporesis
a) Autologous, Allogenic and Alternative method
b) Antibody, Allopathy and moderation
c) Autologous, moderation, Emthelial
d) None of the above
19) The____________ and ____________properties of neural cells can form the
basis of a cell-based bio sensing unit.
a) Invivo and Invitro method
b) Myeloid and Lymphoid
c) Electrochemical and chemo electrical
d) None
20) ____________ tissue contains Type I and Type IV collagen.
a) Bone marrow
b) Kidney
c) Liver
d) None

______________
Set A

*SLRBB369*

-3-

SLR-BB 369

Seat
No.

B.E. (Part II) (Biomedical) Examination, 2014


TISSUE ENGINEERING

Day and Date : Monday, 1-12-2014

Marks : 80

Time : 3.00 p.m. to 6.00 p.m.


II. Answer any four questions :

(54=20)

1) Explain about the reconstruction of Epithelial and Endothelial surface.


2) Explain about the shear stress effect on Endothelial cell metabolite secretion.
3) Explain about the effect of aging on stem cells.
4) Explain about the Endothelial cells, adipocytes and fat storing cells.
5) Explain about the stromal cells as feeder layers.
III. Answer any two questions :

(102=20)

1) Explain in detail about the bio reactor design for tissue engineering diagram.
2) Explain about the mechanism of shear stress induced gene regulation diagram.
3) Explain about the directed motile response for invitro system with diagram.

Set A

SLR-BB 369

-4-

*SLRBB369*

IV. Answer any four questions :

(54=20)

1) Explain in detail about the molecular control of hematopoietic growth factor.


2) Discuss in detail for the liver system.
3) Explain about the active usage of channel properties for nervous system.
4) Explain about the injury for skeletal muscle system.
5) Explain about the endocrine function for kidney system.
V. Answer any two question :

(102=20)

1) Explain about the bio artificial kidney growth and function with diagram.
2) Explain about the invitro chrendrogenesis cartilage system.
3) Explain about the invitro neural circuits and biosensors.

_____________________

Set A

SLR-BB 37

*SLRBB37*
Seat
No.

Set

T.E. (Civil) (Part I) Examination, 2014


DESIGN OF STEEL STRUCTURES (Old)
Day and Date : Wednesday, 3-12-2014
Time : 3.00 p.m. to 6.00 p.m.

Max. Marks : 100

Instructions : 1) Q. No. 1 is compulsory. It should be solved in first 30


minutes in Answer book Page No. 3.
2) Answer MCQ/Objective type questions on Page No. 3 only.
Dont forget to mention, Q.P. Set (A/B/C/D) on Top of Page.
MCQ/Objective Type Questions
Duration : 30 Minutes
1. Choose the correct answer :

Marks : 20

1) For welds 91 to 100 angle between fusion faces, the throat thickness is
taken as
a) 0.6 times the size
b) 0.65 times the size
c) 0.7 times the size
d) 0.55 times the size
2) The allowable direct tensile stress in rolled steel sections is
a) 120 MPa
b) 150 MPa
c) 180 MPa
d) 200 MPa
3) The max. Slenderness ratio in steel ties which may be subjected to
compression under wind load should be
a) 250
b) 350
c) 450
d) No limit
4) A vertical member in compression is called as
a) Column
b) Column and strut
c) Strut

1
1

d) Beam

5) The connection of one beam to another beam by means of an angle at the


bottom and an angle at the top, is known as
a) Unstiffened seated connection
b) Stiffened seated connection
c) Seated connection
d) None of these
6) The permissible vertical deflection for electric overhead travelling cranes
upto 500 kN is
a) L / 500
b) L / 750
c) L / 1000
d) L / 600

where L is span of crane runway girder.


P.T.O.

SLR-BB 37

-2-

*SLRBB37*

7) The member of roof truss which supports purlins is called


a) Main tie
b) Main sling
c) Rafter
d) Main strut

8) The effective length of single angle member in welded truss is equal to


a) L
b) 0.85 L
c) 0.7 L
d) 0.65 L

9) The slenderness ratio of the Lacing bars for compression members shall not
exceed
a) 180
b) 145
c) 250
d) 300

10) In plastic analysis, the shape factor for circular sections is


a) 1.5
b) 1.6
c) 1.697
d) None of these

11) The allowable shear stress in M.S. bolts in clearance holes is (in MPa)
a) 80
b) 100
c) 120
d) 250

12) The effective area of welded angle for design purpose is equal to
a) Gross area
b) Full area of welded leg plus certain percentage of outstanding leg
c) a) and b)
d) None of above

13) The average shear stress in a member calculated on the cross section of
the web shall not exceed
b) 0.66 fy
c) 0.4 fy
d) 0.45 fy
a) 0.6 fy

14) The vertical force on rail from electric over head cranes when compared with
the wheel load is about
a) 20%
b) 25%
c) 5%
d) 10%

15) As per codal provisions, the effective buckling length of a steel column
which is effectively held in position and restrained against rotation at both
ends is
a) 0.5 L
b) 1.3 L
c) 0.65 L
d) 2 L

______________

Set A

*SLRBB37*

-3-

SLR-BB 37

Seat
No.

T.E. (Civil) (Part I) Examination, 2014


DESIGN OF STEEL STRUCTURES (Old)
Day and Date : Wednesday, 3-12-2014
Time : 3.00 p.m. to 6.00 p.m.

Marks : 80

Instructions : 1) Solve any three questions from Section I and any three
questions from Section II.
2) Assume necessary data if required and mention it clearly.
3) Figures to right indicate full marks.
4) Use of IS 800 : 1984 and IS 875 III codes, steel table is
permitted.
SECTION I
1. a) What are the types of connections used in steel structures ?

b) An ISA 100 7510 mm is welded with flange of a column ISHB 350 @ 674 N/m
as shown in Fig. 1. The bracket carries a load of 100 kN at a distance of
50 mm from face of column. Design welded connection.
10

Fig. 1 [Q. 1b)]


2. a) Draw sketches of common sections used as tension members.

b) A tension member tie has to carry a load of 150 kN. Design a suitable angle
section for it. Determine number of 16 mm bolts required to connect the
member to gusset plates at ends.

Set A

SLR-BB 37

-4-

3. a) What do you mean by effective length of a strut ?

*SLRBB37*
3

b) Calculate the safe compressive load carried by a double angle discontinuous


strut composed of 2ISA 70458 mm with longer legs back to back connected
to opposite side of a gusset plate 10 mm thick with two bolts. The length of
the strut between centre to centre of intersection is 3 m.
Take fy = 250 N/mm2.
10
4. a) Explain web buckling of beams.

b) A simply supported beam has an effective span of 7m and carries a uniform


distributed load of 50 kN/m. Taking fy = 250 N/mm2 and E = 2105 N/mm2,
design the built-up beam if it is laterally supported.
10
SECTION II
5. A gantry girder of span 6 m is subjected to total vertical BM of 520 kNm and
horizontal BM of 40 kNm. Design a suitable section for the gantry girder and
check it for the combined stresses.
14
6. a) Sketch any type of the roof truss and show its components.

b) The principal rafter of roof truss is subjected to a compressive force of 70 kN.


The length of rafter is 2.5 m. Design the member if it is also subjected to a
tensile force of 50 kN. Assume joints are welded.
10
7. An ISLB 500 @ 750 N/m is subjected to an axial compressive force of 550 kN
and a BM of 40 kNm about the major axis. Check adequacy of the section if its
effective length is 3.5 m.
13
8. a) Explain theorem of plastic analysis.

b) A steel column ISHB 300 @ 0.630 kN/m resting on a steel base plate is
supported on concrete foundation. The column carries a central load of
1500 kN. Design the base plate. The permissible stress of concrete is 4 MPa. 10

_____________________

Set A

SLR-BB 370

*SLRBB370*
Seat
No.

Set

S.E. (Part I) (Electrical and Electronics) Examination, 2014


ENGINEERING MATHEMATICS III
Day and Date : Tuesday, 9-12-2014
Time : 10.00 a.m. to 1.00 p.m.

Max. Marks : 100

N. B. : 1) Q. No. 1 is compulsory. It should be solved in first 30 minutes in Answer


Book Page No. 3. Each question carries one mark.
2) Figures to the right indicate full marks.
3) Use of calculator is allowed.
4) Answer MCQ/Objective type questions on Page No. 3 only. Dont forget
to mention, Q.P. Set (A/B/C/D) on Top of Page.
MCQ/Objective Type Questions
Duration : 30 Minutes

Marks : 20

1. Choose the correct answer :


20
2
2
1) The complementary function of the differential equation (D + 1)y = x is
a) (c1 + c2x)ex
b) c1ex + c2ex
c) (cosx + sinx)ex d) c1cosx + c2sinx
2)

1 x
D3

a)

X
2

=?
b) ex

1
8

X
2

c)
d2 y

3) PI of the differential equation

dx

a)

e2x

b)

x
3

d) e x . x

1
9

+ 3 dy
+ 2y = 2e2x is
dx

x e 2x
6

c) 2e2x

d)

1 e 2x
6

4) The PI of the differential equation (x2D2 + xD + 1)y = x is


a)

ex
2

b)

x
2

5) The general solution of (x + a)2


a) y = c1(x + a)2 + c2(x + a)3
c2
c1
+
c) y =
2
(x + a)
(x + a)3
6) The solution of q = 3p2 is
a) z = 3ax + a2y + c

d2 y
dx 2

4(x + a) dy
+ 6y = 0 is
dx

b) z = 3ax2 + by + c
2
d) z = ax + a y + c

7) The solution of xp + yq = z is

(z

b) y = c1x2 + c2x3
d) y = c1(x + a)5 + c2(x + a)

c) z = ax + 3a2y + c

a) x , y 2 z2 = 0

d) e x

c) x

( )

y
b) x , = 0
z z

y x
c) z , z = 0

d) y 2 + z2 , x = 0
z

P.T.O.

SLR-BB 370

*SLRBB370*

-2-

1 1
8) z z 2 for 2<|z| is

b) 2k1 k 1
a) 2k1 k 1
9) If z{xk} = f(z) then z{ak.xk} = ?

()

b) a F az

a) f(az)

c) 2k k 1

d) 2k k 1

c) a1 F(az)

d) F az

( )

10) If {f(k)} = 4k k 0 then its z-transform is defined for _______


a) |z| > 4
b) |z| > 0
c) |z| < 4
d) |z| = 0
11) A vector function F is called solenoidal if
a) curl F = 0
b) divF = 0
c) gradF = 0

d) div F 0

12) The directional derivative of = e2xcosyz at (0, 0, 0) is


a) 2i
b) 2j
c) 2k

d) i + j + k

1
13) L1
=?
2
(s 3) + 16
e3t
e3t
sin4t
c) e3t cos4t
sin4t
b)
4
4
14) The convolution of two functions f1(t) and f2(t) is defined as
a)

a)

f1(t) f2 (t u)dt
0

b)

d) e3t cos4t

f1(u)f2 (t u)du

c)

f1(u) f2 (t u)du

d)

f1(t) f2 (t u)dt
0

15) L1{(s)} = ?
a)

L1{(s)}
t

b) t L1{(s)}

c) L1 {(s)}

d) s L1{(s)}

16) The conditions for expansion of a function in a Fourier series are known as
a) Harmonic b) Riemann conditions c) Periodic
d) Dirichlets conditions
17) Which of the following functions cannot be expanded in Fourier series in the
interval ( , )
a) e x
b) |x|
c) cosecx
d) x2
18) In the interval (0, ) the constant term in the sine series of f(x) = x is
a) 2

b)

d) 4

c) 0

19) If Fc( ) is the cosine transform of f(x), then f(x) = ?

a)

c)

1
2

Fc () cos x d

Fc () cos x d

b)

Fc ( ) cos x d

d) None of these

20) The Fourier sine transform of f(x) = ex, x 0 is


a)

s
1+ s2

s
b) .
c)
1 + s2
1 s2
______________

d)

s
1 + s2

Set A

*SLRBB370*

-3-

SLR-BB 370

Seat
No.

S.E. (Part I) (Electrical and Electronics) Examination, 2014


ENGINEERING MATHEMATICS III
Day and Date : Tuesday, 9-12-2014
Time : 10.00 a.m. to 1.00 p.m.

Marks : 80

Instructions : 1) Attempt any three questions from each Section.


2) Figures to right indicate full marks.
3) Use of calculator is allowed.
SECTION I
2. a) Solve
b) Solve

d2 y
dx2
(D2

c) Solve x2

+ 4y = x2 + cos2x.

+ 2)y = ex.cosx.

d2 y
dx

+ 2x dy
6y = x3logx.
dx

3. a) Solve (1 + x)2

d2 y
dx 2

+ (1 + x) dy
+ y = [log(1 + x)]4 + cos log(1 + x).
dx

b) An electric circuit consists of an inductance L, a condenser of capacity C and


e.m.f. E = E0 cos t, so that the charge Q satisfies the differential equation
d2 Q
dt 2

E
+ Q = o cos t . If =
CL
L

1
LC

and initially Q = Q0 at t = 0 and the current

i = i0 at t = 0. Find the charge Q at time t.


4. a) Solve p2x2 + q2y2 = z2.

7
4

b) Solve (2z y)p + (x + z)q = (2x + y).

c) Solve z(p2 q2) = x y.

5. a) Find z{2k sin (3k + 2)} k 0.

b) Find z{k2 ak 1}, k 0.

OR
2x
b) Solve (D2 + 4D + 4)y = e 5 .

z
c) Find the inverse z-transform of f(z) = (z 1)(z
, |z| > 2.
2)

Set A

SLR-BB 370

*SLRBB370*

-4-

SECTION II
6. a) Show that the vector field F given by F = (x2 yz)i + (y2 zx)j + (z2 xy)k
is irrotational and hence, find its scalar potential such that F = .
b) Find the angle between the normals to the surface xy = z2 at the points
(1, 4, 2) and (3, 3, 3).

( )

c) Prove that . rn r = (n + 3)rn where r and r have their usual meaning.

6
4
4

OR
c) Find the directional derivative of = x2y2+ y2z2 + z2x2 at (1, 1, 2).
,
x
7. a) Obtain half range sine series for f(x) =
2 x ,
2
8

1
2

1
3

1
5

1
72

0< x < 1
1< x < 2

. Hence deduce that

+ .... .

,
b) Find the Fourier series for periodic function f(x) =
x ,
8. a) Express the function, f(x) = sin x ,
=0

< x < 0
0< x <

0 x , as Fourier Sine integral and


x>

evaluate

d
.
sin 1x.sin

a
b) If Fs( ) = e
, then find f(x). Hence obtain the inverse sine transform of 1 .

s + a
9. a) Find L1 log
.
s
+
b

4
t

b) Find Laplace transform of

4u

.sin3u du .
u

c) Using convolution theorem find the inverse Laplace transform of


_____________________

s2
.
(s2 + 1) (s2 + 4)

Set A

SLR-BB 371

*SLRBB371*
Seat
No.

Set

S.E. (Electrical and Electronics Engineering) (Part I) (New)


Examination, 2014
TRANSFORMERS AND DC MACHINES
Day and Date : Thursday, 11-12-2014
Time : 10.00 a.m. to 1.00 p.m.

Max. Marks : 100

Instructions : 1) Q. No. 1 is compulsory. It should be solved in first 30 minutes


in Answer Book Page No. 3. Each question carries one mark.
2) Answer MCQ/Objective type questions on Page No. 3 only.
Dont forget to mention, Q.P. Set (A/B/C/D) on Top of Page.
MCQ/Objective Type Questions
Duration : 30 Minutes

Marks : 20

1. Fill in the blanks :

20

1) In DC generator armature reaction is produced actually by


a) Its field current
b) Field pole winding
c) Armature conductor
d) Load current in armature
2) Back E.M.F. in a DC motor
a) Opposes the applied voltage
b) Aids the applied voltage
c) Aids the armature current
d) None of the above
3) ___________ motor should be never started at no load.
a) Series
b) Shunt
c) Long compound

d) Short compound

4) Which of the load would be best driven by DC compound motor ?


a) Reciprocating pump
b) Electric locomotive
c) Centrifugal pump
d) Fan
5) The speed of DC motor can be controlled by varying
a) Its flux per pole
b) Resistance of armature circuit
c) Applied voltage
d) All of the above
6) In DC generator the effect of armature reaction on the main pole flux is to
a) Reduce it
b) Distort it
c) Reverse it
d) Both a) and b)
7) Which of the following is power equation ?
a) VIa = EbIa + Ia ^2Ra
b) V = EbIa + Ia^2Ra
c) Ia^2V = EbIa^2 + Ia^2Ra
d) V = Eb + IaRa
8) The E.M.F. induced in dc machine is ______________ E.M.F.
a) dynamically
b) statically
c) both a) and b) d) none of the above
P.T.O.

SLR-BB 371

-2-

*SLRBB371*

9) In dc motor field control method is used to control speed ________ rated one.
a) below
b) above
c) both a) and b) d) none of these
10) The primary reason for providing compensating winding in a DC generator is to
a) Compensate for decrease in main flux
b) Neutralize armature M.M.F.
c) Neutralize cross-magnetizing flux
d) Maintain uniform flux distribution
11) A transformer transfers electric power from primary to secondary usually
with change in
a) Frequency
b) Voltage
c) Power
d) Time period
12) The transformer has a full load Cu loss of 400 W, the Cu loss at half load is
a) 100W
b) 1000W
c) 200W
d) 400W
13) The efficiency of single phase transformer is given by
a) = (VA rating*cos )/[(VA rating*cos ) + Pi + Pcu]
b) = (V2I2cos )/[(V2I2cos ) + Pi + Pcu]
c) = (VA rating*cos )/[(VA rating*cos ) + Pi ^ 2+]
d) Both a) and b)
14) A universal motor is one which
a) Is available universally
b) Can be marketed internationally
c) Can be operated either on DC or AC supply
d) Runs at dangerously high speed on no load
15) A transformer having 1000 primary turns is connected to 200V AC supply,
for a secondary voltage of 400 V, the no. of turns for the secondary should be
a) 1600
b) 2000
c) 2500
d) 1250
16) Back to back test of transformer is also called as ________ test.
a) break
b) swinburn
c) heat run
d) sumpner
17) In parallel operation of 3 ph transformer it is essential to have ________
phase sequence.
a) 120
b) 90
c) same
d) opposite
18) The primary and secondary of a transformer are ________ coupled.
a) Electrically
b) Magnetically
c) Electrically and Magnetically
d) None of the above
19) Transformer works on the principle of
a) Self inductance
b) Mutual inductance
c) Saturated inductance
d) Leakage inductance
20) For connection DD0(delta-delta zero) there will be _________ phase difference
between primary and secondary.
a) 0
b) 180
c) +30
d) 30
______________
Set A

*SLRBB371*

-3-

SLR-BB 371

Seat
No.

S.E. (Electrical and Electronics Engineering) (Part I) (New)


Examination, 2014
TRANSFORMERS AND DC MACHINES
Day and Date : Thursday, 11-12-2014
Time : 10.00 a.m. to 1.00 p.m.

Marks : 80

SECTION I
2. Solve any four :

20

a) Draw and explain the power stages of a DC machine.


b) Why starter is necessary ? Explain three point starter.
c) With neat diagram explain simplex lap and simplex wave winding.
d) How Brake test is performed on DC shunt motor to calculate its efficiency ?
e) A 230V DC series motor is taking 50A. Resistance of armature and series
field winding is 0.45 and 0.5 respectively. Calculate.
a) Back E.M.F.
b) Power wasted in armature
c) Mechanical power developed.
3. Solve any two :

20

a) Draw and explain torque speed, armature current-torque and armature


current-speed characteristics of d.c. shunt and series motor.

b) A 250V, 14.9KW, 8 pole D.C. motor has single turn coils. The armature is
wave would with 94 commutator segments. If the brushes are given a lead
of 2 commutator segments at full load, calculate :
a) Total armature reaction ampere turns per pole.
b) Cross magnetizing ampere turns per pole. Assume efficiency of 80%.
c) Which are the different types of DC motor ? Explain any two. And also write
down the equations for the armature current and back e.m.f. for any two
types of motor.
Set A

SLR-BB 371

-4-

*SLRBB371*

SECTION II
4. Solve any four :

20

a) Why transformer is called constant flux machine ?


b) Explain operation of Universal Motor on both AC and DC.
c) Draw phaser diagram of single phase transformer on resistive and capacitive
load.
d) Explain the construction and working principle of single phase transformer.
e) What is ideal transformer ? Derive the emf equation for single phase
transformer.
f) In a 50KVA transformer, the iron loss is 500W and full load Cu loss is 800W.
Find the efficiency at full load and half load at 0.8 p.f. lagging.
6. Solve any two :

20

a) Explain use of transformer having vector group YY 0, DY 1, DD0 and DY 11.


Draw the vector group with connection diagram for each group.
b) Two single phase transformers with equal turns have impedance of (0.5 + j3) ohm
and (0.6 + j10) ohm w.r.t. the secondary. If they operate in parallel, determine
how they will share total load of 2000KW at pf. 0.8 lagging.
c) A 100KVA, 3 , 50Hz, 3300/400V transformer is delta connected on the H.V .
side and star connected on the L.V. side. The resistance of the H.V. winding
is 3.5 per phase and that of L.V. winding 0.02 per phase. Calculate the
iron losses of the transformer at normal voltage and frequency if its full load
efficiency is 95.8% at 0.8 p.f. lagging.
_____________________

Set A

SLR-BB 372

*SLR-BB-372*
Seat
No.

Set

S.E. (Part I) (Electrical and Electronics Engg.) Examination, 2014


ELECTRICAL NETWORK
Day and Date : Saturday, 13-12-2014
Time : 10.00 a.m. to 1.00 p.m.

Max. Marks : 100

Instructions : 1) Assume the suitable data whenever necessary.


2) Q. No. 1 is compulsory. It should be solved in first 30 minutes in
Answer Book Page No. 3. Each question carries one mark.
3) Attempt any two questions from each Section.
4) Answer MCQ/Objective type questions on Page No. 3 only.
Dont forget to mention, Q.P. Set (A/B/C/D) on Top of Page.
MCQ/Objective Type Questions
Duration : 30 Minutes

Marks : 20

1. Choose the correct answer :


1) An electric current of 5A is same as
a) 5 J/C
b) 5 V/C

20
c) 5 C/sec.

d) 5 w/sec.

2) Two resistances R1 and R2 give combined resistance of 4.5 ohms when in series
and 1 ohm when in parallel. The resistances are
a) 2.5 ohms and 2 ohms
b) 3.5 ohms and 1 ohms
c) 1.5 ohms and 3 ohms
d) 1.5 ohms and 0.5 ohms
3) In order to get maximum power transfer from a capacitive source, the load must
a) Have a capacitive reactance equal to circuit resistance
b) Have impedance that is the complex conjugate of the source impedance
c) Be as capacitive as it is inductive
d) None of the above
4) To determine the polarity of the voltage drop across a resistor, it is necessary to
know
a) Value of current through the resistor
b) Direction of current through the resistor
c) Value of resistor
d) None of the above
5) Which of the following is non-linear circuit parameter ?
a) Inductance b) Transistor
c) Wire wound resistor d) Condenser
6) In a series parallel circuit, any two resistances in the same current path must be in
a) Series with the voltage source
b) Parallel with each other
c) Series with each other
d) Parallel with the voltage source
P.T.O.

SLR-BB 372

*SLR-BB-372*

-2-

7) Millmans theorem yields


a) Equivalent resistance
c) Equivalent voltage source

b) Equivalent impedance
d) Equivalent voltage or current source

8) For three phase star connected circuit


a) Line voltage = 3 phase voltage
c) Line current = 3 phase current

b) Line voltage = phase voltage


d) None of the above

9) At t = 0+ with zero initial condition, which of the following acts as short circuit ?
a) Inductor
b) Capacitor
c) Resistor
d) All of the above
10) Time constant of an inductive circuit
a) Decreases with increase of inductance and decrease of resistance
b) Decreases with the increase of inductance and the increase of resistance
c) Decreases with decrease of inductance and decrease of resistance
d) Decreases with decrease of inductance and increase of resistance
11) The transient currents are associated with the
a) Changes in the stored energy in the inductors and capacitors
b) Impedance of the circuit
c) Applied voltage to the circuit
d) Resistance of the circuit
12) Kirchhoffs current law is related to
a) Junction currents b) Battery E.M.Fs c) Junction voltages d) None of the above
13) In a series resonant circuit, the impedance of the circuit is
a) Minimum
b) Maximum
c) Zero

d) None of the above

14) In R-L-C series resonant circuit magnitude of resonance frequency can be changed
by changing the value of
a) R,L or C
b) L only
c) C only
d) L or C
15) The circuit has resistors, capacitors and semi-conductor diodes. The circuit will
be known as
a) Non-linear circuit
b) Linear circuit
c) Bilateral circuit
d) None of the above
16) In series resonance reactance below resonant frequency is
a) Capacitive
b) Resistive
c) Inductive
d) None of the above
17) In series RLC circuit which phasers are in phase opposition.
a) VL, VC
b) VL, VR
c) VC , VR
d) None of the above
18) Second order circuit is over damped when
a) LC>4R2C2
b) LC=4R2C2
c) LC<4R2C2
eat cos t

19) Laplace transform of


is
2
2
2
a) (s+a)/[(s+a) + ] b) a/[(s+a) + 2]

c) /[(s+a)2+ 2]

d) None of the above


d) None of the above

20) The power at half power frequencies is ___________ time power at resonance.
a) Same
b) Half
c) 0.637
d) 0.707
______________
Set A

*SLR-BB-372*

-3-

SLR-BB 372

Seat
No.

S.E. (Part I) (Electrical and Electronics Engg.) Examination, 2014


ELECTRICAL NETWORK
Day and Date : Saturday, 13-12-2014
Time : 10.00 a.m. to 1.00 p.m.

Marks : 80

Instructions : 1) Assume the suitable data whenever necessary.


2) Attempt any two questions from each Section.
SECTION I
2. Solve any four :
a) Show that Rs = RL in maximum power transfer theorem.

(54=20)

b) Determine current through the 5 resistor using Thevenins theorem.

c) Find the value of resistance in between the terminals a-b

d) Find power dissipated by 70 resistor.

e) Write a note on Principle of Duality.


f) State and explain Millmans theorem.
Set A

SLR-BB 372

-4-

*SLR-BB-372*

3. Solve any two :

(102=20)

a) Find the Thevenins and Nortons equivalent circuit.

b) Find Vo in the circuit using node analysis

c) Using Superposition theorem find V6

SECTION II
4. Solve any four :
1) Derive an expression for response given by RL circuit.
2) Derive H-parameter in terms of Z and transmission parameters.

20

3) A series RLC circuit has following parameters. R = 100 , L= 0.02H, C = 0.02 F


when a variable frequency voltage of 50V is applied to it, calculate resonant
frequency, frequencies at which voltage across L and C is maximum, maximum
current ?
Set A

*SLR-BB-372*

-5-

SLR-BB 372

4) For the network shown find the Y-Parameter and Z-Parameter.

5) Show that 0 = (1 * 2 )1/ 2 for series resonance.


6) The switch in the network is open at t=0. Find iL (t) for t >0 and t = 10 msec.

5. Solve any two :

20

1) Derive an expression of frequencies for which voltage across inductor and


capacitor is Maximum at resonant condition.
2) Determine the expression for VL(t) in the network by Laplace transform, when

A) Vs(t) = (t)

B) Vs(t) = et.

3) The switch in the circuit is closed at t=0. Find Vc(t).

_____________________

Set A

Set A

SLR-BB 373

*SLRBB373*
Seat
No.

Set

S.E. (Electrical and Electronics Engg.) (Part I) Examination, 2014


ANALOG ELECTRONICS
Day and Date : Tuesday, 16-12-2014
Time : 10.00 a.m. to 1.00 p.m.

Total Marks : 100

Instructions : 1) Q. No. 1 is compulsory. It should be solved in first


30 minutes in Answer Book Page No. 3. Each question carries
one mark.
2) Answer MCQ/Objective type questions on Page No. 3 only.
Dont forget to mention, Q.P. Set (A/B/C/D) on Top of Page.
MCQ/Objective Type Questions
Duration : 30 Minutes

Marks : 20

1. Choose the correct answer :


1) MOSFET can be used as a
a) Current controlled capacitor
c) Current controlled inductor

20
b) Voltage controlled capacitor
d) Voltage controlled inductors

2) The ideal Op-amp has the following characteristics.


a) Ri = , A = , R0 = 0
b) Ri = 0, A = , R0 = 0
d) Ri = 0, A = , R0 =
c) Ri = , A = , R0 =
3) Slew rate is defined by _____________
a) dv / dt (max)
b) di/dt (max)
c) Both a) and b)

d) None of the above

4) The main function of clipping circuit is to


a) Remove certain portion of input signal above or below certain level
b) Suppress amplitude variation in input signal
c) Restore dc value of signal
d) Both a) and b)
5) When Q point is at center of ac load line the maximum peak to peak output
voltage excess
a) VCEQ

b) 2VCEQ

c)

1
2

VCEQ

d) (VCEQ)2

6) The output of class B amplifier


a) Is distortion free
b) Consist of positive half cycle only
c) Is like a output of full wave rectifier
d) Comprise short duration of current pulses
7) The UJT has
a) One emitter and two base
c) Anode, cathode and one gate

b) Two emitter and one base


d) Two anode and one gate
P.T.O.

SLR-BB 373

*SLRBB373*

-2-

8) The pin numbers used for output offset voltage null are
a) Pin 2 and pin 3
b) Pin 1 and pin 5 c) Pin 7 and pin 4

d) None of above

9) The monostable multivibrator


a) Gives one output pulse for one input trigger pulse
b) Gives two output pulse for one input trigger pulse
c) Has one stable state
d) Both a) and b)
10) Two input terminal of op-amp are
a) Positive and negative
c) Inverting and non inverting

b) Differential and non-differential


d) High and low

11) The main job of voltage regulator is to provide a nearly __________ output voltage.
a) Sinusoidal
b) Constant
c) Smooth
d) Fluctuating
12) The change in op-amp i/p offset voltage caused by variation in supply voltage is
a) SVRR
b) PSS
c) PSRR
d) All of above
13) The pin number 7 of IC 555 is
a) Reset
b) Discharge

c) Trigger

d) Ground

14) When transistor is used as switch its operation is configured in


a) Saturation region b) Cut-off region c) Both a) and b) d) Active region
15) The efficiency of full wave rectifier
a) 40.6%
b) 81.2%

c) 50%

16) A transistor has


a) One P-N junction b) Two P-N junction c) Three P-N junction

d) 100%
d) Four P-N junction

17) The best method for locating Q-point at center of load line
a) Fixed bias method
b) Voltage divider bias method
c) Emitter bias method
d) None of above
18) Refer to the given figure. This circuit is known as

a) Non-inverting amplifier
c) An integrator

b) A differentiator
d) A summing amplifier

19) The output of a Schmitt trigger is a


a) Pulse waveform
c) Sinusoidal waveform

b) Saw tooth waveform


d) Triangle waveform

20) In class B amplifier the current in output circuit flows for


a) 360
b) 180
c) 90

______________

d) Less than 90

Set A

*SLRBB373*

-3-

SLR-BB 373

Seat
No.

S.E. (Electrical and Electronics Engg.) (Part I) Examination, 2014


ANALOG ELECTRONICS
Day and Date : Tuesday, 16-12-2014
Time : 10.00 a.m. to 1.00 p.m.

Marks : 80

Instructions : 1) All questions are compulsory.


2) Figure to the right indicates maximum marks.
3) Assume the suitable data whenever necessary.
SECTION I
2. Solve any four :

20

1) Explain full wave rectifier in detail.


2) Explain transistor as an amplifier.
3) Derive and explain emitter bias configuration for BJT.
4) Define clipper. Enlist its types. Explain any one in detail.
5) Explain feedback pair connection.
6) Calculate the DC bias voltage and currents in the circuit.

3. Solve any two :

20

1) Describe construction and operation of depletion type MOSFET. Also draw their
static characteristics.
2) Explain class C and class D amplifier.

Set A

SLR-BB 373

-4-

*SLRBB373*

3) Determine the following for emitter bias network.


a) IB

b) IC

c) VCE

d) VC

e) VE

f) VB

g) VBC

SECTION II
4. Solve any four :

20

1) What is op-amp ? Explain block diagram of op-amp.


2) Explain Schmitt trigger with circuit diagram.
3) Explain ZCD comparator with neat diagram.
4) Explain differentiator circuit with derivation.
5) Explain frequency response of Op-amp.
6) Explain phase shifter with neat diagram.
5. Solve any two :

20

1) Explain IC 555 as monostable multivibrator.


2) Explain following terms :
a) CMRR

b) Input offset voltage

c) Slew rate

d) Thermal drift

e) SVRR
3) Explain instrumentation amplifier with neat circuit diagram.

_____________________

Set A

SLR-BB 374

*SLRBB374*
Seat
No.

Set

S.E. (Electrical and Electronics Engg.) (Part I) Examination, 2014


ELECTRICAL POWER GENERATION
Day and Date : Thursday, 18-12-2014
Time : 10.00 a.m. to 1.00 p.m.

Total Marks : 100

Instructions : 1) Figure to the right indicates maximum marks.


2) Assume the suitable data whenever necessary.
3) Q. No. 1 is compulsory. It should be solved in first 30 minutes in
Answer book Page No. 3. Each question carries one mark.
4) Answer MCQ/Objective type questions on Page No. 3 only. Dont
forget to mention, Q.P. Set (A/B/C/D) on Top of Page.
MCQ/Objective Type Questions
Duration : 30 Minutes

Marks : 20

1. Choose the correct answer :

20

1) A graphical representation between discharge and time is known as


A) Monograph
B) Hectograph
C) Topograph
D) Hydrograph
2) A Francis turbine is
A) Inward flow reaction turbine
C) Outward flow reaction turbine

B) Inward flow impulse turbine


D) Outward flow impulse turbine

3) In a super-heater
A) Pressure raises, temperature drops
B) Pressure raises, temperature remains constant
C) Pressure remains constant and temperature rises
D) Both pressure and temperature remains constant
4) Which of the following material can be used as a moderator ?
A) Graphite
B) Heavy water
C) Beryllium

D) Any of the above

5) Particles having the same atomic number but different mass numbers arc called
A) Positrons
B) Beta particles
C) Isotopes
D) Decayed particles
6) In a thermal power plant, heat from the flue gases is recovered in
A) Chimney
B) De-super heater
C) Economizer
D) Condenser
7) For the same plant size, initial cost of which plant is highest ?
A) Steam power plant
B) Diesel engine plant
C) Nuclear power plant
D) Gas turbine plant
8) The path followed by the gases discharged from chimney called the plume, depends on
A) Thermal properties of gases
B) Dynamic properties of gases
C) Wind direction
D) All of the above

P.T.O.

SLR-BB 374

*SLRBB374*

-2-

9) Pulverized coal is
A) Coal free from ash
C) Coal which burns for long time

B) Non-smoking coal
D) Coal broken into fine particles

10) For low head and high discharge, the hydraulic turbine used is
A) Kaplan turbine
B) Francis turbine
C) Pelton wheel
D) Jonual turbine
11) A two stroke engine may be identified by
A) Piston size
C) Cooling system

B) Absence of valves
D) Lubrication system

12) An air filter is used in


A) Nuclear power plants
C) Diesel engine power plants

B) Steam power plants


D) Hydro-power plants

13) Photovoltaic solar energy conversion system makes use of


A) Fuel cell
B) Solar cell
C) Solar pond

D) None of the above

14) Tidal energy mainly makes use of


A) Kinetic energy of water
B) Potential energy of water
C) Both kinetic as well as potential energy of water
D) None of the above
15) In geothermal power plants waste water is
A) Re-circulated after cooling in cooling tower
B) Discharged into sea
C) Discharged back to earth
D) Evaporated in ponds
16) In a 2 stroke engine there is one power stroke in
A) 90 of crank rotation
B) 180 of crank rotation
C) 360 of crank rotation
D) 720 of crank rotation
17) In a fuel cell positive electrode is of
A) Oxygen
B) Ammonia

C) Hydrogen

D) Carbon monoxide

18) The ratio maximum demand of the installation to the sum of individual maximum demands
is known as
A) Demand factor
B) Plant use factor
C) Diversity factor
D) Plant capacity factor
19) Which meter is installed at the premises of a consumer for recovery of charges of electrical
energy ?
A) Voltmeter
B) Ammeter
C) KVA meter
D) KWH meter
20) Local winds are caused by
A) Differential heating of land and water
B) Differential heating of plains and mountains
C) Any of the above
D) None of the above
______________

Set A

*SLRBB374*

SLR-BB 374

-3-

Seat
No.

S.E. (Electrical and Electronics Engg.) (Part I) Examination, 2014


ELECTRICAL POWER GENERATION
Day and Date : Thursday, 18-12-2014
Time : 10.00 a.m. to 1.00 p.m.
Instructions :

Marks : 80

1) All questions are compulsory.


2) Figure to the right indicates maximum marks.
3) Assume the suitable data whenever necessary.
SECTION I

2. Solve any four :

(54=20)

1) Write a short note on hydrology.


2) Explain pelton turbine with neat diagram.
3) Discuss factors to be considered for selection of site for thermal power plant.
4) Explain the process of nuclear fission.
5) Give classification of hydroelectric power plant on the basis of head.
6) Explain boiling water reactor with neat diagram.
3. Solve any two :

(102=20)

1) Draw typical layout of thermal power plant and explain it briefly.


2) Draw typical layout of hydroelectric power plant and explain it briefly.
3) Explain single line diagram of typical AC power system.
SECTION II
4. Solve any four :

(54=20)

1) Explain construction and working of solar photovoltaic cell.


2) Explain geo-thermal power plant with neat diagram also state its application.
3) Explain construction and working of fuel cells.
4) Explain load duration curve with example.
5) State application of diesel power stations.
6) A generating station has the following load cycle :
Time (Hours)
Load (MW)

06

6 10

10 12

12 16

16 20

20 24

40

50

60

50

70

40
Set A

SLR-BB 374

*SLRBB374*

-4-

Draw the load curve and find :


i) Maximum demand
ii) Units generated per day
iii) Average load
iv) Load factor
5. Solve any two :

(102=20)

1) With a neat diagram explain tidal power plant with its advantages and limitations.
2) What is operating principle of wind power plant ? With the help of block diagram explain
wind power plant.
3) A group of 2 consumers has the following electricity demand pattern on a typical winter
day :
Consumer A : connected load
Load from 12 midnight to 5 am

2.5 KW
100 W

From 5 am to 6 am

1.1 KW

From 6 am to 8 am

200 W

From 8 am to 5 pm

Nil

From 5 pm to 12 midnight

500 W

Consumer B : connected load


Load from 11 pm to 7 am

3 KW
Nil

From 7 am to 8 am

300 W

From 8 am to 10 am

1 KW

From 10 am to 6 am

200 W

From 6 pm to 11 pm

600 W

a) Calculate the demand factor of both the consumers.


b) Plot the variations in demand v/s time of the day for each consumer and group.
c) Find the group diversity factor.
d) Find energy consumed by each consumer in 24 hours.
e) Find the maximum energy which each consumer would consume in 24 hours if his
load were constant and equal to his maximum demand.
f) Find the ratio of actual energy to maximum energy for each.
_____________________

Set A

SLR-BB 375

*SLRBB375*
S

S.E. (Part II) (Electrical and Electronics) Examination, 2014


LINEAR ALGEBRA (Old)
Day and Date : Tuesday, 25-11-2014
Time : 3.00 p.m. to 6.00 p.m.

Max. Marks : 100

Instructions : 1) Q. No. 1 is compulsory. It should be solved in first 30 minutes in Answer


book Page No. 3. Each question carries one mark.
2) Attempt any three questions from each Section.
3) Figures to the right indicate full marks.
4) Use of non programmable calculator is allowed.
5) Answer MCQ/Objective type questions on Page No. 3 only. Dont forget
to mention, Q.P. Set (A/B/C/D) on Top of Page.
MCQ/Objective Type Questions

Duration : 30 Minutes

Marks : 20

1. Tick mark the correct alternative :




(201=20)

1) Rank of the matrix

is

a) 1

b) 2

c) 3

d) None of these

2) If T : R3
R2, T(e1) = (1, 3), T(e2) = (4, 7) and T(e3) = (5, 4), where e1, e2, e3 are columns
of identity matrix then standard matrix of T is


A





a)

"

b)

c)

"

d) None of these

"




"

!


3) Choose the correct alternative for a matrix Am n


a) rank A + dim column = m
b) rank A + dim Null A = m
c) rank A + dim column = n
d) rank A + dim Null A = n



%

~


&

then dimensions for Null A is




"

4) If A =

"

a) 03

b) 02

c) 01

d) None of these

5) The system AX = B, where A is a square matrix has a unique solution if |A| = __________
a) 0
b) non zero
c) 1
d) none of these



6) The eigen values of A =

"


are

a) 2, 0, 1

b) 0, 1, 3

7) State which of the following is false


a) A is invertible iff |A|
0
c) |A T | = |A|


c) 1, 4, 7

d) 2, 1, 3

b) |AB| = |A| |B|


d) |A B| = |A|+|B|


P.T.O.

SLR-BB 375

*SLRBB375*

-2-

8) If a vector space V has a basis of n-vectors then every basis of V must consist
a) less than n vectors
b) 2n vectors
c) exactly n vectors
d) greater than n vectors
9) Given a scalar r the transformation T : R2
a) dilation
b) contraction
10) Given that

is an eigen vector of




R2 defined by T(x) = rx, when 0 r 1 is called


c) translation
d) none of these


, what is the corresponding eigen value ?




a) 3

b) 2

11) The transformation W =

c) 7

d) 3

is
1

a) translation
c) inversion

b) rotation and magnification


d) none of these

12) The

dZ where C is the circle |Z| = 2 is equal to


3

a)

b)

c) 2

d) none of these

13) The transformation W =


a

a) ad bc = 1

is said to be normalized bilinear transformation if

b) ad bc = 0

c) ad + bc = 1

d) none of these

byx)2

d) none of these

14) The co-efficient of correlation r is given by


a)
>

15) If u =
a) [0, 4, 6]


b) bxy

>

and v =


c) 15

c)

d) none of these

d)

O
O

17) For the two lines of regressions 8x 10y = 66, 40x 18y = 214, the mean
N

N
N

a)

then u v is


c) (bxy

b) 10

16) Cauchy-Riemann equations are


a)
b)
K

byx


= 10,
O

= 12

b)
N

= 12,
O

= 10

c)

= 13,
N

18) A function which satisfies Laplaces equation is called


a) Analytic
b) Halomorphic
c) Harmonic

= 17

d)

and
N

are given by

= 17,
O

= 13

d) Non-Harmonic

19) Let u be a non-zero vector. Then a unit vector in direction of u is


a) || u ||

b) || u ||


c)
K

d) None of these

20) The coefficient of correlation


a) can not be negative
c) can not be positive

b) can be positive as well as negative


d) is always positive
______________

Set A

*SLRBB375*
S

SLR-BB 375

-3-

S.E. (Part II) (Electrical and Electronics) Examination, 2014


LINEAR ALGEBRA (Old)

Day and Date : Tuesday, 25-11-2014


Time : 3.00 p.m. to 6.00 p.m.
Instructions :

Marks : 80

1) Attempt any three questions from each Section.


2) Figures to the right indicate full marks.
3) Use of non programmable calculator is allowed.
SECTION I

2.

a) Solve the equation Ax = b by using LU factorization for given A




A=


,b=



$

"

, A = LU =








b) For what value of h will y be span of {v1, v2, v3 } if v1 =




, v2 =





"

, v3 =


3x1 7x2 + 7x3 = 8;

and y =


"

. 5


4x1 + 6x2 x3 = 7.
%

&

3.

5


c) Solve : x1 3x2 + 4x3 = 4 ;




a) Determine whether the columns of given matrix A =

"

span R4.

'


$

'

b) Find the general traffic pattern in the free way network shown in the figure




c) Let a1 =





, a2 =


, and b =


. Determine whether b can be generated as a




"




linear combination of a1 and a2.





4.

a) With T defined by T(x) = Ax, find a vector x whose image under T is b, where A =


,b=



!


!

.5





b) If A =

"


"

'

'

.Find rank of A and dim Null A.

"




Hence find bases for Co1A, Null A.

5
5


c) Check whether the following vectors are linearly independent. Justify your answer.

"

#


"

4


with X0 =

a) Diagonalise the matrix, if possible ? A =




b) Apply power method to A =


4





,


,

5.

. Stop when K = 5 and estimate the




dominant eigen value of a corresponding eigen vector of A.

7
Set A

SLR-BB 375

*SLRBB375*

-4-

SECTION II
6.

a) Write down the quadratic form corresponding to the following matrices





i)

ii)




b) Consider the set of vectors S in R4 as S = {u1, u2, u3} where u1 =

, u2 =




,











u3 =

, u4 =

. Show that S is orthogonal.

'

c) Is Q(x) = 3
7.

8.

,5

+3
N

2x2x3 + 2x3x1 2x1x2, positive definite ?

a) Find the equatoins of lines of regression and coefficient of correlation from the following data :
x : 61
68
68 64
65 70
63
62
64
77
y : 112 123 130 115 110 125 100 113 116 126

b) Calcualte the coefficient of correlation from the following data :


x : 64
65
66 67
68 69
70
y : 66
67
65 68
50 68
71

a) Find the bilinear transformation which maps the points Z = 1, i, 1 into the points W = 2, i, 2.

b) Show that W =

N
N

c) Evaluate


is an analytic function and find


O

.
w

O
d

(x2 iy)dy along the path

ii) y = x2.

i) y = x
9.

a) Following are the equations of lines of regression 6y = 5x + 90 and 15x = 8y + 130. Find
the means
N

and
O

and the coefficient of correlation.

b) Given
X series
Y series
Mean
18
100
S. D.
14
20
and r = 0.8.
Find the equations of the lines of regression. Find out the most probable value of y if x is 70 and
the most probable value of x if y is 90.

c) Find the image of Z =

"

'

under the transformation W =

'

_____________________

Set A

*SLRBB376*
Seat
No.

SLR-BB 376
Set

S.E. (Electrical and Electronics Engg.) (Part II) (Old) Examination, 2014
AC MACHINES
Day and Date : Wednesday, 26-11-2014
Time : 3.00 p.m. to 6.00 p.m.

Total Marks : 100

Instructions : 1) All questions are compulsory.


2) Assume the suitable data whenever necessary.
3) Q. No. 1 is compulsory. It should be solved in first 30 minutes
in Answer book Page No. 3. Each question carries one mark.
4) Answer MCQ/Objective type questions on Page No. 3 only.
Dont forget to mention, Q.P. Set (A/B/C/D) on Top of Page.
MCQ/Objective Type Questions
Duration : 30 Minutes

Marks : 20

1. Solve the following :


(120=20)
1) When load is placed on a 3-phase induction motor, its rotor induced EMF,
a) Increases
b) Decreases c) Remain constant d) None of the above
2) The torque developed by a 3-phase induction motor depends on the following
three factors.
a) Speed, frequency, number of poles
b) Voltage, current and stator impedance
c) Synchronous speed, rotor speed and frequency
d) Rotor EMF, rotor current and rotor P.F.
3) If the stator voltage and frequency of an induction motor are reduced proportionately,
its
a) Locked rotor current is reduced
b) Torque developed is increased
c) Both a) and b)
d) Magnetising current is decreased
4) The efficiency and P.F. of a SCIM increases in proportion to its
a) Speed
b) Mechanical load c) Voltage
d) Rotor torque
5) Under which of the following starting methods an induction motor draws high
starting currents
a) Star-delta starter
b) Auto transformer starter
c) Direct on line starter
d) Reduced voltage starter
6) A Double cage induction motor has,
a) Two series windings in stator
b) Two series windings in rotor
c) Two parallel windings in rotor
d) None of the above
7) Slip ring induction motors have the advantage of
a) High starting torque and high over load capacity
b) Very rugged construction
c) Cheap in cost
d) Both a) and b)
P.T.O.

SLR-BB 376

-2-

*SLRBB376*

8) As compared to other single phase ac motors a universal motor has,


a) Speed ratio of 2:1 on ac/dc
b) Highest efficiency on 50 Hz supply
c) High horse power/kg ratio
d) All of the above
9) The disadvantage of shaded pole motor is
a) Low starting torque
b) Low efficiency
c) Very little over load capacity
d) All of the above
10) The starting torque of a capacitor start motor is
a) Zero
b) Low
c) Same as rated torque
d) More than rated torque
11) When an alternator is supplying unity power factor load, the armature reaction will
produce
a) Magnetization of the main field
b) Demagnetization of the main field
c) Distortion of the main field
d) None of the above
12) For the same power rating, a lower voltage alternator will be
a) More efficient
b) Larger in size
c) Operating at high rpm
d) More costly
13) A 50 Hz alternator will run at the greatest speed, if it is wound for _________
poles.
a) 2
b) 4
c) 6
d) 8
14) The angle between no load EMF and terminal voltage in an alternator is called as
__________ angle.
a) Synchronizing b) Torque
c) Power
d) None of the above
15) If the numbers of poles of an alternator are increased, the generated EMF will be,
a) Increases b) Decreases c) Remain constant d) None of the above
16) Which motor can conveniently operate on lagging as well as leading power factor ?
a) Squirrel cage induction motor
b) Wound rotor induction motor
c) Synchronous motor
d) Any of the above
17) A synchronous motor working on leading power factor and not driving any
mechanical, is known
a) Synchronous induction motor
b) Spinning motor
c) Synchronous condenser
d) None of the above
18) The constant speed of a synchronous motor can be changed to new fixed value by
a) Changing the applied voltage
b) Interchanging any two phases
c) Changing the load
d) Changing the frequency of supply
19) In a synchronous motor, the torque angle is
a) The angle between the rotating stator flux and rotor poles
b) The angle between magnetizing current and back EMF
c) The angle between the supply voltage and the back EMF
d) None of the above
20) The hunting in a synchronous motor takes place when
a) Friction in bearings is more
b) Air gap is less
c) Load is variable
d) Load is constant

______________

Set A

*SLRBB376*

-3-

SLR-BB 376

Seat
No.

S.E. (Electrical and Electronics Engg.) (Part II) (Old) Examination, 2014
AC MACHINES
Day and Date : Wednesday, 26-11-2014
Time : 3.00 p.m. to 6.00 p.m.

Marks : 80

Instructions : 1) All questions are compulsory.


2) Assume the suitable data whenever necessary.
SECTION I
2. Solve any four :

(45=20)

a) Find the condition for maximum starting torque of 3 O


/ induction motor.
b) A 50 Hz, split phase induction motor has resistance of 5 and inductive reactance
of 20 in main winding as well as starting winding. Determine the value of
resistance and capacitance to be added in series with starting winding to send
same current in each winding with phase difference of 90.
Draw and explain power stage diagram of 3 O
/ induction motor.
c) With neat diagram explain auto transformer starter of 3 O
/ induction motor.
d) Draw and explain an equivalent circuit of 3 O
/ induction motor.
e) A 6 pole, 3 O
/ , 50 Hz induction motor takes 60A at full load speed of 940 rpm
develops a torque of 150 N-m. starting current at rated voltage is 300A. What is starting
torque ? If star-delta starter is used, determine the starting torque ?
f) Explain the construction and working of universal motor. State its advantages.
3. Solve any two :

(210=20)

a) Describe the procedure to draw circle diagram to find out various parameters.
b) Explain the principle of operation of 3 O
/ induction motor. With the help of phasor
diagram explain how rotating magnetic field is produced ?
c) A 240 V, 2 pole, 50 Hz, 1O
/ induction motor has following parameter refer to stator
R1 = 2.2 , X1 = 3 , R2= 3.8 , X2= 2.1 , Xm = 86 .
If iron and frictional losses are 50 W and motor operate at full load speed of 2820 rpm
using approximate circuit. Find stator current and P.F., power input, total mechanical
power developed, useful shaft power, shaft torque, efficiency.

Set A

SLR-BB 376

*SLRBB376*

-4-

SECTION II
4. Solve any four :

(45=20)

a) Distinguish between salient and cylindrical rotor of an alternator.


b) Derive an expression for power equation of salient pole alternator.
c) A 12 pole, 3 O
/ , star connected alternator has 72 slots flux/pole is 0.0988 wb.
calculate,
1) Speed if frequency of generated EMF is 50 Hz.
2) Terminal EMF for full-pitched coils and 8 conductor/slot.
3) The terminal EMF if the coil span is reduced to 2/3 of pole pitch.
d) Explain synchronous condenser with phaser diagram. State its advantages and
disadvantages.
e) A 2 pole, 50 Hz, 3 O
/ , star connected turbo-alternator is excited to generate the
busbar voltage of 11 KV at no load. Calculate synchronizing power/degree
mechanical displacement of rotor, synchronizing torque. The machine has short
circuit current for this excitation is 1200A.
f) Explain the need of synchronization of alternators. State different conditions of
synchronization.
5. Solve any two :

(210=20)

a) A 50 KVA, 500V, 1 O
/ alternator gave the following data for O.C.C. and S.C.C.
Field Current (A)

10

15

20

25

30

EMF(V)

125

250

370

480

566

640

S.C. Armature Current (A) :

73

146

220

Using MMF method find full load voltage regulation at


1) Unity P.F.
2) 0.8 P.F. lagging. Effective armature resistance is 0.2 .
b) Explain the following terms related to alternator. Armature resistance, armature
leakage reactance, armature reaction, pitch factor, distribution factor.
c) With neat vector diagram explain V and inverted V curves of synchronous motor.
Draw both the curve.
_____________________

Set A

SLR-BB 377

*SLRBB377*
Seat
No.

Set
S.E. (Electrical and Electronics Engg.) (Part II) (Old) Examination, 2014
ELECTRICAL MEASUREMENT

Day and Date : Thursday, 27-11-2014


Time : 3.00 p.m. to 6.00 p.m.

Total Marks : 100

Instructions : 1) Figure to the right indicate maximum marks.


2) Assume the suitable data whenever necessary.
3) Q. No. 1 is compulsory. It should be solved in first 30 minutes in Answer
Book Page No. 3. Each question carries one mark.
4) Answer MCQ/Objective type questions on Page No. 3 only. Dont forget to
mention, Q.P. Set (A/B/C/D) on Top of Page.
MCQ/Objective Type Questions
Duration : 30 Minutes

Marks : 20

1. Tick the correct options :

20

1) The dimensions of magnetizing force in C.G.S.E.M. system are


a) [ 1/ 2 M1/ 2 L3 / 2 T 1]

b) [ 1/ 2 M1 L3 / 2 T 1]

c) [ 1 / 2 M1/ 2 L1/ 2 T 1]

d) [1/ 2 M1/ 2 L1 T 1]

2) The unit whose sizes cannot be chosen independently are called


a) Derived unit
b) Fundamental unit
c) Auxiliary fundamental unit
d) Absolute unit
3) Error in moving coil instrument
a) Spring weakening due to temperature and aging
b) Strength of permanent magnet weakens due to temperature and aging
c) Change if coil resistance due to temperature change
d) All of the above
4) Creeping in energy meter can be prevented by
a) Using extra turns on voltage coil
c) Using stronger magnet

b) Having two holes on opposite sides of the disc


d) Using steel laminations of high permeability

5) When measuring power with an electrodynamometer wattmeter in a circuit having low power
factor
a) The current coil should be connected on load side
b) The current coil should be connected on supply side
c) The pressure coil should be connected on load side
d) A compensated wattmeter with pressure coil connected on load side should be used
6) The value of resistance of an earthing electrode depends upon
a) Shape and material of electrode
b) Depth to which electrode is driven into earth
c) Specific resistance of soil
d) All of these
7) 1 mA ammeter has resistance of 100 , it is to be converted to a 10 V voltmeter. The value of
multiplier resistance
a) 999
b) 9999
c) 9900
d) 990
P.T.O.

SLR-BB 377

*SLRBB377*

-2-

8) In electrodynamometer type wattmeter the inductance of pressure coil circuit produces error
a) Which is constant irrespective of power factor of the load
b) Which is higher at low power factor
c) Which is lower at low power factor
d) None of the above
9) The power in a 3-phase 4-wire circuit can be measured by using
a) 2 wattmeter
b) 3 wattmeter
c) 4 wattmeter

d) 1 wattmeter

10) If an induction meter runs fast it can be slowed down by


a) Lag adjustment
b) Light load adjustment
c) By adjusting the position of breaking magnet and making in come closer to the center of disc
d) By adjusting the position of breaking magnet and making it move away from center of disc
11) Which of the following optical transducer is an active transducer ?
a) Photo emissive cell
b) Photo diode
c) Photo transistor
d) Photovoltaic cell
12) The switching time of LED is of the order of
a) 1 s
b) 1 ms

c) 10 s

d) 1 ns

13) Thermocouples
a) Are most commonly used temperature transducers
b) Requires reference junction compensation
c) Have a low output voltage level
d) All of the above
14) The Lissajous pattern on CRO screen is shown in figure below. The frequency ratio of vertical to
horizontal one is,

a) 3 : 2

b) 5 : 1

c) 1 : 5

d) 2 : 3

15) A phase sequence indicator rotates clockwise for phase sequence of RYB. If the phase sequence
is changed to RBY it will rotate
a) Clockwise
b) Anticlockwise
c) Both a) and b)
d) None of above
16) Which of the following A/D converter is used in DSO ?
a) Ramp type
b) Successive approximation type
c) Dual slope type
d) Parallel type
17) The deflection of a hot wire instrument depend upon the ______ value if alternating current.
a) Instantaneous
b) Peak
c) Average
d) r.m.s.
18) Q-meter is based on principle of
a) Series resonance
c) Mutual inductance

b) Parallel resonance
d) None of these

19) A Nixie tube requires


a) 150 V 220 V

c) 150 mv 220 mv

b) 100 V 150 V

d) 100 mv 150 mv

20) A true rms reading voltmeter uses two thermocouple in order


a) To increase sensitivity
b) That second thermocouple cancels out non-linear effect of first thermocouple
c) To prevent drift in dc amplifier
d) All of these
______________

Set A

*SLRBB377*

-3-

SLR-BB 377

Seat
No.
S.E. (Electrical and Electronics Engg.) (Part II) (Old) Examination, 2014
ELECTRICAL MEASUREMENT
Day and Date : Thursday, 27-11-2014
Time : 3.00 p.m. to 6.00 p.m.
Instructions :

Marks : 80

1) All questions are compulsory.


2) Figure to the right indicate maximum marks.
3) Assume the suitable data whenever necessary.
SECTION I

2. Solve any four :

(54=20)

1) Derive the dimensional equation of following quantities in C.G.S.E.S. system of unit :


a) Charge
b) Current
c) Potential difference
d) Capacitance
e) Resistance.
2) Explain real power measurement in 3 circuit by using one wattmeter method.
3) Explain the following terms related to instrument transformer
a) Transformation ratio
b) Nominal ratio
c) Turns ratio
d) Ratio correction factor.
4) Explain Andersons bridge for the measurement of unknown inductance and also draw its Phasor
diagram.
5) Derive the expression for current through galvanometer in a whetstones bridge for small Unbalance.
6) The four arms of a bridge are :
arm ab : an imperfect capacitor C1 with equivalent series resistance r1
arm bc : a non-inductive resistance R3, arm cd : a non-inductive resistance R4
arm da : an imperfect capacitor C2 with an equivalent series resistance of r2 series with resistance R2.
A supply of 450 Hz is given between terminal a and c and the detector is connected between b and
d at balance b and d at balance R2 = 4.8 , R3 = 2000 , R4 = 2850 and C2 = 0.5 F and r2
= 0. Calculate the value of c1 and r1.

Set A

SLR-BB 377

-4-

*SLRBB377*

3. Solve any two :

(102=20)

1) Explain construction and working of LPF wattmeter in detail.


2) Explain real power measurement in 3 circuit by using two wattmeter method for
a) Star connected load
b) Delta connected load
also find its power factor.
3) A regular whetstones bridge is used to measure resistances (in mega ohm range). The bridge has
ratio arms of 10,000 and 10 . The adjustable arms has a maximum value of 10,000 . A
battery of 10 V emf and negligible resistance is connected from junction of ratio arms to opposite
corner.
a) What is maximum resistance that can be measured by this arrangement ?
b) If the galvanometer has sensitivity of 200 mm/ A and resistance of 50 , how much unbalance
is needed to give a galvanometer deflection of 1 mm for maximum resistance of part (a) ?
c) If the galvanometer of part (b) is replaced by a galvanometer of sensitivity of 1000 mm/ A and
resistance of 1000 , calculate the change in resistance to cause deflection of 1 mm.
SECTION II
4. Solve any four :

(54=20)

1) Explain construction and operation 1 electrodynamometer power factor meter.


2) Explain with neat sketch X-Y recorder also state its advantages and disadvantages.
3) Draw and explain block diagram of generalized data acquisition system.
4) Explain the factor influencing for the choice of transducers.
5) Explain Nixie tube with neat diagram.
6) Explain working of Q-meter with neat diagram.
5. Solve any two :

(102=20)

1) Explain ramp type digital voltmeter with neat diagram.


2) Explain construction and working of LVDT also state its advantages and disadvantages.
3) Explain different types of phase sequence indicator with neat diagram.
_____________________

Set A

SLR-BB 378

*SLRBB378*
Seat
No.

Set

S.E. (E & E) (Part II) Examination, 2014


SIGNALS AND SYSTEMS (Old)
Day and Date : Friday, 28-11-2014
Time : 3.00 p.m. to 6.00 p.m.
Instructions :

Max. Marks : 100

1)
2)
3)
4)

All questions are compulsory.


Figures to the right indicated full marks.
Assume suitable data if necessary.
Q. No. 1 is compulsory. It should be solved in first 30 minutes in
Answer book Page No. 3. Each question carries one mark.
5) Answer MCQ/Objective type questions on Page No. 3 only. Dont
forget to mention, Q.P. Set (A/B/C/D) on Top of Page.
MCQ/Objective Type Questions

Duration : 30 Minutes
I.

Marks : 20

Choose the correct answer :

20

1) The energy and power of the signal u(t) u(t 1) is


a) E = 0, P = 1
b) E = 1, P = 0
d) E = 0, P = 0
c) E = , P =
2) In the signal x(n) = anej (won + ) for |a| < | the amplitude of the sinusoidal sequence
a) Increases exponentially
b) Remains constant
c) Decays exponentially
d) None
3) The period of the signal x(n) = (1)n is
a) 1
b) 2

c) 3

d) 4

4) Consider the following signal x(t) and y(t) the relation between x(t) and y(t) t can be y(t) =

a) x (t 2)

b) x (2t + 3)

c) x( 3 2 t)

d) None

5) The Laplace transform of u(t) is L(s) and the Fourier transform of u(t) is F(jw) then
a) F(jw) = L(s)/s = jw
c) L(s)

1
1
but F( jw )
s
jw

b) L(s) =

1
1
but F( jw)
s
jw

1
d) L(s) 1s but F( jw )
jw
P.T.O.

SLR-BB 378

*SLRBB378*

-2-

6) The convolution of the sequence x1(n) = {1, 2, 3, 1} and x2(n) = {4, 3, 2, 1} is


a) 15, 16, 21, 15
b) 15, 21, 15, 16
c) 21, 16, 15, 15
d) None

7) If the Laplace transfer of u(t) is 2


then the value of limt x( t) is
s + 2
a) Cannot be determined
b) Is zero
c) is infinity
d) Is unity
8) Given system y(n) = x(2n) is
a) Dynamic, Linear
c) Dynamic, unstable

b) Linear, shift invariant


d) Non linear, non causal

9) The integration of

x(t)(t to)dt

is

c) x (to)

d) (t )

10) h(n) = 0 for n < 0 shows the system is


a) Stable
b) Periodic

c) Causal

d) Linear

11) In Fourier analysis, an represents.


a) Cosine components
c) DC components

b) Sine components
d) Fourier series

a) 1

b) 0

12) In z-transform an x(n) gives


a) a n x(z)

b)

x( z)
an

c) x(az)

13) Concept of sampling will be lost if bandwidth is


a) Finite
b) Infinite
c) Ts = 0
14) In Fourier analysis bn represents.
a) DC components
c) Fourier series

z
d) x
a
d) Ts =

b) Cosine components
d) Sine components

15) A single pole inside unit circle on real axis with a positive describes a ___________
signal.
a) Right handed growing
b) Right handed decaying
c) Left handed growing
d) Left handed decaying
16) Interpolation is the process of conversion
a) CT to CT
b) CT to DT

c) DT to CT

d) DT to DT

c) 2

d) 12

17) If m(t) = t for a period of 1 second, a0 =


a) 0

b) 1

18) A single pole inside unit circle on real axis with negative value of a expressed a
_____________ signal.
a) Signed decaying
b) Unsigned decaying
c) Signed growing
d) Unsigned growing
19) Process of CT signal in digital domain is
a) Z transform
b) Fourier Transform c) Sampling
20) If m(t) =
a) e

et

with 1 second period, a0 =


c) 1 e
b) e 1
______________

d) Interpolation
d) 1 e1
Set A

*SLRBB378*

-3-

SLR-BB 378

Seat
No.

S.E. (E & E) (Part II) Examination, 2014


SIGNALS AND SYSTEMS (Old)
Day and Date : Friday, 28-11-2014
Time : 3.00 p.m. to 6.00 p.m.
Instructions :

Marks : 80

1) All questions are compulsory.


2) Figures to the right indicated full marks.
3) Assume suitable data if necessary.
SECTION I

II. Answer any four :


(45=20)
a) Determine whether the following signals are periodic or not . If periodic find the fundamental
period :
i) x(n) = (1)n
ii) x(t) = cost + sin

2 t.

b) Determine the energy and power of the following signals


x(t) = r(t) r(t 2).
c) Find the convolution of

( )

x(n) = 2n u(n) and h(n) = 1 u(n) .


3
d) State and prove the time convolution property of Laplace transform.
e) Find the step response for the LTI system with impulse response h(n) given

( 2) u(n) + ( 13) u(n) .

h(n) = 1

III. Answer any two :

(210=20)

a) i) Check whether the given system is


a) Static or dynamic
b) Linear or non linear
c) Causal or non causal
d) Time variant or time invariant
y(t) = 0 {u( t)} .
ii) State and prove the commutative property of convolution.

Set A

SLR-BB 378

*SLRBB378*

-4-

b) An LTIC system in specified by equation


d 2 y( t )
dy(t )
dx( t)
+5
+ 6 y( t ) =
+ 4x( t) with x(t) = et u(t) and initial condition y(o+) = 3,
2
dt
dt
dt
dy(o + )
= 0 . Find the total response y(t).
dx
c) Find inverse Laplace transform of x(s) =

2s
. For ROC
(s + 3) (s + 1)

i) 1 Re(s) < 3
ii) Re(s) < 3
iii) Re(s) > 1
SECTION II
IV. Attempt any two :

(45=20)

( )

a) Convert the signal in z domain if x(n) = 1 u(n)


2
b) Calculate Fourier transform of unit impulse signal. Also sketch magnitude spectrum.
c) What is aliasing ? Why it occurs ? Explain the procedure to avoid aliasing.
d) Derive the convolution property in Fourier transform.
1 1
z
2
.
e) Find x(n) by long division method where x(n) is right sided signal if x(z)=
1 1
1+ z
2
1

V. Solve any two :

(102=20)

1) Find exponential Fourier transform of half wave rectified signal in time domain.
2) Explain the concept of ROC, poles and zeros of systems and sequences in z
transformation. Also explain significance of ROC.
3) If m(t) = 4 COS 100 t, Calculate sampling rate. If Fs = 200 Hz, Determine discrete
sequence m(n) and obtain m(t) reconstructed. Conclude your result.
_____________________

Set A

SLR-BB 379

*SLRBB379*
Seat
No.

Set

S.E. (Electrical and Electronics Engineering) (Part II)


Examination, 2014
DIGITAL TECHNIQUES (Old)
Day and Date : Saturday, 29-11-2014
Time : 3.00 p.m. to 6.00 p.m.

Total Marks : 100

Instructions : 1) Assume suitable data wherever necessary.


2) Non-programmable calculators are permitted.
3) Q. No. 1 is compulsory. It should be solved in first 30
minutes in Answer Book Page No. 3. Each question carries
one mark.
4) Answer MCQ/Objective type questions on Page No. 3
only. Dont forget to mention, Q.P. Set (A/B/C/D) on Top
of Page.
MCQ/Objective Type Questions
Duration : 30 Minutes

Marks : 20

1. Objective questions :

(201=20)

1) A logic circuit for the subtraction of B(subtrahend) from A(minuend) where A


and B are 1 bit numbers is referred to as a
a) Half subtractor
b) Full subtractor
c) Full adder
d) None of these
2) Gray to binary code converters consists of how many EX-OR gates
a) 2
b) 3
c) 5
d) 1
3) A decoder with 64 output lines has _________ data inputs.
a) 2
b) 5
c) 4
d) 6
4) SOP form of logic expression consists ___________ terms.
a) min
b) max
c) max and min d) none
5) Which of the following IC is a decade counter ?
a) IC 74191
b) IC 7490
c) IC 72181

d) IC 7485

6) A digital counter in which all the flip-flops are not triggered simultaneously
are called
a) Asynchronous counter
b) Synchronous counter
c) Decade counter
d) Ripple counter
P.T.O.

SLR-BB 379

*SLRBB379*

-2-

7) A tabular form of representing the behavior of sequential circuit


a) State table
b) Program table
c) Truth table
d) Sequential table
8) ALU stands for
a) Arithmetic logic unit
c) Ahead logic unit

b) Array logic unit


d) None

9) A measure of the noise which can be tolerated by a logic circuit is


a) Noise immunity b) Noise margin c) Noise gain
d) Noise measure
10) ___________ is the fastest logic family.
a) ECL
b) TTL
c) CMOS

d) None

11) Fan-in signifies the ____________ of gate.


a) Number of inputs
b) Number of outputs
c) Number of select input
d) No. of pins
12) In an S-R flip flop ___________ is undefined state.
a) S = 0, R = 1
b) S = 1, R = 0 c) S = 1, R = 1

d) S = 0, R = 0

13) The gate whose output is low if and only if all inputs are high
a) NOR
b) NAND
c) OR
d) AND
14) The minimum no. of flip-flops required for a decade counter is
a) 4
b) 2
c) 6
d) 5
15) The Excess-3 code for decimal number 8 is ___________
a) 1100
b) 1011
c) 0011
d) 1010
16) Adding (7F)16 and (BA)16 result is
a) 100111001
b) 00110011

c) 10100011

d) 00010001

17) A 4 variable logic expression can be realized using single _______ multiplexer.
a) 4 : 1
b) 16 : 1
c) 8 : 1
d) 2 : 1
18) A shift register in which data can be shifted in both the directions, i.e., from
left to right and right to left is
a) Bi-directional
b) SIPO
c) Universal
d) PISO
19) The minterms corresponding to decimal number 15 is
a) ABCD

b) ABCD

c) A + B + C + D d) A + B + C + D

20) Master slave configuration is used in J-K flip flop to eliminate


a) Race around condition
b) Toggle condition
c) Undefined state
d) Defined state
______________
Set A

*SLRBB379*

-3-

SLR-BB 379

Seat
No.

S.E. (Electrical and Electronics Engineering) (Part II)


Examination, 2014
DIGITAL TECHNIQUES (Old)
Day and Date : Saturday, 29-11-2014
Time : 3.00 p.m. to 6.00 p.m.

Marks : 80

Instructions : 1) Assume suitable data wherever necessary.


2) Non-programmable calculators are permitted.
SECTION I
2. Attempt any four :

(46=24)

1) Minimize 4 variable logic function using K map.


F = m(0, 1, 2, 3, 8, 9, 10, 11, 13, 15) + d(5, 7)
2) Describe with waveforms RACE AROUND CONDITION.
3) Explain the concept of multiplexer tree.
4) Describe 2 bit digital comparator.
5) Describe BCD to 7 segment decoder.
3. Attempt any two :

(28=16)

1) Implement 256 : 1 multiplexer using 32 : 1 MUX.


2) Explain SR to JK flip flop conversion.
3) Explain the terms :
i) Noise margin
ii) Fan in
iii) Fan out
iv) Power dissipation
v) Current and voltage parameters.
Set A

SLR-BB 379

-4-

*SLRBB379*

SECTION II
4. Attempt any four :

(64=24)

a) Write a note on IC 7495.


b) Draw and explain 3bit synchronous counters using D flip flop.
c) Explain SISO and SIPO register.
d) Design MOD-7 counter using IC 7490.
e) Write a note on Johnsons counter.
5. Attempt any three :

(82=16)

a) Explain :
i) Ring counter
ii) Sequence generator.
b) Explain programmable logic devices in detail.
c) Design BCD to Excess-3 code converter and implement using PLA.
_____________________

Set A

SLR-BB 38

*SLRBB198*
Seat
No.

Set

T.E. (Civil) (Part I) (Old) Examination, 2014


GEOTECHNICAL ENGG. I
Day and Date : Friday, 5-12-2014
Time : 3.00 p.m. to 6.00 p.m.

Total Marks : 100

Instructions : a) Q. No. 1 is compulsory. It should be solved in first 30 minutes


in Answer Book Page No. 3.
b) Answer MCQ/Objective type questions on Page No. 3 only.
Dont forget to mention, Q.P. Set (A/B/C/D) on Top of Page.
c) Assume additional data if required and state it clearly.
d) Figures on the right indicate full marks.
MCQ/Objective Type Questions
Duration : 30 Minutes

Marks : 20

1. Choose the correct answer :


1) A change in voids ratio per unit change in pressure is termed as ______
a) Coefficient of volume change
b) Coefficient of compressibility
c) Coefficient of consolidation
d) None of these

20
1

2) The actual lateral earth pressure at a particular depth in relation to the


vertical pressure at that depth _____________
1
a) Greater
b) Less
c) Equal
d) Sometimes less, sometimes greater
3) A soil sample has G = 2.7, degree of saturation = 40% and water content of 20%.
The void ratio of the sample is ______________
1
a) 1.35
b) 0.03
c) 5.4
d) None of these
4) As per the plasticity chart of Indian standard classification system, the soil is
of medium compressibility, when the liquid limit is
1
a) < 35%
b) 35% to 50%
c) > 50%
d) 25% to 34%
5) The degree of saturation of a saturated soil is
a) 0.25
b) 0.5
c) 1.0

1
d) 0.0

P.T.O.

SLR-BB 38

*SLRBB198*

-2-

6) The critical exit gradient for seepage of water out of a soil is _______
a)

G 1
1 e

b)

G 1
1+ e

c)

G+1
1 e

7) A Proctor needle is used to determine


a) Water content b) Dry density
c) Both

d)

G+1
1+ e

1
d) Neither

8) Which of the following is correct Mohr-Colomb law ?


a) = C tan
b) = C + tan
c) = C + cot

d) None of these

9) In a sieve analysis the following data is obtained, D10 = 2; D30 = 12; and
D60 = 20 Coefficient of curvature of the soil is _________
a) 10
b) 120
c) 3.6
d) 1.8

10) Relation between the consistency limit is

a) L < P > S

b) L > P > S

c) L > P < S

d) none of the above

11) If the void ratio of a soil mass changes from e1 to e2 due to increase in
effective pressure from 1 to 2 , the value of the compression index will be
a)

e1 e2

log10 2
1

b)

e1 + e 2

log10 2
1

c)

e1 e 2

loge 2
1

e1 + e2

loge 2
1

12) A flow net is drawn for a weir. The total head loss is 8 m. Number of potential
drops are 8 and the length of the flow path of last flow field is 1 m. The exit
gradient is
2
a) 1
b) 1.6
c) 2.3
d) 2.4
13) A clayey soil is having unconfined compressive strength of 40 kPa, its
cohesion is
a) 80 kPa
b) 20 kPa
c) 10 kPa
d) 90 kPa

14) A soil which has never been subjected an effective stress greater than the
existing pressure and also which is completely consolidated, is called
a) Normally consolidated
b) Over consolidated
c) Under consolidated
d) None of these
15) Depth of an unsupported vertical cut in cohesive soil having c = 40 kN/m2,
= 20 kN/m3 is
a) 1 m
b) 2 m
c) 8 m
d) 4 m
______________

Set A

*SLRBB198*

SLR-BB 38

-3-

Seat
No.

T.E. (Civil) (Part I) (Old) Examination, 2014


GEOTECHNICAL ENGG. I
Day and Date : Friday, 5-12-2014
Time : 3.00 p.m. to 6.00 p.m.

Marks : 80

Instructions : a) Solve any three questions from Section I.


b) In Section II, Q. No. 6 is compulsory, solve any two from
the remaining.
c) Assume additional data if required and state it clearly.
d) Figures on the right indicate full marks.
SECTION I
2. a) Derive the relationship between e, , G and S.
b) Following are the results obtained from the tests conducted on two soils A and
B. Classify the soils as per IS. Show the salient steps involved :
Soil

Gravel
(Size)%

Sand Fines
(Size)%
%

01

96

04

Cu

Cc

LL(%)

PL(%)

03

7.0

1.5

96

60

25

3. a) Derive the expression for the permeability using constant head test.
b) It is observed that in 12 minutes, 800 ml of water passes through a soil
sample 10 cm thick and 75 cm2 of cross sectional area, under a head of 60 cm.
Determine the discharge velocity and coefficient of permeability. On oven
drying the sample weighs 0.00685 kN. Compute the seepage velocity assuming
the sp. Gr. of soil solids as 2.7.

4. a) Write a note on box shear test.


b) In a shear box test, the following results were obtained :

Normal Stress (kN/m2)

250

350

Shear Stress (kN/m2)

110

130

What would be the deviator stress at failure, if a triaxial test is carried out on
the same soil with a cell pressure of 150 kN/m2 ?

Set A

SLR-BB 38

*SLRBB198*

-4-

5. Write short notes on any four :


(3.54=14)
i) Relation between voids ratio and porosity with the help of three phase diagram.
ii) Classification of fine grained soils.
iii) Differences between triaxial shear test and box shear text.
iv) Pycnometer method of finding specific gravity.
v) Determination of permeability by constant head method.
SECTION II
6. Solve any four :
a) Earth pressure at rest.
b) Use of Geotextiles in Civil Engineering.
c) Write a note on Field Compaction Control.
d) Derive the relation for coefficient of active earth pressure.
e) Difference between consolidation and compaction.

(44=16)

7. a) The following data have been obtained in a standard Proctor test :


Water Content (%)

7.7

11.5

14.6

17.5

21.2

Wt. of wet soil (N)

16.67

18.54

19.52

19.23

18.83

Take G = 2.77, Volume of the mould = 944 cm3,


i) Plot compaction curve
ii) Determine the optimum moisture content maximum dry unit weight
iii) What is the degree of saturation at optimum condition ?

b) Distinguish between Standard and Modified Proctor test.

8. a) A retaining wall 7.5 m high, retains a cohesionless backfill. The top 3 m fill
has a unit weight of 18 kN/m3 and angle of internal friction of 30 and the rest
has a unit weight of 24 kN/m3 and angle of internal friction of 20. Determine
the pressure distribution on the wall. Find total active pressure and its point of
application.

b) Explain Rehbanns method of construction to determine the active earth


pressure at the back of a retaining wall.

9. a) What is preconsolidation pressure ?

b) 20 mm thick undisturbed sample of saturated clay tested in laboratory with


drainage through top and bottom. Sample reached 50% consolidation in
35 minutes. If the clay layer from which the clay sample was obtained is 3 m
thick and is free to drain through top and bottom surfaces, calculate the time
required for the same degree of consolidation in the field. What is the time
required if the drainage in the field is through top only ?

_____________________

Set A

SLR-BB 380

*SLRBB380*
Seat
No.

Set

S.E. (Elect. and Electronics) (Part II) Examination, 2014


LINEAR ALGEBRA (New)
Day and Date : Tuesday, 25-11-2014
Time : 3.00 p.m. to 6.00 p.m.

Max. Marks : 100

Instructions : 1) Use of non-programmable calculator is allowed.


2) Q. No. 1 is compulsory. It should be solved in first 30 minutes
in Answer book Page No. 3. Each question carries one mark.
3) Answer MCQ/Objective type questions on Page No. 3 only.
Dont forget to mention, Q.P. Set (A/B/C/D) on Top of Page.
MCQ/Objective Type Questions
Duration : 30 Minutes
1. Tick (9) the correct alternative :
1 2 3

1) If A = 0 3 5 then the eigen values of A3 are


0 0 2

Marks : 20
20

b) 1, 13 , 12
c) 1, 3, 2
d) 1, 27, 8
The dimension of Col A is
a) The number of Columns of A
b) The number of Rows in A
c) The number of pivot Columns of A
d) None
3 1 7 0 6

0
2 4 0 3

then dim Null A and dim Row A are


If B =
0
0 0 1 1

0 0 0 0
0
a) 4, 1
b) 2, 3
c) 4, 0
d) 1, 3
Which of the following is not characteristic of invertible matrix ?
a) The equation AX = b has at least one solution for each b in Rn
b) AT is an invertible matrix
c) AX = 0 has only the non trivial solution
d) The columns of A span Rn
Let T : Rn Rm be a linear transformation and let A be the standard matrix for T. Then
a) T is 1 1 iff the Columns of A are LI
b) T is 1 1 iff the Columns of A are LD
c) T is 1 1 iff the Rows of A are LI
d) T is 1 1 iff the Rows of A are LD
a) 1, 27, 8

2)

3)

4)

5)

1 2
6) The eigen values of matrix A =
are
3
1
a) 1 i
b) 2 i
c) i
0
7) Consider a basis B = {b1, b2} for R2, where b1 = and b2 =
1
2
has the co-ordinate vector [x]B = then x is
3
1
a)
2

2
b)
3

6
c)
1

d) 4 2i
1
2
suppose an x in R
2

1
d)
6
P.T.O.

SLR-BB 380

*SLRBB380*

-2-

8) Which system of equations corresponds to the following augmented matrix ?


1
3 0 2

0 1 2 7
0
1 1 6

a) x + 3y = 2, y + 2z = 7, y + z = 6
b) u + 3v = 2, v + 2w = 7, v = w + 6
c) r + 3s = 2, s + 2t = 7, s = t + 6
d) a + 3b = 2c, b 2c = 7, b c = 6
9) The homogeneous equation AX = 0 has a non-trivial solution iff the equation has
a) At least one free variable
b) No free variable
c) At least one basic variable
d) Can not be determined
2 3 5
10) Rank of matrix 2 3 5 is

2 3 5
a) 2
b) 3
11) If an analytic function

c) None

f(z) = u(x, y) + iv (x, y) then f (z) = _________


a) ux + vx
b) uy + vx
c) ux + ivx

d) 1

d) ux + ivx

dz + b
a point in z-plane is mapped onto
cz + d
a) Unique point on the w-plane
b) More than one point on the w-plane
c) More than two point on the w-plane
d) None of the above
13) Which of the following equation is called Laplace equation ?
12) In a bilinear transformation W =

a)

2
x

2
y

=0

b)

2
x

2
y

=0

c)


+
=0
x y

d)

=0
x y

5
8
, bxy = , then r = __________
18
5
2
1
d) 3
a)
b)
c) 2
5
2
3
2
15) If the points in a scatter diagram are scattered on all directions, then ______
a) r = 0
b) r = 1
c) r = 1
d) r = 2
16) The co-efficient of correlation r is given by

14) If byx =

a) bxy byx

b)

bxy.byx

c) (bxy.byx)2

d) None of the above

17) If u = [3, 4, 6] and v = [0, 1, 1], then u.v. = _________


a) 10
b) 15
c) [0, 4, 6]
d) None of the above
18) Let u be a non-zero vector. Then a unit vector in direction of u is
u
b) u u
d) None of the above
c)
a) u
u
3
2
2
19) Let (n) = x1 8x1x 2 5x 2 , then (n) = __________ for x = .
1
a) 20
b) 16
c) 20
d) 28
x
20) The harmonic conjugate of e cosy is
a) ex cosy + c
b) ex + c
c) ex + c
d) ex siny + c

______________

Set A

*SLRBB380*

-3-

SLR-BB 380

Seat
No.

S.E. (Elect. and Electronics) (Part II) Examination, 2014


LINEAR ALGEBRA (New)
Day and Date : Tuesday, 25-11-2014
Time : 3.00 p.m. to 6.00 p.m.

Marks : 80

Instructions : 1) Attempt any three questions from each Section.


2) Figures to the right indicate full marks.
3) Use of non-programmable calculator is allowed.
SECTION I
1

2. a) Determine b, if it is a linear combination of a1, a2, a3. a1 = 2 , a2 =
0

2
5
0
, a = 6 b = 1 . 3

1 3
6
8
2
b) Show that T is linear transformation and find standard matrix of T(x1, x2) = (2x2 3x1,
x1 4x2, 0, x2).
4
3 7 2
7

c) Factorize the matrix and hence find the solution A = 3


5
1 , b = 5 .
6 4
2
0

1 1
3. a) Find the eigen values of A and also find basis for each eigen space A =
.
0.4 0.6

2 1

5
5
.
3
2

8 8

1 4 0

3 12 1
b) Find a basis for Null space and column space of A. Where A =
2
8 1

5 20 2

6 7
4. a) Use power method
to find largest Eigen value and corresponding eigen vector.
8 5
1
X0 = upto K= 4.
0
b) If A is row equivalent to B. List rank A, dim Null A. Then find bases for Col A, Row A and
Null A.

0 1 5
1 4 9 7
1

5 6
A = 1 2 4 1 B = 0 2
.
,
5 6 10 7
0
0
0
0

Set A

SLR-BB 380

*SLRBB380*

-4-

c) Let B = {b1, b2} and c = {c1, c2} be bases for R2. Find the change of co-ordinates matrix
9
5
1
3
from B to C. where b1= , b2 = , c1= , c2 = .
1
1
4
5

3
4 2

5. a) Diagonalize the matrix if possible A = 1 1 3 .


2 4
9

b) Compute the transfer matrix of a ladder network in the following figure. Also design a
1 8
ladder network whose transfer matrix is 1
.


5
2

SECTION II
6. a) Find the equations of lines of regression and also coefficient of correlation from the following
data :
x : 62 64

65

69

70

71 72

74

y : 126 125 139 145 165 152 180 208

b) The following table gives the no. of motor-vehicle with licences and the no. of motor vehicle
accidents in a city. Calculate the coefficient of correlation between the number of
motor-vehicles and the number of motor-vehicle accidents.
No. of motor vehicles
in thousands
No. of accidents
in hundreds
7. a) Show that w =

x
2

x +y

2.6

2.8

2.9

3.1

3.2

2.3

2.5

1.8

5.9

6.0

6.2

6.2

7.6

7.0

7.5

5.5

iy
2

x +y

is an analytic function and find

dw
.
dz

b) Prove that u = ex cosy is harmonic function. Determine its harmonic conjugate V and the
analytic function f(z).

c) Find the bilinear transformation which maps the points z = 1, 1, onto the points
w = 1 + i, 1 i, 1.

Set A

*SLRBB380*

SLR-BB 380

-5-

8. a) The equations of lines of regression are x + 2y = 5 and 2x + 3y = 8. Find x, y and r.

b) Write down the quadratic form corresponding to the following matrices :

3
1
ii)
2

0 5 1

i) 5 1 6
1 6 2

c) Evaluate

1
2
0
6

6 .

z2 z + 1
dz, where C is the circle z = 1.
z 1

9. a) Show that {V1, V2, V3} is an orthonormal basis of IR3 where

V1 =

10

3 , V =
2
20

20

0
10

1 , V = 1 .
3 2
20

1
1
2
20

3
4
1
b) If u = , v = and w = 1 , then compute :

6
2
5
i) u.u, v.u and

v.u
u.u

ii)

1
w
w.w

u.v
v
iii)
v.v

_____________________

Set A

Set A

SLR-BB 381

*SLRBB381*
Seat
No.

Set

S.E. (Electrical and Electronics Engg.) (Part II) Examination, 2014


A.C. MACHINES (New)
Day and Date : Wednesday, 26-11-2014
Time : 3.00 p.m. to 6.00 p.m.
Instructions :

Total Marks : 100

1) All questions are compulsory.


2) Assume the suitable data whenever necessary.
3) Q. No. 1 is compulsory. It should be solved in first 30 minutes in
Answer book Page No. 3. Each question carries one mark.
4) Answer MCQ/Objective type questions on Page No. 3 only.
Dont forget to mention, Q.P. Set (A/B/C/D) on Top of Page.
MCQ/Objective Type Questions

Duration : 30 Minutes

Marks : 20

1. Solve the following :


(120=20)
1) A 3-phase, 4-pole, 50-Hz induction motor runs at a speed of 1440 rpm. The rotating field
produced by the stator rotates at a speed of __________ rpm.
a) 1500
b) 1440
c) 60
d) 0
2) Line joining tangent and output line in a circle diagram gives
a) Copper loss
b) Stator loss
c) Maximum output
d) Maximum Torque
3) Skew of the rotor bar eliminates
a) The effect of space harmonics
b) The effect of crawling
c) Magnetic noise
d) None of the above
4) Shape of the torque slip curve is
a) Rectangular Hyperbola
b) Parabola
c) Ellipse
d) Cosine
5) The no-load speed of an induction motor is of the order of
a) 8%
b) 4%
c) 2%
d) 6%
6) Circle diagram of an induction motor can be used to determine the
a) Power factor
b) Efficiency
c) Slip
d) All the above
7) A Double cage induction motor has
a) Two series windings in stator
b) Two series windings in rotor
c) Two parallel windings in stator
d) Two parallel windings in rotor
8) Iron loss in 3 phase induction motor is negligible. This is due to
a) Very low frequency of emf induced in the rotor
b) Very low induced emf is induced in the rotor
c) Very low flux density in rotor parts
d) Constant magnitude of the flux linking the rotor core
9) In case of split phase motors the phase shift is usually limited to
a) 3 degrees
b) 60 degrees
c) 90 degrees
d) 150 degrees
P.T.O.

SLR-BB 381

-2-

*SLRBB381*

10) For ceiling fans generally the single phase motor used is
a) Split phase type
b) Capacitor start type
c) Capacitor start and run type
d) Permanent capacitor type
11) The efficiency of shaded pole motor is in the range
a) 80 to 95 percent
b) 70 to 80 percent
c) 50 to 70 percent
d) 5 to 35 percent
12) For an alternator when the power factor of the load is lagging
a) The armature flux will have square waveform
b) The armature flux will be demagnetizing
c) The armature flux will be cross-magnetizing
d) The armature flux will reduce to zero
13) Which kind of rotor is most suitable for turbo alternators which are designed to run at
high speed ?
a) Salient pole type
b) Non-salient pole type
c) Both a) and b)
d) None of the above
14) For 50 Hz system the maximum speed of an alternator can be
a) 6000 rpm
b) 1500 rpm
c) 3600 rpm
d) 3000 rpm
15) If the input to the prime mover of an alternator is kept constant but the excitation is changed,
then
a) Reactive component of the output is changed
b) Active component of the output is changed
c) Power factor of the load remains constant
d) Power factor of the load reduces
16) In synchronous alternator, which of the following coils will have emf closer to sine
waveform.
a) Concentrated winding in full pitch coils
b) Concentrated winding in short pitch coils
c) Distributed winding in full pitch coils
d) Distributed winding in short pitch coils
17) An over excited synchronous motor draws current at
a) Lagging power factor
b) Leading power factor
c) Unity power factor
d) Depends on the nature of load
18) In a synchronous motor if the back emf generated in the armature at no load is
approximately equal to the applied voltage, then
a) The torque generated is maximum
b) The excitation is said to be zero percent
c) The excitation is said to be 100%
d) The motor is said to be fully loaded
19) A synchronous motor is switched on to supply with its field windings shorted on themselves.
It will
a) Not start
b) Run as an induction motor
c) Start as an induction motor and then run as a synchronous motor
d) None of the above
20) The magnitude of stator back emf depends on
a) Load only
b) Load and speed only
c) Excitation only
d) Load, speed and Excitation
______________
Set A

*SLRBB381*

-3-

SLR-BB 381

Seat
No.
S.E. (Electrical and Electronics Engg.) (Part II) Examination, 2014
A.C. MACHINES (New)
Day and Date : Wednesday, 26-11-2014
Time : 3.00 p.m. to 6.00 p.m.
Instructions :

Marks : 80

1) All questions are compulsory.


2) Assume the suitable data whenever necessary.
SECTION I

2. Solve any four :

(45=20)

a) Draw and explain shaded pole induction motor.


b) A 4 pole main induction motor and 6 pole auxiliary induction motor are connected in
cumulative cascade. The frequency in secondary circuit of 6 pole is 1Hz. Determine the
combined speed of set and slip in each machine. Take a supply frequency of 50 Hz.
c) A 200W, 230V, 50Hz, capacitor start motor has following winding parameters.
Main winding :

R = 4.5 , XL = 3.7

Starting winding :

R = 9.5 , XL = 3.5

Find the value of capacitance that will result in maximum starting torque.

/ induction motor.
d) With neat diagram explain DOL starter of 3 O
/ induction motor.
e) Find the condition for maximum starting torque of 3 O
/ induction motor.
f) Explain torque-slip characteristic of 3 O
3. Solve any two :

(210=20)

/ induction motor is not self starting ? How to make it self start ? Explain cross field
a) Why 1 O
theory ?
b) Describe the procedure to draw circle diagram to find out various parameters.

/ , 220V induction motor has following data.


c) A 1 O
N.L. Test

200V, 6A, 350W.

S.C. Test

125V, 15A, 530W.

Stator resistance is 1.2 with d.c.


Calculate full load P.F. and efficiency if slip is 0.05.

Set A

SLR-BB 381

*SLRBB381*

-4-

SECTION II
4. Solve any four :

(45=20)

a) Explain magnetizing and De-magnetizing effect in an alternator.


b) Distinguish between Salient and Cylindrical rotor of an alternator.
c) A synchronous motor improves the PF of load of 200KW from 0.8 lagging to 0.9 lagging.
Simulenously the motor carries a load of 80KW. Find the leading KVAR supplied by the
motor, KVA rating of the motor, the PF at which the motor operates.
d) Describe significance of Short Circuit Ratio.

/ , star connected turbo-alternator is excited to generate the bus bar


e) A 2 pole, 50Hz, 3 O
voltage of 11KV at N.L. calculate synchronizing power/mechanical degree displacement
of rotor and corresponding synchronizing torque. The machine has short circuit current
for this excitation is 1200A.
f) Describe the working principle of Synchronous motor.
5. Solve any two :

(210=20)

a) Explain pitch factor and distribution factor. Also derive an expression for EMF equation
of alternator.
b) Describe Armature reaction in an alternator.
/ alternator gave the following data for O.C.C. and S.C.C.
c) A 50 KVA, 500V, 1 O
Field current (A)

10

15

20

25

30

125

250

370

480

566

640

S.C. Armature Current (A) :

73

146

220

EMF (V)

Using MMF method, find full load voltage regulation at unity P.F. and 0.8 P.F. lagging.
Effective armature resistance is 0.2 .
_____________________

Set A

SLR-BB 382

*SLRBB382*
Seat
No.

Set

S.E. (Electrical and Electronics Engg.) (Part II) Examination, 2014


ELECTRICAL AND ELECTRONICS MEASUREMENT (New)
Day and Date : Thursday, 27-11-2014
Time : 3.00 p.m. to 6.00 p.m.

Total Marks : 100

Instructions : 1) Figure to the right indicates maximum marks.


2) Assume the suitable data whenever necessary.
3) Q. No. 1 is compulsory. It should be solved in first 30 minutes in Answer
Book Page No. 3. Each question carries one mark.
4) Answer MCQ/Objective type questions on Page No. 3 only. Dont forget
to mention, Q.P. Set (A/B/C/D) on Top of Page.
MCQ/Objective Type Questions
Duration : 30 Minutes

Total Marks : 20

1. Tick the correct options.


1) The dimension of inductance in C.G.S.E.S. system is

20

a)
b)
c)
d)
2) Measuring range of voltmeter can be extended by using
a) High shunt resistance
b) High series resistance
c) Low shunt resistance
d) Low series resistance
3) The power in a 3-phase 4-wire circuit can be measured by using
a) 2 wattmeter
b) 3 wattmeter
c) 4 wattmeter
d) 1 wattmeter
4) The nominal ratio of current transformer is
a) Primary winding current/secondary winding current
b) Rated primary winding current/rated secondary winding current
c) No. of secondary winding turns/no. of primary winding turns
d) All of the above
5) When secondary winding of current transformer is open-circuited with primary winding is
energized ?
a) The whole of primary current produces large value of flux in the core thereby producing a large
voltage in the secondary wining
b) The large voltage may act as safety hazard for the operator and may even rupture the insulation
c) When large magnetizing force is taken off it leaves large value of residual magnetism
d) All of the above
6) In spring controlled moving iron instrument, the scale is
a) Uniform
b) Cramped at the lower end and expanded at the upper end
c) Expanded at the lower end and cramped at the upper end
d) Cramped both at lower end and upper end
7) High ac voltage are usually measured with
a) Magnetic voltmeter
b) Inductive voltmeter
c) Potential transformer with voltmeter
d) Current transformers and voltmeter
8) The disadvantages of using multipliers with voltmeter at high voltages are
a) The power consumption of multipliers becomes large at larger voltages
b) The multipliers at high voltages have to be shielded in order to prevent capacitive currents
c) The metering circuit is not isolated electrically from power circuit
d) All of the above
P.T.O.

SLR-BB 382

-2-

*SLRBB382*

9) The breaking torque provided by permanent in single phase induction energy meter can be changed by
a) Providing a magnetic shunt and changing its position
b) Changing the distance of permanent magnet from center of revolving disc
c) Both a) and b)
d) None of the above
10) In order that A.C. Bridge is balanced when
a) I1 = I3 and I2 = I4
b) Z1Z4 = Z2Z3
c) 1 + 4 = 2 + 3
d) All of the above
11) The deflection of a hot wire instrument depend upon the ___________ value of alternating current.
a) instantaneous
b) peak
c) average
d) r.m.s.
12) The switching time of LED is of the order of
a) 1 s
b) 1 ms
c) 10 s
d) 1 ns
13) A phase sequence indicator rotates clockwise for phase sequence of RYB. If the phase
sequence is changed to RBY it will rotate
a) Clockwise
b) Anticlockwise
c) Both a) and b)
d) None of above
14) The accuracy of a measuring instrument is determined by
a) Closeness of the value indicated by it to the correct value of measured
b) Repeatability of measured value
c) Speed with which the instruments reading approaches the final value
d) Least change in the value of measured that could be detected by instrument
15) X-Y recorders
a) Records one quantity with respect to another quantity
b) Record one quantity on X-axis with respect to time on Y-axis
c) Record one quantity on Y-axis with respect to time on X-axis
d) None of the above
16) An electrodynamometer type wattmeter is connected in a 3-phase supply having 3-phase balanced
load, E and I are the values of phase voltage and current and is phase angle between them. The
reading of wattmeter will be

17)

18)

19)
20)

a) Proportional to EI Sin
b) Proportional to EI cost
c) Proportional to EI tan
d) Zero
A true rms reading voltmeter uses two thermocouple in order
a) To increase sensitivity
b) That second thermocouple cancels out non-linear effect of first thermocouple
c) To prevent drift in dc amplifier
d) All of these
The source of emission of electrons in a CRT is
a) PN junction diode
b) A barium and strontium oxide coated cathode
c) Accelerating anode
d) Post accelerating anode
Q-meter is based on principle of
a) Series resonance
b) Parallel resonance c) Mutual inductance d) None of these
Which of the following optical transducer is an active transducer ?
a) Photo emissive cell b) Photo diode
c) Photo transistor
d) Photovoltaic cell
______________
Set A

*SLRBB382*

-3-

SLR-BB 382

Seat
No.

S.E. (Electrical and Electronics Engg.) (Part II) Examination, 2014


ELECTRICAL AND ELECTRONICS MEASUREMENT (New)
Day and Date : Thursday, 27-11-2014

Marks : 80

Time : 3.00 p.m. to 6.00 p.m.


Instructions : 1) All questions are compulsory.
2) Figure to the right indicates maximum marks.
3) Assume the suitable data whenever necessary.
SECTION I
2. Solve any four :

(54=20)

1) Explain construction and operation of repulsion type MI instrument.


2) Derive the expression for bridge sensitivity for Whetstones bridge with equal arms.
3) Explain Maxwells bridge for the measurement of unknown inductance and also draw its phasor
diagram.
4) A milliammeter if 2.5 resistance reads upto 100 milliampere. Calculate the resistance which is
necessary to enable it to be used as
a) A voltmeter reading upto 10 V
b) An ammeter reading upto 10 A.
5) Explain measurement of power in 1 circuit by using 3 voltmeter method.
3. Solve any two :

(102=20)

1) Explain construction and operation of current transformer and potential transformer in detail.
2) Explain construction and working of 1 induction energy meter with neat diagram.
3) The four arms of a bridge are :
arm ab : an imperfect capacitor C1 with equivalent series resistance r1
arm bc : an non-inductive resistance R3
arm cd : a non-inductive resistance R4
arm da : an imperfect capacitor C2 with an equivalent series resistance of r2 series with
resistance R2
A supply of 450 Hz is given between terminal a and c and the detector is connected between b and
d at balance R2 = 4.8 , R3 = 2000 , R4 = 2850 and C2 = 0.5 F and r2 = 0.4 . Calculate
the value of c1 and r1.

Set A

SLR-BB 382

-4-

*SLRBB382*

SECTION II
4. Solve any four :

(54=20)

1) Explain construction and operation of Weston frequency meter with neat diagram.
2) Explain front panel control details of dual trace oscilloscope.
3) Explain LCD display with diagram also states its advantages and disadvantages.
4) Explain the factor influencing for the choice of transducers.
5) Explain with neat sketch X-Y recorder also state its advantages and disadvantages.
5. Solve any two :

(102=20)

1) Explain different types of phase sequence indicator with neat diagram.


2) Explain ramp type digital voltmeter with neat diagram
3) Explain working of signal generator with help of block diagram.
_____________________

Set A

SLR-BB 383

*SLRBB383*
Seat
No.

Set

S.E. (E and E) (Part II) Examination, 2014


SIGNAL AND SYSTEMS (New)
Day and Date : Friday, 28-11-2014
Time : 3.00 p.m. to 6.00 p.m.

Max. Marks : 100

Instructions : 1) Figures to the right indicate full marks.


2) Assume suitable data if necessary.
3) Q. No. I is compulsory. It should be solved in first 30 minutes in Answer
Book Page No. 3. Each question carries one mark.
4) Answer MCQ/Objective type questions on Page No. 3 only. Dont forget to
mention, Q.P. Set (A/B/C/D) on Top of Page.
MCQ/Objective Type Questions
Duration : 30 Minutes

Marks : 20

I. Choose correct answer :

20

1) The period of the function cos


a)

1
sec
8

(t 1) is
4

b) 8 sec

c) 4 sec

d)

1 sec
4

2) The odd and even components of sequence x (n) = {1, 1, 0.5}


a) {0.25, 0, 0.25}, {1.5, 1, 1.5}
b) {0.25, 1, 0.25}, {0.5, 0, 0.5}
c) {1, 1, 0.5}, {0.5, 0.5, 1}
d) {0.25, 0.25, 0.5}, {0.5, 0.5, 5.2}
3) A continuous time system is governed by the equation 3y3(t) + 2y2(t) + y(t) = x2(t) + x(t) the
system is
a) Linear and dynamic
b) Linear and non-dynamic
c) Non-linear and dynamic
d) Non-linear and non-dynamic
4) Which of the following system is BIBO unstable ?

a) y(n) = x(K)
b) y(n) = ax(n)
c) y(n) = | x(n)|
K =

d) y(n) = log10 [x(n)]

5) The system represent which equation ?

a) y(n) = x(n) [h1(n) + h2(n)]


c) y(n) = x(n) * [h1(n) + h2(n)]
6) Power signals are the signal with
a) 0 < E < , P = 0
c) 0 < P < , E =

b) y(n) = x(n) * [h1(n) * h2(n)]


d) y(n) = x(n) * [h1(n) / h2(n)]
b) 0 < E < , P =
d) 0 < P < , E = 0
P.T.O.

SLR-BB 383

*SLRBB383*

-2-

7) The real exponential signal for x(t) = Aeat for a = 0

a)

b)

c)

d)

8) Z and Laplace transform are related by


a) S = ln z

b) S =

ln Z
T

c) S = Z

d) S = T ln Z

9) SINC (x) =
Sin x
Sin x
b)
x
x
at
10) Laplace transform of e cos(bt) is equal to
S+a
Sa
a)
b)
2
(S + a ) + b 2
(S a )2 + b 2

a)

c)

Sin x
x

d)

Sin2 x
x

c)

Sa
(S + a )2 + b 2

d)

S+a
(S b )2

11) Z-transform of S(n) is


a) 0
b)
c) 1
12) The region of convergence of the Z-transform of a unit step function is
a) | Z| > 1
b) | Z| < 1
c) (Real part of Z) > 0
13) Zero order hold is used to
a) Reconstruct the signal
b) Low pass filter
c) Hold the signal till next sample
d) All the above

d)
d) (Real part of Z) < 0

14) If x(t) is even then x( j )


a) Imaginary and even
b) Imaginary and odd
c) Real and even
d) Real and odd
15) The Fourier transform of a continuous time aperiodic signal is
a) Continuous and aperiodic
b) Discrete and aperiodic
c) Continuous and periodic
d) Discrete and periodic
16) What is Nyquist sampling rate of the following signal : x(t) = Sin c2 (100 t) ?
a) 50 Hz
b) 100 Hz
c) 200 Hz
d) 400 Hz
n
n
17) The ROC of the Z-transform of the signal 2 u(n) 3 u(n1) is
a) |Z| > 1
b) |Z | < 1
c) 2 < |Z| < 3
d) does not exist
18) Z-transform converts convolution of time signals to
a) Addition
b) Substraction
c) Multiplication
d) Division
19) Sampling theorem state that
1
1
2 fm
a) FS 2 fm
b) TS 2 Tm
c)
d) TS 2 fm
TS
1
+ ()
j
b) Cos t
c) Sgn(t)
______________

20) Inverse Fourier transform of the function F( j) =


a) Sin t

d) u(t)
Set A

*SLRBB383*

SLR-BB 383

-3-

Seat
No.

S.E. (E and E) (Part II) Examination, 2014


SIGNAL AND SYSTEMS (New)
Day and Date : Friday, 28-11-2014
Time : 3.00 p.m. to 6.00 p.m.

Marks : 80

Instructions : 1) All questions are compulsory.


2) Figures to the right indicate full marks.
3) Assume suitable data if necessary.
SECTION I
II. Answer any four :
1) Explain the properties of convolution.

(45=20)

2) Find even and odd components of :


i) x(t) = Cos t + Sin t + Cos t . Sin t
ii) x(n) = { 2, 1, 2, 1, 3}.
3) For the signal x(t) shown in Fig. find the following signals and sketch :

Fig. 1.
i) x (2t + 3)

(2)

ii) x 3 t .
n

1
4) Find whether the signal is energy or power x(n) = u(n) .
3

5) Find the Laplace transform of the following signal x(t) = 2e2t u(t) + 4e4t u(t).

III. Answer any two :

(210=20)

3S 2 + 8S + 6
1) Find inverse Laplace transform of
.
(S + 2) (S2 + 2S H)

2) Find the step response of the following system whose impulse response is given by :
i) h(t) = u(t+1) u(t 1)
ii) h(t) = e3t u(t) e2t u(t).
3) Find the convolution of the signals :

,
Fig. 1

Fig. 2

Set A

SLR-BB 383

-4-

*SLRBB383*

SECTION II
IV. Solve any four :

(54=20)

1) Find the Z-transform of the following :

( ) + ( 13)

x(n) = 1
2

u(n) .

2) Find 4-point DFT of the sequence :


i) x(n) = 1 0 n 2.
3) Explain the effect of under sampling.
4) Find the inverse Z-transform of the following :
X(Z) =

1 Z 1
4
.
1 1

1
1 Z 1 14 Z

5) A signal x(t) = Sin c (150 t) is sampled at the rate of a) 100 Hz, b) 200 Hz, c) 300 Hz for each
of these 3 cases, explain if you can recover the signal x(t) from the sampled signal.
V. Attempt any two :

(102=20)

1) Explain the sampling theorem in time domain.


2) Find the inverse Z-transform X(Z) =

Z+4
.
2
Z 4Z + 3

3) Find the trigonometric Fourier series for periodic signal x(t) as shown in Fig. 1.

Fig. 1
_____________________

Set A

SLR-BB 384

*SLRBB384*
Seat
No.

Set

S.E. (E&E) (Part II) Examination, 2014


DIGITAL TECHNIQUES (New)
Day and Date : Saturday, 29-11-2014
Time : 3.00 p.m. to 6.00 p.m.

Total Marks : 100

Instructions : 1) Figures to the right indicate full marks.


2) Assume suitable data wherever necessary.
3) Q. No. 1 is compulsory. It should be solved in first 30 minutes
in Answer book Page No. 3. Each question carries one mark.
4) Answer MCQ/Objective type questions on Page No. 3 only.
Dont forget to mention, Q.P. Set (A/B/C/D) on Top of Page.
MCQ/Objective Type Questions
Duration : 30 Minutes

Marks : 20

1. Choose the correct answer :


1) The NOR NOR realization is equivalent to _________
a) AND OR Realization
b) OR AND Realization
c) AND OR NOT Realization
d) AND AND Realization
2) Multiplexer is represented by
b) 1 2n
c) 2n n
a) 2n 1
3) A de-multiplexer is _________ implementation.
a) OR
b) AND OR
c) AND

(201=20)

d) n 2n
d) OR AND

4) The statement x(x + y) = x * y represent _________


a) Associative law
b) DeMorgans theorems
c) Distributive law
d) Commutative law
5) The given Excess 3 number is 0111, its equivalent binary number is
a) 0100
b) 0011
c) 1010
d) 1001
6) The expression f = m3 + m5 + m6 + m7 is equivalent to _________
a) f = m3*m5*m6*m7
b) f = M0 + M1 + M2 + M4
c) f = M0*M1*M2*M4
d) f = M3 + M5 + M6 + M7
7) The _________ is a Boolean expression containing AND terms with one or
more literals each.
a) sum of products
b) standard POS
c) products of sums
d) standard SOP
P.T.O.

SLR-BB 384

*SLRBB384*

-2-

8) _________ code is an examples of non-weighted codes.


a) BCD
b) Excess 3
c) 2421
d) 8,4,2,1
9) _________ is the number of inputs available in a gate.
a) Fan-out
b) Fan-in
c) Noise Margin

d) None of above

10) Simplification of the Boolean expression (x + y) (x + y ) is


c) xy
d) x
a) y
b) x y
11) State reduction gives
a) reduction in number of flip-flops
c) (a) or (b)

b) number of flip-flop remains same


d) none of the above

12) Which one of the following is used as toggle switch ?


a) D flip-flop
b) T flip-flop
c) SR flip-flop
d) None of the above
13) The output Qn of JK flip-flop is 1. It changes to 0 when a clock pulse is
applied. The input Jn and Kn are respectively.
a) 1 and X
b) 0 and X
c) X and 0
d) X and 1
14) The characteristic equation of T flip-flop is given by
c) Q + = TQ + Q
a) Q + = TQ + TQ b) Q + = TQ
15) A decade counter has ____________ different states.
a) 8
b) 9
c) 10

d) Q + = TQ + TQ
d) 15

16) In initial content of 4-bit serial-in-parallel out, right shift, shift register shown in
figure is 0110. After three clock pulses are applied contents of shift register will be

a) 1010

b) 0101

c) 0011

d) 1011

17) A 4-bit pre-settable down counter has preset input 0100. The preset operation
takes place as soon as counter becomes minimum 0000. The modulus of
counter is
a) 6
b) 5
c) 4
d) 8
18) When a flip-flop is reset, its output will be
a) Q = 0, Q = 0
b) Q = 1, Q = 0 c) Q = 0, Q = 1 d) Q = 1, Q = 1
19) A flip-flop is used to store
a) one bit of information
b) two bits of information
c) one byte of information
d) one nibble of information
20) The number of states in 5 bit Johnsons counter are
a) 8
b) 4
c) 16
d) 10
______________
Set A

*SLRBB384*

-3-

SLR-BB 384

Seat
No.

S.E. (E&E) (Part II) Examination, 2014


DIGITAL TECHNIQUES (New)
Day and Date : Saturday, 29-11-2014
Time : 3.00 p.m. to 6.00 p.m.

Marks : 80

Instructions : 1) All questions are compulsory.


2) Figures to the right indicate full marks.
3) Assume suitable data wherever necessary.
SECTION I
2. Solve any four :

(44=16)

1) Explain what do you mean by Dont care condition with suitable example.
2) Minimise the four variable logic function using k-map.
f(A, B, C, D) = m (0, 1, 2, 3, 5, 7, 8, 9, 11, 14).
3) Draw and explain parallel adder.
4) Write a short note on priority encoder.
5) Explain NMOS inverter.
3. Solve any three :

(83=24)

1) Simplify the following expression using k-map.


f = AB + A C + AD + A B C + ABC
2) Implement full adder using half adder.
3) Design a 2-bit comparator using k-map to compare the relative magnitude of
2 binary number A and B, having 3 outputs indicating whether A > B, A < B or
A = B.
4) Give the detailed comparison between TTL and CMOS.
Set A

SLR-BB 384

-4-

*SLRBB384*

SECTION II
4. Attempt any four :

(44=16)

1) Give characteristics equation and excitation table for J-K flip-flop.


2) Draw and explain Johnsons counter.
3) With down the different application of counter.
4) Explain clocked SR flip-flop with present and clear.
5) Explain Serial in Serial out shift register.
5. Solve any three :

(83=24)

1) Draw and explain bidirectional shift register.


2) Design 3-bit synchronous down counter.
3) Explain the general model for conversion of flip-flop. Convert SR flip-flop to
J-K flip-flop.
4) Design 4-bit ripple counter using flip-flop and differentiate asynchronous and
synchronous counter.
_____________________

Set A

SLR-BB 385

*SLRBB385*
Seat
No.

Set

T.E. (Electrical and Electronics) (Part I) Examination, 2014


ELECTROMAGNETIC ENGINEERING (Old)
Day and Date : Wednesday, 3-12-2014
Time : 3.00 p.m. to 6.00 p.m.

Max. Marks : 100

Instructions : 1) Non programmable calculator is allowed.


2) Missing data may be suitably assumed.
3) Q. No. 1 is compulsory. It should be solved in first 30
minutes in Answer book Page No. 3. Each question carries
one mark.
4) Answer MCQ/Objective type questions on Page No. 3 only.
Dont forget to mention, Q.P. Set (A/B/C/D) on Top of Page.
MCQ/Objective Type Questions
Duration : 30 Minutes

Marks : 20

1. MCQ questions solve all twenty :

(120=20)

1) Electric field intensity is


a)

Volts
meter

b)

Amperes
meter

c)

Newton
coulomb

d) a) and c)

2) Vector is the quantity which is completely defined by its


a) Magnitude
b) Direction
c) Both a) and b)

d) None of these

3) If cross product of two vector is zero then vectors are


a) Co-linear
b) Parallel
c) Perpendicular

d) None of these

4) .D = 1 is
a) Ohms law
H
is
t
a) Amperes law

b) Faradays law c) Coulombs law d) Gausss law

5) E =

6) Energy is
a) Power/sec

b) Faradays law c) Gausss law

d) Poynting law

b) Power sec

d) Joules

c) Watts

7) . ( H) is
a) Curl of divergence of H
c) Divergence of H

b) Divergence of Curl of H
d) Curl of H

8) . ( A ) is
a) 1

c) 0

b) 2

d) 3
P.T.O.

SLR-BB 385

*SLRBB385*

-2-

9) A B C value is
a) B(A.C) C(A.B)

b) A(B.C) + B(A.C)

c) B(A.C) + C(A.B)

d) C(A.B) A(B.C)

10) Volume charge density v is


a) v =
11) dB =

ch arg e
volume

Idl sin

4r 2
a) Joules law

b)

ch arg e
area

c)

ch arg e
unit length

d) None of these

is

D
is
t
a) Amperes law

b) Amperes law

c) Gausss law

d) Biot and Savart law

12) H = J +

13)

b) Maxwells law c) Faradays law

D
is known as
t
a) Diffusion current density
c) Displacement current density

d) Gausss law

b) Drift current density


d) Conduction current density

14) Intrinsic impedance for free space is


a) 400
b) 360

c) 377

d) 250

15) H = J is known as
a) Faradays law
b) Amperes law

c) Joules law

d) Ohms law

16) Uniform plane waves means


a) Waves are travelling in homogeneous medium
c) u and E are same every where in medium

b) Heterogeneous medium
d) u and E are different

17) Ey Hz is the plane wave propagating in


a) Positive x direction
c) Negative y direction

b) Negative x direction
d) Positive y direction

18) Velocity of propagating wave is given by

19) Wave length, frequency and velocity are related by


a) v =

a) v =

b)

c)

b) v = f

c) v =

d) None of the above

d) None of these

20) Velocity of wave in free space is 3 108 m/s. Then its velocity in a medium having
r = 1 r = 9 is
a) 3 108
b) 5 108
c) 7 108
d) 1 108
______________
Set A

*SLRBB385*

-3-

SLR-BB 385

Seat
No.

T.E. (Electrical and Electronics) (Part I) Examination, 2014


ELECTROMAGNETIC ENGINEERING (Old)
Day and Date : Wednesday, 3-12-2014
Time : 3.00 p.m. to 6.00 p.m.

Marks : 80

Instructions : 1) Non programmable calculator is allowed.


2) Missing data may be suitably assumed.
3) Marks for each question are indicated to the right of
question.
SECTION I
2. Solve any four questions :
a) State and prove gausss law.

(45=20)

d
H
.
prove the result E =
dt
dt
c) State and prove Divergence theorem.

b) Starting from Faradays law v =

d) Find the charge in the volume bounded by 0 x 1, 0 y 1, 0 z 1 if


v = 30 x 2 y c m 3 .
e) Charge is uniformly distributed along infinite straight line with linear charge density
of l c/m find the expression for E at any general point P on line.
3. Solve any two questions :

(210=20)

a) With usual notations show that E = V .


40
nc is uniformly distributed in the form of disc of radius 2m. Find
3
the potential due to this charge at a point on the axis 2m from the disc. Compare
the result with that result if all the charges are at the centre of the disk.

b) Total charge of

c) Two pith balls of mass m kg each are hanged at a point by means of two equal
length L of thread. Then each is given a +9 charge. In equilibrium position each
thread makes an angle with the vertical then show that
tan 3
1 + tan 2

92
16 MgL2

where 9 = acceleration due to gravity.

Set A

SLR-BB 385

-4-

*SLRBB385*

SECTION II
4. Solve any 4 questions :

(45=20)

a) State and explain Biot and Savart law.


b) Find an expression for B at a perpendicular distance R meter away from finite
length of a conductor uniformly charged with l c/m. Use Biot and Savart law.
c) From amperes work law deduce the result H = J .
d) A circular cross section of radius 1.5m carries a conduction current of ic = 5.5 sin
(4 1010t) A . What is the amplitude of the displacement current density it
= 35 MS/m. and r = 1 .
e) An area of 0.65 m2 in z = 0 plane is enclosed by filamentary conductor. Find the
induced voltage given that
ay + az
B = 0.05 cos 10 3 t
2

web/m2.

5. Solve any two questions :

(210=20)

a) Derive three dimensional wave equation in E in terms of , and for


propagating wave.
b) Find the boundary conditions in magnetic field between two media having
1 and 2 .
c) Calculate intrinsic impedance propagation constant and wave velocity v for
conducting medium in which = 58 MS/m ur = 1 at frequency f = 100 MH.
_____________________

Set A

SLR-BB 386

*SLRBB386*
Seat
No.

Set

T.E. Electrical and Electronics Engineering (Part I) Examination, 2014


ELECTRIC POWER GENERATION AND UTILIZATION (Old)
Day and Date : Friday, 5-12-2014
Time : 3.00 p.m. to 6.00 p.m.

Total Marks : 100

Instructions : 1) Assume suitable data wherever necessary.


2) Non-programmable calculators are permitted.
3) Q. No. 1 is compulsory. It should be solved in first 30 minutes
in Answer book Page No. 3. Each question carries one mark.
4) Answer MCQ/Objective type questions on Page No. 3 only.
Dont forget to mention, Q.P. Set (A/B/C/D) on Top of Page.
MCQ/Objective Type Questions
Duration : 30 Minutes

Marks : 20

1. Choose the correct option for the following :


(201=20)
1) A Francis turbine is
a) Inward flow reaction turbine
b) Inward flow impulse turbine
c) Outward flow reaction turbine
d) Outward flow impulse turbine
2) Reflecting mirrors used for exploiting solar energy are called
a) Diffusers
b) Ponds
c) Heliostats
d) Mantle
3) In geothermal power plants waste water is
a) Recirculated after cooling in cooling towers b) Discharged into sea
c) Discharge back to earth
d) Evaporated in ponds
4) A graphical representation between discharge and time is known as
a) monograph
b) hectograph
c) topograph
d) hydrograph
5) A photovoltaic cell converts
a) Heat energy into mechanical energy b) Chemical energy into electrical energy
c) Solar energy into electrical energy
d) Electrical energy into chemical energy
6) A graphical representation between discharge and time is known as
a) monograph
b) hectograph
c) topograph
d) hydrograph
7) In pumped storage
a) power is produced by means of pump
b) water is stored by pumping to high pressures
c) downstream water is pumped upstream during off load period
d) water is recirculated through turbine
8) When maximum and average loads are equal, the load factor will be
a) zero
b) 1.0
c) 0.01
d) 0.5
P.T.O.

SLR-BB 386

-2-

*SLRBB386*

9) A low utilization factor for a plant indicates that


a) Plant is used for stand by purpose only
b) Plant is used for base load only
c) Plant is under maintenance
d) Plant is used for peak load as well as base load
10) During load shedding
a) System voltage is reduced
b) Some loads are switched off
c) System frequency is reduced
d) System power factor is changed
11) Steel pipes are manufactured by
a) Arc welding
b) Thermite welding
c) Resistance welding
d) Argon arc welding
12) Which of the following is of high value in case of induction heating ?
a) Frequency
b) Voltage
c) Current
d) All of the above
13) Highest power factor can be expected in which method of heating ?
a) electric arc heating
b) induction heating
c) dielectric heating
d) resistance heating
14) In dielectric heating, if the capacitor is loss-free, the heat produced will be
a) Zero
b) Proportional to the value of capacitance
c) Infinity
d) Proportional to the frequency
15) As the thickness of the part to be welded increases, which of the following
parameter for ac welding should also increase ?
a) Voltage
b) Current
c) Frequency
d) All of the above
16) A train has a schedule speed of 36 km per hour on a level track. If the distance
between the stations is 2 km and the stoppage is 30 seconds the actual time
of run will be
a) 260 seconds
b) 230 seconds c) 200 seconds d) 170 seconds
17) Which of the following factor affects specific energy consumption ?
a) Distance between stops
b) Gradient
c) Retardation and acceleration values
d) All of the above
18) When the speed of the train is estimated taking into account the time of stop
at a station in addition to the actual running time between stops, is known as
a) Average speed
b) Schedule speed
c) Notching speed
d) Free running speed
19) The advantage of electric braking is
a) instantaneous
b) more heat is generated during braking
c) it avoids wear of track d) motor continue to remain loaded during braking
20) The coasting retardation on trains is approximately
a) 0.16 km phps
b) 1.6 km phps c) 16 km phps
d) 25 km phps
______________
Set A

*SLRBB386*

SLR-BB 386

-3-

Seat
No.

T.E. Electrical and Electronics Engineering (Part I) Examination, 2014


ELECTRIC POWER GENERATION AND UTILIZATION (Old)
Day and Date : Friday, 5-12-2014
Time : 3.00 p.m. to 6.00 p.m.

Marks : 80

Instructions : 1) Assume suitable data wherever necessary.


2) Non-programmable calculators are permitted.
SECTION I
2. Solve any four :
(45=20)
a) Explain Pelton Turbine with neat diagram.
b) Draw neat diagram of advanced gas reactor.
c) The load curve of an electrical system is linear with the following values at
different times of the day.
Time :

06

6 10

20

25

Load (mw) :

10 12 12 16 16 20 20 24
30

25

35

20

Draw the load curve and find :


i) maximum demand
ii) units generated per day
iii) average load
iv) load factor.
d) Write a short on Penstock and Spillway.
e) Explain the terms :
1) Maximum demand
2) Average demand
3) Plant load factor.
f) Explain geothermal power plant.
Set A

SLR-BB 386

-4-

*SLRBB386*

3. Attempt any two :

(210=20)

a) What is pumped storage plant ? Explain various parts of hydroelectric


power plant with their functions.
b) What is a site selection criterion for nuclear power plant ? Explain nuclear
power plant with the help of block diagram.
c) What is operating principle of wind power plant ? With the help of block
diagram explain various component of wind power plant.
SECTION II
4. Solve any four :

(45=20)

a) Draw a neat sketch of spot welding machine and explain its working.
b) Explain primary and secondary system.
c) What is specific energy consumption of train and discuss various factors
affecting it ?
d) State the advantages of electrical heating.
e) Explain construction and working of liquid metal cooled reactor.
f) Define the terms :
1) Crest speed
2) Average speed
3) Schedule speed.
5. Attempt any two :

(210=20)

a) A train is required to run between two stations 1.6 km apart at an average


speed of 40 kmph. The run is to be made to a simplified quadrilateral speed
time curve. If the maximum speed is to be limited to 64 kmph, acceleration to
2.0 kmphps and coasting and breaking retardation to 0.16 kmphps and 3.2
kmphps respectively, determine the duration of acceleration, coasting and
braking periods.
b) Explain simplified trapezoidal speed time curve and quadrilateral speed time
curve.
c) What are the desirable properties of material for heating element should have
and explain design procedure of heating element when power and volt of the
oven are known ?
_____________________

Set A

SLR-BB 387

*SLRBB387*
Seat
No.

Set

T.E. (Electrical and Electronics Engineering) (Part I) Examination, 2014


MICROPROCESSOR (Old)
Day and Date : Monday, 8-12-2014
Time : 3.00 p.m. to 6.00 p.m.

Max. Marks : 100

Instructions : 1) Figures to the right indicates full marks.


2) Assume suitable data if necessary.
3) Q. No. 1 is compulsory. It should be solved in first 30 minutes
in Answer Book Page No. 3. Each question carries one mark.
4) Answer MCQ/Objective type questions on Page No. 3
only. Dont forget to mention, Q.P. Set (A/B/C/D) on Top
of Page.
MCQ/Objective Type Questions
Duration : 30 Minutes

Marks : 20

1. Select suitable option :

(201=20)

1) A resolution of an ADC depends upon


a) supply voltage
b) number of bits
c) conversion time
d) ramp slope
2) Take odd man out with reference to 8259 : cascade mode, polled mode,
strobe mode, fully nested mode
a) cascade mode
b) polled mode
c) strobed mode
d) fully nested mode
3) Which of below can be used as an output port
a) buffer
b) decoder
c) latch

d) all of these

4) The program that allow to set break points is


a) assembler
b) linker
c) breaker

d) debugger

5) Which of below is true for stack


a) it is located in RAM
c) it is accessed for 16 bit data

b) it is LIFO
d) all of these

6) Instruction SIM is used for


a) setting interrupt mask
c) both a) and b)

b) outputting serial data


d) none of these
P.T.O.

SLR-BB 387

*SLRBB387*

-2-

7) The instructions used for data transfer in memory mapped I\O are
a) IN, OUT
b) LDA, STA
c) IN, STA
d) OUT, LDA
8) Instruction to call a subroutine if sign flag is reset is
a) CC
b) CNC
c) CM

d) CP

9) In an instruction OUT BCH, what is BC


a) memory location address
b) port address
c) register pair
d) immediate data
10) MC 1488 is
a) line driver/receiver
c) line received

b) line driver
d) voltage doubler

11) Which of below can be used for clock source of 8085 ?


a) LC
b) RC
c) Crystal

d) All of these

12) In RIM instruction format, bit D3 is


a) SID
b) IE

c) M 7.5

d) M 6.5

13) Output of DAC 0808 is _________


a) voltage
b) current

c) power

d) none of these

14) If memory map of a memory IC is 2000H to 3FFFH, then memory size is


a) 2 K
b) 4 K
c) 8 K
d) 16 K
15) First machine cycle for the instruction CALL ABCDH is _________
a) memory read b) opcode fetch c) stack puch
d) invalid instruction
16) Which of below is control word for 8251
a) mode word
b) command word
c) both a) and b)
d) none of these
17) Interfacing one stepper motor and eight linear keys requires
a) two 8255
b) one 8255
c) one 8255 and one 8254
d) one 8255 and one 8251
18) For 8254, make odd man out : CLK1, A1, VCC, OUT2
a) CLK1
b) A1
c) OUT2

d) VCC

19) Which of below is not an arithmetic instruction ?


a) INR
b) ADI
c) RAR

d) DCX

20) Which of below is not a functional block of 8254 ?


a) counter 0
b) modem control
c) transmit control
d) receive buffer
______________
Set A

*SLRBB387*

-3-

SLR-BB 387

Seat
No.

T.E. (Electrical and Electronics Engineering) (Part I) Examination, 2014


MICROPROCESSOR (Old)
Day and Date : Monday, 8-12-2014
Time : 3.00 p.m. to 6.00 p.m.

Marks : 80

Instructions : 1) All questions are compulsory.


2) Figures to the right indicates full marks.
3) Assume suitable data if necessary.
SECTION I
2. Solve any three :

(38=24)

a) Interface an eight LEDS and eight switches with 8085 using suitable output
and input ports. Assume suitable addresses. Write a program to input status
of switches and show it on LEDs.
b) Write a program to produce a software time delay of exact 3.2 seconds.
Show all necessary calculations.
c) Draw and explain timing diagram for instruction MVI C, FFH.
d) With suitable diagram explain demultiplexing of address and data bus.
3. Solve any four :

(44=16)

a) Two arrays of ten bytes each are located from 2000H and 2010H onwards.
Write a program to add respective numbers in these arrays and store results
in third array starting from 2020H onwards.
b) With suitable example (one each) explain logical instructions, branching
instructions.
c) Write for all hardware interrupts of 8085 priority, masking status, vectoring
address, level or edge triggering.
d) With suitable example explain use of a stack.
e) Explain the control and status signals of 8085.
Set A

SLR-BB 387

-4-

*SLRBB387*

SECTION II
4. Solve any three :

(38=24)

a) Draw and explain detailed interfacing of 8254 with 8085. Assume suitable
addresses.
b) With suitable example explain how serial communication is supported using
SID and SOD pins.
c) Interface a temperature sensor with 8085 using ADC 0808. Assume suitable
addresses. Write a program to input a digital count from ADC and store it in
memory.
d) Draw and explain internal block diagram of 8259.
5. Solve any four :

(44=16)

a) With suitable timing diagram explain Mode 4 of 8254.


b) Explain cascade mode of 8259.
c) Explain binary weighted type DAC. What are its disadvantages ?
d) Explain frequency measurement using 8085.
e) Explain BSR mode of 8255.
_____________________

Set A

SLR-BB 388

*SLRBB388*
Seat
No.

Set

T.E. (Electrical and Electronics Engineering) (Part I) (Old) Examination, 2014


DIGITAL SIGNAL PROCESSING
Day and Date : Wednesday, 10-12-2014
Time : 3.00 p.m. to 6.00 p.m.

Total Marks : 100

Instructions : 1) Assume suitable data if necessary.


2) Q. No. 1 is compulsory. It should be solved in first 30 minutes in
Answer Book Page No. 3. Each question carries one mark.
3) Answer MCQ/Objective type questions on Page No. 3 only.
Dont forget to mention, Q.P. Set (A/B/C/D) on Top of Page.
MCQ/Objective Type Questions
Duration : 30 Minutes

Marks : 20

1. Choose the correct answer :

20

1) The direct evaluation DFT requires _________ complex multiplications.


a) N(N 1)

b) N2

2) DFT is ___________
a) Discrete Fourier series
c) Discrete time Fourier transform

c) N(N + 1)

d)

N(N 1)
2

b) Discrete Fourier transform


d) Discrete time Fourier series

3) The fast Fourier transform is an algorithm used to compute the _________


a) DFT
b) FT
c) Convolution
d) Circular convolution
4) Circular convolution of the sequences x(n) = {1, 2, 1} and h(n) = {1, 2, 2} is
a) {1, 2, 1}
b) {3, 2, 1}
c) {3, 2, 1}
d) {3, 2, 2}
5) The methods to calculate filtering of long duration sequences is
a) Overlap save
b) Overlap add
c) FIR and IIR
d) Both a) and b)
6) When the system has poles inside the unit circle in Z-domain ?
a) The system is stable and its impulse response is a decaying function
b) Time domain behavior will be exponentially rising signal
c) The system is unstable
d) The impulse response is marginally constant
7) If X(k) is N-point DFT of finite duration sequence x(n), then
a) x(n + N) = x(n)
b) x(n N) = x(n)
c) x(n + N) = x(n)
d) None of the above
8) The Direct form II realization requires ___________ no. of multiplications.
a) M N + 1
b) M + N + 1
c) M + N
d) M N
P.T.O.

SLR-BB 388

*SLRBB388*

-2-

9) Fourier transform of x(t) can be given as


a) x( j) =
c) x( j) =

x(t ) e jt dt

b) x( j) =

x(t ) e jt dt

x(t) e jnt dt

d) None

10) Main disadvantage of direct form II realization


a) Quantization error
b) Requires more memory
c) Sensitive to error
d) All above
11) IIR filter design is based on _________ filter design.
a) Analog
b) Digital
c) DT

d) None of above

12) Aliasing is present in which filter technique ?


a) Impulse invariant technique
b) Bilinear transformation
c) Approximation of derivatives
d) The matched Z-transform
13) IIR filters are _________ type.
a) Non Recursive b) Recursive

c) Causal

d) None

14) Using impulse invariant technique which type of filters are designed ?
a) Only High pass filters
b) Only LPF
c) Only BPF
d) Both LPF and BPF
15) The mapping for BLT method is
a) One to one
b) Many to one

c) Many to many

d) None of above

16) Memory requirement and execution time required for FIR filter is
a) High
b) Low
c) Very high
d) Very low
17) FIR filters are always
a) Unstable
b) Quasi stable

c) Stable

d) None of above

18) FIR filters have constant


a) Group delay
b) Phase delay

c) Both a) and b)

d) None

19) FIR filter has a ____________


a) Non linear phase response
c) Linear phase response

b) Quasi phase response


d) All above

20) The main lobe width of hanning window is __________ that of the rectangular
window.
a) Twice
b) Thrice
c) Four times
d) Five times
______________

Set A

*SLRBB388*

-3-

SLR-BB 388

Seat
No.

T.E. (Electrical and Electronics Engineering) (Part I) (Old) Examination, 2014


DIGITAL SIGNAL PROCESSING
Day and Date : Wednesday, 10-12-2014

Marks : 80

Time : 3.00 p.m. to 6.00 p.m.


Instructions : 1) All questions are compulsory.
2) Assume suitable data if necessary.
SECTION I
2. Attempt any four :

(45=20)

1) Perform circular convolution using concentric circle method for x1(n) = {1, 1, 2, 1}
and x2(n) = {1, 2, 3, 4}.
2) Differentiate between linear convolution and circular convolution.
3) What is zero padding ? What are its uses ?
4) Compute four point DFT of a sequence x(n) = {0, 1, 2, 3} using DIT FFT algorithm.
5) Find DFT of x(n) = an.
3. Attempt any two :

(210=20)

1) Find the DFT of a sequence x(n) = {1, 2, 3, 4, 4, 3, 2, 1} using DIT algorithm.


2) Find the output value of a filter whose impulse response is h(n) = {1, 1, 1} and input
signal is x(n) = {3, 1, 0, 1, 3, 2, 0, 1, 2, 1} using
a) overlap save method and
b) overlap add method.
3) Obtain the cascade realization with minimum number of multipliers
H(z) = (1 + z 1) 1 1 z 1 + 1 z 2 .
2
2 4

Set A

SLR-BB 388

*SLRBB388*

-4-

SECTION II
4. Attempt any four :

(45=20)

1) Explain VLIW architecture.


2) Design an ideal LPF with a frequency response Hd (e j ) = 1 for .
2
2
= 0 for
2
Find values of h(n) for N = 4 using rectangular window.
3) Explain adaptive noise cancellation.
4) By impulse invariance method obtain the digital filter transfer function of analog
filter H(s) =

1 .
s+1

5) Explain in detail LMS filter and prove how it is adaptive filter.


5. Attempt any two :

(210=20)

1) Explain in detail floating point digital signal processor TMS320C54XX.


2) Apply impulse invariant method and find H(z) for H(s) =

1
with
s 2 + 2s + 1

T = 1 sec.
/ / 3 .
4
4
= 0 for otherwise

3) Design an ideal BPF with a frequency response Hd (e j ) = 1 for


Find values of h(n) for N = 11 plot frequency response.
_____________________

Set A

SLR-BB 389

*SLRBB389*
Seat
No.

Set

T.E. (Electrical and Electronics) (Part I) Examination, 2014


FEEDBACK CONTROL SYSTEM (Old)
Day and Date : Friday, 12-12-2014
Time : 3.00 p.m. to 6.00 p.m.

Max. Marks : 100

Instructions : i) Q. No. 1 is compulsory. It should be solved in first 30 minutes in Answer


Book Page No. 3. Each question carries one mark.
ii) Assume suitable data, if necessary.
iii) Use of non-programmable calculator is allowed.
iv) Figures to the right indicates full marks.
v) Answer MCQ/Objective type questions on Page No. 3 only. Dont forget to
mention, Q.P. Set (A/B/C/D) on Top of Page.
MCQ/Objective Type Questions
Duration : 30 Minutes

Marks : 20

1. Choose the correct answer :

(201=20)

1) An open loop control system has its


a) Control action independent of the output or desired quantity
b) Controlling action, depending upon human judgement
c) Internal system changes automatically taken care of
d) Both a) and b)
2) A good factor for Mp should be
a) Less than 1
c) More than 2.2

b) Lying between 1.1 and 1.5


d) Zero

3) The valid relation between setting time ts and rise time tr is


a) tr > ts
b) ts > tr
c) ts = tr

d) none of these

4) Peak overshoot of step-input response of an underdamped second-order system explicitly


indicative of
a) Settling time
b) Rise time
c) Natural frequency
d) Damping ratio
5) A transfer function of a system is a Laplace transform of its
a) Square wave response
b) Step response
c) Ramp response
d) Impulse response
6) Introduction of the feedback reduces of effects of
a) Noise signal
b) Disturbances
c) Error signals
d) Noise signals and disturbances
7) Regenerative feedback
a) Implies feedback with positive sign
b) Is sometimes used to increase the loop gain of the feedback system
c) Has the transfer function with a negative sign in the denominator
d) All the above
8) Transfer function of the control system depends on
a) Initial conditions of input and output
b) System parameters alone
c) Nature of the input
d) Nature of the output
P.T.O.

SLR-BB 389

*SLRBB389*

-2-

9) Error constants of a system are measure of


a) Relative stability
b) Transient state response
c) Steady state response
d) Steady state as well as transient state response
10) The steady state error of a control system can be reduced by increasing
a) Gain constant of the system
b) Time constant of the system
c) Both the gain constant and time constant of the system
d) None of the above
11) The integral control
a) Increases the steady state error
c) Increases the noise and stability

b) Decreases the steady state error


d) Decreases the damping coefficient

12) The frequency and time domain are related through which of the following ?
a) Laplace Transform and Fourier Integral
b) Laplace Transform
c) Fourier Integral
d) Either b) or c)
13) In a PID controller, the offset has increased. The integral time constant has to be
________________ so as to reduce offset.
a) Reduced
b) Increased
c) Reduced to zero
d) None of the above
14) An increase in gain, in most systems, leads to
a) Smaller damping ratio
c) Constant damping ratio

b) Larger damping ratio


d) None of the above

15) A conditionally stable system exhibits poor stability at


a) Low frequencies
b) Reduced values of open loop gain
c) Increased values of open loop gain
d) None of the above
16) In a proportional temperature controller, if the quantity under the heater increases the offset will
a) Increase
b) Reduce
c) Remain unaffected
d) None of the above
17) Proportional band of the controller is expressed as
a) Gain
b) Ratio
c) Percentage
d) Range of control variables
18) The state variable approach can be applied to ________________ system(s).
a) Time variant
b) Non-linear
c) Linear and time-invariant
d) All of the above
19) With the knowledge of state space representation the transfer function of the system
a) Can be determined partly
b) Can be determined completely
c) Cannot be determined
d) None of the above
20) Which of the following is used to increase the bandwidth of a control system ?
a) Phase lag compensator
b) Phase lead compensator
c) Phase lag-lead compensator
d) All of these
______________
Set A

*SLRBB389*

-3-

SLR-BB 389

Seat
No.

T.E. (Electrical and Electronics) (Part I) Examination, 2014


FEEDBACK CONTROL SYSTEM (Old)
Day and Date : Friday, 12-12-2014

Marks : 80

Time : 3.00 p.m. to 6.00 p.m.


Instructions : 1) All questions are compulsory.
2) Figures to the right indicates full marks.
3) Assume suitable data wherever necessary.
SECTION I
2. Solve any four :

(54=20)

a) Derive the transfer function of field controlled DC servomotor.


b) Explain any five block diagram reduction rules in detail.
c) Give the transfer function of series RLC circuit.
d) What is Routh-Hurwitz stability criterion ? Explain.
e) Explain the linearizing effect of feedback.
3. Solve any two :

(102=20)

a) Explain concept of Root Locus with an example.


b) For the physical system shown below draw its equivalent system and write equilibrium equations.
Hence draw its electrical analogous circuits based on :
i) Force Voltage
ii) Force Current Method.

c) With neat sketch explain all the time domain specifications.


Set A

SLR-BB 389

*SLRBB389*

-4-

SECTION II
4. Solve any four :

(54=20)

a) Derive the expression for bandwidth of the system.


b) Explain PD Controller with diagram.
c) Draw the architecture of PLC.
d) Explain Controllability and Observability of the control system.
e) Draw the polar plot for the system with G(s) = s/(s + 1).
5. Solve any two :

(102=20)

a) Sketch the Bode Plot for the system with TF given by G(s) =

80
and hence
s(s + 2) (s + 20)

determine the gain margin and the phase margin of the system.
0 3
b) Obtain the state transition matrix for the state model whose matrix A =
.
1 4

c) Derive the TF for Lag compensator and explain the PID Controller.
_____________________

Set A

SLR-BB 39

*SLRBB39*
Seat
No.

Set

T.E. (Civil) (Part I) (Old) Examination, 2014


BUILDING PLANNING AND DESIGN
Day and Date : Monday, 8-12-2014
Time : 3.00 p.m. to 7.00 p.m.

Total Marks : 100

Instructions : 1) Q. No. 1 is compulsory. It should be solved in first 30


minutes in Answer book Page No. 3. Each question carries
one mark.
2) Answer MCQ/Objective type questions on Page No. 3 only.
Dont forget to mention, Q.P. Set (A/B/C/D) on Top of Page.
MCQ/Objective Type Questions
Duration : 30 Minutes

Marks : 20

1. State whether following statement is correct or incorrect :

20

1) In AutoCAD, the Arc command creates a line similar to a construction line


except that it extends infinitely in only one direction from the first pick point.
2) Rainwater harvesting is strictly prohibited in green buildings.
3) While drawing AutoCAD drawing, the Ellipse arc command can be used to
draw top view of European style W C pan.
4) Concept of FSI is used only in residential building and not at all in public
buildings.
5) In architecture, pattern is the repetition of all or some of the elements of
design, such as lines, shapes, and forms.
6) In case of fire hazard in a public building it is desirable to use staircase
instead of lift.
7) Contrast can be recognized in a building, when two adjacent parts are very
different from one another, different materials, colors and textures.

P.T.O.

SLR-BB 39

-2-

*SLRBB39*

8) ICU in hospital, means illness cure unit.


9) One point perspective is easier to draw.
10) Unit of fire load is Kcal/Sq.m/hr.
11) As per SP : 7-1970 type I buildings have highest fire resistance.
12) Lime stone is excellent fire resisting material.
13) Number of cycles or vibrations per second is known as pitch or frequency.
14) Dead spots occur because of formation of sound foci.
15) Excessive reverberation occurs because of insufficient absorption.
16) In case of classrooms minimum sound of 78 dB shall be received by pupil in
last row.
17) For indoor patient wards, there should be atleast one WC for every 8 beds or
parts thereof.
18) Area of 15 Sq. m. shall be considered per bus while designing bus station.
19) In banks, height of public counters shall be about 0.5 m.
20) Minimum width of corridors in public buildings is 0.9 m.
______________

Set A

*SLRBB39*

-3-

SLR-BB 39

Seat
No.

T.E. (Civil) (Part I) (Old) Examination, 2014


BUILDING PLANNING AND DESIGN
Day and Date : Monday, 8-12-2014
Time : 3.00 p.m. to 7.00 p.m.
Instructions : 1)
2)
3)
4)
5)
6)

Marks : 80

All questions are compulsory.


Use both sides of full imperial drawing sheet for Section I.
Figures on right indicate full marks.
Assume suitable data wherever needed and mention it.
Use answer book for Section II.
Retain all projections / construction lines on drawing sheet.

SECTION I
2. A sugar factory wish to construct a public Bazar (Super Market) for general and
staff on its campus. A plot of 60 m 40 m is demarketed with a 12 m wide road
along one of its 60 m side.
Provide for following areas :
1) Entrance Lobby with Porch
2) Rolling Trolley stand
3) Goods display hall
4) Cash counters (Payment Counters) and Exit Lounge
5) Security guard room
6) Ladies changing room
7) Administrative office
8) CCTV serviliance room
9) Sanitary block

Ladies one
Gents one.

A) Plan and draw to scale 1 : 100 regular plan for building permission.

16

B) Draw vertical section to scale 1:100 of the building planned above.


OR
Draw to scale 1:50 furniture layout for goods display hall.

12
Set A

SLR-BB 39

*SLRBB39*

-4-

3. Draw perspective of the object using the data given as per enclosed sketch. Use
scale 1: 100. Symbols have usual meaning.
12
Fig shows a plan and elevation of a small structure. Draw a 2 point perspective
taking eye level at 2.0 m above G.L. The observer stands at 4.5 m along central
visual ray. Retain all construction lines.

Fig. not to scale

Elevation
SECTION II

4. Answer any five questions :

40

a) Explain the following w.r.t. architectural composition of a public building :


a) Unity
b) Mass composition
c) Contrast
b) Write a short note on Landscape and Interior Designing.
c) Write a note on :Rain Water Harvesting and future scope of it.
d) Explain in detail different types of fire hazards.
e) Explain salient features of a green building and its scope w.r.t public buildings.
f) Explain various acoustic defects which require due consideration for improving
the acoustical conditions.
g) Explain Form and Function w.r.t. architectural composition of a building.
_____________________

Set A

SLR-BB 390

*SLRBB390*
Seat
No.

Set

T.E. (Electrical and Electronics Engg.) (Part I) (New) Examination, 2014


ELECTROMAGNETIC ENGINEERING
Day and Date : Wednesday, 3-12-2014
Time : 3.00 p.m. to 6.00 p.m.

Max. Marks : 100

N. B. : 1) Figure indicates full marks.


2) Q. No. 1 is compulsory. It should be solved in first 30 minutes in
Answer book Page No. 3. Each question carries one mark.
3) Answer MCQ/Objective type questions on Page No. 3 only. Dont
forget to mention, Q.P. Set (A/B/C/D) on Top of Page.
MCQ/Objective Type Questions
Duration : 30 Minutes

Marks : 20

1. Choose the correct options :

(201=20)

1) The surface = constant is a __________ in spherical coordinate system.


a) Sphere

b) Circle

c) Rectangle

d) Cone

c) one

d) zero

2) The value of a y x( a x ) is __________


b) a z

a) a z

3) Electric field intensity E is ___________ of potential V.


a) Positive gradient

b) Curl

c) Negative divergence

d) Negative gradient

4) The unit of is___________


a) 1m
b) 1sec
5) E =

c) 1Joule

d) 1m sec

is ___________

a) Gauss law

b) Maxwell first equation

c) Both a) and b)

d) None of these

6) Energy density in electrostatic field is given by __________


2
a) 12 0 E

2
b) 12 0 E

c) 0 E 2

d) E0 E

7) If 2 V = 6 , then the scalar potential field V does not satisfy ________ equation.
a) Poissons

b) Couombs

c) Gausss

d) Laplace

P.T.O.

SLR-BB 390

*SLRBB390*

-2-

8) Capacitance of a coaxial cable is given by _________


a) C =

2 L
ln b
a

( )

b) C = 4 L

l (a )
b

c) C =

4 L
ln b
a

( )

d) C = 2 L

ln

(a b )

Where a = radius of inner conductor and b = radius of outer conductor.


9) The continuity equation of current is given by ____________
b) J = 0
c) J = 2 t v
a) J = v

d) J = d dt v

10) Any field that satisfies O


E dt = 0 is said to be _________ field.
a) Irrotational

b) Conservative

c) Solenoidal

d) Both a) and b)

11) The vector magnetic potential A is defined as __________


a) B = A

b) B = A

c) B = A

d) None of these

12) If the flux density is 20 wb/m2 and the area of the coil is 2m2, the flux is ________
a) 10 Wb

b) 40 Wb

c) 5 Wb

d) 60 Wb

c) Amp./m

d) Volt/m

13) The unit of scalar magnetic potential is


a) Ampere

b) Volt

14) When divergence is positive, then that region is said to contain _________
a) Sink

b) Source

c) Both a) and b)

d) Vector

15) According to gauss law which is the Maxwells equation ?


a)

B dS = 0

b)

B dL = 0

c)

B dv = 0

d) Both a) and b)

16) Application of Amperes circuital law to unsymmetrical surfaces yielded the concept of
___________
a) Dot

b) Curl

c) Divergence

d) None of these

c) N /I

d) N/ I

c) Weber/m.

d) Weber/sq-m.

17) Self-inductance L is given by ___________


a) NI/

b) I/N

18) Permittivity is expressed in


a) Farad/m.

b) Farad/sq-m.

19) In case of perfect dielectric the characteristics of wave impedance is given as


a) = /

b) = /

c) =

d) None of these

20) Wave equation in phasor form is known as _________


a) Maxwell Equation

b) Helmoltzs Equation

c) Amperes Law

d) Gauss Law

______________

Set A

*SLRBB390*

SLR-BB 390

-3-

Seat
No.

T.E. (Electrical and Electronics Engg.) (Part I) (New) Examination, 2014


ELECTROMAGNETIC ENGINEERING
Day and Date : Wednesday, 3-12-2014
Time : 3.00 p.m. to 6.00 p.m.

Marks : 80

SECTION I
2. Attempt any four questions :

(45=20)

a) Given A = ax ay , B = 2az and C = ax + 3ay , then find A B C and A B C .


b) With the help of Gauss law show that 2 V = v / 0 .
c) Two infinite uniform sheets are located at x = 1m. If s = 20c / m2 on each sheets then
determine E in all regions.

d) If D = 2y 2 + z ax + 4 xy ay + xaz C / m 2 . Then find the total charge enclosed by the cube


situated with one of its corners at origin and side 1m.
e) Derive the boundary condition for perfect dielectric in electrostatic field.
f) Two small identical spheres (conducting) have charges of 2109C and 0.5109C
respectively. When they are placed 4cm apart, what is the force between them. If they are
brought into contact and then separated by 4cm. what is the force between them ?
3. Attempt any two questions :

( )

(210=20)

3
2
a) Given that D = 10 3 x ax c / m . Evaluate both sides of divergence theorem for the
volume of a cube 2m on the edge centered at origin and with edges parallel to axis.

b) Derive an expression for capacitance of coaxial cable. What is dielectric strength and on
what factors does it depend on ?
c) Given potential field V = 2x2y 5z. Find the following :
1) Potential V
2) Electric field intensity E
3) Direction of E
4) Electric flux density D
5) v at P( 4, 3, 6).

Set A

SLR-BB 390

-4-

*SLRBB390*

SECTION II
4. Attempt any four :

(45=20)

1) State and prove Amperes Circuital Law.


2) The point charge Q = 18 nC has a velocity of 5106 m/s in the direction
av = 0.04 ax 0.05 ay + 0.2 az. Calculate the magnitude of force exerted on the charge
by the field B = 3ax + 4 ay + 6az T.
3) Two homogeneous linear isotropic material medium have an interface at X = 0 at which
there is a surface current K = 100 ay A/M2 for x<0, r 1= 2 and H1 = 150 ax 400 ay + 250 az A /M.
In region x > 0 and r 2 = 5. Find H2, B1 , B2.
4) Derive Poissons equation from Maxwells equation.
5) Write a short note on scalar and vector magnetic potential.
5. Attempt any two :

(210=20)

1) A 6580 MHz uniform plane wave is propagating in a medium of r = 2.25 , if the


amplitude of electric field intensity 500 v/m and material assumed to be lossless.
Find , Wavelength, velocity, , H.
2) Derive Maxwells equations in point form and integral form for harmonically varying
fields.
2
3) If Jd= 5 cos(2108t kz)ax A / m in a material for which = 0. = 5 0 , = 4 0 .
Find D, E, B, H, K.S.
_____________________

Set A

SLR-BB 391

*SLRBB391*
Seat
No.

Set

T.E. (Electrical and Electronics) (Part I) (New) Examination, 2014


INSTRUMENTATION TECHNIQUES
Day and Date : Friday, 5-12-2014
Time : 3.00 p.m. to 6.00 p.m.

Max. Marks : 100

Instructions : 1) Assume suitable data wherever necessary.


2) Figure to the right indicates full marks.
3) Q. No. 1 is compulsory. It should be solved in first 30 minutes in
Answer book Page No. 3. Each question carries one mark.
4) Answer MCQ/Objective type questions on Page No. 3 only. Dont
forget to mention, Q.P. Set (A/B/C/D) on Top of Page.
MCQ/Objective Type Questions
Duration : 30 Minutes

Marks : 20

1. Multiple choice questions :

(120=20)

1) The measurand is
a) Output
c) Secondary Signal

b) Measured variable
d) Any above

2) The input resistant of a CRO is of order of


a) Tens of
b) Mega

c) K

d) Fraction of

3) Uncertainty distribution is used for


a) Analysis of multisample data
b) Analysis of single sample data
c) Analysis of both single and multi sample data
d) None of above
4) Which of following errors can arise as a result of mistakes reading, improper instrument
location and inadequate lighting ?
a) Construction error
b) Transmission error
c) Observation error
d) Translation error
5) A buffer amplifier has gain of
a) Infinity
c) Unity

b) Zero
d) Dependent upon circuit parameter

6) AC amplifier are best suited for


a) Steady state signal
c) Rapidly varying signal

b) Low frequency signals


d) None of above

7) What is the desirable features in an electronic amplifier ?


a) High output impedance
b) Low input impedance
c) Good frequency response
d) All of above
8) The output from frequency modulation system is
a) ac voltage
b) dc voltage
c) ac and dc voltage
d) any of above
P.T.O.

SLR-BB 391

*SLRBB391*

-2-

9) The overall gain or amplification of system of two amplifier arranged in series is where
G1 and G2 are gains ____________
a) G1 + G2

b) (G1 G2)

c) G1 G2

d) G1 / G2

c) Only MF

d) MF and HF

10) The properties of an ideal op-amp are


a) It should have zero input impedance
b) It should have high input impedance
c) Should have a zero open loop gain
d) None of above
11) The FM systems operate at
a) VHF and UHF
b) Only VHF

12) FM System as compared to AM system are


a) Less affected by noise
b) Equally affected by noise
c) More affected by noise
d) Highly affected by noise since they operate at VHF and UHF
13) Time Division multiplexing requires.
a) Constant data transmission
c) Transmission of data at random

b) Transmission of data samples


d) None of above

14) Modem is an acronym of


a) Modulation
c) Modulation and Demodulation

b) Demodulation
d) All of above

15) TDM is used when the data to be transmitted


a) Is slow changing
b) Has small bandwidth
c) Is fast changing and has low bandwidth
d) Is fast changing and has low bandwidth
16) FM telemetry as compared with AM telemetry requires a channel that is
a) Smaller than what is required for AM telemetry
b) Equal of that of AM telemetry
c) 10 times that required for AM telemetry
d) 100 times that required for AM telemetry
17) An LCD requires a power of approximate
a) 20 W

b) 20 mW

c) 20 W

d) 20 nW

18) Which of the following has highest frequency response and least response time ?
a) X-Y plotter
b) U-Y recorder
c) Pen recorder
d) CRO
19) In cathode ray tube the focusing anode is located
a) Between pre-accelerating and accelerating anode
b) After accelerating anode
c) Before accelerating anode
d) None of above
20) The time bases of an oscilloscope are generated by
a) Horizontal amplifier
b) Vertical amplifier
c) Sweep generator
d) Storage oscilloscope
______________

Set A

*SLRBB391*

-3-

SLR-BB 391

Seat
No.

T.E. (Electrical and Electronics) (Part I) (New) Examination, 2014


INSTRUMENTATION TECHNIQUES
Day and Date : Friday, 5-12-2014

Marks : 80

Time : 3.00 p.m. to 6.00 p.m.


Instructions : 1) Assume suitable data wherever necessary.
2) Figure to the right indicates full marks.
SECTION I
2. Solve any four :

(54=20)

1) Derive equations for second order low pass butterworth filter.


2) Explain Data Acquisition System in detail.
3) Draw block diagram of Instrumentation system and explain function of each block.
4) Explain method of measurement of displacement with LVDT.
5) Derive equations for first order high pass Butterworth filter.

3. Solve any two :

(102=20)

1) With neat diagram explain programmable gain amplifier. Also derive the expression for
gain.
2) Explain with diagram method of voltage to frequency conversion and derive its equation.
3) Draw and explain instrumentation amplifier in detail.

Set A

SLR-BB 391

-4-

*SLRBB391*

SECTION II
4. Solve any four :

(54=20)

a) Explain rule of PLC in automation.


b) Describe the working of storage oscilloscope.
c) Explain about function generator.
d) Write a short notes on LED and LCD displays.
e) Explain position telemetry system.
5. Solve any two :

(102=20)

a) What are the operating principles of LCD display ? Explain seven segment LED display.
b) What are the types of PLC system ? Explain ladder diagram with one industrial
application.
c) Define telemetry ? Explain current and voltage telemetry.
_____________________

Set A

SLR-BB 392

*SLRBB392*
S

T.E. (Electrical and Electronics Engineering) (Part I) (New)


Examination, 2014
MICROPROCESSOR AND ITS APPLICATIONS
Day and Date : Monday, 8-12-2014
Time : 3.00 p.m. to 6.00 p.m.

Max. Marks : 100

Instructions : 1) Figures to the right indicates full marks.


2) Assume suitable data if necessary.
3) Q. No. 1 is compulsory. It should be solved in first 30
minutes in Answer Book Page No. 3. Each question carries
one mark.
4) Answer MCQ/Objective type questions on Page No. 3
only. Dont forget to mention, Q.P. Set (A/B/C/D) on Top
of Page.
Duration : 30 Minutes

MCQ/Objective Type Questions

Marks : 20

1. Select suitable option.

(201=20)

1) Instruction MOV B, M is an example of _____________ addressing mode.


a) direct
b) immediate
c) register
d) none of these
2) Mode 4 of 8254 is
a) square wave generator
c) software triggered strobe

b) rate generator
d) hardware triggered strobe

3) With reference to 8259, take odd man out INT,


a) INT
b) SP/EN
c) IR1
4) A0 and A1 lines of 8255 are used to
a) select 8255
b) select ports
5) ADC 0809 is _________ converter.
a) 8 channel 8 bit
c) 9 channel 8 bit
6) DMA is
a) Digital Memory Access
c) Direct Memory Access

c) reset 8255

, IR1, EOI
d) EOI
d) none of these

b) 8 channel 9 bit
d) none of these
b) Digital Minimum Access
d) None of these

7) Instruction PUSH B will require _________ T states.


a) 4
b) 6
c) 12

d) 14

P.T.O.

SLR-BB 392

*SLRBB392*

-2-

8) Which of below is not a flag of 8085 ?


a) overflow
b) sign
c) auxiliary carry d) carry
9) Take odd man out-EPROM, Static RAM, Content Accessible RAM, Hard Disk
a) EPROM
b) Static RAM
c) Content Accessible RAM
d) Hard Disk
10) Vector location for RST7 is
a) 003CH
b) 0038H
11) _________ ADC is the fastest.
a) Successive approximation
c) Single slope

c) 0034H

d) 002CH

b) Dual slope
d) Flash

12) Which of below is not a part of dynamic display system ?


a) IC 8253
b) transistor
c) IC 7447
d) IC 8255
13) With reference to 8255, take odd man out strobed I/O, BSR, status driven
I/O, storbed Bi directional I/O
a) strobed I/O
b) BSR
c) status driven I/O
d) strobed bi directional I/O
14) Starting and ending address of memory chip are 1000H and 1FFFH respectively.
Size of memory is
a) 512 bytes
b) 1 Kbytes
c) 2 Kbytes
d) 4 Kbytes
15) Which of below is not a RS232C signal ?
a) RTS
b) CTS
c) CS

d) DSR

16) In an instruction OUT BCH, what is BC


a) memory location address
b) port address
c) register pair
d) immediate data
17) Take odd man out : editor, assembler, emulator, loader
a) editor
b) assembler
c) emulator
d) loader
18) MC 1488 is
a) line driver/receiver
c) line receiver

b) line driver
d) voltage doubler

19) Status of S1 S0 line during interrupt acknowledge machine cycle is


a) 0 0
b) 0 1
c) 1 0
d) 1 1
20) When all address lines are used for selecting a device/memory it is called ?
a) absolute decoding
b) partial decoding
c) complete decoding
d) full addressing
______________

Set A

*SLRBB392*
S

-3-

SLR-BB 392

T.E. (Electrical and Electronics Engineering) (Part I) (New)


Examination, 2014
MICROPROCESSOR AND ITS APPLICATIONS
Day and Date : Monday, 8-12-2014

Marks : 80

Time : 3.00 p.m. to 6.00 p.m.


Instructions : 1) All questions are compulsory.
2) Figures to the right indicates full marks.
3) Assume suitable data if necessary.
SECTION I
2. Solve any three :

(38=24)

a) With suitable example explain instructions DAA, SHLD, CMP, XCHG.


b) Write a program to count the even and odd numbers from the given array
of 10 elements.
c) With suitable hardware show how a single stepping is supported in 8085.
d) With suitable timing diagram show how a slower memory can be interfaced
with 8085 using Ready signal.
3. Solve any four :

(44=16)

a) With suitable example explain use of a stack.


b) Write an assembly program to multiply three numbers.
c) What is Tri-state logic ? Why it is required ?
d) Explain software interrupts of 8085.
e) Explain flag register of 8085.
Set A

SLR-BB 392

-4-

*SLRBB392*

SECTION II
4. Solve any three :

(38=24)

a) With suitable hardware and program explain DC motor speed control


using 8085.
b) Show detailed interfacing of 8259 with 8085. Assume suitable addresses.
c) Show interfacing of DAC 0808 with 8085. Assume suitable addresses. Write
a program to generate triangular waveform using DAC 0808.
d) With suitable example explain how serial communication is supported using
SID and SOD pins.
5. Solve any four :

(44=16)

a) With suitable timing diagram explain Mode 0 of 8254.


b) With suitable example explain BSR mode of 8255.
c) Explain successive approximation type ADC.
d) Explain synchronous Vs asynchronous serial communication.
e) With suitable example explain R-2R DAC.
_____________________

Set A

SLR-BB-393

*SLRBB393*
S

T.E. (Electrical and Electronics Engineering) (Part I) Examination, 2014


ELEMENTS OF POWER SYSTEM (New)

Day and Date : Wednesday, 10-12-2014


Time : 3.00 p.m. to 6.00 p.m.

Max. Marks : 100

Instructions : 1) Q. No. 1 is compulsory. It should be solved in first 30 minutes in


Answer Book Page No. 3. Each question carries one mark.
2) Answer MCQ/Objective type questions on Page No. 3 only. Dont
forget to mention, Q.P. Set (A/B/C/D) on Top of Page.
Duration : 30 Minutes

MCQ/Objective Type Questions

Marks : 20

1. Choose the correct answer.

20

1) The diameter of each strand is d then the diameter of n-layer stranded conductor will
be _____________
a) (2n + 1) d
b) (2n 1) d
c) 3 (n + 1) d
d) 3 (n 1) d
2) With increase in temperature, the sag in overhead transmission line _________
a) Increases
b) Have no effect c) Decreases
d) None of above
3) The function of steel wire in an ACSR conductor is to _____________
a) Compensate for skin effect
b) Provide additional mechanical strength
c) Take care of surges
d) Reduce inductance
4) In a suspension type insulator, the potential drop is _____________
a) Maximum across the lowest disc
b) Uniformly distributed over the discs
c) Maximum across the top most disc
d) None of above
5) String efficiency can be improved by _____________
a) Using long cross arms
b) Using a guard ring
c) Grading of the insulators
d) Any of above
6) To reduce corona effect usually _____________
a) Distance between the conductors is reduced
b) Bundled conductors are used
c) Conductor diameter is reduced
d) Stranded conductors are used
7) The conductor carries more current on the surface in comparison to its core. This
phenomenon is called as
a) Skin effect
b) Ferranti effect
c) Corona
d) Lenzs law
8) The inductance of a transmission line is minimum when ___________
a) GMD is high
b) Both GMD and GMR are high
c) GMR is high
d) GMD is low and GMR is high
P.T.O.

SLR-BB-393

*SLRBB393*

-2-

9) The effective resistance of a conductor will be same as ohmic resistance when


a) Voltage is low
b) Current is pure sinusoidal
c) Current is uniformly distributed in the x-section of the conductor
d) None of these
10) Transposition of transmission line is done to
a) Reduce line losses
b) Reduce skin effect
c) Balance line voltage drop
d) Reduce corona
11) Which of the following is neglected while analyzing a short transmission line ?
a) Series impedance
b) Power losses
c) Shunt admittance
d) None of these
12) The values of A, B, C and D constants for a short transmission line are respectively
a) Z, 0, 1 and 1
b) 0, 1, 1 and Z
c) 1, Z, 0 and 1
d) 1, 1, Z and 0
13) In any transmission line AD BC = ___________
a) 1
b) Infinite
c) 0

d) None of above

14) % voltage regulation of a transmission line is given by the expression


a)
8

100

b)
8

100

c)
8

100

d)
8

100

15) Sheaths are used in cables to


a) Provide proper insulation
c) Provide mechanical strength

b) Prevent ingress of moisture


d) None of above

16) Paper as an insulating material has the main drawback that it


a) Is hygroscopic
b) Has low insulation resistivity
c) Has poor dielectric strength
d) Has high capacitance
17) Charging current in underground cable is ___________
a) less than
b) same as
c) more than

d) none of above

18) Theoretically, maximum value of power factor can be __________


a) 1
b) 0
c) 0.5
d) 0.9
19) An overexcited synchronous motor on no load is known as ___________
a) Induction motor
b) Reluctance motor
c) Induction generator
d) Synchronous condenser
20) Power factor can be improved by
a) Capacitor banks
b) Phase advancers
c) Synchronous condenser
d) All of above
______________

Set A

*SLRBB393*
S

SLR-BB-393

-3-

T.E. (Electrical and Electronics Engineering) (Part I) Examination, 2014


ELEMENTS OF POWER SYSTEM (New)

Day and Date : Wednesday, 10-12-2014


Time : 3.00 p.m. to 6.00 p.m.

Marks : 80

SECTION I
2. Solve any four questions :

(45=20)

1) What are the main components of transmission lines ? Explain criteria for choosing
conductor material.
2) A three phase line has conductors of radius 0.8 cm spaced equilaterally 2.5 m apart.
If the dielectric strength of air is 30 KV/cm, determine the disruptive critical voltage at
which corona will occur. Take relative air density factor


= 0.96 and irregularity factor

m0 = 0.94.
3) Find the inductance per km of a three phase transmission line using 1.24 cm diameter
conductor when these are placed at the corners of an equilateral triangle of each side 2 m.
4) What are different types of insulators that are used in overhead transmission line ? Write
advantages of pin type and suspension type insulator.
5) Define corona. Describe advantages and disadvantages of corona.
6) Derive an expression for inductance of 1 2 wire line.


3. Solve any two questions :

(210=20)

1) Derive an expression for sag of transmission line having equal and unequal level of
supports.
2) In a 33 KV overhead line, there are three units in the string of insulators. If the capacitance
between each pin and earth is 11% of self capacitance of each insulator, find :
a) Distribution of voltage over three insulator discs
b) String efficiency.
3) Write short note on skin effect and proximity effect.
Set A

SLR-BB-393

-4-

*SLRBB393*

SECTION II
4. Solve any four questions :

(45=20)

1) Describe properties of insulating material used in underground cable. List the different
insulating materials that are used in underground cables.
2) Derive an expression for capacitance of single core underground cable with neat diagram.
3) Derive an expression for voltage regulation, sending end power factor and efficiency of
short transmission line long with the equivalent circuit diagram and phasor diagram.
4) Describe the causes and disadvantages of low power factor.
5) A three phase, 5 KW induction motor has a power factor of 0.75 lagging. A bank of
capacitors is connected in delta across the supply mains and power factor is raised to
0.9 lagging. Determine the KVAR rating of the capacitors connected in each phase.
6) Derive an expression for insulation resistance of single core cable.
5. Solve any two questions :

(210=20)

1) Derive the expression for generalized circuit constants of long transmission line of following
methods :
a) Nominal T method
b) Nominal


method.

2) A three phase, 50 Hz overhead transmission line of 100 km length has following constants.
Resistance/km/phase = 0.1


Inductive reactance/km/phase = 0.2


Capacitance susceptance/km/phase = 0.04 104 S
Calculate :
a) Sending end voltage
b) Sending end current
c) Sending end power factor
d) Transmission efficiency
e) Voltage regulation.
when supplying a balanced load of 10000 KW at 66 KV, 0.8 P.F. lag. Use nominal T method.


3) Explain following methods of capacitance grading :


a) Capacitance grading
b) Intersheath grading.

_____________________

Set A

SLR-BB 394

*SLRBB394*
S

T.E. (Electrical and Electronics Engineering) (Part I) (New) Examination, 2014


CONTROL SYSTEM I
Day and Date : Friday, 12-12-2014
Time : 3.00 p.m. to 6.00 p.m.

Total Marks : 100

Instructions : 1) Assume suitable data wherever necessary.


2) Non-programmable calculators are permitted.
3) Q. No. 1 is compulsory. It should be solved in first 30 minutes in
Answer book Page No. 3. Each question carries one mark.
4) Answer MCQ/Objective type questions on Page No. 3 only. Dont
forget to mention, Q.P. Set (A/B/C/D) on Top of Page.
MCQ/Objective Type Questions

Duration : 30 Minutes

Marks : 20

1. Choose the correct answer :

(201=20)

1) Transfer function of the control system depends on


a) initial conditions of input and output
b) system parameters alone
c) nature of the input
d) nature of the output
2) Control systems are normally designed with damping factor
a) less than unity
b) more than unity
c) zero
d) unity
3) If the characteristic equation of a closed-loop system is s2 + 2s + 2 = 0, then the system is
a) over damped
b) critically damped
c) underdamped
d) undamped
4) A signal flow graph is a
a) special type of graph for analyzing the modern control system
b) pictorial representation of the simultaneous equations describing a system
c) polar graph
d) log graph
5) For the system in the given figure the characteristic equation is

a)

c) K(s + 1) (s + 3) = 0

b)

d) s(s + 2) = 0

P.T.O.

SLR-BB 394

*SLRBB394*

-2-

6) For the system of the given figure, the damping ratio of closed loop poles is

a) 1.5

b) 1

c) 0.5

d) 0.25

7) A system has its two poles on the negative real axis and one pair of poles lies on j axis.
The system is
a) stable
b) unstable
c) critically stable
d) either a) or c)
8) A lag compensator is essentially a
a) low pass filter
b) high pass filter c) band pass filter
d) none of these
9) The equation 2s4 + s3 + 3s2 + 5s + 10 = 0 has _____________ roots in the left half of
s-plane.
a) one
b) two
c) three
d) four
10) For a type one system, the steady-state error due to step input is equal to
a) infinite
b) zero
c) 0.25
d) 0.5
11) In root locus a centroid is a point where
a) there is a breakaway point
b) the root locus intersects the imaginary axis
c) all the asymptotes meet
d) none of the above
12) In force-voltage analogy, damper is analogous to
a) resistance
b) current
c) inductance
d) capacitance
13) The phase margin of a system specifies
a) relative stability
b) absolute stability
c) frequency response
d) none of these
14) In an integral controller
a) the output is proportional to input
b) the rate of change of output is proportional to input
c) the output is proportional to rate of change of input
d) none of the above
15) Bode magnitude plot is drawn between
b) dB magnitude and log
a) magnitude of network function and
c) dB magnitude and
d) loge (magnitude) and log


16) With feedback ________________ increases.


a) system stability
b) sensitivity
c) gain
d) effects of disturbing signals
17)


is a


a) type 0 system
b) type 1 system
c) type 2 system
d) type 3 systems
18) A system with gain margin close to unity or a phase margin close to zero is
a) highly stable
b) oscillatory
c) relatively stable d) unstable
19) A system with more than one input variable or more than one output variable is known by
what name ?
a) Robust control system
b) Closed-loop feedback system
c) Open-loop feedback system
d) Multivariable control system
20) The transient response of a system is mainly due to
a) inertia forces
b) internal forces
c) stored energy
d) friction
______________

Set A

*SLRBB394*
S

-3-

SLR-BB 394

T.E. (Electrical and Electronics Engineering) (Part I) (New) Examination, 2014


CONTROL SYSTEM I
Day and Date : Friday, 12-12-2014
Time : 3.00 p.m. to 6.00 p.m.

Marks : 80

Instructions : 1) Assume suitable data wherever necessary.


2) Non-programmable calculators are permitted.
SECTION I
2. Answer the following (any 4) :

20

a) Explain open loop and closed loop system with example.


b) Derive the transfer function of armature controlled servo motor.
c) Obtain transfer function by Masons Gain formula.

d) Explain the transient response specification with diagram.


e) Obtain C(s)/R(s) using block diagram reduction technique.

Set A

SLR-BB 394

-4-

*SLRBB394*

3. Answer the following (any 2) :

20

a) Sketch the complete root locus for a unity feedback system, whose transfer function is
given by

.
k

Show all the salient points on graph and comment on stability.


b) Obtain C(s)/R(s) using block diagram reduction technique.

c) Obtain differential equations describing the mechanical system shown in fig. Draw the
electric network using force-voltage analogy.

Set A

*SLRBB394*

-5-

SLR-BB 394

SECTION II
4. Answer following (any 4) :

20

1) Explain in detail lead compensator.


2) Explain PD controller in detail.
3) Write the correlation between time domain and frequency domain.
4) A given system oscillates with frequency 2rad/sec. Find the values of kmar and p. No
poles in R.H.S.

5) Explain the steps to design bode plot.


5. Answer the following (any 2) :

20

1) Explain in detail Lag-Lead Compensator. Write the advantages of Lag-Lead compensator.


2) Explain the following :
a) State
b) State variables
c) State vector
d) State space
e) State trajectory.
3) Sketch bode plot for the following the system. Obtain the value of gain margin and phase
margin.
1

.


_________________

Set A

Set A

SLR-BB 396

*SLRBB396*
Seat
No.

Set

T.E. (Part II) (Electrical and Electronics) Examination, 2014


POWER ELECTRONICS
Day and Date : Tuesday, 25-11-2014
Time : 10.00 a.m. to 1.00 p.m.

Max. Marks : 100

Instructions : 1) Q. No. 1 is compulsory. It should be solved in first 30 minutes


in Answer book Page No. 3. Each question carries one mark.
2) Answer MCQ/Objective type questions on Page No. 3 only.
Dont forget to mention, Q.P. Set (A/B/C/D) on Top of Page.
MCQ/Objective Type Questions
Duration : 30 Minutes
1. MCQs :
1) A DIAC is equivalent to
a) Pair of SCRs
c) Pair of four layers SCRs

Marks : 20
(120=20)
b) Triac with two gates
d) Diode with transistors

2) The maximum di/dt in SCR is


a) Directly proportional to Vm of supply voltage
b) Inversely proportional to Vm of supply voltage
c) Directly proportional to L in the circuit
d) None
3) MOSFETs are not suitable for low voltage switching.
a) True
b) False
4) Suppose the anode current of conducting SCR is 50A. If the gate current is
reduced to one-fifth, its anode current will become
a) 10A
b) 50A
c) 25A
d) Zero
5) A resistor connected across the gate and cathode of an SCR
a) Increases dv/dt rating of SCR
b) Increases holding current of SCR
c) Decreases noise immunity of SCR d) Increases turn-off time of SCR
6) SMPS are superior to linear power supplies in respect of
a) size and efficiency
b) efficiency and regulation
c) regulation and noise
d) noise and cost
7) A single phase half wave controlled rectifier has 400 sin 314t as the input
voltage and R as the load. For the firing angle of 60 for the SCR, the average
output voltage is
a) 400/
b) 300/
c) 240/
d) 200/
P.T.O.

SLR-BB 396

*SLRBB396*

-2-

8) When the firing angle of 1- fully controlled rectifier fording constant d.c.
current in to a load is 30, the displacement factor of the rectifier is
a) 1

b) 0.5

c)

d)

9) Which power semiconductor device out of these following is not a current


triggered device ?
a) Thyristor
b) GTO
c) TRIAC
d) MOSFET
10) For continuous conduction in a single phase full converter each pair of SCRs
conducts for
a)
b)
c)
d) +
11) A cyclo converter is
a) ac-dc converter
b) dc-ac converter
c) dc-dc converter
d) ac-ac converter
12) If tON and tOFF are the turn-on and turn-off times of the thyristor, then
b) tON > tOFF
a) tON and tOFF are almost equal
c) tOFF >> tON
d) tON is about twice tOFF
13) Induction heating systems requires low frequency investors.
a) True
b) False
14) The two modes of operation of 3-phase bridge investors known as
a) 60 mode and 120 mode
b) 90 mode and 120 mode
c) 120 mode and 180 mode
d) 120 mode and 240 mode
15) In a load commutated chopper, the commutating element is
a) Capacitor
b) Capacitor and inductor
c) Capacitor, inductor and thyristor d) Any of the above depending on load
16) In a half bridge investor each thyristor is gated at a frequency f. The output
frequency is
a) 2f
b) f
c) 0.5 f
d) 0.25 f
17) Multiphase choppers are used in
a) Low power circuits
b) High power circuits
c) Both low and high power circuits d) None of these
18) A bridge type 1- cycloconverter changes the frequency f to f/3. Then one
half wave of output contains
a) 3 full wave of inputs
b) 3 half waves of input
c) 6 full wave of inputs
d) 6 half waves of input
19) A 3-phase series inverter has
a) 12 thyristors
b) 6 thyristors
c) 3 thyristors
d) 3 or 6 thyristors
20) In a sinusoidal PWM for inverters the width of distant pulses are not the same.
a) True
b) False
______________
Set A

*SLRBB396*

-3-

SLR-BB 396

Seat
No.

T.E. (Part II) (Electrical and Electronics) Examination, 2014


POWER ELECTRONICS
Day and Date : Tuesday, 25-11-2014
Time : 10.00 a.m. to 1.00 p.m.

Marks : 80

Instructions : 1) All questions are compulsory.


2) Figures to the right indicate full marks.
SECTION I
2. Solve any four :
(44=16)
a) Sketch the output characteristics of enhancement type MOSFET. What are
the basic differences in control of BJT and MOSFET ?
b) Distinguish between natural commutation and forced commutation for SCR
with illustrative examples.
c) Using two transistor model, explain how a small gate current can turn-on a
SCR, when blocking forward voltage.
d) Obtain the expression for input power factor for a full wave controlled
rectifier feeding a purely resistive load.
e) What is SMPS ? List the various types of SMPS. Describe SMPS with push
pull configuration.
f) What is IGBT ? Draw its circuit diagram and discuss switching characteristics.
3. Solve any three :

(38=24)

a) Mention the applications of convertors. Explain the principle of operation of


single phase half wave controlled rectifier with R-L load and fly wheel diode.
b) Describe briefly the following thyristor applications :
i) UPS
ii) Arc welding.
c) A single phase bridge rectifier is operated with extinction angle control with
P = 45. The ac supply voltage is 160V (rms) and a load resistance of 23 is
connected at the output of the rectifier. Determine the
i) distortion factor
ii) displacement factor
iii) input power factor
iv) average load
v) average current.
d) Draw circuit and V-I characteristics of the following devices :
i) GTO
ii) TRIAC.
Set A

SLR-BB 396

-4-

*SLRBB396*

SECTION II
4. Solve any four :

(44=16)

a) With necessary waveforms explain the operation of single phase bridge inverter.
b) Discuss single phase current source inverter. Compare VSI with CSI.
c) A cycloconverter converts high frequency in to low frequency. Justify through
waveforms. Draw a neat circuit diagram and waveforms.
d) A single phase half bridge inverter feeds a resistive load R = 7.5 . The d.c.
voltage of the inverter is given as 220 V. Determine :
i) Average and peak currents of the thyristors
ii) PIV.
e) Describe the operation of four quadrant chopper with neat diagrams.
f) A step-up chopper is used to deliver load voltage of 400 V from a 220 V D.C.
source. If the blocking period of thyristor is 60 sec. Compute the required
pulse width.
5. Solve any three :

(38=24)

a) Describe the principle of operation of a single phase bridge type step down
cycloconverter feeding a R-L load.
b) For the basic chopper circuit Edc = 50 volts, R = 80 , duty cycle =30%.
Find out :
i) The average output voltage and current
ii) Output current at the instant of commutation
iii) Rms values of output voltage and currents.
c) Explain working of three VSI with 120 degree conduction with resistive
load. Draw wave forms.
d) What is a chopper ? Give classification and explain principle of operation of
Jhones chopper.
_____________________

Set A

SLR-BB 397

*SLRBB397*
Seat
No.

Set

T.E. (Electrical and Electronics Engineering) (Part II)


Examination, 2014
ELECTRONICS COMMUNICATION ENGINEERING
Day and Date : Wednesday, 26-11-2014
Time : 10.00 a.m. to 1.00 p.m.

Max. Marks : 100

Instructions : 1) Assume suitable data if required.


2) Figures to the right indicate full marks.
3) Q. No. 1 is compulsory. It should be solved in first 30 minutes
in Answer Book Page No. 3. Each question carries one mark.
4) Answer MCQ/Objective type questions on Page No. 3
only. Dont forget to mention, Q.P. Set (A/B/C/D) on Top
of Page.
MCQ/Objective Type Paper
Duration : 30 Minutes

Marks : 20

1. Choose the correct answer :


1) Discrete signal is represented by
a) Coding
c) Demodulation

(120=20)
b) Modulation
d) Sequence of samples

2) Frequency modulation has


a) One carrier
b) One carrier with two side band frequencies
c) One carrier with infinite frequencies
d) None of these
3) In amplitude modulation, frequency is
a) Constant
b) Zero
c) Variable

d) One

4) In order to reduce interference in transmission, the signal should be


a) Amplified
b) Multiplied
c) Demodulated d) Modulated
5) Which of the following is not an analog-to-analog conversion ?
a) AM
b) PM
c) FM
d) QAM
6) The Hamming distance between 100 and 001 is ____________
a) 0
b) 1
c) 2
d) None of the above
P.T.O.

SLR-BB 397

-2-

*SLRBB397*

7) Which multiplexing technique involves signals composed of light beams ?


a) FDM
b) TDM
c) WDM
d) None of the above
8) A _________ error means that two or more bits in the data unit have changed.
a) Burst
b) Double-bit
c) Single-bit
d) None of the above
9) A technique used is speech coding to achieve a smaller dynamic range in
quantization levels sends only the difference between consecutive samples.
This technique is known as _____________
a) PCM modulation
b) Delta modulation
c) Random delta modulation
d) On-off keying
10) Video signals in TV are
a) Amplitude modulated
b) De modulated
c) Frequency modulated
d) None of these
11) In amplitude modulation, amplitude is
a) Constant
b) Zero
c) Variable
d) None of these
12) The maximum power in AM, when modulated index is
a) 0
b) 0.5
c) 0.7
d) 1
13) Communication is a process of
a) Transfer of information
b) Transfer of energy
c) Transfer of channel
d) None of these
14) Slope overload distortion occur in
a) PCM
b) ADM
c) DM
d) None of these
15) Messages travel from transmitter to receiver with help of
a) Transmitter
b) Receiver
c) Channel
d) Antennas
16) Which of the following is a valid probability value for a discrete random
variable ?
a) 0.2
b) 1.5
c) 0.7
d) All of the above
17) Which of the following is not a major communication medium ?
a) Free space
b) Water
c) Wires
d) Fiber optic cable
18) The word ________ refers to the portion of a ________ that carries a
transmission.
a) Channel; link
b) Link; channel c) Line; channel d) Line; link
19) The step size is constant in ___________
a) PCM
b) DM
c) ADM
d) All of the above
20) fc + fm is _____________ of A.M.
a) Upper band frequency
b) Lower band frequency
c) Mid band frequency
d) None of these
Set A
______________

*SLRBB397*

-3-

SLR-BB 397

Seat
No.

T.E. (Electrical and Electronics Engineering) (Part II)


Examination, 2014
ELECTRONICS COMMUNICATION ENGINEERING
Day and Date : Wednesday, 26-11-2014
Time : 10.00 a.m. to 1.00 p.m.

Marks : 80

SECTION I
2. Attempt any four :

(45=20)

a) Define amplitude modulation and frequency modulation with modulated


waveform. Also explain the limitation in AM.
b) Explain electronic communication system. Also state the applications of
communication system.
c) Define modulation index for AM. Draw the amplitude modulated waveform for
1) M = 1
2) M > 1
3) M < 1
d) Explain super heterodyne receiver in detail.
e) Define signal. Explain the different operations done on the signals.
3. Attempt any two :

(210=20)

a) Define modulation. What is the necessity of modulation. Give classification


of modulation system.
b) Explain SSB and DSB generation with its types in detail.
c) With noise triangle show that frequency modulation is more immune to noise
than AM and PM.
Set A

SLR-BB 397

-4-

*SLRBB397*

SECTION II
4. Attempt any four :

(45=20)

a) Explain the PCM transmitter in detail.


b) Define multiplexing. Explain FDM in detail.
c) Generate the hamming code for data 111011001 with even parity.
d) Define following :
1) Codeword
2) Hamming weight
3) Quantization
4) Line coding.
e) Describe basic digital communication system.
5. Attempt any two :

(210=20)

a) Explain delta modulation in detail with suitable diagram. Also explain


Adaptive delta modulation and compare its performance with delta
modulation.
b) Explain following :
1) Central limit theorem
2) Autocorrelation and co-variance function.
c) Define sampling theorem with its waveform. When aliasing effect occurs on
signal and how to minimize it ?
_____________________

Set A

SLR-BB 398

*SLRBB398*
Seat
No.

Set

T.E. Electrical and Electronics Engineering (Part II) Examination, 2014


POWER SYSTEM I
Day and Date : Thursday, 27-11-2014
Time : 10.00 a.m. to 1.00 p.m.

Max. Marks : 100

Instructions : 1) Assume suitable data wherever necessary.


2) Non-programmable calculators are permitted.
3) Q. No. 1 is compulsory. It should be solved in first 30 minutes in
Answer Book Page No. 3. Each question carries one mark.
4) Answer MCQ/Objective type questions on Page No. 3 only.
Dont forget to mention, Q.P. Set (A/B/C/D) on Top of Page.
MCQ/Objective Type Questions
Duration : 30 Minutes

Marks : 20

1. Objective questions :

(201=20)

1) Skin effect depends on


a) size of the conductor
b) Frequency of the current
c) Resistivity of the conductor material
d) All of the above
2) Ten discs usually suggest that the transmission line voltage is
a) 11 kV
b) 33 kV
c) 66 kV
d) 132 kV
3) Corona usually occurs when the electrostatic stress in the air around the
conductor succeeds
a) 30 kV (maximum value)/cm
b) 22 kV (maximum value)/cm
c) 11 kV (rms value)/cm
d) 6.6 kV (rms value)/cm
4) Between two supports, due to sag the conductor takes the form of
a) Catenary
b) Triangle
c) Ellipse
d) Semi-circle
5) Presence of ozone as a result of corons is harmful because
a) it gives bad odor
b) it corrodes the material
c) it transfers energy to the ground
d) reduces power factor
6) Pin insulators are normally used up to voltage of about
a) 100 kV
b) 66 kV
c) 33 kV

d) 25 kV

7) Corona effect can be detected by


a) Hissing sound
b) Faint luminous flow of bluish color
c) Presence of ozone detected by odor
d) All of the above
P.T.O.

SLR-BB 398

*SLRBB398*

-2-

8) ACSR conductor implies


a) All conductors surface treated and realigned
b) Aluminum conductor steel reinforced
c) Anode current sinusoidal run
d) Anodized core smooth run
9) The effect of wind pressure is more predominant on
a) Transmission lines
b) Neutral wires
c) Insulator
d) Supporting towers
10) In transmission system a feeder feeds power to
a) Service mains
b) Generating stations
c) Distributors
d) All of the above
11) The bundling of conductors is done primarily to
a) Reduce reactance
b) Increase reactance
c) Increase ratio interference
d) Reduce radio interference
12) Which of the following regulation is considered to be the best ?
a) 2%
b) 30%
c) 70%
d) 98%
13) Which of the following is not an insulator material ?
a) Rubber
b) Varnished rubber
c) Varnished cambric
d) Galvanised steel
14) The gas used in pressure cable is
a) Nitrogen
b) Hydrogen

c) Sulphur

d) All are used

15) Varnished cambric is also know as


a) Empire tape
b) Rubber tape

c) Paper tape

d) PVC tape

16) The process of achieving uniformity in the dielectric stress by using layers of different
Dielectrics is known as
a) Stranding process
b) Grading of cables
c) Stress distribution
d) Capacitance grading
17) Belted Cables are used upto _____________ kV.
a) 100 kV
b) 11 kV
c) 33 kV

d) 25 kV

18) In any transmission line, the constants A and D are


a) Dimensionless
b) Ohms and siemen
c) Mega ohm and volts
d) Volt and ampere
19) In short transmission line the effect of _____________ is neglected.
a) Resistance
b) Capacitance
c) Inductance
d) Conductance
20) End Condenser method is used for which transmission line
a) Very short
b) Medium
c) Short
d) Long
______________
Set A

*SLRBB398*

-3-

SLR-BB 398

Seat
No.

T.E. Electrical and Electronics Engineering (Part II) Examination, 2014


POWER SYSTEM I
Day and Date : Thursday, 27-11-2014
Time : 10.00 a.m. to 1.00 p.m.

Marks : 80

Instructions : 1) Assume suitable data wherever necessary.


2) Non-programmable calculators are permitted.
SECTION I
2. Solve any four :
(45=20)
a) Explain comparisons between DC and AC system.
b) What is a spacing, clearance and span length in transmission line. Explain in
details.
c) What are the desirable properties of line conductors ?
d) Define Kelvins law and explain the limitations of Kelvins law with points.
e) Explain Factors effecting Corona.
3. Attempt any two :

(210=20)

a) Explain the concept of self GMD and mutual GMD ?


b) What is SAG ? Explain the calculation of SAG at equal levels and unequal levels.
c) What are descriptive critical voltage, visual critical voltage and power loss due to
corona ? Explain the methods of Reducing Corona Effect.
SECTION II
4. Solve any four :
(45=20)
a) Explain the synchronous condenser method for improving factor.
b) Derive an expression for insulation resistance of single core cable.
c) Explain Oil-filled cables with neat diagram.
d) Derive an expression for dielectric stress in a single core cable.
e) Determine the generalized Constants for Medium line using Nominal T metod.
5. Attempt any two :
(210=20)
a) Draw and explain the general construction of underground cable. What are the
requirements of the Underground cables ?
b) Derive an expression for sending end and receiving end voltage and current for
long transmission line.
c) Draw circuit diagram and vector diagram of a medium transmission line and
derive an expression for voltage regulation and transmission efficiency using
nominal II-method.
Set A
_____________________

Set A

SLR-BB 399

*SLRBB399*
Seat
No.

Set

T.E. (Electrical and Electronics) (Part II) Examination, 2014


NON-LINEAR AND DIGITAL CONTROL SYSTEM
Day and Date : Friday, 28-11-2014
Time : 10.00 a.m. to 1.00 p.m.

Max. Marks : 100

Instructions : i) Assume suitable data, if necessary.


ii) Use of non-programmable calculator is allowed.
iii) Q. No. 1 is compulsory. It should be solved in first 30 minutes in Answer
Book Page No. 3. Each question carries one mark.
iv) Answer MCQ/Objective type questions on Page No. 3 only. Dont
forget to mention, Q.P. Set (A/B/C/D) on Top of Page.
MCQ/Objective Type Questions
Duration : 30 Minutes

Marks : 20

1. Choose the correct answer :

20

1) Differentiators are avoided in the system design as they


a) Are costlier
b) Reduces the response of the system
c) Develop noise and saturate the amplifier
d) Cannot be formed with capacitors and inductors
2) Phase lag compensation will
a) improve relative stability
c) increase bandwidth

b) increase the speed of response


d) increase overshoot

3) In a feedback system, lead compensator


a) increase the margin of stability
b) increase the system error constant to some extent
c) speeds up the transient response
d) all of these
4) In order to increase the damping of a badly underdamped system which of following
compensator may be used
a) Phase-Lead
b) Phase-Lag
c) Both a) and b)
d) None of the above
5) The phase lag produced by the transportation relays
a) is independent of frequency
b) is inversely proportional to frequency
c) increases linearly with frequency
d) decreases linearly with frequency
6) Addition of zero
a) has no effect on speed of response
c) slows down the response

b) speeds up the response


d) none of the above

7) Usually a lag network for compensation consist of


a) R element
b) R and L element c) R and C elements d) R, 1 and C elements
8) Indicate which one of the following TF represent phase lead compensator ?
s+1

a) s + 2

6s + 3

b) 6s + 2

s+5

c) 3s + 2

s+8

d) s2 + 5s + 6
P.T.O.

SLR-BB 399

*SLRBB399*

-2-

9) The eigen values of a linear system are the locations of


a) finite poles
b) poles of the system
c) zeros of the system
d) none of the above
1
0
10) The system matrix of a discrete system is given by A =
. The characteristic
3 5
equation is given by
a) z2 + 5z + 3 = 0
b) z2 3z 5 = 0
c) z2 + 3z + 5 = 0
d) z2 + z + 2 = 0

 = AX, where X is the


11) The state variable description of a linear autonomous system is, X
0 2
two dimensional state vector and A is the system matrix given by A =
. The poles of
2 0
the system are located at
a) j2 and + j2
b) + 2 and + 2
c) 2 and + 2
d) 2 and 2
12) Which of the following is an example of intentional nonlinear system ?
a) Contractor serves
b) Nonlinear pitch or dampers in aircraft control
c) On-off transducers
d) All of the above
13) In nonlinear control system limit cycle is self-sustained oscillations of
a) fixed frequency
b) variable frequency
c) variable amplitude
d) fixed frequency and amplitude
14) For which of the following reasons nonlinearities are introduced ?
a) To simplify the system
b) To achieve special characteristics
c) Both a) and b)
d) None of the above
15) The term backlash is associated with
a) servomotors
b) induction relays

c) gear trains

d) none of these

16) The transfer function of ZOH (Zero Order Hold) is


Ts
a) 1 e

Ts
b) 1 e

c)

1 e Ts
s

d)

1 e Ts
s

17) In order to recover the original signal from the sampled one, what is the condition to be
satisfied for sampling frequency s and highest frequency component m ?
a) m < m 2m

b) s 2m

c) s m

d) s = m

18) Which one of the following rules determines the mapping of s-plane to z-plane ?
a) Right half of the s-plane maps into outside of the unit circle in z-plane
b) Left half of s-plane maps into inside of the unit circle
c) Imaginary axis in s-plane maps into the circumference of the unit circle
d) All of the above
19) The TF of a phase-lead compensator is given by Gc (s) =
phase shift provided by such a compensator is

1 + 3 Ts
where T> 0. The maximum
1+ Ts

b) 3
c) 4
d) 6
a) 2
20) For a unity feedback system, the origin of the s-plane is mapped in the z-plane by
transformation z = esT to which one of the following ?
a) origin
b) 1 + j0
c) 1 + j0
d) 0 + j1
______________

Set A

*SLRBB399*

SLR-BB 399

-3-

Seat
No.

T.E. (Electrical and Electronics) (Part II) Examination, 2014


NON-LINEAR AND DIGITAL CONTROL SYSTEM
Day and Date : Friday, 28-11-2014
Time : 10.00 a.m. to 1.00 p.m.

Marks : 80

Instructions : i) All questions are compulsory.


ii) Assume suitable data, if necessary.
iii) Use of non-programmable calculator is allowed.
SECTION I
2. Solve any four :
a) Explain the procedure to design lead compensator using frequency response.
b) Write the steps to design lag compensator using root locus technique.
c) Derive the realization of lag-lead compensator.
d) The transfer function of the system is given as :
T(s) =

(54=20)

s+3
s + 5s 2 + 8s + 4
3

Obtain the state variable model of the system.


1 1
1
e) Find out the eigen and eigen vectors A =
B= .
0 1
1
0 3
3. a) Obtain state transition matrix for the following system matrix A =
.
1 4
b) A unity feedback system has OLTF G(s) =

10

K
. Design a lead compensator to
s (1 + 0.2s)
2

meet the following specifications. Ka = 10 Phase Margin = 35.

10

OR
b) The forward path TF of a unity feedback control system is given by G(s) =

K
.
s(s + 8s + 17)
2

The system has to have percentage overshoot for unit step input 16% and steady state

error for unit input 17

design a lag compensator using root locus.


70 rad / sec .

10

Set A

SLR-BB 399

*SLRBB399*

-4-

SECTION II
4. Solve any four :

(54=20)

a) Explain in short backlash and dead-zone nonlinearity.


b) Explain the limit cycle phenomenon in nonlinear control system.
c) Derive the pulse transfer function of cascaded elements.
d) Find the inverse Z-transform of
E(z)

z(z 0.7)
.
(z 1) (z 0.6)

e) Determine the inverse z-transform of z-domain function F(z) =

3z 2 + 2z + 1
.
z 2 + 3z + 2

5. Solve any two :

(102=20)

a) Derive the describing function of dead-zone nonlinearity.


b) Explain the steady state error analysis for discrete time control systems.
c) Check the stability of the sampled data control system represented by the equation :
F(z) = z 3 0.2z 2 0.25z + 0.05 = 0 .
_____________________

Set A

SLR-BB 4

*SLRBB4*
Seat
No.

Set

F.E. (Part I) (New) Examination, 2014


APPLIED MECHANICS
Day and Date : Monday, 15-12-2014
Time : 10.00 a.m. to 1.00 p.m.

Total Marks : 70

Instructions : 1) Q. No. 1 is compulsory. It should be solved in first 30 minutes


in Answer Book Page No. 3. Each question carries one mark.
2) Answer MCQ/Objective type questions on Page No. 3 only.
Dont forget to mention, Q.P. Set (A/B/C/D) on Top of Page.
MCQ/Objective Type Questions
Duration : 30 Minutes
1. Choose the correct answer :
1) The process of finding out the resultant force is called
a) Composition
b) Resolution
c) Idealization

Marks : 14
(114=14)
d) Analytical

2) When a force is resolved in two mutually perpendicular components, they


are
a) Orthogonal components
b) Rectangular components
c) Reciprocal components
d) None of these
3) The angle of inclination of the plane at which the body begins to move down
the plane is called
a) Angle of friction
b) Angle of projection
c) Angle of repose
d) None of these
4) A beam, 10 m long, carries uniformly distributed load of 8 kN/m and supported
at its two ends. What is the reaction at each support.
a) 8 KN
b) 80 KN
c) 40 KN
d) 4 KN
5) The M.I. of a triangular section of base (b) and height (h) about an axis
through its base is given by
b) bh3/18
c) bh3/36
d) bh3/64
a) bh3/12
6) To solve a truss by the method of joints, the number of unknown forces at a
joint should not be
a) Less than two
b) More than two
c) More than three
d) Cannot said
P.T.O.

SLR-BB 4

*SLRBB4*

-2-

7) Co-efficient of friction depends upon


a) Area of contacting only
b) Nature of surface only
c) Both a) and b)
d) None of these
8) Time required to stop a car moving with a velocity 20 m/s within a distance of
40 m, is
a) 2 sec
b) 3 sec
c) 4 sec
d) 5 sec
9) Two trains A and B moving on a parallel track in the same direction having
velocity 100 kmph and 65 kmph respectively. The relative velocity of train A
with respect to B is
a) 35 kmph
b) 35 kmph
c) 100 kmph
d) 165 kmph
10) If and are linear and angular velocities, the centripetal acceleration of a
moving body along a circular path of radius r, will be
r
a) 2

2
b)
r

r
c) 2

2
d)
r

11) The time of flight of a projectile on a horizontal plane is


a)

2u sin
g

b)

2u cos
g

c)

2u sin
2g

d)

2u cos
2g

12) The area under t curve represents


a) Average velocity of particles
b) Instantaneous velocity of the particle
c) Distance travelled by particle
d) Acceleration of the particle
13) When the speed of particle is doubled, its kinetic energy
a) Remains same
b) Increase two fold
c) Increase three fold
d) Increase four fold
14) Coefficient of restitution for elastic impact is
a) 0.25
b) 0.5
c) 0.75
______________

d) 1.0

Set A

*SLRBB4*

-3-

SLR-BB 4

Seat
No.

F.E. (Part I) (New) Examination, 2014


APPLIED MECHANICS
Day and Date : Monday, 15-12-2014
Time : 10.00 a.m. to 1.00 p.m.

Marks : 56

Instructions : 1) Solve any three questions from each Section.


2) Make suitable assumptions if necessary state them clearly.
SECTION I
2. a) State and explain Law of Transmissibility of forces.
b) Two smooth cylinders with radius and weights as shown in Figure are kept in
a groove with slanting surfaces. Determine the reactions at contact points.

3
7

Set A

SLR-BB 4

-4-

*SLRBB4*

3. a) State the laws of friction.


b) The beams AB is loaded and supported as shown in figure. A is a hinge
support and B is a roller support. Determine the support reactions at A and B
due to forces acting on the beam.

4. a) Explain the terms :


1) Perfect frame
2) Deficient frame
3) Redundant frame.
b) Figure shows a truss with hinged supports at E and roller support at A.
Find the forces in members AB, AC, BD and CD of the truss.

Set A

*SLRBB4*

-5-

SLR-BB 4

5. a) State and prove Parallel Axis Theorem.

b) Locate centroid of shaded area shown in figure. Determine the M.I. of the
shaded area about the centroidal X-X axis. Also find M.I. of the shaded area
about the base A-B. All dimensions are in mm.

SECTION II
6. a) Derive the following equation of motion of the body moving in a straight line
with uniform acceleration : s = ut +

1 at 2 .
2

b) On a straight line road, a smugglers car passes a police station with uniform
velocity 10 m/s. After 10 seconds, a police party starts chasing the car in a
jeep, with a uniform acceleration 1 m/s2. Find the time necessary for the jeep
to catch up with the smugglers car.

7. a) Show that the time of flight (T) on horizontal plane, for the projectile projected
with velocity u at an angle of projection with the horizontal is given by :
T=

2u sin
.
g

b) A bombing aircraft moving horizontally at a speed 108 km/hr, at an altitude of


1000 m, towards the target on the ground, releases a bomb which hits the
target. Estimate the horizontal distance of the aircraft from the target, when it
releases the bomb. Calculate also the direction and velocity, with which the
bomb hits the target.

Set A

SLR-BB 4

-6-

*SLRBB4*

8. a) State DAlemberts principle, giving equations. Explain it for a rigid body in


plane motion.

b) A cage of weight 6000 N, carrying additional weight of 10 men each weighing


500 N, starts moving downwards from rest in a vertical mine shaft. The cage
attains a speed of 12 m/s in 20 m distance. Find the pressure exerted by each
man on the floor of the cage.
Also find tension in the cable during this travel.

9. a) What do you understand by the term Energy ? Explain the various forms of
Mechanical Energy.

b) An engine of mass 50 tonnes pulls a train of mass 250 tonnes up a gradient 1


in 125, with a uniform speed of 36 km per hour. Find the power transmitted by
the engine, if the tractive resistance in 60 N per tonne mass. Also find the
power transmitted by the engine, if the engine and train is accelerated with
acceleration 0.2 m/s2 along the same portion of the track. Take g = 9.8 m/s2. 8
_____________________

Set A

SLR-BB 40

*SLRBB40*
Seat
No.

Set

T.E. (Civil) (Part I) (Old) Examination, 2014


ENVIRONMENTAL ENGINEERING I
Day and Date : Wednesday, 10-12-2014
Time : 3.00 p.m. to 6.00 p.m.

Total Marks : 100

Instructions : 1) Q. No. 1 is compulsory. It should be solved in first 30 minutes


in Answer Book Page No. 3. Each question carries one mark.
2) Answer MCQ/Objective type questions on Page No. 3 only.
Dont forget to mention, Q.P. Set (A/B/C/D) on Top of Page.
3) Figures to the right indicate full marks.
4) Assume suitable data wherever necessary and mention it
clearly.
5) Use of non-programmable calculator is allowed.
Duration : 30 Minutes

MCQ/Objective Type Questions

Marks : 20

1. Choose the correct answer :


1) The most widely used coagulant in water treatment is __________
a) Lime soda
b) Alum
c) Ferrous sulphate
d) Chlorinated copperas
2) Cleaning of slow sand filter is done by ____________
a) Scraping and removal of sand
b) Back washing
c) Any of above
d) None of above
3) Which of the following compound can be used for chlorination of water ?
a) Carbon dioxide
b) Bleaching powder
c) Alum
d) All of above
4) Wash water troughs are provided in ______________
a) Rapid sand filter
b) Flocculator
c) Aerator
d) Sedimentation tank
5) The purpose of aeration used in water treatment is ___________
a) to reduce corrosion to water pipes b) to remove iron and manganese
c) remove hydrogen sulphide
d) all the above
6) Strokes law is applied in design of _________ treatment unit.
a) Coagulation
b) Filtration
c) Sedimentation d) Aeration
7) Standard EDTA is used to determine ________ of water.
a) Hardness
b) Alkalinity
c) Turbidity
d) Residual chlorine

20

P.T.O.

SLR-BB 40

-2-

*SLRBB40*

8) The water is said to be safe for drinking when it satisfies ________ standards.
a) Physical
b) Chemical
c) Biological
d) All of above
9) Alkalinity in water is expressed as milli-grams per litre in terms of equivalent
_______________
a) Calcium carbonate
b) Magnesium carbonate
c) Sodium carbonate
d) Calcium hydroxide
10) Which of the following is not a water borne disease ?
a) dysentery
b) cholera
c) typhoid
d) malaria
11) Distribution mains of any water supply, is normally designed for its average
daily requirement _________
a) 100%
b) 150%
c) 200%
d) 225%
12) For determining the velocity of flow of underground water, the most commonly
used non-empirical formula is _________
a) Darcys formula
b) Slichters formula
c) Hazens formula
d) Lacys formula
13) The maximum pressure to which a pipe is subjected ___________ to during
its operation.
a) design pressure
b) pipe pressure
c) working pressure
d) test pressure
14) _________ reduces the conveyance losses such as evaporation and seepage.
a) Open channel b) Tunnels
c) Pipelines
d) Manholes
15) _______ effect is checked for velocity of water in pipes by closing the valves.
a) Temperature stress
b) Water hammer
c) Longitudinal stress
d) Internal pressure
16) In ____________ layout distribution system main pipe runs through centre
and sub mains takeoff from both sides.
a) Dead end system
b) Radial system
c) Circular system
d) Grid iron system
17) __________ joints are used for pipes to considerable difference in temperature.
a) Tyton
b) Coupled
c) Expansion
d) Flanged
18) A vertical pipe that carries water directly from mains to ESR __________
a) Siphon
b) Riser
c) Pressure
d) Centrifuge
19) Pipes are laid parallel to increase the capacity of the water supply and
_____________
a) to provide a means of repairing without closing water supply
b) to discharge at lower portion
c) to lead wastage at supply period
d) to reduce in the size of pumps
20) Storage capacity of distribution reservoir is based on ___________
a) Balancing reserve
b) Breakdown reserve
c) All of above
d) Fire reserve
______________
Set A

*SLRBB40*

-3-

SLR-BB 40

Seat
No.

T.E. (Civil) (Part I) (Old) Examination, 2014


ENVIRONMENTAL ENGINEERING I
Day and Date : Wednesday, 10-12-2014
Time : 3.00 p.m. to 6.00 p.m.

Marks : 80

Instructions : 1) Solve any three questions from Section I, i.e. question


no. 2 to question no. 5. Question no. 6 is compulsory in
Section II and solve any two questions from the remaining.
2) Figures to the right indicate full marks.
3) Assume suitable data wherever necessary and mention it
clearly.
4) Use of non-programmable calculator is allowed.
SECTION I
2. a) Explain working of slow sand filter with neat sketch.
b) Explain types of settling. Also discuss theory of settling for rectangular settling
tank.

7
6

3. a) Enlist different types of aerators and explain slat tray aerator with neat sketch. 5
b) Find the dimensions of a rectangular sedimentation tank for the following
data :
8
i) Volume of water is to be treated = 3 MLd
ii) Detention period = 4 hrs
iii) Velocity of flow = 10 cm/min.
4. a) Explain significance of following tests :
i) Alkalinity
ii) Chlorides
b) Derive equation for settling velocity by using Newtons law.

3
2
8

5. Write short notes on following :


1) Backwashing of filter.

2) Breakpoint chlorination.

3) Geometric increase method of population forecasting.

5
Set A

SLR-BB 40

-4-

*SLRBB40*

SECTION II
6. a) Explain Grid iron system of water distribution with neat sketch.

b) Explain the various pressures required to be considered for design of pressure


pipeline.
7. a) Explain factors contributing pipe corrosion.

6
8

b) Give the advantages of using pressure conduits.

8. a) Explain the equivalent pipe method of analysis of pipe network of a distribution


system.

b) Determine the distribution of flow in the pipe network shown in fig. the head
loss may be assumed as KQ2, flow is turbulent and pipes are rough. Use
Hardy cross method. (Value for K = AB = DA = 2, BC = 4, CD =3 and DB =1),
use two trials.

9. Write short note (any three) :

12

i) Maintenance of distribution system


ii) Newton Raphson method
iii) Intermittent water supply
iv) Corrosion and its control.
_____________________

Set A

SLR-BB 400

*SLRBB400*
Seat
No.

Set

T.E. (E and E) (Part II) Examination, 2014


MICROCONTROLLER AND ITS APPLICATIONS
Day and Date : Saturday, 29-11-2014
Time : 10.00 a.m. to 1.00 p.m.

Max. Marks : 100

N.B. : 1) Figures to right indicate full marks.


2) Assume relevant data whenever necessary.
3) Q. No. 1 is compulsory. It should be solved in first 30
minutes in Answer book Page No. 3. Each question carries
one mark.
4) Answer MCQ/Objective type questions on Page No. 3
only. Dont forget to mention, Q.P. Set (A/B/C/D) on Top
of Page.
Duration : 30 Minutes

MCQ/Objective Type Questions

Marks : 20

1. Objective questions :
(120)
1) In mode 2, the counter rolls over when counter goes upto __________ 4.
a) FFFF
b) FF
c) OO
d) 7 F
2) 8051 has __________ interrupts.
a) 05
b) 06
c) 07
d) 08
3) Which bit from the IP register is to be made high to assign highest priority to
TF1 ?
a) D5
b) D6
c) D3
d) D2
4) RST is active __________ pin.
a) High
b) Low
c) Both
d) None
5) The address of DPH and DPL are __________ 4 and __________ 4.
a) 83, 82
b) 16, 17
c) 84, 85
d) 64, 65
6) __________ bit of DSW register is user definable.
a) 5th
b) 6th
c) 1st
d) 8th
7) After reset, SP register is initialised to address __________
a) 8 H
b) 9 H
c) 7 H
d) 6 H
8) 8051 has __________ bidirectional input/output lines.
a) 8
b) 16
c) 32
d) 64
9) TCON register is __________ bit wide.
a) 4
b) 6
c) 8
d) 16
P.T.O.

SLR-BB 400

*SLRBB400*

-2-

10) The SFR space of 8051 is __________


a) 80 FF

b) 00 IF

c) 20 2 F

d) 30 7 F

11) Control word register address is 33 H. What will be the add. of port C and
port A respectively (8255 IC) ?
a) 31 H, 32 H
b) 30 H, 32 H
c) 32 H, 30 H
d) None
12) The shift register mode and multiprocessor variable mode of 8051
respectively are
a) mode 0, mode 3
c) mode 2, mode 3

b) mode 1, mode 2
d) none

13) In mode 2 and mode 3 if __________ bit of SCON is set will cause enable
multiprocessor communication and is of __________ bit address respectively.
a) SM1, 9 EH
b) TB 8, 9 CH
c) SM 2, 9 DH
d) SM 0, 9 FH
14) In c and LCD interface which line will instruct the LCD that c is
sending data ?
a) DBO

b) RW

c) RS

15) Fastest ADC is


a) Counter type

d) EN

b) Flash

c) Successive approximation

d) Double slope

16) The control word format to use 8253 in mode 0, 8 bit count for counter
0 and binary count is
a) 10 H
b) 11 H
c) 22 H
d) 33 H
17) No. of bits required to transmit 7 bit ASCIJ character in asynchronous data
transfer are
a) 10 bits
b) 7 bits
c) 8 bits
d) 9 bits
18) DAC 0808 is __________ bit and uses __________ method.
a) 8 bit, weighted register

b) 8 bit, R-2R ladder

c) 16 bit, R-2R ladder


d) Both b) and c)
19) The control word format of 8255 for using all parts as output in mode 0 and
BSR mode for setting PC 7 line respectively are
a) 80 H, 1 FH

b) 80 H, 0 FH

20) IC 8253 provides __________


a) Three 8 bit up counters
c) Three 8 bit down counters

c) 90 H, 0 FH

d) None

b) Two 16 bit down counters


d) Three 16 bit down counters

______________
Set A

*SLRBB400*

-3-

SLR-BB 400

Seat
No.

T.E. (E and E) (Part II) Examination, 2014


MICROCONTROLLER AND ITS APPLICATIONS
Day and Date : Saturday, 29-11-2014
Time : 10.00 a.m. to 1.00 p.m.

Marks : 80

N.B. : 1) All questions are compulsory.


2) Figures to right indicate full marks.
3) Assume relevant data whenever necessary.
SECTION I
2. Solve any four :
a) Explain interrupt structure of MCS 51.
b) Explain the following instructions :

(45=20)

1) MOV X @ DPTR, A
2) MOV @ R0, # data
3) DJNZ R1, 800 A.
c) Explain the function of the following pins :
1) ALE
2) PSEN
3) RST
4) EA
5) TXD.
d) Explain the importance of TI flag in serial communication.
e) Explain conditional and unconditional JUMP instructions in detail.
3. a) Ten hex numbers are stored in RAM locations 50 H onwards. Write a program
to find the biggest number in the set. The biggest number should finally be
saved in 60 H.
10
b) Explain the stack of 8051 and its operation using PUSH and POP instructions.

10

OR
b) Design 8051 based system with 16 KB of program ROM and 16 KB of data
RAM.

10
Set A

SLR-BB 400

-4-

*SLRBB400*

SECTION II
4. Solve any four :

(45=20)

a) What is EEPROM ? Explain its interfacing with 8051.


b) Write a program to generate square wave at PCO of 8255 which is connected
to 8051 in BSR mode.
c) Interface DAC to 8051 and write a program to generate triangular wave.
d) Explain with block diagram power factor connector circuits.
e) Explain various pins of LCD and its interfacing with 8051.
5. Solve any two :

(210=20)

a) What is the need of interfacing external 8255 to 8051 ? Interface 8255 with
8051 and write a program to read port A in mode 1. Complement the data
and send to port B in mode 0.
b) What is temp. indicator and control circuit ? With suitable block diagram
and software (flow chart) explain the circuit.
c) Draw an interfacing circuit to interface (4 4) matrix type keyboard and
(16 2) LCD display to 8051. Write a program to display keycode on LCD.
_____________________

Set A

SLR-BB 401

*SLRBB401*
Seat
No.

Set

B.E. (Electrical and Electronics Engineering) (Part I) Examination, 2014


INDUSTRIAL DRIVES AND CONTROL
Day and Date : Tuesday, 2-12-2014
Time : 3.00 p.m. to 6.00 p.m.
Instructions :

Total Marks : 100

1) Assume suitable data wherever necessary.


2) Non-programmable calculators are permitted.
3) Q. No. 1 is compulsory. It should be solved in first 30 minutes in
Answer book Page No. 3. Each question carries one mark.
4) Answer MCQ/Objective type questions on Page No. 3 only. Dont
forget to mention, Q.P. Set (A/B/C/D) on Top of Page.
MCQ/Objective Type Questions

Duration : 30 Minutes

Marks : 20

1. Objective questions :

(120=20)

1) For high frequency chopper, the device preferred is ___________


a) Thyristor
b) TRIAC
c) Transistor
d) GTO
2) During regenerative braking mode, back emf is ________ than supply voltage.
a) Less
b) More
c) Equal to
d) None of above
3) The conduction period for the thyristor in a three phase full converter connected to a high
inductive load is ___________
a) 120
b) 180
c) 60
d) 360
4) The motor commonly used in computers and digital system is __________
a) DC shunt motor
b) Induction motor
c) Stepper motor
d) Synchronous motor
5) What is meant by plugging ?
a) Phase sequence of supply is reversed
b) Disconnecting source
c) Stored energy is dissipated
d) All of above
6) In a three phase controlled bridge rectifier, with an increase of overlap angle, the output
DC voltage ____________
a) Decreases
b) Increases
c) Does not change
d) Depends upon load inductance
7) AC to DC circulating current dual converters are operated with which of the following
relationship between their triggering angles ( 1 and 2 ) ?
a) 1 + 2 = 180

b) 1 + 2 = 360

c) 1 2 = 180

d) 1 + 2 = 90

8) Resonant converters are basically used to ________


a) Generate large peaky voltage
b) Reduce switching losses
c) Eliminate harmonics
d) Convert square wave into sine wave
P.T.O.

SLR-BB 401

*SLRBB401*

-2-

9) In a three phase half wave diode rectifier if Vm is the maximum value of per phase voltage
then each diode is subjected to a peak inverse voltage of _________
a) Vm

b)

3 Vm

c) 2 Vm

d) 3 Vm

10) In a thyristor DC chopper, which type of commutation results in best performance ?


a) Voltage commutation
b) Current commutation
c) Load commutation
d) Supply commutation
11) Wound rotor and squirrel cage motors with high slip which develop maximum torque at
standstill are used for
a) Machine tools
b) Presses and punches
c) Elevators
d) All of above
12) In synthetic fibre mills motor with
a) Constant speed are preferred
c) Variable speed are preferred

b) High starting torque are preferred


d) Low starting torque are preferred

13) Reluctance motor is


a) Self starting type synchronous motors
c) Variable torque motor

b) Low torque variable speed motor


d) Low noise, slow speed motor

14) A single phase diode bridge rectifier supplies highly inductive load. The load current can
be assumed to be ripple free. The AC supply side current waveform will be ________
a) Sinusoidal
b) Constant DC
c) Square
d) Triangular
15) An
a)
b)
c)
d)

electric drive consist of


Motor, transmitting shaft and control equipment
Motor and load
Motor, control equipment and load
Motor, supply system and load

16) In a constant power type load


a) Torque is proportional to speed
b) Torque is proportional to square speed
c) Torque is inversely proportional to speed
d) Torque is independent of speed
17) The maximum horse power up to which 440 V electric motors are used is
a) 200 HP
b) 50 HP
c) 20 HP
d) 10 HP
18) To save the energy during braking
a) Dynamic braking is used
c) Regenerative braking is used

b) Plugging is used
d) Mechanical braking is used

19) Pole changing method of speed control is used in


a) Slip ring IM
b) DC shunt motor c) DC series motor

d) Squirrel cage IM

20) Which of the following pair is used for frequency converter ?


a) Squirrel cage IM and synchronous motor
b) Wound rotor IM and synchronous motor
c) Wound rotor IM and squirrel cage IM
d) All of above
______________

Set A

*SLRBB401*

-3-

SLR-BB 401

Seat
No.

B.E. (Electrical and Electronics Engineering) (Part I) Examination, 2014


INDUSTRIAL DRIVES AND CONTROL
Day and Date : Tuesday, 2-12-2014
Time : 3.00 p.m. to 6.00 p.m.
Instructions :

Marks : 80

1) Assume suitable data wherever necessary.


2) Non-programmable calculators are permitted.
SECTION I

2. Solve any four :

(54)

a) What is dual converter ? Explain how rotation of dc motor is reversed in dual converter.
b) Draw the block diagram of electrical drive and explain.
c) Explain and derive fundamental torque equation.
d) Explain parts of electrical drives in detail.
e) A motor drives two loads. One has rotational motion, it is coupled to the motor through a
reduction gear with a = 0.1 and efficiency of 90%. The load has moment of inertia of 10 kg-m2
and a torque of 10 Nm. Other load has translation motion and consists of 100 kg weight to
be lifted up at an uniform speed of 1.5 m/s. Coupling between load and motor has an efficiency
of 85%. Motor has an inertia of 0.2 kg-m2 and runs at a constant speed of 1420 rpm.
Determine, equivalent inertia referred to motor shaft and power developed by motor.
3. Solve any two :

(102)

a) Explain single phase fully controlled converter with block diagram, waveform and different
modes.
b) A 220 V, 1500 rpm, 50 A separately excited dc motor with armature resistance of 0.5 ohm
is fed from circulating current dual converter with ac source voltage (line) = 165 V.
Determine converter firing angles for
1) Motoring operation at rated motor torque and 1000 rpm.
2) Braking operation at rated motor torque and 1000 rpm.
3) Motoring operation at rated motor torque and (1000) rpm.
4) Braking operation at rated motor torque and (1000) rpm.
c) A 2 pole separately excited dc motor has rating of 220 V, 100 A and 750 rpm. Resistance
of armature is 0.1 . The motor has two field coils which are normally connected in
parallel. It is used to drive a load whose torque is expressed as TL = 500 0.3 N Nm,
where N is the motor speed in rpm. Speed below and above rated are obtained by armature
voltage control and by connecting two field winding in series respectively. Calculate
i) Calculate armature current and speed when armature voltage is reduced to 110 V.
ii) Calculate motor speed and current when field coils are connected in series.

Set A

SLR-BB 401

-4-

*SLRBB401*

SECTION II
4. Solve any four :

(54)

a) Explain current regulated VSI control of induction motor drive.


b) Explain rotor resistance control and conventional methods.
c) Explain static krammer drive with block diagram and waveforms.
d) Explain operation of solar drives.
e) A star connected squirrel cage induction motor has following ratings and parameters 400 V,
50 Hz, 4 pole, 1370 rpm, Rs = 2 , Rr = 3 , Xs = Xr = 3.5 .
For regenerative braking operation of inverter fed induction motor drives. Determine
approximate value of
1) Speed for frequency of 30 Hz and 80% of full load torque.
2) Frequency for a speed of 1000 rpm and full load torque.
5. Solve any two :

(102)

a) Explain chopper control of separately excited dc motor with motoring, braking operation.
b) A 440 V, 50 Hz, 970 rpm, 6 pole, star connected, 3 wound rotor IM has following
parameter referred to stator :
Rs = 0.1 , Rr = 0.08 , Xs = 0.3 , Xr = 0.4 . Stator to rotor turn ratio is 2.
Motor speed is controlled by static scherbius drive. Drive is designed for speed range
of 25% below synchronous speed. Maximum value of firing angle 165. Calculate
i) Transformer turns ratio.
ii) Torque for the speed of 780 rpm and = 140 .
(Note : Rd = 0.01 ]
c) A star connected squirrel cage IM has following rating and parameters :
Rs = 2 , Rr = 3 , Xs = Xr = 3.5
For regenerative braking operation of IM drive, determine approximate values of :
1) Speed for frequency of 30 Hz and 80% of full load torque.
2) Frequency for the speed of 1000 rpm and full load torque.
3) Torque for frequency of 40 Hz and speed of 1300 rpm.
4) What will be answers to 1 to 3 if drive works in dynamic braking ?
_____________________

Set A

SLR-BB 402

*SLRBB402*
S

B.E. (E&E) (Part I) Examination, 2014


ENGG. ECONOMICS AND INDUSTRIAL MANAGEMENT
Day and Date : Thursday, 4-12-2014
Time : 3.00 p.m. to 6.00 p.m.

Total Marks : 100

Instructions : 1) All questions are compulsory.


2) Figures to right indicate full marks.
3) Q. No. 1 is compulsory. It should be solved in first 30 minutes
in Answer book Page No. 3. Each question carries one mark.
4) Answer MCQ/Objective type questions on Page No. 3 only.
Dont forget to mention, Q.P. Set (A/B/C/D) on Top of Page.
Duration : 30 Minutes

MCQ/Objective Type Questions

Marks : 20

1. Choose the correct answer :


20
1) In __________________ type ownership, the owner is liable for all obligations
and debts of the business.
a) single
b) partnership
c) joint stock companies
d) public sector
2) The partner who do not take any active part in the conduct of business is
called as _____________
a) active partner
b) inactive partner
c) sleeping partner
d) share holder
3) The art of getting work done through people is called as
a) business
b) management c) industry
d) ownership
4) Public Ltd., company has to allot shares within _______________ days from
date of prospects.
a) 180
b) 90
c) 365
d) 30
5) The investment of small scale industries is around
a) Rs. 300 lakhs b) Rs. 200 lakhs c) Rs. 400 lakhs d) Rs. 100 lakhs
6) Cottage industries are run by
a) members of family
b) government
c) private company
d) shareholders
7) In ____________ more lines of products, more types and sizes are added to
satisfy the needs of large number of consumers.
a) product diversification
b) variety reduction
c) product improvement
d) product development
P.T.O.

SLR-BB 402

-2-

*SLRBB402*

8) The main aim of value engineering is to study the relationship between the
________________ of a part.
a) Design function and cost
b) time and cost
c) production and design
d) design and time
9) A person responsible for setting up a business or an enterprise is called as
____________
a) Businessman b) Industrialist c) Entrepreneur d) Director
10) The number of shareholders should not be less than _____________, but
there is no limit to their maximum number.
a) five
b) seven
c) ten
d) tweleve
11) The enterprise owned and managed by State is called as ___________
a) private company
b) public sector
c) joint stock company
d) partnership
12) In private limited company the capital is collected from ______________
a) single partner
b) government
c) sleeping partner
d) private partners
13) MIS stands for
a) Management Information System b) Management Integrated System
c) Managerial Information System
d) Managerial Investigation System
14) The control of the enterprise is effected through ______________
a) planning
b) directing
c) administration d) controlling
15) Effective managing doesnt require
a) education
b) leadership
c) punctuality
d) money
16) _______________ is not the type of industry.
a) Cottage industries
b) Small scale industries
c) Multiscale industries
d) Large scale industries
17) ______________ is the process by which managers select, train, promote
and retire their subordinates.
a) Directing
b) Staffing
c) Controlling
d) Planning
18) Small scale industries employ ________________
a) 10 to 100
b) 20 to 50
c) 10 to 50
d) 50 to 100
19) Region bounded by horizontal line in breakeven chart is called as
a) variable cost
b) total cost
c) purchasing cost d) fixed cost
20) Make and buy decision depend upon
a) cost
b) quality
c) urgency
d) variety
______________

Set A

*SLRBB402*
S

-3-

SLR-BB 402

B.E. (E&E) (Part I) Examination, 2014


ENGG. ECONOMICS AND INDUSTRIAL MANAGEMENT
Day and Date : Thursday, 4-12-2014

Marks : 80

Time : 3.00 p.m. to 6.00 p.m.


Instructions : 1) All questions are compulsory.
2) Figures to right indicate full marks.
SECTION I
2. a) Explain joint stock companies.

b) Explain economics applied to industries in brief.

3. Explain in detail type of business organisations.

10

OR
Explain infrastructure of Indian economy and trends in performance of infrastructure
sector from independence.
4. Write short notes on any four :

10
20

1) Economic lot size


2) Cost control and cost ratio
3) Line and staff organisation
4) Organizational structure and its characteristics
5) Energy sector related to Indian economy
6) Private sector companies.
Set A

SLR-BB 402

-4-

*SLRBB402*

SECTION II
5. Explain in detail functions of management.

10

6. Explain in detail types of industries.

10

OR
Explain entrepreneurial Vs managerial style.
7. Write short notes on any four :

10
20

1) Project planning function


2) Indian Electricity Act, 2003
3) Advantages of small scale industry
4) Management information system
5) Project proposal process
6) Characteristics of project.
_____________________

Set A

SLR-BB 403

*SLRBB403*
Seat
No.

Set

B.E. (Electrical and Electronics) (Part I) Examination, 2014


INSTRUMENTATION TECHNIQUES
Day and Date : Saturday, 6-12-2014
Time : 3.00 p.m. to 6.00 p.m.

Max. Marks : 100

Instructions : 1) Q. No. 1 is compulsory. It should be solved in first 30


minutes in Answer Book Page No. 3. Each question carries
one mark.
2) Figure to right indicate full marks.
3) Make suitable assumptions if necessary.
4) Answer MCQ/Objective type questions on Page No. 3 only.
Dont forget to mention, Q.P. Set (A/B/C/D) on Top of Page.
MCQ/Objective Type Questions
Duration : 30 Minutes

Marks : 20

1. Select one right answer :

(120)

1) In order to build a 3 bit simultaneous A/D converter ____________ no, of


comparator required.
a) 7
b) 8
c) 15
d) 16
2) Transmission channels for telemetry are
a) Optical links
b) Radio links
c) Ultrasonic and magnetic induction data link
d) All of the above
3) LED consumes ____________ power than LCD.
a) Less
b) More
c) Moderate

d) None of these

4) The dynamics characteristics of capacitive transducers are similar to


those of
a) Low pass filter
b) High pass filter
c) Notch filter
d) Band stop filter
5) High value pot resistance leads to
a) Low sensitivity
c) Low non-linearity

b) High sensitivity
d) Less error

6) Strip chart recorders have advantage of


a) Long period run
b) Change in speed of chart
c) Uniform resolution
d) All of above
P.T.O.

SLR-BB 403

*SLRBB403*

-2-

7) An op-amp is basically
a) Low gain ac amplifier
b) High gain ac amplifier
c) High gain RC coupled amplifier
d) Low gain transformer coupled amplifier
8) Doppler shift principal is used in the measurement of
a) Temperature
b) Frequency
c) Speed

d) Pressure

9) Range of an analog transducer is 0 10 V. For a resolution of 5 mV the bits of


ADC will be
a) 8
b) 9
c) 10
d) 11
10) The desirable static characteristics in the measurement system is/are
a) Sensitivity
b) Reproducibility
c) Accuracy
d) All of these
11) Q factor of Q meter is
a) W0R/L
b) LR/w0

c) w0L/R

12) Strain gauge rosettes are used when


a) Hoop stress is not known
c) Principal stress is known

b) Principal stress is not known


d) Longitudinal stress is not known

d) None of these

13) A force digital transducer measures the pressure of the range of 0 200 N with
the resolution of 0.1% of full scale. The smallest charge it can measures is
a) 0.2 N
b) 0.4 N
c) 0.5 N
d) 1.000 N
14) The lower limits of useful working range of a transducer is determined by its
a) Error
b) Noise
c) Both a) and b) d) None of these
15) The largest deviation from mean is
a) Range of doubt
b) Possible error
c) Either a) or b)
d) Standard deviation
16) Scan time and execution time of PLC is depends on
a) No. of inputs
b) Power supply c) Processor
d) No. of outputs
17) In ac circuits the connection of measuring instruments causes loading errors which
may affect the measurands ?
a) Magnitude
b) Phase
c) Waveform
d) All of the above
18) Inverse transducer converts
a) Electrical energy to any form of energy
b) Electrical energy to light energy
c) Mechanical displacement to electrical signal
d) Electrical energy to mechanical form
19) A strip chart recorder is
a) an active transducer
c) an output transducer

b) an inverse transducer
d) both b) and c)

20) The of gain of a 741 OP amp falls at a low frequency of


a) 10 KHZ
b) 10 hz
c) 100 HZ
______________

d) 1000 HZ
Set A

*SLRBB403*

-3-

SLR-BB 403

Seat
No.

B.E. (Electrical and Electronics) (Part I) Examination, 2014


INSTRUMENTATION TECHNIQUES
Day and Date : Saturday, 6-12-2014

Marks : 80

Time : 3.00 p.m. to 6.00 p.m.


Instructions : 1) All questions are compulsory.
2) Figure to right indicate full marks.
3) Make suitable assumptions if necessary.
SECTION I
2. Solve any four :

(54)

1) Derive the expression for resistance change in strain gauge.


2) What is the different circuits are used for temperature measurement by RTD ?
3) What are the different types of undesirable characteristics in instrumentation
system ?
4) Explain programmable gain amplifier with neat circuit diagram.
5) Derive the output for first order band pass filter.
6) Explain the transducer required for acceleration measurement.
3. Solve any two :

(102)

1) Explain TDM and FDM in detail.


2) Explain instrumentation system block diagram in detail.
3) Derive the output expression for cylindrical strain gauge with neat diagram.

Set A

SLR-BB 403

-4-

*SLRBB403*

SECTION II
4. Solve any four :

(54)

1) Explain data acquisition system with neat diagram.


2) What is working principle of magnetic tape recorder ? Explain tape recorder with
neat diagram.
3) Explain various types of LCD.
4) What are the selection criteria for PLC ?
5) Explain frequency to voltage and voltage to frequency converter.
6) Amongst all ADCs explain the fastest one.
5. Solve any two :

(102)

1) What is instrumental set up required for measurement of temperature of furnace


wall ?
2) Design a system for measurement of flow rate and explain how it can be used in
PLC to control the valve ?
3) Explain the recorder which can record the one quantity with respect to other
quantity ?
_____________________

Set A

SLR-BB 404

*SLRBB404*
Seat
No.

Set

B.E. (Electrical and Electronics Engineering) (Part I) Examination, 2014


POWER SYSTEM II
Day and Date : Tuesday, 9-12-2014
Time : 3.00 p.m. to 6.00 p.m.

Max. Marks : 100

Instructions : 1) Assume suitable data wherever necessary.


2) Non-programmable calculators are permitted.
3) Q. No. 1 is compulsory. It should be solved in first 30 minutes in
Answer Book Page No. 3. Each question carries one mark.
4) Answer MCQ/Objective type questions on Page No. 3 only. Dont
forget to mention, Q.P. Set (A/B/C/D) on Top of Page.
MCQ/Objective Type Questions
Duration : 30 Minutes

Marks : 20

1. Choose the correct answer :

(120=20)

1) The correct relation is


a) 2 = 0.5 + j 0.866

b) 2 = 0.5 + j 0.866

c) 2 = 0.5 j 0.866

d) 2 = 0.5 j 0.866

2) In a synchronous generators, the positive sequence impedance Z1 and negative sequence


impedance Z2 are related as
a) Z1 = Z2
b) Z1 > Z2
c) Z1 < Z2
d) None of these

3) Unsymmetrical faults
a) Introduce unbalance in the system
b) Indicate abnormal conditions in the system
c) Are more frequent than symmetrical faults
d) All of the above
4) The positive sequence impedance of a transmission lines is 3 . Its negative sequence
impedance will be
a) 3
b) 1
c) 9
d) 6
5) A 250 KVA, 11 KV synchronous generator has 0.4 p.u. synchronous reactance. The per
unit synchronous reactance on the base values of 100 KVA and 22 KV is
a) 0.5
b) 0.25
c) 0.04
d) 0.004
6) For a transformer with primary delta connected and secondary is delta connected, the
zero sequence equivalent network is
a)

b)

c)

d)
P.T.O.

SLR-BB 404

-2-

*SLRBB404*

7) In a power system the time constant of governor mechanism and turbine is


a) same as that of field winding time constant b) greater than field winding time constant
c) less than field winding time constant
d) none of the above
8) The incremental cost characteristics of two generators delivering 100 MW are as follows
df1 = 1.0 + 0.01P1 df 2 = 0.8 + 0.01P2
dp2
dp1
For economic operation, the generations P1 and P2 should be
a) P1 = P2 = 50 MW
b) P1 = 60 MW, P2 = 40 MW
c) P1 = 40 MW, P2 = 60 MW
d) P1 = 70 MW, P2 = 30 MW
9) The function of a governor in a power system is to
a) control the load current
b) adjust the speed
c) adjust the terminal voltage
d) none of these
10) In the optimum generator scheduling of different power plants, the minimum fuel cost is
obtained when
a) only the incremental fuel cost of each plant is same
b) the incremental fuel cost of each plant multiplied by its penalty factor is same
c) the ratio of the incremental fuel cost to the penalty factor of each plant is same
d) the penalty factor of each plant is same
11) If the excitation of a synchronous generator connected to an infinite bus and delivering
power at a lagging power factor is increased.
a) the terminal voltage increases
b) voltage angle increases
c) current delivered increases
d) all of these
12) Zero sequence fault currents are absent for
a) S LG fault
b) L-L fault
c) L-L-G fault
d) all of the above
13) The method of neutral grounding affect
a) positive sequence network
b) negative sequence network
c) zero sequence network
d) all of the above
14) A 90 MVA 11 KV generator has an inertia constant H = 3. The stored energy in the rotor
at synchronous speed will be
a) 30 KJ
b) 270 KJ
c) 270 MJ
d) 810 MJ
15) During a disturbance on a power system, if the angle continues to increase indefinitely
a) the system is stable
b) the system is unstable
c) stability cannot be predicted
d) this data is not sufficient
16) If the time taken for clearing a fault is greater than critical clearing time
a) the generator looses synchronism
b) the system is stable
c) data is not sufficient
d) none of the above
17) Stored energy in a synchronous machine is equal to
d) none of above
a) GH
b) G/H
c) G2 H
18) In symmetrical components
a) the angle between positive sequence phasors is 120
b) the angle between negative sequence phasors is 120
c) magnitudes of all phasors of positive sequence components are equal
d) all of the above
19) Solution of swing equation
a) gives an idea about system stability
b) helps in finding the fault current
c) both a) and b)
d) none of the above
20) Which of the following is a symmetrical fault ?
a) Single line to earth
b) Phase to phase
c) All three phase to earth
d) Two phases to earth
______________

Set A

*SLRBB404*

-3-

SLR-BB 404

Seat
No.

B.E. (Electrical and Electronics Engineering) (Part I) Examination, 2014


POWER SYSTEM II
Day and Date : Tuesday, 9-12-2014
Time : 3.00 p.m. to 6.00 p.m.
Instructions :

Marks : 80

1) Assume suitable data wherever necessary.


2) Non-programmable calculators are permitted.
SECTION I

2. Solve any four :

(54)

a) The line currents in a three phase system are Ia = 5 < 60 A, Ib = 5, 60 A and Ic = 0 find
the symmetrical components.
b) Analyze a single line to ground fault and show interconnections of sequence networks.
c) What is per unit system ? State its advantages.
d) Explain steady state stability and transient stability.
e) Derive the expression for economic generation of power when two machines are feeding
a common load.
f) Explain equal area criterion.
3. Solve any two :

(102)

a) Derive swing equation and explain M and H constants.


b) Explain critical clearing angle and its effect on stability.
c) Explain point by point solution of a swing equation.
SECTION II
4. Solve any four :

(54)

a) Draw zero sequence networks for star/star, star/delta, delta/delta and delta/star
connections of transformers.
b) A 30 MVA, 15 KV generator with solidly grounded neutral has Z1 = Z2 = j 0.2 p.u.,
Z0 = j 0.05 p.u. A line to ground fault occurs on the generator terminals. Find the fault
current.
c) Explain concept of symmetrical components.

Set A

SLR-BB 404

-4-

*SLRBB404*

d) Draw zero sequence networks for star/star, star/delta, delta/delta and delta/star connections
of transformers where all star connections have their star points solidly grounded to earth.
e) State the properties of operator in relation with symmetrical components.
f) Show that a line to ground fault is more severe than a LLLG fault of an unloaded alternator.
5. Solve any two :

(102)

a) Draw the zero sequence network for the system shown below. All reactances are in p.u.
to the same base.

b) A synchronous generator has its neutral grounded through a reactance Xn. The generator
has balanced e.m.f.s and sequence reactances are X1 = X2 >> X0.
Draw the sequence networks of the generator as seen from the terminals and derive an
expression for solid line to ground fault on phase a.
c) Explain with a block diagram, automatic voltage control system.
_____________________

Set A

SLR-BB 405

*SLR-BB-405*
Seat
No.

Set

B.E. Electrical and Electronics Engineering (Part I ) Examination, 2014


Elective I : RENEWABLE ENERGY SOURCES
Day and Date : Thursday, 11-12-2014
Time : 3.00 p.m. to 6.00 p.m.

Total Marks : 100

Instructions : 1) Q. No. 1 is compulsory. It should be solved in first 30 minutes in


Answer Book Page No. 3. Each question carries one mark.
2) Assume suitable data wherever necessary.
3) Non-programmable calculators are permitted.
4) Answer MCQ/Objective type questions on Page No. 3 only.
Dont forget to mention, Q.P. Set (A/B/C/D) on Top of Page.
MCQ/Objective Type Questions
Duration : 30 Minutes

Marks : 20

1. Choose the correct answer :

(120=20)

1) A PV hybrid system in installed with a back-up system of


a) Steam generator
b) Petrol generator
c) Diesel generator
d) All of the above
2) India has a potential of _____________ of wind power.
a) 40,000 MW
b) 20,000 MW
c) 25,000 MW

d) 35,000 MW

3) Central Receiver collector uses as _____________ reflector.


a) Heliostats
b) Fresnels lens c) Aluminium
d) Steel
4) The ultimate capacity of korba thermal power station in M.P is _____________
a) 3000 MW.
b) 2500 MW.
c) 2100 MW.
d) 2700 MW.
5) The main issue in the tower design is the _____________
a) Availability of sufficient and suitable place
b) Availability of material
c) Structural dynamics
d) All of these
6) The maximum power coefficient is of the order of _____________ for vertical axis
rotor. (Savonius rotor)
a) 0.15
b) 0.25
c) 0.30
d) 0.35
7) Lift and drag both are proportional to _____________ and _____________ of
wind speed.
a) Air density and square of wind speed
b) Volume and direction of wind
c) Air density and cube of air speed
d) None of the above
P.T.O.

SLR-BB 405

-2-

*SLR-BB-405*

8) Wind turbines of electrical rating of 100 KW and above are of _____________


a) Variable speed type
b) Constant speed type
c) Both a) and b)
d) None of these
9) The capacity of battery is expressed in _____________
a) volts-amperes
b) ampere-hours c) ampere-minutes d) volt-hours
10) The following material can be used to fabricate Solar cell.
a) Carbon Sulphides
b) Cadmium Sulphide
c) Chlorium Sulphide
d) Potassium Sulphide
11) Which of the following is not a hydrothermal convective system ?
a) Dry steam fields
b) Liquid dominated systems
c) Petro thermal
d) Hot water fields
12) Cowdung originally consists of _____________ of solids.
a) 18%
b) 20%
c) 22%

d) 25%

13) The approximate heat content of Biogas is _____________


a) 20-25 MJ/m3
b) 25-30 MJ/m3
c) 23-30 MJ/m3
d) 31-36 MJ/m3
14) Which of the following is the method of producing hydrogen using solar energy ?
a) Steam reformation
b) Photo-electrolysis
c) Electrolysis
d) Thermochemical method
15) In a solid storage the hydrogen is stored in the form of
a) Non-metal hydrogen
b) Metal hydrides
c) Non-metal hydrides
d) Metal mixture
16) The Methane gas reforming reaction is _____________
a) Exothermic
b) Giving Co2 as product
c) Produces heat and water
d) Endothermic
17) The close cycle is also called as _____________
a) Steam cycle
b) Carnot cycle
c) Anderson cycle d) Rankine cycle
18) The binary cycle system is basically based on _____________
a) Claude cycle
b) Rankine cycle c) Barjot cycle
d) Hybrid cycle
19) Earthquakes and volcanoes are largely located in the
a) Longitudes
b) Altitudes
c) Plate boundaries
d) Local boundaries
20) A potential geothermal source region should have _____________
a) Low thermal gradient
b) High thermal gradient
c) Both a) and b)
d) None of above
______________

Set A

*SLR-BB-405*

-3-

SLR-BB 405

Seat
No.

B.E. Electrical and Electronics Engineering (Part I ) Examination, 2014


Elective I : RENEWABLE ENERGY SOURCES
Day and Date : Thursday, 11-12-2014

Marks : 80

Time : 3.00 p.m. to 6.00 p.m.


Instructions : 1) Assume suitable data wherever necessary.
2) Non-programmable calculators are permitted.
SECTION I
2. Solve any four :

(54)

a) What are the prospects of renewable energy sources ?


b) How are the concentrating collectors classified ? Explain with help of neat chart.
c) Draw and explain Solar Photovoltaic system in detail.
d) Explain Liquid flat plate collector with neat sketch.
e) Explain solar distillation.
f) Explain wind energy conversion system with neat diagram.
3. Solve any two :

(102)

a) What are the design consideration of Horizontal axis wind machines and also
explain Rotor and Wind stream variation ?
b) Write short notes on :
(Draw diagram)
i) Sensible heat storage and its types.
ii) Latent heat storage.
c) Derive and expression for forces on blades and axial thrust on turbines.
Set A

SLR-BB 405

-4-

*SLR-BB-405*

SECTION II
4. Solve any four :

(54)

a) Write short notes on Estimates and nature of Geothermal energy.


b) Explain digester design considerations for Biogas plant.
c) Explain hybrid cycle for OTEC.
d) Explain how wave energy conversion devices are classified.
e) Explain electrolysis process for Hydrogen Production.
f) Explain double stage process for Biogas plant.
5. Solve any two :

(210)

a) Derive an expression for energy and power from waves.


b) Explain open cycle for OTEC and also explain the double-basin, linked basin
plant with neat sketch.
c) Explain :
i) Fuel Properties of Biogas.
ii) Site selection factors for Biogas plant.
_____________________

Set A

SLR-BB 406

*SLRBB406*
Seat
No.

Set

B.E. (Electrical and Electronics Engg.) (Part II) Examination, 2014


SWITCHGEAR AND PROTECTION
Day and Date : Tuesday, 25-11-2014
Time : 3.00 p.m. to 6.00 p.m.

Total Marks : 100

Instructions : 1) All questions are compulsory.


2) Assume the suitable data whenever necessary.
3) Q. No. 1 is compulsory. It should be solved in first 30 minutes
in Answer Book Page No. 3. Each question carries one mark.
4) Answer MCQ/Objective type questions on Page No. 3 only.
Dont forget to mention, Q.P. Set (A/B/C/D) on Top of Page.
Duration : 30 Minutes

MCQ/Objective Type Questions

Marks : 20

1. Write the correct choice :


(120=20)
1) The R.R.R.V. depends upon
a) system voltage
b) circuit p.f. only
c) switching condition only
d) both b) and c)
2) Electro-magnetic relays may be operated by
a) electro-magnetic attraction
b) electro-magnetic induction
c) thermal effect
d) any of the above
3) Which of the following is distance relay ?
a) Impedance
b) Reactance
c) Mho
d) All of these
4) Differential protection principle is used in the protection of
a) generators
b) transformers
c) feeders
d) all of the above
5) In air blast circuit breakers, the pressure of air is of the order of
a) 100 mm Hg
b) 1 kg/cm2
c) 20 to 30 kg/cm2
d) 200 to 300 kg/cm2
6) The fuse blows off by
a) arcing
b) burning
c) melting
d) none of these
7) A circuit breaker is
a) power factor correcting device
b) a device to neutralize the effect of transients
c) a waveform correcting device
d) a current interrupting device
8) The function of protective relay in a circuit breaker is
a) to each any stray voltages
b) to close the contacts when the actuating quantity reaches a certain
predetermined value
c) to limit arcing current during the operation of circuit breaker
d) to provide additional safety in the operation of circuit breaker
P.T.O.

SLR-BB 406

*SLRBB406*

-2-

9) For magnetic blow-off arc, the magnetic field is produced


a) in the load circuit
b) at right angles to the axis of arc
c) in line with axis of arc
d) any of above
10) Which of the following circuit breakers is preferred for EHT application ?
a) Air blast circuit breakers
b) Minimum oil circuit breakers
c) Bulk oil circuit breakers
d) SF6 oil circuit breakers
11) The acting contacts for a circuit breakers are made of
a) Stainless steel
b) Hard pressed carbon
c) Porcelain
d) Copper tungsten alloy
12) A Merz-price protection is suitable for
a) transformers
b) alternators

c) feeders

d) transmission lines

13) Breaking capacity of a circuit breaker is usually expressed in terms of


a) Amperes
b) Volts
c) MW
d) MVA
14) MHO relay is used for
a) rectifiers
c) transmission lines

b) circuit breakers
d) feeders

15) The over-voltage surges in power systems may be caused by


a) lightning
b) switching
c) resonance
d) any of the above
16) SF6 gas
a) is yellow in color
c) is highly toxic

b) has pungent odor


d) is non-inflammable

17) In modem EHV circuit breakers, the operating time between instant of receiving
trip signal and final contact separation is, of the order of
a) 0.001 sec
b) 0.015 sec
c) 0.003 sec
d) 0.03 sec
18) Fusing factor for a HRC fuse is
a) Minimum fusing current / Current rating
b) Minimum fusing current / Minimum rupturing time
c) Maximum fusing current / Minimum fusing current
d) Minimum fusing current / Prospective current of circuit
19) The transient voltage that appears across the contacts at the instant of arc extinction
is called
a) recovery voltage
b) re-striking voltage
c) supply voltage
d) peak voltage
20) A relay used for protection of motors against overload is
a) Impedance relay
b) Electromagnetic attraction type
c) Thermal relay
d) Buchholzs relay
______________

Set A

*SLRBB406*

-3-

SLR-BB 406

Seat
No.

B.E. (Electrical and Electronics Engg.) (Part II) Examination, 2014


SWITCHGEAR AND PROTECTION
Day and Date : Tuesday, 25-11-2014

Marks : 80

Time : 3.00 p.m. to 6.00 p.m.


Instructions : 1) All questions are compulsory.
2) Assume the suitable data wherever necessary.
SECTION I
2. Solve any four :

(54=20)

1) Write a short note on earth Blades.


2) Write a short notes on current chopping.
3) Give the difference between MCB and M.C.C.Bs.
4) With neat diagram explain HRC fuse.
5) Explain Principle and operation of Air-break C.B.
3. Solve any two :

(102=20)

1) Explain the construction and working principle of Air blast C.B.


2) Define restriking voltage and recovery voltage and RRRV. Derive expression
for restriking voltage and RRRV in terms of system voltage, inductance and
capacitance.
3) With the help of neat diagram explain Minimum Oil Circuit Breaker.

Set A

SLR-BB 406

-4-

*SLRBB406*

SECTION II
4. Solve any four :

(54=20)

1) Describe the loss of excitation of an alternator.


2) Write a short note on surge arrestor and absorbers.
3) Basic requirement of quantities of protective devices (Relays).
4) Explain the details operation of Buchholdz relay.
5) Explain the instantaneous over current relay, inverse over current relay.
5. Solve any two :

(102=20)

1) Explain the different schemes for the protection of A.C. Generator.


2) Describe the microprocessor based over current relay with its block diagram and
flow chart to realise its characteristics.
3) Explain the causes of over-voltages.
_____________________

Set A

SLR-BB 407

*SLR-BB-407*
Seat
No.

Set

B.E. (Electrical and Electronics Engg.) (Part II) Examination, 2014


ELECTRICAL MACHINE DESIGN
Day and Date : Thursday, 27-11-2014
Time : 3.00 p.m.to 6.00 p.m.

Max. Marks : 100

Instructions : 1) All questions are compulsory.


2) Assume the suitable data wherever necessary.
3) Q. No. 1 is compulsory. It should be solved in first 30 minutes
in Answer book Page No. 3. Each question carries one mark.
4) Answer MCQ/Objective type questions on Page No. 3 only.
Dont forget to mention, Q.P. Set (A/B/C/D) on Top of Page.
MCQ/Objective Type Questions

Duration : 30 Minutes

Marks : 20

1. Solve the following :

20

1) In an induction motor air gap is increased


A) Speed will reduce
B) Efficiency will improve
C) Power factor will be lowered
D) Breakdown torque will reduce
2) Line joining tangent and Torque line in a circle diagram gives
A) Copper loss
B) Stator loss
C) Maximum output
D) Maximum Torque
3) Specific magnetic loading is not governed by
A) Maximum flux density in iron parts B) Armature reaction and commutation
C) Iron losses
D) Magnetizing current
4) High current density increases
A) copper cost
B) copper loss

C) efficiency

D) none of the above

5) The rotor slots in 3 phase induction motor are kept inclined. This phenomenon
is known as
A) skewing
B) crawling
C) cogging
D) none of the above
6) The output of rotating machine is limited by
A) peripheral speed
B) temperature rise
C) size of machine
D) none of the above
7) CRGO has _____________ permeability in the direction of grain orientation.
A) nil
B) maximum
C) minimum
D) none of the above
8) Eddy current are reduced in high content silicon steel as it
A) increases resistivity
B) reduces resistivity
C) short circuits
D) none of the above
P.T.O.

SLR-BB 407
9) Stepped core is used to
A) increase output
C) decrease the cost of copper

*SLR-BB-407*

-2-

B) reduces the cost


D) increases efficiency

10) With increases in voltage, window space factor


A) increases
B) remain same C) decreases

D) none of the above

11) Iron losses of a machine are


A) Directly proportional to flux density
B) Directly proportional to the square of flux density
C) Inversely proportional to flux density
D) Inversely proportional to the square of flux density
12) The thickness of insulation between HV and LV is given by relation
A) (5+0.9KV) mm B) (9+0.5KV) mm C) (0.5+0.9KV) mmD) (4+0.5KV) mm
13) With larger value of L/T, the insulation cost
A) increases
B) remain same C) decreases

D) none of the above

14) Usual values of ac recommended for turbo alternator are


A) 5000-10000 A/m
B) 10000-20000 A/m
C) 20000-50000 A/m
D) 50000-100000 A/m
15) Induction motors are provided with open slots because
A) Exciting current is high
B) PF is low
C) PF is high
D) Both A) and B)
16) Current density in rotor bars of an induction motor is
A) 3-4 A/mm2
B) 4-7 A/mm2
C) 3-10 A/mm2

D) 10-15 A/mm2

17) Which component of the no load current of the transformer is opposite in phase to
the Induced EMF ?
A) Magnetizing component
B) Core loss component
C) Both A) and B) above
D) None of the above
18) The effect of harmonics in rotating machines can be minimized by
A) Use of longer air gap
B) Skewing the poles
C) Use of distributed winding
D) All of the above
19) An alternator with higher value of SCR has
A) Poor voltage regulation and lower stability limit
B) Poor voltage regulation and higher stability limit
C) Better voltage regulation and higher stability limit
D) None of the above
20) In case of Induction motor the air gap flux density is usually taken is
A) 0.1T 0.2T
B) 0.2T 0.35T C) 0.3T 0.6T
D) IT 1.2T
______________

Set A

*SLR-BB-407*

-3-

SLR-BB 407

Seat
No.

B.E. (Electrical and Electronics Engg.) (Part II) Examination, 2014


ELECTRICAL MACHINE DESIGN
Day and Date : Thursday, 27-11-2014
Time : 3.00 p.m.to 6.00 p.m.

Marks : 80

Instructions : 1) All questions are compulsory.


2) Assume the suitable data wherever necessary.
SECTION I
2. Solve any four :

(45=20)

a) For a transformer show that, Et = K KVA . Where Et. Emf per turn,
KVA = Rating of Transformer.
b) Determine the overall dimensions of 100KVA, 11000/415V, three phase and 50 Hz
core type transformer. Use following data :
Emf per turn = 5 V, Maximum flux density = 1.4wb/Sq.m, Current density = 2.6 A/Sq.mm,
Kw = 0.3, stacking factor = 0.9, use three stepped core for which width of largest
stamping is 0.9d. Ai= 0.6d2.
c) Explain the methods of improving starting torque of three phase induction motor.
d) Estimate magnetizing current per phase for 11000/415V, three phase, delta-star
core type transformer from following data :
i) Gross cross section area of limb and yoke = 200 cm2
ii) Height of each limb = 130 cm
iii) Length of each yoke = 100 cm
iv) Stacking factor = 0.9
v) Density of steel = 7600 Kg/m3
vi) Reactive VA/Kg = 5
e) Derive the expression for bar current and end ring current for a three phase induction
motor.
f) Derive an expression for winding resistance of three phase induction motor.
3. Solve any two :

(210=20)

a) Determine the main dimensions and turns per phase of 187.5 KW, 3-phase, 50 Hz,
400 V, 1410 rpm slip ring induction motor. Assume Bav = 0.5 web/m2, ac = 30000 A/m,
efficiency = 0.9, power factor = 0.9, slot space factor = 0.4, current density = 3.5 A/mm2,
stacking factor = 0.955, the ratio of length to pole pitch is 1.2. The machine is delta
connected.
Set A

SLR-BB 407

-4-

*SLR-BB-407*

b) Explain the design procedure for stator of single phase induction motor.
c) Write a short note on :
i) Factors affecting choice specific magnetic loading and specific electric
loading
ii) Harmonic induction and harmonic sub synchronous torques.
SECTION II
4. Solve any four :

(45=20)

a) Write short note on damper winding used in synchronous machine.


b) A 500 KVA, 600 rpm, 3.3 KV, 50 Hz, 3-phase salient pole alternator has 180 turns
per phase. Estimate the length of air gap if the average flux density is 0.54 web/m2.
The ratio of pole arc to pole pitch = 0.65, the short circuit ratio = 1.2 and gap
extension coefficient = 1.15. The mmf required for gap is 80% of no load field mmf.
Winding factor = 0.955.
c) Derive an expression for capacitance required for maximum torque in capacitor
start single phase induction motor.
d) A 1250 KVA, 3 phase, 20 poles, 50 Hz, 6600 V salient pole alternator has the
following data :
Air gap diameter = 1.6m, length of core = 0.45m specific electric loading = 28000 ac/m,
pole arc/pole pitch = 0.68, stator slot pitch = 28 mm, current density in damper
bar = 3 A/mm2 design suitable damper winding for the machine.
e) Explain the design procedure rotor of single phase induction motor.
f) What are different pole constructions are used for salient pole alternator.
5. Solve any two :

(210=20)

a) State and explain different leakage reactance in case of single phase induction
motor.
b) Explain the design procedure field winding in an alternator.
c) A 3-phase 50 Hz, star connected alternator has the following design data :
Terminal Voltage = 3300 V, RPM = 300, rotor diameter = 2.3m, core length = 0.35 m,
stator winding has single layer, Kw = 0.955, Bav = 0.9 Tesla. Estimate,
a) No. of slots
b) No. of conductors/slot.
_____________________

Set A

SLR-BB 408

*SLRBB408*
Seat
No.

Set

B.E. (Electrical and Electronics Engg.) (Part II) Examination, 2014


FLEXIBLE AC TRANSMISSION SYSTEM
Day and Date : Saturday, 29-11-2014
Time : 3.00 p.m. to 6.00 p.m.

Total Marks : 100

Instructions : 1) Assume the suitable data wherever necessary.


2) Q. No. 1 is compulsory. It should be solved in first 30 minutes in
Answer Book Page No. 3. Each question carries one mark.
3) Answer MCQ/Objective type questions on Page No. 3 only.
Dont forget to mention, Q.P. Set (A/B/C/D) on Top of Page.
MCQ/Objective Type Questions
Duration : 30 Minutes

1. Solve the following :

Marks : 20

(120=20)

1) In power oscillating damping the VAR output is controlled in


a) Linear manner
b) Bang blast manner
c) Non-linear manner
d) Bang Bang manner
2) In FC-TCR to decrease the capacitance output the current in the reactor is
increased by
a) Increase angle
b) Decrease angle
c) Comparing angle
d) None
3) Following controller is used for power transmission management in a Multiple
Substrain
a) IPFC
b) UPFC
c) SVC
d) TCSC
4) IPFC provides capability directly transformer power between compensated lines
a) Active
b) Real
c) Reactive
d) Apparent
5) PWD converter for fact technology is high power, high voltage and
a) High frequency
b) Low frequency
c) Medium frequency
d) None
6) FACTS provide
a) Stability transfer capability and controllability
b) Stability and transient response
c) Phase sequence and comparability
d) None
7) Combined series controller used for
a) Maximum utilization of a transmission line
b) Maximum stabilization of a transmission line
c) Maximum comparison of a transmission line
d) None
P.T.O.

SLR-BB 408

*SLRBB408*

-2-

8) Midpoint VAR compensator exchange only power with the transmission line in
this process.
a) Reactive
b) Active
c) Apparent
d) None
9) In SVC the thyristor used is
a) With GTO capability
c) With T off capability

b) Without GTO capability


d) Without T off capability

10) In FACTS technology the ultimate limit will be


a) Thermal and dielectric
b) Stability
c) Dielectric and Stability
d) None
11) Series controller inject ___________ to control the power flow.
a) Current
b) Voltage
c) Both VTG and current
d) None
12) The ultimate objective of applying reactive shunt compensation in a
transmission system is to
a) Increase the transmittable power
b) Decrease the transmittable power
c) Improve the system performance
d) To improve loading capability
13) If the TCR switching is restricted to a fixed delay angle, usually = 0, then it
becomes
a) TSR
b) TSC
c) FC-TCR
d) None
14) In the linear operating range the V-I characteristics and the functional compensation
capabilities of the STATCOM and SVC are
a) Different
b) Similar
c) Exactly opposite
d) None
15) AC power transmission over long lines is primarily limited by
a) Series active reactance
b) Shunt active reactance
c) Series reactive impedance
d) Shunt reactive impedance
16) The objective of GCSC scheme is to control the
a) ac voltage across the capacitor
b) dc voltage across the capacitor
c) ac current through the capacitor
d) dc current through the capacitor
17) The TSSC can control the degree of series compensation by
a) Inserting series inductance
b) Inserting series capacitor
c) Bypassing series capacitor
d) Both b) and c)
18) The Synchronous Voltage Source (SVS) is analogous to
a) Ideal transformer
b) Ideal generator
c) Ideal motor
d) Ideal condenser
19) The transmitted power by phase angle regulator at the ends of line is given by
2
V2
V2
V
sin( ) d) P =
cos( )
a) P = V sin( ) b) P = sin( ) c) P =
X
X
X
X
20) In SVC capacitor is used for
a) Light load condition
b) Heavy load condition
c) Transient condition
d) None
Set A
______________

*SLRBB408*

-3-

SLR-BB 408

Seat
No.

B.E. (Electrical and Electronics Engg.) (Part II) Examination, 2014


FLEXIBLE AC TRANSMISSION SYSTEM
Day and Date : Saturday, 29-11-2014
Time : 3.00 p.m. to 6.00 p.m.

Marks : 80

Instructions : 1) All questions are compulsory.


2) Assume the suitable data wherever necessary.

SECTION I

2. Solve any four :

(54=20)

a) What limits the loading capability ? Explain.


b) Explain the power flow in parallel paths and meshed transmission system.
c) Explain the objectives of shunt compensation.
d) With the help of neat diagram and waveforms explain TSC-TCR type Var
generator.
e) Explain the direct and indirect control in STATCOM.
f) With the help of circuit diagram and V-I characteristics explain GCSC.
3. Solve any two :

(102=20)

a) Explain the basic types of FACTS controllers.


b) Compare STATCOM and SVC.
c) With the help of circuit diagram and V-I characteristics explain TSSC in
detail.

Set A

SLR-BB 408

-4-

*SLRBB408*

SECTION II

4. Solve any four :

(54=20)

a) Explain functional external control scheme for the SSSC.


b) With the help of circuit diagram and V-I characteristics explain TCSC.
c) Explain the power flow control by phase angle regulators.
d) How a transient stability is improved with phase angle regulators ? Explain.
e) Explain the basic operating principle and characteristics of IPFC.
f) Compare UPFC with phase angle regulator.
5. Solve any two :

(102=20)

a) With the help of neat block diagrams explain internal control scheme for
SSSC employing a directly controlled converter.
b) Explain with neat diagram hybrid phase angle regulators.
c) With the help of block diagram explain basic UPFC control scheme.
_____________________

Set A

SLR-BB 409

*SLRBB409*
Seat
No.

Set

B.E. (Electrical and Electronics Engineering) (Part II)


Examination, 2014
Elective II : HIGH VOLTAGE DC TRANSMISSION
Day and Date : Monday, 1-12-2014
Time : 3.00 p.m. to 6.00 p.m.

Total Marks : 100

Instructions : 1) Q. No. 1 is compulsory. It should be solved in first 30 minutes


in Answer Book Page No. 3. Each question carries one mark.
2) Answer MCQ/Objective type questions on Page No. 3 only.
Dont forget to mention, Q.P. Set (A/B/C/D) on Top of Page.
3) Assume suitable data wherever necessary.
4) Non-programmable calculators are permitted.
MCQ/Objective Type Questions
Duration : 30 Minutes

Marks : 20

1. Choose the correct alternative :

20

1) The initial HVDC valves were ____________


a) IGATS
b) Thyristors
c) Mercury arc rectifiers
d) None of above
2) The output voltage of converter is changed by varying ________
a)
b)
c)
d) Any of the above , or
3) In a monopolar system usually the pole is _____________
a) positive
b) negative
c) positive and negative
d) alternatively positive and negative
4) Thyristors valves came into operation in the year __________
a) 1950
b) 1954
c) 1972
d) 2000
5) 12-pulse converters are used in modern converters because of ________
a) reduced current
b) reduced ripple
c) increased voltage and reduced harmonics
d) both b) and c)
6) Power transfer in DC line is depends on _____________
a) sending and receiving end voltages
b) number of pulses in the rectifier
c) line resistance
d) none of the above
7) The common control done in the converters is __________
a) rectifier as both voltage and current controller
b) inverter as both voltage and current controller
c) inverter as current controller
d) none of above

P.T.O.

SLR-BB 409

*SLRBB409*

-2-

8) HVDC-VSC scheme employs ________________


a) IGBT valves
b) Light or optically triggered thyristor valves
c) Mercury arc valves
d) MOSFETs and GTO valves
9) During commutation in a converter ____________
a) voltage is exchanged
b) Current is transformed from one valve to the other
c) DC voltage is blocked
d) None of the above
10) Modern HVDC system are all _____________
a) 3-pulse converters
b) 6-pluse converters
c) 24-pluse converters
d) 12-pulse converters
11) In 12-pulse connections, transformers are connected ___________
a) Delta/Delta (both)
b) Star/Star (both)
c) Star/Delta (both)
d) One Star/Star and other Star/Delta
12) If a angle of advance is 30 and overlap angle is 12, the extension angle
will be __________
a) 42
b) (30/2)
c) 18
d) 21
13) A surge diverter is used across the DC CB to _____________
a) limit recovery voltage
b) limit fault current
c) absorb the arc energy
d) all of the above
14) Which of the following is a series connected FACTS device ?
a) UPFC
b) STATCOM
c) TCSC
d) TCPST
15) Multiterminal systems are ____________
a) Series connected
b) Parallel connected
c) Ring connected
d) All of above
16) Firing angle control in modern HV converter is ____________
a) IFC
b) EPC
c) IPC
d) Both a) and b)
17) TIF factor usually lies between ____________
a) 10 to 25
b) 20 to 30
c) 25 to 50

d) 50 to 100

18) Most frequency type of fault in DC system is _________


a) Converter internal fault
b) DC line fault
c) Commutation failure
d) Arc back and arc through
19) A system is said to be weak if SCR is _________
a) Less than 3
b) Less than 1
c) More than 5

d) 3 to 5

20) An HVDC line using same current density in conductors as that of DC will have
conductor losses ___________
a) About 3 times
b) 1.5 times
c) Same as that of DC
d) None of the above

______________
Set A

*SLRBB409*

-3-

SLR-BB 409

Seat
No.

B.E. (Electrical and Electronics Engineering) (Part II)


Examination, 2014
Elective II : HIGH VOLTAGE DC TRANSMISSION
Day and Date : Monday, 1-12-2014

Marks : 80

Time : 3.00 p.m. to 6.00 p.m.


Instructions : 1) Assume suitable data wherever necessary.
2) Non-programmable calculators are permitted.
SECTION I
2. Solve any four :

(54=20)

1) Explain EPC scheme in detail.


2) What is misfire ? Explain causes and remedies.
3) Explain the various applications of HVDC transmission.
4) Draw and explain the 12 pulse converter used in typical HVDC converter station.
5) Draw and explain over current protection.
6) Explain system control hierarchy.
3. Solve any two :

(102=20)

1) Compare EHVAC and HVDC transmission with suitable comments.


2) Explain the over voltage protection of converter in detail.
3) Explain with neat diagram the different types of DC links.

Set A

SLR-BB 409

-4-

*SLRBB409*

SECTION II
4. Solve any four :

(54=20)

1) Explain Static Var systems.


2) Explain the current margin method for protection of MTDC system.
3) Write a short note on SVC.
4) Explain DC Filters.
5) What are the types of MTDC systems ? Explain series type in detail.
6) Write a short note on protection of MTDC system.
5. Solve any two :

(102=20)

1) What are the potential applications of MTDC systems ? Explain each in detail.
2) Derive an expression for characteristic harmonics.
3) Explain the concept of reactive power compensation.
_____________________

Set A

SLR-BB 41

*SLRBB41*
S

T.E. (Civil) (Part I) Examination, 2014


ENGINEERING MANAGEMENT I (Old)
Day and Date : Friday, 12-12-2014
Time : 3.00 p.m. to 6.00 p.m.

Max. Marks : 100

Instructions : 1) Q. No. 1 is compulsory. It should be solved in first 30


minutes in Answer Book Page No. 3. Each question
carries one mark.
2) Answer MCQ/Objective type questions on Page No. 3
only. Dont forget to mention, Q.P. Set (A/B/C/D) on Top
of Page.
Duration : 30 Minutes

MCQ/Objective Type Questions

Marks : 20

1. Choose against only one correct answer option for the following multiple
choice questions :
(201=20)
1) Functional organization is invented by
a) Gilbreth
b) Taylor
c) Feyol

d) Mayo

2) Work is as natural as play of rest is assumption of


a) Theory W
b) Theory Z
c) Theory Y
d) Theory X
3) In ABC analysis least monitoring and control is required for
a) A class items b) B class items c) C class items d) All of these
4) In dynamic programming state is defined as
a) Point where decision is made
b) Information describing problem at each stage
c) Decision making rule
d) Optimal policy
5) Transportation problem can be solved if
a) No. of rows = no. of columns
b) No. of rows > no. of columns
c) No. of rows < no. of columns
d) All of these
6) Folding Back method is used for problems of
a) Transportation b) Games
c) Assignment
d) Decision tree
7) Functioning of human brain has inspired
a) Artificial neural network
b) Fuzzy logic
c) Genetic algorithm
d) Dynamic programming

P.T.O.

SLR-BB 41

-2-

*SLRBB41*

8) Monte Carlo simulation method is suitable for


a) Transportation problem
b) Queuing problem
c) Assignment problem
d) None of these
9) Linear programming deals with the optimization of a function of variable is
known as ________________
a) Subjective function
b) Objective function
c) Constraints
d) All of these
10) Marketing channels means
a) Intermediaries for distribution
b) Television channels
c) Costumer
d) Intermediaries for advertise
11) Queuing theory means
a) Optimization process
b) Descriptive process
c) Maximization process
d) Minimization process
12) The most important resources used in construction are
a) Men and money
b) Materials
c) Machines
d) All of these
13) Games without a saddle point require player to play ________________
a) Mixed strategies
b) Pure strategies
c) Dominated strategies
d) None of these
14) The Vogels approximation method is also called as ________________
a) Row minima method
b) Least cost method
c) Penalty method
d) None
15) Motion study is recommended for
a) Developing motion picture
b) Preventing indiscipline
c) Eliminating wasteful motions
d) None of these
16) Materials in material management consists of
a) Procurement process
b) Finishing process
c) Work in progress materials
d) All of these
17) The Monte-Carlo simulation used
a) Sampling technique
b) Decision technique
c) Mathematical technique
d) Both a) and b)
18) The biological process, of mutation has inspired
a) Artificial Neural Network
b) Genetic Algorithm
c) Fuzzy logic
d) Dynamics programming
19) The solution of decision tree is obtained by
a) Folding back method
b) Games theory
c) Laplace criteria
d) Dynamic programming
20) If possible outcomes of alternate course of action and probability of possible
outcome is not known, then the problem is of decision under
a) Certainty
b) Risk
c) Uncertainty
d) None
______________
Set A

*SLRBB41*
S

SLR-BB 41

-3-

T.E. (Civil) (Part I) Examination, 2014


ENGINEERING MANAGEMENT I (Old)
Day and Date : Friday, 12-12-2014
Time : 3.00 p.m. to 6.00 p.m.

Marks : 80
SECTION I

Q. No. 2 is compulsory. Attempt any two of Q. 3 to Q. 5.


2. a) Explain fourteen principles of classical management.

b) What are types of plans ? Explain each in short.

3. a) Explain the functions of management.

b) Explain briefly the functions of store.

4. a) Derive the formula for EOQ.

b) What are the methods of sales promotion ?

5. a) Explain theory X and theory Y.

b) What are the methods of sales promotion ?

SECTION II
Q. No. 6 is compulsory. Attempt any two out of Q. No. 7 to Q. No. 9.
6. a) Solve the following LPP using Simplex method.
Max Z = A B + 3C
S.t.
AB C
2A B
2A 2B + 3C
A, B, C 0

10
2
0


b) Determine IBFS of the following transportation model using VAM.


A

"

"

"

'

Set A

SLR-BB 41

*SLRBB41*

-4-

7. a) Determine the decision under Maximax and Maximin criteria for the following
decision process.
A

"

&

"

>

>

b) Find the value of the game

&

"

8. a) Write short note on :


1) Genetic Algorithm

2) Process of Dynamic Programming.

b) Find the duel of :


Min Z =
S.t.

6A + 3B
C + 6A 3B
4B C 3A
A, B, C 0


2
5

9. a) Explain in short about applications of waiting line theory.


b) What is assignment model ? Discuss.

8
5

_____________________

Set A

SLR-BB 410

*SLRBB410*
Seat
No.

Set

B.E. (Civil) (Part I) Examination, 2014


Elective I : ADVANCED DESIGN OF STEEL STRUCTURES
Day and Date : Thursday, 11-12-2014
Time : 3.00 p.m. to 6.00 p.m.

Total Marks : 100

Instructions : 1) Use of IS 800, IS 811, steel table and non-programmable calculator is


allowed.
2) Draw neat sketches where required and assume suitable data with clear
notification.
3) Q. No. I is compulsory. It should be solved in first 30 minutes in Answer
Book Page No. 3.
4) Answer MCQ/Objective type questions on Page No. 3 only. Dont forget
to mention, Q.P. Set (A/B/C/D) on Top of Page.
MCQ/Objective Type Questions
Duration : 30 Minutes

Marks : 20

I. Choose the correct answers :

20

1) When all beams and columns of a multistory steel building can carry bending moment, shear force
and axial thrust, such building is known as
a) Rigid frame
b) Building bent
c) Bent
d) All of above

2) For simply supported composite beams the deflection due to dead load, live load and impact load
should preferably not exceed ____________ of the span.
a) 1/600th of the span b) 1/800th of the span c) 1/900th of the span d) 1/1000th of the span

3) Truss girder bridges are also known as


a) Open web girder bridges
c) Triangulated framed structures

1
b) Lattice girder bridges
d) All of these

4) Which of the following are approximate method of analysis of multistory steel building subjected to
lateral loads is more accurate as compared to other approximate methods ?
a) Portal method
b) Cantilever method c) Factor method
d) None of these

5) Which of the following statement is wrong in the cantilever method of analysis of a multistory
steel building subjected to lateral loads ?
1
a) A point of contraflexure occurs at the centre of each beam
b) A point of contraflexure occurs at the centre of each column
c) The axial stress in each column of a storey is proportional to the horizontal distance of the column
from centre of gravity of all the columns of the storey under consideration
d) The total horizontal shear at each storey is distributed between the columns of that storey
6) Which of the following is not true with regards to the assumptions made in the plastic analysis ?
1
a) The deformation and strains are small
b) Plane section before bending remains plane after bending
c) Cross section of the member is symmetrical about an axis in the plane perpendicular to the plane of
bending
d) Stress-strain curve is an ideal elasto-plastic curve

P.T.O.

SLR-BB 410

*SLRBB410*

-2-

7) The basic design stress adopted for the design of light gauge steel members is
a) The permissible stress
b) The yield stress
c) The stress at limit of proportionality
d) The yield stress multiplied by 0.6

8) The approximate methods of analyzing the multi storey buildings are still adopted even when large
size and high speed computers are available because the approximate methods
a) Are easy and quick methods
b) Give the guidelines to select the initial size of members
c) Give fairly correct results
d) Give quite rough results
9) The shape or form factor is introduced in the design of compression members to represent the
effect of
a) Diagonal buckling
b) Post buckling
c) Local buckling
d) Vertical buckling

10) The plastic section modulus ZPY of a rectangular section of width b and depth d is given by
a)

bd2/6

b)

db2/6

c)

bd2/4

d)

db2/4

11) The thickness of gusset plate in light truss girder bridges is kept in between
a) 10 mm to 12 mm
b) 20 mm to 22 mm
c) 14 mm to 16 mm
d) None of these

12) If the number of possible plastic hinges are 4 and the degree of indeterminacy of the structure is
2 then the number of possible independent mechanism(s) n will be
a) 6
b) 4
c) 2
d) 1

13) The steel sheets used for light gauge steel members confirm to
a) IS : 1079 1973
b) IS : 1893 2002
c) IS : 456 2000
d) IS : 800 2007

14) Composite materials are classified based on


a) Type of matrix
c) Both

1
b) Size-and-shape of reinforcement
d) None

15) The number of possible independent mechanisms for a portal frame as shown in Figure 1 below is

Figure 1
a) 1

b) 2

c) 3

d) 4

16) Spacing of anchors when used alone shall not be less than ____ times the depth of the slab and
shall not be greater than ____ times depth of the slab.
a) 0.7, 2.0
b) 0.9, 2.5
c) 0.8, 3.0
d) 0.5, 2.0

17) The collapse load for the following beam is

Figure 2
a) 0.75 MP

b) Mp/I

c) 2 MP/I
______________

d) 1.5 MP/I

Set A

*SLRBB410*

-3-

SLR-BB 410

Seat
No.

B.E. (Civil) (Part I) Examination, 2014


Elective I : ADVANCED DESIGN OF STEEL STRUCTURES
Day and Date : Thursday, 11-12-2014
Time : 3.00 p.m. to 6.00 p.m.

Marks : 80

Instructions : 1) Answer any two questions from each Section.


2) Use of IS 800, IS 811, steel table and non-programmable calculator is
allowed.
3) Draw neat sketches where required and assume suitable data with clear
notification.
SECTION I
II. The effective span of a through type girder railway bridge is 52 m for a single lane B.G. track. The cross
girders a speed at 4.5 m apart. The stringers are spaced at 2.5 m between centre line. The weight of
stock and check rails are 0.5 kN/m and 0.4 kN/m. Sleepers are placed 0.55 m c/c. Weight of P.S.C.
sleepers is 25 kN/m3. The main girders are provided at 4.5 m apart, determine the design forces in top,
bottom, vertical and diagonal members of central panel. Design the bottom closed member and vertical
member. The bridge is to carry an equivalent U.D.L. line load of 3800 kN for B.M. and 4200 kN for shear
force.

20

III. Determine the shears and moments in columns and beams of a building frame with moment resisting
joints in Figure 1, by cantilever method. Assume the area of bottom storey column as 2 A and the area
of top storey column as A.

20

Figure 1

Set A

SLR-BB 410

-4-

*SLRBB410*

IV. a) Write on special design considerations for light gauge steel compression element.

b) Write on design procedure for light gauge beams.

c) A hot section 100 mm 50 mm 4 mm with lip 22 mm is to be used as a concentrically loaded


column of 3 m effective length. Determine the allowable load.

10

SECTION II
V. a) Design a cased column to carry a load of 1200 kN. The effective length of column is 4 m.

10

b) Design a composite foot bridge having clear width of 3 m and effective span of 15 m. The bridge is
to be designed foe live load of 4 kN/m2. Assume kerb 40 cm 30 cm and two steel girders at 1.8 m c/c
are provided.

10

VI. a) Explain different collapse mechanisms.

b) A fixed beam of span 4.8 m carries a uniform distributed load 5 kN/m on the right hand 4 m portion
of the beam. The load factor is 1.75 and the shape factor is 1.15. The yield stress is 250 MPa.
Calculate the section modulus of the beam and locate the position of plastic hinges.

12

VII. a) Write the design procedure of composite beams.

b) Determine the values of fully plastic moment of the frame, when loaded up to portal collapse. The
portal frame has vertical layer AB = 3 m, CD = 5 m and horizontal portion BC = 5.5 m. The end A is
fixed and is also fixed. A horizontal load of 30 kN towards B is acting at B and a vertical load of
45 kN acting downwards is a 2.2 m from B on BC portion. The plastic moment of the frame is uniform
throughout. Draw BMD also.

14

_____________________

Set A

SLR-BB 411

*SLRBB411*
Seat
No.

Set

B.E. (Civil) (Part I) Examination, 2014


(Elective I) OPEN CHANNEL AND RIVER HYDRAULICS
Day and Date : Thursday, 11-12-2014
Time : 3.00 p.m. to 6.00 p.m.

Max. Marks : 100

Instructions : 1) Q. No. 1 is compulsory. It should be solved in first 30 minutes


in Answer Book Page No. 3.
2) Answer MCQ/Objective type questions on Page No. 3 only.
Dont forget to mention, Q.P. Set (A/B/C/D) on Top of Page.
MCQ/Objective Type Questions
Duration : 30 Minutes

Marks : 20

1. Choose correct answer :


1) Steady flow in open channel exists when the
a) Flow is uniform
b) Depth does not change wtin time
c) Channel is friction less
d) Channel bed is not curved

2) In the uniform flow in a channel of small bed slope, the hydraulic grade line
a) Coincides with bed
b) Is considerably below the free surface
c) Is considerably above the free surface
d) Essentially coincides with free surface

3) The momentum correction factor is given

a)

1
3

V A

3
V dA

b)

1
VA

VdA

c)

1
2

V A

2
V dA

d)

1
3

V A

2
V dA

4) The difference between total head line and piezometric head line represents
a) Velocity head
b) Pressure head
c) Elevation of the bed of the channel d) Depth of flow

5) The momentum equation in X direction Fx = Q ( Vx 2 Vx1) has the


assumption the flow is
a) Steady
b) Unsteady
c) Uniform
d) Frictionless

6) The two alternate depth in 4.0 m wide rectangular channel are 3.86 and 1.0 m.
The discharge in the channel in m3/sec is
a) 15
b) 1.5
c) 7.76
d) 31.0

7) For a traingular channel of side slopes M horizontal :1 vertical the froude number is
given by F =
1
a)

M
gy

b)

V
2gy

c)

V 2
gy

d)

V
gy

P.T.O.

SLR-BB 411

*SLRBB411*

-2-

8) Standard step method aims to solve


a) Differential equation of GVF
c) Bernoulli equation

1
b) Differential energy equation of GVF
d) Momentum equation

9) The hydraulic jump is a phenomenon


a) In which water surface connects the alternate depth
b) Which occours only in frictionless channel
c) Which occours only in rectangular channel
d) None

10) The specific force is constant


a) In all frictionless channels irrespective of magnitude of the longitudinal slope
b) In horizontal frictionless channel of any shape
c) In all horizontal channel of any shape
d) In any open channel

11) A current meter measures the velocity of flow if it is held


a) At bottom surface of channel
b) At the surface of channel
c) At the centroid of channel section
d) At any point within the cross section

12) The channel velocity given by Mannings formula is affected by


a) Hydraulic mean radius
b) Slope of river bed
c) Roughness of bed and sides
d) All of the above

13) River plains are made of


a) Alluvim
b) Red soils

14) Silting of reservior


a) Reduces efficiency of dams
c) Raises reservior water level

c) Black soil

d) None
1

b) Reduces storage capacity


d) None

15) River training works server the following purposes


a) Increase or decrease river discharge
b) Protect the river bed and banks
c) Direct the river flow in desired direction
d) Protect the surrounding land

16) Extreme condition of the Meanders is called


a) Cut-off
b) Spur
c) Island

1
d) None

17) A spur pointing upstream forms a still pocket against its upstream face. It has an
effect of
a) Repelling the flow from bank
b) Attracting the flow towards the bank
c) Deflecting the flow at the site
d) None
18) The dimensions of pressure gradient in fluid flow are
b) ML3 T 2
c) ML2T 2
a) ML1T2

d) M1L3T 2

19) Distorted models are required to prepared for


a) Rivers
b) Dams across very wide rivers
c) Harbours
d) All

______________

1
1

Set A

*SLRBB411*

-3-

SLR-BB 411

Seat
No.

B.E. (Civil) (Part I) Examination, 2014


(Elective I) OPEN CHANNEL AND RIVER HYDRAULICS
Day and Date : Thursday, 11-12-2014
Time : 3.00 p.m. to 6.00 p.m.

Marks : 80

Instruction : All questions are compulsory. Draw neat sketches wherever


it is necessary.
SECTION I
2. Attempt any four :

24

a) Derive the equation for Chezys formulae.


b) Find the rate of flow and conveyance K for rectangular channel 6 m wide for
uniform flow at a depth of 1.5 m. The bed slope of channel is 1:1000 and
C = 50. Also state that the flow is tranquil or rapid.
c) Derive the equation for modified Gradually Varied Flow (GVF).
d) What is specific energy curve ? Draw a neat sketch and show all details.
e) An overflow spillway is 40.0 m high. At the design head of 2.5 m over spillway, find
the sequent depths and energy loss in hydraulic jump formed on horizontal apron
at the toe of spillway. Neglect energy loss due to flow over spillway face (assume
Cd = 0.74).
3. Any two :

16

a) What do you mean by most economical section of an open channel and prove
that, for Trapezoidal channel ?
Half of top width = length of one of the sloping side.
b) A spill way discharges flood flow at a rate of 9m3/s/m. At the downstream horizontal
apron depth of flow is 0.70 m. What tail water depth is needed to form hydraulic
jump ? If jump is found find its type, length and energy loss as a percentage of the
initial energy and profile.
c) Derive an expression for the depth of flow after the jump in a rectangular channel
in terms of depth before the jump. (Draw neat sketch).

Set A

SLR-BB 411

-4-

*SLRBB411*

SECTION II
4. Attempt any two :

16

a) Design a regime channel for a discharge of 60 cumecs and silt factor 1.15. Using
Laceys theory.
b) What is sediment ? How it is transported in stream ? Write an equation giving the
relation between sediment transport and stream flow.
c) I) Define similitude and write types of similarities.
II) A model of water meter is tested in a pipe of 100 mm diameter. The discharge
was 50 lit/sec and pressure difference was 0.1 N/mm2. What will be the discharge
in a pipe 500 mm diameter and what will be the pressure drop ?
5. Attempt any four :

24

a) Derive Reynolds law for model and state giving examples where it can be used.
b) Explain the following terms :
1) Suspended load
2) Regime channel
3) Threshold motion of the sediment.
c) Explain the term River training and what are its types ? (Draw sketches).
d) Define stream gauging and draw a neat sketch of current meter and explain its
working.
e) Design an irrigation channel to carry 50 cumecs discharge. The channel is laid at
a slope of 1/4000. The critical velocity ratio is 1.10. Use Kutters roughosity

coefficient as 0.023.

_____________________

Set A

SLR-BB 412

*SLRBB412*
S

T.E. (Mechanical) (Part I) Examination, 2014


FLUID MACHINERY AND FLUID POWER (New) (Elective I)
Day and Date : Wednesday, 10-12-2014

Max. Marks : 100

Time : 3.00 p.m. to 6.00 p.m.


Instructions : 1) Draw neat sketches wherever necessary.
2) Use of non-programmable calculators is permitted.
3) Q. No. 1 is compulsory. It should be solved in first 30 minutes
in Answer book Page No. 3.
4) Answer MCQ/Objective type questions on Page No. 3 only.
Dont forget to mention, Q.P. Set (A/B/C/D) on Top of Page.
MCQ/Objective Type Questions
Duration : 30 Minutes

Marks : 20

1. Choose the correct answer :

20

1 and 2 statements in column I may have one or more matches with statements
in column II. (Each question 3 marks).
1)

Column I

Column II

a) Pelton wheel

p) Ns = 8.5 to 51 rpm

b) Francis turbine

q) Ns = 30 to 51 rpm

c) Kaplan turbine

r) Ns = 52 to 255 rpm

s) Ns = 256 to 860 rpm


2)

Column I

Column II

a) FRL unit

p) Pilot operated

b) Air brake

q) Pneumatic system

c) Pressure control valve

r) Automobiles

s) Hydraulic system
P.T.O.

SLR-BB 412

*SLRBB412*

-2-

Questions 3 to 5 are multiple correct type questions (2 marks each) :


3) Weight loaded accumulator provides
a) high pressure oil
b) nearly constant pressure for full stroke of piston
c) varying pressure for different positions of piston
d) low pressure oil

4) Francis turbine is
a) impulse turbine
c) high head turbine

b) reaction turbine
d) mixed flow turbine

5) Direction control valves are used to


a) control pressure of fluid
c) direct fluid flow

b) control fluid flow


d) direct and control fluid flow

Q. No. 6 to 13 with single correct answers (1 mark each) :


6) Forces on the buckets of Pelton turbine are calculated by
a) continuity equation
b) momentum equation
c) energy equation
d) force balance

7) Specific speed of a centrifugal pump is given by

a)

b)

c)

d)

8) Re-heat is related to
a) Francis turbine b) Kaplan turbine c) Gas turbine

d) Pelton wheel

9) Maximum efficiency of Pelton wheel is


a)
1

b)
1

1
c)
1

d)
1

10) Pressure rating for brass piping is


a) 250 bar
b) 200 bar

c) 2500 bar

d) 125 bar

11) Mostly in railways following brake is used


a) pneumatic brake system
b) hydraulic brake system
c) mechanical brake system
d) none of the above

12) Speed control circuit is


a) regenerative circuit
c) counterbalance circuit

13) Mostly FRL is used in


a) compressor outlet
c) pump

b) meter in circuit
d) all of the above
b) compressor inlet
d) cylinder
______________

Set A

*SLRBB412*
S

-3-

SLR-BB 412

T.E. (Mechanical) (Part I) Examination, 2014


FLUID MACHINERY AND FLUID POWER (New) (Elective I)
Day and Date : Wednesday, 10-12-2014

Marks : 80

Time : 3.00 p.m. to 6.00 p.m.


Instructions : 1) Question No. 2 and 5 are compulsory.
2) Solve any one question from each Section out of the remaining
questions.
3) Draw neat sketches wherever necessary.
4) Use of non-programmable calculators is permitted.
SECTION I
2. Attempt any five of the following :

(45=20)

a) Explain governing of Pelton wheel.


b) Derive Eulers equation of work done for turbines.
c) Why the multistaging of centrifugal pump is required ? Explain series type multistage
pump.
d) What are the different methods of improving thermal efficiency of gas turbine ?
Explain regeneration method by showing it on T-S diagram.
e) Explain different heads and efficiencies for a centrifugal pump.
f) Explain the significance and types of draft tubes.
g) Derive an expression for air-standard efficiency of an ideal Joules cycle. Show it
on P-V and T-S diagrams.
3. a) Draw the different types of velocity triangles if the inlet and outlet vane angles are
1) Equal to zero
2) Less than 90
3) Equal to 90
4) Greater than 90.
Label the diagrams correctly.

Set A

SLR-BB 412

*SLRBB412*

-4-

b) A centrifugal pump has an impeller 0.5 meter outer diameter and when running at
600 rpm discharges water at the rate of 8000 1pm, against a head of 8.5 meter.
The water enters the impeller without whirl and shock. The inner diameter is 0.25
meter and the vanes are set back at outlet at an angle of 45 and the area of flow
which is constant from inlet to outlet of the impeller is 0.06 m2. Determine
i) The manometric efficiency of the pump
ii) The vane angle at inlet
iii) The list speed at which the pump commences to work.

c) A 137 mm diameter jet issuing from nozzle impinges on the bucket of Pelton wheel
and the jet is deflected through an angle of 165 by the buckets. The head available
at the nozzle is 400 meters. Assuming co-efficient of velocity as 0.97, speed ratio
as 0.46 and reduction in relative velocity while passing through the buckets as
15%, find
i) The force exerted by the jet on buckets in tangential direction
ii) The power developed.

4. a) Explain in detail with the help of T-S diagram a closed cycle gas turbine plant.
What are the merits and demerits of it ?

b) A Francis turbine with overall efficiency of 75% is required to produce 148.25 kW.
It is working under a head of 7.62 m. The peripheral velocity is 0.26*
radial velocity of flow at inlet is 0.96*
C

and

. The wheel runs at rpm and the hydraulic

losses in the turbine are 22% of the available energy. Assuming radial discharge
determine
i) The guide blade angle
ii) The wheel vane angle
iii) Diameter of wheel at inlet
iv) Width of wheel at inlet.
c) Define and derive the expression for a specific speed of a turbine.

8
6

Set A

*SLRBB412*

-5-

SLR-BB 412

SECTION II
5. Attempt any five of the following :

(54=20)

a) Explain the importance of seals. Classify seals.


b) Draw symbols of DCV with ports and two positions and pressure compensated
adjustable flow control valve with by pass.
c) Explain 3/2 seat type DCV in pneumatics.
d) Explain advantages of fluid power.
e) Explain regenerative circuit.
f) Explain time delay valve.
g) Explain with neat sketch hydraulic intensifier.
6. a) Explain construction and working of pressure compensated flow control valve.

b) Define actuator. Explain different types of actuators.

c) Explain meter in circuit in speed control.

7. a) Draw a neat block diagram of pneumatic system with its elements.

b) Explain counter balancing circuit.

c) Draw a neat sketch of hydraulic and pneumatic clamping system.

_____________________

Set A

Set A

SLR-BB 413

*SLRBB413*
Seat
No.

Set

T.E. (Mech.) (Part I) Examination, 2014


MATERIAL HANDLING SYSTEM (New)
(Professional Elective I)
Day and Date : Wednesday, 10-12-2014
Time : 3.00 p.m. to 6.00 p.m.

Max. Marks : 100

Instructions : 1) Figures to the right indicate full marks.


2) Draw neat diagram where necessary.
3) Q. No. 1 is compulsory. It should be solved in first 30 minutes
in Answer book Page No. 3. Each question carries one mark.
4) Answer MCQ/Objective type questions on Page No. 3 only.
Dont forget to mention, Q.P. Set (A/B/C/D) on Top of Page.
MCQ/Objective Type Questions
Duration : 30 Minutes
1. Objective questions :

Marks : 20
(201)

1) The main objective of the material handling system is to __________ material


movement.
a) reduce
b) Maximum
c) Medium
d) Light movement
2) Assembly chart shows ______________
a) Layout of the product
b) Sequences in which components are assembled
c) Figures of components d) None of these
3) To transport ferrous material vertically, ______________ conveyor is used.
a) Wheel conveyors
b) Chain conveyors
c) Magnetic conveyors
d) Chute conveyors
4) The equipment used to handle material at a single location so that, it is in the
correct position for Machining ______________
a) Transport equipment
b) Storage equipment
c) Positioning equipment
d) None of these
5) Cranes are used to move material over ______________ paths.
a) Fixed
b) Diverted
c) Variable
d) None of these
6) Hoisting equipments works in conjection with ______________ and workstation
cranes.
a) Roller
b) Industrial trucks
c) Elevator
d) Overhead crane
P.T.O.

SLR-BB 413

-2-

*SLRBB413*

7) For ________ type material flow having less difficulty in returning empty containers.
a) Straight line
b) Inverted
c) L shaped
d) U shaped
8) Interlocks be available in the controls of _____________ safety, so that worker
can safely service the area.
a) Electrical
b) Robotics
c) Chemical
d) Radiation
9) The material handling equation consists of the material characteristics, the
move requirement and ______________
a) The method capabilities b) Human capacity c) Load
d) Gravity
10) Storage equipments are used for ______________ material over a period of time.
a) Catching
b) Holding
c) Hanging
d) None of these
11) The handling of a quantity designed to be treated as a single mass __________
a) Unit load
b) Unit mass
c) Unit wt.
d) None of these
12) For automated material handling system ______________ materials are required.
a) Low volume
b) Medium volume c) High volume
d) None of these
13) Equipment evaluation sheet consists of equipment characteristics,
utilization and ______________
a) Safety
b) Vendor characteristics c) Flexibility
d) Unit load
14) In power equipment automatic correction according to ______________
a) Gravity
b) Manual
c) Signal
d) Equipment
15) From to chart gives the information about ______________
a) Density of moves between depts. b) Equipment
c) Selection procedure
d) None of these
16) Symbol represents for
a) Operation
b) Store
c) Inspection
d) Movement
17) Flow dia, is used to study ______________
a) Back tracking
b) Sequence of operations
c) m/c maintenance
d) Operator skill
18) Procedure chart gives the information about ______________
a) Inventory
b) Inspection c) Communication d) Material requirement
19) As the distance or time factor increases, the cost per unit of product
Handled ______________
a) Increases
b) Decreases
c) Same
d) None of these
20) There are three basic characteristics of material handling, picking up the
load, transporting the load and ______________
a) Catching the load
b) Rush the load
c) Setting the load down
d) Up the load
______________
Set A

*SLRBB413*

-3-

SLR-BB 413

Seat
No.

T.E. (Mech.) (Part I) Examination, 2014


MATERIAL HANDLING SYSTEM (New)
(Professional Elective I)
Day and Date : Wednesday, 10-12-2014

Marks : 80

Time : 3.00 p.m. to 6.00 p.m.


Instructions : 1) Figures to the right indicate full marks.
2) Draw neat diagram where necessary.
SECTION I
Solve any two questions :
2. a) Define material handling system. And explain its principles.

b) Explain the general characteristics of Crane and describe with Fig. any two
types of Cranes.
c) Explain objective, benefits and scope of the material handling system.
3. a) Compare conventional and CIMS material handling system.

7
6
7

b) What is meant by material handling equipments and give its classification.

c) Explain concept of Unit load containerization and palletisation.

4. Write a short notes (any three) :


a) Hoisting equipments.

b) Industrial robots.

c) Types of Belts.

d) Storing equipments.

7
Set A

SLR-BB 413

-4-

*SLRBB413*

SECTION II
Solve any two questions :
5. a) Explain in brief procedure for selection of material handling equipment.

b) Describe with figure material handling equation.

c) Explain in brief material handling safety and training.

6. a) Explain selection criteria for material handling equipment.

b) Explain different material handling systems.

c) Explain with Fig. assembly chart.

7. Write a short notes (any three) :


a) Types of material flow.

b) Equipment evaluation sheet.

c) From To chart.

d) Selection of material handling equipments in Foundry.

_____________

Set A

SLR-BB 414

*SLRBB414*
S

S.E. (Electronics Engg.) (Part I) (Old) Examination, 2014


DIGITAL TECHNIQUES
Day and Date : Tuesday, 2-12-2014
Time : 10.00 a.m. to 1.00 p.m.

Max. Marks : 100

Instructions : 1)
2)
3)
4)

All questions are compulsory.


Figures to the right indicate full marks.
Assume suitable data wherever necessary.
Q. No. 1 is compulsory. It should be solved in first 30
minutes in Answer book Page No. 3. Each question
carries one mark.
5) Answer MCQ/Objective type questions on Page No. 3
only. Dont forget to mention, Q.P. Set (A/B/C/D) on Top
of Page.
MCQ/Objective Type Questions

Duration : 30 Minutes
1. Choose the correct answer :

Marks : 20
(201=20)

1) The NOR-NOR realization is equivalent to __________


a) AND-OR realization
b) OR-AND realization
c) AND-OR-NOT realization
d) AND-AND realization
2) Multiplexer is represented by
b) 1 2n
c) 2n n
d) n 2n
a) 2n 1
3) A de-multiplexer is _______________ implementation.
a) only Ex-OR
b) AND-OR
c) Only AND
d) OR-AND
4) The statement x(x + y) = x * y represent _______________
a) Associative law
b) DeMorgans theorem
c) Distributive law
d) Commutative law
5) The given binary number is 10000, its equivalent BCD number is
a) 0001 0000
b) 0000 0110
c) 0001 0001
d) 0001 0110
6) The expression f = m0 + m2 + m6 + m7 is equivalent to ____________
a) f = m3 * m5 * m6 *m7
b) f = M0 + M1 + M2 + M4
c) f = M1 * M3 * M4 * M5
d) f = M1 + M3 + M4 + M5
7) The _______________ is a Boolean expression containing OR terms with
one or more literals each.
a) sum of products
b) standard POS
c) products of sums
d) standard SOP

P.T.O.

SLR-BB 414

-2-

*SLRBB414*

8) ______________ code is an examples of non-weighted codes.


a) BCD
b) Gray
c) 2421
d) 8, 4, 2, 1
9) ______________ is the number of outputs a gate can drive.
a) Fan-out
b) Fan-in
c) Noise Margin d) None of above
10) Simplification of the Boolean expression (x + y) (x + y) is
a) y
b) xy
c) xy
d) x
11) State reduction gives
a) reduction in number of flip-flops b) number of flip-flop remains same
c) (a) or (b)
d) none of the above
12) Which one of the following is used as toggle switch ?
a) D flip-flop
b) T flip-flop
c) SR flip-flop
d) None of the above
13) The output Qn of JK flip-flop is 1. It changes to 0 when a clock pulse is
applied. The input Jn and Kn are respectively.
a) 1 and X
b) 0 and X
c) X and 0
d) X and 1
14) The characteristic equation of S flip-flop is given by
a) Q+ = S + RQ b) Q+ = SQ + R c) Q+ = S + RQ d) Q+ = SQ + R
15) A MOD-11 counter has _______________ different states.
a) 11
b) 12
c) 10
d) 15
16) The initial content of 4-bit serial-in-parallel out, right shift, shift register shown
in figure is 0110. After three clock pulses are applied contents of shift register
will be

17)

18)
19)
20)

a) 1010
b) 0101
c) 0011
d) 1011
A 4-bit pre-settable down counter has preset input 0111. The present operation
takes place as soon as counter becomes minimum 0000. The modulus of
counter is
a) 6
b) 5
c) 4
d) 8
When a flip-flop is reset, its output will be
a) Q = 0, Q = 0
b) Q = 1, Q = 0 c) Q = 0, Q = 1 d) Q = 1, Q = 1
The number of states in 6 bit Johnsons counter are
a) 8
b) 4
c) 12
d) 10
A flip-flop is used to store
a) one bit of information
b) two bits of information
c) one byte of information
d) one nibble of information
______________
Set A

*SLRBB414*
S

SLR-BB 414

-3-

S.E. (Electronics Engg.) (Part I) (Old) Examination, 2014


DIGITAL TECHNIQUES
Day and Date : Tuesday, 2-12-2014
Time : 10.00 a.m. to 1.00 p.m.

Marks : 80

Instructions : 1) All questions are compulsory.


2) Figures to the right indicate full marks.
3) Assume suitable data wherever necessary.
SECTION 1
2. Attempt any four :
(44=16)
a) Explain design of full subtractor using half-subtractor in details with proof.
b) Design a 4 1 multiplexer using NAND gates. Implement a full subtractor
with 4 1 multiplexers.
c) Construct a 16 1 multiplexer with 4 1 multiplexers. Explain with truth
table.
d) Simplify the following Boolean function F, together with the dont-care conditions
d. F(A, B, C, D) =
(1, 3, 8, 10, 15) and d =
(0, 2, 9). Implement using
AND-OR gates only.
e) Design and implement 1-bit comparator circuit using basic gates.


3. a) Design a combinational circuit with 3 inputs (x, y and z) and 2 outputs (y1, y2).
The output y1 is equal to 1 if the input variables have more 1s than 0s. The
output y1 is 0 otherwise. The other output y2 is equal to 1 when the binary
value of the inputs is an even number. Design the circuit by finding the circuits
truth table, Boolean equation, and a logic diagram.

b) Attempt any two :


(82=16)
i) An XOR gate is a 2 input gate whose output is 1 if and only if exactly one
of its inputs is 1. Write truth table. Implement the gate using (a) AND-OR
(b) OR-AND (c) NAND-NAND (d) NOR-NOR implementation.
ii) Design a combinational circuit that converts a four-bit binary code to a
four-bit Gray number.
iii) Design a 4-input priority encoder and implement using basic gates.
Set A

SLR-BB 414

-4-

*SLRBB414*

SECTION 2
4. Attempt any four :

(44=16)

a) Explain a 4-bit ring counter using shift register.


b) Show that JK flip-flop can be used as T flip-flop.
c) Derive the characteristic equation for D flip-flop.
d) Write a short note on APL.
e) Design a MOD-4 ripple counter using T flip-flop.
5. a) Design a decade asynchronous counter using JK flip-flop. Draw neat circuit
diagram and waveforms.
b) Attempt any two :

(82=16)

i) Design a sequential circuit specified by state diagram shown using D flip-flop.

ii) What is a shift register ? Draw and explain different types of shift registers.
iii) Explain Melay machine with example. Compare Moore and Melay machine.
_____________________

Set A

SLR-BB 415

*SLRBB415*
S

S.E. (E and TC) (Part I) (Old) Examination, 2014


DATA STRUCTURES
Day and Date : Tuesday, 2-12-2014
Time : 10.00 a.m. to 1.00 p.m.

Max. Marks : 100

Instructions : 1) Figures to the right indicate full marks.


2) Q. No. 1 is compulsory. It should be solved in first 30
minutes in Answer Book Page No. 3. Each question
carries one mark.
3) Answer MCQ/Objective type questions on Page No. 3
only. Dont forget to mention, Q.P. Set (A/B/C/D) on Top
of Page.
Duration : 30 Minutes

MCQ/Objective Type Questions

Marks : 20

1. Choose the correct answer :


1) Empty condition of stack is
a) Top = 1
b) front = 1
c) rear = 1
d) front = = rear
2) After doing enq operation, following variables get affected
a) rear = rear + 1
b) front = front + 1
c) top = top + 1
d) none of the above
3) The pointer pointing to first node of linked list is called as
a) Internal pointer
b) external pointer
c) traversal pointer
d) none of the above
4) Queues in which we can insert item or delete item from any position based
on some property is known as
a) Deque
b) Circular Queue
c) Priority Queue
d) None of the above
5) The last node link of circular list is holding the value _________
a) Null
b) Integer
c) Character
d) Address of first node
6) The following functions are present in alloc.h file
a) printf
b) malloc ( )
c) calloc ( )
d) both b), c)
7) A vail is _______ pointer of available linked list
a) External
b) Traversal
c) Internal
d) None of the above
P.T.O.

SLR-BB 415

*SLRBB415*

-2-

8) A tree is called a binary tree if each node have


a) max. 3 children
b) max. 2 children
c) max. 4 children
d) max .1 children
9) The getnode operation of linked list uses
a) Insert first algorithm
b) Delete first algorithm
c) Insert second algorithm
d) None of above
10) Josephus problem can be solved by using
a) Singly linked list
b) Doubly linked list
c) Circular linked list
d) None of above
11) Searching by dividing the array into two equal parts is known as
a) Sequential
b) Binary
c) Linear
d) None of the above
12) Recursion function can be implemented using stacks
a) True
b) False
13) One way to represent graph in computer memory is
a) Stack
b) Queue
c) Adjancy matrix d) None of the above
14) ___________is not a collision resolution technique.
a) Linear probing
b) Double hashing
c) Folding method
d) None of the above
15) Sort can be classified as internal if
a) The record that it is sorting are in main memory
b) The record that it is sorting are in auxiliary memory
c) All of the above
d) None of the above
16) The edge in graph contains
a) Non-weighted b) Weighted
c) Isolated
d) Cyclic
17) Two records that can occupy same position. This situation is known as
a) Rehashing
b) Hash collision c) Chaining
d) None of the above
18) Quick sort is also known as
a) Fast sort
c) Partition exchange sort

b) Insertion sort
d) None of the above

19) DFS traversal uses


a) Stack
b) Queue
c) All of the above
d) None of the above
20) Adjacency list representation of a graph requires advance knowledge of no.
of nodes.
a) True
b) False
______________
Set A

*SLRBB415*
S

-3-

SLR-BB 415

S.E. (E and TC) (Part I) (Old) Examination, 2014


DATA STRUCTURES
Day and Date : Tuesday, 2-12-2014

Marks : 80

Time : 10.00 a.m. to 1.00 p.m.


Instructions : 1) All questions are compulsory.
2) Figures to the right indicate full marks.
SECTION I
2. Answer any four :

(54=20)

1) Explain the concept of treenode and getnode with respect to avail list.
2) Write a note on priority queue .
3) Write an algorithm to add two polynomials.
4) Write ENq and DEq functions of circular queue.
5) Suppose following values are inserted into binary search tree, in the order
given, 10, 15, 12, 7, 8, 18, 6, 20. Draw the resultant binary search tree
3. a) Solve any one :

(110=10)

1) Implement stack using dynamic memory allocation.


2) Write a program of queue using array, with ENq, DEq, display, functions.
b) Define tree. Explain the following terms with example.

(110=10)

i) Root
ii) Parent
iii) Leaf nodes
iv) Interior nodes
v) Siblings.
Set A

SLR-BB 415

-4-

*SLRBB415*

SECTION II
4. For following graph, draw its linked representation and matrix (adjacent matrix)
representation and also find its in degree and out degree.

5. Attempt any four :

10

(54=20)

a) Explain collision resolution with open addressing.


b) What is recursion ? Explain advantages and disadvantages of recursion.
c) Sort the following using merge sort 12, 44, 32, 65, 15, 24, 39, 19.
d) Explain following terms related to graph.
1) Graph
2) Weighted graph
3) Directed graph
4) Cycle
5) Adjacency matrix.
e) What is the purpose of big O notation ?
6. Attempt any one :

(110=10)

a) Sort following using :


48 77 33 13 69 28 56 98
1) Bubble sort
2) Selection sort.
b) What is hashing ? Explain hash collision.
_____________________

Set A

SLR-BB 416

*SLRBB416*
Seat
No.

Set

S.E. (CS & E) (Part I) Examination, 2014


DATA STRUCTURE I (Old)
Day and Date : Tuesday, 2-12-2014
Time : 10.00 a.m. to 1.00 p.m.

Max. Marks : 100

Instructions : 1) Q. No. 1 is compulsory. It should be solved in first 30 minutes


in Answer Book Page No. 3. Each question carries one mark.
2) Answer MCQ/Objective type questions on Page No. 3 only.
Dont forget to mention, Q.P. Set (A/B/C/D) on Top of Page.
MCQ/Objective Type Questions
Duration : 30 Minutes

Marks : 20

1. Choose the correct answer :


1) The declaration in (*P) (10) is
a) Pointer to array
c) Pointer to function

20
b) Array of pointer
d) None of the above

2) The number of arguments used in realloc( ) is


a) 0
b) 1
c) 2
3) In ascending priority queue only
a) Smallest
c) any item can be removed

d) 3

b) largest
d) none

4) Can we used string constants in switch cases ?


a) yes
b) no
5) Which of the following is a FIFO list ?
a) Priority queue b) List
c) Stack

d) Queue

6) A pointer to character accesses the following bytes of memory


a) 1
b) 2
c) 4
d) 8
7) In evaluation of postfix expression stack is used to store
a) operands
b) operators
c) both a) and b) d) none
8) Recursive procedure are implemented by
a) queue
b) stack
c) linked list

d) string
P.T.O.

SLR-BB 416
9) Free( ) function is used to
a) Release memory for node
c) To unlink first and last node

*SLRBB416*

-2-

b) To unlink the node


d) None of the above

10) Which of the following statement terminate the execution of loop ?


a) else
b) break
c) return
d) goto
11) Linked list is
a) sequential storage
c) both a) and b)

b) random storage
d) none of the above

12) The number of parameters passed to Fopen( ) function


a) 1
b) 2
c) 3

d) no parameters

13) A static variable is initialized to the value


a) Garbage
b) 0
c) 1

d) 2

14) The contents of the file can be modified when it is opened in following mode
a) r
b) w
c) a+
d) w+
15) The rewind function returns a value which is of the data type ?
a) void
b) int
c) char
d) float
16) The following operator cannot be applied on pointers
a) addition
b) multiplication c) subtraction
17) Is n++ is fast or n = n + 1 is fast ?
a) n++ is fast
c) all of the above

d) decrement

b) n = n + 1 is fast
d) none of these

18) Can we apply modulus operator to float type operands ?


a) Yes
b) no
19) Two functions can have the same name.
a) True
b) False
20) Giving a structure tag is a must.
a) True
b) False
______________

Set A

*SLRBB416*

-3-

SLR-BB 416

Seat
No.

S.E. (CS & E) (Part I) Examination, 2014


DATA STRUCTURE I (Old)
Day and Date : Tuesday, 2-12-2014
Time : 10.00 a.m. to 1.00 p.m.

Marks : 80

Instructions : 1) Each Section carries 40 marks.


2) Attempt all questions from each Section.
SECTION I
2. Solve any four :

(45=20)

a) What is pointer ? Explain pointer to structure with example.


b) Explain declaration and initialization of string.
c) Define Array, Stack, Priority queue, String, Static variable.
d) What do you understand by linear data structure ? Explain with example.
e) Write a C program to display contents of file on screen.
f) What is recursion ? Explain advantage and disadvantage.
3. Solve any one :

10

a) Explain array of pointer and pointer to array with example.


b) What is a string ? Write an algorithm to compare two strings without using
library functions.
4. Explain dynamic memory allocation. Compare it with static allocation with example. 10
SECTION II
5. Answer any four from the following :

(45=20)

a) What is priority queue ? Explain applications of priority queue.

b) Compare stack and queue.

5
Set A

SLR-BB 416

-4-

*SLRBB416*

c) Consider the stack of colours :


STACK : Red, Green, Blue, Yellow, ......
Describe the stack as the following operations take place :
i) Push (STACK, Orange)
ii) Pop (STACK, ITEM)
iii) Pop (STACK, ITEM).

d) Write a C function to insert new node at the end of singly linked list.

e) Define queue. Explain primitive operations on linear queue.

f) What is circular linked list ? Explain types of circular linked list.

6. a) Define circular queue. Explain need for circular queue and write
C functions for :
i) Inserting new element in circular queue
ii) Displaying contents of circular queue.

10

OR
b) Write a C program to convert given infix form expression into its equivalent
postfix form expression.
7. Write a C program for implementation of stack using linked list.
______________

10
10

Set A

SLR-BB 417

*SLRBB417*
Seat
No.

Set

S.E. (CSE) (Part I) (Old) Examination, 2014


SWITCHING THEORY AND LOGIC DESIGN
Day and Date : Thursday, 4-12-2014
Time : 10.00 a.m. to 1.00 p.m.

Total Marks : 100

Instructions : 1)
2)
3)
4)

Figures to the right indicate full marks.


Draw neat diagrams.
Assume suitable data if necessary.
Q. No. 1 is compulsory. It should be solved in first 30 minutes in
Answer Book Page No. 3. Each question carries one mark.
5) Answer MCQ/Objective type questions on Page No. 3 only. Dont
forget to mention, Q.P. Set (A/B/C/D) on Top of Page.
MCQ/Objective Type Questions

Duration : 30 Minutes

Marks : 20

1. Choose the correct answer :

(201=20)

1) The OR operation can be produced with __________


a) One NAND gates b) two NAND gates c) three OR gates

d) two OR gates

2) Max term corresponding to the decimal number 15 is _________


a) A B C D

b) A B C D

c) A + B + C + D

d) A + B + C + D

3) Simplified form or Boolean expression ( A + B + A B)C is ________


a) 1

b) 0

4) A three variable K-map has __________


a) 8 minterms
b) 4 minterms

c) C

d) C

c) 16 minterms

d) 32 minterms

5) A combinational circuit has inputs A, B and C and its K-map is shown below. The O/P
of the circuit is given by

a) ( AB + A B )C

b) ( AB + A B )C

c) ( A B C )

d) A B C

6) To perform multi bit addition ___________ is used.


a) half adder
b) half subtractor c) full adder

d) full subtractor

7) A MOD-13 counter must have


a) 13 flip flops
b) 3 flip flops

d) 2 flip flops

c) 4 flip flops

P.T.O.

SLR-BB 417

*SLRBB417*

-2-

8) In ripple counter, if Q output of flip flop is fed to clock input of next stage flip flop, this
counter results in ___________
a) up counter
b) down counter
c) up-down counter
d) none of these
9) A _________ shift register can be used as SISO, SIPO, PISO, PIPO shift register.
a) bi-directional
b) universal
c) unidirectional
d) none of these
10) The O/P Qn of a J-K flip flop is 0. It change to 1 when a clock pulse is applied. The input
Jn and Kn are respectively
a) 1, X
b) 0, X
c) X, 0
d) X, 1
11) For decoder circuit 8 : 3, the number of select lines
a) 0
b) 2
c) 3

d) 4

12) In PLA, letter A stands for __________


a) Array
b) Access

d) None of these

c) Ahead

13) Which of the following device has more number of inputs and single output ?
a) demultiplexer
b) decoder
c) multiplexer
d) none of these
14) Associate memory is ____________
a) EPROM
b) RAM

c) ROM

d) CAM

15) A memory that loses it contents when power is turned off __________
a) static memory
b) dynamic memory
c) volatile memory
d) non volatile memory
16) IC 74148 is ___________
a) multiplexer
b) demultiplexer

c) decoder

17) In register level design __________ IC technology is used.


a) SSI
b) MSI
c) LSI

d) encoder
d) VLSI

18) ROM contains ____________


a) Programmable AND and programmable OR array
b) Programmable AND and non programmable OR array
c) Non programmable AND and programmable OR array
d) Non programmable AND and non programmable OR array
19) A full adder can be made out of ____________
a) Two half adders
b) Two half adders and a NOT gate
c) Two half adders and one OR gate
d) Two half adders and an AND gate
20) Data selector is basically same as ____________
a) decoder
b) demultiplexer
c) multiplexer
______________

d) encoder

Set A

*SLRBB417*

-3-

SLR-BB 417

Seat
No.

S.E. (CSE) (Part I) (Old) Examination, 2014


SWITCHING THEORY AND LOGIC DESIGN
Day and Date : Thursday, 4-12-2014

Marks : 80

Time : 10.00 a.m. to 1.00 p.m.


Instructions :

1) All questions are compulsory.


2) Figures to the right indicate full marks.
3) Draw neat diagrams.
4) Assume suitable data if necessary.
SECTION I

2. Solve any four :

(45=20)

a) Implement the Boolean expression using only NOR gates w = ( x + y )z D .

b) Simplify the function using K-map F(w, x, y, z) = m(1, 3, 7, 11, 15) + d (0, 2, 5).
c) Draw and explain full adder using half adder.
d) What is excitation table ? Draw excitation table of J-K flip flop and D-type flip flop.
e) Write a short note on a comparator.
f) Explain serial in serial out left shift register. Draw waveforms.
3. Design a combinational logic circuit that has four inputs and one output. The output produces
1 when an input is greater than 1000.
4. a) Design MOD-10 asynchronous counter using T-flip flop.

10
10

OR
b) With the neat diagram and waveforms, explain 3-bit asynchronous counter.

Set A

SLR-BB 417

-4-

*SLRBB417*

SECTION II
5. Attempt any four :

(54=20)

a) Implement 8 line to 256 line decoder using 4 line to 16 line decoders tree.
b) Design 4 digit 7 segment LED display system using 7447 with leading blanking.
c) Explain the concept of PLA.
d) Compare the working of multiplexer and demultiplexer with suitable ICs.
e) Implement 32 : 1 multiplexer using two 16 : 1 multiplexer and one 2 :1 multiplexer.
6. Attempt any two :

(210=20)

a) Write a short note on :


i) Sequential circuit design
ii) Problems with MSI circuits.
b) Explain gate level design, register level design, and processor level design with their
components and examples in detail.
c)

i) Explain BCD to 7 segment decoder functions like LT, RBI, BI, RBD.
ii) Set up a single 7 segment LED display using 7447 BCD to 7 segment decoder/driver.
iii) Design 4 digit 7 segment LED display system using 7447 with leading zero blanking.

_____________________

Set A

SLR-BB 418

*SLRBB418*
Seat
No.

Set

S.E. (I.T.) (Part I) Examination, 2014


DATA STRUCTURE I (Old)
Day and Date : Tuesday, 2-12-2014
Time : 10.00 a.m. to 1.00 p.m.

Max. Marks : 100

Instructions : 1) Q. No. 1 is compulsory. It should be solved in first 30 minutes


in Answer Book Page No. 3. Each question carries one mark.
2) Answer MCQ/Objective type questions on Page No. 3 only.
Dont forget to mention, Q.P. Set (A/B/C/D) on Top of Page.
MCQ/Objective Type Questions
Duration : 30 Minutes

Marks : 20

1. Choose the correct answer :


1) The declaration in (*P) (10) is
a) Pointer to array
c) Pointer to function

20
b) Array of pointer
d) None of the above

2) The number of arguments used in realloc( ) is


a) 0
b) 1
c) 2
3) In ascending priority queue only
a) Smallest
c) any item can be removed

d) 3

b) largest
d) none

4) Can we used string constants in switch cases ?


a) yes
b) no
5) Which of the following is a FIFO list ?
a) Priority queue b) List
c) Stack

d) Queue

6) A pointer to character accesses the following bytes of memory


a) 1
b) 2
c) 4
d) 8
7) In evaluation of postfix expression stack is used to store
a) operands
b) operators
c) both a) and b) d) none
8) Recursive procedure are implemented by
a) queue
b) stack
c) linked list

d) string
P.T.O.

SLR-BB 418
9) Free( ) function is used to
a) Release memory for node
c) To unlink first and last node

*SLRBB418*

-2-

b) To unlink the node


d) None of the above

10) Which of the following statement terminate the execution of loop ?


a) else
b) break
c) return
d) goto
11) Linked list is
a) sequential storage
c) both a) and b)

b) random storage
d) none of the above

12) The number of parameters passed to Fopen( ) function


a) 1
b) 2
c) 3

d) no parameters

13) A static variable is initialized to the value


a) Garbage
b) 0
c) 1

d) 2

14) The contents of the file can be modified when it is opened in following mode
a) r
b) w
c) a+
d) w+
15) The rewind function returns a value which is of the data type ?
a) void
b) int
c) char
d) float
16) The following operator cannot be applied on pointers
a) addition
b) multiplication c) subtraction
17) Is n++ is fast or n = n + 1 is fast ?
a) n++ is fast
c) all of the above

d) decrement

b) n = n + 1 is fast
c) none of these

18) Can we apply modulus operator to float type operands ?


a) Yes
b) no
19) Two functions can have the same name.
a) True
b) False
20) Giving a structure tag is a must.
a) True
b) False
______________

Set A

*SLRBB418*

-3-

SLR-BB 418

Seat
No.

S.E. (I.T.) (Part I) Examination, 2014


DATA STRUCTURE I (Old)
Day and Date : Tuesday, 2-12-2014
Time : 10.00 a.m. to 1.00 p.m.

Marks : 80

Instructions : 1) Each Section carries 40 marks.


2) Attempt all questions from each Section.
SECTION I
2. Solve any four :

(45=20)

a) What is pointer ? Explain pointer to structure with example.


b) Explain declaration and initialization of string.
c) Define Array, Stack, Priority queue, String, Static variable.
d) What do you understand by linear data structure ? Explain with example.
e) Write a C program to display contents of file on screen.
f) What is recursion ? Explain advantage and disadvantage.
3. Solve any one :

10

a) Explain array of pointer and pointer to array with example.


b) What is a string ? Write an algorithm to compare two strings without using
library functions.
4. Explain dynamic memory allocation. Compare it with static allocation with example. 10
SECTION II
5. Answer any four from the following :

(45=20)

a) What is priority queue ? Explain applications of priority queue.

b) Compare stack and queue.

5
Set A

SLR-BB 418

-4-

*SLRBB418*

c) Consider the stack of colours :


STACK : Red, Green, Blue, Yellow, ......
Describe the stack as the following operations take place :
i) Push (STACK, Orange)
ii) Pop (STACK, ITEM)
iii) Pop (STACK, ITEM).

d) Write a C function to insert new node at the end of singly linked list.

e) Define queue. Explain primitive operations on linear queue.

f) What is circular linked list ? Explain types of circular linked list.

6. a) Define circular queue. Explain need for circular queue and write
C functions for :
i) Inserting new element in circular queue
ii) Displaying contents of circular queue.

10

OR
b) Write a C program to convert given infix form expression into its equivalent
postfix form expression.
7. Write a C program for implementation of stack using linked list.
______________

10
10

Set A

SLR-BB 419

*SLRBB419*
S

S.E. (Information Technology) (Part I) (Old) Examination, 2014


SWITCHING THEORY AND LOGIC DESIGN
Day and Date : Thursday, 4-12-2014
Time : 10.00 a.m. to 1.00 p.m.

Max. Marks : 100

Instructions : 1) Q. No. 1 is compulsory. It should be solved in first 30 minutes


in Answer Book Page No. 3. Each question carries one mark.
2) Answer MCQ/Objective type questions on Page No. 3 only.
Dont forget to mention, Q.P. Set (A/B/C/D) on Top of Page.
MCQ/Objective Type Questions

Duration : 30 Minutes

Marks : 20

1. Choose the correct answer :

20

1) _____________ is the dual for P + PR = P.


a) P (PR) = P

b) P (P + R) =

c) P (PR) =

d) P (P + R) = P
4

2) Write the sum of product expression for 7 =


a)
b)
c)


M (2, 3, 4, 5) is ____________
d) none of these
(

3) An example of canonical SOP is _______________


a) ABC + BC + AB

b) AB

c) ABC + AC

d)
)

4) Six variable function contain _______________ number of cells in the K-map.


a) 8
b) 16
c) 32
d) 64
5) In an S-R flip-flop S = R = 1 ___________ permitted.
a) is
b) is not
c) toggle value

d) none of these

6) How is a JK flip-flop made to toggle ?


a) J = 0, K = 0
b) J = 1, K = 0

d) J = 1, K = 1

c) J = 0, K = 1

7) On a possible edge triggered S-R FF, the output reflects the input condition when
_____________
a) the clock pulse is low
b) the clock pulse is high
c) the clock pulse transitions from low to high
d) the clock pulse transition from high to low
8) A BCD counter is _____________
a) Binary counter
c) Full modulus counter

b) Decade counter
d) Divide by 10 counter
P.T.O.

SLR-BB 419

*SLRBB419*

-2-

9) A final output of a modulus-8 counter occurs one time for every __________
a) 8 clock pulse
b) 24 clock pulse
c) 32 clock pulse
d) 16 clock pulse
10) Once an up/down counter begins its count sequence, if cannot be reversed
a) True
b) False
11) Following integrated circuit is used for BCD to 7 segment decoder/driver.
a) 74148
b) 74157
c) 7447
d) 7485
12) The size of memory chip is specified by two numbers as M N bits, where N
stands for
a) No. of words
b) No. of bits at each location
c) No. of location
d) None of these
13) IC 74151 is used as ______________
a) Multiplexer
b) Demultiplexer
c) Decoder
d) Encoder
14) A PLA has ______________
a) Programmable AND array and fixed OR array
b) Fixed AND array and fixed OR array
c) Programmable NAND array and fixed NOR array
d) Fixed AND array and programmable OR array
15) The types of DRAMs are
a) FPM DRAM
c) BEDO DRAM

b) EDO DRAM
d) All the above

16) For selecting one out of 2048 location, the number of address lines required is
___________
a) 10
b) 02
c) 11
d) 15
17) In demultiplexer, the number of output lines in n and the number of select lines is
m, where n = ________________
a) 2 m
b) 2m
c) 2m n
d) m2
18) An EAROM is _______________ erasable.
a) ultraviolet radiation
b) intense head radiation
c) electrically
d) none of these
19) The major computer design levels are
a) Register
b) Processor

c) Gate

20) Multiplexer also known as ________________


a) Encoder
b) Decoder
c) Data selector

d) All the above


d) DMUX-EX

______________

Set A

*SLRBB419*
S

-3-

SLR-BB 419

S.E. (Information Technology) (Part I) (Old) Examination, 2014


SWITCHING THEORY AND LOGIC DESIGN
Day and Date : Thursday, 4-12-2014

Marks : 80

Time : 10.00 a.m. to 1.00 p.m.


Instructions : 1) All questions are compulsory.
2) Figures to the right indicate full marks.
3) Assume suitable data if necessary.
SECTION I
2. Attempt any four :

(45=20)

a) Realise the following function using NAND gates :


f2 =


(0, 1, 2, 3, 11, 12, 14, 15)

b) Write a note on Arithmetic logic unit.


c) Explain SR and JK flip flop.
d) Design full adder and full subtractor using K-map.
e) Minimise the logic function in POS form :
f(A, B, C, D) =


(0, 1, 2, 3, 5, 7, 8, 9, 11, 14).

3. Attempt any two :

(210=20)

a) Draw and explain operation of carry look ahead adder. Obtain the expression for
carry at each stage for 4 bit adder.
b) Explain mod-10 (Decade) asynchronous counter using JK-FF.
c) Design bit comparator using K-map and implement it using logic gates.
Set A

SLR-BB 419

-4-

*SLRBB419*

SECTION II
4. Attempt any four :

(45=20)

a) Implement the expression using 4 : 1 multiplexer.


y = F (A, B, C) =


(1, 3, 5, 6).

b) Realize a 32 : 1 Mux using 16 : 1.


c) Write a short note on register level design.
d) Explain the different types of memories.
e) Explain sequential circuit design with examples.
5. Attempt any two :

(210=20)

a) Design and explain driver for 7 segment display.


b) What are the components of gate level design ? Explain each component of it
with examples in detail.
c) Explain with the help of decoder a example of decoder tree. Give the
significance of decoder tree.
_____________________

Set A

SLR-BB 42

*SLRBB42*
Seat
No.

Set

T.E. (Civil) (Part I) (Old) Examination, 2014


TRANSPORTATION ENGINEERING I
Day and Date : Monday, 15-12-2014
Time : 3.00 p.m. to 6.00 p.m.

Max. Marks : 100

Instructions : 1) Q. No. 1 is compulsory. It should be solved in first 30 minutes


in Answer Book Page No. 3. Each question carries one mark.
2) Answer MCQ/Objective type questions on Page No. 3 only.
Dont forget to mention, Q.P. Set (A/B/C/D) on Top of Page.
MCQ/Objective Type Questions
Duration : 30 Minutes

Marks : 20

1. Choose the correct answer :

20

1) Which of the following is not included in PIEV time of driver ?


a) Perception time b) Volition time
c) Interfering time d) Emotion time
2) Which of the following is not included in PIEV time of driver ?
a) Perception time b) Volition time
c) Interfering time d) Emotion time
3) Bottom most layer of pavement is known as
a) Wearing course b) Base course c) Sub-base course d) Subgrade
4) Rapid curing cutback bitumen is produced by blending bitumen with
a) Kerosene
b) Benzene
c) Diesel
d) Petrol
5) The total area of the district is 14,000 sq.km. and 15 towns as per 1981 census.
The length of MDR is
a) 1350 km
b) 1200 km
c) 1050 km
d) 1450 km
6) The terrain be classified as rolling terrain if the cross slope of land is
a) Between 10% to 25%
b) Between 25% to 60%
c) Between 60% to 80%
d) Between more than 80%
7) The background color of the informatory sign board is
a) Red
b) White
c) Green

d) Yellow

8) Classification of the bridge depending on function is


a) Aqueduct
b) Pedestrian
c) Pipeline

d) All of the above

9) The vertical cutting of river bed is known as


a) Afflux
b) Waterway
c) Scour

d) None of the above

10) The maximum width of vehicle as recommended by IRC is


a) 1.85 m
b) 4.72 m
c) 3.81 m

d) 2.50 m

P.T.O.

SLR-BB 42

*SLRBB42*

-2-

11) Design of horizontal curves on highways is based on


a) Permissible maximum super-elevation
b) Design speed of vehicle
c) Permissible friction on the road surface
d) All the above
12) Which of the following is not matched correctly ?
Type of highways
Minimum ruling radius of horizontal curve
a) National highways
360 m
b) State highways
270 m
c) Major district roads
230 m
d) Other district roads
155 m
13) The maximum thickness of expansion joint is rigid pavement is
a) 5 mm
b) 25 mm
c) 50 mm
d) 100 mm
14) On earthen roads with heavy rainfall area, the camber should preferably be
a) 1 in 10
b) 1 in 25
c) 1 in 15
d) None of the above
15) Normal scour depth in case of a quasi-alluvial streams for contracted water way is
determined by formula

Q
a) D = 0.473
f

13

f
c) D = 0.473
Q

2 3

2 3

b) D = 0.473

13

f
d) D = 0.473
Q

16) Needle beam method is used for


a) Tunneling in soft ground
c) Tunneling in soft and hard ground

b) Tunneling in hard strata


d) None of above

17) In case of RUB


a) Road Passes Under Bridge
c) Both a) and b) are correct

b) Roadway Passes Over Bridge


d) None of above

18) The explosive required for one meter cube of hard rock is
a) 2 kg
b) 5.7 kg
c) 10 kg

d) None of above

19) In liner plate method for tunneling in soft rock, the standard size of plate is
a) 40 cm 48 cm
b) 23 cm 22 cm
c) 42 cm 30 cm
d) 91 cm 41 cm
20) Cut holes, reliever holes and rim holes are blasted in sequence
a) Cut holes, reliever holes and rim holes
b) Reliever holes rim holes, cut holes
c) Rim holes, reliever holes, cut holes
d) None of the above
______________
Set A

*SLRBB42*

-3-

SLR-BB 42

Seat
No.

T.E. (Civil) (Part I) (Old) Examination, 2014


TRANSPORTATION ENGINEERING I
Day and Date : Monday, 15-12-2014
Time : 3.00 p.m. to 6.00 p.m.

Marks : 80

Instruction : All questions are compulsory.


SECTION I
2. Solve any five (8 marks each) :
(58=40)
a) Discuss different factors that affect highway alignment.
b) Describe highway development in India.
c) What is OSD ? Derive the formula for the same.
d) Explain the importance of gradient compensation and super elevation.
e) Define camber. State its different types and values adopted under different road
conditions.
f) Discuss different traffic junctions.
g) Enlist different tests carried out on Bituminous material. Explain any one in detail
with its practical application.
h) What is highway drainage ? How it is carried out ?
SECTION II
3. Solve any three :
a) Discuss factors affecting site selection of a bridge.

b) Write in short about scour depth, afflux.

c) How will you carryout deck construction of a steel bridge ?

d) Explain different bridge foundations.

4. Solve any three :


a) How will you decide size and shape of tunnel ?

b) Explain tunneling in hardwork.

c) Describe how will you carrying out mucking operation in tunneling ?

d) Discuss importance of tunnel lining.

_____________________

Set A

Set A

SLR-BB 420

*SLRBB420*
Seat
No.

Set

S.E. (Electrical) (Part I) (Old) Examination, 2014


PULSE AND DIGITAL CIRCUIT
Day and Date : Thursday, 4-12-2014
Time : 10.00 a.m. to 1.00 p.m.

Total Marks : 100

Instructions : 1) Q. No. 1 is compulsory. It should be solved in first 30 minutes


in Answer Book Page No. 3. Each question carries one mark.
2) Answer MCQ/Objective type questions on Page No. 3 only.
Dont forget to mention, Q.P. Set (A/B/C/D) on Top of Page.
MCQ/Objective Type Questions
Duration : 30 Minutes
1. Choose the correct answer :
1) The Schmitt trigger have ____________ feedback.
a) negative
b) positive
c) both positive and negative
d) none of above

Marks : 20
(120=20)

2) Monostable multivibrator has _______________


a) No stable state
b) One stable state
c) Two stable states
d) None of above
3) Differentiator circuit is a simple RC circuit with output taken across ________
a) R
b) C
c) Both R and C d) None of above
4) A clamping circuit adds _____________ component to the signal.
a) dc
b) ac
c) both ac and dc d) none
5) Which of the following code is used in a K-map ?
a) 8421
b) Gray code
c) Excess 3 code d) BCD
6) In four variable K-map, if we covered 8 adjacent cells as a group then the
resultant term contents __________ number of variables.
a) 4
b) 3
c) 2
d) 1
7) Multiplexer is represented by
a) 2n 1
b) 2n n
c) n 2n
d) 1 2n
8) What is the number of inputs, outputs of a decoder that accepts 64 different
combinations ?
a) 5
b) 6
c) 64
d) 8
9) The logic family which has highest noise margin is _____________
a) T TL
b) IIL
c) MOS
d) CMOS

P.T.O.

SLR-BB 420

*SLRBB420*

-2-

10) The fan out of TTL logic is about _______________


a) 5
b) 10
c) 20

d) 50

11) For a flip-flop formed from NAND gates as shown in the given figure, the
unusable state corresponds to

a) X = 0, Y = 0

b) X = 0, Y = 1

c) X = 1, Y = 0

d) X = 1, Y = 1

12) While converting the SR flip-flop into D flip-flop, the SR inputs are
a) S = D, R = D

b) S = D, R = D

c) S = R = D

d) S = R = D

13) IC 7490 can act as mod N counter. The value of N is


a) 12
b) 16
c) 15
d) 10
14) A 4-bit pre settable up counter has preset input 0101. The preset operation takes
place as soon as counter becomes maximum 1111. The modulus of counter is
a) 5
b) 10
c) 11
d) 15
15) A ________ shift register can be used for SISO, SIPO, PIPO, PISO of data.
a) Formal
b) Global
c) Universal
d) Optimum
16) A binary multiplier uses
a) Right shift register
b) Counter
c) Left shift register
d) None of these
17) A PLA has
a) Programmable AND array and fixed OR array
b) Programmable AND array and programmable OR array
c) Fixed AND array and fixed OR array
d) Fixed AND array and programmable OR array
18) The number of address lines required in memory of 1 kb is ________
a) 16
b) 8
c) 1
d) 10
19) A divide by 78 counter can be realized by using _____________
a) 6 number of mod 13 counters
b) 13 number of mod 6 counters
c) one mod 13 counter followed one mod 6 counter
d) none of these
20) To display a character on LCD, the input data must be in ______________
a) Hex
b) ASCII
c) BCD
d) None
______________

Set A

*SLRBB420*

-3-

SLR-BB 420

Seat
No.

S.E. (Electrical) (Part I) (Old) Examination, 2014


PULSE AND DIGITAL CIRCUIT
Day and Date : Thursday, 4-12-2014

Marks : 80

Time : 10.00 a.m. to 1.00 p.m.


2. Solve any four :

(45=20)

1) Explain RC integrator.
2) Explain working of Schmitt trigger with circuit diagram and waveform.
3) Design full Subtractor.
4) Implement 1 : 4 Demultiplexer using 1 : 2 Demultiplexer.
5) Draw and explain CMOS inverter.
6) Compare CMOS and TTL logic family.
3. a) Explain astable multivibrator using BJT. Draw circuit diagram and
waveforms.

10
OR

a) Reduce the following function using K-map and implement using NOR gate : 10
F = m(1, 5, 6, 12, 13, 14) + d(2, 4).
b) Implement the following logic function using single 8 : 1 MUX. Also realize
the same using 16 : 1 MUX.
F = m(0, 1, 3, 5, 7, 8, 10, 11, 14, 15).

10

Set A

SLR-BB 420

-4-

*SLRBB420*

4. Solve any four :

(45=20)

1) Convert JK flip flop into T flip flop.


2) Design MOD 9 Counter using IC 7490.
3) What are the modes of operation of shift register ? Explain any one with
waveform.
4) Write a note on seven segment display.
5) Explain the operation of LCD display.
6) Differentiate Static and Dynamic RAM.
5. Attempt any two :

(210=20)

1) Explain JK flip flop. What is Race around condition in JK flip flop and how it
will be overcomed ?
2) Draw and explain with waveform 4 Bit Asynchronous up down counter.
3) Explain different types of memories.
_____________________

Set A

SLR-BB 421

*SLRBB421*
Seat
No.

Set

S.E. (Biomedical Engg.) (Part I) Examination, 2014


ELECTRICAL NETWORK ANALYSIS AND SYNTHESIS (Old)
Day and Date : Tuesday, 2-12-2014
Time : 10.00 a.m. to 1.00 p.m.

Max. Marks : 100

Instructions : 1) Figures to right indicate full marks.


2) Assume suitable data if necessary.
3) Q. No. 1 is compulsory. It should be solved in first 30 minutes in Answer
Book Page No. 3. Each question carries one mark.
4) Answer MCQ/Objective type questions on Page No. 3 only. Dont forget to
mention, Q.P. Set (A/B/C/D) on Top of Page.
MCQ/Objective Type Questions
Duration : 30 Minutes
Marks : 20
1. Objective type questions (one mark each) :
20
1) To find thevenius equivalent impedance (resistance) for the circuit __________
a) All independent voltage sources are shorted and all independent current sources are open
circuited
b) All independent voltage sources are open circuited and all independent current sources are
short circuited
c) All independent voltage and current sources are open circuited
d) All independent voltage and current sources are short circuited
2) The Vth and Rth for the following circuit is

a) Vth = 6 V, Rth = 1.5


b) Vth = 3 V, Rth = 6
c) Vth = 6 V, Rth = 5.5
d) Vth = 3 V, Rth = 5.5
3) In circuit shown, what is the mass power transferred to load

a) 5 watt
b) 2.5 watt
4) A passive network has
a) No voltage source but current source
c) No voltage source and no current source

c) 10 watt

d) 25 watt

b) No current source but voltage source


d) None of the above

5) Three resistors of 9 are connected in delta what is the resistance in one of the aim in an
equivalent star network
a) 9
b) 3
c) 27
d) None of these
6) The tie set schedule gives the relation between
a) Branch currents and link currents
b) Branch voltage and link current
c) Branch current and link voltage
d) None of these
P.T.O.

SLR-BB 421

-2-

*SLRBB421*

7) If 10 V battery is connected across parallel combination of resistors of the values 3 , 5 , 10 ,


20 . How much voltage is there across the 5 resistance ?
a) 10 V
b) 5 V
c) 50 V
d) 20 V
8) If one of the resistance in parallel resistance circuit is removed what happens to the total resistance
of circuit
a) Decreases
b) Increases
c) Remains same
d) Doubled
9) For mesh analysis which of the following is correct ?
a) No. of unknowns in equation are equal to no. of meshes
b) Uses of KVL
c) Both a) and b)
d) Either a) or b)
10) If a wire conductor of resistance 0.4 is doubled in length, its resistance will become
b) 0.2
c) 0.8
d) None of these
a) 0.4
11) Four terminal network constitutes a
a) One Port Network
b) Two Port Network c) Three Port Network d) Four Port Network
12) A pole of driving point impedance function signifies
a) A open circuit at that complex freq.
b) A short circuit at that complex freq.
c) Both a) and b)
d) None of these
2

13) The poles of E V (s) are


a) Always simple
b) Always imaginary
c) Always in multiple form
d) Either simple or of higher multiplicity
14) In an impedance function a pole at infinity is realized with use of
a) A capacitance in series
b) An inductor in series
c) An inductor in parallel with driving point terminals
d) None of these
15) Inductor does not allow sudden change in
a) Current
b) Voltage
c) Either a) or b)
d) None of these
16) The function is said to be positive read when
a) The poles and zeros lie an left half of s-plane
b) The poles and zeros lie on right half of s-plane
c) The poles and zeros are simple and lie on imaginary axis
d) Both a) and c)
17) A polynomial q(s) is Hurwitz if
a) q(s) is real when s is real
b) q(s) is real and have real roots which are zero or ve
c) it has conjugate pair of complex roots
d) both b) and c)
18) A minimum reactance function is one which has
a) no zeros at origin
b) no poles at origin
c) no zero an imaginary axis
d) no poles an imaginary axis
19) Steady state value of correct in following circuit will be

a) 5 A
b) 0 A
c)
d) None of these
20) In the given inductive circuit the following equation represents circuit correctly

a) V = L di dt

b) V = IR

c) V = IR + L di dt
______________

1
d) V = IR + di dt
L

Set A

*SLRBB421*

-3-

SLR-BB 421

Seat
No.

S.E. (Biomedical Engg.) (Part I) Examination, 2014


ELECTRICAL NETWORK ANALYSIS AND SYNTHESIS (Old)
Day and Date : Tuesday, 2-12-2014
Time : 10.00 a.m. to 1.00 p.m.

Marks : 80

Instructions : 1) Figures to right indicate full marks.


2) Assume suitable data if necessary.
SECTION I
2. Attempt any four :

(45=20)

a) State and explain superposition theorem with one example.


b) Define the following terms :
i) Trees and co-trees
ii) Tie set matrix and
iii) Tree branch voltages.
c) For the network shown. Find the current flowing through all resistances by using superposition
theorem.

d) Find the equivalent resistance between terminals A and B for the network.

e) Find equivalent Nortous circuit for the network.

Set A

SLR-BB 421

-4-

*SLRBB421*

3. Attempt any two :

(210=20)

a) For the network shown. Find the voltage across 4 resistor by using node analysis.

b) Determine current I in the circuit by using super position analysis (all resistor are in )

c) State and explain maximum power transfer theorem and also find the value of RL and max. power
absorbed by RL.

SECTION II
4. Attempt any four :
(45=20)
a) Derive the expression for parallel connected two port network.
b) Derive the expression for short circuit admittance parameters.
c) For the circuit shown. Find the complete expression for the current when switch is closed at t = 0.

d) Derive and explain DC response of RC ckt.


e) Explain the conditions for positive real function.
5. Attempt any two :
a) Two identical sections of network shown in fig. are to be connected in parallel obtain Y
parameters of the combination.

(210=20)

b) Obtain interrelation of ABCD parameters with Z and Y parameters.


c) Test whether the polynomial
P(s) = s5 + 3s3 + 2s is Hurwitz.
_____________________

Set A

SLR-BB 422

*SLRBB422*
Seat
No.

Set

S.E. (Part I) (Electrical and Electronics Engg.)(Old) Examination, 2014


ELECTRICAL CIRCUIT ANALYSIS
Day and Date : Tuesday, 2-12-2014
Time : 10.00 a.m. to 1.00 p.m.
Instructions :

Max. Marks : 100

1) Q. No. 1 is compulsory. It should be solved in first 30 minutes in


Answer Book Page No. 3. Each question carries one mark.
2) Answer MCQ/Objective type questions on Page No. 3 only. Dont
forget to mention, Q.P. Set (A/B/C/D) on Top of Page.
MCQ/Objective Type Questions

Duration : 30 Minutes

Marks : 20

1. Solve the following :

(201=20)

1) Which of the following quantities are the same in all parts of series circuit.
a) Voltage
b) Current
c) Resistance
d) None of the above
2) Two resistances of equal value, when connected in parallel give an equivalent resistances
of R . If these resistances are connected in series, the equivalent resistance will be
a) R

b) 4 R

c) 2 R

d) R/2

3) For the circuit shown, an equivalent resistance will be

a) 36

b) 12

4) An ideal voltage source should have


a) Zero source resistance
c) Finite source resistance

c) 4

d) 6

b) Infinite source resistance


d) None of the above

5) Which of the following statements is incorrect ?


a) Resistance is passive element
b) Inductor is a passive element
c) Current source is a passive element
d) Voltage source is an active element
6) The terminals across the source are ____________ , if a current source is to be replaced.
a) Short circuited
b) Open circuited
c) Inductor
d) Capacitor
7) For determining the polarity of voltage drop across a resister, it is necessary to know the
a) Value of resistor
b) Value of current
c) Value of emf in the circuit
d) Direction of current through resistor
8) Kirchoffs laws are valid for
a) Linear circuits only
c) Passive networks only

b) Non-linear circuits only


d) Both linear and non-linear circuits

P.T.O.

SLR-BB 422

*SLRBB422*

-2-

9) Superposition theorem is applicable only for


a) Single source network
b) Two or more source network
c) Voltage source network
d) None of the above
10) In Thevenins circuit, the voltage VTh is,
a) Short-circuit terminal voltage
c) EMF in the network

b) Open-circuit terminal voltage


d) EMF of battery nearest terminals

11) In star arrangement of resistances has each resistances of 3 , The equivalent delta
arrangement will have each resistance of value.
a) 9
b) 6
c) 3
d) 1.5
12) The theorem that enables no. of voltage/current sources to be combined directly into
single voltage or current source is the ____________ theorem.
a) Millmans
b) Reciprocity
c) Superposition
d) Maxwells
13) The Y11 for the two-port network is

14) A leading power factor implies that,


a) Current lags behind the voltage
c) Current leads the voltage

c) 0.1

b) 0.2

d) 0.4

a) 0.5

b)Voltage leads the current


d) None of the above

15) In an ac circuit, low reactive power in comparison with true power gives.
a) Low power factor
b) High power factor
c) Unity power factor
d) None of the above
16) The phasors for which of the following are in phase opposition in series RLC circuit.
a) VL and VR
b) VC and VR
c) VL and VC
d) None of the above
17) The delta connection has
a) Neutral point does not exist
c) Neutral point is exist

b) 3 phase 3 wire system


d) Both a) and b)

18) A series RLC circuit at resonance has


a) Maximum impedance
c) Current is minimum

b) Maximum admittance
d) Power factor is zero

19) In series resonant circuit, the reactance above resonant frequency is


a) Capacitive
b) Resistive
c) Inductive
d) Zero
20) The Q-factor of R-LC circuit will increase if
a) R decreases
b) R increases
c) Voltage increases
d) None of the above
______________
Set A

*SLRBB422*

SLR-BB 422

-3-

Seat
No.

S.E. (Part I) (Electrical and Electronics Engg.) Examination, 2014


ELECTRICAL CIRCUIT ANALYSIS (Old)
Day and Date : Tuesday, 2-12-2014
Time : 10.00 a.m. to 1.00 p.m.
Instructions :

Marks : 80

1) All questions are compulsory.


2) Assume data as applicable
SECTION I

2. Solve the following (any four) :

20

1) Find the voltage node x-using source transformation.

2) For the graph of network shown in Fig. find :


i) Incidence matrix

ii) F- cutset matrix

iii) F-Tieset matrix

3) Using Millmans theorem, find the current in (3 + j4) .

Set A

SLR-BB 422

-4-

*SLRBB422*

4) Find the currents Ix and Iy by mesh analysis.

5) Determine the value of Ix using Thevenins theorem.

6) Define the following terms :


a) Network

b) Circuit

c) Node

d) Branch

e) Loop

f) Mesh

g) Dependent sources
3. Solve any two :

20

1) Find the current through j1 using Nortons theorem.

2) State and explain Thevenins and Maximum power transfer theorem. Write steps to solve
the any network by it.
Set A

*SLRBB422*

SLR-BB 422

-5-

3) Find the value of current I by using superposition theorem.

SECTION II
4. Solve the following (any four) :

20

1) Describe the current-frequency curve at resonance and also define bandwidth and
Q-factor at series resonance.
2) Find the response produced by R-C circuit by using Laplace transform for :
i) Unit step

ii) Unit impulse function

3) For the circuit shown, switch is closed at t = 0. Find ic(t) at t 0.

4) A series RLC circuit has R = 10 , L = 0.01H, C = 100 F. Calculate :


i) Resonant frequency
iii) Bandwidth

ii) Q-Factor
iv) Half Power Frequencies

5) Determine i1 and iL in the circuit shown, when switch is open at t = 0.

Set A

SLR-BB 422

-6-

*SLRBB422*

6) Find Y-parameter for the network shown

5. Solve any two :

20

1) Find out VL(t) in the network shown when :


i) Vs (t) =

(t),

ii) Vs(t) = et u(t) by Laplace

2) Derive the Y-parameter interns of


i) Z-parameter
ii) H-parameter
iii) ABCD parameter
3) In Fig. shown, iL(0) = 40A, V(0) = 40V, L = 12.5 mH find an expression
for v(t).

______________

Set A

SLR-BB 423

*SLRBB423*
Seat
No.

Set

S.E. (Electrical and Electronics Engg.) (Part I) Examination, 2014


DATA STRUCTURES (Old)
Day and Date : Thursday, 4-12-2014
Time : 10.00 a.m. to 1.00 p.m.

Max. Marks : 100

Instructions : 1) Q. No. 1 is compulsory. It should be solved in first 30


minutes in Answer book Page No. 3. Each question carries
one mark.
2) Answer MCQ/Objective type questions on Page No. 3
only. Dont forget to mention, Q.P. Set (A/B/C/D) on Top
of Page.
MCQ/Objective Type Questions
Duration : 30 Minutes

Marks : 20

1. Choose correct alternatives :

20

1) int a[5] [5] array declare how many elements of integer.


a) 5
b) 10
c) 25
d) 15
2) If array elements are not initialized, then they contain
a) 0 values
b) 1 value
c) garbage

d) all

3) Which specifier is used to print the address of variable ?


a) % d
b) % lf
c) % u
d) % f
4) More than one argument can be passed to function.
a) True
b) False
c) Cant say

d) None

5) Which bitwise operator is used for right shift ?


a) >
b) >>
c) <

d) <<

6) Which of the function places the pointer to beginning of file ?


a) fseek()
b) rewind()
c) fread()
d) fwrite()
7) A _________ integer is an integer that is always positive.
a) short
b) long
c) unsigned
d) all
8) Which of the following is represented as ADT ?
a) data type
b) array
c) structures

d) all
P.T.O.

SLR-BB 423

-2-

*SLRBB423*

9) Which operations are performed from top of stack ?


a) insertion
b) deletion
c) both a) and b) d) none
10) A postfix AB + C is equivalent to infix
a) (A + B)/C
b) A + B C
c) A + (B C)

d) A B + C

11) Which data structure is used to implement recursion in programming language ?


a) Linked list
b) queue
c) stack
d) array
12) Worst case complexity of Quick sort is
a) O(n2)
b) O(logn)
c) O(nlogn)

d) O(n/2 logn)

13) The complexity of linear search algorithm is


a) O(n)
b) O(log n)
c) O(n2)

d) O(n log n)

14) When new data are to be inserted into a data structure, but there is no
available space; this situation is usually called
a) underflow
b) overflow
c) full
d) saturated
15) How many fields are contained in each node in circular singly linked list ?
a) At least two
b) Exactly two c) Three
d) One
16) The prefix form of AB/ (C * D ^ E) is
a) /*^ ACBDE
b) ABCD*^DE
c) A/B*C^DE
d) A/BC*^DE
17) Which of the following linked list below have last node of the list pointing to
the first node ?
a) Singly linked list
b) Doubly linked list
c) Circular linked list
d) None of above
18) Binary search technique require which of the following condition
a) list should be unsorted
b) list should be sorted
c) list must contain only characters
d) list cannot exceed 100 elements
19) Finding the location of the element with a given value is
a) Traversal
b) Search
c) Sort
d) None of above
20) When Hash function produce same address for two different keys, then
__________ occurs.
a) collision
b) clustering
c) probing
d) overflow
______________
Set A

*SLRBB423*

-3-

SLR-BB 423

Seat
No.

S.E. (Electrical and Electronics Engg.) (Part I) Examination, 2014


DATA STRUCTURES (Old)
Day and Date : Thursday, 4-12-2014
Time : 10.00 a.m. to 1.00 p.m.

Marks : 80

Instruction : All questions are compulsory.


SECTION I
2. Attempt any four (each carries 5 marks) :

20

A) What is array ? Describe multidimensional array.


B) What is ADT ? Explain.
C) List and explain all C data types.
D) Write a note on structure as an ADT.
E) Describe fread and fwrite functions.
F) Compare structures and unions.
3. What is function ? Explain. Describe call by value and call by reference using
examples.

10
OR

Write a program in C for implementation of a stack of size 5. Describe and write


push and pop operations and stack.

10

4. Perform conversion of following infix expression into prefix and postfix ($ having
highest precedence)
i) (A + B) * (C$ (D E) + F) G
ii) (A + B) * (C D) $ E * F.

10

Set A

SLR-BB 423

-4-

*SLRBB423*

SECTION II
5. Attempt any four :

(45=20)

1) Explain any three types of Recursion.


2) Explain Sequential search and give its timing analysis.
3) With the help of neat diagram explain enqueue and dequeue operations of queue.
4) Write C code to implement bubble sort for 10 numbers.
5) Give advantages and disadvantages of Doubly Linked List.
6) What is Dynamic Memory Allocation ? How it is done in C ?
6. Attempt any one :

10

1) What is Sorting ? Explain Quick Sort Method.


Sort the following numbers using quick sort
45, 26, 77, 14, 68, 61, 97, 39, 99, 90
2) What is Singly Linked List ? Write a C code to implement following operations
on Singly Linked List.
a) Insert at end
b) Insertion at specified location
c) Deletion at specified location
d) Display linked list.
7. What is Hashing ? Why it is useful than linear and binary search. Explain all the
techniques of Open Hashing to resolve Hash Collision.
10
_____________________

Set A

SLR-BB 43

*SLRBB43*
Seat
No.

Set

T.E. (Civil) (Part I) Examination, 2014


DESIGN OF STEEL STRUCTURES (New)
Day and Date : Wednesday, 3-12-2014
Time : 3.00 p.m. to 6.00 p.m.
Instructions :

Total Marks : 100

1) Q. No. 1 is compulsory. It should be solved in first 30 minutes in


Answer Book Page No. 3.
2) Answer MCQ/Objective type questions on Page No. 3 only. Dont
forget to mention, Q.P. Set (A/B/C/D) on Top of Page.
MCQ/Objective Type Questions

Duration : 30 Minutes

Marks : 20

1. Choose the correct answer :


1) For welds 91 to 100 angle between fusion faces, the throat thickness is taken as
a) 0.6 times the size
b) 0.7 times the size
c) 0.65 times the size
d) 0.55 times the size

2) The max. Slenderness ratio in steel ties which may be subjected to compression under wind
load should be
1
a) 250
b) 450
c) 350
d) No limit
3) A structural member subjected to compressive stress in a direction parallel to its
longitudinal axis, is generally known as
a) Strut
b) Post
c) Column
d) All the above

4) The effective length in the plane of end gusset of double angle discontinuous strut in
welded truss is equal to
a) L
b) 0.7 L
c) 0.85 L
d) (0.7 to 0.85) L

5) The design tensile strength of a plate due to rupture of critical section is given by
a) 0.8 Anfu/ m1
b) 0.8 Anfy/ m0
c) 0.9 Anfu/ m1
d) 0.9Anfy/ m0

6) If the outstand element of compression flange of a rolled section is less than ______ the
section is classified as plastic.
b) 10.5
c) 9.4
d) 15.7
a) 8.4
7) The structural advantage of using steel as a structural material is
a) Small weight to strength
b) Speed of dismantling
c) Speed of erection
d) All of above
8) If the number of possible plastic hinges are 4 and the degree of indeterminacy of the
structure is 2 then the number of possible independent mechanisms n will be
a) 6
b) 4
c) 2
d) 1
9) In plastic analysis, the shape factor for circular sections is
a) 1.5
b) 1.697
c) 1.6
10) A beam section is selected and provided on the basis of
a) Section modulus b) Deflection
c) Shear

1
1

1
2

d) None of these
1
d) All the above
P.T.O.

SLR-BB 43

*SLRBB43*

-2-

11) The slenderness ratio of a web of beam section from diagonal buckling consideration is
given by
a)

d
tw

b) 2.45

d
tw

c) 85

d
tw

d) 67

d
tw

12) A beam section is classed as low shear case when the factored shear force is less than
a) 0.4 Vd
b) 0.6 Vd
c) 0.8 Vd
d) Vd

13) In case of an axially loaded column machined for full bearing, the fastenings connecting the
column to the base plate in gusseted base are designed for
1
a) 100% of the column load
b) 50% of the column load
c) 25% of the column load
d) Erection loads only
14) A column base is subjected to moment. If the intensity of bearing pressure due to axial
load is equal to stress due to moment then the bearing pressure between the base and
concrete is
a) Uniform compression throughout
b) Zero at one end and compression at other end
c) Tension at one end and compression at the other end
d) Uniform tension throughout
15) The length of the anchor bolt may be found by
a)

A .Fy
4bdm0

b)

A .Fy
bd m

c)

1
d .Fy
4bdm0

d)

d .Fu
bdm1

16) The economical spacing of roof trusses are generated kept as


a)

1
1
to
span
3
5

b)

1
1
to
span
4
3

c)

1
2
to
span
4
3

1
d)

1
1
to
span
4
2

17) The principal rafter of a roof truss with purlins placed at intermediate points on the panel
length can be analysed by _______
a) Method of joint
b) Method of section
c) Graphical method
d) Moment distribution method
18) Live load for roof trusses should not be less than ____________
b) 0.4 KN/m2
c) 0.75 KN/m2
d) 1.5 KN/m2
a) 0.2 KN/m2
19) The partial safety factors for dead load and wind load for a roof truss for limit state of
strength are respectively __________
a) 1.0 and 1.5
b) 1.5 and 1.5
c) 1.2 and 1.5
d) 1.2 and 1.2

______________

Set A

*SLRBB43*

-3-

SLR-BB 43

Seat
No.

T.E. (Civil) (Part I) Examination, 2014


DESIGN OF STEEL STRUCTURES (New)
Day and Date : Wednesday, 3-12-2014
Time : 3.00 p.m. to 6.00 p.m.

Marks : 80

Instructions : 1) Solve any three questions from Section I and II.


2) Assume suitable data if required and mention it clearly.
3) Figures to right indicate full marks.
4) Use of IS : 800-2007, IS 875 Part III and steel table is permitted in
examination.
SECTION I
1. a) Define pitch, edge distance and end distance.

b) In a truss, a discontinuous strut 4 m long consists of 21SA 1001008mm connected to


each side of 12 mm thick gusset plate by weld. Calculate the design strength of the
member. Use steel of grade Fe410, fy = 250 MPa.

11

2. a) Explain partial safety factors and provide the values in tabular form.

b) An unequal angle 1.5 m long is connected to the gusset plate. It carries an ultimate tension
of 250 kN. Design the section using 4 mm weld. Assume fy = 250 MPa and fu = 410 MPa.

12

3. Design a tension member using double unequal angle sections back to back on opposite side
gusset plate of 10 mm thick which carries an axial factored load of 350 kN using 20 mm black
bolt. Assume Fe 410 grade of steel, fy = 250 MPa and also show design details.

13

4. Design a double angle discontinuous strut placed on same sides of 12 mm thick gusset plate
to carry a factored load of 145 kN. The length of the strut is 3.0 m between intersections. The
two angles are placed back to back (with long legs connected) and are tack bolted. Use steel
of grade Fe 410 fy = 250 MPa.

13

Set A

SLR-BB 43

-4-

*SLRBB43*

SECTION II
5. Check whether ISMB 250 is suitable or not as a simply supported beam over an effective
span of 6.0 m. It carries UDL of 15kN/m. (including self weight).
Properties of ISMB 250 are
bf = 125 mm
tf = 12.5 mm
tw = 6.9 mm
Ixx = 5731.6104mm4
Zxx = 410103 mm3
r1 = 13.0 mm
Zp = 465.71103mm3.

13

6. Design the principal rafter of a fink type roof truss for the following data :
i)

13

Design compressive load = 165 kN(DL+LL)

ii) Design tensile load = 60 kN (DL + WL)


iii) Length of rafter panel = 2.235m
iv) Grade of steel = Fe410
v) Grade of bolts = 4.6
7. Design a built up column consisting of two rolled steel I sections to resist an axial load of
3700 kN. The length of the column is 4.5 m. It is restrained against rotation and translation at
bottom and restrained against rotation and free at top. Take fy = 250 MPa. Also design the
lacing system.

14

8. A column ISHB 350@ 661.2 N/m carries an axial compressive factored load of 1700 KN.
Design a suitable bolted gusset base. The base rests on M15 grade concrete pedstal. Use 24 mm
diameter bolts of grade 4.6 for making the connection. Also draw a neat sketch.

13

_____________________

Set A

SLR-BB 44

*SLRBB44*
Seat
No.

Set

T.E. (Civil) (Part I) Examination, 2014


GEOTECHNICAL ENGINEERING I (New)
Day and Date : Friday, 5-12-2014
Time : 3.00 p.m. to 6.00 p.m.

Max. Marks : 20+80=100

Instructions : 1) Assume additional data, if required and state it clearly.


2) Figures to the right indicate full marks.
3) Q. No. 1 is compulsory. It should be solved in first 30
minutes in Answer Book Page No. 3.
4) Answer MCQ/Objective type questions on Page No. 3 only.
Dont forget to mention, Q.P. Set (A/B/C/D) on Top of Page.
MCQ/Objective Type Questions
Duration : 30 Minutes

Marks : 20

1. Give the correct option for following MCQ .


1) In the modified proctor test, the compaction energy with respect to
standard compaction test is
a) Less
b) Same
c) More
d) No relation
2) The soil having particles of variety of sizes is said to be
a) Well graded
b) Uniformly graded
c) Poorly graded
d) Gap graded
3) The critical exit gradient for the seepage of water out of a soil is
a)

G 1
1 e

b)

G1
1 + e

c)

G +1
1 e

d)

G+1
1 + e

4) The toughness index of clayey soil is given by,

a) Plasticity index/flow index


c) Liquidity index/plastic limit

b) Liquid limit/plastic limit


d) Plastic limit/liquidity index

5) An unconfined compression test is good for


a) Granular soils
b) Saturated cohesive soils
c) Both granular and cohesive soils d) None of these

6) Piping in soil occurs when


1
a) The soil is highly porous
b) Sudden change in permeability occurs
c) Effective pressure becomes zero d) The soil highly stratified
P.T.O.

SLR-BB 44

-2-

*SLRBB44*

7) In a compaction test, as a compaction effort is increased, the optimum


water content
a) Decreases
b) Remains same
c) Increases
d) Increases first and then decreases
8) During seepage through an earth mass, the direction of seepage is
a) Parallel to the equipotential line
b) Perpendicular to the stream line
c) Perpendicular to the equipotential line
d) Along the direction of gravity

9) When a retaining wall moves away from the backfill, the pressure exerted on
the wall is termed as
1
a) Passive earth pressure
b) Swelling pressure
c) Pore pressure
d) Active earth pressure
10) When the degree of consolidation is 50%, Time factor will be ________
a) 0.1963
b) 0.382
c) 4
d) 1

11) In a given soil sample, Degree of saturation is 90%, G = 2.7, Voids ratio
is 0.3. The water content of sample is
a) 10%
b) 13.5%
c) 22.5%
d) 35%

12) A sand deposit has a porosity of 0.375 and a specific gravity of 2.6, the
critical hydraulic gradient for the sand deposit is
a) 2.975
b) 2.225
c) 1
d) 0.75

13) In a drained triaxial compression test, a saturated specimen of cohesionless


sand fails under a deviatric stress of 3 kg/cm2. The effective angle of shearing
resistance of sand is about
a) 37
b) 45
c) 53
d) 20
14) A concentrated load of 50 T acts vertically at a point on the surface. If
Boussinesqs equation is applied for computation of stress, then the ratio of
vertical stress at depth of 3 m and 5 m, respectively, vertically below the
point of application of load will be
a) 0.36
b) 0.6
c) 1.66
d) 2.77
15) A rigid retaining wall 5 m high supports a backfill of cohesionless soil with
/ = 30, what will be the coefficient of active earth pressure.
O
a) 1/3
b) 3
c) 1/2
d) 2

______________

Set A

*SLRBB44*

-3-

SLR-BB 44

Seat
No.

T.E. (Civil) (Part I) Examination, 2014


GEOTECHNICAL ENGINEERING I (New)
Day and Date : Friday, 5-12-2014
Time : 3.00 p.m. to 6.00 p.m.

Marks : 80

Instructions : 1) Solve any three questions from Section I.


2) In Section II, Q. No. 6 is compulsory, solve any two
from remaining questions.
3) Assume additional data, if required and state it clearly.
4) Figures to the right indicate full marks.
SECTION I
2. A) Define the following :
i) Density index
ii) Coefficient of uniformity
iii) Specific gravity
iv) Air content
v) Porosity.
B) Draw a neat labeled 3-phase diagram, using this diagram derive relation.
3. A) With a neat labelled sketch, explain characteristics of flownet.
B) For a homogeneous Earth dam 32 m high and 2 m free board, a flow net was
constructed with four flow channels. The no of potential drops are 20. The
dam has a horizontal filter at the base near the toe. The coefficient of
permeability of the soil is 9102 mm/sec. Determine the anticipated seepage,
if the length of the dam is 100 metres.

8
5

4. A) Derive relation between angle made by failure plane with major principal plane. 5
B) The following results were obtained from a direct shear test on a sandy clay
sample.
Normal load (N)
Shear load proving ring reading (Division)
360
13
720
19
1080
26
1440
26
If the box is 60 mm 60 mm square and the proving ring constant is 20 N per
division. Estimate the shear strength parameters of the soil.
8
Set A

SLR-BB 44

-4-

*SLRBB44*

5. A) Write short notes on any three :


a) Atterberg limits
b) GSD curve
c) Stress isobar
d) Sensitivity of soil.

B) Write a detail note on Boussinesqs equation for stress under the


concentrated load.

SECTION II
6. Write short note on following (any four) :

(44=16)

a) Effect of cohesion on earth pressure


b) Coefficient of volume change
c) Field compaction control
d) Assumptions in Rankines earth pressure theory
e) Proctor needle
f) e-logp curve.
7. A) Draw the sketch of the rammer used for light compaction test (standard
compaction test). Give the specifications of it as per IS.

B) Explain the factors affecting the compaction in detail.

8. A) Distinguish between normally consolidated and over consolidated soil.

B) Under a consolidation pressure of 0.2N/mm2 the thickness and water content


of a saturated specimen were found to be 20.5 mm and 16%. On increasing
the pressure to 0.4 N/mm2 the specimen got compressed by 1.05 mm. Find
the compression index of soil. Take G = 2.76.
9. A) Write a note on Coulombs Theory of earth pressure.

B) A cantilever retaining wall having vertical back face retains moist sandy soil
as backfill. It carries a uniform surcharge of 15 kN/m2. Height of backfill is
5 m. Backfill has following properties.
Dry unit weight of soil 16kN/m3, O/ = 30, water content = 15%.
Draw active earth pressure diagram and determine total active force on
retaining wall.
_____________________

Set A

SLR-BB 45

*SLRBB45*
S

T.E. (Civil) (Part I) Examination, 2014


BUILDING PLANNING AND DESIGN (New)
Day and Date : Monday, 8-12-2014
Time : 3.00 p.m. to 7.00 p.m.

Max. Marks : 100

N. B. : 1) All questions are compulsory.


2) Section I to be drawn on full imperial drawing sheet.
Use both sides of the sheet.
3) Section II to be written in answer book.
4) Figures to the right indicate full marks.
5) Assume suitable data wherever necessary and mention
it clearly.
6) Retain all projection/construction lines on drawing sheet.
7) Q. No. 1 is compulsory. It should be solved in first 30
minutes in Answer Book Page No. 3. Each question carries
one mark.
8) Answer MCQ/Objective type questions on Page No. 3
only. Dont forget to mention, Q.P. Set (A/B/C/D) on Top
of Page.
Duration : 30 Minutes

MCQ/Objective Type Questions

Marks : 20

1. State whether following statements is correct or incorrect.


1) FSI in case of public buildings is always less than one.
2) If electronic components catches fire, it is needed that water shall be
used to prevent fire.
3) CCU in hospital means critical care unit.
4) Unit of fire load is Kcal/sqm.
5) RCC structure can withstand fire easily upto 1000C.
6) Dead spots and sound foci are interrelated.
7) In hotels pantry is provided.
8) Width of bus is approximately 2.5 m.
9) A motel, is a hotel along highways.
10) LEED is the Green Building Certification System.
P.T.O.

SLR-BB 45

*SLRBB45*

-2-

Select the most appropriate alternative.


11) Locally available building materials are the best because
A) less transportation cost
C) cheapter cost

B) readily available
D) they give native touch

12) Cinema theatres are to be designed at the rate of


A) 0.6 sq.m. per head
C) 1 sq.m. per head

B) 1.5 sq.m. per head


D) 2 sq.m. per head

13) Reverberation are desirable to add to the


A) musical quality
C) better audibility

B) clarity of speech
D) better acoustics

14) Echo is
A) Dull sound
C) Repeated sound

B) Reflected sound
D) Multiplied sound

15) For library, the width of shelves to store books is


A) 0.2 m
B) 0.3 m
C) 0.5 m
16) Mezzanine floor may be provided for
A) Book stocking area

D) 0.6 m

B) Reading area

C) Periodical section
D) Lending section
17) The desirable number of storeys for a school building is
A) Not more than 2

B) Not more than 3

C) Not more than 4


D) Only one
18) In library, clear spacing between the shelves is
A) 1 m

B) 1.5 m

C) 2 m

D) 3 m

19) The desirable width of a class room is not more than


A) 4 m
B) 6 m
C) 7 m
D) 9 m
20) Minimum width of a door opening in a school is
A) 0.9 m

B) 1.65 m
C) 1.0 m
______________

D) 1.5 m

Set A

*SLRBB45*
S

-3-

SLR-BB 45

T.E. (Civil) (Part I) Examination, 2014


BUILDING PLANNING AND DESIGN (New)
Day and Date : Monday, 8-12-2014
Time : 3.00 p.m. to 7.00 p.m.
N. B. :

Marks : 80

1) All questions are compulsory.


2) Section I to be drawn on full imperial drawing sheet.
Use both sides of the sheet.
3) Section II to be written in answer book.
4) Figures to the right indicate full marks.
5) Assume suitable data wherever necessary and mention
it clearly.
6) Retain all projection/construction lines on drawing sheet.
SECTION I

2. A Primary Health Centre is to be provided for a village. The following accommodation


needs to be provided :
Waiting Hall
50 sq.m.
Consulting rooms 2 Nos. each 12.5 sq.m.
Dispensary and drug store
22.5 to 25 sq.m.
Office room
15 sq.m.
Dressing room
15 sq.m.
Mini operation theater
25 to 30 sq.m.
Detached but close ward for 8 to 10 Beds with nurses room.
The plot admeasures 0.8 hectares and it faces 12 m wide road on east. All other
provisions such as sanitary units, Verandah etc. should be provided as per
requirements. The Buildings is RCC framed structure with single storey. Assume
additional data if required.
Draw :
1) A detailed plan (Scale 1 : 100).
12
2) A site plan to suitable scale and mention it on drawing.
10
3) Detailed furniture arrangement for typical ward (Scale 1 : 50).
8
Set A

SLR-BB 45

-4-

*SLRBB45*

3. Figure below shown a plan and elevation of object. Assume eye level at 1.5 m
above GL. Draw perspective to suitable scale. Retain all construction lines.

10

SECTION II

4. Attempt any five questions.


40
A) Write a short note on : Rain water harvesting and future scope of it.
B) Explain different factors influencing planning of public building.
C) What do you understand by submission drawing ? What is the necessity of it ?
D) Explain the benefits of green buildings.
E) Explain Form and Function w.r.t. architectural composition of a building.
F) Define Fire load and how is it calculated.
G) Explain sound reflection from different kinds of surfaces.
_____________________

Set A

SLR-BB 46

*SLRBB46*
Seat
No.

Set

T.E. (Civil) (Part I) (New) Examination, 2014


ENVIRONMENTAL ENGINEERING I
Day and Date : Wednesday, 10-12-2014
Time : 3.00 p.m. to 6.00 p.m.

Total Marks : 100

Instructions : 1) Solve any three questions from Section I i.e. Question No. 2
to Question No. 5. Question No. 6 is compulsory in Section II
and solve any two questions from the remaining.
2) Figures to the right indicate full marks.
3) Assume suitable data wherever necessary and mention it
clearly.
4) Use of non-programmable calculator is allowed.
5) Q. No. 1 is compulsory. It should be solved in first 30 minutes
in Answer Book Page No. 3. Each question carries one mark.
6) Answer MCQ/Objective type questions on Page No. 3 only.
Dont forget to mention, Q.P. Set (A/B/C/D) on Top of Page.
MCQ/Objective Type Questions
Duration : 30 Minutes

Marks : 20

1. Choose the correct answer :

20

1) Indicator/s used in alkalinity test is/are __________


a) Phenolphthalein (PP)
b) Methyl Orange (MO)
c) PP and MO
d) None of these
2) A condition that tends to increase corrosiveness of water on metals is
__________
a) High DO content
b) Low DO content
c) Polyphosphate
d) Low TDS
3) Design period of water works depends upon __________
a) Funds availability
b) Life of pipe material and other structural materials
c) Rate of interest on loan taken to complete the project
d) All of above
4) Generally __________ supply will reduce rate of demand.
a) Continuous
b) Intermittent
c) Both a) and b)
5) __________ causes scaling in boilers.
a) Hardness
b) Colour
c) Odour

d) None of these
d) Taste

6) For design of flocculator the value of G.t should be __________


b) 100 to 1000
c) 106 to 109
d) All of above
a) 104 to 105
P.T.O.

SLR-BB 46

-2-

*SLRBB46*

7) Immediately after addition of Alum, __________ mixing is necessary.


a) Gentle
b) Rapid
c) No
d) None of these
2
3
8) For flow of 15 mld with SOR 20 m /m /d, surface area of sedimentation
tank is __________ m2.
a) 750
b) 450
c) 650
d) 550
9) For ideal settling basin vs is settling velocity. If particle is having settling velocity
(vs) more than that of vs, then particle will removed with __________ efficiency.
a) 100%
b) Less than 100%
c) 0% (no removal)
d) None of these
10) The process in which chlorination is done beyond break point is __________
a) Pre chlorination
b) Super chlorination
c) Post chlorination
d) Break point chlorination
11) The storage capacity of a reservoir may be divided into three zones. The lowest
zone is
a) Surcharge storage
b) Dead storage
c) Useful storage
d) None of these
12) In which pipes the total discharge is equal to sum of discharge in each pipes.
a) Pipes in series b) Rough pipes
c) Pipes in parallel d) Smooth pipes
13) Corrosion of a pipe occurs __________
a) Reduces its life span
b) Adds colour to water
c) Adds odour to water
d) All the above
14) Water supply includes not only collection, transportation and treatment of water
but __________
a) Provision of hydrants for fire fighting b) Chlorination
c) Pressure in pipes
d) Control of leakages
15) __________ is a primary connection from distribution system to the consumer.
a) Fixture spacing
b) Service connection
c) Process piping
d) Tree connection
16) To control the wastage of water __________ measures are taken.
a) Pipe joints
b) Water tax
c) Zoning system
d) All of these
17) Total head loss is composed of __________
a) Composition of pipe material
b) Replacement of pipes
c) Water hammer
d) Friction and types of fitting
18) Analysis of pipe networks of distribution system is calculated by __________
a) Discharge in pipelines
b) Equivalent pipe method
c) Computation of pressure
d) Mass curve method
19) By studying the nature of hydraulic gradient __________ can be detected.
a) Leakages
b) Corrosion
c) Number of joints
d) Velocity
20) __________ is a type of distribution system in which whole area is dived in square
zones.
a) Grid iron system
b) Reticulation system
c) Radial system
d) Ring system

______________

Set A

*SLRBB46*

-3-

SLR-BB 46

Seat
No.

T.E. (Civil) (Part I) (New) Examination, 2014


ENVIRONMENTAL ENGINEERING I
Day and Date : Wednesday, 10-12-2014
Time : 3.00 p.m. to 6.00 p.m.

Marks : 80

Instructions : 1) Solve any three questions from Section I i.e. Question No. 2
to Question No. 5. Question No. 6 is compulsory in Section II
and solve any two questions from the remaining.
2) Figures to the right indicate full marks.
3) Assume suitable data wherever necessary and mention it
clearly.
4) Use of non-programmable calculator is allowed.
SECTION I
2. a) Draw a flow sheet of water treatment plant for a town of 15 lac population with river
as a source of water supply. Also mention function of each treatment
unit.
b) Write drinking water standards as per BIS : 10500
i) pH
ii) Alkalinity
iii)) Hardness
iv) Turbidity
v) MPN
vi) Chlorides.
3. a) Write a note on backwashing of filter.
b) Design a flocculator for a flow of 10 mld. Assume suitable data.
4. a) Explain chemistry of chlorination.

7
6

5
8
5

b) A settling tank is designed for an overflow rate of 4000 lit/m2/hr. What percentage
of particles of diameter a) 0.05 mm and b) 0.02 mm, will be removed in this tank at
10C ?
8
5. Write short notes on following :
1) Lime soda process of water softening.

2) Inlet and outlet arrangement for sedimentation tank.

3) Fire demand.

Set A

SLR-BB 46

-4-

*SLRBB46*

SECTION II
Instruction : Question No. 6 is compulsory in Section II and solve any two questions
from the remaining.
6. a) Explain the advantages and disadvantages of Grid iron system.

b) Explain with neat sketch service connection from the water mains.

7. a) Explain the mass curve method for finding the capacity of a reservoir.

b) Explain the corrosion control measures taken for pipes.

8. a) Explain equivalent pipe method.

b) Analyse the pipe network shown in Fig. k values in the expression for loss of head
hf = kQ2 and AB = 1, BD = CD = 2 AC = 3 and AD = 2, find discharge through each
pipe and indicate direction of flow. Take two trials

9. Write short note (any three) :

12

i) Water hammer
ii) Air relief valve
iii) GI pipes
iv) Balancing heads.
_____________________

Set A

SLR-BB 47

*SLRBB47*
Seat
No.

Set

T.E. (Civil) (Part I) (New) Examination, 2014


ENGINEERING MANAGEMENT I
Day and Date : Friday, 12-12-2014
Time : 3.00 p.m. to 6.00 p.m.

Max. Marks : 100

Instructions : 1) Q. No. 1 is compulsory. It should be solved in first 30 minutes


in Answer Book Page No. 3. Each question carries one mark.
2) Answer MCQ/Objective type questions on Page No. 3 only.
Dont forget to mention, Q.P. Set (A/B/C/D) on Top of Page.
Duration : 30 Minutes

MCQ/Objective Type Questions

Marks : 20

1. Choose the correct answer :


20
1) Materials in material management consist of _____________
a) Purchased material
b) Finished goods
c) Work in progress materials
d) All of these
2) The cost of keeping items in inventor is called ___________
a) Setup cost
b) Finished goods
c) Holding cost
d) None of these
3) If primal is maximize 40X1 + 35X2, subjected to 2X1 + 3X2 60 and
4X1 + 3X2 96, X1, X2 0 the dual will have objective function as ______
b) 3y1 3y2 (maximize)
a) 3y1 + 3y2 (minimize)
d) 60y1 96y2 (maximize)
c) 60y1 + 96y2 (minimize)
4) Hungerian trial and error method is suitable for solving
a) Transportation problem
b) Assignment problem
c) Games theory
d) Simplex method
5) EOQ decreases when the cost of item
a) Increases
b) Decreases
c) Zero
d) None of these
6) An LPP is said to be _______ if has no solution that satisfies all the constraints.
a) Feasible
b) Bounded
c) Infeasible
d) None of these
7) Transportation problem can be solved if _________
a) No. of rows = no. of columns
b) No. of rows no. of columns
c) No. of rows < no. of columns
d) All of these
8) In inventory control theory, the economic order quantity is
a) Average level of inventory
b) Optimum lot size
c) Capacity of a warehouse
d) Lot size corresponding to break-even analysis
P.T.O.

SLR-BB 47
9) EOQ model helps to find
a) Optimum size of order
c) Both of these

-2-

*SLRBB47*
b) Time interval between order
d) None of these

10) Moderate inventory control is sufficient for ______________


a) A class items b) C class items c) B class items d) All of these
11) Motile Carlo simulation method is suitable for
a) Transportation problem
b) Queuing problem
c) Assignment problem
d) None of these
12) In decision tree problems the event mode is shown by
a) Circle
b) Triangle
c) Rectangle
d) Square
13) Queuing theory is
a) Maximization process
c) Optimization process

b) Minimization process
d) Descriptive process

14) Most popular barcode reader used in conveyor system is


a) Wand type reader
b) Gun type reader
c) Fixed scanners
d) None of these
15) Acceptance quality level for inspection in stored normally range between
a) 0.5 3%
b) 50 100%
c) 0 0.5%
d) 20 50%
16) Which of the following statement is correct ?
a) A-B-C analysis is based on Paretos principle
b) Simulation can be used for inventory control
c) Economic order quantity formula ignores variations in demand pattern
d) All of the above
17) The biological process of mutation has inspired
a) Artificial Neural Network
b) Fuzzy number
c) Genetic algorithm
d) Dynamic programming
18) In break even analysis, total cost consists of
a) Fixed cost + sales revenue
b) Variable cost + sales revenue
c) Fixed cost + variable cost
d) Fixed cost + variable cost + profit
19) The maps that form an important part of a disaster managers resource include
a) Topographic maps
b) Demographic maps
c) Land-use maps
d) None of the above
20) In the chronology of disaster the preparatory phase includes
a) Preparedness b) Mitigation
c) Warning
d) All of the above
______________

Set A

*SLRBB47*

SLR-BB 47

-3-

Seat
No.

T.E. (Civil) (Part I) (New) Examination, 2014


ENGINEERING MANAGEMENT I
Day and Date : Friday, 12-12-2014
Time : 3.00 p.m. to 6.00 p.m.

Marks : 80
SECTION I

2. Attempt any three (10 marks each) :


a) A factory is considering three courses of action S1 = Sub-contracting, S2 = Construct
new facilities and S3 = overtime.
Correct choice depends on future demand as low, medium or high with
probabilities as .1, .5, .4.
The cost analysis reveals effect upon the profit as shown. Prepare decision
tree. Indicate most preferred decision and expected value.
Course of action

Profit if
demand is

S1

S2

S3

Low P = .1

10,000

20,000

1,50,000

Med. P = .5

50,000

60,000

20,000

High P = .4

50,000

1,00,000

2,00,000

b) Solve the assignment problem to minimize the time. Matrix represents process
time in hours.

Job

P
Q
R
S
T

1
10
7
13
12
8

Operators
2
3
4
12
15
12
16
14
14
14
7
9
10
11 13
13
15
11

5
8
11
9
10
15

c) Solve the following transportation problem using least cost method.

Demand

19
70
40
5

30
30
08
8

50
40
70
7

10
60
20
14

Supply
7
9
18

Set A

SLR-BB 47

-4-

*SLRBB47*

d) Solve the following LPP using simplex method.


Max. Z = 6x1 + 3x2
S/t 2x1 + x2 8
3x1 + 3x2 18
x2 3
x1, x2 0.
e) Write notes on :
1) Waiting line theory
2) Dynamic programming.
3. Write notes on any two (5 marks each) :
a) Duality in LPP
c) Evolution of management

b) Simulation
d) Game theory.
SECTION II

4. Solve any five (8 marks each) :


a) Calculate EOQ with following data :
i) Annual usage 36,000/ii) Procurement cost Rs. 25 per order
iii) Cost per price Rs. one/item
iv) Inventory carrying cost 20%
Also calculate total cost of inventory.
b) Explain with example role of mitigation in Disaster Management.
c) What are responsibilities of a storage management ? Discuss with example.
d) Comment on Importance of material management.
e) What is meant by SQC ? How X and R chart are used for quality control.
f) Write notes on :
1) Quality circle
2) TQM.
g) Discuss what precautions you will take after disaster has happened ? Take
case study as flood in Kashmir.
h) Calculate BEP from following data and explain its significance in two cases.
1) By without considering profit
2) By considering profit
1) Profit ` 40,000/2) Fixed cost ` 2,00,000/3) Sale price ` 20 per unit
4) Variable cost ` 2 per unit.
_____________________

Set A

SLR-BB 48

*SLRBB48*
Seat
No.

Set

T.E. (Civil) (Part I) (New) Examination, 2014


TRANSPORTATION ENGINEERING I
Day and Date : Monday, 15-12-2014
Time : 3.00 p.m. to 6.00 p.m.

Max. Marks : 100

N.B. : 1) Q. No. 1 is compulsory. It should be solved in first 30 minutes


in Answer Book Page No. 3. Each question carries one mark.
2) Answer MCQ/Objective type questions on Page No. 3 only.
Dont forget to mention, Q.P. Set (A/B/C/D) on Top of Page.
MCQ/Objective Type Questions
Duration : 30 Minutes

Marks : 20

1. Choose the correct answer (one mark each) :


1) Which of the following is not included in PIEV time of driver ?
a) Perception time
b) Volition time
c) Interfering time
d) Emotion time
2) OSD is required for
a) one way traffic in two lane road
c) both a) and b)

b) two way traffic in four lane road


d) none

3) Bottom most layer of pavement is called as


a) wearing course b) base course c) subgrade

d) none

4) Rapid curing cutback bitumen is produced by blending bitumen with


a) kerosene
b) benzene
c) diesel
d) petrol
5) MDR means
a) Minor District Road
c) Maintained District Road

b) Minor Development Road


d) None

6) The terrain may be classified as rolling terrain if cross slope of kind is between
______________ %.
a) 10 to 25
b) 25 to 50%
c) 50 to 75
d) 75 to 90
7) The background colour of informatory sign board is
a) red
b) white
c) green

d) yellow
P.T.O.

SLR-BB 48

*SLRBB48*

-2-

8) Classification of bridge depending on function is


a) aqueduct
b) pedestrian
c) pipe line

d) all the above

9) The vertical cutting of river bed is known as


a) afflux
b) waterway
c) scour

d) none

10) The maximum width of vehicle as recommended by IRC is


a) 1.85 m
b) 4.72 m
c) 3.81 m
d) 2.5 m
11) Design of horizontal curves on highways is based on
a) Permissible maximum super elevation
b) Design speed of vehicle
c) Permissible friction on the road surface
d) All the above
12) First twenty year road development plan is called as __________ plan.
a) Bombay
b) Nagpur
c) Lucknow
d) Delhi
13) The maximum thickness of expansion joint in rigid pavement is _____ mm.
a) 5
b) 25
c) 50
d) 100
14) On earth road with heavy rainfall area, preferable camber should be _________
a) 1 : 10
b) 1 : 15
c) 1 : 20
d) 1 : 25
15) Normal scour depth in case of a quasi-alluvial streams for contracted
1
3

Q
water-way is determined by formula D = _____________ R .
f

a) .473

b) .373

c) .273

d) .173

16) Needle beam method is used for ____________ ground.


a) soft
b) hard
c) both a) and b) d) none
17) In case of RUB
a) Road passes under bridge
c) Both a) and b)

b) Road passes over bridge


d) None

18) The explosive required for one meter of hard rock is


a) 2 kg
b) 5.7 kg
c) 10 kg

d) none

19) In linear plate method for tunneling in soft rock, standard size of plate is
a) 40 48 cm
b) 23 22 cm
c) 42 30 cm
d) none
20) In tunneling ventilation can be done using
a) mechanically
b) artificially
c) both a) and b)
d) none
______________
Set A

*SLRBB48*

-3-

SLR-BB 48

Seat
No.

T.E. (Civil) (Part I) (New) Examination, 2014


TRANSPORTATION ENGINEERING I
Day and Date : Monday, 15-12-2014

Marks : 80

Time : 3.00 p.m. to 6.00 p.m.


N.B. : All questions are compulsory.
SECTION I
2. Attempt any five (8 marks each) :

(85=40)

a) The speed of overtaking and overtaken vehicle are 70 and 40 kmph resp. on
a two way traffic road. If acceleration of over taking vehicle is .99 m/sec2
[take t = 2 sec.]
1) Calculate OSD
2) Mention minimum length of overtaking zone
3) Draw a sketch of overtaking zone and show positions of sign posts.
b) Enlist different modes of transportation. Discuss importance of any two.
c) Explain principles of highway planning.
d) What is super elevation ? State its importance. Derive the formula for the
same.
e) Discuss volume studies and speed studies.
f) Describe traffic control devices.
g) Explain importance of Aggregate impact test and aggregate abrasion test.
h) Define camber. State its importance. Explain different types of cambers.

Set A

SLR-BB 48

*SLRBB48*

-4-

SECTION II
3. Solve any five (8 marks each ) :

(85=40)

a) Compute the radius of relative stiffness of 15 cm thick cement concrete slab


from following data for two cases of k :
1) Modulus of Elasticity = 2,10,000 kg/cm2 of cement concrete
2) Poissons ratio for concrete = .13
3) Modulus of sub grade reaction k :
i) k = 3 kg/cm3
ii) k = 7.5 kg/cm3
b) Discuss the role of temp. stresses and joints in design of rigid pavement.
c) Explain classification of Bridges.
d) What is Highway drainage ? How it is achieved ? State its effects.
e) Describe different types of bridge bearings.
f) State and explain size and shapes of tunnels.
g) What is lining of tunnel ? How and why it is done ?
h) Find the economical span of a T beam bridge from the following data :
Span (m)
Approx cost of one span
of superstructure (Rs.)

4000

16000

36000

50000

Approx cost of one pier


with foundation (Rs.)

26200

26000

26500

26800

_____________________

Set A

SLR-BB 50

*SLRBB50*
Seat
No.

Set

T.E. (Civil) (Part II) Examination, 2014


STRUCTURAL MECHANICS III
Day and Date : Tuesday, 25-11-2014
Time : 10.00 a.m. to 1.00 p.m.

Max. Marks : 100

Instructions : 1) Assume suitable data if required and mention it clearly.


2) Figures to the right indicate full marks.
3) Q. No. 1 is compulsory. It should be solved in first 30 minutes in
Answer Book Page No. 3.
4) Answer MCQ/Objective type questions on Page No. 3 only. Dont
forget to mention, Q.P. Set (A/B/C/D) on Top of Page.

MCQ/Objective Type Questions


Duration : 30 Minutes

Marks : 20

Tick mark the correct answer :


1. In the following fig. 1 determine degree of static indeterminency

fig. 1
a) 2

b) 4

c) 3

d) 1

2. A two hinge parabolic arch when subjected to udl on the entire span is subjected to
a) Normal thrust only
b) Radial shear only
c) Normal thrust and bending moment
d) Normal thrust, bending moment and radial shear

3. If a unit force is acting on a structure producing displacement, the flexibility of the structure will be
a) Inverse of displacement value
b) Equal to displacement value
c) 1
d) Zero

4. A fixed beam of length 4m carries a point load of 120 KN at 1m from left support. The fixed end
moments are
1
a) 67.5 KN, 22.5 KN
b) 65.5 KNm, 25.5 KNm
c) 87.5 KNm, 52.9 KNm
d) 67.5 KNm, 27.5 KNm
5. Strain energy due to bending of a structure is given by
2

a)

M
EI dx

b)

M2
2EI dx

c)

M
EI dx

1
d)

2M2
EI dx

P.T.O.

SLR-BB 50

*SLRBB50*

-2-

6. Flexibility matrix is
a) Rectangular matrix
b) Unsymmetrical about major diagonal axis
c) Symmetrical about minor diagonal axis d) None of the above

7. For the analysis by the force method the following condition is required
a) Equilibrium equation
b) Compatibility condition
c) Displacements
d) None of these

8. Maxwell reciprocal theorem in structural analysis can be applied in


a) Plastic structures
b) All elastic structures
c) Symmetric structures
d) Prismatic element structures

9. A two hinged semicircular arch of radius R carries a concentrated load W at the crown the
horizontal thrust is
a) W 3
b) W
c) W 4
d) 8W

10. The principle of superposition is applicable when


a) The action of applied forces will be affected by small deformations of the structure
b) Material obeys hooks law
c) Deflections are linear functions of applied force
d) All of the above

11. Slope deflection method is


a) Equilibrium method
c) Stiffness coefficient method

1
b) Deformation method
d) All of the above

12. The slope deflection equations give the relationship between


a) Slope and deflection only
b) B. M. and rotation only
c) B. M. and vertical deflection only
d) B. M. rotation and deflection

13. In portal frames, sway is produced due to ___________


a) Eccentric loading on the frame
b) Horizontal loading on the column
c) Different end condition of column
d) All of the above

14. Stiffness matrix is


a) Unsymmetric about major diagonal axisb) Rectangular matrix
c) Symmetric about major diagonal axis d) All of above

15. In the displacement method of analysis the basic unknowns are


a) Displacement and forces
b) Displacements
c) Forces
d) None of these

16. In a linearly elastic structural element


a) Stiffness is equal to flexibility
b) Stiffness is inversely proportional to flexibility
c) Stiffness is directly proportional to flexibility
d) Stiffness is not related to flexibility

17. Degree of kinematic indeterminacy of a fixed beam is


a) Four
b) Two
c) Zero

d) Six

18. The fixed end moment MFAB for following beam is _________

a) 13.33 kNm

b) 16.66 kNm

c) 40 kNm

2
2

d) 10 kNm

______________
Set A

*SLRBB50*

SLR-BB 50

-3-

Seat
No.

T.E. (Civil) (Part II) Examination, 2014


STRUCTURAL MECHANICS III
Day and Date : Tuesday, 25-11-2014
Marks : 80
Time : 10.00 a.m. to 1.00 p.m.
Instructions :
1) In Section I, solve any three questions.
2) In Section II, Question No. 5 is compulsory, solve any two from
remaining.
3) Assume suitable data if required and mention it clearly.
4) Figures to the right indicate full marks.
SECTION I
1. Solve any three:
(43=12)
a) Determine degree of static and kinematic indeterminency of the structures as shown in
fig. 1(a) and (b).

Figure 1(a)

Figure 1(b)

b) Distinguish between force and displacement method.


c) Explain castiglianos second theorem with its application.
d) State properties of flexibility matrix.
2. Draw BMD for the following beam in fig. 2 using consistent displacement method.

14

Figure 2

3. Analyse the frame shown in fig. 3 and draw BMD using energy method.

14

Figure 3

4. Analyse continuous beam as shown in fig. 4 using flexibility matrix method.

Figure 4

14

Set A

SLR-BB 50

-4-

*SLRBB50*

SECTION II
5. Solve any three of the following :

(34=12)

a) Explain displacement method.


b) Define :
i) Distribution factor

ii) Carry over moment

c) Generate the stiffness matrix for the beam with respect to the co-ordinates as shown in
Figure 5.

Figure 5

d) Write a general stiffness equation and explain meaning of each term.


6. Analyze the portal frame loaded as shown in Figure 6 by the slope deflection
method and sketch the BMD.

14

Figure 6

7. Analyze the structure loaded as shown in Figure 7 below by the moment


distribution method and sketch the BMD.

14

Figure 7

8. Generate the stiffness matrix for the beam with respect to the co-ordinates as
shown in Figure 8 below.

Figure 8
_____________________

14

Set A

SLR-BB 500-1

*SLRBB5001*
S

T.E. (All Branches) (Part I) Examination, 2014


SELF LEARNING : PROFESSIONAL ETHICS AND HUMAN VALUES (New)
Day and Date : Tuesday, 16-12-2014
Time : 3.00 p.m. to 5.00 p.m.

Max. Marks : 50

Instructions : 1) Figures at right indicate full marks.


2) Q. No. 1 is compulsory. It should be solved in first
20 minutes in Answer book Page No. 3. Each question
carries one mark.
3) Answer MCQ/Objective type questions on Page No. 3
only. Dont forget to mention, Q.P. Set (A/B/C/D) on Top
of Page.
Duration : 20 Minutes

MCQ/Objective Type Questions

Marks : 10

1. Select the correct answer from the given options/alternatives :

10

1) Breaking the law/norm is called as _____________


a) Crime
b) Punishment
c) Value
d) None of above
2) Dr. Maslow has demonstrated the hierarchy of ___________
a) Beliefs
b) Needs
c) Team
d) None of above
3) Unity of thoughts, words, honest deeds and open mindedness is called as
____________
a) Ethics
b) Moral
c) Integrity
d) None of above
4) Alignment to goals and adherence to ethical principles during the activities
can be referred as ___________
a) Commitment
b) Co-operation
c) Empathy
d) Confidence
5) In SWOT analysis T stands for _______________
a) Temperature
b) Threats
c) Table
d) Teacher
P.T.O.

SLR-BB 500-1

*SLRBB5001*

-2-

6) CSR stands for __________


a) Corporate Social Responsibility
b) Co-operation in Society Representation
c) Class Social Representative
d) None of above
7) H R means _____________
a) Human Resource
c) Head Resource

b) Honorary Responsibility
d) Human Resistance

8) Employees dissatisfaction and downsizing leads to _______________


a) Motivation
b) Low morale
c) Positive attitude
d) None of above
9) Kohlberg theory is related to __________
a) Moral development
b) Human values
c) Motivation
d) Team working
10) Any occupation/job/vocation that requires expertise, skill, knowledge, self
regulation and concerned service to the public is called as _____________
a) Leadership
b) Profession
c) Ethics
d) None of above
______________

Set A

*SLRBB5001*
S

-3-

SLR-BB 500-1

T.E. (All Branches) (Part I) Examination, 2014


SELF LEARNING : PROFESSIONAL ETHICS AND HUMAN VALUES (New)
Day and Date : Tuesday, 16-12-2014

Marks : 40

Time : 3.00 p.m. to 5.00 p.m.


Instructions : 1) Attempt any four from the following questions.
2) Figures at right indicate full marks.
2. Explain the meaning of professional ethics. Also explain objectives of studying
the professional ethics and human values.

10

3. Explain the meaning of human values and morals. Explain the difference between
them.

10

4. What are the engineering ethics ? Explain the approach and scope of the same. 10
5. Explain variety of moral issues.

10

6. What is safety ? Explain the various factors that influence the perception of risk. 10
7. Explain the risk benefit analysis.

10

_____________________

Set A

Set A

SLR-BB 500-2

*SLRBB5002*
Seat
No.

Set

T.E. (Part I) (All Branches) Examination, 2014


ECONOMICS (New)
(Self Learning H.S.S. Course)
Day and Date : Tuesday, 16-12-2014
Time : 3.00 p.m. to 5.00 p.m.

Total Marks : 50

Instructions : 1) Q. No. 1 is compulsory. It should be solved in first 20 minutes


in Answer Book Page No. 3. Each question carries one mark.
2) Answer MCQ/Objective type questions on Page No. 3 only.
Dont forget to mention, Q.P. Set (A/B/C/D) on Top of Page.
MCQ/Objective Type Questions
Duration : 20 Minutes

Marks : 10

1. Choose the correct answer :

10

1) Microeconomics is a study of _________________


A) Aggregates

B) Units

C) Both of them

D) None of them

2) Macroeconomics studies ___________


A) Individual income

B) National income

C) Both above

D) None of the above

3) India adopted New Economic Policy in the year __________


A) 2001

B) 1980

C) 1991

D) Never

4) _____________ is a market where there is single producer/seller.


A) Perfect competition

B) Imperfect competition

C) Oligopoly

D) Monopoly

5) Quantity demanded normally ___________ with increased price as per law


of demand.
A) increases

B) decreases

C) remains constant

D) none of the above

P.T.O.

SLR-BB 500-2

*SLRBB5002*

-2-

6) Saving means that portion of income which is ___________


A) Spent on consumption
B) Deposited in fixed deposit a/c
C) Paid rental of house
D) Purchased a car
7) ______________ bank is empowered to create legal tender money.
A) Commercial Bank

B) Central Bank

C) Both above

D) None of the above

8) International trade is a trade between ____________


.

A) different regions

B) different industries

C) different countries

D) none of the above

9) Balance of payment is adverse when _________


A) import > export

B) import < export

C) import = export

D) none of the above

10) Monetary Policy is formulated and implemented by ___________


A) Commercial Bank

B) Central Bank

C) Central Govt.

D) None of the above


______________

Set A

*SLRBB5002*

-3-

SLR-BB 500-2

Seat
No.

T.E. (Part I) (All Branches) Examination, 2014


ECONOMICS (New)
(Self Learning H.S.S. Course)
Day and Date : Tuesday, 16-12-2014

Marks : 40

Time : 3.00 p.m. to 5.00 p.m.


Instructions : 1) Attempt any 4 out of Q. 2 to Q. 7.
2) All questions carry equal marks.
2. Define Economics and state basic economic problems faced by India presently. 10
3. Write a detailed note on features and achievements of New Economic Policy in
India.

10

4. Explain in details characteristics and price determination in Oligopoly Market.

10

5. What do you mean by National Income ? How it is determined ?

10

6. What do you mean by inflation ? Explain causes and controls of inflation.

10

7. Explain the concepts Monetary and Fiscal Policy and state the distinction therein.

10

_____________________

Set A

Set A

SLR-BB 500-3

*SLRBB5003*
Seat
No.

Set

T.E. (Part I) (All Branches) Examination, 2014


SOCIOLOGY (New)
Introduction to Sociology
(Self Learning)
Day and Date : Tuesday, 16-12-2014
Time : 3.00 p.m. to 5.00 p.m.
N. B. :

Max. Marks : 50

1) Q. No. 1 is compulsory. It should be solved in first 20 minutes


in Answer Book Page No. 3. Each question carries one mark.
2) Attempt any 4 from the remaining questions.
3) Figures to the right indicate full marks.
4) Answer MCQ/Objective type questions on Page No. 3 only.
Dont forget to mention, Q.P. Set (A/B/C/D) on Top of Page.
MCQ/Objective Type Questions

Duration : 20 Minutes

Marks : 10

1. Choose the correct answer :

10

I) Human society is basically a web of __________ relationships.


a) Economic
c) Individual

b) Social
d) Cultural

II) Social institutions are approved by ___________


a) Religion
b) Society
c) Family

d) Education

III) Social change is a change in ____________


a) Social structure
b) Governance
c) Individual

d) Industry

IV) ____________ is primary associated with Lokpal Andolan.


a) Nana Patekar

b) Anna Hazare

c) Rajendra Singh

d) T. N. Kanchan

V) Promotion is an example of _________ mobility.


a) Downward mobility

b) Horizontal mobility

c) Upward mobility

d) None of the above

P.T.O.

SLR-BB 500-3

*SLRBB5003*

-2-

VI) __________ is a primary social institution.


a) Industry

b) Family

c) Education

d) All the above

VII) ___________ is called to be the Father of Sociology.


a) Karl Marx

b) Auguste Comte

c) Max Weber

d) Emile Durkheim

VIII) Urbanization is primary associated with ___________


a) Educational growth
b) Industrialization
c) Growth of non-agricultural production
d) All the above
IX) The emerging concept of Smart City is an example of _______
a) Sanskritization

b) Modernization

c) Growth

d) Cultural change

X) The concept of Sanskritization originated from the term _____________


a) Socialization
c) Modernization

b) Brahminization
d) Westernization
______________

Set A

*SLRBB5003*

-3-

SLR-BB 500-3

Seat
No.

T.E. (Part I) (All Branches) Examination, 2014


SOCIOLOGY (New)
Introduction to Sociology
(Self Learning)
Day and Date : Tuesday, 16-12-2014
Time : 3.00 p.m. to 5.00 p.m.

Marks : 40

N. B. : 1) Attempt any 4 from the remaining questions.


2) Figures to the right indicate full marks.
2. Define the concept of sociology and explain the importance of achieved social
status.

10

3. Define social institution and explain the characteristics of caste institution.

10

4. Elaborate the nature and characteristics of industrialization.

10

5. Define socialization and explain its importance in human life.

10

6. Define urbanization and discuss the trends of urbanization in India.

10

7. Define social movement and explain the objectives of Lokpal Movement in


India.

10

_____________________

Set A

Set A

SLR-BB 500-4

*SLRBB5004*
S

T.E. (All Branches) (Part I) Examination, 2014


Self Learning : STRESS AND COPING (New)
Day and Date : Tuesday, 16-12-2014

Max. Marks : 50

Time : 3.00 p.m. to 5.00 p.m.


Instructions : 1) Q. No. 1 is compulsory. It should be solved in first
20 minutes in Answer book Page No. 3. Each question
carries one mark.
2) Answer MCQ/Objective type questions on Page No. 3
only. Dont forget to mention, Q.P. Set (A/B/C/D) on Top
of Page.
MCQ/Objective Type Questions
Duration : 20 Minutes

Marks : 10

1. A) Choose among the right alternative :

1) The word stress is derived from Latin word Stringere which means
_________
a) Draw tight

b) Stimulus

c) Force

d) Attitude

2) ____________ is the number of people who are available to provide support


to relieve stress.
a) Cognitive Restructuring

b) Social Loafing

c) Self-control

d) Social network

3) ____________ is a technique for transforming some aspects of


physiological behavior into electrical signals like visual/auditory.
a) Meditation

b) Yoga

c) Bio-feedback

d) Problem solving
P.T.O.

SLR-BB 500-4

*SLRBB5004*

-2-

4) _______________ arises when the role occupant feels that he/she lacks
in required skills for effective performance.
a) Personal inadequacy

b) Resource inadequacy

c) Role ambiguity
d) Role expectations
5) Absenteeism, turnover and less productivity are _____________ symptoms
of stress.
a) Medical

b) Psychological

c) Behavioral

d) None of these

B) Match the pairs :


Set A

5
Set B

1) Locus of control

1) Organisational stressors

2) Leadership and structure


3) Anxiety and depression

2) Social support
3) Cognitive restructuring

4) Friends and colleagues

4) Personality

5) Reframing thinking

5) Psychological symptoms
___________________

Set A

*SLRBB5004*
S

-3-

SLR-BB 500-4

T.E. (All Branches) (Part I) Examination, 2014


Self Learning : STRESS AND COPING (New)
Day and Date : Tuesday, 16-12-2014

Marks : 40

Time : 3.00 p.m. to 5.00 p.m.


Note : Solve any 4 from Q. No. 2 to Q. No. 7.
2. Define stress and explain in detail various sources of stress.

10

3. Explain Medical, Psychological and behavioral consequences of stress in detail.

10

4. Elaborate the role of social support in mitigating stress.

10

5. Explain in detail various methods of stress management.

10

6. Discuss the nature of stress response.

10

7. Explain the historical perspective of stress.

10

_____________________

Set A

Set A

SLR-BB 500-5

*SLRBB5005*
Seat
No.

Set

T.E. (All Branches) (Part I) Examination, 2014


SELF LEARNING : INTELLECTUAL PROPERTY RIGHTS FOR
TECHNOLOGY DEVELOPMENT AND MANAGEMENT (New)
Day and Date : Tuesday, 16-12-2014
Time : 3.00 p.m. to 5.00 p.m.
N.B. :

Max. Marks : 50

1) Q. No. 1 is compulsory. It should be solved in first 20 minutes


in Answer Book Page No. 3. Each question carries one mark.
2) Answer MCQ/Objective type questions on Page No. 3 only.
Dont forget to mention, Q.P. Set (A/B/C/D) on Top of Page.
MCQ/Objective Type Questions

Duration : 20 Minutes

Marks : 10

1. Choose the correct answer :


1) The result of intellectual property rights is
a) General agreement on trade and tariff
b) World trade organisation
c) Trade related aspects of intellectual property rights
d) All of the above
2) Intellectual property rights relates to
a) Copy right
b) Incorporeal policy
c) Intangible property
d) Both b) and c)
3) Intellectual property rights are result of
a) Physical work
b) Communication
c) Mental work
d) Technical work
4) Intellectual property rights does not relate to
a) Copy rights
b) Expenditure
c) Trade marks
d) Trade secretes

P.T.O.

SLR-BB 500-5

*SLRBB5005*

-2-

5) Copy rights is guaranteed to protect intellectual efforts in the filed of


a) Literature
b) Sound recording
c) Computer programs
d) All of the above
6) Trade mark patent does not relate to
a) Research method
b) Advertise
c) identify
d) Distinguish
7) The Trade Mark Act of India 1958 a mended and reconstructed in
a) 1999
b) 2002
c) 1995
d) 2005
8) Graphical indications means an indication, which identifies certain goods as
originating or manufactured from a particular
a) Material and quantity
b) Locality with specific quality
c) Natural source
d) None of the above
9) Industrial design relates to
a) Shape
c) Pattern

b) Configuration
d) All of the above

10) In India the industrial design is governed by the design act of


a) 1995
b) 1998
c) 2000
d) 2005
______________

Set A

*SLRBB5005*

-3-

SLR-BB 500-5

Seat
No.

T.E. (All Branches) (Part I) Examination, 2014


SELF LEARNING : INTELLECTUAL PROPERTY RIGHTS FOR
TECHNOLOGY DEVELOPMENT AND MANAGEMENT (New)
Day and Date : Tuesday, 16-12-2014

Marks : 40

Time : 3.00 p.m. to 5.00 p.m.


Instructions : 1) Attempt all questions.
2) Figures to the right indicate full marks.
2. Elaborate the Indian Patent Act, 1970.

10

3. What are essential requirements for granting patent ? Brief in detail.

10

OR
3. Explain the evolution of law on patenting computer programs and software.
4. Write short notes on any four :

20

1) Industrial designs patent


2) Publication and examination of patent applications
3) Trade marks
4) Biotechnology inventions
5) Semiconductor chips and integrated circuit patents
6) Copy rights.
_____________________

Set A

Set A

SLR-BB 51

*SLRBB51*
Seat
No.

Set

T.E. (Civil) (Part II) Examination, 2014


GEOTECHNICAL ENGINEERING II
Day and Date : Wednesday, 26-11-2014
Time : 10.00 a.m. to 1.00 p.m.

Max. Marks : 100

Instructions : 1) Q. No. 1 is compulsory. It should be solved in first 30 minutes


in Answer book Page No. 3.
2) Solve any three questions from each Section.
3) Assume additional data, if required and state it clearly.
4) Figures to the right indicate full marks.
5) Answer MCQ/Objective type questions on Page No. 3
only. Dont forget to mention, Q.P. Set (A/B/C/D) on Top
of Page.
MCQ/Objective Type Questions
Duration : 30 Minutes

Marks : 20

1. 1) Method of locating position of centre of critical slip circle is given by


a) Taylor
b) Terzaghi
c) Fellenious
d) None
2) For toe failure of slope the depth factor should be
a) = 1
b) < 1
c) > 1

1
1

d) > 10

3) Ideal location for the anchor rods in case of anchored sheet pile wall is
a) top of wall
b) bottom of wall
c) near dredge level
d) near top of wall

4) The maximum pressure intensity which a soil can carry without shear failure
is known as its
a) safe bearing capacity
b) ultimate bearing capacity
c) net safe bearing capacity
d) net ultimate bearing capacity
5) It is very difficult to state the limiting condition for which a local or general
shear failure takes place for a given soil. However, which of the following
parameters can be used as a rough guide to determine the type of failure ?
a) density index
b) penetration test
c) angle of shear resistance
d) all of above

P.T.O.

SLR-BB 51

-2-

*SLRBB51*

6) What were the values for soil parameters used by Terzaghi for his local
shear failure analysis ?
1
a) m = 2 tan , Cm = 2 c
b) m = 3 tan , C = 3 c
3
3
4
4
c) m = 1 tan , C = 1 c
d) none of these
2
2
7) A cyclic load test is performed to determine a piles
1
a) ultimate load capacity under repetition
b) skin resistance and base resistance separately
c) skin resistance
d) tip resistance
8) The efficiency of pile group depends on
1
a) soil type
b) method of pile installation
c) pile spacing
d) all of these
9) Geophysical surveys are not useful for
1
a) large areas
b) complex boundary layers
c) underground cavities
d) locating water tables
10) Sand drains are provided
1
a) to accelerate the consolidation process
b) to increase the rate of gain of shear strength
c) in saturated clays
d) for all of these
11) Three piles are arranged in triangular form; efficiency of this pile group by
Felds rule is
2
a) 33.33%
b) 50%
c) 75%
d) 87.5%
12) If the diameter of pile is doubled then bearing component of pile capacity will
be increased by _________ times.
2
a) one
b) two
c) three
d) four
13) Critical height for the slope of height 10 m made of clayey soil having
c = 25 kPa and = 19 kN/m3.
Assuming stability number 0.064 is
2
a) 10 m
b) 15 m
c) 20.6 m
d) 26.0 m
14) Position of centroid of trapezoidal footing having parallel sides 1 m and 2 m,
distance between parallel sides 4.5 m from the shorter side is
2
a) 2.5 m
b) 2 m
c) 3 m
d) none
2
15) The ultimate bearing capacity of a soil is 310 kN/m , the depth of foundation
is 0.8 m and is 20 kN/m2. If the factor of safety is 3, what will be the net
safe bearing capacity ?
2
2
2
2
2
a) 98.00 kN/m
b) 98.86 kN/m
c) 00.98 kN/m
d) 100.86 kN/m
______________
Set A

*SLRBB51*

-3-

SLR-BB 51

Seat
No.

T.E. (Civil) (Part II) Examination, 2014


GEOTECHNICAL ENGINEERING II
Day and Date : Wednesday, 26-11-2014
Time : 10.00 a.m. to 1.00 p.m.

Marks : 80

Instructions : 1) Solve any three questions from each Section.


2) Assume additional data, if required and state it clearly.
3) Figures to the right indicate full marks.
SECTION I
2. A) Explain General shear failure and Local shear failure. How do you ascertain
whether a foundation soil is likely to fail in General shear failure or Local
shear failure ?

B) A circular footing is resting on a stiff saturated clay with unconfined


compressive strength of 250 kN/m2. The depth of foundation is 1.8 m.
Determine the diameter of the footing if the column load is 800 kN. Assume a
factor of safety as 3. The bulk unit weight of soil is 20 kN/m3.

3. A) Explain the guidelines of IS code to conduct the plate load test.

B) Explain different types of augurs with neat sketch.

4. A) What do you mean by collapsible soil ? What care has to be taken while
doing construction in this type of soil ?

B) What do you understand by reinforced earth ? Explain the mechanism of its


action in resisting the active earth pressure.
5. A) State the assumptions in the Terzaghis bearing capacity analysis. Also
emphasize the limitations of this theory.
B) Write a note on :
i) Collapsible soil
ii) Corrosive soil.

6
Set A

SLR-BB 51

-4-

*SLRBB51*

SECTION II
6. A) Differentiate shallow and deep foundation. Explain different types of shallow
foundation with their suitability.
B) Write in detail the design principle of Combine footing.

7
6

7. A) Draw sectional elevation of pneumatic caisson. Explain the function of each


component.

B) What are tilts and shifts of well foundation ? How tilts and shifts are rectified ? 6
8. A) Find depth of embedment for the anchored sheetpile wall from following data :
i) Height of wall 6.5 m
ii) Anchor rods are at a distance of 1.2 m below ground surface
iii) Soil above dredge level and below dredge level granular.
iv) Properties of granular soil (c = 0, = 33, = 20.6 kN/m3).

B) Write a short note on negative skin friction.

9. A) Check the stability of a slope by using friction circle method by using following
data :
i) Height of slope 12 m
ii) Inclination of slope 35 degree with horizontal
iii) Slip circle radius 20 m through toe of slope and cut the top at a horizontal
distance of 4 m from top edge of slope
iv) C.G. of slope is at a distance of 11 m from centre of rotation
v) Properties of soil (c = 30 kPa, = 10, = 20 kN/m3).
B) Write short note on Taylors stability number.

10
4

_____________________

Set A

SLR-BB 510

*SLRBB510*
S

S.E. (E &TC) (Part I) Examination, 2014


DATA STRUCTURES (Old)
Day and Date : Thursday, 18-12-2014
Time : 10.00 a.m. to 1.00 p.m.

Max. Marks : 100

Instructions : 1) Figures to the right indicates full marks.


2) Assume suitable data if necessary.
3) Q. No. 1 is compulsory. It should be solved in first 30
minutes in Answer book Page No. 3. Each question carries
one mark.
4) Answer MCQ/Objective type questions on Page No. 3
only. Dont forget to mention, Q.P. Set (A/B/C/D) on Top
of Page.
Duration : 30 Minutes

MCQ/Objective Type Questions

Marks : 20

1. Choose the correct answer :


(201=20)
1) Overflow condition of Queue is ____________
A) rear = = max-1
B) front = = max-1
C) top = = max-1
D) both A) and B)
2) ___________________ is the application of stack.
A) Execution of sub program
B) Conversion of infix to prefix
C) Both A) and B)
D) None of the above
3) The nodes of trees are called siblings
A) if they have different parents
B) if they have one parent
C) if it is a root node
D) none of the above
4) The length of the longest path from root node is known as ___________
A) depth of tree
B) length of tree
C) width of tree
D) none of the above
5) The degree of node is ___________
A) the number of arcs incident to it
B) the number of arcs that have node as head
C) the number of arcs that have node as tail
D) none of above
6) Graph is ___________ data structure.
A) linear
B) non linear
C) both A) and B) D) none of above
P.T.O.

SLR-BB 510

-2-

*SLRBB510*

7) Quick sort is also known as ___________


A) fast sort
B) insertion sort
C) partition exchange sort
D) none
8) One way to represent graph in computer memory is ___________
A) stack
B) queue
C) adjacency matrix
D) none
9) Following sequence of operation is performed on stack. Push (1), Push (2),
Pop, Push (1), Push (2), Pop, Pop, Pop, Push (2), Pop. The sequence of
popped out values are ___________
A) 22122
B) 22112
C) 21221
D) 21222
10) A binary tree in which every non-leaf node has non-empty left and right sub
trees is called a strictly binary tree. Such a tree with 4 leaves has ___________
nodes.
A) 7
B) 6
C) 9
D) 5
11) Linked list are not suitable for ___________
A) insertion sort
B) binary search
C) radix sort
D) polynomial manipulation
12) The faster searching technique is ___________
A) linear search B) binary search C) both B) and C) D) none
13) Merge sort merges two ___________ list.
A) sorted list
B) unsorted list C) linked list
D) none of above
14) The process of accessing data stored in tape is similar to manipulating data
on a ___________
A) stack
B) queue
C) array
D) heap
15) The number of possible binary trees with 3 nodes are ___________
a) 1
B) 4
C) 6
D) 5
16) Traversing a binary tree first left, then root and then right subtree called as
___________ traversal.
A) postorder
B) inorder
C) preorder
D) none of above
17) In Heap, element with the greatest key is always in the ___________ node.
A) Leaf
B) Internal
C) Root
D) None of the above
18) How many fields are contained in each node of Doubly linked list ?
a) At least two
B) Exactly two C) Three
D) One
19) Merge sort uses ___________
A) Divide and conquer strategy
B) Back tracking approach
C) Greedy approach
D) None
20) The level of first node of tree is ___________
A) Zero
B) One
C) Two
D) Three
______________
Set A

*SLRBB510*
S

-3-

SLR-BB 510

S.E. (E &TC) (Part I) Examination, 2014


DATA STRUCTURES (Old)
Day and Date : Thursday, 18-12-2014
Time : 10.00 a.m. to 1.00 p.m.

Marks : 80

Instructions : 1) All questions are compulsory.


2) Figures to the right indicate full marks.
3) Assume suitable data if necessary.
SECTION I
2. Answer any four :

(54=20)

1) Give the difference between singly, doubly and circular linked list.
2) Define binary search tree. Explain representation of binary search tree using
array in computer memory.
3) Write an algorithm to perform Push and Pop operation of stack.
4) Give the advantages and disadvantages of B+ tree over B tree.
5) Define data structure. Enlist its types. Give different applications of tree data
structure.
3. Answer any two :

(102=20)

1) What is B tree ? State the properties of B tree. Draw B tree of order 3 for
following elements.
10, 24, 23, 31, 16, 26, 35, 29, 20, 46.
2) Write a program to implement singly linked list for performing following
operations :
a) Insert at the end
b) Insert at specified location
c) Delete at specified location
d) Display list elements.
3) What is linked list ? Enlist its types and explain them in detail.
Set A

SLR-BB 510

-4-

*SLRBB510*

SECTION II
4. Answer any four :

(54=20)

1) Define graph. Explain the following terms with respect to graph.


a) Directed graph
b) Undirected graph
c) Weighted graph
d) Cycle.
2) Explain the bubble sort technique for sorting an array of ten elements.
3) What is purpose of big O notation ?
4) Sort the following using merge sort.
12, 44, 32, 65, 15, 24, 39, 19
5) Write the searching algorithm of binary search.
5. Answer any two :

(102=20)

1) What are different tree traversal methods ? Explain any one in detail with
example.
2) What is meant by hash collision ? Explain the types of collision resolving
techniques in detail.
3) What is recursion ? Write a recursive program for factorial by showing
recursive calls.
_____________________

Set A

SLR-BB 52

*SLRBB52*
Seat
No.

Set

T.E. (Civil) (Part II) Examination, 2014


ENVIRONMENTAL ENGINEERING II

Day and Date : Thursday, 27-11-2014


Time : 10.00 a.m. to 1.00 p.m.

Total Marks : 100

Instructions : 1) Solve any three questions from Section I,. i.e. Question No. 2 to
Question No. 5. Question no. 6 is compulsory in Section II
and solve any two questions from the remaining.
2) Figures to the right indicate full marks.
3) Assume suitable data wherever necessary and mention it clearly.
4) Use of non programmable calculator is allowed.
5) Q. No. 1 is compulsory. It should be solved in first 30 minutes in
Answer Book Page No. 3. Each question carries one mark.
6) Answer MCQ/Objective type questions on Page No. 3 only. Dont
forget to mention, Q.P. Set (A/B/C/D) on Top of Page.
MCQ/Objective Type Questions
Duration : 30 Minutes

Marks : 20

1. Choose the correct option:


1) The self cleansing velocity in siphons should be _________
a) 3 m/s
b) 1 m/s
c) 0.5 m/s

(201=20)
d) 2.1 m/s

2) __________ process can be used of removal of nitrogen in wastewater treatment.


a) Coagulation
b) Ammonia striping
c) Carbon adsorption
d) Reverse osmosis
3) A velocity control device is necessary to be provided in a ______________
a) Trickling filter
b) Rapid sand filter
c) Aeration tank
d) Grit chamber
4) Detention period adapted for oxidation pond is ___________
a) 2 6 hrs
b) 4 8 hrs
c) 12 36 hrs

d) 2 4 days

5) BOD5 represent 5 days biochemical oxygen demand at temperature of _______


a) 0C
b) 20C
c) 30C
d) None of these
6) When there is no recirculation of sewage then circulation factor is _______
a) zero
b) one
c) infinity
d) none of these
7) Algae dies out, through fishes life may survive, in a river zone, called ______
a) Zone of degradation
b) Zone of active decomposition
c) Zone of recovery
d) None of these
8) The specific gravity for grit particle is ____________ at 15.5c temperature of wastewater.
a) 2.95
b) 2.65
c) 3.0
d) 1.7
P.T.O.

SLR-BB 52

*SLRBB52*

-2-

9) The decomposition of organic matter produced by gases in domestic waste water causes.
a) Algal growth
b) Anaerobic decomposition
c) Odour
d) Biosolids
10) Turbulent flow occurs in wastewater due to _____________
a) Continuous flow
b) Laminar flow
c) Radial flow
d) Rapid mixing
11) ______________ is a waste treatment process that involves the combustion of organic
substances contained in waste materials.
a) Composting
b) Incineration
c) Sanitary landfill
d) Sorting
12) _______________ is a proven management for incorporating environmental concerns in
development process and decision making.
a) Environmental management
b) Sustainable management
c) Environmental impact assessment
d) Life cycle assessment
13) Effective waste management system includes _______________
a) Resource recovery through waste processing
b) Erosion and stability problem
c) Centralized sorting
d) Landfill method
14) ___________ system is most widely adopted for thermal treatment process in municipal
solid waste.
a) Aerobic process
b) Landfill
c) Anaerobic digestion
d) Combustion
15) ______________ is a main source of fuel combustion.
a) Hydrogen sulphide
b) Sulphur dioxide
c) Hydrogen chloride
d) Ozone
16) ____________ are used to remove particulates above 40 diameter.
a) Bag filter
b) Settling chamber
c) Cyclone separator
d) Scrubber
17) ___________ particle in finely divided state and capable of temporary suspension in air.
a) Particulate matter
b) Contaminants
c) Gases
d) Carcinogen
18) Natural sources of air pollution come from ____________
a) internal combustion engines
b) coal-fired electrical plants
c) soil dust, volcanic dust
d) birds
19) ____________ is a primary pollutant.
a) Ozone
c) Carbon monoxide

b) Photochemical smog
d) Formaldehyde

20) ____________ is a control device that traps gas borne particulates by forcing gas.
a) Bag house filter
b) Scrubber
c) Separator
d) Precipitators
______________
Set A

*SLRBB52*

-3-

SLR-BB 52

Seat
No.
T.E. (Civil) (Part II) Examination, 2014
ENVIRONMENTAL ENGINEERING II

Day and Date : Thursday, 27-11-2014


Time : 10.00 a.m. to 1.00 p.m.

Marks : 80

Instructions : 1) Solve any three questions from Section I,. i.e. Question No. 2 to
Question No. 5. Question no. 6 is compulsory in Section II
and solve any two questions from the remaining.
2) Figures to the right indicate full marks.
3) Assume suitable data wherever necessary and mention it clearly.
4) Use of non programmable calculator is allowed.
SECTION I
2. a) Explain an inverted siphon with a neat sketch.

b) Design a primary sedimentation tank of circular shape for a town having population of
80,000 with a water supply of 200 liters per capita per day.

3. a) Calculate the velocity of flow and discharge in a sewer of circular section having diameter
1.2 m laid at gradient of 1 in 400. Use Mannings N = 0.012. Assume that sewer runs at
0.6 depth.

b) The 5-day BOD of waste is 280 mg/l. The ultimate BOD is reported as 410 mg/l. At what
rate is the waste being oxidized ?

4. a) Explain the activated sludge process of waste water treatment.


b) Design a high rate trickling filter for following data :
i) Sewage flow 3 mld
ii) Recirculation ratio 1.5
iii) BOD of raw sewage 210 mg/l
iv) BOD removal in primary clarifier 30%
v) Final effluent BOD desired 25 mg/l
5. a) Explain Oxidation pond in detail.
b) Write short notes on :
i) Septic Tank
ii) Anaerobic Pond.

6
7

6
8

Set A

SLR-BB 52

*SLRBB52*

-4-

SECTION II
6. a) Draw a neat sketch showing the zones of pollution in stream.

b) Explain trenching method of composting.

7. a) Using following data find out DO at the end 2 days :


River

Wastewater

Flow (m3/s)

30

DO (mg/1)

9.17

200

5-day BOD (mg/1)

Take deoxygenation constant as 0.1 per day and reoxygenation constant as 0.3 per day.
Take saturation DO as 9.17 mg/1.
b) Explain the incineration method of Solid waste disposal. Mention its advantages.
8. a) Explain the working of gravity settling chamber with neat sketch.
b) Explain the effect of sulphur dioxide on human health and materials.
9. Write short notes on any three :

7
7
7
7
14

i) Control of Vehicular air pollution


ii) Solid waste management
iii) D.O. Sag Curve
iv) Sources of air pollution.
_____________________

Set A

SLR-BB 53

*SLRBB53*
Seat
No.

Set

T.E. (Civil) (Part II) Examination, 2014


ENGINEERING MANAGEMENT II
Day and Date : Friday, 28-11-2014
Time : 10.00 a.m. to 1.00 p.m.

Max. Marks : 100

Instructions : 1) Q. No. 1 is multiple choice type for 20 marks.


2) In Section I, Q. No. 2 is compulsory. Solve any two out of
remaining three questions.
3) Section II consists of Q. No. 6 to Q. No. 9. Solve any three out
of these four questions.
4) Use of nonprogrammable engineering calculator is allowed.
5) Q. No. 1 is compulsory. It should be solved in first 30 minutes in
Answer book Page No. 3. Each question carries one mark.
6) Answer MCQ/Objective type questions on Page No. 3 only.
Dont forget to mention, Q.P. Set (A/B/C/D) on Top of Page.
MCQ/Objective Type Questions
Duration : 30 Minutes

Marks : 20

1. Make tick mark against correct answer option/options only, for the following questions. 20
1) Work break down structure helps in
a) Identifying the activities
b) Identifying the functional element of a project and their interrelationship
c) Breaking the project into several elements
d) Breaking the project into system
2) One of the main disadvantages of the bar chart is
a) Progress of the work cannot be monitored
b) Does not show the inter dependencies of the activities
c) The time schedule is not shown properly
d) The financial aspect is not shown
3) The decision networks are represented by
a) A-O-N diagram b) A-O-A diagram c) Both (a) and (b) d) None of these
4) PERT (Programme Evaluation and Review Technique) is mainly useful
a) Only small projects
b) Only large projects
c) Research and development
d) Small as well as large and complex projects
5) There is no need for updating if
a) All the activities are not progressing according to schedule
b) All the activities are partially progressing according to schedule
c) All the activities are progressing well according to schedule
d) All of these
P.T.O.

SLR-BB 53

-2-

*SLRBB53*

6) The stage of the work at which cost control is not effected is at


a) Planning stage
b) Designing stage
c) Construction stage
d) None of these
7) Resource smoothening means
a) Optimization and economical utilization of resources
b) Complete revamping of resources to suit the requirements
c) Gradual increase in resources
d) Adjustment of resources to have the least variations
8) Construction costs mainly consists of expenditure on
a) Labour and materials
b) Plants and equipments
c) Overheads
d) All of these
9) Precedence networks are more commonly used when
a) Repetitive tasks are involved
b) Overlapping of activities occur
c) Interdependencies exist
d) All of these
10) The reduction in project time normally results in
a) Decreasing the direct cost and increasing the indirect cost
b) Increasing the direct cost and decreasing the indirect cost
c) Increasing both direct and indirect costs
d) Decreasing both direct and indirect costs
11) BOT means
a) Built Own and Transfer
b) Buy Operate and Transfer
c) Built Operate and Transfer
d) None
12) If an interest rate is being compounded quarterly, the term m in formula
i = (1 + r/m)1 is equal to
a) 4
b) 3
c) 2
d) 1
13) ________ is the discounting method of economic comparison.
a) PWM
b) Pay back period
c) Break even analysis
d) Return of investment
14) If the bike is sold before its useful life we will get ____________
a) Scrap value
b) Salvage value c) Esteem value d) None
15) When demand is more than supply, the situation will be ____________
a) Inflation
b) Depreciation c) Deflation
d) None
16) If amount is to be deposited in the bank then __________ method of compounding
gives maximum benefit to the depositor.
a) Compounded monthly
b) Compounded biannually
c) Compound yearly
d) Compounded continuously
17) USSFF means uniform series
a) Sinking Fast Factor
b) Starting Fund Factor
c) Sinking Fund Factor
d) Starting Fast Factor
18) In P Chart the word P means __________
a) Percentage
b) Process
c) Proportion
d) None
19) The uniform amount to be invested at the end of each period in order to produce a
fixed amount can be calculated using __________
a) U S C A F
b) U S C R F
c) U S P W F
d) U S S F F
20) For Variable Sampling following chart is used __________
a) X chart
b) C chart
c) P chart
d) None
______________
Set A

*SLRBB53*

-3-

SLR-BB 53

Seat
No.

T.E. (Civil) (Part II) Examination, 2014


ENGINEERING MANAGEMENT II
Day and Date : Friday, 28-11-2014
Time : 10.00 a.m. to 1.00 p.m.

Marks : 80

Instructions : 1) In Section I Q. No. 2 is compulsory. Solve any two out of


remaining three questions.
2) Section II consists of Q. No. 6 to Q. No. 9. Solve any three out
of these four questions.
3) Use of nonprogrammable engineering calculator is allowed.
SECTION I
2. a) Draw a bar chart for the following project data :

Table 1
Activity

Immediate
Predecessors

Duration
(days)

E,F

I,J

Determine the project completion time.


b) Draw CPM network and indicate critical path for the project data indicated in the
Table 1. Determine project completion time.
9
Set A

SLR-BB 53

*SLRBB53*

-4-

3. a) Draw the PERT network for following data of a project.

Table 2
Duration (days)
Activity Immediate Predecessors
Optimistic Most Likely Pessimistic
A

4.5

0.5

1.5

B,C

B,C

1.5

E,F

E,F

D,H

2.5

2.75

4.5

G,I

b) Calculate variance and expected time for each of the activities in PERT network
as per Table 2. Calculate slack for each of the activities. Calculate variance of
the critical path. Also calculate duration of critical path.

4. a) Explain time cost relationship with the help of Time-Cost trade off graph. Indicate
typical curves for direct cost, indirect cost, total cost. Locate minimum total cost
and optimal project duration.
b) Write note on :

5
(4+4=8)

A) Resource smoothening
B) Updating of the network.
5. a) Explain precedence network with suitable sketch.
b) Explain work measurement.

5
8
Set A

*SLRBB53*

SLR-BB 53

-5-

SECTION II
6. a) Make comparison between Present Worth Method (PWM) and Equivalent Annual
Cost (EUAC) method of economic comparison.
b) Briefly explain the control charts used for variables and attributes.

8
5

7. a) Draw cash flow diagram and derive an expression for Capital Recovery
Factor (CRF).

b) Differentiate between the following :


1) Nominal and effective rate of interest.
2) Discrete and continuous compounding.

c) If credit plan charges interest rate of 18% per year compounded monthly, then
what is its effective interest rate ?

8. a) Explain the Interest Rate of Return (IRR) method used for economic comparison.

b) Compare the machines below on the basis of their present worth using an interest
rate of 18% per year.

Machine A Machine B
i) Initial cost (in Rs.)
ii) Salvage value (in Rs.)
iii) Life in years
iv) Annual maintenance cost (in Rs.)

8 lakhs

11 lakhs

1.5 lakhs

02 lakhs

75,000

90,000

9. Write a note on :
a) Time Value of Money.

b) Total Quality Management.

c) Statistical Quality Control.

5
_______________

Set A

Set A

SLR-BB 54

*SLRBB54*
Seat
No.

Set

T.E. (Civil) (Part II) Examination, 2014


TRANSPORTATION ENGINEERING II
Day and Date : Saturday, 29-11-2014
Time : 10.00 a.m. to 1.00 p.m.

Max. Marks : 100

Instructions : 1) Q. No. 1 is compulsory. It should be solved in first 30 minutes


in Answer Book Page No. 3. Each question carries one mark.
2) Answer MCQ/Objective type questions on Page No. 3 only.
Dont forget to mention, Q.P. Set (A/B/C/D) on Top of Page.
Duration : 30 Minutes

MCQ/Objective Type Questions

Marks : 20

1. Choose the correct answer :


20
1) Single slip consists of
a) two sets of points
b) three sets of points
c) single set of points
d) one point
2) The angle formed between the gauge line of the stock rail and tongue rail is
known as
a) switch angle
b) switch
c) crossover
d) none of the above
3) At the dead ends of sidings to prevent escape of vehicles ________ is provided.
a) switch
b) cross over
c) buffer stop
d) none of the above
4) The best material for performing the functions of a ballast in a railway track is
a) Gravel
b) Cinders
c) Sand
d) Broken Stone
5) The rails are connected at their ends by
a) Fish plate
b) Bearing plate c) Base plate
d) None of above
6) Find out the number of sleepers for rails of 13 meter length with sleeper
density as (n + 3)
a) 12
b) 13
c) 14
d) 16
7) If ruling gradient is 1 in 200 an a particular section of B.G. and if a curve of 4
is situated on this ruling gradient, what should be actual ruling gradient ?
a) 12%
b) 22%
c) 25%
d) 0.34%
8) While laying rails ____________ is placed between the two rails to provide
for the necessary expansion gap.
a) joint
b) spikes
c) shim
d) none of the above
9) The difference in cross levels of tack is known as
a) Buckles
b) Twist
c) Turn
d) None of the above
P.T.O.

SLR-BB 54

*SLRBB54*

-2-

10) Safe speed on N.G. is given by


a) 4.4 R 70

b) V = 3.6 R 6

c) V = 2.4 R 70

d) 4.2 R 70

11) Bcafort number for Gale wind


a) 7
b) 9

c) 8

d) 10

12) The component of the wave, along the coast, give rise to current which
carries sediment along with it is known as
a) Littoral drift
b) Abyssal drift
c) Continental drift
d) None of the above
13) Size of the airport depends on
a) Elevation of the airportsize and volume of traffic
b) Metrological conditions
c) Performance characteristics of aircraft
d) All of the above
14) Which of the following is not imaginary surface related to airport ?
a) Approach surface
b) Conical surface
c) Horizontal surface
d) Holding appron
15) In case of airport clear zone is provided _________
a) At start of runway
b) At end of runway
c) At middle of runway
d) None of these
16) ________________ is a structure constructed at the tip of a breakwater near
the harbour entrance.
a) Dolphin
b) Wharf
c) Pierheads
d) None of these
17) A ship designed to carry containers is
a) Lo-Lo-Ships
b) Ro-Ro Cargo carries
c) DWT
d) None of these
18) Fender is cushion provided between the
a) Ship and dock wall
b) Jetties and ship
c) Wharf and ship
d) None of these
19) The portion lying between the harbour entrance and harbour basin is called
a) approach channel
b) Inner channel
c) both a) and b)
d) none of these
20) _____________ harbour is protected on sides by head lands and requires
manmade protection only to the entrance.
a) Natural harbour
b) Semi-natural harbour
c) Artificial harbour
d) None of these
______________
Set A

*SLRBB54*

-3-

SLR-BB 54

Seat
No.

T.E. (Civil) (Part II) Examination, 2014


TRANSPORTATION ENGINEERING II
Day and Date : Saturday, 29-11-2014
Time : 10.00 a.m. to 1.00 p.m.

Marks : 80

Instruction : All questions are compulsory.


2. Answer any three (8 marks each) :
a) Explain grade compensation on curve after solving the following example :
If the ruling gradient is 1 in 200 on a particular section of B.G. and if a curve of
4 is situated on the ruling gradient, what should be the actual ruling gradient ?
Assume grade compensation = .04% per degree curve.
b) A B.G. track has sleeper density (n + 6). If the track is laid with welded rails
of 26 m length, work out the number of sleepers on rail length.
c) What is super elevation and negative super elevation. Explain the importance
of both with sketch.
d) What is crossing angle ? Explain any two methods of calculating angle of
crossing. Use the notations and N.
e) Explain any 4 special signals.
3. Answer any three : (8 marks each) :
a) Enlist the seven topics that are to be considered carefully while planning a
heliport. Explain any one in detail.
b) Which conditions are assumed to calculate basic runway length ?
c) Enlist eight elements of geometric standards for taxiway. Discuss one in
detail.
d) Discuss Airport Lighting.
e) Write in short about Heliport Marking.

Set A

SLR-BB 54

-4-

*SLRBB54*

4. Answer any three (8 marks each) :


a) Explain Littoral Drift.
b) What do you know about :
1) Natural harbour
2) Semi Natural harbour
3) Artificial harbour
4) Commercial harbour ?
c) Discuss types of gravity wharves.
d) Explain Spipway.
e) Discuss :
1) Light house
2) Light ship.
5. Answer any one (8 mark) :
a) Write in brief about :
Corrosion prevention for hydrocarbon transport by pipe line.
b) Explain Points and Crossings in Railway Engineering.
_____________________

Set A

SLR-BB 55

*SLRBB55*
Seat
No.

Set

B.E. (Civil) (Part I) Examination, 2014


DESIGN OF CONCRETE STRUCTURES I
Day and Date : Tuesday, 2-12-2014
Time : 3.00 p.m. to 6.00 p.m.

Total Marks : 100

Instructions : 1)
2)
3)
4)
5)

Use of non programmable calculator is allowed.


Use of IS 456-2000 is allowed while solving Q. No. II to VI.
Assume suitable data if necessary and state it clearly.
Draw neat sketches wherever necessary.
Q. No. 1 is compulsory. It should be solved in first 30 minutes in
Answer Book Page No. 3. IS 456-2000 shall not be allowed to refer
while solving MCQ.
6) Answer MCQ/Objective type questions on Page No. 3 only. Dont
forget to mention, Q.P. Set (A/B/C/D) on Top of Page.
MCQ/Objective Type Questions

Duration : 30 Minutes

Marks : 20

I. Choose the correct answer :

20

1) If standard deviation is 4N/mm2, what should be the mean strength of concrete, if desired
characteristic strength is 20 N/mm2.
a) 26.4 N/mm2
b) 30.87 N/mm2
c) 20.18 N/mm2
d) 17.53 N/mm2

2) The yield strength of a corrugated bar as compared to an ordinary mild steel bar is nearly 1
a) 25% more
b) 50% less
c) 50% more
d) 25% less
3) The unit weights of various materials used for building construction are given in
___________
a) IS 456-2000
b) IS 875(Part-1)-1987
c) IS 12119-1987
d) IS 5816-1999
4) Effective width of flange for T beam is ___________
a)

+ bw + 6 Df

b)

+ bw + 3 Df

c)

+ bw + 6 Df

d)

+ 6 bw + Df

5) Design bond stress in limit state method for plain bars in compression for M25 grade
concrete is ____________ N/mm2.
a) 1.2
b) 1.75
c) 1.5
d) 1.3
6) The diameter of the polygonal links or lateral ties shall not be less than
a) 6 mm
b) max of a) and d)
c) least of a) and d)

1
1

d) 14 diameter of larger longitudinal bar


P.T.O.

SLR-BB 55

*SLRBB55*

-2-

7) Slenderness limit for beam to ensure lateral stability for simply supported or continuous
beams shall be so proportional that the clear difference for the lateral restraining does not
exceed _________ b or _________ b2/d whichever is less.
a) 30, 90
b) 60, 250
c) 25, 100
d) 50, 50
8) For member subjected to bending and torsion two legged closed hoops enclosing the
corner longitudinal bars shall have an area of cross-section Asv given by,
a) Asv = [(Tu.Sv/b1d1(0.47fy)] + [Vu.Su/2.5d1(0.47fy)]
b) Asv = (Tu.Sv/b1d1(0.87fy) + Vu Tu)
c) Asv(Tu.Sv/b1d1(0.85fy)
d) Asv = [(Tu.Sv/b1d1(0.87fy)] + [Vu.Su/2.5d1(0.87fy)]

9) Effective width in L beam is ___________

a)

+ bw + 6 Df

b)

+ bw + 3 Df

c)

+ 3 bw + Df

d)

+ 6 bw + Df

10) Design a square footing for a short axially loaded column of size 230 300 mm carrying
500 kN load. Use M20 concrete and fe415 steel. SBC of soil is 180 kN/m2.
a) 2m 2m
b) 2.5m 2.5m
c) 3m 3m
d) 1m 1m

11) An R.C. beam has cross section 300 550 mm and is subjected to the shear force
95kN-m and torsional Moment 40 kN-m. Determine the equivalent bending moment.
a) 295.63 kN
b) 310.60 kN
c) 308.33 kN
d) 320.53 kN

12) An R.C. beam has cross section 400 600 mm and is subjected to the BM115 kN-m and
torsional moment 45 kN-m. Determine the equivalent bending moment.
a) 194.41 kN-m
b) 199.70 kN-m
c) 196.78 kN-m
d) 198.50 kN-m

13) The diameter of reinforcing bar shall not exceed _________ of the total thickness of the
slab.
a)

1
6

b)

1
8

c)

1
3

d)

1
4

14) The stability of structure as a whole against overturning shall be assumed so that the
restoring moment shall not be less than sum of __________ times maximum overturning
D.L. and ___________ times maximum overturning L.L.
a) 1.5, 1.2
b) 1.2, 1.5
c) 1.2, 1.4
d) 1.4, 1.2
15) The permissible bearing stress for M20 concrete column resting on a rectangular concrete
footing in limit state method of design should not be more than.
a) 11 Mpa
b) 9 Mpa
c) 10 Mpa
d) 20 Mpa
16) In limit state method of concrete structures the strain distribution is assumed to be
a) parabolic
b) non linear
c) linear
d) parabolic and rectangular
______________

1
1

Set A

*SLRBB55*

SLR-BB 55

-3-

Seat
No.

B.E. (Civil) (Part I) Examination, 2014


DESIGN OF CONCRETE STRUCTURES I
Day and Date : Tuesday, 2-12-2014
Time : 3.00 p.m. to 6.00 p.m.
Instructions : 1)
2)
3)
4)

Marks : 80

Use of non programmable calculator is allowed.


Use of IS 456-2000 is allowed while solving Q. No. II to VI.
Assume suitable data if necessary and state it clearly.
Draw neat sketches wherever necessary.
SECTION I

II. The portico of a guest house building consist of cantilever beams of effective span 2.8 m,
spaced at 2.2 m centre. The beams support 120 mm thick slab. Live load on slab is 1.5 kN/m2.
Using concrete mix M20 and steel Fe415, design an intermediate beam, if slab is flush with top
of beams.
OR
The portico of a guest house building consist of cantilever beams of effective span 3.4 m,
spaced at 2.2 m center. The beams support 120 mm thick slab. Live load on slab is 1.5 kN/m2.
Using concrete mix M20 and steel Fe415, design an intermediate beam, if slab is flush with
bottom of beams.

14

14

III. A singly reinforced concrete beam has a width of 300 mm and a overall depth 600 mm with a
clear cover of 40 mm is reinforced with 4 bars of 20 mm. Find the flexural strength and hence
a safe udl on the simply supported beam of span 6.3 m. Use M20 concrete and Fe-415 steel. 13
OR
A doubly reinforced beam 300 mm 500 mm overall reinforced with 4 bars of 20 mm diameter
on tension side and 2 bars of 20 mm diameter on compression side with an effective cover of
35 mm. Determine the safe udl the beam can carry over a simply supported span of 4 m. Use
M20 concrete and Fe415 steel.
13
Stress Level

0.8fy

0.9fy

fy = 415 N/mm2

Stress in N/mm2

288.7

324.8

360.9

IV. A rectangular beam 230 mm width. The clear span of the beam is 6.5 m. The beam is to have
support width 350 mm. The characteristic super imposed load is 14 kN/m. Using M20 concrete
and Fe415 steel design the beam.
OR
Design the rectangular beam of section 300 mm 600 mm of effective span 6.5 m. Effective
cover for reinforcement should be kept at 50 mm. Imposed load on the beam is 43 kN/m. Use
M20 concrete and Fe415 steel. Refer table of Q. III for fsc.

13

13

Set A

SLR-BB 55

-4-

*SLRBB55*

SECTION II
V. Design a rectangular beam section subjected to a factored shear force of 80 kN, torsional
moment of 40 kN-m and moment of 80 kN-m. Use M25 concrete and Fe500 steel.

14

OR
Design a reinforced concrete combined rectangular slab footing for two columns located at 4.5
m apart. The overall size of the columns are 400 mm 400 mm and 600 mm 600 mm and
they are transferring 580 kN and 980 kN respectively. The SBC of soil 180 kN/m2. Use M20
concrete and Fe500 steel. Sketch the reinforcement details.

14

VI. Design a circular column with helical reinforcement subjected to a working load of 1900 kN.
The column has unsupported length of 4 m and in effectively held in position and direction at
both ends. Use M25 and Fe500 steel.

13

OR
a) Design a column 3.4 m long restrained in position and direction at both ends to carry an
axial load of 1800 kN. Use M20 concrete and Fe500 steel.
b) Explain interaction curve to be used for designing column subjected to axial load and
uniaxial bending.

10
3

VII. Design a square footing for a short axially loaded column of size 350 mm 350 mm carrying
650 kN load. Use M20 concrete and Fe415 steel. SBC of soil is 180 kN/m2. Sketch the details
of reinforcement.

13

OR
Design a sloping footing for a column of size 450 mm 450 mm carrying a load at 2200 kN.
SBC of soil 200 kN/m2. Use M25 concrete and Fe500 steel.

13

_____________________

Set A

SLR-BB 56

*SLRBB56*
Seat
No.

Set

B.E. (Civil) (Part I) Examination, 2014


QUANTITY SURVEYING AND VALUATION
Day and Date : Thursday, 4-12-2014
Time : 3.00 p.m. to 7.00 p.m.

Max. Marks : 100

Instructions : 1) Q. 2 and Q. 9 are compulsory. Attempt any two out of


Q. 3 to Q. 5 and any two out of Q. 6 to Q. 8.
2) Assume suitable data if necessary but mention it clearly.
3) Q. No. 1 is compulsory. It should be solved in first 30 minutes
in Answer book Page No. 3. Each question carries one mark.
4) Answer MCQ/Objective type questions on Page No. 3 only.
Dont forget to mention, Q.P. Set (A/B/C/D) on Top of Page.
MCQ/Objective Type Questions
Duration : 30 Minutes

Marks : 20

1. Choose the correct answer :


1) For RCC work, task work per mason may be taken as
a) 1 Cu.m.
b) 2 Cu.m.
c) 3 Cu.m.

d) None of these

2) For 12 mm plaster in cement mortar, task work per mason may be taken as
a) 2 Cu.m.
b) 4 Cu.m.
c) 8 Cu.m.
d) None of these
3) For Brickwork dry mortar may be taken as ___________% of volume of brickwork.
a) 20
b) 30
c) 40
d) 50
4) For Rubble stone masonary, dry mortar may be taken as ___________% of
volume of masonary.
a) 21
b) 32
c) 42
d) 52
5) For Rubble stone masonary, stones may be taken as ___________% of volume
of masonary.
a) 100
b) 125
c) 150
d) None of these
6) For Brickwork with standard bricks, mortar joint thickness is assumed as
a) 1 cm
b) 2 cm
c) 3 cm
d) None of these
7) For Brickwork with standard bricks, breakage and wastage of brick is assumed
as
a) 2%
b) 5%
c) 10%
d) None of these
P.T.O.

SLR-BB 56
8) Detailed specifications decides
a) Quality
b) Quantity

*SLRBB56*

-2-

c) Both a) and b) d) None of these

9) Select the overhead charges out of following


a) Labour
b) Electric power required for construction
c) Telephone and light bill of site office
d) All the above
10) For M20 concrete mixer with the charge be rotated for a minimum period of
a) 60 sec
b) 120 sec
c) 180 sec
d) None of these
11) RCC M20 is required to be cured for a period of
a) 7 days
b) 14 days
c) 21 days
d) 28 days
12) Select the act of god
a) Flood
b) Earthquake

c) Both of these

d) None of these

13) If net income from property is Rs. 5,000/year, and years purchase is 15,
then capitalized value is
a) Rs. 75,000
b) Rs. 7,50,000 c) Rs. 7,500
d) None of these
14) Mode of measurement and payment for down-take PVC pipe having 100 mm dia. is
a) Kg
b) Numbers
c) Per meter run d) None of these
15) If wages are paid weekly, the muster roll can be maintained in
a) Single copy
b) In duplicate
c) In triplicate
d) None of these
16) A brick wall in elevation 10000 mm long and 8300 mm height is to be plastered
externally, if plaster thickness is 25 mm the measurement written shall be
a) 10.05 mL, 8.3 mH
b) 10 mL, 8.45 mL
c) 10.00 mL, 8.3 mH
d) None of the above
17) In UCR masonary wall thickness is 450 mm and number of headers is X, if
wall thickness is 900 mm, number of headers required shall be
a) X
b) 0.5 X
c) 2X
d) 3X
18) If UCR masonary 10m3 is to be constructed quantity of rubble required in m3 will be
a) 8
b) 10
c) 12
d) 15
19) In the above problem if wall thickness is 500 mm and height of masonary is
2000 mm the number of headers required will be
a) 10
b) 20
c) 30
d) None of these
20) French polishing is to be done for teakwood frame and shutter, polishing is to
be done at
a) night
b) bright sunshine
c) weather is cloudy
d) any time
______________
Set A

*SLRBB56*

-3-

SLR-BB 56

Seat
No.

B.E. (Civil) (Part I) Examination, 2014


QUANTITY SURVEYING AND VALUATION
Day and Date : Thursday, 4-12-2014
Time : 3.00 p.m. to 7.00 p.m.

Marks : 80

Instructions : 1) Q. 2 and Q. 9 are compulsory. Attempt any two out of


Q. 3 to Q. 5 and any two out of Q. 6 to Q. 8.
2) Assume suitable data if necessary but mention it clearly.
2. Estimate the quantities of the following items of the building shown in the figure.
(Assume all the missing data suitably)
14

3. a) Prepare a preliminary estimate of a residential building project with a total


plinth area of all the buildings being 500 Sq.m. The following information is
given :

a) Plinth area rate : Rs. 20,000 per m2


b) Additional amount allowed for special architectural features : 1.5% of the
building cost
c) Extra for water supply and sanitary fittings : 5% of building cost
d) Extra for building services = 7.50% of building cost
Set A

SLR-BB 56

-4-

*SLRBB56*

e) Contingencies : 2% of overall cost


f) Supervision charges = 7.5% of overall cost.
b) State the methods of approximate estimate, explain any one.

4. A) Write the detailed specifications for :

a) Painting in new building.


b) Earthwork in roads.
B) Write note on Provisional quantities and Provisional sums.

5. A) Carry out rate analysis for the following items.

a) First coat bituminous painting road.


b) Random Rubble masonary in C.M. (1 : 5).
B) Differentiate between long wall short wall and center line method.

6. A) List various types of contract and explain BOT contract.

B) What is EMD and Security Deposit ? What is its significance in Tender


Documents ?

7. A) List various methods of valuation and describe Belting method of valuation of


land with suitable example.
B) Define :

8
5

6) Book Value
7) Accommodation value
8) Speculation value
9) Scrap value
10) Potential value.
8. A) A temporary shed has been constructed for Rs. 12,000/-. Assuming its salvage
value at the end of 6 years as Rs. 3,000/-. Determine the depreciation and
book value for each year by constant percentage method.
B) Explain Easement Rights.

8
5

9. What do you mean by valuation ? What is its necessity ? List various methods
of valuation and describe any three of them giving examples.

14

_____________________

Set A

SLR-BB 57

*SLRBB57*
S

B.E. (Civil) (Part I) Examination, 2014


EARTHQUAKE ENGG.
Day and Date : Saturday, 6-12-2014
Time : 3.00 p.m. to 6.00 p.m.

Max. Marks : 100

Instructions : 1) Q. No. 1 is compulsory. It should be solved in first 30


minutes in Answer Book Page No. 3. Each question carries
two marks.
2) Answer MCQ/Objective type questions on Page No. 3
only. Dont forget to mention, Q.P. Set (A/B/C/D) on Top
of Page.
MCQ/Objective Type Questions

Duration : 30 Minutes
1. 1) The expression

Marks : 20

gives
K

(210=20)

a) W

b) T

c) F

2) The expression for logarithmic decrement


a)


b)
4

d) R
is


c)
4

d)
H

H


3) The expression for damped circular frequency


a) W


b) W
4

c)
9

d) W
4

4) The dynamic displacement of vibrating body is


a) Ydy = Magnification factor Y Static
b) Ydy = Magnification factor / Y static
c) Ydy =
M

Y static
1

d) Ydy = 2 Y static
5) The magnification factor for undamped forced vibration of SDOF system is
a)

b)


c)


d) (1 r2)


P.T.O.

SLR-BB 57

*SLRBB57*

-2-

6) The damped time period of SDOF system is


a) TD =

b) TD =


H
9

c) TD =

d) TD =


7) The ratio of importance factor and response reduction factor (I/R) shall not
be greater than
a) 1.0
b) 1.5
c) 2.0
d) 0.5
8) Earthquake resistant design is
a) Design for gravity loading
b) Design for lateral loading
c) Design for lateral and gravity loading
d) None of above
9) What is maximum percentage of steel can be spliced at one location in RCC
members ?
a) 25%
b) 50%
c) 40%
d) 30%
10) As per IS 13920, what is the anchorage length for flexural member
a) Ld
b) Ld + 10 db
c) Ld/10 db
d) Ld 10 db
______________

Set A

*SLRBB57*
S

-3-

SLR-BB 57

B.E. (Civil) (Part I) Examination, 2014


EARTHQUAKE ENGG.
Day and Date : Saturday, 6-12-2014

Marks : 80

Time : 3.00 p.m. to 6.00 p.m.


Instructions : 1) Solve any three questions from each Section.
2) Figures to right indicate full marks.
3) Use of only IS 1893 is allowed.
4) Assume suitable data if necessary and mention it clearly.
SECTION I
1. a) What are the remedial measures to control soil liquefaction ?

b) What do you understand by magnitude of an earthquake ? Explain the different


scales that are commonly used.

10

2. From first principle derive the governing differential equation for free vibration of
a undamped SDOF system. Obtain the general solution of this differential equation.
Plot the graph if system is given initial displacement y0.

13

3. Derive the government differential equation of undamped forced vibration of a


SDOF system subjected to harmonic loading. Also obtain complete solution of
differential equation and plot graph of magnification factor vs frequency ratio r. 13
4. a) Explain the concept of Duhamels integral for damped systems. Derive its
expression.
b) What is combined spectrum ? What are its characteristics ?

5
8

Set A

SLR-BB 57

-4-

*SLRBB57*

SECTION II
5. It is proposed to construct a R.C.C. four storied commercial building having plan
dimensions as shown in fig. 1 in zone III with following data. Determine the
lateral forces and base shear. The all column sizes are 250 450 mm and
beams sizes are 250 450 mm. The slab thickness is 140 mm and thk. Of walls
is 230 mm. The ht. of floor is 3.2 m and Live load is 2.5 kN/m2 IS 13920 will be
used. The strata is hard.
14

Figure no. 1 Q. 5.
6. What do understand by a weak storey ? How weak storey differs from soft
storey ?

13

7. What do you understand by ductility and what is its importance ?

13

8. What is the basic difference between Earthquake resistant features in RCC


and masonry construction ?

13

_____________________

Set A

SLR-BB 58

*SLRBB58*
S

B.E. (Civil) (Part I) Examination, 2014


WATER RESOURCES ENGINEERING II
Day and Date : Tuesday, 9-12-2014
Time : 3.00 p.m. to 6.00 p.m.

Max. Marks : 100

Instructions : 1) Draw neat labeled sketches wherever necessary.


2) Assume suitable data if necessary and state it clearly.
3) Q. No. 1 is compulsory. It should be solved in first 30 minutes in Answer
Book Page No. 3. Each question carries one mark.
4) Answer MCQ/Objective type questions on Page No. 3 only. Dont
forget to mention, Q.P. Set (A/B/C/D) on Top of Page.
Duration : 30 Minutes
1. Choose the correct answer :

MCQ/Objective Type Questions

Marks : 20
(120=20)

1) The surcharge storage in a dam reservoir is the volume of water stored between
a) minimum and maximum reservoir levels
b) minimum and normal reservoir levels
c) normal and maximum reservoir levels
d) none of the above
2) Trap efficiency of a storage reservoir is defined as
a) (Total annual sediment inflow)/(Reservoir capacity)
b) (Total sediment deposited in a given period)/(Total sediment inflow in that period)
c) (Total annual sediment deposited in the reservoir)/(Dead storage capacity of the servoir)
d) None of the above
3) The useful storage in a dam reservoir is the volume of water stored between
a) minimum and maximum reservoir levels
b) minimum and normal reservoir levels
c) normal and maximum reservoir levels
d) none of the above
4) In order to economise on the provided section of a concrete gravity dam, attempts are made to
reduce the uplift, by
a) providing drainage gallery to collect seepage water
b) constructing cut-off under upstream face
c) pressure grouting in dam foundation
d) all the above methods
5) According to IS 6512 1984, the wind set up in a reservoir having maximum fetch of F km, and
average reservoir depth along this fetch length as D m, as caused by the wind velocity of V km/h,
is given by
a) (V2F)/(62000D)
b) (V2F)/(D)
c) (V2F)/(6200D)
d) (V2F)/(620D)
6) The horizontal component of the earthquake wave, producing instability in dam, is the one, which
acts
a) towards the reservoir b) towards the dam
c) both (a) and (b)
d) none of the above
7) The most preferred soil for the central impervious core of a zoned embankment type of an earthen
dam is
a) highly impervious clay
b) highly previous gravel
c) coarse sand
d) clay mixed with fine sand
8) The base width of a rack fill dam, in comparison to that of an earthen dam, is
a) much larger
b) much smaller
c) sometimes larger sometimes smaller
d) almost equal
P.T.O.

SLR-BB 58

*SLRBB58*

-2-

9) When the tail water depths in the river, downstream of a spillway are quite low, such that the tail
water curve, at all discharges, lies below the post jump depth curve, then the energy dissipation can
be affected best by
a) a roller bucket
b) a ski-jump bucket c) either (a) or (b)
d) none of them
10) The only spillway among the following, through which the discharge is almost at its capacity rate,
even from the start of its functioning, is
a) chute spillway
b) side channel spillway
c) ogee spillway
d) siphon spillway
11) Critical exit gradient, as enunciated in Khoslas theory of design of weirs and barrages on previous
foundations, is
a) the left out pressure in the seeping water at the downstream end point, where it emerges out on
the river bed
b) the rate of loss of pressure of the seeping water at the downstream emerging point, which is just
enough to lift the soil grains at that point
c) the actual pressure gradient of the seepage, at the downstream emerging point
d) none of the above
12) For the head regulator, the most severe condition of uplift pressure on the floor occurs when
a) the flow in the river is at flood level and canal is running at full supply depth
b) the canal runs dry and the river flow is at high flood level
c) the canal runs at full supply depth and the river flow is a pond level
d) the canal runs dry and the river flow is at pond level
13) The arrangement made in a canal network, which acts as its safety valve, is
a) cattle crossing
b) canal ladder
c) canal escape
d) canal module
14) Point out the choice among the following, which is not a function of a distributary head regulator ?
a) it serves as a meter for measuring discharge in the off-taking canal
b) it serves to control silt entry into the off-taking canal
c) it helps in controlling and regulating supplies in the entire downstream canal network
d) none of the above
15) Point out the incorrect statement in relation to the behaviour of a meandering alluvial river
a) primary cause for meandering is the silting action due to excess sediment at the available
discharge and surface slope
b) secondary cause for meandering is the scouring action due to deficiency of sediment at the
available discharge and surface slope
c) meandering tendency is more in an alluvial river during floods, because of high variation in
discharge
d) none of these
16) The upstream angle of inclination of a repelling groyne with normal to the bank line, is of the order of
a) 5 to 10
b) 10 to 30
c) 30 to 50
d) 70 to 90
17) Alkaline soils are best reclaimed by
a) leaching
b) addition of gypsum to soil
c) providing good drainage
d) addition of gypsum to soil and leaching
18) Pick up the incorrect statement
a) moderately saline lands are represented by visible whitish deposits of salts on the top and
upper layers of soil
b) Na2, CO3 is the most harmful salt for causing salinity
c) NaCl is the least harmful salt for causing salinity
d) none of the above
19) An impulse turbine like Peltons turbine is
a) an open body exposed to atmosphere
c) a partially open and partially closed

b) a closed body not exposed to atmosphere


d) none of these

20) The only statement, which is incorrect in regard to hydropower is


a) the system efficiency of a hydroplant is quite high
b) the installation cost of a hydroplant is very high
c) the running cost of a hydro power plant is very low
d) the hydraulic turbines takes a lot of time in being put off and on
______________

Set A

*SLRBB58*
S

-3-

SLR-BB 58

B.E. (Civil) (Part I) Examination, 2014


WATER RESOURCES ENGINEERING II
Day and Date : Tuesday, 9-12-2014
Time : 3.00 p.m. to 6.00 p.m.
Instructions :

Marks : 80

1) Solve any three questions from Section I (Questions 2, 3, 4, 5) and


any three questions from Section II (Questions 6, 7, 8, 9).
2) Draw neat labeled sketches wherever necessary.
3) Assume suitable data if necessary and state it clearly.
SECTION I

2.

a) Write a note on reservoir sedimentation. How do you estimate the probable life of a reservoir ?

b) Explain the mass curve method that can be used for determining reservoir capacity for fulfilling given
demand.

3. Figure shows a section of a non overflow portion of a gravity dam built of concrete.

Calculate the maximum vertical stress at the heel and toe of the dam. Assume unit weight of concrete as
23.5 kN/m3. Neglect earthquake forces.

13

Set A

SLR-BB 58
4.

-4-

*SLRBB58*

a) What are the criteria for the safe design of earth dam ?

b) A homogeneous earth dam 42 m high is built to store water for irrigation requirements with a free
board of 2 m. A horizontal filter of 30 m length is provided as its downstream end. The coefficient of
permeability of the dam material is 2 103 cm/sec. A flow net was constructed through the body of
the dam giving following results. Number of potential drops = 25, Number of flow channels = 4.
Calculate the seepage flow per meter length of a dam in liters per second.

5. a) What is meant by energy dissipater ? Discuss the various methods used for energy dissipation
below spillways.

b) Draw a neat labeled sketch of Wet Intake Tower and describe its working.

SECTION II
6. a) Briefly outline Khoslas theory on the design of weirs on permeable foundation. Enumerate the various
corrections that are needed in the application of this theory.

b) Write short notes with suitable sketches :


i) Canal siphon
ii) Level crossing.

7. a) What is meant by water-logging ? What are its ill effects ? Describe some anti-water-logging measures
with suitable sketches.

b) Write short notes on the following :


i) High water, low water and mean water training
ii) Use of levees for protecting cities from floods.

8. a) Name and describe the various types of canals which are required to be constructed while planning a
canal irrigation system.

b) How will you justify economically the necessity of lining an existing canal ? What added benefits you
will expect if the canal to be lined is new and yet to be constructed ?

9. a) Classify the hydroelectric power schemes according to the available head. Draw a sketch to show a
general layout of high head scheme.

b) A runoff river plant has installed capacity of 15000 kW and it operates at 30% load factor when it
serves as peak load station. What should be the minimum discharge in the stream so that it may
serve as a peak load station ? The plant efficiency may be assumed as 82% when working under a
head of 25 meter. Also calculate the maximum load factor of the plant when the discharge in the
stream is 32 cumec.

_____________________

Set A

SLR-BB 59

*SLRBB59*
Seat
No.

Set

B.E. (Civil) (Part I) Examination, 2014


Elective : I : AIR POLLUTION AND CONTROL
Day and Date : Thursday, 11-12-2014
Time : 3.00 p.m. to 6.00 p.m.
Instructions :

Total Marks : 100

1) Solve any three questions from Section I i.e. Question no. 2 to question
no. 5. Question no. 6 is compulsory in Section II and solve any two
questions from the remaining.
2) Figures to the right indicate full marks.
3) Assume suitable data wherever necessary and mention it clearly.
4) Use of non programmable calculator is allowed.
5) Q. No. 1 is compulsory. It should be solved in first 30 minutes in
Answer Book Page No. 3. Each question carries one mark.
6) Answer MCQ/Objective type questions on Page No. 3 only. Dont
forget to mention, Q.P. Set (A/B/C/D) on Top of Page.
MCQ/Objective Type Questions

Duration : 30 Minutes

Marks : 20

1. Choose the correct answer :

20

1) Aerosols are defined as ______________


a) Finely divided liquid droplets or solid particles
b) Finely divided liquid droplets
c) Solid particles
d) Toxic gases
2) A device which consist of bags to which dust in a gas flowing in it attaches itself is called
______________
a) Cyclone
b) Fabric filter
c) Separator
d) Filter
3) Carbon monoxide in air effects ____________
a) Heart
b) Skin
c) Eye

d) Hair

4) When ELR equals to ALR and both lines coincide the environment is called ____________
a) Metastable
b) Unstable
c) Stable
d) None of these
5) Electrostatic precipitators are used as pollution control device for separation of
_____________
b) Nox
a) So2
c) Hydrocarbons
d) Particulate matter
6) The vertical temperature gradient is known as ____________
a) Inversion
b) Lapse rate
c) Mixing height

d) Humidity

7) Most dangerous hydrocarbons found in the automobile exhaust are


____________
a) Methanes
b) Olefins
c) Alkynes
d) Ketones

P.T.O.

SLR-BB 59

*SLRBB59*

-2-

8) Main impact of green house effect is ____________


a) Increase in vegetation
b) Decrease in vegetation
c) No effect on vegetation
d) None of these
9) Measurement of suspended particulates and gaseous contaminants is represented with
unit ______________
b) Mg/cm3
c) g/cm3
d) g /m3
a) Mg/cm2
10) Size of the particle of which size can be seen by naked eye is _________
a) 50 m

b) 30 m

c) 50 m

d) 20 m

11) _______________ is a ratio of actual vapour pressure of air to the saturation vapour
pressure.
a) Relative humidity
b) Pressure drop
c) Synergistic effect
d) Stack
12) A device for removing and retaining pollutants from air or other gases is
_____________
a) Separator
b) Collector
c) Filter
d) Precipitatio
13) _______________ is the physical and chemical measure of concentration of
contaminants in atmosphere.
a) Air pollution
b) Air pollution index
c) Air quality index
d) Ambient air quality
14) Weather pattern are largely determined in the ______________
a) Stratosphere
b) Mesosphere
c) Troposphere

d) Lithosphere

15) _______________ plume behaviour occurs when atmospheric inversion begins from the
ground level and continues.
a) Looping
b) Fumigation
c) Coning
d) Fanning
16) The spray tower can be used to control ____________
a) Gaseous pollutants only
b) Particulate pollutants only
c) Both a) and b)
d) None of these
17) ______________ is (are responsible for ozone layer depletion.
A) CFC
B) HFC
a) Only A)

b) Only B)

c) Both A) and B)

18) The acid rains are caused by the pollutants ______________


a) SO2 and O3
b) SO2 and NOx
c) NOx and O3

d) None of these
d) CO and SO2

19) The intensification of greenhouse effect is attributed to the increased level of ____________
a) Carbon dioxide
b) Carbon monoxide
c) Chloroflurocarbons
d) Sulphur dioxide
20) ________________ is a representation to show distribution of wind direction.
a) Stack
b) Ringelmann chart
c) Venture
d) Wind rose
______________

Set A

*SLRBB59*

SLR-BB 59

-3-

Seat
No.
B.E. (Civil) (Part I) Examination, 2014
Elective : I : AIR POLLUTION AND CONTROL
Day and Date : Thursday, 11-12-2014
Time : 3.00 p.m. to 6.00 p.m.
Instructions :

Marks : 80

1) Solve any three questions from Section I i.e. Question no. 2 to question
no. 5. Question no. 6 is compulsory in Section II and solve any two
questions from the remaining.
2) Figures to the right indicate full marks.
3) Assume suitable data wherever necessary and mention it clearly.
4) Use of non programmable calculator is allowed.
SECTION I

2. a) Define Air pollution. What are primary and secondary pollutants ?

b) Draw a neat sketch profile of atmosphere and explain various layers.

3. a) Differentiate between adiabatic lapse rate and environmental lapse rate.

b) A power plant uses a coal of 750 kg/hr if the coal contains 28% ash and 2% sulphur
determine the emission rate of SPM and SO2 also determine the level concentration
downward at distance of 1.5 km and 3.5 km if the wind speed is 4.2m/sec and the stability
class is B and values of y and z are

For 1.5 km

For 3.5 km

170

480

125

300

H = 150 m

4. a) What is meant by wind rose diagram ? How is it prepared ?

b) Explain meteorological factors influencing Air Pollution ?

5. Write shorts note on any three :

15

a) Effect of SO2 and SPM on man


b) Inversion
c) Plume rise
d) Photochemical Smog.

Set A

SLR-BB 59

-4-

*SLRBB59*

SECTION II
Instruction :

Question no. 6 is compulsory in Section II and solve any two


questions from the remaining.

6. a) Draw a neat sketch of particulate sampling train in stack sampling. Write basic equation to
determine the velocity of flow inside the stack.
b) What are basic consideration required for Air sampling ?

8
4

7. a) A fabric filter is to be constructed using bags having 30 cm diameter and 4.5 m height. The
bag house is to receive 12m3/s of air, and filtering velocity is restricted to 2m/min. Determine
the number of bags required for a continuously cleaned operation.

b) What is meant by standard cyclone ? Give the values of cyclone dimension for it in terms
of cyclone diameter.

8. a) What factors affect the efficiency of Bag Filter ? Comment on merits and demerits of this
equipment.

b) What is meant by photochemical Smog ? Explain the effects.


9. Write short note on any three :

6
14

a) Radiation Inversion
b) Global Air Pollution Aspects
c) London Smog
d) Acid Rain.
_____________________

Set A

SLR-BB 60

*SLRBB60*
Seat
No.

Set

B.E. (Civil) (Part I) Examination, 2014


Elective I : DESIGN OF FOUNDATIONS
Day and Date : Thursday, 11-12-2014
Time : 3.00 p.m. to 6.00 p.m.

Max. Marks : 100

Instructions : 1) Answer any three questions from each Section.


2) Make suitable assumption if necessary and mention it
clearly.
3) Figures to the right indicates full marks.
4) Q. No. 1 is compulsory. It should be solved in first 30 minutes
in Answer Book Page No. 3.
5) Answer MCQ/Objective type questions on Page No. 3 only.
Dont forget to mention, Q.P. Set (A/B/C/D) on Top of Page.
MCQ/Objective Type Questions
Duration : 30 Minutes

Marks : 20

1. Choose the correct answer :

20

1) Connection between pile and superstructure is called


a) Pile shaft
b) Pile tip
c) Pile cap

1
d) None

2) Load resisted by pile along its shaft is called


a) bearing load
b) skin friction
c) negative skin friction
d) uplift

3) Magnification factor is the ratio of


a) Dynamic force to static force
b) Static force to dynamic force
c) Dynamic force to static deflection d) Static force to static deflection

4) A vibrating system is said to be overdamped when

a) C < 2mn

b)

c
K
<
2m
m

c) C > 2mn

d)

5) Allowable bearing pressure for a foundation depends upon


a) Allowable settlement only
b) ultimate bearing capacity of soil only
c) Both allowable settlement and ultimate bearing capacity
d) None of above

c
K
=
2m
m

P.T.O.

SLR-BB 60

*SLRBB60*

-2-

6) Which one of the following is the correct Terzaghi general bearing capacity
equation ?
a) qu = C Nc + D Nq + 0.5 B N

b) qu = C Nc D Nq + 0 .5 B N

c) qu = C Nc + 0.5 D Nq + B N

d) None

7) In a plate-load test, the load is applied in one of the following increments of


load
b) 1/20th
c) 1/10th
d) 1/5th
a) 1/30th

8) In case of relatively hard leveled bed of river which type of caisson foundation
is preferred.
a) Open
b) Pneumatic
c) Box
d) None

9) Site investigation is necessary for


a) Foundation design
b) Ground water studies
c) Construction planning
d) All of these
10) In SPT, we terminate the test for no. of blows which are obtained to drive the
required 30 cm
a) 70 blows
b) 80 blows
c) 90 blows
d) 100 blows
11) Surface area of pile having square cross section of size 0.5m and 10m
embedded in soil is
a) 10 sq. m
b) 50 sq.m
c) 20 sq.m
d) 25 sq.m

12) Two columns carrying loads 500kN and 600 kN separated by 5.5m c/c, position
of resultant column load with respect to heavier column load is
a) 2.5m
b) 3.5m
c) 4.5m
d) None
13) A soil sample has external diameter as 7 cm and wall thickness of 2.5 mm.
The area ratio in percentage will be
a) 12.88
b) 18.22

2
c) 15.97

d) 16.85

14) Three piles are arranged in triangular form; efficiency of this pile group by
Felds rule is
a) 33.33%
b) 50%
c) 75%
d) 87.5%
15) In Shallow foundation if Rw1 = 1 and Rw2 = 0.5 than where the water table
lies
a) At base of footing
b) Below the footing
c) At the ground level
d) Any where at the mid

______________

Set A

*SLRBB60*

SLR-BB 60

-3-

Seat
No.

B.E. (Civil) (Part I) Examination, 2014


Elective I : DESIGN OF FOUNDATIONS
Day and Date : Thursday, 11-12-2014
Time : 3.00 p.m. to 6.00 p.m.

Marks : 80

Instructions : 1) Answer any three questions from each Section.


2) Make suitable assumption if necessary and mention it clearly.
3) Figures to the right indicates full marks.
SECTION I
2. A) Explain the IS code equation to determine the bearing capacity. How the shape
factor, depth factor and inclination factors are calculated in this equation ?
B) A plate load test was performed on a uniform deposit of sand and the following
observations were recorded.
Load
(kN/m2)

50

100

200

300

400

500

600

Settlement
(mm)

4.5

8.5

16

31.3

50

74

104

Size of plate was 20 20 cm. Plot load settlement curve and determine the
load on a footing of size 1.75 m 1.75 m can carry safely if the settlement is
not to exceed 30 mm.

3. A) Describe the procedure for the design of strap footing (geometry only).
B) Two reinforced column 800 800 mm and 600 600 mm in size carry axial
load of 2850 kN and 1700 kN respectively. The columns are spaced 2.8 m
apart. The available space by side of 1700 kN column is 1.3 m only from
centre of column. The SBC of soil is 250 kN/m2. Use M-20 concrete and Fe-415
steel. Design combined trapezoidal footing (Geometry only).

4. A) Describe conventional method of design of raft foundation.


B) Design a raft foundation (beam column type) for 4 columns spaced at a
distance of 4m center to center in either direction (square pattern). All four
columns carry an equal load of 800 kN. Assume SBC - 110 kN/m2. Use M-20
concrete and Fe-415 steel.

Set A

SLR-BB 60

-4-

*SLRBB60*

5. Write a note on any four :

16

a) Assumptions of Terzaghis bearing capacity theory.


b) Suitability of raft foundation.
c) Requirement of CNS soil.
d) Provisions for foundations on expansive soil
e) Effect of water table on bearing capacity.
SECTION II
6. A) What is negative skin friction ? Where it is developed ? What is its effect on
the load carrying capacity of pile ?

B) A square group of 9 piles was driven into a soft clay extending to a large
depth. The diameter and length of the piles were 30 cm and 9 cm respectively.
If the unconfined compressive strength of clay is 90kN/m2, and the spacing is
90 cm centre to centre, What is the capacity of group ? Assume FOS 2.5 and
adhesion factor 0.75.

7. A) A column carries load of 1800 kN is supported by four piles of 450 mm dia.


The size of the column is 400 mm 400 mm. The center to center distance
between the pile is 1.3 m in either direction. Design suitable pile cap. Assume
M-20 concrete and Fe=415 steel.

B) Describe the load transfer mechanism in case of drilled pier.

8. A) What do you understand by scour depth and grip length ? What is its importance
in well foundation ?

B) Show Yawing, Rocking and pitching with neat sketch.

C) Discuss the criteria for satisfactory performance of machine foundation.

9. Write a short note on any four :

16

a) Pile group efficiency


b) Types of machine foundation
c) Caisson disease
d) Rock socketing
e) Criteria for satisfactory performance of machine foundation.
_____________________

Set A

SLR-BB 61

*SLRBB61*
Seat
No.

Set

B.E. (Civil) (Part I) Examination, 2014


Elective I : MANAGERIAL TECHNIQUES
Day and Date : Thursday, 11-12-2014
Time : 3.00 p.m. to 6.00 p.m.

Max. Marks : 100

Instructions : 1) Solve any three questions from each Section.


2) Q. No. 1 is compulsory. It should be solved in first 30
minutes in Answer Book Page No. 3. Each question carries
one mark.
3) Answer MCQ/Objective type questions on Page No. 3 only.
Dont forget to mention, Q.P. Set (A/B/C/D) on Top of Page.
MCQ/Objective Type Questions
Duration : 30 Minutes

Marks : 20

1. Choose correct alternative (one mark each) :


1) Activity sampling and work sampling are
a) Different
b)
c) Same
d)
2) An amount paid to individual worker is
a) Direct incentive
b)
c) Both a) and b)
d)

Dependent on each other


None
Indirect incentive
None

3) Outline process chart includes


a) Survey and record of overall processes
b) All the events
c) Activity of operator
d) None
4) Classification method is method of
a) Merit rating
b) Performance rating
c) Work sampling
d) None
5) The aspects which are considered before selecting a job in method study
are
a) Quality development aspects
b) Managerial aspects
c) Human reactions
d) All the above
6) Work study includes
a) Method study b) Time study

c) Both a) and b) d) None


P.T.O.

SLR-BB 61

-2-

*SLRBB61*

7) The successful application of work study is related to


a) Relation between management and workers
b) Relation between employer and workers
c) Relation between worker and representative and worker
d) All the above
8) The benefit of productivity for customers to reducing
a) Higher wages
b) Higher standard of living
c) Price of material
d) None
9) Methods study involves
a) Reduction in fatigue of worker
b) Improved method
c) Both a) and b)
d) None
10) Basic time is also called as
a) Observed time
b) Standard time
c) Both a) and b)
d) None
11) In TQM we study _________ S technique.
a) One
b) Three
c) Two
d) None
12) Analysis for existing product is
a) Value engineering
b) Value estimation
c) Value analysis
d) None
13) The symbols are used in FTA are grouped as events, gates and ________
symbols.
a) Main
b) Exclusive
c) Transfer
d) None
14) The use of value engineering is
a) Balance cost and performance
b) Cost reduction technique
c) Prevent ever design of component d) All the above
15) If the part of system fails, there is an alternative success path (backup) is
provided. It is called as
a) Reliability
b) Redundancy c) FEMA
d) PDF
16) Percentage of working time is represented by letter.
a) p
b) q
c) r
d) s
17) PDCA cycle is developed by
a) Juran
b) Kaizen
c) Deming
d) None
18) PDCA cycle is
a) Produce, develop, create, action b) Plan, develop, check, action
c) Produce, do, check, act
d) Plan, do, check, act
19) Juran Trilogy is based on Quality planning, Quality improvement and
a) Quality measurement
b) Quality control
c) Quality system
d) Quality cost
20) Motion study is developed by
a) Taylor
b) Deming
c) Juran
d) None
______________
Set A

*SLRBB61*

-3-

SLR-BB 61

Seat
No.

B.E. (Civil) (Part I) Examination, 2014


Elective I : MANAGERIAL TECHNIQUES
Day and Date : Thursday, 11-12-2014
Time : 3.00 p.m. to 6.00 p.m.

Marks : 80

Instruction : Solve any three questions from each Section.


SECTION I
2. a) Enlist the different steps in method study. Explain any one in detail.
b) Write in short about Performance Rating.

9
5

3. a) What is the role of trade union in work study ?


b) State the importance of Allowances in calculating standard time.

8
5

4. a) Draw the flow process chart (man type) for concreting of slab at first floor.
b) Write a note on PMTS.

8
5

5. a) Discuss the different methods of Job Evaluation.


b) What do you know about Kaizen ?

8
5

SECTION II
6. a) Pilot study showed the percentage of occurrence of an activity as 50%.
Determine number of observations for 95% confidence level and accuracy
of 2%.
b) Tell in short about Sample size.

9
5

7. a) Explain in detail Random sampling.


b) Write in short about use of control charts with respect to work sampling.

8
5

8. a) Explain Bath tub curve used in Reliability engineering.


b) Write in short about FEMA.

8
5

9. a) Explain value analysis procedure.


b) Discuss Types of values.

8
5

_____________________

Set A

Set A

SLR-BB 62

*SLRBB62*
Seat
No.

Set

B.E. (Civil) (Part I) Examination, 2014


Elective I : ADVANCED CONCRETE TECHNOLOGY
Day and Date : Thursday, 11-12-2014
Time : 3.00 p.m. to 6.00 p.m.

Max. Marks : 100

Instructions : 1)
2)
3)
4)

Figures to right indicate full marks.


Use of non-programmable calculator is allowed.
Assume suitable data, if necessary and mention it clearly.
Q. No. 1 is compulsory. It should be solved in first 30
minutes in Answer Book Page No. 3.
5) Answer MCQ/Objective type questions on Page No. 3 only.
Dont forget to mention, Q.P. Set (A/B/C/D) on Top of Page.
MCQ/Objective Type Questions

Duration : 30 Minutes

Marks : 20

1. Choose the correct answer. Each MCQ carries two marks.


1) Measurement of concrete workability is done by
a) Compacting factor test
b) Abrasion test
c) Soundness test
d) Setting time test
2) Final and Initial setting time of cement is
a) Minimum 30 min. and maximum 600 min.
b) Maximum 30 min. and Minimum 600 min.
c) Minimum 600 min. and Maximum 30 min.
d) Maximum 600 min. and Minimum 30 min.
3) In case concrete is to be transported by pumping, the slump should be
a) More than 100 mm
b) Between 50 mm to 70 mm
c) Between 25 mm to 50 mm
d) Less than 25 mm
4) True slump shows characteristic of
a) Homogenous material

b) Non-cohesive material

c) Segregation

d) Bleeding
P.T.O.

SLR-BB 62

*SLRBB62*

-2-

5) Modulus of elasticity of concrete can be assumed as


a) 5000 f ck

b) 0.7 f ck

c) 200 KN/mm2

d) 0.27 f ck

6) Which of the following is not a effect of creep ?


a) Due to creep there is a volume changes in concrete
b) Creep increases the deflection of structure with time
c) Creep will lead to buckling
d) It may cause cracking in interior of structure
7) For concrete mix pH value of water shall not be less than
a) 7

b) 6

c) 8

d) 9

8) For determination of flexural strength, the test of concrete can be done by


a) Centre point loading method and third point loading method
b) Centre point loading method and three point loading method
c) Midpoint loading method and three point loading method
d) One point loading method and third point loading method
9) The apparatus for conducting the slump test consists of a metallic mould of a
cone having the internal dimensions
a) Bottom diameter = 20 cm, Top diameter = 10 cm, Height = 31.5 cm
b) Bottom diameter = 20 cm, Top diameter = 10 cm, Height = 30 cm
c) Bottom diameter = 10 cm, Top diameter = 20 cm, Height = 31.5 cm
d) Bottom diameter = 10 cm, Top diameter = 20 cm, Height = 30 cm
10) For RCC slab the slump of the concrete should be
a) 0 25 mm

b) 25 50 mm

c) 25 100 mm

d) 50 100 mm
______________

Set A

*SLRBB62*

-3-

SLR-BB 62

Seat
No.

B.E. (Civil) (Part I) Examination, 2014


Elective I : ADVANCED CONCRETE TECHNOLOGY
Day and Date : Thursday, 11-12-2014
Time : 3.00 p.m. to 6.00 p.m.
Instructions : 1)
2)
3)
4)

Marks : 80

Solve any three questions from each Section.


Figures to right indicate full marks.
Use of non-programmable calculator is allowed.
Assume suitable data, if necessary and mention it clearly.
SECTION I

2. a) What is the difference between an additive and an admixture ?


b) Differentiate between :
i) An accelerator and
ii) A set-accelerating admixture.
3. How would you improve the bond of fresh concrete to hardened concrete ?

10
13

4. How can aggregate cause efflorescence in concrete ? How does the maximum
size of aggregate affect the workability of concrete with a given water content ? 13
5. a) List the various aspects of HPC.
b) What are the factors which control the performance of HPC ?

4
9

SECTION II
6. a) What is meant by a crack arrester in concrete ?
b) Discuss crack propagation in concrete.
7. a) What is the influence of mixing time on the strength of concrete ?
b) What are the particular requirements for pumpability of a concrete mix ?

4
9
4
9

8. What are the advantages and disadvantages of revibration of concrete ?

13

9. What are the two main categories of ready-mixed concrete ? Explain in detail.

14

_____________________

Set A

Set A

SLR-BB 63

*SLR-BB-63*
Seat
No.

Set

B.E. (Civil) (Part II) Examination, 2014


DESIGN OF CONCRETE STRUCTURES II
Day and Date : Tuesday, 25-11-2014
Time : 3.00 p.m. to 6.00 p.m.

Max. Marks : 100

Instructions : Question No. I is MCQ, which is compulsory and to be solved


in 30 minutes in Answer Book Page No. 3 while solving MCQ
IS 456-2000 and I.S 1343 are not allowed to refer.
Write the correct option of each question.
Answer MCQ/Objective type questions on Page No. 3 only.
Dont forget to mention, Q.P. Set (A/B/C/D) on Top of Page.
MCQ/Objective Type Questions

Duration : 30 Minutes
I.

Marks : 20

Choose the correct option :


1) For two way slab with shorter span up to 3.5 m with Tor steel for the span to overall
depth ratio for continuous slab.
a) 30
b) 40
c) 28
d) 32
2) Torsion reinforcement provided at mid-span in middle strip of two way slab
restrained shall extend in lower part of slab to width ___________ L of a
continuous edge.
a) 0.2
b) 0.25
c) 0.15
d) 0.3
3) The maximum working load that a structure has to withstand and for which it is to
be designed is called as
a) ultimate load
b) characteristic load
c) factored load
d) none of the above
4) Minimum grade of concrete used for pre stressed work should have a cube strength
of ______________ N/mm2 for post-tensioned.
a) 25
b) 30
c) 35
d) 40
5) The effective span of staircase supported at top and bottom risers by beams
spanning in parallel with the riser is
a) Clear distance between beams
b) Distance between c/c of beams
c) Clear distance between beams+d
d) None of the above
6) Effective span of continuous beam, if the width of support is greater than 1/12th clear
span or 600 mm, whichever is less for end span with one end free and other
continuous shall be
a) clear span between supports
b) c/c distance between supports
c) clear span + d
d) clear span + d/2

P.T.O.

SLR-BB 63

-2-

*SLR-BB-63*

7) The cable profile in prestressed concrete beam to resist u.d.1 throughout the span
shall be preferably.
a) Along N.A.
b) Parallel to N.A. c) Triangular
d) Parabolic
8) Which of the following is a type of retaining wall

1
1

I) Simply supported retaining wall


II) Counterfort retaining wall
III) Cantilever retaining wall

9)

10)
11)

12)

13)

14)

15)

16)

a) I and II only
b) II and III only
c) I and III only
d) I, II and III
In Cantilever retaining wall stem, heal slab and toe slab are all acting as
cantilever due to pressure from
a) Backfill only b) Soil pressure only c) Both a) and b) d) Neither a) or b)
In C.C.L. standard system____________ number of wires is tensioned at a time.
a) 2
b) 4
c) 1
d) 6
In Lee McCall system the diameter of the high tensile alloy steel is _________ mm and
above.
a) 6
b) 8
c) 10
d) 20
Plane section normal to the axis remains plane even after bending means _______
a) Stress along the depth of c/s is linear
b) Stress along the depth of c/s is parabolic
c) Strain along the depth of c/s is linear
d) Strain along the depth of c/s is parabolic
The main reinforcement of RC slab consists of 10 mm bars @ 10 cm spacing. It
is desired to replace 10 mm bars by 12 mm bars and then the spacing of 12 mm
bars should be
a) 12 cm
b) 14 cm
c) 14.40 cm
d) 16 cm
The shear coefficient of continuous beam of uniform c/s which supports udl over
three or more spans which do not differ by more than 10% of the long span are
____________ for D.L. and ____________ for L.L. at all other interior supports.
a) 0.4, 0.45
b) 0.6, 0.6
c) 0.55, 0.6
d) 0.5, 0.6
Limit state of serviceability for deflection including the effects due to creep, shrinkage
and temperature occurring after erection of partition and application of finishes as
applicable to floors and roofs is restricted to
a) Span/150
b) Span/200
c) Span/250
d) Span/350
For maximum sagging B.M.at support in a continuous RC beam, Live Load should
be placed on
a) Spans adjacent to support plus alternate spans
b) All the spans except the spans adjacent to the support
c) Spans next to adjacent spans of the support plus alternate spans
d) Spans adjacent to supports only
______________

1
1

1
1

Set A

*SLR-BB-63*

-3-

SLR-BB 63

Seat
No.

B.E. (Civil) (Part II) Examination, 2014


DESIGN OF CONCRETE STRUCTURES II
Day and Date : Tuesday, 25-11-2014
Time : 3.00 p.m. to 6.00 p.m.

Marks : 80

Instructions : Answer any three questions from each Section.


Use of I.S. 456 and non-programmable calculator is allowed.
Draw neat sketchs where required and assume suitable data
with clear notification.
SECTION I
II. Design a cantilever slab of 1.8 m span carrying a superimposed load of 5 kN/m2. The
slab is cantilevering out from a beam 250 mm 450 mm. The thickness of wall is 250 mm.
Use M20 mix and Fe 415 grade of steel. Draw neat sketch of distribution of steel.
III.

13

Design a dog legged stair case for an office building in a room measuring 2.5 m 5.8 m clear.
Vertical distance between floors is 3.6 m. Width of flight is 1.25 m. allow a live load of
3 kN/m2. Sketch the details of reinforcement. Use M20 concrete and Fe 415 steel.
Assume the stairs are supported on 230 mm walls at the end of outer edges of landing
slabs. Assume a rise of 150 mm and tread 300 mm respectively.

13

IV. Design the toe slab of a cantilever retaining wall, to retain an earth embankment with
a horizontal top 3.5 m above ground level. Density of earth 18 kN/m3, angle of internal
friction is 30 and S.B.C of soil 200 kN/m2. Co-efficient of friction
between soil and concrete is 0.5. Use M20 mix and Fe 500 grade of steel.

14

V. Design a three span continuous rectangular beam of effective span 6.25 m each to
carry a dead load of 13 kN/m and live load of 9.25 kN/m. The beam is supported by
columns. Use M20 mix and Fe 500 grade of steel. Sketch the reinforcement
details.

13

Set A

SLR-BB 63

-4-

*SLR-BB-63*

SECTION II
VI. A P.S.C. beam of rectangular section 230 mm 600 mm has a pan of 10 m. The
effective prestressing force is 980 kN at an eccentricity of 120 mm. The dead load of
the beam is 4.5 kN/m and live load of 7.5 kN/m. Determine the extreme stresses.
a) At the end of section
b) At mid span with out live load
c) At mid span with live load.

13

VII. A P.S.C. pile 250 mm 250 mm is prestressed by 60 wires of 3 mm diameter. The


wires are uniformly distributed over the section. The wires are initially subjected to a
pull of 300 kN. Find the final stress in concrete after all the losses. Take
ES = 2 105 N/mm2.
EC = 3 105 N/mm2, Relaxation loss percentage 5%.
esh = 1.9 104, Creep strain 25 106 N/mm2 of stress.

13

VIII. a) A rectangular PSC beam has a span of 15 m bears a LL of 15 kN/m excluding selfweight. The given permissible stresses in concrete and steel are 14 N/mm2
and 1100 N/mm2. Design the beam using 6 mm tendon.
10
b) Enumerate the different systems of prestressing and explain any one of them
with neat sketching.

IX. The end block of a post tensioned bridge girder is 600 mm wide and 1200 mm deep.
Two cables each comprising of 90 high tensile wires of 7 mm diameter are anchored
using square anchor plate with side 400 mm with their centers located 300 mm from
top and bottom edges of the beam. The jacking force in each cable is 3000 kN. Design
a suitable anchorage zone reinforcement using Fe 415 and HYSD bars. Design the
end block using IS 1343 code provisions.
13
_____________________

Set A

SLR-BB 64

*SLRBB64*
S

B.E. (Civil) (Part II) Examination, 2014


CONSTRUCTION PRACTICES AND TOWN PLANNING
Day and Date : Thursday, 27-11-2014
Time : 3.00 p.m. to 6.00 p.m.

Total Marks : 100

Instructions : 1)
2)
3)
4)

Use separate answer book for each section.


Figures on right indicate full marks.
Assume suitable data wherever required and mention it.
Q. No. 1 is compulsory. It should be solved in first 30
minutes in Answer book Page No. 3.
5) Answer MCQ/Objective type questions on Page No. 3
only. Dont forget to mention, Q.P. Set (A/B/C/D) on Top
of Page.

Duration : 30 Minutes

MCQ/Objective Type Questions

Marks : 20

1. A) Fill in the gaps with correct words :

10

1) The four essential objectives of town planning are ____________,


___________, _____________ and ____________
2) ____________ indicates the town is in the worst stage.

4
1

3) ____________ Cranes are capable of travelling on public roads.

4) The concept of Garden city is contribution of Sir ___________


5) The 3 basic parts of shovel family are ___________, ___________ and
_____________

1. B) Match A group items with correct B group items.


A Group

3
10

B Group

1) Selection of Equipment

1) The village community based on agriculture.

2) The output of power shovel

2) Removal of material in thin layers.

3) Natural growth of town


4) Growth according to
direction

3) Township for a defense establishment.


4) The type, size and other particulars of
the equipment.
P.T.O.

SLR-BB 64

-2-

5) Eopolis

*SLRBB64*

5) Association of population based on


specialization and mechanization.

6) Polis

6) Horizontal growth.

7) Stripping

7) Local planning prepared by keeping in


view local condition.

8) Cantonment

8) Improve stability of crane.

9) Development Plan of city

9) Type of material, depth of cut and angle


of swing.

10) Out riggers

10) Concentric spread.

______________

Set A

*SLRBB64*
S

-3-

SLR-BB 64

B.E. (Civil) (Part II) Examination, 2014


CONSTRUCTION PRACTICES AND TOWN PLANNING
Day and Date : Thursday, 27-11-2014

Marks : 80

Time : 3.00 p.m. to 6.00 p.m.


Instructions : 1) Use separate answer book for each section.
2) Figures on right indicate full marks.
3) Assume suitable data wherever required and mention it.
4) All questions are compulsory.
SECTION I
(Construction Practices)
2. Attempt any five questions :

(58=40)

a) Explain the terms operating cost and salvage value.


b) Enlist various Compaction Equipments and describe any one in detail.
c) Explain the safety measure practices in construction industry.
d) Explain the basic parts and operation of a Power Shovel.
e) Discuss the suitability and merit of prefabricated construction and insitu
construction.
f) Write a note on Reclaimed Asphalt Pavements.
g) Discuss the significance of Disaster Management in construction industry.
Set A

SLR-BB 64

-4-

*SLRBB64*

SECTION II
(Town Planning)
3. Attempt any five questions :

(58=40)

a) Describe in brief the objects of Town Planning.


b) Why are surveys conducted before town planning ? What is the chief subject
of surveys ?
c) Explain the advantages of zoning.
d) Describe Indus Valley Civilization.
e) Discuss planning of the modern town from town planning perspective.
f) Explain the forms of planning.
g) Explain Geddesian Triad by Sir Patrick Geddes.
_____________________

Set A

SLR-BB 65

*SLRBB65*
Seat
No.

Set

B.E. (CIVIL) (Part II) Examination, 2014


Elective II : GROUND IMPROVEMENT TECHNIQUES
Day and Date : Saturday, 29-11-2014
Time : 3.00 p.m. to 6.00 p.m.
Instructions :

Max. Marks : 100

1)
2)
3)
4)
5)

Q. No. 1 is multiple choice questions. Choose the correct option.


In Section I and II, solve any three questions.
Figures to the right indicate full marks.
Assume suitable data if required.
Q. No. 1 is compulsory. It should be solved in first 30 minutes in
Answer Book Page No. 3. Each question carries one mark.
6) Answer MCQ/Objective type questions on Page No. 3 only. Dont
forget to mention, Q.P. Set (A/B/C/D) on Top of Page.
MCQ/Objective Type Questions

Duration : 30 Minutes

Marks : 20

1. Choose the correct option :


1) In a cohesionless or coase-grained soil the densification is primarily
accomplished by re-orientation of grains which is resisted by
a) Inter particle force
b) Friction between the particles
c) Both a) and b)
d) None of above
2) Zero air void line is also called as
a) 0% saturation line
c) 100 % saturation line

b) 50% saturation line


d) All of above

3) The factor that must be considered in the selection of best technique include the following.
a) Material availability
b) Environmental consideration
c) Local experiences and preferences
d) All the above
4) Vibrating smooth drum rollers are most effective in compacting
a) Clean granular soil
b) Organic soil
c) Clay and silt
d) None of above
5) Sheep foot rollers are suitable for __________
a) Clay and silt-clay soil
b) Granular soil
c) Boulder
d) None of above
6) Dynamic compaction can density the soil
a) Upto 2 m
c) Only surface area

b) Upto 20 m
d) None of above

7) In the dewatering method of ground improvement __________


a) Water table is increased (elevated)
b) Water table is maintained at same level
c) Water table is covered
d) All the above
P.T.O.

SLR-BB 65

*SLRBB65*

-2-

8) The process of gradual load transfer from pore water to soil skeleton and the corresponding
gradual compression is called __________
a) Compaction
b) Consolidation
c) Elastic compression
d) None of above
9) The primary consolidation is normally __________
a) More than secondary consolidation
b) Less than secondary consolidation
c) Equal to secondary consolidation
d) None of above
10) In sand drains, the maximum particle size of sand should not be higher than __________
a) 10 mm
b) 2 mm
c) 20 mm
d) 4 mm
11) In electro-osmosis method, flow of water takes place __________
a) From cathode to anode
b) From anode to cathode
c) Cant say
d) All of above
12) Vibro-compaction method is most effectively used in __________
a) Saturated cohesive soil
b) Gravelly soil
c) Boulders
d) Saturated cohesionless soil
13) In blasting technique, compaction is done in more than one tier, if depth is more than
__________
a) 20 m
b) 5 m
c) 10 m
d) 25 m
14) Stone columns are very much suitable for
a) Soft, inorganic cohesive soil
c) Clean, granular soil

b) Dense silty soil


d) All of above

15) Low viscosity grouts are suitable for __________


a) Highly densed coarse grained soil
b) High permeable soil
c) Fine grained soil with low permeability d) None of above
16) The process in which soil gradation or arrangement is improved is called
a) Mechanical stabilization
b) Chemical stabilization
c) Electro-osmosis
d) Consolidation
17) The process in which clay minerals or clay-water systems are altered, called as _________
a) Commenting
b) Mechanical stabilization
c) Void filling
d) Physico-chemical alteration
18) Bituminous stabilization is suitable usually for soil having plasticity index __________
a) More than 20%
b) More than 30% c) Less than 18%
d) All the above
19) The main function of geosynthetics is
a) Separation
c) Reinforcement

b) Fluid transmission
d) All above

20) Polypropylene is mainly available in __________


a) Granular form
b) Liquid form
c) Gas form

______________

d) None of above

Set A

*SLRBB65*

-3-

SLR-BB 65

Seat
No.

B.E. (CIVIL) (Part II) Examination, 2014


Elective II : GROUND IMPROVEMENT TECHNIQUES
Day and Date : Saturday, 29-11-2014
Time : 3.00 p.m. to 6.00 p.m.

Marks : 80

Instructions : 1) In Section I and II, solve any three questions.


2) Figures to the right indicate full marks.
3) Assume suitable data if required.
SECTION I
2. A) Explain with neat sketch electro-osmotic method of ground improvement.
B) Write a note on stabilization by thermal technique.

8
5

3. A) Write in detail various factors to be considered in the selection of best soil improving
technique.
B) What is mechanical modification ? Give examples.

8
6

4. A) Discuss the principle of sand drain method and explain the procedure with suitable
sketch, how soils are densified by sand drain method.
B) Write a note on wickdrains.

8
5

5. A) With a neat sketch. Explain the procedure of installing stone column.


B) Discuss the suitability of stone column.

8
5

SECTION II
6. A) What is difference between Geogrid and Geotextile ? Explain field application of geogrid
and geotextile.
B) List various material to be used in soil reinforcement.

8
5

7. A) Enlist slope stabilization techniques. Write detail note on any one.


B) Explain field compaction control.

8
5

8. A) Write a note on soil-flyash stabilization. Give some examples where this method can
be used.
B) Discuss about soil line reaction.

8
5

9. A) Discuss in detail, the engineering situation where grouting can be advantageously


resorted to

B) Discuss suspension and solution grouting.


_____________________

Set A

Set A

SLR-BB 66

*SLRBB66*
Seat
No.

Set

B.E. (Civil) (Part II) Examination, 2014


TRAFFIC ENGINEERING AND CONTROL (Elective II)
Day and Date : Saturday, 29-11-2014
Time : 3.00 p.m. to 6.00 p.m.

Total Marks : 100

Instructions : 1) Q. No. 1 is compulsory. It should be solved in first 30 minutes


in Answer book Page No. 3. Each question carries one mark.
2) Answer MCQ/Objective type questions on Page No. 3 only.
Dont forget to mention, Q.P. Set (A/B/C/D) on Top of Page.
MCQ/Objective Type Questions
Duration : 30 Minutes
Marks : 20
1. Choose the correct answer.
1. In desire-line diagram
a) Width of desire-line is proportional to the number of trips in one direction
b) Length of desire-line is proportional to the number of trips in both directions
c) Width of desire-line is proportional to the number of trips in both directions
d) Both length and width of desire line are proportional to the number of trips
in both directions
2. The maximum allowable width, height and length of vehicle have been
Standardized by the
a) Indian Road Committee
b) Indian Road Congress
c) Centre Government
d) Indian Automobile Group
3. Vision is quite sensitive within a visual cone of
a) 5 or 6
b) 7 or 8
c) 9 or 10
d) 11 or 12
4. Physical characteristics of human includes
a) Hearing
b) Vision
c) Fatigue
d) All the above
5. Percentage of parking bays actually occupied by parked vehicles as compared
to the theoretical number available is called
a) Parking demand
b) Parking index
c) Parking load
d) None of the above
6. Provision of one-way streets results in
a) Increase in capacity
b) Increase in speed
c) Decrease in time of travel
d) All the above
7. Radar gun is used to find out
a) Delay
b) Journey speed c) Spot speed
d) Running speed
8. Which one of the following method is used for collecting O-D data ?
a) Licence plate method
b) Post card method
c) Home interview method
d) All the above
P.T.O.

SLR-BB 66

-2-

*SLRBB66*

9. Which one of the following method is not used for speed and delay study ?
a) Floating car method
b) Enoscope method
c) Licence plate method
d) Elevated observer method
10. Consider the following statements :
Collision diagram is used to
1. Study accident pattern.
2. Eliminate accidents.
3. Determine remedial measures.
4. Make statistical analysis of accidents.
Which of these statements are correct ?
a) 1 and 2
b) 3 and 4
c) 1 and 3
d) 2 and 4
11. Which of the following is correct ?
a) Running speed is greater than journey speed
b) Journey speed is greater than running speed
c) Running speed is equal journey speed
d) Both a) and c)
12. Maximum number of vehicles can be parked in
a) Parallel parking
b) 60 parking
c) 90 parking
d) 45 parking
13. Which one of the following is not in a road safety programme ?
a) Engineering
b) Technology
c) Education
d) Enforcement
14. Accident record includes
a) Location files b) Spot maps
c) Fatality records d) All the above
15. The community parking space required for a car can be considered as
a) 2 4 m
b) 2.5 5 m
c) 2.5 5.5 m
d) 3 6 m
16. The designation of various sections of highway for different values of safe
speed limits is called as
a) Speed zoning
b) Speed control area
c) Speed boundary
d) None of the above
17. By installing traffic signal at the intersection, especially _________ type of
accidents reduces.
a) Rear-End collision
b) Head on collision
c) Right angled collision
d) All the above
18. Marking intended for restriction on waiting and parking is __________ colour.
a) White
b) Yellow
c) Black
d) Black and White
19. Curb parking creates which one of the following problem ?
a) Decreased street capacity
b) Increased accidents
c) More traffic congestion
d) All the above
20. In the signal design, which of the following criteria is taken in to consideration
for computing duration of yellow light ?
a) Stopping time b) Clearance time c) Both a) and b) d) None of the above
______________
Set A

*SLRBB66*

SLR-BB 66

-3-

Seat
No.

B.E. (Civil) (Part II) Examination, 2014


TRAFFIC ENGINEERING AND CONTROL (Elective II)
Day and Date : Saturday, 29-11-2014
Time : 3.00 p.m. to 6.00 p.m.
Instructions : 1)
2)
3)
4)

Marks : 80

Answer any two questions from each Sections.


Figures to the right indicates full marks.
Draw neat sketch wherever it is necessary.
Any missing data can be suitably assumed.
SECTION I

1. a) What are the resistances considered for determining the power performance
of the vehicles ? And discuss them briefly with neat sketch.
b) Explain PIEV theory. Also list out the factors affecting the total reaction time
of the driver. What is the IRC recommended total reaction time of a driver ?
c) Explain :
1) Eye movement

8
6
6

2) Peripheral Vision.

2. a) Define O-D studies and its application. Briefly explain Desired line diagram
with neat sketch.
b) What are the objectives of Traffic Volume Count ? Briefly explain any one
method of traffic volume count.
c) A two-lane two way straight road is intersecting with another two-lane two
way straight road at right angle. Show the conflict points in the diagram. And
find out the total number of conflict points.
3. a) What are the objectives of parking studies ? Also justify how this parking
study data is useful in reducing traffic congestion and accidents.
b) Two vehicles A and B approaching at right angles. A from west and B from
south collide with each other. After the collision, vehicle A skids in a direction
50 north of west and vehicle B 60 East of North. The initial skid distances
of the vehicles A and B are 38 and 20 m respectively, before collision. The
skid distances after collision are 15 m and 36 m respectively. If the weights
of vehicles A and B are 4.4 and 6 tonnes respectively. Calculate the
original speeds of the vehicles. The average skid resistance of the pavement
is found to be 0.55.
c) List out the common methods of on-street parking and discuss any one.

8
8
4
8

8
4

Set A

SLR-BB 66

-4-

*SLRBB66*

SECTION II

4. Write a short note on (any four) :


1) Driver licencing.
2) Financial responsibility.
3) Vehicle registration.
4) Pedestrian rules.
5) One-way streets.
6) Problems created by curb parking.

20

5. a) Discuss the basic requirements of traffic control devices.

b) What are the basic requirements of traffic signals ?


c) The average normal flow on cross roads A and B during design period are 400
and 250 per/hr. The saturation flows are 1250 and 1000 per/hr. respectively.
The all red time required for pedestrian crossing in 12 sec. Design a twophase traffic signal by Websters method.
6. a) Write a short note on traffic marking.
b) Explain :

8
8
12

1) Bump integrator.
2) Radar gun.
3) Portable skid resistance meter.
_________________

Set A

SLR-BB 67

*SLRBB67*
Seat
No.

Set

B.E. (Civil) (Part II) Examination, 2014


SOLID AND HAZARDOUS WASTE MANAGEMENT (Elective II)
Day and Date : Saturday, 29-11-2014
Time : 3.00 p.m. to 6.00 p.m.

Max. Marks : 100

Instructions : 1) Right sides figure shows marks.


2) Draw the neat sketch wherever necessary.
3) Solve any three questions from Section I. However
Q. No. 2 is compulsory.
4) Solve any three questions from Section II. However
Q. No. 6 is compulsory.
5) Q. No. 1 is compulsory. It should be solved in first 30 minutes in
Answer Book Page No. 3. Each question carries one mark.
6) Answer MCQ/Objective type questions on Page No. 3 only. Dont
forget to mention, Q.P. Set (A/B/C/D) on Top of Page.
MCQ/Objective Type Questions
Duration : 30 Minutes

Marks : 20

1. Choose the correct answer :

(201=20)

1) The most serious environmental effect posed by hazardous wastes is


a) Air pollution
b) Contamination of groundwater
c) Increased use of land for landfills
d) Destruction of habitat
2) Landfills are generally places where
a) Microbes successfully break down the majority of trash produced
b) Composting occurs, as there is plenty of water and oxygen
c) Castoff materials find a final burial ground
d) Biodegradation readily occurs
3) Recovery of energy in the form of heat is obtained from
a) Combustion process
b) Sanitary landfilling
c) Anaerobic process
d) None of these
4) Which of the following is NOT a major option for managing hazardous waste ?
a) Reducing the production of waste while increasing the amount generated
b) Reducing the volume and/or hazard of the waste
c) Long-term storage or disposal
d) Dumping at sea
5) Mechanical size reduction operation includes
a) Shredding
b) Grinding
c) Milling

d) All of the above

6) Which of the following is not a method for the disposal of hazardous wastes ?
a) ISV
b) Landfills
c) Deep-will injection
d) Surface impoundments
P.T.O.

SLR-BB 67

*SLRBB67*

-2-

7) Leachate is colored liquid that comes out of


a) Septic tank
b) Sanitary landfill c) Aerated lagoon

d) None of the above

8) Tub grinders are used for grinding


a) Cloth waste
b) Yard waste

d) None of these

c) Glass waste

9) Which of the following is not a source reduction activity ?


a) Products package reuse
b) Reducing use by modifying practices
c) Saving energy by using recycled materials
d) Package or product design that reduces material or toxicity
10) Which is not true about incineration of solid waste ?
a) There are drastic reductions in the volume and weight of wastes
b) The incinerators are relatively cheap to build
c) The ash can contain heavy metals and other toxic substances
d) The incinerators can be built to generate electricity
11) Which of the following is a practice used to reduce and manage MSW ?
a) Waste combustion
b) Source reduction
c) Recycling of materials
d) All of the above
12) Which of the following is/are negative effects on the soil and water due to conventional
mechanized farming practices ?
a) Soil compaction
b) Reduction in soil organic matter
c) Soil erosion
d) None of the above
13) High level radioactive waste can be managed in which of the following ways ?
a) Open dumping b) Composting c) Incineration
d) Dumping in sealed containers
14) Biomedical waste may be disposed of by ?
a) Incineration
b) Autoclaving
c) Land filling

d) Both b) and c)

15) Which of the following is a biodegradable organic chemical/substance ?


a) Plastics
b) Oils
c) Pesticides
d) Garbage
16) The largest single component of Municipal Solid Waste (MSW) is
a) Plastics
b) Food wastes
c) Yard wastes
d) Paper and paper products
17) An environmental problem with unsecured landfills is
a) Groundwater pollution
b) Methane production
c) Incomplete decomposition
d) All of the above
18) A modern landfill is required to
a) Be sited on a location well above the water table
b) Be surrounded by groundwater monitoring wells
c) Have a leachate collection system
d) All of the above
19) Which of the following components of municipal solid wastes can be recycled ?
a) Metals
b) Paper and paper products
c) Glass
d) All of the above
20) Which of the following is not a material in MSW ?
a) agricultural wastes b) food wastes
c) glass and plastics
______________

d) wood wastes
Set A

*SLRBB67*

SLR-BB 67

-3-

Seat
No.

B.E. (Civil) (Part II) Examination, 2014


SOLID AND HAZARDOUS WASTE MANAGEMENT (Elective II)
Day and Date : Saturday, 29-11-2014
Time : 3.00 p.m. to 6.00 p.m.
Instructions :

Marks : 80

1) Right sides figure shows marks.


2) Draw the neat sketch wherever necessary.
3) Solve any three questions from Section I. However
Q. No. 2 is compulsory.
4) Solve any three questions from Section II. However
Q. No. 6 is compulsory.
SECTION I

2. a) What does the functional elements transfer and transport mean in solid waste management. 7
b) Write the air pollution problems associated with incineration.

3. a) Write a short note on recycling of solid waste.

b) Explain the term incineration with neat sketch. Enlist merits and demerits of incinerator.
4. a) Describe the effect of following on composting process :
a) C/N ratio

b) Temperature

c) Moisture content

7
d) pH

b) Describe the classification of solid waste in detail.

5. a) Estimate the moisture content of solid waste sample of 100 kg using the following data :
Sr. No.

Component

%by mass

%by M.C.

Food waste

16

65

Paper

36

08

Cardboards

05

05

Plastics

10

02

Grass

12

55

Wood

08

04

Metals

13

03

b) Enlist the various types of composting methods. Explain any one in detail.

6
Set A

SLR-BB 67

-4-

*SLRBB67*

SECTION II
6. a) What are the various types of landfilling techniques ? Explain the control measures to be
taken while landfilling techniques.
b) Explain in brief how will you minimize the damage due to manmade hazard.

8
6

7. a) Define leachate and explain in short any two methods used to control the flow of leachate. 7
b) What are the factors governing composting techniques ?

8. a) Writes a detailed note on waste minimization.

b) Write the steps for recycling of solid waste.

c) Explain with one example 3R formulae.

9. a) What is recycling ? What are some items at your home that can be recycled ?
b) Write in detail about risk assessment.

7
6

_____________________

Set A

SLR-BB 68

*SLRBB68*
S

B.E. (Civil) (Part II) Examination, 2014


Elective II : DESIGN OF BRIDGES
Day and Date : Saturday, 29-11-2014
Time : 3.00 p.m. to 6.00 p.m.

Max. Marks : 100

Instructions : 1)
2)
3)
4)

Solve any three questions each from Section I and Section II.
Figures to the right indicates full marks.
Assume suitable data if necessary and mention it clearly.
Q. No. 1 is compulsory. It should be solved in first 30 minutes in
Answer Book Page No. 3. Each question carries one mark.
5) Answer MCQ/Objective type questions on Page No. 3 only. Dont
forget to mention, Q.P. Set (A/B/C/D) on Top of Page.
MCQ/Objective Type Questions

Duration : 30 Minutes

Marks : 20

1. Give the correct option for the following questions.

(120=20)

1) Aqueduct is the classification of bridge according to


a) Material
b) Function
c) Type of superstructure
d) Inter-span relationship
2) Section VII of IRC bridge code deals with
a) Composite construction
c) Cement concrete

b) Bearings
d) Foundations and substructure

3) To provide for the possible variation of the direction of the water current, allowance for
an extra variation of water current direction is
a) 15
b) 20
c) 25
d) 35
4) The minimum clearance for class AA loading is
a) 0.15 m
b) 1.0 m
c) 1.1 m

d) None of these

5) The minimum height of Kerb may be taken as ________ above the road level.
a) 250 mm
b) 225 mm
c) 600 mm
d) 300 mm
6) Permissible shear stress (
) for M-25 concrete is
a) 0.28 N/mm2
b) 0.34 N/mm2
c) 0.4 N/mm2

d) 0.45 N/mm2

7) Pigeauds method is most useful when K is more than


a) 0.44
b) 0.50
c) 0.55

d) 0.60

8) Pigeauds curves are used to calculate


a) Bending moment coefficients
c) Impact factor

b) Load factor
d) Effective span
P.T.O.

SLR-BB 68

*SLRBB68*

-2-

9) Load factor K by Courbons theory for uniform MI is given by,


a)

b)




n


W




n

n
X

c)

d)

W



n



E

X



n

n
X





E

10) Permissible maximum compressive stress of plain concrete M20 is


a) 7.0 MPa
b) 2.7 MPa
c) 5.0 MPa
d) 1.5 MPa
11) Expansion bearing allow
a) Rotation only
c) Both rotational as well as translation

b) Translation only
d) Restrict rotation as well as translation

12) Following is not type of Expansion bearing


a) Sliding plate beam
b) Steel roller-cum-rocker bearing
c) Elastomeric bearing
d) Rocker bearing
13) Elastomeric having ultimate tensile strain
a) 300 percent maximum
c) 400 percent maximum

b) 300 percent minimum


d) 400 percent minimum

14) Guideline for the design of bridge foundation are available in IRC bridge code section
____________
a) I
b) III
c) V
d) VII
15) Which of the following statement is not true ?
a) Seismic and wind forces shall be considered to act simultaneously
b) Horizontal force due to seismic effect shall be taken to act through the C.G. of all
the force under consideration
c) Centrifugal force shall be considered to act at the height of 1.2 m above the level of
carriage way
d) None of above
16) Expansion joint are provided at
a) Support
b) Foundation

c) Abutment

d) None of these

17) For selecting bearing, following factor is considered


a) High vertical load taking capability
b) Movement capability to cope with horizontal movements
c) Rotational capability
d) All
18) Rocker bearing are suitable for
a) R.C.C. bridge
c) Straight steel bridges

b) Timber bridge
d) All

19) For pre-stress concrete bridge minimum grade of concrete is


a) M-30
b) M-20
c) M-40

d) M-50

20) Hardness of elastomeric bearing shall be _________ on IRHD.


a) 75
5 degrees
b) 25
5 degrees
c) 60
5 degrees
d) 80
5 degrees
______________


Set A

*SLRBB68*
S

-3-

SLR-BB 68

B.E. (Civil) (Part II) Examination, 2014


Elective II : DESIGN OF BRIDGES
Day and Date : Saturday, 29-11-2014
Time : 3.00 p.m. to 6.00 p.m.

Marks : 80

Instructions : 1) Solve any three questions each from Section I and Section II.
2) Figures to the right indicates full marks.
3) Assume suitable data if necessary and mention it clearly.
SECTION I
2. Answer the following :
A) What are the basic components of bridge structure ? Explain with the help of sketch.

B) What is the importance of subsoil exploration in the design of major bridges ? List the
data to be obtained from such explorations.

C) Give a critical review of IRC loading for bridges.

3. Find the design bending moment of two lane bridge solid deck slab for following data :
a) Effective span 6.5 m
b) Carriage way width 9 m
c) Kerb 600 275 on both side
d) Live load IRC Class A (Two lane)
e) Wearing coat 100 mm thick
f) Use M-30 concrete and Fe-415 steel
g) Use
= 2.77
Find the percentage change in the design bending moment if the Live load of IRC class AA
tacked is used.

13

4. A RCC T beam type bridge having deck slab of 220 mm thick, wearing coat of 80 mm thick,
three longitudinal girders and five cross girders. Determine the design bending moment for all
the longitudinal girders. Use following additional data :
a) Carriage way width 9 m
b) Span of bridge 16 m
c) Live load IRC class AA Tracked
d) Kerb 600 mm wide, 400 mm deep
e) Web thickness for Longitudinal and cross girder 300 mm
f) Longitudinal Girder Spacing 3 m
g) Use M-30 concrete and Fe-415 steel
Find the percentage change in the design bending moments if four longitudinal girders are
provided with spacing of 2.3 m.

13

Set A

SLR-BB 68

-4-

*SLRBB68*

5. Design a slab panel having size of 2.7 m 3.375 m. Consider IRC class AA tracked loading.
Use Pigeauds chart. Consider thickness of slab as 200 mm and wearing coat thickness as
85 mm. Use M-30 concrete and Fe-415 steel. Refer Fig. 1 and 2 for Pigeuads coefficient.

13

Set A

*SLRBB68*

-5-

SLR-BB 68

Set A

SLR-BB 68

-6-

*SLRBB68*

SECTION II
6. Verify the adequacy of pier for the following data :
Top width of pier 1.8 m, Height of pier upto springing level 12 m, C/C distance of bearing 1.2 m,
Side batter 1 : 14, HFL 1.5 m below the bearing level, Span of bridge 16 m, Self weight of
the superstructure = 250 kN/m, Live load IRC class AA tracked, Material of pier = M20
concrete.
13
7. Verify the suitability of abutment as shown in the fig 7.1. Use following data Density
= 32. Coefficient of friction 0.6, Live load IRC
of soil 17.5 kN/m3, Friction angle of soil
class AA tracked. DL reaction of superstructure 1400 kN.

13

Fig. No. 7.1


8. A) Design a elastomeric unreinforced bearing pad for following data
Vertical load (sustained) = 185 kN,
Vertical load (dynamics) = 60 kN,
Horizontal force = 85 kN
Modulus of rigidity of elastomer 1.2 N/mm2
Coefficient of friction = 0.35.
B) Write a note on Expansion Joints.
9. Write a note on following (any four) :
a) Types of bridge pier with their suitability
b) Functions of bearing
c) Approach slab
d) Reinforced earth retaining wall
e) Importance of bridge inspection.

8
5
(3.54=14)

_____________________

Set A

SLR-BB 69

*SLRBB69*
Seat
No.

Set

B.E. (Civil) (Part II) Examination, 2014


Elective II : INFRASTRUCTURAL ENGINEERING
Day and Date : Saturday, 29-11-2014
Time : 3.00 p.m. to 6.00 p.m.

Max. Marks : 100

Instructions : 1) Q. No. 1 is compulsory. It should be solved in first


30 minutes in Answer Book Page No. 3. Each question
carries two marks.
2) Answer MCQ/Objective type questions on Page No. 3 only.
Dont forget to mention, Q.P. Set (A/B/C/D) on Top of Page.
MCQ/Objective Type Questions
Duration : 30 Minutes

Marks : 20

1. Choose the correct answer :


1) What is meant by contract type O & M.
a) Operation and maintenance
b) Own and maintenance
c) Operate and manage
d) Own and manage
2) Which of the following is correct term for DBFMO ?
a) Design Build Finance Maintain Own
b) Design Build Finance Maintain Operate
c) Design Build Finance Manage Operate
d) Design Build Finance Manage Own
3) Which of the following is not principle of Public Private Partnership ?
a) Ownership by stakeholder
b) Joint venture
c) Simplicity in usage and management
d) Flexibility
P.T.O.

SLR-BB 69

*SLRBB69*

-2-

4) Which of the following is benefit for public sector of Public Private Partnership ?
a) Lifecycle cost management
b) Business opportunities
c) Export

d) None of the above

5) Which of the following is benefit for Private sector of Public Private


Partnership ?
a) Innovative solutions
c) Construction management

b) Export opportunities
d) Sharing the assets

6) Which of the following is not risk in infrastructure ?


a) Demand risk

b) Economic risk

c) Agricultural risk

d) Political risk

7) Which of the following is role of private sector in Public Private Partnership ?


a) Access to private finance
b) Sufficient return to investors
c) Providing the facility and service
d) All of the above
8) What is meant by the term OBA contract ?
a) Output Based Assistance

b) Output Build Aid

c) Operate Build Assistance

d) Output Based Aid

9) How many contributing factors are considered for risk emergence ?


a) 9
b) 10
c) 11
d) 12
10) Which of following is principle aspect of sustainable development ?
a) Energy

b) Environmental

c) Simplicity

d) Amenity
______________

Set A

*SLRBB69*

-3-

SLR-BB 69

Seat
No.

B.E. (Civil) (Part II) Examination, 2014


Elective II : INFRASTRUCTURAL ENGINEERING
Day and Date : Saturday, 29-11-2014
Time : 3.00 p.m. to 6.00 p.m.

Marks : 80

Instructions : 1) Solve any three questions from Section I and any three
questions from Section II.
2) Assume necessary data if required and mention it clearly.
3) Figures to right indicate full marks.
SECTION I
1. a) Explain the term Design Build Finance Maintain (DBFM).
b) State the major problems with the transport sector in India.
2. a) What is meant by Public Private Partnership ?
b) What is the role and responsibilities of Government in Public Private
Partnership ?
3. a) What are the salient features of a Public Private Partnership ?
b) What are the role and responsibilities of private agencies in Public Private
Partnership ?
4. a) Draw a neat diagram for model of public private partnership.
b) Explain in detail the management contracts ?

4
10
4
9
4
9
4
9

Set A

SLR-BB 69

-4-

*SLRBB69*

SECTION II
5. a) Draw a schematic diagram of four phases of the project life cycle.

b) What activities are considered in project initiation ?

10

6. Discuss the points to be considered for risk allocation frame for BOT (toll)
project.

13

7. a) Write in brief on risk management in Public Private Partnership.

b) Explain the role of asset management in risk reduction.

8. a) What are the benefits of green infrastructure projects ?

b) What are the main advantages of adopting sustainability principles for building
and infrastructure ?

_____________________

Set A

SLR-BB 70

*SLRBB70*
Seat
No.

Set

B.E. (Civil) (Part II) Examination, 2014


ADVANCED DESIGN OF CONCRETE STRUCTURES (Elective III)
Day and Date : Monday, 1-12-2014
Time : 3.00 p.m. to 6.00 p.m.
Instructions :

Duration : 30 Minutes
I.

Total Marks : 100

i) Q. No. I is compulsory. It should be solved in first 30 minutes in


Answer Book Page No. 3. Each question carries one mark.
ii) Answer any two questions from each Section.
iii) Use of IS 456 and IS 3370.
iv) Assume suitable data if necessary.
v) Draw neat sketches wherever necessary.
vi) Answer MCQ/Objective type questions on Page No. 3 only.
Dont forget to mention, Q.P. Set (A/B/C/D) on Top of Page.
MCQ/Objective Type Questions

Marks : 20

Choose the correct answer :


1) In water tanks the compressive (permissible) stress in column subjected to direct
load is ____________ N/mm2 for HYSD bars.
a) 115
b) 125
c) 175
d) 150
2) Thickened part of a flat slab over its supporting column, is technically known as
a) Drop panel
b) Capital
c) Column head
d) None of these
3) Middle strip in flat slab means the design strip bounded on each of its opposite
sides by
a) Panel
b) Column strip
c) Drop
d) Column heads
4) In case of grid slab the in situ ribs shall not be less than ________ mm wide.
a) 60
b) 50
c) 65
d) 75
5) The minimum longitudinal reinforcement in pile should not be less than _____%
of c/s area of pile for piles having length up to 30 times their least lateral dimension.
a) 1
b) 2
c) 1.25
d) 1.5
6) The tanks situated underground, the walls of the tanks are to be generally designed
for
a) Earth pressure only
b) Water pressure only
c) Both Earth and water pressure
d) All of these
7) The floor of the underground water tanks are designed for _________ pressure,
for the empty tank condition.
a) Uplift
b) Water
c) Earth
d) All of these
8) For design of water tanks the permissible stress in bending tension in HYSD bars
is for is ____________ N/mm2.
a) 115
b) 140
c) 150
d) 230

P.T.O.

SLR-BB 70

*SLRBB70*

-2-

9) In case of water tanks the permissible shear stress for M20 grade concrete is
_______ c N/mm2
a) 1
b) 1.2
c) 1.25
d) 1.7
10) The rectangular footing is required in following situations
a) If loaded footing of columns overlap
b) Columns may be near to property line
c) S. B. C. of soil being less
d) All a), b) and c)
11) The minimum spacing of piles shall be _________ to _________ times diameter
of the pile.
a) 2 to 3
b) 2.5 to 3
c) 1.5 to 2
d) 1 to 1.5
12) The minimum percentage in walls, floors and roofs in each of the two directions at
right angles shall have an area of __________ percent of the concrete section in
that direction for sections up to 100 mm thickness.
a) 0.3
b) 0.25
c) 0.2
d) 0.35
13) The walls of underground water tanks are designed for __________ pressure,
specially under the condition when tank is empty.
a) Water
b) Earth
c) Both a) and b) d) All of these
14) For a circular slab circumferential moment per unit length is given by M = _______
a) M = C2qR 0.5

b) M = C 2qR 3

c) M = C 2qR 4

d) M = C 2qR 2

15) If W is the load on a circular slab of radius R, the maximum circumferential moment
at the centre of the slab, is
a) WR2/16
b) 2WR2/16
c) 3WR2/16
d) Zero
16) Piles are usually driven by
a) Diesel operated hammer
c) Single acting steam hammer

b) Drop hammer
d) All the above

17) A raft foundation is provided if its area exceeds the plan area of the building by
a) 10%
b) 20%
c) 40%
d) 50%
18) Grid slabs are economical up to size __________ m spans.
a) 5-10 m
b) 10-25 m
c) 8-10 m
d) 6-9 m
19) According to I.S. : 456, 1978 the thickness of reinforced concrete footing on piles
at its edges, is kept less than
a) 20 cm
b) 30 cm
c) 40 cm
d) 50 cm
20) The permissible tensile stress in M.S. bars on liquid retaining face is _____ N/mm2.
a) 115
b) 100
c) 125
d) 150
______________

Set A

*SLRBB70*

-3-

SLR-BB 70

Seat
No.

B.E. (Civil) (Part II) Examination, 2014


ADVANCED DESIGN OF CONCRETE STRUCTURES (Elective III)
Day and Date : Monday, 1-12-2014

Marks : 80

Time : 3.00 p.m. to 6.00 p.m.


Instructions :

i) Answer any two questions from each Section.


ii) Use of IS 456 and IS 3370.
iii) Assume suitable data if necessary.
iv) Draw neat sketches wherever necessary.
SECTION I

1. a) Design the interior panel of a flat slab 5.6 m 6.6 m in size, for a super imposed
load of 7.75 kN/m2. Provide two way reinforcement. Use M20 grade of concrete
and Fe415 steel.

12

b) Sketch the reinforcement details for grid slab.

2. An R.C. column 450 mm 450 mm carrying a load of 550 kN is supported on three


piles 450 mm 450 mm in section. The centre to centre distance between the pile is
2.5 m. Design a suitable pile cap. Use M25 grade of concrete and Fe415 steel.

20

3. a) Design combined rectangular footing for two columns A and B carrying loads of
500 kN and 700 kN respectively. Column A is 300 mm 300 mm in size and
column B is 400 mm 400 mm in size. The centre to centre spacing of the columns
is 3.4 m. The safe bearing capacity of soil may be taken as 200 kN/m2. Use M20
grade of concrete and Fe415 Steel.
b) Explain the significance of raft foundation.

15
5

Set A

SLR-BB 70

-4-

*SLRBB70*

SECTION II
4. a) A rectangular water tank 4.5 m long 2.25 m wide and 2.25 m high has its walls
rigidly jointed at the critical edges. Design the tank by IS code method, it is supported
an all sides under the wall. Use M25 grade of concrete and Fe415 Steel.

12

b) Explain the design procedure for approximate method of rectangular water tank
resting on the ground.

5. Design an underground water tank 4.5 m 10 m 3.5 m deep. The subsoil consists of
sand having angle of repose of 30 and saturated unit weight of 18 kN/m3. The water
table is likely to rise up to ground level. Use M25 grade of concrete and Fe415 Steel.
Take unit weight of water as 9.81 kN/m3.

20

6. Design a circular water tank with fixed base for capacity of 3 lakh litres. The depth of
water is to be 3.8 m including a free board of 200 mm. Use M25 grade of concrete and
Fe500 Steel. Assume = 0. The tank is free at top and rest on the ground. Take unit
weight of water as 9.8 kN/m3.

20
_____________________

Set A

SLR-BB 71

*SLRBB71*
Seat
No.

Set

B.E. (Civil) (Part II) Examination, 2014


Elective III : INDUSTRIAL WASTE TREATMENT
Day and Date : Monday, 1-12-2014
Time : 3.00 p.m. to 6.00 p.m.
Instructions :

Total Marks : 100

1)
2)
3)
4)

Figures to the right indicate full marks.


Assume suitable data wherever necessary and mention it clearly.
Use of non programmbale calculator is allowed.
Q. No. 1 is compulsory. It should be solved in first 30 minutes in
Answer book Page No. 3. Each question carries one mark.
5) Answer MCQ/Objective type questions on Page No. 3 only. Dont
forget to mention, Q.P. Set (A/B/C/D) on Top of Page.
MCQ/Objective Type Questions

Duration : 30 Minutes

Marks : 20

1. Choose the correct option :

20

1) The biochemical reactions within stream exert BOD resulting _____________ of the stream.
a) Reareation
b) Dissolved oxygen
c) Deoxygenation
d) Self purification
2) The intensity of microbial activities is reflected by ____________
a) Chemical oxygen demand
b) Biological oxygen demand
c) Dissolved oxygen
d) Nitrogenous oxygen demand
3) The _____________ phase of microorganisms are forced to oxidize for energy.
a) Endogenous growth
b) Log growth
c) Declining growth
d) All of above
4) Aerobic condition in effluent treatment can be achieved by the use of
_____________
a) Dissolved oxygen
b) Diffused aeration
c) Coagulation
d) Sludge control
5) ______________ is meant for reduction of moisture content of sludge.
a) Dewatering
b) Conditioning
c) Thickening
d) Incineration
6) ______________ is a process of removal of non biodegradable organics using fixed bed
of activated carbon column.
a) Adsorption
b) Reverse osmosis
c) Air stripping
d) Chemical precipitation
7) The substrate removal in trickling filters is directly related to the surface area of
____________
a) Organic loading
b) Recirculation
c) Effluent characteristics
d) Filter media
P.T.O.

SLR-BB 71

-2-

*SLRBB71*

8) For a conventional sludge digesters detention period of _____________ is provided


a) 20 to 30 minutes
b) 30 to 90 days
c) 4 to 6 hrs.
d) 1 to 2 days
9) _______________ is a convenient device for measuring flow in a sewer
a) Manometer
b) Thermometer
c) Parshall flume
d) Water meters
10) _____________ are used to improve settling and separation of coagulated substance.
a) Alum
b) Precipitation
c) Activated carbon
d) Polyelectrolytes
11) ______________ mill waste contains the presence of lignin derived from cellulosic material
a) Textile
b) Pulp and paper
c) Sugar
d) Tannery
12) Yeast sludge containing rich in proteins, carbohydrates vitamins are treated separately
for ______________
a) Segregation
b) Recylce
c) Byproduct recovery
d) High efficiency
13) In a tannery industry hides are limed with a paste of lime and ___________
a) Hydrogen sulphide
b) Calcium sulphide
c) Nitrogen sulphide
d) Sodium sulphide
14) ____________ is a process in which yarn is strengthened by loading starch
a) Mercerising
b) Slashing
c) Weaving
d) Carding
15) In a sugar mill the clarified juice is bleached by ____________ process.
a) Sulphitation
b) Dechlorination c) Aeration
d) Coagulation
16) _____________ involves the exposure of waste in increasing the concentration of
microbiological population.
a) Nitrifying of bacteria
b) Denitrifying of bacteria
c) Acclimatization of bacteria
d) Photosynthesis
17) Malt making and Brewing is one of the process of ___________ Industry.
a) Pulp and paper
b) Distillery Industry
c) Sugar Industry
d) Dairy Industry
18) Carbonates and nitrogen are ______________ substance to encourage the growth of
micro-plants in body of water.
a) Organic
b) Toxic
c) Colour-producing d) Inorganic
19) _____________ involves oxidation of ammonia to nitrate via nitrite under aerobic condition.
a) Activated biological treatment
b) Biological nitrification
c) Aeration
d) Reoxygenation
20) Addition of sodium sulphate to concentrated black liquor result in formation of ___________
a) Green liquor
b) White liquor
c) Spent liquor
d) Lime mud
______________

Set A

*SLRBB71*

-3-

SLR-BB 71

Seat
No.

B.E. (Civil) (Part II) Examination, 2014


Elective III : INDUSTRIAL WASTE TREATMENT
Day and Date : Monday, 1-12-2014

Marks : 80

Time : 3.00 p.m. to 6.00 p.m.


Instructions :

1) Solve any three questions from Section I i.e. Question no. 2 to


Question no. 5. Question no. 6 is compulsory in Section II and
solve any two questions from the remaining.
2) Figures to the right indicate full marks.
3) Assume suitable data wherever necessary and mention it clearly.
4) Use of non programmbale calculator is allowed.
SECTION I

2. a) Discuss in detail about the Physical unit processes commonly used in waste water treatment. 8
b) Explain the process for removal of color from waste water by activated carbon treatment.

3. a) City discharge sewage at the rate of 1200 litres per second into the stream whose minimum
flow is 5000 litres per, the temperature of both being 20C. The 5 day BOD at 20C for
sewage is 160 mg/I and that of river water is 2 mg/I. The DO content of sewage is zero
while that of stream is 90% of the saturation Do. Find out the degree of treatment required
if the minimum DO to be maintained in the stream is 4 mg/I. Assume deoxygenation coefficient
as 0.1 and reoxygenation coefficient as 0.3. Given saturation DO at 20C as 9.17 mg/I.
b) What is meant by self purification capacity of stream ?
4. a) Explain By-product recovery from wastewater.
b) Enlist various methods of waste strength reduction. Explain any one in details.

9
6
5
7

5. Write short note on :


I) Equipment change

II) Electrodialysis process.

7
Set A

SLR-BB 71

-4-

*SLRBB71*

SECTION II
Instruction : Question no. 6 is compulsory in Section II and solve any two
questions from the remaining.
6. a) Explain with flow diagram the Massive Lime Treatment for color removal in pulp and
paper mill.

b) What are the effects of pulp and paper mill waste on receiving water sewers.

7. Give the characteristics of wastewater, draw the wastewater treatment flow diagram and
explain in detail.
a) Pulp and paper industry

b) Sugar industry.

8. Draw manufacturing process flow diagram, indicate the sources wastes and explain in detail.
a) Tannery Industries

b) Distillery industries.

9. Write short note on :

14

I) Operation and maintenance requirement


II) Process sensitivity.
_____________________

Set A

SLR-BB 72

*SLRBB72*
Seat
No.

Set

B.E. (Civil) (Part II) Examination, 2014


Elective III : WATER POWER ENGINEERING
Day and Date : Monday, 1-12-2014
Time : 3.00 p.m. to 6.00 p.m.
Instructions :

Max. Marks : 100

1) Solve any three questions from Section I (Questions 2, 3, 4, 5)


and any three questions from Section II. (Questions 6, 7, 8, 9)
2) Draw neat labeled sketches wherever necessary.
3) Assume suitable data if necessary and state it clearly.
4) Q. No. 1 is compulsory. It should be solved in first 30 minutes in
Answer book Page No. 3. Each question carries one mark.
5) Answer MCQ/Objective type questions on Page No. 3 only. Dont
forget to mention, Q.P. Set (A/B/C/D) on Top of Page.
MCQ/Objective Type Questions

Duration : 30 Minutes

Marks : 20

1. Multiple choice type questions :

(120=20)

1) A nuclear power plant is invariably used as a


a) peak load plant
b) base load plant c) stand-by plant

d) none of the above

2) Which plant can never have 100 percent load factor ?


a) Peak load plant
b) Base load plant
c) Nuclear power plant
d) Hydro electric plant
3) Power plant having maximum demand more than the installed rated capacity will have
utilisation factor
a) equal to unity
b) less than unity c) more than unity d) none of the above
4) Annual installment towards depreciation reduces as rate of interest increases with
a) sinking fund depreciation
b) straight line depreciation
c) reducing balances depreciation
d) none of the above
5) A low utilization factor for a plant indicates that
a) plant is used for stand by purpose only
b) plant is under maintenance
c) plant is used for base load only
d) plant is used for peak load as well as base load
6) During load shedding
a) system power factor is changed
c) system voltage is reduced

b) some loads are switched off


d) system frequency is reduced

7) With reference to a power station which of the following is not a fixed cost ?
a) Fuel cost
b) Interest on capital
c) Insurance charges
d) Depreciation
P.T.O.

SLR-BB 72

*SLRBB72*

-2-

8) Which of the following is not a method for estimating depreciation charges ?


a) Sinking fund method
b) Straight line method
c) Diminishing value method
d) Halseys 50-50 formula
9) Which of the following is an impulse turbine ?
a) Pelton turbine
b) Francis turbine c) Kaplan turbine
10) Alternate formation and breakage of bubbles are called
a) water hamming
b) cavitations
c) surgification

d) None of above
d) all of the above

11) In hydroelectric power plants


a) both operating and initial cost are high
b) both operating and initial cost are low
c) operating cost is low and initial cost is high
d) operating cost is high and initial cost is low
12) The function of a surge tank is to
a) produce surge in the pipeline
c) supply water at constant pressure

b) relieve water hammer pressure in the penstock


d) none of above

13) Gross head of a hydroelectric power station is


a) the height of water level in the river where the storage is provided
b) the difference of water level between the level in storage and tail race
c) the height of water level in the river where the tail race is provided
d) all of above
14) Storage requirement can be determined from
a) flow duration curve b) hydro-graph
c) mass curve

d) both b) and c)

15) Where is the draft tube of a hydroelectric plant that is an airtight pipe located ?
a) At the beginning of the penstock
b) Near the surge tank
c) In between the runner exhaust and the tail race
d) In between the penstock and runner
16) For harnessing low variable water heads, the suitable hydraulic turbine with reaction and
adjustable vanes is
a) Pelton
b) Kaplan
c) Impeller
d) Francis
17) In Francis turbine runner, the number blades is usually of the order of
a) 3-6
b) 6-8
c) 12-14
d) 16-24
18) The specific speeds of Kaplan, Francis and Pelton turbines are in
a) the decreasing order
b) the increasing order
c) neither increasing nor decreasing order d) none of the above
19) On what factors does the selection of a water turbine depend ?
a) Working head
b) Speed
c) Nature of load

d) All of above

20) In high head hydroelectric power plant, the velocity of water flow in penstock is around
a) 2 m/s
b) 4 m/s
c) 8 m/s
d) 10 m/s
______________
Set A

*SLRBB72*

SLR-BB 72

-3-

Seat
No.
B.E. (Civil) (Part II) Examination, 2014
Elective III : WATER POWER ENGINEERING
Day and Date : Monday, 1-12-2014
Time : 3.00 p.m. to 6.00 p.m.

Marks : 80

Instructions : 1) Solve any three questions from Section I (Questions 2, 3, 4, 5)


and any three questions from Section II. (Questions 6, 7, 8, 9)
2) Draw neat labeled sketches wherever necessary.
3) Assume suitable data if necessary and state it clearly.
SECTION I
2. a) What is the importance of thermal power development in the country ? Describe its
development during the plan period.

b) What do you understand by base load and peak load power plants ? What type of power
plants are used as base load and peak load plants and why ? What factors are considered
in selecting a plant as base load plant or peak load plant ?

3. a) Explain the construction of flow duration curve and discuss its importance in comparing
the power potentiality of different storages used for power generation.

b) The following data refers to a proposed hydroelectric power plant. Available head = 27 m,
Catchment area = 430 square km, Rainfall = 150 cm/year, Percentage of total
rainfall utilized = 65%, Penstock efficiency = 95%, Turbine efficiency = 80%, Generator
efficiency = 86%, Load factor = 0.45. Calculate the power that can be developed. Suggest
suitable turbine for the plant.

4. a) The runoff data of a river at a particular site is tabulated as following :


Month

Mean Discharge MCM Month

Mean Discharge MCM

January

30

July

80

February

25

August

100

March

20

September

110

April

October

65

May

10

November

45

June

50

December

30

i) Draw the hydrograph and find the mean flow


ii) Draw flow duration curve
iii) Find the power developed if the available head is 90 m and the overall efficiency of
generation is 86%.
b) What is meant by economical diameter of the penstock ? How can it be found out ?

8
6

Set A

SLR-BB 72

-4-

*SLRBB72*

5. a) What do you understand by water hammer in a pipe line ? Derive the expression for the
water hammer pressure in case of rigid pipe and in case of elastic pipe.
b) What are the functions of surge tank ? Describe with neat sketches, the behaviour of
various types of surge tanks.

6
7

SECTION II
6. a) What topographical features are in favors of underground power house ? What are the
different types underground power stations ? Draw neat layout of each.

b) What factors are mainly considered in selecting a prime-mover for :


i) run-off river plant
ii) storage plant
iii) pumped storage plant.

7. a) What are the advantages of reaction turbine over the Pelton wheel in respect of efficiency,
size, cost and maintenance ?
b) A Pelton wheel has to be designed for the following specification. Power to be
developed = 6000 kW. Net head available = 300 m. Speed = 550 rpm. Ratio of jet diameter
to wheel diameter = 1/10. Hydraulic efficiency = 0.85. Assuming the velocity coefficient
Cv = 0.98 and speed ratio f = 0.46, find :
i) the number of jets
ii) diameter of jets
iii) diameter of wheel and
iv) the quantity of water required.

8. a) Describe working of a pumped storage hydropower plant. What are various arrangements
possible for higher and lower reservoir in case of pumped storage plant ?

b) A closed cycle pumped storage hydropower plant with gross head 350 m has a head race
tunnel 4 m diameter and 700 m long. The power hose discharges directly in the lower reservoir.
The flow velocity is 6.5 seconds and the friction factor f = 0.018. If the overall efficiency of
pumping and generation are 85% and 88% respectively, estimate the plant efficiency.

9. a) Explain with the sketches, the Double Basin System of tidal power generation. What are
the limitations of this method ?

b) Explain why the discharge conditions at part load are less favourable for the high specific
speed runner than for the low specific speed runner.

__________________

Set A

SLR-BB 73

*SLRBB73*
S

B.E. (Civil) (Part II) Examination, 2014


ENTREPRENEURSHIP (Elective III)
Day and Date : Monday, 1-12-2014
Time : 3.00 p.m. to 6.00 p.m.
Instructions :

Max. Marks : 100

1) Q. No. 1 is compulsory. It should be solved in first 30 minutes in


Answer book Page No. 3. Each question carries one mark.
2) Answer MCQ/Objective type questions on Page No. 3 only. Dont
forget to mention, Q.P. Set (A/B/C/D) on Top of Page.

Duration : 30 Minutes

MCQ/Objective Type Questions

Marks : 20

PART I
1. Choose the correct answer :
1) Entrepreneur is a person who
a) starts new projects
c) both a) and b)

(1 mark each)
b) make efforts to complete them
d) none of these

2) The main function of RBI in India is


a) Granting license to commercial for opening branches
b) Accepting deposits from the bank
c) Regulating foreign trade
d) Acting as a note issuing authority, bankers bank and bankers to the government
3) A project report is prepared before the execution of the project is called
a) Project report
b) Project preliminary report
c) Project assessment
d) Project feasibility
4) The ________________ format is prepared before hand for starting any small scale
industry.
a) project report
b) project preliminary report
c) project assessment
d) project feasibility
5) _______________ capital is invested in the plant, equipment, land and buildings.
a) Working
b) Fixed
c) Both a) and b)
d) Cash
6) The study or relationship between the raising of finance and development of finance is
called as ___________
a) Organized group efforts
b) Business
c) Financial management
d) Feasibility
7) NABARD, an apex developing bank was set up on the recommendation of
a) Craficard committee
b) Jilami committee
c) Nayak committee
d) None of these

P.T.O.

SLR-BB 73

*SLRBB73*

-2-

8) Following is one of the factors which is not considered in case of project feasibility
a) Financial
b) Technical
c) Environmental
d) Economical
9) The registration of SSI can be
a) Permanent
b) Provisional

c) Both a) and b)

d) None of these

10) RBI is the lender of the last resort because


a) RBI can grant loan up to any extent
b) Any loan proposal rejected by a bank can be considered by RBI
c) RBI can direct any bank to grant loan to a needy borrower
d) None of the above
11) ________________ is the main motive of business enterprise.
a) Employment
b) Stability
c) Financial assistance
d) Profit
12) The ______________ of message is a starting point of communication system.
a) Sender
b) Manager
c) Receiver
d) Board of directors
13) SIDBI commend its operations with effective from
a) April 02, 1990
b) April 02, 1991
c) April 04, 1992

d) None of these

14) The industrial financial corporation of India grants


a) Finance to large industries
b) Refinance to commercial banks under new bill market scheme
c) Revival package to sick SSI units
d) All of these
15) The part of wealth which is used for further production of wealth is called as
a) Cost
b) Capital
c) Price
d) Finance
16) The tax which is levied annually by individual or company is called
a) Sales tax
b) Excise duty
c) Income tax
d) None of these
17) The stored items which form an integral part of finished goods is called
a) Stock
b) Direct inventory
c) Indirect inventory
d) All of these
18) The definition of small scale industry in India is based on
a) Sales of the unit
b) Investment in machines and equipments
c) Market coverage
d) Export capacity
19) Which of the following is not a phase in business plan preparation ?
a) Setting business goals
b) Locating resources
c) Tracing competitions
d) Developing the labour market
20) SEZ stands for
a) Small Entrepreneurship Zone
c) Sick Economic Zone

b) Special Entrepreneurship Zone


d) Special Economic Zone

______________

Set A

*SLRBB73*
S

-3-

SLR-BB 73

B.E. (Civil) (Part II) Examination, 2014


ENTREPRENEURSHIP (Elective III)
Day and Date : Monday, 1-12-2014
Time : 3.00 p.m. to 6.00 p.m.

Marks : 80

PART II
Instruction : Solve any three questions from each Section.
SECTION I
2. a) Define the term entrepreneur. Explain the various qualities of a good entrepreneur.

b) Write in detail the procedure for registration of small scale industry.

3. a) Define SSI. Discuss about the role and scope of SSI. Write in detail the characteristics of
small scale industries.
b) Explain the factors to be considered or step to start a SSI.

8
5

4. Write in detail Whom to approach for what for setting SSI.

13

5. a) Explain the concept of market survey. Write in detail the various information to be collected
for market survey of a particular product.
b) Write a note on Small INDUSTRIES Development Bank of India (SIDBI).

8
5

SECTION II
6. Following is the balance sheet of a limited company as on 31st March 2013.
Liabilities
Share capital
Reserve and surplus

Rs.

Assets

Rs.

2,00,000 Land and Building

2,40,000

70,000 Plant and Machinery

12% Debentures

4,20,000 Stock on trade

Creditors

1,00,000 Bills and receivables

Bills payable
Total

50,000 Bank Balance


8,40,000 Total

3,50,000
2,00,000
10,000
40,000
8,40,000

Set A

SLR-BB 73

-4-

*SLRBB73*

The sales of the company were Rs. 4 lacs. The gross profit was Rs. 25,000 and net profit was
Rs. 20,000. Calculate
i) Current ratio
ii) Quick ratio
iii) Gross profit ratio
iv) Net profit ratio
v) Current assets to fixed assets
vi) Proprietary ratio and
vii) Inventory to working capital.

14

7. Write a detail project report on Ceramic Glazed Wall Tiles as a small scale manufacturing
enterprise.

13

8. a) What do you mean by Project Feasibility ? Explain the various factors of project feasibility. 7
b) Define the term Capital. Explain different types of Capital.

9. a) Define communication. Explain various elements of communication and also indicate the
various barriers to communication.
b) What do you mean by Taxation ? Explain the various types of it.

7
6

_____________________

Set A

SLR-BB 74

*SLRBB74*
S

B.E. (Civil) (Part II) Examination, 2014


OPTIMIZATION TECHNIQUES (Elective III)
Day and Date : Monday, 1-12-2014
Time : 3.00 p.m. to 6.00 p.m.
Instructions :

Duration : 30 Minutes

Max. Marks : 100

1) Q. No. 1 is compulsory. It should be solved in first 30 minutes in


Answer Book Page No. 3. Each question carries one mark.
2) Answer MCQ/Objective type questions on Page No. 3 only. Dont
forget to mention, Q.P. Set (A/B/C/D) on Top of Page.
MCQ/Objective Type Questions

1. Choose the correct answer option :

Marks : 20
(201=20)

1) The inventory carrying costs are also called as


a) procurement costs
b) set-up costs
c) storage costs
d) none of above
2) If either the objective functions and/or one or more of the constraints are non-linear in
X(x1, x....xn) then problem is called as
a) Statistical problem
b) Simulation problem
c) Replacement problem
d) Non-linear programming problem
3) _______________ of a function is a value which is higher than all the values of the function.
a) Global minimum
b) Global maximum
c) Both a) and b)
d) None
4) If more than one states of nature exist and there is enough information available to assign
probability to each of the possible states then that decision is known as
a) Decision making under condition of certainty
b) Decision making under condition of risk
c) Decision making under condition of uncertainty
d) All of these
5) The shortest acyclic route network problems can be solved by
a) Arithmetic method
b) Graphical method
c) Statistical method
d) Simulation method
6) The Monte-Carlo simulation used
a) Sampling technique
c) Mathematical technique

b) Decision technique
d) Both a) and b)

7) Games without a saddle point require player to play


a) mixed strategies
b) pure strategies
c) dominated strategies
d) none of these
8) Which of the following criterion for decision making uses optimistic and pessimistic
decisions ?
a) Minim ax
b) Maxim in
c) Laplace
d) Hurwitz
P.T.O.

SLR-BB 74

*SLRBB74*

-2-

9) O.R. was used in industry firstly by


a) British industries
c) Indian industries

b) U.S. industries
d) Japanese industries

10) The Vogels approximation method is also called as


a) Row minima method
b) Least cost method
c) Penalty method
d) None
11) In simplex method, the elimination of all infeasible basic solutions are achieved by starting
with a basic solution which is ______________
a) Negative
b) Non-negative
c) Less than-1
d) All of above
12) In artificial variables techniques, the artificial variables are added to the constraints of
and = type
b)
and = type
a)
d) all of these
c) , and = type


13) The graphical variables Techniques, the artificial variables are added to the
constraints of
a) objective function equation
b) constraint equation
c) linear equation
d) All of these
14) One disadvantages of using N-W-C rule to find initial solution to the transportation problem
is that
a) It is complicated to use
b) It does not take into account the cost of transportation
c) It leads to a degenerate initial solution
d) All of the above
15) A type of decision making environment is _____________
a) Certainty
b) Uncertainty
c) Risk

d) All of the above

16) Which of the following characteristics apply to queuing system ?


a) Customer population
b) Arrival process
c) Both a) and b)
d) Neither a) nor b)
17) A L.P.P. in which some or all of the variables must take non-negative integer values is
commonly referred to as _____________
a) Mathematical model
b) Simulation model
c) Integer model
d) None
18) The unconstrained optimization is a type of ____________
a) Non-linear program problem
b) Linear program problem
c) Simulation model
d) None of above
19) _____________ is a representation of reality through the use of a model or other device
which will react in the same manner as reality under a given set of conditions.
a) Queuing model
b) Inventory model
c) Simulation model
d) None of above
20) Linear programming deals with the optimization of a function of variable is known as
____________
a) Subjective function
b) Objective function
c) Constraints
d) All of these
______________
Set A

*SLRBB74*
S

-3-

SLR-BB 74

B.E. (Civil) (Part II) Examination, 2014


OPTIMIZATION TECHNIQUES (Elective III)
Day and Date : Monday, 1-12-2014
Time : 3.00 p.m. to 6.00 p.m.

Marks : 80
SECTION I

Instruction : Q. 3 is compulsory, solve any two of Q. 2, Q. 4 and Q. 5.


2. a) Define optimization. Write the necessary and sufficient conditions for the
relative maximum of a function of a single variable.

b) Find the optimum value of the function f(x) = (x 2)4 and also state if the
function attains a maximum or a minimum.

3. a) In a car servicing station, it takes exactly 20 min. to service a car. If average


arrival rate of car is 30 min. Calculate the average waiting time of a car.

b) Use BigM method to solve following LPP :


Minimize
y + 2x
20
Subject to
x+y
2x + 5y
70
7x + 3y
84

x


0, y


0.

4. a) In figure labels on the arcs indicate the flow capacities in both the directions. For example,
on the arc joining A to B, the label near A indicates flow in direction A to B, while label near B
indicates flow from B to A. Determine maximum total flow from A to G.

Set A

SLR-BB 74

*SLRBB74*

-4-

b) Find the shortest path from node 1 to all other nodes. Find the other distances,
in order of increasing distance from node 1.

5. a) Determine the initial basic feasible solution for the following transportation model.

>

Use Vogels Approximation Method.

b) A decision maker has three alternative plans with two possible states of nature and
respective monitory profits (in rupees) mentioned in following matrix.

Set A

*SLRBB74*

SLR-BB 74

-5-

Determine the decisions by Maximax, Maximin and Laplace equally likely approach. What
will be the Expected Value of Perfect Information ?
OR
b) Prepare a decision tree for the above mentioned decision making problem. Advise a correct
decision. (Assume the possible states of nature with equal probability of occurrence.)

SECTION II
Instructions : Q. 6 is compulsory. Solve any two of Q. 7 to Q. 9. Make suitable
assumption.
6. a) Explain the following terms :

i) Hurwitz criteria
ii) Saddle point
iii) Pure and mixed strategy.
b) What are the rules for elimination of columns or rows of the matrix of game
using the theory of dominance ?

7. a) What is simulation ? Name three simulation techniques. Explain Monte Carlo


technique.

b) Explain suitability of integer programming in decision making problems.

8. a) Explain characteristics of dynamic programming.

b) Explain deterministic model for inventory management.

9. a) A plant producing a line of hydraulic valves can supply the factory warehouse at the rate
of 750/month. The warehouse ship 3000 valves per year at a unit selling price of Rs. 250.
Considering the plants ordering and set up cost of Rs. 300 and inventory carrying cost
rate of 20%. What quantities should the warehouse order from the plant ?

b) Write the expressions for expected average queue length, expected average total time
and expected average waiting time in terms of


= service rate,


= arrival rate and

. Assume M/D/1 case (random arrival, deterministic service, and one service

channel).

5
_____________________

Set A

Set A

SLR-BB 75

*SLRBB75*
Seat
No.

Set

B.E. (Civil) (Part II) Examination, 2014


Elective III : DISASTER MANAGEMENT
Day and Date : Monday, 1-12-2014
Time : 3.00 p.m. to 6.00 p.m.
Instructions :

Max. Marks : 100

i) Figures to the right indicate full marks.


ii) Draw the sketches wherever necessary.
iii) Q. No. 1 is compulsory. It should be solved in first 30 minutes in
Answer book Page No. 3.
iv) Answer MCQ/Objective type questions on Page No. 3 only.
Dont forget to mention, Q.P. Set (A/B/C/D) on Top of Page.
MCQ/Objective Type Questions

Duration : 30 Minutes

Marks : 20

1. Choose the correct option :


I) _________________ is the measure of expected losses due to hazardous event
of a particular magnitude occurring in a specific area over a specific time period.
A) Hazard
B) Vulnerability C) Risk
D) All of the above
II) Which of the following is NOT a natural disaster ?
A) Flood
B) Drought
C) Thunderstorm

1
1

D) Deforestation

III) _________________ is the nodal department for wind detection, tracking and
forecasting of cyclones in India.
A) Indian Hydrological Department
B) Archaeological Survey of India
C) Indian Meteorological Department D) Indian Ministry of Earth Science

IV) The avalanche is commonly known as _________________


A) Snow fall
B) Snow slide
C) Hailstorm
D) All of the above

V) FEMA stands for _________________


A) Federal Emergency Management Authority
B) Federal Emergency Management Agency
C) Federation of Emergency Management Agency
D) Federation of Emergency Management Activities

VI) Disaster risk is the product of _________________


A) Disaster and Hazard
B) Hazard and Emergency
C) Hazard and Vulnerability
D) Emergency and Vulnerability

VII) DRR stands for_________________


A) Disaster Risk Rate
C) Disaster Response Rate

1
B) Disaster Risk Reduction
D) Disaster Response Reduction

P.T.O.

SLR-BB 75

-2-

*SLRBB75*

VIII) _________________ is the sum total of actions taken by people and institutions
in the emergency phase.
A) Disaster Recovery
B) Disaster Risk
C) Disaster Response
D) Disaster Mitigation

IX) Gustnado is one of the type of _________________as a disaster.


A) Hurricanes
B) Cyclones
C) Tornadoes
D) Tsunamis

X) The District Disaster Management Authority is headed by the _____________


A) District Superintendent of Police
B) District Magistrate
C) Elected representative of local body D) None of the above

XI) The disaster which is caused by the exposure of living organism to germs and
toxic substances, is known as _________________
A) Chemical Disaster
B) Biological Disaster
C) Geological Disaster
D) Nuclear Explosion

XII) The effective disaster response policies must be_________________


A) Rigid
B) Flexible
C) Both A and B D) None of these

XIII) Primary appraisal is the term used to describe the process of ____________
A) Identifying of the stressor
B) Evaluation of the stressor
C) Both A and B
D) None of the above

XIV) _________________is the sensor that uses a laser beam to find the rising or
lowering of magma levels beneath ground.
A) Sphygmomanometer
B) Tiltmeter
C) Tensiometer
D) Bottom pressure recorders
XV) The explosive materials that deflagrate are known as _____________
A) Low explosives
B) High explosives
C) Mild explosives
D) Severe explosives
XVI) According to IS-1893 Part I : 2002, India has grouped into ______________
seismic zones.
A) Three
B) Four
C) Five
D) Six
XVII) Which of the following is a natural metrological disaster ?
A) Earthquakes
B) Tsunamis
C) Tornadoes

1
1

D) Landslides

XVIII) Match the followings with correct pairs


3
i) Minimizing the effects of a disaster
A) Mitigation
B) Preparedness
ii) Returning community to normal life
iii) Planning how to respond life disaster
C) Recovery
iv) Efforts to minimize the hazard created by a disaster

______________

Set A

*SLRBB75*

-3-

SLR-BB 75

Seat
No.

B.E. (Civil) (Part II) Examination, 2014


Elective III : DISASTER MANAGEMENT
Day and Date : Monday, 1-12-2014
Time : 3.00 p.m. to 6.00 p.m.
Instructions :

Marks : 80

i) Figures to the right indicate full marks.


ii) Draw the sketches wherever necessary.
iii) Q. No. 2 from Section I and Q. No. 6 from Section II are
compulsory.
iv) Solve any two questions from 3, 4 and 5 in Section I.
v) Solve any two questions from 7, 8 and 9 in Section II.
SECTION I

2. A) What is drought ? Explain its types.


B) What do you mean by disaster response ? State and explain the sequential
action steps in disaster response.
3. A) Write a note on environmental hazards.
B) Discuss the causes of following disasters.

6
8
5
8

i) Volcanic eruptions
ii) Sedimentation processes.
4. A) Write a note on Population Explosion.
B) What is Disaster Management Act, 2005 ? Also state its salient features.
5. A) Define Disaster Management Cycle. Also explain its various phases in short.
B) What do you mean by soil erosion ? Discuss its causes and preventive
measures.

5
8
5
8

Set A

SLR-BB 75

-4-

*SLRBB75*

SECTION II
6. A) Discuss the immediate relief measures to be taken for disaster victims.

B) What is the role of Prediction and Warning System in Disaster Management ?

7. A) What are the measures of adjustment to natural hazards ?

B) Write the functions and role of following organizations in Disaster Management.

i) National Disaster Management Authority.


ii) The National Executive Committee.
8. A) Explain the role of geographical information system in disaster Management.
B) Medias plays a key role in Disaster Management. Comment on this statement
also give your justification.
9. A) What do you understand by coastal disasters ? Explain in short.
B) Write a note on :

5
8
5
8

i) Role of Panchayats in Disaster Management


ii) Remedies for Disasters.

_____________________

Set A

SLR-BB 76

*SLRBB76*
Seat
No.

Set

S.E. (Mechanical Engineering) (Part I) Examination, 2014


ANALYSIS OF MECHANICAL ELEMENTS
Day and Date : Tuesday, 9-12-2014
Time : 10.00 a.m. to 1.00 p.m.
Instructions :

Max. Marks : 100

1) Q. No. 1 is compulsory. It should be solved in first 30 minutes in


Answer Book Page No. 3. Each question carries one mark.
2) Answer MCQ/Objective type questions on Page No. 3 only.
Dont forget to mention, Q.P. Set (A/B/C/D) on Top of Page.
MCQ/Objective Type Questions

Duration : 30 Minutes

Marks : 20

1. Choose the correct answer :

20

1) For a simply supported beam of span (L) carries a uniformly distributed load over the whole span,
the S.F.D. will be
a) A rectangle
b) A triangle
c) Two equal and opposite rectangles
d) Two equal and opposite triangles
2) The bending moment on a section is maximum where shearing force is
a) Minimum
b) Maximum
c) Zero
d) Changing sign
3) At a point of contra-flexure
a) Bending moment is zero
b) Shear stress is zero
c) Shear stress is maximum
d) Bending moment is maximum
4) The torsion equation is given by
T
L
T C
T C
= =
= =
= =
a)
b)
c)
d) None of these
J R C
R J
L
J R
L
5) The assumption made, while determining the shear stress in a circular shaft subjected to torsion,
is that
a) The material of the shaft is uniform
b) The twist along the shaft is uniform
c) C/S of the shaft is plane and circular before and after the twist
d) All the above
6) The thermal stress is given by
ET
1
E
b)
c)
d)
a) E T

ET
T
7) The elongation product in a rod (by its own weight) of length (L) and diameter (d) rigidly fixed at the
upper end and hanging is equal to
wL
w L2
w L3
w L4
a)
b)
c)
d)
2E
2E
2E
2E
8) The work-done in producing strain on a material per unit volume is called
a) Resilience
b) Elasticity
c) Both a) and b)
d) None of these
9) The normal stress on an oblique plane at an angle to the cross section of a body which is
subjected to a direct stress ( ) is equal to

sin 2
a)
b) cos
c) cos2
d) sin2
2
P.T.O.

SLR-BB 76

*SLRBB76*

-2-

10) If the member is subjected to an axial tensile load, the plane normal to the axis of loading carries
a) Minimum normal stress
b) Maximum normal stress
c) Maximum shear stress
d) None of these
11) The ratio of crippling load, for a column of length (l) with both ends fixed to the crippling load of the
same column with one end fixed and other end hinged, is equal to
a) 2.0
b) 4.0
c) 8.0
d) 16.0
12) Buckling factor is defined as the ratio of
a) Equivalent length of the column to the minimum radius of gyration
b) Length of the column to the minimum radius of gyration
c) Length of the column to the area of cross-section of the column
d) Maximum radius of gyration to the area of cross-section of the column
13) The equivalent length is equal to half of the actual length of the column with
a) One end fixed and other end free
b) Both ends fixed
c) One end fixed and other end hinged
d) Both ends hinged
14) A cantilever of length (l) carries a uniformly distributed load of w per unit length for a distance x
from the free end, then the deflection at the free end will be
a)

Wx4 Wx 3 L
+
8 EI
6 EI

b)

W x4
8 EI

c)

W L4
8 EI

d)

Wx 4 Wx 3
+
(L x)
8 EI
6 EI

15) The proof resilience is given by


2E
2
2 2
b)
c)
2E
2
E
16) For an I section
a) Bending stress is maximum at the neutral axis
b) Shear stress is zero at the neutral axis
c) Bending stress is zero at the edges
d) Shear stress is maximum at the neutral axis

a)

d)

E
2 2

17) Strain energy will stored in a body be maximum if


a) Load is applied gradually
b) Load is applied suddenly without an impact
c) Load is applied with an impact
d) Load applied is cyclic
18) Rankines formula is an empirical formula which is used for
a) Long columns
b) Short columns
c) Both long and short columns
d) None of the above
19) Choose the correct statement
a) Section modulus of a hollow circular section of external diameter D and internal diameter d is
equal to

D4 d4
64 D

b) Section modulus of a circular section of diameter D is


c) Section modulus of rectangular section is

D4
32

b d2
12

b4
12
20) A simply supported beam is of rectangular section. It carries a udl over the whole span. The
deflection at centre is y. If the depth of the beam is doubled, the deflection at the centre would be
a) 2y
b) 4y
c) y/2
d) y/8
Set A
______________

d) Section modulus of a square section is

*SLRBB76*

-3-

SLR-BB 76

Seat
No.

S.E. (Mechanical Engineering) (Part I) Examination, 2014


ANALYSIS OF MECHANICAL ELEMENTS
Day and Date : Tuesday, 9-12-2014
Time : 10.00 a.m. to 1.00 p.m.
Instructions :

1)
2)
3)
4)

Marks : 80

Solve any two questions from each Section.


Use of calculator is allowed.
Figures to the right indicate full marks.
Assume additional suitable data, if necessary and state it clearly.
SECTION I

2. a) Define Youngs modulus and modulus of rigidity. Derive relation between Youngs modulus E, Modulus
of rigidity C in terms of Poissons.
b) A metallic bar 25105 cm is loaded as in the fig. 2. Find the change in volume, take
E = 2000 tonn/cm2 and Poissons is 0.25. Also find the change that should be made in the 400 tonn.
load in order that there should be no change in volume of bar.

Fig. 2 (Figure is not to the scale)


c) Derive the torsion equation for a hollow shaft.
3. a) Determine torque that may be applied to solid shaft of 90 mm diameter without exceeding an allowable
shear stress of 75 MPa. If the above shaft is replaced by a hollow shaft by same mass and of
100 mm outer dia. What would be its inner diameter and torque, if allowable shear stress is same.
b) Draw S.F.D. and B.M.D. for the beam as shown in Fig. (3). Indicate all significant points. Also
calculate maximum bending moment :

8
12

Fig. 3 (Figure is not to the scale)


Set A

SLR-BB 76

-4-

*SLRBB76*

4. a) Draw S.F.D. and B.M.D. for a simply supported beam of length L carrying a symmetrical U.V.L. with
intensity varying from zero at each support upto maximum w/m at the centre.

b) At a point in a strained material, the principal stresses are 100 N/mm2 and 40 N/mm2 both compressive.
Determine the result stress in magnitude and direction on a plane inclined to 60 to the axis of the
major principal stress. Also determine the maximum intensity of shear stress in the
material at the point.

c) The tensile stresses at a point across two mutually perpendicular planes are 120 N/mm2 and
60 N/mm2. Determine the normal, tangential and resultant stresses on a plane inclined at 30 to the
axis of the major stress by Mohrs circle method.

SECTION II
5. a) A hollow C.I. column whose outside diameter is 200 mm has a thickness of 20 mm. It is 4.5 m long
and is fixed at both ends. Calculate the safe load by Rankines formula using a factor of safety of 4.
Calculate the slenderness ratio and the ratio of Eulers and Rankines critical loads.
Take c = 550 N/mm2 and a = 1/1600 in Rankines formula and E = 9.4 104 N/mm2.

b) A cast iron beam is of I section as shown in fig. 5-b. The beam is simply supported on a span of
5 m. If the tensile stress is not to exceed 20 N/mm2, find the safe uniformly load which the beam can
carry. Find also the maximum compressive stress.

Fig. 5 -b
c) A uniform metal bar has a cross-sectional area 700 mm2 and a length of 1.5 m. With an elastic limit
of 160 MPa, what will be its modulus of resilience. Determine also the maximum values of an applied
load which may be suddenly applied without exceeding elastic limit. Calculate the gradually applied
load which will produce the same extension as produced by the suddenly applied load above.
E = 200 GPa.

Set A

*SLRBB76*

-5-

SLR-BB 76

6. a) A beam ABC of length 9 m has one support of the left end and the other support at a distance of
6 m from the left end. The beam carries a point load of 1 kN at the right end and also carries a
uniformly distributed load of 4 kN/m over a length of 3 m as shown in fig. 5-a. Determine the slope
and deflection at point C. Take E = 2 105 N/mm2 and I = 5 108 mm4.

10

Fig. 6-a
b) In an I Section of a beam having flanges 18 cm 6 cm giving overall depth of 36 cm, draw the
shear stress distribution by calculating shear stresses at 0, 3, 6, 9, 12, 14, 16, 18 cm from the
neutral axis. What is the maximum shear stress ? Draw the shear stress diagram across the flange.
The maximum shear force in the beam is 50 kN.

10

7. a) A cantilever of length 3 m carries two point loads of 2 kN at the free end and 4 kN at a distance of
1 m from the free end. Find the deflection at the free end. Take E = 2 105 N/mm2 and
I = 5 108 mm4.
b) Derive the expression for crippling load when both ends of column are fixed.

7
7

c) A tension bar 5 m long is made up of two parts, 3 m of its length has a cross-sectional area of
10 cm2 while the remaining 2 m has a cross-sectional area of 20 cm2. An axial load of 80 kN is
gradually applied. Find the total strain energy produced in the bar and compare this value with that
obtained in a uniform bar of the same length and having the same volume when under the same load.
Take E = 2 105 N/mm2.

_____________________

Set A

Set A

SLR-BB 77

*SLRBB77*
Seat
No.

Set

S.E. (Mechanical) (Part I) Examination, 2014


APPLIED THERMODYNAMICS
Day and Date : Thursday, 11-12-2014
Time : 10.00 a.m. to 1.00 p.m.

Max. Marks : 100

Instructions : 1) Use of steam tables and mollier diagram is allowed.


2) Use of scientific calculator is allowed.
3) Q. No. 1 is compulsory. It should be solved in first 30 minutes
in Answer Book Page No. 3. Each question carries one mark.
4) Answer MCQ/Objective type questions on Page No. 3 only.
Dont forget to mention, Q.P. Set (A/B/C/D) on Top of Page.
MCQ/Objective Type Questions
Duration : 30 Minutes

Marks : 20

1. Choose the correct answer :


1) Internal energy of ideal gas is function of
a) Temperature
b) Pressure
c) Volume

(201=20)
1
d) Specific volume

2) The unit of specific volume is


a) m3
b) m3/kg
c) m2
d) m
3) Area under the curve on T S chart indicates.
a) Net heat transfer
b) Net work transfer
c) Cycle efficiency
d) None of the above

1
1

4) Carnot cycle is a reversible cycle because


a) All the processes are reversible
b) All the processes are irreversible
c) All the processes are adiabatic
d) All the processes are isothermal

5) The process of heating in the boiler is considered as


a) Steady flow process
b) Unsteady flow process
c) Constant temperature process
d) None of the above
6) Steam trap is
a) Mounting of the boiler
b) Accessory of the boiler
c) Fitting of the steam carrying pipe d) None of the above

7) Saturation temperature of the water___________ as per pressure.


a) Decreases
b) Increases
c) Remains constant
d) None of the above

P.T.O.

SLR-BB 77
8) Economizer of the boiler is used
a) Decrease the efficiency
c) Increase the efficiency

*SLRBB77*

-2-

1
b) To keep the pressure constant
d) Keep temperature constant

9) Parsons reaction turbine U having degree of reaction equal to


a) 0.5
b) 1
c) 0.75
d) 0.25

10) Steam turbine is an _____________combustion thermal prime mover.


a) Internal
b) Rotary
c) Reciprocating d) External
11) Enthalpy drops in impulse turbine in
a) Moving blades
b) Both in fixed and moving blades
c) Only in fixed blades
d) None of the above

12) Cooling of water takes plane in cooling tower mainly due to


a) Mixing of air and water
b) Evaporative cooling
c) Addition of heat into water
d) None of the above

13) In a supersaturated state the dryness fraction of steam is


a) Less than one b) More than one c) Equal to one d) Zero
14) As polytropic index of compression increases the compression curve on
P V chart goes on becoming
a) Flatter and flatter
b) Steeper and Steeper
c) Does not change
d) None of the above

15) A 7m3/min compressor means that it


a) Compresses 7m3/min of standard air
b) Compresses 7m3/min of free air
c) Delivers 7m3/min of compressed air
d) Delivers 4m3/min of compressed air at delivery

16) Due to intercooling the required workdone for compression.


a) Remains constant
b) Increases
c) Decreases
d) None of the above
17) Carnots cycle refrigerator works on
a) Rankine cycle
b) Reversed joule cycle
c) Brayton cycle
d) Reversed Carnot cycle

18) For defining process at least __________ are required.


a) Two states
b) Three states c) Four states
d) One state

19) In a boiler which type of heaping is carried out.


a) Only sensible
b) Only latent
c) Only superheating
d) All of the above
20) In a radially inward flow type of turbine the blade speed is ___________
a) Constant at inlet and outlet
b) Different at inlet and outlet
c) Different for each blade
d) None of the above
______________

Set A

*SLRBB77*

-3-

SLR-BB 77

Seat
No.

S.E. (Mechanical) (Part I) Examination, 2014


APPLIED THERMODYNAMICS
Day and Date : Thursday, 11-12-2014
Time : 10.00 a.m. to 1.00 p.m.
Instructions : 1)
2)
3)
4)

Marks : 80

Answer any two questions from each Section.


Use of steam tables and Mollier diagram is allowed.
Assume suitable data if necessary and state it clearly.
Use of scientific calculator is allowed.
SECTION I

2. a) Define following terms :


i) Adiabatic flame temperature
iii) Excess air

7
ii) Enthalpy of combustion
iv) Standard enthalpy of reaction.

b) Calculate standard heat of reaction for the reaction :

2C2H6(g) + 7 O2(g) 4 CO2 + 6 H2O


Assume enthalpies of formation for C2H6 = 74.9 kJ/mol, CO2 = 393 kJ/mol,
H2O = 242 kJ/mol.
c) Explain refrigerator with respect to first law of thermodynamics.

3. a) A reversible engine receives heat from two constant temperature sources at


870 K and 580 K. It rejects 3000 kJ/min to a sink at 290K. The engine develops
85kW. Determine the heat supplied by each source and the efficiency of the
engine.

b) Explain regenerative Rankine cycle and derive the relation for cycle efficiency. 7
c) Define the following :
a) Boiler
c) Boiler efficiency

6
b) Equivalent evaporation
d) Incomplete combustion

4. a) In a boiler trial the following data were recorded

Feed water = 640 Kg/hr, temperature of feed water = 30C, steam pressure = 10 bar,
quality of steam is dry saturated, coal burnt = 75 Kg/hr, calorific value of
coal = 31800 KJ/kg, flue gases = 1580 Kg/hr, temperature of flue gases = 305C,
unburnt coal and ash = 5.5 Kg/hr, calorific value of ash and unburnt
coal = 2900 KJ/kg, boiler room temperature = 28C, specific heat of flue
gases = 1.005 KJ/kgK. Determine boiler efficiency and draw heat balance sheet.
Set A

SLR-BB 77

-4-

*SLRBB77*

b) Steam at 15 bar and 300C is supplied to an engine working on Rankine


cycle. The exhaust takes place at 0.8 bar. Calculate :
a) Condition of steam after isentropic expansion
b) Rankine cycle efficiency
c) Improvement of efficiency if exhaust takes place at 0.5 bar.
c) Compare Rankine cycle and Carnot cycle.
SECTION II
5. a) Derive the relation for critical pressure ration for the nozzle flour.

6
7

b) Steam with absolute velocity 360 m/s enters the stage of an impulse turbine
provided with single row wheel. The nozzles are inclined at 20 to the plane of
the wheel. The blade rotor with diameter 95.5 cm rotates with a speed of
3000 rpm. Find: a) Suitable inlet and outlet angle for the moving blade so that
there is no axial thrust and no friction on surface of the blades. Also work out
power developed.

c) Write the comparison between jet condenser and surface condenser.

6. a) A single stage single acting reciprocating compressor delivers 150 m3/min,


compressing it from 1 bar to 8 bar. The speed of compressor is 300 rpm. If
1
the clearance is 16 th swept volume, find diameter and stroke of the
compressor. Take L/D = 1.5, where L is stroke and D is diameter. The value
of n can be taken as 1.3.

b) Define nozzle efficiency and derive the relation for exit velocity of nozzle in
terms of enthalpy drop.

c) Differentiate between impulse and reaction turbine.

7. a) Steam enters a group of convergent and divergent nozzles at 21 bar and 270C,
the discharge pressure being 0.07 bar. The expansion is in equilibrium throughout
and the loss of friction in the converging portion of the nozzle is negligible but
the loss by friction in the divergent section of the nozzle is equivalent to 10%
of the enthalpy drop available in that section. Calculate the throat and exit
areas to discharge 14 Kg/sec of stream. Also sketch total heat entropy chart
and show on it various stages of the expansion. Assume n = 1.3.
7
b) Derive the relation for power developed in Parsons reaction turbine and draw
velocity triangle.

c) 20m3 of air per minute is compressed from 1 bar and 20C to 10.24 bar.
Determine :

a) The minimum power required to drive the compressor with two stage
compression and compare it for single stage compression.
b) The heat removed in the intercooler for perfect intercooling.
Set A
___________________

SLR-BB 78

*SLRBB78*
Seat
No.

Set

S.E. (Mechanical Engg.) (Part I) Examination, 2014


ENGINEERING MATHEMATICS III
Day and Date : Saturday, 13-12-2014
Time : 10.00 a.m. to 1.00 p.m.
N.B. :

Max. Marks : 100

1) Q. No. 1 is compulsory. It should be solved in first 30 minutes in


Answer Book Page No. 3. Each question carries one mark.
2) Answer MCQ/Objective type questions on Page No. 3 only.
Dont forget to mention, Q.P. Set (A/B/C/D) on Top of Page.
3) Figures to the right indicate full marks.
MCQ/Objective Type Questions

Duration : 30 Minutes

Marks : 20

1. Choose the correct answer :

20

1) The particular integral of the differential equation (D3 D)y = ex + ex, D = d is


dx

x
x
x
1 (e + e )
x (e e x )
a) 2
b) xcoshx
c) xsinhx
d) 2
2
dy
2 d y
+ 2 y = e x becomes
2) If D = d , t = logx then the differential equation x
2 + 4x
dx
dx
dt
t
b) (D 2 + 3D + 2)y = e t
a) (D2 + 3D + 2)y = e e
c) (D 2 3D + 2)y = e 2 t
1 ax
3) f(D) e X =
ax
1
a) e f(D) X

ax
1
b) e f(D a) X

d) (D 2 3D + 2)y = e e

ax
1
c) f(D + a ) e X

ax
1
d) e f(D + a) X

dy
d2 y
2 y = 8x 2 2x + 3 with
4) The Legendres linear equation, (2 x 1)2 2 + (2 x 1)
dx
dx
substitution (2x 1) = et on R.H.S. gives
2t
2t
2t
et
et + 4
et + 2
a) e2t + 3et + 2
b) e + 3
c) e + 3
d) e + 3
2
2
2
5) The complementary function of (D4 a4) y = 0 is
a) y = c1eax + c2eax + c3cosax + c4sinax
b) y = (c1 + c2x)eax + c3cosax + c4sinax
c) y = c1eax + c2e2ax + c3cosax + c4sinax
d) y = c1eax + c2eax + (c3 + c4x)e2ax
6) One of the solution of the linear partial differential equation xzp + yzq = xy is
x
a) xy = c1
b) x2 + y2 = c1
c) y = c1
d) none of above
7) Solution of p2 + q2 = m2, m is constant is

a) z = ax + by + c

b) z = ax + 1 a2 y + c

c) z = ax + y (m2 a2)+ c

d) z = ax + y (m 2 a 2 ) + c
P.T.O.

SLR-BB 78

*SLRBB78*

-2-

8) The complete solution of a linear differential equation contains


a) no. of arbitrary constant equal to order of linear diff. equation
b) no arbitrary constant in C.F.
c) no. of arbitrary constant in C.F. = no. of arbitrary constant in P.I.
d) none of the above
9) If is a differential operator then,
a) = scalar
b) v = scalar

c) v = scalar

d) all are true

10) If a vector V is irrotational, then there exist a scalar potential such that

a) V = div

b) V = curl

c) V = grad

d) none of the above

11) For a Poisson distribution, which of the following is true ?


a) Mean < Variance b) Mean = Variance c) Mean > Variance d) Mean Variance = 1
12) A discrete probability distribution is given by :
X

: 0 1 2
p(X = x) : 19 2 9 2 3 then p(X 1) = ____________
a) 19
b) 2 9
c) 2 3

d) 13

1 , p (x = 2) = 1 , then p(x = 0) is ?
13) The random variable x has Poisson distribution . If p(x= 3) = 6
3
3

a) e 2
b) e 2
c) e 3
d) e
14) z1 cos zdz = ___________ , where C is the ellipse qx2 + 4y2 = 1.

12

a) 0
b) 2
c) 2i
15) Which of the following equation is called Laplace equation ?
a)

2 2
2
2 = 0
y
x

2 2
b) x 2 + y 2 = 0


c) x + y = 0

16) The function f(z) = (x + ay) + i (bx + cy) is analytic if


a) a = 1, b= 2, c = 3 b) a = b, c = 1
c) a = b = c = 1
17) Standard error of statistic t is
a) Mean of t values
c) Standard deviation of t values

d) none of these

d) x y = 0
d) none of these

b) Variance of t values
d) Error in t values

18) If X denote number of units possessing a given attribute in population containing N units,
then the proportion of units possessing given attribute is
X
X
X2
b)
c)
d)
a) N
X
N
N
N

( )

19) Square of standard normal variable is called


a) Chi-square
b) t- distribution
c) Standard normal variate
d) None of these
20) For small sample of size n, from a normal population with mean and variance 2 .
If sample variance is not known then an unbiased estimate of population variance is given by
2
1
2
d) ( x x )
a) n 1 (x x )
c) (x x )2
b) n1 (x x )
______________
Set A

*SLRBB78*

SLR-BB 78

-3-

Seat
No.
S.E. (Mechanical Engg.) (Part I) Examination, 2014
ENGINEERING MATHEMATICS III
Day and Date : Saturday, 13-12-2014
Time : 10.00 a.m. to 1.00 p.m.
N.B. :

Marks : 80

1) Attempt any three question from each Section.


2) Figures to the right indicate full marks.
3) Use of non-programmable calculator and statistical tables are
allowed.
4) Assume necessary data whenever required.
5) Natations : L.O.S. Level of significance.
DF Degree of Freedom.

SECTION I
d3 y
dy
+4
= sin 2x .
3
dx
dx

b) Solve (D2 3D + 2)y = xe3x + sin 2x.

2. a) Solve

c) Solve (2 x 1)2

d2 y
dy
+ (2 x 1)
2 y = 8x 2 2x + 3 .
2
dx
dx

3. a) The differential equation of the motion of a body is

d2 x
+ k 2 x = k 2 a sin nt. If at t = 0, x = 0,
2
dt

dx
ka
= 0 . Show that, x = 2
(k sin nt n sin kt ) .
dt
k n2
b) Solve,

d4z
z = cos t cosh t .
dt 4

OR
b) Solve, (D2 1)y = t sin 3t + cost.

4. a) Solve, 4(1 + z3) = 9z4 pq.


2
b) Prove that F = F +

2
F .
r

c) Find div F and curl F if F = grad , where = x3 + y3 + z3 3xyz.

6
5
4

4
Set A

SLR-BB 78

*SLRBB78*

-4-

5. a) Solve Z2(p2 q2) = (x y).

b) Solve xzp + yzq = xy.

c) A particle moves along the curve x =

t3

+ 1, y =

t2,

z = 2t + 3 where t is the time.

Find the components of its velocity and acceleration at t =1 in the direction i + j + 3k .

SECTION II
6. a) A random variable x has following probability distribution function :
x:

p(x) : 0

2k

2k

3k

k2

2k2

7k2 + k

the find :
i) K
ii) p(x 6) .
b) The average monthly sales of 5000 firms are distributed normally. Its mean and standard
deviation are Rs. 36,000 and Rs. 10,000 respectively. Find
i) The number of firms, the sales of which are over Rs. 40,000.
ii) The percentage of firms, the sales of which will be between Rs. 38,500 and
Rs. 41,000.
[Given : The area under normal curve
between z = 0 and z =0.4 is 0.1554,
between z = 0 and z = 0.25 is 0.0987,
between z = 0 and z = 0.5 is 0.1915].

c) A radio shop sells, on an average 200 radios per day with a standard deviation of 50 radios.
After an extensive advertising campaign, the management will compute the average sells
for the next 25 days to see whether an improvement has occurred. Assume that the daily
sells of radios is normally distributed,
i) Write down the null and alternative hypothesis.
ii) Test the hypothesis at 5% level of significance if X = 216 .
iii) How large must X be in order that the null hypothesis is rejected at 5% level of
significance ?
[Given : At 5% level of significance critical value of Z is |z| = 1.645].

7. a) A manufacturer claimed that atleast 95% of equipments which he supplied to a factory


confirmed to specifications. An examination of a sample of 200 pieces of equipment revealed
that 18 were faulty. Test the claim at level of
i) 0.05,
ii) 0.01 significance.
[Given : Critical value of z for 5% L.O.S. (left tailed) is 1.645 and critical value of Z for 1%
L.O.S. (left tailed ) is 2.33].

Set A

*SLRBB78*

SLR-BB 78

-5-

b) In a sample of 600 students of a certain college 400 are found to use dot pens. In another
college from a sample of 900 students, 450 were found to use dot pens. Test whether the
two colleges are significantly different with respect to the habit of using dot pens.
[Given : Critical value of z at 5% L.O.S. (two tailed test) is 1.96].

c) One fifth percent of blades produced by a blade manufacturing factory turn out to be
defective. The blades are supplied in a packets of 10. Use Poisson distribution to calculate
the approximate number of packets containing no defective, one defective and two defective
blades respectively in a consignment of 100000 packets.
8. a) Determine p such that the function f(z) =

px
1
log( x 2 + y 2 ) + i tan 1 be an analytic function. 4
2
y

b) Find the analytic function whose imaginary part is

sinh 2 y
.
cos 2 x + cosh 2y

c) A die is thrown 120 times with the following results :


face :

frequency : 16

30

22

18

14

20

Test the hypothesis that the die is unbiased. Use 5% level of significance to test the
hypothesis.
[ Given : Value of Chi-square at 5% L.O.S. with 5 degrees of freedom = 11.07].

9. a) A sample of size 9 from a normal population gave x = 15.8 and variance 10.3. Find a
99% confidence interval for population mean.

b) An automobile manufacturing firm is bringing out a new model. In order to map out its
advertising campaign, it wants to determine whether the model will appeal most to a
particular age group or equally to all age groups.

The firm takes a random sample from persons attending a pre-view of the new model and
obtained the results as follows
Persons
who

Age Group
Under
20

20 40

40 60

60 and
above

Total

Liked the
car

200

70

60

70

400

Disliked
the car

50

30

20

50

150

Total

250

100

80

120

550

On the basis of this data, can it be concluded that the new model appeals equally to all age
groups ? Test it at 1% L.O.S. [Given : Value of Chi-square at 1% with 3DF = 11.345].

Set A

SLR-BB 78

-6-

*SLRBB78*

1+ i

c) Evaluate

(x y + ix 2 ) dz :

i) along the line from z = 0 to z = 1 + i


ii) along the real axis from z = 0 to z = 1 and along the line parallel to the imaginary axis
from z = 1 to z = 1 + i.
OR
c) Evaluate

z+3
dz where C is the circle | z i| = 2.
2z + 3z 2
2

_____________________

Set A

SLR-BB 79

*SLRBB79*
Seat
No.

Set

S.E. (Mechanical) (Part I) Examination, 2014


MACHINE TOOLS AND PROCESSES
Day and Date : Tuesday, 16-12-2014
Time : 10.00 a.m. to 1.00 p.m.

Max. Marks : 100

Instructions : 1) Q. No. 1 is compulsory. It should be solved in first 30 minutes


in Answer Book Page No. 3.
2) Answer MCQ/Objective type questions on Page No. 3 only.
Dont forget to mention, Q.P. Set (A/B/C/D) on Top of Page.
MCQ/Objective Type Questions
Duration : 30 Minutes

Marks : 20

1. Choose the correct answer :

20

Type 1 : Match the correct pair (1 mark each) :


1) Eccentric Turning

A) Drill Chuck

2) Job holding on Capstan lathe

B) Drilling operation

3) Spot facing

C) Shaping machine

4) Clapper Box

D) Four Jaw chuck

5) Holding of straight shank drill

E) Collet chuck

Type 2 : Choose proper answer (2 marks each) :


1) A conical shape head of screw must flush with surface after drilling a hole.
Which operation is required after drilling ?
A) Counter heading
B) Counter sinking
C) Counter drilling
2) Back gear mechanism on lathe will
A) Decrease the speed
B) Increase the speed
C) None of them
3) Swing over Diameter is the specification of lathe which gives
A) Distance between lathe centre to the bed
B) Distance between job and bed
C) Distance between chuck and tool post

P.T.O.

SLR-BB 79

*SLRBB79*

-2-

4) If more than one cutter is mounted on arbor of milling machine, then it is


A) Form milling

B) Slab milling

C) Gang milling

5) A dielectric fluid is required in between tool and job in


A) ECM

B) USM

C) EDM

Type 3 : Select proper answer (True/False) (1 mark each) :


1) Non conducting materials can be machined by ECM.
A) True

B) False

2) If pitch of lead screw is in mm and pitch of job is in inches, still we can


perform thread cutting on lathe
A) True

B) False

3) Horizontal milling machine can have X, Y, Z axes motions of the job


A) True

B) False

4) Grinding of hard and brittle materials require fine grit size.


A) True

B) False

5) In broaching operation, number of reciprocating cutting strokes is required.


A) True

B) False
______________

Set A

*SLRBB79*

-3-

SLR-BB 79

Seat
No.

S.E. (Mechanical) (Part I) Examination, 2014


MACHINE TOOLS AND PROCESSES
Day and Date : Tuesday, 16-12-2014
Time : 10.00 a.m. to 1.00 p.m.

Marks : 80

Instructions : 1) Attempt any three questions from Section I and any three
questions from Section II.
2) Draw neat sketches wherever necessary.
3) Assume suitable data wherever necessary and mention
it clearly.
SECTION I
2. a) List different types of attachments used on Lathe machine. Explain any one
of them in detail with neat sketch.
b) Explain with neat diagram Bar Feed mechanism of Capstan lathe.
3. a) What are the characteristics of Nonconventional machining operations ?

7
6
4

b) Explain various operations carried out on Drilling machine.

c) Explain with neat sketch construction and working of Shaping machine.

4. a) Explain with neat sketch Standard Double Housing Planning Machine.


b) Differentiate clearly between Shaping and Planing.
5. Write short notes on following (any three) :

7
6
13

a) Electro Chemical Machining (ECM)


b) Taper Turning attachment on centre lathe
c) Automats
d) Limitations of Ultrasonic machining
e) Specifications of Drilling machine.

Set A

SLR-BB 79

-4-

*SLRBB79*

SECTION II
6. a) Explain with neat sketch various milling operations.

b) Explain operations that are carried out on Boring machine.

7. a) Explain with neat sketch Push and Pull broaching operations.

b) Give classification of grinding processes. Explain in detail cylindrical grinding. 7


8. a) Explain in detail Gear Hobbing Process.

b) What are grinding wheel types ?

c) List down at least Six types of Milling cutters.

9. Write short notes on following (any three) :

13

a) Difference between up milling and down milling


b) Gear finishing processes
c) CNC machines
d) Vertical milling attachment
e) Grinding wheel bonds.
_____________________

Set A

SLR-BB 80

*SLRBB80*
S

S.E. (Mech.) (Part I) Examination, 2014


MACHINE DRAWING
Day and Date : Thursday, 18-12-2014
Time : 10.00 a.m. to 2.00 p.m.

Max. Marks : 100

Instructions : 1)
2)
3)
4)
5)

Retain all constructional details.


Figures to the right indicate full marks.
Assume suitable data if necessary and mention it clearly.
Only first angle method of projection should be used.
Q. No. 1 is compulsory. It should be solved in first 30
minutes in Answer book Page No. 3.
6) Answer MCQ/Objective type questions on Page No. 3
only. Dont forget to mention, Q.P. Set (A/B/C/D) on Top
of Page.

Duration : 30 Minutes

MCQ/Objective Type Questions

1. Choose the correct answer :

Marks : 20
20

Type A :
a) Match the pairs.
Column I
1)
2)
3)

Column II
a)
b)
c)
d)

N11
N8
N5
N3

b) Match the pairs.


Column I
Welding jt. symbol
1)
2) *
3) XXX


3
Column II
Welding jt. type
a)
b)
c)
d)

Seam weld
Spot weld
Square butt
Fillet weld

P.T.O.

SLR-BB 80

*SLRBB80*

-2-

Type B : Write correct or incorrect.


1)

2)

This is a symbol for internal thread geometry.

is an example of bilateral tolerance.

3) A hole whose lower deviation is zero is called as basic hole.


4) An Oldhams coupling connects two intersecting shaft.
Type C : Multiple correct answers :

1) Which of the following are allowable sizes for


a) 100.00
b) 100.005
c) 100.04


2) Different lay patterns shown on production drawing are


a) M
b) Y
c) C

d) 100.103
d) P

Type D : Single correct answer :

1) Journal bearings can support only ____________


a) Axial load
b) Radial load
c) Inclined load d) None of these
2) The angle of buttress thread is ______________ degrees.
a) 90
b) 50
c) 45
d) 29
3)

represents ______________ section.

a) full
b) half
c) partial
d) revolved
4) Universal joint is used to connect two ______________ shafts.
a) parallel
b) intersecting c) perpendicular d) skew
5) Which of the following symbol is used to show that Machining is prohibited ?
a)

6)

b)

c)

d)

indicates which type of dimensioning.


a) Aligned

b) Multidirectional c) Unidirectional
______________

d) Non-linear
Set A

*SLRBB80*
S

-3-

SLR-BB 80

S.E. (Mech.) (Part I) Examination, 2014


MACHINE DRAWING
Day and Date : Thursday, 18-12-2014
Time : 10.00 a.m. to 2.00 p.m.

Marks : 80

Instructions : 1) Attempt any two questions from Section I from Q. No. 2


to Q. No. 4.
2) Question 5 from Section II is compulsory.
3) Attempt any one question from Section II from the
remaining.
4) Retain all constructional details.
5) Figures to the right indicates full marks.
6) Assume suitable data if necessary and mention it clearly.
7) Only first angle method of projection should be used.
SECTION I
2. a) Show conventional representation with actual machine section of the
following :

(42=8)

i) Slotted head
ii) Spur/Helical gears.
b) Draw neat and proportionate free-hand sketches of the following :

(62=12)

i) Foot-step bearing
ii) Single revetted butt joint.
3. a) Show the BIS conventions of the following :
i) Copper and its alloys

(24=8)

ii) Splined shaft


iii) Compression spring with square section
iv) Straight knurling.
b) Draw neat and proportionate free-hand sketches of the following :
i) Socket and spigot joint (cotter).
ii) Wing nut.
iii) Knuckle threads.

6
3
3
Set A

SLR-BB 80

-4-

*SLRBB80*

4. a) Represent by drawing a hole basis system and shaft basis system.

b) Fig. No. 01 shows front view, incomplete top view and incomplete auxiliary
view of a machine component. Redraw the given views and complete the top
view and auxiliary view.
15

Fig. No. 1
Set A

*SLRBB80*

-5-

SLR-BB 80

SECTION II
5. Refer Fig. No. 2. Prepare an assembly drawing of a Drill Jig with the help of
detailed parts shown in the fig. Draw
i) Sectional front view
ii) Top view
Also prepare a bill of materials.

Fig. No. 2

30

Set A

SLR-BB 80

-6-

*SLRBB80*

6. Refer Fig. 3 and draw


i) Sectional F.V. (Section along A-A)
ii) Right hand side view and
indicate the following parameters with proper datum representation.
i) Cylindrical feature of


10

80 has circular run out within 0.01 mm.

ii) Surface A is grounded to obtain circular lay pattern and Ra value of


10 microns.
iii) Surface B is flat within 0.02 mm.
iv) The position of 30 hole is fixed within 0.01 mm.


Fig. No. 3
7. Calculate the working dimensions, recognize the type of fit and calculate the
maximum clearance/interference for 100 H7n6 combination.


10

Represent the limit sizes by drawing proportionate fig.


Set A

*SLRBB80*

-7-

_____________________

SLR-BB 80

Set A

Set A

SLR-BB 81

*SLRBB81*m
S

S.E. (Mechanical) (Part II) (Old) Examination, 2014


FLUID MECHANICS
Day and Date : Tuesday, 25-11-2014
Time : 3.00 p.m. to 6.00 p.m.

Max. Marks : 100

Instructions : 1) Q. No. 1 is compulsory. It should be solved in first 30 minutes in


Answer book Page No. 3.
2) Neat diagrams must be drawn wherever necessary.
3) Use of single memory non-programmable calculator is allowed.
4) Assume suitable data if necessary and state it clearly.
5) Answer MCQ/Objective type questions on Page No. 3 only. Dont
forget to mention, Q.P. Set (A/B/C/D) on Top of Page.
MCQ/Objective Type Questions
Duration : 30 Minutes
Marks : 20
Type 1 : Match the following :
1.

Column I
A)


u


"

P) Steady flow

C)

B)

Column II

Q) In compressible flow

R) Uniform and nonuniform flow

S) Steady and unsteady flow


2.

Column I
A) Centroid
B) Centre of Pressure
C) Presurface

p)
q)
r)
s)

Column II
Resultant force
Area moment zero
Constant pressure
Point of application of displaced weight

Type 2 : Assertion and Reason Type Attempt any one from two questions. Each question
carries two marks.
It consists of two statements, one labelled as statement 1 and other labelled as statement 2.
Select the answer using following code :
A) Both statement 1 and Statement 2 are true and statement 2 is the correct explanation of statement 1.
B) Both statement 1 and statement 2 are true, but statement 2 is not the correct explanation of statement 1.
C) Statement 1 is true, but statement 2 is false.
D) Statement 1 is false, but statement 2 is true.
3. Statement 1 : According to Bernoullis theorem, total head with flowing fluid is constant.
Statement 2 : In a Pitot tube pressure head is converted to kinetic head.

4. Statement 1 : In Venturimeter the velocity of liquid at the throat is always higher than at inlet.
Statement 2 : In Venturimeter the pressure of liquid at the throat is always higher than
at inlet.

2
P.T.O.

SLR-BB 81

*SLRBB81*

-2-

Type 3 : Multiple correct answer type : Answer any four from 5 questions given. Each carries 2 marks.
5. Equipotential lines
2
A) These are the lines drawn along the stream lines
B) These are the lines drawn perpendicular to the stream lines
= constant
C) Represented by
= constant
D) Represented by


6. In case of parallel pipe


A) Loss of head in each branch pipe is same
B) Loss of head in each branch pipe is different
C) Discharge in main pipe is equal to sum of rate of flow through branch pipe
D) Discharge in main pipe decreases

7. Equivalent pipes
2
A) Loss of head and discharge equal to loss of head and discharge in compound pipe
B) Loss of head and discharge are different to loss of head and discharge in compound pipe
C) Length of equivalent pipe is sum of lengths of compound pipe
D) Length of equivalent pipe is multiplication of length of compound pipes
8. The component of acceleration in x, y direction are
A) ax = u
C) ay = u

+v
K

+v

+w

+w

B) ax = u

2


D) ay = u

+w

+v
L




+v
K

+w

z


9. Properties of a velocity potential function


A) Its differential in any direction gives velocity in perpendicular direction to that direction
B) Its negative differential in any direction gives velocity in that direction
C) Irrotational flow exists it is continuous
D) It is a continuous function it any flow exists

Type 4 : Straight objective type questions. Answer any four from 5 questions given. Each carries 1 mark.
10. The continuity equation can be written as in the form as
B) 1 A1V1 = 2 A2V2
A) A1V1 = A2V2
C) A1 1 = A2 2
D) P1A1V1 = P2 A2V2


11. The total drag on the body is sum of


A) Pressure drag and velocity drag
C) Friction drag and velocity drag

B) Pressure drag and friction drag


D) None of above

12. The boundary layer separation takes place if


A) Pressure gradient is zero
B) Pressure gradient is negative
C) Pressure gradient is positive
D) None of a above
13. The body is called stream lined body when it is placed in a flow and surface of the body
A) coincides with the stream lines
B) does not coincides with the stream lines
C) is perpendicular to the stream lines
D) None of above
14. Similarity of forces between model and prototype
A) Geometric similarity
B) Dynamic similarity
C) Kinematic similarity

D) None of above

______________

Set A

*SLRBB81*
S

-3-

SLR-BB 81

S.E. (Mechanical) (Part II) (Old) Examination, 2014


FLUID MECHANICS
Day and Date : Tuesday, 25-11-2014
Time : 3.00 p.m. to 6.00 p.m.

Marks : 80

Instructions : 1) Answer three questions from each Section Question No. 2 and
Question No. 6 are compulsory.
2) Neat diagrams must be drawn wherever necessary.
3) Use of single memory non-programmable calculator is allowed.
4) Assume suitable data if necessary and state it clearly.
SECTION I
2. a) Define the terms of velocity potential function and stream function. What are the conditions
for the flow to be irrotational ?
7
b) Describe the doublet and strength of doublet.

3. a) Explain the different types of fluid flows in detail.

b) A cylindrical vessel 12 cm in diameter and 30 cm deep is filled with water up to the top.
The vessel is open at the top. Find the quantity of liquid left in vessel, when it is rotated
about its vertical axis with a speed of 3000 rpm.

4. a) In a two dimensional incompressible flow, the fluid velocity components are given by
u = x 4y and v = y 4x. Show that velocity potential exists and determine it.

b) State the Archimedes principle. Derive the expression for the centre of pressure and total
pressure for vertical submerged body.
6
5. a) Define the terms Buoyancy and Centre of Buoyancy. Explain with neat sketch the
condition of equilibrium for floating bodies.

b) A rectangular plane surface 3 m wide, 4 m deep lies in water in such a way that its, plane
makes an angle 30 with the free surface of water. Determine the total pressure force and
position of centre of pressure when the upper edge is 2 m below the free surface.
6
SECTION II
6. a) Give the importance of CFD and write its applications in detail.

b) A pipe, through which water is flowing is having diameter 40 cm and 20 cm at the cross
section 1 and 2 respectively. The velocity of water at a section 1 is 5.0 met/sec. Find the
velocity head at section 1 and 2 and also rate of discharge.

c) Explain the term equivalent pipe.

4
Set A

SLR-BB 81

*SLRBB81*

-4-

7. a) Two tanks whose water surface levels differ by 10 m are connected by three pipes of
different diameters 250 mm, 150 mm and 200 mm in series. Their lengths are 300 m, 250 m
and 150 m respectively. Their co-efficient of frictions are 0.005, 0.006 and 0.0055
respectively. Find the rate of flow through the pipe neglecting minor losses. Also find the
values of the individual losses of head.

b) The frictional torque T of a disc of diameter D rotating at speed N in a fluid viscosity




and density


in turbulent flow is given by T = D5N2







8. a) Derive an expression for rate of flow through orificemeter.

b) Why the length of the divergent outlet part in a venturimeter is usually made longer as
compared to that of the converging part ?

9. a) Derive an expression for power transmission through pipes. What is the condition for
maximum transmission of power ?

b) Prove that velocity distribution for viscous flow between two parallel plates when both
plates are fixed across a section is parabolic in nature. Also prove that maximum velocity
is equal to one and half times the average velocity.

_____________________

Set A

SLR-BB 82

*SLRBB82*
Seat
No.

Set

S.E. (Mech.) (Part II) Examination, 2014


THEORY OF MACHINES I (Old)
Day and Date : Wednesday, 26-11-2014
Time : 3.00 p.m. to 6.00 p.m.

Total Marks : 100

Instructions : 1)
2)
3)
4)
5)
6)
7)

Make suitable assumptions if necessary and state them clearly.


Draw neat diagrams wherever necessary.
Figures to the right indicate full marks.
Figures in the question papers are not drawn to the scale.
Use of non-programmable scientific calculator is allowed.
Solve graphical method on blank drawing sheets provided.
Q. No. 1 is compulsory. It should be solved in first 30 minutes in Answer book
Page No. 3. Each question carries one mark.
8) Answer MCQ/Objective type questions on Page No. 3 only. Dont forget to mention,
Q.P. Set (A/B/C/D) on Top of Page.
MCQ/Objective Type Questions

Duration : 30 Minutes
1.

Marks : 20

1)

Choose the correct statement


a) Toothed gears in mesh constitute a higher kinematic pair
b) Belt on pulley drive constitute a higher kinematic pair
c) Chain and sprocket drive constitute a higher kinematic pair
d) All of the above
2) In a kinematic chain, a quaternary joint is equivalent to
a) Two binary joints
b) Three binary joints c) Four binary joints
d) None of the above
3) Instantaneous center of rotation of a round disc rolling on straight horizontal path without slipping has its
centre of rotation
a) At the centre of gravity of the disc
b) At the centre of the circle whose diameter is equal to the diameter of the disc
c) At the point of contact of the disc with straight horizontal path
d) None of the above
4) Two kinematic links 1 and 2 of a mechanism have absolute angular velocity 1 (clockwise) and
2 (anticlockwise) respectively. The angular velocity of the link relative to link 2 is
a) 1 + 2

b) 1 2

c) 2 1

d) None of the above

5) Coriolis component of acceleration exists whenever a point moves along a path that has
a) Linear motion
b) Rotational motion
c) Tangential acceleration
d) Centripetal acceleration
6) Watt straight line mechanism is
a) Mathematically correct method of generating straight line path
b) Approximate method of generating straight line
c) Method for generating mathematically correct circular path
d) Approximate method of generating circular path
7) The angular velocity ratio of Hookes joint is unity when,
a) tan = cos

b) tan = cos

c) tan =

tan

d)

tan = tan

8) In a reciprocating engine, the thrust on the sides of the cylinder perpendicular to cylinder walls is
a) FN = FP
c) FN =

sin2
(n sin2 )1.5
2

FP
(n2 sin 2 )1.5

b) FN = FP

sin
(n2 sin2 )0.5

d) FN = FP

n sin
(n2 sin 2 )0.5

9) The throw of a cam is the maximum distance of the follower from


a) Base circle
b) Pitch circle
c) Prime circle

d) Pitch curve
P.T.O.

SLR-BB 82

*SLRBB82*

-2-

10) Pitch point as referred to cam is


a) Any point on pitch curve
b) The point on pitch curve with maximum pressure angle
c) The point on pitch curve with minimum pressure angle
d) Any point on pitch circle
11) The pressure angle of cam
a) Decreases with decrease in base circle diameter b) Decreases with increase in base circle diameter
c) Is independent of base circle diameter
d) Is maximum for a particular base circle diameter
12) For the same pitch, the efficiency of screw jack with square threads is
a) More than that with V-threads
b) Less than that with V-threads
c) Same as that with V-threads
d) Dependent only on the load on the jack
13) Maximum efficiency of screw jack
a) Increases with increase of load on the jack
b) Increases with increase in pitch of the screw
c) Increases with increase in co-efficient of friction
d) Is true for all the above statements
14) In flat pivot bearing the total power lost in friction
a) For uniform wear is greater than that for uniform pressure
b) For uniform wear is less than that for uniform pressure
c) For uniform wear is same as that for uniform pressure
d) Is unpredictable
15) Considering for safe design, the friction clutch should be designed
a) Assuming uniform pressure
b) Assuming uniform wear
c) Assuming any criteria, either uniform wear or uniform pressure
d) Assuming uniform pressure for high torque and uniform wear for low torque
16) Refer Fig. 16. It shows a band brake arrangements. The brake to work with force at A upwards, and direction
of drum clockwise

Fig. 16
a) a = b
b) a > b
c) a < b
d) independent of ratio a/b
17) Refer Fig. 17. It shows band brake. F is the force at lever end acts downwards. The condition for self locking is

Fig. 17

b
for counter clockwise direction
a)
a <e
b
for both directions
c)
a <e

b)

b
for clockwise direction
a <e

d) None of the above

18) Choose the correct statement


a) Hartung governor is gravity controlled type
b) Pickering governor is gravity controlled type
c) Hartnell governor is gravity controlled type
d) Proel governor is gravity controlled type
19) The frictional resistance at the sleeve of a governor
a) Increases sensitivity of governor
b) Decreases sensitivity of governor
c) Does not affect the sensitivity of governor
d) May increase or decrease sensitivity depending upon when speed is increasing or decreasing
20) The main objective of controlling force is to
a) Determine stability of governor
b) Determine sensitiveness of governor
c) Determine power of governor
d) All the above
______________

Set A

*SLRBB82*

-3-

SLR-BB 82

Seat
No.
S.E. (Mech.) (Part II) Examination, 2014
THEORY OF MACHINES I (Old)
Day and Date : Wednesday, 26-11-2014
Time : 3.00 p.m. to 6.00 p.m.
Instructions :

1)
2)
3)
4)
5)
6)
7)

Marks : 80

Attempt any 2 questions from each Section.


Make suitable assumptions if necessary and state them clearly.
Draw neat diagrams wherever necessary.
Figures to the right indicate full marks.
Figures in the question papers are not drawn to the scale.
Use of non-programmable scientific calculator is allowed.
Solve graphical method on blank drawing sheets provided.
SECTION I

2.

a)

Describe the inversions of double slider crank chain.

b)

Locate the instantaneous centers and draw the circle diagram for the mechanism in Fig. 2b.

Fig. 2b

3.

c)

Explain how Tchebihoffs mechanism can be used to produce straight line motion.

a)

For the mechanism shown in fig-3a the wheel pivoted at O is rotating at 1800 rpm and is driving the wheel
7 pivoted at Q without slipping. There is a rolling contact. Construct the acceleration diagram and determine
the linear acceleration of the slider block 5 and also the angular acceleration of links 3, 4 and 6 clearly
mentioning the sense of angular acceleration.

12

Fig. 3a
b)

Explain Ackerman steering mechanism.

8
Set A

SLR-BB 82
4.

a)

*SLRBB82*

-4-

Explain the effect of the inertia of the connecting rod on the crank shaft and determine the total torque thus
exerted.

b)

Explain with examples a machine, a mechanism and a structure.

c)

Two shafts are coupled together by universal coupling, the driving shaft rotating uniformly at 600 rpm. Find
the greatest permissible angle if the maximum speed of the follower shaft is not to exceed 630 rpm. What is
then the minimum speed of the driven shaft ?

SECTION II
5.

a)

b)

It is required to set out the profile of a cam to give the following motion to the follower :
i) Follower to move outward through 31.4 mm during 180 of cam rotation, with cycloidal motion.
ii) Follower to return with cycloidal motion during 180 of cam rotation.
The base circle of the cam is 30 mm diameter and the roller diameter of the follower is 10 mm. The axis of the
roller is offset by 7.5 mm to the right.

12

In the band and block brake system as shown in fig.-5b the band is lined with 10 blocks. Each block subtends
an angle of 16 at the centre of the wheel. For the given arrangement calculate the maximum force F required
8
at lever end for the bracket to absorb 225 kW at 300 rpm. Take = 0.4. Effective diameter is 80 cm.

Fig. 5b
6.

a)

b)

7.

a)

Derive an expression for determining the efficiency of a square threaded screw jack when
i) The nut is fixed
ii) The nut rotates.
In a spring loaded governor of the Hartnell type the weight of each ball is 5.45 kg and the lift of the sleeve
is 6.3 cm. The speed at which the governor begins to float is 240 rpm and at this speed the radius of the ball
path is 14 cm. The mean working speed of the governor is 18 times the range of speed when the effect of
friction is neglected. Calculate the stiffness of the spring when the length of the vertical arm is 15.3 cm, the
length of the horizontal arm is 12.7 cm and the fulcrum of the lever O from the axis of the vertical spindle
is 17.7 cm.
For governors explain :
i) sensitiveness

ii) stability

iii) isochronisms.

12

b)

With suitable examples classify cams.

c)

A multiple disc clutch has five plates having four active parts of frictional surfaces. Determine the maximum
axial intensity of pressure between the discs for transmitting 18 kW at 500 rpm if the outer and inner radii of
the friction surface are 12.5 cm and 7.5 cm respectively. Assume uniform wear and take = 0.3.

_____________________
Set A

SLR-BB 83

*SLRBB83*
Seat
No.

Set

S.E. (Mech.) (Part II) (Old) Examination, 2014


MANUFACTURING PROCESSES
Day and Date : Thursday, 27-11-2014
Time : 3.00 p.m. to 6.00 p.m.

Total Marks : 100

Instructions : i) Draw neat sketches wherever necessary.


ii) Q. No. 1 is compulsory. It should be solved in first 30 minutes
in Answer Book Page No. 3.
iii) Answer MCQ/Objective type questions on Page No. 3 only.
Dont forget to mention, Q.P. Set (A/B/C/D) on Top of Page.
Duration : 30 Minutes

MCQ/Objective Type Questions

Marks : 20

Type 1 :
1. Match the pairs :
A) SMAW
p) Oxy-acetylene gas
B) TIG
q) Inert gas
C) Brazing
r) Granular flux
s) Non-ferrous filler metal

Type 2 : Attempt the following :

2. i) Removal of core is a fettling operation.


ii) Fettling is operation of forging process.
A) Statement i) is correct and ii) is incorrect
B) Statement i) is incorrect and ii) is correct
C) Both i) and ii) are correct
D) Both i) and ii) are incorrect
Type 3 : Attempt the following :

(2 mark each)

3. Which of the following statements are incorrect ?


A) In soldering, filler metal having liquidus not exceeding 427C is used
B) In brazing, filler metal having liquidus not exceeding 427C is used
C) In SMAW, the shield is provided to arc pool by inert gas
D) Bare wire can be used as filler metal in gas welding

P.T.O.

SLR-BB 83

*SLRBB83*

-2-

4. In hot chamber die casting


A) Melting pot is separate from the machine
B) Melting pot is an integral part of the machine
C) Melting pot may have any location
D) High temperature and pressure is used
5. In which process, bare wire is used as filler metal ?
A) Gas welding B) Arc welding C) Brazing

D) Resistance welding

6. Which of the following process uses consumable electrode ?


A) TIG
B) MIG
C) SMAW
D) Resistance welding
Type 4 : Attempt the following :

(1 mark each)

7. In a ______________, the molten metal is poured and allowed to solidify while


mould is rotating.
A) Die casting
B) Sand moulding
C) Permanent mould casting
D) Centrifugal casting
8. The purpose of adding wood flour or saw dust in foundry sand is to improve
A) Mouldability B) Dry strength C) Hot strength
D) Collapsibility
9. Greater forging capacity is achieved with
A) Mechanical press
B) Power hammer
C) Hydraulic press
D) None of them
10. Foundry crucible is made up of
A) Stainless steel
C) Lead

B) Graphite
D) Mild steel

11. In cold working process


A) Good surface finish is produced
C) Close tolerance is difficult to obtain

B) Scales formed
D) All of the above

12. Tube shell is produced from billet by


A) Forging
B) Extrusion
C) Rolling

D) None

13. Wire is produced by


A) Drawing
B) Forging

D) Extrusion

C) Casting
______________

Set A

*SLRBB83*

-3-

SLR-BB 83

Seat
No.

S.E. (Mech.) (Part II) (Old) Examination, 2014


MANUFACTURING PROCESSES
Day and Date : Thursday, 27-11-2014

Marks : 80

Time : 3.00 p.m. to 6.00 p.m.


Instructions : i) Attempt any three questions from each Section.
ii) Draw neat sketches wherever necessary.
SECTION I
1. a) Define pattern. Enlist different types of patterns. Explain any one in detail.

b) Explain with neat sketch cupola furnace in detail.

2. a) What are components of gating system ? Explain functions of each part.

b) What is hot chamber die casting ? Explain goose-neck type hot chamber die
casting.

3. a) What are different types of furnaces used in foundries ? Explain any arc
furnace used in steel foundry.

b) What are different types of equipments used in fettling operation ? Explain Air
blasting in detail.

4. Write short notes on :


a) Inspection procedure of casting.

b) Foundry mechanization.

c) Pouring equipments.

d) Semi-centrifugal casting.

5
Set A

SLR-BB 83

-4-

*SLRBB83*

SECTION II
5. a) What is extrusion ? How are extrusion processes are classified ?

b) What do you understand by the term forging ? How does cold forging differ
from hot forging.

c) Explain with neat sketch wire drawing process.

6. Draw neat self explanatory sketches (No explanation is required) :


a) Backward extrusion.

b) Cold heading.

c) Three-high rolling mill.

7. a) Give comparison between TIG and MIG welding processes.

b) Name atleast three components that are manufacture by forging and Extrusion
process.

8. Write short notes on :


a) Brazing and soldering.

b) Four-High rolling mill.

c) Forging presses.

5
_____________________

Set A

SLR-BB 84

*SLRBB84*
S

S.E. (Mechanical) (Part II) (Old) Examination, 2014


ELECTRICAL AND ELECTRONICS TECHNOLOGY
Day and Date : Tuesday, 2-12-2014
Time : 3.00 p.m. to 6.00 p.m.

Max. Marks : 100

Instructions : 1) Assume suitable data if necessary.


2) Draw neat diagrams whenever necessary.
3) Q. No. 1 is compulsory. It should be solved in first 30 minutes
in Answer book Page No. 3. Each question carries one mark.
4) Answer MCQ/Objective type questions on Page No. 3 only.
Dont forget to mention, Q.P. Set (A/B/C/D) on Top of Page.

Duration : 30 Minutes

MCQ/Objective Type Questions

1. Choose the correct answer :

Marks : 20
(201=20)

1) The starting torque of a D.C. motor is independent of which of the following ?


a) flux
b) armature current
c) flux and armature current
d) speed
2) In induction heating _______________ is abnormally high.
a) phase angle
b) frequency
c) current
d) voltage
3) Induction furnaces are employed for which of the following ?
a) heat treatment
b) heat of insulators
c) melting aluminum
d) none of the above
4) Resistance welding cannot be used for
a) dielectrics
b) ferrous materials
c) non-ferrous metals
d) none of these
5) Motors preferred for rolling mill drive
a) d.c. motors
b) ac slip ring motors with speed control
c) any of the above
d) none of the above
6) A single phase energy meter has the rating 1200 resolutions/kWh. If a 500 W
electric gadget is used for 4 hours, the energy meter will make
a) 1200 revolutions
b) 1800 revolutions
c) 2100 revolutions
d) 2400 revolutions
7) In series motor which of the following methods can be used for changing the flux
per pole ?
a) tapped field control
b) divert field control
c) series-parallel control
d) any of these
P.T.O.

SLR-BB 84

*SLRBB84*

-2-

8) DC motors are still preferred for use in


a) electric excavators, steel mills and cranes
b) lathes and machine tools
c) flour mills and jaw crushers
d) paper industry
9) Moving iron and PMMC instruments can be distinguished from each other by
looking at
a) pointer
b) terminal size
c) scale
d) scale range
10) All meters used for measuring current, voltage and resistance are basically
a) voltmeters
b) ammeters
c) multimeters
d) tachometers
11) Basic building block of combinational digital circuit are
a) gates only
b) flip-flop only
c) gate and flip-flop only
d) none
12) 8051 microcontroller has inbuilt ________________
a) timers
b) I/O ports
c) limited memory d) all of above
13) The output of Nand gate is zero when _____________
a) all inputs are zero
b) when any input is zero
c) all inputs are one
d) when any input is one
14) n cascade flip-flop count _____________ binary numbers.
a) 2(n 1)
b) 2n 1
c) 2n
d) 2n + 1
15) Input resistance and output resistance of ideal op amp respectively are
a) zero, infinite
b) infinite, zero
c) infinite, finite
d) none
16) The 2s complement of (13)10 is
a) (0010)2
b) (0011)2

c) (1100)2

d) (1110)2

17) In microcontroller 8051 address bus is of ______________ bits wide.


a) 64
b) 8
c) 16
d) 32
18) Non-inverting amplifier using op-amp has voltage gain of
a)

b)

c)

d)


4


19) The flip-flop which is free from race around problem is _____________
a) RS FF
b) Master slave JK FF
c) JK FF
d) All
20) Which type of the following memory is non-volatile and may be written only once ?
a) RAM
b) EEPROM
c) EPROM
d) PROM
______________

Set A

*SLRBB84*
S

-3-

SLR-BB 84

S.E. (Mechanical) (Part II) (Old) Examination, 2014


ELECTRICAL AND ELECTRONICS TECHNOLOGY
Day and Date : Tuesday, 2-12-2014
Time : 3.00 p.m. to 6.00 p.m.

Marks : 80

Instructions : 1) Assume suitable data if necessary.


2) Draw neat diagrams whenever necessary.
SECTION I
2. Attempt any four of the following :

(45=20)

a) What is an electrical drive ? Explain what are the factors on which the selections
of electrical drive depend ?
b) Explain the necessary of starter in a dc motor and describe three point starter.
c) Discuss the relative merits and demerits of direct and indirect arc furnaces.
d) Explain with neat diagram working of digital tachometer and compare with analog
tachometer.
e) A 500 V shunt motor runs at its normal speed of 250 rpm when the armature current
is 200A. The resistance of armature is 0.12 ohm. Calculate the speed when a
armature is inserted in the field reducing the shunt field to 80% of normal value
and the armature current is 100 Ampere.
f) What are the advantages of electric heating over other systems of heating and
write the priorities of high frequency eddy current heating.
3. Attempt any two of the following :

(210=20)

a) Describe the constructional details of single phase induction type energy meter.
Explain, why the phase of shunt flux is made exactly in quadrature with that of
applied voltage so as to produce a deflecting torque exactly proportional to power.
b) Explain in briefly how heating is done in the following cases ?
i) Resistance heating
ii) Induction heating
iii) Dielectric heating.

c) What are the various methods of controlling the speed of three induction motors ?
Explain briefly the basic principles of operation of these methods.

Set A

SLR-BB 84

-4-

*SLRBB84*

SECTION II
4. Attempt any four of the following :

(45=20)

a) Explain with block diagram PLC system.


b) Explain three input summing amplifier in inverting configuration.
c) Explain memory organisation of 8051 microcontroller.
d) Explain working of basic 4-bit register using D flip-flop.
e) Explain Program Status Word (PSW) resistor of microcontroller 8051.
f) Explain characteristics of an ideal op-amp.
5. Attempt any two of the following :

(210=20)

a) Explain with neat diagram operation of op-amp as differentiator and integrator.


b) Explain working of JK flip flop in detail.
c) Explain interrupt structure of microcontroller 8051.
_____________________

Set A

SLR-BB 85

*SLRBB85*
Seat
No.

Set

S.E. (Mech.) (Part II) Examination, 2014


ENGINEERING MATHEMATICS III (Old)
Day and Date : Saturday, 29-11-2014
Time : 3.00 p.m. to 6.00 p.m.

Max. Marks : 100

N. B. : 1) Q. No. 1 is compulsory. It should be solved in first 30 minutes in


Answer book Page No. 3. Each question carries one mark.
2) Attempt any three questions from each Section.
3) Figures to the right indicate full marks.
4) Use of non-programmable calculator is allowed.
5) Answer MCQ/Objective type questions on Page No. 3 only. Dont
forget to mention, Q.P. Set (A/B/C/D) on Top of Page.
MCQ/Objective Type Questions
Duration : 30 Minutes

Marks : 20

1. Choose the correct option :


1) The general solution of the diff. equation (D + 1)2 (D 2)y = 0 is
a) y = c1 cosx + c2sinx + c3e2x
b) y = (c1 + c2x)ex + c3e2x
c) y = c1ex + c2e2x
d) y = (c1 + c2x)ex + c3e2x

2) Particular integral of (D2 + 6D + 9)y = e3x is


2
3 x
a) x e
12

2
3 x
b) x e
4

2
3 x
c) x e
2

3 x
d) e
18

d
d2 y
dy
3) If D
and z = logx, then the diff. equation x 2 + 2
= 6x becomes
dz
dx
dx
a) D(D + 1)y = 6e2z b) D(D + 1)y = 6ez c) D(D 1)y = 6e2z d) None of these
d2 y
dy
4( x + 2)
+ 6 y = 0 is
2
dx
dx
b) y = c1e2x + c2e3x
a) y = c1(x + 2)2 + c2(x + 2)3

4) The general solution of ( x + 2)2

c) y = c1(x + 2)3 + c2(x + 2)2


5)

d) y = c1(x + 2)2 + c2(x + 2)3

1 x = _______
D3

x1
x+1
b)
3 9
3 9
6) The general solution of P.D.E., pq = 1 is
a)

3x x
1
c) e +
3 9

x 1
d)
3 9

1
a) z = ax + y + c
a

b) z = a(x + y) + c c) z = ax 1 y + c d) None of these


a
7) The solution of 2p + 3q = 1 is
a) (3x + 2y, y + 3z) = 0
b) (3x 2y, y 3z) = 0
c) (3x + 2y, y 3z) = 0

d) (3x 2y, 3y + z) = 0

P.T.O.

SLR-BB 85

*SLRBB85*

-2-

8) A function can be expanded in Fourier series if it satisfies


a) Fouriers conditions
b) Eulers conditions
c) Bernoullis formula
d) Dirchlets conditions
9) If f(x) = x2 is expanded as a cosine series in (0, ) then constant term is
a)

3
3

b)

2
3

c)

3 2
2

d)

2 3
3

10) The Fourier series of the function f( x) = x + 1, x 0 contains


= x + 1,
0x
a) Only cosine terms
b) Only sine terms
c) Both sine and cosine terms
d) Cannot be predicted
11) The value of integral
a) 1243

e 3 t t 5 dt = _____________

b) 4 243

c) 40 243

d) 80 243

1
12) L1
= ___________
3/2
( s 2)
a) e 2 t t

1
2

2 2 t

13) L t e
a)

2
b) e 2 t t
3!

c) e t t

} = ___________

1
s+2

1
b) (s + 2)2

1
2

1
c) (s + 2)3

2
d) e t t
5

2
d) (s + 2)3

14) The function f(z) = 1 log( x 2 + y 2 ) + i tan 1(y x ) is analytic. For P = ____________
2
a) 1
b) 0
c) 1
d) 2
15) In the mapping W = 4z, the region x = 0, y = 0, x + y = 1 is transformed into __________
a) Square
b) Circle
c) Triangle
d) None of these
16) If F(z) = 2x + ixy2 then F(z) = ___________
a) 2 + iy2
b) x + iy2
c) 2x + 2iy2

d) Does not exist

17) If r = xi + yj + zk and a is a constant vector then ( a r ) = ____________


a) 2 a

b) a

c) r

d) 2 r

18) If f = ( x + 3 y ) i + ( y 2 z) j + ( x + az ) k is solenoidal then a = ____________


a) 2
b) 2
c) 1
d) 0
19) The directional derivative of = x2 + y2 + z2 at (1, 1, 1) along z-axis is __________
a) 0
b) 1
c) 2
d) 2i

20) If a is constant vector then curl( a ) = _______


a) 1
b) 0
c) 0
______________

d) None of these

Set A

*SLRBB85*

-3-

SLR-BB 85

Seat
No.

S.E. (Mech.) (Part II) Examination, 2014


ENGINEERING MATHEMATICS III (Old)
Day and Date : Saturday, 29-11-2014
Time : 3.00 p.m. to 6.00 p.m.
N. B. :

Marks : 80

1) Attempt any three questions from each Section.


2) Figures to the right indicate full marks.
3) Use of non-programmable calculator is allowed.
SECTION I

2. a) Solve (D3 + 3D2 4D 12)y = 12xe2x.

b) Solve (x2D2 + xD + 1)y = logx sin(logx).

c) Solve (D3 D2 6D)y = 1 + x2.

3. a) The diff. equation of a body projected upward in a resisting medium is given by

V
gt
g
d 2 s + K ds = g
ds = V
s = 2 + 0 1 e Kt .
.
If
at
t
=
0,
s
=
0
and
0 show that
2
K
K
dt
dt
dt
K

b) Solve p2 pq = 1 z2.

c) Solve pz = qz + z2 + (x + y)2.

4. a) Solve the following partial diff. equation

u
u
=4
where u(0, y) = 8e3y by method of
y
x

separation of variables.

b) Solve (D2 + 3D + 2)y = sinex.


2
c) Solve ( x + a )

d2 y
dy
4(x + a)
+ 6y = x .
2
dx
dx

OR
c) Find the half-range sine series of f(x) = lx x2 in (0, l).

5. a) Find the Fourier series of f(x) = 2x x2 in the interval (0, 3). Hence deduce that
2 = 1 + 1 + 1 + ....
6 12 2 2 3 2
b) Find the Fourier series of f(x) =
=

7
,
x,

< x < 0.
0<x<

Set A

SLR-BB 85

*SLRBB85*

-4-

SECTION II
6. Attempt the following :
a) Find the angle between the surfaces xy2z = 3x + z2 and 3x2 y2 + 2z = 1 at (1, 2, 1).

b) Prove that, 2 f(r ) = f (r ) + 2 f (r ) .


r

c) Find L t e t sin2 3t .

7. Attempt the following :

s + 29
a) Find L1
.
2
( s + 4) ( s + 9)

b) Show that

sin at dt = .
2
t

c) Show that u = cos x cosh y is harmonic function. Find its harmonic conjugate and
corresponding analytic function.

8. Attempt the following :


2

a) A vector field f = ( y sin z sin x ) i + ( x sin z + 2 yz) j + ( xy cos z + y ) k . Prove that it is


irrotational and hence find its scalar potential .
x 2 y 5 ( x + iy)
,
4
10
b) Show that f(z) = x + y

0
,

z0
z=0

is not analytic at origin although C-R equations are satisfied.

c) Find L1 tan 1(s + 1) .

9. Attempt the following :


a) Find the values of a, b, c if the directional derivative of = axy2 + byz + cz2x3 at (1, 2, 1)
has maximum magnitude 64 in the direction parallel to z-axis.

b) Find the bilinear transformation which maps z = 2, 1, 0 onto w = 1, 0, i.

c) Construct analytic function whose real part u = ex(xcosy ysiny).

_____________________

Set A

SLR-BB 86

*SLRBB86*
Seat
No.

Set

S.E. (Mech.) (New) (Part II) Examination, 2014


FLUID MECHANICS
Day and Date : Tuesday, 25-11-2014
Time : 3.00 p.m. to 6.00 p.m.

Max. Marks : 100

Instructions : 1) Q. No. 1 is compulsory. It should be solved in first 30 minutes in


Answer book Page No. 3. Each question carries one mark.
2) Do not fold or mutilate the answer sheet.
3) Answer MCQ/Objective type questions on Page No. 3 only. Dont
forget to mention, Q.P. Set (A/B/C/D) on Top of Page.
MCQ/Objective Type Questions
Duration : 30 Minutes
Marks : 20
1. Classical multi choice questions.
(201=20)
1) The flow of liquid at constant rate in a conically tapered pipe is classified as
a) Steady, uniform flow
b) Steady, non-uniform flow
c) Unsteady, uniform flow
d) Unsteady, non-uniform flow
2) The Cd of an orifice is always
a) Greater than Cc
b) Equal to Cv
c) Equal to Cc
d) Less than Cc
3) For a sphere falling at terminal velocity in the stokes law range, the drag coefficient CD is
given by
a) 24 Re
b) 64
c) 24
d) None of the above
Re
Re
4) Loss of head due to sudden expansion of pipe
a)

2(V1 V2 )
2g

b)

(V1 V2 )
2g

V 2 V 2
1
2
c)
2g

d)

(V1 V2 )2
2g

5) The velocity of flow through a pipeline according to chezys formula is given by


a) V = m Ci

b) V = i mC

c) V = C mi

d) V =

Cmi

6) An ice cube floating in a glass of water melts, after melting, the water level in the glass
a) Rises
b) First falls and then rises
c) Remains the same d) Falls
7) Pitot tube is used to measure
a) Discharge
b) Average velocity
8) In calculating the lift force
a) Always frontal area is used
c) Actual surface area of the body is used

c) Velocity at a point

d) Pressure at a point

b) Always planform area is used


d) None of the above

9) A large metacentric height


a) Improves stability and makes periodic time of oscillation longer
b) Has no effect on stability
c) Improves stability and makes periodic time of oscillation shorter
d) Has no effect on stability or periodic time of oscillations
P.T.O.

SLR-BB 86

*SLRBB86*

-2-

10) In two dimensional flow the equation of streamline is given by


dy
b) dx
u = v

dy dx
u = v

u = dy
c) dx
v

dy
d) dx = u,
=v
dt
dt
11) TEL is always higher than HGL, the vertical distance between the two represents
a) The pressure head
b) The piezometric head
c) The velocity head
d) None of the above
a)

12) Irrotational flow means


a) The fluid does not rotate while moving
b) The fluid moves in straight line
c) The net rotation of fluid particles about their mass centre is zero
d) None of the above
13) When pipes are connected in parallel, the total loss of head
a) Is equal to the sum of the loss of head in each pipe
b) Is same in each pipe
c) Is equal to the reciprocal of the sum of loss of head in each pipe
d) None of the above
p
v2
14) In Bernoullis equation g + 2g + z each term represents
a) Energy in Kg-m/Kg mass of fluid
b) Energy in N-m/Kg mass of fluid
c) Energy in N-m/N weight of fluid
d) Power in KW/Kg mass of fluid
15) A streamlined body is one for which
a) The skin friction is zero
b) The skin friction is minimum
c) The thickness of the body is minimum
d) The separation point occurs on the far downstream part of the body
16) Loss of head at the entrance of a pipe

V2
a) 0.5 2g

V2
b) 2g

V2
c) 0.1 2g

V2
d) 0.3 2g

17) The total head in a flow is the sum of


a) Piezometric head and datum head b) Piezometric head and pressure head
c) Piezometric head and velocity head d) Piezometric head, velocity head and datum head
18) A floating body displaces
a) A volume of liquid equal to its own volume
b) A volume of liquid equal to its own submerged weight
c) A weight of liquid equal to its own weight
d) A weight of liquid which depends upon the volume of the container
19) The head loss due to sudden contraction of
a)

2
V2

1 1
2g
CC

b)

1 1
CC

2
V2

2g

c)

2
V2

1 1
CC 2g

1
d) C 2

2
V2
2g

20) The square root of ratio of inertia force to pressure force is known as
a) Reynolds number b) Froud number
c) Mach number
d) Euler number
______________

Set A

*SLRBB86*

-3-

SLR-BB 86

Seat
No.

S.E. (Mech.) (New) (Part II) Examination, 2014


FLUID MECHANICS
Day and Date : Tuesday, 25-11-2014
Time : 3.00 p.m. to 6.00 p.m.
Instructions : 1)
2)
3)
4)

Marks : 80

Answer any two questions from each Section.


Neat diagrams must be drawn wherever necessary.
Use of non-programmable calculator is allowed.
Assume suitable data if necessary and state it clearly.
SECTION I

2. A) A rectangular plane surface 2m wide lies in water in such a way that its plane (or longer
side) makes an angle of 30 with the free surface of water. Determine the total pressure
force and position of center of pressure when the upper edge and lower edge of a rectangular
plane surface is 3m and 6m below the free surface of water respectively.
B) Water is flowing through a taper pipe having diameter 20cm at A to 50cm at B. Pressure
at A and B are 78.5 KN/m2 and 58.9 KN/m2 respectively with the end B being 3m
higher than A. If the flow in pipeline is 200 liters/sec find the direction of flow and head lost
in friction between A and B.
C) What is continuity equation ? Obtain an expression for continuity equation in Cartesian
Co-ordinates.
3. A) A body weighs 1030 N and 1200 N when weighted under submerged conditions in water
and kerosene (sp gr = 0.8) respectively. Determine the volume and weight of the body in
air. Also find specific gravity of the body.
B) The maximum velocity of laminar flow in a pipe of diameter 240 mm is 2.8 m/s. Determine
the mean velocity and the radius at which it occurs. Also find the velocity at 40mm from
the wall of the pipe.
C) Explain the principle of venturimeter with a neat sketch. Derive an expression for the
discharge through a venturimeter.
4. A) A nozzle is situated at a distance of 1m above the ground level and is inclined at an angle
of 45 to the horizontal. The diameter of the nozzle is 50mm and the jet of water from the
nozzle strikes the ground at a horizontal distance of 4m. Find
i) the rate of flow of water
ii) the maximum height attained by the jet above the ground
iii) the horizontal distance at which this maximum height is obtained.

7
6

7
6

B) Check whether the stream function = 5 xy is irrotational and if so, determine the
corresponding potential function ( ) .

C) For viscous flow through circular pipe of length(L) radius(R) the average velocity (u ) is

( )

p 2
given by u = 81 x R . Derive an expression for Hagen Poiseuille formula.

6
Set A

SLR-BB 86

-4-

*SLRBB86*

SECTION II
5. A) A partially submerged body is towed in water. The resistance (R) or force to its motion can
be expressed in the form
2

R = L V

Lg

,
.
VL V 2

Where density ( ) , viscosity of water ( ) , length of the body (L), velocity of the body (V)

and acceleration due to gravity (g).

B) A spherical balloon of diameter 3m and weight 135 N contains helium whose density is
0.22 Kg/m3 when the surrounding air density is 1.2 Kg/m3. If the balloon is tied to the
ground by a cable wire, what would be the inclination of the cable when wind blows
horizontally with a velocity of 10 km/hr ? Also find tension(T) in the cable Take CD = 0.2.

C) Prove that the head loss due to friction is equal one third of the total head at inlet for
maximum power transmission through pipes.

6. A) What do you mean by separation of boundary layer ? What are the different methods of
preventing the separation of boundary layers ?

B) A pipe system consists of three pipes connected in series


i) 1800m long, 50 cm in diameter
ii) 1200m long, 40 cm in diameter
iii) 600m long, 30 cm in diameter.
Assume coefficient of friction is same for all pipes (Neglect minor losses)
i) Determine the equivalent length of pipe diameter 40 cm
ii) What is the equivalent size of a pipe 3600m long.

C) What do you mean by terminal velocity of a body ? What is the relation between weight of
the body, drag force and buoyant force when the body acquires terminal velocity ?

7. A) Water from a main canal (reservoir) is siphoned to a branch canal (reservoir) over an
embankment by means of wrought iron pipe of 9 cm diameter. The length of pipeline up to
the summit is 25 m and the total length is 65 m. The water surface level in the branch canal
is 10 m below that of the main canal. What is the maximum height of the summit above the
water level in the main canal so that the water pressure at summit may not fall below
19.62 KPa absolute ? The barometer reading being 10 m of water. Take ? = 0.03. (Neglect
minor losses).

B) A pipe of diameter 1.5 m is required to transport an oil of sp gr 0.90 and viscosity 0.03
poise at the rate of 3000 liter/sec. Tests were conducted on 15 cm diameter pipe using
water at 25C. Find the velocity and rate of flow in the model. Take viscosity of water as
0.01 poise.

C) What is CFD ? Explain solution procedure for CFD analysis.

_____________________

Set A

SLR-BB 87

*SLRBB87*
S

S.E. (Mechanical) (Part II) Examination, 2014


THEORY OF MACHINES I (New)
Day and Date : Wednesday, 26-11-2014

Max. Marks : 100

Time : 3.00 p.m. to 6.00 p.m.


Instructions : 1) Figures to right indicate full marks.
2) Scratching, rubbing of answers are not allowed.
3) Q. No. 1 is compulsory. It should be solved in first 30 minutes in
Answer book Page No. 3.
4) Answer MCQ/Objective type questions on Page No. 3 only. Dont
forget to mention, Q.P. Set (A/B/C/D) on Top of Page.
MCQ/Objective Type Questions
Duration : 30 Minutes

Marks : 20

1. Objective questions :

20

I) Match the pairs :


List (I)

List (II)

A) Friction clutches

P) Use of springs

B) Centrifugal clutch

Q) Thick layer of lubricant

C) Boundary friction

R) Thin layer of lubricant

D) Film friction

S) Efficiency less than 50%

E) Self locking screws

T) Uniform wear theory preferred

II) Multiple choice questions each question carries 2 marks :

A) ______________ are the examples of spherical pair.


a) Ball and socket joint

b) Attachment of car mirror

c) Pen stand

d) Toothed gearing

B) ____________ acceleration is perpendicular to the velocity of the particle at any instant.


a) Centripetal

b) Radial

c) Tangential

d) None of above
P.T.O.

SLR-BB 87

*SLRBB87*

-2-

C) Following are the graphical methods to determine velocity and acceleration in


mechanism.
a) Kleins construction

b) Ritterhauss construction

c) Bennets construction

d) None of above

D) ____________ are dead weight centrifugal governor.


a) Porter

b) Proell

c) Pickering

d) Watt

III) Single correct answer type, each question carries 1 mark.

A) The magnitude of linear velocity of a point B on a link AB relative to point A is


a) w.AB

b) w.(AB)2

c) w2 (AB)

d) (wAB)2

B) Which of the following mechanism is used to enlarge or reduce the size of drawing ?
a) Grasshopper mechanism

b) Watt mechanism

c) Pantograph

d) None of these

C) Offset is provided to cam follower mechanism to


a) Minimize the side thrust

b) Accelerate

c) Avoid jerk

d) None of these

D) The brakes commonly used in railway trains is


a) Shoe brake

b) Band brake

c) Band and block brake

d) Internal expanding brake

E) A hartnell governor is a ______________ governor.


a) Pendulum type

b) Spring loaded

c) Pickering

d) Hartnell

F) Which of the following governor is used to drive a gramophone ?


a) Watt

b) Porter

c) Pickering

d) Hartnell

G) Which of the following is spring controlled governor ?


a) Hartnell

b) Hartung

c) Pickering

d) All of above

______________

Set A

*SLRBB87*
S

-3-

SLR-BB 87

S.E. (Mechanical) (Part II) Examination, 2014


THEORY OF MACHINES I (New)
Day and Date : Wednesday, 26-11-2014
Time : 3.00 p.m. to 6.00 p.m.
Instructions : 1)
2)
3)
4)

Marks : 80

Attempt any two questions from each Section.


Make suitable assumptions if required and state them clearly.
Neat diagrams must be drawn wherever necessary.
Figures to right indicate full marks.
SECTION I

2. a) With neat sketch explain any two inversions of single slider crank mechanism.

b) In the toggle mechanism shown in figure (II-b), the slider D is constrained to move on a
horizontal path. The crank OA is rotating in the anticlockwise direction at a speed of
180 rpm increasing at the rate of 50 rad/sec2. The dimensions of various links are as
follows, OA = 180 mm, CB = 240 mm, AB = 360 mm, BD = 540 mm. For the given
configuration find
i) Velocity of slider D and angular velocity of BD.
ii) Acceleration of slider D and angular acceleration of BD.

Figure (2-b)

12

Set A

SLR-BB 87

-4-

*SLRBB87*

3. a) Explain Peaucellier mechanism with neat sketch.

b) In a slider crank mechanism, the length of the crank and connecting rod are 150 mm and
600 mm respectively. The crank position is 60 from IDC. The crankshaft speed is
450 rpm clockwise. Using Kleins construction determine
i) Velocity and acceleration of the slider.
ii) Velocity and acceleration of point D on the connecting rod which is 150 mm from
crank C.
iii) Angular velocity and angular acceleration of the connecting rod.

c) Two shafts are connected by a universal joint. The driving shaft rotates at a uniform speed
of 1200 rpm. Determine the greatest possible angle between the shaft axes so that the
total fluctuation of speed does not exceed 100 rpm. Also calculate the maximum and
minimum speeds of the driven shaft.
4. a) Explain Pantograph with neat sketch.

8
4

b) Derive an expression for forces acting in single slider crank mechanism.

c) A vertical double acting steam engine has a cylinder 300 mm diameter and 450 mm stroke
and runs at 200 rpm. The reciprocating parts has a mass of 225 kg and the piston rod is
50 mm diameter. The connecting rod is 1.2 m long. When the crank has turned through
125 from top dead centre, the steam pressure above the piston is 30 kN/m2 and below
the piston is 1.5 kN/m2. Calculate the effective turning moment on the crank shaft.

SECTION II
5. a) Derive an expression for torque transmitted in case of single plate clutch using uniform
pressure and uniform wear theory.

b) A cam is to be designed for a knife edge follower with the following data
1) Cam lift = 40 mm during 90 of cam rotation with simple Harmonic motion.
2) Dwell for next 30.
3) During the next 60 of cam rotation the follower returns to its original position with
simple Harmonic motion.
4) Dwell during remaining 180. Draw the profile of the cam when the line of stroke is
offset 20 mm from the axis of cam shaft. The radius of base circle of the cam is 40 mm.
6. a) Draw displacement, velocity and acceleration diagram for a follower with uniform
acceleration and uniform retardation.

12
4

b) A thrust shaft of a ship has 6 collars of 600 mm external diameter and 300 mm internal dia.
The total thrust from the propeller is 100 kN. If coefficient of friction is 0.2 and speed of the
engine 90 rpm. Find power absorbed in friction at the thrust block assuming
i) Uniform pressure
ii) Uniform wear.

Set A

*SLRBB87*

-5-

SLR-BB 87

c) In a spring loaded governor of Hartnell type, the mass of each ball is 1 kg, length of vertical
arm of the bell crank lever is 100 mm and that of the horizontal arm is 50 mm. The distance
of fulcrum of each bell crank lever is 80 mm from the axis of rotation of the governor. The
extreme radii of rotation of the balls are 75 mm and 112.5 mm. The maximum equilibrium
speed is 5 percent greater than the minimum equilibrium speed which is 360 rpm. Find
neglecting obliquity of arms, initial compression of the spring and equilibrium speed
corresponding to the radius of rotation of 100 mm.
7. a) Derive an expression for braking torque in case of single block brake when the line of
action of braking force passes through a distance a above the fulcrum O.

8
8

b) What is Isochronous Governor ? Check the condition for Porter and Hartnell Governor.

c) In a winch, the rope supports a load W and is wound round a barrel 450 mm diameter. A
differential band brake acts on a drum 800 mm diameter which is keyed to the same shaft
as the barrel. The two ends of the bands are attached to pins on opposite sides of the
fulcrum of the brake lever and at distances of 25 mm and 100 mm from the fulcrum. The
angle of lap of the brake band is 250 and coefficient of friction is 0.25. What is the maximum
load W which can be supported by the brake when a force of 750 N is applied to lever at
a distance of 3000 mm from the fulcrum ?

Figure (7-c)
_____________________

Set A

Set A

SLR-BB 88

*SLRBB88*
Seat
No.

Set

S.E. (Mech.) (Part II) (New) Examination, 2014


MANUFACTURING PROCESSES
Day and Date : Thursday, 27-11-2014
Time : 3.00 p.m. to 6.00 p.m.

Max. Marks : 100

Instructions : 1) Q. No. 1 is compulsory. It should be solved in first 30 minutes in


Answer book Page No. 3.
2) Answer MCQ/Objective type questions on Page No. 3 only.
Dont forget to mention, Q.P. Set (A/B/C/D) on Top of Page.
MCQ/Objective Type Questions
Duration : 30 Minutes

Marks : 20

Type 1 :
1. Match the appropriate pairs :
A) Match plate patterns
B) Large casting symmetrical shapes
C) Gated pattern

2. Match the appropriate pairs :


A) Butt joint
B) T joint
C) Lap joint

3
p)
q)
r)
s)

Large castings
Small casting
Sweep patterns
Piston ring of I.C. engine
3

p)
q)
r)
s)

Longitudinal shear
Strongest brazing joint
Up to 30 mm material thickness
Up to 25 mm material thickness

Type 2 : Attempt any one of the following (3 and 4) :


3.

i) Fettling operations are essentially carried on casting.


ii) Use of forehand welding technique for thinner metals (i.e. less than 5 mm)
A) Statement (i) is correct (ii) is incorrect B) Statement (ii) is correct (i) is incorrect
C) Both statement (i) and (ii) are correct D) Both statement (i) and (ii) are incorrect

4.

i) TIG welding uses a permanent non-consumable tungsten electrode.


ii) MIG uses consumable continuous coil electrode of same chemical composition.
A) Statement (i) is correct (ii) is incorrect B) Statement (ii) is correct (i) is incorrect
C) Both statement (i) and (ii) are correct D) Both statement (i) and (ii) are incorrect
P.T.O.

SLR-BB 88

*SLRBB88*

-2-

Type 3 : Attempt any four of the following (5 to 9) :


5. Arc welding process include
A) Metal arc welding
C) Resistance welding

(2 Marks each)

B) Carbon arc welding


D) Projection welding

6. The basic features common to various casting are


A) Pattern
B) Mould
C) Pouring molten metal
D) None of above
7. Which of the following additives are added in green sand ?
A) Starch
B) Saw dust
C) Bentonite
D) Coal dust
8. Which of the following are forged products ?
A) Spanners
B) Connecting rod
C) Lathe bed
D) I sections
9. Which of the following are rolling products ?
A) Crane hook
B) Crank shaft
C) L sections

D) Rail sections

Type 4 : Attempt any four of the following (10 to 14) :

(1 Mark each)

10. When melting cast iron in cupola, the flux used is


A) Lime stone
B) N2
C) Helium

D) Chlorine

11. For investment casting the pattern material used is


A) Wax
B) Metal
C) Wood

D) Plastic

12. The highest shrinkage allowance is provided for casting the material of
A) Gray cast iron B) Brass
C) Al alloy
D) Manganese steel
13. Projection welding is
A) Continuous spot welding process B) Multi spot welding process
C) Used to form rectangle
D) Something else
14. The average composition of silver solder is
A) 60% Silver, 30% Cu, 10% Zn
B) 30% Silver, 60% Cu, 10% Zn
C) 30% Silver, 40% Cu, 30% Zn
D) 25% Silver, 70% Cu, 5% Zn
______________

Set A

*SLRBB88*

-3-

SLR-BB 88

Seat
No.

S.E. (Mech.) (Part II) (New) Examination, 2014


MANUFACTURING PROCESSES
Day and Date : Thursday, 27-11-2014

Marks : 80

Time : 3.00 p.m. to 6.00 p.m.


Instructions : 1) Attempt any three questions from Section I and three from
Section II. Each Section carries 40 marks.
2) Draw neat sketches wherever necessary.
SECTION I
1. a) What is casting process ? Explain basic steps in casting process.

b) Enlist different elements of gating system. Explain various types of gates used in
gating system.

2. a) Explain different properties that are generally required in moulding materials.

b) Explain with neat sketch cold chamber die casting process. Write its advantages,
limitations with applications.
3. a) Explain the indirect electric are furnace with neat sketch.

7
7

b) What is blow moulding ? Explain extrusion blow moulding process with neat
sketch.
4. Write a note on (any 3) :

6
14

a) Need for fettling


b) Computer application in foundry process
c) Loose piece pattern
d) Draft allowance in pattern making
e) Turn over method of Green sand moulding.

Set A

SLR-BB 88

-4-

*SLRBB88*

SECTION II
5. a) Differentiate clearly between hot and cold rolling of metals.
b) Explain basic forging operations.
6. a) Classify extrusion process. Explain cold extrusion forging process with neat
sketch.

7
6

b) Explain methods of wire drawing.

c) Explain tube drawing process with neat sketch.

7. a) Draw self explanatory sketches of following :


i) Types of oxy-acetylene flames
ii) Resistance welding.

b) Explain shielded metal arc welding in detail.


8. Write a note on (any 3) :

7
14

a) Difference between low pressure and high pressure gas welding process.
b) Hydrostatic extrusion.
c) Cold heading.
d) Manufacturing sequence of any one rolled product.
e) Classification of forming processes.
_____________________

Set A

SLR-BB 89

*SLRBB89*
S

S.E. (Mech.) (Part II) (New) Examination, 2014


NUMERICAL METHODS

Day and Date : Friday, 28-11-2014


Time : 3.00 p.m. to 6.00 p.m.
Instructions :

1) Figures to the right indicate full marks.


2) Use of scientific calculator is allowed.
3) Q. No. 1 is compulsory. It should be solved in first 30 minutes in
Answer Book Page No. 3. Each question carries one mark.
4) Answer MCQ/Objective type questions on Page No. 3 only. Dont
forget to mention, Q.P. Set (A/B/C/D) on Top of Page.
MCQ/Objective Type Questions

Duration : 30 Minutes

1. 1) The Newtons iterative formula for finding


a)

d)


where N is a positive real number is


n

Marks : 20

c)

b)

Max. Marks : 100

2) Using Bisection method, a second approximation to the root of equation x3 4x 9 = 0


between 2 and 3 is
a) 2.25
b) 2.5
c) 2.75
d) 2.625
3) For finding root of equation f(x) = 0. The Newtons Raphson method fails, if
is negative
b)
is positive c)
is zero
d) never fails
a)
4) When Gauss Jordan method is used to solve set of equation AX = B, matrix A is
transformed to
a) Lower triangular matrix
b) Diagonal matrix
c) Singular matrix
d) Upper triangular matrix
5) Power method is used to find
a) Smallest eigen value of the matrix
b) Largest eigen value of the matrix
c) Only integer valued eigen value
d) A roof of algebraic equation
6) Which of the following is iterative method ?
a) Gauss Elimination b) Gauss Jordan c) Gauss Jacobis d) Crouts method
7) The Newtons divided difference formula is used to find polynomial, if the given values of
x are
a) Equally-spaced b) Unequally spaced c) Odd numbers
d) Even numbers
8) To fit the straight line y = ax + b to n observations, the normal equations are
b)
a)
c)
d) None of these


9) Given the table :


N

The quadratic polynomial by Lagranges formula is


O

a)

b)
N

c)

!
N

d)

!
N

P.T.O.

SLR-BB 89

*SLRBB89*

-2-

10) The line of regression of x on y is given by


b)
a)
c)
d) None of these
N

>

>

>

>

11) If I1 and I2 represents approximate value of I =

in the Rombergs method then


N

I=
a)

b)

c)

d)




12) In Newtons cotes formula if f(x) is interpolated at equally spaced nodes by a polynomials
of degree two then it represents
H

a) Trapezoidal rule

b) Simpsons

J
@

rule


c) Simpsons

rule
!

d) None of these

&

13) Which of the following is Gaussian two point formula ?




a)

b)




c)

d) None of these

14) Ordinary differential equation involves


a) Only one independent variable
c) More than two independent variables
15) Given initial value problem
(n + 1)th iteration is
a) yn + 1 = yn +
c) yn + 1 = yn +


y

with y(x0) = y0. The Runge Kutta formula for

(k1 + k2 + k3 + k4)

b) yn + 1 = yn +

(2k1 + k2 + k3 + 2k4)

d) None of the above

16) The Laplace equation uxx + uyy = 0 is


a) Parabolic
b) Hyperbolic
17) The differential equation





d) Circular

b) Parabolic
d) None of above

18) A method that does not restrict


k

c) Elliptic

(k1 + 2k2 + 2k3 + k4)

a) Elliptic
c) Hyperbolic

is
K

b) Two independent variables


d) None of these

and also reduces the volume of calculation where

= mesh ratio parameter is

a) Crank-Nicolson

b) Bender Schmidt c) Libman

d) None of these

19) In solving a ODE by Runge-Kutta method, if y(0) = 1, h = 0.1, k1 = 0.1, k2 = 0.1150,


k3 = 0.1171 and k4 = 0.1359 then the value of y (0.1) = ___________
a) 0.11668
b) 0.16186
c) 1.11668
d) 0.16186
20) For the PDE Auxx + Buxy + Cuyy + Dux + Euy + Fu = G where A, B, C, D, E, F, G are any
functions of x or constant then the afore mentioned of PDE is Elliptic if
a) B2 4AC > 0
b) B2 4AC = 0
2
c) B 4AC < 0
d) None of these
______________

Set A

*SLRBB89*
S

SLR-BB 89

-3-

S.E. (Mech.) (Part II) (New) Examination, 2014


NUMERICAL METHODS
Day and Date : Friday, 28-11-2014
Time : 3.00 p.m. to 6.00 p.m.

Marks : 80

Instructions : 1) Attempt any three questions from each Section.


2) Figures to the right indicate full marks.
3) Use of scientific calculator is allowed.
SECTION I
2. a) Find the positive root of x4 x = 10 correct to three decimal places, using Newton-Raphson
method. Take x0 = 2.

b) Use the method of false position, to find the fourth root of 32 correct to three decimal
places. (Take four iterations only)

c) Solve the system of non-linear equations


y0 = 1.8.

x2

+ y = 11,

y2

+ x = 7 taking x0 = 3.5 and




3. a) Find the largest eigen value and the corresponding eigen vector of the matrix

using power method.


Take [1, 0, 0]T as initial eigen vector. (Take five iterations only)
b) Using Gauss elimination method, solve the equations
x + 2y + 3z u = 10
2x + 3y 3z u= 1
2x y + 2z + 3u = 7
3x + 2y 4z + 3u = 2.

c) Apply factorization method to solve the equations :


3x + 2y + 7z = 4
2x + 3y + z = 5
3x + 4y + z = 7
Given that












!


!







!
!










"

$


&










i.e. A = LU.

#


4
OR

Set A

SLR-BB 89

*SLRBB89*

-4-

c) Solve the equations :


27x + 6y z = 85
x + y + 54z = 110
6x + 15y + 2z = 72
by Gauss-Seidal method. (Perform four iterations only)

4. a) To find correlation coefficient of a bivariate data following results were obtained :


n

At the time of checking it was discovered that two pairs of (x, y), (8, 12) and (6, 8) were
wrongly recorded as (6, 14) and (8, 6).
Find the correct correlation.

b) The following results were obtained from marks in physics and chemistry in an examination
Mark in Physics (x)

Marks in Chemistry (y)

Mean

47.5

39.5

S. D.

16.8

10.8

(Standard Deviation)
If r = 0.95, find both the regression equations. Also estimate the value of y for x = 30.

5. a) If P is the pull required to lift a load W by means of a pulley block, find a linear law of the
form P = mW + C connecting P and W, using the following data.
P:

12

15

21

25

W:

50

70

100

120

where P and W are taken in kg-wt. Compute P when W = 150 kg.

b) Given the values :


x:
f(x) :

11

13

17

150

392

1452

2366

5202

evaluate f(g), using Newtons divided difference formula.

c) Find the normal equations for the power curve y = axb.

SECTION II
1

6. a) Evaluate

by Weddles Rule. After finding true value of the integral

estimate the error.

b) Evaluate

by using Rombers method correct to 4 decimal places.




Set A

*SLRBB89*

SLR-BB 89

-5-

7. a) Using Runge Kutta method of fourth order solve


@

with y (0) = 1 at x = 0.2, 0.4. 6

b) Using Shooting method, solve the boundary value problem :


d

and y(1) = 1.17.

8. a) Using Crank-Nicholson method solve




subject to u(x, 0) = 0, u(0, t) = 0 and

J


u(1, t) = t by taking h = 0.5, k =

.


&

b) Solve uxx +uyy = 0 over the square mesh of side 4 units satisfying following boundary
conditions.
i) u(0, y) = 0

for 0


ii) u (4, y) = 12 + y for 0


iii) u(x, 0) = 3x for 0
iv) u(x, 4) =

x2

for 0

y


x


x


4


4


4.


9. a) Solve uxx 2ut = 0. Given u(0, t) = 0, u(4, t) = 0, u(x, 0) = x(4 x). Assume h = k = 1.
Find the value of u up to t = 5, using Bender-Schmidt method.
b) Solve


over the square mesh with sides x = 0, y = 0, x = 3, y = 3

with u = 0 on the boundary and mesh length 1 unit.

OR
b) Using Crank-Nicholsons scheme solve uxx = 16 ut 0 < x < 1, t > 0 given
u (x, 0) = 0 = u (0, t), u(1, t) = 100 t.

_____________________

Set A

Set A

SLR-BB 90

*SLRBB90*
Seat
No.

Set

S.E. (Mechanical) (Part II) (New) Examination, 2014


ELECTRICAL AND ELECTRONICS TECHNOLOGY
Day and Date : Saturday, 29-11-2014
Time : 3.00 p.m. to 6.00 p.m.

Max. Marks : 100

Instructions : 1) Q. No. 1 is compulsory. It should be solved in first 30 minutes


in Answer Book Page No. 3. Each question carries one mark.
2) Answer MCQ/Objective type questions on Page No. 3 only.
Dont forget to mention, Q.P. Set (A/B/C/D) on Top of Page.
MCQ/Objective Type Questions
Duration : 30 Minutes

Marks : 20

1. MCQ Questions :
20
1) Which of the following motor is preferred for blowers ?
a) wound rotor induction motor
b) squirrel cage induction motor
c) dc shunt motor
d) dc series motor
2) Motors preferred for rolling mill drive is
a) dc motors
b) ac slip ring motors with speed control
c) any of the above
d) none of the above
3) If a single phase motor runs hot, the probable cause may be
a) overload
b) low voltage
c) high voltage
d) any of the above
4) In arc heating, the electrodes used are made up of
a) carbon
b) graphite
c) tungsten
d) aluminium
5) In a series motor, the field flux is
a) proportional to the armature current
b) is nearly constant
c) inversely proportional to the armature current
d) proportional to the square of the armature current
6) The output indicated on the name plate of any motor is always the
a) gross power
b) power drawn in volt amperes
c) power drawn in watts
d) output power at the shaft
7) The direction of rotation of dc shunt motor can be reversed by interchanging
a) The supply terminals
b) The field terminals
c) The armature terminals only
d) Either field or armature terminals
P.T.O.

SLR-BB 90

*SLRBB90*

-2-

8) A permanent split single phase capacitor motor does not have


a) centrifugal switch
b) starting winding
c) squirrel cage rotor
d) high power factor
9) The rotational speed of given stepper motor is determined solely by
a) shaft load
b) step pulse frequency
c) polarity of stator current
d) magnitude of stator current
10) One of the basic requirements of servo motor is that it must produce high
torque at all
a) loads
b) frequencies c) speeds
d) voltages
11) Which of the following Flip-Flop is used as latch ?
a) J-K Flip Flop
b) S-R Flip Flop c) D Flip Flop
d) T Flip Flop
12) In op-amp block diagram level shifting stage works such as
a) o/p stage
b) collector follower
c) i/p stage
d) emitter follower
13) The average of the currents that flow into the inverting and non inverting
input terminal of op-amp
a) input offset current
b) input bias current
c) input current range
d) input resistance
14) The Maximum rate of change of output voltage per unit time is called
_________
a) common mode rejection ratio
b) supply voltage rejection ratio
c) slew rate
d) large signal voltage gain
15) The microcontroller internal RAM of
a) 32 bytes
b) 128 bytes

c) 16 bytes

d) 8 bytes

16) The 8085 microprocessor has _______ registers.


a) 6
b) 8
c) 4
17) The data bus is _________
a) Unidirectional b) Bidirectional c) Parallel

d) Serial

18) Microcontroller has _________ interrupts.


a) 5
b) 6
c) 7

d) 8

d) 2

19) Inverting amplifier using op-amp has voltage gain of __________


a) 1 + (RF/R1)
b) 1 (RF/R1) c) (RF/R1)
d) (RF/R1)
20) The register use to sequence the execution of the instruction
a) Stack pointer
b) Program counter
c) ALU
d) Accumulator
______________
Set A

*SLRBB90*

-3-

SLR-BB 90

Seat
No.

S.E. (Mechanical) (Part II) (New) Examination, 2014


ELECTRICAL AND ELECTRONICS TECHNOLOGY
Day and Date : Saturday, 29-11-2014
Time :3.00 p.m. to 6.00 p.m.

Marks : 80

Instructions : 1) Two Sections should be written in separate answer books.


2) Use non-Programmable calculator.
SECTION I
2. Solve any 4 questions :
(45=20)
a) Write constructional details and working principle of d.c. motor.
b) A dc machine has following details : poles = 4, flux per pole = 58.7 mwb,
armature winding = wave, no. of slots = 48, conductors per slot = 3, armature
resistance = 0.2 ohm, when the machine was connected to 400 V supply, it
run as motor at speed of 1050 r.p.m. Calculate the value of back emf and
hence find the value of the armature torque developed by the motor.
c) Derive the torque equation of 3 phase induction motor under running condition.
d) A 3 ph 50 Hz, 4 pole, wound rotor IM has a rotor standstill emf per phase of
115 V. If the motor is running at 1440 rpm, calculate for this speed 1. The slip
2. The frequency of the rotor induced emf 3. Value of the rotor induced emf
per phase.
e) With neat sketch explain the working of single phase induction motor.
f) Explain clearly high frequency eddy current heating and its applications.
3. Solve any two :
(210=20)
a) Write properties of resistance heating material. With neat diagram explain
resistance heating.
100 kg of tin to be smelt during an hour in a smelting furnace. Determine
suitable rating of furnace, if smelting temperature of tin is 235C, specific
heat 0.055, Latent heat of liquidification 13.3 Kcal/Kg. Take the initial temp. of
metal as 35C.
b) Write note on :
1) Servo Motor
2) Brushless D.C. Motor
c) List the characteristics of D.C. Motor. And explain it for shunt motor. Write
its applications.
Set A

SLR-BB 90

-4-

*SLRBB90*

SECTION II
4. Attempt any four :

(45=20)

A) Explain universal shift register with diagram.


B) Explain op-amp as an integrator.
C) What is ROM and what are the types of ROM ?
D) Explain the Master Slave flip flop.
E) Explain types of addressing modes of microcontroller.
5. Attempt any two :

(210=20)

A) Explain the architecture of microprocessor.


B) Describe the various interrupt of microcontroller.
C) Explain op-amp as adder and substractor with example.
_____________________

Set A

SLR-BB 91

*SLRBB91*
Seat
No.

Set

T.E. (Mechanical) (Part I) Examination, 2014


THEORY OF MACHINES II (Old)
Day and Date : Wednesday, 3-12-2014
Time : 3.00 p.m. to 6.00 p.m.
Instructions :

Max. Marks : 100

1) Assume suitable data if necessary.


2) Q. No. 1 is compulsory. It should be solved in first 30 minutes in
Answer book Page No. 3. Each question carries one mark.
3) Answer MCQ/Objective type questions on Page No. 3 only. Dont
forget to mention, Q.P. Set (A/B/C/D) on Top of Page.
MCQ/Objective Type Questions

Duration : 30 Minutes

Marks : 20

1. Solve following MCQs :


(201=20)
1) The condition of correct gearing is
a) pitch line velocities of teeth be same
b) radius of curvature of two profiles be same
c) common normal to the pitch surface cuts the line of centres at a fixed point
d) none of above
2) When the addenda on pinion and wheel is such that the path of approach and path of
recess are half of their maximum possible values then length of path of contact is given
by
a)

3)
4)
5)

6)

7)

(r + R ) sin

b)

(r + R ) cos

c)

(r + R) tan

d) none of these
2
2
2
Interference occurs in case of
a) cycloidal profile teeth
b) involute profile teeth
c) in both of them
d) in none of them
A differential uses following gear train
a) simple
b) epicyclic
c) reverted
d) compound
In a simple gear train, there is odd number of idlers. The direction of rotation of the
driver and driven gear will be
a) opposite
b) same
c) depends on number of teeth on gear
d) none of above
In a simple gear train of three gear wheels, the first gear has 20 teeth second gear 10
teeth and the third gear 40 teeth. The speed ratio of third gear to first gear and direction
are
1
1
a)
and opposite direction
b)
and same direction
2
2
c) 2 and opposite direction
d) 2 and same direction
The ratio of the maximum fluctuation of energy to the _____________ is called coefficient
of fluctuation of energy.
a) minimum fluctuation of energy
b) work done per cycle
c) maximum speed
d) none of above

8) The maximum fluctuation of energy in a fly wheel is equal to


a) Iw (w1 w2)
b) Iw2 Cs
c) 2ECs

d) all of above
P.T.O.

SLR-BB 91

-2-

*SLRBB91*

9) The magnitude of gyroscopic couple applied to a disc of moment of inertia I, spinning with
an angular velocity w, and having an angular velocity of precession wp is
10)

11)

12)

13)

b) Iw 2w p
c) Iw w p2
a) I2w wp
d) Iww p
The engine of an aeroplane rotates in clockwise direction when seen from the tail end the
aeroplane takes a turn to the left. The effect of gyroscopic couple on the aeroplane will be
a) to raise the nose and dip the tail
b) to dip the nose and raise the tail
c) to raise nose and tail
d) to dip the nose and tail
For complete balancing of the reciprocating parts the condition arrived at is
a) primary force polygon must close
b) secondary force polygon must close
c) primary and secondary couple polygon must close
d) all of the above
In a locomotive the ratio of length of connecting rod to the crank radius is kept very large
in order to
a) start the locomotive quickly
b) minimize the effect of primary forces
c) minimize the effect of secondary forces
d) none of these
The swaying couple is due to the
a) primary unbalanced force
b) secondary unbalanced force
c) two cylinders of locomotive
d) partial balancing

14) For the spring mass system shown in fig. 1 the frequency of vibration is N what will be the
frequency when one more similar spring is added in series as shown in fig. 2.

Fig. 1
a) N/2

15)
16)
17)
18)

Fig. 2

b) N / 2
c) 2 / N
d) 2N
At nodal point in a shaft the amplitude of torsional vibration is
a) zero
b) minimum
c) maximum
d) none of these
In three rotor system if extreme two rotors on one side are rotating in one direction
______________ node obtained.
a) single
b) double
c) triple
d) half
The particles of a body move ____________ its axis in torsional vibrations.
a) in a circle about
b) parallel to
c) perpendicular to d) away from
The ratio of the amplitude of the steady-state response of forced vibrations to the static
deflection under the action of static force is known as
a) damping ratio
b) damping factor
c) transmissibility
d) magnification factor

19) If w/wn is more than 2 in a vibration isolation system, then for all value of damping
factor, the transmissibility is
b) more than 2
c) less than unity
d) more than unity
2
20) In free vibrations velocity vector leads the displacement vector by
a)
b) /2
c) /3
d) 2 /3
______________
a) less than

Set A

*SLRBB91*

-3-

SLR-BB 91

Seat
No.

T.E. (Mechanical) (Part I) Examination, 2014


THEORY OF MACHINES II (Old)
Day and Date : Wednesday, 3-12-2014
Time : 3.00 p.m. to 6.00 p.m.

Marks : 80

Instructions : 1) Solve any two questions from Section I.


2) Solve any two questions from Section II.
3) Assume suitable data if necessary.
SECTION I
2. a) Enlist difference between involute and cycloidal profile of teeth of gear.
b) The following data relate to a pair of 20 involute gears in mesh : module = 6mm, number
of teeth on pinion = 17, number of teeth on gear = 4 g. Addenda on pinion and gear
wheel = 1 module, find :
i) The number of pairs of teeth in contact
ii) The angle turned through by the pinion and the gear wheel when one pair of teeth in
contact and
iii) The ratio of sliding to rolling motion when the tip of a tooth on the larger wheel (a) is just
making contact (b) is just leaving contact with mating tooth (c) is at the pitch point.
c) Define the terms :
i) Turning moment diagram
ii) Maximum fluctuation of energy
iii) Coefficient of fluctuation of energy
iv) Coefficient of fluctuation of speed.
3. a) Explain with neat sketch sun and planet gear train.

12

4
4

Figure 3. a)
b) An epicyclic train of gears is arranged as shown in figure 3 a). How many revolutions
does the arm, to which the pinions B and C are attached, make :
1) When A makes on revolution clockwise and D makes half revolution anticlockwise.
2) When A makes one revolution and D is stationary the number of teeth on the gears A
and D are 40 and 90 respectively.
c) Explain gyroscopic principle for aircraft.

12
4

Set A

SLR-BB 91

-4-

*SLRBB91*

4. a) The turning moment diagram for a multicylinder engine has been drawn to scale 1mm = 600 Nm
vertically and 1mm = 3 horizontally. The intercepted areas between the output curve and
the mean resistance line, taken in order from one end, are as follows :
+52, 124, + 92, 140, +85, 72 and +107 mm2, when the engine is running at a speed
of 600 rpm. If the total fluctuation of speed is not to exceed 1.5% of the mean, find the
necessary mass of flywheel of radius 0.5 M.

10

b) Find the angle of inclination with respect to the vertical of a two wheeler negotiating a turn.
Given : combined mass of vehicle with its rider 250 kg; moment of inertia of the engine
flywheel 0.3 kgm2; moment of inertia of each road wheel 1 kgm2; speed of engine flywheel
5 times that of road wheels and in the same direction; height of centre of gravity of rider
with vehicle 0.6 m; two wheeler speed 90 km/h; wheel radius 300 mm; radius of turn 50 m.

10

SECTION II
5. a) A four cylinder inline engine running at 2000 rpm is having crank and connecting rod lengths of
60 mm and 240 mm respectively. The mass of reciprocating parts of each cylinder is 2 kg. The
cylinders are spaced 160 mm apart and the cranks appear at 90 interval in an end view. If the
firing order of the engine is 1 4 2 3. Determine (i) the unbalanced primary and
secondary force (ii) the unbalanced primary and secondary couples.
b) Derive frequency equation for undamped free cantilever beam having mass at its free end.

14
6

6. a) Explain various types of vibrations and discuss with examples.

b) Define logarithmic decrement and derive expression for logarithmic decrement.

c) A disc of torsional pendulum has a mass moment of inertia of 0.06 kgm2. The brass shaft
attached to it is of 100 mm diameter and 400 mm long. When the pendulum is vibrating the
observed amplitudes on the same side of the rest position for successive cycles are 9, 6
and 4. Find (i) logarithmic decrement (ii) damping torque at unit velocity (iii) time period of
vibration (iv) what would the frequency be if the disc is removed from viscous fluid ?
Assume the modulus of rigidity as 4.4 1010 N/m2.

10

7. a) Derive the expression for amplitude of steady state forced vibration having an excitation
force F0sin t.
b) Explain the term whirling speed or critical speed.

10
2

c) Two identical rotors are attached to the two ends of a stepped shaft. Each rotor weighs
450 kg and has a radius of gyration of 0.38 m. The diameters of the shaft is 750 mm for
first 0.25 m length, 100 mm for next 0.1 m length and 87.5 mm for the remaining length. If
the total length of the shaft is 0.6 m. Find
i) Equivalent length of shaft assuming 87.5 mm as equivalent diameter.
ii) Position of node from left side rotor.

___________________

Set A

SLR-BB 92

*SLRBB92*
Seat
No.

Set
T.E. (Mechanical) (Part I) Examination, 2014
FLUID MACHINERY AND FLUID POWER (Old)

Day and Date : Friday, 5-12-2014

Max. Marks : 100

Time : 3.00 p.m. to 6.00 p.m.


N.B. : 1) Q. No. 1 is compulsory. It should be solved in first 30 minutes in Answer
Book Page No. 3.
2) Draw neat sketches wherever necessary.
3) Answer MCQ/Objective type questions on Page No. 3 only. Dont
forget to mention, Q.P. Set (A/B/C/D) on Top of Page.
MCQ/Objective Type Questions
Duration : 30 Minutes

Marks : 20

1. 1 and 2 statements in column I may have a/b/c matches with statements in column II.
(Each question 3 marks).
1) Column I

Column II

a) Pelton turbine

p) Reaction type

b) Francis turbine

q) Impulse type

c) Kaplan Turbine

r) Vanes adjustable

2) Column I
Pipe materials

Column II
Pressure Rating in bars

a) Copper

p) 125

b) Aluminium

q) 250

c) Brass

r) 200

Question 3 and 4 are multiple correct type questions :

(2 marks each)

3) Models are of _____________ size when compared to actual size.


a) Reduced

b) Enlarged

c) Same

d) None of the above

4) Which of the following are positive displacement pumps ?


a) Centrifugal pump

b) Gear pump

c) Vane group

d) Gerton pump

P.T.O.

SLR-BB 92

*SLRBB92*

-2-

Question 5 and 14 are single correct answer type questions :

(1 mark each)

5) For high head and low discharge, the suitable turbine is


a) Pelton
b) Francis
c) Kaplan

d) None of the above

6) To discharge large quantity of liquid by multi stage centrifugal pump, the impellers are connected
_____________
a) in parallel

b) in series

c) in parallel or in series

d) none of the above

7) A turbine is a device which converts


a) Hydraulic energy into mechanical energy
b) Mechanical energy into hydraulic energy
c) Electrical energy into mechanical energy
d) Kinetic energy into pressure energy
8) In case of impulse steam turbine
a) There is enthalpy drop in fixed and moving blades
b) There is enthalpy drop in moving blades only
c) There is enthalpy drop in nozzles
d) None of the above
9) For Parsons reaction steam tubine, degree of reaction is
a) 75%
b) 100%
c) 50%

d) 60%

10) Hydraulic accumulator is a device used for


a) Lifting heavy weights
b) Storing the energy of fluid in the form of pressure energy
c) Increasing the pressure intensity of fluid
d) None of the above
11) _____________ is suitable for high pressure and low volume requirement.
a) Vane pump
c) Axial flow pump

b) Centrifugal pump
d) None of the above

12) In railways _____________ brake system is used.


a) Hydraulic
c) a) and b)
13) FRL unit is used in
a) Hydraulic system
c) a) and b)

b) Pneumatic
d) None of the above
b) Pneumatic system
d) None of the above

14) Pressure reducing values are normally _____________ pressure control value.
a) Closed
b) Open
c) a) and b)

d) None of the above


______________
Set A

*SLRBB92*

SLR-BB 92

-3-

Seat
No.

T.E. (Mechanical) (Part I) Examination, 2014


FLUID MACHINERY AND FLUID POWER (Old)
Day and Date : Friday, 5-12-2014
Time : 3.00 p.m. to 6.00 p.m.
N.B. :

Marks : 80

1) Question No. 2 and 5 are compulsory.


2) Solve any one question from each Section out of the remaining
questions.
3) Draw neat sketches wherever necessary.
SECTION I

2. Attempt any four of the following :

(45=20)

a) Explain the need of priming for centrifugal pump.


b) Explain importance of model testing.
c) Explain governing of water turbines.
d) Define and explain hydraulic efficiency, mechanical efficiency,overall efficiency of a turbine.
e) A turbine develops 7225 KW power under a head of 25 m 135 rpm. Calculate the specific speed of
the turbine and state the type of turbine.
f) A centrifugal pump delivers water at the rate of 40 lit/sec. for a total head of 30 m. Find the output of
the pump (water power). If the overall efficiency of the set consisting of the pump and electric motor
is 75%, find the power required to drive the motor.
3. a) Show that maximum efficiency of Pelton turbine is

1 + cos
. Where is the angle between
2

direction of relative velocity of jet at outlet and velocity of vane at outlet.


b) A single row impulse steam turbine develops 132 KW at a blade speed of 175 m/s using 2 kg of
steam per second. Steam leaves the nozzle at 400 m/s. Velocity coefficient of the blades is 0.9.
Steam leaves the turbine blades axially. Determine the nozzle angle, blade angles at entry and exit
assuming no shock.

c) Explain multistaging of centrifugal pumps.

7
6

4. a) A Kaplan turbine working under a head of 20 m develops 11772 KW shaft power. The outer diameter
of the runner is 3.5 m and hub diameter is 1.75 m. The guide blade angle at the extreme edge of the
runner is 35. The hydraulic and overall efficiencies of the turbine are 88% and 84% respectively. If
the velocity of whirl is zero at outlet . Determine :
i) Runner vane angles at inlet and outlet at the extreme edge of the runner.
ii) Speed of the turbine.

b) The diameters of an impeller of a centrifugal pump at inlet and outlet are 30 cm and 60 cm respectively.
The velocity of flow at outlet is 2 m/s and the vanes are set back at an angle of 45 at the outlet.
Determine the minimum starting speed of the pump if the manometric efficiency is 70%.

c) What is compounding of steam turbine ? Explain velocity compounding in detail.

6
Set A

SLR-BB 92

-4-

*SLRBB92*

SECTION II
5. Attempt any four of the following :

(45=20)

a) Draw symbol of pressure relief value and accumulator.


b) Explain construction and working of gear pump.
c) Explain with neat sketch piping layout for pneumatic system.
d) Explain construction and working of 3/2 spool type direction control valve.
e) Draw neat sketch of regenerative circuit.
f) Compare air motor with electric motor.
6. a) Explain construction and working of weight loaded accumulator.

b) Explain the criteria used for selection of air compressor.

c) Explain construction and working of pilot operated sequence valve.

7. a) Explain meter in circuit used in hydraulic system.

b) Explain the different materials used for seals used in hydraulic system alongwith the applications.

c) Explain with neat sketch construction and working of regulator used in pneumatic system.

_____________________

Set A

SLR-BB 93

*SLRBB93*
Seat
No.

Set

T.E. (Mech.) (Part I) Examination, 2014


METALLURGY (Old)
Day and Date : Monday, 8-12-2014
Time : 3.00 p.m. to 6.00 p.m.

Total Marks : 100

Instructions : 1) Q. No. 1 is compulsory. It should be solved in first 30 minutes


in Answer Book Page No. 3.
2) Answer MCQ/Objective type questions on Page No. 3 only.
Dont forget to mention, Q.P. Set (A/B/C/D) on Top of Page.
MCQ/Objective Type Questions
Duration : 30 Minutes

Marks : 20

1. Objective type questions :


Type (1) :
1. Match the appropriate pairs :
a) Ferrite
p) FCC
b) Austenite
q) Orthorhombic
c) Cementite
r) BCC
s) BCT

2. Match the appropriate pairs :


a) Austempering
p) Martensite
b) Martempering
q) Coarse Pearlite
c) Patenting
r) Fine pearlite
s) Bainite

Type (2) : MCQs with only one answer correct :

1. Which of the following components are manufactured by P/M ?


a) Ball bearings
b) Clutch lever
c) Torch nozzle
d) Tungsten filament
2. Which of the following test is recommended for detecting internal flaws in welded
vessel ?
a) Gamma ray radiography
b) Ultrasonic test
c) Dye penetrant test
d) Eddy current test

P.T.O.

SLR-BB 93

*SLRBB93*

-2-

3. Modification treatment is associated with


a) Cast iron

b) Tin bronzes

c) Aluminium bronzes

d) Aluminium silicon alloys

4. Which of the following brasses is a two phase brass ?


a) Admiralty brass
b) Guilding brass
c) Cartridge brass

d) Muntz metal

Type (3) : MCQs with more than one answer correct (2 marks each) :

10

1. Which of the following elements act as carbide stabilizer when added to steels ?
a) Cr

b) Ni

2. Babbits contain following elements


a) Sn
b) Mn

c) Mo

d) Si

c) Sb

d) Cu

3. Which of the following cast irons have higher carbon equivalent ?


a) Grey CI

b) White CI

c) Malleable CI

d) SG iron

4. Stability of austenite depends upon


a) Carbon content
c) Cooling rate

b) Heating rate
d) Presence of alloying elements

5. Eutectic alloy has following characteristics


a) Zero freezing range

b) Excellent forming ability

c) Excellent casting properties

d) Higher liquid to solid shrinkage

______________

Set A

*SLRBB93*

-3-

SLR-BB 93

Seat
No.

T.E. (Mech.) (Part I) Examination, 2014


METALLURGY (Old)
Day and Date : Monday, 8-12-2014
Time : 3.00 p.m. to 6.00 p.m.

Marks : 80

Instructions : 1) Attempt any three questions from Section I and any three
questions from Section II.
2) Draw neat sketches wherever necessary.
3) Assume suitable data wherever necessary and mention
it clearly.
SECTION I
2. a) Draw Fe-Fe3C equilibrium diagram, label all the temperatures and constituents
correctly.
5
b) Explain eutectic and eutectoid transformations w.r. to above diagram.
4
c) Draw microstructure of 0.2% C steel and 1.2% C steel.
4
3. a) Explain the effect of Ni, Cr, Mo and Tungsten on properties of steel.
b) Compare between martensitic, Ferritic and Austenitic stainless steels.
c) Explain the effect of inoculation on properties of CI.
4. a) Give typical composition, properties and applications of any three of the
following :
i) Maraging steel
ii) HCHC steel
iii) Muntz metal
iv) Hindalium.
b) Explain the stages in precipitation hardening of Duralimin.

4
6
3
9

5. Write notes on (any three) :


a) Lever arm principle.

b) Types of solid solutions.

c) Grey CI.

d) Tin Bronzes.

5
Set A

SLR-BB 93

-4-

*SLRBB93*

SECTION II
6. a) Draw T-T-T diagram for eutectoide steel.

b) Superimpose following curves/treatments on above diagram.


i) critical cooling rate
ii) annealing
iii) normalizing
iv) hardening
v) austempering
vi) martempering.

c) What is retained austenite ? How to transform retained austenite to martensite ? 3


7. a) Compare between (any three) of the following :
i) Annealing Vs. Normalising
ii) Carburising Vs. Nitriding
iii) Induction hardening Vs. Flame hardening
iv) Charpy test Vs Izod test.

b) Draw S-N curve for mild steel and also explain the effect of corrosion on
S-N curve. What is corrosion fatigue ?

8. a) Explain advantages and limitations of Brinell test over Rockwell and Vickers
test.

b) Explain the setup for Pulse-Echo method of ultrasonic test with its applications. 4
c) Draw flow chart for manufacture of Cemented carbide cutting tools by P/M.

9. Write notes on (any three) :


a) Tempering and its significance.

b) Radiography test.

c) Mechanical methods of powder manufacture.

d) Creep phenomenon.

4
_____________________

Set A

SLR-BB 94

*SLRBB94*m
Seat
No.

Set

T.E. (Mechanical Engineering) (Part I) Examination, 2014


METROLOGY AND MECHANICAL MEASUREMENT (Old)
Day and Date : Wednesday, 10-12-2014
Time : 3.00 p.m. to 6.00 p.m.

Max. Marks :100

Instructions : 1) Q. No. 1 is compulsory. It should be solved in first 30 minutes


in Answer book Page No. 3.
2) Answer MCQ/Objective type questions on Page No. 3 only.
Dont forget to mention, Q.P. Set (A/B/C/D) on Top of Page.
MCQ/Objective Type Questions
Duration : 30 Minutes
Marks : 20
1. Matrix Match Type :
i) Column I
A) LVDT

q)
r)
s)

Column II
Effective diameter
measurement
Mechanical comparator
Optical comparator
Electrical comparator

p)
q)
r)
s)

Column II
Acceleration measurement
Positive displacement
Acoustic measurement
Flow measurement

p)

B) Sigma comparator
C) Floating carriage micrometer
ii) Column I
A) Hot wire anemometer
B) Seismic instrument
C) Domestic water meter

Assertions and Reasoning Type :


It consists of two statements, one labelled as statement I and other labelled as statement II.
Select the answer using the following code :
iii. Statement I : Differential comparator is having balanced type air gauge.
Statement II : Back pressure is developed in differential comparator.
A)

B)

C)

D)

iv. Statement I : Piezometer tube can be used for measurement of negative pressures.
Statement II : Pirani gauge can be used for the measurement of negative pressures.
A)
B)
C)
D)
A) Both statement I and statement II are true, and statement II is the correct explanation
of statement II.
B) Both statement I and statement II are true but statement II is not correct explanation
of statement I.
C) Statement I is true, but statement II is false.
D) Statement I is false, but statement II is true.
P.T.O.

SLR-BB 94

*SLRBB94*m

-2-

Multiple correct answers questions :


v. Following terms are mainly regarded in metrology.
A) Interchangeability
B) Accuracy
C) Precision
D) None of the above
vi. The process of inspecting a gauge to verify that it meets a specific parameter means
A) Certification
B) Standardization
C) Calibration
D) None of the above
vii. Following substances can be used as thermometric substance for temperature
measurement below - 40 degree Celsius.
A) Mercury
B) Creosote
C) Alcohol
D) Pentane
viii. Electrical resistance strain gauges are used for measuring _________
A) Variable resistance
B) Static strain
C) Variable voltage
D) Dynamic strain
Straight objective type questions :
ix.

Vernier caliper gauge improves the _________


A) Sensitivity
B) Accuracy
C) Repeatability

D) Resolution

x.

A GO and NOT GO caliper gauge is used to _____________


A) Determine exact size
B) To check dimension within the design
C) Angle measurement
D) Thickness measurement

xi.

A Venturimeter is preferred to an orifice plate because ____________


A) It is cheaper
B) Its coefficient of discharge has a constant value
C) Energy or head loss is less
D) It is easy to install

xii. Thermistors are __________


A) Bi-metallic strips used in thermometers
B) Semiconductors which have negative coefficient of resistance
C) Metals having small positive coefficient of resistance
D) Transducers for measuring thermal properties of materials

______________
Set A

*SLRBB94*m

-3-

SLR-BB 94

Seat
No.

T.E. (Mechanical Engineering) (Part I) Examination, 2014


METROLOGY AND MECHANICAL MEASUREMENT (Old)
Day and Date : Wednesday, 10-12-2014
Time : 3.00 p.m. to 6.00 p.m.

Marks : 80

Note : 1) Attempt any three questions from Section I and any three
questions from Section II.
2) Draw meaningful sketches wherever necessary with pencil
only.
3) Figures to right indicate full marks.
4) Make suitable assumptions, if required and state them clearly.
SECTION I
2. a) Distinguish between
i) Unilateral and bilateral system
ii) Tolerance and allowance
iii) Hole basis and shaft basis system.
b) Explain construction, working and application of pneumatic comparator.

7
7

3. a) Define following terms :


i) Metrology
ii) International Meter using wavelength standard
iii) Measurement, Metrology and Inspection
iv) Accuracy, precision and error
v) Calibration of slip gauges.

b) What precautions should be taken while using slip gauges ? Give main
specifications of any two slip gauge boxes. How slip gauges are cleaned ?

4. a) What is Sine bar ? With neat sketch explain the application of sine bar for angle
measurement.

b) Explain in detail gauge length interferometer.

6
Set A

SLR-BB 94

-4-

*SLRBB94*m

5. a) Write a note on Bevel protractor as per Indian standard.

b) Explain three wire method for the measurement of effective diameter of screw thread. 7
6. a) What is working principle of optical flat ? Explain its application for the flatness
checking in detail.

b) Explain working principle of gear tooth vernier caliper. Describe detailed procedure
for the measurement of gear tooth thickness.

SECTION II
7. a) Explain Seismic instrument.

b) Explain RTD and thermocouples.


8. a) Explain Hot Wire Anemometer.
b) Explain the functional elements of generalized measurement system.
9. a) Explain working of stroboscope.
b) Explain Pirani gauge.

6
7
6
7
6

10. Write notes on any three :

14

a) Strain gauges for load measurement


b) Dead weight pressure gauge tester
c) Thermistors
d) Sound level meter.

_____________________

Set A

SLR-BB 95

*SLRBB95*
Seat
No.

Set

T.E. (Mech.) (Part I) Examination, 2014


INDUSTRIAL ENGINEERING (Old)
Day and Date : Friday, 12-12-2014
Time : 3.00 p.m. to 6.00 p.m.

Max. Marks : 100

Instructions : 1) Figure to the right indicate full marks.


2) Q. No. 1 is compulsory. It should be solved in first 30
minutes in Answer Book Page No. 3. Each question carries
one mark.
3) Answer MCQ/Objective type questions on Page No. 3
only. Dont forget to mention, Q.P. Set (A/B/C/D) on Top
of Page.
MCQ/Objective Type Questions
Duration : 30 Minutes
1. Choose the correct answer :
i) In MTM unit of time measurement is ___________
a) Hour
b) Minute
c) TMV

Marks : 20
(120=20)
d) Second

ii) __________ allowance is the management discretion.


a) Personal
b) Fatigue
c) Contingency
d) Policy
iii) Start and stop the machine is an example of
a) Constant element
b) Variable element
c) Foreign element
d) Occasional element
iv) __________ is required for drawing a cycle graph.
a) Light source
b) Stop watch
c) String

d) Templet

v) In method study critical examination is done through ___________


a) Work sampling
b) Questioning tech.
c) Stop watch tech.
d) Flow process chart
vi) Gilberth contributed to
a) Time study
b) Motion study c) Project study d) Value engg.
Observed Time
100
a) Cumulative time
b) Performance rating
c) Allowances
d) Standard time

vii) Normal time =

P.T.O.

SLR-BB 95

*SLRBB95*

-2-

viii) Factors affecting working conditions


a) Plant lighting and ventilation elements
c) String dia.

b) Material handling equipments


d) Process chart

ix) Following technique of work measurement is based on statistics


a) Time study
b) Work sampling
c) Analytical estimating
d) Standard data
x) What is the therblig for grasp ?
b)
c) O
d) #
a)
xi) Line balancing is necessary for following type of layout
a) Product
b) Process
c) Combination layout
d) Fixed position layout
xii) Check list method is a type of
a) Job evaluation b) Merit rating

c) Incentive scheme

d) Method study

xiii) In PDCA cycle C stands for


a) check
b) cycle
c) continuous
d) cumulative
xiv) Performance rating of a average qualified worker is ___________
a) 100
b) 50
c) 150
d) 200
xv) Rowan plan is a type of
a) Job evaluation plan
b) Merit rating plan
c) Sampling plan
d) Incentive plan
xvi) Which is recording technique for micromotion study ?
a) Simo chart
b) Activity chart c) Flow process chart
xvii) Merit rating is useful for
a) Rewarding the person
c) Job evaluation

d) MTM

b) Industrial safety
d) Machine allowance

xviii) Which is premium bonus incentive scheme ?


a) Taylor differential piece rate system b) Halsey plan
c) Emmersions plan
d) Group incentive
xix) The various charts is technique of
a) Method study
b) Micromotion study
c) Work measurement
d) MTM
xx) Fixed type layout is suitable for
a) Automobile industry
b) Sugar industry
c) Ship building
d) Textile industry
______________

Set A

*SLRBB95*

-3-

SLR-BB 95

Seat
No.

T.E. (Mech.) (Part I) Examination, 2014


INDUSTRIAL ENGINEERING (Old)
Day and Date : Friday, 12-12-2014
Time : 3.00 p.m. to 6.00 p.m.

Marks : 80

Instructions : 1) Out of remaining questions solve any two from each


Section.
2) Figure to the right indicate full marks.
SECTION I
2. a) Explain with figure flow process chart (man type).

b) Define industrial engg. and explain its functions and scope.

c) Define method study and give its objectives.

3. a) What is meant by micromotion study and explain SIMO chart ?

b) Explain with figure string diagram.

c) Compare cyclegraph and cronocycle graph with neat diagram.

4. a) Explain how, Lighting and noise factors plays an important role in productivity
improvement.

b) Explain with fig. erogonomic consideration at work place (machine shop).

c) Explain the importance of critical examination in method study.

SECTION II
5. a) Define work measurement and explain its basic procedure.

b) Explain PMTS with advantages and limitations.

c) List the various methods of job evaluation and explain any one.

6
Set A

SLR-BB 95

*SLRBB95*

-4-

6. a) Explain different factors affecting on layout design of the industry.

b) Explain principles and functions of material handling and explain material


handling equipments (any two).

c) Write a short note on performance rating.

7. a) Define merit rating and give its objective and advantages.

b) The following table gives time study readings.


Element

Observed time in seconds

Rating

11

10

12

80

16

16

14

14

15

100

40

38

39

40

40

110

Considering 12% allowance, calculate standard time.


c) Write a short note on PDCA cycle.

8
6

_____________________

Set A

SLR-BB 96

*SLRBB96*
S

T.E. (Mech.) (Part I) (Old) Examination, 2014


MACHINE DESIGN I
Day and Date : Monday, 15-12-2014
Time : 3.00 p.m. to 6.00 p.m.

Max. Marks : 100

Instructions : 1) Figures to the right indicate full marks.


2) Assume suitable data if necessary and mention it clearly.
3) Q. No. 1 is compulsory. It should be solved in first 30
minutes in Answer book Page No. 3.
4) Answer MCQ/Objective type questions on Page No. 3
only. Dont forget to mention, Q.P. Set (A/B/C/D) on Top
of Page.
MCQ/Objective Type Questions
Duration : 30 Minutes

Marks : 20

1. Objective type questions :


A) Match the following :
Column I
a)
b)
c)
d)

Goodman line
Soderberg line
Stress concentration
Fatigue failure

4
Column II
p)
q)
r)
s)
t)

B) Match the following :


Column I
a)
b)
c)
d)

Bushed pin coupling


Rigid flange coupling
Mechanical advantage
Fatigue

Localization of high stresses


Finite life
Fluctuating stresses
Syt
Sut
4
Column II

p)
q)
r)
s)
t)

Load arm/Effort arm


Misaligned shafts
Exact aligned shafts
Effort arm/load arm
Endurance limit
P.T.O.

SLR-BB 96

*SLRBB96*

-2-

C) Multiple correct answer :


1) Stress concentration is caused due to

(42=8)

a) Variation in properties of material from point to point in a member


b) Pitting at points or areas at which loads on a member are applied
c) Fine finish
d) All of the above
2) Standardisation deals with
a) dimensions of machine elements b) aesthetic
c) method of testing the product
3) Ergonomics
a) Design of controls

d) all the three


b) Design of displays

c) Look

d) All the three

4) Factor of safety depends on


a) efficiency of the joint
c) type of load

b) Lames equation
d) properties of material

D) Single answer correct :

(41=4)

1) While designing a shaft, pulley and key for a system


a) shaft is the weakest member
b) pulley is the weakest member
c) key is the weakest member

d) all are designed for equal strength

2) If a material fails below its yield point, failure would be due to


a) impact loading
b) fatigue
c) creep

d) sudden loading

3) While of the following is a permanent fastening


a) bolts
b) cotters
c) rivets

d) keys

4) When two springs are in series (having stiffness K), the equivalent, stiffness
will be
a) K

b) K/2

c) 2K

d) K/4

______________

Set A

*SLRBB96*
S

SLR-BB 96

-3-

T.E. (Mech.) (Part I) (Old) Examination, 2014


MACHINE DESIGN I
Day and Date : Monday, 15-12-2014
Time : 3.00 p.m. to 6.00 p.m.

Marks : 80

Instructions : i) Answer any two questions from each Section.


ii) Figures to the right indicate full marks.
iii) Assume suitable data if necessary and mention it clearly.
SECTION I
2. a) Explain significance of theories of failure. Explain maximum principal stress
theory.
b) State and explain different types of loads with suitable example.

5
5

c) The pull in the tie rod of the roof truss is 50 kN. Design a suitable screw joint.
The permissible tensile and shear stresses are 75 N/mm2 and 37.5 N/mm2
respectively. Assume design load as 1.3 times normal load and crushing
stress is 90 N/mm2. Refer Table No. 1.
10
M

(
r

3. a) Explain stress concentration. Explain the methods with diagrams to reduce


stress concentration.
b) Explain Soderberg and Goodman diagrams.

6
6

Set A

SLR-BB 96

-4-

*SLRBB96*

c) A rotating bar made of steel 45C8 (Sut = 630 N/mm2) is subjected to a


completely reversed bending stress. The corrected endurance limit of the bar
is 315 N/mm2. Calculate the fatigue strength of the bar for a life of 90,000
cycles.
4. a) Explain design procedure of Knuckle joint with necessary sketches.

8
6

b) Explain cumulative damage in fatigue.

c) It is required to select a flat belt drive for a compressor running at 720 rpm,
which is driven by a 25 kW, 1440 rpm moor. Space is available for a centre
distance of 3 meter. The belt is open type. (Use the tables given.)

SECTION II
5. a) Explain the Aesthetic considerations in machine design with suitable examples. 6
b) Define the terms :

i) Transmission shaft
ii) Axle
iii) Spindle
iv) Line shaft.
c) Explain the design procedure of rigid flange coupling with neat sketches.
6. a) Explain stress concentration due to the curvature of the coil and how it is
accounted in spring design.
b) Describe springs in series and springs in parallel.
c) Design a helical spring for a spring loaded safety valve for the following
conditions :

10
5
5
10

operating pressure = 1 N/mm2


Maximum pressure when the valve blows off freely = 1.075 N/mm2
Maximum lift of the valve when the pressure is 1.075 N/mm2 = 6mm
Diameter of valve seat = 100 mm
Maximum shear stress = 400 MPa
Modulus of rigidity = 86 kN/mm2
Spring index = 5.5
Consider effect of direct shear and neglect curvature effect.

Set A

*SLRBB96*

SLR-BB 96

-5-

7. a) Give the details of design of machine controls.

b) Discuss the advantages and disadvantages of welded joints over other types
of joints.

c) A steel plate is fixed to a column by means of six rivets as shown in fig. The
plate carries a load of 48 kN as shown. Design the river diameter for maximum
allowable stress of 48 N/mm2.

10

Table 1 : Load correction factor (Fa) :


Type of load

Fa

i) Normal load
ii) Steady load, eg. centrifugal pumps-fans light machine

1.0
1.2

iii) Intermittent load, eg. heavy duty fan blowers-reciprocating


pumps-line shafts-heavy duty machines compressors

1.3

iv) Shock load, eg. vacuum pumps-rolling mills-hammers grinder

1.5

Table 2 : Arc of contact factor (Fd) :



(

Table 3 : The power transmitting capacities of belt :


H

0
d

0
F

Set A

SLR-BB 96

*SLRBB96*

-6-

The above values are based on following two assumptions :


i) The arc of contact is 180
ii) The belt velocity is 5.08 m/s
The standard widths of these belts (in mm) are as follows :
3

Table 4 : Preferred pitch diameters of pulleys :

_____________________

Set A

SLR-BB 97

*SLRBB97*
Seat
No.

Set

T.E. (Mech.) (Part I) Examination, 2014


THEORY OF MACHINE II (New)

Day and Date : Wednesday, 3-12-2014


Time : 3.00 p.m. to 6.00 p.m.

Max. Marks : 100

Instructions : 1) Q. No. 1 is compulsory. It should be solved in first 30 minutes in


Answer Book Page No. 3. Each question carries one mark.
2) Answer MCQ/Objective type questions on Page No. 3 only.
Dont forget to mention, Q.P. Set (A/B/C/D) on Top of Page.
3) Solve any 3 questions from Section I and Solve any 3
questions from Section II.
4) Figures to the right indicate full marks.
5) Assume suitable data wherever required and mention it.
MCQ/Objective Type Questions
Duration : 30 Minutes

Marks : 20

1. Choose the correct answer :


1) Velocity of sliding is proportional to distance form pitch point to ___________
a) point of approach
b) point of contact
c) point of recess
d) point of tangent
2) For helical gears ____________
a) PN = PC tan
b) PC = PN / cos c) PN = PC / cos d) PC . PN = cos
3) When the axes of intersecting shafts are perpendicular to each other it is called as
______
a) Perpendicular shafts
b) Herringbone shafts
c) Mitres
d) Right angled shafts
4) In an differential gear direction of first and last gear is ____________
a) same
b) opposite
c) cant say
d) either same or opposite
5) Ratio of speed of driver to driven is called as _______
a) speed ratio
b) train value
c) train ratio

d) speed value

6) When the ship is pitching upward, the effect of gyroscopic couple in rolling is
__________
a) move towards starboard side
b) no effect
c) move towards port side
d) stern will be lifted
7) In four wheelers condition that inner wheel wont leave ground is __________
a) w/4 + p/2 + q/2 > 0
b) p/2 + q/2 > w/4
c) p/2 + q/2 < w/4
d) p/2 q/2 > w/4
P.T.O.

SLR-BB 97

*SLRBB97*

-2-

8) The air screw of aeroplane is rotating clockwise when looking from the front. If it
makes a left turn, the gyroscopic effect will __________
a) tend to depress the nose and raise the tail
b) till the aeroplane
c) tend to raise the nose and depress the tail
d) none of above
9) Coefficient of steadiness is __________
a) reciprocal of CE b) reciprocal of SC c) reciprocal of CS d) none of above
10) Value of is ___________ in 2 stroke engine.
b) / 2
c) 4
a)

d) 2

11) For dynamic balancing of shaft ____________


a) net dynamic force acting is zero
b) net couple due to dynamic force is zero
c) both a) and b)
d) none of the above
12) Reference plane is used in _________
a) Balancing of single mass by two masses in different plane
b) Balancing of several masses rotating in same plane
c) Balancing of two masses rotating in same plane
d) Balancing of several masses rotating in different plane
13) Balancing of rotating and reciprocating masses of engine is necessary at
_________
a) slow speed
b) medium speed c) high speed
d) mean speed
14) A torsional vibratory system having two rotors connected by a shaft has
__________
a) one node
b) two node
c) three node
d) no node
15) Transmissibility is ___________
a) FT / F
b) F / FT

c) FT / FS

d) F . FT

16) In vibration isolation system if w / w n > 1 then the phase difference between the
transmitted force and the disturbing force is _________________
a) 0
b) 90
c) 270
d) 180
17) At resonance ________________
a) w = 0

b) w / w n = 1

c) w n = 0

d) w = w n

18) Natural frequency of shaft with both ends fixed and UDL is __________
a) fn = 0.751 / s b) fn = 0.571 / s c) fn = 0.4985 / s d) none of the above
19) In torsional vibrations stresses induced are _______
a) compressive
b) shear
c) bending

d) no stresses

20) In under damped vibration system if x1 and x2 are successive values of amplitude
on same side of mean position then logarithmic decrement is __________
a) x1 / x2
b) log(x1 / x2)
c) loge (x1 / x2)
d) log(x1 . x2)
______________

Set A

*SLRBB97*

-3-

SLR-BB 97

Seat
No.
T.E. (Mech.) (Part I) Examination, 2014
THEORY OF MACHINE II (New)

Day and Date : Wednesday, 3-12-2014


Time : 3.00 p.m. to 6.00 p.m.

Marks : 80

Instructions :1) Solve any 3 questions from Section I and Solve any 3
questions from Section II.
2) Figures to the right indicate full marks.
3) Assume suitable data wherever required and mention it.
SECTION I
2. a) Define the following terms :
i) Base circle
ii) Pressure Angle
iii) Circular pitch
iv) Diametral Pitch.

b) Two 20 involute spur gears mesh externally and give a velocity ratio of 3. The
module is 3 mm and the addendum is equal to 1.1 module. If the pinion rotates at
120 rpm, determine the
i) Minimum number of teeth on each wheel to avoid interference
ii) Contact ratio.
10
3. a) Explain with neat sketch differential gear of an automobile .

b) The turning moment diagram for a multicylinder engine has been drawn to a scale
1 mm = 600 N.m vertically and 1 mm = 3 horizontally. The intercepted areas
between output torque curve and the mean resistance line, taken in order from
one end, are as follows :
+ 52, 124, + 92, 140, + 85, 72 and + 107 mm2, when the engine is running at a
speed of 600 r.p.m. If the total fluctuation of speed is not to exceed 1.5% of the
mean speed, find the necessary mass of the flywheel of radius 0.5 m.

Set A

SLR-BB 97

-4-

*SLRBB97*

4. a) Define the following terms :


i) Coefficient of fluctuation of energy.
ii) Coefficient of fluctuation of speed.

b) Fig. I shows an epicyclic gear train with the following details :

A has 40 teeth external (fixed gear) ; B has 80 teeth internal; C D is a compound wheel
having 20 and 50 teeth (external) respectively, E-F is a compound wheel having
20 and 40 teeth (external) respectively, and G has 90 teeth (external). The arm runs at
100 r.p.m. in clockwise direction. Determine the speeds for gears C, E and B.
10
5. a) Derive relation for angle of heel to avoid skidding of the two wheeler.

b) Explain gyroscopic effect on four wheeled vehicle.

SECTION II
6. A shaft is supported in bearings 1.8 m apart and projects 0.45 m beyond bearings at
each end. The shaft carries three pulleys one at each end and one at the middle of its
length. The mass of each end pulley is 48 kg and 20 kg and their center of gravity are
15 mm and 12.5 mm respectively from the shaft axis. The centre pulley has a mass of
56 kg and its centre of gravity is 15 mm from the shaft axis. If pulleys are arranged so as
to give static balance determine :
1) Relative angular positions of the pulleys and
2) Dynamic forces produced on the bearings when the shaft rotates at 300 rpm.

13

Set A

*SLRBB97*

-5-

SLR-BB 97

7. a) A flywheel is mounted on a vertical shaft as shown in fig. VI-a. The both of the shaft
are fixed and its diameter is 50 mm. The flywheel has a mass of 500 kg. Find the
natural frequencies of longitudinal and transverse vibrations.
Take E = 200 GN/m2.

b) A machine of mass 75 kg is mounted on springs and is fitted with a dashpot to


damp out vibrations. There are three springs each of stiffness 10 N/mm and it is
found that the amplitude of vibration diminishes from 38.4 mm to 6.4 mm in two
complete oscillations. Assuming that the damping force varies as the velocity,
determine :
1) the resistance of the dashpot unit velocity
2) the ratio of the frequency of the damped vibration to the frequency of the
undamped vibration ; and
3) the periodic time of the damped vibration.
8. a) Define :

7
5

i) Variation in tractive force


ii) Swaying couple
iii) Hammer blow.

Set A

SLR-BB 97

-6-

*SLRBB97*

b) A single cylinder oil engine drives directly a centrifugal pump. The rotating mass
of the engine, flywheel and the pump with the shaft is equivalent to a three rotor
system as shown in Fig. The mass moment of inertia of the rotors A, B and C are
0.15, 0.3 and 0.09 kgm2. Find natural frequency of the torsional vibration. The
modulus of rigidity for the shaft material is 84 kN/mm2.

9. a) Draw amplitude draw diagram for two rotar system and three rotar system and
show the position of nodes.

b) Define :
i) Damping coefficient
ii) Damping ratio
iii) Critically damped system
iv) Logarithmic Decrement.

c) Explain Whirling of Shafts.

7
_____________________

Set A

SLR-BB 98

*SLRBB98*
Seat
No.

Set

T.E. (Mechanical) (Part I) Examination, 2014


HEAT AND MASS TRANSFER (New)
Day and Date : Friday, 5-12-2014
Time : 3.00 p.m. to 6.00 p.m.

Total Marks : 100

Instructions : 1) Assume suitable data if required.


2) Use of scientific calculator is allowed.
3) Q. No. 1 is compulsory. It should be solved in first 30 minutes
in Answer book Page No. 3. Each question carries one mark.
4) Answer MCQ/Objective type questions on Page No. 3 only.
Dont forget to mention, Q.P. Set (A/B/C/D) on Top of Page.
Duration : 30 Minutes

MCQ/Objective Type Questions

Marks : 20

1. Choose the correct option for the following :


20
1) Heat transfer takes place according to __________ law of thermodynamics.
A) Zeroth
B) First
C) Second
D) Third
2) A satellite in space exchanges heat with surrounding essentially by _________
A) Conduction B) Convection C) Radiation D) Conduction and Convection
3) In which one of the following materials, the energy propagation due to
conduction heat transfer will be minimum ?
A) Lead
B) Copper
C) Water
D) Air
4) The steady state temperature distribution in a very large thin plate with uniform
surface temperatures will be
A) Linear
B) Parabolic
C) Hyperbolic
D) Logarithmic
5) A composite slab has two layers of different materials having thermal
conductivities K1 and K2. if each layer has same thickness, the equivalent
thermal conductivity of slab would be
A)

K1K 2
K1+K 2

B)

K1K 2
2(K1+K 2 )

C)

2K1
K1+K 2

D)

2K1K 2
K1+K 2

6) Up to the critical radius of insulation


A) Heat loss decreases with addition of insulation
B) Heat loss increases with addition of insulation
C) Heat loss remains constant
D) There occurs a decrease in heat flux
7) An increase in fin effectiveness is caused by high value of
A) Convection coefficient only
B) Thermal conductivity only
C) Sectional area only
D) Circumference and thermal conductivity
P.T.O.

SLR-BB 98

-2-

*SLRBB98*

8) For a finned surface, it is considered appropriate that area of cross section be


A) Maintained constant along the length
B) Increased along the length
C) Reduced along the length
D) None
9) In the lumped parameter model, the temperature variation with time is
A) Linear
B) Cubic
C) Exponential
D) Sinusoidal
10) The thermal radiation occurs in the portion of electromagnetic spectrum
between the wavelengths
A) 102 to 104 micron
B) 101 to 102 micron
2
D) 102 micron onwards
C) 0.1 to 10 micron
11) Gases have poor
A) Absorptivity
B) Reflectivity
C) Transmittivity D) None
12) The intensity of solar radiation on earth is
A) 1 Kw/m2
B) 2 Kw/m2
C) 5 Kw/m2

D) 10 Kw/m2

13) For a hemisphere, solid angle is measured in


A) Radian and its maximum value is
B) Degree and its maximum value is 180
C) Steradian and its maximum value is 2
D) Steradian and its maximum value is
14) The Nusselt number in natural transfer is a function of fluid Prandtl number and
A) Grashoff number B) Biot number C) Reynolds number D) None
15) The normal automobile radiator is a heat exchanger of type
A) Direct contact B) Parallel flow C) Counter flow D) Cross flow
16) In a heat exchanger with one fluid evaporating or condensing, the surface
area required is least in
A) Parallel flow
B) Counter flow
C) Cross flow
D) Same in all above arrangements
17) The requirement of transfer of large heat is usually met by
A) Increasing length of tube
B) Decreasing the diameter of the tube
C) Increasing the number of tubes
D) Having multiple tube or shell passes
18) The value of Prandtl number for air is about
A) 0.1
B) 0.4
C) 0.7

D) 1.1

19) Drop wise condensation usually occurs in


A) Oily Surface B) Glazed Surface C) Smooth Surface D) Coated Surface
20) Which one of dimensionless number has a significant role in forced convection
A) Prandtl number
B) Reynolds number C) Mach number D) None
______________

Set A

*SLRBB98*

-3-

SLR-BB 98

Seat
No.

T.E. (Mechanical) (Part I) Examination, 2014


HEAT AND MASS TRANSFER (New)
Day and Date : Friday, 5-12-2014
Time : 3.00 p.m. to 6.00 p.m.

Marks : 80

Instructions : 1) Out of remaining questions, attempt any two questions from


each Section.
2) Assume suitable data if required.
3) Use of scientific calculator is allowed.
SECTION I
2. A) Derive the three dimensional equation for heat conduction in Cartesian
Coordinates for no heat generation.

B) An electric cable of 10 mm diameter is to be laid in atmosphere at 20C. The


estimated surface temperature of the cable due to heat generation is 65C.
Find the maximum percentage increase in heat dissipation when the wire is
insulated with rubber having K = 0.155 w/mk. Take h = 8.5 w/m2k.

C) Explain Biot number and Fourier number in lumped system analysis and give
their significance.

3. A) Define Nusselt number, Reynold and Prandtl number.

B) Derive an expression for temperature distribution and heat flow rate through
a pin fin considering uniform cross section with finite length and fin tip is
insulated.

C) The internal heat generation in a 5 m thick plate of concrete dam (K = 1.4 w/mk)
is 105 w/m3 due to release of hydrogen during pouring process which is for a
short interval of time. If both the dam surface are at 20C calculate the
maximum temperature to which the concrete wall be subjected. Also calculate
the heat flow rate from the dam if dam is 80 m wide and 30 m high.

Set A

SLR-BB 98

-4-

*SLRBB98*

4. A) Liquid mercury flows at a rate of 1.6 kg/s through a copper tube of 20 mm


diameter. The mercury enters the tube at 15C and after getting heated it
leaves the tube at 35C. Calculate the length for constant heat flux at the wall
which is maintained at an average temperature of 50C. Correlation is taken
as Nu = 7 +0.025 (Pr*Re)0.8. Thermophysical properties of mercury at 25C
are =13582 kg/m3, k = 8.69 w/mC, Cp = 140 J/kgC, = 1.5 107 m2/s,
Pr = 0.0248.

B) Using dimensional analysis, obtain the general form of equation for natural
convection heat transfer.

C) Derive critical radius for cylinder.

SECTION II
5. a) Define following terms :

i) Emissive power
ii) Emissivity
iii) Monochromatic emissivity.
b) State Stefan Boltzmanns law and deduce it from Plancks distribution law.

c) Find the percentage reduction in heat transfer with a polished aluminum


radiation shield with emissivity 0.03 when placed between two very large
parallel plates with emissivities 0.6 and 0.8.

6. a) Explain with sketch construction, working and applications of heat pipe.

b) A parallel flow heat exchanger is used to cool 3600 kg/hr of oil (Cp = 2000 J/kgK)
at 150C with the help of water (Cp = 4187 J/kgK) flowing at the rate of
3710 kg/hr. Water enters at 25C. The overall heat transfer co-efficient is
500 W/m2K and surface area is 4.872m2. Calculate exit temperatures of oil
and water by using NTU-effectiveness method.

c) Derive an expression for LMTD of counter flow heat exchanger.

7. a) Explain different modes of mass transfer.

b) Explain different regions of a Pool boiling curve with diagram.

c) Explain in detail Fouling factor.

_____________________

Set A

SLR-BB 99

*SLR-BB-99*
Seat
No.

Set

T.E. (Mech.) (Part I) Examination, 2014


METALLURGY (New)
Day and Date : Monday, 8-12-2014
Time :3.00 p.m. to 6.00 p.m.

Max. Marks : 100

Instructions : 1) Answer MCQ/Objective type questions on Page No. 3


only. Dont forget to mention, Q.P. Set (A/B/C/D) on Top
of Page.
2) Q. No. 1 is compulsory. It should be solved in first 30 minutes
in Answer Book Page No. 3.
MCQ/Objective Type Questions
Duration : 30 Minutes

Marks : 20

1. Type I : MCQ with more than one answer correct (2 marks each) :

(25=10)

1) Which of the following are proeutectic constituents present in Fe-Fe3 diagram ?


a) Alpha Ferrite

b) Delta ferrite

c) Austenite

d) Cementite

2) Which of the following steel can not hardened by Heat treatment ?


a) Austenitic st steel

b) Ferritic st steels

c) Maraging steels

d) HCHC steels

3) Which of the following alloys are having two phase structure ?


a) Cartridge brass

b) Muntz metal

c) Babbits

d) Admiralty brass

4) Which of the following components are manufactured by powder metallurgy ?


a) Tungsten Filament

b) Nano Materials

c) Cermets

d) Ball bearing Balls

5) Which of the following NDT methods are useful in detecting cracks on the
surface of component ?
a) Ultrasonic test

b) Magnetic Particle test

c) Radiography

d) Dye Penetrant test


P.T.O.

SLR-BB 99

*SLR-BB-99*

-2-

Type II MCQ with only one answer correct (1 marks each) :

(110=10)

6) Number of phases present at eutectic point are


a) Zero

b) One

c) Two

d) Three

7) Which of the following alloy has zero freezing range ?


a) Eutectic

b) Eutectoid

c) Peritectic

d) Hypoeutectic

8) Which of the following steel gets work hardened ?


a) Hadfield Si steel

b) Hadfield Mn steel

c) HSLA steel

d) Dual phase steel

9) Babbitts are alloys of


a) Sn-Sb

b) Cu-Pb

c) Bi-Pb

d) Cu-Bi

10) Which of the following element is neutral element added to alloy steel ?
a) W

b) Mn

c) Co

d) Pb

11) Cold worked parts are subjected to


a) Full annealing

b) Spheroidising annealing

c) Homogenising annealing

d) Recrystallization Annealing

12) Which of the following steel is oil hardened ?


a) Austenitic St steel

b) HCHC steel

c) High carbon steel

d) HSLA steel

13) To measure hardness of razor blade _______________ hardness test is used.


a) Brinell

b) Rockwell

c) Vickers

d) Poldi

14) Which of the following NDT is fast method of testing ?


a) Eddy Current b) MP test

c) DP test

d) Radiography

15) During manufacturing of self lubricating bearing, which of the following is


important step ?
a) Sizing

b) Heat treatment c) Inspection

d) impregnation

______________

Set A

*SLR-BB-99*

-3-

SLR-BB 99

Seat
No.

T.E. (Mech.) (Part I) Examination, 2014


METALLURGY (New)
Day and Date : Monday, 8-12-2014
Time :3.00 p.m. to 6.00 p.m.

Marks : 80

Instructions : 1) Attempt any three questions from Section I and any three
questions from Section II.
2) Draw neat sketches wherever necessary.
3) Assume suitable data wherever necessary and mention it
clearly.
SECTION I
2. a) Draw Fe-Fe3C equilibrium diagram, label all the temperatures and
constituents.
b) Explain all the transformations occurring in above diagram.
3. a) Explain the importance of Mn and Cr as a alloying element in the steel.
b) Why Cu is an essential constituents of Babbits ?
c) Differentiate between Gray Cast Iron and White Cast Iron. Why Nodulizing
treatment is given to Gray Cast Iron ?
4. a) Give composition, properties and applications of (any three) :

6
7
4
3
6
9

i) HCHC steel
ii) HSS steel
iii) Bell metal
iv) Duralumin.
b) Explain Lever Arm principle and its applications.

5. Write note on (any three) :


a) Classification of metallic materials

b) Comparison between Interstitial and substitutional solid solution

c) Nano materials

d) Two phase Brasses.

4
Set A

SLR-BB 99

-4-

*SLR-BB-99*

SECTION II
6. a) Draw TTT diagram for eutectoid steel and superimpose following heat
treatments on it :

i) Austempering
ii) Martempering
iii) Patenting
b) What is annealing ? Enlist different types with purpose of each.
7. a) Attempt the following questions (any 3) :

6
9

i) What are the different stages in Quenching ?


ii) Compare between Carburizing and Nitriding.
iii) Draw the set-up for Ultrasonic Test of NDT.
iv) What are advantages of Rockwell Hardness tests over others.
b) What is Hardenability ? Explain the factors affecting Hardenability.

8. a) What are the applications of Powder Metallurgy ? Explain the types of


sintering.

b) What is Fatigue Limit ? Draw S-N curve for steel and Brass.

9. Write a short note on (any 3) :


a) Types of Tempering

b) Magnetic Particle Test

c) Flame Hardening

d) Manufacturing of Cemented carbide by PM.

_____________________

Set A

You might also like